Fuhrman & Zimmerman's Pediatric Critical Care [6 ed.] 9780323672696, 2020930336

Cleaned version with corrected pagination, page orientation and bookmarks.

2,233 152 149MB

English Pages 1628 [2609] Year 2021

Report DMCA / Copyright

DOWNLOAD FILE

Polecaj historie

Fuhrman & Zimmerman's Pediatric Critical Care [6 ed.]
 9780323672696, 2020930336

Table of contents :
Inside Front Cover
Front Matter
Fuhrman & Zimmerman’s Pediatric Critical Care
Copyright
Contributors
Preface
Contents
Section I: Pediatric Critical Care: The Discipline
Chapter 1: History of pediatric critical care medicine
Pearls
Evolution of modern medicine
Anatomy and physiology
Resuscitation and ventilatory support
Contributions of specific disciplines
Pediatric anesthesiology
Pediatric general surgery and pediatric cardiac surgery
Neonatology
Pediatric cardiology
Early use of mechanical ventilation in neonates and children
Poliomyelitis and creation of the first intensive care units
Definitions
Pediatric intensive care unit
Pediatric intensivist
First pediatric intensive care units
Central role of critical care nursing
Role of pediatric anesthesiologists and pediatricians in founding pediatric critical care medicine
Growth of pediatric critical care medicine
Growth in numbers of pediatric intensive care units
Growth in training programs and education
Cost of success in pediatric critical care medicine
Around the world
High-mortality countries
Summary
Acknowledgments
Key references
Chapter 2: High-reliability pediatric intensive care unit: Role of intensivist and team in obtaining optimal outcomes
Pearls
Pediatric intensive care unit as a system
Models of critical care delivery
Structure
Process
Outcomes
Summary
References
Chapter 3: Critical communications in the pediatric intensive care unit
Pearls
Intensive care unit design
Medical record
Huddles
Checklists
Rounds
Closed-loop communication
Transitions of care
Medical training
Debriefing
Team training
Conclusion
Key references
Chapter 4: Professionalism in pediatric critical care
Pearls
Profession
The virtuous doctor
Stakes
Great paradox of the medical profession
Professionalism, the physician charter
Pediatric intensive care unit as a site for medical education and lifelong learning
References
Chapter 5: Leading and managing change in the pediatric intensive care unit
Pearls
National change day: A case study in leading change
History and development of change management
Change management in healthcare
Models and tools to facilitate change leadership and management
Theories of change
Bringing theory to practice
Tools for assessing readiness for change
Tools to implement change
Sustaining change
Conclusion
Key references
Chapter 6: Evolution of critical care nursing
Pearls
Early pediatric critical care nursing
Describing what nurses do: The synergy model
Patient characteristics of concern to nurses
Nurse competencies important to patients and families
Optimal patient outcomes
Patient-level outcomes
Provider-level and system-level outcomes
Nightingale metrics
Leadership
Beacon award
Professional development
Staff development
Orientation
Continuing education
Certification in pediatric nursing and pediatric critical care nursing
Evolution of advanced practice registered nurses into pediatric critical care
Clinical nurse specialists
Pediatric nurse practitioners
Nursing research
Summary
Key references
Chapter 7: Fostering a learning healthcare environment in the pediatric intensive care unit
Pearls
Learning healthcare system
Foundation predicated on professionalism
Pillars of a learning healthcare environment
Best-practice clinical care
Clinical research, including quality improvement
Interdisciplinary educational model
Benefits of a learning healthcare environment
Key references
Chapter 8: Challenges of pediatric critical care in resource-poor settings
Pearls
Child mortality rates
Current trends and health maintenance
Justification for critical care in resource-poor settings
State of critical and intensive care delivery in resource-limited settings
Approach to basic critical care in resource-limited settings
Cost considerations in critical care delivery
Ethics of intensive care in resource-poor settings
Strengthening critical care infrastructure
Healthcare systems
Pediatric critical care capacity building through education
Critical illness during public health emergencies
How to develop an ICU in low- to middle-income countries
Importance of critical care research in limited resource settings
Key references
Chapter 9: Public health emergencies and emergency mass critical care
Pearls
How many pediatric patients could be affected in a public health emergency?
What are the most likely public health emergencies?
Who will make decisions during an emergency?
What is the expected timeline of a public health emergency?
What is a surge and what can be done to meet surge needs?
How can the intensive care unit support the emergency department during a public health emergency?
How can all intensive care units work together?
What steps can be taken to maximize intensive care unit treatment in a disaster?
Patient spaces
Personnel
Mechanical ventilation
Manual ventilation
Equipment and supplies
Medications
How will the intensive care unit evacuate if needed?
How should pediatric patients be tracked?
How will limited services be ethically rationed?
What are the mental health considerations relevant to emergency mass critical care?
What is the role of medical learners in public health emergencies?
Conclusion
Key references
Chapter 10: Lifelong learning in pediatric critical care
Pearls
Adult learning theory in medical education
Graduate medical education
Accreditation council for graduate medical education core competencies, milestones, and entrustable professional activities
Methods of teaching
Bedside teaching
Procedural training
Simulation training
Beyond graduate medical education
Continuing medical education, board certification, and maintenance of certification
New methods of assessment and future challenges
Key references
Section II: Pediatric Critical Care: Tools and Procedures
Chapter 11: Essential concepts in clinical trial design and statistical analysis
Pearls
Purpose of a clinical trial
Clinical trial design
Getting started: Question and hypothesis
Target population: Minimizing variation versus generalizability
Power and sample size
Randomization
Blinding
Outcome selection
Mortality
Morbidity
Organ dysfunction
Resource use
Functional status
Quality of life
Composite end points
Outcome-free time
Surrogate end points
Common trial designs
Phases of clinical trials for new drug approval
Statistical analysis and reporting
Whom to analyze?
Hypothesis testing and determining the study result
Inference and estimate of effect
95% confidence interval
P values
Additional sources and mitigation of bias
Additional methods of exploring study results
Negative studies
Conclusions
Key references
Chapter 12: Prediction of short-term outcomes during critical illness in children
Pearls
Historical perspective
Methods
Conceptual framework
Statistical issues
Current prediction tools for assessment of mortality risk
Neonatal intensive care unit prediction methods
Pediatric intensive care unit prediction tools
Cardiac intensive care unit prediction tools
Additional algorithms in the public domain
Next generation: Morbidity and mortality prediction—trichotomous outcome
Morbidity assessment
Application of prediction tools in pediatric intensive care
Future directions: Predictive analytics and tools for decision support
Key references
Chapter 13: Pediatric critical care transport
Pearls
Pediatric transport systems
Specialized teams improve outcome
Components of a specialized interfacility transport team
Communications
Staffing
Equipment
Safety and quality improvement
Stresses of the transport environment
Referring hospital responsibilities
Summary
Key references
Chapter 14: Pediatric vascular access and centeses
Pearls
Intraosseous infusion
Indications
Contraindications
Supplies and equipment
Technique
Maintenance
Complications
Summary
Arterial catheter placement
Indications
Contraindications
Procedure
Technique
Maintenance of an arterial catheter
Complications
Summary
Central venous line placement
Indications and contraindications
Technique
Internal jugular vein cannulation
Subclavian vein cannulation
Femoral vein cannulation
Use of ultrasound for central venous line placement
Complications
Peripherally inserted central venous catheters
Ultrasound-assisted peripheral venous access
Venous cutdown
Umbilical arterial catheter and umbilical venous catheter placement
Supplies and equipment
Technique
Umbilical arterial cannulation
Umbilical venous cannulation
Proper placement of umbilical arterial and venous catheters
Maintenance
Removal
Complications
Umbilical arterial cannulation
Umbilical venous cannulation
Summary
Pulmonary artery catheterization
Contraindications
Procedure and equipment
Maintenance
Complications
Summary
Thoracentesis
Indications
Contraindications
Preparation
Technique
Complications
Interpretation
Summary
Tube thoracostomy
Contraindications
Supplies and equipment
Technique
Maintenance
Complications
Summary
Pericardiocentesis
Indications
Contraindications
Procedure
Monitoring for pericardiocentesis
Technique
Maintenance
Complications
Summary
Abdominal paracentesis
Indications
Contraindications
Procedure
Technique
Complications
Interpretation
Summary
Key references
Chapter 15: Ultrasonography in the pediatric intensive care unit
Pearls
Ultrasound physics and basics of image optimization
Transducers
Procedural guidance
Venous access
Arterial and peripheral intravenous access
Umbilical access
Drainage procedures
Lumbar puncture
Diagnostic modalities
Pulmonary ultrasound
Abdominal ultrasound
Right upper quadrant (fig. 15.18a)
Left upper quadrant (fig. 15.18b)
Pelvis (figs. 15.18c and 15.18d)
Subcostal cardiac (fig. 15.18e)
Cardiac ultrasound
Pericardiocentesis
Left ventricular function
Right ventricular function
Cardiac arrest
Neurosonology
Translation to practice
Conclusion
Key references
Section III: Pediatric Critical Care: Psychosocial and Societal
Chapter 16: Patient- and family-centered care in the pediatric intensive care unit
Pearls
Definition of “family”
Historical evolution of patient- and family-centered care
Fundamental needs of patients and families in the intensive care unit
Core principles of patient- and family-centered care1,12
Honoring differences and respecting each child and family
Maintaining flexibility in practice and procedures to deliver healthcare within the context of the family
Geography of the intensive care unit
Admission process and visiting hours
Creating a “personalized” room
Sibling participation
Parental presence during cardiopulmonary resuscitation and invasive procedures
Transition points and follow-up care, including bereavement
Sharing information using collaborative communication
Elements of high-quality communication
Family-centered rounds
Structured transdisciplinary care conferences
Providing transdisciplinary support for the family unit
Collaborating and building partnerships with patients and families
Empowering patients and families to facilitate shared medical decision-making
Patient- and family-centered care improves outcomes for all stakeholders
Overcoming barriers and challenges to patient- and family-centered care in the intensive care unit
Summary
Key references
Chapter 17: Pediatric critical care ethics
Pearls
Defining bioethics
Examples of ethical issues in the pediatric intensive care unit
Domains of bioethics
Value-based decision-making
State and national laws and legal precedence
Professional codes and healthcare organization policies and regulations
Communication, negotiation, and mediation
Prevailing ethical theories and norms
Who should address ethical issues in the pediatric intensive care unit?
Critical care team
Ethics consultant
Ethics committee
Approach to bioethics dilemmas in the pediatric intensive care unit
Recognition and clarification of ethical issues
Information gathering
Analysis of ethical issues
Consequentialism
Deontology
Principalism
Virtue-based ethics
Casuistic ethics
Care ethics
Narrative ethics
Communitarian ethics
Communication of recommendations
Support
Address staff distress
Support the patient and family
Ethics of patient and surrogate decision-making
Patient decision-making
Doctrine of informed consent
Emergency exception to informed consent
Advance directives
Child and adolescent decision-making
Shared decision-making
Surrogate decision-making
Surrogate decision-making for previously competent patients
Surrogate decision-making for never-competent patients
Parents as surrogate decision-makers
Limits of parental refusals
Limits of parental demands
Other ethical issues in the pediatric intensive care unit
Research ethics
Resource allocation
Ethical issues at the end of life
Limits to clinician refusals
Medical training
Use of unproven medical therapies
Global health
Medical errors
Relationship boundaries
Preventive ethics
Goals for the ethical practice of the intensivist
Key references
Chapter 18: Ethical issues around death and dying
Pearls
Decision-making at the end of life
Requests for potentially inappropriate treatments in the intensive care unit
Withholding and withdrawing of life-sustaining treatments
Administration of analgesics and sedatives in end-of-life care
Is there a role for neuromuscular blockade in end-of-life care?
Artificial hydration and nutrition
Key references
Chapter 19: Palliative care in the pediatric intensive care unit
Pearls
Palliative care consults in the pediatric intensive care unit
Communication
Suboptimal communication in the intensive care unit
Families with limited english proficiency
Family meeting as an intensive care unit “procedure”
Communication pearls
Phrases to avoid
Limitation of interventions
Do not attempt resuscitation orders
Hospice support in the home
Compassionate extubation
Pain and symptom management
Medication management
Opioids
Methadone
Other pharmacologic agents
Symptom management
Pain
Dyspnea
Agitation and anxiety
Nausea and vomiting
Seizures
Bowel obstruction
Palliative sedation
Care of family and staff after a child’s death
Key references
Chapter 20: Organ donation process and management of the organ donor
Pearls
Process of organ donation
Role of the pediatric intensivist and critical care team in the process of organ donation
Determination of neurologic death
Testing for apnea
Ancillary studies
Brain death physiology
Pediatric donor management
Treatment of hemodynamic instability
Hormonal replacement therapy
Management of pulmonary issues for the potential pediatric organ donor
Fluid and electrolyte disturbances
Diabetes insipidus
Oliguria
Coagulation abnormalities
Thermoregulatory instability
Medical examiner and coroner issues and organ donation for children
Donation after circulatory death
Contraindications to organ donation
Evolving areas of transplantation
Summary
Key references
Chapter 21: Long-term outcomes following critical illness in children
Pearls
Post–intensive care syndrome
Health-related quality of life
Assessing change from baseline
Summary of outcomes in general pediatric intensive care unit populations
Hospital readmission and late mortality
Health-related quality of life
Functional status
Neurocognitive status
Mental health
Family functioning
Outcomes for common pediatric intensive care unit illness categories
Respiratory failure
Sepsis
Trauma
Extracorporeal life support
Examples of postdischarge outcomes in pediatric interventional trials
Strategies to assess long-term outcomes
Strategies to improve follow-up
Follow-up programs for intensive care unit survivors
Other initiatives
Potential targets for interventions
Conclusion
Key references
Chapter 22: Burnout and resiliency
Pearls
Burnout and compassion fatigue in pediatric critical care providers
Critical care societies work to address burnout
Strategies for building resilience
Promoting a healthy work environment
Summary
Key references
Section IV: Pediatric Critical Care: Cardiovascular
Chapter 23: Structure and function of the heart
Pearls
Anatomic development and structure
Segmental anatomy
Innervation of the heart
Ductus arteriosus
Development of the human heart
Microscopic anatomy
Cardiomyocyte
Contractile apparatus
Sarcolemma and sarcoplasmic reticulum
Cytoplasm
Cytoskeleton and extracellular matrix
Physiologic development and function
Myocardial mechanics: Cardiac sarcomere function
Excitation-contraction coupling
Sarcomere length-tension relationships
Myocardial mechanics: Myocardial receptors and responses to drugs
Myocardial mechanics: Integrated muscle function
Relationship between muscle strips and intact ventricles
Pressure-volume loops
Assessing myocardial contractility: Systolic ventricular function
Isovolumic phase indices
Ejection phase indices
Assessing myocardial relaxation: Diastolic ventricular function
Pericardial function
Ventricular interactions
Neural control of the heart
Cardiac output
Myocardial metabolism: Normal myocardial energy metabolism
Basic metabolic processes
Determinants of myocardial oxygen consumption
Myocardial oxygen demand-supply relationship
Effects of myocardial ischemia on cardiac function and metabolism
Systemic vasculature
General anatomy
Physiologic mechanisms
General features
Control of vascular tone
Autoregulation
Key references
Chapter 24: Regional peripheral circulation
Pearls
General features
General anatomy
Basic physiology
Venous return and cardiac output
Critical closing pressure
Autoregulation
Distensibility and compliance
Vascular resistance
Vascular impedance
Local regulatory mechanisms
Innervation and neural processes
Circulating endocrine and neuroendocrine mediators
Local metabolic products
Blood gas composition
Endothelial-derived factors
Myogenic processes
Regional circulations
Pulmonary circulation
Normal fetal circulation
Changes in the pulmonary circulation at birth
Regulation of postnatal pulmonary vascular resistance
Cerebral circulation
Coronary circulation
Myocardial oxygen demand-supply relationship
Gastrointestinal circulation
Renal circulation
Conflicting needs of regional circulations
Key references
References
Chapter 25: Endothelium and endotheliopathy
Pearls
Normal endothelial function
Endothelial cell heterogeneity
Endothelial progenitor cells
Coagulation and fibrinolysis
Anticoagulant mechanisms
Procoagulant mechanisms
Endothelium-derived vasodilators
Nitric oxide
Prostacyclin
Endothelium-derived hyperpolarizing factor
Endothelium-derived vasoconstrictors
Endothelins (endothelium-derived contracting factors)
Reactive oxygen species
Vasoconstrictor prostaglandins
Endothelium and blood cell interactions
Interactions of leukocytes with the vessel wall
Platelet adhesion
Endothelial permeability
Endothelial cell dysfunction
Ischemia-reperfusion injury
Sepsis
Hemolytic-uremic syndrome
Vasculitic disorders
Biomarkers of endothelial activation
Conclusions
Key references
Chapter 26: Principles of invasive cardiovascular monitoring
Pearls
Role of invasive hemodynamic monitoring
Indications for invasive hemodynamic measurements
Principles of measurement
Signal analysis
Measurement systems
Errors in measurement
Calibration
Frequency response
Impedance
Invasive techniques
Central venous catheters
Indications
Interpretation of waveforms
Mixed venous oxygen saturation
Arterial pressure catheters
Indications
Interpretation of waveforms
Pulmonary artery catheters
History and controversy
Indications
Monitoring techniques with the pulmonary artery catheter
Catheter placement
Indirectly measured variables
Measurement of cardiac output
Fick method
Thermodilution method
Calculation of oxygen delivery and consumption
Interpretation of waveforms
Resistance
Calculation of intracardiac shunt
Novel monitoring strategies
Conclusions
Key references
Chapter 27: Assessment of cardiovascular function
Pearls
Cardiovascular function
Quantity of therapy
Variables that determine tissue oxygenation
Monitoring tissue oxygenation
Qualitative assessment of cardiac output
Physical examination
Chest radiography
Quantitative assessment of cardiac output
Thermodilution technique
Fick method
Doppler echocardiography
Pulse oximetry
Other measures of oxygen delivery
Acid-base status
Blood lactate
Serum biomarkers
Gastric tonometry
Urine output
Near-infrared spectroscopy
Systemic arterial blood pressure
Invasive blood pressure monitoring
Noninvasive blood pressure monitoring
Central venous or intracardiac pressure monitoring
High-frequency physiologic data capture and streaming analytics
Assessing variables that affect the quantity of therapy
Ventricular systolic function
Ventricular diastolic function
Rhythm disturbance
Abnormal systemic vascular resistance
Increased pulmonary vascular resistance
Inefficient circulation
Vascular integrity
Pulmonary function
Physiology of the patient with a single ventricle
Key references
Chapter 28: Cardiac failure and ventricular assist devices
Pearls
Pediatric heart failure
Low cardiac output syndrome
Definitions
Assessment
Specific treatments to improve cardiac function
Broad treatment strategies
Mechanical circulatory support in pediatric patients
Extracorporeal life support
Extracorporeal membrane oxygenation indications and contraindications
Myocarditis and extracorporeal life support
Postcardiopulmonary bypass
Extracorporeal cardiopulmonary resuscitation
Bridge to transplantation
Malignant dysrhythmias
Contraindications
Critical care management during extracorporeal life support
Cardiac output
Troubleshooting
Hypovolemia
Hypertension
Cardiac stun
Echocardiography and cardiac catheterization
Single ventricle
Anticoagulation strategies
Ventilation strategies
Fluid, nutrition, and renal
Analgesia and sedation
Infection
Intrahospital transport
Ventricular assist devices
Pulsatile ventricular assist devices
Continuous-flow ventricular assist devices
Short- to medium-term ventricular assist device support
Long-term ventricular assist device support
Total artificial heart
Device selection
Indications and management
Basic management of ventricular assist device patients
Anticoagulation and antiplatelets
Antibiotic prophylaxis
Care of the drive line and cannulas
Ventricular assist device—congenital heart disease and single-ventricle physiology
Outcomes
Intraaortic balloon pump and impella device
Current perspective and future directions
Conclusions
Key references
Chapter 29: Echocardiographic imaging
Pearls
Components of the examination
Modalities
Transthoracic echocardiography
Transesophageal echocardiography
Fetal echocardiography
Intracardiac echocardiography
Cross-sectional imaging
Point-of-care ultrasound
Structural congenital heart disease and intracardiac shunting
Valve anatomy and function
Ventricular function
Assessing volume status
Assessing diastolic function
Assessing systolic function
Left ventricular systolic function
Right ventricular systolic function
Systolic function: Strain
Pulmonary hypertension
Pericardial effusion
Intracardiac vegetations and thrombi
Kawasaki disease and coronary artery anomalies
Intravascular catheters
Balloon atrial septostomy
Extracorporeal membrane oxygenation and ventricular assist devices
Key references
Chapter 30: Diagnostic and therapeutic cardiac catheterization
Pearls
Catheterization laboratory environment
Diagnostic cardiac catheterization
Oxygen saturation
Pressure assessment
Flow calculations
Estimating shunts
Resistance calculations
Angiography
Indications
Pulmonary vascular resistance and vasoreactivity testing
Therapeutic cardiac catheterization
Pericardiocentesis
Atrial septostomy
Pulmonary balloon valvuloplasty
Balloon aortic valvuloplasty
Balloon dilation of pulmonary arteries
Occlusion device insertion
Native and recurrent coarctation of the aorta
Ductal stenting
Hybrid stage I palliation for hypoplastic left heart syndrome
Transcatheter pulmonary valve replacement
Risks and complications
Preprocedural risk stratification
Cardiac catheterization and extracorporeal membrane oxygenation
Conclusion
Key references
Chapter 31: Pharmacology of the cardiovascular system
Pearls
Mechanisms of response
Adrenergic receptors
Signal transduction
β-adrenergic receptors
α-receptors
Receptor downregulation
Polymorphisms
Vasopressin receptors
V1 receptors
Phosphodiesterase regulation of cyclic adenosine monophosphate
Atpase inhibition
Developmental issues
Sympathomimetic amines
Dopamine
Basic pharmacology
Clinical pharmacology
Pharmacokinetics
Clinical role
Adverse effects
Preparation and administration
Interactions
Summary
Norepinephrine
Basic pharmacology
Clinical pharmacology
Pharmacokinetics
Clinical role
Adverse effects
Preparation and administration
Interactions
Summary
Epinephrine
Basic pharmacology
Clinical pharmacology
Pharmacokinetics
Clinical role
Preparation and administration
Adverse effects
Interactions
Summary
Isoproterenol
Basic pharmacology
Clinical pharmacology
Pharmacokinetics
Clinical role
Preparation and administration
Adverse effects
Interactions
Summary
Dobutamine
Basic pharmacology
Clinical pharmacology
Pharmacokinetics
Clinical role
Preparation and administration
Adverse effects
Interactions
Summary
Vasopressin
Basic pharmacology
Clinical pharmacology
Pharmacokinetics
Clinical role
Dosing and administration
Adverse effects
Interactions
Summary
Bipyridines
Milrinone
Clinical pharmacology
Pharmacokinetics
Clinical role
Preparation and administration
Adverse effects
Summary
Digitalis glycosides
Basic pharmacology
Clinical pharmacology
Pharmacokinetics
Clinical role
Preparation and administration
Adverse effects
Interactions
Summary
Conclusion
Key references
Chapter 32: Cardiopulmonary interactions
Pearls
Effects of ventilation on circulation
Right ventricular filling and stroke volume
Systemic venous return
Right ventricular preload and stroke volume
Positive pressure ventilation and right ventricular preload
Critical illness and the effects of positive pressure breathing on rv preload
Respiration and right ventricular afterload
Lung volume
Alveolar pressure
Regulation of pulmonary vascular resistance
Direct effects of airway pressure on pulmonary vascular tone
Respiration and left ventricular preload
Ventricular interdependence
Respiration and left ventricular afterload
Cardiac contractility
Preload dependence versus afterload dependence
Fluid responsiveness during positive pressure ventilation
Pulsus paradoxus in respiratory distress
Positive-pressure ventilation and right ventricular output in acute respiratory distress syndrome
Effects of cardiovascular function on respiration
Shock states and respiratory function
Elevated work of breathing and the circulation
Congestive heart failure/critical heart failure and shock
Cardiomyopathies and congenital heart disease
Glenn and fontan procedures
Key references
Chapter 33: Disorders of cardiac rhythm
Pearls
Classification of arrhythmias
Bradycardias
Appropriate versus normal heart rate
Sinus bradycardia and sinus pauses
Atrioventricular block
Escape rhythms and accelerated rhythms
Tachycardias
Tachycardia mechanisms
Supraventricular tachycardias
Atrioventricular reciprocating tachycardias (AV reentry)
Atrioventricular nodal reentrant tachycardia
Primary atrial tachycardias
Junctional ectopic tachycardia
Ventricular tachycardias
Approach to diagnosis
Monitoring and general assessment
Surface electrocardiogram and bedside monitoring
Bradycardias
Extrasystoles
Tachycardias with normal QRS
Tachycardias with prolonged QRS
Assessment of atrial activation
Diagnostic uses of adenosine
Treatment of rhythm disturbances
Bradycardia therapies
Pharmacologic treatment of bradycardias
Temporary and permanent pacing for bradycardias
Principles of pacing
Temporary pacing
Setting temporary pacing parameters
Permanent pacing—indications and selection
Other indications for pacing
Tachycardia therapies
Vagal maneuvers
Acute pharmacologic therapies
Adenosine
Antiarrhythmic agents
Procainamide. 
Lidocaine. 
β-blocking agents. 
Amiodarone. 
Other QT prolonging (class III) antiarrhythmic drugs
Intravenous sotalol. 
Ibutilide. 
Calcium channel–blocking agents
Magnesium sulfate
Digoxin
Dexmedetomidine
Ivabradine
Cardioversion and defibrillation
Approach to therapy
Extrasystoles
Sustained tachycardias
Unstable patients
Treatment failure
Errors in diagnosis
Unrecognized termination and reinitiation
Improper technique
Specific arrhythmias
Primary arrhythmias
Orthodromic reciprocating tachycardia in infancy
Tachycardia-induced cardiac dysfunction
Chaotic atrial tachycardia
Long QT syndromes
Idiopathic ventricular tachycardias in healthy patients
Bidirectional polymorphic ventricular tachycardias
Secondary rhythm disturbances
Postoperative arrhythmias
Postsurgical atrioventricular block
Junctional ectopic tachycardia
Late postoperative arrhythmias
Metabolic derangements
Electrolyte disturbances
Endocrine disorders (thyroid)
Central nervous system injury
Hypothermia and hyperthermia
Acute myocardial infarction
Arrhythmias resulting from drug toxicity
Digoxin
Cocaine
Tricyclic antidepressants and phenothiazine
Infections
Key references
Chapter 34: Shock states
Definition and physiology
Functional classification and common underlying etiologies
Hypovolemic shock
Cardiogenic shock or congestive heart failure
Obstructive shock
Distributive shock
Septic shock
Endocrine
Mitochondrial
Multisystem effects of shock
Respiratory
Renal
Coagulation
Hepatic
Gastrointestinal
Endocrine
Monitoring
Fick equation for oxygen consumption
Contemporary cardiac output monitoring in pediatric shock
Treatment
General principles
Intubation and mechanical ventilation
Fluid resuscitation
Vasoactive infusions
Age-related therapy concerns
Specific shock state therapy considerations
Hypovolemic shock
Cardiogenic shock
Obstructive shock
Distributive shock
Septic shock
Other therapies
Summary
Key references
Chapter 35: Pediatric cardiopulmonary bypass
Pearls
Background
History
Surgical team
Equipment and preparation for cardiopulmonary bypass
Heart-lung machine console and pumps
Cardiopulmonary bypass circuit
Oxygenators
Tubing
Hemoconcentrators
Circuit prime
Anticoagulation
Cannulation
Cardiopulmonary bypass
Pediatric VS. adult considerations
Initiation of cardiopulmonary bypass
Determining and monitoring effective perfusion flow rate
Arterial pressure
Arterial and venous oxygen saturation
Near-infrared spectroscopy
Methods to optimize physiologic management
Target hematocrit and ultrafiltration
Hypothermia
Deep hypothermic circulatory arrest VS antegrade cerebral perfusion
Ph and partial pressure of arterial carbon dioxide strategy
Myocardial protection
Inflammatory response to cardiopulmonary bypass
Termination of cardiopulmonary bypass
Key references
Chapter 36: Critical care after surgery for congenital cardiac disease
Pearls
Neonatal considerations
Preoperative care
Severe hypoxemia
Excessive pulmonary blood flow
Obstruction of left heart outflow
Ventricular dysfunction
Postoperative care
Assessment
Monitoring
Low cardiac output syndrome
Volume adjustments
Preserving and creating right-to-left shunts
Other strategies
Mechanical cardiac support
Right ventriculotomy and restrictive physiology
Diastolic dysfunction
Pharmacologic support
Managing acute pulmonary hypertension in the intensive care unit
Pulmonary vasodilators
Management of postoperative bleeding
Cardiac tamponade
Diaphragmatic dysfunction, effusions, and pulmonary issues
Chylothorax
Separating from mechanical ventilation
Central nervous system
Renal function and postoperative fluid management
Gastrointestinal issues
Infection
Hyperglycemia
Critical care management of selected specific lesions
Single-ventricle anatomy and physiology
Neonatal preoperative management
Postoperative management
Bidirectional cavopulmonary anastomosis
Fontan procedure
Complications after the fontan procedure
Pleuropericardial effusions
Rhythm disturbances
Premature closure of the fenestration
Persistent hypoxemia
Low cardiac output state
Patent ductus arteriosus
Pathophysiology
Critical care management
Atrial septal defect
Pathophysiology
Critical care management
Ventricular septal defect
Pathophysiology
Critical care management
Critical care management for late postoperative care
Atrioventricular canal defects
Pathophysiology
Critical care management
Truncus arteriosus communis
Pathophysiology
Critical care management
Critical care management for late postoperative care
Total anomalous pulmonary venous connection
Pathophysiology
Critical care management
Critical care management for late postoperative care
Transposition of the great arteries
Pathophysiology
Atrial switch procedure (mustard and senning)
Arterial switch operation (jatene procedure)
Ventricular switch (rastelli procedure)
Critical care management
Late complications
Tetralogy of fallot
Pathophysiology
Critical care management for the early postoperative course
Critical care management for late postoperative care
Pulmonary atresia
Pathophysiology
Critical care management
Critical care management for late postoperative care
Tricuspid atresia
Pathophysiology
Critical care management
Left-sided obstructive lesions
Pathophysiology
Aortic stenosis
Coarctation of the aorta
Interrupted aortic arch
Hypoplastic left heart syndrome
Pathophysiology
Critical care management
Postoperative management
Evolution of treatment strategies. 
Specific considerations for the norwood operation. 
Hybrid approach. 
Summary
Acknowledgment
Key references
Chapter 37: Cardiac transplantation
Pearls
Background
Indications for transplant
Transplant evaluation
Panel reactive antibody
Transplant listing
Management of the potential heart transplant recipient
Anticoagulation
Abo-incompatible listing and transplantation
Critical care management of the orthotopic heart transplant recipient
Intraoperative considerations
Early perioperative management
Management of early heart allograft dysfunction
Immunosuppression and heart allograft rejection
Complications of immunosuppression in heart transplant recipients that occur in the pediatric intensive care unit
Infection
Renal function
Diabetes mellitus
Future management strategies for critical care of infants and children with cardiopulmonary failure
Key references
Chapter 38: Physiologic foundations of cardiopulmonary resuscitation
Pearls
Mechanisms of blood flow
Cardiac versus thoracic pump mechanism
Rate and duty cycle
Chest geometry
Effects of cardiopulmonary resuscitation on intracranial pressure
Newer cardiopulmonary resuscitation techniques
Abdominal binding
Open-chest cardiopulmonary resuscitation
Cardiopulmonary bypass and extracorporeal cardiopulmonary resuscitation
Transcutaneous cardiac pacing
Pharmacology
Adrenergic agonists
Vasopressin
High-dose epinephrine
Atropine
Sodium bicarbonate
Other alkalinizing agents
Calcium
Glucose
Management of ventricular fibrillation
Defibrillation
Antiarrhythmics
Postresuscitation care
Future directions
Key references
Chapter 39: Performance of cardiopulmonary resuscitation in infants and children
Pearls
Four phases of cardiac arrest
Prearrest
No-flow/low-flow
Postarrest
Epidemiology of pediatric cardiac arrest
Optimizing blood flow during cardiopulmonary resuscitation
Pediatric cardiopulmonary resuscitation targets
Chest compression depth
Chest compression rate
Chest compression fraction/minimizing interruptions
Duty cycle
Airway and breathing management during cardiopulmonary resuscitation
Physiologic targets
Arterial blood pressure
End-tidal carbon dioxide
Medications used to treat cardiac arrest
Vasopressors
Epinephrine
Vasopressin
Antiarrhythmics
Amiodarone versus lidocaine
Other medications
Calcium
Sodium bicarbonate
Postarrest interventions
Targeted temperature management
Anticipation and prevention of hypotension
Postarrest oxygenation and ventilation management
Monitoring for and treating seizures
Other considerations
Contemporary methods to improve cardiopulmonary resuscitation quality
Intra-arrest cardiopulmonary resuscitation quality monitoring technology
Point-of-care bedside training
Extracorporeal cardiopulmonary resuscitation
Controversies in pediatric cardiac arrest management
Airway management
Ventilation during pediatric cardiopulmonary resuscitation
Ventricular fibrillation and pulseless ventricular tachycardia
Pediatric automated external defibrillators
Summary
Key references
Section V: Pediatric Critical Care: Pulmonary
Chapter 40: Structure and development of the upper respiratory system
Pearls
Developmental anatomy of the upper airway
Anatomy and physiology of the upper airway
Nasal passages
Mouth (oral cavity) and pharynx
Larynx
Trachea and bronchi
Acknowledgment
Key references
Chapter 41: Structure and development of the lower respiratory system
Pearls
Lower respiratory system
Overview of the lungs
Airways
Definitions of special lung unit and alveolar formation
Alveolar-capillary unit
Lung circulation
Pulmonary vascular system
Bronchial vascular system
Pulmonary lymphatics and bronchus-associated lymphoid tissue
Diaphragm
Summary
Key references
Chapter 42: Physiology of the respiratory system
Pearls
Physiology of the respiratory system
Boyle’s law
Equation of motion of the respiratory system
Transmural pressures
Central nervous system control
Respiratory centers and efferent nerve transmission to the respiratory muscles
Receptors and feedback to the respiratory centers
Higher states
Extrathoracic upper airways
Anatomic considerations for the pediatric upper airway
Ohm’s law and poiseuille’s law
Turbulent versus laminar flow (reynolds number)
Transmural pressure (collapse of upper airway)
Inspiratory flow limitation
Intrathoracic compartment
Flow resistance of the respiratory system
Anatomic considerations for pediatric lower airways
Reactivity of lower airways
Transmural pressure in the presence of lower airway obstruction
Identifying expiratory flow limitation
Administration of positive end expiratory pressure to prevent collapse
Time constants
Elastic properties of the respiratory system
Respiratory system and lung compliance
End-expiratory lung volume and compliance
Functional residual capacity, closing capacity, and age
Work of breathing
Extrathoracic space
Gas exchange
Ventilation-perfusion relationships
Hypoxemia
Hypercarbia
Diffusion of oxygen and carbon dioxide
Key references
Chapter 43: Noninvasive respiratory monitoring and assessment of gas exchange
Pearls
Pulse oximetry
Principles of pulse oximetry
Validation
Sources of error
Probe placement
Tissue oximetry
Near-infrared spectroscopy
Capnometry and capnography
Operating principles of capnometry
Clinical and technical issues
Gas sampling issues
Physiologic basis
Dead space ventilation
Differential diagnosis of abnormal capnograms
Gradually decreasing end-tidal carbon dioxide concentration
Sustained low end-tidal carbon dioxide concentrations without plateaus
Sustained low end-tidal carbon dioxide concentration with good plateaus
Exponential decrease in end-tidal carbon dioxide
Gradual increase in both baseline and end-tidal carbon dioxide
Clinical applications
Transcutaneous monitoring
Oxygen monitoring
Carbon dioxide monitoring
Conclusion
Key references
Chapter 44: Overview of breathing failure
Pearls
Physiology of breathing
Controls of breathing
Respiratory pump
Diaphragm
Intercostal muscles
Accessory muscles of respiration
Breathing failure
Failure of respiratory controls
Recognizing depressed respiratory drive
Failure of neural control
Structural brain lesions
Nonstructural acquired disorders
Chronic respiratory failure
Structural brain lesions
Nonstructural congenital disorders
Nonstructural acquired chronic disorders
Respiratory pump failure
Respiratory plegia, paralysis, and tetany
Muscle exhaustion
Failure of mechanics of breathing
Terminal failure of respiratory control and mechanics
Restrictive versus obstructive respiratory disease
Compensatory mechanisms in breathing failure
Compensatory mechanisms in restrictive lung disease
Compensatory mechanisms with obstructive lung disease
Special conditions
Infancy
Sleep
Thoracic dysfunction
Altered nutritional states: Malnutrition and obesity
Conclusion
Acknowledgments
Key references
Chapter 45: Ventilation/perfusion inequality
Pearls
Distribution of ventilation
Distribution of perfusion
Fractal model of pulmonary blood flow and ventilation
Va/q abnormalities in pulmonary disease
Hypoxemia
Pediatric acute respiratory distress syndrome
Pneumonia
Pleural effusion
Asthma
Pulmonary embolism
Primary pulmonary hypertension
Therapeutic considerations
Positive end-expiratory pressure
Prone positioning
Nitric oxide
Measuring pulmonary ventilation/perfusion
Key references
Chapter 46: Mechanical dysfunction of the respiratory system
Pearls
Pump dysfunction and failure
Pressures
Lung volumes and capacities
Flow/volume relationships
Equation of motion
Resistive forces
Static and dynamic compliance
Airway dynamics
Time constant
Work of breathing
Clinical manifestations of mechanical dysfunction
Effectors of the respiratory pump
Accessory muscles of respiration
Chest wall
Disease of the chest wall
Therapeutic maneuvers to improve mechanical dysfunction of the respiratory system
Positive end-expiratory pressure/continuous positive airway pressure
Rate
Tidal volume
Inspiratory and expiratory times
Inspiratory flow patterns
Key references
Chapter 47: Diseases of the upper respiratory tract
Pearls
Anatomy and physiology
Diagnosis and management
Congenital malformations of the upper airway
Choanal atresia
Laryngomalacia
Laryngeal webs and atresia
Laryngeal cysts
Laryngeal clefts
Vocal cord paralysis
Vascular compression of the trachea
Neoplasms of upper airway disease
Laryngeal papillomatosis
Hemangiomas
Other tumors of the larynx and upper airway
Croup
Epiglottitis
Bacterial tracheitis
Peritonsillar abscess
Retropharyngeal abscess
Iatrogenic and acquired disorders of the upper airway
Laryngotracheal (subglottic) stenosis
Pediatric tracheostomy
Foreign body aspiration
Burn injury to the upper airway
Airway trauma
Key references
Chapter 48: Pediatric acute respiratory distress syndrome and ventilator-associated lung injury
Key words:
Pearls
Clinical features: Pathophysiology
Definition
Epidemiology
Pathobiology
Exudative phase
Direct injury
Indirect injury
Alveolar fluid clearance
Leukocytes and inflammation
Surfactant
Coagulation
Apoptosis
Fibrosis and repair phase
Ventilator management
Considerations for ventilator mode
Tidal volume
Peak or plateau pressure
Positive end-expiratory pressure
Nonconventional ventilation strategies
Ventilator-associated lung injury
Pulmonary ancillary therapies
Exogenous surfactant
Preclinical or adult data
Pediatric data
Nitric oxide
Physiologic rationale
Preclinical and adult studies
Pediatric
Nonpulmonary therapies
Prone positioning
Physiologic rationale
Adult data
Pediatric data
Monitoring
Noninvasive support
Extracorporeal life support
Key references
Chapter 49: Acute viral bronchiolitis
Key words:
Pearls
Microbiology
Epidemiology and risk factors
Pathophysiology
Clinical features and diagnosis
Prevention
Treatment
Hypertonic saline
Inhaled bronchodilators
Corticosteroids
Hydration and nutritional support
Other inhaled therapies
Other systemic therapies
Respiratory support
High-flow nasal cannula
Continuous positive airway pressure
Invasive mechanical ventilation
Other respiratory support
Complications
Key references
Chapter 50: Asthma
Pearls
Epidemiology and risk factors
Pathophysiology
Clinical assessment
History
Physical examination
Radiography
Laboratory data
Arterial blood gas analysis
Electrolytes and complete blood cell count
Muscle enzymes
Electrocardiography
Spirometry
Treatment
Initial management in the emergency department
Admission criteria
Management in the intensive care unit
General
Oxygen
Fluids
Corticosteroids
β-agonists
Albuterol
Terbutaline
Anticholinergic agents
Magnesium sulfate
Methylxanthine agents
Helium-oxygen mixtures
Ketamine
Mechanical ventilation
Indications
Noninvasive ventilation
Intubation
Ventilator settings
Ventilatory monitoring
Analgesia, sedation, and muscle relaxation
Inhalational anesthetic agents
Antibiotics
Bronchoscopy
Extracorporeal life support
Prognosis
Key references
Chapter 51: Neonatal pulmonary disease
Pearls
Acute respiratory disorders
Transient tachypnea of the newborn
Surfactant-deficient respiratory distress syndrome
Pulmonary air leak syndromes
Pulmonary hemorrhage
Pneumonia
Meconium aspiration syndrome
Immature respiratory control
Biological maturational considerations
Apnea of prematurity
Therapeutic options
Pulmonary malformations
Pulmonary hypoplasia
Congenital diaphragmatic hernia
Congenital pulmonary airway malformation
Bronchogenic cysts
Pulmonary parenchymal cysts
Pulmonary sequestrations
Congenital lobar emphysema
Pulmonary agenesis and aplasia
Congenital defects of the lymphatics
Nonpulmonary causes of respiratory distress
Choanal atresia/stenosis
Laryngomalacia
Vocal cord paralysis
Airway vascular tumors/malformations
Tracheobronchomalacia
Tracheoesophageal fistula
Vascular compression
Phrenic nerve paralysis
Eventration of the diaphragm
Pleural effusion
Congenital anomalies of the chest wall
Persistent pulmonary hypertension of the neonate
Hyperviscosity syndrome
Congenital heart disease
Metabolic disorders
Intestinal or renal bicarbonate wasting
Neonatal chronic lung disease of prematurity: Bronchopulmonary dysplasia
Neonatal chronic lung disease: Primary ciliary dyskinesia
Acknowledgment
Key references
Chapter 52: Pneumonitis and interstitial disease
Pearls
Pathogenesis
Pathophysiology
Diagnosis
Laboratory diagnosis
Bacterial pneumonitis
Definition
Pathophysiology
Clinical features
Radiographic features
Diagnosis
Specific pathogens
Group b streptococci
Streptococcus pneumoniae
Haemophilus influenzae
Staphylococcal pneumonia
Mycoplasma pneumonia
Gram-negative bacteria
Legionella pneumophila
Anaerobic bacteria
Complications
Therapy
Viral pneumonitis
Pathophysiology
Diagnosis
Radiographic findings
Specific pathogens
Respiratory syncytial virus
Parainfluenza virus
Adenovirus
Influenza
Measles
Human immunodeficiency virus
Complications
Diagnosis
Prevention and treatment
Vaccination
Chemoprophylaxis
Therapy
Fungal pneumonitis
Primary pulmonary fungi
Pathogenesis
Histoplasmosis
Diagnosis
Complications
Treatment
Blastomycosis
Diagnosis
Complications
Treatment
Coccidioidomycosis
Diagnosis
Complications
Treatment
Opportunistic pulmonary mycoses
Pulmonary aspergillosis
Diagnosis
Complications
Treatment
Pulmonary candidiasis
Pathogenesis
Diagnosis
Complications
Treatment
Pneumocystis jirovecii pneumonia
Clinical features
Diagnosis
Complications
Treatment
Chemical pneumonitis
Aspiration pneumonia
Pathophysiology
Clinical findings
Treatment
Inhalation injury
Pathogenesis
Clinical findings
Treatment
Prognosis
Ingestion/injection of pharmacologic agents
Diagnosis/clinical findings
Idiopathic interstitial lung disease
Pediatric pulmonary hemorrhage
Definition
Pathophysiology
Etiology
Diffuse/nonimmune pulmonary hemorrhage
Diffuse/immune pulmonary hemorrhage
Focal pulmonary hemorrhage
Treatment
General
Specific
Summary
Key references
Chapter 53: Diseases of the pulmonary circulation
Pearls
Definition and classification
Pathology of pulmonary hypertension
Diagnostic evaluation of pulmonary hypertension/pulmonary vascular disease
Pharmacotherapy
Prostacyclin (pgi2) analogs
Endothelin receptor antagonists
Phosphodiesterase 5 inhibitors
Calcium channel blockers
Pulmonary hypertension in the context of specific diseases
Group 1 pulmonary arterial hypertension
Isolated pulmonary arterial hypertension
Autoimmune disorders
Infectious diseases
Group 2 pulmonary arterial hypertension (cardiac disease)
Group 3 pulmonary arterial hypertension (intrinsic pulmonary diseases)
Pulmonary arterial hypertension due to parenchymal lung disease
Pulmonary hypertension secondary to bronchopulmonary dysplasia
Diagnostic ph evaluation in BPD
Bpd-specific therapies
Congenital diaphragmatic hernia
General management of cdh
Ph-specific therapy in cdh
Cystic fibrosis
Diffuse lung disease
Pulmonary hypertension secondary to acute respiratory distress syndrome
Group 5 pulmonary hypertension
Chronic hemolytic anemia
Acute pulmonary hypertension crisis/right ventricular failure
Right ventricular dysfunction in pulmonary hypertension
Adaptive versus maladaptive right ventricular hypertrophy
Sympathetic activation in pulmonary arterial hypertension
Right ventricle ischemia
Metabolism in right ventricular hypertrophy
Right ventricle inflammation and fibrosis
Recent advances in molecular mechanisms and novel therapeutic targets in pulmonary hypertension
Genetic predisposition
Growth factors and apoptosis resistance
Inflammation
Noncalcium ion channels
Transcription factors and transcriptional coregulators
Metabolic reprogramming and mitochondrial dysfunction
Epigenetic modifications
Long-term outcomes of patients with pulmonary vascular disease
Future directions and perspectives
Key references
Chapter 54: Mechanical ventilation and respiratory care
Pearls
Applied respiratory physiology
Lung volumes and capacities
Lung inflation and deflation
Time constant
Work of breathing
Determinants of gas exchange
Indications for mechanical ventilation
Respiratory failure
Other indications for mechanical ventilation
Design and functional characteristics of ventilators
Phases of a breath
Initiating breaths
Patterns of gas flow
Limit
Cycling of breaths
Continuous positive airway pressure and positive end-expiratory pressure
Ventilator modes and phase variables
Selection of parameters for mandatory breaths
Dual-control modes
Automatic tube compensation
Nontraditional modes of ventilation
Airway pressure release ventilation
Proportional assist ventilation
Neurally adjusted ventilatory assist
Ventilation for selected underlying pathophysiology
Primary respiratory muscle failure (“respiratory pump failure”)
Obstructive lung diseases
Restrictive lung diseases
Unilateral lung disease or severely differential lung disease
Heart failure and postoperative management of congenital heart disease
Diseases with abdominal distention
Neurologic and neuromuscular diseases
Patient-ventilator asynchrony
Asynchrony associated with breath triggering
Asynchrony during inspiration (flow asynchrony)
Asynchrony during cycling
Use of neuromuscular blockade
High-frequency ventilation
Mechanism of gas flow in high-frequency ventilation in the normal lung
High-frequency jet ventilation
High-frequency oscillatory ventilation
High-frequency percussive ventilation
Adverse effects of mechanical ventilation
Airway injury
Effects on the lung
Effects on the circulatory system
Specialty gases
Inhaled nitric oxide
Helium-oxygen mixture
Altering pulmonary vascular resistance with adjusted inspired oxygen and carbon dioxide concentrations
Respiratory care during mechanical ventilation
Pulmonary hygiene
Humidification systems
Aerosol therapy
Weaning from mechanical ventilation and extubation
Extubation readiness trial
Extubation
Key references
Chapter 55: Noninvasive ventilation in the pediatric intensive care unit
Pearls
Epidemiology
Physiology and application of noninvasive ventilation
High-flow nasal cannula
Noninvasive positive-pressure ventilation
Negative-pressure ventilation
Negative-pressure ventilation use in chest physiotherapy and secretion clearance
Vibration mode
Cough mode
Neurally adjusted ventilatory assist
Patient selection
Bronchiolitis
Asthma
Pediatric acute respiratory distress syndrome
Immunocompromised patients
Neuromuscular disease
Patient monitoring and complications
Sedation during noninvasive ventilation
Failure of noninvasive ventilation
Long-term use of noninvasive ventilation
Key references
Chapter 56: Extracorporeal life support
Pearls
A trip around the extracorporeal membrane oxygenation circuit
Cannulation techniques
Venoarterial extracorporeal membrane oxygenation (fig. 56.1a)
Venous cannulation
Arterial access
Left heart decompression
Venovenous extracorporeal membrane oxygenation
Venoarteriovenous extracorporeal membrane oxygenation
Arteriovenous
Percutaneous cannulation
Extracorporeal membrane oxygenation circuit
Venous reservoir and venous saturation monitor
Priming
Types of pumps and oxygenators
Roller head pumps
Centrifugal pumps
Negative inlet pressure
Membrane lung
Hollow-fiber membrane lung
Patient populations treated with extracorporeal life support
Neonatal cardiopulmonary failure
Pediatric patients
Trauma patients
Extracorporeal membrane oxygenation and high-risk diseases
Poisonings
Bridge to lung transplant
Why use extracorporeal membrane oxygenation?
Extracorporeal membrane oxygenation for cardiac dysfunction
Extracorporeal membrane oxygenation for resuscitation
Extracorporeal membrane oxygenation and septic shock
Patient selection criteria
Physiology of extracorporeal life support: Gas exchange and oxygen delivery
Oxygenation
Carbon dioxide exchange
Oxygen delivery
Recirculation
Transport
Patient management on extracorporeal membrane oxygenation
Initiation of extracorporeal life support
Patient management during extracorporeal life support
General principles
Anticoagulation monitoring
Nutritional support
Fluid and renal replacement therapy
Ventilator management
Sedation and analgesia
Weaning from extracorporeal membrane oxygenation
Decannulation
When to stop extracorporeal membrane oxygenation support
Complications from extracorporeal membrane oxygenation
Bleeding
Infection
Vascular injury
Other management tidbits
Right ventricular failure
Outcomes from extracorporeal life support
Medical outcomes
Neurodevelopmental outcomes
Long-term neurologic implications
Future of extracorporeal membrane oxygenation
Key references
Chapter 57: Pediatric lung transplantation
Pearls
Indications
Contraindications
Survival and outcomes
Evaluation of the donor
Surgical approach
Presurgical management in the intensive care unit
Postsurgical management
Immunosuppression
Immunobiology
Rejection
Antibody-mediated rejection
Chronic lung allograft dysfunction
Treatment options for clad
Summary
Key references
Section VI: Pediatric Critical Care: Neurologic
Chapter 58: Structure, function, and development of the nervous system
Pearls
Major cell types
Intercellular communication in the nervous system
Electrical synapses
Chemical synapses
Neuromuscular junction
Chemical synapses in the central nervous system
Neurotransmitter systems
Neurotransmitters
Acetylcholine
Catecholamines
Serotonin
Amino acids
Adenosine, peptides, and nitric oxide
Neurotransmitter receptors
Nicotinic acetylcholine receptors
Muscarinic acetylcholine receptors
Glutamate receptors
Gabaa and gabab receptors
Major anatomic organization of the nervous system
Central nervous system
Spinal cord
Medulla
Pons
Midbrain
Reticular formation
Cerebellum
Diencephalon
Basal ganglia
Cerebral hemispheres
Peripheral nervous system
Somatic peripheral nervous system
Visceral or autonomic peripheral nervous system
Sympathetic nervous system
Parasympathetic autonomic nervous system
Meninges
Blood-brain barrier
Anatomy
Selectivity
Blood-brain barrier–deficient areas
Ventricles and cerebrospinal fluid
Ventricular system
Cerebrospinal fluid production and flow
Cerebrospinal fluid composition and function
Vasculature in the central nervous system
Brain vasculature
Spinal cord vasculature
Regulation of cerebral blood flow
Perfusion pressure–related autoregulation
Oxygen-related autoregulation
Hydrogen ion–related autoregulation
Metabolic coupling
Emerging characterization of the “lymphatic” circulation in the central nervous system
Developmental processes relevant to pediatric critical care medicine
Cell origin and differentiation
Synaptogenesis and synaptic pruning
Neurotransmitter system maturation
Myelination
Development of cerebrovasculature and blood-brain barrier
Developmental aspects of cerebral blood flow, autoregulation, and cerebral metabolism
New insights in neurodevelopment relevant to pediatric critical care
Excitatory amino acid inhibition and neurodevelopmental apoptosis
Microbiome and neurodevelopment and function
Conclusion
Key references
Chapter 59: Critical care considerations for common neurosurgical conditions
Pearls
Hydrocephalus
Background
Pathophysiology
Clinical presentation
Causes
Diagnosis
Management
Arachnoid cysts
Background
Clinical presentation
Diagnosis
Management
Chiari malformations
Chiari I malformation
Chiari II malformation
Other chiari malformations
Dandy-walker complex
Encephalocele and meningocele
Spinal dysraphism
Conclusions
Key references
Chapter 60: Neurologic assessment and monitoring
Pearls
Nursing role in the recognition of neurologic complications of critical illness
Anticipatory planning for new neurologic deficits
History and assessment of risk factors
Iatrogenic complications of pharmacotherapy
Vital signs
General physical exam
Importance of observation in the neurologic exam
Assessment of level of consciousness and mental status
Fundoscopic examination
Cranial nerve examination
Approach to the motor exam
Reflexes
Cerebellar function and gait evaluation
Sensory examination
Abnormal movements or altered sensorium in the child with static encephalopathy
Distinguishing functional deficits from nonorganic pathology in the pediatric intensive care unit
Goals of the neurologic examination in the pediatric intensive care unit
Neuroimaging
Intracranial pressure monitoring
Electroencephalography monitoring
Transcranial doppler measurement of cerebral blood flow
Near infrared spectroscopy
Brain tissue oxygen monitoring
Optic nerve sheath diameter measurement
Tympanometry
Cerebral microdialysis
Integrating neurologic monitoring data
Key references
Chapter 61: Neuroimaging
Pearls
Imaging modality overview
Radiography
Ultrasound
Computed tomography
Magnetic resonance imaging
Advanced magnetic resonance imaging techniques
Magnetic resonance angiography
Catheter angiograms
Myelography
Nuclear medicine
Hypoxic ischemic injury and germinal matrix hemorrhage in the neonate
Imaging of neurovascular disorders
Ischemic stroke
Vasculopathy/vasculitis
Intracranial hemorrhage and vascular malformations
Venous infarct
Posterior reversible encephalopathy syndrome
Central nervous system infection
Demyelinating disease
Trauma
Hydrocephalus
Tumor
Seizures
Conclusion
Key references
Chapter 62: Coma and depressed sensorium
Pearls
Definitions
Epidemiology
Relevance
Physiology/pathophysiology
Etiology
Initial treatment of the comatose child
Prehospital care
Initial stabilization
Airway
Breathing
Circulation
History
Timing of the onset of symptoms
Antecedent events
Associated signs or symptoms
Preexisting conditions and comorbidities
Physical examination
Focused neurologic examination
Respiratory pattern
Eye examination
Cranial nerve examination
Motor examination
Herniation syndromes
Diagnostic evaluation
Laboratory tests
Imaging and other studies
Treatment goals: Initial and ongoing
Prognosis and outcomes
Key references
Chapter 63: Intracranial hypertension and monitoring
Pearls
Clinical background
Physiology of the intracranial vault
Intracranial pressure
Utility of hydrodynamic and electrical analog models of intracranial pressure
Cerebral vasodilation and cerebral spinal fluid pressure
Cerebral perfusion pressure and cerebral autoregulation
Measurement of intracranial pressure
Invasive monitoring devices
Noninvasive diagnostic tests of intracranial pressure
Pressure gradients and compartments
Clinical analysis of intracranial pressure
Normal values in intracranial pressure
Choice of type of invasive intracranial pressure monitoring
Choice of whether to use intracranial pressure monitoring
Clinical targets in intracranial pressure and cerebral perfusion pressure levels
Supplementing intracranial pressure monitoring with other monitoring modalities
Mechanism of brain injury in intracranial hypertension
Intracranial pressure monitoring and the postinsult natural history of injury
Key references
Chapter 64: Status epilepticus
Pearls
Definition of status epilepticus
Outcome of status epilepticus
Status epilepticus classification
Seizure types and classification
Febrile seizures and febrile status epilepticus
Seizures in the pediatric intensive care unit
Chemically induced seizures
Dialysis disequilibrium syndrome
Hepatic mechanisms
Hypertensive encephalopathy
Posttraumatic epilepsy
Renal failure
Transplantation
Autoimmune status epilepticus
Neurophysiology and pathology
Cardiorespiratory failure in status epilepticus
Other organ systems
Evaluation and electroencephalography in status epilepticus
Management of status epilepticus
General supportive care
Monitoring and termination of status epilepticus
First- and second-line pharmacotherapy
Current practice protocols
First-line benzodiazepines: Diazepam, lorazepam, or midazolam
Second-line fosphenytoin and phenobarbital
Other therapies
Medications for established status epilepticus
Management of refractory status epilepticus and other forms of treatment-resistant status epilepticus
Super-refractory status epilepticus
Fever-induced refractory epileptic encephalopathy and febrile infection-related epilepsy syndrome
Therapeutic strategies in the rse-srse-norse-fires spectrum
Immunotherapies
Infusions and anesthesia for seizure control
Midazolam
Midazolam infusions for refractory status epilepticus
Ketamine
Ketamine infusions for refractory status epilepticus
High-dose barbiturates
Treatment and dosing strategy with pentobarbital
Ceeg monitoring
Complications of pentobarbital
Concurrent antiseizure medications with pentobarbital
Inhalational anesthetics for (super) refractory status epilepticus
Hypothermia for (super) refractory status epilepticus
Ketogenic diet for (super) refractory status epilepticus
Surgical options
Changing goals of therapy in prolonged srse/fires
Key references
Chapter 65: Hypoxic-ischemic encephalopathy
Pearls
Epidemiology
Mechanisms of hypoxic-ischemic brain injury
Energy failure
Selective vulnerability
Cell death mechanisms
Neuronal death pathophysiology
Reperfusion injury
Anoxia, ischemia, reperfusion pathophysiology
Excitotoxicity and calcium accumulation
Protease activation
Oxygen radical formation
Membrane phospholipid hydrolysis and mediator formation
Endogenous defenses
Clinical pathophysiology
Cerebral blood flow and metabolism after resuscitation
Histopathology of hypoxic-ischemic encephalopathy
Clinical outcome and prognostication after pediatric cardiac arrest
Response of the immature brain to cardiac arrest
Treatment after cardiac arrest
Field interventions
Supportive care in the intensive care unit
Current and novel therapies and interventions
Postresuscitative targeted temperature management
Improving quality of cardiopulmonary resuscitation
Achieving high-quality cardiopulmonary resuscitation
Therapies optimizing postresuscitative cerebral blood flow
Phenotype-directed management
Cognitive rehabilitation after the intensive care unit
Futuristic approaches
Mitochondria targeting strategies
Targeted temperature management in a syringe
Erythropoietin
Stem cell therapy
Extracorporeal life support
Summary
Acknowledgment
Key references
Chapter 66: Pediatric stroke and intracerebral hemorrhage
Pearls
Significance
Arterial ischemic stroke
Etiologies and risk factors
Cerebral arteriopathy
Sickle cell disease
Congenital and acquired heart disease
Hypercoagulable states
Pathophysiology
Presentation
Neuroimaging
Laboratory evaluation
Initial laboratory studies in child with possible acute stroke
Other tests of hypercoagulability (etable 66.1)
Treatment
Supportive therapy
Recanalization therapies
Antithrombotic therapy
Cerebral sinus venous thrombosis
Etiologies and risk factors
Neuroimaging
Laboratory evaluation
Treatment
Prognosis
Spontaneous intracranial hemorrhage
Arteriovenous malformations
Aneurysms
Cavernous malformation
Coagulation disorders
General care of the child with intracerebral hemorrhage
Prognosis
Conclusion
Key references
Chapter 67: Central nervous system infections and related conditions
Pearls
Bacterial meningitis
Epidemiology
Epidemiology of common organisms causing pediatric meningitis
Pathogenesis
Clinical manifestations
Laboratory diagnosis
Treatment
Supportive care
Adjunctive therapy
Outcomes
Prevention
Subdural empyema
Brain abscess
Viral meningoencephalitis
Epidemiology
Pathophysiology/pathogenesis of viral meningoencephalitis
Clinical evaluation
Laboratory manifestations
Neuroimaging
Clinical presentation and course
Treatment
Prognosis
Acute disseminated encephalomyelitis
Key references
Chapter 68: Acute neuromuscular disease and disorders
Pearls
Guillain-barré syndrome
Myasthenia gravis
Congenital and transient neonatal myasthenia gravis
Tick paralysis
Periodic paralyses
Botulism
Diphtheria
Acute intermittent porphyria
Spinal muscular atrophy
Poliomyelitis
Polio-like syndromes
Organophosphate and carbamate poisoning
Key references
Chapter 69: Acute rehabilitation and early mobility in the pediatric intensive care unit
Pearls
Rehabilitation team members in the pediatric intensive care unit
Role of physical therapy in the pediatric intensive care unit
Role of occupational therapy in the pediatric intensive care unit
Role of speech-language pathology in the pediatric intensive care unit
Important considerations for mobility in the pediatric intensive care unit
Pediatric intensive care unit environment
Cardiac and pulmonary status and complications
Sedation
Intensive care unit–acquired weakness
Facilitating infant neurodevelopment during critical illness
Communication
Acute rehabilitation across the intensive care unit continuum
Patient selection: Age-based factors
Patient selection: Acuity of illness
Equipment
Positioning equipment
Sitting equipment
Standing or ambulation equipment
Other equipment
Resource utilization
Implementation of acute rehabilitation in the pediatric intensive care unit: Evidence and strategies
Evidence in support of early mobilization in the pediatric intensive care unit
Early mobilization: The “e” in the abcdef ICU liberation bundle
Implementation of a pediatric intensive care unit early mobility program: Strategies for success
Step 1: Form an interdisciplinary team
Step 2: Summarize the evidence
Step 3: Identify local barriers to implementation
Step 4: Establish performance measures
Step 5: Ensure that all patients receive the intervention—the 4 e’s
Summary
Key references
Section VII: Pediatric Critical Care: Renal
Chapter 70: Renal structure and function
Pearls
Renal anatomy
Renal development
Renal vasculature
Vascular anatomy
Vascular function
Vascular development
The nephron
Nephron development
Glomerulus
Glomerular anatomy
Glomerular function
Tubular anatomy
Proximal tubule
Loop of henle
Distal nephron
Tubular function
Proximal tubule
Loop of henle
Distal tubule
Collecting duct
Interstitium
Development
Structure and function
Summary
Key references
Chapter 71: Fluid and electrolyte issues in pediatric critical illness
Pearls
Sodium
Hyponatremia
Pathophysiology and etiology
Decreased total body sodium
Renal sodium losses
Cerebral salt wasting
Increased total body sodium
Congestive heart failure
Cirrhosis
Nephrotic syndrome
Renal failure
Normal total body sodium
Signs and symptoms
Treatment
Prevention
Therapy
Hypernatremia
Pathophysiology and etiology
Low total body sodium
Normal total body sodium
Increased total body sodium
Signs and symptoms
Treatment
Potassium
Causes of hypokalemia
Hypokalemia without potassium deficit
Hypokalemia with potassium deficit
Renal losses
Gastrointestinal losses
Signs and symptoms
Treatment
Hyperkalemia
Causes
Artifactual
Redistribution
Increased load
Impaired elimination
Manifestations of hyperkalemia
Treatment
Magnesium
Hypomagnesemia
Causes
Signs and symptoms
Treatment
Hypermagnesemia
Cause
Signs and symptoms
Treatment
Calcium
Regulation of calcium
Hormonal regulation of calcium
Hypocalcemia
Clinical and laboratory concerns
Treatment
Hypercalcemia
Treatment
Phosphorus
Hypophosphatemia
Causes of severe hypophosphatemia
Signs and symptoms
Treatment
Hyperphosphatemia
Causes of hyperphosphatemia
Signs and symptoms
Treatment
Key references
Chapter 72: Acid-base disorders
Pearls
Overview of acid-base physiology
Defining acids and bases
Role of water
Principles of electroneutrality
Acids, bases, and buffers
Tools for interpreting acid-base disorders
Henderson-hasselbalch equation
Base excess and standard base excess
Bicarbonate-based approach
Anion gap and corrected anion gap
Serum osmolar gap
Strong ion difference
Urinary anion gap
Summary
Clinical approach to disorders of acid-base balance
Is abnormal ph dangerous?
Blood gases: Arterial, central venous, or capillary samples?
Evaluation of acid-base disorders
Metabolic acidosis
Elevated anion gap acidoses
Lactic acidosis
Ketoacidosis
Toxic compounds that directly provoke acidosis
Other forms of metabolic acidosis with an increased anion gap
Hyperchloremic acidosis: Nonanion gap metabolic acidosis
Exogenous chloride load
Postpyloric gastrointestinal fluid losses
Renal tubular acidosis and drug-mediated tubulopathies
Urinary reconstruction using bowel segments
Treating metabolic acidosis
Sodium bicarbonate
Alternative alkalinizing agents
Dialysis management of metabolic acidosis
Metabolic alkalosis
Treating metabolic alkalosis
Respiratory acid-base derangements
Respiratory acidosis
Treating respiratory acidosis
Respiratory alkalosis
Mixed acid-base derangements
Summary
Key references
Chapter 73: Tests of kidney function in children
Pearls
Assessment of glomerular function and injury
Renal clearance techniques
Renal inulin clearance compared with other glomerular filtration rate measurement techniques
Plasma markers
Creatinine
Cystatin c
Estimating equations
Summary of recommendations regarding use of glomerular filtration rate estimating equations
Neonatal renal function
Biomarkers of acute kidney injury and the next generation
Tubular function
Urine
Urine electrolytes (sodium and chloride)
Urine concentration capacity
Serum blood urea nitrogen/creatinine ratio
Urine microscopy
Proteinuria
Renal acidification
Potassium regulation
Generalized proximal tubulopathy
Integration of kidney function assessment in critical care
Key references
Chapter 74: Glomerulotubular dysfunction and acute kidney injury
Key words:
Pearls
Physiology of glomerular filtration
Normal renal physiology and response of the kidney during stress
Pathogenesis of reduced glomerular filtration rate in acute kidney injury
Morphologic changes in renal injury
Mechanisms of renal cell injury
Alterations in cell membranes
Cellular calcium homeostasis
Production of free radicals
Tubular cell energy metabolism
Glomerulotubular dysfunction
Hemodynamically mediated acute kidney injury
Treatment of acute kidney injury
Prevention/attenuation of acute kidney injury
Diuretics
Calcium entry blockers
Prostaglandins
Renin-angiotensin antagonists
Adenosine and adenosine triphosphate
Atrial natriuretic factor
Free radical scavengers
Thyroxine
Glycine and alanine
Acute kidney injury: Clinical impact
Hyperkalemia
Severe hypertension
Plasma and extracellular volume expansion
Severe metabolic acidosis
Hypocalcemia/hyperphosphatemia
Uremia
Renal disposition of endogenous and exogenous compounds
Specific kidney diseases that may lead to acute kidney injury
Hemolytic uremic syndrome
Pathophysiology
Clinical signs
Complications
Therapy
Prognosis
Acute glomerulonephritis
Acute postinfectious (streptococcal) glomerulonephritis
Clinical signs
Laboratory findings
Treatment
Prognosis
Systemic lupus erythematosus
Clinical signs
Treatment
Prognosis
Other glomerulonephritides
Nephrotic syndrome and acute kidney failure
Tubulointerstitial disease
Acute tubulointerstitial nephritis
Cardiorenal syndrome
Cardiac surgery–related acute kidney injury
Tumor lysis syndrome
Management
Rasburicase
Role of renal replacement therapy
Pigment nephropathy
Pathophysiology
Hypocalcemia
Drug-induced nephrotoxicity
Aminoglycoside nephrotoxicity
Amphotericin b
Vancomycin
Calcineurin inhibitors
Nonsteroidal antiinflammatory drugs
Contrast-induced nephropathy
Acute renal failure after stem cell transplantation
Urinary tract obstruction
Conclusions
Key references
References
Chapter 75: Pediatric renal replacement therapy in the intensive care unit
Pearls
Basic physiology of dialysis and ultrafiltration
Peritoneal dialysis
Physiology
Indications
Technique
Disadvantages and complications
Intensive care unit issues
Intermittent hemodialysis
Physiology
Indications
Technique
Disadvantages and complications
Intensive care unit issues
Continuous renal replacement therapy
Physiology
Indications
Technique
Anticoagulation
Dialysate and infused fluids
Clearance
Disadvantages and complications
Intensive care unit issues
Extended dialysis
Outcomes of renal replacement in critically ill children
Advances in pediatric renal replacement therapies
Summary
Key references
Chapter 76: Pediatric renal transplantation
Pearls
Donor source: Living donor versus deceased donor
Living donor transplantation
Deceased donor transplantation
Timing of transplantation
Histocompatibility
Abo blood group considerations
Hla matching
Pretransplantation crossmatch testing
Sensitized recipient
Surgical procedure
Multiorgan transplantation
Immediate arrival to the pediatric intensive care unit (boxes 76.1, 76.2, and 76.3)
Posttransplantation monitoring
Hemodynamics and tissue perfusion
Fluid and electrolytes
Urine output
Recovery of renal function
Immunosuppression
Infection surveillance and prevention
Posttransplantation complications
Acute kidney injury
Vascular complications
Urologic complications
Rejection
Urinary tract infection
Other infections
Recurrence of primary renal disease
Focal segmental glomerulosclerosis
References
Chapter 77: Renal pharmacology
Pearls
Kidney function and drug disposition
Drug dosing in kidney disease
Drug dosing in dialysis
Kidney as a therapeutic target: Diuretics and agents regulating renal excretion
Carbonic anhydrase inhibitors
Mechanism and sites of action
Efficacy on urinary excretion and therapeutic uses
Adverse effects
Osmotic diuretics
Mechanism and sites of action
Efficacy on urinary excretion and therapeutic uses
Adverse effects
Loop diuretics
Mechanism and site of action
Pharmacokinetics
Effects on urinary excretion and adverse effects
Therapeutic uses
Thiazide and thiazide-like diuretics
Mechanism and site of action
Pharmacokinetics
Effects on urinary excretion and therapeutic uses
Adverse effects
Potassium-sparing diuretics
Mechanism and site of action
Effects on urinary excretion and therapeutic uses
Adverse effects
Vasopressin antagonists
Mechanism of action and therapeutic uses
Adverse effects
Diuretic resistance
Medications for the prevention/reversal of acute kidney injury
Key references
Chapter 78: Acute severe hypertension
Pearls
Terminology
Etiology
Pathophysiology
Endothelial homeostasis
Sympathetic nervous system activation
Renin-angiotensin-aldosterone system
Nitric oxide
Volume overload
Clinical presentation
Patient evaluation and monitoring
Blood pressure measurement and other monitoring
Diagnostic evaluation
Pharmacologic therapy
General considerations
Clonidine
Esmolol
Hydralazine
Isradipine
Labetalol
Nicardipine
Sodium nitroprusside
Other available agents
Special situations
Preeclampsia
Pheochromocytoma
Summary
Key references
Section VIII: Pediatric Critical Care: Metabolic and Endocrine
Chapter 79: Cellular respiration
Pearls
Pathways of cellular respiration
Glycolysis (anaerobic respiration)
Fatty acid β-oxidation
Protein catabolism
Krebs cycle
Mitochondrial oxidative phosphorylation
Oxygen toxicity
Impaired cellular respiration in critical illness
Clinical assessment of oxygen utilization
Lactate
Venous oxygen saturation
Microdialysis
Near-infrared spectroscopy
Optical spectroscopy
Tissue oxygen tension
Magnetic resonance spectroscopy
Blood mitochondrial DNA
Mitochondrial- and bioenergetic-targeted therapy in critical illness
Antioxidants
Glycemic control
Substrate provision
Mitochondrial biogenesis and mitophagy
Membrane stabilizers
Hibernation
Mitochondrial transplantation
Conclusions
Key references
Chapter 80: Biology of the stress response
Key words:
Pearls
Definitions and background
Stress system primary elements
Stress response
Central activation and integration
Peripheral responses
Cellular responses
Stress response in critical illness
Recommendations and conclusions
Key references
Chapter 81: Inborn errors of metabolism
Pearls
Pathophysiology of inborn errors of metabolism
Inheritance of inborn errors of metabolism
Signs and symptoms of inborn errors of metabolism
Laboratory evaluation of suspected inborn errors of metabolism
Postmortem evaluation of a child with suspected inborn errors of metabolism
Emergency treatment of children with suspected inborn errors of metabolism
Classification of inborn errors of metabolism by clinical presentation
Group 2 inborn errors of metabolism: Impaired energy production
Group 3 inborn errors of metabolism: Intoxication
Summary
Metabolic acidosis
Hypoglycemia
Cardiomyopathy and inborn errors of metabolism
Metabolic myopathies and rhabdomyolysis
Neonatal screening
Conclusions
Key references
Chapter 82: Progress towards precision medicine in critical illness
Pearls
Genetic variation and critical illness
Genetic variation and sepsis
Genetic variation and acute respiratory distress syndrome
Transcriptomics and critical illness
Plasma biomarkers and critical illness
Plasma biomarkers and sepsis
Plasma biomarkers and acute respiratory distress syndrome
Summary
Key references
Chapter 83: Molecular foundations of cellular injury
Pearls
Caspase-dependent forms of regulated cell death
Apoptosis
Animal studies on apoptosis
Pyroptosis
Caspase-independent forms of regulated cell death
Necroptosis
Animal studies—necroptosis
Autophagy
Mitochondrial permeability transition–mediated regulated necrosis
Ferroptosis
Parthanatos
Netosis
Cell death as a therapeutic target
Key references
References
Chapter 84: Endocrine emergencies
Pearls
Hypothalamic-pituitary-adrenal axis
Cortisol biochemistry and biology
Actions of cortisol
Immunity
Metabolism
Hemodynamics
Assessing the cortisol stress response
Free cortisol
Adrenal insufficiency in the intensive care unit
Primary adrenal insufficiency
Secondary adrenal insufficiency
Treatment of adrenal insufficiency
Corticosteroid side effects
Alterations of glucose homeostasis
Glucose homeostasis in health
Hyperglycemia
Stress hyperglycemia and outcomes
Pathophysiology of stress hyperglycemia
Mechanisms of stress hyperglycemia adverse outcomes
Clinical trials examining management of critical illness hyperglycemia
Glucose measurement
Hypoglycemia
Clinical manifestations
Pathogenesis
Fasting adaptation
Hypoglycemia treatment
Alterations of thyroid hormone in critical illness
Thyroid biochemistry
Thyroid hormone actions
Hyperthyroidism
Hypothyroidism
Euthyroid sick syndrome in critical illness
Thyroid hormone supplementation in the picu
Key references
Chapter 85: Diabetic ketoacidosis
Pearls
Etiology, definition, and presentation
Epidemiology
Frequency of diabetic ketoacidosis at diagnosis
Frequency of diabetic ketoacidosis in children and adolescents after diagnosis
Morbidity and mortality associated with diabetic ketoacidosis
Management guidelines (fig. 85.1)
Fluids
Insulin
Electrolytes
Correction of acidosis
Monitoring
Diabetic ketoacidosis–associated complications
Hyperglycemic hyperosmolar syndrome
Thrombotic complications
Other complications
Neuropsychologic sequelae
Healthcare costs associated with diabetic ketoacidosis
Key references
Section IX: Pediatric Critical Care: Hematology and Oncology
Chapter 86: Structure and function of the hematopoietic organs
Pearls
Normal peripheral blood values
Structure of the bone marrow
Function of the bone marrow: Hematopoiesis
Regulators of hematopoiesis: Growth factors
Erythropoiesis
Granulopoiesis
Megakaryocyte and platelet production
Lymphopoiesis
Spleen
Key references
Chapter 87: The erythron
Pearls
Oxygen transport
Carbon dioxide transport
Biophysical factors influencing gas transport
Blood rheology
Red blood cell aggregation and adhesion
Red blood cell deformability
Regulation of blood flow distribution by red blood cells
Role of red blood cell–nitric oxide interactions in vasoregulation
Metabolism of endothelium-derived nitric oxide by red blood cells: Historical view
Metabolism of endothelium-derived nitric oxide by red blood cells: Contemporary view
Processing and export of s-nitrosothiols by red blood cells
Metabolism of nitrite by red blood cells
Vasoregulation by red blood cell-derived adenosine triphosphate
Red blood cell energetics and consequences of antioxidant system failure
Acquired red blood cell injury, eryptosis, and clearance
Influence of red blood cells on hemostasis
Summary
Key references
References
Chapter 88: Hemoglobinopathies
Pearls
Globin gene loci
Sickle cell disease
Molecular description and epidemiology
Sickle cell trait
Spectrum of sickle cell disease genotypes and natural history
Laboratory and diagnostics
Pathophysiology
Hemoglobin polymerization
Red cells, inflammation, hemolysis, and the endothelium
Clinical manifestations
Pain
Pathophysiology, diagnosis, and presentation
Management
Sepsis. 
Pathophysiology and etiology
Management
Acute chest syndrome
Presentation
Management
Stroke. 
Natural history
Diagnosis
Management
Aplastic crisis
Splenic sequestration
Pulmonary hypertension
Pathophysiology and etiology
Diagnosis
Management
Multiorgan failure 
Renal conditions
Iron overload
Sleep conditions and depression and suicide in sickle cell disease
Surgery and anesthesia
Disease-modifying and curative therapies for sickle cell disease
Hydroxyurea
Transfusion
Choice of product
Type and goals of transfusion
Transfusion reactions and hyperhemolysis
Hematopoietic stem cell transplantation, gene therapy, and novel therapies for sickle cell disease
Thalassemia
Molecular description and epidemiology
Laboratory and diagnostics
Pathophysiology
Forms and variations
α-thalassemia
β-thalassemia
Hbe/β0-thalassemia
Natural history
Assessment of iron overload
Spectrum of disease
Anemia
Transfusion-related complications
Cardiac complications
Assessments
Management
Hepatic and renal dysfunction in thalassemia
Thrombosis and pulmonary emboli
Pulmonary hypertension
Endocrine
Hematopoietic stem cell transplantation, gene therapy, and novel therapies for thalassemia
Key references
Chapter 89: Coagulation and coagulopathy
Pearls
Overview of hemostasis
Role of platelets and von willebrand factor in hemostasis
Role of endothelial cells in hemostasis
Crosstalk between coagulation and inflammation
Hypercoagulability and thrombophilia
Approach to the patient with an identified or suspected coagulation disorder
Clinical history
Physical examination
Basic tests of hemostasis
Platelet function testing
Thromboelastography and thromboelastometry
Abnormal hemostasis in critical illness
Disseminated intravascular coagulation and other microangiopathic consumptive disorders
Pathogenesis
Clinical presentation and diagnosis
Meningococcal purpura fulminans
Management
Thrombotic thrombocytopenic purpura and hemolytic uremic syndrome
Thrombocytopenia associated multiorgan failure
Abnormal hemostasis in liver disease and hepatic insufficiency
Presentation
Management
Vitamin k deficiency
Management
Circulating anticoagulants
Iatrogenic coagulopathy
Massive transfusion syndrome
Management
Anticoagulant overdose
Warfarin
Management
Heparin
Management
New oral anticoagulants
Management
Platelet disorders
Quantitative platelet disorders
Heparin-induced thrombocytopenia
Qualitative platelet disorders
Management
Defects associated with acquired vwf defects
Uremia
Thrombocytopenia-associated multiorgan failure
Cardiac-induced hemostatic disorders
Systemic diseases associated with factor deficiencies
Thrombophilia and immunothrombosis
Summary
Key references
Chapter 90: Thrombosis in pediatric critical care
Pearls
Developmental hemostasis
Etiology and epidemiology
Central venous access devices
Arterial access
Cardiac surgery
Thrombophilia
Heparin-induced thrombocytopenia
Clinical features
Diagnosis
Venous thrombosis
Pulmonary embolus
Arterial thrombosis
Intracardiac thrombosis
Management
Unfractionated heparin in children
Thromboprophylaxis in the pediatric intensive care unit
Conclusions
Key references
Chapter 91: Transfusion medicine
Pearls
Red blood cells
Red blood cell transfusion: Why and why not
Anemia and oxygen delivery
Oxygen delivery in the critically ill
Adaptive mechanisms with anemia
Oxygen kinetics in the critically ill
Regulation by red blood cells of oxygen delivery to tissue
Risks of anemia
Transfusion of red blood cells: Indications (when)
Hemorrhagic shock or severe bleeding (node 1)
Picu patients with severe anemia without severe bleeding (node 2)
Hemoglobin less than 5 or between 5 and 7 g/dl (node 2)
Hemodynamically unstable patients without serious bleeding (node 3)
Hemodynamically stable noncardiac patients (node 4)
Cardiac patients (node 5)
Prevention of anemia and red blood cell transfusion
Pre–pediatric intensive care unit anemia
Pediatric intensive care unit–associated anemia
Post–pediatric intensive care unit anemia
Types of red blood cell units
Standard red blood cell units
Other types of red blood cell units
Leukocyte-reduced red blood cell units
Washed red blood cell units
Irradiated red blood cell units
Cytomegalovirus-negative red blood cell units
Directed red blood cell units
Autologous red blood cell units
Transfusion of red blood cells: How
Blood types
Volume and number of red blood cell units
Length of storage
Perfusion, warming, and filtration
Plasma
Types of plasma
Transfusion of plasma: Indications (when)
Transfusion of plasma: How
Platelets
Types of platelet products
Standard platelet products
Special platelet concentrates
Leukocyte-reduced platelets
Irradiated platelets
Cytomegalovirus-negative platelets
Cryopreserved platelets
Pathogen-reduced platelets
Platelet transfusion (why)
Transfusion of platelets: Indications (when)
Transfusion of platelets: How
Whole blood
Whole blood: Type of product
Whole blood: Indications
Whole blood: How
Cryoprecipitate
Transfusions reactions and complications
Transfusion reactions
Immediate transfusion reactions
Other acute transfusion reactions
Delayed transfusion reactions
Complications related to massive transfusion
Transfusion-transmitted infections
Transfusion-related immunomodulation
Other transfusion-related complications
Reactions and complications more frequent with plasma transfusion
Reactions and complications more frequent with platelet transfusion
Treatment of transfusion reactions and complications
Conclusion
Key references
Chapter 92: Hematology and oncology problems
Pearls
Hematologic emergencies
Anemia
Hemorrhagic anemia
Anemia secondary to decreased production
Hemolytic anemia
Thrombocytopenia related to decreased platelet production
Thrombocytopenia-related immune-mediated consumption
Thrombocytopenia related to nonimmune consumption
Bleeding in uremia
Oncologic emergencies
Tumor lysis syndrome
Hyperkalemia
Hyperphosphatemia/hypocalcemia
Hyperuricemia
Hyperleukocytosis
Spinal cord compression
Acute airway compromise in anterior mediastinal tumors
Superior vena cava syndrome
Hemophagocytic lymphohistiocytosis
Anthracycline-induced cardiac dysfunction
Posterior reversible encephalopathy
Chimeric antigen receptor t cell–mediated toxicity
Key references
Chapter 93: Critical illness in children undergoing hematopoietic progenitor cell transplantation
Pearls
Sources of hematopoietic progenitor cells and identification of donors
Indications and outcomes
Transplant procedure
Conditioning regimen
Stem cell harvesting/collection/cryopreservation
Reinfusion
Recovery period
Complications
Cardiac complications
Pulmonary complications
Early pulmonary complications
Periengraftment respiratory distress syndrome
Pulmonary cytolytic thrombi
Diffuse alveolar hemorrhage
Idiopathic pneumonia syndrome
Late pulmonary complications
Bronchiolitis obliterans syndrome/bronchiolitis obliterans organizing pneumonia
Pulmonary venoocclusive disease
Critical care for pulmonary complications
Mechanical ventilatory support
Adjunctive therapies
Conclusion
Dilemmas in the diagnosis of pulmonary complications
Hepatic complications
Colitis and other gastrointestinal complications
Myelosuppression and hematologic complications
Myelosuppression and immune dysregulation
Infectious complications
Graft failure
Hematologic complications
Iron overload
Graft-versus-host disease
Neurologic complications
Chimeric antigen receptor t cells
Key references
Section X: Pediatric Critical Care: Gastroenterology and Nutrition
Chapter 94: Gastrointestinal structure and function
Pearls
Intestinal structure, digestion, absorption of nutrients, water, and electrolytes
Water and solute transport across intestinal epithelium
Digestion of carbohydrates
Digestion of proteins
Digestion of lipids
Fat digestion
Intestinal lymphatics
Regulation of electrolyte and water movement
Electrolyte transport
Gastric acid
Intestinal motility
Pancreas
Pancreatic exocrine secretory function
Hepatobiliary system
Examination
Anatomy, structure, and function
Portal circulation
Hepatic function
Degradation and elimination by the liver
Enterohepatic circulation
Hepatic regulatory function
Storage function
Host-defense mechanisms of the gut: Immunology and microbiology
Immunologic processing in the gut
Gastrointestinal and hepatobiliary testing in the intensive care unit
Key references
References
Chapter 95: Disorders and diseases of the gastrointestinal system
Pearls
Gastrointestinal evaluation of the critically ill child
Abdominal examination
Gastrointestinal endoscopy
Gastroesophageal reflux monitoring
Use of colorants to identify aspiration in the intensive care unit
Radiologic procedures
Plain films
Contrast radiography
Ultrasonography, computed tomographic scanning, and magnetic resonance scanning
Radionuclide scanning
Testing for occult blood loss
Stool ph and reducing substances
Intensive care unit as a satellite laboratory facility
Life-threatening complications of gastrointestinal disorders
Systemic
Central line–associated bloodstream infection
Esophagus
Congenital esophageal anomalies
Caustic injury to the esophagus
Esophageal foreign bodies
Gastroesophageal reflux
Stomach and duodenum
Gastric volvulus
Gastric ulcer
Duodenal ulcers
Small intestine and colon
Malrotation
Necrotizing enterocolitis
Low cardiac output syndrome
Food allergy
Hemolytic uremic syndrome
Inflammatory bowel disease
Distal intestinal obstruction syndrome
Hirschsprung disease
Acute colonic pseudo-obstruction
Abdominal compartment syndrome
Acute and chronic pancreatitis
Acute and chronic liver failure
Key references
Chapter 96: Acute liver failure
Pearls
Acute liver failure
Background, definitions, etiology, outcomes
Etiologies
Prognostic assessment
Diagnosis and workup
Specific treatments for particular causes of pediatric acute liver failure
Acetaminophen toxicity
Amanita poisoning
Autoimmune hepatitis
Wilson disease
Metabolic disease
Pediatric intensive care unit complications and management
Neurologic complications: Hepatic encephalopathy and cerebral edema
Cerebral edema
Management of hepatic encephalopathy and cerebral edema
Ventilation
Glucose, electrolytes, and fluid balance
Ascites
Renal function
Coagulopathy
Infection prophylaxis and treatment
Liver support
Decision-making for liver transplantation for acute liver failure
Key references
Chapter 97: Hepatic transplantation
Pearls
Current state of pediatric liver transplantation
Indications for liver transplant
Liver transplant evaluation
Pretransplant considerations
Transplant considerations
Donor considerations
Liver transplant procedure
Complications of liver transplantation
Primary graft nonfunction
Vascular complications
Bleeding
Biliary complications
Rejection
Infection
Posttransplant lymphoproliferative disorder
Posttransplant management in the intensive care unit
Drains and vascular access
Fluid and electrolytes
Graft function
Anticoagulation
Respiratory support
Sedation
Infection prophylaxis
Immunosuppression
Liver transplant outcomes
Patient and graft survival
Retransplantation
Key references
References
Chapter 98: Acute abdomen
Pearls
Anatomic and physiologic considerations
Peritoneum
Visceral blood flow
Physical examination of the abdomen
Laboratory tests
Imaging options
Ultrasonography
Abdominal plain radiographs
Computed tomography
Magnetic resonance imaging
Abdominal conditions requiring treatment in the intensive care unit
Perforated viscera
Ischemia
Neutropenic enterocolitis
Pancreatitis
Hemorrhage
Other specific conditions
Cholecystitis
Spontaneous bacterial peritonitis
Abdominal compartment syndrome
Intraabdominal abscess
Intestine as a source of sepsis
Surgical intervention
Key references
Chapter 99: Nutrition of the critically ill child
Pearls
Malnutrition in the pediatric critically ill patient
Assessment of nutritional status
Body composition
Biochemical assessment
Metabolic consequences of critical illness
Underfeeding and overfeeding in the pediatric intensive care unit
Indirect calorimetry
Protein requirements
Lipid requirements
Micronutrient requirements
Refeeding syndrome
Enteral nutrition in critically ill children
Parenteral nutrition
Carbohydrates
Amino acids
Lipids
Electrolytes/minerals and trace elements
Biochemical monitoring
Obese critically ill children
Revised guidelines for pediatric critical care nutrition
Conclusions
Key references
References
Section XI: Pediatric Critical Care: Immunity and Infection
Chapter 100: Innate immunity
Pearls
Components of innate immune system
Innate immune stimulus: Danger hypothesis
Signal recognition
Signal transduction
Effector pathways
Soluble mediator response
Cellular response
Crosstalk between systems
Regulation of the innate immune response
Clinical manifestations of the innate immune response in the intensive care unit
Therapeutic targets of the innate immune system
Therapeutic questions arising from the innate immune system
Conclusion
Key references
References
Chapter 101: Adaptive immunity
Pearls
Lymphocytes develop to recognize specific antigens
B-cell activation leads to antibody secretion: The humoral immune response
Effector t cells direct cell-mediated immunity
T-cell activation requires interaction with innate immune cells
T-cell activation requires a second signal
Cd8+ t cells differentiate into cytotoxic t cells
Cd4+ t cells differentiate into multiple t helper cell subtypes
Th1 cells
Th2 cells
Th17 cells
T follicular helper (tfh) cells
Th9 cells
Regulatory t cells (treg)
Additional t-cell subtypes
Adaptive immunity in the intensive care unit
Summary
Key references
Chapter 102: Critical illness and the microbiome
Pearls
General concepts in the field of microbiome science
Commensal, pathogenic, and keystone species
Site specificity
Dysbiosis
Studying the microbiome
Development of the microbiome in children
Development of intensive care unit dysbiosis in children
Known and potential clinical consequences of intensive care unit dysbiosis
Microbiome-based therapeutics in the intensive care unit
Probiotics
Fecal microbiota transplantation
Selective gut decontamination
Supplementing/repleting microbial metabolites
Targeted interventions against enterobacteriaceae
Monitoring the intensive care unit microbiome
Future of microbiome science
Key references
Chapter 103: Congenital immunodeficiency
Pearls
Basic framework for understanding the immune system
Compartment 1: Complement
Clinical presentations
Compartment 2: Phagocytes
Clinical presentations
Compartment 3: B cells and antibodies
Clinical presentations
Compartment 4: T cells
Clinical presentations
Specific disorders likely to be encountered in the pediatric intensive care unit
Specific disorders: Complement
C1 inhibitor deficiency
Early pathway defects (c1, c2, c3, c4)
Late pathway components: Membrane attack complex defects (c5, c6, c7, c8, c9)
Complement regulatory protein defects (factor h, factor I, mcp)
Specific disorders: Phagocytes
Severe congenital neutropenia
Leukocyte adhesion deficiency
Whim syndrome
Chronic granulomatous disease
Specific disorders: B cells and antibodies
X-linked agammaglobulinemia
Hyperimmunoglobulin m syndromes
B-cell activation
Pneumocystis jirovecii and cryptosporidium parvum infections among patients with hyperimmunoglobulin m and cd40l mutations
Common variable immunodeficiency syndromes
Genetic defects associated with a common variable immunodeficiency phenotype
Selective immunoglobulin a deficiency
Specific disorders: T cells
22q11 deletion syndrome (digeorge syndrome)
Severe combined immunodeficiency
X-linked severe combined immunodeficiency
Pathogenesis of selected genotype/phenotypes of scid
Adenosine deaminase– and purine nucleotide phosphorylase–deficient severe combined immunodeficiency
Specific disorders: Other complex or combined immunodeficiencies
Wiskott-aldrich syndrome
Genotype/phenotype correlations of mutations in wiscott aldrich syndrome
Cartilage-hair hypoplasia
Radiation-sensitive disorders: Ataxia telangiectasia and nijmegen breakage syndrome
Susceptibility to hemophagocytic lymphohistiocytosis and severe epstein-barr virus infection
Toll-like receptors and innate signaling pathway defects
Chronic mucocutaneous candidiasis syndromes
Autoimmune lymphoproliferative syndrome
Laboratory evaluation of the immune system
Diagnostic testing: Complement
Diagnostic testing: Phagocytes
Diagnostic testing: B cells and antibodies
Diagnostic testing: T cells
Treatment of immune system disorders
Treatment: Complement
Treatment: Phagocytes
Treatment: B cells and antibodies
Immunoglobulin replacement
Prophylactic antibiotics
Treatment: T cells
Key references
References
Chapter 104: Acquired immune dysfunction
Pearls
Immune dysfunction during sepsis, malnutrition, hiv/aids, and other critical illness states
Sepsis and immune dysfunction
Immunosuppression in sepsis
Malnutrition and immune dysfunction
HIV infection and AIDS
Pulmonary complications and respiratory failure
Pneumocystis jirovecii pneumonia
Cytomegalovirus pneumonitis
Other viral pathogens
Mycobacterial pathogens
Fungal infections
Lymphocytic interstitial pneumonitis
Upper airway obstruction
Cardiovascular complications
Abdominal complications
Hematologic and malignancy complications
Central nervous system complications
Occupational human immunodeficiency virus exposure
Other selected causes of secondary immune dysfunction
Immunosuppressive medications
Transfusions
Uremia
Neonatal period
Key references
Chapter 105: Immune balance in critical illness
Pearls
Innate and adaptive immunity
Innate immunity
Adaptive immunity
Proinflammatory and antiinflammatory responses
Systemic inflammatory response syndrome in critical illness
Compensatory antiinflammatory response syndrome in critical illness
Temporal aspects of the systemic inflammatory response syndrome/compensatory antiinflammatory response syndrome response
Cars and clinical outcomes
Immunomodulation in the pediatric intensive care unit
Targeted innate immunostimulation
Targeted adaptive immunostimulation
Unintended immunomodulation
Immune monitoring in the intensive care unit
Conclusion
Key references
Chapter 106: Pediatric rheumatologic disease
Pearls
Rheumatologic diseases: Overview
Rheumatologic diseases: Clinical presentation, diagnosis, and treatment
Juvenile idiopathic arthritis
Systemic juvenile idiopathic arthritis
Key points
Clinical presentation
Laboratory studies
Management
Systemic lupus erythematosus
Key points
Clinical presentation
Mucocutaneous manifestations
Musculoskeletal manifestations
Pulmonary manifestations
Pleuritis and pleural effusion
Pulmonary hemorrhage
Pulmonary embolism
Interstitial lung disease
Pulmonary hypertension
Cardiovascular manifestations
Pericarditis and pericardial tamponade
Myocarditis
Valvular disease
Arrhythmias
Acute coronary syndrome
Renal manifestations
Central nervous system manifestations
Hematologic involvement
Thrombotic thrombocytopenic purpura
Immune dysfunction
Acute abdominal manifestations: Peritonitis, serositis, pancreatitis, and intestinal perforation
Laboratory studies
Auto-antibody testing
Management
Neonatal systemic lupus erythematosus
Raynaud phenomenon
Antiphospholipid syndrome
Key points
Clinical presentation
Laboratory studies
Management
Juvenile dermatomyositis
Key points
Clinical presentation
Laboratory studies
Imaging
Management
Systemic vasculitis
Key points
Small-vessel vasculitis
Immune-complex small-vessel vasculitis
Henoch-schönlein purpura
Treatment of hsp
Pauci-immune small-vessel vasculitis (antineutrophil cytoplasmic antibody–associated vasculitis)
Granulomatosis with polyangiitis
Microscopic polyangiitis
Eosinophilic granulomatosis with polyangiitis
Treatment of anca-associated vasculitis
Medium-vessel vasculitis
Kawasaki disease
Treatment of kd
Polyarteritis nodosa
Large-vessel vasculitis
Takayasu arteritis
Primary vasculitis of the central nervous system
Autoinflammatory syndromes
Key points
Rheumatologic diseases: Conditions and complications in the intensive care unit
Macrophage activation syndrome
Key points
Airway compromise
Organ-specific complications
Treatment-related complications of rheumatologic diseases
Corticosteroids
Gastrointestinal side effects
Infectious side effects
Summary
Key references
Chapter 107: Bacterial and fungal infections
Pearls
Bacterial infections in the intensive care unit
Gram-positive bacteria
Gram-negative bacteria
Anaerobic infections
General considerations for antibiotic therapy
Antibiotic classes
β-lactam antibiotics
β-lactam antimicrobial plus β-lactamase inhibitor combination
Cephalosporins
Carbapenems
Monobactams
Aminoglycosides
Glycopeptides
Macrolides
Fluoroquinolones
Miscellaneous
Clindamycin
Linezolid
Metronidazole
Colistin
Doxycycline
Trimethoprim-sulfamethoxazole
Tigecycline
Daptomycin
Antibiotic resistance and treatment of multidrug-resistant pathogens
Antibiotic resistance mechanisms
Treatment of multidrug-resistent pathogens
Fungal infections and antifungal agents
Candida
Aspergillus and other invasive molds
Antifungal agents
Amphotericin
Triazoles
Echinocandins
Flucytosine
Combination antifungals
Infection diagnostics
Antimicrobial stewardship
Summary
Key references
Chapter 108: Life-threatening viral diseases and their treatment
Pearls
Myocarditis
Epidemiology and etiology
Clinical presentation
Diagnosis and management
Acute liver failure
Epidemiology and etiology
Clinical presentation
Diagnosis and management
Viral pneumonia/pneumonitis
Epidemiology and etiology
Clinical presentation
Diagnosis and management
Central nervous system infections
Epidemiology and etiology
Clinical presentation
Diagnosis and management
Emerging viral diseases
Key references
Chapter 109: Healthcare-associated infections
Pearls
Epidemiology
Risk factors
Pathogen transmission and isolation practices
Infection prevention strategies
Care bundles
Hand hygiene
Personal protective equipment
Antimicrobial resistance and antimicrobial stewardship
Visitation policies
Specific healthcare-associated infections in the pediatric intensive care unit
Bloodstream infections
Hospital-acquired lower respiratory tract infections (lrtis)
Viral lower respiratory tract infections
Ventilator-associated pneumonia
Urinary tract infections
Surgical site infections
Ventriculostomy-related infections
Healthcare-associated diarrhea
Key references
Chapter 110: Pediatric sepsis
Pearls
Epidemiology
Definitions
Clinical presentation
Pathogenesis
Pathogen recognition and signal transduction
Cytokines as principal mediators of the sepsis response
Adhesion molecules
Nitric oxide
Coagulation cascade
Peroxisome proliferator-activated receptor-γ pathway
Myeloid-derived suppressor cells
Paradigm of sepsis as an adaptive immune problem
Genomic medicine and sepsis
Genetic influence and septic shock
Genome-wide expression profiling in children with septic shock
Treatment strategies
Initial resuscitation
Invasive monitoring
Elimination of pathogens
Maintenance of oxygen delivery
Additional management considerations
Immunomodulation
The case for more effective stratification in pediatric septic shock
Conclusion
Key references
References
Chapter 111: Multiple-organ dysfunction syndrome
Pearls
Pathophysiology and targeted therapies
Mitochondrial dysfunction
Innate and adaptive immune response
Microcirculatory dysfunction, ischemia-reperfusion injury
Epithelial dysfunction
Neurohumoral response
Multiple-organ dysfunction syndrome phenotypes
Multiple-organ dysfunction syndrome definitions
Multiple-organ dysfunction syndrome scoring
Epidemiology
Time course and outcomes
Conclusions and research perspectives
Key references
Section XII: Pediatric Critical Care: Environmental Injury and Trauma
Chapter 112: Bites and stings
Pearls
Snakebites
Epidemiology
Venomous snakes in the united states
Pathophysiology and clinical presentation
Emergency and critical care
Prehospital care
Assessment, stabilization, and disposition
Antivenin considerations
Antivenin administration
Other supportive care
Recurrence and therapeutic complications
Follow-up
Prognosis
Pitfalls
Resources
Spider bites
Epidemiology
Widow spiders
Recluse spiders
Pathophysiology and clinical presentation
Widow spider envenomation (latrodectism)
Recluse spider envenomation (loxoscelism)
Emergency and critical care
Latrodectism management
Loxoscelism management
Therapeutic complications
Follow-up
Prognosis
Pitfalls
Future directions
Resources
Scorpion stings
Epidemiology
Clinical presentation
Emergency and critical care
Hymenoptera stings (bees, wasps, and ants)
Massive envenomation
Emergency and critical care
Prognosis
Key references
Chapter 113: Hyperthermic injury
Pearls
Definitions
Epidemiology
Pathophysiology of heat-related illnesses
Acclimatization
Acute-phase response
Clinical features of heat stroke
Central nervous system
Cardiovascular
Pulmonary
Renal
Gastrointestinal
Metabolic
Hematologic
Infectious
Treatment
Key references
References
Chapter 114: Accidental hypothermia
Pearls
Physiology
Central nervous system
Cardiovascular
Respiratory
Renal
Coagulation
Treatment
Resuscitation
Rewarming
Outcome
Key references
Chapter 115: Drowning
Pearls
Definitions
Epidemiology
Pathophysiologic considerations
Type of aspirated fluid
Pulmonary effects
Cardiovascular effects
Central nervous system effects
Effects on other organ systems
Mammalian diving reflex
Preexisting associated conditions
Cold water drowning
Management
Management at the scene
Emergency department evaluation and stabilization
Management in the intensive care unit
Prognosis
Key references
Chapter 116: Burn and inhalation injury
Pearls
Types of burns
Flame burn
Scald burn
Electrical burn
Chemical burn
Normal skin anatomy
Depth and size of burn
Superficial burns
Superficial partial-thickness burns
Deep partial-thickness burns
Full-thickness burns
Zones of injury
Estimating the extent of the burn
Prehospital and early management
Transfer to burn centers
Pain management
Resuscitation
Colloid resuscitation
Complications of resuscitation
Treatment of burn wounds
Topical therapy
Surgery (excision and grafting)
Inhalation injury
Pathophysiology of inhalation injury
Diagnosis of inhalation injury
Management of inhalation injury
Ventilator support
Airway clearance and extubation criteria
Therapeutic adjuncts
Nutrition
Calculating and monitoring nutritional requirements
Enteral support
Parenteral support
Nutritional adjuncts
Rehabilitation
Reconstructions
Prevention
Key references
Chapter 117: Evaluation, stabilization, and initial management after trauma
Pearls
Prehospital care and trauma team activation
Trauma resuscitation
Primary survey
Overview
Establish an airway with cervical spine stabilization (a)
Breathing (b)
Circulation (c)
Disability (d)
Exposure/environment (e)
Secondary survey
Diagnostic assessment
Laboratory studies
Radiographic imaging
Emergency department thoracotomy
Stabilization and definitive care
Conclusions
Key references
Chapter 118: Traumatic brain injury
Pearls
Epidemiology
Pathophysiology
Posttraumatic ischemia
Excitotoxicity
Delayed neuronal death cascades
Cerebral swelling
Cerebral blood volume
Edema
Axonal injury
History
Signs and symptoms
Initial resuscitation
Rapid-sequence induction and intubation
Circulatory stabilization
Herniation
Transition from the emergency department to the pediatric intensive care unit: Computed tomographic scan and intracranial ...
Diagnostic studies and monitoring modalities
Computed tomography
Magnetic resonance imaging
Intracranial pressure monitoring
Advanced monitoring techniques
Monitoring cerebral blood flow
Monitoring cerebral metabolism
Treatment in the pediatric intensive care unit
Intracranial pressure and cerebral perfusion pressure thresholds
Baseline care
Treatment of intracranial hypertension: First-tier therapies
Ventricular cerebrospinal fluid drainage
Osmolar therapy
Sedation analgesia and neuromuscular blockade
Head position
Treatment of intracranial hypertension: Second-tier therapies
Barbiturates
Hyperventilation
Hypothermia
Decompressive craniectomy
Other therapies for refractory intracranial hypertension
Miscellaneous
Linking rehabilitation and acute care
Outcomes
Conclusion
Key references
Chapter 119: Pediatric thoracic trauma
Pearls
Epidemiology and prevention
Anatomic and physiologic considerations
Initial resuscitation and diagnosis
Chest wall injury
Lung and airway injury
Traumatic asphyxia
Cardiac injuries
Esophageal injury
Chylothorax
Diaphragmatic injury
Conclusions
Key references
Chapter 120: Pediatric abdominal trauma
Key words:
Pearls
Mechanisms and patterns of injury
Penetrating abdominal trauma
Recreational and sports injury
Wartime trauma
Evaluation and resuscitation
Physical examination
Laboratory tests
Radiographic assessment
Computed tomography
Sonography
Additional assessment tools
Diagnostic peritoneal lavage
Diagnostic laparoscopy
Management of specific abdominal injuries
Nonoperative management of solid-organ injuries
Embolization of solid-organ injuries
Injury to the spleen
Injury to the liver
Injury to the small bowel
Injury to the duodenum
Injury to the pancreas
Blunt abdominal aortic injury
Renal trauma
Bladder injuries
Pelvic fractures
Key references
Chapter 121: Child abuse
Pearls
Case example
Abuse VS. accident?
Epidemiology
Medical VS. forensic assessment
Sentinel injuries
Case example
Specific syndromes
Abusive head trauma
Presentation
Diagnosis
Retinal hemorrhage
Imaging
Intensive care unit/surgical considerations
Abdominal trauma
Alternative explanations of injuries
Birth
Subdural hematoma or retinal hemorrhage with valsalva
Brief resolved unexplained event
Cerebral venous sinus thrombosis
Benign enlargement of subarachnoid space
Other alternatives: Lumbar puncture, vaccines, bleeding, or bone mineralization disorders
Mandated reporting
Approach to parents
Case example
Multidisciplinary diagnosis
Coroner cases
Conclusions
Key references
Section XIII: Pediatric Critical Care: Pharmacology and Toxicology
Chapter 122: Principles of drug disposition
Pearls
Clinical pharmacokinetics
Clearance
Volume of distribution
Half-life
Bioavailability
Applied pharmacokinetics
Clinical pharmacodynamics
Determinants of effective therapy
Pharmacokinetic processes
Absorption
Enteral absorption
Sublingual/buccal absorption
Transdermal absorption
Intramuscular and subcutaneous absorption
Rectal absorption
Distribution
Age-dependent maturation in protein binding
Age-dependent maturation in body composition
Disease-dependent changes affecting drug distribution
Metabolism
Hepatic phase I metabolism
Cyp3a
Cyp2c
Cyp2d6
Cyp2b6
Other
Hepatic phase II metabolism
Intestinal metabolism
Elimination
Renal excretion
Biliary excretion
Pharmacodynamics changes
Critical care therapeutics
Antimicrobials
Analgesics and sedatives
Cardiovascular agents
Pulmonary agents
Anticoagulants
Steroids
Neuromuscular blockers
Conclusion
Key references
Chapter 123: Molecular mechanisms of drug actions
Pearls
Targets for drug action
Receptors
Ion channels
Enzymes
Carrier proteins
Receptor type and regulation
G protein–coupled receptors
Channel-linked receptors
Enzyme-linked receptors
Nuclear receptors
Receptor regulation
Signal transduction mechanisms: Intracellular messengers and effectors
Second messengers
Cyclic adenosine monophosphate
Cyclic guanosine monophosphate
Arachidonic acid and its metabolites
Diacylglycerol and inositol triphosphate
Calcium ions
Phosphorylation of proteins
Protein kinases
Calcium-binding proteins
Multiple drug targets within an organ system: The myocardium
Drug response and genetic polymorphisms
Genetic polymorphisms and drug disposition
Genetic polymorphisms, drug targets, and signaling mechanisms
Drug response and development
Drug response and disease
Conclusion
Key references
Chapter 124: Adverse drug reactions and drug-drug interactions
Pearls
Adverse drug reactions by organ system
Renal
Hepatic
Cardiovascular
Central nervous system
Hematologic
Endocrine and metabolic
Dermatologic
Drug-drug interactions
Pharmacodynamic drug-drug interactions
Drug-drug interactions by therapeutic class
Cardiovascular agents
Vasopressors
Antiarrhythmics
β-blockers
Calcium channel blockers
Angiotensin-converting enzyme inhibitors
Nitrates
Anticonvulsant medications
Antiinfective and antimicrobial agents
Anesthetic agents and sedatives
Analgesic agents
Anticoagulants
Immunosuppressive agents
Antineoplastic agents
Key references
Chapter 125: Principles of toxin assessment and screening
Pearls
Common agents involved in serious pediatric poisonings
Resources for the clinician
General assessment of the poisoned patient
History
Physical examination
Laboratory tests and toxin screens
Additional investigations
Radiographic studies
Diagnostic trials
Summary
Key references
Chapter 126: Toxidromes and their treatment
Pearls
Opioids
Sedative hypnotics
Sympathomimetic agents
Anticholinergic agents
Cholinergic agents
Methemoglobinemia
Xenobiotic-induced hyperthermia
Metabolic acidosis with increased anion gap
Toxin-induced seizures
Cardiovascular agents
Acetaminophen (paracetamol)
Key references
Section XIV: Pediatric Critical Care: Anesthesia Principles in the Pediatric Intensive Care Unit
Chapter 127: Airway management
Pearls
Anatomic considerations
Basic airway management
Nasopharyngeal airway
Oropharyngeal airways
Oxygen delivery devices
Nasal cannulas
Masks
Noninvasive positive-pressure ventilation
Establishing a functional airway
Endotracheal intubation
Indications
Respiratory failure
Hemodynamic instability
Neuromuscular dysfunction
Failure of central nervous system to regulate ventilatory drive and airway reflexes
Other indications
Physiologic effects of intubation
Recognition of a difficult airway
Process of intubation
Pharmacologic agents to facilitate intubation
Anticholinergic agents
Sedative and analgesic agents
Neuromuscular blocking agents
Preoxygenation
Orotracheal intubation
Nasotracheal intubation
Adjunctive airway approaches
Laryngeal mask airway
Video laryngoscopy
Flexible fiberoptic bronchoscopy
Cricothyrotomy, tracheostomy, and retrograde intubation
Extubation
Complications of endotracheal intubation
Prolonged intubation
Tracheostomy
Special circumstances
Full stomach
Increased intracranial pressure and neurologic dysfunction
Cervical spine instability
Upper airway obstruction
Facial and laryngotracheal injury
Open-globe injury
Key references
Chapter 128: Anesthesia effects on organ systems
Pearls
Anesthetic agents and regional anesthesia methods
Neurologic effects
Cardiovascular effects
Respiratory, gastrointestinal, and renal effects
Key references
Chapter 129: Anesthesia principles and operating room anesthesia regimens
Pearls
Preoperative evaluation
Nothing-by-mouth guidelines
Preoperative medication
Monitoring
Pharmacology of anesthetic agents
Local anesthetic agents
Intravenous anesthetic agents
Opioids
Volatile anesthetic agents and nitrous oxide
Neuromuscular blocking agents
Intraoperative anesthetic care
Maintenance anesthesia
Intraoperative fluid management
Postoperative care
Postoperative analgesia
Conclusions
Key references
Chapter 130: Malignant hyperthermia
Pearls
Pathophysiology
Genetics
Clinical recognition of malignant hyperthermia
Differential diagnosis
Course of malignant hyperthermia
Potential systemic complications
Management of an episode of malignant hyperthermia
Initial steps: Discontinue trigger agents and administer dantrolene
Further management: Expert consultation and symptomatic treatment
Urine and blood tests in malignant hyperthermia
Testing for malignant hyperthermia susceptibility
Muscle contracture testing
Less invasive tests of malignant hyperthermia susceptibility
Disorders associated with malignant hyperthermia
Muscular diseases and malignant hyperthermia
Heat illness and malignant hyperthermia susceptibility
Neuroleptic malignant syndrome
Malignant hyperthermia association and registry
Key references
Chapter 131: Neuromuscular blocking agents
Pearls
Neuromuscular junction
Neuromuscular blocking agents: Depolarizing agents
Neuromuscular blocking agents: Nondepolarizing agents
Pancuronium
Vecuronium
Rocuronium
Rapacuronium
Mivacurium
Atracurium
Cis-atracurium
Reversal of neuromuscular blockade
Acetylcholinesterase inhibitors
Sugammadex
Monitoring neuromuscular blockade
Adverse effects of neuromuscular blockade
Summary: Neuromuscular blocking agents in the picu
Key references
Chapter 132: Sedation and analgesia
Pearls
Posttraumatic stress disorder
Sedation scoring and assessment
Opioids and analgesia in the pediatric intensive care unit
Specific opioid agonists
Morphine
Meperidine
Fentanyl
Sufentanil
Alfentanil
Remifentanil
Codeine
Hydromorphone
Tramadol
Opiate antagonists
Incidental pain syndromes in the pediatric intensive care unit
Sickle cell crisis
Opiate tolerance
Rapid opiate detoxification
Benzodiazepines
Specific benzodiazepines
Diazepam
Midazolam
Lorazepam
Tolerance for and dependence on benzodiazepines
Flumazenil
Other agents for sedation in the pediatric intensive care unit
Chloral hydrate
Butyrophenones and phenothiazines
Haloperidol
Droperidol
Chlorpromazine
Lytic cocktail
Neuroleptic malignant syndrome
Baclofen
Cannabis
Dexmedetomidine
Propofol
Special issue regarding long-term infusion of propofol
Propofol infusion syndrome
Propofol tolerance and withdrawal
Ketamine
Etomidate
Inhalational anesthetic agents
Inhalation agent withdrawal syndrome
Sedation for children with preexisting cognitive impairment
Apoptosis and neurocognitive effects of anesthetic agents
Sedation-related complications
Pharmacoeconomics of analgesia and sedation in the pediatric intensive care unit
Key references
Chapter 133: Tolerance, dependency, and withdrawal
Pearls
History of tolerance and withdrawal in medical practice
Clinical signs and symptoms of withdrawal
Limitation of the development of tolerance and physical dependency
Incidence of tolerance and physical dependency
Scoring systems to identify withdrawal
Weaning from sedative and analgesic agents
Summary
Key references
Chapter 134: Pediatric delirium
Pearls
Background
Etiology
Epidemiology
Risk factors
Outcomes
Clinical presentation
Diagnosis
Treatment
Underlying illness
Iatrogenic factors
Environment
Pharmacotherapy
Prevention
Conclusion
Key references
Chapter 135: Procedural sedation for the pediatric intensivist
Differences between outpatient and inpatient sedation
Outpatient procedural sedation training during pediatric critical care fellowship
Sedation team structure
Classification of sedation
Equipment, monitoring, and rescue drugs
Sedation prescreening
Considerations in choosing commonly used medications
Nonpharmacologic approach to outpatient sedation
Sedation adverse events
Key references
Section XV: Pediatric Critical Care: Board Review Questions
Chapter 136: Board review questions
Chapter 1: History of pediatric critical care medicine
Chapter 2: High-reliability pediatric intensive care unit: role of intensivist and team in obtaining optimal outcomes
Chapter 3: Critical communication in the pediatric intensive care unit
Chapter 4: Professionalism in pediatric critical care
Chapter 5: Leading and managing change in the pediatric intensive care unit
Chapter 7: Fostering a learning healthcare environment in the pediatric intensive care unit
Chapter 8: Challenges of pediatric critical care in resource-poor settings
Chapter 9: Public health emergencies and emergency mass critical care
Chapter 10: Lifelong learning in pediatric critical care
Chapter 11: Essential concepts in clinical trial design and statistical analysis
Chapter 12: Prediction tools for short-term outcomes following critical illness in children
Chapter 13: Pediatric critical care transport
Chapter 14: Pediatric vascular access and centeses
Chapter 15: Ultrasonography in the pediatric intensive care unit
Chapter 16: Patient- and family-centered care in the pediatric intensive care unit
Chapter 17: Pediatric critical care ethics
Chapter 18: Ethical issues around death and dying
Chapter 19: Palliative care in the pediatric intensive care unit
Chapter 20: Organ donation process and management of the organ donor
Chapter 21: Long-term outcomes following critical illness in children
Chapter 22: Burnout and resiliency
Chapter 23: Structure and function of the heart
Chapter 24: Regional and peripheral circulation
Chapter 25: Endothelium and endotheliopathy
Chapter 26: Principles of invasive cardiovascular monitoring
Chapter 27: Assessment of cardiovascular function
Chapter 28: Cardiac failure and ventricular assist devices
Chapter 29: Echocardiographic imaging
Chapter 30: Diagnostic and therapeutic cardiac catheterization
Chapter 31: Pharmacology of the cardiovascular system
Chapter 32: Cardiopulmonary interaction
Chapter 33: Disorders of cardiac rhythm
Chapter 34: Shock states
Chapter 35: Pediatric cardiopulmonary bypass
Chapter 36: Critical care after surgery for congenital cardiac disease
Chapter 37: Cardiac transplantation
Chapter 38: Physiologic foundations of cardiopulmonary resuscitation
Chapter 39: Performance of cardiopulmonary resuscitation in infants and children
Chapter 40: Structure and development of the upper respiratory system
Chapter 41: Structure and development of the lower respiratory system
Chapter 42: Physiology of the respiratory system
Chapter 43: Noninvasive respiratory monitoring and assessment of gas exchange
Chapter 44: Overview of breathing failure
Chapter 45: Ventilation/perfusion inequality
Chapter 46: Mechanical dysfunction of the respiratory system
Chapter 47: Diseases of the upper respiratory tract
Chapter 48: Pediatric acute respiratory distress syndrome and ventilator-associated lung injury
Chapter 49: Acute viral bronchiolitis
Chapter 50: Asthma
Chapter 51: Neonatal pulmonary disease
Chapter 52: Pneumonitis and interstitial disease
Chapter 53: Diseases of the pulmonary circulation
Chapter 54: Mechanical ventilation and respiratory care
Chapter 55: Noninvasive ventilation in the pediatric intensive care unit
Chapter 56: Extracorporeal life support
Chapter 57: Pediatric lung transplantation
Chapter 58: Structure, function, and development of the nervous system
Chapter 59: Critical care considerations for common neurosurgical conditions
Chapter 60: Neurologic assessment and monitoring
Chapter 61: Neuroimaging
Chapter 62: Coma and depressed sensorium
Chapter 63: Intracranial hypertension and monitoring
Chapter 64: Status epilepticus
Chapter 65: Anoxic ischemic encephalopathy
Chapter 66: Pediatric stroke and intracerebral hemorrhage
Chapter 67: Central nervous system infections and related conditions
Chapter 68: Acute neuromuscular disease and disorders
Chapter 70: Renal structure and function
Chapter 71: Fluid and electrolyte issues in pediatric critical illness
Chapter 72: Acid-base balance in critical illness
Chapter 73: Tests of kidney function in children
Chapter 74: Glomerulotubular dysfunction and acute kidney injury
Chapter 75: Pediatric renal replacement therapy in the intensive care unit
Chapter 76: Pediatric renal transplantation
Chapter 77: Renal pharmacology
Chapter 78: Hypertensive urgencies and emergencies
Chapter 79: Cellular respiration
Chapter 80: Biology of the stress response
Chapter 81: Inborn errors of metabolism
Chapter 82: Genetic variation in health and disease
Chapter 83: Molecular foundations of cellular injury
Chapter 84: Endocrine emergencies
Chapter 85: Diabetic ketoacidosis
Chapter 86: Structure and function of the hematopoietic organs
Chapter 87: The erythron
Chapter 88: Hemoglobinopathies
Chapter 89: Coagulation and coagulopathy
Chapter 90: Thrombosis in pediatric critical care
Chapter 91: Transfusion medicine
Chapter 92: Hematology and oncology problems
Chapter 93: Critical illness in children undergoing hematopoietic progenitor cell transplantation
Chapter 94: Gastrointestinal structure and function
Chapter 95: Disorders of the gastrointestinal system
Chapter 96: Acute liver failure
Chapter 97: Hepatic transplantation
Chapter 98: Acute abdomen
Chapter 99: Nutrition of the critically ill child
Chapter 100: Innate immunity
Chapter 101: Adaptive immunity
Chapter 102: Critical illness and the microbiome
Chapter 103: Congenital immunodeficiencies
Chapter 104: Acquired immune dysfunction
Chapter 105: Immune balance in critical illness
Chapter 106: Pediatric rheumatologic disease
Chapter 107: Bacterial and fungal infections
Chapter 108: Life-threatening viral diseases and their treatment
Chapter 109: Healthcare-associated infections
Chapter 110: Pediatric sepsis
Chapter 111: Multiple organ dysfunction syndrome
Chapter 112: Bites and stings
Chapter 113: Hyperthermic injury
Chapter 114: Hypothermic injury
Chapter 115: Drowning
Chapter 116: Burn and inhalation injuries in children
Chapter 117: Evaluation, stabilization, and initial management after trauma
Chapter 118: Traumatic brain injury
Chapter 119: Pediatric thoracic trauma
Chapter 120: Pediatric abdominal trauma
Chapter 121: Child abuse
Chapter 122: Principles of drug disposition
Chapter 123: Molecular mechanisms of drug action
Chapter 124: Adverse drug reactions and drug-drug interactions
Chapter 125: Principles of toxin assessment and screening
Chapter 126: Toxidromes and their treatment
Chapter 127: Airway management
Chapter 128: Anesthesia effects on organ systems
Chapter 129: Anesthesia principles and operating room anesthesia regimens
Chapter 130: Malignant hyperthermia
Chapter 131: Neuromuscular blocking agents
Chapter 132: Sedation and analgesia
Chapter 133: Tolerance, dependency, and withdrawal
Chapter 134: Pediatric delirium
Chapter 135: Procedural sedation for the pediatric intensivist
Inside Back Cover

Citation preview

FUHRMAN

&

ZIMMERMAN’S

PEDIATRIC CRITICAL CARE

SIXTH EDITION

FUHRMAN & ZIMMERMAN’S

PEDIATRIC

CRITICAL CARE JERRY J. ZIMMERMAN, MD, PhD, FCCM

ALEXANDRE T. ROTTA, MD, FCCM

Robert S.B. Clark, MD

Sapna Ravi Kudchadkar, MD, PhD, FCCM

Faculty, Pediatric Critical Care Medicine, Seattle Children’s Hospital, Harborview Medical Center, University of Washington School of Medicine, Seattle, Washington

Professor and Vice Chair, Critical Care Medicine, University of Pittsburgh School of Medicine; Associate Director, Safar Center for Resuscitation Research, University of Pittsburgh, Pittsburgh, Pennsylvania

Bradley P. Fuhrman, MD, FAAP, MCCM

Professor of Pediatrics in Medical Education, Clinical Educator and Gold College Mentor, Paul L. Foster School of Medicine, Texas Tech University HSC El Paso, El Paso, Texas

Division Chief, Pediatric Critical Care Medicine Duke Children’s Hospital; Professor of Pediatrics; Duke University School of Medicine, Durham, North Carolina

Associate Professor, Anesthesiology and Critical Care Medicine, Pediatrics, and Physical Medicine and Rehabilitation, Charlotte R. Bloomberg Children’s Center, Johns Hopkins University School of Medicine, Baltimore, Maryland

Monica Relvas, MD, FAAP, FCCM, MSHA Medical Director, Pediatric Critical Care Medicine, Covenant Children’s Hospital, Associate Clinical Professor, Texas Tech University, Lubbock, Texas

Joseph D. Tobias, MD

Chair, Department of Anesthesiology and Pain Medicine, Nationwide Children’s Hospital; Professor of Anesthesiology and Pediatrics, The Ohio State University, Columbus, Ohio

Elsevier 1600 John F. Kennedy Blvd. Ste 1600 Philadelphia, PA 19103-2899

FUHRMAN AND ZIMMERMAN’S PEDIATRIC CRITICAL CARE, SIXTH EDITION 

ISBN: 978-0-323-67269-6

Copyright © 2022 by Elsevier, Inc. All rights reserved. No part of this publication may be reproduced or transmitted in any form or by any means, electronic or mechanical, including photocopying, recording, or any information storage and retrieval system, without permission in writing from the publisher. Details on how to seek permission, further information about the Publisher’s permissions policies and our arrangements with organizations such as the Copyright Clearance Center and the Copyright Licensing Agency, can be found at our website: www.elsevier.com/permissions. This book and the individual contributions contained in it are protected under copyright by the Publisher (other than as may be noted herein).

Notice Practitioners and researchers must always rely on their own experience and knowledge in evaluating and using any information, methods, compounds or experiments described herein. Because of rapid advances in the medical sciences, in particular, independent verification of diagnoses and drug dosages should be made. To the fullest extent of the law, no responsibility is assumed by Elsevier, authors, editors or contributors for any injury and/or damage to persons or property as a matter of products liability, negligence or otherwise, or from any use or operation of any methods, products, instructions, or ideas contained in the material herein. Previous editions copyrighted 2017, 2011, 2006, 1998, 1992 by Elsevier, Inc. Library of Congress Control Number: 2020930336

Content Strategist: Michael Houston Senior Content Development Specialist: Jennifer Ehlers Publishing Services Manager: Catherine Jackson Senior Project Manager/Specialist: Carrie Stetz Design Direction: Patrick Ferguson

Printed in Canada Last digit is the print number:  9  8  7  6  5  4  3  2  1

Contributors

Isaac Josh Abecassis, MD Resident Physician Department of Neurological Surgery University of Washington Seattle, Washington

Ben D. Albert, MD Program Director, Critical Care Medicine Fellowship Department of Anesthesiology, Critical Care, and Pain Medicine Boston Children’s Hospital Boston, Massachusetts

Jayani Abeysekera, MD, FRCP Assistant Professor Department of Pediatrics Division of Cardiology Dalhousie University/IWK Health Centre Halifax, Nova Scotia, Canada

Alicia Alcamo, MD, MPH Assistant Professor of Critical Care and Pediatrics Department of Anesthesiology and Critical Care Medicine University of Pennsylvania Perelman School of Medicine Children’s Hospital of Philadelphia Philadelphia, Pennsylvania

P. David Adelson, MD, FAAP, FACS, FAANS Diane and Bruce Halle Chair of Children’s Neurosciences Director, Barrow Neurological Institute at Phoenix Children’s Hospital Professor, Department of Child Health University of Arizona College of Medicine; Professor, Department of Neurosurgery Mayo Clinic Phoenix, Arizona

Matthew N. Alder, MD, PhD Assistant Professor Critical Care Medicine Cincinnati Children’s Hospital Medical Center Cincinnati, Ohio

Rachel S. Agbeko, FRCPCH, PhD Consultant Paediatric Intensive Care Unit Great North Children’s Hospital Newcastle upon Tyne Hospitals NHS Trust Newcastle upon Tyne, United Kingdom Michael S.D. Agus, MD Chief of Medical Critical Care Department of Pediatrics Division of Medical Critical Care Boston Children’s Hospital Boston, Massachusetts Mubbasheer Ahmed, MD Cardiac Intensive Care Unit Texas Medical Center Texas Children’s Hospital Houston, Texas Alireza Akhondi-Asl, PhD Departments of Anesthesiology, Critical Care, and Pain Medicine Division of Critical Care Boston Children’s Hospital Boston, Massachusetts

Omar Alibrahim, MD, FAAP Chief, Pediatric Critical Care Division John R. Oishei Children’s Hospital Associate Professor of Pediatrics Jacob’s School of Medicine University of Buffalo Buffalo, New York Veerajalandhar Allareddy, MBBS, MBA Section Chief, Pediatric Cardiac Intensive Care Duke University Medical Center Professor of Pediatrics Duke University School of Medicine Durham, North Carolina Melvin C. Almodovar, MD The George E. Batchelor Chair in Pediatric Cardiology Chief, Pediatric Cardiology University of Miami Miller School of Medicine; Director, Children’s Heart Center Director, Cardiac Intensive Care Hotz Children’s Hospital, Jackson Health System Miami, Florida Alexandra Aminoff, MD Acting Assistant Professor Department of Pediatric Rheumatology Seattle Children’s Hospital Seattle, Washington

v

vi

Contributors

Catherine Amlie-Lefond, MD Professor Department of Neurology University of Washington Seattle, Washington

Adnan M. Bakar, MD Assistant Professor of Pediatrics Section Head, Pediatric Cardiac Critical Care Cohen Children’s Medical Center of Hofstra/Northwell New Hyde Park, New York

Rajesh Aneja, MD Clinical Chief, Division of Pediatric Critical Care Medicine Medical Director, Pediatric Intensive Care Unit Children’s Hospital of Pittsburgh of UPMC; Professor Department of Critical Care Medicine and Pediatrics University of Pittsburgh School of Medicine Pittsburgh, Pennsylvania

Katherine Banker, MD Clinical Assistant Professor Department of Pediatrics, Critical Care Seattle Children’s Hospital Seattle, Washington

Abigail Apple, ARNP Pediatric Critical Care Seattle Children’s Hospital Seattle, Washington Andrew C. Argent, MD Professor School of Child and Adolescent Health University of Cape Town; Medical Director Paediatric Intensive Care Red Cross War Memorial Children’s Hospital Cape Town, South Africa Joan C. Arvedson, PhD Program Coordinator, Feeding & Swallowing Services Speech Pathology and Audiology Children’s Hospital of Wisconsin; Clinical Professor Department of Pediatrics Medical College of Wisconsin Milwaukee, Wisconsin François Aspesberro, MD Pediatric Cardiothoracic ICU Miller Children’s & Women’s Hospital Long Beach Long Beach, California Nir Atlas, MD Clinical Fellow Division of Pediatric Critical Care Children’s Healthcare of Atlanta–Egleston Emory University School of Medicine Atlanta, Georgia John E. Baatz, PhD Professor Department of Pediatrics Medical University of South Carolina Charleston, South Carolina Harris P. Baden, MD Professor and Chief Pediatric Cardiac Critical Care University of Washington Seattle Children’s Hospital Seattle, Washington

Piers C.A. Barker, MD Professor of Pediatrics and Obstetrics/Gynecology Department of Pediatrics Duke University School of Medicine Durham, North Carolina Lee M. Bass, MD Associate Professor of Pediatrics Departments of Gastroenterology, Hepatology & Nutrition Ann & Robert H. Lurie Children’s Hospital of Chicago Northwestern University Feinberg School of Medicine Chicago, Illinois Rajit K. Basu, MD, MS, FCCM Associate Professor of Pediatrics Emory School of Medicine; Research Director Division of Pediatric Critical Care Medicine Children’s Healthcare of Atlanta Atlanta, Georgia Hülya Bayir, MD Safar Center for Resuscitation Research Department of Critical Care Medicine University of Pittsburgh School of Medicine Children’s Hospital of Pittsburgh of UPMC Pittsburgh, Pennsylvania Lance B. Becker, MD Chair, Emergency Medicine Donald and Barbara Zucker School of Medicine at Hofstra/Northwell Hempstead, New York Jamie L. Bell, MD Assistant Professor of Pediatrics Department of Pediatrics Children’s Hospital of Michigan Wayne State University School of Medicine Detroit, Michigan Michael J. Bell, MD Chief, Pediatric Critical Care Medicine Department of Pediatrics Children’s National Medical Center Washington, DC

Contributors

Melania M. Bembea, MD, MPH, PhD Associate Professor Department of Anesthesiology and Critical Care Medicine Johns Hopkins University School of Medicine Baltimore, Maryland

Naomi B. Bishop, MD Assistant Professor Department of Pediatric Critical Care Medicine Weill Cornell Medical College New York, New York

M.A. Bender, MD, PhD Director, Sickle Cell and Hemoglobinopathy Program Odessa Brown Children’s Clinic; Associate Professor Department of Pediatrics University of Washington Seattle, Washington

Julie Blatt, MD Professor Pediatric Hematology Oncology University of North Carolina Chapel Hill, North Carolina

Alexis L. Benscoter, DO Assistant Professor of Clinical Pediatrics University of Cincinnati College of Medicine; Department of Cardiac Critical Care Medicine Division of Cardiology Cincinnati Children’s Hospital Medical Center Cincinnati, Ohio Wade W. Benton, PharmD Chief Development Officer Eicos Sciences, Inc. San Mateo, California Robert A. Berg, MD Professor Departments of Anesthesiology, Critical Care Medicine, and Pediatrics University of Pennsylvania Perelman School of Medicine; Division Chief, Critical Care Medicine Departments of Anesthesiology and Critical Care Medicine Children’s Hospital of Philadelphia Philadelphia, Pennsylvania Emily Berkman, MD Assistant Professor Departments of Pediatrics and Bioethics and Humanities University of Washington School of Medicine; Division of Pediatric Critical Care Department of Pediatric Bioethics Treuman Katz Center for Pediatric Bioethics Seattle Children’s Hospital Seattle, Washington Carol Berkowitz, MD Chief, Division of General Pediatrics Department of Pediatrics Harbor-UCLA Medical Center Torrance, California; Distinguished Professor of Pediatrics David Geffen School of Medicine at UCLA Los Angeles, California Katherine V. Biagas, MD Associate Professor Department of Pediatrics Vice Chairman for Faculty Development Stony Brook Children’s and the Renaissance School of Medicine Stony Brook, New York

Lauren Bodilly, MD Critical Care Fellow Department of Pediatric Critical Care Cincinnati Children’s Hospital Cincinnati, Ohio Robert H. Bonow, MD Resident Physician Department of Neurological Surgery University of Washington Seattle, Washington E. Alexis Bragg, MD Assistant Professor of Anesthesiology and Pediatrics Department of Anesthesia and Critical Care Medicine Children’s Hospital Los Angeles Keck School of Medicine–University of Southern California Los Angeles, California Barbara W. Brandom, MD Retired Professor Department of Anesthesiology University of Pittsburgh; Retired Director North American MH Registry of MHAUS Pittsburgh, Pennsylvania Richard J. Brilli, MD Chief Medical Officer Hospital Administration Nationwide Children’s Hospital; Professor Department of Pediatrics Division of Critical Care Medicine The Ohio State University College of Medicine Columbus, Ohio Thomas V. Brogan, MD Professor Department of Pediatrics University of Washington School of Medicine Seattle, Washington Ronald A. Bronicki, MD, FCCM, FACC Professor Departments of Critical Care Medicine and Cardiology Texas Children’s Hospital and Baylor College of Medicine Houston, Texas

vii

viii

Contributors

Samuel R. Browd, MD, PhD, FAANS, FAAP Professor Department of Neurological Surgery Seattle Children’s Hospital University of Washington Seattle, Washington Timothy E. Bunchman, MD Professor and Director, Pediatric Nephrology Children’s Hospital of Richmond Virginia Commonwealth University School of Medicine Richmond, Virginia Jeffrey P. Burns, MD, MPH Chief and Endowed Shapiro Chair of Critical Care Medicine Chair, Governance Committee of the Intensive Care Units Boston Children’s Hospital Professor of Anesthesia Harvard Medical School Boston, Massachusetts David F. Butler, MD Fellow Department of Pediatrics Division of Critical Care Seattle Children’s Hospital Seattle, Washington Derya Caglar, MD Fellowship Director, Pediatric Emergency Medicine Associate Professor, Pediatrics University of Washington School of Medicine Attending Physician Division of Pediatric Emergency Medicine Seattle Children’s Hospital Seattle, Washington Michael W. Camitta, MD Associate Professor of Pediatrics Department of Pediatrics Duke University School of Medicine Durham, North Carolina M. Jay Campbell, MD, MHA Associate Professor of Pediatrics Department of Pediatrics Duke University School of Medicine Durham, North Carolina Sally Campbell, MBBS(Hons), FRACP, FRCPA Paediatric Haematologist Royal Children’s Hospital Melbourne, Australia Karel D. Capek, MD Fellow Acute Burn, Critical Care, and Reconstruction Shriners Hospitals for Children Galveston and the University of Texas Medical Branch Galveston, Texas

Michael P. Carboni, MD Medical Director, Pediatric Heart Failure & Transplant Department of Pediatrics Duke Children’s Hospital Durham, North Carolina Joseph A. Carcillo, MD Professor of Critical Care Medicine and Pediatrics Children’s Hospital of Pittsburgh University of Pittsburgh School of Medicine Pittsburgh, Pennsylvania Antonio Cassara, MBChB Associate Professor Department of Anesthesia Ruby Memorial Hospital Morgantown, West Virginia Nina Censoplano, MD Attending physician Pediatric Critical Care Medicine INOVA Fairfax Medical Center Falls Church, Virginia Victoria Chadwick, PharmD, BCCCP Critical Care Pharmacist Seattle Children’s Hospital Seattle, Washington Reid C. Chamberlain, MD Pediatric Cardiology Fellow Department of Pediatrics Duke University Hospital Durham, North Carolina Anny Chan, PharmD Hematopoietic Stem Cell Transplant Clinical Pharmacist Seattle Children’s Hospital Seattle, Washington John R. Charpie, MD, PhD Amnon Rosenthal Professor & Director of Pediatric Cardiology University of Michigan Medical School Co-Director, University of Michigan Congenital Heart Center C.S. Mott Children’s Hospital Michigan Medicine Ann Arbor, Michigan Ira M. Cheifetz, MD Professor Departments of Pediatrics and Anesthesiology Duke University Medical Center Durham, North Carolina Saurabh Chiwane, MD Assistant Professor Department of Pediatrics Saint Louis University St. Louis, Missouri

Contributors

Robert H. Chun, MD Associate Professor Department of Pediatric Otolaryngology Medical College of Wisconsin Milwaukee, Wisconsin

Craig M. Coopersmith, MD Professor Department of Surgery Emory University Atlanta, Georgia

Jeff Clark, MD Staff Intensivist Pediatric Critical Care Medicine Vice Chair of Pediatrics Ascension St. John Children’s Hospital Detroit, Michigan

Seth J. Corey, MD, MPH Professor Departments of Pediatrics and Molecular Medicine Cleveland Clinic Cleveland, Ohio

Jonna D. Clark, MD Associate Professor Departments of Pediatrics and Critical Care Medicine University of Washington; Faculty Treuman Katz Center for Pediatric Bioethics Seattle Children’s Hospital Seattle, Washington Robert S.B. Clark, MD Professor and Vice Chair Critical Care Medicine University of Pittsburgh School of Medicine; Associate Director, Safar Center for Resuscitation Research University of Pittsburgh Pittsburgh, Pennsylvania April Clawson, MD Fellow Pediatric Emergency Medicine University of Arkansas Medical Center Little Rock, Arkansas Jason A. Clayton, MD, PhD Assistant Professor Department of Pediatrics Rainbow Babies & Children’s Hospital Cleveland, Ohio Thomas Conlon, MD Assistant Professor Department of Pediatric Critical Care Medicine The Children’s Hospital of Philadelphia Philadelphia, Pennsylvania Carol Conrad, MD Associate Professor Department of Pediatrics Stanford University Palo Alto, California Edward E. Conway Jr, MD, MS Chairman and Pediatrician-in-Chief Milton and Bernice Stern Department of Pediatrics Beth Israel Medical Center; Professor of Pediatrics Icahn School of Medicine at Mount Sinai New York, New York

Mary K. Dahmer, PhD Associate Professor Department of Pediatrics Division of Critical Care University of Michigan Medical School Ann Arbor, Michigan Heidi J. Dalton, MD, MCCM, FELSO Director of ECMO Program Development and Research INOVA Fairfax Medical Center Falls Church, Virginia; Professor of Pediatrics Virginia Commonwealth University Richmond, Virginia; Professor of Clinical Surgery George Washington University Washington, DC Rahul C. Damania, MD, FAAP Clinical Fellow Division of Pediatric Critical Care Children’s Healthcare of Atlanta–Egleston Emory University School of Medicine Atlanta, Georgia Mihaela A. Damian, MD, MPH Clinical Assistant Professor Department of Pediatrics Stanford University Palo Alto, California Lauren Dartois, PharmD Critical Care Clinical Pharmacist Seattle Children’s Hospital Seattle, Washington Peter J. Davis, MD, FAAP Professor Departments of Anesthesiology and Pediatrics University of Pittsburgh School of Medicine; Anesthesiologist-in-Chief Department of Anesthesiology Children’s Hospital of Pittsburgh of UPMC Pittsburgh, Pennsylvania

ix

x

Contributors

Leslie A. Dervan, MD Assistant Professor Department of Pediatrics Seattle Children’s Hospital University of Washington School of Medicine Seattle, Washington Clifford S. Deutschman, MS, MD, MCCM Vice Chair, Research Department of Pediatrics Professor of Pediatrics and Molecular Medicine Donald and Barbara Zucker School of Medicine at Hofstra/Northwell Hempstead, New York; Cohen Children’s Medical Center New York, New York; Professor, Elmezzi Graduate School of Molecular Medicine Professor, Feinstein Institutes for Medical Research Manhasset, New York; Emeritus Professor of Anesthesiology & Critical Care and Surgery Perelman School of Medicine at the University of Pennsylvania Philadelphia, Pennsylvania Cameron Dezfulian, MD, FAHA Director, Adult Congenital Heart Disease ICU Critical Care Medicine Section Texas Children’s Hospital; Senior Faculty Baylor College of Medicine Houston, Texas André A.S. Dick, MD, MPH Associate Professor of Surgery ASTS Transplant Fellowship Director Surgical Director, Pediatric Kidney Transplant Department of Surgery Division of Transplantation Seattle Children’s Hospital University of Washington School of Medicine Seattle, Washington Douglas S. Diekema, MD, MPH Professor Department of Pediatrics University of Washington; Director of Education Treuman Katz Center for Pediatric Bioethics Seattle Children’s Research Institute Seattle, Washington Michael Dingeldein, MD Trauma Medical Director Rainbow Babies & Children’s Hospital Assistant Professor of Surgery Case Western Reserve University Cleveland, Ohio

Allan Doctor, MD Professor of Pediatrics (Critical Care) Director, Center for Blood Oxygen Transport and Hemostasis University of Maryland School of Medicine Baltimore, Maryland John J. Downes, MD Professor Emeritus of Anesthesiology and Critical Care, and Pediatrics University of Pennsylvania Perelman School of Medicine Chair Emeritus of Anesthesiology and Critical Care Medicine Children’s Hospital of Philadelphia Philadelphia, Pennsylvania Christine Duncan, MD Associate Clinical Director of Pediatric HSCT Senior Physician Dana Farber/Boston Children’s Cancer and Blood Disorders Center; Assistant Professor of Pediatrics Harvard Medical School Boston, Massachusetts Christopher M. Edwards, MD Assistant Professor Department of Anesthesiology University of Florida College of Medicine Gainesville, Florida Lauren R. Edwards, MD Assistant Professor Pediatric Critical Care University of Arkansas for Medical Sciences/Arkansas Children’s Hospital Little Rock, Arkansas Chinyere Egbuta, MD Instructor Department of Anesthesia Harvard Medical School; Associate Professor Anesthesia and Critical Care Medicine Boston Children’s Hospital Boston, Massachusetts Howard Eigen, MD Professor of Pediatrics, Retired Department of Pediatrics Indiana University School of Medicine Indianapolis, Indiana Hannah Laure Elfassy, MD, FRCP(c) Chief Department of Allergy and Clinical Immunology Sacré Coeur Hospital Montreal, Quebec, Canada

Contributors

Alison M. Ellis, MD, MBA Assistant Professor Anesthesia and Perioperative Medicine Charleston, South Carolina Idris V.R. Evans, MD Assistant Professor Department of Critical Care Medicine University of Pittsburgh Pittsburgh, Pennsylvania Reid W.D. Farris, MD Associate Professor Department of Pediatrics Division of Critical Care Medicine University of Washington Seattle, Washington Jeffrey R. Fineman, MD Professor Department of Pediatrics University of California, San Francisco San Francisco, California Ericka L. Fink, MD Associate Professor of Critical Care Medicine University of Pittsburgh School of Medicine Children’s Hospital of Pittsburgh of UPMC; Associate Director Safar Center for Resuscitation Research University of Pittsburgh Medical Center Pittsburgh, Pennsylvania Frank A. Fish, MD Professor Department of Pediatrics Vanderbilt Medical Center Nashville, Tennessee Tamara N. Fitzgerald, MD, PhD Assistant Professor Department of Surgery Duke University Durham, North Carolina Gregory A. Fleming, MD, MSCI, FSCAI Associate Professor Department of Pediatrics Duke University Durham, North Carolina Saul Flores, MD Assistant Professor Pediatrics/Critical Care Baylor College of Medicine; Attending Physician Texas Children’s Hospital Houston, Texas

Joseph T. Flynn, MD Chief, Division of Nephrology Department of Pediatrics Seattle Children’s Hospital; Professor Department of Pediatrics University of Washington School of Medicine Seattle, Washington Michael L. Forbes, MD, FCCM Professor and Associate Chair Department of Pediatrics Northeast Ohio Medical University; Director, Critical Care Research & Outcomes Analysis Akron Children’s Hospital Akron, Ohio Joseph M. Forbess, MD Department of Surgery Northwestern University Feinberg School of Medicine Chicago, Illinois Deborah E. Franzon, MD Clinical Professor Department of Pediatric Medicine Division of Critical Care Medicine University of California, San Francisco San Francisco, California W. Joshua Frazier, MD Assistant Professor of Pediatrics Critical Care Medicine Nationwide Children’s Hospital Columbus, Ohio Bradley P. Fuhrman, MD, FAAP, MCCM Professor of Pediatrics in Medical Education Clinical Educator and Gold College Mentor Paul L. Foster School of Medicine Texas Tech University HSC El Paso El Paso, Texas Richard M. Ginther Jr, MBA, BS Pediatric Perfusionist Pediatric Cardiothoracic Surgery UT Southwestern Medical Center Dallas, Texas Nicole Glaser, MD Professor Department of Pediatrics Section of Endocrinology and Diabetes University of California, Davis Sacramento, California Ana Lia Graciano, MD, FCCM Professor of Pediatrics Medical Director CICU Pediatric Critical Care Medicine University of Maryland Children’s Hospital Baltimore, Maryland

xi

xii

Contributors

Megan M. Gray, MD Assistant Professor of Pediatrics Department of Pediatrics Division of Neonatology Fellowship Program Director for Neonatal-Perinatal Medicine Seattle Children’s Hospital University of Washington School of Medicine Seattle, Washington Kristin C. Greathouse, CPNP-AC, PhD Heart Center University of Minnesota Masonic Children’s Hospital Minneapolis, Minnesota Bruce M. Greenwald, MD, FAAP, FCCM Professor of Clinical Pediatrics Executive Vice-Chair, Department of Pediatrics Division of Pediatric Critical Care Medicine Weill Cornell Medical College New York, New York Matthew M. Grinsell, MD, PhD Associate Professor Department of Pediatrics University of Utah Salt Lake City, Utah Jocelyn R. Grunwell, MD Assistant Professor Department of Pediatrics Division of Critical Care Medicine Emory University/Children’s Healthcare of Atlanta Atlanta, Georgia Björn Gunnarsson, MD Department of Research Norwegian Air Ambulance Foundation Drøbak, Norway SAR Helicopter Ørland Main Air Station, Norway Marla Guzman, MD Fellow Pediatric Rheumatology Cohen Children’s Medical Center-Zucker School of Medicine at Hofstra/Northwell New Hyde Park, New York Timothy Hahn, MD Fellow Department of Pediatrics Penn State Children’s Hospital Hershey, Pennsylvania Mark W. Hall, MD Chief, Division of Critical Care Medicine Department of Pediatrics Nationwide Children’s Hospital Columbus, Ohio

Cary O. Harding, MD Professor Department of Molecular and Medical Genetics Oregon Health & Science University Portland, Oregon Mary E. Hartman, MD, MPH Associate Professor Department of Pediatrics Washington University in St. Louis St. Louis, Missouri Silvia M. Hartmann, MD Assistant Professor Critical Care Medicine Seattle Children’s Hospital Seattle, Washington Kevin M. Havlin, MD Assistant Professor Department of Pediatrics Division of Pediatric Critical Care University of Louisville Louisville, Kentucky Kristen Hayward, MD Associate Professor Department of Pediatrics Division of Rheumatology Seattle Children’s Hospital University of Washington School of Medicine Seattle, Washington Patrick J. Healey, MD Professor Department of Surgery University of Washington School of Medicine Division Chief, Pediatric Transplant Surgery Seattle Children’s Hospital Seattle, Washington Christopher M.B. Heard, MBChB, FRCA Clinical Professor Department of Anesthesiology Clinical Professor Division of Pediatric Critical Care Oishei Children’s Hospital Buffalo, New York Julia A. Heneghan, MD Assistant Professor Pediatric Critical Care University of Minnesota Masonic Children’s Hospital Minneapolis, Minnesota David N. Herndon, MD Editor, Burn Care and Research American Burn Association Chicago, Illinois

Contributors

Lynn J. Hernan, MD Associate Professor of Pediatrics Pediatric Clerkship Director Texas Tech University HSC El Paso Paul L. Foster School of Medicine El Paso, Texas

Laura Marie Ibsen, MD Professor of Pediatrics Division Chief, Pediatric Critical Care Department of Pediatrics Oregon Health and Sciences University Portland, Oregon

Kevin D. Hill, MD, MSCI Associate Professor Department of Pediatrics Duke University Medical Center Durham, North Carolina

Hanneke Ijsselstijn, MD, PhD Associate Professor of Pediatrics Department of Pediatric Surgery and Intensive Care Erasmus Medical Center, Sophia Children’s Hospital Rotterdam, The Netherlands

Julien I. Hoffman† Professor (Emeritus) Pediatrics University of California, San Francisco San Francisco, California

Travis C. Jackson, PhD Associate Professor of Molecular Pharmacology & Physiology University of South Florida Morsani College of Medicine USF Health Heart Institute Tampa, Florida

Paula Holinski, MD, FRCPC Clinical Assistant Professor of Anesthesiology University of Alberta Pediatric Anesthesia and Pediatric Cardiac Intensive Care Stollery Children’s Hospital Edmonton, Alberta, Canada Sue J. Hong, MD Assistant Professor Departments of Pediatrics and Neurology Northwestern University Lurie Children’s Hospital of Chicago Chicago, Illinois Simon Horslen, MB ChB Professor Department of Pediatrics University of Washington School of Medicine; Medical Director of Solid Organ Transplantation Department of Gastroenterology, Hepatology & Nutrition Seattle Children’s Hospital Seattle, Washington Aparna Hoskote, MBBS, DCH, MRCP, MD Consultant in Cardiac Intensive Care Honorary Senior Lecturer UCL Great Ormond Street Institute of Child Health Great Ormond Street Hospital for Children NHS Foundation Trust London, United Kingdom Justin C. Hotz, BSRT, RRT-NPS Senior Research Associate Department of Anesthesia and Critical Care Medicine Children’s Hospital Los Angeles Los Angeles, California



Deceased.

Shelina M. Jamal, MD Clinical Assistant Professor Pediatric Critical Care Alberta Children’s Hospital Calgary, Alberta, Canada Prashant Joshi, MD Associate Professor Department of Pediatrics Texas Tech University Health Sciences Center El Paso El Paso, Texas Emily L. Joyce, MD Assistant Professor Department of Pediatrics, Division of Nephrology UH Rainbow Babies and Children’s Hospital Case Western Reserve University School of Medicine Cleveland, Ohio Tom Kallay, MD Chief, Division of Pediatric Critical Care Medicine Director, Simulation and Educational Resource Center for Improved Patient Safety and Outcomes Harbor-UCLA Medical Center Torrance, California; Associate Professor of Clinical Pediatrics David Geffen School of Medicine Los Angeles, California Pradip P. Kamat, MD Medical Director, Sedation Children’s Healthcare of Atlanta Atlanta, Georgia Jason M. Kane, MD, MS, FAAP, FCCM Associate Professor of Pediatrics Department of Pediatric Critical Care Medicine University of Chicago Comer Children’s Hospital Chicago, Illinois

xiii

xiv

Contributors

Prince J. Kannankeril, MD Professor Department of Pediatrics Vanderbilt Children’s Hospital Nashville, Tennessee Oliver Karam, MD, PhD Division Chief John J. Mickell Endowed Associate Professor of Pediatrics Department of Pediatrics Children’s Hospital of Richmond at VCU Richmond, Virginia Katherine L. Kenningham, MD Fellow, Neonatal-Perinatal Medicine Department of Pediatrics Division of Neonatology Seattle Children’s Hospital University of Washington School of Medicine Seattle, Washington Hedieh Khalatbari, MD, MBA Assistant Professor of Pediatric Radiology Department of Radiology Seattle Children’s Hospital and University of Washington Seattle, Washington Robinder G. Khemani, MD, MsCI Associate Director of Research Department of Anesthesiology and Critical Care Medicine Children’s Hospital Los Angeles; Associate Professor of Pediatrics Keck School of Medicine University of Southern California Los Angeles, California Elizabeth Y. Killien, MD, MPH Acting Assistant Professor Department of Pediatrics Division of Pediatric Critical Care Medicine Seattle Children’s Hospital University of Washington Seattle, Washington Yun Kim, MS, OTR/L Pediatric Rehabilitation Clinical Specialist Department of Physical Medicine and Rehabilitation Johns Hopkins Children’s Center Baltimore, Maryland Jenny Kingsley, MD Fellow Pediatric Critical Care Medicine Treuman Katz Center for Pediatric Bioethics Seattle Children’s Hospital Seattle, Washington

Christa Jefferis Kirk, PharmD, BCCP Heart Center Clinical Pharmacy Specialist Seattle Children’s Hospital; Clinical Associate Professor School of Pharmacy University of Washington Seattle, Washington Sonya Kirmani, MD Medical Instructor Department of Pediatrics Duke University Durham, North Carolina Ruth Kleinpell, PhD, RN, APRN-BC Director, Center for Clinical Research and Scholarship Assistant Dean for Clinical Scholarship Vanderbilt University School of Nursing Nashville, Tennessee Patrick M. Kochanek, MD, MCCM Ake N. Grenvik Professor in Critical Care Medicine Department of Critical Care Medicine University of Pittsburgh School of Medicine Pittsburgh, Pennsylvania Keith C. Kocis, MD, MS Director, Pediatric Cardiac ICU Co-Director, Pediatric ECLS Inova Children’s Hospital Inova Heart and Vascular Institute; Professor of Pediatrics VCU School of Medicine Inova Campus Falls Church, Virginia Samuel A. Kocoshis, MD Professor Department of Pediatrics University of Cincinnati College of Medicine; Medical Director, Intestinal Transplantation and Intestinal Care Center Department of Gastroenterology, Hepatology, and Nutrition Cincinnati Children’s Hospital Medical Center Cincinnati, Ohio Ildiko H. Koves, MD, FRACP Associate Professor Department of Endocrinology and Diabetes Seattle Children’s Hospital Seattle, Washington Sapna R. Kudchadkar, MD, PhD Associate Professor of Anesthesiology and Critical Care Medicine, Pediatrics, and Physical Medicine and Rehabilitation Charlotte R. Bloomberg Children’s Center Johns Hopkins University School of Medicine Baltimore, Maryland

Contributors

Thomas J. Kulik, MD Senior Associate in Cardiology Department of Cardiology Boston Children’s Hospital Associate Professor of Pediatrics Harvard Medical School Boston, Massachusetts Jacques Lacroix, MD Professor Department of Pediatrics Sainte-Justine Hospital Montreal, Quebec, Canada Ruth Lebet, PhD, RN, PCNS-BC, CCNS-P Nurse Scientist Center for Pediatric Nursing Research & Evidence-Based Practice; Program Director, Pediatric & Neonatal CNS Programs University of Pennsylvania School of Nursing Philadelphia, Pennsylvania Amy Lee, MD Associate Professor Department of Neurological Surgery University of Washington Seattle, Washington Hallie Lenker, PT, DPT Pediatric Rehabilitation Team Coordinator Department of Physical Medicine and Rehabilitation Johns Hopkins Children’s Center Baltimore, Maryland Daniel L. Levin, MD Professor Emeritus Departments of Pediatrics and Anesthesia Children’s Hospital at Dartmouth Lebanon, New Hampshire Emily R. Levy, MD Assistant Professor Department of Pediatric and Adolescent Medicine Divisions of Pediatric Critical Care Medicine and Pediatric Infectious Diseases Mayo Clinic Rochester, Minnesota Mithya Lewis-Newby, MD, MPH Associate Professor Departments of Pediatrics and Bioethics and Humanities University of Washington School of Medicine; Division of Pediatric Cardiac Critical Care Treuman Katz Center for Pediatric Bioethics Seattle Children’s Hospital Seattle, Washington

xv

John C. Lin, MD Associate Professor Department of Pediatrics Washington University School of Medicine St. Louis, Missouri Aline Maddux, MD Assistant Professor of Pediatrics Department of Pediatric Critical Care University of Colorado Anschutz Medical Campus Aurora, Colorado Matthew P. Malone, MD Assistant Professor Pediatric Critical Care University of Arkansas for Medical Sciences/Arkansas Children’s Hospital Little Rock, Arkansas Mioara Manole, MD Associate Professor of Pediatrics University of Pittsburgh School of Medicine Children’s Hospital of Pittsburgh of UPMC; Associate Director Safar Center for Resuscitation Research University of Pittsburgh Medical Center Pittsburgh, Pennsylvania Anne Marsh, MD Associate Professor of Pediatrics Department of Hematology/Oncology & Bone Marrow Transplant University of Wisconsin School of Medicine and Public Health Madison, Wisconsin Richard J. Martin, MBBS Professor Departments of Pediatrics, Reproductive Biology, and Physiology & Biophysics Case Western Reserve University School of Medicine; Drusinsky-Fanaroff Professor and Director of Neonatal Research Pediatrics/Neonatology Rainbow Babies & Children’s Hospital Cleveland, Ohio Mudit Mathur, MD, MBA, FAAP, FCCM, CPPS Director, Pediatric Critical Care Director, Patient Safety and Risk Management Kaiser Permanente Fontana Medical Center; Associate Professor Department of Clinical Science Kaiser Permanente Bernard J. Tyson School of Medicine Pasadena, California Jennifer McArthur, MD Associate Member St. Jude’s Research Hospital Pediatric Critical Care Memphis, Tennessee; Adjunct Associate Professor of Pediatrics Medical College of Wisconsin Milwaukee, Wisconsin

xvi

Contributors

Christine McCusker, MD Associate Professor Department of Pediatrics McGill University; Director Division of Allergy, Immunology and Dermatology Montreal Children’s Hospital Montreal, Quebec Ruth A. McDonald, MD Professor of Pediatrics Division of Nephrology University of Washington Chief Medical Officer, Hospital Operations Seattle Children’s Hospital Seattle, Washington Nilesh M. Mehta, MD, FASPEN Professor of Anesthesia Harvard Medical School; Director of Quality and Outcomes Director of Critical Care Nutrition Associate Medical Director Division of Critical Care Medicine Chair, Critical Care Nutrition and Metabolism Department of Anesthesiology, Critical Care, and Pain Medicine Boston Children’s Hospital Boston, Massachusetts; Past President American Society for Parenteral & Enteral Nutrition Ann J. Melvin, MD, MPH Professor Department of Pediatrics Division of Infectious Disease Seattle Children’s Hospital University of Washington Seattle, Washington Shina Menon, MD Assistant Professor Department of Pediatrics Division of Nephrology Seattle Children’s Hospital University of Washington Seattle, Washington Paul Monagle, MBBS, MD Professor Department of Paediatrics University of Melbourne; Haematologist Royal Children’s Hospital; Group Leader Haematology Research Murdoch Children’s Research Institute Melbourne, Victoria, Australia

Ryan W. Morgan, MD, MTR Assistant Professor of Anesthesia, Critical Care, and Pediatrics Department of Anesthesiology and Critical Care Medicine University of Pennsylvania Philadelphia, Pennsylvania Peter F. Morgenstern, MD Assistant Professor Department of Neurosurgery Icahn School of Medicine at Mount Sinai New York, New York Michael J. Morowitz, MD Associate Professor Pediatric General and Thoracic Surgery UPMC Children’s Hospital of Pittsburgh Pittsburgh, Pennsylvania Wynne Morrison, MD, MBE Ingerman Endowed Chair in Palliative Care Attending Physician, Critical Care and Palliative Care Children’s Hospital of Philadelphia Associate Professor of Anesthesiology and Critical Care Perelman School of Medicine at the University of Pennsylvania Philadelphia, Pennsylvania Raj Munshi, MD Associate Professor Department of Pediatrics Seattle Children’s Hospital Seattle, Washington Jennifer A. Muszynski, MD, MPH Assistant Professor of Pediatrics Critical Care Medicine Nationwide Children’s Hospital Columbus, Ohio Vinay M. Nadkarni, MD Professor Departments of Anesthesiology, Critical Care, and Pediatrics University of Pennsylvania Perelman School of Medicine Philadelphia, Pennsylvania Jessica A. Naiditch, MD Trauma Medical Director Dell Children’s Medical Center of Central Texas; Assistant Professor of Surgery Department of Surgery & Perioperative Care Dell Medical School, University of Texas Austin; Pediatric Surgeon Austin Pediatric Surgery Austin, Texas Thomas A. Nakagawa, MD, FAAP, FCCM Professor Department of Pediatrics Division of Critical Care Medicine University of Florida College of Medicine; Medical Director, Pediatric Intensive Care Wolfson Children’s Hospital Jacksonville, Florida

Contributors

xvii

Vu Nguyen, MD Assistant Professor Department of Pediatric Critical Care Virginia Commonwealth University Richmond, Virginia

Todd Otteson, MD, MPH Chief Division of Pediatric Otolaryngology University Hospitals/Rainbow Babies and Children’s Hospital Cleveland, Ohio

Jenna R. Nickless, PharmD Critical Care Pharmacist Inpatient Pharmacy Seattle Children’s Hospital Seattle, Washington

Yves Ouellette, MD, PhD Assistant Professor Division of Critical Care Medicine Department of Pediatrics Mayo Clinic Rochester, Minnesota

Akira Nishisaki, MD Associate Director, Clinical Implementation Program, ICU Department of Anesthesiology and Critical Care Medicine The Children’s Hospital of Philadelphia Philadelphia, Pennsylvania Victoria F. Norwood, MD Robert J. Roberts Professor of Pediatrics Chief of Pediatric Nephrology Department of Pediatrics University of Virginia Charlottesville, Virginia Daniel A. Notterman, MA, MD Professor Department of Molecular Biology Princeton University Princeton, New Jersey Peter Oishi, MD Professor of Pediatrics University of California, San Francisco; Executive Medical Director Inpatient Services and Regional Access UCSF Beinoff Children’s Hospital San Francisco, California Jeffrey Ojemann, MD Professor Department of Neurological Surgery University of Washington Seattle, Washington Michelle L. Olson, MD Department of Pediatric Critical Care Children’s Hospital of Richmond at VCU Richmond, Virginia Jessie O’Neal, PharmD, BCCCP Critical Care Pharmacist Seattle Children’s Hospital Seattle, Washington Kirsten Orloff, MD Pediatric Critical Care Medicine Fellow Department of Pediatrics Duke University Hospital Durham, North Carolina

Daiva Parakininkas, MD Associate Professor Department of Pediatrics Medical College of Wisconsin Milwaukee, Wisconsin Robert I. Parker, MD Professor Emeritus Department of Pediatrics Stony Brook Children’s Hospital Stony Brook University Renaissance School of Medicine Stony Brook, New York Katherine Ratzen Peeler, MD Department of Pediatrics Division of Medical Critical Care Boston Children’s Hospital Boston, Massachusetts Francisco A. Perez, MD, PhD Assistant Professor of Pediatric Radiology Department of Radiology Seattle Children’s Hospital and University of Washington Seattle, Washington Melvin G. Perry Jr, MD Pediatric Intensivist Department of Pediatrics Piedmont Henry Hospital Stockbridge, Georgia; Pediatric Intensivist Department of Pediatrics WellStar Hospital Marietta; Georgia Mark J. Peters, MBChB, MRCP(UK), FRCPCH, PhD Professor of Paediatric Intensive Care Respiratory, Critical Care and Anaesthesia Unit UCL Great Ormond Street Institute of Child Health; Honorary Consultant Paediatric Intensivist Paediatric Intensivist Care Unit Great Ormond Street Hospital for Children NHS Trust London, United Kingdom

xviii

Contributors

Brent J. Pfeiffer, MD, PhD Assistant Professor Department of Pediatrics Division of Critical Care Medicine University of Miami Miami, Florida Rachel Phelan, MD, MPH Assistant Professor of Pediatrics Division of Hematology/Oncology/Blood and Marrow Transplant Medical College of Wisconsin Milwaukee, Wisconsin Joseph Philip, MD, FAAP Associate Professor Medical Director Pediatric Cardiac ICU Congenital Heart Center University of Florida Gainesville, Florida Neethi Pinto, MD, MS Assistant Professor Children’s Hospital of Philadelphia Philadelphia, Pennsylvania Murray M. Pollack, MD, MBA Professor of Pediatrics Department of Pediatrics George Washington University School of Medicine and Health Sciences; Director, Clinical Outcomes Research Department of Critical Care Children’s National Medical Center Washington, DC Thomas J. Preston, BS, CCP, FPP Innovative ECMO Concepts, Inc. Arcadia, Oklahoma Parthak Prodhan, MD Professor Pediatric Critical Care University of Arkansas/Arkansas Children’s Hospital Little Rock, Arkansas Lawrence Quang, MD Professor Department of Pediatrics College of Medicine University of Arkansas for Medical Sciences Little Rock, Arkansas Michael W. Quasney, MD Associate Professor Department of Pediatrics Division of Critical Care University of Michigan Medical School Ann Arbor, Michigan

Thomas M. Raffay, MD Assistant Professor Department of Pediatrics and Neonatology Rainbow Babies and Children’s Hospital Cleveland, Ohio Prakadeshwari Rajapreyar, MD, FAAP Pediatric Critical Care Medical College of Wisconsin Milwaukee, Wisconsin Lauren Rakes, MD Clinical Assistant Professor Department of Pediatrics Division of Critical Care Medicine University of Washington Seattle, Washington Rafael G. Ramos-Jimenez, MD Surgery Resident Department of General Surgery UPMC Presbyterian Shadyside Pittsburgh, Pennsylvania Samiran Ray, MBBChir, MA Consultant Department of Paediatric Intensive Care Medicine Great Ormond Street Hospital NHS Foundation Trust London, United Kingdom Christopher R. Reed, MD Resident Physician Department of Surgery Duke University Medical Center Durham, North Carolina James J. Reese Jr, MD Associate Professor of Neurology Department of Neurology University of New Mexico Albuquerque, New Mexico Kyle J. Rehder, MD Associate Professor Department of Pediatrics Duke Children’s Hospital; Physician Quality Officer Duke Center for Healthcare Safety and Quality Duke University Health System Durham, North Carolina Kenneth E. Remy, MD, MSCI, FCCM Assistant Professor of Pediatrics and Internal Medicine Adult and Pediatric Critical Care Medicine Washington University in St. Louis St. Louis, Missouri Jorge D. Reyes, MD Professor and Chief, Transplant Surgery University of Washington School of Medicine Seattle, Washington

Contributors

Eileen Rhee, MD, MS Assistant Professor Division of Pediatric Critical Care Medicine Division of Bioethics and Palliative Care Seattle Children’s Hospital Seattle, Washington Clare Richardson, MD Fellow Division of Pediatric Otolaryngology Seattle Children’s Hospital University of Washington School of Medicine Seattle, Washington Joan S. Roberts, MD Associate Professor Department of Pediatrics and Critical Care Seattle Children’s Hospital Seattle, Washington Luciana Rodriguez Guerineau, MD Staff Physician Division of Cardiac Critical Care Medicine The Hospital for Sick Children; Assistant Professor of Paediatrics Department of Paediatrics University of Toronto Toronto, Ontario Canada

Ahmed Said, MD, PhD Assistant Professor of Pediatrics (Critical Care) Center for Blood Oxygen Transport and Hemostasis University of Maryland School of Medicine Baltimore, Maryland Colin J. Sallee Pediatric Critical Care Fellow Seattle Children’s Hospital/Harborview Medical Center University of Washington Seattle, Washington Britt Julia Sandler, MD Fellow, Pediatric Critical Care Medicine University of Washington School of Medicine Seattle, Washington Jhuma Sankar, MD Associate Professor Department of Pediatrics All India Institute of Medical Sciences New Delhi, India Ajit A. Sarnaik, MD Associate Professor Department of Pediatrics Wayne State University Detroit, Michigan

Stephen Rogers, PhD Assistant Professor of Pediatrics Center for Blood Oxygen Transport and Hemostasis (CBOTH) University of Maryland School of Medicine Baltimore, Maryland

Ashok P. Sarnaik, MD Professor of Pediatrics Department of Pediatrics Children’s Hospital of Michigan Wayne State University School of Medicine Detroit, Michigan

Alexandre T. Rotta, MD, FCCM Division Chief Pediatric Critical Care Medicine Duke University Medical Center Professor of Pediatrics Duke University School of Medicine Durham, North Carolina

Robert Sawin, MD Surgeon-in-Chief General Surgery, Oncology, and Transplantation Professor of Surgery Seattle Children’s Hospital University of Washington Seattle, Washington

Mark E. Rowin, MD Associate Professor of Pediatrics University of Tennessee College of Medicine Chattanooga, Tennessee

Kenneth A. Schenkman, MD, PhD Associate Professor Department of Pediatrics University of Washington Seattle, Washington

Randall Ruppel, MD Assistant Professor Department of Pediatrics Virginia Tech Carilion School of Medicine Roanoke, Virginia Rita M. Ryan, MD Professor of Pediatrics (Neonatology) UH Rainbow Babies & Children’s Hospital Case Western Reserve University Cleveland, Ohio; Adjunct Professor of Pediatrics Medical University of South Carolina Charleston, South Carolina

xix

Stephen M. Schexnayder, MD Professor and Chief Pediatric Critical Care University of Arkansas for Medical Sciences/Arkansas Children’s Hospital Little Rock, Arkansas Charles L. Schleien, MD, MBA Philip Lanzkowsky Professor and Chair of Pediatrics Cohen Children’s Medical Center Hoftstra Northwell School of Medicine New Hyde Park, New York

xx

Contributors

Stephanie P. Schwartz, MD Assistant Professor Department of Pediatrics Division of Critical Care Medicine University of North Carolina at Chapel Hill Chapel Hill, North Carolina Steven M. Schwartz, MD, FRCPC Head, Division of Cardiac Critical Care Medicine The Hospital for Sick Children; Professor of Paediatrics Department of Paediatrics University of Toronto Toronto, Ontario, Canada Jay Shah, MD Medical Director, Aerodigestive Clinic Division of Pediatric Otolaryngology University Hospitals/Rainbow Babies and Children’s Hospital Cleveland, Ohio Sareen Shah, MD Assistant Professor of Pediatrics Department of Critical Care Medicine Cohen Children’s Medical Center New Hyde Park, New York Dennis W.W. Shaw, MD Professor of Pediatric Radiology Department of Radiology Seattle Children’s Hospital and University of Washington Seattle, Washington Steven L. Shein, MD Associate Professor Chief of Pediatric Critical Care Medicine Department of Pediatrics Rainbow Babies & Children’s Hospital Cleveland, Ohio Michael Shoykhet, MD, PhD Attending Physician Pediatric Critical Care Children’s National Medical Center Principal Investigator Center for Neuroscience Research Children’s National Research Institute Washington, DC Dennis W. Simon, MD Assistant Professor Department of Critical Care Medicine University of Pittsburgh School of Medicine Pittsburgh, Pennsylvania V. Ben Sivarajan, MD, MS, FRCPC Division Head, Pediatric Cardiac Intensive Care Unit Pediatric Critical Care Medicine Associate Professor of Pediatrics & Critical Care Medicine Faculty of Medicine & Dentistry, University of Alberta Stollery Children’s Hospital & University of Alberta Hospitals Edmonton, Alberta, Canada

Katherine N. Slain, DO Assistant Professor Department of Pediatrics Rainbow Babies & Children’s Hospital Cleveland, Ohio Jodi M. Smith, MD, MPH Professor Department of Pediatrics University of Washington, Seattle Seattle, Washington Lincoln S. Smith, MD Associate Professor Department of Pediatrics University of Washington Seattle, Washington Anthony A. Sochet, MD, MSc, FAAP Assistant Professor, Anesthesia and Critical Care Medicine Associate Fellowship Program Director, Pediatric Critical Care Medicine Johns Hopkins University School of Medicine Johns Hopkins All Children’s Hospital St. Petersburg, Florida Lauren R. Sorce, PhD, RN, CPNP-AC/PC, FCCM Founder’s Board Nurse Scientist Associate Director, Nursing Research Pediatric Critical Care Nurse Practitioner Ann and Robert H. Lurie Children’s Hospital of Chicago; Assistant Professor Division of Pediatric Critical Care Medicine Northwestern University Feinberg School of Medicine Chicago, Illinois Linda E. Sousse, PhD, MBA Assistant Professor Department of Surgery University of Texas Medical Branch Galveston, Texas Michael C. Spaeder, MD Associate Professor Division of Pediatric Critical Care University of Virginia School of Medicine Charlottesville, Virginia Richard H. Speicher, MD Associate Professor Department of Pediatric Critical Care Rainbow Babies & Children’s Hospital Cleveland, Ohio Philip C. Spinella, MD Director, Translational Research Program Department of Pediatrics Professor of Pediatric Critical Care Washington University School of Medicine St. Louis, Missouri

Contributors

Erika L. Stalets, MD, MS Associate Professor of Clinical Pediatrics University of Cincinnati College of Medicine Clinical Director, Division of Critical Care Medicine Cincinnati Children’s Hospital Medical Center Cincinnati, Ohio

Erik Su, MD Assistant Professor Department of Pediatrics Division of Pediatric Critical Care Medicine McGovern Medical School Houston, Texas

Stephen Wade Standage, MD Assistant Professor Department of Pediatrics Cincinnati Children’s Hospital Medical Center Cincinnati, Ohio

Corinne Summers, MD Assistant Member Clinical Research Division Fred Hutchinson Cancer Research Center; Assistant Professor Department of Pediatrics University of Washington, Seattle Children’s Hospital Seattle, Washington

Rebecca Stark, MD Assistant Professor Department of Pediatric Surgery Seattle Children’s Hospital; Assistant Professor Department of Surgery University of Washington Seattle, Washington Michelle C. Starr, MD, MPH Assistant Professor Department of Pediatrics Division of Nephrology Indiana University and Riley Children’s Hospital Indianapolis, Indiana David M. Steinhorn, MD Professor of Pediatrics Department of Critical Care Children’s National Medical Center Washington, DC Kurt R. Stenmark, MD Professor of Pediatrics and Medicine Department of Pediatric Critical Care University of Colorado Anschutz Medical Campus Aurora, Colorado Claire A. Stewart, MD, MEd Division of Critical Care Medicine Department of Pediatrics Nationwide Children’s Hospital The Ohio State University College of Medicine Columbus, Ohio Lindsay M. Stollings, MD Assistant Professor Department of Anesthesiology and Perioperative Medicine Children’s Hospital of Pittsburgh of UPMC Pittsburgh, Pennsylvania Casey Stulce, MD Assistant Professor Department of Pediatrics University of Chicago Chicago, Illinois

xxi

Robert M. Sutton, MD, MSCE Associate Professor of Anesthesia, Critical Care, and Pediatrics Department of Anesthesiology and Critical Care Medicine University of Pennsylvania Philadelphia, Pennsylvania Jordan M. Symons, MD Professor Department of Pediatrics University of Washington School of Medicine; Attending Nephrologist Division of Nephrology Seattle Children’s Hospital Seattle, Washington Nathaniel R. Sznycer-Taub, MD Assistant Professor of Pediatrics Division of Pediatric Cardiology University of Michigan Ann Arbor, Michigan Robert T. Tamburro Jr, MD Professor Department of Pediatric Critical Care Medicine Janet Weis Children’s Hospital Danville, Pennsylvania Christian Tapking, MD, MMS Fellow Department of Surgery University of Texas Medical Branch Galveston, Texas; Resident Hand, Plastic and Reconstructive Surgery, Microsurgery, Burn Trauma Center BG Trauma Center Ludwigshafen University of Heidelberg Ludwigshafen, Germany Robert C. Tasker, MBBS, MD Professor of Anesthesia (Pediatrics) Harvard Medical School; Department of Neurology Departments of Anesthesia (Pediatrics), Pain and Perioperative Medicine Division of Critical Care Boston Children’s Hospital Boston, Massachusetts

xxii

Contributors

Gregory H. Tatum, MD Associate Professor of Pediatrics Department of Pediatrics Duke University School of Medicine Durham, North Carolina Ann H. Tilton, MD Professor of Neurology and Pediatrics Section Chair, Child Neurology Louisiana State Health Sciences Center; Director, Children’s Hospital Rehabilitation Center Children’s Hospital of New Orleans New Orleans, Louisiana Matthew R. Timlin, DO Pediatric Intensivist Department of Pediatrics Madigan Army Medical Center Joint Base Lewis-McChord, Washington Pierre Tissieres, MD, DSc Professor Pediatric Intensive Care AP-HP Paris Saclay University Le Kremlin-Bicetre, France Joseph D. Tobias, MD Chair Department of Anesthesiology and Pain Medicine Nationwide Children’s Hospital; Professor of Anesthesiology and Pediatrics The Ohio State University Columbus, Ohio Philip Toltzis, MD Professor Department of Pediatrics Rainbow Babies & Children’s Hospital Cleveland, Ohio Alexis A. Topjian, MD, MSCE Associate Professor of Anesthesia, Critical Care, and Pediatrics Department of Anesthesiology and Critical Care Medicine University of Pennsylvania Philadelphia, Pennsylvania Troy Torgerson, MD, PhD Associate Professor Department of Pediatrics University of Washington Seattle, Washington Chani Traube, MD, FAAP, FCCM Associate Professor of Clinical Pediatrics Division of Pediatric Critical Care Medicine Weill Cornell Medical College New York, New York

Marisa Tucci, MD Professor Department of Pediatrics Sainte-Justine Hospital University of Montreal Montreal, Quebec, Canada David Tuggle, MD Associate Trauma Medical Director Department of Surgery Dell Children’s Medical Center of Central Texas Austin, Texas Jennifer L. Turi, MD Medical Director Pediatric Cardiac Intensive Care Unit Duke University Medical Center Associate Professor of Pediatrics Duke University School of Medicine Durham, North Carolina David A. Turner, MD Associate Director Graduate Medical Education Duke University Health System; Section Chief, Pediatric Intensive Care Associate Professor, Department of Pediatrics Division of Pediatric Critical Care Duke Children’s Hospital and Health System Durham, North Carolina Alisa Van Cleave, MD Clinical Assistant Professor Division of Pediatric Critical Care Medicine Division of Bioethics and Palliative Care Seattle Children’s Hospital Seattle, Washington Meredith G. van der Velden, MD Senior Associate of Critical Care Medicine Boston Children’s Hospital Assistant Professor of Anesthesia Harvard Medical School Boston, Massachusetts Adam M. Vogel, MD Associate Professor Departments of Surgery and Pediatrics Texas Children’s Hospital; Associate Professor Departments of Surgery and Pediatrics Baylor College of Medicine Houston, Texas Christine Vohwinkel, MD, PhD Associate Professor of Pediatrics Department of Pediatric Critical Care University of Colorado Anschutz Medical Campus Aurora, Colorado

Contributors

Amelie von Saint Andre-von Arnim, MD Associate Professor Department of Pediatrics Division of Pediatric Critical Care Medicine Seattle Children’s Hospital University of Washington Seattle, Washington

Scott L. Weiss, MD, MSCE Associate Professor of Anesthesiology, Critical Care, and Pediatrics Department of Anesthesiology and Critical Care The Children’s Hospital of Philadelphia University of Pennsylvania Perelman School of Medicine Philadelphia, Pennsylvania

Surabhi B. Vora, MD, MPH Associate Professor Department of Pediatric Infectious Disease Seattle Children’s Hospital Seattle, Washington

Jesse Wenger, MD Assistant Professor of Pediatrics Department of Pediatric Critical Care Medicine University of Washington Seattle Children’s Hospital Seattle, Washington

Alpana Waghmare, MD Assistant Professor Department of Pediatrics Division of Infectious Disease Seattle Children’s Hospital University of Washington Seattle, Washington Mark S. Wainwright, MD, PhD Professor Department of Neurology University of Washington Seattle, Washington Jessica S. Wallisch, MD Assistant Professor Department of Pediatrics Division of Pediatric Critical Care University of Missouri–Kansas City Children’s Mercy Hospital Kansas City, Missouri R. Scott Watson, MD, MPH Professor Department of Pediatrics Division of Pediatric Critical Care Medicine Seattle Children’s Hospital University of Washington School of Medicine Seattle, Washington Kevin Watt, MD, PhD Associate Professor Department of Pediatrics Chief Division of Clinical Pharmacology University of Utah Salt Lake City, Utah Maria Weimer, MD Associate Professor of Clinical Neurology Department of Neurology Louisiana State University Health Sciences Center New Orleans, Louisiana

Derek S. Wheeler, MD, MMM, MBA Professor of Pediatrics Feinberg School of Medicine Northwestern University Executive Vice President and Chief Medical Officer Ann & Robert H. Lurie Children’s Hospital of Chicago Chicago, Illinois Beth Wieczorek, DNP, CRNP-AC Nurse Practitioner Anesthesia and Critical Care Medicine Pediatric Intensive Care Unit Johns Hopkins University Baltimore, Maryland Michael Wilhelm, MD Associate Professor Department of Pediatrics University of Wisconsin, Madison Madison, Wisconsin Hector R. Wong, MD Critical Care Medicine Cincinnati Children’s Hospital Medical Center Cincinnati, Ohio Charles R. Woods Jr, MD Chair Department of Pediatrics University of Tennessee College of Medicine; Chief Medical Officer Children’s Hospital Erlanger Health System Chattanooga, Tennessee George A. (Tony) Woodward, MD, MBA Professor of Pediatrics Chief, Division of Emergency Medicine University of Washington School of Medicine Medical Director, Emergency and Transport Services Seattle Children’s Hospital Seattle, Washington

xxiii

xxiv

Contributors

Amy C. Yang, MD Assistant Professor Department of Molecular and Medical Genetics Oregon Health and Science University Portland, Oregon Heidi Yu, PharmD Critical Care Pharmacist Seattle Children’s Hospital Seattle, Washington Nicole R. Zane, PharmD, PhD Research Scientist Center for Clinical Pharmacology Children’s Hospital of Philadelphia Philadelphia, Pennsylvania Danielle M. Zerr, MD, MPH Professor Department of Pediatrics Seattle Children’s Hospital University of Washington Seattle, Washington Hui Zhang, MD, PhD Instructor Department of Pediatric Critical Care University of Colorado Anschutz Medical Campus Aurora, Colorado Hengqi (Betty) Zheng, MD Acting Instructor Department of Gastroenterology and Hepatology Seattle Children’s Hospital Seattle, Washington

Jerry J. Zimmerman, MD, PhD, FCCM Faculty, Pediatric Critical Care Medicine Seattle Children’s Hospital Harborview Medical Center University of Washington School of Medicine Seattle, Washington Kanecia Zimmerman, MD, MPH Assistant Professor Department of Pediatrics Duke University Medical Center Duke Clinical Research Institute Durham, North Carolina Matt S. Zinter, MD Assistant Professor Department of Pediatrics Division of Critical Care Medicine University of California, San Francisco San Francisco, California Athena F. Zuppa, MD, MSCE Professor Department of Anesthesia and Critical Care The Children’s Hospital of Philadelphia Philadelphia, Pennsylvania

Preface

From our home working spaces, in the midst of the COVID-19 pandemic, welcome to the sixth edition of Pediatric Critical Care. The world has finally been provided a realistic glimpse of intensive care units, the work that occurs there, and the dedicated providers who provide critical care, sometimes at their own peril. This sixth edition, which now reflects a lifetime work product for many contributors, is dedicated to the multidisciplinary team that makes critical care a reality. What the pandemic has taught all of us is the importance of being able to adapt to change. As readers page through this new edition, many changes will become apparent. Brad Fuhrman, who ultimately deserves the credit for creating this publishing adventure, decided to move from Co-editor to Section Editor. More than once, likely while sipping single malt scotch together, Brad noted that Pediatric Critical Care might be our most important professional contribution. Quality and sustainability over six editions prove his prediction correct. Meanwhile, Alex Rotta, appropriately one of Brad’s early apprentices, has provided outstanding organizational leadership as Co-editor for the sixth edition. Thanks also to Section Editors Bob Clark, Sapna Kudchadkar, Monica Relvas, and Joe Tobias. Perusing the list of contributors similarly ascertains established and burgeoning pediatric critical care contributors. Accordingly, the sixth edition is truly a multigenerational effort.

For pediatric critical care medicine fellows who may read the textbook cover to cover, and for others who need an updated reference for anything related to pediatric critical care, the new edition will not disappoint. Each color-coded section provides best-evidence clinical approaches to pediatric critical care issues based on contemporary genetic, biochemical, and physiologic infrastructure. Because authors typically want to include all relevant details in their discussion and a hard copy textbook has physical limitations, readers are encouraged to make use of the expanded electronic content included with the sixth edition. Board review questions, composed for most chapters, will be valuable for new and repeat readers and are also available in the electronic content. Finally, all of us must honor the children and families who provide the meaning for our life work. With the publishing of the sixth edition of Pediatric Critical Care, we collectively acknowledge the clinical challenges that critical illness presents. However, these challenges facilitate curiosity and imagination, growth and experience, and ultimately personal enrichment. Hopefully the sixth edition of Pediatric Critical Care can serve as a valuable tool for addressing longstanding as well as novel critical care challenges. Jerry J. Zimmerman, MD, PhD, FCCM Alexandre T. Rotta, MD, FCCM

xxv

Contents

Section I:  Pediatric Critical Care: The Discipline  1

Section II:  Pediatric Critical Care: Tools and Procedures  75

   1 History of Pediatric Critical Care Medicine  2

11 Essential Concepts in Clinical Trial Design and Statistical Analysis  76

Daniel L. Levin and John J. Downes

Leslie A. Dervan, R. Scott Watson, and Mary E. Hartman

   2 High-Reliability Pediatric Intensive Care Unit: Role of Intensivist and Team in Obtaining Optimal Outcomes  16 Claire A. Stewart, Derek S. Wheeler, and Richard J. Brilli

   3 Critical Communications in the Pediatric Intensive Care Unit  22

12 Prediction of Short-Term Outcomes During Critical Illness in Children  82 Julia A. Heneghan, Michael C. Spaeder, and Murray M. Pollack

13 Pediatric Critical Care Transport  89 Lauren Rakes, Reid W.D. Farris, and George A. (Tony) Woodward

Shelina M. Jamal, Katherine Banker, and Harris P. Baden

   4 Professionalism in Pediatric Critical Care  26 Bradley P. Fuhrman and Lynn J. Hernan

   5 Leading and Managing Change in the Pediatric Intensive Care Unit  29 John C. Lin

   6 Evolution of Critical Care Nursing  36 Lauren R. Sorce and Ruth Lebet

   7 Fostering a Learning Healthcare Environment in the Pediatric Intensive Care Unit  47 Melvin G. Perry Jr and Jerry J. Zimmerman

   8 Challenges of Pediatric Critical Care in ResourcePoor Settings  51 Amélie von Saint André–von Arnim, Jhuma Sankar, Andrew Argent, and Ericka Fink

   9 Public Health Emergencies and Emergency Mass Critical Care  59 Katherine L. Kenningham and Megan M. Gray

10 Lifelong Learning in Pediatric Critical Care  66 Stephanie P. Schwartz, Laura Marie Ibsen, and David A. Turner

14 Pediatric Vascular Access and Centeses  94 Lauren R. Edwards, Matthew P. Malone, Parthak Prodhan, and Stephen M. Schexnayder

15 Ultrasonography in the Pediatric Intensive Care Unit  114 Erik Su, Akira Nishisaki, and Thomas Conlon

Section III:  Pediatric Critical Care: Psychosocial and Societal  135 16 Patient- and Family-Centered Care in the Pediatric Intensive Care Unit  136 Jenny Kingsley and Jonna D. Clark

17 Pediatric Critical Care Ethics  144 Mithya Lewis-Newby, Emily Berkman, and Douglas S. Diekema

18 Ethical Issues Around Death and Dying  154 Meredith G. van der Velden and Jeffrey P. Burns

19 Palliative Care in the Pediatric Intensive Care Unit  158 Alisa Van Cleave, Eileen Rhee, and Wynne Morrison

20 Organ Donation Process and Management of the Organ Donor  163 Thomas A. Nakagawa, Mudit Mathur, and Anthony A. Sochet

xxvii

xxviii

Contents

21 Long-Term Outcomes Following Critical Illness in Children  175 Elizabeth Y. Killien, Jerry J. Zimmerman, François Aspesberro, and R. Scott Watson

22 Burnout and Resiliency  183 Ruth Kleinpell and Jason M. Kane

35 Pediatric Cardiopulmonary Bypass  363 Richard M. Ginther Jr and Joseph M. Forbess

36 Critical Care After Surgery for Congenital Cardiac Disease  380 Paula Holinski, Jennifer Turi, Veerajalandhar Allareddy, V. Ben Sivarajan, and Alexandre T. Rotta

Section IV:  Pediatric Critical Care: Cardiovascular  187

37 Cardiac Transplantation  411

23 Structure and Function of the Heart  188

38 Physiologic Foundations of Cardiopulmonary Resuscitation  420

Luciana Rodriguez Guerineau, Jayani Abeysekera, V. Ben Sivarajan, and Steven M. Schwartz

24 Regional Peripheral Circulation  203 Peter Oishi, Julien I. Hoffman, Bradley P. Fuhrman, and Jeffrey R. Fineman

25 Endothelium and Endotheliopathy  218 Yves Ouellette

26 Principles of Invasive Cardiovascular Monitoring  227 Matthew R. Timlin and Kenneth A. Schenkman

27 Assessment of Cardiovascular Function  239 Nathaniel R. Sznycer-Taub, Thomas J. Kulik, John R. Charpie, and Melvin C. Almodovar

28 Cardiac Failure and Ventricular Assist Devices  248 Ana Lia Graciano, Joseph Philip, and Keith C. Kocis

Sonya Kirmani and Michael Carboni

Adnan M. Bakar, Kenneth E. Remy, Sareen Shah, and Charles L. Schleien

39 Performance of Cardiopulmonary Resuscitation in Infants and Children  444 Ryan W. Morgan, Robert A. Berg, Alexis A. Topjian, Vinay M. Nadkarni, and Robert M. Sutton

Section V:  Pediatric Critical Care: Pulmonary  453 40 Structure and Development of the Upper Respiratory System  454 Robert H. Chun and Joan C. Arvedson

41 Structure and Development of the Lower Respiratory System  462 John E. Baatz and Rita M. Ryan

42 Physiology of the Respiratory System  470 Robinder G. Khemani and Justin C. Hotz

29 Echocardiographic Imaging  270 M. Jay Campbell, Michael W. Camitta, Piers C.A. Barker, and Gregory H. Tatum

43 Noninvasive Respiratory Monitoring and Assessment of Gas Exchange  483 David F. Butler and Kenneth A. Schenkman

30 Diagnostic and Therapeutic Cardiac Catheterization  289 Reid C. Chamberlain, Kevin D. Hill, and Gregory A. Fleming

31 Pharmacology of the Cardiovascular System  300 Naomi B. Bishop, Bruce M. Greenwald, and Daniel A. Notterman

32 Cardiopulmonary Interactions  320 Ronald A. Bronicki, Mubbasheer Ahmed, Saul Flores, and Bradley P. Fuhrman

33 Disorders of Cardiac Rhythm  329 Frank A. Fish and Prince J. Kannankeril

34 Shock States  352 Lincoln Smith, Alicia Alcamo, Joseph A. Carcillo, and Rajesh Aneja

44 Overview of Breathing Failure  492 Katherine V. Biagas, Michael Wilhelm, and Bradley P. Fuhrman

45 Ventilation/Perfusion Inequality  503 Silvia M. Hartmann and Thomas V. Brogan

46 Mechanical Dysfunction of the Respiratory System  509 Jeff Clark, Saurabh Chiwane, and Ashok P. Sarnaik

47 Diseases of the Upper Respiratory Tract  524 Todd Otteson, Clare Richardson, and Jay Shah

48 Pediatric Acute Respiratory Distress Syndrome and Ventilator-Associated Lung Injury  536 Colin J. Sallee, Robinder G. Khemani, and Lincoln S. Smith

Contents

49 Acute Viral Bronchiolitis  546 Katherine N. Slain and Steven L. Shein

50 Asthma  552 Steven L. Shein, Richard H. Speicher, Howard Eigen, and Alexandre T. Rotta

51 Neonatal Pulmonary Disease  568 Thomas M. Raffay and Richard J. Martin

xxix

64 Status Epilepticus  779 Edward E. Conway Jr and Robert C. Tasker

65 Hypoxic-Ischemic Encephalopathy  793 Ericka L. Fink, Mioara Manole, Robert S.B. Clark, Cameron Dezfulian, and Patrick M. Kochanek

66 Pediatric Stroke and Intracerebral Hemorrhage  811 Catherine Amlie-Lefond and Jeffrey Ojemann

52 Pneumonitis and Interstitial Disease  585 Daiva Parakininkas

53 Diseases of the Pulmonary Circulation  608 Hui Zhang, Christine Vohwinkel, Aline Maddux, and Kurt R. Stenmark

54 Mechanical Ventilation and Respiratory Care  625 Kyle J. Rehder and Ira M. Cheifetz

55 Noninvasive Ventilation in the Pediatric Intensive Care Unit  644

67 Central Nervous System Infections and Related Conditions  823 Kevin M. Havlin, Charles R. Woods Jr, and Mark E. Rowin

68 Acute Neuromuscular Disease and Disorders  837 Maria Weimer, James J. Reese Jr, and Ann H. Tilton

69 Acute Rehabilitation and Early Mobility in the Pediatric Intensive Care Unit  845 Hallie Lenker, Yun Kim, Beth Wieczorek, and Sapna R. Kudchadkar

Omar Alibrahim and Katherine N. Slain

56 Extracorporeal Life Support  655 Heidi J. Dalton, Nina Censoplano, Tom Preston, Hanneke Ijsselstijn, and Aparna Hoskote

57 Pediatric Lung Transplantation  679 Carol Conrad

Section VII:  Pediatric Critical Care: Renal  855 70 Renal Structure and Function  856 Matthew M. Grinsell and Victoria F. Norwood

71 Fluid and Electrolyte Issues in Pediatric Critical Illness  866 Idris V.R. Evans and Emily L. Joyce

Section VI:  Pediatric Critical Care: Neurologic  689

72 Acid-Base Disorders  882

58 Structure, Function, and Development of the Nervous System  690

73 Tests of Kidney Function in Children  896

Robert S.B. Clark and Michael Shoykhet

59 Critical Care Considerations for Common Neurosurgical Conditions  710 Peter F. Morgenstern, Robert H. Bonow, Isaac Josh Abecassis, Samuel R. Browd, and Amy Lee

60 Neurologic Assessment and Monitoring  720 Mark S. Wainwright and Sue J. Hong

61 Neuroimaging  735 Francisco A. Perez, Hedieh Khalatbari, and Dennis W.W. Shaw

62 Coma and Depressed Sensorium  756 Neethi Pinto and Casey Stulce

63 Intracranial Hypertension and Monitoring  768 Robert C. Tasker and Alireza Akhondi-Asl

Michelle C. Starr and Shina Menon

Rajit K. Basu

74 Glomerulotubular Dysfunction and Acute Kidney Injury  907 Timothy E. Bunchman, Vu Nguyen, and Michelle L. Olson

75 Pediatric Renal Replacement Therapy in the Intensive Care Unit  923 Raj Munshi and Jordan M. Symons

76 Pediatric Renal Transplantation  930 Jodi M. Smith, André A.S. Dick, and Ruth McDonald

77 Renal Pharmacology  937 Jenna R. Nickless, Victoria Chadwick, and Heidi Yu

78 Acute Severe Hypertension  945 Joseph T. Flynn

xxx

Contents

Section VIII:  Pediatric Critical Care: Metabolic and Endocrine  959

Section X:  Pediatric Critical Care: Gastroenterology and Nutrition  1129

79 Cellular Respiration  960

94 Gastrointestinal Structure and Function  1130

Scott L. Weiss, Clifford S. Deutschman, and Lance B. Becker

80 Biology of the Stress Response  971 Stephen Wade Standage

Lee M. Bass and David M. Steinhorn

95 Disorders and Diseases of the Gastrointestinal System  1141 Lauren Bodilly and Samuel A. Kocoshis

81 Inborn Errors of Metabolism  976 Cary O. Harding and Amy Yang

96 Acute Liver Failure  1155 Hengqi (Betty) Zheng, Mihaela A. Damian, and Simon Horslen

82 Progress Towards Precision Medicine in Critical Illness  991 Mary K. Dahmer and Michael W. Quasney

83 Molecular Foundations of Cellular Injury  996 Jocelyn R. Grunwell and Craig M. Coopersmith

97 Hepatic Transplantation  1162 Patrick J. Healey, Britt Julia Sandler, Abigail Apple, Thomas V. Brogan, and Jorge D. Reyes

98 Acute Abdomen  1170 Robert Sawin, Rebecca Stark, and Derya Caglar

84 Endocrine Emergencies  1003 Katherine Ratzan Peeler and Michael S.D. Agus

99 Nutrition of the Critically Ill Child  1177 Ben D. Albert and Nilesh M. Mehta

85 Diabetic Ketoacidosis  1016 Ildiko H. Koves and Nicole Glaser

Section XI:  Pediatric Critical Care: Immunity and Infection  1189

Section IX: Pediatric Critical Care: Hematology and Oncology  1023

100 Innate Immunity  1190

86 Structure and Function of the Hematopoietic Organs  1024

101 Adaptive Immunity  1199

Seth J. Corey and Julie Blatt

87 The Erythron  1033 Allan Doctor, Ahmed Said, and Stephen Rogers

88 Hemoglobinopathies  1040 M.A. Bender and Anne Marsh

Samiran Ray, Rachel S. Agbeko, and Mark J. Peters

Jennifer A. Muszynski, W. Joshua Frazier, and Kristin C. Greathouse

102 Critical Illness and the Microbiome  1208 Rafael G. Ramos-Jimenez, Dennis Simon, and Michael J. Morowitz

103 Congenital Immunodeficiency  1215 Hannah Laure Elfassy, Troy Torgerson, and Christine McCusker

89 Coagulation and Coagulopathy  1052 Robert I. Parker

104 Acquired Immune Dysfunction  1229 Brent J. Pfeiffer

90 Thrombosis in Pediatric Critical Care  1073 Sally Campbell and Paul Monagle

105 Immune Balance in Critical Illness  1242 Mark W. Hall

91 Transfusion Medicine  1082 Jacques Lacroix, Marisa Tucci, Oliver Karam, and Philip C. Spinella

92 Hematology and Oncology Problems  1101 Jesse Wenger, Corinne Summers, and Joan S. Roberts

106 Pediatric Rheumatologic Disease  1249 Marla Guzman, Timothy Hahn, Alexandra Aminoff, and Kristen Hayward

107 Bacterial and Fungal Infections  1263 Deborah E. Franzon, Emily R. Levy, and Matt S. Zinter

93 Critical Illness in Children Undergoing Hematopoietic Progenitor Cell Transplantation  1113 Prakadeshwari Rajapreyar, Jennifer McArthur, Christine Duncan, Rachel Phelan, Robert T. Tamburro Jr

108 Life-Threatening Viral Diseases and Their Treatment  1273 Surabhi B. Vora, Alpana Waghmare, Danielle M. Zerr, and Ann J. Melvin

Contents

109 Healthcare-Associated Infections  1284 Alexis L. Benscoter, Richard J. Brilli, Derek S. Wheeler, and Erika L. Stalets

110 Pediatric Sepsis  1293 Matthew N. Alder, Lauren Bodilly, and Hector R. Wong

111 Multiple-Organ Dysfunction Syndrome  1310

xxxi

124 Adverse Drug Reactions and Drug-Drug Interactions  1464 Jessie O’Neal, Lauren Dartois, Anny Chan, Wade W. Benton, and Christa Jefferis Kirk

125 Principles of Toxin Assessment and Screening  1486 April Clawson and Lawrence Quang

Pierre Tissieres and Melania M. Bembea

126 Toxidromes and Their Treatment  1496

Section XII:  Pediatric Critical Care: Environmental Injury and Trauma  1317 112 Bites and Stings  1318 Kirsten Orloff and Kanecia Zimmerman

113 Hyperthermic Injury  1327 Jason A. Clayton and Philip Toltzis

114 Accidental Hypothermia  1332 Björn Gunnarsson and Christopher M.B. Heard

115 Drowning  1337 Jamie L. Bell, Ajit A. Sarnaik, and Ashok P. Sarnaik

116 Burn and Inhalation Injury  1347 Christian Tapking, Linda E. Sousse, Karel D. Capek, and David N. Herndon

117 Evaluation, Stabilization, and Initial Management After Trauma  1363 Jessica A. Naiditch, Michael Dingeldein, and David Tuggle

118 Traumatic Brain Injury  1375 Patrick M. Kochanek, Michael J. Bell, Dennis W. Simon, Hülya Bayır, Jessica S. Wallisch, Michael L. Forbes, Randall Ruppel, P. David Adelson, Travis C. Jackson, and Robert S.B. Clark

119 Pediatric Thoracic Trauma  1401 Tamara N. Fitzgerald and Christopher R. Reed

120 Pediatric Abdominal Trauma  1408 Adam M. Vogel and Michael Dingeldein

121 Child Abuse  1417 Tom Kallay and Carol Berkowitz

Section XIII:  Pediatric Critical Care: Pharmacology and Toxicology  1425 122 Principles of Drug Disposition  1426 Nicole R. Zane and Athena F. Zuppa

123 Molecular Mechanisms of Drug Actions  1446 Kevin Watt

Prashant Joshi

Section XIV:  Pediatric Critical Care: Anesthesia Principles in the Pediatric Intensive Care Unit  1509 127 Airway Management  1510 Chinyere Egbuta, E. Alexis Bragg, and Sapna R. Kudchadkar

128 Anesthesia Effects on Organ Systems  1535 Lindsay M. Stollings, Peter J. Davis, Alison M. Ellis, and Antonio Cassara

129 Anesthesia Principles and Operating Room Anesthesia Regimens  1544 Joseph D. Tobias

130 Malignant Hyperthermia  1560 Christopher M. Edwards and Barbara W. Brandom

131 Neuromuscular Blocking Agents  1567 Joseph D. Tobias

132 Sedation and Analgesia  1583 Christopher M. B. Heard, Omar Alibrahim, and Alexandre T. Rotta

133 Tolerance, Dependency, and Withdrawal  1611 Joseph D. Tobias

134 Pediatric Delirium  1617 Chani Traube and Bruce M. Greenwald

135 Procedural Sedation for the Pediatric Intensivist  1624 Nir Atlas, Rahul C. Damania, and Pradip P. Kamat

Section XV:  Pediatric Critical Care: Board Review Questions  e1 136 Board Review Questions  e2 Index  1629

SECTION

I

Pediatric Critical Care: The Discipline 1. History of Pediatric Critical Care Medicine, 2 2. High-Reliability Pediatric Intensive Care Unit: Role of Intensivist and Team in Obtaining Optimal Outcomes, 16 3. Critical Communications in the Pediatric Intensive Care Unit, 22 4. Professionalism in Pediatric Critical Care, 26 5. Leading and Managing Change in the Pediatric Intensive Care Unit, 29  



 



 

6. Evolution of Critical Care Nursing, 36 7. Fostering a Learning Healthcare Environment in the Pediatric Intensive Care Unit, 47 8. Challenges of Pediatric Critical Care in Resource-Poor Settings, 51 9. Public Health Emergencies and Emergency Mass Critical Care, 59 10. Lifelong Learning in Pediatric Critical Care, 66  





 



 



 

 

1

1 History of Pediatric Critical Care Medicine DANIEL L. LEVIN AND JOHN J. DOWNES

“In critical care, it strikes one that the issues are three: realism, dignity, and love.”

Jacob Javitz, 1986 (Posthumous Inspirational Award Honoree, Society Of Critical Care Medicine)

PEARLS •

• •











The evolution of pediatric critical care medicine reflects long progress in anatomy, physiology, resuscitation and ventilation, anesthesiology, neonatology, pediatric general surgery, pediatric cardiac surgery, and pediatric cardiology. The role of nursing is absolutely central to the evolution of critical care units. Until the 1950s and 1960s, intensive care units were organized by grouping patients with similar diseases. However, in the 1960s, neonatal intensive care units grouped children according to age and severity of illness, and pediatric intensive care units followed this example. Sophisticated interhospital transfer services proved significant in reducing morbidity and mortality of critically ill children

Evolution of Modern Medicine The evolution of pediatric critical care medicine (PCCM) reflects a long series of contributions from anatomy, physiology, resuscitation and ventilation, anesthesiology, neonatology, pediatric general surgery, pediatric cardiac surgery, pediatric cardiology, and the many individuals responsible for the discoveries and innovations.1,2 Intensive care units were originally organized by grouping together patients with the same or similar diseases. However, when neonatologists grouped children according to age and severity of illness, pediatric intensive care units (PICUs) followed their example. Transport, or retrieval medicine, developed and nurses took on a major role in providing care to critically ill and injured children.

Anatomy and Physiology What seems simple and obvious today took a great deal of time, effort, and insight to understand. This section discusses some of the contributions that advanced the practice of medicine, enabled 2









starting in the 1970s. This retrieval medicine holds great promise for future improvements in care. In pediatric critical care medicine, there have been remarkable achievements in the ability to understand and treat critical illness in children as well as progress in the organization of pediatric critical care medicine, education, and research in the field. Increasing use of improved technology has advanced the care of critically ill children but has not eliminated errors, complications, or potentially long-term sequelae, and it is associated with a need for greater focus on establishing a humane, caring environment for the patients and their families.

the development of cardiorespiratory support, and eventually led to the establishment of intensive care. Andreas Vesalius (1514–1564), the Flemish anatomist, corrected many previous mistakes in the understanding of anatomy and provided positive pressure ventilation via a tracheotomy tube to asphyxiated fetal lambs. Michael Servetus of Spain (1511–1553) correctly described the pumping action of the heart’s ventricles and the circulation of blood from the right heart through the lungs to the left heart. Matteo Realdo Columbo (1515–1559) described pulmonary circulation and the concept that the lungs added a spirituous element to the blood by the admixture of air. William Harvey (1578–1657) confirmed the function of the heart and arterial and venous circulations through both animal experiments and observations in humans. He published De Motu Cordis3 (On the Motion of the Heart) in 1628. Because he did not yet have the microscope, he could not see the capillaries and thus could not include the mechanism for transfer of blood from the arterial to the venous systems of the pulmonary circulation. Capillaries were first described by Marcello Malpighi (1628–1694, Italian) in De Pulmonibus (On the Lungs) in 1661. Thomas Willis (1611–1675)



CHAPTER 1

and, eventually, William Cullen (1710–1790) led the way to the understanding of the role of the nervous system as the site of consciousness and the regulation of vital phenomena. Richard Lower (1631–1691) proved that it was the passage of blood through the lungs, ventilation of the lungs, and gas exchange with blood that vivified the blood and turned it red. Stephen Hales (1677–1761) measured blood pressure with a brass tube connected to a 9-foot glass tube in a horse. Joseph Black (1728–1799) identified carbon dioxide as a gas expired from human lungs. Karl Wilhelm Scheele (1742–1786) isolated oxygen, as did Joseph Priestley (1733–1804), who named it “dephlogisticated air” and determined its vital role in supporting combustion. Antoine-Laurent Lavoisier (1743–1794) identified oxygen as the vital element taken up by the lungs that maintains life and gave it its name (literally “acid generator”). Oxygen’s essential role in physiology and biochemistry was not clarified until the late 19th century when Felix Hoppe-Seyler (1825–1895) described the transportation of oxygen in blood by hemoglobin. Giovanni Morgagni (1682–1771) initiated the field of anatomic pathology in his classic book De sedibus et causis morborum per anatomen indagatis, published in 1761. He described in detail his observations of the diseased organs in more than 700 autopsies of persons with a wide variety of disorders and made correlations with the patient’s appearance and symptoms, the initial clinicalpathologic basis of medicine. In 1842, Crawford Long in Georgia and in 1846, William Morton in Boston demonstrated the efficacy and safety of ether anesthesia, thereby opening the era of modern surgery. Joseph Lister (1827–1912), one of the founders of modern surgery, reasoned that bacteria were the source of pus in rotten organic material and in 1865 used carbolic acid in surgical fields and in wound dressings to eliminate bacteria. This technique dramatically improved patient outcomes after surgery. Robert Koch (1843–1910) developed his postulates in 1882 in order to attribute the etiology of a disease to a particular microorganism in a logical, scientific manner. He also identified the tubercle bacillus as the cause of tuberculosis and was awarded the Nobel Prize in 1905. Wilhelm Conrad von Röntgen (1845–1923) discovered x-rays in 1895. Scipione Riva-Rocci (1863–1937), in 1896, measured blood pressure using the sphygmomanometer, and Nikolai Korotkoff (1874–1920) introduced his auscultation method of determining systolic and diastolic pressure in 1905.1

Resuscitation and Ventilatory Support The key to understanding the present practice of intensive care for children lies in knowing the history of scientific study of cardiorespiratory anatomy and physiology and the discovery of techniques to support ill patients. Although one could think that current practice suddenly emerged with the late 20th century, technical discoveries and accomplishments in the development of resuscitation and ventilation taken for granted today date back to the Bible, and numerous events and contributions led to current practice. In a biblical story,1,4,5 Elisha resurrected a young boy who was dead when “he climbed onto the bed and stretched himself on top of the child, putting his mouth to his mouth, his eyes to his eyes, and his hands to his hands, and as he lowered himself onto him the child’s flesh grew warm....Then the child sneezed and opened his eyes.” In 117 CE, Antyllus performed tracheotomies for patients with upper airway obstruction.6 Paracelsus, a 16th-century Swiss alchemist and physician, first

History of Pediatric Critical Care Medicine

3

provided artificial ventilation to both animals and dead humans using a bellows.6 Andreas Vesalius, the aforementioned Flemish professor of anatomy, in De Humani Corporis Fabrica, reported ventilating open-chest dogs, fetal lambs, and pigs using a tracheostomy and fireplace bellows in 1543.7–9 The French obstetrician Desault, in 1801, described how to successfully resuscitate apneic or limp newborns by digital oral tracheal intubation with a lacquered fabric tube and then blowing into the tube.1 In 1832, Dr. John Dalziel in Scotland developed a bellows-operated intermittent negative pressure device to assist ventilation.8 In 1864, Alfred F. Jones, of Lexington, Kentucky, built a body-enclosing tank ventilator; in the 1880s, Alexander Graham Bell developed a so-called vacuum jacket driven by hand-operated bellows.8 In 1876 in Paris, Woillez built what was probably the first workable cuirass ventilator, which was strikingly similar to the “iron lung” respirator introduced by McKhann and Drinker in 1929 and manufactured for widespread use by Emerson in 1931.10 Braun developed an infant resuscitator, as described by Doe in 1889, which was used successfully in 50 consecutive patients. A respirator developed by Steuart in 1918 in Cape Town, South Africa, apparently successfully treated a series of polio patients, but he did not report it.8 In 1888, Joseph O’Dwyer, a physician working at the New York Foundling Hospital who was concerned about the high death rate in croup and laryngeal diphtheria, instituted the manual method of blind oral laryngeal intubation using short, tapered brass tubes that entered the subglottic lumen. Despite severe criticism, he persisted in developing a series of various-diameter tubes for the palliation of severe adult and pediatric laryngeal edema due to infections, including diphtheria. They were used until the 1930s. George Fell, another New York physician, devised a method of ventilation with a foot-operated bellows and exhalation valve connected by rubber tubing to the O’Dwyer tube.8 In 1898, Rudolph Matas of New Orleans adapted the FellO’Dwyer technique to ventilate patients’ lungs during chest wall surgery. In the early 1900s, George Morris Dorrance of Philadelphia used the technique to perform resuscitations.8 In 1910, at the Trendelenburg Clinic in Leipzig, two thoracic surgeons. A. Lawen and R. Sievers, developed a volume-preset, positive-pressure, electrically powered piston-cylinder ventilator with a draw-over humidifier. It was used successfully with a tracheotomy tube during and after thoracic surgery and for a variety of disorders causing respiratory failure.1 Chevalier Jackson (1858–1955), a surgeon at Temple University in Philadelphia, developed a highly specific series of techniques for laryngoscopy, bronchoscopy, and tracheotomy.1 He revolutionized the procedure of tracheotomy and developed a detailed protocol of airway care. His design of tubes, made of silver, for patients of all ages set the standard for tracheotomy tubes for more than the first half of the 20th century. In 1958, Peter Safar, then at the Baltimore City Hospital, published studies proving that the long-standing pulmonary resuscitation technique of chest pressure and arm lift was virtually worthless. In effect, he went back to Elisha and proved jaw thrust and mouth-to-mouth resuscitation superior.11 Soon after, W.B. Kouwenhoven and James Jude at Johns Hopkins published work on the effectiveness of closed-chest cardiac massage.12 In 1946 Beck and his team demonstrated open-chest electrical defibrillation. In 1952, Zoll and coworkers proved the efficacy of external defibrillation and, in 1956, the effectiveness of external cardiac pacing.13

4

SECTION I



Pediatric Critical Care: The Discipline

Contributions of Specific Disciplines Pediatric Anesthesiology PCCM developed initially through the efforts of pediatric anesthesiologists, as well as pediatric general surgeons and pediatric cardiac surgeons and neonatologists. In fact, most of the original PICUs were founded by pediatric anesthesiologists (Table 1.1).1,4,14–23 Before discrete, geographically separate, ICUs evolved, critically ill children often received close monitoring,

intensive nursing care, and pulmonary support in the postanesthetic recovery room. There, the anesthesiologists were the attending physicians. In addition to those PICUs noted in Table 1.1, there were certainly others that were not as well documented.

Pediatric General Surgery and Pediatric Cardiac Surgery Dr. William E. Ladd (1880–1967) at Boston Children’s Hospital (BCH), the first full-time pediatric surgeon, pioneered the

TABLE Some Early Pediatric Intensive Care Units and Programsa 1.1

Year

Institution/Location

Medical Director(s)

Director(s) Specialtyb

1955

Children’s Hospital, Goöteborg, Sweden

G. Haglund

Ped Anesth.

1961

St. Goran’s Children’s Hospital, Stockholm, Sweden

H Feychting

Ped Anesth.

1961

Great Ormond Street Children’s Hospital, London, England

W. Glover

Ped Anesth.

1963

Hospital St. Vincent de Paul, Paris, France

J.B. Joly G. Huault

Neonatology Neonatology

1963

Royal Children’s Hospital, Melbourne, Australia

I.H. McDonald J. Stocks

Ped Anesth. Ped Anesth.

1963

Adelaide Children’s Hospital, Adelaide, Australia

T. Allen I. Stevens

Ped Anesth. Ped Anesth.

1964

Alden Hey Children’s Hospital Liverpool, England

G.J. Rees

Ped Anesth.

1967

Children’s Hospital of Philadelphia, Philadelphia, Pennsylvania

J.J. Downes

Ped Anesth.

1967

Children’s Memorial Hospital, Chicago, Illinois

D. Allen F. Seleny J. Cox

Ped Anesth. Ped Anesth. Ped Anesth.

1968c

Children’s Hospital District of Columbia, Washington, DCd

C. Berlin

Ped.

1968

Children’s Hospital Calvo Mackenna, Santiago de Chile

E. Bancalari

Ped.

1969

Children’s Hospital of Pittsburgh, Pittsburgh, Pennsylvania

S. Kampschulte

Ped Anesth.

1969

Yale–New Haven Medical Center, New Haven, Connecticut

J. Gilman N. Talner

Ped Anesth. Ped Cardiol.

1970e

Hospital for Sick Children, Toronto, Canada

A. Conn

Ped Anesth.

1971

Massachusetts General Hospital, Boston, Massachusetts

D. Shannon I.D. Todres

Ped Pulm. Ped & Ped Anesth.

1971

Long Island Jewish Hospital, New York

B. Holtzmann

Ped Pulm.

1971

Montefiore Hospital, New York

R. Kravath

Ped Pulm.

1972

Sainte Justine Hospital, Montreal, Canada

M. Weber A. Lamarre

Ped. Ped Pulm.

1972

Children’s Hospital “Dr. R. Guiterrez,” Buenos Aires, Argentina

J. Sasbon

Ped.

1972

Children’s Hospital “Pedro Elizade,” Buenos Aires, Argentina

C. Bonno

Ped.

1972

Hospital for Sick Children, Edinburgh, Scotland

H. Simpson

Pulmonology

1974

Red Cross Children’s War Memorial Hospital, Cape Town, South Africa

M. Klein

Ped Pulm.

1975

Private Hospital, Uruguay

M. Gajer

Ped.

1975

Children’s National Hospital Medical Center, Washington, DC

P.R. Holbrook A. Fields

Ped. Ped.

1975

Children’s Medical Center, Dallas, Texas

D. Levin F. Morriss

Ped. Ped. & Ped Anesth.

1976

Hospital Infantil La Paz, Madrid, Spain

F. Ruza

Ped.



CHAPTER 1

History of Pediatric Critical Care Medicine

5

TABLE Some Early Pediatric Intensive Care Units and Programsa—cont’d 1.1

Year

Institution/Location

Medical Director(s)

Director(s) Specialtyb

1977

Johns Hopkins Medical Center, Baltimore, Maryland

M.C. Rogers S. Nugent

Ped. & Ped Anesth.

1977

Sheba Medical Center, Israel

F. Barzilay

Ped.

1977

Children’s Hospital of San Diego, San Diego, California

B. Peterson

Ped. & Ped Anesth.

1977

Hospital de Clinicas, Sao Paulo, Brazil

A. Wong

Ped.

1978

Hospital Sãa Lucas da PUCRS, Porto Alegre, Brazil

P. Celiny Garcia

Ped.

1978

Sophia’s Children’s Hospital, Rotterdam, Netherlands

E. van der Voort H. van Vught

Ped. Ped.

1978

Children’s Hospital of Los Angeles, Los Angeles, California

E. Arcinue

Ped.

1979

University of Minnesota Hospital, Minneapolis, Minnesota

B. Fuhrman

Ped.

1979

Hospital de Clinicas de Porto Alegre, Brazil

P.R. Carvalho

Ped.

1980

Moffett Hospital, San Francisco, California

G. Gregory

Ped Anesth.

1980

Children’s Hospital Boston, Boston, Massachusetts

R. Crone

Ped. & Ped Anesth.

Ped., Pediatrics; Ped Anesth., pediatric anesthesiology; Ped Pulm., pediatric pulmonology. a This is not intended to be a complete list. It is primarily composed of units well documented in the literature and personally known to the authors. b Primary specialties (not all-inclusive). c Although conceptual development of unit started in 1965, Dr. Berlin states that the first year of operation of the present ICU was in 1969 (opened December 1968). d Columbia Hospital District of Columbia was a precursor of Children’s National Hospital Medical Center. e This 20-bed state-of-the-art unit followed an experience with four designated beds in the PACU beginning in 1964. Data from references 1, 4, 14–23.

development of many techniques to operate on noncardiac congenital malformations. His protégé, Dr. Robert Gross, first successfully operated on patent ductus arteriosus in 1937 and later on other congenital cardiac lesions. Dr. C. Crawfoord in Sweden and Dr. Gross in Boston both successfully repaired a coarctation of the aorta in 1945. In the same year, at Johns Hopkins, Dr. Alfred Blalock (surgeon) and Dr. Helen Taussig (cardiologist) with Mr. Vivien Thomas (laboratory assistant) created the subclavian-to-pulmonary artery shunt for tetralogy of Fallot. Dr. John Gibbon at Jefferson Medical College Hospital in Philadelphia performed the first successful open-heart surgery using cardiopulmonary bypass for closure of an atrial septal defect in an adolescent girl in 1953.1 These advances in pediatric surgery created the need for excellent and often complex postoperative care. Dr. C. Everett Koop, who had completed surgical residency at the University of Pennsylvania in 1945, then trained in Boston with Dr. Gross for 6 months. He returned to the University of Pennsylvania and the Children’s Hospital of Philadelphia (CHOP) in 1946. With the help of Dr. Leonard Bachman, director of anesthesiology, and the nursing staff, Dr. Koop developed the first neonatal surgical ICU in 1962. Dr. Bachman and his young associate, John J. Downes, subsequently set up North America’s first PICU service with a full-time medical and nursing staff in 1967 at CHOP.

Neonatology Pediatric critical care owes a great debt to neonatologists and their special care nurseries.1,4,24 The first and most prominent of these

was established in the 1880s in Paris by obstetrician Etienne Tarnier and his young associate Pierre Budin at the Hôpital la Charitre with a unit that had a full-time dedicated nursing staff, an antiseptic environment, incubators, and gavage feeding of breast milk. The practices reduced hospital preterm infant mortality in less than a decade from 197 per 1000 live births to 46 per 1000 live births. Their work presaged the development of modern neonatal intensive care in the 20th century. In 1914, the first premature infant center in the United States was opened at Michael Reese Hospital in Chicago by Dr. Julius Hess (1876–1955). Canadian pediatrician Dr. Alfred Hart performed exchange transfusions involving peripheral artery cannulation in 1928. In 1932, Drs. Louis Diamond, Kenneth Blackfan, and James Batey at BCH determined the pathophysiology of hemolytic anemia and jaundice of erythroblastosis fetalis. In 1948, they described exchange transfusions using a feeding tube inserted into the umbilical vein. In the 1950s and 1960s, Dr. Geoffrey Dawes at the Nuffield Institute for Medical Research at Oxford University described for the first time the fetal and transitional circulation of mammalian newborns using fetal and newborn lambs. In the late 1950s, Columbia University’s obstetrical anesthesiologist, Virginia Apgar, who had devised the Apgar score for assessing birth asphyxia, recruited Dr. L. Stanley James to develop animal and human investigation of transitional pulmonary-cardiovascular adaptation during labor, delivery, and the postnatal period. Dr. James and his team at Columbia and Dr. Abraham Rudolph, a South African pediatric cardiologist, and his team at Albert Einstein Medical Center in New York City and subsequently at the Cardiovascular Research Institute in San Francisco, performed extensive studies

6

SECTION I



Pediatric Critical Care: The Discipline

in fetal lambs, rhesus monkeys, and term and preterm human newborns that defined the human cardiopulmonary adaptation to delivery and postnatal life. They also determined the biochemical factors and time course of birth asphyxia and recovery. In 1959, a research fellow at Harvard, Dr. Mary Ellen Avery (with mentor Dr. Jere Mead), discovered deficiency of alveolar surfactant in lungs of newborns dying from respiratory distress syndrome (RDS). This discovery led to a better understanding of neonatal pulmonary disorders and eventually led to the intratracheal instillation of surfactant in newborn preterm infants to prevent or mitigate the severity of RDS. In the 1960s, state-of-the-art neonatal ICUs were established at Columbia-Presbyterian Hospital (Dr. William Silverman), University of Pennsylvania (Dr. Thomas Boggs), Vanderbilt University (Dr. Mildred T. Stahlman), Toronto Hospital for Sick Children (Dr. Paul Swyer), and the University of California at San Francisco (Dr. William H. Tooley).

Pediatric Cardiology As previously indicated, the vision of Dr. Taussig in devising a method to treat “blue babies” and successful cardiac operations led to infants and children who survived surgery and needed postoperative intensive care. Advances in technology, especially in imaging, have allowed clinicians to “see” into living patients with astounding accuracy. Increased understanding of anatomy and physiology has led to improved surgical and nonsurgical care for children with complex cardiopulmonary problems. Developments in cardiac catheterization and interventional radiology have enabled clinicians to treat many lesions without open-heart surgery and potentially difficult postoperative intensive care. This concept was introduced in 1968 by Dr. William Rashkind at the Children’s Hospital of Philadelphia (CHOP) with the introduction of the balloon atrial septostomy for infants with transposition of the great arteries. Growth of techniques that allow effective intervention in many complex cardiac conditions, both nonsurgical and surgical, has resulted in many pediatric centers creating specific cardiac ICUs, often run by pediatric cardiac intensivists. Cognitive impairment in some infants with complex lesions or chromosomal abnormalities and the occasional development of chronic respiratory failure with dependence on mechanical ventilation for months or years are two of the occasional major sequelae of these highly successful endeavors. The value of PCCM for these cardiac patients and other critically ill children has been well documented by Dr. Jacqueline Noonan, who noted, “Much success of the surgery can be attributed to a group of pediatric intensivists, pediatric intensive care units, improved ventilator support, and trained respiratory therapists.”25

Early Use of Mechanical Ventilation in Neonates and Children The first series of carefully observed infants and children treated for respiratory failure was published in 1959. In that year, Drs. P.M. Smythe (pediatrician) and Arthur Bull (anesthesiologist) reported the first real success in mechanical ventilation of a series of neonates with respiratory failure caused by neonatal tetanus. These infants were paralyzed with curare to relax the tetanic muscle spasms and ventilated for 4 to 14 days using tracheotomy and a modified Radcliff adult ventilator.26 Until that time, infants or children were rarely given ventilator support for more than a few hours, with either adult ventilators or manual ventilation.

Neither specifically designed pediatric ventilators nor small-volume blood gas analysis was available. Dr. Smythe had to overcome these obstacles by innovation. Due to local cultural practices, Bantu children from tribal areas were particularly prone to develop tetanus. On July 13, 1957, at Groote Schuur Hospital, he performed a tracheostomy and began intermittent positive pressure ventilation for these infants with the assistance of anesthesiologist Dr. Bull. This was truly a landmark event in the evolution of PCCM. Although considered a success story in that it was the first time that infants survived up to weeks of positive-pressure mechanical ventilation, the first seven of nine patients died. Eventually, their survival rate reached 80% to 90%. Drs. Smythe and Bull commented, “No praise can be too high for the nursing staff, who were all student nurses and without any special training.” David Todres, a medical student at that time, was giving curare to and observing these infants, sparking his interest in critical care. In 1963 to 1964 in Toronto, Drs. Paul Swyer, Maria DelivoriaPapadopoulos, and Henry Levison were the first to successfully treat a series of moribund premature infants with RDS and respiratory failure. They used positive-pressure mechanical ventilation and supportive care27 and emphasized the importance of a fulltime team, including dedicated nurses and therapists as well as physicians. In 1968 Dr. George Gregory and colleagues at the University of California at San Francisco demonstrated improved survival with early use of continuous positive airway pressure without assisted ventilation or with positive end-expiratory pressure added to the mechanical ventilation regimen.28 An important contribution to the development of intensive care and long-term mechanical ventilation was the use of plastic endotracheal tubes for prolonged intubation and ventilation. Dr. Bernard Brandstater, an Australian working at the American Hospital in Beirut, Lebanon, reported prolonged nasotracheal intubation as an alternative to the tracheostomy at the First European Congress of Anesthesia in Vienna in 1962.29

Poliomyelitis and Creation of the First Intensive Care Units Poliomyelitis epidemics occurred worldwide in the early 20th century but seemed especially severe in Western Europe and North America. There was no treatment and, until the late 1920s, no effective life support for those victims with respiratory failure. Fortunately, the confluence of great scientific and clinical minds and the organizational efforts of physicians, nurses, and therapists addressing the needs of polio patients led to the creation of dedicated polio respiratory care units for patients of all ages. In 1929, Philip Drinker, an engineer—with pediatricians Louis Shaw and Charles F. McKhann at BCH—published their experience with an electrically powered negative pressure, body-enclosing mechanical ventilator, later termed the iron lung.10,30 Polio outbreaks occurred in the summer months worldwide in the 1930s and 1940s. The polio epidemics of the early 1950s were very severe in Los Angeles and Copenhagen. In 1952, Dr. H.C. Lassen, the chief epidemiologist at Blegdam Hospital in Copenhagen, described treating 2772 patients with polio. Of these, 316 were in respiratory failure and initially received assisted ventilation with iron lungs in a large respiratory care unit. During that summer, they had as many as 70 patients in respiratory failure in that unit. Unfortunately, the mortality of patients supported by an iron lung ventilator was nearly 90%, with the cause of death frequently being unrecognized upper airway obstruction. When



CHAPTER 1

History of Pediatric Critical Care Medicine

7

the number of patients in respiratory failure exceeded the available number of iron lung ventilators, Bjorn Ibsen, the chief of anesthesiology at the hospital, with the help of his medical staff and nurse anesthetists, performed tracheal intubation and then tracheostomy along with manual positive pressure ventilation with 50% oxygen and tracheal suctioning. This treatment was carried out in 200 patients with respiratory failure. To provide continuous manual ventilation on a 24-hour basis, Ibsen recruited, trained, and used 200 nursing students and aides along with 200 medical students, each working 8-hour shifts to provide manual ventilation, as well as 27 technicians per day to care for the patients. The mortality in patients receiving this treatment decreased from 90% to 40%.31–33 At that time, patients from outlying areas were transported to hospitals in ambulances without sufficient attendants or airway care and arrived moribund. Lassen and Ibsen started to send socalled retrieval teams in ambulances out to pick up the patients in the countryside, with marked improvements in status on arrival. They also started passing stomach tubes early on for nutrition, and the rubber-cuffed tracheostomy tubes were replaced with a silver cannula that caused less tracheal mucosal damage. Even with all of these improvements, Dr. Ibsen noted, “Naturally we ran into a lot of complications.”33 Drs. Ibsen and Lassen also received help from other people who were focusing their efforts on treating polio. The clinical biochemist Dr. Poul Astrup developed a micro method to measure capillary arterialized pH and PCO2 in infants, children, and adults. C.G. Engstrom, a Swedish anesthesiologist, designed and clinically tested the first modern volume-preset positive pressure mechanical ventilator. This spectacular and thrilling story culminated in a cohort of patients with respiratory failure being treated in a single geographic area and cared for by full-time physicians, nurses, and technicians: the first modern ICU. Although these units tended to disband after the summer-fall polio season, they led to the creation of full-time respiratory care units at the Radcliff Infirmary of Oxford University and elsewhere in Europe and North America in the 1950s. Soon after these events, in 1958, Peter Safar led development of the first multidisciplinary ICU in North America at Baltimore City Hospital.34 In 1960, Barrie Fairley and colleagues created the ICU at Toronto General Hospital, followed in 1962 by the ICU at Massachusetts General Hospital under Drs. Henning Pontoppidan and Henrik Bendixen.

Pediatric Intensivist

Definitions

Although many sources emphasize the role of advanced technology in the creation of adult, neonatal, and pediatric critical care,1,19 skilled nursing care was even more important in this evolving process. Porter41 and others remind us of the vital role of nursing in triage and organization of care for patients by degree of illness. Long before the organizational efforts of the 20th century, Florence Nightingale (1820–1910) organized a volunteer service with 20 nurses and created a clean environment at the British military hospital at Skutari, Turkey, in 1854 during the Crimean War. Although the care consisted mostly of hygiene and nutrition, within 6 months of her arrival the mortality rate dropped from 40% to 2%.42 Nightingale provided the definition of nursing as “helping the patient to live.”42 These efforts were continued in the United States by Dorothea Dix (1802–1887) and Clara Barton (1821–1912), the “angel of the battlefield” during the American Civil War. Barton also brought the Red Cross to America in 1882. As the complexity of medical and surgical care evolved in the late 19th and early 20th century, the need to cohort sick patients

Some of the difficulty in relating the history of PCCM is defining a PICU and pediatric intensivist. The current definitions are as follows.

Pediatric Intensive Care Unit An ad hoc committee of the American Academy of Pediatrics (AAP), Diseases of the Chest Section established Guidelines for the Organization of Children’s Intensive Care Units in July 1975.35 In 1983, the AAP and Society of Critical Care Medicine (SCCM) published Joint Guidelines for Pediatric Intensive Care Units,36 which were updated in 199337 and 200438 and then retired in 2013.39 The committee defined a PICU as “a hospital unit which provides treatment to children with a wide variety of illnesses of life-threatening nature including children with highly unstable conditions and those requiring sophisticated medical and surgical treatment.”

Randolph and coworkers40 defined a pediatric intensivist (in the United States) as “any one of the following: (a) a pediatrician with subspecialty training in PCCM and subspecialty certification from the American Board of Pediatrics (ABP); (b) a pediatric anesthesiologist with special competency in critical care with subspecialty certification from the American Board of Anesthesiology; (c) a pediatric surgeon with special competency in critical care with subspecialty certification from the American Board of Surgery; (d) a physician (as above) eligible for subspecialty certification by the appropriate respective board.” Similar requirements for training exist or are in development elsewhere in the world.

First Pediatric Intensive Care Units In 1955 Dr. Goran Haglund at the Children’s Hospital of Göteborg, Sweden,18 developed the first PICU, which he called a pediatric emergency ward. The patient who inspired Dr. Haglund to organize the unit was a 4-year-old boy who was operated on in 1951 for a ruptured appendix. Postoperatively, he lapsed into a coma; his surgeon declared that he had done all he could and the boy would die of bacteriotoxic coma. The anesthesiologist offered to help and the boy was intubated, given manual positive-pressure respiration with generous oxygen, tracheostomized, and given a large blood transfusion. After about 8 hours, the boy’s bowels started to function, and 4 hours later he was out of coma. After 20 hours, he had spontaneous respiration and had been successfully treated for respiratory insufficiency and shock. This new unit had seven acute care beds with full-time nurses and nursing assistants providing 24-hour coverage. In the first 5 years, the team treated 1183 infants and children, with a mortality rate of 13.6%. Haglund went on to state, “But what we did was something else. It was the application of the basic physiology to clinical practice. Our main purpose was not to heal any disease; it was to forestall the death of the patient. The idea was—and is—to gain time, time so that the special medical or surgical therapy can have desired effects.”18 Haglund was also careful to point out: “There are few jobs more exciting, demanding, and taxing than emergency nursing. Our nurses and nurse assistants are tremendous. They must be!”18

Central Role of Critical Care Nursing

8

SECTION I



Pediatric Critical Care: The Discipline

and provide skilled nursing care became apparent, especially for premature newborns and victims of poliomyelitis, as cited earlier. Then, as now, the recovery of the critically ill pediatric or adult patient depended on the skilled nurse at the bedside who was trained to use the life support and monitoring equipment at hand but to remain focused on the stability and comfort of the person in the bed.43 In the mid- to late 1970s, as pediatric cardiovascular surgery for more complex lesions in infants was developing, nurses provided postoperative care in designated units. Children with Reye syndrome suddenly appeared, requiring complex multisystem care. In addition, in the 1980s, emergency medical services systems began transporting severely injured children to hospitals, where they required rapid assessment and intervention by nurses and physicians and initiation of cardiorespiratory and neurologic support.44 Pediatric critical care nurses joined the SCCM from its beginning in 1970 and the American Association of Critical Care Nurses emphasizing the care of children. In the mid-1990s, pediatric critical care nurses founded their own society and established a peer-reviewed journal. Also in the 1990s, advanced practice nurses and nurse practitioners began to specialize in pediatric critical care. They continue to function as important critical care team members to augment both physician and nursing care as well as conduct clinical research.43,44

Role of Pediatric Anesthesiologists and Pediatricians in Founding Pediatric Critical Care Medicine An important early physician-directed multidisciplinary PICU in North America was established at CHOP in 1967 as an outgrowth of a hospital-wide respiratory intensive care service.1,45 The unit consisted of an open ward of six beds equipped with bedside electronic monitoring and respiratory support capabilities and an adjacent intensive care chemistry laboratory staffed 24 hours per day. The nurses were assigned full-time to the unit; most had previously served in the recovery room, infant ICU, or cardiac surgery postoperative ward. Dr. John Downes was the medical director and worked closely with two other anesthesiologists, Dr. Leonard Bachman, chief of anesthesiology, and Dr. Charles Richards, and a pediatric allergist/pulmonologist, Dr. David Wood. Four pediatric anesthesiology/critical care fellows provided 24-hour in-unit service. Dr. C. Everett Koop (chief of surgery), Dr. William Rashkind (the father of interventional pediatric cardiology), Dr. John Waldhausen (one of the nation’s few full-time pediatric cardiac surgeons), Dr. Sylvan Stool (a pioneer in pediatric otolaryngology), and other staff and residents provided close collaborative patient care, education, and clinical research. By 1975, with the establishment of the new CHOP building, the acute PICU was expanded to 20 beds with an adjacent 10-bed intermediate step-down unit. In 1969, Dr. Peter Safar and his trainee, Stephen Kampschulte, developed a 10-bed PICU at the Children’s Hospital of Pittsburgh. That same year, James Gilman, a pediatric anesthesiologist, and Norman Talner, a pediatric cardiologist, established a six-bed PICU at the Yale–New Haven Medical Center. In 1970, at the Hospital for Sick Children in Toronto, Dr. Alan Conn resigned as director of the Department of Anesthesiology to become director of a new multidisciplinary 20-bed PICU, by far the largest and most sophisticated unit in North

America. During the prior decade, Dr. Conn and his colleagues had treated critically ill infants and children in a sequestered area of the postanesthesia care facility where they had developed considerable expertise in critical care. The new state-of-the-art PICU was the forerunner of units developed in major pediatric centers throughout North America spanning the 1970s and beyond. Dr. Geoffrey Barker, who went on to develop one of the largest multinational fellowship training programs in the world, followed Dr. Conn as director of the PICU. Also in 1971, Dr. David Todres, an anesthesiologist and pediatrician, and Dr. Daniel Shannon, a pediatric pulmonologist, founded a 16-bed multidisciplinary unit for pediatric patients of all ages at the Massachusetts General Hospital.1,4 The units in Philadelphia, Toronto, and Boston established vibrant training programs in critical care medicine and conducted clinical research. Among their numerous accomplishments, Dr. Conn became a noted authority on the management of near-drowning victims, and Dr. Todres and Dr. Downes pioneered long-term mechanical ventilation for children at home with chronic respiratory failure. These early PICUs and their training programs had a favorable impact on mortality and morbidity rates, particularly those associated with acute respiratory failure, leading to the development of similar units and programs in most major pediatric centers in North America, Western Europe, and Japan during the 1970s and early 1980s. The development of the PICU at Children’s Memorial Hospital (CMH), Northwestern University Medical School, Chicago, illustrates how many of the early PICUs evolved. The unit was first started as a four-bed area set in one of the postoperative care wards by pediatric anesthesiologists David Allen and Frank Seleny. Anesthesiologist Dr. John Cox arrived in August of 1964 and was named director. He has stated that the unit never formally opened. It began in the four-bed unit in the postoperative ward in 1964 and became a 14-bed separate designed unit in late 1967. Dr. Cox was the director until 1975, when he was succeeded by Dr. Richard Levin. During this time, Dr. Hisashi Nikaidoh, who was a surgery resident from 1966 to 1967, remembers taking care of a renal transplant patient; the care was provided by nephrology, general surgery, and immunology without a centralized PICU service. Dr. Zehava Noah, who was educated in Israel and trained in the United Kingdom, did a critical care fellowship in anesthesia at CMH, developed a closed medical-surgical PICU in 1979, and was named the director in 1981. There was also an associate surgical director.46–49 Some of the early PICUs were directed by pediatricians. In 1966, Dr. Max Klein joined Drs. H. de V. Heese and Vincent Harrison in a two-bed neonatal research unit at the Groote Shuur Hospital in Cape Town. Their research resulted in many significant papers, not the least of which was “The Significance of Grunting in Hyaline Membrane Disease,”50 demonstrating that oxygen tensions fell when infants had tracheal intubation, eliminating the ability to grunt on exhalation. By 1969, at Red Cross War Memorial Children’s Hospital in Cape Town, South Africa, pediatric patients with respiratory failure (e.g., Guillain-Barré syndrome) were ventilated on the general wards. Although outcomes improved, deaths were still common. Dr. Max Klein encouraged Dr. Malcolm Bowie (consultant) to start a six-bed ICU, or “high-care ward.” After further training in South Africa and at the University of California San Francisco (UCSF), Dr. Klein returned to Cape Town in 1974, where he combined the neonatal tetanus ward of Dr. Smythe and the six-bed ICU of Dr. Bowie into the first full-time PICU in South Africa.51



CHAPTER 1

The path for pediatricians providing care for the sickest patients on a full-time basis remained unclear for an extended period. Subsequent early leaders in the field each carved out their own path. Dr. Daniel Levin completed pediatric cardiology and neonatology fellowships to learn the care of sick children. However, he found few Chairs of Pediatrics interested in hiring an “intensivist.” Then, in 1975, Drs. Levin and Frances Morriss (trained in pediatrics and pediatric anesthesia) were recruited to start a PICU at Children’s Medical Center of Dallas. There were so few of this new breed of intensivists that many became directors upon completion of residency and fellowship. At the beginning, few other physicians wanted to be responsible for pediatric intensive care.23 Eventually, more pediatricians decided to devote their careers to being members of a multidisciplinary team taking care of the sickest children in hospitals on a full-time basis. In 1975, the CHOP program started to accept PCCM trainees who were pediatricians without anesthesia training. In 1967, Dr. Peter Holbrook as a medical student at the University of Pennsylvania began a part-time job in the PICU at CHOP and developed a strong interest in PCCM. Informed at the time that one needed anesthesia training to successfully work in the PICU, Holbrook shelved the idea and entered pediatric residency training at Johns Hopkins. When the PCCM idea resurfaced, he found that many still felt a physician needed anesthesia training to function in the PICU. Disagreeing with the reasoning behind such a requirement, he pursued critical care

History of Pediatric Critical Care Medicine

9

training with Dr. Peter Safar in Pittsburgh, who welcomed him as a fellow in critical care medicine. In 1975, Dr. Holbrook and pediatrician Dr. Alan Fields, who also trained in Pittsburgh, were recruited to the new, modern Children’s Hospital National Medical Center (Washington, DC) as pediatricians in the Department of Anesthesia to direct the PICU. Dr. Bradley Peterson,52 after pediatric and neonatology training and an anesthesiology residency at Stanford University, became director of the new PICU at Children’s Hospital of San Diego in 1977. Dr. Bradley Fuhrman, following pediatric cardiology and neonatology fellowships, started the first PICU at University of Minnesota Hospital in 1979.53 Dr. George Lister,54 after a pediatric residency at Yale and a fellowship in cardiopulmonary physiology at UCSF, joined the staff at the UCSF Moffitt Hospital San Francisco in 1977 as an attending in its combined adult-pediatric ICU. Due to the director’s illness, he quickly found himself the co-director of the unit.54 He eventually returned to Yale as an attending in the PICU. Dr. Mark Rogers, after completion of a pediatric residency at BCH, an anesthesiology residency at Massachusetts General Hospital, and a pediatric cardiology fellowship at Duke, became director of the first PICU at Johns Hopkins Hospital in 1977.55 Subsequently, in 1980, Dr. Rogers became chair of the Department of Anesthesiology and Critical Care Medicine at Johns Hopkins and chief editor of a major textbook of pediatric intensive care (Table 1.2).

TABLE Textbooks in Pediatric Critical Care Medicine 1.2

First Edition

Title

Editors

Reference

1971

Care of the Critically Ill Child

R. Jones, J.B. Owen-Thomas

56

1971

Pediatric Intensive Care: Manual

K. Roberts, J. Edwards

57

1972

Smith’s The Critically Ill Child: Diagnosis and Medical Management

J. Dickerman, J. Lucey

58

1977

Pediatrie d’urgence

G. Huault, H. Labrune

59

1979 through 1997

A Practical Guide to Pediatric Intensive Care, first and second editions (and accompanying Essentials volumes)

D. Levin, F. Morriss, G. Moore

60–63

1980

Tratado de Cuidados Intensivos Pediatrucos (Textbook of Pediatric Intensive Care)

F.J. Ruza

64

1980

Core Curriculum for Pediatric Critical Care Nursing

M.C. Slota

65

1983

Pediatric Critical Care

J. Bloedel Smith

66

1984

Nursing Care of the Critically Ill Child

M.F. Hazinski

67

1984

Textbook of Critical Care

W.K. Shoemaker, W.L. Thompson, P.R. Holbrook

68

1984

Pediatric Intensive Care

E. Nussbaum

69

1985

Temas em Terapia Intensiva (Critical Care Issues in Pediatrics)

J. Piva, P. Carvalho, P. Celiny Garcia

70

1985

Critical Care Pediatrics

S. Zimmerman, J.H. Gildea

71

1987

Pediatric Intensive Care

J.P. Morray

72

1988

Textbook of Pediatric Intensive Care

M.C. Rogers

73

1992

Pediatric Critical Care

B.P. Fuhrman, J.J. Zimmerman

74

1993

Textbook of Pediatric Critical Care

P.R. Holbrook

75

1994

Urgences & Soins Intensif Pediatriques (Pediatric Emergency and Critical Care)

J. Lacroix, M. Gauthier, P. Hubert, et al.

76

Continued

10

SECTION I



Pediatric Critical Care: The Discipline

TABLE Textbooks in Pediatric Critical Care Medicine—cont’d 1.2

First Edition

Title

Editors

Reference

1995

Critical Heart Disease in Infants and Children

D.G. Nichols, D.E. Cameron, W.J. Greeley, et al.

77

1996

Critical Care of Infants and Children

I.D. Todres, J.H. Fugate

78

1996

Critical Care Nursing of Infants and Children

M.A. Curley, J. Bloedel-Smith, P.A. Moloney Harmon

79

1997

Illustrated Textbook of Pediatric Emergency and Critical Care Procedures

R.A. Dieckmann, D.H. Fiser, S.M. Selbst

80

1997

Pediatric Intensive Care

N.S. Morton

81

2003

Essentials of Pediatric Intensive Care

C.G. Stack, P. Dobbs

83

2005

Medicinia Intensiva em Pediatria

J. Piva, P. Celiny Garcia

84

2005

Cuidudo Intensivo Pediatrico y Neonatal

J. Forero, J. Alarcon, G. Cassalett

85

2006

Pediatric Critical Care Medicine

A.D. Slonim, M.M. Pollack

86

2006

Manual de Cuidado Intensivo Cardiovascular Pediatrico

G. Casselett, M.C. Patarroyo

87

2007

Pediatric Critical Care Medicine: Basic Science and Clinical Evidence

D.S. Wheeler, H.R. Wong, T.P. Shanley

88

2010

Critical Care of Children with Heart Disease

R. Munoz, V. Monell, E. da Cruz, C.G. Vetterly

89

2012

Comprehensive Critical Care: Pediatric Medicine

Society of Critical Care Medicine

90

2012

Pediatric Critical Care Study Guide

S.E. Lucking, F.A. Maffei, R.F. Tamburro, N.J. Thomas

91

2015

Pediatric Critical Care Nutrition

P.S. Goday, N.M. Mehta

92

2017

Pediatric Intensive Care

S. Watson, A. Thompson

93

Growth of Pediatric Critical Care Medicine The field of PCCM grew rapidly in the late 1970s and 1980s. However, there was a struggle for authority in both adult and pediatric units. The culture of intensive care was changing from one in which each specialty service cared for its part of the patient to one in which a full-time critical care service cared for the whole patient, with help of consulting specialties.2,94 For PCCM to achieve its full potential, it required several elements: a national organization to provide a venue in which to meet and communicate, acceptance and validation of pediatric critical care as a subspecialty, nationally approved training requirements, and academic credibility with meaningful research. A small group of interested physicians met at the SCCM National Meeting in 1979 and decided to petition the SCCM to form a section on pediatrics. The society had no subsections, but the petition was successful. The pediatric section with Dr. Russell Raphaely as chair was formed in 1980.1 In 1983, a committee of the SCCM developed guidelines for organization of PICUs36 that were regularly updated37,38 until January 2013, after which time they were retired.39 In 1984, after petitions by pediatric intensivists, a Section of Critical Care Medicine was established in the AAP with Dr. Russell Raphaely as chair.95 These organizations then petitioned for recognition of PCCM fellowships from the

American College of Graduate Medical Education (ACGME) and for the subspecialty of PCCM by the American Board of Pediatrics (ABP). Legitimization of the subspecialty was achieved with establishment of a new subboard of Pediatric Critical Care Medicine of the ABP in 1985 and the first certifying examination in 1987, at which time 182 subspecialists were certified.95 Certification provided a clear basis for hospital credentialing of PCCM physicians.96 In addition to certification by the ABP, the American Board of Anesthesiology and the American Board of Surgery confer subspecialty certification with special competency in critical care. In 1989, special requirements for training in PCCM were developed by the ACGME, with formally accredited programs first recognized in 1990.97

Growth in Numbers of Pediatric Intensive Care Units In 1979, there were 150 PICUs of four or more beds identified, and another 42 thought to exist.98 Most were just special care nursing units, and only 40% had a pediatric intensivist available at all times. Forty percent of the units had fewer than seven beds and only one half had affiliated transport systems. Pediatric ward beds decreased by 22.4% between 1980 and 1989—by 10.8% between 1990 and 1994 and by 15.7% between 1995 and 2000. During the same three time periods, PICU beds



CHAPTER 1

increased by 26.2%, 19.0%, and 12.9%, respectively.40 Between 2001 and 2016, the US pediatric population grew 1.9% to greater than 73.6 million children, and PICU hospitals decreased 0.9% from 347 to 344 (58 closed and 55 opened). In contrast, PICU bed numbers increased 43% (4135 to 5908 beds). Sixty-three PICU hospitals (18%) accounted for 47% of PICU beds.40a According to the FY2017 American Hospital Association (AHA) survey database, there are 399 hospitals in the United States and territories that have a PICU in their hospital.100 Although not all children’s hospitals are members of the Children’s Hospital Association, of the 155 children’s hospitals that contribute data to the fiscal year 2017 Children’s Hospital Association Annual Benchmark Report Survey, 128 (82%) stated they had staffed PICU beds.101

Growth in Training Programs and Education In 1983 to 1984, there were 32 PCCM training programs; the ACGME accredited 28 of them in 1990. By 2018 to 2019, the number had increased to 68 accredited training programs with 527 enrolled fellows, of whom 336 (63.8%) are women.99 Since its inception, the subboard has certified 2693 subspecialists.99 Educational programs in PCCM have progressed considerably at the annual SCCM, AAP, Pediatric Academic Societies, American Thoracic Society, and American College of Chest Physicians meetings, as well as at independent meetings such as the Pediatric Critical Care Colloquium and the World Federation of Pediatric Intensive Critical Care Societies (WFPICCS). Dr. Barker envisioned the need to bring together pediatric intensive care from many parts of the world. This led to his founding directorship of the WFPICCS, which has done much to foster development of pediatric critical care around the world, bringing vital critical care skills and experience to benefit multiple countries. Numerous textbooks on PCCM have appeared in many languages (see Table 1.2), and the journal Pediatric Critical Care Medicine was launched in 2000.102 Academic credibility that results from meaningful scientific research has come slowly. In the early days, intensivists were mostly consumed by clinical care and research and administrative responsibilities. High-quality basic science, epidemiology, and translational studies addressing a broad range of problems have gradually emerged. Multiinstitutional organizations have allowed studies that require more patients than can be drawn from a single institution to be designed, funded, and completed. In the early 1990s, the Pediatric Critical Care Study Group was formed.103 It was followed by the Pediatric Acute Lung Injury and Sepsis Investigators (PALISI) network,104–106 which employed the successful programming model of research developed by the Canadian Critical Care Trials Group.107–109 PALISI has grown and prospered through the voluntary collaboration of currently 94 member PICUs110 and has supported more than 200 articles addressing the spectrum of PCCM.111 The virtual PICU was started in 1997 to bring data management technologies to critical care. In 2004, Virtual PICU Systems (VPS) was formed by Drs. Thomas Rice and Ramesh Sachdeva (Children’s Hospital and Health System of Milwaukee) and Dr. Randall Wetzell (Children’s Hospital Los Angeles) in conjunction with the National Association of Children’s Hospitals and Related Institutions to develop a PICU registry to facilitate quality improvement and research. VPS currently has more than 125 members and a massive database describing more than 1 million critical care admissions.112,113

History of Pediatric Critical Care Medicine

11

In April 2004, the Eunice Kennedy Shriver National Institute of Child Health and Human Development (NICHD) established funding for the first federally supported network for pediatric critical care research, the Collaborative Pediatric Critical Care Research Network. The network is a multicentered program designed to investigate the safety and efficacy of treatment and management strategies to care for critically ill children as well as the pathophysiologic basis of critical illness and injury in childhood.114–117 The NICHD has also supported research in PCCM by developing and supporting young investigators in the field through the Pediatric Critical Care and Trauma Scientist Development Program (PCCTSDP), a K-12 research training program. The PCCTSDP has been funded since 2004 and is directed by Dr. Heather Keenan at the University of Utah. Eligible applicants are board-eligible or board-certified PCCM faculty, or pediatric trauma surgery faculty.114 Perhaps most notably, in 2013 the NICHD created an independent branch, the Pediatric Trauma and Critical Illness Branch, to further support research in pediatric critical illness and injury. The mission of the new branch is to prevent and reduce all aspects of childhood trauma and critical illness and to enhance health outcomes for all children across the continuum of care.114,116,117 The growth of education and research in PCCM has coincided with, and presumably resulted in, better care for children as reflected in the decrease in mortality from septic shock. Between 1958 and 1966, in patients younger than 16 years of age at the University of Minnesota, mortality in septic shock was 95%; now, with PICU care, it is less than 10%.118 Drs. Murray Pollack and Timothy Yeh established the basis for studying severity-adjusted mortality in pediatrics and demonstrated that patients do better when cared for by pediatric intensivists.119 Although many would attribute these improvements to technology and scientific advances, Dr. Yeh and others remind us that the presence of a fulltime nursing and medical team and attention to basic principles rather than exotic high technology improve outcomes.120 This is echoed by Dr. Shann’s two rules of PCCM: (1) “the most important thing is to get the basics exactly right all of the time,” and (2) “organizational issues are crucially important.”23 In addition, Yeh as well as Ibsen33 and Orr have emphasized the important contributions of regionalization and the quality of PCCM transport teams in improving outcomes.121,122 Modern medical simulation originated in pediatrics and has made significant contributions to education. In 1960, shortly after resuscitating his 2-year-old son following a drowning, Asmun Laerdal, the owner of a Norwegian doll factory, partnered with the Red Cross to create the first medical simulation mannequin. In 1988, Laerdal partnered with the American Heart Association and the AAP to create Pediatric Acute Life Support simulationbased training. Since that time, evolving pediatric residency and fellowship requirements, duty hour restrictions, and an increased focus on medical safety have catalyzed exponential growth in simulation training.123–125 The International Network for SimulationBased Pediatric Innovation Research and Education has documented an increase in pediatric simulation centers from 50 to 268 in the past 7 years. A recent meta-analysis documented 57 studies and over 3500 learners engaged in pediatric simulation education. Studies compared simulation education with no intervention and found large effects for outcomes of knowledge, behavior with patients, and time to task completion.126 Dr. Elizabeth Hunt along with pioneers in simulation at Johns Hopkins have been able to document progressive acquisition of

12

SECTION I



Pediatric Critical Care: The Discipline

resuscitation skills during pediatric residency through the use of rapid-cycle deliberate practice in simulation training127 as well as retention of skills in pediatric trauma management 6 months after simulated pediatric trauma education in emergency departments.128 Researchers at the University of Michigan have demonstrated significant correlation between simulation-based education and a 50% improvement in pediatric patient cardiopulmonary arrest survival rates.129 Simulation continues to grow in the education of pediatric residents, fellows, and attendings worldwide as a platform for mastery through deliberate practice providing the opportunity for immediate feedback in high-risk, low-frequency events without compromising patient safety. Many pediatric intensivists have contributed to local and national organizations and have been rightfully recognized for that. Some of the individuals from organizations in the United States are listed in Tables 1.3 through 1.6.

TABLE Chairs, Pediatric Critical Care Medicine 1.3 Subboard, American Board of Pediatricsa 1985–1987

Peter Holbrook, MD

1988–1990

Bradley Fuhrman, MD

1991–1992

Thomas Green, MD

1993–1996

Ann Thompson, MD

1997–1998

TABLE Chairs, Executive Committee, Section on Critical 1.5 Care Medicine, American Academy of Pediatrics 1984–1987

Russell C. Raphaely, MD

1987–1990

Fernando Stein, MD

1990–1992

J. Michael Dean, MD

1992–1996

Kristan Outwater, MD

1996–2000

Timothy Yeh, MD

2000–2004

M. Michele Moss, MD

2004–2008

Alice Ackerman, MD

2008–2012

Donald Vernon, MD

2012–2016

Edward Conway Jr, MD

2016–2018

Michael Agus, MD

2018–2021

Elizabeth Mack, MD

TABLE Chairs, Pediatric Section, Society of Critical Care 1.6 Medicine 1980–1981

Russell C. Raphaely, MD

1981–1983

Peter R. Holbrook, MD

1983–1984

Bernard Holtzman, MD

1984–1985

Bradley Fuhrman, MD

Daniel Notterman, MD

1985–1986

Frank R. Gioia, MD

1999–2000

David Nichols, MD

1986–1987

Timothy S. Yeh, MD

2001–2002

Jeffrey Rubenstein, MD

1987–1988

Fernando Stein, MD

2003–2004

Alice Ackerman, MD

1988–1989

Thomas B. Rice, MD

2005–2006

Donald Vernon, MD

1989–1991

Ann E. Thompson, MD

2007–2008

Karen Powers, MD

1991–1994

J. Michael Dean, MD

2009–2010

M. Michele Mariscalco, MD

1994–1996

Debra H. Fiser, MD

2011–2012

Laura Ibsen, MD

1996–1998

Thomas P. Green, MD

2013–2014

Susan Bratton, MD

1998–2000

Daniel A. Notterman, MD

2015–2016

James Fortenberry, MD

2000–2002

Richard J. Brilli, MD

2017–2018

Jeffrey Burns, MD

2002–2004

M. Michele Moss, MD

Folafoluwa Odetola, MD

2004–2006

Stephanie A. Storgion, MD

2006–2008

Edward E. Conway Jr, MD

2008–2010

Vicki L. Montgomery, MD

2010–2012

Jeffrey P. Burns, MD

2012–2014

Ken Tegtmeyer, MD

2014–2016

Derek Wheeler, MD

2016–2018

Thomas A. Nakagawa, MD

2018–2020

David A. Turner, MD

2020–2022

Alexandre T. Rotta, MD

2019–2020 a

Medical Editor, 1985–2004, George Lister, MD; 2004, Jeffrey Rubenstein, MD.

TABLE Pediatric Intensivists Serving as President 1.4 of Society of Critical Care Medicine 1982–1983

George Gregory, MD

1984–1985

Dharmpuri Vidyasagar, MD

1988–1989

Peter R. Holbrook, MD

1992–1994

Russell C. Raphaely, MD

2001–2002

Ann E. Thompson, MD

2004–2005

Margaret M. Parker, MD

2018–2019

Jerry J. Zimmerman, MD

Cost of Success in Pediatric Critical Care Medicine Everything comes at a cost. In the field of PCCM, as in many others, advances have led to increased financial cost, survivors with chronic disease, medical errors, and occasional dehumanization of

12.e1

Canada As described earlier, Dr. Alan Conn, anesthetist-in-chief at the Hospital for Sick Children, Toronto, envisioned and successfully opened a multidisciplinary PICU for medical and surgical patients in 1971.138 At the Children’s Hospital of Montreal, a medical PICU was created in 1972 by a pediatrician, Dr. Michael Weber, and pulmonologist Dr. Andre Lamarre. Drs. Marie Gauthier, Jacques Lacroix (University of Montreal), and John Gordon (McGill University) in 1992 were active in the development and implementation of a fellowship program in PCCM supervised by the Royal College of Physicians and Surgeons of Canada.139

South Africa As noted earlier, in 1959 Drs. P.M. Smythe and Arthur Bull conceived a brilliant therapeutic plan to treat infants afflicted with neonatal tetanus from infected umbilical cord stumps.26 Dr. Max Klein continued their tradition at Red Cross Children’s War Memorial Hospital, opening a special unit for critically ill children in 1974 with full-time intensivists Drs. Louis Reynolds, Jan Vermeulen, Paul Roux, and, later, Andrew Argent.4,5,140

Japan In the 1960s, Dr. Seizo Iwai, chief of anesthesia at the National Children’s Hospital in Tokyo, was the first Japanese physician to introduce mechanical ventilation and arterial blood gas analysis of critically ill infants, fostering a tradition of anesthesiologists taking care of critically ill infants and children outside of the operating room. He was a strong force in developing a close relationship with other Asian countries, inviting trainees from those countries to promote teaching and pediatric critical care development in their homeland. His close working relationship with Drs. Conn and Barker in Toronto paved the way for Dr. Katsuyuki Miyasaka to study in Toronto with Dr. Conn and in Philadelphia with Dr. Downes. Dr. Miyasaka returned to Japan in 1977. In October 1994, he opened the first geographically distinct PICU in Japan at the National Children’s Hospital and helped to found the Japanese Society of Pediatric Intensive Care. Dr. Miyasaka continues to foster the development of a new generation of pediatric intensivists as a hospital director, playing a major role in facilitating this process.141

India Neonatal intensive care units (NICUs) in India were established in the 1960s, first at the All India Institute of Medical Sciences, Delhi, and subsequently at teaching hospitals in other major cities.142–144 The first PICUs were established at major postgraduate centers (Delhi, Chennai, Chandigarh, Mumbai, and Lucknow).145 Growth of PICUs had been mainly in the private sector, although major government teaching hospitals are also improving the PICUs in their locations. An intensive care chapter of the Indian Academy of Pediatrics was formed in 1997. The Pediatric Section of Intensive Care of the Indian Society of Critical Care Medicine (ISCCM) was formed in 1998.146 There are four streams of formal training available: (1) a 3-year doctor of medicine program at three institutions, (2) a 2-year fellowship offered at 12 centers by the National Board of Examinations, (3) 1- and 2-year fellowships offered at 33 centers by the

College of Pediatric Critical Care, and (4) a 1-year fellowship offered at 18 centers by the Indian Academy of Pediatrics— Intensive Care Chapter.147,148 Whereas the government-supported transport service has spread to many states, it transports PICU patients to government hospitals only. There is a growing awareness that outcomes are better if patients are transported by specialized teams; therefore, an increasing number of critically ill children are transported by dedicated transport teams maintained by major PICUs in private hospitals.149

Australia and New Zealand As in the United States and Canada, Australian PICUs started forming in the early 1960s, arising from wards that performed postoperative recovery care for children following congenital heart surgery,150–157 with continuing development of PICUs in major cities. There was creative development in many centers throughout Australia, some of which are in Melbourne, Adelaide, Camperdown, Brisbane, and Perth. We will present some of the history and refer the reader to a more detailed account with AUSPIC News edited by Frank Shann in 1993.150 At the Free Hospital for Sick Children Melbourne, pediatric anesthesiologists Jan H. McDonald and John Stocks in 1963 developed a 10-bed multidisciplinary PICU. They conducted clinical studies, including a large study demonstrating the safety and efficacy of oral and nasal plastic endotracheal tubes for airway management of children requiring mechanical ventilation.158 In Adelaide Children’s Hospital, Australia, in the early 1960s, Tom Allen and Ian Steven from the Department of Anesthesia began treating upper airway obstruction with prolonged oral and then nasal intubation.159 Long-term mechanical ventilation using Bird ventilators commenced in 1963.160 Pediatric intensive care was established at Princess Margaret Hospital in Perth by Nerida Dilworth (anesthesia) in the early 1960s. Prolonged nasal intubation was first performed in September 1963. In May 1969, a dedicated area in one of the medical wards was set aside for the care of critically ill children. The first full-time intensivists were appointed in 1986 (Alan Duncan, director, and Paul Swan, specialist). The modern 10-bed PICU was opened in April 1987.161 Matthew Spence, an anesthesiologist originally from Glasgow, pioneered critical care medicine for adults and children in Auckland Hospital, New Zealand, opening the first adult and pediatric ICU in 1958. Cardiac surgery for children in New Zealand started at Green Lane Hospital, Auckland, in the 1950s. A dedicated intensive care unit was established there in 1963, led by the cardiac surgeon Sir Brian Barrett-Boyes and anesthesiologists Drs. Marie Simpson and Eve Scelye. The first specialized pediatric emergency transport service commenced in Victoria in 1980.162–166 In March 1991, Elizabeth Segedin was appointed as a full-time pediatric intensivist (with no PICU!) to plan for the development of a pediatric unit. With the building of the new hospital for children in Auckland, intensive care for children was reorganized to a single PICU at the Starship, which opened on December 2, 1991, with Dr. Segedin as director.

Europe In Europe, pediatric intensive care followed shortly after the poliomyelitis epidemic in Denmark in 1952. As described

12.e2

previously, in 1955 Dr. Goran Haglund, a pediatric anesthesiologist, established the first medical-surgical PICU for infants and children at the Children’s Hospital in Göteborg in Sweden.18 In 1961, Dr. Hans Feychting, also a pediatric anesthesiologist, established the first PICU at St. Goran’s Children’s Hospital in Stockholm, Sweden, and became recognized as a pioneer in the development of pediatric intensive care in Europe. In France, in 1963, a newborn presented with tetanus and was admitted to L’Hôpital des Enfants Malades of Paris. Shortly afterward, Dr. Gilbert Huault and J.B. Joly, both neonatologists, opened the first multidisciplinary PICU in France at Saint Vincent de Paul Children’s Hospital. This unit was the first pediatrician-directed PICU in Europe; it soon became a major influence on the development of PICUs. Drs. Francois Beaufils, Jean Christophe Mercier, and Denis Devictor were to play an important role in further development of European pediatric critical care medicine.167 In London, a pediatric anesthesiologist, William Glover, opened a unit for care of postoperative cardiac patients in 1961 at the Hospital for Sick Children on Great Ormond Street. Soon, all patients needing ventilator care were admitted to that unit.168 In 1964, a well-designed discrete 13-bed PICU was developed by Dr. G. Jackson Reese, a pediatric anesthesiologist, at the Alder Hey Children’s Hospital in Liverpool. Other units soon followed, essentially serving as areas allowing prolonged postoperative support.169 In Spain, pediatrician Dr. Francisco Ruza started working in neonatal surgical intensive care in 1969. In 1976, he opened a multidisciplinary medical-surgical PICU for older infants and children at Hospital Infantil La Paz in Madrid. This center, directed by Dr. Ruza, has served as a major training center for pediatric intensivists not only from Spain but from South America as well.170 From this center, Dr. Ruza has also promoted the teaching and high-quality research related to pediatric critical pathology. The first PICUs in the Netherlands were established in the late 1970s and early 1980s at Rotterdam’s Sophia Children’s Hospital under Edwin van de Voort and Hans van Vught in Rotterdam. PICUs were also developed at Wilhelmina Children’s Hospital in Utrecht and Emma Children’s Hospital at the Academic Medical Center in Amsterdam.171 These PICUs are multidisciplinary, and all are part of university teaching hospitals. In 1995, the Dutch Pediatric Association established a section on Pediatric Intensive Care Medicine that certifies training of nearly all Dutch pediatric intensivists. A nationwide transport system connects this centralized care system of pediatric critical care. Units were opened in Germany172 and Slovakia173 as well as in Krakow, Poland, and many other European locations.

Israel Although located in the Middle East, Israel has traditionally been part of European scientific organizations, and most pediatric intensivists in Israel have trained in North America. The first PICU in Israel was established in 1977 by Dr. Zohar Barzilay as a fivebed facility located within Children’s Hospital at Sheba. Israel now has 12 PICUs and two cardiac PICUs. Extracorporeal membrane oxygenation services as well as cardiac transplantation are provided nationwide as part of the national health insurance program. About 30% of the patients in many of the PICUs in Israel come from the Palestinian Authority. Palestinian physicians trained in PCCM in Israel established the first PICU in Gaza.174,175

Latin America The first PICU in Latin America was established in Argentina at the Dr. Ricardo Gutierrez Children’s Hospital in Buenos Aires in 1969 as part of a general surgery ward. In 1972, Dr. Jorge Sasbon became the first staff director of the PICU. In 1972, a PICU was set up in Pedro de Elizalde Children’s Hospital, Buenos Aires, under guidance of Dr. Clara Bonno.23 In Brazil in the 1970s, epidemics of polio and meningococcal disease, with a high mortality, led to the creation of small units for care of these patients. These units were precursors of PICUs later established at Hospital das Clínicas São Paulo by Dr. Anthony Wong (1977), at Hospital São Lucas in Porto Alegre by Dr. Pedro Celiny (1978), at Hospital de Clínicas de Porto Alegre (1979) by Paulo R. Carvalho, and in Rio de Janeiro. In 1982, Dr. Jefferson Piva opened a 13-bed PICU at Hospital da Criança Santo Antonio in Porto Alegre.176 One of the pediatric critical care pioneers in Latin America was Dr. Mauricio Gajer, a dedicated physician from Uruguay. Dr. Gajer traveled to France, where he worked with Professors Huault and Beaufils. After returning to Uruguay, he created the first private PICU in Montevideo, Uruguay, in 1975. In Colombia, pediatric intensive care started in the early 1960s with postoperative care of cardiovascular patients in Clinica Shaio of Bogotá, with adult cardiologists in charge. In the 1970s, Dr. Merizalde, a pediatrician with training in pediatric cardiology, provided care for pediatric cardiovascular patients.177 In 1956 at Luis Calvo Mackenna Children’s Hospital in Santiago, Chile, a single-bed postoperative care unit was started by Drs. Helmut Jager (cardiac surgeon) and Fernando Eimbecke (cardiologist). In 1968, this evolved into a five-bed PICU led by Dr. Eduardo Bancalari, a neonatologist. He was later joined by pediatricians Drs. Patricio Olivio and Jaime Cordero. In the 1980s, additional PICUs developed, including one with Dr. Carlos Casar at Roberto del Rio Children’s Hospital in Santiago, Chile, later directed by Dr. Bettina von Dessauer (pediatrician).178,179 Intensivists there have devoted great effort toward developing a network and transport systems to overcome the impact of Chile’s challenging geography. Similarly, the first intensive care unit in San Jose, Costa Rica, was opened in 1969 at Hospital Nacional de Niños “Dr. Carlos Sáenz H” as a postoperative cardiac care unit. It was initially a nine-bed unit run by anesthesiologists and surgeons. Eventually, pediatricians without special PCCM training became involved. In 1982, Dr. Aristides Baltodano trained in PCCM in Toronto, becoming the first pediatric intensivist in Costa Rica at Hospital Nacional de Niños “Dr. Carlos Sáenz H.” At present, the PICU is a multidisciplinary unit with active cardiovascular and multiorgan transplant programs.180 Over time, a critical care network throughout Latin America has improved access, transport, and specific critical care knowledge in all countries. However, there is still work to do to facilitate access to critical care and achieve results comparable to those in high-resource countries. Sociedad Latin Americana de Cuidados Intensivos Pediatricos (SLACIP) has played a crucial role. Every 2 years, a Latin American Pediatric Intensive Care Congress takes place. A SLACIP symposium prior to each WFPICCS World Congress has become a tradition since the first world congress in Baltimore in 1992. The common language among more than 70 countries with many cultures and challenges forms a common bond.



CHAPTER 1

patients. Accurate estimates of the extraordinary but often necessary financial costs of modern care of the critically ill child are difficult to obtain but are frequently substantial, even for life-years saved.130,131 Children with preexisting chronic disease and an acute critical illness have prolonged PICU stays, frequent readmissions, and the need for intensive care at home or in the rare pediatric subacute facility. Most patients with these conditions did not survive in the 1960s and early 1970s. Although many very sick children return to complete health, a small number, often with associated complex disorders, survive but live with chronic neurologic, respiratory, cardiac, or renal disease. These children and their families usually require extraordinary medical and social support and advocacy to thrive. As PICUs have evolved, intensivists have developed greater sophistication in dealing with individual family concerns, pain management, ethical issues, palliative care, social and spiritual needs,132 cultural differences, and the value of involving families as members of the team by including them on daily rounds.133 These issues are described in more detail elsewhere in this text. Since the inception of PCCM, members of the team have experienced long hours of stressful work and occasional feelings of despair and frustration that their efforts are not making a difference. This can lead to emotional distress and a sense of loss of fulfillment in their professional lives. Understanding of this problem by local medical and nursing leaders helps the team members realize that they are making an important difference through their efforts and dedication, thereby reducing burnout and enhancing staff morale.134–136

Around the World People around the world have made many contributions to the evolution of PCCM, through innovative treatment of specific diseases, creating PICUs (see Table 1.1) and advancing education (see Table 1.2).137 See ExpertConsult.com for a summary discussion of the global origins of pediatric critical care.

High-Mortality Countries In 2013, 6.3 million children worldwide died before age 5 years.181 Over 95% of these deaths occurred in high-mortality

History of Pediatric Critical Care Medicine

13

countries of Asia and sub-Saharan Africa and 52% were caused by infections—especially pneumonia, diarrhea, and malaria— that could be prevented or treated at low cost.182 Asia, for example, is the world’s hot spot for the emergence and reemergence of infectious diseases that threaten the world’s population (i.e., severe acute respiratory syndrome, COVID-19, and influenza) but still has huge burdens of the traditional infectious diseases (malaria, tuberculosis, HIV, diarrhea, dengue, etc.). Asia also leads the world in the emerging and global export of drug resistance to many pathogens, despite undergoing a period of unprecedented economic growth. Approximately 96% of children in the world live and die in resource-poor countries. Replicating success demonstrated in many countries, as in India and elsewhere, will have an immense impact on national resources. For example, in India, because of the high birth rates (annual births of 25 million) and large pediatric population (35% of total or approximately 300 million), as well as the need for trained people and material resources to service them, the required number of NICU and PICU beds would be enormous. It would therefore seem prudent for all district hospitals (750 in the country) to be upgraded to provide level 2 services that will meet the needs of rural communities.183 Increased access to PCCM for those at all economic levels should improve survival and eventually decrease the birth rate once people are more confident that their children will live. Although the development of PICUs is essential for the overall improvement of child survival in developing countries, the high cost of intensive care limits patients’ access to PICU services.184,185 Basic and cost-effective care have proved to have a major impact on improving the survival of infants and children. For example, a study in New Guinea demonstrated that the systematic use of pulse oximetry and supplemental oxygen reduced mortality from pneumonia by 35%.186 However, the cost was $51 for each child, which is beyond the means for many low-income countries where a high proportion of child deaths occur.187 Because hospital care is often not available to children in high-mortality countries, several authors have emphasized the need for preventive measures and improved primary healthcare.188–193 It is therefore important to train healthcare personnel and families in early detection of infants and children at high risk for mortality from infections and sepsis, which lead to critical disorders such as respiratory failure. Prompt initiation of treatment can reduce the need for critical care services (Table 1.7).

TABLE Accessibility to Pediatric Critical Care Medicine in High-Mortality Countries 1.7

Purposes

References

Pediatric Fundamental Critical Care Support (PFCCS)

1. Prioritize needs 2. Appropriate tests 3. Identify and respond to changing vital signs 4. Need for transport

194, 195

Boston Children’s Hospital

OPEN Pediatrics

Free online educational platform

196, 197

American Academy of Pediatrics

Helping Babies Breathe (HBB)

Neonatal resuscitation

198, 199

American Hospital Association

Saving Children’s Lives and Pediatric Emergency Assessment Programs (PEARS)

Resuscitation training

200

World Health Organization

Integrated Management of Childhood Illness (IMCI)

Identify sick children early

201











Program



Organization Society of Critical Care Medicine

14

SECTION I



Pediatric Critical Care: The Discipline

Summary The evolution of PCCM has been long and complex. It owes a great deal to innovations in anatomy, physiology, ventilation and resuscitation, anesthesiology, neonatology, pediatric general surgery and pediatric cardiac surgery, pediatric cardiology, and nursing as well as to the many individuals who advanced these fields around the world. Any attempt to relate the history of PCCM is inherently incomplete. Several individuals have been recognized for their contributions to PCCM in general and are noted in Tables 1.8, 1.9, and 1.10.

TABLE Distinguished Career Awardees, Section on 1.9 Critical Care, American Academy of Pediatrics

Year

Name

1995

I. David Todres, MD

1996

John Downes, MD

1997

Peter Holbrook, MD

1998

George Gregory, MD

1999

George Lister, MD

2000

Russell C. Raphaely, MD

2001

Murray Pollack, MD

2002

Daniel Levin, MD

Country

2003

Ann Thompson, MD

Alan Conn, MD

Canada

2004

Bradley Fuhrman, MD

John Downes, MD

United States

2005

J. Michael Dean, MD

Hans Feychting, MD

Sweden

2006

David Nichols, MD

Maurico Gajer, MD

Uruguay

2007

Ashok Sarnaik, MD

Gilbert Huault, MD

France

2008

Patrick Kochanek, MD

Seigo Iwai, MD

Japan

2009

Jerry J. Zimmerman, MD

Max Klein, MD

South Africa

2010

M. Michele Moss, MD

John Stocks, MD

Australia

2011

Timothy Yeh, MD

2012

Niranjan Kissoon, MD

2013

Vinay Nadkarni, MD

2014

Barry Markovitz, MD

2015

Thomas Rice, MD

2016

Alice Ackerman, MD

2017

Richard Brilli, MD

2018

James Fortenberry, MD

2019

Fernando Stein, MD

TABLE International Pioneer Awards World Federation 1.8 of Pediatric Intensive Critical Care Societiesa

Name

a

Awarded Montreal, 2000.

Acknowledgments We thank the following individuals and organizations for gathering material for this chapter and especially for helping us get facts and dates correct: Andrew Argent, Aristides Baltonado, Geoffrey Barker, Zahar Barzilay, John Beca, Jeffrey Burns, Gabriel Cassalett, Ira Cheifetz, Edward E. Conway, Jr., Mark Coulthard, John Cox, Peter Cox, Robert Crone, Martha Curley, J. Michael Dean, Bettina von Dessauer, Denis Devictor, Alan Duncan, Gideon Eshel, Alan Fields, Ericka Fink, Bradley Fuhrman, Melissa Fussell, Jonathan Gillis, William Glover, Denise Goodman, Thomas Green, George Gregory, David Hatch, Mary Fran Hazinski, Peter Holbrook, Robin Horak, Hector James, Tamara Jenkins, Niranjan (Tex) Kissoon, Max Klein, Patrick Kochanek, Jacques LaCroix, Jos Latour, George Lister, George Little, Kathryn Maitland, Barry Markowitz, Neil Matthews, M. Michele Mariscalo, Peter Meaney, M. Michele Moss, David Nichols, Hisashi Nikaidoh, Zehava Noah, Folafoluwa Odetola, John Pearn, Bradley Peterson, Jefferson Piva, Arnold Platzker, Bala Ramachandran, Adrienne Randolph, Russell Raphaely, Thomas Rice, Mark Rogers, Francisco Ruza, Hiro Sakai, David Schell, Frank Shann, Janice Bloedel

Smith, Gregory Stidham, Sue Tellez, James Thomas, Neal Thomas, Ann Thompson, Ron Trubuhovich, Robert Tumburro, Edwin vander Voort, Amir Vardi, Dharmapuri Vidyasagar, Randall Wetzel, Gary Williams, Douglas Willson, Timothy Yeh, the American Academy of Pediatrics (AAP), the Critical Care Medicine Subboard of the American Board of Pediatrics (ABP), the American Council of Graduate Medical Education (ACGME), the American Hospital Association (AHA), the Children’s Hospital Association, the Society of Critical Care Medicine (SCCM), the World Federation of Pediatric Intensive and Critical Care Societies (WFPICCS), and the National Institutes of Health (NIH). In Memoriam: Dr. Nick Anas, 1950–2018.



CHAPTER 1

History of Pediatric Critical Care Medicine

15

TABLE Pediatric Award Recipients, Society of Critical Care Medicine 1.10

Year

Name

Year

Name

Asmund S. Laerdal Memorial Award Lecture

2012

Timothy S. Yeh, MD

1993

Bradley P. Fuhrman, MD

2013

M. Michele Moss, MD

2002

Patrick M. Kochanek, MD

2019

Mohan R. Mysone, MD

2008

Vinay M. Nadkarni, MD

2019

Bruce M. Greenwald, MD

2012

Richard A. Berg, MD

American College of Critical Care Medicine Distinguished Investigator Award

Shubin-Weil Master Clinician/Teacher: Excellence in Bedside Teaching Award

2002

Murray M. Pollack, MD Patrick M. Kochanek, MD

1990

John J. Downes, MD

2007

1993

Alan I. Fields, MD

2011

Arno L. Zaritsky, MD

Barry A. Shapiro Memorial Award for Excellence in Critical Care Management

2013

Robert D. Truog, MD

2014

Madelyn D. Kahana, MD

2015

David G. Nichols, MD

Grenvik Family Award for Ethics 1993

Robert D. Truog, MD

1999

I. David Todres, MD

2013

Wynne E. Morrison, MD

2015

Jeffrey P. Burns, MD

Distinguished Service Award

2010

M. Michele Mariscalco, MD

2014

Ann E. Thompson, MD

2015

Richard J. Brilli, MD

Lifetime Achievement Award 2010

John J. Downes, MD

2017

Patrick M. Kochanek, MD

Norma J. Shoemaker Award for Critical Care Nursing Excellence 2011

Lauren R. Sorce, RN

2002

Patrick M. Kochanek, MD

Dharmapuri Vidyasagar Award for Excellence in Pediatric Critical Care Medicine

2004

Ann E. Thompson, MD

2015

Patrick Kochanek, MD

2007

Margaret M. Parker, MD

2016

Timothy S. Yeh, MD

2009

Richard J. Brilli, MD

2017

Desmond J. Bohn, MD

2009

Alan I. Fields, MD

2018

Jeffrey P. Burns, MD

2012

Edward Conway Jr, MD

2019

Nick G. Anas, MD

Key References Downes JJ. Development of pediatric critical care medicine: how did we get here and why? In: Wheeler DS, Wong HR, Shanley TP, eds. Pediatric Critical Care Medicine: Basic Science and Clinical Evidence. London: Springer; 2007:3-30. Downes JJ. Historic origins and role of pediatric anesthesiology in child health care. Pediatr Clin North Am. 1994;41:1-14. Drinker P, McKhann CF, The use of a new apparatus for the prolonged administration of artificial respiration. I. A fatal case of poliomyelitis. JAMA. 1929;92:1658-1660. Haglund G, Werkmaster K, Ekstrom-Jodal B, McDougall DH. The pediatric emergency ward-principles and practice after 20 years. In: Stetson JB, Swyer PR, eds. Neonatal Intensive Care. St. Louis: WH Green; 1976:73-87. Harrison VC, Heese H de V, Klein M. The significance of grunting in hyaline membrane disease. Pediatrics. 1968;41:549-559. Lassen HCA. A preliminary report on the 1952 epidemic of poliomyelitis in Copenhagen: with special reference to the treatment of acute respiratory insufficiency. Lancet. 1953;1:37-41.

Levin D, Morriss F, Moore G. A Practical Guide to Pediatric Intensive Care. St. Louis: CV Mosby; 1979. Rogers MC, ed. Textbook of Pediatric Intensive Care. Philadelphia: Williams & Wilkins; 1988. Rogers MC. The history of pediatric intensive care around the world. In: Nichols DG, ed. Roger’s Textbook of Pediatric Intensive Care. Philadelphia: Lippincott Williams & Williams; 2008:3-17. Safar P, Escarraga LA, Elam JO. A comparison of the mouth-to-mouth and mouth-to-airway methods of artificial respiration with the chestpressure arm-lift methods. N Engl J Med. 1958;258:671-677. Safar P. On the history of modern resuscitation. Crit Care Med. 1996; 24(suppl):S3-S11. Smythe PM, Bull A. Treatment of tetanus neonatorum with intermittent positive-pressure respiration. Br Med J. 1959;2:107-113.

The full reference list for this chapter is available at ExpertConsult.com.

e1









































































1. Downes JJ. Development of pediatric critical care medicine: how did we get here and why? In: Wheeler DS, Wong HR, Shanley TP, eds. Pediatric Critical Care Medicine: Basic Science and Clinical Evidence. London: Springer; 2007:3-30. 2. Levin DL. History of pediatric critical care medicine. In: Fuhrman BP, Zimmerman JJ, eds. Pediatric Critical Care Medicine. 4th ed. Elsevier; 2011:3-19. 3. Harvey W. Exercitio Anatomica de Motu Cordis et Sanguinia Animalibus. The Classics of Medicine Library [Keynes G, Trans.]. London: Nonesuch Press; 1978. 4. Todres ID. History of pediatric critical care. In: Fuhrman BP, Zimmerman JJ, eds. Pediatric Critical Care. 3rd ed. St. Louis: CV Mosby; 2005:7-14. 5. Second Book of Kings 4: 29-37. The Jerusalem Bible. New York: Doubleday; 1968:396-397. 6. Rosengart MR. Critical care medicine: landmarks and legends. Surg Clin North Am. 2006;86:1305-1321. 7. Grenvik A, Eross B, Powers D. Historical survey of mechanical ventilation. Int Anesthesiol Clinc. 1980;18:1-10. 8. Sommerson SJ, Sicilia MR. Historical perspectives on the development and use of mechanical ventilation. J Am Assoc Nurse Anesth. 1992;60:83-94. 9. Vesalius A. De Humani Corporis Fabrica. Basel: Libri Septem, Switzerland; 1543:659. 10. Drinker PA, McKhann CF III. The iron lung. JAMA. 1986;255:14761480. 11. Safar P, Escarraga LA, Elam JO. A comparison of the mouth-to-mouth and mouth-to-airway methods of artificial respiration with the chestpressure arm-lift methods. N Engl J Med. 1958;258:671-677. 12. Kouwenhoven WB, Jude JR. Closed-chest cardiac massage. JAMA. 1960;174:1064-1067. 13. Safar P. On the history of modern resuscitation. Crit Care Med. 1996;24(suppl):S3-S11. 14. Costarino AT, Downes JJ. Pediatric anesthesia historical perspective. Anesthesiol Clin North Am. 2005;23:573-595. 15. Snider GL. Historical perspective on mechanical ventilation: from simple life support systems to ethical dilemma. Am Rev Respir Dis. 1989;140:S2-S7. 16. Downes JJ. Historic origins and role of pediatric anesthesiology in child health care. Pediatr Clin North Am. 1994;41:1-14. 17. Downes JJ. The historical evolution, current status, and prospective development of pediatric critical care. Crit Care Clin. 1992;8:1-22. 18. Haglund G, Werkmaster K, Ekstrom-Jodal B, McDougall DH. The pediatric emergency ward-principles and practice after 20 years. In: Stetson JB, Swyer PR, eds. Neonatal Intensive Care. St. Louis: WH Green; 1976:73-87. 19. Epstein S, Brill JJ. A history of pediatric critical care medicine. Pediatr Res. 2005;58:987-996. 20. Kampshulte S, Safar P. Development of a multidisciplinary pediatric intensive care unit. Crit Care Med. 1973;1:308-315. 21. Berlin C. The pediatric intensive care unit. Med Ann Dist Columbia. 1970;39:483-486. 22. Holbrook P. Address to the section on critical care medicine of the AAP, 25th Anniversary 1984-2009, Washington, DC. October 18, 2009. 23. Rogers MC. The history of pediatric intensive care around the world. In: Nichols DG, ed. Roger’s Textbook of Pediatric Intensive Care. Philadelphia: Lippincott Williams & Williams; 2008:3-17. 24. Philip AGS. The Evolution of Neonatology. Pediatr Res. 2005;58: 799-815, 25. Noonan JA. A history of pediatric specialties: the development of pediatric cardiology. Pediatr Res. 2004;56:298-306. 26. Smythe PM, Bull A. Treatment of tetanus neonatorum with intermittent positive-pressure respiration. Br Med J. 1959;2:107-113. 27. Delivonia-Papadopoulos M, Swyer PR. Assisted ventilation in terminal hyaline membrane disease. Arch Dis Child. 1964;39:481-484.

28. Gregory G, Kitterman J, Phibbs R, et al. Treatment of the idiopathic respiratory distress syndrome with continuous positive airway pressure. N Engl J Med. 1971;284:1333-1340. 29. Brandstater B. Prolonged Intubation An Alternative to Tracheostomy in Infant Procedures. Vienna: First European Congress of Anesthesiology; 1962:106. 30. Drinker P, McKhann CF, The use of a new apparatus for the prolonged administration of artificial respiration. I. A fatal case of poliomyelitis. JAMA. 1929;92:1658-1660 and 1986;225: 1473-1475. 31. Ibsen B. Treatment of respiratory complications in poliomyelitis: the anesthetist’s viewpoint. Dan Med J. 1954;1:9-12. 32. Lassen HCA. A preliminary report on the 1952 epidemic of poliomyelitis in Copenhagen: with special reference to the treatment of acute respiratory insufficiency. Lancet. 1953;1:37-41. 33. Ibsen B. The anesthetist’s viewpoint on the treatment of respiratory complication in poliomyelitis during the epidemic in Copenhagen, 1952. Proc R Soc Med. 1954;42:72-74. 34. History of the Multidisciplinary ICU. Johns Hopkins Bayview Medical Center. www.hopkinsbayview.org/icu50th/history/html. 35. Chernick, V, Tooley WH, Cox JM, Warren RH. Guidelines for Organization of Children’s Intensive Care Units. American Academy of Pediatrics; July 1, 1975. 36. Committee on Hospital Care and the Pediatric Section of Critical Care Medicine the Society of Critical Care Medicine: guidelines for pediatric intensive care units. Crit Care Med. 1983;11:753-860. 37. Committee on Hospital Care and the Pediatric Section of the Society of Critical Care Medicine: guidelines and levels of care of pediatric intensive care units. Pediatrics. 1993;92:166-175. 38. Rosenberg DI, Moss MM. The APP Section on Critical Care and Committee on Hospital Care. Guidelines and levels of care for pediatric intensive care units. Pediatrics. 2004;114:1114-1124. 39. AAP Publications reaffirmed or retired. Pediatrics. 2013;131(5): e1707. 40. Randolph AG, Gonzales CA, Cortellini L, Yeh TS. Growth of pediatric intensive care units in the United States from 1995-2001. J Pediatr. 2004;144:792-798. 40a. Horak RV, Griffin JF, Brown A, et al. Pediatric Acute Lung Injury and Sepsis Investigators (PALISI) Network. Growth and changing characteristics of pediatric intensive care 2001-2016. Crit Care Med. 2019;47(8):1135-1142. 41. Porter R. The Greatest Benefit to Mankind. New York: WW Norton; 1997:178-461. 42. Nightingale F. Notes on Hospital. 3rd ed. London: longmanm Green, longman, Roberts and Green; 1863:86. 43. Mary Fran Hazinski, personal communication. 44. Martha Q. Curley, personal communication. 45. Bachman L, Downes JJ, Richards CC, et al. Organization and function of an intensive care unit in a children’s hospital. Anesth Analg. 1967;45:570-574. 46. Thomas Green, personal communication. 47. Zehara Noah, personal communication. 48. John Cox, personal communication. 49. Hiashi Nikaidoh, personal communication. 50. Harrison VC, Heese H de V, Klein M. The significance of grunting in hyaline membrane disease. Pediatrics. 1968;41:549-559. 51. Max Klein, personal communication. 52. Bradley Peterson, personal communication. 53. Bradley Fuhrman, personal communication. 54. George Lister, personal communication. 55. Mark Rogers, personal communication. 56. Jones R, Owens-Thomas J, eds. Care of the Critically Ill Child. London: Edward Arnold; 1971. 57. Roberts K, Edward JM. Paediatrics Intensive Care: A Manual for Resident Medical Officers and Senior Nurses. Oxford: Blackwell; 1971. 58. Dickerman JD, Lucey JF. Smith’s The Critically Ill Child: Diagnosis and Medical Management. Philadelphia: WB Saunders; 1972.



References

e2

























































































90. Society of Critical Care Medicine. Comprehensive Critical Care: Pediatric (9780936145891) Medicine 2012 Pub Society of Critical Care Medicine. 91. Lucking SE, Maffei FA, Tamburro RF, Thomas NJ, eds. Pediatric Critical Care Study Guide. Google Books, 2012. 92. Goday PS, Mehta NM, eds. Pediatric Critical Care Nutrition. New York: McGraw Hill Medical; 2015. 93. Watson S, Thompson A, eds. Pediatric Intensive Care. Oxford University Press. Oxford, England; 2017. 94. Calvin JE, Habet K, Pannillo JE. Critical care in the United States: who are we and how did we get here? Crit Care Clin. 1997;13:363-376. 95. Lister G. Pediatric critical care medicine. In: Pearson HA, ed. The American Board of Pediatrics 1933-2008. Chapel Hill, NC: American Board of Pediatrics; 2008:168-172. 96. Pediatric critical care medicine. Clin Privil White Paper Ped Crit Care Med. 2007;1-16. 97. Althouse LA, Stockman JA III. Pediatric workforce: a look at pediatric critical care medicine data from the Academic Board of Pediatrics. J Pediatr. 2006;149:390-392. 98. Ross Planning Associates. Pediatric Intensive Care Questionnaire 1979. Timothy Yeh, personal communication. 99. American Council of Graduate Medical Education, Graduate Medical Education /data Resource Book. http://acgmeweb/tabid/259/ graduatemedicaleducation/graduatemedicaleducationdataresourcebook. aspx. 100. American Hospital Association, personal communication, Rachel Wedeward. 101. Children’s Hospital Association, personal communication, Amy Keegan. 102. Patrick Kochanek, personal communication. 103. Gregory Stidham, personal communication. 104. Randolph AG, Wypii D, Venkataraman ST, et al. Effect of mechanical ventilator weaning protocols on respiratory outcomes in infants and children: a randomized controlled trial. JAMA. 2002; 288:2561-2568. 105. LaCroix J, Hebert PC, Hutchinson JS, et al. Transfusion strategies for patients in pediatric intensive care units. N Engl J Med. 2007; 356:1609-1619. 106. Willson DF, Thomas NJ, Markowitz BP, et al. Effect of exogenous surfactant (calfactant) in pediatric acute lung injury: a randomized controlled trial. JAMA. 2005;293:470-476. 107. Fineman LD, LaBrecque ME, Shih MC, Curley, MA. Prone positioning can be safely performed in critically ill infants and children. Pediatr Crit Care Med. 2006;7:413-422. 108. Marshall JC, Cook DJ, Canadian Critical Care Trials Group. Investigator-Led Clinical Research Consortia: The Canadian Critical Care Trials Group. Crit Care Med. 2009;37(suppl1):S165-S172. 109. Adrienne Randolph, personal communication. 110. Ira Cheifetz, personal communication. 111. Neal Thomas, personal communication. 112. Randall Wetzell, personal communication. 113. Thomas Rice, personal communication. 114. J. Michael Dean, personal communication. 115. Douglas Willson, personal communication. 116. Robert Tamburro, personal communication. 117. Tamara Jenkins, personal communication. 118. Dupont HL, Spink WW. Infection due to gram-negative organism: an analysis of 860 patients with bacteremia at the University of Minnesota Medical Center 1958-1966. Medicine (Baltimore). 1969;48:307-332. 119. Pollack MM, Cuerdon TT, Patel KM, et al. Impact of quality-ofcare factors on pediatric intensive care unit mortality. JAMA. 1994;272:941-946. 120. Yeh TS. Regionalization of pediatric critical care. Crit Care Clin. 1992;8:23-35. 121. Orr RA, Felmet KA, Han Y, et al. Pediatric specialized transport teams are associated with improved outcomes. Pediatrics. 2009; f124:40-48.



























































59. Huault G, Labrune B. Pediatric d’urgence. Paris: Ilammarion MédicineSciences; 1977. 60. Levin D, Morriss F, Moore G. A Practical Guide to Pediatric Intensive Care. St. Louis: CV Mosby; 1979. 61. Levin D, Morriss F, eds. A Practical Guide to Pediatric Intensive Care. 2nd ed. St. Louis: CV Mosby; 1984. 62. Levin D, Morriss F, eds. Essentials of Pediatric Intensive Care. St. Louis: Quality Medical Publications; 1990. 63. Levin D, Morriss F, eds. Essentials of Pediatric Intensive Care. 2nd ed. St. Louis, New York: Quality Medical Publications, Churchill Livingston; 1997. 64. Ruza FJ. Tratado de Ciududos Intensivos Pediatricas (Textbook of Pediatric Intensive Care). Madrid: Editorial Norma-Capitel; 1980. 65. Slota MC. Core Curriculum for Pediatric Critical Care Nursing. Philadelphia: WB Saunders; 1980. 66. Smith B. Pediatric Critical Care. New York; John Wiley & Sons; 1983. 67. Hazinski MF, ed. Nursing Care of the Critically Ill Child. St. Louis: CV Mosby; 1984. 68. Shoemaker WC, Thompson WL, Holbrook PR, eds. Textbook of Critical Care. Philadelphia: WB Saunders; 1984. 69. Nussbaum E, ed. Pediatric Intensive Care. Mount Kisco, NY: Futura; 1984. 70. Piva JP, Celiny-Garcia P, eds. Temas em Terapia Intensiva Pediatria. (Critical Care Issues in Pediatrics). Brazil: AMRIGS; 1985. 71. Zimmerman SS, Gildea JH, eds. Critical Care Pediatrics. Philadelphia: WB Saunders; 1985. 72. Morray JP, ed. Pediatric Intensive Care. Norwalk, CN: Appleton & Lange; 1987. 73. Rogers MC, ed. Textbook of Pediatric Intensive Care. Philadelphia: Williams & Wilkins; 1988. 74. Fuhrman BP, Zimmerman JJ, eds. Pediatric Critical Care. St. Louis: Mosby Year Book; 1992. 75. Holbrook PR, ed. Textbook of Pediatric Critical Care. Philadelphia: WB Saunders; 1993. 76. Lacroix J, Gauthier M, Hubert P, et al., eds. Urgences et Sions Intensifs Pediatriques. (Pediatric Emergency and Critical Care). Montreal & Paris: Les Presses de Universite de Montreal & Dion; 1994. 77. Nichols DG, Cameron DE, Greeley WJ, et al., eds. Critical Heart Disease in Infants and Children. St. Louis: Mosby; 1995. 78. Todres ID, Fugate JH, eds. Critical Care of Infants and Children. Boston: Little, Brown; 1996. 79. Curley MAQ, Bloedel-Smith J, Moloney-Harmon PA, eds. Critical Care Nursing of Infants and Children. Philadelphia: WB Saunders; 1996. 80. Dieckmann RA, Fiser DH, Selbst SM, eds. Illustrated Textbook of Pediatric Emergency and Critical Care Procedures. St. Louis: Mosby; 1997. 81. Morton NS, ed. Pediatric Intensive Care. Oxford University Press; 1997. 82. Lopez-Herce Cid J, Lorente Acosta MJ, Baltodano Aguero A. Manual de Cuidados Intensivos Pediatricos. Madrid: Publimed; 2001. 83. Stack CG, Dobbs P, eds. Essentials of Pediatric Intensive Care. London: Greenwich Medical Media Limited; 2003. 84. Piva J, Garcia P, eds. Medcina Intensiva em Pediatria. Rio de Janeiro; Editoria Revinter 2005. 85. Forero G, Alarcon J, Cassalett B, eds. Cuidado Intensivo Pediatrico Y Neonatal. Bogota, Columbia: Editorial Distribuna; 2005. 86. Slonim AD, Pollack MM, eds. Pediatric Critical Care Medicine. Philadelphia: Lippincott, Williams & Wilkins; 2006. 87. Cassalett G, Patarryo MC. Manual de Cuidado Intensivo Cardiovascular Pediatrico. (Cardiovascular Pediatric Intensive Care Handbook). Bogota, Columbia: Distribuna; 2006. 88. Wheeler DS, Wong HR, Shanley TP, eds. Pediatric Critical Care Medicine: Basic Science and Clinical Evidence. London: Springer; 2007:64. 89. Munoz R, Monell V, Da Cruz E, Vetterly CG, eds. Critical Care of Children with Heart Disease: Basic Medical and Surgical Concepts. Springer, New York; 2010.

e3









































































151. Yule P. The Second Floor. Anesthesia, Intensive Care, Neonatology and the Operating Suite, in the History of the Royal Children’s Hospital: A History of Faith, Science and Love. New South Wales: Halstead Press Pub; 1999. 152. Webb D, Warren D. Intensive Care Unit, “A New Deal for Little Adults” in “Safe in Our Hands.” 100 Years of Princess Margaret Hospital for Children Foundation, Inc. Pub Western Australia; 2009. 153. Alan Duncan, personal communication. 154. Neil Matthews, personal communication. 155. Mark Coulthard, personal communication. 156. David Schell, personal communication. 157. Gary Williams, personal communication. 158. Brandstate B. Prolonged intubation an alternative to tracheostomy in infant procedures. First European Congress of Anesthesiology Vienna. 1962:106. 159. Allen T, Steven I. Prolonged endotracheal intubation in infants and children. Br Med J. 1965;37:566. 160. McDonald IH, Stock JG. Prolonged nasotracheal intubation. A review of it’s development in a paeditric hospital. Br J Anesth. 1965;37:161-173. 161. Duncan AW, Mullins GG, Kent M, Phelan PD. A paediatric emergency transport service: one year’s experience. Med J Aust. 1981; 26:673-676. 162. John Beca, personal communication. 163. Ron Trubuhovich, personal communication. 164. Trubuhovich RV, Judson JA. Intensive Care in New Zealand. A History of the New Zealand Region of ANZICS with Notes on the Development of Intensive Care in New Zealand. Auckland, NZ: Trubuhovich RV and Judson JA publishers; 2001:150. 165. Trubuhovich RV. Some prehistory of New Zealand intensive care medicine. Anaesth Intensive Care. 2009;37(suppl1):16-29. 166. Trubuhovich RV. Pioneering paediatric intensive care medicine in New Zealand. Anaesth Intensive Care. 2013;41:655-670. 167. Denis Devictor, personal communication. 168. David Hatch, personal communication. 169. Sinclair J. The Spectrum of Paediatric Intensive Care, in Pediatric Intensive Care. Oxford: Oxford University Press; 1997. 170. Francisco Ruza, personal communication. 171. Edwin van der Voort, personal communication. 172. Prien T, Meyer J, Lawin P. Development of intensive care medicine in Germany. J Clin Anesth. 1991;3:253-258. 173. Benedekova M. Children in the focus of the attention of pediatricians and pediatric surgeons. Bratisl Lek Listy. 2003;104:255-258. 174. Amir Vardi, personal communication. 175. Gideon Eshel, personal communication. 176. Jefferson Piva, personal communication. 177. Gabriel Cassalett, personal communication. 178. Campos-Nino S, Sasbon JS, von Dessauer B. Los cuidados intensivos pediatricos en Latin America. Med Intensiva. 2012;36:3-10. 179. Bettina von Dessauer, personal communication. 180. Aristides Baltodano, personal communication. 181. Lui L, Oza S, Hogan D, et al. Global, regional, and national causes of child mortality in 2000-13, with projections to inform post2015 priorities: an updated systematic analysis. Lancet. 2015;385: 430-440. 182. Kathryn Maitland, personal communication. 183. Khilnani P, Sarma D, Singh R, et al. Demographic profile and outcome analysis of a tertiary level pediatric intensive care unit. Indian J Pediatr. 2004;71:587-591. 184. Argent AC, Ahrens J, Marrow BM, et al. Pediatric intensive care in South Africa: an account of making optimum use of limited resources at the Red Cross War Memorial Children’s Hospital. Pediatr Crit Care. 2014;15:7-14. 185. Kissoon N, Burns J. Who should get pediatric intensive care when not all can? A call for international guidelines on allocation of pediatric intensive care resources. Pediatr Crit Care Med. 2014;15: 82-83.































































122. McPherson ML, Graf JM. Speed isn’t everything in pediatric medical transport. Pediatrics. 2009;124:381-383. 123. Ojha R, Liu A, Rai D, Nanan R. Review of Simulation in Pediatrics: The Evolution of a Revolution. Front Pediatr. 2015 November; 3:106. 124. Cheng A, Duff J, Grant E, Kissoon N, Grant V. Simulation in paediatrics: an educational revolution. Paediatr Child Health. 2007; 12(6):465-468. 125. Lopreiato J, Sawyer T. Simulation-based medical education in pediatrics. Acad Pediatr. 2015;15(2):134-142. 126. Cheng A, Lang T, Starr S, Pusic M, Cook D. Technologyenhanced simulation and pediatric education: a meta-analysis. Pediatrics. 2014;133;e1313. 127. Hunt E, Duval-Arnould J, Nelson-Macmillan K, et al. Pediatric resident resuscitation skills improve after “rapid cycle deliberate practice” training. Resuscitation. 2014;85:945-951. 128. Hunt E, Heine M, Hohenhaus S, et al. Simulated pediatric trauma team management: assessment of an educational Intervention. Pediatr Emerg Care. 2007:23;796-804. 129. Andreatta P, Saxton E, Thompson M, Annich G. Simulation-based mock codes significantly correlate with improved pediatric patient cardiopulmonary arrest survival rates. Pediatr Crit Care Med. 2011; 12(1):33-36. 130. Chalom R, Raphaely RC, Costarino AT. Hospital costs of pediatric intensive care. Crit Care Med. 1999;27:2079-2085. 131. Downes JJ, Parra MM. Costs and reimbursement issues in long term ventilation. In: Hill NS, ed. Long Term Mechanical Ventilation. New York: Marcel Dekker; 2001:353-374. 132. Martinez M. Psychosocial aspects of pediatric intensive care: the parent. In: Levin DL, Morriss FC, eds. Essentials of Pediatric Intensive Care Medicine. 2nd ed. St. Louis: Quality Medical Publisher and New York, Churchill Livingstone; 1997:1035-1038. 133. Jarvis JD, Levin D. Joint Decision Making Rounds in the PICU Results in Positive Outcomes and Improved Satisfaction. Oral Abstract. Washington, DC: America Academy of Pediatrics NCE, Section on Critical Care; 2005. 134. Rehder KJ, Cheifetz IM, Markowitz BP, Tuner DA, PALISI: Survey of in-house, coverage by pediatric intensivists: characterization of twenty-four-seven in-hospital pediatric critical care faculty coverage. Pediatr Crit Care. 2014;15:97-104. 135. Fields, AI. Do you smell something burning? Could it be you? Pediatr Crit Care Med. 2014;15:788-789. 136. Garcia TT, Garcia PCR, Moloy ME, et al. Prevalence of burnout in pediatric intensivists: an observational comparison with general pediatricians. Pediatr Crit Care Med. 2014;15:e347-e353. 137. Kochanek P. Pediatric critical care medicine, our expanding global mission. Pediatr Crit Care Med. 2001;2:106-107. 138. Geoffrey Barker, personal communication. 139. Jacques LaCroix, personal communication. 140. Andrew Argent, personal communication. 141. Hirokazu Sakai, personal communication. 142. Vidyasagar D, Singh M, Bhakoo ON, et al. Evolution of neonatal and pediatric critical care in India. Crit Care Clin. 1997;13:331-346. 143. Nangia S, Saili A, Dutta AK, et al. Neonatal ventilation-experience at a level II centre. Indian J Pediatr. 1998;65:291-296. 144. Govil YC. Pediatric intensive care in India: time for introspection and intensification. Indian Pediatr. 2006;43:675-678. 145. http://www.isccm.org/issm/Aboutpediatric.aspx 146. www.piccindia.org 147. Bala Ramachandran, personal communication. 148. Bhalala U, Khilnani P. Pediatric critical care medicine training in India. Past, present and future. Front Pediatr. 2018;6:34. 149. Prabhudesai S, Kasala M, Manwarei N, Krypanandan R, Ramachandran B. Transport-related adverse events in critically-ill children: the role of a dedicated transport team. Indian Pediatr. 2017; 54(11):942-45. 150. Shann F. Australian paediatric intensive care newsletter, AUSPIC NEWS, 2:Dec 1993.

e4























193. Shann F. Role of intensive care in countries with a high child mortality rate. Pediatr Crit Care Med. 2011;12:114-115. 194. Edward E. Conway, Jr., personal communication. 195. Society of Critical Care Medicine. Pediatric Fundamental Critical Care Support. Society of Critical Care Medicine, Mount Prospect, IL; 2008. 196. OPENPediatrics. www.openpediatrics.org 197. Jeffrey Burns, personal communication. 198. George Little, personal communication. 199. Little GA, Keenan WJ, Singhal N, Neirmeyer S. International perspectives: helping babies breathe: evolution of a global neonatal resuscitation program for resource limited areas. NeoReviews. 2014;15:e1-e12. 200. Peter Meaney, personal communication. 201. Gove S. for the WHO Working Group on Guidelines for Integrated Management of the Sick Child. Integrated management of childhood illness by outpatient health workers: technical basis and overview. Bull World Health Organ. 1997;75:7-24.





















186. Duke T, Mgone J, Frank D. Hypoxemia in children with severe pneumonia in Papua New Guinea. Int J Tuberc Lung Dis. 2001; 5:511-519. 187. Frank Shann, personal communication. 188. Maitland K, et al. Mortality after fluid bolus in African children with severe infection. N Engl J Med. 2011;364:2483-2495. 189. Molyneux E, Ahmad S, Robertson A. Improved triage and emergency care for children reduces inpatient mortality in a resource-constrained setting. Bull World Health Organ. 2006;84: 314-319. 190. Bhutta Z, Ali S, Cousens S, et al. Alma-Ata: rebirth and revision 6 interventions to address maternal, newborn, and child survival: what differences can integrated primary health care strategies make? Lancet. 2008;372:972-989. 191. Baker T. Pediatric emergency and critical care in low-income countries. Paediatr Anaesth. 2009;19:23-27. 192. Kissoon N. Out of Africa: a mother’s journey. Pediatr Crit Care Med. 2011;12:73-79.

e5

Abstract: Pediatric critical care owes a great debt to the expertise of the practitioners of anesthesiology, neonatology, pediatric cardiology, pediatric general surgery, pediatric cardiovascular surgery, and nursing for its evolution. The modern pediatric critical care medicine unit and service are more the result of the need to treat and organize care for critically ill and injured patients, such as

those with poliomyelitis, than the result of any developments in technology. Key Words: resuscitation, mechanical ventilation, neonatology, pediatric cardiology, anesthesia, pediatric general surgery, pediatric cardiac surgery, nursing

2 High-Reliability Pediatric Intensive Care Unit: Role of Intensivist and Team in Obtaining Optimal Outcomes CLAIRE A. STEWART, DEREK S. WHEELER, AND RICHARD J. BRILLI

PEARLS •











A modern pediatric intensive care unit (PICU) is a complex system, operating in conjunction with other complex systems (inpatient units, operating rooms, and emergency departments) within and between hospitals. Incorporating principles of high reliability is necessary to bring about system changes that will lead to continued improvements in PICU outcomes. As the multidisciplinary PICU team leader, pediatric intensivists should understand and collaborate in managing all system aspects, not just clinical care.

An often-repeated quotation states, “Every system is perfectly designed to achieve the results it gets.” Improved outcomes and higher quality of care in pediatric intensive care units (PICUs) derives from system change. The Institute of Medicine report Crossing the Quality Chasm raised significant concerns about healthcare quality in the United States and called for fundamental redesign of the US healthcare delivery system.1 Since that time, the healthcare industry has begun to adopt high-reliability practices of industries such as nuclear power, commercial aviation, and aircraft carrier flight deck operations. These industries operate within complex and inherently dangerous environments. Despite these challenging work environments, they have achieved near flawless outcomes by applying many of the high-reliability principles outlined by Weick et al., earning them the designation high-reliability organizations (HROs).2 We posit that, to dramatically improve outcomes, ICUs must become HROs (Table 2.1). Studying HROs yields five interrelated principles that characterize how individuals in these organizations behave and think and how the systems where they work adapt to these behaviors, decreasing the likelihood of error and enhancing an organization’s ability to quickly recover when an error occurs (Table 2.2).2,3 “Preoccupation with failure” means treating even minor errors as potential catastrophes and learning opportunities. “Sensitivity to operations” implies that leaders pay attention to operations at all 16













PICU structure, processes, and outcomes are key system components requiring management for success. Anticipated future changes in reimbursement methodology will likely influence the value proposition of current diagnostic and treatment models in an increasingly high-technology PICU world. Despite inherent challenges, severity-adjusted comparative data for PICU outcomes are available and useful for assessing outcomes within and between hospitals.

levels—especially at the front line, relying heavily on transparent, nonpunitive information dissemination to all parties. “Reluctance to simplify” recognizes the dangers of oversimplification in complex systems and looks for differing viewpoints when analyzing events. “Deference to expertise” means that in traditionally hierarchical organizations such as hospitals, understanding of operational details for any process usually resides within frontline staff. When possible, shifting operational responsibility to the front line (and, recently, even to parents and families) may optimize results. “Commitment to resilience” specifies that when unforeseen events occur, HROs adapt swiftly, communicate rapidly, and problem solve creatively. To date, high-reliability care in the US healthcare system remains elusive. Data from pediatric hospitals suggest that recommended care is delivered only 55% of the time.4 Multiple pediatric hospital systems report significant harm reduction by applying HRO principles as part of robust quality improvement (QI) programs.5–7 In addition, Berry et al. describe a link between PICU harm reduction and safety and teamwork culture as part of a larger comprehensive hospital initiative focused on high-reliability principles.8 Further, pediatric hospitals have banded into statewide and national collaboratives using HRO principles and QI science to lower rates of serious safety events and hospital acquired conditions (HACs).9,10



CHAPTER 2

High-Reliability Pediatric Intensive Care Unit: Role of Intensivist and Team in Obtaining Optimal Outcomes

17

TABLE System Characteristics as They Relate to Differing Levels of Reliability 2.1

LOW Reliability (Generally More Basic and Inconsistent) Individual preference prevails Intent to perform well Individual excellence rewarded Human vigilance for risk, error, harm Hard work, trying harder after failures Codified policies, procedures, guidelines Personal checklists Retrospective performance feedback Didactic training/retraining Awareness raising Basic standardization (equipment, brands, forms)



RELIABILITY



Personnel informed by reliability science Implementation of human factors Standardization of processes is norm Ambiguities in standard work eliminated Workaround solutions eliminated Reminders and decision support built in Standard checklists (real-time compliance) Good habits/behaviors leveraged Error proofing: warnings, sensory alerts Deliberate redundancy in critical steps Key tasks scheduled/assigned Some simulation training for emergencies Real-time performance feedback

High Reliability (Generally More Robust and Effective) Sophisticated organizational design Integrated hierarchies, processes, teams Error proofing: forced function, shutdown Failure modes and effects analysis Routine simulation for training/reinforcing Strong teamwork climate Strong safety culture Staff perception of psychological safety Preoccupation with failure Reluctance to simplify interpretations Sensitivity to operations Deference to expertise Commitment to resilience

Data from Weick KE, Sutcliffe KM. Managing the Unexpected: Resilient Performance in an Age of Uncertainty. San Francisco, CA: Jossey-Bass, 2007; Nolan TRR, Haraden C, Griffin FA. Improving the Reliability of Health Care. IHI Innovation Series white paper; 2004; Hines S, Luna K, Lofthus J, et al. Becoming a High Reliability Organization: Operational Advice for Hospital Leaders. AHRQ Publication No. 08-0022. https://psnet.ahrq.gov/issue/becoming-high-reliability-organization-operational-advice-hospital-leaders; and Niedner MF, Muething SE, Sutcliffe KM. The high-reliability pediatric intensive care unit. Pediatr Clin North Am. 2013;60:563–580.

There is marked complexity in systems, processes, technology, and work The environment is socially and/ or politically unforgiving The consequences for failure are potentially catastrophic The cost of failure precludes learning through trial and error or traditional experimentation

Preoccupation with failure Reluctance to simplify interpretations Sensitivity to operations Deference to expertise Commitment to resilience

From Niedner MF, Muething SE, Sutcliffe KM. The high-reliability pediatric intensive care unit. Pediatr Clin North Am. 2013;60:563–580.

PICUs are an essential, rapidly expanding component of hospital systems. Critically ill patients usually receive care in the PICU but, in some cases, they receive care by the PICU team outside the PICU. Niedner and colleagues describe the current state of US PICUs and their journey toward high reliability.11 They conclude that, while progress has been made and many PICUs are on this journey, currently no hospital or PICU truly fulfills all of the HRO criteria. Furthermore, leaders in any PICU system must pay attention to aspects of the system well beyond clinical care, which adds challenges that must be managed.12 This chapter’s purpose is to equip pediatric intensivists, as leaders of multidisciplinary PICU teams, with an enhanced understanding of the PICU as a system within the greater hospital system, including management and evaluation of PICU operations and outcomes. Integrated into this discussion are specific challenges to attaining high-reliability care delivery. It is

Pediatric Intensive Care Unit as a System Industrial engineering and operations research would describe hospitals and PICUs as emergent systems.13 Emergence is a phenomenon in which larger systems arise from combined interactions of smaller systems in such a way that the whole is greater than the sum of the individual components. The PICU is a system with inputs and outputs. PICU inputs include critically ill patients admitted from all units within the healthcare system. Outputs include patients transferred back to those same units, including home. Therefore overall quality of care in the PICU cannot be viewed in isolation. Indeed, the whole system (critical care) is greater than the sum of the individual components (e.g., emergency department, operating room, hospital inpatient unit, interhospital transport, and rehabilitation unit). Improvements at the whole-system level will primarily occur if it is viewed in its entirety rather than as a collection of multiple individual components. This concept has fostered a relatively new way of thinking about the modern ICU as an “ICU without walls” rather than as a distinct geographic unit. As McQuillan et al. stated, “the greatest impact on the outcome for intensive care units may come from improvements in the input to intensive care, particularly in the quality of acute care. . . .”14 The “PICU without walls” concept is perhaps best understood in the context of reduced morbidity and mortality associated with implementation of rapid response systems, watch-stander or “watcher” programs, and enhanced focus on situation awareness.15–19 Intensivists—and often pediatric critical care medicine (PCCM) fellows—are expected, with increasing frequency, to provide consultation for children outside the ICU, often in conjunction with hospital medicine colleagues. Additionally, as children recover from critical illness, long-term sequelae often persist. Thus intensivists are recognizing the  

Characteristics



Contexts

through implementation of administrative and quality knowledge, in combination with clinical expertise, that highly reliable PICUs could soon flourish.



TABLE High-Reliability Organizations: Contexts 2.2 and Adaptive Characteristics

18

SECTION I



Pediatirc Critical Care: The Discipline

importance of early and frequent involvement of rehabilitation specialists, both while the patient is still in the ICU and after transfer out of the PICU.20,21 Another example of the PICU without walls concept is telemedicine. The American College of Critical Care Medicine Task Force on Models of Critical Care specifically mentions provision of tele-ICU services as key elements for the ideal critical care delivery model.22 Berrens et al. describe use of telemedicine at a satellite facility with rapid response and code team activations staffed remotely by pediatric intensivists.23 They note a similar standard of response team activation and response at their satellite facility when compared with the main institution using telemedicine. However, they also note that an effective system must be in place at the main institution for full telemedicine effectiveness. Telemedicine is likely an effective and important tool for improving critical care delivery, especially when used in conjunction with many of the other high-reliability design principles discussed in this chapter.

Avedis Donabedian, an early systems thinker in healthcare, stated that healthcare quality should be based on three dimensions: structure, processes, and outcomes (Fig. 2.1).24 In this model, processes are effective (1) when the right structures are in place to support them and (2) when outcomes are measured, so that these processes can be evaluated for effectiveness and modified to produce better results. Structure refers to the setting in which care is delivered. Structure elements include patients, providers, technology, and therapy.13 Providers include the entire PICU team. When organized according to HRO principles, the team is optimally led jointly by a physician and nurse and is inclusive of all disciplines that touch the patient (Fig. 2.2). A recent trend is greater subspecialization of PICUs based on specific subpopulations of critically ill children (e.g., neurointensive care, cardiac intensive care). These subspecialty PICUs are discussed elsewhere in this textbook; however, the structure and processes to achieve optimal outcomes apply regardless of subspecialization of a particular ICU. Process refers specifically to how care is provided, including incorporation of high-reliability principles into daily activities. Outcomes refer to end points of care, including commonly used quality and safety measures, and other key outcomes, such as length of stay (LOS), patient/family experience, and cost/value.

Patients Providers Technology Therapy

• Fig. 2.1

Process

Outcome

Bundles

Patient/family experience

Checklists Situation awareness Communication Teamwork

Morbidity Cost Length of stay Mortality

​Donabedian’s model for quality. (Modified from Stewart C, Brilli RJ. Does a medical emergency team activation define a new paradigm of mortality risk? Pediatr Crit Care Med. 2017;18:601–602.)  





ICU pharmacist

Nursing leader

Respiratory therapist

Social worker

Pastoral care

Physical therapist

ICU nurses

Dietitian

Trainees

• Fig. 2.2

​Typical pediatric intensive care unit (PICU) team structure incorporating high-reliability organization principles. A physician and nurse leading the team together signals both deference to expertise and reluctance to simplify. Inclusion of all other disciplines serving the PICU population at the same level incorporates the concept of sensitivity to operations.  



Structure

Models of Critical Care Delivery

Structure

Intensivist

The critical care team is perhaps the most important structural component driving care quality and high-reliability processes in the PICU. The exact number of physicians practicing pediatric critical care medicine is unknown. A 2017 survey by the American Board of Pediatrics reported 2603 board-certified pediatric intensivists in the United States (up from 1881 in 2011) but did not account for physicians no longer practicing critical care or others (cardiologists, anesthesiologists) staffing PICUs.25 Pediatric intensivists are aging—the average age is 50 years. Many PICUs now use a 24/7 in-house attending coverage model. One recent PICU survey revealed that 53% of responding hospitals had full in-house attending coverage, 21% had mixed coverage (some nights in-house, some nights home coverage), and only 26% had exclusive home coverage.26 Perhaps in response to the aging intensivist population and the increased staffing needs associated with increased in-house attending coverage, PCCM training programs are growing rapidly. The number of PCCM fellows increased from 271 in 2001 to 548 in 2017.25 The field of PCCM is also becoming increasingly female, with the number of first-year female PCCM fellows increasing from 41 in 2001 to 120 in 2017.25,27 Staffing models that incorporate more advanced practice nurses, physician assistants, and hospitalists—as well as telemedicine and care regionalization—have been proposed as solutions to the potential critical care physician shortage.28–33 Critical care services regionalization can be both a problem and a solution. Halpern et al. report that 91% of hospitals with intensivists are located in metropolitan areas and 93% of all ICU beds are located in metropolitan locations,34 raising concern for a shortage of intensivists and ICU beds in rural areas and potentially making care more difficult to access for patients living away from metropolitan areas. PICU physician workforce challenges are clearly multifactorial, including career path opportunities, work environment, and job satisfaction. PICU physicians report the highest number of work hours per week as compared with all other pediatric medical subspecialties.35,36 They also report the smallest percentage of a part-time workforce as compared with other pediatric subspecialties.27 Given the work environment (long work hours and high stress), the prevalence of burnout among pediatric critical care physicians, as reported in a Critical Care Societies Statement, was 71%.37 This rate is higher than critical care physicians as a whole and more than twice the rate of general pediatricians.



CHAPTER 2

High-Reliability Pediatric Intensive Care Unit: Role of Intensivist and Team in Obtaining Optimal Outcomes

Methods to mitigate burnout are multifactorial and involve both individual and institutional/organizational buy-in.38,39 As Perlo et al. discuss in their Institute for Healthcare Improvement white paper entitled “Joy in Work,” a focus on factors that bring enjoyment may be more effective than a focus on what causes burnout40 (see also Chapter 22). Importantly, 94% of pediatric critical care physicians report that they would encourage a resident to pursue a career in PCCM and 87% report that they would choose a career in PCCM again.26 Finally, burnout scores were not different between those who provide in-house coverage versus those that take home calls.26,41 When surveyed, 36% of intensivists planned to reduce their workload in the next 5 years, with nearly 10% planning to retire. These physicians cite long work hours and stress as contributing factors to their decisions.36 In sum, the impact of burnout and work environment on PCCM physician staffing remains unclear but certainly warrants attention intended to minimize factors that contribute to the stress of working in an ICU. Another key element that pertains to PICU staffing is ICU strain. ICU strain is “a discrepancy between the availability of ICU resources and demand to admit and provide high-quality care for patients with critical illness.”42 Rewa et al. identified several indicators of ICU strain, including ICU census (capacity to admit new patients), ICU queuing (delay in time to physical admission from decision to admit), ICU readmission, early ICU discharge, after-hours discharges, and surgery cancellations.42 All of these factors have potential impacts on patient safety and outcomes. Opgenorth et al. surveyed providers and found that ICU strain increased stress levels and contributed to burnout among providers, reduced perceived quality of patient care, and contributed to high staff turnover. Strategies to mitigate ICU strain include increasing ICU bed capacity; increasing nonacute care beds, which can improve ICU throughput; improving care transitions; better ICU staff training; and increased nurseto-patient ratios.43 Optimal physician coverage and direct presence in the PICU remains unclear. As discussed previously, 24/7 in-house attending coverage is increasingly common among PICUs. A 2013 survey indicated that 86% of pediatric intensivists believed in-house PICU attending coverage improved patient care and more than 80% thought patient care was safer and timelier with in-house attending coverage.26 Several published studies comparing ICUs with and without 24/7 attending critical care physician coverage suggest mixed results. Demonstrating a mortality difference between 24/7 in-house attending coverage and home coverage is difficult, especially in the context of a low overall mortality rate in PICUs. However, Gupta et al. demonstrated a significantly lower mortality rate, lower cardiac arrest incidence, and decreased mortality after cardiac arrest with a 24/7 staffing model. They also demonstrated shorter PICU LOS and decreased mechanical ventilation duration.44 Another study by Gupta, examining children after cardiac surgery, revealed a lower incidence of cardiac arrest, decreased use of extracorporeal membrane oxygenation, earlier tracheal extubation, and shorter central venous line duration.45 Nishisaki and colleagues45a reported that 24/7 in-hospital intensivist coverage decreased duration of mechanical ventilation and PICU LOS but did not affect risk-adjusted mortality. In sum, while the optimal PICU attending staffing model remains unclear, recent data suggest improved outcomes with a 24/7 inhouse attending staffing model. Optimal ICU physician staffing ratios are also unclear. A study conducted in adult ICUs demonstrated a relationship between staffing ratios and mortality.46 Other studies demonstrated a

19

higher nursing workload, which can negatively impact nurse retention, burnout, and patient safety.37,47 The Society of Critical Care Medicine published guidelines regarding how to address this important issue but did not recommend specific physician-patient ratios.48 The society did note: “In academic medical ICUs, there is evidence that intensivist/patient ratios more than 1:14 negatively impacted perceptions of teaching quality, stress, patient care, and workforce stability.” Structure also encompasses ICU physical design, monitoring and support equipment, and information systems preferably equipped with decision support.49,50 Optimal ICU design may improve patient safety. As Leaf et al. demonstrated, those patients who were in low visibility rooms (i.e., not visible from the nursing station) were at as high a risk of mortality as equally sick patients in high-visibility rooms.51 Halpern outlines key principles that should guide ICU patient room design, such as single-occupancy rooms and rooms that are similarly designed so that staff can move seamlessly between rooms.52 Devices in rooms (monitors and ventilators), should be informatics based and have the capability to continuously send and store data to a central location.53 The PICU is a technical environment in which the interface between technology and humans can both improve care and increase error risk. For example, computerized physician order entry (CPOE) can significantly improve quality of care provided in the ICU setting by decreasing LOS and reducing medication error.54 However, even with electronic health records (EHRs) and CPOE, risk for error remains (e.g., ordering medication on the wrong patient, relying on the computer for dosage) and some inefficiencies persist (e.g., increased time associated with computer documentation).55 There is growing interest in using predictive analytics, based on objective data pulled automatically from the medical record, to predict patient outcomes.56–58

Process Process refers specifically to how care is delivered in the ICU. The aforementioned structural elements interact with key processes to drive improved outcomes. It is important to emphasize that structural elements cannot compensate for the lack of an appropriate institutional climate that supports standardized, reliable, evidence-based processes and uses high-reliability principles.5,13,22 Adequate structure, accompanied by rigorous use of highreliability principles, is essential to drive improved outcomes. For example, optimal nurse and physician staffing structure will not consistently prevent an adverse drug event from causing harm if rigorous medication double-check techniques and the five rights of medication administration are not consistently (reliably) performed with every medication administration. There must also be a robust quality culture—with focus on safety, teamwork, and engagement—emanating from the top and permeating throughout the organization. The role of senior hospital leadership and the board of directors, who should emphasize to everyone the importance of quality and safety, is critical. As Randall et al. state, “increased CEO involvement and accountability are key opportunities to move closer to high reliability.”59 Multiple studies demonstrate significant patient harm reduction by implementing standardized reliable processes, including—but not limited to—the use of care bundles, checklists, and clinical pathways (Table 2.3). A care bundle is a relatively short list of standardized, generally evidence-based or bestpractice interventions for a patient population or disease that, when implemented consistently, lead to improved outcomes. It is

20

SECTION I



Pediatirc Critical Care: The Discipline

TABLE Key PICU Processes 2.3

Rounding Process

Transitions of Care

Medication reconciliation Daily multidisciplinary rounds (including medical and surgical subspecialists) Daily goals sheet

Operating room to ICU hand-offs

Daily shift huddles

Other hand-offs

Primary nursing assignments

Discharge planning

PICU leadership walk rounds

Bundles

Protocols

Central line insertion bundle

Clinical pathways Standard care protocols (e.g., extubation readiness testing, sedation/analgesia protocols)

VARI bundle

Condition-specific protocols (e.g., status asthmaticus, sepsis, diabetic ketoacidosis, traumatic brain injury)

Urinary catheter care bundle Pressure ulcer prevention bundle Peripheral IV care bundle IV, Intravenous; PICU, pediatric intensive care unit; VARI, ventilator-associated respiratory infection. Modified from Riley C, Poss WB, Wheeler DS. The evolving model of pediatric critical care delivery in North America. Pediatr Clin North Am. 2013;60:545–562.

the combination of elements performed consistently and in aggregate that drive improvement. The ABCDEF ICU liberation bundle is a recent example of large-scale bundle use in adult ICUs that, when used reliably, is linked to improved survival, reduced delirium, and decreased ICU readmissions.60 Large collaborations between children’s hospitals, such as the Children’s Hospitals’ Solutions for Patient Safety, have contributed to the development of pediatric specific best practices and HAC prevention clinical care bundles, including those aimed at surgical site infection, catheter-associated urinary tract infection, and pressure injury reductions. The use of these bundles leads to HAC rate reduction.61–64 Finally, sepsis recognition and treatment bundles are increasingly common. Work published by Evans et al. assessing the use of the New York sepsis guidelines demonstrated improved outcomes when a sepsis bundle (antibiotics, cultures, and fluid) was completed within 1 hour.65 Balamuth et al. demonstrated improved outcomes with use of a sepsis identification bundle in the emergency department in combination with the electronic medical record to generate automatic alerts to identify patients at risk of sepsis.66,67 Checklists have also been used to improve care of critically ill children admitted to the PICU. The NEAR4KIDS group developed an Airway Bundle Checklist to identify tracheal intubation risk factors, improve team situation awareness, improve the use of time-outs, and mitigation plan generation.68 Clinical pathways are flowcharts or algorithms to guide provider decision-making and offer education to learners about evidence behind clinical care recommendations.69 Consistent use of pathways decreases

hospital LOS, increases adherence to evidence-based practices, and improves flow through emergency departments when applied to care for common pediatric conditions such as asthma and acute gastroenteritis.69–71 Finally, structured communication during multidisciplinary rounds and hand-offs are additional key processes driving quality and improved outcomes in the PICU.72–74 An example of the efficacy of structured hand-off communication is I-PASS (illness severity, patient summary, action list, situational awareness, and synthesis by receiver). Reliable use of this nursing hand-off bundle demonstrated improvements in hand-off completeness and quality. However, there was no evidence that clinical outcomes improved.75 When elements of structure and process come together successfully, a unit begins to function as an HRO. Several PICUs are adopting characteristics of HROs to drive improvements in safety and quality.11,76 To the extent that these principles (see Table 2.1) are operational, errors and adverse events should dramatically decrease, thus improving outcomes. Safety II is another important concept in the pursuit of improved outcomes and error reduction that is receiving increased attention in healthcare, in particular in high-functioning organizations and systems. Safety I involves a retrospective investigation of adverse events (find and fix) while Safety II focuses on what is going right (proactive anticipation of harm to come) in high-functioning systems.77 Erik Hollnagel, an early Safety II pioneer, identified four components of the concept: monitoring, anticipating, responding, and learning.78 Merandi et al. analyzed staff behaviors related to safety in a complex quaternary PICU environment, which already had good safety outcomes.8,77 Four Safety II behavior themes emerged: (1) relying on teamwork when novel therapies or approaches are considered, (2) using teams to respond to challenging circumstances, (3) maintaining healthy skepticism, and (4) bringing atypical approaches from other environments.77 As organizations become more reliable and adverse events are less common, Safety II is likely to become an increasingly important field of study with an opportunity to significantly improve outcomes.

Outcomes It is a challenge to compare outcomes between PICUs given the marked heterogeneity in case mix and illness severity. No single measure is sufficient to adequately summarize the overall quality of care that a critically ill child receives in a particular PICU. We believe that a panel of metrics, including both process and outcome measures, is necessary. Some panels, albeit without process measures, are already in use. The Center for Medicare and Medicaid Service’s Agency for Healthcare Research and Quality has three PICUspecific outcomes in their Pediatric Quality Measure Program.79 These include risk assessment for pressure ulcers, appropriateness of red blood cell transfusion, and baseline screening of nutritional status within 24 hours of PICU admission. Other outcome measures, such as HAC rates, should also be included. The Preventable Harm Index is currently used to aggregate total events of harm over a given time for hospitals or groups of hospitals and can be customized to specific units, such as a PICU.2,3,9 Virtual Pediatric Systems is a data collecting network of over 135 PICUs across the country used for numerous research endeavors, including recalibration of commonly used mortality and length-of-stay prediction models.80 Variations on these models have existed for many years. The two most commonly used today (PIM 2 and PRISM III) represent several iterations and recalibrations of their original forms (see Chapter 12). PIM 2 focuses on the first hour of intensivist



CHAPTER 2

High-Reliability Pediatric Intensive Care Unit: Role of Intensivist and Team in Obtaining Optimal Outcomes

contact; PRISM III focuses on the first 12 to 24 hours to predict risk of mortality. These models are physiologically based severityof-illness scoring systems. They are used to identify observed-topredicted outcomes, in particular for mortality and LOS. These observed-to-predicted ratios can be used to compare outcomes between ICUs with similar severity of illnesses.81 The Virtual PICU, using recurrent neural networks, has developed a severityof-illness model that continually updates as new data are added from participating PICUs.82 An outcome in Donebedian’s construct that deserves further mention is value. Today, this is an increasingly important consideration when evaluating any process, particularly in healthcare. Schleien documented many financial aspects related to PICU care delivery and discusses ways to enhance revenue and decrease cost in a system, which in the United States is largely based on fee for service.83 He anticipates that, over time, the overall payment system will shift to capitation and many incentives currently in place for overutilization of various diagnostic and therapeutic resources may shift. If this results in limitations on use of expensive unproven technologies and medications, when these services and products would have been available previously, it may present challenges for pediatric intensivists to provide high-quality care with optimal value-based outcomes.

21

Further impacting the cost of care is the previously described shortage in pediatric intensivists. This factor, in addition to higher staffing demands on intensivists, growth of specialized ICUs—such as cardiac intensive care units (CICUs) and neuro-ICUs—and changes in duty hour requirements for trainees, will impact the professional cost of providing pediatric intensive care.

Summary A modern PICU is a complex system interacting with multiple other complex systems. To attain, or even approach, HRO-level performance, the pediatric intensivist will need more than just clinical expertise. As leader of a multidisciplinary PICU team, the pediatric intensivist must consistently reinforce and model HRO principles while understanding and managing delivery of optimal clinical care, including addressing the realities of personnel shortages, rapidly changing reimbursement mechanisms, and the everincreasing expectations of patients and their families. Intensivists have a history of rising to great challenges in the past—there is every reason to expect that spirit to continue. The full reference list for this chapter is available at ExpertConsult.com.

e1







































32.





33.





34.





35.





36.





37.





38.





39.





40.





41.





42.





43.





44.



45.





















31.





30.





29.





28.





27.





26.









25.





24.









1. Institute of Medicine. Crossing the Quality Chasm: A New Health System for the 21st Century. Washington, DC: Institute of Medicine; 2001. 2. Weick KE, Sutcliffe KM. Managing the Unexpected: Resilient Performance in An Age of Uncertainty. 2nd ed. San Francisco: JosseyBass; 2007. 3. Weick KE, Sutcliffe KM, Obstfeld D. Organizing for high reliability: processes of collective mindfulness. In: Sutton RI, Staw BM, ed. Research in Organizational Behavior. Greenwich, CT: JAI Press; 1999:81-124. 4. Mangione-Smith R, DeCristofaro AH, Setodji CM, et al. The quality of ambulatory care delivered to children in the United States. N Engl J Med. 2007;357:1515-1523. 5. Muething SE, Goudie A, Schoettker PJ, et al. Quality improvement initiative to reduce serious safety events and improve patient safety culture. Pediatrics. 2012;130:e423-431. 6. Peterson TH, Teman SF, Connors RH. A safety culture transformation: its effects at a children’s hospital. J Patient Saf. 2012;8:125-130. 7. Brilli RJ, McClead Jr RE, Crandall WV, et al. A comprehensive patient safety program can significantly reduce preventable harm, associated costs, and hospital mortality. J Pediatr. 2013;163:16381645. 8. Berry JC, Davis JT, Bartman T, et al. Improved Safety culture and teamwork climate are associated with decreases in patient harm and hospital mortality across a hospital system. J Patient Saf. 2016. 9. Lyren A, Brilli R, Bird M, et al. Ohio Children’s Hospitals’ solutions for patient safety: a framework for pediatric patient safety improvement. J Healthc Qual. 2016;38:213-222. 10. Lyren A, Brilli RJ, Zieker K, et al. Children’s hospitals’ solutions for patient safety collaborative impact on hospital-acquired harm. Pediatrics. 2017;140(3):e20163494. 11. Niedner MF, Muething SE, Sutcliffe KM. The high-reliability pediatric intensive care unit. Pediatr Clin North Am. 2013;60:563-580. 12. St Andre A. The formation, elements of success, and challenges in managing a critical care program: part I. Crit Care Med. 2015;43: 874-879. 13. Riley C, Poss WB, Wheeler DS. The evolving model of pediatric critical care delivery in North America. Pediatr Clin North Am. 2013;60:545-562. 14. McQuillan P, Pilkington S, Allan A, et al. Confidential inquiry into quality of care before admission to intensive care. BMJ. 1998;316: 1853-1858. 15. Bonafide CP, Roland D, Brady PW. Rapid response systems 20 years later: new approaches, old challenges. JAMA Pediatr. 2016;170: 729-730. 16. Halpern NA. Early warning systems for hospitalized pediatric patients. JAMA. 2018;319:981-982. 17. McKelvie B MJ, Chan J, Momoli F, Ramsay C, Lobos AT. Increased Mortality and length of stay associated with medical emergency team review in hospitalized pediatric patients: a retrospective cohort study. Pediatr Crit Care Med. 2017;18(6):571-579. 18. Stewart C, Brilli RJ. Does a medical emergency team activation define a new paradigm of mortality risk? Pediatr Crit Care Med. 2017;18:601-602. 19. Brady PW, Muething S, Kotagal U, et al. Improving situation awareness to reduce unrecognized clinical deterioration and serious safety events. Pediatrics. 2013;131:e298-e308. 20. Wheeler DS, Dewan M, Maxwell A, et al. Staffing and workforce issues in the pediatric intensive care unit. Transl Pediatr. 2018;7: 275-283. 21. Treble-Barna A, Beers SR, Houtrow AJ, et al. PICU-based rehabilitation and outcomes assessment: a survey of pediatric critical care physicians. Pediatr Crit Care Med. 2019;20(6):e274-e282. 22. Weled BJ, Adzhigirey LA, Hodgman TM, et al. Critical care delivery: the importance of process of care and ICU structure to improved outcomes: an update from the American College of Critical Care



23.

References

Medicine Task Force on Models of Critical Care. Crit Care Med. 2015;43:1520-1525. Berrens ZJ, Gosdin CH, Brady PW, et al. Efficacy and safety of pediatric critical care physician telemedicine involvement in rapid response team and code response in a satellite facility. Pediatr Crit Care Med. 2019;20:172-177. Donabedian A. Evaluating the quality of medical care. Milbank Mem Fund Q. 1966;44(3):166-206. The American Board of Pediatrics. Pediatric Physicians Workforce Data Book, 2017-2018. Chapel Hill, NC: American Board of Pediatrics; 2018. Rehder KJ, Cheifetz IM, Markovitz BP, et al. Survey of in-house coverage by pediatric intensivists: characterization of 24/7 inhospital pediatric critical care faculty coverage. Pediatr Crit Care Med. 2014;15:97-104. Freed GL, Boyer DM, Van KD, et al. Variation in part-time work among pediatric subspecialties. J Pediatr. 2018;195:263-268. Pastores SM, Kvetan V, Coopersmith CM, et al. Workforce, workload, and burnout among intensivists and advanced practice providers: a narrative review. Crit Care Med. 2019;47:550-557. Verger JT, Marcoux KK, Madden MA, et al. Nurse practitioners in pediatric critical care: results of a national survey. AACN Clin Issues. 2005;16:396-408. Sorce L, Simone S, Madden M. Educational preparation and postgraduate training curriculum for pediatric critical care nurse practitioners. Pediatr Crit Care Med. 2010;11:205-212. Siegal EM, Dressler DD, Dichter JR, et al. Training a hospitalist workforce to address the intensivist shortage in American hospitals: a position paper from the Society of Hospital Medicine and the Society of Critical Care Medicine. Crit Care Med. 2012;40:1952-1956. Randolph AG, Gonzales CA, Cortellini L, et al. Growth of pediatric intensive care units in the United States from 1995 to 2001. J Pediatr. 2004;144:792-798. Mathur M, Rampersad A, Howard K, et al. Physician assistants as physician extenders in the pediatric intensive care unit setting-A 5-year experience. Pediatr Crit Care Med. 2005;6:14-19. Halpern NA, Tan KS, DeWitt M, et al. Intensivists in U.S. Acute care hospitals. Crit Care Med. 2019;47:517-525. Rimsza ME, Ruch-Ross HS, Clemens CJ, et al. Workforce trends and analysis of selected pediatric subspecialties in the United States. Acad Pediatr. 2018;18:805-812. Radabaugh CL, Ruch-Ross HS, Riley CL, et al. Practice patterns in pediatric critical care medicine: results of a workforce survey. Pediatr Crit Care Med. 2015;16:e308-e312. Moss M, Good VS, Gozal D, et al. An official critical care societies collaborative statement-burnout syndrome in critical care healthcare professionals: a call for action. Chest. 2016;150:17-26. Colville G, Dalia C, Brierley J, et al. Burnout and traumatic stress in staff working in paediatric intensive care: associations with resilience and coping strategies. Intensive Care Med. 2015;41:364-365. Vernon DD. Burnout in the ICU: What do we do now? Pediatr Crit Care Med. 2017;18:725-726. Perlo J, Balik B, Swensen S, Kabcenell A, Landsman J, Feeley D. IHI framework for improving joy in work. Cambridge, MA: IHI; 2017. Shenoi AN, Kalyanaraman M, Pillai A, et al. Burnout and psychological distress among pediatric critical care physicians in the United States. Crit Care Med. 2018;46:116-122. Rewa OG, Stelfox HT, Ingolfsson A, et al. Indicators of intensive care unit capacity strain: a systematic review. Crit Care. 2018;22:86. Opgenorth D, Stelfox HT, Gilfoyle E, et al. Perspectives on strained intensive care unit capacity: a survey of critical care professionals. PLoS One. 2018;13:e0201524. Gupta P, Rettiganti M, Rice TB, et al. Impact of 24/7 in-hospital intensivist coverage on outcomes in pediatric intensive care. A multicenter study. Am J Respir Crit Care Med. 2016;194:1506-1513. Gupta P, Rettiganti M, Jeffries HE, et al. Association of 24/7 inhouse intensive care unit attending physician coverage with outcomes in children undergoing heart operations. Ann Thorac Surg. 2016;102:2052-2061.

e2













































64. Toltzis P, O’Riordan M, Cunningham DJ, et al. A statewide collaborative to reduce pediatric surgical site infections. Pediatrics. 2014;134:e1174-e1180. 65. Evans IVR, Phillips GS, Alpern ER, et al. Association between the New York sepsis care mandate and in-hospital mortality for pediatric sepsis. JAMA. 2018;320:358-367. 66. Balamuth F, Alpern ER, Abbadessa MK, et al. Improving recognition of pediatric severe sepsis in the emergency department: contributions of a vital sign-based electronic alert and bedside clinician identification. Ann Emerg Med. 2017;70:759-768. e752. 67. Balamuth F, Weiss SL, Fitzgerald JC, et al. Protocolized treatment is associated with decreased organ dysfunction in pediatric severe sepsis. Pediatr Crit Care Med. 2016;17:817-822. 68. Li S, Rehder KJ, Giuliano Jr JS, et al. Development of a quality improvement bundle to reduce tracheal intubation-associated events in pediatric ICUs. Am J Med Qual. 2016;31:47-55. 69. Rutman L, Atkins RC, Migita R, et al. Modification of an established pediatric asthma pathway improves evidence-based, efficient care. Pediatrics. 2016;138:e20161248. 70. Rutman L, Klein EJ, Brown JC. Clinical pathway produces sustained improvement in acute gastroenteritis care. Pediatrics. 2017;140: e20164310. 71. Lee J, Rodio B, Lavelle J, et al. Improving anaphylaxis care: the impact of a clinical pathway. Pediatrics. 2018;141:e20171616. 72. Joy BF, Elliott E, Hardy C, et al. Standardized multidisciplinary protocol improves handover of cardiac surgery patients to the intensive care unit. Pediatr Crit Care Med. 2011;12:304-308. 73. Starmer AJ, Spector ND, Srivastava R, et al. Changes in medical errors after implementation of a handoff program. N Engl J Med. 2014;371:1803-1812. 74. Vats A, Goin KH, Villarreal MC, et al. The impact of a lean rounding process in a pediatric intensive care unit. Crit Care Med. 2012;40:608-617. 75. Starmer AJ, Schnock KO, Lyons A, et al. Effects of the I-PASS Nursing Handoff Bundle on communication quality and workflow. BMJ Qual Saf. 2017;26:949-957. 76. LaRovere JM, Jeffries HE, Sachdeva RC, et al. Databases for assessing the outcomes of the treatment of patients with congenital and paediatric cardiac disease: the perspective of critical care. Cardiol Young. 2008;18 (suppl 2):130-136. 77. Merandi J, Vannatta K, Davis JT, et al. Safety II behavior in a pediatric intensive care unit. Pediatrics. 2018;141:e20180018. 78. Hollnagel E, Wears RL, Braithwaite J. From Safety-I to Safety-II: A White Paper. The Resilient Health Care Net. University of Southern Denmark, University of Florida, and Macquarie University; 2015. 79. Agency for Healthcare Research and Quality. Pediatric Quality Measures Program. 2019. www.ahrq.gov/pqmp/index.html. 80. LLC VPS. Virtual Pediatric Systems. 2019. www.myvps.org. 81. Visser IH, Hazelzet JA, Albers MJ, et al. Mortality prediction models for pediatric intensive care: comparison of overall and subgroup specific performance. Intensive Care Med. 2013;39:942-950. 82. Laura P. and, Leland K. Whittier Virtual PICU. www.vpicu.net. 83. Schleien CL. The pediatric intensive care unit business model. Pediatr Clin North Am. 2013;60:593-604.









































45a. Nishisaki A, Pines JM, Lin R et al. The impact of 24-hr, in-hospital pediatric critical care attending physician presence on process of care and patient outcomes. Crit Care Med. 2012;40:2190-5. 46. Dara SI, Afessa B. Intensivist-to-bed ratio: association with outcomes in the medical ICU. Chest. 2005;128:567-572. 47. Carayon P, Gurses AP. A human factors engineering conceptual framework of nursing workload and patient safety in intensive care units. Intensive Crit Care Nurs. 2005;21:284-301. 48. Ward NS, Afessa B, Kleinpell R, et al. Intensivist/patient ratios in closed ICUs: a statement from the Society of Critical Care Medicine Taskforce on ICU Staffing. Crit Care Med. 2013;41:638-645. 49. Bartley J, Streifel AJ. Design of the environment of care for safety of patients and personnel: does form follow function or vice versa in the intensive care unit? Crit Care Med. 2010;38:S388-S398. 50. Valentin A, Ferdinande P. Recommendations on basic requirements for intensive care units: structural and organizational aspects. Intensive Care Med. 2011;37:1575-1587. 51. Leaf DE, Homel P, Factor PH. Relationship between ICU design and mortality. Chest. 2010;137:1022-1027. 52. Halpern NA. Innovative designs for the smart ICU: part 2: The ICU. Chest 2014;145:646-658. 53. Halpern NA. Innovative designs for the smart ICU: part 3: Advanced ICU informatics. Chest. 2014;145:903-912. 54. Han JE, Rabinovich M, Abraham P, et al. Effect of Electronic health record implementation in critical care on survival and medication errors. Am J Med Sci. 2016;351:576-581. 55. Caldwell NA, Power B. The pros and cons of electronic prescribing for children. Arch Dis Child. 2012;97:124-128. 56. Alarhayem AQ, Muir MT, Jenkins DJ, et al. Application of electronic medical record-derived analytics in critical care: Rothman Index predicts mortality and readmissions in surgical intensive care unit patients. J Trauma Acute Care Surg. 2019;86:635-641. 57. Despins LA. Automated deterioration detection using electronic medical record data in intensive care unit patients: a systematic review. Comput Inform Nurs. 2018;36:323-330. 58. Kipnis P, Turk BJ, Wulf DA, et al. Development and validation of an electronic medical record-based alert score for detection of inpatient deterioration outside the ICU. J Biomed Inform. 2016;64:10-19. 59. Randall KH, Slovensky D, Weech-Maldonado R, et al. Self-reported adherence to high reliability practices among participants in the Children’s Hospitals’ Solutions for Patient Safety Collaborative. Jt Comm J Qual Patient Saf. 2019;45:164-169. 60. Pun BT, Balas MC, Barnes-Daly MA, et al. Caring for critically ill patients with the ABCDEF bundle: results of the ICU Liberation Collaborative in over 15,000 adults. Crit Care Med. 2019;47:3-14. 61. Frank G, Walsh KE, Wooton S, et al. Impact of a Pressure injury prevention bundle in the solutions for patient safety network. Pediatr Qual Saf. 2017;2:e013. 62. Davis KF, Colebaugh AM, Eithun BL, et al. Reducing catheterassociated urinary tract infections: a quality-improvement initiative. Pediatrics. 2014;134:e857-e864. 63. Nordin AB, Sales SP, Besner GE, et al. Effective methods to decrease surgical site infections in pediatric gastrointestinal surgery. J Pediatr Surg. 2017.

e3

Abstract: Pediatric intensivists, as leaders of multidisciplinary pediatric intensive care unit (PICU) teams, require an enhanced understanding of the PICU as a system within the greater hospital system, including management and evaluation of PICU operations and outcomes. Integrated into this chapter’s discussion are specific challenges to attaining high-reliability care delivery. We

examine three key dimensions: structure (the setting in which care is delivered, including ICU staffing and physical design), process (how care is provided), and outcomes (end points of care). Key Words: High reliability, intensivist-led ICU, safety culture, quality outcomes, multidisciplinary teams

3 Critical Communications in the Pediatric Intensive Care Unit SHELINA M. JAMAL, KATHERINE BANKER, AND HARRIS P. BADEN





“Safety First” is the mantra of twenty-first century American healthcare since the Institute of Medicine’s 1999 publication, “To Err Is Human: Building a Safer Health System.”1 That groundbreaking report drew attention to the high rate of medical errors resulting from ineffective communication and teamwork and compelled an industry-wide transformation in healthcare delivery systems and practices. As identified by the Joint Commission on Quality and Patient Safety, “communication failures are frequent in healthcare and have been identified as a root cause in approximately 65% of sentinel events reported to The Joint Commission.”2 Contemporary pediatric intensive care units (ICUs) are highly technical and data-rich environments with specialized, multidisciplinary care teams that are ever rotating to provide 24/7 coverage. Accordingly, establishment of an effective and reliable system of communication is imperative in every ICU. A variety of schemes and tools are available to optimize communication and mitigate risk. Common to all is the focus on ensuring that every member of the healthcare team, including the patient, is on the same page. For example, borrowing from the US Department of Defense and other high-reliability industries, the Agency for Healthcare Research and Quality developed TeamSTEPPS, a collection of strategies and tools to promote situational awareness and development of a shared mental model by fostering communication, leadership, situation monitoring, and mutual support, all rooted in team structure and dynamics.3 Dr. Mica Endsley, Chief Scientist of the United States Air Force, pioneered the development and evaluation of systems to support 22









Patient safety is paramount to outstanding healthcare, and effective communication is critical to sustaining a safe patient care environment. Strategies and tools to promote situational awareness and shared mental models in the healthcare setting were developed after a groundbreaking 1999 report from the Institute of Medicine describing of a high rate of medical errors resulting from ineffective communication and teamwork. Using specific design elements in the intensive care unit (ICU) can enhance patient surveillance and nonverbal communication.













PEARLS Applying standardized work—such as organized huddles, checklists, and structured rounds—can result in less variability and more consistent communication within multidisciplinary ICU care teams. Intentional and ongoing education and assessment of communication skills—such as closed loop communication using techniques such as simulation and debriefing—is vital. Implementation of interdisciplinary team training provides skills that improve teamwork, enhance communication, and contribute to patient safety.

human situational awareness and decision-making, which she defined as “the perception of elements in the environment within a volume of time and space, the comprehension of their meaning, and the projection of their status in the near future.”4 Kenneth Craik, philosopher and psychologist, first described the concept of mental models5 as an explanation of an individual’s thought process about a particular situation that can be influenced by the surrounding circumstances, team member dynamics, and a person’s intuitive perception. Mental models shape behavior and set an approach, or personal algorithm, to solving problems. In the team setting, it is imperative that all individual members share the same mental model. This chapter describes communication tools and other techniques to enhance situational awareness and the development of a shared mental model as key methods of improving patient safety in the critical care setting. Comprehensive patient safety efforts encompass ICU design, monitoring systems, electronic medical records, patient flow schemes, closedloop communication, staffing models, and team training. We describe verbal and visual communication strategies that have been deployed in the medical setting with successful and sustainable results.

Intensive Care Unit Design According to the 2012 Society of Critical Care Medicine’s guidelines, optimal ICU design can (1) help reduce medical errors, (2) improve patient outcomes, (3) reduce length of stay, (4) increase



CHAPTER 3 Critical Communications in the Pediatric Intensive Care Unit

Huddles Huddles are brief gatherings that bring team members together to create a shared mental model regarding a distinct procedure or event or a status update across the entire ICU.12–15 They should include team introductions, review of planned activities, anticipation of problems that may arise, and creation of contingency plans. Huddles serve to activate teams, empowering each team member to share responsibility in the completion of the task, while encouraging openness and trust among the team,



Checklists have been widely adopted by the healthcare industry, with demonstrated reductions in morbidity, mortality, and preventable errors.20–23 In 2004, The Joint Commission Board of Commissioners created the Universal Protocol to address the wrong site, wrong procedure, and wrong person surgery and other procedures.24,25 A 2010 survey found that greater than 90% of respondents agreed or strongly agreed that there was benefit in using the Universal Protocol in hospital units where invasive procedures are performed. Well-constructed checklists: • Function as a communication tool with demonstrated benefit in routine procedures (e.g., tracheal extubation, procedural sedation, magnetic resonance imaging screening, and preoperative screening) and less frequent occurrences (e.g., extracorporeal membrane oxygenation cannulation, computer downtime).26–32 • Increase the reliability of care processes. Checklists performed at the end of rounds have been shown to reduce central line–associated bloodstream infections, optimize nutrition, wean sedation, and more.33–37 • Review daily care plans. Use of daily goals sheets have demonstrated reduction in ICU length of stay and significant reductions in mortality.2,38,39 • Serve as an evaluation/audit tool. Although creation and implementation of checklists are important, shifting the culture and behaviors of those using the checklist is what determines success. Implementing mandatory checklists with limited focus on transformation of attitudes can result in no change in outcomes.40,41

Extracting useful information from the electronic medical record can be challenging. Alerts and notifications regarding code status, difficult airway status, medication allergies, or important social issues allow rapid orientation to the patient’s status and care plan. Existing monitoring and evolving prediction models take patient data (i.e., from the telemetry monitors) and identify patterns that might warn of impending clinical deterioration. Likewise, these technologies are helpful in retrospective reviews for quality improvement purposes.

Checklists



Medical Record

facilitating communication, and improving overall situational awareness.12,16 Keys to successful implementation of huddles in a medical setting include, but are not limited to15,17,18: • Designating a leader • Mandatory participation of all team members • Incorporation into standard work practice • Limiting to 10 minutes or less • Holding in a central location Huddles can be used in a variety of scenarios: • Admission: facilitate communication to review events, create a care plan, and highlight risks/concerns. • Periprocedure: orient the team, define roles, and identify potential pitfalls and contingency plans. • ICU day/night shift: review expected procedures, admissions, transfers, discharges, and high-acuity patients • ICU workflow: discuss planned ICU admissions and discharges along with their impact on staffing, bed availability, and hospital census. • Daily check-in (a Healthcare Performance Institute initiative)19: This is a focused and directed conversation to address safety/quality issues from the last 24 hours, anticipated safety/ quality issues in the next 24 hours, and status reports on issues identified that day or the day before.



















social support for patients, and (5) play a role in reducing patient cost.6 Private rooms enhance the patient and family experience, and minimizing noise and disturbances can promote the healing process.7–9 On the other hand, published reports describe a correlation between lower ICU visibility and increased mortality.10 As such, reliable monitoring systems are crucial to patient safety and quality of care. This includes not only bedside monitoring but also remote monitoring of patients from central workstations and throughout the ICU. Several design elements can enhance situational awareness, patient surveillance, and nonverbal communication both in patient rooms and within the ICU. In patient rooms: • Monitoring should be visible from the door as well as the care team’s workspace. • Display boards in the room can be used for daily care plans, patient and family questions, and family contact information.11 • Signage in the room can convey information to care team members, ancillary staff, and families (e.g., isolation requirements, fracture risk, fall risk, difficult airway or an open chest). • Boards outside the patient’s room identify the nurse, responsible physician, and contact information. Within the ICU: • Remote centralized monitoring (e.g., monitors at various places in the unit itself, conference rooms, call rooms) or remote access monitoring (e.g., web-based applications) should be in place so that even when not in the vicinity of the patient, the patient’s monitoring is visible or accessible. • A central board near the main workstation shows the physical layout of the unit, patient location, nursing assignment, admits and transfers for the day, the care team members, and their contact information. • Signage denoting that a sterile procedure is in progress creates a physical barrier to encourage nonessential personnel to avoid the area and promotes situational awareness within the greater ICU team. • Remote access to operating room monitors and intraoperative cameras allow the ICU team to follow a patient’s progress and be prepared for patient arrival.

23

Rounds Performance of daily rounds in a standardized format results in less variability and more consistent communication within the team. To ensure a shared mental model and optimize situational

24

SECTION I



Pediatirc Critical Care: The Discipline

awareness, all team members (e.g., physicians, nurses, pharmacists, nutritionists, family members) should participate on rounds in preassigned roles.42 Structured reporting of data and presentation of information ensures that no issues are omitted and all concerns are addressed. Daily rounds conclude with review of daily safety checks and order read-back—closed-loop communication with all team members that reenforces the shared mental model.2,37

Additionally, evaluation of a trainee’s ability to communicate is changing with the development of clinical competencies and milestones. Intentional education and assessment of communication skills is a growing expectation.53 There are many ways to incorporate communication into medical training: • A longitudinal curriculum during undergraduate or graduate medical training • Strategies specific to a certain rotation or environment • Focus on communication as a marker for ongoing quality improvement One effective means of teaching and evaluating communication in the ICU is through the use of simulation.54–56 Simulation scenarios focused on closed-loop communication skills—such as clarity of roles and responsibilities, order repeat-backs, clarifying questions, knowledge sharing, reevaluation and summarizing, communication with families, and mutual respect—give ICU staff the opportunity to practice in a high-fidelity, low-risk environment.57–63 Some scenarios, such as an extracorporeal life support cannulation, allow for incorporation of multiple disciplines (e.g., physicians, trainees, nurses, respiratory therapists) leading to more realistic simulation of infrequent, high-risk events that require precision teamwork.64,65 Simulation is further enhanced with thorough debriefing.



Closed-Loop Communication Deficiencies in verbal communication impair the development of team structure, collaboration, and task performance.43,44 Standardized methods of communication have been developed to promote safety and efficiency, thereby reducing the risk of team breakdowns.45,46 A common method used in healthcare settings is closed-loop communication, which involves three steps: • Sender transmits a message using standardized terminology • Receiver accepts the message and verbally acknowledges receipt and understanding • Sender verifies that the message has been received and interpreted correctly





Transitions of Care



Reliable communication is essential at times of transitions of care. Duty hour restrictions for physicians in training have led to increased handoffs and the potential for discontinuity in patient care.47 Consistent use of a handoff tool (e.g., I-PASS: illness severity, patient summary, action list, situation awareness and contingency plans, synthesis by receiver) has been associated with reductions in medical errors and preventable adverse events along with improvements in communication, without a negative effect on workflow.48 Another tool that has been demonstrated to improve patient safety, especially when used to structure communication over the phone, is SBAR (situation, background, assessment, recommendation).49 Transfers between hospitals to the ICU can be especially high risk, as they involve transport of critically ill patients, variable team members and skill sets, and resources limited by space and mobility. Transmission of clear, concise, and accurate information is imperative. Creation of a “communication center” can facilitate this process by: • Use of a single phone number for all referring hospitals and providers wishing to initiate transfer of a patient • Recorded phone conversations to clarify information transmitted as well as quality improvement initiatives • Ability to conference in multiple team members • Prompt and reliable access to the transport team while in route

Debriefing

Medical Training

The Institute of Medicine calls for interdisciplinary team training programs for critical care settings.1 Extensive team training curricula based on concepts central to crew resource management exist and continue to evolve.80–85 Examples include TeamSTEPPS and VA Medical Team Training. Some common themes among these programs include85: • Developing communication strategies that flatten hierarchy and encourage team member assertiveness • Cross-training to tasks, duties, and responsibilities of all team roles • Simulating errors and contingencies • Facilitated debriefings





Debriefing originated in the military (analyzing a mission after it is completed) and gained traction in critical incident reviews (mitigating stress following critical events).66 According to Kolb, the process of reflective observation is a cornerstone of lifelong adult learning.67 In medical simulation, debriefing is facilitated reflection that leads participants to analyze and learn from an event.66 Debriefing is also helpful after high-risk, infrequent events and after any event that team members find particularly challenging. Effective debriefing can improve skill acquisition and retention and staff satisfaction.51,68–71 In general, debriefing includes the following72–79: • Structure: phases include description, analogy/analysis, and application. A communication tool ensures that key points are addressed and the process is standardized. • Content: focus can include communication, medical management processes, and logistics. • participation: team members should be active participants in self-reflection. • Action items: development of a reliable method for follow-up is essential.











The Accreditation Council for Graduate Medical Education requires that Pediatric Critical Care Medicine fellows are able to “demonstrate interpersonal and communication skills that result in the effective exchange of information and collaboration with patients, their families, and health professionals.”50 Communication is also a cornerstone of effective team leadership and is a key feature of Pediatric Advanced Life Support training.51 Staff perceptions of teamwork and team behavior are related to the improvement of quality and safety of patient care as well as communication and teamwork that lead to safer patient care.52

Team Training











CHAPTER 3 Critical Communications in the Pediatric Intensive Care Unit





• Creating shared mental models and situational awareness • Encouraging closed-loop communication Deliberate and continued training, evaluation, and modification are imperative to sustaining the improvement in patient safety that can be achieved when team communication is a focus of a healthcare organization.86–93

Conclusion ICUs are high-stakes, high-risk environments. Reliable and accurate communication is essential to optimizing patient care and safety. An ICU focused on safety is characterized by thoughtful design, reliable organizational systems of communication, and unwavering commitment to teamwork. These elements foster a shared mental model and improve situational awareness among all team members, leading to greater efficiency and safer healthcare delivery.

Key References Brass SD, Olney G, Glimp R, et al. Using the patient safety huddle as a tool for high reliability. Jt Comm J Qual Saf. 2018;44:219-226. Foronda C, MacWilliams B, McArthur E. Interprofessional communication in healthcare: an integrative review. Nurse Educ Pract. 2016;19:36-40.

25

Lane D, Ferri, M, Lemaire J, et al. A systematic review of evidenceinformed practices for patient care rounds in the ICU. Crit Care Med. 2013;41:2015-2029. Low XM, Horriganb D, Brewster D. The effects of team-training in intensive care medicine: a narrative review. J Crit Care. 2018;48:283289. O’Brien A, O’Reilly K, Dechen T, et al. Redesigning rounds in the ICU: standardizing key elements improves interdisciplinary communication. Jt Comm J Qual Patient Saf. 2018;44(10):590-598. Pronovost P, Berenholtz S, Dorman T, et al. Improving communication in the ICU using daily goals. J Crit Care. 2003;18:71-75. Salas E, Wilson KA, Murphey CE, et al. Communicating, coordinating, and cooperating when lives depend on it: tips for teamwork. Jt Comm J Qual Patient Saf. 2008;34:333-341. Shaw DJ, Davidson JE, Smilde RL, et al. Multidisciplinary team training to enhance family communication in the ICU. Crit Care Med. 2014;42(2):265-271. Thompson DR, Hamilton DK, Cadenhead CD, et al. Guidelines for intensive care unit design. Crit Care Med. 2012;40(5):1586-1600. Weaver SJ, Dy SM, Rosen MA, et al. Team-training in healthcare: a narrative synthesis of the literature. BMJ Qual Saf. 2014;23:359372.

The full reference list for this chapter is available at ExpertConsult.com.

e1











































































































1. Institute of Medicine. To Err is Human: Building a Safer Health System. Washington, DC: National Academies Press; 2000. 2. O’Brien A, O’Reilly K, Dechen T, et al. Redesigning rounds in the ICU: standardizing key elements improves interdisciplinary communication. Jt Comm J Qual Patient Saf. 2018;44(10):590-598. 3. TeamSTEPPS 2.0: Core Curriculum. Rockville, MD: Agency for Healthcare Research and Quality; 2014. http://www.ahrq.gov/ professionals/education/curriculum-tools/teamstepps/instructor/ index.html. 4. Endsley, MR. Toward a theory of situation awareness in dynamic systems. Hum Factors. 1995;37(1):32-64. 5. Craik, KJW. The Nature of Explanation. Cambridge: Cambridge University Press; 1967. 6. Thompson DR, Hamilton DK, Cadenhead CD, et al. Guidelines for intensive care unit design. Crit Care Med. 2012;40(5):1586-1600. 7. Hagerman I, Rasmanis G, Blomkvist V, et al. Influence of intensive coronary care acoustics on the quality of care and physiological state of patients. Int J Cardiol. 2004;98:267-270. 8. Graven SN. Clinical research data illuminating the relationship between the physical environment and patient medical outcomes. J Healthc Des. 1997;9:15-9. 9. Xie H, Kang J, Mills GH, et al. Clinical Review: The impact of noise on patients’ sleep and the effectiveness of noise reduction strategies in intensive care units. Crit Care. 2009;13208. 10. Leaf DE, Homel P, Factor PH, et al. Relationship between ICU design and mortality. Chest. 2010;137:1022-1027. 11. Justice LB, Cooper DS, Henderson C, et al. Improving communication during cardiac ICU multidisciplinary rounds through visual display of patient daily goals. Pediatr Crit Care Med. 2016;17:677683. 12. Meeting Tools: Huddles. Cambridge, MA: Institute for Health care Improvement; 2015. http://www.ihi.org/resources/Pages/Tools/ Huddles.aspx. 13. Institute for Healthcare Improvement. Use Regular Huddles and Staff Meetings to Plan Production and to Optimize Team Communication. Cambridge, MA: Institute for Healthcare Improvement; 2015. http:// www.ihi.org/resources/Pages/Changes/UseRegularHuddlesandStaffMeetingstoPlanProductionandtoOptimizeTeamCommunication. aspx. 14. Houck S. What Works: Effective Tools & Case Studies to Improve Clinical Office Practice. Boulder: HealthPress Publishing; 2004. 15. Stewart EE, Johnson BC. Huddles: improve office efficiency in mere minutes. Fam Pract Manag. 2007;14(6):27-29. 16. Brass SD, Olney G, Glimp R, et al. Using the patient safety huddle as a tool for high reliability. Jt Comm J Qual Saf. 2018;44: 219-226. 17. Donnelly LF. Daily management systems in medicine. Radiographics. 2014;34(2):549-555. 18. Medication huddles slash adverse drug events, promote safety culture across all hospital units, including the ED. ED Manag. 2014;26(3):s1-s4. 19. Stockmeier C, Clapper C. Daily Check-in for Safety: From Best Practice to Common Practice. HPI White Paper series. Virginia Beach, VA: Healthcare Performance Improvement; 2010. http://hpiresults. com/publications/HPI%20White%20Paper%20-%20Daily%20 Check-In%20REV%200%20SEP%202010.pdf. 20. Bergs J, Hellings J, Cleemput I et al. Systematic review and metaanalysis of the effect of the WHO surgical safety checklist on postoperative complications. Br J Surg. 2014;101(3):150-158. 21. Haynes AB, Weiser TG, Berry WR, et al. A surgical safety checklist to reduce morbidity and mortality in a global population. N Engl J Med. 2009;360(5):491-499. 22. Hales BM, Pronovost PJ. The checklist: a tool for error management and performance improvement. J Crit Care. 2006;21(3):231-235. 23. Bosk CL, Dixon-Woods M, Goeschel CA, et al. Reality check for checklists. Lancet. 2009;374:444-445.

24. Universal Protocol. Oakbrook Terrace, IL: The Joint Commission; 2015. http://www.jointcommission.org/standards_information/up.aspx 25. Makary MA, Mukherjee A, Sexton JB, et al. Operating room briefings and wrong-site surgery. J Am Coll Surg. 2007;204(2):236-243. 26. Li S, Rehder KJ, Giuliano Jr JS, et al. Development of a quality improvement bundle to reduce tracheal intubation-associated events in pediatric ICUs. Am J Med Qual. 2014. Epub ahead of print. 27. Davis DA, Mazmanian PE, Fordis M, et al. Accuracy of physician self-assessment compared with observed measures of competence: a systematic review. JAMA. 2006;296:1094-1102 . 28. Pronovost PJ, Goeschel CA, Colantuoni E, et al. Sustaining reductions in catheter related bloodstream infections in Michigan intensive care units: observational study. BMJ. 2010;340:c309. 29. Haynes AB, Weiser TG, Berry WR, et al. Changes in safety attitude and relationship to decreased postoperative morbidity and mortality following implementation of a checklist-based surgical safety intervention. BMJ Qual Saf. 2011;20:102-107. 30. Berenholtz SM, Pham JC, Thompson DA, et al. Collaborative cohort study of an intervention to reduce ventilator-associated pneumonia in the intensive care unit. Infect Control Hosp Epidemiol. 2011;32:305-314. 31. Lipitz-Snyderman A, Steinwachs D, Needham DM, et al. Impact of a statewide intensive care unit quality improvement initiative on hospital mortality and length of stay: retrospective comparative analysis. BMJ. 2011;342:d219. 32. Bennett SC, Finer N, Halamek LP, et al. Implementing delivery room checklists and communication standards in a multi-neonatal ICU quality improvement collaborative. 2016;42(8):369-376. 33. Blot K, Bergs J, Vogelaers D, et al. Prevention of CLABSI through QI interventions: a systematic review and meta-analysis. Clin Infect Dis. 2014;59(1):96-105. 34. Shaughnessy L, Jackson J. Introduction of a new ward round approach in a cardiothoracic critical care unit. Nurs Crit Care. 2015. Epub ahead of print. 35. Sharma S, Peters MJ, PICU/NICU Risk Action Group. “Safety by DEFAULT”: introduction and impact of a pediatric ward round checklist. Crit Care. 2013;17(5):R232. 36. Centofanti JE, Duan EH, Hoad NC et al. Use of daily goals checklist for morning ICU rounds: a mixed-methods study. Crit Care. 2014;42(8):1797-1803. 37. Lane D, Ferri, M, Lemaire J, et al. A systematic review of evidenceinformed practices for patient care rounds in the ICU. Crit Care Med. 2013;41:2015-2029. 38. Pronovost P, Berenholtz S, Dorman T, et al. Improving communication in the ICU using daily goals. J Crit Care. 2003;18:71-75. 39. Checkley W, Martin GS, Brown SM, et al. Structure, process, and annual ICU mortality across 69 centers: United States Critical Illness and Injury Trials Group Critical Illness Outcomes Study. Crit Care Med. 2014;42:344-356. 40. Urbach DR, Govindarajan A, Saskin R, et al. Introduction of surgical safety checklists in Ontario, Canada. N Engl J Med. 2014;370:1029-1038. 41. Davis KF, Napolitano N, Li S, et al. Promoters and barriers to implementation of tracheal intubation airway safety bundle: a mixedmethods analysis. Pediatr Crit Care Med. 2017;18(10):965-972. 42. Donovan AL, Aldrich JM, Gross AK, et al. Interprofessional care and teamwork in the ICU. Crit Care Med. 2018;46:980-990. 43. Cooper S, Wakelam A. Leadership of resuscitation teams: “lighthouse leadership.” Resuscitation. 1999;42:27-45. 44. Lingard L, Whyte S, Espin S, et al. Towards safer interprofessional communication: constructing a model of “utility” from preoperative team briefings.J Interprof Care. 2006;20:471-83. 45. Burke CS, Salas E, Wilson-Donnelly K, et al. How to turn a team of experts into an expert medical team: guidance from the aviation and military communities. Qual Saf Health Care. 2004;13:96-104. 46. Salas E, Wilson KA, Murphey CE, et al. Communicating, coordinating, and cooperating when lives depend on it: tips for teamwork. Jt Comm J Qual Patient Saf. 2008;34:333-341.

References

e2





















































65. Anderson JM, Murphy AA, Boyle KB, et al. Simulating extracoporeal membrane oxygenation emergencies to improve human performance. Part II: assessment of technical and behavioral skills. Simul Healthc. 2006;1(4):228-232. 66. Fanning RM, Gaba DM. The role of debriefing in simulation-based learning. Simul Healthc. 2007;2(2):115-125. 67. Kolb D. Experiential Learning: Experience as a Source of Learning and Development. Upper Saddle River, NJ: Prentice Hall; 1984. 68. Percarpio KB, Harris FS, Hatfield BA, et al. Code debriefing from the Department of Veterans Affairs (VA) Medical Team Training program improves the cardiopulmonary resuscitation code process. Jt Comm J Qual Patient Saf. 2010;36(9):424-429, 385. 69. Savoldelli GL, Naik VN, Park J, et al. Value of Debriefing during simulated crisis management. Anesthesiology. 2006;105(2):279-285. 70. Edelson DP, Litzinger B, Arora V, et al. Improving in-hospital cardiac arrest process and outcomes with performance debriefing. Arch Intern Med. 2008;168(10):1063-1069. 71. Cheng A, Hunt EA, Donoghue A, et al. Examining pediatric resuscitation education using simulation and scripted debriefing: a multicenter randomized trial. JAMA Pediatr. 2013;167(6):528-536. 72. Eppich W, Cheng A. Promoting excellence and reflective learning in simulation. development and rationale for a blended approach to health care simulation debriefing. Simul Healthc. 2015;10(2):106-115. 73. Rudolph JW, Simon R, Dufresne RL, et al. There’s no such thing as “nonjudgmental” debriefing: theory and method for debriefing with good judgment. Simul Healthc. 2006;1(1):49-55. 74. Raemer D, Anderson M, Cheng A, et al. Research regarding debriefing as part of the learning process. Simul Healthc. 2011;6(7):S52-S57. 75. Edelson DP, LaFond CM. Deconstructing debriefing for simulationbased education. JAMA Pediatr. 2013;167(6):586-587. 76. Dismukes RK, Gaba DM, Howard SK. So many roads: facilitated debriefing in healthcare. Simul Healthc. 2006;1(1):23-25. 77. Ahmed M, Arora S, Russ S, et al. Operation debrief: a SHARP improvement in performance feedback in the operating room. Ann Surg. 2013;258(6):958-963. 78. Mullan PC, Wuestner E, Kerr TA, et al. Implementation of an in situ qualitative debriefing tool for resuscitations. Resuscitation. 2013;84(7):946-951. 79. Kolbe M, Weiss M, Grote G, et al. TeamGAINS: a tool for structured debriefings for simulation-based team trainings. BMJ Qual Saf. 2013;22:541-553. 80. Baker DP, Gustafson S, Beaubien JM, et al. Medical team training programs in health care. In: Henriksen K, Battles JB, Marks ES, et al, eds. Advances in Patient Safety: From Research to Implementation. Vol 2. Rockville, MD: Agency for Healthcare Research and Quality; 2005. 81. Weaver SJ, Dy SM, Rosen MA, et al. Team-training in healthcare: a narrative synthesis of the literature. BMJ Qual Saf. 2014;23:359-372. 82. Thomas EJ. Improving teamwork in healthcare: current approaches and the path forward. BMJ Qual Saf. 2011;20(8):647-650. 83. Buljac-Samardzic M, Dekker-van Doorn CM, van Wijngaarden JDH, et al. Interventions to improve team effectiveness: a systematic review. Health Policy. 2010;94:183-195. 84. Cumin D, Boyd MJ, Webster CS, et al. A systematic review of simulation for multidisciplinary team training in operating rooms. Simul Healthc. 2013;8(3):171-179. 85. Weaver SJ, Lyons R, DiazGranados D, et al. The anatomy of health care team training and the state of practice: a critical review. Acad Med. 2010;85(11):1746-1760. 86. Eppich WJ, Brannen M, Hunt EA. Team training: implications for emergency and critical care pediatrics. Curr Opin Pediatr. 2008;20:255-260. 87. Mayer CM, Cluff L, Lin W, et al. Evaluating efforts to optimize TeamSTEPPS implementation in surgical and pediatric intensive care units. Jt Comm J Qual Patient Saf. 2011;37(8):365-374.











































47. DeRienzo CM, Frush K, Barfield ME, et al. Handoffs in the era of duty hours reform: a focused review and strategy to address changes in the Accreditation Council for Graduate Medical Education Common Program Requirements. Acad Med. 2012;87(4): 403-410. 48. Starmer AJ, Spector ND, Srivastava R, et al. Changes in medical errors after implementation of a handoff program. N Engl J Med. 2014;371:1803-1812. 49. Muller M, Jurgens J, Redaelli M, et al. Impact of the communication and patient hand-off tool SBAR on patient safety: a systematic review. BMJ Open. 2018;8:e022202. 50. ACGME. Program Requirements For Graduate Medical Education In Pediatric Critical Care Medicine. 2007. https://www.acgme.org/ acgmeweb/Portals/0/PFAssets/2013-PR-FAQ-PIF/323_critical_ care_peds_07012013.pdf 51. Cheng A, Rodgers DL, van der Jagt E, et al. Evolution of the Pediatric Advanced Life Support course: enhanced learning with a new debriefing tool and web-based module for PALS instructors. Pediatr Crit Care Med. 2012;13(5):589-595. 52. Manser T. Teamwork and patient safety in dynamic domains of healthcare: a review of the literature. Acta Anaesthesiol Scand. 2009; 53:143-151. 53. Turner DA, Mink RB, Lee KJ, et al. Are pediatric critical care medicine fellowships teaching and evaluating communication and professionalism? Pediatr Crit Care Med. 2013;14(5):454-461. 54. McGaghie WC, Issenberg SB, Cohen ER, et al. Does simulationbased medical education with deliberate practice yield better results than traditional clinical education? A meta-analytic comparative review of the evidence. Acad Med. 2011;86(6):706-711. 55. Mundell W, Kennedy C, Szostek J, Cook D. Simulation technology for resuscitation training: A systematic review and metaanalysis. Resuscitation. 2013;84:1174-1183. 56. Cortegiani A, Russotto V, Gregoretti C, Giarratano A, Antonelli M. Medical simulation for ICU staff: Does it influence safety of care? Intensive Care Med. 2016;42:635. 57. Steadman RH, Coates WC, Huang YM et al. Simulation-based training is superior to problem-based learning for the acquisition of critical assessment and management skills. Crit Care Med. 2006;34(1):151-157. 58. Merien AE, van de Ven J, Mol BW, et al. Multidisciplinary team training in a simulation setting for acute obstetric emergencies: a systematic review. Obstet Gynecol. 2010;115(5):1021-1031. 59. Allan CK, Thiagarajan RR, Beke D, et al. Simulation-based training delivered directly to the pediatric cardiac intensive care unit engenders preparedness, comfort, and decreased anxiety among multidisciplinary resuscitation teams. J Thorac Cardiovasc Surg. 2010; 140(3):646-652. 60. Gordon JA, Wilkerson WM, Shaffer DW, et al. “Practicing” medicine without risk: students’ and educators’ responses to high-fidelity patient simulation. Acad Med. 2001;76(5):469-472. 61. Cuoto TB, Kerrey BT, Taylor RG, et al. Teamwork skills in actual, in situ, and in-center pediatric emergencies: performance levels across settings and perceptions of comparative educational impact. Simul Healthc. 2015;10(5):76-84. 62. Shaw DJ, Davidson JE, Smilde RI, Sondoozi T, Agan D. Multidisciplinary team training to enhance family communication in the ICU. Crit Care Med. 2014;42(2):265-271. 63. Foronda C, MacWilliams B, McArthur E. Interprofessional communication in healthcare: an integrative review. Nurse Educ Pract. 2016;19:36-40. 64. Brum R, Rajani R, Gelandt E, et al. Simulation training for extracorporeal membrane oxygenation. Ann Card Anaesth. 2015;18(2): 185-190.

e3











91. Low XM, Horrigan D, Brewster D. The effects of team-training in intensive care medicine: a narrative review. J Crit Care. 2018;48:283289. 92. Mayo AT, Williams Woolley A. State of the art and science teamwork in health care: maximizing collective intelligence via inclusive collaboration and open communication. AMA J Ethics. 2016; 18(9):933-940. 93. Salas E, Weaver SJ, DiazGranados D, Lyons R, King H. Sounding the call for team training in health care: some insights and warnings. Acad Med. 2009;84(10):S128-S131.











88. Van Schaik SM, Plant J, Diane S, et al. Interprofessional team training in pediatric resuscitation: a low-cost, in situ simulation program that enhances self-efficacy among participants. Clin Pediatr (Phila). 2011;50(9):807-815. 89. Stocker M, Allen M, Pool N, et al. Impact of an embedded simulation team training programme in a pediatric intensive care unit: a prospective, single centre, longitudinal study. Intensive Care Med. 2012;38;99-104. 90. Katinakis PA, Spronk PE. The effects of structural crew resource management (CRM)/medical team work (MTW) training in the ICU: The MTW Impact And Evaluation Study. Am J Respir Crit Care Med. 2016;193:A3207.

e4

Abstract: Recognition of a high rate of medical errors resulting from ineffective communication and teamwork compelled development of strategies and tools to promote situational awareness and a shared mental model in the healthcare setting. Using specific design elements in the intensive care unit (ICU) can enhance patient surveillance and nonverbal communication. Applying standardized work—such as organized huddles, checklists, and structured rounds—can result in less variability and more consistent communication within multidisciplinary ICU care

teams. Intentional and ongoing education and assessment of communication skills—such as closed-loop communication using techniques such as simulation and debriefing—is vital. Implementation of interdisciplinary team training provides skills that improve teamwork, enhance communication, and contribute to patient safety. Key Words: Communication, safety, situational awareness, shared mental model

4 Professionalism in Pediatric Critical Care BRADLEY P. FUHRMAN AND LYNN J. HERNAN

PEARLS •

• •







The medical profession is largely self-regulated by a system composed of state medical licensing boards, subspecialty boards, and credentialing and accrediting bodies. This system confers many benefits on its members. It is the purview of society to allow us this autonomy. To sustain these professional benefits, which include control of entry into our profession and to maintain the autonomy of our credentialing and accrediting bodies, we must honor our contracts with society.













Professionalism is, in its simplest form, putting the patient first, placing altruism before self-interest, as is expected of us. Beyond that, professionalism is a more complete charter that ties altruism to the concrete realities of the doctor-patient relationship and the marketplace in which we practice. The Physician Charter is grounded in the principles of altruism, patient autonomy, and social justice. It codifies the physician’s contract with society.

Profession

The Virtuous Doctor

Pediatric critical care is a profession. To be certified and practice as a pediatric intensivist, one must master several bodies of special knowledge, complete apprenticeships in pediatrics and pediatric critical care, earn various educational certificates, pass examinations (the culmination of which is the American Board of Pediatrics certifying examination in Pediatric Critical Care), and be granted a license to practice by a state medical board. Our profession oversees that process much as a guild controls its members and membership. As a profession, we train ourselves, test ourselves, credential ourselves, and discipline ourselves. We derive many benefits from our status as professionals. We are paid as professionals, respected as professionals, and valued by society as professionals. The autonomy of our profession is granted and allowed by society. There are, accordingly, unwritten contracts between our profession and society. In exchange for the benefits that “professionalism” confers and the autonomy, self-governance, and control of licensure that is ceded to us, society expects us to meet its needs within the boundaries of our expertise. We gain the advantages listed here because we provide the quality of service that society requires. In the end, professionalism is the set of responsibilities and behaviors that fulfill our contracts with society. These characteristics exemplify the good and virtuous doctor because that is the model society would hold us to, not because virtue has intrinsic value (though it does), but because deviation from virtue breaks our contract.

Most would agree as to what characteristics exemplify the good and virtuous doctor. We have watched exemplary characters portrayed on television over the years in dramas, comedies, and in advertisements. These are the professionals that our parents, our patients, and the rest of society expect us to emulate, imitate, dress like, and act like. We have long had a sense of the characteristics portrayed. However, over the past several decades, professional medical organizations have obsessed over defining, teaching, assessing, and understanding the behaviors that connote professionalism. Now, why is that?

26

Stakes Healthcare now consumes (produces) about 17% of the US gross domestic product.1 It has become increasingly technology intense and engages financial giants in the form of insurers, pharmaceutical companies, device/equipment manufacturers, both freestanding and gigantic nationwide inpatient facilities, and information storage/management enterprises. Healthcare employs about 14,000,000 workers,2 some of whom are independent practitioners and many of whom are contracted to large enterprises. Two of the largest third-party payers in the United States are Medicare and Medicaid, which together expend 4.7% of the gross domestic product on healthcare. Those payments flow from the general tax coffers to physicians and other recipients. We now live in a medical marketplace,3 and hear patients referred to by administrators as customers or by insurers as covered lives. How physicians manage



CHAPTER 4 Professionalism in Pediatric Critical Care

patients in this context is of great financial import; thus it should be no surprise that many of the traits characterized as professionalism have financial implications. Professionalism has, accordingly, received renewed attention and scrutiny.

Great Paradox of the Medical Profession

















A constant tension pervades medicine where principles of selfinterest and altruism coexist.4 In most human endeavors, it is considered appropriate to identify one’s own best interests and make decisions accordingly. It is in one’s best interest to obey the law, brush one’s teeth, and to work for a living. Yet, it is in the best interest of society for physicians to treat patients altruistically, whether that benefits the professional or not. The physician’s pledge to society is to be altruistic in dealing with patients, to put the patient first, before oneself. These two principles, self-interest and altruism, are polarized and often conflict. With each medical decision comes the question: “Was this for the patient or for the doctor? Whose interest did this serve?” In its simplest sense, professionalism in medicine comes down to putting the patient first. Here’s a concrete example: It is 4 am. I haven’t slept a wink and I’m hard at work in the pediatric intensive care unit (PICU) trying to finish my documentation so I can catch some shuteye. The emergency department (ED) resident calls. He has a child with a fever who looks sick (to him). Would I mind taking a look to see if he needs to come to the PICU? What should I do? 1. Go take a look. 2. Have him admitted to general pediatrics. 3. Hold him in the ED until 7 am when relief arrives for the day shift. 4. Just bring him up to the PICU. 5. Send him home; it’s just a fever. Altruism says: “Go take a look.” That would be best for the patient. It could improve the quality of the triage decision and optimize patient care. Self-interest says: “2, 3, or 5 and it’s not my problem. I’m exhausted.” The professionalism issue here is altruism, “patient first,” but look at the financial overlay. The PICU provides expensive care and is a costly resource. A decision to admit to the PICU should not be made casually. General pediatrics may not be able to safely care for the patient, and a bad outcome may mean patient suffering, additional hospital or patient expense, or a lawsuit. Don’t use the hospital if you don’t have to. It costs money. Holding the patient in the ED is a dissatisfier, will damage the hospital in the eyes of the community, and will interfere with the ED workflow. Professionalism is a larger issue than merely resolving the patient first medical paradox; the concept of professionalism received a much more thorough examination in the late 1990s and first decade of the new millennium. Many of the groups that regulate medicine on our behalf weighed in. Among them were the American Board of Internal Medicine Foundation, the American College of Physicians, the American Society of Internal Medicine, and the European Federation of Internal Medicine, all of which worked together to draft the document “Medical Professionalism in the New Millennium: A Physician Charter.”5

Professionalism, The Physician Charter The Charter adopted three principles and made 10 commitments to fulfill the medical profession’s contract with society: Principle 1: Primacy of Patient Welfare. Altruism demands that the patient’s needs be given precedence over self-interest, market forces, societal pressure, and administrative exigency.

27

Principle 2: Patient Autonomy. Physicians must be honest with their patients and, whenever possible, empower them to make informed decisions. Principle 3: Social Justice. There should be fair distribution of healthcare resources. Commitment 1: Professional Competence. Each individual physician must ensure one’s own competence, and the profession as a whole must ensure its members’ competence. Professionals are responsible for putting mechanisms in place to ensure lifelong learning, competence, and skills. Commitment 2: Honesty with Patients. Patients must be completely and honestly informed. Medical errors must be acknowledged. Mistakes must be analyzed to improve the quality of healthcare. Commitment 3: Patient Confidentiality. Patient trust demands that confidences be protected. Trust is essential to the doctorpatient (patient-doctor) relationship. Commitment 4: Appropriate Relations to Patients. Patients are inherently vulnerable. Professionalism demands that they not be exploited. Commitment 5: Improve Quality of Care. Not only must we maintain clinical competence, we must work collaboratively to reduce medical errors, increase patient safety, minimize overuse of healthcare resources, and optimize outcomes of care. Commitment 6: Improve Access to Care. Physicians must strive to reduce barriers to equitable healthcare and to foster uniform and adequate standards of care. Commitment 7: Just Distribution of Finite Resources. The physician must make wise and cost-effective use of limited resources. Commitment 8: Scientific Knowledge. Where possible, care should be evidence based. Commitment 9: Manage Conflicts of Interest. To maintain patient trust, physicians must recognize, disclose, and deal with conflicts of interest that arise in the course of their professional activities. Commitment 10: Ensure the Integrity of Professional Responsibilities. The profession must define, organize, and ensure the standards of its current and future members.

Pediatric Intensive Care Unit as a Site for Medical Education and Lifelong Learning The renaissance of interest in professionalism has fostered an endeavor to weave the topic into medical education, build it into curricula, and focus on it in coursework.6 Despite that interest, medical student altruism, social interest, and other qualities of positive social value have been noted to decline as the student progresses through medical school and the early phase of clinical training.7–11 The altruistic freshman is transformed by clinical experiences into the cynical senior. It has been argued that this growth of cynicism reflects the gap between what we say as teachers (the formal curriculum) and what we do as practitioners (the hidden curriculum).12 When we do not “walk the talk,” we plant the seeds of cynicism and nonprofessionalism. An example: Dr. Blunt and an impressionable medical student are suturing a central line in place. Their patient is in a chemically induced coma from self-medication compounded by subsequent hypoxia. His story is tragic and the student knows that the teenager deserves sympathy and respect. As Dr. Blunt ties the last knot he comments, “It would have been easier to get this line in if he

28

SECTION I



Pediatirc Critical Care: The Discipline

hadn’t been so fat.” What did the student just learn about sympathy and respect? In the PICU, doctors, students, and nurses are compressed into a very small, intense space. They are all engaged in lifelong learning as they play their separate roles in patient care. Life is, after all, an open-book test. Thus, throughout our development and maturation as professionals, we in the PICU are continuously learning professionalism and cynicism from our colleagues. The team aspect of PICU care adds a dimension to professionalism. Not all care providers will be aligned in their views of each patient’s self-interest or society’s collective interest in distributing healthcare resources. A good example is care of the dying or severely disabled child. We may not all agree on limits of care,

management of family members, or end-of-life issues. Transfer of primary responsibility at end of shift; division of responsibility among team members such as nurses, physicians, and consultants; and appropriate delegation of tasks (given our job-specific scopes of practice) all stress our ability to work together. Collaborative care necessitates that we share our ethical views among team members and include appropriate team members in the decisionmaking process. We should clean up our act in the PICU, take care that our words and actions reflect the formal curriculum, fulfill our contract with society, and behave toward our patients—and toward each other—like the professionals we, at first, set out to be. The full reference list for this chapter is available at ExpertConsult.com.

e1





9.









10.



12.





11.





8.





7.









1. WHO. Global Health Expenditure Database. 2015. Available at: http://data.worldbank.org/indicator/SH.XPD.TOTL.ZS. 2. Congressional Budget Office. Federal Spending and the Government’s Major Health Care Programs is Projected to Rise Substantially Relative to GDP. 2013. Available at: http://www.cbo.gov/publication/44582. 3. Frankford DM, Konrad TR. Responsive medical professionalism: integrating education, practice and community in a market-driven era. Acad Med. 1998;73:138-145. 4. Jonsen AR. Watching the doctor. N Engl J Med. 1983;308:1531-1535. 5. ABIM Foundation, ACP-ASIM Foundation and EFIM. Medical Professionalism in the New Millennium: a physician charter. Ann Int Med. 2002;136(3):243-246. 6. Inui TS. A Flag in the Wind: Educating for Professionalism in Medicine. Association of American Medical Colleges. 2003. Available at: https://



References

members.aamc.org/eweb/upload/A%20Flag%20in%20the%20Wind% 20Report.pdf. Wear D. Professional development of medical students: problems and promises. Acad Med. 1997;72:1056-1062. Testerman JK, Morton KR, Loo LK, et al. The natural history of cynicism in physicians. Acad Med. 1996;71:S43-45. Feudtner C, Christakis DA, Christakis NA. Do clinical clerks suffer ethical erosion? Students’ perceptions of their ethical environment and personal development. Acad Med. 1994;69:670-679. Marcus ER. Empathy, humanism, and the professionalization process of medical education. Acad Med. 1999;74(11):1211-1215. Crandall SJS, Volk RJ, Loemker V. Medical students’ attitudes toward providing care for the underserved. J Amer Med Assoc. 1993;269:2519-2523. Coulehan J. Today’s professionalism: engaging the mind but not the heart. Acad Med. 2005;80(10):892-898.

e2

Abstract: The medical profession is largely self-regulated by a system comprising state medical licensing boards, subspecialty boards, and credentialing and accrediting bodies. This system confers many benefits on its members. It is the purview of society to allow us this autonomy. To sustain these professional benefits, which include control of entry into our profession and to maintain the autonomy of our credentialing and accrediting bodies, we must honor our contracts with society. Professionalism is, in its simplest form, putting the patient first, placing altruism before

self-interest, as is expected of us. Beyond that, professionalism is a more complete charter that ties altruism to the concrete realities of the doctor-patient relationship and the marketplace in which we practice. The Physician Charter is grounded in the principles of altruism, patient autonomy, and social justice. It codifies the physician’s contract with society. Key Words: Profession, professionalism, autonomy, social justice, conflict of interest

5 Leading and Managing Change in the Pediatric Intensive Care Unit JOHN C. LIN

“If we could change ourselves, the tendencies in the world would also change. As a man changes his own nature, so does the attitude of the world change towards him. We need not wait to see what others do.” (in other words, be the change you want to see in the world.)

—Attributed to Mahatma Gandhi “And it ought to be remembered that there is nothing more difficult to take in hand, more perilous to conduct, or more uncertain in its success, than to take the lead in the introduction of a new order of things. . . . This coolness arises partly from fear . . . and partly from the incredulity of men, who do not readily believe in new things until they have had a long experience of them.”  













—Niccolò Machiavelli. The Prince, 1513. Chapter 6

PEARLS • A modern pediatric intensive care unit (PICU) faces constant pressures to implement new clinical care practices, introduce new equipment, or assimilate new systems in response to rapidly evolving healthcare regulatory, economic, and patientcentered demands while maximizing healthcare value. • To meet new challenges and advance PICU care optimally, the process of change requires a combination of leadership and management in order to develop an intentional strategy and carry out a structured yet adaptable implementation approach.

Leading a change initiative in a complex system such as a pediatric intensive care unit (PICU) poses multiple challenges that cannot be accomplished by any single person. Just as caring for a critically ill child requires interprofessional team collaboration to achieve a desired patient outcome, introducing new care initiatives also requires a deliberate plan and customized approach. Without intentional leadership and management, change initiatives too often fail to achieve the desired outcomes, create tension that undermines a unit’s morale, and are soon forgotten even when initial success occurs. This chapter uses examples from successful initiatives in healthcare, specifically from the fields of adult and pediatric critical care medicine, to review the history of change management. It also describes models and tools that have been developed and used to facilitate and sustain change.

• The PICU team can increase the likelihood of successful and sustainable change in care practices by understanding the strengths and weaknesses of existing interprofessional team function and empowering distributed leadership, personal agency, and group identity among the diverse people who comprise the PICU team. • The fields of business administration and management, dissemination and implementation science, and quality improvement offer models and tools that can guide a PICU team embarking on new initiatives.

National Change Day: A Case Study in Leading Change On March 13, 2013, the United Kingdom’s National Health Service (NHS) held a National Change Day1 in response to the 2010 Francis Report.2 This government oversight report condemned systemic and cultural failings on an organizational scale that led to “appalling and unnecessary suffering of hundreds of people [. . . who were] failed by a system which ignored the warning signs and put corporate self-interest and cost control ahead of patients and their safety.” Founded as a grassroots movement, Change Day challenged NHS employees from all positions and professions to “Do Something Better Together” and to identify a concrete change in their own work and behavior that would  





29

30

SECTION I



Pediatric Critical Care: The Discipline

impact positively on an aspect of their local patient care processes. The organizers set an initial goal of 65,000 pledges, which doubled to 130,000 by the morning of Change Day, ended the day at 182,000, and reached a total of 189,000 individual pledges by the end of March 2013.3 The NHS Change Day transformed the world’s largest health system and was subsequently awarded the Harvard Business Review/McKinsey Leaders Everywhere Challenge in September 2013. Incredibly, this movement, credited as the single “biggest ever day of collective action to improve healthcare” evolved from an impromptu conversation between a general practitioner and a lecturer speaking on “Building Contagious Commitment to Change.” This chance encounter between a conference attendee and a program lecturer evolved into a core team made up of 12 people: a pediatric resident trainee, a family practitioner, and an NHS graduate management trainee who were mentored by five improvement leaders and supported by one administrator and three social media and communications experts.4 Since then, building on this overwhelming success, the NHS holds Change Day annually. Similar movements have spread to 17 other countries. The success and spread of Change Day reflect change management principles that developed beginning in the 1960s, have evolved through the ensuing 5 decades, and have steadily been adopted in manufacturing and service industries, including healthcare. Change Day serves as a model for creating distributed leadership within a hierarchical system. It also highlights the potential for inspirational leadership combined with intentional management to design, implement, and sustain changes in behavior.

History and Development of Change Management The beginning of change management as a field of expertise began when leaders recognized the importance of addressing the psychology of change as an integral component of leading and managing change. This new field of expertise developed insights from the 1969 book On Death and Dying, by Elisabeth Kubler-Ross.5 This seminal book described how terminally ill patients and their loved ones reacted to a health-related event that resulted in the personal loss of a previously held image of both present and future selves. Kubler-Ross proposed five stages of grief: denial, anger, bargaining, depression, and acceptance. Adopting the psychological insights from the stages outlined by Kubler-Ross to address the common negative emotions triggered by an event leading to the experience of loss of current roles, changes to work environment or status, and a shift from the “old way,”6 initial change management models emphasized the importance of approaching change with more than just an idea and a plan. By the 1980s, Julien Phillips at McKinsey & Company proposed three critical components for successful organizational change: (1) new strategic vision, (2) new organizational skills/capabilities, and (3) political support. Phillips further described that these three core factors would have to carry out four sequential and often overlapping phases of the change process: (1) creating a sense of concern, (2) developing a specific commitment to change, (3) pushing for major change, and (4) reinforcing and consolidating the new course.7 Over the next 2 decades, Phillips and his contemporaries from the “big 6” accounting and consulting firms of the time helped create the change management industry. Recognizing the

importance of leveraging a crucial event to trigger Phillips’s first two phases of change, Daryl Conner coined the term “burning platform” based on the 1988 North Sea Piper oil rig fire disaster. Soon, the phrase “create a burning platform” became widely used to represent both creating a sense of urgency and establishing a commitment to change.8 Common to all of these initial change management approaches was the implied belief that organizational change could and should be accomplished using a top-down approach that inspired, convinced, cajoled, or forced frontline employees to enact and embrace the changes that leaders deemed necessary. By the turn of the century, business leaders recognized the weaknesses of using a top-down approach when trying to enact lasting change. Further, as the rapidity of change accelerated, change leaders recognized the importance of distributed leadership. This strategic shift promoted individual initiative and created a business environment in which change was encouraged and able to develop incrementally in continuous fashion rather than in large disruptive shifts that occurred in a change-stasis-change pattern. Lean management principles, originally focused on elimination of waste, shifted the emphasis of change management to creating a pattern of behavior that allowed and encouraged continuous improvement led by frontline staff that fit within the broader goal of increasing efficiency and quality. The role and importance of employee engagement in achieving sustained changes in workplace behavior was formally introduced into change models in 1996 by John Kotter in his book Leading Change. In his book, Kotter outlined an 8-step Leading Change Model (Table 5.1) that distinguished change leadership from change management, described the importance of both of these executive skills in guiding change, and highlighted the role of distributed leadership and employee empowerment in the change process.9 The pace of extrinsic forces that drive the need for change has only increased. In the preface of his 2012 edition of Leading Change, Kotter highlighted each of these points as the foundation to achieving both efficiency and quality: • Management makes a system work. It helps you do what you know how to do. Leadership builds systems or transforms old ones. It takes you into territory that is new and less well known, or even completely unknown to you. • These trends (of an ever-increasing speed of change) demand more agility and change-friendly organizations; more leadership from more people, and not just top management. • Speed of change is the driving force. Leading change competently is the only answer.9





Change Management in Healthcare In 2006, the book entitled Redefining Health Care: Creating ValueBased Competition on Results,10 introduced the concept known as “value agenda.” In this approach, the goal of increasing efficiency and quality was reframed as maximizing healthcare value for the patient. This replaced simply reducing costs, increasing market share, or improving quality. In this sense, the audaciously titled article, “The Strategy that Will Fix Health Care,” defined value as “improving outcomes that matter to patients relative to the cost of achieving those outcomes.”11 In this paradigm, maximizing healthcare quality becomes a balancing force for minimizing or shifting healthcare costs and maximizing market share. The NHS Change Day epitomized all these described principles. In change management terms, the 2010 Francis Report created the “burning platform” that galvanized not just people



CHAPTER 5 Leading and Managing Change in the Pediatric Intensive Care Unit

31

TABLE Change Leadership/Management Models 5.1

Principles 1. Unfreeze. • Determine what needs to change. • Ensure strong support from senior management. • Create the need for change. • Manage and understand doubts and concerns. 2. Change. • Communicate often. • Dispel rumors. • Empower action. • Involve people in the process. 3. Refreeze. • Anchor changes into culture. • Develop ways to sustain change. • Celebrate success.

Ronald Lippitt

Phases of Change Theory

1. 2. 3. 4. 5. 6. 7.

Identify the problem. Assess motivation, capacity, and readiness for change. Identify available resources. Define desired change. Define change agent’s role (e.g., advocate, facilitator, consultant, expert). Maintain the change. Terminate change agent’s role.

Everett Rogers

Five-Stage Change Theory (Diffusion of Innovation Theory)

1. 2. 3. 4. 5.

Knowledge: Expose individual to the new idea. Persuasion: Convince individual to adopt the new idea. Decision: Individual decides to adopt or reject the new idea. Implementation: Individual adopts the change. Confirmation: Individual accepts the change as advantageous.

John Kotter

Leading Change Model

1. 2. 3. 4. 5. 6. 7. 8.

Establish a sense of urgency. Create the guiding coalition. Develop a vision and strategy. Communicate the change vision. Empower employees for broad-based action. Generate short-term wins. Consolidate gains and produce more change. Anchor new approaches in culture.



























































































Name Theory of Planned Change



Author Kurt Lewin

within the NHS but also the public at large. The report criticized and described the systemic failures of the NHS, misguided focus on healthcare costs rather than healthcare value, and provided a multitude of examples in which the inability of the NHS to address its known flaws led to unnecessary patient suffering. This unmitigated language created an overwhelming and uniformly shared sense of urgency and concern that led to a unifying commitment for change from political leaders and the British public at large, allowing development of a guiding coalition.2,3 The inclusion of social media and communication experts among the core leadership team ensured that the vision for change was communicated broadly and consistently. Local groups were empowered to seek out input from community members and leaders about how the NHS could better serve patients’ needs and improve their healthcare value. Individuals were encouraged to each participate in their own way. As a result, a broad-based network of leaders, unified under the umbrella of the Change Day initiative, pursued and led team-specific actions and change initiatives. While this all-accepting approach allowed participants to make individual pledges as simple as “to meet and greet patients with a smile,” it also created space for city and regional health

commissions to fold their local efforts into a national movement. Subsequent analysis of the psychological factors that led to Change Day success highlighted the impact of allowing daily participation and commitment to self-initiated, small tests of change. These small successes, in turn, affirmed both personal agency and group efficacy, promoting and restoring a sense of “vocational and organizational identity.”12

Models and Tools to Facilitate Change Leadership and Management Theories of Change Others have described change theories with many similarities to Kotter’s eight-step model. Examples include Lewin’s Theory of Planned Change,13–15 Lippitt’s Phases of Change Theory,16 and Rogers’s Diffusion of Innovation Theory17 (see Table 5.1). The advantages of Kotter’s model over these four include the treatment of change as a continuous rather than a discrete event (Lewin), establishing distributed leadership and empowering frontline

32

SECTION I



Pediatric Critical Care: The Discipline

initiative and action rather than focusing on the change agent and a top-down approach (Lippitt), and description of active leadership rather than passive management and subsequent undirected diffusion of ideas and change (Rogers). Despite these advantages, Kotter’s Leading Change model lacks specific details on how to best accomplish each of the eight steps. Additionally, Kotter’s model does not include a step that assesses current attitudes and receptivity for change or a step that focuses on identifying facilitators and barriers to change. Understanding current organizational culture, behavior, biases, attitudes, and knowledge provides extremely useful guidance. Change leaders and managers increase the likelihood of success if they analyze how various aspects of the existing organization must interact with internal and external variables when implementing a specific intervention. Without this tactical step that defines how to accomplish the strategic goal, the inspirational vision remains nebulous and can be dismissed as a grand idea but too hard or even impossible to enact.

Bringing Theory to Practice The Consolidated Framework for Implementation Research (CFIR)18 provides a bridge between the inspirational vision and practical question of “How do we get there?” Proposed by Damschroder et al. in 2009 as a framework to understand adoption of new initiatives in health services, the CFIR gives a roadmap not only for accomplishing implementation research but also for achieving successful implementation of a proposed change.18 Five domains comprise the CFIR: intervention characteristics, outer setting, inner setting, individuals involved, and the implementation process (eFig. 5.1). In turn, key constructs define each of these domains and are explicitly defined to facilitate use. Within 5 years of introduction, the CFIR model was described in 26 separate publications, a testament to the framework’s success and utility when introducing change in the form of new healthcare initiatives.19 One of these studies used the CFIR model to evaluate the implementation of the ICU Liberation improvement initiative across five adult ICUs and two additional specialty care units in a single tertiary care academic medical center. The authors outlined in great detail how their adoption of the ICU Liberation initiative fit within the CFIR model and provided a “lessons learned” summary of points to consider during ICU Liberation implementation.20 By applying the CFIR to their work, this multiprofessional group of ICU specialists outlined a roadmap to guide other centers seeking to replicate the work.

Tools for Assessing Readiness for Change Kotter’s change model and the inner setting domain of the CFIR require change leaders to understand their team’s culture, readiness to accept change, capacity to absorb new information, and willingness to adopt new practice. Interprofessional team collaboration encompasses these aspects and can be assessed using several different surveys. One example, the Assessment of Interprofessional Team Collaboration Scale (AITCS), prompts participants to answer 37 questions in the three domains of partnership/shared decision-making, cooperation, and coordination in order to understand how people perceive the quality of interactions among team members during their daily work.21 During the ICU Liberation collaborative, adult and pediatric participating centers used the AITCS to assess their degree of interprofessional collaboration

before and after ICU Liberation participation.22,23 Staff responses allowed each site’s change leaders to identify specific domains in which implementation work should focus. Among the pediatric centers, the before and after responses suggested that participation in the ICU Liberation quality initiative coincided with higher AITCS domain scores.23 While the AITCS discerns team culture, it does not specifically address an organization’s openness to adopting a specific clinical care initiative. For this, the Organization Readiness for Change Assessment (ORCA) provides insight.24 Developed by the Veterans Health Administration, the ORCA asks respondents to answer questions directed to a specific change initiative. Designed to be administered as part of the preparation for introducing a new clinical practice initiative, the ORCA asks 20 questions within the three domains of evidence, context, and facilitation. The evidence domain assesses baseline understanding and perception of the strength and quality of medical evidence upon which the planned clinical practice change is based. The context domain overlaps with the AITCS questions about teamwork but attempts to draw a distinction between whether resistance to change is based on the specific proposed intervention or is a more global resistance to change of any sort. Last, the facilitation domain of the ORCA focuses on understanding how respondents perceive their leadership and management team’s ability to develop consensus, define clear roles and responsibilities, ensure adequate resources, and regularly and transparently report the impact of the change initiative on meaningful patient outcomes. When used together with the AITCS, the ORCA can further focus implementation activities and resources on expressed needs and concerns, help identify facilitating and resisting forces impacting change, and predict the likelihood of success at change implementation given the current inner setting as described by the CFIR model. Additional tools to identify individual tasks needed to enact change must also be applied within the CFIR model. Lewin’s Force Field Analysis presents one mechanism to identify the driving forces that facilitate change, the restraining forces that resist it, and the relative impact each force has in promoting or preventing movement to the new desired state (Fig. 5.2). Once identified and assigned a relative level of importance, each of these identified forces can then be analyzed further using process improvement tools. The Institute for Healthcare Improvement (IHI) has assembled an open-access 10-item quality improvement (QI) toolkit to guide healthcare teams in this work (Table 5.2).25 With this toolkit, the guiding coalition and bedside PICU team members can work with implementation facilitators to develop not just a step-by-step plan to achieve individual aims but also to determine appropriate performance measures that demonstrate the impact, success, or failure of the work in meeting the stated goals. Conducting focus groups can also be extremely useful in prioritizing issues and needs, eliciting common opinions that might facilitate or resist the change initiative, and generating new ideas on how to approach change planning. The Agency for Healthcare Research and Quality describes focus groups as “a collection of several individuals who all discuss a particular subject, voicing and discussing their opinions and ideas on that subject.” Focus groups should be led by a facilitator with specific training and experience in leading these discussions to maximize open and honest exchanges among participants while preventing off-topic conversations that derail the discussion. Exact composition, size, and number of sessions held can vary greatly depending on the

32.e1

Inner setting Individuals involved

Intervention (adapted)

Adaptable periphery

Core components

Outer setting

Core components

Adaptable periphery

Intervention (unadapted)

Process

• eFig. 5.1



 

Consolidated Framework for Implementation Research (CFIR) domains.18 Intervention: This domain refers to the characteristics of the planned intervention. A key element of this domain relates to how well the intervention fits within the current team dynamics and function. In most instances, any externally designed intervention must be adapted in some way to meet the specific characteristics and needs of the team. As the puzzle piece cutout of the figure displays, without adaptation, a poor fit occurs. Team member perception of the intervention’s legitimacy will also impact implementation success. Aspects impacting perception include whether the intervention was developed externally or internally, the expertise and reputation of the developers, the quality of supporting evidence, the applicability and advantages over other options, and the quality and comprehensiveness of the presentation. Tactical considerations will also impact acceptance of the intervention. These include whether the intervention can be adapted to meet local requirements and needs, if it can be first tested on a small scale and easily reversed if proven ineffective, and how disruptive the change would be to existing workflow. Last, the cost of intervention implementation will be important. Costs include needed time and effort, money, equipment, and opportunity costs of implementing the change. Outer setting: This domain reflects how factors external to the people carrying out the intervention will impact implementation. Factors include patient needs, perceived value to the patient, peer pressure from external competitors doing the same work, and existing organizational policies and incentives that impact the ease with which the change can be completed. Inner setting: This domain addresses the team’s structure, existing behaviors and culture, communication quality both among team members and to external groups, the ability to and readiness for change, and capacity and resource availability to implement the intervention. Individuals involved: This domain addresses characteristics of the individual members of the team. Aspects include skill and educational level, self-belief that one’s skills and knowledge are sufficient to perform the intervention, sense of personal agency and identity as a valued part of the team, and other personal character traits that impact individual response to change. Process: This domain addresses the way in which the clinical practice change and intervention are planned and executed. Factors include the engagement of formal and informal team leaders and influencers, recruitment of change champions, input from external consultants, and the transparency and quality of quantitative and qualitative reporting of the intervention’s impact on meaningful outcomes.



CHAPTER 5 Leading and Managing Change in the Pediatric Intensive Care Unit

Beginning State

Current State

33

Desired State

Driving Forces

Restraining Forces



 



CHANGE CONTINUUM Fig. 5.2 Lewin’s force field analysis. In any change process, a continuum exists defined along time or specific milestones. From a beginning to a current to a desired end state, various forces can facilitate or hinder movement along the continuum. These forces can be strong or weak (line width), continuous or intermittent (solid or dashed line), or begin or end at different points along the change continuum.18



TABLE Institute for Healthcare Improvement Quality Improvement Essentials Toolkit 5.2

Tool

Purpose

Cause and effect diagram

Identifies individual causes and their relationships that contribute to a specific outcome. Typically, five categories of causes are considered: people, environment, materials, methods, and equipment. This is also known as a fishbone diagram.

Driver diagram

Identifies the primary and secondary forces or drivers that contribute to the overall aim of the change initiative. In contrast to the Cause and Effect Diagram, the Driver Diagram develops specific interventions designed to impact behaviors or processes and permits small tests of change.

Failure modes and effects analysis

Specifically evaluates steps in the process in which adverse or undesired actions (i.e., failure modes) could occur, what causes contribute to those failures, and the potential consequences (i.e., failure effects) of those failures on the overall system.

Flowchart

Develops a visual graphic of each step in the current process to create a shared understanding among all team members. As part of the brainstorming process for designing the change initiative, this provides a valuable tool for identifying steps that create bottlenecks or that do not add value, steps in which communication breakdowns can occur, and points in which interventions identified in the Driver Diagram can be tested.

Histogram

Presents summary data and metrics in graphic form.

Pareto chart

Evaluates the frequency of individual factors that impact an overall effect in order to identify the smaller subset of factors that have the largest contribution to the end result.

Plan-do-study-act worksheet

Allows repeated evaluation of the impact of small tests of change.

Project planning form

Provides a timeline representation of each of the action steps identified in the other tools in this list.

Run chart and control chart

These two graphs provide visual presentation of metrics such as guideline compliance and discrete outcomes. After compiling at least 15 points for the Run Chart, a Control Chart allows a higher-level summary and takes into account common and uncommon variation to create an expected vs. unexpected range of variation (i.e., upper and lower control limits) in the specific metric.

Scatter diagram

Provides a graphical representation of the relationship between two variables to determine cause-and-effect relationships. The specific variables to be graphed can be selected by those identified in the Cause-and-Effect Diagram or the Failure Modes and Effects Analysis.

These tools are available at http://www.ihi.org/resources/Pages/Tools/Quality-Improvement-Essentials-Toolkit.aspx.

complexity of the questions being asked and the presence of preexisting tensions or biases among team members. Recent methodologic studies suggest that having eight participants per group is the most frequently recommended size and that 90% of identified themes and ideas occur by the sixth session.26

Tools to Implement Change While these tools play key roles in planning change and outlining specific steps, implementing change requires keen focus on communication, education, and transparency about the impact on meaningful outcomes. These aspects are crucial when considering

34

SECTION I



Pediatric Critical Care: The Discipline

the second half of Kotter’s eight-step model and the process and inner setting domains of the CFIR model. The NHS Change Day offers an example. By including three social media and communication experts in the core team, the leaders ensured frequent and consistent communication to its large workforce during the initiative. Social media platforms, community outreach initiatives, and public release of a Change Day video and website met the NHS’s goals of educating the public and garnering support at the national, local, and individual levels.1 However, communication cannot be limited to advertising that a change initiative is about to occur. In leading and managing change, communication becomes an educational initiative that provides the rationale for why change is needed and offers the evidence supporting the effectiveness of the proposed clinical practice change. Summaries of the work accomplished using the CFIR model and IHI QI tools can be used to communicate the rationale behind, urgency for, and process of change. Communicating the impact of change on meaningful outcomes allows discussion of short-term successes and becomes part of the education process. By committing to transparent discussion of the performance measures and outcomes following the change in clinical practice, frontline staff gain trust in the process and can see for themselves how the work is improving patient care. Debriefing events and open results review can be quite impactful on communication and education during a change initiative. These sessions provide an opportunity to discuss outcomes in a transparent fashion that acknowledges successes, highlights lessons learned, and outlines opportunities for improvement. The ICU-Resuscitation (ICU-RESUSC) study provides an example in the PICU setting. This 10-site US collaborative introduces postarrest interprofessional debriefing as part of an ongoing, unitwide initiative to improve cardiopulmonary resuscitation (CPR) quality. In a single center, these debriefing events have both sustained the CPR training efforts and correlated with improved post-arrest neurologic outcomes.27 By debriefing all personnel from in-unit cardiac arrests within 3 weeks of the CPR event, the authors created a forum open to all PICU staff that allowed honest dialogue about a stressful event, displayed CPR quality metrics, and reviewed relevant literature.27,28 The inclusion of open discussions of ongoing CPR training interventions and patient outcomes following CPR made the CPR QI efforts transparent. This provided a way to discuss short-term results openly and identify opportunities for improvement. Since then, this comprehensive CPR training and debriefing program is being implemented in 10 US PICUs as part of the ICU-RESUSC study evaluating post-arrest neurologic outcomes.29 As described in Kotter’s eightstep model, not only have these investigators anchored the change in their own unit, they are also introducing similar change in other tertiary care PICUs across the country. Successful change leadership and management requires a blend of educational approaches in addition to debriefing events or reporting results. Computer-based learning (CBL) has become a common component of many healthcare initiatives. Advantages of this educational platform include cost-effectiveness by decreasing the number of instructors needed to reach the target audience, increased accessibility by eliminating the time and location restrictions of traditional classroom teaching, flexibility in allowing learners to complete or review the material at their own pace or as just-in-time training, and automated tracking of completion rates among required staff. Disadvantages include inability to answer questions not covered in the material, inability to interact in real time with other students, and lack of spontaneous discussions that

promote deeper understanding. Unfortunately, the superiority of CBL compared with in-person education in achieving sustained knowledge retention has not been demonstrated in a rigorous fashion, and the best method and format of CBL modules has not been proven.30 Nevertheless, CBL has demonstrated effectiveness in discrete tasks such as arterial blood gas interpretation among ICU nurses31 and for more broad-based education in postgraduate nursing critical care courses.32 Pediatric residents working on an in-patient oncology rotation also had favorable reaction to use of CBL modules as a supplement to traditional didactic education when used in a just-in-time format during their 4-week rotation.33 Peer coaching has also been an effective component of education initiatives that helps bridge the gap between didactic or independent learning and bedside practice. As a nonevaluative partnership between colleagues, peer coaching has also been described as having identified “super-users” available during patient care to serve as a real-time resource. These super-users often undergo additional interactive education regarding the new change in practice and typically are recognized as informal leaders or individuals with particular expertise. Waddell and Dunn named the essential components of peer coaching in nursing staff development to be (1) recognizing the time-sensitive need for education or information transfer, (2) hands-on training of the new practice, (3) demonstration of competency in the new skill, (4) nonevaluative feedback, (5) opportunity for questioning and clarification, and (6) self-assessment.34 Examples of peer coaching success in inpatient and ICU settings include increased use of ceiling lifts for patient transfers35 and improved recognition of delirium in ICU patients.36

Sustaining Change The ability to sustain gains following change implementation also requires specific attention. Without intentional planning on how to anchor the new clinical practice as part of standard behavior, old habits and practices are likely to creep back. For example, a multiprofessional team of physicians, nurses, and pharmacists from one tertiary care PICU spent 6 months developing a nursedriven sedation protocol. Upon completion of the protocol, an extensive education program reached all PICU staff. These 1-hour, small-group training sessions included a review of PICU sedation literature, pharmacology review, and specific direction on use of the sedation protocol and its interface with computerized order entry and electronic medical record documentation. Peer coaching by a nurse educator with bedside nursing occurred as part of just-in-time training in order to answer questions and evaluate for appropriate protocol application. Pharmacy staff took responsibility for reviewing accuracy of medication ordering and titration and performed daily audits of protocol compliance. In a subsequent analysis of consecutive PICU admissions requiring invasive mechanical ventilation 1 year before and 1 year after sedation protocol implementation, the group observed a significant decrease in duration of lorazepam and morphine exposure with a trend toward decreased duration of mechanical ventilation and length of PICU stay.37 However, within 3 years of this observed success, resources dedicated to protocol education, monitoring, and communication were redistributed to other initiatives. Consequently, all of the observed improvements in the year following protocol implementation had completely reverted back to preimplementation ranges for duration of sedation exposure, duration of mechanical ventilation, and length of PICU stay.38 The



CHAPTER 5 Leading and Managing Change in the Pediatric Intensive Care Unit

accompanying editorial identified the importance of postimplementation monitoring of protocol adherence and ongoing review of protocol impact on identified metrics. The editorial concluded with a call to develop “structural methods for evaluation of sustainability.”39 One such tool is currently under development: the Clinical Sustainability Assessment Tool (CSAT), which followed the success of the Program Sustainability Assessment Tool (PSAT), freely available at https://sustaintool.org. The PSAT tool was designed for, validated in, and successfully used by public health programs and has had wide use since introduction.40–42 In contrast, the CSAT is specifically designed for use in clinical medicine. Designed to be completed by all involved medical care team members either as part of the change planning phase or as an assessment of an existing practice, the CSAT asks respondents to identify and rank resources, attitudes, and personnel across seven domains encompassing team, system infrastructure, and administrative characteristics that would be most important to ensuring sustainability of the specific clinical initiative being implemented. With input from clinical care specialists from multiple professions—including adult- and pediatric-based practices, inpatient-based and outpatient-based locations, medical and surgical specialties, and implementation scientists—the CSAT tool has undergone pilot testing and is now awaiting validation.43

Conclusion Change leaders face a daunting task. A diverse group of individuals comprise the PICU team. This team functions as part of a larger hospital or academic system that is navigating the everchanging landscape of pediatric healthcare. Change leaders and managers can easily be overwhelmed with a feeling of dread and futility. As Machiavelli wrote centuries ago, human nature resists change. To be successful, the change process must start with an intentional strategy that combines inspirational leadership to provide the guiding vision that triggers a positive visceral response

35

with concerted management that adheres to a structured yet adaptive tactical approach. All of this must include a strategy for sustaining desired change over time. Change in systems as complex as the PICU and healthcare must create a sense of belonging and group identity, must foster and support distributed leadership, and must demonstrate ongoing patient value by balancing the ideals of providing the highest quality care and the pragmatic reality of rising healthcare costs and limited resources.

Key References Balas MC, Burke WJ, Gannon D, et al. Implementing the awakening and breathing coordination, delirium monitoring/management, and early exercise/mobility bundle into everyday care: opportunities, challenges, and lessons learned for implementing the ICU Pain, Agitation, and Delirium Guidelines. Crit Care Med. 2013;41(9 Suppl 1): S116-127. Damschroder LJ, Aron DC, Keith RE, Kirsh SR, Alexander JA, Lowery JC. Fostering implementation of health services research findings into practice: a consolidated framework for advancing implementation science. Implement Sci. 2009;4:50. Kirk MA, Kelley C, Yankey N, Birken SA, Abadie B, Damschroder L. A systematic review of the use of the Consolidated framework for implementation research. Implement Sci. 2016;11:72. Kotter JP. Leading Change. Boston: Harvard Business Press; 2012. NHS. Leaders Everywhere: The Story of NHS Change Day. A learning report 2013. https://www.slideshare.net/NHSIQ/the-story-of-change-day?from_ action5save. Porter ME, Lee TH. The strategy that will fix health care. Harv Bus Rev. 2013;91(10):50-70. Yaghmai BF, Di Gennaro JL, Irby GA, Deeter KH, Zimmerman JJ. A pediatric sedation protocol for mechanically ventilated patients requires sustenance beyond implementation. Pediatr Crit Care Med. 2016;17(8):721-726.

The full reference list for this chapter is available at ExpertConsult.com.

e1



































































































1. National Health Service 2013 Change Day Video. https://www. youtube.com/watch?v5h7A9rohysZw. Accessed April 17, 2019. 2. Francis R. Press Statement. Report of the Mid Staffordshire NHS Foundation Trust Public Inquiry: Chairman’s Statement. London, United Kingdom: Stationery Office; 2013. 3. NHS. Leaders Everywhere: The Story of NHS Change Day. A learning report 2013. https://www.slideshare.net/NHSIQ/the-story-of-changeday?from_action5save. 4. Bevan H. Biggest Ever Day of Collective Action to Improve Healthcare that Started with a Tweet. https://www.mixprize.org/story/biggestever-day-collective-action-improve-healthcare-started-tweet-0. 5. Kubler-Ross E. On Death and Dying. New York, NY: Macmillan Publishing Company; 1969. 6. Scheck CL, Kinikci AJ. Identifying the antecedents of coping with an organizational acquisition: a structural assessment. J Organiz Behav. 2000;21:627-648. 7. Phillips JR. Enhancing the effectiveness of organizational change management. Human Resource Management. 1983;22(1/2):183-199. 8. Conner DR. Managing at the Speed of Change: How Resilient Managers Succeed and Prosper Where Others Fail. New York, NY: Random House; 1992. 9. Kotter JP. Leading Change. Boston, MA: Harvard Business Press; 2012. 10. Porter ME, Teisberg EO. Redefining Health Care: Creating Value-Based Competition on Results. Boston, MA: Harvard Business Press; 2006. 11. Porter ME, Lee TH. The strategy that will fix health care. Harv Bus Rev. 2013;91(10):50-70. 12. Moskovitz L, Garcia-Lorenzo L. Changing the NHS a day at a time: the role of enactment in the mobilisation and prefiguration of change. J Soc Polit Psychol. 2016;4(1):196-219. 13. Lewin K. Frontiers in group dynamics. In: Cartwright D, ed. Field Theory in Social Science. London: Social Science Paperbacks; 1947. 14. Burnes B. Kurt Lewin and the planned approach to change: a reappraisal. J Manage Stud. 2004;41(6):977-1002. 15. MindTools. Lewin’s Change Management Model: Understanding the Three Stages of Change. https://www.mindtools.com/pages/article/ newPPM_94.htm. 16. Lippitt R, Watson J, Westley B. The Dynamics of Planned Change: a comparative study of principles and techniques. New York: Harcourt, Brace, & World, Inc.; 1958. 17. Rogers EM, Shoemaker FF. Commuincation of Innovations: A CrossCultural Approach. New York: Free Press; 1971. 18. Damschroder LJ, Aron DC, Keith RE, Kirsh SR, Alexander JA, Lowery JC. Fostering implementation of health services research findings into practice: a consolidated framework for advancing implementation science. Implement Sci. 2009;4:50. 19. Kirk MA, Kelley C, Yankey N, Birken SA, Abadie B, Damschroder L. A systematic review of the use of the consolidated framework for implementation research. Implement Sci. 2016;11:72. 20. Balas MC, Burke WJ, Gannon D, et al. Implementing the awakening and breathing coordination, delirium monitoring/management, and early exercise/mobility bundle into everyday care: opportunities, challenges, and lessons learned for implementing the ICU Pain, Agitation, and Delirium Guidelines. Crit Care Med. 2013;41(9 Suppl 1): S116-127. 21. Orchard CA, King GA, Khalili H, Bezzina MB. Assessment of Interprofessional Team Collaboration Scale (AITCS): development and testing of the instrument. J Contin Educ Health Prof. 2012;32(1):58-67. 22. Pun BT, Balas MC, Barnes-Daly MA, et al. Caring for critically Ill patients with the ABCDEF bundle: results of the ICU liberation collaborative in over 15,000 adults. Crit Care Med. 2019;47(1): 3-14.

23. Arteaga G, Kawai Y, Rowekamp D, et al. Bundling the bundles: can we change culture with a holistic approach to patient care in the ICU? Crit Care Med. 2018;46(suppl1):629. 24. Helfrich CD, Li YF, Sharp ND, Sales AE. Organizational readiness to change assessment (ORCA): development of an instrument based on the Promoting Action on Research in Health Services (PARIHS) framework. Implement Sci. 2009;4:38. 25. QI Essentials Toolkit. http://www.ihi.org/resources/Pages/Tools/ Quality-Improvement-Essentials-Toolkit.aspx. 26. Guest G, Namey E, McKenna K. How many focus groups are enough? Building an evidence base for nonprobability sample sizes. Field Methods. 2017;29(1):3-22. 27. Wolfe H, Zebuhr C, Topjian AA, et al. Interdisciplinary ICU cardiac arrest debriefing improves survival outcomes. Crit Care Med. 2014;42(7):1688-1695. 28. Zebuhr C, Sutton RM, Morrison W, et al. Evaluation of quantitative debriefing after pediatric cardiac arrest. Resuscitation. 2012;83(9):11241128. 29. Reeder RW, Girling A, Wolfe H, et al. Improving outcomes after pediatric cardiac arrest: the ICU-Resuscitation Project: study protocol for a randomized controlled trial. Trials. 2018;19(1):213. 30. Taveira-Gomes T, Ferreira P, Taveira-Gomes I, Severo M, Ferreira MA. What are we looking for in computer-based learning interventions in medical education? A systematic review. J Med Internet Res. 2016;18(8):e204. 31. Schneiderman J, Corbridge S, Zerwic JJ. Demonstrating the effectiveness of an online, computer-based learning module for arterial blood gas analysis. Clin Nurse Spec. 2009;23(3):151-155. 32. Patel R. Evaluation and assessment of the online postgraduate critical care nursing course. Stud Health Technol Inform. 2007;129(Pt 2):1377-1381. 33. Mangum R, Lazar J, Rose MJ, Mahan JD, Reed S. Exploring the value of just-in-time teaching as a supplemental tool to traditional resident education on a busy inpatient pediatrics rotation. Acad Pediatr. 2017;17(6):589-592. 34. Waddell DL, Dunn N. Peer coaching: the next step in staff development. J Contin Educ Nurs. 2005;36(2):84-89; quiz 90-81. 35. Alamgir H, Drebit S, Li HG, Kidd C, Tam H, Fast C. Peer coaching and mentoring: a new model of educational intervention for safe patient handling in health care. Am J Ind Med. 2011;54(8):609-617. 36. Gordon SJ, Melillo KD, Nannini A, Lakatos BE. Bedside coaching to improve nurses’ recognition of delirium. J Neurosci Nurs. 2013; 45(5):288-293. 37. Deeter KH, King MA, Ridling D, Irby GL, Lynn AM, Zimmerman JJ. Successful implementation of a pediatric sedation protocol for mechanically ventilated patients. Crit Care Med. 2011;39 (4):683-688. 38. Yaghmai BF, Di Gennaro JL, Irby GA, Deeter KH, Zimmerman JJ. A pediatric sedation protocol for mechanically ventilated patients requires sustenance beyond implementation. Pediatr Crit Care Med. 2016;17(8):721-726. 39. Ista E, van Dijk M. How to sustain quality improvements in sedation practice? Pediatr Crit Care Med. 2016;17(8):792-794. 40. Schell SF, Luke DA, Schooley MW, et al. Public health program capacity for sustainability: a new framework. Implement Sci. 2013;8:15. 41. Luke DA, Calhoun A, Robichaux CB, Elliott MB, Moreland-Russell S. The Program Sustainability Assessment Tool: a new instrument for public health programs. Prev Chronic Dis. 2014;11:130184. 42. Calhoun A, Mainor A, Moreland-Russell S, Maier RC, Brossart L, Luke DA. Using the Program Sustainability Assessment Tool to assess and plan for sustainability. Prev Chronic Dis. 2014;11:130185. 43. Luke DA, Malone S, Prewitt K, Hackett R, Lin JC. The clinical sustainability assessment tool (CSAT): Assessing sustainability in clinical medicine settings. 11th Annual Conference on the Science of Dissemination and Implementation in Health. Washington D.C. December 2018.

References

e2

Abstract: A modern pediatric intensive care unit (PICU) faces constant pressures to implement new clinical care practices, introduce new equipment, or assimilate new systems in response to rapidly evolving healthcare regulatory, economic, and patientcentered demands while maximizing healthcare value. To meet new challenges and advance PICU care optimally, the process of change requires a combination of leadership and management in order to develop an intentional strategy and carry out a structured yet adaptable implementation approach. The PICU team can increase the likelihood of successful and sustainable change in care practices by understanding the strengths and weaknesses of

existing interprofessional team function and empowering distributed leadership, personal agency, and group identity among the diverse people who comprise the PICU team. The fields of business administration and management, dissemination and implementation science, and quality improvement offer models and tools that can guide a PICU team embarking on new initiatives. Key words: change leadership, change management, interprofessional team, consolidated framework for implementation research, quality and process improvement, sustainability

10 6 Chapter Title Evolution of Critical Care Nursing CHAPTERR.AUTHOR LAUREN SORCE AND RUTH LEBET

PEARLS • • •











To gain basic of the development of theintensive eye. As part of theknowledge multiprofessional team of dedicated To develop understanding at famicare experts,essential nurses are pivotal in thehow careabnormalities of children and various stages of development can arrest or hamper normal lies during critical illness. formationa of the ocularenvironment structures and visual pathways. Building humanistic that endorses parents as unique individuals capable of providing essential elements of care to their children constitutes the foundation for familycentered care. Caring practices include a constellation of nursing activities responsive to the uniqueness of the patient/family and create a compassionate and therapeutic environment with the aim of promoting comfort and preventing suffering.

Pediatric critical care nursing has evolved tremendously over the years. The nurse plays a vitally important role in the pediatric intensive care unit (PICU) by fostering an environment in which critically unstable, highly vulnerable infants and children benefit from vigilant care and the highly coordinated actions of a skilled team of patient-focused healthcare professionals. Pediatric critical care nursing practice encompasses staff nurses who provide direct patient care, nursing leaders and clinical nurse specialists who facilitate an environment of excellence, professional staff development that ensures continued nursing competence and professional growth, acute care pediatric nurse practitioners who manage patients as providers and contribute to staff nurse professional growth, and nurse scientists who generate knowledge to support the practice of pediatric critical care nursing. This chapter discusses the evolution of pediatric critical care nursing as well as the current framework for PICU nursing practice.

Early Pediatric Critical Care Nursing The evolution of critical care dates to the days of the Crimean War when Florence Nightingale grouped the sickest patients in a cohort so that they could be more closely observed. The first PICU was opened in 1955 in Sweden with seven acute care beds and five stepdown beds (see also Chapter 1). While others followed in Europe and Australia, the first multiprofessional PICU in the United States was opened in 1967 by Dr. John J. Downes at the Children’s Hospital of Philadelphia.1 This PICU was fully equipped with monitoring and required devices for six beds. Although critically ill children had been previously studied in a 36













To acquire adequate information about anatomy of a Excellence in a pediatric critical care unitnormal is achieved through the eye and related structures andisdevelop a strong foundation combination of many factors and highly dependent on a for the understanding of common ocular problems and healthy work environment as well as training beyond thetheir techconsequences. nical requirements of the nursing role. Research has demonstrated that better patient outcomes are achieved when nurses are educated at the baccalaureate level and have specialty certification. A successful critical care professional advancement program recognizes varying levels of clinical nurse knowledge and expertise and fosters advancement through a wide range of clinical learning and professional development experiences.

cohort as a result of acute poliomyelitis outbreaks, this PICU was the first unit in the United States to care for critically ill children with a variety of diagnoses. Over the next 4 years, three additional PICUs opened on the East Coast. With the expansion of pediatric critical care medicine, the need for specialty trained nurses became vital for the care of these complex pediatric patients. Nursing care in early PICUs focused on close observation with limited technology, primarily basic ventilators, arterial and central venous lines and simple intracranial pressure monitoring devices (Fig. 6.1). As the discipline has evolved, PICU nurses have learned to manage and monitor increasingly complex technology, including multiple types of ventilators, invasive lines, cerebral monitors, renal replacement therapy, circulatory assist devices, extracorporeal circulatory membranous oxygenation, and electronic medical records (Fig. 6.2). The complexity of these systems increases nurses’ mental workload and results in the need for a highly skilled PICU nursing workforce. In order to manage multiple competing priorities, safety technologies have been developed supporting the safe provision of nursing care and quality outcomes.

Describing What Nurses Do: The Synergy Model The Synergy Model (Table 6.1) describes nursing practice based on the needs and characteristics of patients and their families.2 The fundamental premise of this model is that patient characteristics



CHAPTER 6 Evolution of Critical Care Nursing

37

drive required nurse competencies. When patient characteristics and nurse competencies match and synergize, optimal patient outcomes result. The major components of the Synergy Model encompass patient characteristics of concern to nurses, nurse competencies important to the patient, and patient outcomes that result when patient characteristics and nurse competencies are in synergy. A detailed description of the Synergy Model can be found at the American Association of Critical-Care Nurses (AACN) website.3

Patient Characteristics of Concern to Nurses

• Fig. 6.1



​Nursing care in early pediatric intensive care units focused on close observation and limited technology, primarily basic ventilators, arterial and central venous lines, and simple intracranial pressure monitoring devices. (From The Alan Mason Chesney Medical Archives of The Johns Hopkins Medical Institutions.)

All patients and family members uniquely manifest the following characteristics during the PICU experience. These characteristics— stability, complexity, predictability, resiliency, vulnerability, participation in decision-making, participation in care, and resource availability—span the continuum of health and illness. Each characteristic is operationally defined as follows. Stability refers to the person’s ability to maintain a steady state. Complexity is the intricate entanglement of two or more systems (e.g., physiologic, family, therapeutic). Predictability is a summative patient characteristic that allows the nurse to expect a certain trajectory of illness. Resiliency is the patient’s capacity to return to a restorative level of functioning using compensatory and coping mechanisms. Vulnerability refers to an individual’s susceptibility to actual or potential stressors that may adversely affect outcomes. Participation in decision-making and participation in care are the extents to which the patient and family engage in decision-making and in aspects of care, respectively. Resource availability refers to resources that the patient, family, and community bring to a care situation and include personal, psychosocial, technical, and fiscal resources. This classification system allows nurses to have a common language to describe patients that is meaningful to all care areas. Each of these eight characteristics forms a continuum, and individuals fluctuate around different points along each continuum. For example, in the case of the critically ill infant in multisystem organ failure, stability can range from high to low, complexity from atypical to typical, predictability from uncertain to certain, resiliency from minimal reserves to generous reserves, vulnerability from susceptible to safe, family participation in decision-making and care from no capacity to full capacity, and resource availability from minimal to extensive. Compared with existing patient classification systems, which are primarily based on the number of therapies and procedures, these eight dimensions better describe the needs of patients that are of concern to nurses.

Nurse Competencies Important to Patients and Families

• Fig. 6.2



​Pediatric intensive care unit nurses have learned to manage and monitor increasingly complex technology, including multiple types of ventilators, invasive lines, cerebral monitors, renal replacement therapy, circulatory assist devices, extracorporeal circulatory membranous oxygenation, and electronic medical records.

Nursing competencies, which are derived from the needs of patients, also are described in terms of essential continua: clinical judgment, clinical inquiry, caring practices, response to diversity, advocacy/moral agency, facilitation of learning, collaboration, and systems thinking. Clinical judgment is clinical reasoning that includes clinical decision-making, critical thinking, and a global grasp of the situation coupled with nursing skills acquired through a process of integrating formal and experiential knowledge. Clinical inquiry is the ongoing process of questioning and evaluating practice, providing informed practice based on available data, and innovating through research and experiential learning. The nurse engages in clinical knowledge development to promote the best patient outcomes.

38

SECTION I



Pediatric Critical Care: The Discipline

TABLE Synergy Model Nurse Competencies, Expanded 6.1

Nurse Competency

Activities

Supports

Clinical Judgment: Skilled clinical knowledge, use of discretionary judgment, and the ability to integrate complex multisystem data and understand the expected trajectory of illness and human response to critical illness

• • • • •

As nurses develop their knowledge base and skill set, they move from novice to expert.











Anticipate the needs of patients Predict patient’s trajectory of illness Forecast patient’s level of recovery Prevent untoward effects and complications Facilitate safe passage for patients and families through critical illness • Help patient and family move toward a greater level of self-awareness, knowledge, or health • Transition through the acute care environment or stressful events • Peaceful death





Clinical Inquiry: Studying the clinical effectiveness of care and how it influences patient outcomes

• Optimizes the delivery of evidence-based care • Provides information that helps balance cost and quality • CPGs are driven by patient needs and provide evidence linking interventions to patient outcomes • Eliminate interventions that are steeped in tradition and opinion but do not actually benefit patients







1. Quality improvement methods use multidisciplinary teams working together to help systems operate in a way that promotes the best interests of patient care. 2. Collaborative practice groups work with CPGs to initiate evidence-based and expert consensus-based interventions. 3. CPGs are patient-centered multidisciplinary, multidimensional plans of care driving evidence-based practice that improve the process of care delivery.





Caring Practices: Activities that are meaningful to the patient and family and enhance their feelings that the healthcare team cares about them

• Bring clinical judgment into view • Vigilance: alert and constant watchfulness, attentiveness, and reassuring presence • Essential to limit the complications associated with a patient’s vulnerabilities2 • Coordinate the patient’s and family’s experiences by continuous attention to the person who exists underneath all of the advanced technology that is employed. • Near-continuous presence with patients, unique to the profession of nursing2 • Preserve the patient’s humanness through activities such as surrounding patients with their possessions and favorite music, talking with and orienting unresponsive patients and teaching this process to family members, facilitating interaction with their critically ill loved one. • Integrating family-centered care into the practice of critical care • Building a humanistic environment endorsing parents as unique individuals capable of providing essential elements of care to their children. Pediatric critical care nurses have gone beyond the identification of family needs to illustrating interventions that patients and families find helpful2 and providing families with what they need to help their child. Parents believe the most important contribution pediatric critical care nurses make is to serve as the interpreter by translating their critically ill child’s responses to others within the PICU environment.











1. Families equate caring behaviors with competent behaviors. 2. Families trust that nurses will be vigilant. 3. Steady attention can make an important difference by helping patients and their families better tolerate the experience of critical illness. 4. Nursing research ascertains that parents a. have the need for hope, information, and proximity b. must believe that their loved one is receiving the best care possible c. seek the opportunity to be helpful, to be recognized as important, and to talk with other parents who have similar issues































Response to Diversity: Honors the differences that exist among people and individuals

• Requires that care be delivered in a nonjudgmental, nondiscriminatory manner

1. Effective communication with patients and families at their level of understanding may require customizing the healthcare culture to meet the diverse needs and strengths of families. 2. Skilled nurses foresee differences and beliefs within the team and negotiate consensus in the best interest of the patient and family.









CHAPTER 6 Evolution of Critical Care Nursing

TABLE Synergy Model Nurse Competencies, Expanded—cont’d 6.1

Facilitation of Learning: Ensure that patients and their families become knowledgeable about the healthcare system and make informed choices

• Employ teaching as a continuous process that involves helping the patient and family understand the critical care environment and therapies involved in critical care. • Reinforce the patient’s experience and how, most likely, the infant or child will cope with the ICU experience.

Education provides patients with the capacity to help themselves manage the experience and for parents to help their infants and children.

Collaboration

• Hospitals with good collaboration and a lower mortality rate had a comprehensive nursing educational support program that included a clinical nurse specialist and clinical protocols that staff nurses can independently initiate. • Studies examining the relationship between nursephysician collaboration and adverse patient outcomes (falls, hospital-acquired pressure ulcers, and the development of hospital-acquired infections in critically ill adults) demonstrate that nurse-physician collaboration was inversely related to the incidence of falls, hospitalacquired pressure ulcers, ventilator-associated pneumonia, and central line–associated infections.13 • Donovan and colleagues58 reviewed the quality improvement literature specific to critical care and found a large body of evidence demonstrating that patient outcomes are improved when care is provided by a collaborative interdisciplinary team and that nurses are key team members. • Knaus and associates12 found an inverse relationship between actual and predicted patient mortality and the degree of interaction and coordination of multidisciplinary intensive care teams.

Collaboration requires commitment by the entire multidisciplinary team.

Systems Thinking: Ability to understand and effectively manipulate the complicated relationships involved in complex problem solving

• Design, implement, and evaluate whole programs of care. • Manage units. • Determine whether healthcare system is meeting patient needs.57 • Create a safe environment. • Help patients make transitions between elements of the healthcare system using systems knowledge and intradisciplinary collaboration.

1. 2. 3. 4. 5.













CPGs, Clinical practice guidelines.





































Supports 1. The holistic view of the patient that nurses often possess is a reflection of moral awareness. 2. Including family members during pediatric resuscitation is not a universal practice. A systematic review of family presence during resuscitation in the PICU supports the belief that parents who are able to be present are better able to adjust to their child’s death and better able to cope.55 Parents who were not able to stay described more anguish. 3. Local guidelines and education have been developed to facilitate parental presence during resuscitation. Importantly, physicians and nurses report increased comfort with parental presence when they, the professionals, are prepared to help support parent presence.56

Activities • Acknowledges the particular trust inherent within nurse-patient relationships • When a cure is no longer possible, nurses turn their focus to ensuring that death occurs with dignity and comfort.54 • Supports the practice of family presence during procedures and resuscitation

Nurse Competency Advocacy/Moral Agency: Speaking on the patient’s behalf in an effort to preserve a patient’s lifeworld53

Patient-centered culture Strong leadership Continuous multidisciplinary communication Collaborative problem solving Conflict management29

39

40

SECTION I



Pediatric Critical Care: The Discipline

Caring practices are a constellation of nursing activities that are responsive to the uniqueness of the patient/family and create a compassionate and therapeutic environment with the aim of promoting comfort and preventing suffering. Caring behaviors include vigilance, engagement, and responsiveness. Response to diversity is the sensitivity to recognize, appreciate, and incorporate patient- and family-specific differences into the provision of care. Differences may include individuality, cultural practices, spiritual beliefs, gender, race, ethnicity, disability, family configuration, lifestyle, socioeconomic status, age, values, and alternative care practices involving patients/families and members of the healthcare team. Advocacy/ moral agency is defined as working on another’s behalf and representing the concerns of the patient, family, and community. For example, the nurse serves as a moral agent in identifying and helping to resolve ethical and clinical concerns within the clinical setting. Facilitation of learning is the ability to use the process of providing care as an opportunity to enhance the patient’s and family’s understanding of the disease process, its treatment, and its likely impact on the child and family. Collaboration is working with others (e.g., patients, families, and healthcare providers) in a way that promotes and encourages each person’s contributions toward achieving optimal and realistic patient goals. Collaboration involves intradisciplinary and interdisciplinary work with colleagues. Systems thinking is appreciating the care environment from a perspective that recognizes the holistic interrelationships that exist within and across healthcare systems. These competencies illustrate a dynamic integration of knowledge, skills, experience, and attitudes needed to meet patients’ needs and optimize patient outcomes. Nurses require competence within each domain at a level that meets the needs of their patient population. Logically, more compromised patients have more severe or complex needs; this, in turn, requires the nurse to possess a higher level of knowledge and skill in an associated continuum. For example, if a patient is stable but unpredictable, minimally resilient, and vulnerable, primary competencies of the nurse center on clinical judgment and caring practices (including vigilance). If a patient is vulnerable, unable to participate in decision-making and care, and has inadequate resource availability, the primary competencies of the nurse focus on advocacy/moral agency, collaboration, and systems thinking. Although all eight competencies are essential for contemporary nursing practice, each assumes more or less importance depending on a patient’s characteristics. Optimal care is most likely when there is a match between patient needs and characteristics and nurse competencies. Table 6.1 provides further detail on each nurse competency.

Optimal Patient Outcomes According to the Synergy Model, optimal patient outcomes result when patient characteristics and nurse competencies synergize. A nurse-sensitive outcome, a term first coined by Johnson and McCloskey,4 defines a dynamic patient or family caregiver state, condition, or perception that is responsive to nursing interventions. Brooten and Naylor5 noted, “The current search for ‘nursesensitive patient outcomes’ should be tempered in the reality that nurses do not care for patients in isolation and patients do not exist in isolation.”

Patient-Level Outcomes Major patient-level outcomes of concern to pediatric critical care nurses include the presence or absence of complications and mortality. Outcomes related to limiting iatrogenic injury and complications of therapy demonstrate the potential hazards present in

illness and in the critical care environment. Odds of postoperative complications in pediatric cardiac surgery patients are reduced in units with a greater percentage of nurses with Bachelor of Science degrees and in hospitals with a greater percentage of nurses with Critical Care Registered Nurse certification.6,7 Furthermore, mortality rates are reduced in units with a greater proportion of nurses with more than 2 years of experience.6 Odds of patient death decreases in PICUs where critical care nurses have 11 or more years of experience. In contrast, in units with 20% or more of nurses having 2 years or less experience, the odds of death increased.8 Patient and family satisfaction ratings are subjective measures of health or the quality of health services. Patient satisfaction measures involving nursing care typically include technical and professional factors, trusting relationships, and education experiences. Patient-perceived functional status and quality of life are multidisciplinary outcome measures.9,10 Linking patient satisfaction, functional status, and quality of life is important because the three factors are often related.

Provider-Level and System-Level Outcomes Provider-level and system-level outcomes may be intertwined and difficult to isolate. It is known that nurse-physician collaboration and positive interaction are associated with lower mortality rates, high patient satisfaction with care, and low hospital-acquired infections.11–13 Clear and effective communication between physicians and nurses is positively correlated with collaborative practice.14 Furthermore, collaborative practice within the team improves the quality of care delivered and decreases burnout.15 Hospitals that decreased burnout by 30% had a reduction in healthcare-associated infections (urinary tract and surgical site infections) with an annual savings of $68 million.16 In the absence of collaborative practice and team communication, there is an indirect relationship to increased hospital associated infections.17 Nightingale Metrics One population-specific approach to measurement of nurse-sensitive outcomes is the Nightingale Metrics program.18 This program was developed so that bedside nurses could be actively involved in identifying nurse-sensitive metrics important to their unique patient and family practice. Nurses give care in an environment that should support the capacity of the patient and family to heal. In addition to supportive care, a large aspect of nursing is preventive care that often is not measured; thus care is often invisible. When measuring outcomes, it is important to account for the invisible aspects of nursing that have a tremendous impact on patients. This might include steps taken, according to the best understanding of what works, to prevent a specific complication. For example, invisible are the large numbers of pressure ulcers that never develop because of good nursing care. The Nightingale Metrics reflect unit-specific current standards of care, are based on evidence, are measurable, and reflect concerns specific to nurses working in a specific setting (Box 6.1).

Leadership Excellence in a pediatric critical care unit is achieved through a combination of many factors and is highly dependent on effective leadership.19 Numerous studies have demonstrated the importance of leadership in creating an environment where both nurses and patients can flourish. Specialized units such as PICUs must have staff with the expert knowledge and skill required to meet the multifaceted needs of patients and families. A healthy work environment should improve



CHAPTER 6 Evolution of Critical Care Nursing

• BOX 6.1











• • • • •





• •









a

Pediatric Intensive Care Unit: Example of Nightingale Metricsa

State Behavioral Scale scores every 4 hours In patients with a central venous line, changing the dressing every 7 days Development of an enteral feeding guideline Mouth care every 4 hours Venous thromboembolism: risk factors, central line removal, prophylactic medications Parental presence Pressure ulcer bundle: If patient is immobile, documentation of position change every 2 hours and positioning of heels off the bed; if not on bed rest, documentation of patient being out of bed or held in parent’s or nurses’ arms in previous 24 hours Ventilator-associated pneumonia bundle: head of bed elevation at 30 to 45 degrees; documentation of oral hygiene twice in 24 hours; peptic ulcer prophylaxis (in patients not receiving tube feedings); discussion of extubation readiness test on rounds; daily holiday from sedation or chemical paralysis “Time to critical intervention”: response to panic laboratory value, the time intervals from sending specimen to laboratory to first intervention to correct laboratory value

Metrics developed for unit-specific needs.

retention and recruitment. An evidence-based practice working group at one facility piloted several leadership proposals to enrich the nursing work environment. The criteria instituted by the AACN Standards for Establishing and Sustaining Healthy Work Environments—which include skilled communication, true collaboration, effective decision-making, appropriate staffing, meaningful recognition, and authentic leadership—were the basis for the proposals. When these standards were integrated with qualities of the staff— such as clinical proficiency, personal values, and management experience—the results showed improvement in absenteeism, patient and staff satisfaction, and nursing quality indicators.20 The literature demonstrates that an established and proficient workforce improves patient outcomes. A study conducted by Aiken and colleagues21 observed the effect of nurse staffing levels on patient outcomes and factors affecting nurse retention. A total of 10,184 nurses from 168 hospitals were surveyed. After adjusting for patient and hospital characteristics, each additional patient per nurse was associated with a 7% increase in the likelihood of dying within 30 days of admission and a 7% increase in the odds of failure to rescue (death subsequent to a complication that develops during the hospital stay). In addition, after adjusting for nurse and hospital characteristics, each additional patient per nurse was associated with a 23% increase in the odds of burnout and a 15% increase in the odds of job dissatisfaction. Aiken and colleagues22 have continued their work by assessing the net effects of work environments on nurse and patient outcomes. Using data from the same hospitals and nurses, they investigated whether better work environments were related to lower patient mortality and better nurse outcomes independent of nurse staffing and the education of the registered nurse workforce in hospitals. Work environments were evaluated according to the practice environment scales of the Nursing Work Index. Three of the five subscales studied were nursing foundations for quality of care; nurse manager ability, leadership, and support; and collegial registered nurse/physician relationships. Outcomes studied included job satisfaction, burnout, intent to leave, quality of care, mortality, and failure to rescue. They found that a greater percentage of nurses working in hospitals with unsupportive care

41

environments reported higher burnout levels and dissatisfaction with jobs. They also found that work environment had a significant effect on nurses’ plans to leave their units. When all patient and nurse factors were considered, the likelihood of patients dying within 30 days of admission was 14% lower in hospitals with healthier care environments. These findings support the observation that nursing leaders have at least three major opportunities to boost nurse retention and patient outcomes. These opportunities include increasing nurse staffing, using a more highly educated nurse workforce, and enhancing the work environment. Work conducted by the same investigators validated their previous findings. An observational study using discharge data for 422,730 patients aged 50 years or older who underwent common surgeries in 300 hospitals in nine European countries demonstrated that an increase in a nurse’s workload boosted the probability of inpatient mortality by 7%. In addition, a greater number of nurses with bachelor’s degrees was associated with a 7% lower risk of mortality.23 Aiken and colleagues have also examined the negative effect of unfavorable work environments and increased nurse workload on pediatric patient outcomes, specifically, missed nursing care. They found that missed nursing care was more common in poor work environments and more care was missed with higher nursing workloads.24 One of the best examples of a work environment that champions the nurse at the bedside is Magnet Recognition for healthcare organizations. Data demonstrate that hospitals that use the structure for Magnet designation achieve significant improvements in their work environments.22 Hospitals that have even some of the Magnet characteristics exhibit improved nurse and patient outcomes. Characteristics of Magnet-designated hospitals that have the most impact on nurse and patient outcomes are investments in staff development, superior management, frontline manager supervisory skill, and good nurse/physician collaboration.22 Nurses who work in Magnet-designated hospitals identify their environments as healthy. A study of 12,233 nurses confirmed healthy work environments in 82% of 540 clinical units and provided evidence that applying structures supporting inter- and intradisciplinary collaboration and decision-making promote the development of healthy work environments.25 As previously noted, the AACN has championed healthy work environments, providing standards for establishing and sustaining a healthy work environment, tools to assess current state, and strategies to improve the environment in order to increase nurse satisfaction and improve patient outcomes.26 The importance of a healthy work environment cannot be stressed enough as the means to ensure a viable, competent, and caring workforce. Nurses look for a culture that respects the nurse’s experience, skills, abilities, and unique contributions.

Beacon Award The Beacon Award for Critical Care Excellence, created by the AACN, distinguishes adult critical care, adult progressive care, and pediatric critical care units that attain high-quality outcomes. This prestigious award provides the critical care community with a means of recognizing achievements in professional practice, patient outcomes, and the health of the work environment. A pediatric critical care unit can achieve the Beacon Award by meeting several criteria in the areas of recruitment and retention; education, training, and mentoring; evidence-based practice and research; patient outcomes; healing environment; and leadership and organizational ethics. Together, these characteristics provide a

42

SECTION I



Pediatric Critical Care: The Discipline

comprehensive view of any given ICU. To date, 31 pediatric critical care units have received the Beacon Award for Critical Care Excellence.27

Professional Development A critical aspect of development for the nurse is the ability to advance and be recognized professionally. A successful critical care professional advancement program recognizes varying levels of staff nurse knowledge and expertise and fosters advancement through a wide range of clinical learning and professional development experiences. Essential components of this program include an orientation program, a continuing education plan, and an array of other opportunities for clinical and professional development. Unit-based advancement programs are most effective when they are linked to the nursing department’s professional advancement program. A professional advancement program that recognizes and rewards evolving expertise contains elements of both clinical and professional development strategies. Nurses require a broad body of knowledge to meet patient and organizational needs. This requirement necessitates a lifelong process of professional development targeted to specific levels of clinical practice. Nurses can choose from many learning options, such as academic education, continuing education programs, participation in research, collaborative learning, case studies, and simulations. Nurses view the availability of continuing education as very important.28

Staff Development The goal of nursing staff development programs is safe, competent practice. Comprehensive programs provide the critical resources to support and promote practice. In addition, professional nursing standards of practice, healthcare laws, regulations, and accreditation requirements focus on the components of competent patient care to protect the healthcare consumer. The establishment of a staff development program that is linked to clinical practice is key to the success of professional nurse development. Technical training alone is no longer sufficient to meet the care delivery needs of the nurse in the critical care environment. In addition to knowledge about disease processes and physiologic instability associated with them, critical care nurses require broad knowledge and expertise in areas such as communication, critical thinking, and collaboration.29 They need to attain the diverse skills necessary to meet the complex needs of their patients and families. Theory and science are required to meet the Synergy competencies and include topics such as specific disease processes, nursing procedures, cultural awareness, moral and ethical principles and reasoning, research principles, and learning theories. This information can be presented using a variety of methods attending to the specific needs of the learners and adult learning principles. Realistic clinical scenarios, case studies, and simulations that represent the dynamic and ambiguous clinical situations nurses encounter daily are most effective.28 Bedside teaching is particularly helpful in the development of clinical judgment and caring practice skills. Expert nurses are role models of many of the competencies delineated by the Synergy Model; novice nurses learn by watching these expert nurses and emulating their behaviors. Communicating and demonstrating clinical knowledge focuses learning, positively affects patient outcomes, and adds to the total body of nursing knowledge.30

Simulation-based learning is now routine in pediatric critical care nursing practice and advanced practice nursing as a state-ofthe-art educational approach. Simulation serves several purposes, such as enhancing patient safety, increasing clinical competence, and promoting effective teamwork. Simulation provides a nonthreatening environment where participants can integrate cognitive, psychomotor, and affective skill attainment without fear of hurting patients.31 All pediatric critical care nursing practice should ideally be evidence based. Although most nursing programs introduce the concepts of evidence-based practice, practicing nurses require continued support to ensure that they can access and evaluate the literature, make appropriate decisions regarding implementing needed practice changes, and evaluate the effectiveness of new practices in improving patient outcomes. Information about research and research use builds clinical inquiry and system thinking skills. Demystifying research, outcome, and quality processes contributes to the development of these key skills. The use of journal club formats and supporting staff involvement in research helps develop clinical inquiry skills. Building knowledge in the areas of healthcare trends and political action expands system thinking skills. The development of critical thinking skills and problemsolving skills also assists with the development of system thinking. Developing excellent communication skills is an essential part of nurses’ professional development plans. In addition to their value in enhancing relationships with patients, families, and colleagues, good communication skills are critical for teaching less experienced staff. Presenting clinical teaching strategies and helping staff to determine learner readiness and to assess understanding will facilitate learning. The importance of developing patience, flexibility, and a nonconfrontational style is reinforced. Negotiation, conflict resolution, time management, communication, and team building are components of collaboration skills. Role-playing, role modeling, and clinical narratives are methodologies that have been used to develop collaboration skills. Nurses learn technical skills and scientific principles in many ways, but caring practices and advocacy are developed only through relationships that evolve over time.32 Nurturing, professional relationships with experienced staff allow novices to integrate their evolving perspectives into practice. Expert nurses who share their clinical knowledge and coach other nurses have a tremendous impact on novice nurses. Nurses who coach do so because they are able to clinically persuade and guide less experienced staff in challenging situations. They demonstrate expert skills and expedite the ongoing clinical development of others. A variety of staff development programs exist, but most fall into either orientation or continuing education programs.

Orientation Orientation programs help acclimate new staff to unit-based policies, procedures, services, physical facilities, and role expectations in a work setting. A specific type of orientation that has developed in response to the nursing shortage is the critical care internship or nurse residency program. These programs have been developed as a mechanism to recruit, train, and retain entry-level nurses. They are designed to transition nurses with little or no nursing experience into the complex critical care environment. They provide extended clinical support for novice nurses and introduce new knowledge more deliberately than do traditional orientation programs. Basic information, skill acquisition, and socialization are the core features of these programs. This foundation builds on the knowledge and skills acquired in nursing school



CHAPTER 6 Evolution of Critical Care Nursing

programs. Teaching usually is under the direction of a hospital or unit-based educator and generally involves less senior staff members as preceptors. Typically, the novice nurse starts with providing care to the least complex patients. The program establishes a foundation on which the novice can develop into a competent clinician.33–36 The AACN partnered with Pediatric Learning Solutions, a division of the Children’s Hospital Association, to offer the Essentials of Pediatric Critical Care Orientation program.37 This program provides a bridge for the knowledge gap between what nurses learn in their basic education program and what they need to know in order to achieve initial clinical competence with critically ill pediatric patients. The program consists of seven interactive modules providing 34 courses, including case scenarios and practice activities that augment knowledge and improve job satisfaction. This program provides flexibility because it is a self-paced, didactic e-learning course that can be incorporated into a blended learning environment combining traditional educational activities, such as preceptorships, discussion groups, workshops, and simulation experiences.

Continuing Education New medical developments, legislation, regulations, professional standards, and expectations of healthcare consumers help determine the need for continuing education and often lend themselves to in-service education programs. In-services involve specific workplace learning experiences that help staff to perform assigned functions and maintain competency. Continuing nursing education includes planned, organized learning experiences designed to expand knowledge and skills beyond the level of basic education.28 The focus of continuing education is information that is not specific to one institution and it builds on previously acquired knowledge and skills. For continuing development, pediatric critical care nurses attend, present at and organize a wide variety of regional, national, and international conferences with pediatric-specific content sponsored by critical care nursing organizations as well as critical care societies. Table 6.2 presents information on organizations providing pediatric critical care content.

Certification in Pediatric Nursing and Pediatric Critical Care Nursing In 1975, the AACN Certification Corporation was established to formally recognize the professional competence of critical care nurses. The mission of the AACN Certification Corporation is to certify and promote critical care nursing practice that optimally contributes to desired patient outcomes. The program establishes the body of knowledge necessary for critical care registered nurse (CCRN) certification, tests the common body of knowledge needed to function effectively within the critical care setting, recognizes professional competence by granting CCRN status to successful certification candidates, and assists and promotes the continual professional development of critical care nurses. In 1997, the unique competencies of pediatric, neonatal, and adult critical care nurses were rearticulated using the Synergy Model2 as a conceptual framework. In order to ensure that the certification reflects current practice, the AACN completes a job analysis at least every 5 years and revises the test plan as needed. The most recent test plan was revised in 2019 (Box 6.2). To date, more than 6400 pediatric critical care nurses hold CCRNPediatric certification, and nearly 300 hold the Pediatric CCRNK.38 The AACN also provided a clinical nurse specialist certification

43

TABLE Selected Critical Care Nursing Organizations 6.2 and Professional Associations

Organization

Website

American Association of Critical Care Nurses

https://www.aacn.org

Australian College of Critical Care Nurses

https://www.acccn.com.au/

Canadian Association of Critical Care Nurses

https://www.caccn.ca/

European Federation of Critical Care Nursing Associations

https://www.efccna.org/

European Society of Pediatric and Neonatal Intensive Care

https://espnic-online.org/

Pediatric Cardiac Intensive Care Society

https://www.pcics. org/?s5nursing

Southeast Asian Federation of Critical Care Nurses

http://seafccn.com/

World Federation of Critical Care Nurses

https://wfccn.org/

World Federation of Pediatric Intensive & Critical Care Societies

https://www.wfpiccs.org/

in pediatric critical care, but the examination for new applicants was retired in 2015 and a new Acute Care Clinical Nurse Specialist (CNS)-Pediatric was developed that complies with the Consensus Model for Advanced Practice Registered Nurse (APRN) Regulation. More information about these certifications can be found at https://www.aacn.org/certification/ get-certified.39 In addition to certification by the AACN, pediatric critical care nurses may choose to be certified by the Pediatric Nursing Certification Board (PNCB). Originally established in 1975 to develop a program to certify primary care pediatric nurse practitioners (CPNP-PC), the PNCB has expanded to certification of acute care pediatric nurse practitioners and pediatric nurses. In 1989, the PNCB launched the Certified Pediatric Nurse (CPN) examination; while CPNs work in a variety of settings, many CPNs work in pediatric critical care today (www.pncb.org/about).40 In 2005, the PNCB launched the only certification examination for acute care pediatric nurse practitioners (CPNP-AC); many currently are practicing in pediatric critical care.

Evolution of Advanced Practice Registered Nurses into Pediatric Critical Care Clinical Nurse Specialists Clinical Nurse Specialists (CNSs), the first APRN designation, evolved from the need for a more specialized focus and increased knowledge to care for specific populations of patients, ensuring quality nursing care. Initially, nurses with focused knowledge were referred to as nurse clinicians, though this terminology ultimately transitioned to CNS. During the early years of the CNS, role competencies lacked clarity and although formal master’s education became a requirement for

44

SECTION I

• BOX 6.2



Pediatric Critical Care: The Discipline

AACN Critical Care Registered Nurse-Pediatric Test Plan

I. Clinical Judgment (80%) A. Cardiovascular (15%) 1. Acute pulmonary edema 2. Cardiac surgery (e.g., congenital defects) 3. Cardiac/vascular catheterization a. Diagnostic b. Interventional 4. Cardiogenic shock 5. Cardiomyopathies (e.g., hypertrophic, dilated, restrictive, idiopathic) 6. Dysrhythmias 7. Heart failure 8. Hypertensive crisis 9. Myocardial conduction system defects 10. Structural heart defects (acquired and congenital, including valvular disease) B. Pulmonary (16%) 1. Acute pulmonary embolus 2. Acute respiratory distress syndrome (ARDS), to include acute lung injury (ALI) and respiratory distress syndrome (RDS) 3. Acute respiratory failure 4. Acute respiratory infections (e.g., pneumonia) 5. Air leak syndromes (e.g., pneumothorax, pneumopericardium) 6. Aspiration (e.g., aspiration pneumonia, foreign body, meconium) 7. Bronchopulmonary dysplasia, asthma, chronic bronchitis 8. Congenital anomalies (e.g., diaphragmatic hernia, tracheoesophageal fistula, choanal atresia, pulmonary hypoplasia, tracheal malacia, tracheal stenosis) 9. Chronic conditions (e.g., asthma, bronchitis) 10. Failure to wean from mechanical ventilation 11. Pulmonary hypertension 12. Status asthmaticus 13. Thoracic surgery 14. Thoracic trauma (e.g., fractured ribs, lung contusions, tracheal perforation) C. Endocrine/Hematology/Gastrointestinal/Renal/Integumentary (19%) 1. Endocrine a. Acute hypoglycemia b. Diabetes insipidus c. Diabetic ketoacidosis d. Hyperglycemia e. Inborn errors of metabolism f. Syndrome of inappropriate secretion of antidiuretic hormone (SIADH) 2. Hematology/Immunology a. Anemia b. Coagulopathies (e.g., immune thrombocytopenia [ITP], disseminated intravascular coagulation [DIC], heparin-induced thrombocytopenia [HIT]) c. Immune deficiencies d. Leukopenia e. Oncologic complications f. Sickle cell crisis g. Thrombocytopenia 3. Gastrointestinal (GI) a. Acute abdominal trauma b. Acute GI hemorrhage c. Bowel infarction/obstruction/perforation (e.g., mesenteric ischemia, adhesions) d. Gastroesophageal reflux e. GI abnormalities (e.g., omphalocele, gastroschisis, volvulus, imperforate anus, Hirschsprung disease, malrotation, intussusception) f. GI surgeries g. Hepatic failure/coma (e.g., portal hypertension, cirrhosis, esophageal varices, fulminant hepatitis, biliary atresia) h. Malnutrition and malabsorption



















































































































































































































3. Renal/Genitourinary a. Acute kidney injury (AKI), acute renal failure, acute tubular necrosis (ATN) b. Chronic kidney disease c. Infections d. Life-threatening electrolyte imbalances 4. Integumentary a. IV infiltration b. Pressure ulcer c. Wounds i. Infectious ii. Surgical iii. Trauma D. Musculoskeletal/Neurology/Psychosocial (16%) 1. Musculoskeletal a. Infections 2. Neurology a. Acute spinal cord injury b. Brain death c. Congenital neurologic abnormalities (e.g., arteriovenous malformation, myelomeningocele, encephalocele) d. Encephalopathy e. Head trauma f. Hemorrhage i. Intracranial (ICH) ii. Intraventricular (IVH) iii. Subarachnoid (traumatic or aneurysmal) a. Hydrocephalus b. Ischemic stroke c. Neurologic infectious disease (e.g., viral, bacterial, fungal) d. Neuromuscular disorders e. Neurosurgery f. Seizure disorders g. Space-occupying lesions (e.g., brain tumors) h. Spinal fusion i. Traumatic brain injury (e.g., epidural, subdural, concussion, nonaccidental trauma) 3. Behavioral/Psychosocial a. Abuse, maltreatment, neglect b. Agitation c. Developmental delays d. Failure to thrive e. Medical nonadherence f. Suicidal ideation and/or behaviors E. Multisystem (14%) 1. Asphyxia 2. Comorbidity in patients with transplant history 3. End of life 4. Healthcare-associated infections (HAI) a. Central line–associated bloodstream infections (CLABSI) b. Catheter-associated urinary tract infection (CAUTI) c. Ventilator-associated pneumonia (i.e., ventilator-associated event [VAE]) 5. Hemolytic uremic syndrome (HUS) 6. Hypotension 7. Infectious diseases a. Multidrug-resistant organisms (e.g., methicillin-resistant Staphylococcus aureus [MRSA], vancomycin-resistant Enterococcus [VRE], carbapenem-resistant Enterobacteriaceae [CRE]) b. Influenza (e.g., pandemic or epidemic) 8. Multiorgan dysfunction syndrome (MODS) 9. Multisystem trauma 10. Pain





































































































































































































































CHAPTER 6 Evolution of Critical Care Nursing

AACN Critical Care Registered Nurse-Pediatric Test Plan—cont’d





II. Professional Caring and Ethical Practice (20%)

















A. B. C. D. E. F. G.















11. Palliative care 12. Sepsis continuum (systemic inflammatory response syndrome (SIRS), sepsis, severe sepsis, septic shock) 13. Shock states a. Distributive (e.g., anaphylactic, neurogenic) b. Hypovolemic 14. Sleep disruption (including sensory overload) 15. Submersion injuries 16. Thermoregulation 17. Toxic ingestions/inhalations (e.g., drug/alcohol overdose) 18. Toxin/drug exposure (including allergies)

Advocacy/Moral Agency Caring Practices Response to Diversity Facilitation of Learning Collaboration Systems Thinking Clinical Inquiry

Data from AACN Certification Corporation. CCRN Exam Handbook. 2019. https://www.aacn.org/,/ media/aacn-website/certification/get-certified/handbooks/ccrnexamhandbook.pdf























• BOX 6.2

45

the role, the curriculum varied from school to school. As a result, there was confusion about the role and unique skill set of the CNS. By the mid-1990s, standards and competencies for CNSs were developed, resulting in improved role definition.41 The components of the CNS role, similar to other APRN roles, became expert clinician, educator, researcher, and administrator. However, a contemporary view of CNSs identifies change agent, strategist, bridge builder, and communicator in addition to expert clinician as key components of the role.42 CNSs work within three spheres of influence: (1) patients (clinical experts supporting direct care at the bedside), (2) nurses (advancing and improving nursing practice) and (3) systems (working with interdisciplinary teams and fostering system innovations).43–45 CNSs in the PICU date back to the 1980s. As technology continued to expand in the PICU environment and patient acuity increased, nursing care became more complex. CNSs were uniquely prepared to guide and support expanding nursing practice. CNSs in the PICU worked to improve quality of care, provide education through a variety of mechanisms, structure and organize orientation for new nurses, and promote best practice. With the changes seen in healthcare finance in the 1990s, particularly the emergence of managed care, many hospitals restructured their organizations to the detriment of the CNS role. Although critically important to the functioning of the PICU, financing the work of the CNS was no longer supported. As a result, the CNS role was eliminated or phased out in many institutions. Because the CNS did important work, the responsibilities were transitioned to other professionals. Yet, in some institutions, the role was maintained. Today, it is being reimplemented with the changing financial landscape, which rewards quality and value-based healthcare.42

Pediatric Nurse Practitioners Another APRN role, the Pediatric Nurse Practitioner (PNP), came into healthcare in the 1960s when the need to care for more children arose.46 PNPs were educated in specialty training programs focusing on delivery of preventive and primary care. Graduate-level degrees were mandated for this role in many states by the early 1980s and today is the minimum expected degree for entry into practice across the United States. Although currently not fully adopted nationally, in 2004, the American Association of Colleges of Nursing recommended the practice doctorate (Doctor of Nursing Practice [DNP]) for the nurse practitioner as the entry into practice degree.47 In the 1990s, PNP practice extended to pediatric critical care with the rise in number of PICUs and number of PICU beds48 and the need for additional personnel. The addition of PNPs in the

PICU was also endorsed by the American Academy of Pediatrics.49 By the early 2000s, the numbers of PNPs in critical care grew even more as a result of restricted resident duty hours set forth by the Accreditation Council for Graduate Medical Education.50 Owing to the need for specialized education for these areas, master’s programs were created that focused on acute care and critical care. To standardize APRN practice across the country, a consensus model was developed for regulation of licensure, accreditation, certification, and education.51 Concisely, this model serves as a guide to inform organizations about selection of the best fitting candidate for an open APRN position, aligning education, licensure, certification, and practice (e.g., hiring a CPNP-AC for the PICU and CPNP-PC for the primary care office). Because this model is yet to be adopted by all states and the availability of CPNP-ACs is limited, some PICUs continue to hire either CPNP-PCs or family nurse practitioners for the provision of patient care. Given this issue, postgraduate training of PNPs not educated in acute care is critical to ensure competency and delivery of high-quality care.

Nursing Research Pediatric critical care nursing is a science as well as an art. It is vital that nursing interventions supporting the care of the critically ill child and family be grounded in high-quality evidence generated through pediatric-specific research. Whereas knowledge generated in the larger neonatal and adult populations can inform pediatric practice, the developmental and maturational differences in our unique patient population require independent study. In 2015, a group of international pediatric nurse scientists convened to summarize the state of the science in pediatric critical care nursing and to prioritize a list of nursing research topics for future focus. The group identified four top research priorities, including end-of-life care and decision-making, interventions impacting the child and family experiencing withdrawal of lifesustaining therapies, long-term psychosocial outcomes of pediatric critical illness, communicating clinical assessments and improving teamwork, and articulating core nursing competencies that prevent unstable situations from deteriorating into crises.52 Today, there are increasing numbers of pediatric critical care nurse scientists internationally who collaborate and address these and many other research priorities.

Summary Pediatric critical care nursing has evolved into a specialty in its own right. Pediatric critical care nurses make significant and unique contributions to the healthcare of children. A pediatric

46

SECTION I



Pediatric Critical Care: The Discipline

critical care nurse requires knowledge and skills in both the art and science of nursing. A supportive, empowered environment and support for professional advancement are essential to the development of knowledge and skills. Ongoing research for the advancement of pediatric critical care nursing is important.

Key References Aiken LH, Clarke SP, Sloane DM, et al. Hospital nurse staffing and patient mortality, nurse burnout, and job dissatisfaction. JAMA. 2002;288:1987-1993. Cimiotti JP, Aiken LH, Sloane DM, et al. Nurse staffing, burnout, and health-care associated infection. Am J Infect Control. 2012;40:486490. Curley MA, Hickey PA. The Nightingale metrics. Am J Nurs. 2006; 106:66-70.

Curley MAQ. Patient-nurse synergy: optimizing patients’ outcomes. Am J Crit Care. 1998;7:64-72. Downes JJ. The historical evolution, current status and prospective development of pediatric critical care. Crit Care Clin. 1992;46:1-22. Hickey PA, Gauvreau K, Porter C, Connor JA. The impact of critical care nursing certification on pediatric patient outcomes. Pediatr Crit Care Med. 2018;19(8):718-724. Ma C, Park SH, Shang J. Inter- and intra-disciplinary collaboration and patient safety outcomes in U.S. acute care hospital units: a crosssectional study. Int J Nurs Stud. 2018;85:1-6. Watson RS, Asaro LA, Hutchins L, et al. Risk factors for functional decline and impaired quality of life after pediatric respiratory failure. Am J Respir Crit Care Med. 2019;200(7):900-909.

The full reference list for this chapter is available at ExpertConsult.com.

e1















Th

































































































1. Downes JJ. The historical evolution, current status and prospective development of pediatric critical care. Crit Care Clin. 1992;46:1-22. 2. Curley MAQ. Patient-nurse synergy: optimizing patients’ outcomes. Am J Crit Care. 1998;7:64-72. 3. American Association of Critical-Care Nurses. AACN Synergy Model for Patient Care. https://www.aacn.org/nursing-excellence/ aacn-standards/synergy-model. 4. Johnson M, McCloskey JC. Quality in the nineties. In: Series on Nursing Administration. Vol. 3. Delivery of Quality Health Care. St. Louis: Mosby Year Book; 1992. 5. Brooten D, Naylor MD. Nurses’ effect on changing patient outcomes. Image (IN). 1995;7:95-99. 6. Hickey PA, Pasquali SK, Gaynor JW, et al. Critical care nursing’s impact on pediatric patient outcomes. Ann orac Surg. 2016;102(4): 1375-1380. 7. Hickey PA, Gauvreau K, Porter C, Connor JA. The impact of critical care nursing certification on pediatric patient outcomes. Pediatr Crit Care Med. 2018;19(8):718-724. 8. Hickey PA, Gauvreau K, Curley MA, Connor JA. The effect of critical care nursing and organizational characteristics on pediatric cardiac surgery mortality in the United States. J Nurs Adm. 2013;43(12): 637-44. 9. Watson RS, Asaro LA, Hertzog JH, et al. Long-term outcomes after protocolized sedation versus usual care in ventilated pediatric patients. Am J Respir Crit Care Med. 2018;197(11):1457-1467. 10. Watson RS, Asaro LA, Hutchins L, et al. Risk factors for functional decline and impaired quality of life after pediatric respiratory failure. Am J Respir Crit Care Med. 2019;200(7):900-909. 11. Ma C, Park SH, Shang J. Inter- and intra-disciplinary collaboration and patient safety outcomes in U.S. acute care hospital units: a crosssectional study. Int J Nurs Stud. 2018;85:1-6. 12. Knaus WA, Draper EA, Wagner DP, Zimmerman JE. An evaluation of the outcome from intensive care in major medical centers. Ann Intern Med. 1985;104:410-418. 13. Boev C, Xia Y. Nurse-physician collaboration and hospital-acquired infections in critical care. Crit Care Nurse. 2015;35:66-72. 14. Wazner MB, Wojtaszczyk AM, Kelly J. Nurses’ perceptions of physicians’ communication: the relationship among communication practices, satisfaction, and collaboration. Health Commun. 2009;24:683691. 15. Guirardello EB. Impact of critical care environment on burnout, perceived quality of care and safety attitude of the nursing team. Rev Lat Am Enfermagem. 2017;25:e2884. 16. Cimiotti JP, Aiken LH, Sloane DM, Wu ES. Nurse staffing, burnout, and health-care associated infection. Am J Infect Control. 2012; 40:486-490. 17. Galletta M, Portoghese I, D’Aloja E, et al. Relationship between job burnout, psychosocial factors and health care-associated infections in critical care units. Intensive Crit Care Nurs. 2016;34:59-66. 18. Curley MA, Hickey PA. The Nightingale metrics. Am J Nurs. 2006;106:66-70. 19. Dent RL, Armstead C, Evans B. Three structures for a healthy work environment. AACN Adv Crit Care. 2014;25:94-100. 20. Nayback-Beeb AM, Forsythe T, Funari T, et al. Using evidence-based leadership initiatives to create a healthy nursing work environment. Dimens Crit Care Nurs. 2013;32:166-173. 21. Aiken LH, Clarke SP, Sloane DM, Silber JH. Hospital nurse staffing and patient mortality, nurse burnout, and job dissatisfaction. JAMA. 2002;288:1987-1993. 22. Aiken LH, Clarke SP, Sloane DM, Cheney T. Effects of hospital care environments on patient mortality and nurse outcomes. J Nurs Adm. 2008;38:223-229. 23. Aiken LH, Sloane DM, Bruyneel L, et al. Nurse staffing and education and hospital mortality in nine European countries: a retrospective observational study. Lancet. 2014;383:1824-1830.



24. Lake ET, de Cordova PB, Barton S, et al. Missed nursing care in pediatrics. Hosp Pediatr. 2017;7(7):378-384. 25. Kramer M, Maquire P, Brewer BB. Clinical nurses in Magnet hospitals confirm productive, healthy unit work environments. J Nurs Manag. 2011;19:5-17. 26. American Association of Critical-Care Nurses. Healthy Work Environments. https://www.aacn.org/nursing-excellence/healthy-workenvironments?tab5Patient%20Care. 27. American Association of Critical-Care Nurses. Beacon Awards. 2019. https://www.aacn.org/nursing-excellence/beacon-awards. 28. Skees J. Continuing education: a bridge to excellence in critical care nursing. Crit Care Nurs Q. 2010;33:104-116. 29. American Association of Critical-Care Nurses. AACN standards for establishing and sustaining healthy work environments: a journey to excellence. Am J Crit Care. 2005;14:187-197. 30. Guilhermino MC, Inder KJ, Sundin D, Kuzmiuk L. Nurses’ perceptions of education on invasive mechanical ventilation. J Contin Educ Nurs. 2014;45:225-232. 31. Roh YS, Lee WS, Chung HS, Park YM. The effects of simulationbased resuscitation training on nurses’ self efficacy and satisfaction. Nurse Educ Today. 2011;33:123-128. 32. Benner P, Tanner C, Cheslea C. Expertise in Nursing Practice: Caring, Clinical Judgment, and Ethics. 2nd ed. New York: Springer Publishing; 2009. 33. Welding N. Creating a nursing residency: decrease turnover and increase clinical competence. Medsurg Nurs. 2011;20:37-40. 34. Eckerson CM. The impact of nurse residency programs in the United States on improving retention and satisfaction of new nurse hires: An evidence-based literature review. Nurse Educ Today. 2018;71:84-90. 35. Van Camp J, Chappy S. The effectiveness of nurse residency programs on retention: a systematic review. AORN J. 2017;106(2):128-144. 36. Walsh, AL. Nurse residency programs and the benefits for new graduate nurses. Pediatr Nurs. 2018;4(6):275-279. 37. American Association of Critical-Care Nurses. Essentials of Pediatric Critical Care Orientation (EPCCO) program. https://www.aacn. org/education/online-courses/essentials-of-pediatric-critical-careorientation. 38. American Association of Critical-Care Nurses. Exam Stats and Scores. Available at: https://www.aacn.org/certification/preparation-tools-andhandbooks/exam-stats-and-scores. 39. American Association of Critical-Care Nurses. Get Certified. Available at: https://www.aacn.org/certification/get-certified. 40. Pediatric Nursing Certification Board. CPNP-AC Fact Sheet. Available at: www.pncb.org/about. 41. McClelland M, McCoy MA, Burson R. Clinical nurse specialists: then, now, and the future of the profession. Clin Nurse Spec. 2013; 27(2):96-102. 42. Davidson PM, Rahman A. Time for the renaissance of the clinical nurse specialist role in critical care? AACN Adv Crit Care. 2019;30:61-64. 43. Coombs M, Chaboyer W, Sole ML. Advanced nursing roles in critical care—a natural or forced evolution? J Prof Nurs. 2007;23(2):83-90. 44. Mohr LD, Coke LA. Distinguishing the clinical nurse specialist from other graduate nursing roles. Clin Nurse Spec. 2018;32(3):139-151. 45. National Association of Clinical Nurse Specialists. What is a CNS? Available at: https://nacns.org/about-us/what-is-a-cns/. 46. Ford LC, Silver HC. The expanded role of the nurse in child care. Nurs Outlook. 1967;15:43-45. 47. American Association of Colleges of Nursing. AACN Position Statement on the Practice Doctorate In Nursing. Available at: www.aacn.nche. edu/DNP/DNPPositionStatement.htm. 48. Randolph AG, Gonzales CA, Cortellini L, Yeh TS. Growth of pediatric intensive care units in the United States from 1995 to 2001. J Pediatr. 2004;144:792-798. 49. Schaeffer HA, Hardy DR, Jewett PH, et al. The role of the nurse practitioner and physician assistant in the care of hospitalized children. Pediatrics. 1999;103:1050-1051.

References

e2













Th













55. McAlvin SS, Carew-Lyons A. Family presence during resuscitation and invasive procedures in pediatric critical care. Am J Crit Care. 2014; 23:477-485. 56. Curley MAQ, Meyer EC, Scoppettuolo LA, et al. Parent presence during invasive procedures and resuscitation: evaluating a clinical practice change. Am J Respir Crit Care Med. 2012;186:1133-1139. 57. O’Grady TP. A new age for practice: creating the framework for evidence. In: Malloch K, O’Grady TP, eds. Introduction to Evidence-Based Practice in Nursing and Healthcare. 2nd ed. Sudbury, MA: Jones and Bartlett; 2009. 58. Donovan AL, Aldrich JM, Gross AK, et al. Interprofessional care and teamwork in the ICU. Crit Care Med. 2018:46;980-990.









50. Philibert I, Friedmann, P, Williams WT for the members of the ACGME Work Group on Resident Duty Hours. New requirements for resident duty hours. JAMA. 2002;288:1112-1114. 51. APRN Consensus Work Group & National Council of State Boards of Nursing APRN Advisory Committee. e Consensus Model for APRN Regulation: Licensure, Accreditation, Certification, Education. 2008. https://www.ncsbn.org/Consensus_Model_for_APRN_ Regulation_July_2008.pdf. 52. Tume LN, Coetzee M, Dryden-Palmer K, et al. Pediatric critical care nursing research priorities—initiating international dialogue. Pediatr Crit Care Med. 2015;16:e174-e182. 53. Hooper-Kyriakidis P, Stannard D. Clinical Wisdom and Interventions in Acute and Critical Care. 2nd ed. New York, NY: Springer; 2011. 54. Curley MAQ. The essence of pediatric critical care nursing. In: Curley MAQ, Moloney-Harmon PA, eds. Critical Care Nursing of Infants and Children. Philadelphia: WB Saunders; 2001.

e3

Abstract: As the critical care environment became increasingly complex, nurses caring for critically ill children and their families ensured that their practice evolved to best meet patient and family needs. Nurses spend the most time in direct contact with patients and families and are essential members of the critical care team. Quality of care and morbidity and mortality outcomes of patients are positively impacted by nursing care, nurses’ level of experience,

and certification. Roles for nurses working in pediatric critical care have evolved with the specialty. Nurses create an environment to safely shepherd children and their families during the critical care experience. Key Words: Nursing science, quality of care, Nightingale metrics, synergy, professional development, pediatric critical care nursing

7 Fostering a Learning Healthcare Environment in the Pediatric Intensive Care Unit MELVIN G. PERRY JR AND JERRY J. ZIMMERMAN









A learning healthcare system occurs when patient care, interdisciplinary education, and clinical research are so integrated and intercalated that they are basically inseparable. Each element informs and benefits from the other. Professionalism provides the foundation for a learning healthcare environment and encompasses the concepts of accountability, respect, and an inclusive, diverse team. Standardization, in the form of clinical standard work, represents the infrastructure for iterative improvement. Without standardization, measurements of improvement are not possible.

Incredibly important as it is, there is more to working in the pediatric intensive care unit (PICU) than providing care for critically ill children. When critical care providers converse about what they do, the discussion typically includes ensuring rapid and accurate diagnosis and treatment, providing support for dysfunctional/failed organ systems, and preventing complications of critical illness and its treatment. However, this chapter considers another activity that will enrich any PICU, including the physical space, the actual work, and the people who provide and receive treatment. This overlooked but key aspect of critical care may be described as fostering a learning healthcare environment.

Learning Healthcare System A learning healthcare system occurs when patient care, interdisciplinary education, and clinical research are so integrated and intercalated that they are basically inseparable (Fig. 7.1). Each element synergistically benefits from and informs the other.1 Such continuous and reciprocal learning and knowledge translation ultimately facilitates enhanced performance and improved outcomes for both patients and providers.













PEARLS Because pediatric critical care providers should ideally base practice on science and not empiricism, the pediatric intensive care unit must serve as the clinical laboratory for generation of evidence. Primary benefits of a learning healthcare environment include identification of best available evidence to support best practice and promotion of wellness and resiliency among critical care providers.

Foundation Predicated on Professionalism Professionalism provides the foundation for a learning healthcare environment and encompasses the concepts of accountability, respect, and teamwork. In the PICU, accountability means practicing value-based care that considers both the cost and the quality of care delivered2 plus demanding a culture of safety.3 Critical care practitioners work at the intersection of complex patients, complex therapies, and a complex environment that collectively provide the antecedents for a potential perfect storm for inadvertent adverse events. A culture of safety includes being personally accountable, practicing clinical standard work, engaging multidisciplinary teams, focusing on systems, and anticipating unintended consequences,4 all of which are linked with effective communication (Fig. 7.2).5 Respect within professionalism centers around inclusion and diversity. Unfortunately, research in workforce diversity and inclusion, including healthcare industry workforce as a whole and within different specialties and subspecialties, is sparse. However, numerous studies have concluded that race and ethnicity are social constructs with profound social, economic, legal, financial, and health implications for the global population, including the 47

48

SECTION I



Pediatirc Critical Care: The Discipline

PICU

Interdisciplinary education

Clinical research

Best practice clinical care

Learning healthcare environment

Professionalism • Fig. 7.1



​Learning healthcare environment.

Complex patients

Complex therapies

Complex environment

• Fig. 7.2



​Intensive care complexity.

PICU workforce. A 2018 study concluded: “There is an urgent need for greater diversity, with respect to gender, race, ethnicity, and sexual orientation in the U.S. healthcare workforce. While society, in general, is becoming more diverse, the same cannot be said of American medicine.”6,7 Interestingly, at least by some measures, the most diverse countries are considered to be in sub-Saharan Africa, while the least diverse tend to be in Europe and Northeast Asia, with the United States in the middle, ranking slightly above Russia and slightly below Spain.8 Census data from 2017 demonstrated that the United States is becoming more diverse, with Asian and mixedrace populations leading the way.9 American healthcare, including PICU staffing, has not kept pace with changing general population demographics. A 1997 study found that the supply of minority physicians will need to double for Hispanic and black physicians, triple for Native American physicians, and decrease by two-fifths for white physicians and two-thirds for Asian and Pacific Island physicians to meet US healthcare needs with racial and ethnic population parity, based on managed care–based recommendations of 218 physicians per 100,000 population.10 In defining the scope of diversity, race, gender (including nonbinary identities), genetics, and socioeconomics must be considered and respected. At the lowest bar, institutional and individual healthcare providers can be disbarred from third-party private and governmental payers for treating patients differently

based on diversity characteristics noted earlier. As healthcare moves into quality outcomes as a determinant for reimbursement, these issues take on even greater importance. Studies have shown that concordance between physician and patient demographics improves quality measures.11-13 In pediatric populations, studies have found that racial and ethnic minority children have disparities in their healthcare access and outcomes versus their white counterparts.14 Enrollment in the State Children’s Health Insurance Program decreases but does not eliminate these disparities, which vary by state.15 A recent British study found that concordance did not affect a patient’s assessment of hospitalist performance.16 This is good news for the PICU population in terms of quality measures. Historically, clinical research, one of the pillars of a learning healthcare environment, was not necessarily designed and conducted to measure and analyze the different responses of racial/ ethnic and gender minorities to a given treatment. There is evidence that research scholarship generated by diverse research teams yields research that is higher quality and more impactful.17,18 The National Institutes of Health (NIH) has been encouraging diversity in research enrollment for years, and the Scientific Workforce Diversity Office leads the NIH’s effort to diversify the national scientific workforce (e.g., #GREATMINDSThinkDifferently [https://diversity.nih.gov/]). The Department of Health and Human Services recently offered funding opportunities to increase minority participation in multidisciplinary PICU conferences (R13, Pediatric Critical Care Conferences Initiative, RFA-HD-20-012). Nevertheless, African-American, LatinX, Native Alaskan, Native American, and Native Hawaiian populations remain underrepresented in medicine as compared with their proportion of the general population. For example, African Americans and LatinX constitute about 13% and 17% of the general population, respectively, but represent only about 4.2% and 4.6%, respectively, of physicians.19 Native American, Alaskan Native, Native Hawaiian, and Pacific Islander are represented even less.6,7,19 Providers from underrepresented minority groups are more likely to practice in an underserved area.20,21 Even less is known about LGBT physician workforce numbers. A recent survey of Pediatric Department Chairs in the United States found that only 0.4% of faculty identified as LGBT.20 Pediatrics appears to be doing better than other medical specialties and subspecialties regarding physician diversity, particularly from a gender standpoint. A 2017 workforce study found that 73% of residents, 64% of fellows, and 54% of faculty were female.22 American Board of Pediatrics data from 2017 showed that 40% of all certified PICU physicians were women and 60% of first-year fellows were female.12 Most African-American and LatinX physicians are first-generation doctors, unlike their white counterparts. Moreover, the majority of the underrepresented minorities enter primary care, not medical or surgical subspecialties. For critical care physicians, the most recent preliminary data of the Society of Critical Care Medicine’s Diversity and Inclusion Committee suggests that 1% to 2% of intensivist members are African American and about 10% are LatinX. Despite the demonstrable benefits of a diverse physician workforce, disparities and outright discrimination remain evident. The literature surrounding this issue has primarily focused on women, but there is no reason to believe that the discriminatory behaviors toward women remain, while those toward ethnic and racial minorities would have ceased, since women represent a greater percentage of physicians, at 37%, than all racial minorities combined.19 A third of female physicians have experienced sexual



CHAPTER 7 Fostering a Learning Healthcare Environment in the Pediatric Intensive Care Unit

harassment23; women are less likely to be introduced as “doctor”24; women are less likely to be first authors in top-tier journals18; women are less likely to be included on expert guideline consensus panels25; there are fewer women in leadership, including editorial board positions, even in pediatrics6,22,26; there are fewer women full professors6,27; women receive less research startup funding 18,28; and a significant gender pay gap persists in medicine.29,30 Studies in other industries have demonstrated that management teams with higher gender diversity outperformed those with less diversity,31 and greater gender diversity increased overall business performance, including number of customers, revenue, and profits.32 Similarly, companies with a racially and ethnically diverse workforce financially outperform their competitors by 35%. Yet 97% of US companies have an executive/senior leadership that fails to reflect the country’s ethnic and racial diversity.33 Healthcare also fails in this regard—including pediatrics—and therefore likely also pediatric critical care. The case for a diverse healthcare workforce includes advancing culture competency, increasing access to high-quality healthcare services, strengthening the medical research agenda, and ensuring optimal management of the healthcare system.21 The third aspect of professionalism is teamwork—and nowhere is this more important than in the PICU.34 Consider, for example, early mobilization and the cast of providers required to make it successful: physical therapy, nursing, respiratory therapy, nutrition services, pharmacy, physicians, and the patient and family. Effective teamwork in the PICU must acknowledge patients and families first,35 celebrate the interdisciplinary care team, include clinical research personnel, and promote wellness and resiliency. Such teamwork facilitates well-being; this, in turn, supports improved patient-clinician relationships, a high-functioning care team, and an engaged and effective workforce (https://nam. edu/initiatives/clinician-resilience-and-well-being/).

Pillars of a Learning Healthcare Environment Best-Practice Clinical Care Characteristics of clinical standard work include being consciously developed and documented; evidence based whenever possible, consensus derived when evidence is absent, followed by everyone performing the work, “owned” by someone, describes a clinical pathway/patient trajectory, is measurable, and represents the basis for improvement.36 Standardization facilitates identifying and eliminating waste, communicating between providers, establishing a baseline for continuous improvement, and minimizing noise/controlling for nuisance variables. Standardization represents the infrastructure for iterative improvement. Without standardization, measurements of improvement are not possible.37 Clinical standard work benefits from continuous process improvement longitudinal plan-do-study-act (PDSA) cycles embedded in clinical research or quality improvement. Probably the best illustration of this concept, iterative research protocols implemented into clinical practice, is the amazing history of acute lymphocytic leukemia (ALL). In the 1950s, ALL was a death sentence for a child within a few months; today, almost all children with ALL experience long-term survival.38 Advantages of protocolized critical care have been summarized and include avoiding errors of omission, improving PICU efficiency, decreasing cost and improving value, and maintaining and improving the standard of care.39

49

Clinical Research, Including Quality Improvement Currently, of the three determinants that affect clinical decisionmaking by critical care practitioners—education, experience, and evidence—the latter is least abundant. However, Claude Bernard astutely noted that while most people regard medicine as the art of healing, it is more appropriate to regard medicine as the science of healing because providers should ideally arrive at a cure scientifically and not empirically.40 For a fundamental discovery to transpire, appropriate clinical stimuli must interact with scientific training41—what better place than the PICU? But for this to happen, a real collaboration must exist among the principal investigator, patient and family, and the entire bedside care team. Primary clinical faculty must work alongside physician-scientists to ensure success of clinical research in the PICU.42 One example of the power of critical care research is the story of central line–associated bloodstream infections (CLABSIs) among critically ill children. Previously, such infections were viewed as almost inevitable among critically ill children but are known to be associated with increased duration of stay, prolonged antibiotic therapy, ongoing need for venous access, increased morbidity and mortality, and increased healthcare costs.43 National quality improvement research focused on insertion and maintenance bundles for central venous catheters (CVCs) resulted in a decrease in PICU CLABSI from 5.8 to 1.4 infections per 1000 CVC days over a 5-year intervention interval.44 Today, a CLABSI typically evokes root cause analysis, reviewed as a serious adverse PICU event. In a similar fashion, investigators have critically examined the utility of the Society of Critical Care Medicine’s ICU Liberation bundle of clinical standard work elements as infrastructure for usual care provided to critically ill adults.45 ABCDEF Bundle elements are summarized in Table 7.1. Cohort analyses from two independent investigations demonstrated that proportional compliance with ABCDEF bundle elements resulted in significant and dose-related improvements in outcomes—specifically survival, duration of mechanical ventilation, neurologic organ dysfunction (i.e., delirium and coma), physical restraint use, ICU readmission rates, and discharge disposition of ICU survivors.46,47 These positive intervention effect sizes were not subtle.

Interdisciplinary Educational Model In a learning healthcare environment, everyone is a teacher and everyone is a student. Benefits of an interdisciplinary model for teaching/education have been summarized and include developing teamwork; engaging in realistic simulations; expanding tolerance

TABLE Elements of Society of Critical Care Medicine’s 7.1 ICU Liberation Bundle A

Always prioritize assessment, prevention, and management of pain

B

Both spontaneous awakening and breathing trials at least daily

C

Cognizant choice of analgesics and sedatives

D

Delirium: assess, prevent, and manage

E

Early mobilization and exercise

F

Family engagement and empowerment in the care plan

50

SECTION I



Pediatirc Critical Care: The Discipline

and respect; practicing complex communication; acknowledging patient, family, and other perspectives; sharing trust, value, and power; and thinking about systems.48 For example, if a particular PICU is interested in successfully transitioning patients receiving active cardiopulmonary resuscitation onto extracorporeal life support (ECPR), the simulation must include surgeons, intensivists, nurses, pump technicians, respiratory care personnel, and social work providers. Educational siloes related to ECPR cannot achieve the desired outcome. As noted previously, all of critical care is a team activity, and team education around any clinical standard work must be an essential component of continuous process improvement that will inform design for the next PDSA cycle. Realtime team debriefing around critical events (doing in context) represents a particularly effective interdisciplinary simulation teaching modality.49

clinical practices that are supported by high-quality evidence. It includes the following recurring steps: (1) conduct quarterly evidence searches, (2) decide which evidence-based practices to implement, (3) support implementation of selected practices, and (4) monitor progress. The second, less obvious benefit is promoting wellness and resiliency among critical care providers. Constant, significant stressors related to provision of pediatric intensive care represent real risk factors for burnout syndrome and a number of related adverse outcomes for PICU practitioners49 (see Chapter 22). Participation of the interdisciplinary team in shared education and research/quality improvement activities affords opportunities for critical care providers to unwind, debrief, and reflect, to provide mutual support, and to reinvigorate a sense of purpose for the important work of pediatric critical care.

Benefits of a Learning Healthcare Environment

Key References

In a learning healthcare environment, the activities of patient care, clinical research, and shared education are inexorably linked to two common purposes (Fig. 7.3). The first obvious benefit is generation or identification of best available evidence to support best practice. In addition to facilitating and participating in clinical research related to pediatric critical care, PICUs might also consider implementation of E-SCOPE—evidence scanning for clinical, operational, and practice efficiencies.49 E-SCOPE is a systematic approach to identify and then rapidly implement Facilitates identification, delivery of high value patient and family care

Fostering A Learning Healthcare Environment

Ely EW. The ABCDEF Bundle: Science and philosophy of how ICU liberation serves patients and families. Crit Care Med. 2017;45(2):321-330. Lane-Fall MB, Miano TA, Aysola J, Augoustides JGT. Diversity in the emerging critical care workforce: analysis of demographic trends in critical care fellows from 2004 to 2014. Crit Care Med. 2017; 45(5):822-827. Meade MO, Ely EW. Protocols to improve the care of critically ill pediatric and adult patients. JAMA. 2002;288(20):2601-2603. Mendoza FS, Walker LR, Stoll BJ, et al. Diversity and inclusion training in pediatric departments. Pediatrics. 2015;135(4):707-713. Rivara FP, Alexander D. Randomized controlled trials and pediatric research. Arch Pediatr Adolesc Med. 2010;164(3):296-297. Rotenstein LS, Jena AB. Lost Taussigs: the consequences of gender discrimination in medicine. N Engl J Med. 2018;378(24):2255-2257. Smith MD, et al., eds, for the Committee on the Learning Health Care System in America. Best Care at Lower Cost: The Path to Continuously Learning Health Care in America. Washington DC: National Academy Press; 2013. Walrath JM, Muganlinskaya N, Shepherd M, et al. Interdisciplinary medical, nursing, and administrator education in practice: the Johns Hopkins experience. Acad Med. 2006;81(8):744-748.

Promotes wellness for the community ICU practitioners and patients

• Fig. 7.3



​Fostering a learning healthcare environment.

The full reference list for this chapter is available at ExpertConsult.com.

e1











































































































1. Smith MD, et al, eds, for the Committee on the Learning Health Care System in America. Best Care at Lower Cost: The Path to Continuously Learning Health Care in America. Washington DC: National Academy Press; 2013. 2. Hernu R, Cour M, de la Salle S, Robert D, Argaud L, for the Costs in French ICUs Study Group. Cost awareness of physicians in intensive care units: a multicentric national study. Intensive Care Med. 2015;41(8):1402-1410. 3. Thornton KC, Schwarz JJ, Gross AK, et al. Preventing Harm in the ICU-building a culture of safety and engaging patients and families. Crit Care Med. 2017;45(9):1531-1537. 4. Leape LL, Berwick DM. Five years after To Err Is Human: what have we learned? JAMA. 2005;293(19):2384-2390. 5. Piazza O, Cersosimo G. Communication as a basic skill in critical care. J Anaesthesiol Clin Pharmacol. 2015;31(3):382-383. 6. Lane-Fall MB, Miano TA, Aysola J, Augoustides JGT. Diversity in the Emerging critical care workforce: analysis of demographic trends in critical care fellows from 2004 to 2014. Crit Care Med. 2017;45(5):822-827. 7. Xierali IM, Castillo-Page L, Zhang K, Gampfer KR, Nivet MA. AM last page: the urgency of physician workforce diversity. Acad Med. 2014;89(8):1192. 8. Alesina A, Devleeschauwer A, Easterly W, Kurlat S, Wacziarg R. Fractionalization. EconPapers Harvard Institute of Economic Research Working Papers. 2002; Harvard Institute of Economic Research (1959). 9. Vespa J, Armstrong DM, Medina L. Demographic turning points for the United States: Population Projections for 2020 to 2060. https:// www.census.gov/content/dam/Census/newsroom/press-kits/2018/ jsm/jsm-presentation-pop-projections.pdf. 10. Libby DL, Zhou Z, Kindig DA. Will minority physician supply meet U.S. needs? Health Aff (Millwood). 1997;16(4):205-214. 11. Saha S, Komaromy M, Koepsell TD, Bindman AB. Patientphysician racial concordance and the perceived quality and use of health care. Arch Intern Med. 1999;159(9):997-1004. 12. Cooper LA, Roter DL, Johnson RL, Ford DE, Steinwachs DM, Powe NR. Patient-centered communication, ratings of care, and concordance of patient and physician race. Ann Intern Med. 2003;139(11):907-915. 13. Laveist TA, Nuru-Jeter A. Is doctor-patient race concordance associated with greater satisfaction with care? J Health Soc Behav. 2002;43(3):296-306. 14. Shone LP, Dick AW, Brach C, et al. The role of race and ethnicity in the State Children’s Health Insurance Program (SCHIP) in four states: are there baseline disparities, and what do they mean for SCHIP? Pediatrics. 2003;112(6 Pt 2):e521. 15. Shone LP, Dick AW, Klein JD, Zwanziger J, Szilagyi PG. Reduction in racial and ethnic disparities after enrollment in the State Children’s Health Insurance Program. Pediatrics. 2005;115(6):e697-705. 16. Crawford D, Paranji S, Chandra S, Wright S, Kisuule F. The effect of racial and gender concordance between physicians and patients on the assessment of hospitalist performance: a pilot study. BMC Health Serv Res. 2019;19(1):247. 17. Valantine HA, Collins FS. National Institutes of Health addresses the science of diversity. Proc Natl Acad Sci U S A. 2015;112(40): 12240-12242. 18. Rotenstein LS, Jena AB. Lost Taussigs - The Consequences of gender discrimination in medicine. N Engl J Med. 2018;378(24):2255-2257. 19. Nivet MA, Castillo-Page, L. Diversity in the Physician Workforce; Facts & Figures. 2014. Washington DC: Association of American Medical Colleges; 2014. 20. Committee on Pediatric Workforce. Enhancing pediatric workforce diversity and providing culturally effective pediatric care: implications for practice, education, and policy making. Pediatrics. 2013;132(4):e1105-e1116.

21. Cohen JJ, Gabriel BA, Terrell C. The case for diversity in the health care workforce. Health Aff (Millwood). 2002;21(5):90-102. 22. Mendoza FS, Walker LR, Stoll BJ, et al. Diversity and inclusion training in pediatric departments. Pediatrics. 2015;135(4):707-713. 23. Jagsi R. Sexual harassment in medicine: #MeToo. N Engl J Med. 2018;378(3):209-211. 24. Files JA, Mayer AP, Ko MG, et al. Speaker introductions at internal medicine grand rounds: forms of address reveal gender bias. J Womens Health (Larchmt). 2017;26(5):413-419. 25. Mehta S, Rose L, Cook D, Herridge M, Owais S, Metaxa V. The speaker gender gap at critical care conferences. Crit Care Med. 2018;46(6):991-996. 26. Maxwell AR, Riley CL, Stalets EL, Wheeler DS, Dewan M. State of the unit: physician gender diversity in pediatric critical care medicine leadership. Pediatr Crit Care Med. 2019;20(7):e362-e365. 27. Jena AB, Khullar D, Ho O, Olenski AR, Blumenthal DM. Sex differences in academic rank in US medical schools in 2014. JAMA. 2015;314(11):1149-1158. 28. Sege R, Nykiel-Bub L, Selk S. Sex differences in institutional support for junior biomedical researchers. JAMA. 2015;314(11):1175-1177. 29. Weaver AC, Wetterneck TB, Whelan CT, Hinami K. A matter of priorities? Exploring the persistent gender pay gap in hospital medicine. J Hosp Med. 2015;10(8):486-490. 30. Lo Sasso AT, Richards MR, Chou CF, Gerber SE. The $16,819 pay gap for newly trained physicians: the unexplained trend of men earning more than women. Health Aff (Millwood). 2011;30(2):193201. 31. Darwin JR, Selvaraj PC. The effects of work force diversity on employee performance in Singapore organisations. Int J Bus Admin. 2015;6(2). 32. Herring C. Does diversity pay?: Race, gender, and the business case for diversity. Sociol Rev. 2009;74(2). 33. Tulshyan R. Racially diverse companies outperform industry norms by 35%. Forbes. 2015 January 30. 34. Donovan AL, Aldrich JM, Gross AK, et al. Interprofessional Care and Teamwork in the ICU. Crit Care Med. 2018;46(6):980990. 35. Zimmerman BA. A piece of my mind. Patient’s sister, seeking job. JAMA. 2013;309(19):2003-2004. 36. Womack JP, Jones DT. Lean Thinking. 2nd ed. New York: Simon & Schuster, Inc.; 2003:397. 37. Ohno T. Toyota Production System: Beyond Large-Scale Production. Portland, OR: Productivity Press; 1988. 38. Ma H, Sun H, Sun X. Survival improvement by decade of patients aged 0-14 years with acute lymphoblastic leukemia: a SEER analysis. Sci Rep. 2014;4:4227. 39. Meade MO, Ely EW. Protocols to improve the care of critically ill pediatric and adult patients. JAMA. 2002;288(20):2601-2603. 40. Bernard C. Pensées: Notes Detachées. Bailliere et Fils. Paris; 1937. 41. Goldstein JL. On the origin and prevention of PAIDS (paralyzed academic investigator’s disease syndrome). J Clin Invest. 1986;78(3): 848-854. 42. Rivara FP, Alexander D. Randomized controlled trials and pediatric research. Arch Pediatr Adolesc Med. 2010;164(3):296-297. 43. Nowak JE, Brilli RJ, Lake MR, et al. Reducing catheter-associated bloodstream infections in the pediatric intensive care unit: Business case for quality improvement. Pediatr Crit Care Med. 2010;11(5):579587. 44. Edwards JD, Herzig CT, Liu H, et al. Central line-associated blood stream infections in pediatric intensive care units: longitudinal trends and compliance with bundle strategies. Am J Infect Control. 2015;43(5):489-493. 45. Ely EW. The ABCDEF bundle: science and philosophy of how ICU liberation serves patients and families. Crit Care Med. 2017; 45(2):321-330. 46. Barnes-Daly MA, Phillips G, Ely EW. Improving hospital survival and reducing brain dysfunction at seven california community

References

e2







48. Walrath JM, Muganlinskaya N, Shepherd M, Awad M, Reuland C, Makary MA, et al. Interdisciplinary medical, nursing, and administrator education in practice: the Johns Hopkins experience. Acad Med. 2006;81(8):744-748. 49. Allen JA, Reiter-Palmon R, Crowe J, Scott C. Debriefs: teams learning from doing in context. Am Psychol. 2018;73(4):504-516.





hospitals: implementing PAD guidelines via the ABCDEF bundle in 6,064 patients. Crit Care Med. 2016. 47. Pun BT, Balas MC, Barnes-Daly MA, et al. Caring for critically ill patients with the ABCDEF bundle: results of the ICU Liberation Collaborative in over 15,000 adults. Crit Care Med. 2019;47(1): 3-14.

e3

Abstract: A learning healthcare system occurs when patient care, interdisciplinary education, and clinical research are so integrated and intercalated that they are basically inseparable. Each element synergistically benefits from and informs the other. Benefits of a learning healthcare system include generation or identification of best available evidence to support best practice and promoting wellness and resiliency among critical care providers.

Key Words: Learning healthcare system, diversity, inclusion, bestpractice clinical care, clinical research, quality improvement, shared educational model, evidence-based medicine, burnout, wellness, resiliency

8 Challenges of Pediatric Critical Care in Resource-Poor Settings AMÉLIE VON SAINT ANDRÉ–VON ARNIM, JHUMA SANKAR, ANDREW ARGENT, AND ERICKA FINK





Life-threatening illnesses are a global phenomenon with markedly disparate outcomes depending on available resources and access to care. Low- to middle-income countries (LMICs) are economies defined by a gross national income per capita of $995 or less, and $996 to $3895 in 2017, respectively (eFig. 8.1).1 In high-income countries (HICs), caring for critically ill patients involves a coordinated system of (1) triage, (2) transport networks, (3) emergency and intensive care provided in wellresourced units and by trained personnel with (4) access to contemporary laboratory services, (5) imaging, (6) transfusion, and (7) surgical services. This cohesive system is resource intensive and, hence, less affordable for many LMICs, where care is fragmented. The burden of critical illness remains inordinately high in LMICs, despite an overall decrease in global childhood mortality (Fig. 8.2).2 Thus, access to quality care for the critically ill child with sudden and serious reversible disease, in addition to trauma and postoperative critical care support, should be a universal shared goal. Delivery of critical care in low-resource settings (LRSs) is in need of a tiered approach to scaling toward a gold standard that includes both strengthening capacity for public health and critical care services. For the purposes of this chapter, we define pediatric critical care as the care of children who suffer an acutely life-threatening illness or injury regardless of the location where care is provided. For example, irrespective of the setting—whether in a district health center with minimal resources and personnel or a tertiary care













Global child mortality is declining due to decreasing poverty and increasing basic medical care access and quality. Given the large burden and high mortality of critical illness and availability of low-cost therapies, there is ample rationale for expanding critical care services in least-developed countries. Pediatric critical care services do not have to be costly, nor do they need to be overtly reliant on high-end technology.









PEARLS Publicly funded intensive care unit treatment remains limited in low-income countries (LICs), and its introduction requires careful resource allocation. Healthcare systems improvements for the critically ill should involve a graded approach of strengthening capacity to provide health maintenance, basic critical care, and publicly funded intensive care services as overall health indices improve. Critical care research from LICs is sorely needed to guide effective and efficient care and advocate for resources.

setting—treatment of severe lower respiratory infections, malaria, or diarrhea with dehydration is critical care.5 In contrast, intensive care is defined as care provided for the critically ill or injured or those who have undergone major surgical procedures in an intensive care unit (ICU) with mechanical ventilators and equipment for close patient monitoring.

Child Mortality Rates Current Trends and Health Maintenance Globally, child and adolescent deaths decreased 51.7%, from 13.77 million in 1990 to 6.64 million in 2017.6 However, aggregate disability increased 4.7% to a total of 145 million years lived with disability globally.6 Progress was uneven and inequity increased, with low- and low- to middle-income regions experiencing 82.2% of deaths, up from 70.9% in 1990. The gains are partly attributable to attention by individual countries to the Millennium Development Goals (MDGs), especially MDG 4, which was related to decreasing the under-5-years-old mortality rate by two-thirds by 2015 from 1990 baseline. The overall improvements in other sectors—poverty, water, sanitation and hygiene, and socioeconomic indices—along with increasing vaccination rates, basic education, access to perinatal and other medical care and improving quality of care, have further helped to reduce mortality in infants and children globally. 51

51.e1 The world by income

Classified according to World Bank estimates of 2016 GNI per capita (current US dollars, Atlas method) Low income (less than $1,005) Lower middle income ($1,006–$3,955) Upper middle income ($3,956–$12,235)

High income (more than $12,235) No data

Note: The World Bank classifies economies as low-income, lower-middle-income, upper-middle-income or high-income based on gross national income (GNI) per capita. For more information see https://datahelpdesk.worldbank.org/knowledgebase/articles/906519-world-bank-country-and-lending-groups.

East Asia and Pacific American Samoa Australia Brunei Darussalam Cambodia China Fiji French Polynesia Guam Hong Kong SAR, China Indonesia Japan Kiribati Korea, Dem. People’s Rep. Korea, Rep. Lao PDR Macao SAR, China Malaysia Marshall Islands Micronesia, Fed. Sts. Mongolia Myanmar Nauru New Caledonia New Zealand Northern Mariana Islands Palau Papua New Guinea Philippines Samoa Singapore Solomon Islands Thailand Timor-Leste Tonga Tuvalu Vanuatu Vietnam

Europe and Central Asia Upper middle income High income High income Lower middle income Upper middle income Upper middle income High income High income High income Lower middle income High income Lower middle income Low income High income Lower middle income High income Upper middle income Upper middle income Lower middle income Lower middle income Lower middle income Upper middle income High income High income High income High income Lower middle income Lower middle income Upper middle income High income Lower middle income Upper middle income Lower middle income Upper middle income Upper middle income Lower middle income Lower middle income

•  eFig. 8.1

Albania Andorra Armenia Austria Azerbaijan Belarus Belgium Bosnia and Herzegovina Bulgaria Channel Islands Croatia Cyprus Czech Republic Denmark Estonia Faroe Islands Finland France Georgia Germany Gibraltar Greece Greenland Hungary Iceland Ireland Isle of Man Italy Kazakhstan Kosovo Kyrgyz Republic Latvia Liechtenstein Lithuania Luxembourg Macedonia, FYR Moldova Monaco

Upper middle income High income Lower middle income High income Upper middle income Upper middle income High income Upper middle income Upper middle income High income Upper middle income High income High income High income High income High income High income High income Lower middle income High income High income High income High income High income High income High income High income High income Upper middle income Lower middle income Lower middle income High income High income High income High income Upper middle income Lower middle income High income

Montenegro Netherlands Norway Poland Portugal Romania Russian Federation San Marino Serbia Slovak Republic Slovenia Spain Sweden Switzerland Tajikistan Turkey Turkmenistan Ukraine United Kingdom Uzbekistan

Upper middle income High income High income High income High income Upper middle income Upper middle income High income Upper middle income High income High income High income High income High income Lower middle income Upper middle income Upper middle income Lower middle income High income Lower middle income

Latin America and the Caribbean Antigua and Barbuda Argentina Aruba Bahamas, The Barbados Belize Bolivia Brazil British Virgin Islands Cayman Islands Chile Colombia Costa Rica Cuba Curaçao Dominica Dominican Republic Ecuador El Salvador

High income Upper middle income High income High income High income Upper middle income Lower middle income Upper middle income High income High income High income Upper middle income Upper middle income Upper middle income High income Upper middle income Upper middle income Upper middle income Lower middle income



​The world by income. The World Bank classifies economies as low income, lower middle income, upper middle income or high income based on gross national income (GNI) per capita.3

52

SECTION I



Pediatric Critical Care: The Discipline

CAUSES OF UNDER-5 DEATHS IN LMIC Typhoid and other SIDS 1% Salmonella 1% Liver and digestive disorders Other infections 1% 1% Drowning 1%

Tuberculosis 1%

Whooping cough 1%

Measles 2% HIV/AIDS 2% Others 5% Injuries and falls 2%

Neonatal disorders 34%

Sexually transmitted infections excluding HIV 2% Protein-energy malnutrition 2%

Meningitis 3%

Malaria 7%

Congenital birth defects 8%

Diarrheal diseases 11%

Lower respiratory infections 16%

•  Fig. 8.2

​Causes of under-5-years-old mortality in low to middle sociodemographic index (SDI) settings globally.4 HIV/AIDS, Human immunodeficiency virus/acquired immunodeficiency syndrome; LMICs, lowermiddle-income countries; SIDS, sudden infant death syndrome.  

However, in regions such as sub-Saharan Africa, 1 out of every 12 children still dies before age 5 years, nearly 16 times the average rate in HICs.7 The majority of childhood deaths under the age of 5 years are related to neonatal problems (34%), followed by lower respiratory (16%) and diarrheal illnesses (11%), as well as malaria (7%). In 5- to 14-year-old children, road injuries (8%) also play an important role (see Fig. 8.2). The five countries with the highest number of under-5-years-old deaths in 2017 were Somalia, Chad, Central African Republic, Sierra Leone, and Mali.1 In more than a quarter of all countries globally, urgent action is needed to accelerate reductions in child mortality to reach the Sustainable Development Goals (SDG) targets. These targets include ending preventable child deaths and decreasing under-5-years-old mortality to at least as low as 25 deaths per 1000 live births by 2030.8 It is possible that critical care services will be necessary in combination with more basic public health measures to achieve this goal. Critical care is just the continuum of care provided to any child with a

life-threatening illness or injury beginning with the time of presentation to a healthcare facility. However, extracting optimal value from critical care treatment in LMICs depends on a deep understanding of the resources required to provide incremental levels of care to critically ill children, as well as a focus on the interventions that will make the most difference.

Justification for Critical Care in Resource-Poor Settings The full scope of the burden of critical illness in resource-limited settings is unknown, but the majority of global child deaths occur in LRSs.9,10 Additionally, children younger than 15 years represent 50% of the population in LMICs.11 Deficiencies in timely and equal access to quality healthcare, emergency triage and transport, and lack of early recognition contribute to increased child deaths in LRSs.12–14 The need for critical care support is likely to



CHAPTER 8

Challenges of Pediatric Critical Care in Resource-Poor Settings

rise with increasing urbanization, more frequent epidemics, and natural disasters.15–17 More work is necessary to better define the burden of pediatric critical illness in LRSs to support provision of resources for critical care delivery. Current approaches to estimate the global burden of critical illness have significant limitations18: (1) counting patients admitted to ICUs around the world, (2) extrapolating from resource-rich countries’ epidemiology, and (3) using the assumption that all deaths occurring in a region had a critical illness at some stage before their demise. The first two approaches will likely lead to an underestimation; the third will lead to an overestimation of the burden of critical illness in resource-poor settings. Effective, low-cost strategies for the management of critical illnesses are becoming more available, providing ample rationale for expanding pediatric critical care services to LRSs, especially in countries with under-5-years-old mortality rates of less than 30 per 1000, while regions with higher mortality rates are advised to focus on public health capacity.19

State of Critical and Intensive Care Delivery in Resource-Limited Settings In HICs, critical care services usually involve “a coordinated system of triage, emergency management, and ICUs” providing contemporary standards of care to the population.20 While some urban university and private hospitals in sub-Saharan Africa, India, and China may offer critical and intensive care services approaching that of HICs, in many LMICs, healthcare systems are less organized, human and material resources are scant, and intensive care services are few or nonexistent—especially at districtlevel hospitals, where about 50% of medical care in Africa is administered.21–25 Even quality of care for common childhood illnesses such as pneumonia and diarrheal diseases is poor in these settings.26 Logistic and financial limitations, poorly resourced supporting disciplines (e.g., laboratories, radiology, nursing), underlying malnutrition, delayed presentation of severely sick children, and suboptimal care contribute to comparatively high mortality.25–28 An ICU in a public hospital in LRSs may provide pressurized air or oxygen, but mechanical ventilation, renal replacement therapy, and basic supplies are limited.29 Systematic efforts to understand the disease epidemiology of a region, its prognosis, and development of policies and guidelines of critical care in LRSs are required to best use available resources.27 Most pediatric intensive care in LMICs is performed in mixed adult-pediatric ICUs. The majority of the pediatric ICUs (PICUs) are staffed by general pediatricians and lack specialized services.30 Where ICUs are available, the most common reasons for admission are for postsurgical and trauma care, infectious diseases, and peripartum maternal or neonatal complications.27 These conditions are major contributors to the global burden of disease. Hence, building intensive care capacity around the relevant disciplines where they already exist is a reasonable method to increase capacity.

Approach to Basic Critical Care in Resource-Limited Settings Pediatric critical care services do not have to be costly, nor do they need to be overtly reliant on high-end technology. Critically ill children in LMICs may benefit from timely care and closer monitoring even without an ICU. Critical care services can help improve outcomes if combined with a focus on community

53

recognition of serious illness, early access to care, referral, and safe transport.30 Some successful interventions include training villagers in basic first aid and resuscitation31; provision of low-cost simplified antimicrobial regimens and rapid diagnostic tests to rural healthcare workers, day clinics, and homes32,33; quality improvement of district hospital services34,35; reorganization of emergency services at referral hospitals36,37; provision of oxygen therapy for hypoxemic children with pneumonia in district clinics; and medical treatment given by village workers or parents.38–40 Noninvasive respiratory support, such as bubble-CPAP (continuous positive airway pressure) has been successfully and cost-effectively introduced for support of neonates and infants with respiratory compromise in LRSs.41–47 Use of simple nurse-initiated CPAP protocols for children up to 5 years of age has been safely introduced to nontertiary care hospitals in Ghana, where invasive mechanical ventilation is not routinely available, and has led to significantly decreased mortality for infants in the CPAP group.48

Cost Considerations in Critical Care Delivery Intensive care involving ICUs and mechanical ventilation is expensive, in contrast to the much lower-cost basic critical care interventions described earlier in this chapter. It is important to consider the relative costs of intensive care services.49 For example, it is estimated that the costs of 1 day in an HIC ICU in 2009 was around $1000.20 This would be equivalent to approximately 20% of the annual per capita expenditure on health in an HIC, but approximately 30 times the annual per capita expenditure on health in a low-income country (LIC). The cost of delivering some aspects of critical care in poorer countries may be substantially lower than this: in India, 1 day in a private ICU for cancer patients was reported as $57. However, in settings where a substantial proportion of healthcare costs is covered by families’ outof-pocket expenditure, that amount has to be related to the family income, here estimated to be approximately 100 times the average per capita household income.50 Hence, intensive care services have the potential to devastate the financial structure of a family in LMICs.51 Cost-effectiveness analyses for intensive care in LICs must also consider the potential hidden infrastructure costs required (e.g., transport, electricity, water provision and sewage disposal, medical gas supply systems, technical maintenance support) to support complex medical care.49 The real cost of providing intensive care in poorer countries may be substantially higher in these circumstances, although possibly offset by the lower salaries for healthcare workers. The differences in access to high-cost healthcare between rich and poor people may vary substantially in many LMICs, with a small proportion of the population having access to state-of-theart medical services while the majority of people within the same country may have limited or virtually nonexistent access to healthcare services.52 Hence, cost-effectiveness of intensive care services in LMICs is complex. Care must be taken to not burden fragile systems with costly interventions that may take away limited resources from other public healthcare sectors while having only a limited impact or poor outcomes with long-term cost. It is important to note, however, that ICUs in high-income countries function in large part by admitting and caring for complex patients with often incurable or chronic diseases, whereas in many resource-limited settings, the ICU provides basic rescue interventions for children and young adults who are ill with curable diseases. Hence, a short duration of intensive care that (selectively) treats reversible acute life-threatening illnesses affecting millions

54

SECTION I



Pediatric Critical Care: The Discipline

of young people in LRSs worldwide may be cost-effective long term.53 Examples of some high-cost interventions associated with excellent outcomes and minimal long-term costs might be mechanical ventilation for pneumonia in an otherwise healthy child or intensive care to enable a major curative surgery. As mentioned earlier, there is evidence that relatively low-cost interventions in the care of critically ill children and improved organization of emergency services within a hospital36 may be associated with substantial reductions in mortality without significant added expense. Implementation of nasal CPAP in Nicaragua could provide improved outcomes while reducing invasive mechanical ventilation,54 while mechanical ventilation in LICs, on the other hand, has been associated with relatively high mortality.55,56 Research into cost-effective interventions in the setting of curable diseases is needed to further understand which high- versus low-cost interventions are appropriate and sustainable and related to good versus less desirable outcomes. This approach could help allocate resources, mutually benefiting HICs and LMICs in reducing the cost of care and improving quality of care.57

Ethics of Intensive Care in ResourcePoor Settings There is no ethical justification for differences in healthcare access for children across the world. However, dealing with the realities of those differences remains profoundly challenging, especially in terms of development of publicly funded intensive care services in resource-poor settings. The focus areas include global justice, family and cultural preferences, and resource allocation. 1. Global justice: According to the global justice argument, healthcare services are a fundamental and universal human right that must be available to everyone.27 The just distribution of healthcare services across all human populations remains a serious challenge. Global justice would imply that those in resource-rich regions have a responsibility to combat critical illness and strengthen healthcare infrastructure for those in resource-poor settings.58 In countries with per-capita healthcare investments less than $100 per person per year, the cost of intensive care renders its provision for everyone impractical. To attempt to provide access to ICUs at the standard of care accepted generally in the HICs will lead to serious distortion of healthcare budgets and have detrimental effects on overall health of the nation. Alternatively, access to different standards of care or differences in quality and breadth of critical care will develop or already has developed in LMICs. In such an inegalitarian healthcare policy, which is common in many developing nations, “centers of excellence can be maintained where critical care technologies can be nurtured so that as a country develops and increases its standards of care, it will have its home-grown critical care resources on which to draw.”59 Such centers can harbor the skilled personnel within a country who can serve as catalysts in expanding high-quality care as economic resources increase and mortality gains are realized. In these systems, the key lies in extreme efficiency measures and meticulous attention to balancing the needs of individuals versus those of the population. 2. Family and cultural preferences: Providing ICU-level care also requires respecting family, cultural, and religious preferences. Different practices are present in different countries with respect to who is accepted as the appropriate decision maker on







initiation or discontinuance of critical care services. In particular, such decisions are often made by the family rather than patient.60 In many African countries, the idea of advanced directives and code status have yet to be discussed at the medical community and national judicial level.61 If the outcome is long-term morbidity or death, the decision to provide advanced or further care needs to be made based on the risk of impoverishment of the family.62 Financial nonaffordability is being stated as reason for withdrawal of care even for patients improving in their course of illness in some countries.63 3. Resource allocation: Resource allocation of intensive care services in deprived areas of the world is challenging. Children must not come to the ICU to die, yet many who are critically ill, with an increased risk of death, may have a better chance of recovery if treated in the ICU.64 Hence, each institution offering ICU-level care should define ICU admission and exclusion criteria. As an example, the Red Cross War Memorial Hospital in South Africa published explicit patient exclusion criteria for offering critical care in an attempt to provide a reasonable process for fair and equitable utilization of scarce resources.65 Some of these exclusion criteria include children declared brain dead or status post–cardiac arrest without establishment of normal respiratory pattern; children with underlying lethal conditions, such as children with burns on more than 60% of their body surface area; children with chronic renal failure with no ability to commit to long-term dialysis; children with severe/lethal chromosomal anomalies; children with malignancies nonresponsive to therapy; or inoperable cardiac lesions. Implementation of such ICU admission policies should be undertaken with great sensitivity to both the family and staff or it is likely to lead to failure, anger, and cynicism. Careful decisions regarding the extent of ICU support need to be made in the context of potential impoverishment of the entire extended family, especially if the end result is a child’s death or long-term morbidity. Suggested ways to address ethical dilemmas of resource allocation in resource-poor settings include the following: obtaining data on disease prognosis with and without ICU care to inform clinical decision-making; development of procedures for addressing levelof-care decisions openly and honestly; and articulating hospital policies on the use of critical care services, including policies regarding appropriate ICU admissions, cardiopulmonary resuscitation, and ventilator candidates.61



Strengthening Critical Care Infrastructure Healthcare Systems The Ebola epidemic in 2014 to 2016 brought to light the fragile state of healthcare systems in the affected LMICs.27 The needs of Ebola patients are what constitute the basics of public health: early identification, good supportive care along with transport services and isolation practices, and safe disposal of medical waste. Building critical care services in LICs requires a tiered approach focused on delivering evidence-based, basic, effective healthcare interventions to scale and carefully making decisions about how to improve existing care.66,67 To inform such decisions, information is needed on local and regional major causes of morbidity and mortality, what interventions are being delivered to whom, and with what outcomes. Marked variability in care and outcomes may demonstrate the need for efforts to support implementation of key interventions or wider system strengthening in



CHAPTER 8

Challenges of Pediatric Critical Care in Resource-Poor Settings

LICs. In Kenya, a clinical information network (CIN) focused on hospitals’ inpatient pediatric units has been established to foster better generation and ultimately better use of information, since more comprehensive health information systems remain limited.68,69 Partners in this effort include the Ministry of Health, the Kenya Paediatric Association, local hospitals, and a research team. Initial data from this CIN in Kenya show that quality of care in district hospitals varies across hospitals. Disease conditions and processes of care suggest that there is significant opportunity for quality improvement in pediatric care.68 These CINs and health information systems can then play an important cross-cutting role in supporting local improvement efforts, benchmarking, and tracking adoption of interventions.70,71 As an example, a survey of Kenyan government hospitals revealed a median availability of essential antibiotics in only 36% of the 22 surveyed facilities, with a wide range of essential resource availability from 49% to 93%.26 Health workers at these hospitals were unable to provide appropriate care for severely ill newborns or children owing to inadequacies in key tasks, such as prescription of antibiotics and feeds, even when resources were available. In attempting to rectify these deficiencies in care, the level of engagement of senior and particularly midlevel clinical managers was important.72 The simple availability of authoritative World Health Organization and national guidelines alone does not improve hospital care for children as hoped. Broader, more system-oriented interventions addressing the many important influences on provider or user behavior are necessary.73 A multifaceted approach addressing deficiencies in knowledge, skills, motivation, and organization of care using face-to-face feedback on performance, supportive supervision, and provision of a local facilitator resulted in more sustained improvements of pediatric care in Kenya.35 Building regional centers of excellence to grow publicly funded intensive care resources and capacity over time may be a reasonable first step. However, there is no approach to improve healthcare systems that is suitable for all countries, and not all approaches are congruent with local values and ideologies or acceptable to all governments or their constituencies. Thus, healthcare strengthening must be seen as a long-term process that involves complex systems and requires carefully orchestrated action on a number of fronts.58 Over the last 2 decades, there has been increasing private participation in the healthcare systems of LMICs, especially in-service delivery. The increase in the number of private providers is driven by both rising incomes and the failure of public services to meet expectations.74 The engagement of the private sector is a topic of considerable controversy, seen by some as inviting the privatization of healthcare and making it a commodity. However, when the capacity of the public sector is limited and there is a concentration of human resources in the private sector, seeking a mix of public and private provision of services can be seen as a pragmatic response and may spur increases in public services and care delivery.75

Pediatric Critical Care Capacity Building Through Education The health workforce shortage remains a huge problem in LMICs.76 Insufficient training capacity and the “brain drain” of health professionals from Africa are principal drivers of the current situation.76,77 Health professional schools in LMICs face notable limitations in physical space, equipment, curricula,

55

training materials, faculty, administrative staff, and funding,78–80 making it challenging to expand the number and diversity of training programs and to improve the quality of training. Practicing health professionals are often overwhelmed by the grinding work of delivering health services in undersupplied and overcrowded healthcare facilities, inadequately compensated for their work, and demoralized by a lack of career opportunities.76,77,81 Several innovative training initiatives in sub-Saharan Africa funded by the US government have tried to address these issues, including the Medical Education Training Partnership Initiative, Nursing Training Partnership Initiative, Rwanda Human Resources for Health Program, and the Global Health Service Partnership. The best practices adopted by these initiatives are (1) alignment to local priorities; (2) country ownership; (3) competency-based training; (4) institutional capacity building; and (5) the establishment of long-lasting partnerships with international stakeholders.82 While some of these programs support components of emergency medicine and critical care, pediatric critical care training opportunities in LMICs are limited (Table 8.1). Most of the world’s sickest children are cared for outside of conventional ICUs (pediatric or mixed) by caregivers without pediatric or critical care training.30,83 The African Paediatric Fellowship Programme at the University of Cape Town provides pediatric subspecialty training for African child health professionals, by Africans, within Africa.84 Trainees identified by partner academic institutions spend 6 months to 2 years training in a pediatric subspecialty, including pediatric critical care. Graduates then return to their home institution to build practice, training, research, and advocacy. Similarly, the University of Nairobi, in collaboration with the University of Washington, have recently implemented the first pediatric emergency and critical care fellowship program in East Africa for African pediatricians.85 In this model, the limited number of pediatric emergency and critical care physicians in Kenya are supported in the teaching process by visiting subspecialists and online educational resources. Pediatric critical care nursing and midlevel provider training programs are also scarce in LMICs and much needed. In Nigeria, for example, there are only 380 ICU-trained nurses in a country of 140 million people.86 “Training the trainer” is widely used as an efficient and effective approach to addressing the shortage of healthcare workers in LMICs through upskilling and to improving their performance, commitment, and— ultimately—retention. However, long-term sustainability of this model depends on multiple factors.87 Standardized pediatric emergency and critical care curricula— such as emergency, triage, assessment and treatment; pediatric basic assessment and support intensive care; and pediatric fundamentals of critical care study—are other educational initiatives used for supporting critical care training in resource-limited settings.88 These courses provide a number of advantages even if they require some degree of adaptation to be effective in resource-poor settings. Standardized courses are readily deployable for various training levels in a range of settings; they are consistent and comprehensive.89,90 Short courses—such as the pediatric, emergency, assessment, recognition and stabilization, and pediatric advanced life support courses—have been bundled and used on systemwide levels to improve the care of sick children in Botswana and India.91 However, data on the utility of short-term training programs, both in HICs and LMICs, are limited—they have been shown to improve short-term knowledge in emergency and critical care in resource-poor settings.92,93 Short courses on trauma care have been shown to identify deficiencies, increase provider skills, and improve trauma outcomes—including mortality—in

56

SECTION I



Pediatric Critical Care: The Discipline

TABLE Examples of Pediatric Critical Care Education Available in Lower-Middle-Income Countries 8.1

Programs

Organization

Program Details

The African Paediatric Fellowship Programme

University of Cape Town, South Africa

African doctors spend 6 months to 2 years training in a pediatric subspecialty, including pediatric critical care. Graduates then return to their home institution to build a practice, training, research, and advocacy.

Pediatric Emergency and Critical Care–Kenya

University of Nairobi and AIC Kijabe Hospital, Kenya in collaboration University of Washington, Seattle, Washington

Two-year program in pediatric emergency and critical care for African pediatricians. Curriculum based on East African disease spectrum and resources.

Emergency, triage, assessment and treatment (ETAT)

World Health Organization

Teaches health workers of all levels to appropriately triage sick children on arrival to a health facility and to provide emergency treatment for lifethreatening conditions.

Pediatric basic assessment and support intensive care (BASIC)

Created by leaders in pediatric critical care education

Teaches nonintensivists the essential principles of recognizing and initiating care for the critically ill child in the absence of an intensivist; low cost.

Pediatric fundamentals of critical care study (PFCCS)

Society of Critical Care Medicine

Teaches nonintensivists the essential principles of recognizing and initiating care for the critically ill child in the absence of an intensivist. Tiered course pricing based on a country’s gross domestic income.

Pediatric advanced life support (PALS)

American Heart Association

Teaches pediatric healthcare provider in developing the knowledge and skills necessary to efficiently and effectively manage critically ill infants and children; targeted toward pediatricians, emergency physicians, family physicians, physician assistants, nurses, nurse practitioners, and paramedics.

Pediatric, emergency, assessment, recognition and stabilization (PEARS)

American Heart Association

Teaches assessment, early recognition, prompt communication, and initial intervention in pediatric critical illness by using high-performance team dynamics; targeted toward physicians and nurses not specializing in pediatrics, nurse practitioners, and physician assistants.

Trauma team training (TTT)

Canadian Network for International surgery grants access to course materials

Low-cost course designed to teach a multidisciplinary team approach to trauma evaluation and resuscitation with the limited resources found at African rural hospitals and health centers.

Fellowships

Short Courses

LMICs, such as Trinidad, India, Ecuador, and Tanzania.94–97 In fields such as pediatric surgery and obstetrics, short-term, specialized training courses have been correlated to increased knowledge retention.98,99 Training of Kenyan doctors taking the Fundamental Critical Care Study course has been shown to increase their knowledge of and confidence in new critical care skills.100 A Cochrane review assessed the effects of in-service emergency care training on health professionals’ treatment of severely ill newborns and children in LICs.101 It included only two neonatal resuscitation studies, both of which suggested a beneficial effect on health provider outcomes (resuscitation practices, assessment of breathing, resuscitation preparedness) in the short term. However, the effects on neonatal mortality outcomes were inconclusive, and improvement in health provider practices after training was not generalizable. Therefore, decisions to scale up life support courses in LICs “must be based on consideration of costs and logistics associated with their implementation, including the need for adequate numbers of skilled instructors, appropriate locally adapted training materials and the availability of basic resuscitation equipment.”101 With the broad availability of mobile technology in LMICs, there are increasing opportunities for mobile apps to improve training of healthcare providers, for example, as a mode for continued medical education or to assistance with resuscitation algorithms.102 In addition, the use of telemedicine can be beneficial

for LMICs, both for improved access to basic and subspecialty care and to promote learning and professional development.103 The global community can help by supporting country-led processes of reform and capacity building in education and by helping to create a stronger evidence base that contributes to cross-country learning.

Critical Illness During Public Health Emergencies As past H1N1 influenza, severe acute respiratory syndrome, and Ebola outbreaks have clearly illustrated, improved preventive and disease surveillance strategies are necessary in LMICs, but also coordinated emergency and critical care resources are critical for saving lives during epidemics. Globally, increasing urbanization, ease of travel, natural disasters, and regional war and conflict increase the risk of infectious outbreaks and need for critical care resources for potentially large numbers of seriously ill patients within a short period of time.104 Mass critical care preparedness in resource-limited settings is only recently being more systematically addressed.105 Use of technology to identify and communicate outbreaks may limit the impact of outbreaks and facilitate triage to critical care resource locations.106,107 Further, an international consensus statement emphasizes the need to develop resilient



CHAPTER 8

Challenges of Pediatric Critical Care in Resource-Poor Settings

healthcare systems to prepare for disaster and mass critical care preparedness in resource-poor settings.108 Recommendations include strengthening the primary care, basic emergency care, and public health systems and building critical care capacity in the fields with the highest burden of disease—such as surgery, obstetrics, internal medicine, and pediatrics.108 District- and regionallevel health centers should develop at least a minimal level of critical care. Therefore, to improve capacity building and quality of care at district hospitals, performance improvement activities should be instituted. Prehospital care and transport of the critically ill could be improved through community-level education of medical and nonmedical laypersons. Highlighting implementation of these ideas, regional capacity building, emergency preparedness training, and other public health efforts to improve future response to outbreaks were recently described for West African nations affected by Ebola.109

How to Develop an ICU in Low- to MiddleIncome Countries Organizing a PICU in resource-poor settings is associated with many challenges necessitating appropriate planning and proper utilization of limited available resources. The building blocks of pediatric critical care services in any setting comprise specialized training of healthcare professionals, including nursing staff, in terms of knowledge, skills and teamwork, resource-appropriate equipment selection, and adequate space for each patient. Also key is support of administration and leaders for logistics and appropriate settings to provide services.30 Team-based training is essential for the success of intensive care programs. The aim should be to master the basic skills and knowledge essential for pediatric critical care, such as resuscitation skills, with effective communication among team members. Simulation-based training has been shown to reinforce skills and teamwork, improving outcomes, and is feasible in LMICs.14,30,61 The bed space as per PICU guidelines in HICs may not be feasible in LMICs because of lack of infrastructure and manpower. Judicious use of space is needed for appropriate care of high patient burden and to prevent cross-infection at the same time. Low-cost substitutes for expensive equipment may be used as appropriate and preferably with locally available technological services.30,61,110 As mentioned earlier, in order to improve outcomes and judicious use of resources, it is important to establish patient selection criteria for admission and discharge for every PICU along with policies for end-of-life care decisions.111 Other key important aspects in PICU development in LMICs are to have (1) infection control policies, (2) continuous supply of consumable items and medications, (3) laboratory and radiology support, and (4) data collection systems for measurement of PICU outcomes and complications.111 The success of an intensive care program depends on early recognition, timely referral, safe transport, good triage, and emergency care. Hence, it is important to strengthen these aspects of care at the community level as well as improving emergency services.30

Importance of Critical Care Research in Limited Resource Settings A large majority of published critical care research occurs in HICs. However, research findings in HICs may not directly translate into improved outcomes in LRSs. Recent breakthroughs in

57

pediatric resuscitation and critical care interventions in research programs conducted in LRSs have impacted clinical care in the following conditions: severe sepsis, in terms of fluid management; the benefits of early norepinephrine in patients with vasodilatory septic shock; the increased risk of dopamine (vs. epinephrine) as a first-line vasoactive agent in fluid refractory septic shock; and insights into the pathophysiology of cerebral malaria that informed an active clinical trial.112–118 There continues to be a vital need for investment in quality research programs that serve the unique needs of these children, as the limited evidence in this field hinders effective and efficient care and advocacy for resources.10,61 Reasons for this gap in evidence include lack of funding; lack of local critical care providers, researchers, and research staff; lack of academic mentorship, infrastructure, and training to do research; and barriers to producing publishable research.119 Additionally, establishing research networks in LRSs should be explored to leverage resources to support training, science quality, and capacity to accomplish more than would be done by individuals. The research agenda should prioritize increasing evidence regarding critical illness epidemiology and its outcomes in LRSs. A more accurate estimate of the potential lives saved through critical care would serve to prove its role in healthcare systems in resource-poor settings.18 Efficacy must be measured and validated for critical care interventions, with limited resources targeted to those practices that save lives, time, and resources. Dissemination of evidence and experience from successes and failures could help accelerate the pace of critical care infrastructure improvements. Data on critical care capacity and access to both critical care resources and healthcare professionals are essential for health system planning but generally are lacking for pediatrics. Recognizing that implementation of clinical care guidelines created using evidence from and for critical care provided in higher-resources settings may not be feasible, efforts should be made to generate and evaluate critical care guidelines for LRSs, especially for common conditions such as sepsis and coma.29,120 Cost-effectiveness analyses of current and proposed critical care practices need to be emphasized.61 Patient triage and clinical research would benefit from simple severity of illness scoring systems adapted and validated for resource-poor settings. With increased survival from pediatric critical illness has come the realization of post-ICU sequelae and late mortality and thus the need for services to support continued recovery from critical illness and successful community reintegration. For example, children admitted to the hospital in Uganda with infections had a 4.9% increased risk of mortality in the first 6 months following discharge.121 Further, in severely malnourished Bangladeshi children initially treated in the PICU for severe pneumonia, postdischarge (3 months) mortality was 8.6%.122 Identifying risk factors of children at high risk or morbidity and late mortality before hospital discharge and providing effective interventions should be another research priority. Low-cost critical care technology, such as noninvasive positivepressure ventilation, is much needed to support critical care in LRSs. Locally available, ubiquitous technology such as cell phones should be used to enable better healthcare seeking and delivery and solve clinical challenges. While mobile apps for critical care and resuscitation are becoming available, there is significant need for quality control.123,124 Technology development must be tightly woven into solving implementation challenges that result from not only technology cost and availability but also complexity of the political, social, and professional systems in LMICs.120

58

SECTION I



Pediatric Critical Care: The Discipline

Key References Argent AC, Ahrens J, Morrow BM, et al. Pediatric intensive care in South Africa: an account of making optimum use of limited resources at the Red Cross War Memorial Children’s Hospital. Pediatr Crit Care Med. 2014;15:7-14. Argent AC. Considerations for assessing the appropriateness of high-cost pediatric care in low-income regions. Front Pediatr. 2018; 6:68. Child GBD, Adolescent Health C, Reiner Jr RC, et al. Diseases, injuries, and risk factors in child and adolescent health, 1990 to 2017: findings from the global burden of diseases, injuries, and risk factors 2017 study. JAMA Pediatr. 2019:e190337. English M, Gathara D, Mwinga S, et al. Adoption of recommended practices and basic technologies in a low-income setting. Arch Dis Child. 2014;99:452-456. Opiyo N, English M. In-service training for health professionals to improve care of seriously ill newborns and children in low-income countries. Cochrane Database Syst Rev. 2015;5:CD007071.

Sankar J, Ismail J, Sankar MJ, C PS, Meena RS. Fluid bolus over 15-20 versus 5-10 minutes each in the first hour of resuscitation in children with septic shock: a randomized controlled trial. Pediatr Crit Care Med. 2017;18:e435-e445. Shann F. Role of intensive care in countries with a high child mortality rate. Pediatr Crit Care Med. 2011;12:114-115. Slusher TM, Kiragu AW, Day LT, et al. Pediatric critical care in resourcelimited settings-overview and lessons learned. Front Pediatr. 2018;6:49. von Saint Andre-von Arnim AO, Attebery J, Kortz TB, et al. Challenges and priorities for pediatric critical care clinician-researchers in lowand middle-income countries. Front Pediatr. 2017;5:277. Wilmshurst JM, Morrow B, du Preez A, Githanga D, Kennedy N, Zar HJ. The African pediatric fellowship program: training in Africa for Africans. Pediatrics. 2016;137(1). Wilson PT, Baiden F, Brooks JC, et al. Continuous positive airway pressure for children with undifferentiated respiratory distress in Ghana: an openlabel, cluster, crossover trial. Lancet Glob Health. 2017;5:e615-e623.

The full reference list for this chapter is available at ExpertConsult.com.

e1







































































































1. Databank. Agriculture & Rural Development. https://data.worldbank. org/indicator. 2. Child GBD, Adolescent Health C, Reiner Jr RC, et al. Diseases, injuries, and risk factors in child and adolescent health, 1990 to 2017: findings from the global burden of diseases, injuries, and risk factors 2017 study. JAMA Pediatr. 2019;173(6):e190337. 3. The World Bank. The World by Income and Region. 2019. https:// datatopics.worldbank.org/world-development-indicators/the-worldby-income-and-region.html. 4. Global burden of disease compare. https://vizhub.healthdata.org/ gbd-compare/. 5. Kissoon N, Argent A, Devictor D, et al. World Federation of Pediatric Intensive and Critical Care Societies-its global agenda. Pediatr Crit Care Med. 2009;10:597-600. 6. Global Burden of Disease Study 2017. Institute for Health Metrics and Evaluation (IHME). 2018. http://ghdx.healthdata.org/gbd-results-tool. 7. Collaborators GBDCM. Global, regional, national, and selected subnational levels of stillbirths, neonatal, infant, and under-5 mortality, 1980-2015: a systematic analysis for the Global Burden of Disease Study 2015. Lancet. 2016;388:1725-1774. 8. UNICEF Data: Monitoring the situation of children and women. Child Survival and the SDGs. https://data.unicef.org/topic/childsurvival/child-survival-sdgs/. 9. Children ACfHoWa. Accountability for Women’s and Children’s Health. Geneva: The World Health Organization; 2014. 10. Adhikari NK, Fowler RA, Bhagwanjee S, Rubenfeld GD. Critical care and the global burden of critical illness in adults. Lancet. 2010;376: 1339-1346. 11. 2018 World Population Data Sheet. 2018. http://www.prb.org. 12. Mendsaikhan N, Begzjav T, Lundeg G, Dunser MW. The epidemiology and outcome of critical illness in Mongolia: A multicenter, prospective, observational cohort study. Int J Crit Illn Inj Sci. 2016;6: 103-108. 13. Dunser MW, Baelani I, Ganbold L. A review and analysis of intensive care medicine in the least developed countries. Crit Care Med. 2006;34:1234-1242. 14. Murthy S, Adhikari NK. Global health care of the critically ill in low-resource settings. Ann Am Thorac Soc. 2013;10:509-513. 15. Murray CJ, Lopez AD. Measuring the global burden of disease. N Engl J Med. 2013;369:448-457. 16. Annez PC. An Agenda for Research on Urbanization in Developing Countries: A Summary of Findings From a Scoping Exercise. Geneva: The World Bank; 2010. 17. Adhikari NK, Rubenfeld GD. Worldwide demand for critical care. Curr Opin Crit Care. 2011;17:620-625. 18. Murthy SS, Adhikari KJ. Critical Care in Low-Resource Settings. New York: Springer Science1Business; 2014. 19. Shann F. Role of intensive care in countries with a high child mortality rate. Pediatr Crit Care Med. 2011;12:114-115. 20. Baker T. Critical care in low-income countries. Trop Med Int Health. 2009;14:143-148. 21. Duke T, Tamburlini G, Silimperi D, Paediatric Quality Care G. Improving the quality of paediatric care in peripheral hospitals in developing countries. Arch Dis Child. 2003;88:563-565. 22. Reyburn H, Mwakasungula E, Chonya S, et al. Clinical assessment and treatment in paediatric wards in the north-east of the United Republic of Tanzania. Bull World Health Organ. 2008;86:132-139. 23. Nolan T, Angos P, Cunha AJ, et al. Quality of hospital care for seriously ill children in less-developed countries. Lancet. 2001;357:106-110. 24. English MLC, Ngugi I, Smith PC. The District Hospital. Chapter 65. Disease Control Priorities in Developing Countries. Washington DC: World Bank; 2006:1211-1228. 25. English M, Esamai F, Wasunna A, et al. Assessment of inpatient paediatric care in first referral level hospitals in 13 districts in Kenya. Lancet. 2004;363:1948-1953.

26. Gathara D, Opiyo N, Wagai J, et al. Quality of hospital care for sick newborns and severely malnourished children in Kenya: a two-year descriptive study in 8 hospitals. BMC Health Serv Res. 2011;11:307. 27. Turner EL, Nielsen KR, Jamal S, von Saint André-von Arnim A, Musa NL. A review of pediatric critical care in resource-limited settings: a look at past, present and future directions. Front Pediatr. 2016;4:5. 28. Friberg IK, Kinney MV, Lawn JE, et al. Sub-Saharan Africa’s mothers, newborns, and children: how many lives could be saved with targeted health interventions? PLoS Med. 2010;7:e1000295. 29. Dunser MW, Festic E, Dondorp A, et al. Recommendations for sepsis management in resource-limited settings. Intensive Care Med. 2012;38:557-574. 30. Slusher TM, Kiragu AW, Day LT, et al. Pediatric critical care in resource-limited settings-overview and lessons learned. Front Pediatr. 2018;6:49. 31. Tiska MA, Adu-Ampofo M, Boakye G, Tuuli L, Mock CN. A model of prehospital trauma training for lay persons devised in Africa. Emerg Med J. 2004;21:237-239. 32. African Neonatal Sepsis Trial Group, Tshefu A, Lokangaka A, et al. Simplified antibiotic regimens compared with injectable procaine benzylpenicillin plus gentamicin for treatment of neonates and young infants with clinical signs of possible serious bacterial infection when referral is not possible: a randomised, open-label, equivalence trial. Lancet. 2015;385:1767-1776. 33. Baqui AH, Arifeen SE, Williams EK, et al. Effectiveness of homebased management of newborn infections by community health workers in rural Bangladesh. Pediatr Infect Dis J. 2009;28:304-310. 34. English M, Ntoburi S, Wagai J, et al. An intervention to improve paediatric and newborn care in Kenyan district hospitals: understanding the context. Implement Sci. 2009;4:42. 35. Ayieko P, Ntoburi S, Wagai J, et al. A multifaceted intervention to implement guidelines and improve admission paediatric care in Kenyan district hospitals: a cluster randomised trial. PLoS Med. 2011;8:e1001018. 36. Molyneux E, Ahmad S, Robertson A. Improved triage and emergency care for children reduces inpatient mortality in a resourceconstrained setting. Bull World Health Organ. 2006;84:314-319. 37. Molyneux E. Emergency care for children in resource-constrained countries. Trans R Soc Trop Med Hyg. 2009;103:11-15. 38. Ashraf H, Mahmud R, Alam NH, et al. Randomized controlled trial of day care versus hospital care of severe pneumonia in Bangladesh. Pediatrics. 2010;126:e807-e815. 39. Addo-Yobo E, Anh DD, El-Sayed HF, et al. Outpatient treatment of children with severe pneumonia with oral amoxicillin in four countries: the MASS study. Trop Med Int Health. 2011;16:995-1006. 40. Duke T, Wandi F, Jonathan M, et al. Improved oxygen systems for childhood pneumonia: a multihospital effectiveness study in Papua New Guinea. Lancet. 2008;372:1328-1333. 41. Bassiouny MR, Gupta A, el Bualy M. Nasal continuous positive airway pressure in the treatment of respiratory distress syndrome: an experience from a developing country. J Trop Pediatr. 1994;40: 341-344. 42. Pieper CH, Smith J, Maree D, Pohl FC. Is nCPAP of value in extreme preterms with no access to neonatal intensive care? J Trop Pediatr. 2003;49:148-152. 43. Tapia JL, Urzua S, Bancalari A, et al. Randomized trial of early bubble continuous positive airway pressure for very low birth weight infants. J Pediatr. 2012;161:75-80.e1. 44. Tagare A, Kadam S, Vaidya U, Pandit A, Patole S. Bubble CPAP versus ventilator CPAP in preterm neonates with early onset respiratory distress—a randomized controlled trial. J Trop Pediatr. 2013;59:113-119. 45. van den Heuvel M, Blencowe H, Mittermayer K, et al. Introduction of bubble CPAP in a teaching hospital in Malawi. Ann Trop Paediatr. 2011;31:59-65. 46. Koyamaibole L, Kado J, Qovu JD, Colquhoun S, Duke T. An evaluation of bubble-CPAP in a neonatal unit in a developing country: effective respiratory support that can be applied by nurses. J Trop Pediatr. 2006;52:249-253.

References

e2





















































69. Kihuba E, Gathara D, Mwinga S, et al. Assessing the ability of health information systems in hospitals to support evidence-informed decisions in Kenya. Glob Health Action. 2014;7:24859. 70. Gachau S, Ayieko P, Gathara D, et al. Does audit and feedback improve the adoption of recommended practices? Evidence from a longitudinal observational study of an emerging clinical network in Kenya. BMJ Glob Health. 2017;2:e000468. 71. Akech S, Ayieko P, Irimu G, Stepniewska K, English M, Clinical Information Network a. Magnitude and pattern of improvement in processes of care for hospitalised children with diarrhoea and dehydration in Kenyan hospitals participating in a clinical network. Trop Med Int Health. 2019;24:73-80. 72. English M, Nzinga J, Mbindyo P, Ayieko P, Irimu G, Mbaabu L. Explaining the effects of a multifaceted intervention to improve inpatient care in rural Kenyan hospitals—interpretation based on retrospective examination of data from participant observation, quantitative and qualitative studies. Implement Sci. 2011;6:124. 73. World Health Organization. Systems Thinking for Health System Strengthening. Geneva: World Health Organization; 2009. 74. Morel CM, Lauer JA, Evans DB. Cost effectiveness analysis of strategies to combat malaria in developing countries. BMJ. 2005; 331:1299. 75. Balabanova D, Mills A, Conteh L, et al. Good Health at Low Cost 25 years on: lessons for the future of health systems strengthening. Lancet. 2013;381:2118-2133. 76. WHO. The World Health Report 2006 - Working Together for Health. Geneva: WHO Press; 2006. 77. Hongoro C, McPake B. How to bridge the gap in human resources for health. Lancet. 2004;364:1451-1456. 78. Celletti F, Reynolds TA, Wright A, Stoertz A, Dayrit M. Educating a new generation of doctors to improve the health of populations in low- and middle-income countries. PLoS Med. 2011;8:e1001108. 79. Mullan F, Frehywot S, Omaswa F, et al. Medical schools in subSaharan Africa. Lancet. 2011;377:1113-1121. 80. Kolars JC, Cahill K, Donkor P, et al. Perspective: partnering for medical education in Sub-Saharan Africa: seeking the evidence for effective collaborations. Acad Med. 2012;87:216-220. 81. Willis-Shattuck M, Bidwell P, Thomas S, Wyness L, Blaauw D, Ditlopo P. Motivation and retention of health workers in developing countries: a systematic review. BMC Health Serv Res. 2008;8:247. 82. Cancedda C, Farmer PE, Kerry V, et al. Maximizing the Impact of Training Initiatives for Health Professionals in Low-Income Countries: Frameworks, Challenges, and Best Practices. PLoS Med. 2015;12:e1001840. 83. Canarie MF, Shenoi AN. Teaching the Principles of Pediatric Critical Care to Non-Intensivists in Resource Limited Settings: Challenges and Opportunities. Front Pediatr. 2018;6:44. 84. Wilmshurst JM, Morrow B, du Preez A, Githanga D, Kennedy N, Zar HJ. The African pediatric fellowship program: training in Africa for Africans. Pediatrics. 2016;137(1). 85. Pediatric Emergency and Critical Care Kenya. www.pecc-kenya.org. 86. Rid A, Emanuel EJ. Why should high-income countries help combat Ebola? JAMA. 2014;312:1297-1298. 87. Mormina M, Pinder S. A conceptual framework for training of trainers (ToT) interventions in global health. Global Health. 2018;14:100. 88. Barasa EW, Ayieko P, Cleary S, English M. A multifaceted intervention to improve the quality of care of children in district hospitals in Kenya: a cost-effectiveness analysis. PLoS Med. 2012;9:e1001238. 89. Wright SW, Steenhoff AP, Elci O, et al. Impact of contextualized pediatric resuscitation training on pediatric healthcare providers in Botswana. Resuscitation. 2015;88:57-62. 90. Joynt GM, Zimmerman J, Li TST, Gomersall CD. A systematic review of short courses for nonspecialist education in intensive care. J Crit Care. 2011;26:533 e1-e10. 91. Meaney PA, Sutton RM, Tsima B, et al. Training hospital providers in basic CPR skills in Botswana: acquisition, retention and impact of novel training techniques. Resuscitation. 2012;83:1484-1490.



















































47. Wilson PT, Morris MC, Biagas KV, Otupiri E, Moresky RT. A randomized clinical trial evaluating nasal continuous positive airway pressure for acute respiratory distress in a developing country. J Pediatr. 2013;162:988-992. 48. Wilson PT, Baiden F, Brooks JC, et al. Continuous positive airway pressure for children with undifferentiated respiratory distress in Ghana: an open-label, cluster, crossover trial. Lancet Glob Health. 2017;5:e615-e623. 49. Argent AC. Considerations for assessing the appropriateness of highcost pediatric care in low-income regions. Front Pediatr. 2018;6:68. 50. Kulkarni AP, Divatia JV. A prospective audit of costs of intensive care in cancer patients in India. Indian J Crit Care Med. 2013;17: 292-297. 51. Miljeteig I, Johansson KA, Sayeed SA, Norheim OF. End-of-life decisions as bedside rationing. An ethical analysis of life support restrictions in an Indian neonatal unit. J Med Ethics. 2010;36:473-478. 52. Beerenahally TS. No free bed with ventilator: experience of a public health specialist. Indian J Med Ethics. 2017;2:56-57. 53. Firth P, Ttendo S. Intensive care in low-income countries—a critical need. N Engl J Med. 2012;367:1974-1976. 54. Rezzonico R, Caccamo LM, Manfredini V, et al. Impact of the systematic introduction of low-cost bubble nasal CPAP in a NICU of a developing country: a prospective pre- and post-intervention study. BMC Pediatr. 2015;15:26. 55. Mukhtar B, Siddiqui NR, Haque A. Clinical Characteristics and Immediate-Outcome of Children Mechanically Ventilated in PICU of Pakistan. Pak J Med Sci. 2014;30:927-930. 56. Khanal A, Sharma A, Basnet S. Current State of Pediatric Intensive Care and High Dependency Care in Nepal. Pediatr Crit Care Med. 2016;17:1032-1040. 57. Cubro H, Somun-Kapetanovic R, Thiery G, Talmor D, Gajic O. Cost effectiveness of intensive care in a low resource setting: A prospective cohort of medical critically ill patients. World J Crit Care Med. 2016;5:150-164. 58. Caney S. Justice Beyond Borders. Oxford: Oxford University Press; 2005. 59. Engelhardt Jr HT. Critical care: why there is no global bioethics. Curr Opin Crit Care. 2005;11:605-609. 60. Murthy S, Leligdowicz A, Adhikari NK. Intensive care unit capacity in low-income countries: a systematic review. PLoS One. 2015;10: e0116949. 61. Riviello ED, Letchford S, Achieng L, Newton MW. Critical care in resource-poor settings: lessons learned and future directions. Crit Care Med. 2011;39:860-867. 62. Baker T. Pediatric emergency and critical care in low-income countries. Paediatr Anaesth. 2009;19:23-27. 63. Rodgers A, Ezzati M, Vander Hoorn S, et al. Distribution of major health risks: findings from the Global Burden of Disease study. PLoS Med. 2004;1:e27. 64. Ballot DE, Davies VA, Cooper PA, Chirwa T, Argent A, Mer M. Retrospective cross-sectional review of survival rates in critically ill children admitted to a combined paediatric/neonatal intensive care unit in Johannesburg, South Africa, 2013-2015. BMJ Open. 2016; 6:e010850. 65. Argent AC, Ahrens J, Morrow BM, et al. Pediatric intensive care in South Africa: an account of making optimum use of limited resources at the Red Cross War Memorial Children’s Hospital*. Pediatr Crit Care Med. 2014;15:7-14. 66. Xu K SP, Evans D. Health Financing and Access to Effective Interventions. World Health Report 2010. Background Paper No. 8. Geneva: World Health Organization; 2010. 67. Lewin S, Lavis JN, Oxman AD, et al. Supporting the delivery of cost-effective interventions in primary health-care systems in lowincome and middle-income countries: an overview of systematic reviews. Lancet. 2008;372:928-939. 68. Ayieko P, Ogero M, Makone B, et al. Characteristics of admissions and variations in the use of basic investigations, treatments and outcomes in Kenyan hospitals within a new Clinical Information Network. Arch Dis Child. 2016;101:223-229.

e3

































110. Rosenberg DI, Moss MM, American College of Critical Care Medicine of the Society of Critical Care M. Guidelines and levels of care for pediatric intensive care units. Crit Care Med. 2004; 32:2117-2127. 111. Rosenberg DI, Moss MM, American Academy of Pediatrics Section on Critical C, American Academy of Pediatrics Committee on Hospital C. Guidelines and levels of care for pediatric intensive care units. Pediatrics. 2004;114:1114-1125. 112. Maitland K, Kiguli S, Opoka RO, et al. Mortality after fluid bolus in African children with severe infection. N Engl J Med. 2011;364: 2483-2495. 113. Seydel KB, Kampondeni SD, Valim C, et al. Brain swelling and death in children with cerebral malaria. N Engl J Med. 2015;372: 1126-1137. 114. Sarmin M, Ahmed T, Bardhan PK, Chisti MJ. Specialist hospital study shows that septic shock and drowsiness predict mortality in children under five with diarrhoea. Acta Paediatr. 2014;103: e306-e311. 115. Ranjit S, Aram G, Kissoon N, et al. Multimodal monitoring for hemodynamic categorization and management of pediatric septic shock: a pilot observational study. Pediatr Crit Care Med. 2014; 15:e17-e26. 116. Ranjit S, Natraj R, Kandath SK, Kissoon N, Ramakrishnan B, Marik PE. Early norepinephrine decreases fluid and ventilatory requirements in pediatric vasodilatory septic shock. Indian J Crit Care Med. 2016;20:561-569. 117. Sankar J, Ismail J, Sankar MJ, Meena RS. Fluid bolus over 15-20 versus 5-10 minutes each in the first hour of resuscitation in children with septic shock: a randomized controlled trial. Pediatr Crit Care Med. 2017;18:e435-e445. 118. Sankar J, Dhochak N, Kumar K, Singh M, Sankar MJ, Lodha R. Comparison of international pediatric sepsis consensus conference versus sepsis-3 definitions for children presenting with septic shock to a tertiary care center in India: a retrospective study. Pediatr Crit Care Med. 2019;20:e122-e129. 119. von Saint Andre-von Arnim AO, Attebery J, Kortz TB, et al. Challenges and priorities for pediatric critical care clinician-researchers in low- and middle-income countries. Front Pediatr. 2017;5:277. 120. English M, Gathara D, Mwinga S, et al. Adoption of recommended practices and basic technologies in a low-income setting. Arch Dis Child. 2014;99:452-456. 121. Wiens MO, Kumbakumba E, Larson CP, et al. Postdischarge mortality in children with acute infectious diseases: derivation of postdischarge mortality prediction models. BMJ Open. 2015;5:e009449. 122. Chisti MJ, Graham SM, Duke T, et al. Post-discharge mortality in children with severe malnutrition and pneumonia in Bangladesh. PLoS One. 2014;9:e107663. 123. Richardson B, Dol J, Rutledge K, et al. Evaluation of mobile apps targeted to parents of infants in the neonatal intensive care unit: systematic app review. JMIR Mhealth Uhealth. 2019;7:e11620. 124. Metelmann B, Metelmann C, Schuffert L, Hahnenkamp K, Brinkrolf P. Medical Correctness and user friendliness of available apps for cardiopulmonary resuscitation: systematic search combined with guideline adherence and usability evaluation. JMIR Mhealth Uhealth. 2018;6:e190.







































92. Robertson MA, Molyneux EM. Description of cause of serious illness and outcome in patients identified using ETAT guidelines in urban Malawi. Arch Dis Child. 2001;85:214-217. 93. Tamburlini G, Di Mario S, Maggi RS, Vilarim JN, Gove S. Evaluation of guidelines for emergency triage assessment and treatment in developing countries. Arch Dis Child. 1999;81:478-482. 94. Tchorz KM, Thomas N, Jesudassan S, et al. Teaching trauma care in India: an educational pilot study from Bangalore. J Surg Res. 2007;142:373-377. 95. Bergman S, Deckelbaum D, Lett R, et al. Assessing the impact of the trauma team training program in Tanzania. J Trauma. 2008;65:879-883. 96. Aboutanos MB, Rodas EB, Aboutanos SZ, et al. Trauma education and care in the jungle of Ecuador, where there is no advanced trauma life support. J Trauma. 2007;62:714-719. 97. Ali J, Adam RU, Gana TJ, Williams JI. Trauma patient outcome after the Prehospital Trauma Life Support program. J Trauma. 1997;42:1018-1021; discussion 21-22. 98. Homaifar N, Mwesigye D, Tchwenko S, et al. Emergency obstetrics knowledge and practical skills retention among medical students in Rwanda following a short training course. Int J Gynaecol Obstet. 2013;120:195-199. 99. Butler MW, Ozgediz D, Poenaru D, et al. The Global Paediatric Surgery Network: a model of subspecialty collaboration within global surgery. World J Surg. 2015;39:335-342. 100. MacLeod JB, Gravelin S, Jones T, et al. Assessment of acute trauma care training in Kenya. Am Surg. 2009;75:1118-1123. 101. Opiyo N, English M. In-service training for health professionals to improve care of seriously ill newborns and children in low-income countries. Cochrane Database Syst Rev. 2015;5:CD007071. 102. Edgcombe H, Paton C, English M. Enhancing emergency care in low-income countries using mobile technology-based training tools. Arch Dis Child. 2016;101:1149-1152. 103. Organization WH. Telemedicine - Opportunities and Developments in Member States. Report on the Second Global Survey on eHealth. Geneva: WHO Press; 2010. 104. Alirol E, Getaz L, Stoll B, Chappuis F, Loutan L. Urbanisation and infectious diseases in a globalised world. Lancet Infect Dis. 2011; 11:131-141. 105. Mould-Millman NK, Dixon JM, Sefa N, et al. The State of Emergency Medical Services (EMS) Systems in Africa. Prehosp Disaster Med. 2017;32:273-283. 106. Leligdowicz A, Bhagwanjee S, Diaz JV, et al. Development of an intensive care unit resource assessment survey for the care of critically ill patients in resource-limited settings. J Crit Care. 2017;38:172-176. 107. El-Khatib Z, Shah M, Zallappa SN, et al. SMS-based smartphone application for disease surveillance has doubled completeness and timeliness in a limited-resource setting - evaluation of a 15-week pilot program in Central African Republic (CAR). Confl Health. 2018;12:42. 108. Geiling J, Burkle Jr FM, Amundson D, et al. Resource-poor settings: infrastructure and capacity building: care of the critically ill and injured during pandemics and disasters: CHEST consensus statement. Chest. 2014;146:e156S-e167S. 109. Marston BJ, Dokubo EK, van Steelandt A, et al. Ebola Response Impact on Public Health Programs, West Africa, 2014-2017. Emerg Infect Dis. 2017;23:S25-S32.

e4

Abstract: Low- and middle-income countries continue to carry the largest burden of critical illness and pediatric mortality yet have the least critical care resources. Basic low-cost critical care interventions can be successfully provided in resource-poor settings without an intensive care unit (ICU). However, publicly funded ICU treatment remains limited in low-income countries, and its introduction requires careful resource allocation. Healthcare systems improvements for the critically ill should involve a graded approach of strengthening capacity to provide health

maintenance, basic critical care, and then publicly funded intensive care services as overall health indices improve. Critical care research from low-income countries is sorely needed to guide effective and efficient care and advocate for resources. Key words: Low-resource settings, low- and middle-income countries, resource allocation, pediatric critical care training, costeffectiveness

9 Public Health Emergencies and Emergency Mass Critical Care KATHERINE L. KENNINGHAM AND MEGAN M. GRAY

“Keep your eye on the ball.”

Bob Kanter, Trailblazer for Children in the World of Disaster Medicine









Emergency mass critical care (EMCC) is limited, essential critical care during disasters when intensive care demands surpass resources. Pediatric, neonatal, and cardiac intensivists must be engaged with hospital and regional public health emergency (PHE) planning and preparedness efforts to ensure that they are familiar with strategies and resources to rapidly increase care capacity. Pediatric intensive care units (ICUs) should plan to care for three times their usual census for 10 days without outside help during a severe PHE.

During a public health emergency (PHE) such as a natural disaster or pandemic, a large number of infants, children, and young adults may need critical care in order to survive. During such an event, the incident command system (ICS) provides a framework to support decision-making and coordinate efforts across affected sites. Because pediatric critical care is highly specialized and because few nonpediatric providers are comfortable caring for severely ill or injured children, pediatric, neonatal, and cardiac intensive care units (ICUs) represent an essential aspect of patient management during a PHE and should be included within a structured response. Planning and preparedness for PHEs can save lives. Unfortunately, critical care providers receive little training in disaster medicine and response and are often underinvolved in hospital disaster preparedness efforts. Recent public health emergencies have exposed a lack of PHE awareness, training, and preparation by critical care providers. The goal of this chapter is to educate the pediatric or neonatal ICU provider in the principles and tools of PHE preparedness and emergency mass critical care (EMCC). Learning from smaller or more local PHEs and preparing for critical care during anticipated PHEs may help intensivists better prepare for future catastrophic events that require EMCC. Forward consideration of how to scale up response and conserve, ration, and allocate critical care services can reduce dangerous uncertainty and save time in the event of a larger-scale PHE.













PEARLS The surge capacity continuum that spans conventional to contingency to crisis capacity should be employed during a PHE in order to extend resources to meet needs. All ICU disaster planning efforts should consider protocols for triage teams to support the emergency department, patient tracking and reunification, victim and staff mental health, and the role of medical learners in EMCC efforts.

How Many Pediatric Patients Could Be Affected in a Public Health Emergency? If a PHE affected persons of all ages equally, children aged 0 to 14 years would account for 20% of all patients.1 However, younger patients may be more vulnerable to infections, dehydration, toxins, and trauma. Therefore, they may be overrepresented in the patient population during a PHE.2 Events involving a child-specific location, such as school, may result in a patient population predominantly made up of children. Under usual circumstances, survival rates from high-risk pediatric conditions tend to be higher when children receive care at pediatric hospitals.3–6 A national survey estimated a pediatric ICU (PICU) peak capacity of 54 beds per million pediatric population.7 Typical PICU occupancy in excess of 50% leaves fewer than 30 vacant PICU beds per million age-specific population, with even fewer cardiac ICU (CICU) beds generally available. The younger the patient, the more age-specific and specialized the treatment requirements become, culminating in the extremely preterm neonate who requires equipment unavailable outside of a regional neonatal ICU (NICU). There are approximately 4 million newborns born in the United States annually and 5700 total NICU beds per million age-specific population. In contrast to PICU capacity, occupancy of NICU beds is higher at baseline, 59

60

SECTION I



Pediatric Critical Care: The Discipline

with 6% of low-risk term infants and 97% of very-low-birthweight infants requiring NICU care.8 Because each region may be served by only a few or even a single pediatric-capable hospital, events that disable one hospital may disproportionately degrade regional pediatric and neonatal care. Quantitative models indicate that survival during a PHE would improve if a pediatric patient surge is distributed to pediatric beds throughout a larger geographic area rather than overwhelming facilities near the epicenter of an emergency.9 Unfortunately, control of patient distribution may be limited in a severe PHE. As a result, all hospitals must be prepared to care for some children for an extended period of time.10–12 Whether or not patients are distributed optimally, outcomes from a large PHE are likely to improve with EMCC approaches.9,14 Additionally, using telemedicine to connect available pediatric critical care and subspecialty physicians to facilities unaccustomed to caring for pediatric patients may significantly extend the reach of pediatric EMCC during a PHE.

Incident commander

Operations chief Medical director

Public info officer

Safety officer

Planning chief

Logistics chief

Labor pool

Finance and admin chief

Materials & supplies

ICU leader

•  Fig. 9.1

​Incident command system diagram. admin, Administrative; ICU, intensive care unit; info, information.  

What Are the Most Likely Public Health Emergencies?

What Is the Expected Timeline of a Public Health Emergency?

Public health emergency risks vary depending on hospital location, population demographics, and local resources. Each hospital is tasked with maintaining emergency plans for its most likely PHEs. This list is usually generated via a hazards vulnerability analysis (HVA), which leverages local expertise in medical management and infrastructure risks, and combines this with frequency and severity data on past and potential events.15 Events such as information technology (IT) and electronic medical record (EMR) outages are experienced nearly universally in healthcare settings and are included in HVAs. Primary planning for IT/EMR outages should include protocols for downtime procedures, paper charting, and nonelectronic communication. Hospitals should engage in drills, tabletop exercises, and simulations targeted to address issues identified by their HVA. Smaller and less severe events, such as planned EMR downtimes and routine adverse weather should be used as opportunities to clarify and test protocols for larger PHEs.

When a sudden-impact PHE occurs, the hospital’s ICS is activated. The initial priority is to perform an assessment of the current state of the hospital; units rapidly assess their bed capacity, including potential beds that could be mobilized. Patients potentially no longer requiring ICU care should be identified for rapid transfer or discharge, and on-site staff able to be redirected to patient care should be tallied. If the PHE directly affects the structure or function of the hospital building, the initial assessment should also include numbers of newly injured staff, visitors, and patients, as well as any damage to the unit. These initial assessment numbers are collected from each unit, and a rough estimate of potential incoming patients is, in turn, shared with area leaders. Based on initial assessment and estimates, the incident commander will direct the response to ensure adequate staff supplies, equipment, and clinical space, and will communicate decisions regarding whether to mobilize potential bed capacity identified in the initial assessment. Information from the ICS should be communicated to front-line staff early on to reduce uncertainty and ensure a clear and united message to patients and families. ICS-directed response may include assigning and sometimes reassigning current staff, calling in additional staff, ordering and distributing additional supplies, and identifying when standards of care should change. Area leaders should be called in as soon as possible to aid in coordinating the response and offload clinical staff of administrative duties. As information about the event becomes available, ICU leaders and educators may need to provide incident-specific just-in-time teaching to staff.

Who Will Make Decisions During an Emergency? Responses to major public health emergencies are organized within a National Response Framework, as outlined by the federal US Department of Homeland Security.16 Emergency responses are coordinated at the most local jurisdiction possible, usually at the city or county level, until those resources are outpaced. The hospital ICS further provides a leadership framework within and among organizations responding to an emergency, representing a simplified and clear chain of command in order to speed decision-making. A hospital ICS includes clinical and nonclinical representation, provides flexible logistical support, and helps to prioritize key functions. Disaster plans at every hospital should incorporate ICS principles, regardless of the size of the hospital; ICS planning guides are widely available to aid in plan development. Identifying the ICU role within the local ICS framework is an essential part of PHE preparedness (Fig. 9.1).

What Is a Surge and What Can Be Done to Meet Surge Needs? Critical care responses to PHEs are scaled according to the size and severity of the emergency (Fig. 9.2).17,18 Emergency surges are categorized as minor, moderate, or major. A minor event would require up to 20% increase above usual peak hospital capacity; conventional surge methods would likely suffice to provide normal standards of critical care to all who need it in this scenario.23 Conventional critical care surge needs can be met by canceling



CHAPTER 9

Public Health Emergencies and Emergency Mass Critical Care

61

Maximum ICU capacity

Level of care available

Crisis standards of Major surge: crisis response care Functionally equivalent Moderate surge: contingency response care

Minor surge: conventional response

Expanding ICU capacity

Usual care Usual volume

Usual ICU capacity

# of patients



•  Fig. 9.2 ​Surge volume, expansion of intensive care unit (ICU) capacity, and the effect on standards of care.

Conventional strategies Conserve resources Utilize on-call staff Substitute for equivalent items Utilize supply caches



• Fig. 9.3

Contingency strategies Adapt other ICUs and similar care areas

Crisis strategies

Extend usual staff

Adapt non-ICU and noncare areas

Reuse select supplies

Utilize non-ICU staff

Substitute where reasonable

Reallocate resources

​Conventional, contingency, and crisis strategies to extend critical care resources. ICU, intensive

care unit.

elective admissions, quickly discharging all patients who can safely leave the ICU, mobilizing staff, and adding bed space (Fig. 9.3). Moderate emergency surges result in an increased patient population of between 20% and 100% of usual capacity, necessitating a contingency response to most effectively use limited resources. Contingency surge methods include all elements of the conventional response with additional strategies to expand coverage. Non-ICU patient care areas may be repurposed for ICU-level care. Staff are leveraged by changing provider-to-patient ratios or by using non-ICU staff in a tiered approach whereby non-ICU providers provide care and are, in turn, supervised by ICU providers. Supplies and equipment are conserved when possible; some substitutions, adaptations, and reuse may be necessary when safe. The goal of the contingency surge response is to significantly increase capacity while minimally affecting patient care practices. EMCC and crisis standards of care (CSC) are required when a large PHE threatens to overwhelm critical care resources despite fully deployed conventional and contingency surge responses. Following a sudden-impact PHE, there may be an initial emergency department (ED) surge lasting a few hours and a subsequent ICU phase of weeks, while prolonged events such as pandemics

could require both EDs and ICUs to sustain contingency or crisis strategies for months. It is recommended that hospitals with PICUs be able to care for up to three times the usual number of critically ill pediatric patients for up to 10 days without outside help.19 In these circumstances, population-based goals will attempt to maximize the number of survivors by reallocating lifesaving interventions to persons who are more likely to benefit from them. This represents an escalation from usual standards of care to CSC. PHE powers are defined on a state-by-state basis; thus, ICU leaders must be familiar with their own state and hospital incident command process for determining when CSC should be activated.20 Sudden-impact events that stress the resources of a community may require the implementation of temporary reactive mass critical care. However, no historical precedents exist for sustained mass critical care such as might occur with major regional damage or severe pandemic.21 During the initial wave of COVID-19 in New York City, temporary mass critical care via rapid expansion of COVID-19 units and staff was utilized as a means to address the overwhelming care needs of patients. EMCC, whether temporary or sustained, should attempt to provide these five priority

62

SECTION I



Pediatric Critical Care: The Discipline

lifesaving interventions: (1) respiratory support, (2) fluid resuscitation, (3) vasopressors, (4) antidotes and antibiotics, and (5) analgesia and sedation. In order to provide adequate essential care to a greater number of patients, some more resource-intensive interventions may need to be delayed or foregone in EMCC settings. Examples include strict monitoring and frequent recording of vital signs and fluid balance, parenteral nutrition, invasive hemodynamic monitoring, intracranial pressure monitoring, renal replacement therapy, and extracorporeal life support.2,22 Similarly, lifesaving EMCC interventions can be extended to much larger than usual numbers of patients by conservation of resources, substitution, adapting personnel, supplies, and spaces, reusing selected items, and reallocation of resource-intense interventions from patients not expected to survive to patients with a higher likelihood of quick recovery and survival. Patient care plans may need to be based more on physical examination findings than on ancillary studies; relying less on laboratory and imaging studies may represent a fundamental shift in patient management paradigms. The degree of deviation from usual practices should be proportional to the gap between patient needs and existing resources, and EMCC should be implemented in an organized way by each hospital’s ICS with the input of public health experts.

How Can the Intensive Care Unit Support the Emergency Department During a Public Health Emergency? To provide continuity of patient care and maintain situational awareness, ICU teams must interact closely with the ED. Rapidly accommodating patients from the ED or operating room will be essential to allow those areas to continue receiving new patients. Triage of patients to match needs with available resources evolves as the PHE unfolds, according to shifting needs and available resources. Initial triage categories are assigned in the ED by an experienced clinician whose sole role is to act as triage officer and should be based on existing triage algorithms. In some cases, ICU staff may be temporarily reassigned to work in the ED as a triage team to speed this process and ensure appropriate patient allocation. Physiologic triage identifies patients needing immediate lifesaving interventions. Physiologic triage tools identify patients in five categories: (1) those needing immediate lifesaving interventions; (2) those who need significant intervention that can be delayed; (3) those needing little or no treatment; (4) those who are so severely ill or injured that survival is unlikely despite major interventions; and (5) those who have already died. Care of patients triaged to group 4, often referred to as “expectant,” will deviate most significantly from usual approaches to intensive care. Because of overall demands on the system, scarce resources must be allocated to other patients who are more likely to survive, and expectant patients should receive appropriate palliative care. While no single triage tool is always rapid, completely accurate, appropriate for all ages and disorders, and already familiar to all providers, triage and ED staff should be familiar with the physiologic triage tools in use locally.24 The local chosen tools should be made available online and in printed form to all relevant areas so that patients are triaged and treated in a standardized manner. Pediatric experts should partner with regional healthcare coalitions to provide standardized pediatric healthcare education, such as pediatric triage and other specialized pediatric topics, prior to a PHE.25 When decontamination or infection control are central to the PHE at hand, these should be incorporated into the ED and triage

response. Decontamination reduces toxic effects for the victim and mitigates contamination of providers, staff, and the hospital facility. Antidotes are given after cleaning an area of the body for administration. Consideration should be given to risks of hypothermia by using warm water preferentially for those at highest risk of thermal instability. Respiratory support during decontamination may be necessary and should be planned for.26–28 For PHEs with highly virulent transmissible infections, infection control must begin outside the ED entrance and continue without interruption in the hospital while the patient is infectious.29,30

How Can All Intensive Care Units Work Together? Pediatric hospitals often have more than one ICU with at least some patient flux between the NICU, PICU, and CICU depending on census. There may be flux between the PICU and adult ICUs depending on patient age, size, underlying conditions, and disease process. During a PHE, usual boundaries for these areas should be evaluated and stretched to accommodate the greatest number of critically ill patients (Fig. 9.4). Critical care may be represented by a single ICU leader within the ICS in order to facilitate awareness of the global pool of ICU beds, staff, equipment, and supplies. As many more infants require ICU admission compared with older children, there is a notably larger pool of NICU beds within any given region. There is considerable variation in equipment and staffing between the four levels of NICUs, but all NICUs have at least one nurse with resuscitation and stabilization training in the hospital at all times (Table 9.1).31 All NICUs have devices to deliver positive-pressure ventilation; intubation supplies, including endotracheal tubes between 2.5 to 4.0; warmer beds; and a supply of medications in pediatric doses. Adult hospitals experiencing a surge of pediatric patients should engage local neonatal and pediatric providers and staff to aid in triage and stabilization of infants and children until transport to regional PICUs and pediatric hospitals becomes available.

What Steps Can Be Taken to Maximize Intensive Care Unit Treatment in a Disaster? Patient Spaces Single-patient spaces may be converted for use by two or three patients with careful discussion of how to monitor additional patients if centralized monitoring is limited. After exhausting PICU space, additional space for EMCC may also be created by Young disproportionately affected

NICU

CICU

PICU

Adult ICU Adults disproportionately affected

•  Fig. 9.4

​Intensive care unit flux and the continuum of critical care in surge events. CICU, Cardiac intensive care unit; ICU, intensive care unit; NICU, neonatal intensive care unit; PICU, pediatric intensive care unit.  



CHAPTER 9

TABLE Levels of Neonatal Intensive Care Unit Treatment 9.1 and Expected Pediatric Specific Resources

Respiratory Equipmenta

Level

Population

Staffing

I: Nursery

Late preterm to term

Pediatrician off site

PPV

II: Special Care Nursery

Moderately preterm to term

APP, pediatrician, or neonatologist on site or home call 24/7

PPV HFNC 6 CPAP

III: NICU

Extremely preterm to term

APP or resident in house 24/7 Neonatologist on site or home call 24/7

PPV HFNC CPAP CMV 6 High-frequency ventilator

IV: Regional NICU

Infants with subspecialty needs

APP or resident in house 24/7 Neonatologist usually on site 24/7

PPV HFNC CPAP CMV High-frequency ventilator 6 ECMO

a Lower-level NICUs may have a limited supply of ventilators for pretransport stabilization. APP, Advanced practice provider (nurse practitioner, physician assistant); CMV, conventional mechanical ventilator; CPAP, continuous positive airway pressure; ECMO, extracorporeal membranous oxygenation; HFNC, high-flow nasal cannula; NICU, neonatal intensive care unit; PPV, positive pressure ventilation.

Public Health Emergencies and Emergency Mass Critical Care

63

hospitals use a single type of ventilator for patients of all sizes, with appropriate circuits and software algorithms. In other hospitals, ventilators usually used for adults that have high compliance circuits and adult algorithms may have to be adapted for use in infants or small children. When local supplies have been exhausted in a major PHE, adult-focused pediatric-adaptable ventilators and supplies may be accessed through the Strategic National Stockpile.33 Some difficulties in adapting equipment may be encountered. The inspiratory flow or pressure sensor may not be sensitive to an infant’s inspiratory effort—triggering of inspiration may fail for synchronized intermittent mandatory ventilation, assist control, or pressure support. Likewise, ventilator algorithms to terminate inspiration pressure support may fail in the presence of air leaks around endotracheal tubes if incorrectly sized tubes are being used owing to limited supply. In a volume-controlled mode, adult ventilators may be unable to provide small tidal volumes and inspiratory flow appropriate for a small infant. Extremely preterm infants often require tidal volumes of less than 5 mL of air and are especially at risk for adverse effects from dead space. Pressure-dependent losses of tidal volume in compressible spaces of adult ventilator circuits exaggerate breath-to-breath variation in delivered tidal volume if peak inspiratory pressure varies with patient effort or changing respiratory mechanics. Difficulties in providing small tidal volumes and variation in ventilation due to leaks around uncuffed endotracheal tubes may necessitate using a time-cycled, pressure-limited mode of ventilation. Supplemental providers need considerable assistance in caring for an infant on a ventilator, especially if nonstandard equipment and techniques are being employed.

Manual Ventilation adapting intermediate care units, postanesthesia care units, EDs, procedure suites, or non-ICU hospital rooms. Considerations for adapting non-ICU spaces include the availability of equipment, monitoring, and staff and whether these areas are needed as part of non-ICU surge activities. The hospital ICS should coordinate these decisions to ensure overall resource optimization. Overflow of critically ill adolescents or young adults may be shared between PICUs and adult ICUs, while younger infants and children should be shared with local NICUs. CICUs provide an additional pool of critical care services. Nonhospital facilities should be used for EMCC only if hospitals become unusable.

Personnel Supplemental providers may include healthcare workers who have skills in non-ICU pediatrics or nonpediatric critical care. Rapid credentialing procedures, just-in-time education, and local or distant supervision by experienced pediatric and neonatal ICU clinicians can help extend the provider pool. Hospitals should expect and plan for a need for significant psychosocial support for patients and providers during and after a PHE, especially for those who were asked to work beyond their usual scope of care.32

Mechanical Ventilation Most hospitals have only a small number of extra ventilators and support devices. It may be necessary to consider temporary use of transport and anesthesia ventilators, bilevel positive-pressure breathing devices, and noninvasive support devices. Some pediatric

Few hospitals stockpile enough mechanical ventilators to support three times the usual number of ICU patients. The temporary use of manual ventilation with a self-inflatable bag may need to be considered. Manual ventilation has been used successfully via tracheostomy tubes for days in a polio epidemic, and for hours in a power failure and during weather emergencies.34–38 It provides similar gas exchange compared with mechanical ventilation when provided via an ETT.39–41 However, manual ventilation is labor intensive, may expose staff to infection risks as a result of close and prolonged bedside contact, and may prove to be insufficient respiratory support to meet patient needs. In extreme circumstances, family members or nonclinical staff could be tasked with providing manual ventilation with just-in-time training to free up clinical staff.

Equipment and Supplies Mass critical care can be provided only if essential equipment and supplies are available on-site, as resupply and rental deliveries may be limited during a PHE. Thus, hospitals must balance the benefits of an adequate stockpile against the costs of maintaining items on-site that may expire or become defunct before being needed. The Task Force on Mass Critical Care has recommended that a hospital should first target a mass critical care capacity of three times the usual maximum ICU capacity for 10 days, but decisions regarding equipment stockpiles should be made by individual hospitals.2 Each hospital should also maintain information on how to contact neighboring hospitals and clinical spaces to evaluate capacity for sharing supplies and equipment locally.

64

SECTION I



Pediatric Critical Care: The Discipline

Nonpediatric hospitals must also consider stocking critical pediatric equipment to care for children until transport and pediatric hospital bed spaces become available. Although it may be possible to carry out many interventions by adapting nearly equivalent equipment and supplies, some adult equipment cannot be adapted to infants and small children. It is essential to stock adequate numbers of resuscitation masks, endotracheal tubes, suction catheters, chest tubes, intravenous catheters, and gastric tubes in pediatric sizes. If cuffed endotracheal tubes are used, it may be possible to cover the majority of pediatric needs with 3.0-, 4.0-, 5.0-, and 6.0-mm cuffed tubes without stocking intermediate sizes.2,42

Medications In order to extend medication stockpiles in mass critical care, rules should be formulated prior to PHEs regarding appropriate substitutions, dose and frequency reductions, reasonable parenteral to enteral conversions, restrictive indications, and shelf-life extension.2 Experience in recent PHEs indicates that large quantities of analgesics and sedatives will be needed.21,43 Weight-based dosing may be simplified to improve efficiency by specifying a limited number of weight range categories. When time constraints make it difficult to weigh patients, length-based estimates of weight may suffice.44

How Will the Intensive Care Unit Evacuate if Needed? ICU providers must be aware of processes to ensure a safe and timely evacuation in the event that this is ordered by the ICS or government authorities. Hurricane Sandy demonstrated a lack of ICU evacuation knowledge, processes, and tools.45 Pediatric patients are especially vulnerable during ICU evacuation, as few hospitals can serve as recipient hospitals and few transport agencies are familiar with pediatric and neonatal critical care. Thus PICU, NICU, and CICU evacuation is critically dependent on regional coordination of resources.46 ICU evacuation best practices are available from the Mass Critical Care Taskforce, which include tools such as ICU evacuation checklists and job action sheets that should be used for preparedness and just-in-time training.47

How Will Limited Services Be Ethically Rationed? If a PHE overwhelms resources despite EMCC approaches, rationing of resources may be needed. Rationing might occur on a first-come, first-served basis or by selecting patients most likely to survive as a result of brief lifesaving interventions.2,48 Proposed eligibility criteria to receive intervention include absence of severe chronic conditions, predicted mortality risk below a threshold chosen by the ICS or public health officials, and improving clinical status on periodic reevaluations. Suggested algorithms exist for both children and adults.49 In pediatrics, however, there is little consensus about, or data to support, which mortality risk score to use, especially in light of typical PICU mortality of less than 5% and NICU mortality rate of less than 1%.19,50 Rationing should only occur using a formal hospital system or regional triage policy or protocol and should be performed by triage officers with critical care training under the direction of the ICS. At present, neither evidence nor consensus of opinion supports a particular rationing strategy. Thus, local ICSs need to evaluate needs and resources in real time in order to guide the triage team.51,52 Medical ethicists and community members are key partners in planning for crisis standards of care and potential rationing. Difficult questions surrounding the ethics of triaging patients as expectant, removing life support, and which patient factors should be considered when deciding who will (and who will not) be offered life support all benefit from careful consideration with trained bioethicists as well as community leaders. Ideally, these discussions happen as part of the PHE planning phase to allow thorough discussion and input from members of the medical community and the public. Public input can be accomplished with focus groups representative of local population demographics and structured discussions of relevant issues. The Institute of Medicine specifically recommends community consultation during development of CSC to ensure that the final recommendations “reflect the ethical values and priorities of the community.” This form of planning ensures transparency and provides reassurance to both affected patients and staff members during a PHE that issues have been sufficiently thought through beforehand. A carefully considered plan created with public input creates a legal basis and liability protections.53

How Should Pediatric Patients Be Tracked?

What Are the Mental Health Considerations Relevant to Emergency Mass Critical Care?

Hospital care of children is more efficient, more effective, and less stressful when children are accompanied by a familiar caregiver. Unaccompanied children must be properly identified, tracked, and reunited with their families. Proper identification of adult caregivers is necessary before releasing children. Examples of child identification and tracking documents are available online.12 A tracking and communication center should be activated by a designee within the ICS in order to centralize patient tracking and field calls from caregivers. Every pediatric patient should have a patient-specific tracking identification number assigned upon arrival to the ED, and the tracking center should be provided any potentially identifying information to aid in reunification (such as physical features, clothing, location where the patient was initially found, information provided directly by verbal children, and a photo whenever possible).

Situations requiring EMCC cause stress and trauma to patients, families, and staff. Requiring care for a significant disaster-related illness or injury is a risk factor for severe mental health deterioration. Use of a mental health triage system such as psySTART can aid in allocation of psychiatric, behavioral, and psychosocial resources.54 For children at risk for significant mental health effects, attention to this should start as soon as possible. Tracking children with the goal of identification and reunification, recording of individual exposures with known mental health effects, and protecting children from additive harm are key early steps. For alert and interactive victims, ICU staff should follow basic support, including establishing safety and security, orienting to the situation in developmentally appropriate ways, and facilitating communication with familiar caregivers and trained support staff. Simple messaging that the child is in a safe place and that the family will join the child as soon as possible is appropriate for all pediatric patients.



CHAPTER 9

Public Health Emergencies and Emergency Mass Critical Care

65

Staff of all disciplines may also have significant mental health effects during PHEs.55 Anxiety regarding risks to themselves, their family, and their coworkers can combine with fatigue and trauma from caring for multiple dead and dying patients in a short period of time and lead to emotional and physical exhaustion. To mitigate these effects, clear protocols and procedures for varying levels of PHE should be created in the planning phase and shared with front-line staff. Job aids and just-in-time tools are important methods to support staff. Rest breaks and basic self-care—such as access to bathrooms, food, and water—are necessary for any sustained response. In longer events, hospitals may consider shortening shifts to allow recovery between intense exposures.

PHE responses, care should be taken to ensure that they can safely participate to the benefit of the patients and their own education.

What Is the Role of Medical Learners in Public Health Emergencies?

Key References

Medical learners are a vital component of the healthcare team at many pediatric centers. Whenever a significant PHE occurs, the needs of the patients and learners must be balanced. Residents and fellows provide extensive patient care in ICUs. They are routinely trained to care for patients with contagious infections and high-risk conditions; with supervision, they can provide care to a large number of critically ill patients at a time. Their value as patient care providers must be weighed against potential risks to their learning, their own health, and their families’ health. Severe PHEs—in which supplies of personal protective equipment are inadequate, training insufficient, or supervision limited—place learners at risk. In this scenario, their role as junior team members may discourage speaking up about these risks. Very junior learners, such as observers and students, are especially vulnerable to these issues—the decision to include them in PHE responses should be carefully considered by their program, especially in settings with limited PPE. When learners of any stage are used in

Conclusion It is essential that critical care providers are knowledgeable about and active in hospital and regional disaster planning, EMCC triage protocols, and surge strategies to be prepared for future events and maximize the survival of pediatric patients. Preparedness efforts should include education on the local ICS, surge protocols, methods to extend care capacity, and triage techniques.

American Academy of Pediatrics. American College of Critical Care Medicine. Consensus report for regionalization of services for critically ill or injured children. Pediatrics. 2000;105:152-155. EMSC National Resource Center. Checklist of Essential Pediatric Domains and Considerations for Every Hospital’s Disaster Preparedness Policies. Washington, DC: EMSC National Resource Center; 2014. Kanter RK, Andrake JS, Boeing NM, et al. A method for developing consensus on appropriate standards of disaster care. Disaster Med Public Health Prep. 2009;3:27-32. Kanter RK. Strategies to improve pediatric disaster surge response: potential mortality reduction and tradeoffs. Crit Care Med. 2007;35:2837-2842. Phillips SJ, Knebel A. Mass Medical Care with Scarce Resources: A Community Planning Guide. Rockville, MD: Agency for Healthcare Research and Quality (AHRQ Publication No. 07–0001); 2007. Schreiber M. The psySTART Rapid Mental Health Triage and Incident Management System. Center for Disaster Medical Sciences, University of California; 2010.

The full reference list for this chapter is available at ExpertConsult.com.

e1









































































































1. US Census Bureau. Age and Sex, Table S0101, American Community Survey. Washington, DC: US Census Bureau; 2006. 2. Christian MD, Devereaux AV, Dichter JR, et al. Introduction and executive summary: care of the critically ill and injured during pandemics and disasters: CHEST consensus statement. Chest. 2014; 146(suppl 4):8S-34S. 3. Tilford JM, Simpson PM, Green JW, et al. Volume outcome relationships in pediatric intensive care units. Pediatrics. 2000;106: 289-294. 4. Pollack MM, Alexander SR, Clarke N, et al. Improved outcomes from tertiary center pediatric intensive care: a statewide comparison of tertiary and nontertiary care facilities. Crit Care Med. 1991;19: 150-159. 5. Osler TM, Vane DW, Tepas JJ, et al. Do pediatric trauma centers have better survival rates than adult trauma centers? An examination of the National Pediatric Trauma Registry. J Trauma. 2001;50: 96-101. 6. Densmore JC, Lim HJ, Oldham KT, et al. Outcomes and delivery of care in pediatric injury. J Pediatr Surg. 2006;41:92-98. 7. Randolph AG, Gonzales CA, Cortellini L, et al. Growth of pediatric ICUs in the US from 1995 to 2001. J Pediatr. 2004;144:792-798. 8. Goodman DC, Little GA, Harrison WN, et al, eds. The Dartmouth Atlas of Neonatal Intensive Care. Lebanon, NH: The Dartmouth Institute of Health Policy & Clinical Practice, Geisel School of Medicine at Dartmouth; 2019. 9. Kanter RK. Strategies to improve pediatric disaster surge response: potential mortality reduction and tradeoffs. Crit Care Med. 2007;35:2837-2842. 10. American Academy of Pediatrics. American College of Critical Care Medicine. Consensus report for regionalization of services for critically ill or injured children. Pediatrics. 2000;105:152-155. 11. EMSC National Resource Center. Checklist of Essential Pediatric Domains and Considerations for Every Hospital’s Disaster Preparedness Policies. Washington, DC: EMSC National Resource Center; 2014. 12. Auf der Heide E. The importance of evidence-based disaster planning. Ann Emerg Med. 2006;47:34-49. 13. Deleted in review. 14. King MA, Koelemay K, Zimmerman J, et al. Geographical maldistribution of pediatric medical resources in Seattle-King County. Prehosp Disaster Med. 2010;25(4):326-332. 15. Hazards Vulnerability Analysis. Emergency Preparedness. California Hospital Association; 2017. https://www.calhospitalprepare.org/hazardvulnerability-analysis. 16. US Department of Homeland Security. National response framework. 4th ed. Washington, DC: Department of Homeland Security; 2019. https://www.fema.gov/media-librar y/assets/documents/ 117791. 17. Phillips SJ, Knebel A. Mass Medical Care with Scarce Resources: A Community Planning Guide. Rockville, MD: Agency for Healthcare Research and Quality (AHRQ Publication No. 07–0001); 2007. 18. Kanter RK, Cooper A. Mass critical care: pediatric considerations in extending and rationing care in public health emergencies. Disaster Med Public Health Prep. 2009;3:S166-S171. 19. Kissoon N, Task Force for Pediatric Emergency Mass Critical Care. Deliberations and recommendations of the Pediatric Emergency Mass Critical Care Task Force: executive summary. Pediatr Crit Care Med. 2011;12(suppl 6):S103-S108. 20. Gostin LO, Sapsin JW, Teret SP, et al. The Model State Emergency Health Powers Act. JAMA. 2002;288:622-628. 21. Mahoney EJ, Harrington DT, Biffl WL, et al. Lessons learned from a nightclub fire: institutional disaster preparedness. J Trauma. 2005; 58:487-491. 22. Eriksson CO, Uyeki TM, Christian MD, et al. Care of the child with Ebola virus disease. Pediatr Crit Care Med. 2015;16:97-103. 23. Kanter RK, Moran JR. Pediatric hospital and intensive care unit capacity in regional disasters. Pediatrics. 2007;119:94-100.

24. Chevalier MS, Chung W, Smith J, et al. Ebola virus disease cluster in the United States–Dallas County, Texas, 2014. MMWR Morb Mortal Wkly Rep. 2014. https://www.cdc.gov/mmwr/preview/ mmwrhtml/mm63e1114a5.htm. 25. Lerner EB, Schwartz RB, Coule PL, et al. Mass casualty triage: an evaluation of the data and development of a proposed national guidance. Disaster Med Public Health Prep. 2008;2:S25-S34. 26. Kenningham K, Koelemay K, King MA. Pediatric disaster triage education and skills assessment: a coalition approach. J Emerg Manag. 2014;12:141-151. 27. Fertel BS, Kohlhoff SA, Roblin PM. Lessons from the “Clean Baby 2007” pediatric decontamination drill. Am J Disaster Med. 2009;4: 77-85. 28. Freyberg CW, Arquilla B, Fertel BS, et al. Disaster preparedness: hospital decontamination and the pediatric patient—guidelines for hospitals and emergency planners. Prehosp Disaster Med. 2008;23: 166-173. 29. US Department of Homeland Security. Patient Decontamination in a Mass Chemical Exposure Incident: National Planning Guidance for Communities. 2014. https://www.dhs.gov/sites/default/files/ publications/Patient%20Decon%20National%20Planning%20 Guidance_Final_December%202014.pdf. 30. Siegel JD, Rhinehart E, Jackson M, et al. Guideline for Isolation Precautions: Preventing Transmission of Infectious Agents in Healthcare Settings. 2007; https://www.cdc.gov/infectioncontrol/guidelines/ isolation/index.html/Isolation2007.pdf. 31. American Academy of Pediatrics Committee on Fetus and Newborn. Levels of neonatal care. Pediatrics. 2012;130(3):587-597. 32. Kanter RK, Andrake JS, Boeing NM, et al. A method for developing consensus on appropriate standards of disaster care. Disaster Med Public Health Prep. 2009;3:27-32. 33. Strategic National Stockpile. Public Health Emergency. US Department of Health & Human Services; 2020. https://www.phe.gov/ about/sns/Pages/default.aspx. 34. Rubinson L, Vaughn F, Nelson S, et al. Mechanical ventilators in US acute care hospitals. Disaster Med Public Health Prep. 2010;4(3):199-206. 35. West JB. The physiological challenges of the 1952 Copenhagen poliomyelitis epidemic and a renaissance in clinical respiratory physiology. J Appl Physiol. 2005;99:424-432. 36. O’Hara JF, Higgins TL. Total electrical power failure in a cardiothoracic intensive care unit. Crit Care Med. 1992;20:840-845. 37. Norcross ED, Elliott BM, Adams DB, et al. Impact of a major hurricane on surgical services in a university hospital. Am Surg. 1993;59:28-33. 38. Nates JL. Combined external and internal hospital disaster: impact and response in a Houston trauma center intensive care unit. Crit Care Med. 2004;32:686-690. 39. Barkmeyer BM. Practicing neonatology in a blackout: the University Hospital NICU in the midst of Hurricane Katrina: caring for children without power or water. Pediatrics. 2006;117:S369-S374. 40. Gervais HW, Eberle B, Konietzky D, et al. Comparison of blood gases of ventilated patients during transport. Crit Care Med. 1987;15:761-763. 41. Hurst JM, Davis K, Branson RD, et al. Comparison of blood gases during transport using two methods of ventilatory support. J Trauma. 1989;29:1637-1640. 42. Johannigman JA, Branson RD, Johnson DJ, et al. Out-of-hospital ventilation: bag valve device vs transport ventilator. Acad Emerg Med. 1995;2:719-724. 43. Branson RD, Johannigman JA, Daugherty EL, et al. Surge capacity mechanical ventilation. Respir Care. 2008;53:78-90. 44. Kumar A, Zarychanski R, Pinto R, et al. Critically ill patients with 2009 Influenza A (H1N1) infection in Canada. JAMA. 2009;302: 1872-1879. 45. Luten R, Zaritsky A. The sophistication of simplicity: optimizing emergency dosing. Acad Emerg Med. 2008;15:461-465. 46. Espiritu M, Patil U, Cruz H, et al. Evacuation of a neonatal intensive care unit in a disaster: lessons from Hurricane Sandy. Pediatrics. 2014;134(6):e1662-e1669.

References

e2















52. Kanter RK. Would triage predictors perform better than first-come, first-served in pandemic ventilator allocation? Chest. 2015;147(1): 102-108. 53. Johnson EM, Diekema DS, Lewis-Newby M, et al. Pediatric triage and allocation of critical care resources during disaster: northwest provider opinion. Prehosp Disaster Med. 2014;29(5): 455-460. 54. Schreiber M. The psySTART Rapid Mental Health Triage and Incident Management System. Center for Disaster Medical Sciences, University of California; 2010. 55. Eriksson, CA, Foy, DW, Larson, LC: When the helpers need help: early intervention for emergency and relief services personnel. In: Litz BT, ed. Early Intervention for Trauma and Traumatic Loss. New York: Guilford Press; 2004:241-262.

















47. Kanter RK. Regional variation in critical care evacuation needs for children after a mass casualty incident. Disaster Med Public Health Prep. 2012;6(2):146-149. 48. Foltin GL, Schonfeld DJ, Shannon MW. Pediatric Terrorism and Disaster Preparedness: A Resource for Pediatricians. Rockville, MD: Agency for Healthcare Research and Quality (AHRQ publication No. 06-0056-EF); 2006. 49. Powell T, Christ KC, Birkhead GS. Allocation of ventilators in a public health disaster. Disaster Med Public Health Prep. 2008;2:20-26. 50. Ely DM, Driscoll AK, Mathews TJ. Infant Mortality By Age at Death in the United States, 2016. NCHS Data Brief, no 326. Hyattsville, MD: National Center for Health Statistics; 2018. 51. Christian MD, Sprung CL, King MA, et al. Triage: care of the critically ill and injured during pandemics and disasters: chest consensus statement. Chest. 2014;146(suppl 4):e61S-e74S.

e3

Abstract: During a public health emergency (PHE) such as a natural disaster or pandemic, a large number of infants, children, and young adults may need critical care in order to survive. During such an event, the incident command system provides a framework to support decision-making and coordinate efforts across affected sites. Because pediatric critical care is highly specialized and because few nonpediatric providers are comfortable

caring for severely ill or injured children, pediatric, neonatal, and cardiac intensive care units represent an essential aspect of patient management during a PHE and should be included within a structured response. Key words: Emergency mass critical care, EMCC, public health emergency, PHE, natural disaster, pandemic

10 Lifelong Learning in Pediatric Critical Care STEPHANIE P. SCHWARTZ, LAURA MARIE IBSEN, AND DAVID A. TURNER





Pediatric critical care medicine (PCCM) is a discipline dedicated to the care of the critically ill child, focusing on the sick child as a whole and including the impact of disease on all organ systems. In addition, pediatric intensivists must address and understand the physical, mental, and emotional needs of the child and the child’s family. The complex needs of the critically ill child also require that intensivists be prepared to assume a leadership role in the coordination of care among team members from multiple disciplines. The pediatric intensivist must develop an understanding of the ethics of critical care medicine and be able to balance complex and high-technology care with humanistic principles and respect for the patient as a human being. The intensivist must be knowledgeable in patient safety and quality improvement methodology and lead these efforts in the intensive care unit (ICU) environment. Skills for evaluating medical literature, clinical and/or basic science research, and the ability to teach learners at different levels and from a variety of disciplines effectively are also invaluable. Development of this complex array of knowledge and skills begins in medical school, with the goal of mastery over the course of an individual’s career. Becoming a master in the specialty of pediatric critical care hinges on lifelong learning, which implies that the described individual has a voluntary interest in self-development and learning for the sake of learning. This enjoyment associated with learning is thought to be moldable and 66





The practice of pediatric critical care medicine requires a broad knowledge base and skill set that necessitates lifelong learning throughout an intensivist’s career to achieve mastery. Based on adult learning principles, education efforts should emphasize active participation and practice, examples of which include bedside teaching, procedural training, debriefing, and simulation. Training in critical care medicine should reflect a structured process that progressively transfers increasing levels of responsibility for decision-making to the learner. Entrustable professional activities describe an ability to perform a task or responsibility without direct supervision once















PEARLS sufficient competency is attained. Milestones provide behavioral descriptors that indicate developmental progression along competencies. Continuing medical education and maintenance of certification programs are working together to incorporate adult learning principles. The mature clinician reflects on one’s daily medical experiences to place them in a larger context of previous encounters and critically evaluates one’s own performance, acknowledging both effective and ineffective aspects of patient care.

able to be influenced, even developed and promoted through the use of adult learning principles.

Adult Learning Theory in Medical Education Adult learning theory was theorized and modeled by Malcom Knowles in the 1970s.1 He identified six principles of adult learning (Box 10.1). Knowles drew on the work of Kolb,2 using these principles to emphasize that there is not a one-size-fits-all approach to learning. For example, the reader might imagine two individuals who purchase a new electronic device. Whereas one may take the device out of the box, immediately turn it on, and begin experimenting with its features, the other purchaser may not even remove it from the box before reading the entire instruction manual. Adult learning theory celebrates the differences in the approach to learning while making these differences overt and explicit. In designing and implementing curricula and assessments, medical educators may design curricula and evaluations that use these concepts. Kolb described effective learning as a progression through a cycle of stages— having a concrete experience, followed by reflection on that experience, leading to information synthesis and future testable hypotheses. For those familiar with quality improvement principles, it is not unlike plan-do-study-act,3 in which small tests

• BOX 10.1 Knowles Principles of Adult Learning 1. Adults are: • internally motivated and self-directed • goal oriented • relevancy oriented • practical 2. Adults bring life experiences and knowledge of learning experiences 3. Adult learners like to be respected

of change are implemented, observed, and the necessary modifications determined. Building on these principles, a key element of medical education is to use learner assessment to drive teaching methods. Stuart and Hubert Dreyfus developed a model of skill acquisition based on their studies of fighter pilots.4 The Dreyfus



CHAPTER 10

Lifelong Learning in Pediatric Critical Care

67

model proposes that skill acquisition is not different from the continuum of human development, with stages of skill acquisition designated as novice, advanced beginner, competent, proficient, expert, and—finally—master. The learner needs to acquire certain skills and learn certain concepts at each level; therefore teaching methods have to match the level of development (Table 10.1).5 Adult learning is fundamentally different from childhood learning because of the greater depth and breadth of experiences and knowledge on which adults build new experiences.6,7 In order to assimilate new information, adults must be able to integrate new ideas with what they already know, and information that conflicts with this knowledge may not be quickly integrated.8 Adults are self-directed and autonomous. They learn best when they are active participants in the learning process and are allowed to practice newly acquired skills and concepts.7,9 As a consequence, education for adults is typically most effective when

TABLE Dreyfus and Dreyfus Model of Skill Development Applied to the Development of a Competence in the 10.1 Subspecialty of Critical Care Medicine

Level of Learning and Characteristics

Examples of Learner Level in Critical Care Medicine

Teaching Implications Teach basic critical care concepts Point out subtle but meaningful diagnostic information in the history and physical examination Eliminate irrelevant information Highlight discriminating features and their importance to the diagnosis Encourage reading about 2 diagnostic hypotheses at the same time

Novice

First-Year Fellow

Rule driven Uses analytical reasoning and rules to link cause and effect Synthesis of information is based on knowledge acquired during residency training Big picture elusive

Interviews patient and performs a physical exam that is focused on the critical illness May not be able to focus the information on the basis of a differential diagnosis Does not see the big picture

Advanced Beginner

Second-Year Fellow

Sorts through rules and information to decide what is relevant on the basis of past experience Uses analytical reasoning and pattern recognition to solve problems Able to abstract from concrete and specific information to more general aspects of a problem

Expose learner to clinical cases proceeding from common to uncommon Can generate more specific differential diagnosis Emphasize the use of semantic qualifiers while obtaining history and physical exam Encourage formulation and verbalization of Capable of filtering relevant information to fordifferential diagnosis and treatment plan mulate a unified summary of the case Good coaching: help learner become attentive to Can abstract pertinent positives and negatives the meaningful pieces from the review of systems and incorporate them into the history of present illness

Competent

Third-Year Fellow

Emotional buy-in allows learner to feel appropriate level of responsibility More expansive experience tips the balance on clinical reasoning from methodical and analytic to identifiable pattern recognition of common clinical problems Sees the big picture Complex/uncommon problems still require reliance on analytical reasoning

Balance supervision with autonomy in decisionmaking Recognizes common patterns of illness based on Hold learners accountable for their decisions previous encounters Do not tell learners what to do; ask what they want Sees consequences of clinical decisions, which to do leads to emotional buy-in to learning Critical for learner to see a breadth and depth of Will methodically attempt to reason through patient encounters to construct and store in complex or uncommon problems memory a large repertoire of illness scripts Responsible for decision-making process Tip the balance from clinical reasoning to pattern recognition

Proficient

Clinical Instructor

Breadth of past experience allows reliance on pattern recognition of illness Problem solving intuitive Still needs to fall back to methodical and analytic reasoning for managing problems because exhaustive number of permutations and responses to management have provided less experience in this regard than in illness recognition Is comfortable with evolving situations, able to extrapolate from a known situation to an unknown situation Can live with ambiguity

Starts to match findings with those encountered in past experience Data gathering more effective and efficient Sees patient through different lens than the student Engages in process of clinical reasoning to find the best intervention

Needs to work alongside and be mentored by an expert Must develop capacity to know ones’ limitations and step back and call on additional resources when stretched beyond one’s capabilities

68

SECTION I



Pediatirc Critical Care: The Discipline

TABLE Dreyfus and Dreyfus Model of Skill Development Applied to the Development of a Competence in the 10.1 Subspecialty of Critical Care Medicine—cont’d

Level of Learning and Characteristics

Examples of Learner Level in Critical Care Medicine

Expert

Assistant Professor

Thought, feeling, and action align into intuitive problem recognition and intuitive situational responses and management Open to noticing the unexpected Clever Discriminates features that do not fit a recognizable pattern

Teaching Implications

Keep up the challenge Broad repertoire of illness scripts, based on clini- Needs ongoing experience and ongoing exposure to interesting and complex cases to avoid complacal experience that allows immediate action cency and to help transcend beyond this level for majority of clinical encounters Should be apprenticed to a master who models the Likes to deal with diagnostic dilemmas skills of the reflective practitioner and a comWhen presented with diagnostic dilemma, will mitment to lifelong learning slow down and look it up

Master

Associate Professor/Professor

Exercises practical wisdom Goes beyond the big picture to that of culture and context of each situation Deep level of commitment to the work Great concern for right and wrong decisions that fosters emotional engagement Intensely motivated by emotional engagement to pursue ongoing learning and improvement Reflects in, on, and for action

The clinician that everyone goes to with problem cases Recognizes subtle features of a current case reminiscent of cases seen over the years Painstakingly revisits past cases or identifies common thread that will help treat the current clinical problem Vision extends beyond individual practice Contributes to bigger context to improvements in the field Intense internal drive to learn and improve Practical wisdom

Self-motivated to engage in lifelong learning and practice improvement

Modified from Carraccio CL, Benson BJ, Nixon LJ, Derstine PL. From the educational bench to the clinical bedside: Translating the Dreyfus developmental model to the learning of clinical skills. Acad Med. 2008;83(8):761–767.

programs facilitate self-learning with specific goals of acquiring practical information. Efforts to be inclusive of curricular methods that support adult learning principles are occurring in undergraduate, graduate, and continuing medical education. Problem-based and small-group learning, flipped classrooms, and simulation exercises allow many venues for reaching learners in different ways. Didactic learning remains firmly in place. It should be emphasized that no one method of instruction has been definitively proven to produce better learning outcomes than another.9–11 Table 10.2 depicts varied instructional techniques with potential benefits and costs. If assessment truly drives learning, medical educational curricula must be increasingly grounded in the assessment of knowledge and skills acquisition, now defined as abilities (or entrustable professional activities) and composed of individual competencies. For example, one could consider a teenager first learning to drive a car. The teenager must be competent in many individual areas, such as knowledge of the laws of the road and the skills of braking, using turn signals, mirrors, and seatbelts before embarking on this activity and being entrusted to drive the car. Like supervising a learner performing a technical procedure on a critically ill child, the trust that a parent affords a child in independent driving is fluid. The teenager may initially receive parental permission for driving around the neighborhood. When demonstrating responsible and safe driving conduct, the teenager may gain parental trust to drive on the freeway or with friends. Likewise, the graduated autonomy that a supervising intensivist will allow learners in performing central line placement will vary according to individual knowledge and skills, but it is also highly contextual. As is reflected in the 2004 guidelines for critical care medicine training and continuing medical edition published by the Society for Critical Care Medicine, training in critical care medicine should

reflect a structured process that progressively transfers increasing levels of responsibility for decision-making and that ensures continued training in practical aspects of care.12

Graduate Medical Education The landscape of graduate medical education (GME) has dramatically evolved since its apprenticeship/house officer origins in the early 1900s. In the past decade, increasing scrutiny has been placed on GME, with a specific focus on duty hours. In 2011 the Accreditation Council for Graduate Medical Education (ACGME) placed restrictions on duty hours in an effort to increase safety for patients and learners based on some data to suggest that sleep deprivation and fatigue causes errors, and that alertness and performance vary within different points in the circadian rhythm.13–15 These restrictions undeniably changed the landscape of learning. For example, duty hour regulations led to an increase in the number of times that care of a patient was transferred to a different provider, which prompted educational reform around transitions of care with programs such as I-PASS (illness severity, patient summary, action list, situational awareness, and synthesis by receiver).16 Following two large randomized control trials showing noninferiority with regard to patient outcomes and resident satisfaction or well-being,17,18 the ACGME issued revised guidelines in 2017, allowing for more flexibility with regard to duty hours and, most importantly, stressing the importance of teamwork, physician well-being, flexibility, and patient safety.19 Along with changes in expectations around hours worked, expectations for GME programs have also evolved to focus more on patient safety, quality, and teamwork, along with the traditional specialty- and subspecialty-specific content that is important for new physicians.



CHAPTER 10

Lifelong Learning in Pediatric Critical Care

69

TABLE Instructional Techniques According to Potential Costs and Benefits 10.2

Potential Benefits

Potential Costs

Didactic learning

Traditional lecture

Teacher led

Easy to organize Inexpensive (both in teacher time and facility cost) Allows for independent learning

Noninteractive Noncollaborative

Problem-based learning

Small group cases

Student led Teacher facilitated

Active learning Motivation for learning Allows for complex thought Collaborative and interprofessional

Facilitator skill required High faculty, time, and space costs Students may have different styles and learning needs that are incompatible

Team-based learning

Out-of-class preparation followed by in- class application

Student led Teacher facilitated

Active learning Motivation for learning Allows for complex thought Collaborative and interprofessional Multiple assessments

Facilitator skill required High faculty, time, and space costs Advance student preparation required Students may have different styles and learning needs that are incompatible

Simulation

Practice of scenarios in simulated environments

Teacher led Students active

Hands-on skills practiced Literature from other industries (airlines) supports utility Collaborative and interprofessional

Resource and time expensive Can seem contrived/unrealistic

Social media

Online Internet platform Student and teacher Real-time discussion facilitated for knowledge sharing Collaborative and interprofessional and discussion Instant access and knowledge of most up-to-date literature

Accreditation Council for Graduate Medical Education Core Competencies, Milestones, and Entrustable Professional Activities The quality of education in training programs is critical to resident and fellow development. In 1999, the ACGME initiated an outcome project to design a conceptual framework for education and training according to six general domains of competency.10 The objective of the outcome project was to “ensure and improve the quality of GME.”11 The ACGME recommends that trainees demonstrate (1) patient care that is compassionate, appropriate, and effective for the treatment of health problems and the promotion of health; (2) medical knowledge regarding established as well as evolving biomedical, clinical, and cognitive sciences with the ability to apply these concepts to patient care; (3) practice-based learning and improvement involving self-evaluation with regard to patient care, appraisal, and utilization of scientific evidence; (4) interpersonal and communication skills that result in effective information exchange and partnership with patients, their families, and other health professionals; (5) professionalism manifested through a commitment to professional responsibilities, adherence to ethical principles, and sensitivity to a diverse patient population; and (6) an awareness of and responsiveness to the healthcare system and the ability to use system resources to provide optimal care in a systems-based practice.10 These core domains of competency should be used to guide and coordinate evaluation of all residents or fellows in their development.20 In order to fulfill the promise of the Outcome Project21 to use educational outcome data in accreditation, the Pediatric Milestone Project, a partnership between the ACGME and American Board of Pediatrics (ABP), was designed for the evaluation of resident physicians participating in ACGME-accredited residency

Potential distraction Not curated or monitored

or fellowship training programs. Milestones, which are now routinely used in GME evaluations, describe the performance levels that residents and fellows are expected to demonstrate for skills, knowledge, and behaviors in the six competency domains. They are intended to provide a developmental framework of observable behaviors and attributes for learner assessment. Although they are not intended to address all competencies, milestones are anchored contextually in the development of the physician in key elements of the competencies. The use of milestone assessment has a number of benefits for residents and fellows, including increased transparency of performance requirements by more explicit expectations, better feedback, and enhanced opportunities for early identification of underperformers. In addition to the milestones, which have now become standard assessment tools in GME training, most specialties and subspecialties have begun developing entrustable professional activities (EPAs), which can be defined as a representation of the tasks associated with all of the work within a specific field.22,23 Leaders in pediatrics and the pediatric subspecialties, through the work of the ABP, have developed a set of EPAs for both pediatrics and for the pediatric subspecialties. PCCM has three subspecialtyspecific EPAs, which supplement several EPAs that cross all specialties, along with those for general pediatrics.24 While EPAs have not yet become a standard method for assessment of learners, data are beginning to emerge on their implementation and use as an assessment tool in this specialty.25,26

Methods of Teaching Given the complexity of the ICU environment and the wide range of learners and educators, leaders of training programs should consider the importance of the quality of teaching

70

SECTION I



Pediatirc Critical Care: The Discipline

methods. The faculty members responsible for supervision in the ICU are the content experts and are often also expected to be the facilitators/educators for learners at a range of levels and from a variety of disciplines. To be an effective educator, there must be a clear understanding that a gap often exists between educators and learners regarding perceptions of adequate teaching and optimal teaching techniques.27 One must overcome recognized barriers to education, which include lack of dedicated teaching time, high

clinical workload, lack of continuity between faculty and learners, and balancing autonomy and supervision.27–34 These factors are increasingly challenging in the current era.34 Education in the ICU consists of teaching basic principles of pathophysiology and therapeutics but should also include an ongoing, dynamic integration of new medical knowledge and technological advances. Table 10.3 demonstrates the broad scope of educational objectives for critical care medicine fellows and

TABLE Essential Clinical and Administrative Learning Points 10.3

Clinical

Administrative

Identify and teach others to identify the need for/provide care for all critically ill patients.

Evaluate ICU policies and suggest improvements.

Provide and teach others resuscitation for any patient sustaining a life-threatening event.

Triage critically ill patients to optimize care delivery within the institution.

Initiate, manage, and wean patients from mechanical ventilation and teach others new methods and devices for management of respiratory failure.

Improve resource utilization and maintain patient care quality by facilitating triage of patients to limited institutional critical care beds and caregivers.

Initiate critical care to stabilize and manage patients who require transport.

Develop programs and change unit practice to improve care of critically ill patients.

Instruct other qualified caregivers and the lay public in the theory and techniques of cardiopulmonary resuscitation.

Develop programs for patient safety monitoring and error reduction.

Treat cardiogenic, traumatic, hypovolemic, and distributive shock with conventional and state-of-the-art approaches.

Actively participate in quality assurance processes, including morbidity and mortality conferences, process improvement teams, and the Joint Commission preparation.

Recognize potential for multiple organ failure and institute measures to avoid or reverse this syndrome.

Support the process of assessing patient and family satisfaction and participate in tool development and implementation.

Identify life-threatening electrolyte/acid-base disturbances, provide treatment/monitor outcome.

Encourage and enhance good relationships with other healthcare providers.

Identify and initiate discussions involving ethical issues and parents/patients’ wishes in making treatment decisions using advance directives and other methods.

Understand advanced concepts important for compensation of critical care services and contractual issues related to providing critical care services and performing the business of medicine.

Diagnose and treat common and uncommon poisonings.

Develop skills for teaching critical care.

Teach appropriate use and monitoring of procedural sedation and use advanced pain management strategies.

Develop and evaluate curriculum changes for ICU caregivers, fellows, and residents.

Diagnose malnutrition and use/monitor advanced nutrition support methodologies.

Evaluate, modify, and approve ICU hospital policies.

Provide invasive and noninvasive monitoring for titrating therapy. Prioritize complex data to support action plan.

Improve resource utilization and maintain patient care quality by planning for future needs for institutional and regional critical care resources.

Use and teach medication safe practice guidelines and determine cost- effectiveness of therapeutic interventions.

Develop programs and change unit, institution, and regional practice to improve care of critically ill patients.

Develop skills of ICU nurses and ancillary personnel in caring for critically ill patients and provide in-service education.

Use existing tool sets to assess patient and family satisfaction and direct the development of new tools when appropriate.

Use, teach, and help enforce methods of infection control.

Develop programs and document improvement in patient safety monitoring and error reduction.

Communicate effectively with patients, families, and other involved members of the healthcare team about all treatment decisions and patient prognosis.

Develop high-quality relationships with other healthcare providers.

Continue to augment knowledge by assimilating peer-reviewed published medical literature through self-directed learning and CME activities.

Teach the business of medicine.

Diagnose and treat a sufficient number of patients with critical illness using conventional and state-of-the-art approaches to maintain clinical proficiencies.

Develop collaborative and productive relationships with other specialist physicians and model joint clinical planning in managing complex ICU problems.

Modified from Dorman T, Angood PB, Angus DC et al: Guidelines for critical care medicine training and continuing medical education. Crit Care Med. 2004;32(1):263–272.

intensivists per the guidelines from the Society of Critical Care Medicine and includes two broad areas of learning: clinical and administrative. Teaching tools should be designed and selected to optimize improvement of both physician performance and healthcare outcomes. Curricular development should focus on development of effective programs that include sequenced and multifaceted activities.9 A review that evaluated 37 studies of continuing medical education activities demonstrated that the use of multiple media, a variety of instructional techniques, and exposures to content to meet instructional objectives are all needed to improve clinical outcomes.35 A recent review of teaching techniques in critical care demonstrates the importance and benefit of multiple interactive strategies that apply principles of adult learning.36,37 The practice of medicine is evolving at a rapid pace, and teaching strategies must also continue to evolve. While bedside teaching and didactic lectures are important, innovative strategies are needed to maximize education in the current era. For example, debriefing is a teaching strategy that is integrated into simulationbased education and is increasingly being used within the context of clinical care in the ICU to teach important principles. A debrief is a review of a situation led by an experienced facilitator to allow learners to explore steps that went well and identify opportunities for improvement and learning.38 A survey of PCCM program directors demonstrated that faculty role modeling is the most common technique used in pediatric critical care programs to teach the competencies of professionalism and communication.39

Bedside Teaching In 1964, Reichsman et al. demonstrated a 75% “incidence” of teaching at the bedside during attending rounds.40 That number had declined to 16% in 197841 and may be lower today. Little data exist on the frequency of bedside teaching in the ICU in the modern era of medical education. Experiential learning and casebased learning at the bedside (both of which are based on principles of adult learning) are traditionally thought to be the most effective means of educating clinicians in the understanding of disease processes and evaluation and management of critically ill patients. The impact of experiential learning at the bedside caring for a patient with septic shock and multiorgan dysfunction is extremely valuable and difficult to quantify. Less dramatic but potentially equally effective are instances when one palpates a thrill, hears a gallop, feels an enlarged liver or spleen, listens to wheezing or stridor, or performs a detailed neurologic examination in a patient who has experienced a stroke or a spinal cord injury. Medical technology can be leveraged to enhance bedside teaching, as when flow-volume loops in a child with asthma are used to teach the principles of mechanical ventilation in the setting of bronchospasm, or the waveforms on bedside monitors allow a demonstration of the impact of ventilation on hemodynamics. Showing learners extracorporeal membrane oxygenation circuits, continuous venovenous hemodialysis and ultrafiltration machines, high-frequency ventilators, and ventricular assist devices while they are being used in patient care reinforces prior learning and may provide the “a-ha!” moment that is difficult to recreate in didactic lectures. In addition, when trainees present the historical data and physical findings of their patients to the attending physician and the team during bedside rounds, they can be taught to describe the information relating to their patients in a succinct manner and discriminate between important and less important information, develop a list of differential diagnoses, and formulate a treatment plan. Bedside and experiential learning teaches the



CHAPTER 10

Lifelong Learning in Pediatric Critical Care

71

important skill of thinking on one’s feet, which will assist in drawing correct conclusions regarding diagnosis and making management decisions. Bedside teaching and rounds are also important for the demonstration and role modeling of professionalism. It is important to remember that patients are often the best teachers, and developing into an outstanding practitioner hinges on exposure to a large and varied number of clinical situations to develop all of the domains of competency.

Procedural Training Procedural expertise is crucial in the care of critically ill children. Specific skills that must be acquired during fellowship include peripheral arterial and venous catheterization, central venous catheterization, endotracheal intubation, thoracostomy tube placement, and procedural sedation.42 In addition, knowledge of the indications, technique, limitations, and complications of other procedures—including cricothyroidotomy, tracheostomy, pericardiocentesis, and abdominal paracentesis—is recommended.12,43,44 There are a wide range of approaches to teaching procedures, which include didactics, video and web-based training, and simulation. Regardless of the specific techniques implemented, initial performance under direct supervision is paramount, with progressive development of autonomy through the course of fellowship training. A recent single-center study demonstrated a statistically significant decrease in the number of central lines and arterial lines performed by PCCM fellowship trainees over the past 10 years.45 While medical advances in recent years have allowed for less invasive forms of interventions, it is critical that educators assess the potential impact of fewer procedures to ensure competency of those in the field.45

Simulation Training Simulation training is an interactive technique used to replicate real-world situations and experiences.46 Modern medical simulation originated in the early 1960s with simple resuscitation manikins that were used in the field of anesthesiology; this technique has both expanded and evolved substantially over time. Technological advances in computer programming and manikin development as well as increased affordability have contributed to increasing use of this educational modality.47 For all levels of learners, simulation is a valuable educational modality for critical care.48 Medical simulation can be generally divided into five different categories: role-play, standardized patients, partial task trainers, computerized patients, and electronic patients. The most comprehensive form of simulation training is the electronic patient or high-fidelity, instructor-driven manikin.46 These manikins are programmed to display normal and abnormal physical examination findings. The training session is divided into three sections: prebriefing, scenario exercise, and debriefing. Debriefing plays an important role in the educational process, allowing the instructor to reemphasize the objectives of the exercise and address knowledge gaps that may have been unmasked during the scenario. Debriefing also allows the trainees to reflect on performance and discuss areas for improvement.49–52 Simulation training in medical education has gained widespread acceptance as a result of the ease of simulating critical events, safety of the learning environment, and reproducibility of the clinical scenarios.53 Learners practice and develop skills without the consequences of negative patient outcomes. For procedures that are performed infrequently, simulation allows repeated opportunities to practice. Scenarios can be created to simulate a

72

SECTION I



Pediatirc Critical Care: The Discipline

range of complications that the learner might otherwise not witness or experience. Evidence demonstrates that learners enjoy participation in medical simulation, and individual and team member performance is enhanced. In addition, self-confidence, knowledge, and operational performance improve with simulation training.54–57 Simulation-based medical education with deliberate practice to attain constant skill improvement is superior to traditional clinical education for acquisition of a wide range of medical skills.58 Some centers have implemented intense simulation-based orientation training for first-year pediatric critical care medicine fellows. In addition to procedural skill improvement, simulation has been used to facilitate education in communication involving difficult conversations and end-of-life care.59,60 The ICU is a highly dynamic setting with complex patients and frequent high-risk situations and invasive procedures that need a well-functioning multidisciplinary team. In situ portable simulation provides the added advantage of replicating the true working environment of the ICU. The learners are placed in the patient care setting and have the opportunity to use the actual equipment within the environment in which they work. In situ simulation has been used to identify safety threats and reinforce resource location and teamwork behaviors.61 It allows multidisciplinary team training in the setting of their clinical work.62 Highfidelity simulations using multidisciplinary teams in the ICU have been linked to improvements in teamwork and efficiency as well as improved times to initiate key clinical tasks during resuscitation.63 In addition to improving teamwork, in the extracorporeal membrane oxygenation literature, a high-fidelity simulation curriculum among PICU personnel showed significantly improved times for response to emergencies, including changing the oxygenator and managing air embolisms.64 From the traditional code team to the performance of highrisk procedures, personnel must work together to accomplish many tasks. Training in teamwork and communication, known as crew (or crisis) resource management (CRM), was developed and implemented in the aviation industry to reduce or eliminate the contribution of human errors to catastrophic events. These principles have been adapted for use in anesthesia crisis management protocols and focus on competent team management, dynamic decision-making, interpersonal behavior, situational awareness, effective use of resources, and stress management.65 CRM training improves team functioning and dynamics in pediatric intensive care settings.66–68 Clinical skills acquired during medical simulation have been shown to directly improve patient care practices and may improve patient outcomes.53 Additional studies are needed to identify the optimal modality, frequency of exposure, quality of assessment tools, and impact of simulation education on patient care. A multicenter pediatric network (International Network for Simulation-based Innovation, Research, and Education) has been formed to measure the impact of simulation on clinical outcomes. Despite some limitations, simulation training remains an important adjunct to traditional medical education in the ICU.

Beyond Graduate Medical Education Continuing Medical Education, Board Certification, and Maintenance of Certification The speed of medical and technological advancement makes ongoing education for those who have completed formal training paramount. Education for physicians practicing in the clinical

environment falls under the umbrella of Continuing Medical Education (CME), which is designed to promote the knowledge, skills, and ongoing performance of physicians.69 CME requirements are locally managed at the institutional level, and also are determined and monitored by state licensing boards. Many states set specific standards for the amount of CME credit required annually for all licensed physicians.70 Individual institutions, private agencies, and the American Academy of Pediatrics offer CME activities to fulfill these requirements. States may also require additional education; in some states, physicians are required to complete additional pain management or opiate prescribing education. Traditionally, methods of delivering CME for physicians involve didactic experiences, such as grand rounds or other lecture series. It is reasonable to question whether these experiences are sufficient to drive outcomes in physician knowledge, practice, and—most important—patient-centered outcomes. Furthermore, evidence suggests that physicians, like all learners, are unreliable self-assessors,71 highlighting the need for external review to produce and ensure learning outcomes. Board certification and maintenance of certification complements GME and CME in certifying practitioners in standards of excellence, which are intended to lead to high-quality health outcomes for their patients. Accreditation boards such as the American Board of Pediatrics72 primarily serve the public, ensuring that physicians participate in recertification through retesting and maintenance of certification activities. The required elements for the education and training of pediatric intensivists in patient care have been defined over time and continue to evolve. The ABP and the subboard of Pediatric Critical Care Medicine have developed a list of content specifications for the subspecialty examination.43 While this list is not intended to serve as a curriculum, the pediatric intensivist sitting for the examination is expected to be familiar with the wide range of topics covered by the content specification document. The content outline is available at https://www.abp.org/sites/abp/files/ pdf/critical_care_content_outline.pdf. To qualify for the ABP subspecialty examination, applicants are required to have a valid unrestricted allopathic and/or osteopathic license to practice, initial certification in general pediatrics, and successful completion of critical care medicine training in a program accredited by the ACGME. The applicant must also provide evidence of meaningful scholarship during training, which can include research, an educational project, or a quality project. Following initial certification, the Maintenance of Certification (MOC) program ensures that the intensivist invests in ongoing knowledge and skills necessary for the delivery of quality care. MOC is based on the six competency domains that are evaluated during fellowship training and includes four parts: professional standing (medical license), lifelong learning and self-assessment, passing an examination, and performance in practice with participation in ABP-approved quality improvement activities in patient care. MOC has evolved with time. Following an extensive evaluation and an international meeting devoted to the “Future of Education in Pediatrics,” MOCA-Peds, an ongoing assessment tool that replaces the intermittent MOC examination, was developed. It is currently active in general pediatrics and being rolled out to subspecialties. Although medical knowledge may be assessed through standardized testing, it is unclear whether this or other activities for certification truly lead to improved patient outcomes. To achieve this goal, the medical community must prioritize practice-based improvement activities and experiences that incorporate adult learning principles, are highly relevant to the learner, and require

active participation. Additionally, they must be authentic on a local level, addressing local problems and opportunities. Organizations can actively incorporate CME/MOC activities into daily workflow for flexible, real-time learning and assessment. Several activities currently available allow physicians to receive both CME and MOC credit. The end of formal training as a fellow represents the beginning of a lifelong process of refining one’s knowledge and judgment, constituting the essence of professional development and maintenance of competency. While the medical profession and society at large have historically assumed physician proficiency by virtue of their continued practice, it is dangerous to assume that longevity is tantamount to expertise. Instead, mature clinicians reflect on their daily medical experiences to place them in a larger context of previous encounters, critically evaluate their own performance, acknowledging both effective and ineffective aspects of patient care, and seek to fill in gaps in knowledge or performance.52,56 Critical care is practiced in an atmosphere of innate uncertainty. In this environment, the practicing intensivist must be able to adapt in real time and, once a situation is resolved, to step back and pursue generalizable knowledge for the next encounter. Another aspect of lifelong learning is the role that teaching plays in ongoing self-directed learning. By reviewing and synthesizing contemporary literature, whether for a didactic lecture for the learner or for family and patient information, the seasoned intensivist creates an opportunity to address all aspects of professional development.

New Methods of Assessment and Future Challenges Educators and those responsible for certification at all levels are currently actively engaged in discussion of new methods and concepts of assessment. There is a growing appreciation for the value in multiple low-stakes assessments that include rich narrative feedback73 and the integration of these assessments into a composite whole. Formative and summative assessment occurs along a spectrum instead of at discrete points in time. Innovations in higher-stakes testing assessment are occurring in all phases of the test development cycle, including design and implementation of items and tests (assessment engineering, automated item generation); items and tasks used within the test (game-based assessment, computer-based simulations, use of Internet or key online resources during a test, and alternative test item types, such as drag-and-drop or multiple correct answers); item selection during the test (computer-adaptive testing); and delivery of the test (online proctoring versus test center proctoring vs. no proctoring). The rapid development of computer and Internet technology makes innovation inevitable; the resulting change needs to be managed carefully, with an eye to testing validity and reliability as well as test taker approval. Although beyond the scope of this chapter, we would be remiss to not mention factors such as the advent of the electronic health record, compensation and billing requirements, collaborative



CHAPTER 10

Lifelong Learning in Pediatric Critical Care

73

learning through social media communities (e.g., #PedsICU)74 and faculty-driven care, which have undeniably impacted the learning environment. Students, residents, and fellows have very different opportunities for learning and experiences relative to those in the past. More opportunities for hands-on care by faculty and closer supervision of learners at all levels most likely are better for individual patients, but they may not provide some of the most powerful learning experiences of past eras. Even documentation experience may be limited—in some settings, students are not permitted to document patient information in the electronic health record, raising questions about their opportunities to write notes and, more important, organize their thoughts into coherent patient assessments and plans. Intensivists and physicians in general must be attuned to environmental and technological advances, which change practices and the ways that they teach, practice, and learn.

Key References Accreditation Council for Graduate Medical Education. Common program requirements. Available at: http://www.acgme.org/Portals/0/PDFs/ Nasca-Community/Section-VI-Memo-3-10-17.pdf ?ver5201703-10-083926-603. Bilmoria KY, Chung JW, Hedges LV et al. National cluster-randomized trial of duty-hour flexibility in surgical training. N Engl J Med. February 25, 2016;374(8):713-727. Carraccio CL, Benson BJ, Nixon LJ, Derstine PL. From the educational bench to the clinical bedside: translating the dreyfus developmental model to the learning of clinical skills. Acad Med. 2008; 83:761-767. Cheng A, Hunt EA, Donoghue A, et al. Examining pediatric resuscitation education using simulation and scripted debriefing: a multicenter randomized trial. JAMA Pediatrics. 2013;167:528-536. Cook DA, Hatala R, Brydges R, et al. Technology-enhanced simulation for health professions education: a systematic review and metaanalysis. JAMA. 2011;306:978-988. Davis DA, Mazmanian PE, Fordis M, et al. Accuracy of physician self-assessment compared with observed measures of competence: a systematic review. JAMA. 2006;296:1094-1102. Dormon T, Angood PB, Angus DC, et al. Guidelines for critical care medicine training and continuing medical education. Crit Care Med. 2004;32(1):263-272. Engorn BM, Newth CJL, Klein MJ, et al. Declining procedures by pediatric critical care medicine fellowship trainees. Front Pediatr. 2018; 6:365. Knowles M. Self-Directed Learning. Chicago: Follet; 1975. Nasca TJ, Philibert I, Brigham T, Flynn TC. The next graduate medical education accreditation system: rationale and benefits. N Engl J Med. 2012;366(11):1051-1056. Silber JH, Bellini LM, Shea JA, et al. Patient safety outcomes under flexible and standard resident duty-hour rules. N Engl J Med. 2019; 380(10):905-914. Tainter CR, Wong NL, Bittner EA. Innovative strategies in critical care education. J Crit Care. 2015;30(3):550-556.

The full reference list for this chapter is available at ExpertConsult.com.

e1











































































































1. Knowles M. Self-Directed Learning. Chicago: Follett; 1975. 2. Kolb DA. Experiential Learning: Experience as the Source of Learning and Development. Upper Saddle River, NJ: Prentice-Hall; 1984. 3. Institute for Healthcare Improvement Plan-Do-Study-Act Worksheet. Available at: http://www.ihi.org/resources/Pages/Tools/ PlanDoStudyActWorksheet.aspx. 4. Dreyfus SE, Dreyfus HL. A five stage model of the mental activities involved in direct skill acquisition (Air Force Office of Scientific Research under contract F49620-79-C-0063), Berkeley, CA: University of California; 1980. Available at: https://apps.dtic.mil/dtic/tr/ fulltext/u2/a084551.pdf. 5. Carraccio CL, Benson BJ, Nixon LJ, Derstine PL. From the educational bench to the clinical bedside: translating the dreyfus developmental model to the learning of clinical skills. Acad Med. 2008;83: 761-767. 6. O’Brien G. What are the principles of adult learning? Available at: www.southernhealth.org.au/meu/articles/adult-learning.htm. 7. Collins J. Education techniques for lifelong learning principles of adult learning. Radiographics. 2004;24(5):1483-1489. 8. Zemke R, Zemke S. 30 things we know for sure about adult learning. Innovation Abstracts VI (8), March 9, 1984. Available at: http://www.muskegoncc.edu/include/CTL%20DOCS/XXIX_No4.pdf. 9. Mazmanian P, Davis D. Continuing medical education and the physician as a learner: guide to the evidence. JAMA. 2002;288(9): 1057-1060. 10. Accreditation Council for Graduate Medical Education. Common Program Requirements: General Competencies. Available at: http:// www.acgme.org/outcome/comp/GeneralCompetenciesStandards21307.pdf. 11. Accreditation Council for Graduate Medical Education. Common Program Requirements (Fellowship). Available at: https://www.acgme. org/Portals/0/PFAssets/ProgramRequirements/CPRFellowship2019.pdf. 12. Dormon T, Angood PB, Angus DC, et al. Guidelines for critical care medicine training and continuing medical education. Crit Care Med. 2004;32(1):263-272. 13. Nasca TJ, Day SH, Amis Jr ES. ACGME Duty Hour Task Force. The new recommendations on duty hours from the ACGME Task Force. N Engl J Med. 2010 8;363(2):2e3. 14. Asch DA, Parker RM. The libby zion case. N Engl J Med. 1988;318: 771-775. 15. Ulmer C, Wolman DM, Johns MME, eds. Resident Duty Hours: Enhancing Sleep, Supervision, and Safety. Washington, DC: National Academies Press; 2009. 16. Starmer AJ, Spector ND, Srivastava R, et al. Changes in medical errors after implementation of a handoff program. N Engl J Med. 2014;371:1803-1812. 17. Bilmoria KY, Chung JW, Hedges LV, et al. National cluster-randomized trial of duty-hour flexibility in surgical training. N Engl J Med. 2016 25;374(8):713-727. 18. Silber JH, Bellini LM, Shea JA, et al. Patient safety outcomes under flexible and standard resident duty-hour rules. N Engl J Med. March 7, 2019;380(10):905-914. 19. Accreditation Council for Graduate Medical Education. Common Program Requirements. Available at: http://www.acgme.org/Portals/ 0/PDFs/Nasca-Community/Section-VI-Memo-3-10-17. pdf?ver52017-03-10-083926-603. 20. Lurie S, Mooney C, Lyness J. Measurement of the general competencies of the Accreditation Council for Graduate Medical Education: a systematic review. Acad Med. 2009;84(3):301-309. 21. Nasca TJ, Philibert I, Brigham T, Flynn TC. The next graduate medical education accreditation system – rationale and benefits. N Engl J Med. 2012;366(11):1051-1056. 22. Ten Cate O. Entrustability of professional activities and competency-based training. Med Educ. 2005;39:1176-1177.

23. Ten Cate O, Hart D, Ankel F, et al. Entrustment Decision Making in Clinical Training. Acad Med. 2016;91(2):191-198. 24. American Board of Pediatrics, entrustable professional activities for subspecialties. Available at: www.abp.org/subspecialty-epas 25. Mink RB, Schwartz A, Herman BE, et al. Validity of level of supervision scales for assessing pediatric fellows on common pediatric subspecialty entrustable professional activities. Acad Med. 2018; 93(2):283-291. 26. Carraccio C, Englander R, Gilhooly J, et al. Building a framework of entrustable professional activities, supported by competencies and milestones to bridge the educational continuum. Acad Med. 2017; 92(3):324-330. 27. McCann L, Naden G, Child S. Doctors as teacher: what do they think? N Z Med J. 2009;122(1292):16-22. 28. d’Apollonia S, Abrami PC. Navigating student ratings of instruction. Am Psychol. 1997;52(11):1198-1208. 29. Wilkerson L, Irby DM. Strategies for improving teaching practices: a comprehensive approach to faculty development. Acad Med. 1998;73(4):387-396. 30. Srinivasan M, Li ST, Meyers FJ, et al. “Teaching as competency”: competencies for medical educators. Acad Med. 2011;86(10):12111220. 31. Turner DA, Narayan AP, Whicker SA, Bookman J, McGann KA. Do pediatric residents prefer interactive learning? Educational challenges in the duty hours era. Med Teach. 2011;33(6):494-496. 32. Armstrong E & Parsa-Parsi R. How can physicians’ learning styles drive educational planning? Acad Med. 2005;80(7):680-684. 33. Vaughn L, Baker R. Do different pairings of teaching styles and learning styles make a difference? Preceptor and resident perceptions. Teach Learn Med. 2008;20(3):239-247. 34. Rehder KJ, Cheifetz IM, Willson DF, Turner DA. Pediatric Acute Lung Injury and Sepsis Investigators Network. Perceptions of 24/7 in-hospital intensivist coverage on pediatric housestaff education. Pediatrics. 2014;133(1):88-95. 35. Mazmanian P, Davis D. Continuing medical education effect on clinical outcomes: effectiveness of continuing medical education: American College of Chest Physicians evidence-based education guidelines. Chest. 2009;135(3):549-555. 36. Tainter CR, Wong NL, Bittner EA. Innovative strategies in critical care education. J Crit Care. 2015;30(3):550-556. 37. Chud gar SM, Cox CE, Que LG, et al. Current teaching and evaluation methods in critical care medicine: has the Accreditation Council for Graduate Medical Education affected how we practice and teach in the Intensive Care Unit? Crit Care Med. 2009;37(1):49-60. 38. Clay A, Que L, Petrusa E, et al. Debriefing in the intensive care unit: a feedback tool to facilitate bedside teaching. Crit Care Med. 2007;35(3):738-754. 39. Turner DA, Mink RB, Lee KJ, et al. Education in Pediatric Intensive Care (EPIC) Investigators. Are pediatric critical care medicine fellowships teaching and evaluating communication and professionalism? Pediatr Crit Care Med. 2013;14(5):454-461. 40. Reichsman F, Browning FE, Hinshaw JR. Observations of undergraduate clinical teaching in action. J Med Educ. 1964;39:147-163. 41. Collins GF, Cassie JM, Daggett CJ. The role of the attending physician in clinical training. J Med Educ. 1979;53:429-431. 42. ACGME Program Requirements for Graduate Medical Education in Pediatric Critical Care Medicine. Available at: https://www.acgme.org/Portals/0/PFAssets/ProgramRequirements/323_PediatricCricitalCareMedicine_2019_TCC.pdf?ver52019-02-19141955-183. 43. The American Board of Pediatrics Content Outline Pediatric Critical Care Medicine. Subspecialty In-Training, Certification, and Maintenance of Certification (MOC) Examinations. Available at: https:// www.abp.org/sites/abp/files/pdf/critical_care_content_outline.pdf. 44. Guidelines for advanced training for physicians in critical care. American College of Critical Care Medicine of the Society of Critical Care Medicine. Crit Care Med. 1997;25:1601-1607.

References

e2



































60. Bateman LB, Tofil NM, White ML, et al. Physician communication in pediatric end-of-life care: a simulation study. Am J Hosp Palliat Care. 2016;33(10):935-941. 61. Wheeler DS, Geis G, Mack EH, LeMaster T, Patterson MD. Highreliability emergency response teams in the hospital: improving quality and safety using in situ simulation training. BMJ Qual Saf. 2013;22:507-514. 62. Weinstock PH, Kappus LJ, Garden A, Burns JP. Simulation at the point of care: reduced-cost, in situ training via a mobile cart. Pediatr Crit Care Med. 2009;10:176-181. 63. Gilfoyle E, Koot DA, Annear JC. Improved clinical performance and teamwork of pediatric interprofessional resuscitation team with a simulation-based educational intervention. Pediatr Crit Care Med. 2017;18(2):e62-e69. 64. Di Nardo M, David P, Stoppa F, et al. The introduction of a highfidelity simulation program for training pediatric critical care personnel reduces the time to manage extracorporeal membrane oxygenation emergencies and improves teamwork. J Thorac Dis. 2018; 10(6):3409-3417. 65. Howard SK, Gaba DM, Fish KJ, Yang G, Sarnquist FH. Anesthesia crisis resource management training: teaching anesthesiologists to handle critical incidents. Aviation Space Environ Med. 1992;63:763-770. 66. Thomas EJ, Williams AL, Reichman EF, Lasky RE, Crandell S, Taggart WR. Team training in the neonatal resuscitation program for interns: teamwork and quality of resuscitations. Pediatrics. 2010;125:539-546. 67. Allan CK, Thiagarajan RR, Beke D, et al. Simulation-based training delivered directly to the pediatric cardiac intensive care unit engenders preparedness, comfort, and decreased anxiety among multidisciplinary resuscitation teams. J Thor Cardiov Surg. 2010;140:646-652. 68. Nishisaki A, Nguyen J, Colborn S, et al. Evaluation of multidisciplinary simulation training on clinical performance and team behavior during tracheal intubation procedures in a pediatric intensive care unit. Pediatr Crit Care Med. 2011;12:406-414. 69. Accreditation Council for Continuing Medical Education. Available at: http://www.accme.org/. 70. Oregon Medical Board. Available at: http://www.oregon.gov/omb/ Pages/index.aspx. 71. Davis DA, Mazmanian PE, Fordis M, et al. Accuracy of physician self-assessment compared with observed measures of competence: a systematic review. JAMA. 2006;296:1094-1102. 72. The American Board of Pediatrics. https://www.abp.org. 73. Pelgrim EAM, et al. Quality of written narrative feedback and reflection in a modified mini-clinical evaluation exercise: an observational study. BMC Med Educ. 2012;12:97. 74. SS Barnes, Kaul V, Kudchadkar SR. Social media engagement and the critical care medicine community. J Intensive Care Med. 2018.





































45. Engorn BM, Newth CJL, Klein MJ, et al. Declining procedures by pediatric critical care medicine fellowship trainees. Front Pediatr. 2018;6:365. 46. Gaba DM. The future vision of simulation in health care. Qual Saf Health Care. 2004;13 (suppl 1):i2-10. 47. Rosen KR. The history of medical simulation. J Crit Care. 2008; 23:157-166. 48. McBride ME, Beke DM, Fortenberry JD, et al. Education and training in pediatric cardiac critical care. World J Pediatr Congenit Heart Surg. 2017;8(6):707-714. 49. Rudolph JW, Simon R, Raemer DB, Eppich WJ. Debriefing as formative assessment: closing performance gaps in medical education. Acad Emerg Med. 2008;15:1010-1016. 50. Fanning RM, Gaba DM. The role of debriefing in simulation-based learning. Simulation in healthcare. J Soc Simul Healthcare. 2007; 2:115-125. 51. Savoldelli GL, Naik VN, Park J, Joo HS, Chow R, Hamstra SJ. Value of debriefing during simulated crisis management: oral versus video-assisted oral feedback. Anesthesiol. 2006;105:279-285. 52. Cheng A, Hunt EA, Donoghue A, et al. Examining pediatric resuscitation education using simulation and scripted debriefing: a multicenter randomized trial. JAMA Pediatrics. 2013;167:528-536. 53. Cook DA, Hatala R, Brydges R, et al. Technology-enhanced simulation for health professions education: a systematic review and metaanalysis. JAMA. 2011;306:978-988. 54. Ottestad E, Boulet JR, Lighthall GK. Evaluating the management of septic shock using patient simulation. Crit Care Med. 2007;35:769-775. 55. Kim J, Neilipovitz D, Cardinal P, Chiu M, Clinch J. A pilot study using high-fidelity simulation to formally evaluate performance in the resuscitation of critically ill patients: The University of Ottawa critical care medicine, high-fidelity simulation, and crisis resource management I study. Crit Care Med. 2006;34:2167-2174. 56. Blackwood J, Duff JP, Nettel-Aguirre A, Djogovic D, Joynt C. Does teaching crisis resource management skills improve resuscitation performance in pediatric residents? Pediatr Crit Care Med. 2014;15: e168-e174. 57. van Schaik SM, Von Kohorn I, O’Sullivan P. Pediatric resident confidence in resuscitation skills relates to mock code experience. Clin Pediatr. 2008;47:777-783. 58. McGaghie WC, Issenberg SB, Cohen ER, Barsuk JH, Wayne DB. Does simulation-based medical education with deliberate practice yield better results than traditional clinical education? A meta-analytic comparative review of the evidence. Acad Med. 2011;86:706-711. 59. Johnson EM, Hamilton MF, Watson RS, et al. An intensive, simulation-based communication course for Pediatric Critical Care Medicine Fellows. Pediatr Crit Care Med. 2017;18(8)e348-e355.

e3

Abstract: Pediatric critical care medicine is a discipline dedicated to the care of the critically ill child, focusing on the sick child as a whole and including the impact of disease on all organ systems. Becoming a master in our specialty hinges on lifelong learning, which is best accomplished using adult learning principles. The mature clinician reflects on one’s daily medical experiences to

place them in a larger context of previous encounters and critically evaluates one’s own performance, acknowledging both effective and ineffective aspects of patient care. Key Words: adult learning, graduate medical education, maintenance of certification, continuing medical education

SECTION

II

Pediatric Critical Care: Tools and Procedures 11. Essential Concepts in Clinical Trial Design and Statistical Analysis 76 12. Prediction of Short-Term Outcomes During Critical Illness in Children 82 13. Pediatric Critical Care Transport 89  



14. Pediatric Vascular Access and Centeses 94 15. Ultrasonography in the Pediatric Intensive Care Unit  114



75

11 Essential Concepts in Clinical Trial Design and Statistical Analysis LESLIE A. DERVAN, R. SCOTT WATSON, AND MARY E. HARTMAN

“You can best learn statistical methods by applying them to data which interest you.” —PETO, PIKE, ARMITAGE, ET AL.

PEARLS • Clinical trials require appropriate design, conduct, and analysis in order to provide valid, unbiased, and reliable results that will be useful to clinicians. • Trial design is more important than analysis; while statistical analysis can be adjusted at any time, study design flaws that introduce bias cannot always be corrected after a study is complete. • Critical design elements include selecting the appropriate population, determining the necessary sample size, and

selecting a clinically meaningful, readily interpreted outcome to study. • Randomization, blinding, and intention-to-treat analysis reduce bias in study results. • Statistical analysis aims to estimate the treatment effect, calculate uncertainty around that estimate, and calculate the likelihood that the effect was identified by chance.

Purpose of a Clinical Trial

Clinical Trial Design

Before recommending an intervention (e.g., a medication, therapy, or practice) to a patient, clinicians need to know whether it is effective. When asking whether a medication is more effective than placebo, or whether it is superior to another medication, or whether a new screening practice improves outcome, a trial creates a standardized setting in which researchers can determine whether that intervention has the desired effect. The ability of a clinical trial to demonstrate the effect of an intervention reliably rests on its design. Trials that answer a clinically relevant question, adequately control bias, and reduce errors due to chance with strong external validity will most inform clinical practice.1 The gold standard in assessing the efficacy of a therapy remains the randomized controlled trial (RCT). RCTs provide the bulk of evidence about interventions of interest to the medical community2 and are the focus of this chapter. RCTs require careful design, conduct, analysis, reporting, and interpretation.3 By understanding these fundamental aspects of clinical trials, clinicians can better interpret the results of new medical research and better incorporate them into practice.

Getting Started: Question and Hypothesis

76

The first step in clinical trial design is identifying a clinical question for which a randomized trial is feasible. Feasibility requires that the investigator be able to control the treatment exposure, that equipoise exists about whether it works, and that the outcome of interest happens commonly enough and soon enough for it to be observed in a study.4 Randomized trials cannot answer all questions; questions about the epidemiology, risk factors, and natural history of an illness require observational studies. If the treatment and outcome support conducting a clinical trial, researchers must next demonstrate equipoise about the treatment. Individual equipoise refers to a clinician having no preference between two treatment options. Collective equipoise refers to uncertainty or disagreement among clinicians as a community about the superiority of one treatment. Ethical oversight committees require researchers to demonstrate that sufficient collective equipoise exists to justify subjecting patients to the potential risks of a study.3,5 The clinical question must be worded in a testable format, clearly specifying the population, intervention, comparison, and outcome (PICO criteria)6 of interest. For interventional trials,



CHAPTER 11 Essential Concepts in Clinical Trial Design and Statistical Analysis

researchers state a null hypothesis, usually that the intervention will have no effect, against which the results will be judged. The null hypothesis states what the researcher is trying to disprove. For example, if a researcher wants to examine whether surfactant improves mortality in children with pediatric acute respiratory distress syndrome (PARDS), the null hypothesis might state: “Mortality in children with PARDS who receive surfactant is equal to the mortality of children with PARDS who do not receive surfactant.” Statistical analysis then allows comparison of the observed outcome to the null hypothesis, including mathematical estimates of uncertainty and calculations of the probability that the observed effect is due to chance.7

77

Another challenge in study design is identifying the optimal study population. A more homogeneous population may help the study identify an effect of an intervention at the expense of limiting the generalizability of the study results and potentially slowing enrollment. Conversely, in a broader, more generalizable population, the effect may be diluted—or even lost—if the intervention is only efficacious in a subset of study subjects.3 Enrolling a truly representative population for the clinical question reduces selection bias. For example, in the theoretical PARDS study, selection bias may occur if clinicians only refer patients with mild PARDS for enrollment. The results of this study may not accurately describe the risks and benefits of surfactant therapy among patients with severe PARDS; therefore, the results may not be generalizable to the actual patient population of interest.

.04 means that 1 out of 25 times, investigators will find a difference where none actually exists. The appropriate P value threshold for a study depends on what is at stake with the clinical question. The sample size calculation also needs to be sensitive to the desired power for a study. It is possible for researchers not to reject the null hypothesis when they should (incorrectly concluding that there is no difference when one actually exists). This is termed type II error, denoted by b. Power refers to the likelihood of avoiding a type II error, equal to 1 2 b. A study with 80% power to detect a difference between treatment groups will incorrectly reject the null hypothesis (i.e., observe no difference when one truly exists) 20% of the time, or in 1 in 5 studies. Depending on the question under consideration, this conventional threshold may be too high, but designing a higherpowered study will require enrolling more patients. Sample size calculations identify how many patients must be enrolled to estimate the treatment effect with appropriate a and b boundaries. Calculations are based on the expected outcome frequency in the control group, what difference in outcome the treatment group should experience, and what type I and type II errors the investigators find acceptable. eFig. 11.1 provides an example of how sample size, effect size, number, and a thresholds influence study power. Sometimes, external constraints—such as cost, time, number of eligible patients, and consent rates—may limit the pool of available study subjects, leading investigators to modify study assumptions, such as increasing the expected difference in outcome in the treatment group. However, such modifications make it harder to identify a smaller, but important, treatment effect. Clinicians need to know when negative results are due to an underpowered study, as such studies are not necessarily evidence that a treatment has no effect.

Power and Sample Size

Randomization

A study needs to enroll a sufficient number of patients to obtain an accurate estimate of the true treatment effect; the ideal sample size is a consequence of several factors. A smaller magnitude of intervention effect and a lower baseline outcome frequency will both increase the sample size needed to achieve a given power. For example, if a disease typically causes 30% mortality, which will be seen in the control arm, and a treatment is expected to reduce mortality to 10%, the estimate of effect is a 20% absolute risk reduction (and a 67% relative risk reduction) in mortality. This will be easier to identify and will therefore need a smaller sample size than a study of a treatment expected to reduce mortality to only 25% (a 5% absolute risk reduction and 17% relative risk reduction). If the baseline mortality is only 10% to begin with, then observing enough events to see a difference between the control and treatment groups will also require a larger sample size. The next step is setting an acceptable threshold of observing a given treatment effect by chance alone, when no effect actually exists. This is referred to as type I error, represented by a. Larger sample sizes are needed to achieve smaller type I error. The P value is the probability that the observed difference (or one greater) in a study would have been found by chance or coincidence if no effect were present. If the mortality in a study’s intervention arm is 10% versus 30% in the control arm, with a P value of .01, then in only 1 out of 100 trials would we find this 20% (or greater) absolute difference in mortality if no effect actually existed (if the null hypothesis were true). This seems unlikely, although not impossible. Conventionally, P values that are below an a threshold of .05, indicating a less than 1 in 20 probability that the study results are due to chance, are considered statistically significant. However, because the P value is a probability, even a significant P value of

Randomization is an essential feature of clinical trial design. The outcome of a study subject is affected by factors other than the study intervention, which can obscure the effect of the intervention on the outcome. Confounding classically refers to factors associated both with the exposure and the outcome outside of the causal pathway. For example, one might observe that duration of antibiotic therapy is associated with longer pediatric intensive care unit (PICU) length of stay (LOS). However, both are affected by the presence of a positive blood culture. By adjusting for presence of a positive blood culture, a less biased estimate of the true association between antibiotic duration and PICU LOS can be calculated. Statistical analyses, such as stratification or multivariable modeling, can adjust for known confounding factors after a nonrandomized trial, but nothing can be done about unknown confounding factors. Instead, if subjects are assigned to treatment groups by chance alone, both known and unknown confounding factors should be balanced across groups. These factors can no longer be associated with the treatment group assignment (which was made randomly) and will therefore no longer confound the results. In a randomized trial it is essential that study staff be unable to determine or guess which treatment group a particular patient might be enrolled into. For example, if a provider believes that an intervention is likely to be effective and is able to determine treatment group assignment in advance, that provider may try to enroll a sicker patient when the sicker patient is likely to be assigned to the treatment group. This selection bias will affect the study results because severity of illness will now be unbalanced between groups. Methods of randomization range from simple (flipping a coin) to complex (centralized, computer-generated randomized lists).8 Block randomization restricts the randomization process so

Target Population: Minimizing Variation Versus Generalizability

77e.1

Estimated power for a two-sample proportions test at two alpha thresholds and different sample sizes: When α = .01

When α = .05

Power (1 – β)

1

.5

0 100

200

300

400 100

200

300

400

Total sample size (N) Experimental-group proportion (p2) .2

.3

Parameters: p1 = .1 Horizontal lines reference 80% (dashed) and 90% (solid) power.

• eFig. 11.1





Estimated power for a two-sample proportions test under varying conditions. Power is higher when comparing a greater difference in outcome between treatment and control groups, with higher sample size, and when accepting a greater probability that the results could be due to chance (higher a). Note which conditions reach typically accepted thresholds of power; these studies are more likely to be able to observe a treatment effect when one is present.

78

SECTION II



Pediatric Critical Care: Tools and Procedures

that an equal number of group allocations is ensured within “blocks,” or small groups of serially enrolled patients, such as those at a study site in a multicenter trial. This ensures balanced numbers of patients in each treatment arm within each block. Despite randomization, a study could still have imbalance in crucial factors between study groups by chance. Stratification at randomization can help control unwanted variation, which is particularly important in smaller studies. In the PARDS example, if children under 2 years old with PARDS have twice the mortality of children over the age of 2 years, then the investigators would need to ensure that an equal number of children under age 2 are enrolled in each study arm. To accomplish this, investigators can randomize within strata of age so that equal numbers of patients within each age group are randomized to the treatment versus control groups.9 A combination of these two practices—stratified block randomization—is the most common randomization practice in RCTs.8

Blinding Blinding indicates that individuals involved in a trial—clinical providers, investigators, and enrolled patients—cannot determine which treatment group study subjects are in. Blinding minimizes the chance that the trial results will be influenced by a change in behavior by providers or subjects based on their knowledge (and opinions) of the intervention. Single-blind studies are composed of subjects who do not know their treatment group assignment. Double-blind studies conceal treatment group assignment from both enrolled patients and those caring for them. It is crucial to blind study staff who determine any subjective outcomes of subjects in order to reduce observer bias, also known as ascertainment bias.3 Concealing treatment group assignment may not always be feasible, such as in trials of extracorporeal life support. If an intervention is highly effective, it is also sometimes possible for clinicians to guess which patients are receiving the intervention. Some efforts to blind participants to treatment group allocation, such as using sham surgical procedures to compare to a surgical treatment arm, may subject patients to additional risk. Special ethical considerations apply in these cases.10,11

Outcome Selection Multiple aspects of study design, including sample size calculations, the timing of study observations, and statistical analysis, are dependent on the choice of the primary outcome. Since it is so influential in study design, a study should have only one primary outcome, although many studies also evaluate additional (secondary) outcomes that are observed alongside the primary outcome but do not influence study design. Each outcome measure has particular strengths and weaknesses; which measure is most appropriate for a particular study depends on the question being asked.

Mortality Reporting death is accurate, with indisputable clinical relevance. However, using mortality as a primary outcome is not entirely straightforward. Many studies attempt to define deaths as related or unrelated to the critical illness, but ascribing death to a particular illness can be highly subjective. Mortality is sometimes too infrequent, as it is in critically ill children, to be a feasible outcome for RCTs.12 Two traditional measures of mortality are hospital discharge status and mortality at a fixed time point (e.g., day 28). Status at hospital discharge can be difficult to interpret, as local practice at

a study site (e.g., a preference for early discharge to a nearby rehabilitation hospital) may influence discharge timing and in-hospital mortality. Mortality at a fixed time point avoids this difficulty but may fail to capture the entire risk period of the illness. For example, a patient with severe sepsis may remain critically ill on life support at day 28. This patient’s outcome for study purposes is survival, although this is meaningless if the patient dies shortly thereafter. The risk of mortality can persist for months13–15 and remains high for up to 3 years among children following critical illness.16–18 Failure to capture delayed illness-associated mortality may falsely support therapies that delay, but do not prevent, death.

Morbidity Intensive care may save a life only to produce a survivor wracked by infirmity, with low quality of life (QOL) and high reliance on healthcare. Morbidity can be considered in terms of physiologic change in organ function,19–22 effects on resource use (e.g., hospital costs, hospital readmission, cost of home nursing),23–25 functional status,26–28 or quality of life.23,24,29–35 Organ Dysfunction

Organ dysfunction (or organ failure) scores are relatively easily quantified. These scores can describe the severity of a patient’s acute illness, indicate the need for hospital resources, and predict risk of in-hospital death. Many organ dysfunction scoring systems exist.19,36,37 The pediatric logistic organ dysfunction (PeLOD) score was the first score developed and validated to predict hospital mortality in critically ill children.38 Subsequently, multiple organ dysfunction syndrome (MODS),39 pediatric MODS,40 and pediatric sequential organ failure assessment (p-SOFA) scores have also been associated with in-hospital outcomes for critically ill children.22,40–42 However, the extent to which timing, severity, and the differential effects of different organs failing influence long-term outcome remains poorly understood.43

Resource Use

Resource use, including measures such as duration of mechanical ventilation and ICU LOS, captures information related to the severity of organ dysfunction and critical illness. Analysis of studies with resource use as the primary outcome require special handling of mortality and other competing risks to ongoing resource use, as patients who die during a study use no additional resources and accrue no additional costs, but this can hardly be considered a positive outcome.44

Functional Status

Research tools that quantify functional status have supported the expansion of critical care outcomes research beyond mortality and the hospital setting. The most commonly employed tools are well validated, easily administered, and suitable for retrospective collection in large populations. Examples include the Pediatric Overall Performance Category, the Pediatric Cerebral Performance Category,45 and the Functional Status Scale.28 On a large scale, approximately 5% of children surviving critical illness experience new functional morbidity, although this risk varies by age, comorbidity, and diagnosis.46

Quality of Life

QOL blends survival with the perceived value of survival. Along with functional status, QOL ranks highest on patient-nominated outcomes of interest.47 QOL measures are being increasingly used in studies of critical illness,48,49 including two large pediatric critical care RCTs in progress (Stress Hydrocortisone in Pediatric



CHAPTER 11 Essential Concepts in Clinical Trial Design and Statistical Analysis

Septic Shock #NCT03401398; Prone and Oscillation Pediatric Clinical Trial #NCT03896763; both at clinicaltrials.gov). It can be difficult to ascertain whether a decrease in QOL is due to critical illness as opposed to being the result of an underlying disease (although paired assessments in relation to a baseline address this challenge), and QOL can be measured only in survivors. However, large RCTs incorporating QOL outcomes can be quite powerful, as underlying disease is usually balanced between treatment groups. Well-validated tools allow comparison with healthy population norms and with other ill populations.50

Composite End Points Composite end points combine multiple different clinical outcomes into one outcome measure. This increases the event rate, which can reduce the sample size needed to see an effect. However, careful selection is needed to ensure that the combined end point is meaningful. Combined outcomes can be difficult to interpret, as they may combine differently valued outcomes (e.g., combined death and neurologic disability at 28 days) that individually occur with different frequency across the study groups.51 The least important outcomes usually have the highest event rate; unfortunately, such studies may be underpowered to identify a difference in the individual outcomes considered most important by patients, families, and clinicians.

Outcome-Free Time To address difficulties in interpreting duration-based clinical outcomes in which mortality does not contribute additional days but is clearly undesirable, many investigators have turned to a composite outcome of outcome-free time. Unfortunately, very different individual outcomes can yield identical outcome-free time. For example, a study group with 10% mortality and median 14 days ventilation among survivors and a study group with 40% mortality and median 5 days ventilation among survivors both have a median of 10 ventilator-free days within a 28-day period. If these were trial results, we would conclude that there was no difference between groups, although patients and providers would likely disagree that these outcomes are the same.44 At a minimum, simple statistical tests are inadequate to evaluate this type of data; more sophisticated methods are required. Surrogate End Points A surrogate outcome is an alternative outcome, often a biomarker, used to stand in for a clinical primary outcome of interest. The word biomarker was first used in research in 1973 to describe extraterrestrial biological markers52,53; medical research adopted this term over time. The US Food and Drug Administration (FDA) began accepting biomarkers as surrogate primary end points in clinical trials in 1992, primarily to speed development of antiretrovirals to combat HIV/AIDS. Surrogate outcomes are attractive in medical research, as they are often more easily measured, cost less, or occur sooner than the ideal clinical outcome. However, concerns persist about the validity and interpretation of surrogate end points. To be valid, surrogates must have a wellestablished, strong, independent association with the relevant clinical outcome in observational studies, with a plausible biological relationship. Then an intervention with an effect on the surrogate marker needs to be consistently associated with a simultaneous effect on the relevant clinical outcome in clinical trials.3 Difficulties with interpretation stem from the leap that clinicians must make between an effect on the surrogate to the relevant clinical outcomes. A statistically significant reduction in low-density lipoprotein serum cholesterol, for example, may not have any impact on clinical

79

outcomes if the magnitude of reduction is too small or if the biomarker-based study did not last long enough to identify important side effects that would influence long-term adherence and safety.53 Studies using surrogate measures as the primary outcome must be considered in the context of medical knowledge surrounding that surrogate measure as it relates to actual patient outcomes.

Common Trial Designs The simplest structure for an RCT involves a single intervention arm compared with a single control arm. However, investigators often wish to evaluate several possible interventions. A multiarmed study is possible but would require adding a study arm’s worth of patients for each intervention. The factorial trial design alleviates this problem. In a 2 3 2 factorial trial, an equal number of patients are allocated to four groups: control, intervention A, intervention B, and intervention A 1 B. This allows for patients in both the intervention A and A 1 B groups to be evaluated for the effect of intervention A, which can reduce total study size if there is no interaction between the effect of A and B. This design can also allow for evaluation of the interaction between A and B, although powering the study to evaluate for an interaction requires a larger sample size.54,55 For some conditions and indications, it is challenging to recruit enough patients to carry out a standard randomized trial design while maintaining balance in other clinically important factors between study groups. A crossover trial may alleviate this problem. This within-subject study design exposes all patients to placebo and to intervention. Patients are randomized to the order in which they are exposed, with a washout period in between. Since all patients receive both treatment options, the number needed is reduced; all patients are assumed to function as their own controls. However, the washout period must be sufficiently long and the effect of the intervention must happen soon enough for it to be observed during the treatment period. The order of receiving treatment may have an effect on the outcome and should be included in the analysis. For example, if the intervention is effective but takes longer than anticipated, the effect could appear during the placebo period for the group that received the treatment first.56 As new medications are developed and tested against previously approved medications, sometimes the question changes from “is the treatment better than the control?” to “is the treatment as good as the control?” This is especially important if the new treatment offers other benefits, such as lower cost, more favorable pharmacokinetics, or fewer side effects. Noninferiority trials approach study design with a different null hypothesis—namely, that the new treatment is worse than the old treatment (often referred to as an active control). Rejecting the null hypothesis requires that the estimate of effect and the entire 95% confidence interval do not cross a predetermined threshold of “worse.” These studies must be powered sufficiently to achieve a narrow confidence interval to appropriately identify noninferiority when present. Additionally, a lower than anticipated outcome rate may make it inappropriately easy for a new treatment to be deemed noninferior.54 Noninferiority trials require a unique approach to statistical analysis and reporting.57

Phases of Clinical Trials for New Drug Approval When a drug, procedure, or treatment appears safe and effective in laboratory and preclinical experiments, investigators proceed with evaluation in human trials. Clinical trials for human drug

80

SECTION II



Pediatric Critical Care: Tools and Procedures

development in the United States are classified into four phases by the FDA. In each progressive phase, the number of patients, complexity, duration, and costs of the trial increase. The complete process is long; the average interval from drug development to market is approximately 10 years.58 The longest portion of this interval is usually drug testing in clinical trials, which collectively occur over 5 to 7 years. Phase I trials are dosing trials, in which a small number of human subjects receive several doses of the study drug to assess its pharmacokinetics, pharmacodynamics, and side effects in either healthy volunteers or patients with terminal conditions with few remaining treatment options. A phase II trial evaluates drug efficacy, usually in the patient population of ultimate interest, while continuing to monitor safety and side effects in more subjects over a longer period. Finally, phase III trials are rigorously designed RCTs, with strictly defined outcomes or clinical end points, enrolling hundreds to thousands of patients across multiple centers. Phase IV trials are postmarket trials conducted following drug approval by the FDA. These studies, in addition to consumer and clinician reporting of drug-associated safety concerns, allow for ongoing evaluation of rare adverse events associated with the drug and evaluation in new populations.58

effects. Also, performing multiple additional statistical tests increases the likelihood of a type I error or identifying an effect by chance simply because so many tests were done. For these reasons, subgroup analyses should be prespecified and limited in number.60

Hypothesis Testing and Determining the Study Result Inference and Estimate of Effect

The results of research studies are judged by their reliability and validity. A trial is reliable if it is repeated under the same circumstances and the same results are achieved. A study has internal validity if the results are real and not due to bias, chance, or confounding; it has external validity if its results can be generalized to a broader population.

A clinical trial observes the effect of an intervention in a small sample of patients; however, researchers want to generalize these results to the entire (theoretical) population. Statistical analysis allows this generalization to be made. Based on the study design and distribution of study measurements, researchers choose an appropriate statistical test to compare the study results against the null hypothesis. For binary outcome measures (e.g., mortality), the estimate of effect is usually expressed as a relative risk or a risk ratio: the proportion of subjects with the outcome in one group divided by the proportion of subjects with the outcome in the second group. Alternatively, relative odds or odds ratio may be reported. Odds are calculated as the ratio of number of events to nonevents, and the odds ratio is the odds of the event in one group divided by the odds of the event in the second group. For rare events, the odds ratio will approximate the relative risk. The odds ratio is amenable to mathematical operations and can be generated from logistic regression analyses, which can include adjustment for confounding factors in calculating the estimate of effect.

Whom to Analyze?

95% Confidence Interval

All randomized studies should be analyzed based on original group assignment, or intention to treat. By including all patients randomized into each group—by the treatment that they were intended to receive—all consequences of the treatment and all benefits of balance achieved by randomization are preserved. It can be tempting to analyze based on actual receipt or completion of treatment, termed per-protocol analysis, which excludes patients who crossed over between treatment groups or excludes patients after randomization for other reasons. While these analyses will reduce dilution and possibly identify a greater treatment effect, they will also lose the benefits of randomization and will introduce selection bias in the result.4 However, the most appropriate approach to analysis depends on the trial. Pragmatic trials, which aim to test the real-world performance of an intervention in a broad population of patients in a clinical setting, differ from explanatory trials, which aim to identify the biological effect of an intervention in a more idealized setting.4 In pragmatic trials in which loss to follow-up and incomplete adherence are an expected part of the intervention, per-protocol analysis may help translate trial results to alternate clinical settings in which differences in adherence, demographics, and other factors may substantially influence the effect of the intervention.59 Subgroup analyses focus on the effects of a treatment within a particular group of study participants, such as women, those within a given age strata, or among members of a specific race. These analyses are typically performed when there is a suspicion based on observational data or biology that the treatment effects may differ among groups, also known as effect modification or interaction. Since these analyses involve smaller numbers compared with the whole trial, they are typically underpowered to definitively identify treatment

Next, the authors consider the certainty of the estimate of effect observed in the study. The 95% confidence interval (CI) describes the range of true effect values that would plausibly yield the observed effect in the study. If the estimate of effect in a study is a relative risk reduction of 50%, then a 95% CI of 40% to 60% indicates that it would reasonably have obtained this study result if the true effect was anywhere from 40% to 60%. A higher-powered study will usually achieve a narrower 95% CI, giving more certainty about the true magnitude of effect. When comparing binary outcomes, a CI that crosses 100% (or 1.0) for odds ratio or relative risk ratio translates to no statistically significant difference between groups. Similarly, for continuous outcomes, a CI that crosses 0 (i.e., no risk difference) translates to no statistically significant difference.

Statistical Analysis and Reporting

P Values Statistical testing assesses whether the results support rejecting the null hypothesis within some margin of error. Researchers calculate the probability that the observed difference (or one greater) would have occurred by chance, if the null hypothesis were true—the P value. A low probability indicates that the results are unlikely to have occurred by chance and would support rejecting the null hypothesis. Notably, even study results with an imprecise estimate of effect (e.g., those with a very wide 95% CI, or wide range of true values that could be consistent with the observed results) can meet statistical significance (i.e., P , .05). The method for calculating the P value varies by study design and outcome measure. Parametric tests assume that the outcome measure has a normal distribution, indicating that it can be fully described by its mean and standard deviation. For this kind of



CHAPTER 11 Essential Concepts in Clinical Trial Design and Statistical Analysis

outcome, a t-test is used to compare means from one or two samples; the analysis of variance test is used to compare means from more than two samples. Nonparametric tests do not depend on a normal distribution, but because they make fewer assumptions about the distribution of the data, they are typically less powerful. These can be more appropriate for data that are highly skewed (e.g., length of stay, which is typically right skewed, as some patients have very long lengths of stay) or otherwise expected to have a nonnormal distribution. Nonparametric tests include the Wilcoxon rank sum or Mann-Whitney U test (for unpaired comparison of the median in two groups) and the KruskalWallis test (for comparison of medians across multiple groups). Different tests are used for categorical data (e.g., ethnicity, gender, pediatric operational performance category score). The chi square test compares observed to expected values in a 2 3 2 table. Fisher’s exact test is similar but calculates a more accurate P value when small cell numbers are present. McNemar’s test is used for paired categorical data. For time-to-event data, survival curves are often used to display data. This allows investigators to handle varying times of observation prior to events, as well as partial data from patients who are observed for some time but not observed to have an event during follow-up (censored data). A hazard ratio is typically reported for survival data, describing the ratio of “hazard” rate (moment-to-moment outcome or event rates) between groups.

Additional Sources and Mitigation of Bias Bias results in differences between study populations that are not due to chance. Some features of design that reduce or eliminate bias, including study population selection, randomization, and blinding, have already been discussed. Some additional features of study analysis can further reduce bias. Bias due to loss of data occurs when data from subjects are eliminated from the final analyses. Protocol violations, postrandomization exclusion, or unequal dropout or loss of follow-up can all result in missing data. Data that are unequally missing between groups, or missing not at random, can introduce bias. For example, if the study treatment was poorly tolerated by a subgroup of the study population, then those patients might be more likely to drop out, and their data would be missing from the final study results. The total amount of missing data can also impact study results. One review of 71 major RCTs in toptier medical journals identified that 13 of the trials (18%) were missing outcome data on 20% or more of enrolled subjects.61 Imputation, sensitivity analyses, and other advanced statistical methods can be used to explore how much the results might be affected by missing data.

Additional Methods of Exploring Study Results A clear distinction should be made between relative risk reduction and absolute risk reduction in study results. A reduction in mortality from 60% to 20% and a reduction from 3% to 1% both represent a relative risk reduction—1 minus the risk ratio—of 66%. However, the absolute risk reduction, or the difference in risk between groups, is substantially different: 40% versus 2%. The number needed to treat (NNT) is a related statistic that describes what number of patients would have to be exposed to the intervention to result in one “saved” outcome, equal to 1 divided by absolute risk reduction. In the examples above, the NNT would be only 2.5 patients for the intervention

81

that reduces mortality from 60% to 20% but would be 50 patients for the intervention that reduces mortality from 3% to 1%.7 The NNT can facilitate comparing results from different studies and consideration of side effects, costs, and other aspects of an intervention. It can also be adjusted for a particular patient’s baseline risk compared with the average risk of patients in the study; among patients with twice the baseline risk of the outcome, the NNT would be cut in half.62 These terms and additional common calculations are summarized in eTables 11.1 and 11.2. The fragility index, another method used to describe study results, refers to the number of outcomes (or events) that would have to be changed to nonoutcomes in order to raise the P value from statistically significant (classically, ,.05) to nonsignificant or to raise the likelihood that the observed study results occurred by chance beyond a threshold of acceptability.63–65 Across 43 published RCTs in pediatric critical care, a median of only two event switches would have been required to alter the study results from statistically significant to nonsignificant.66 This methodology can only be applied to studies with a binary outcome and is subject to the same methodologic concerns as the P value itself.67

Negative Studies The design and analysis of interventional trials is geared toward testing a null hypothesis. Rejecting the null hypothesis (i.e., not finding evidence of a difference) is not the same as finding evidence of no difference. Evidence of no difference would be a study in which the estimate of effect was close to unity, with a high degree of confidence in the result (e.g., a low P value and a narrow CI). While many clinicians refer to a “negative” study as one in which P . .05, this indicates simply a reasonable likelihood that the study result was identified by chance. If the estimate of effect favored the intervention, but P . .05, this study may be merely underpowered to identify a true effect. Clinicians should not necessarily conclude that such a study supports equal outcomes between treatment and control.

Conclusions Good trial design is more important than statistical analysis. Once a trial is completed, shortcomings in design cannot be mitigated, whereas statistical analyses can be modified or corrected. The most common shortcomings in trial design are the introduction of, or failure to accommodate, bias and imprecision in estimating the treatment effect, leading to an inability to address the initial study question.

Key References Koepsell TD, Weiss NS. Randomized Trials. Epidemiologic Methods: Studying the Occurrence of Illness. 1st ed. New York: Oxford University Press; 2003. Piantadosi S. Clinical Trials: A Methodologic Perspective. 2nd ed. Hoboken, NJ: John Wiley and Sons, Inc; 2005. Pocock SJ, McMurray JJ, Collier TJ. Making sense of statistics in clinical trial reports: part 1 of a 4-part series on statistics for clinical trials. J Am Coll Cardiol. 2015;66(22):2536-2549. Pocock SJ, Clayton TC, Stone GW. Design of major randomized trials: part 3 of a 4-part series on statistics for clinical trials. J Am Coll Cardiol. 2015;66(24):2757-2766.

The full reference list for this chapter is available at ExpertConsult.com.

81.e1

eTABLE Two-by-Two Table for Calculations 11.1 DISEASE OR OUTCOME

Test or exposure

Present

Absent

Positive

a

b

Negative

c

d

eTABLE Selected Terms and Definitions 11.2 Absolute risk reduction (ARR): The difference in event rates in treated patients compared with control patients. Note that the order is reversed compared with the attributable risk (see below). ARR 5 [c/(c 1 d)] 2 [a/(a 1 b)] Ascertainment bias: Observer bias; bias introduced by study staff or investigators knowing or being able to determine treatment group assignment in randomized studies. Attributable risk (AR): The effect of an exposure on the risk of disease in those exposed compared with those unexposed. AR 5 (Frequency in exposed group) 2 (Frequency in unexposed group) 5 [a/(a 1 b)] 2 [c/c 1 d)] Blinding: Obscuring study treatment group assignment from individuals in a trial (patients, research study staff, investigators, and/or other clinical providers). Confidence interval (CI): The range of values likely to include the true value for the entire population. The standard is 95%, in which 95% of such intervals will contain the true population mean. Confounding: An effect of a third factor, one associated with both a predictor and an outcome (but not on the causal pathway between the predictor and outcome), that may influence the observed effect of a predictor on outcome. Effect modification: An effect of a third factor that influences the magnitude of the observed effect of a predictor on outcome. Estimate of effect: The observed effect of an intervention in a particular study, usually presented along with an estimate of a range of effect sizes that would be consistent with the study’s result (e.g., a relative risk and its 95% confidence interval). Explanatory trials: Designed to observe the true biological effect, or efficacy, of an intervention, typically under tightly controlled circumstances. Intention-to-treat analysis: Data are analyzed according to the groups to which subjects were assigned, regardless of what treatment subjects actually received (analyzed as randomized). Negative predictive value: The proportion of people with a negative test who are free of disease. NPV =

d c+d

Number needed to treat (NNT): The number of patients needed to treat to achieve one outcome. It is the inverse of the attributable risk ratio. NNT =

1

a   c = 1/  2  ARR c + d a + b

Odds: The ratio of events to nonevents (i.e., chances of something happening divided by chances against something happening). This is not the same as risk (which has a different denominator; see definition below). The odds of getting heads when flipping a coin are 1:1 (one to one). Odds ratio (OR), or relative odds: The odds of an event in a treated patient versus the odds in a control patient. In case-control studies, relative risk (RR) cannot be calculated because subjects are selected on the basis of outcome, not exposure. For rare outcomes (e.g., ,10% of the population), RR can be estimated by OR. a /c ad OR = = b/d bc Per-protocol analysis: Analysis based on subjects who received the intended intervention or adhered to treatment; this analysis loses the benefits of randomization but may be helpful in pragmatic studies. Positive predictive value (PPV): The proportion of people with a positive test who have disease. PPV =

a a+b

Pragmatic trials: Designed to test the effectiveness of an intervention in a real-world scenario, often involving a clinical environment and a broadly selected population.

81.e2

eTABLE Selected Terms and Definitions—cont’d 11.2 P value: The probability that the observed difference, or a larger one, would have been found by chance in a particular study if no effect is truly present. Sensitivity: The proportion of people with disease who have a positive test, 5 a/(a 1 c). Specificity: The proportion of people free of disease who have a negative test, 5 d/(b 1 d). Relative risk (RR): The risk of development of disease in the exposed group relative to those who were not exposed (also called risk ratio). RR 5

Prevalence in exposed group Prevalence in unex p osed group

5

a / (a + b) c / (c + d)

Relative risk reduction (RRR): Percent reduction in events in treated versus untreated groups. RRR 5 (1 2 [a/(a 1 b)]/[c/(c 1 d)]) 3 100% Risk (probability): The ratio of events to all possible events (i.e., the chances of something happening divided by the total number of chances). The risk (probability) of getting heads when flipping a coin is 0.5, or 50%. Type I error (a): The chance that a difference between treated and control groups studied is found when, in reality, there is no difference. Type II error (b): The chance that no difference between treated and control groups studied is found when, in reality, there is a difference. Power (1 – b): Statistical power is the ability of an experiment to observe a significant difference between groups when a difference truly exists. Power is equal to 1 minus the type II error (b). Validity: Internal validity refers to results that are real and not due to bias, chance, or confounding. External validity refers to results that can be generalized to a broader population.

e3



































































































1. Piantadosi S. The study cohort, Treatment allocation. Clinical Trials: A Methodologic Perspective. 2nd ed. Hoboken, NJ: John Wiley and Sons, 2005. 2. Koepsell TDW, Weiss NS. Overview of Study Designs. Epidemiologic Methods: Studying the Occurrence of Illness. New York: Oxford University Press; 2003. 3. Pocock SJ, Clayton TC, Stone GW. Design of Major randomized trials: part 3 of a 4-part series on statistics for clinical trials. J Am Coll Cardiol. 2015;66:2757-2766. 4. Koepsell TDW, Weiss NS. Randomized Trials. Epidemiologic Methods: Studying the Occurrence of Illness. New York: Oxford University Press; 2003. 5. Johnson N, Lilford RJ, Brazier W. At what level of collective equipoise does a clinical trial become ethical? J Med Ethics. 1991;17:3034. 6. Doig GS, Simpson F. Efficient literature searching: a core skill for the practice of evidence-based medicine. Intensive Care Med. 2003;29:2119-2127. 7. Pocock SJ, McMurray JJ, Collier TJ. Making sense of statistics in clinical trial reports: part 1 of a 4-part series on statistics for clinical trials. J Am Coll Cardiol. 2015;66:2536-2549. 8. Lin Y, Zhu M, Su Z. The pursuit of balance: an overview of covariate-adaptive randomization techniques in clinical trials. Contemp Clin Trials. 2015;45:21-25. 9. Suresh K. An overview of randomization techniques: An unbiased assessment of outcome in clinical research. J Hum Reprod Sci. 2011;4:8-11. 10. Horng S, Miller FG. Ethical framework for the use of sham procedures in clinical trials. Crit Care Med. 2003;31:S126-S130. 11. Savulescu J, Wartolowska K, Carr A. Randomised placebo-controlled trials of surgery: ethical analysis and guidelines. J Med Ethics. 2016;42:776-783. 12. Menon K, McNally JD, Zimmerman JJ, et al. Primary outcome measures in pediatric septic shock trials: a systematic review. Pediatr Crit Care Med. 2017;18:e146-e154. 13. Quartin AA, Schein RM, Kett DH, Peduzzi PN, Group ftDoVASSCS. Magnitude and duration of the effect of sepsis on survival. JAMA. 1997;277:1058-1063. 14. Kaplan V, Clermont G, Griffin MF, et al. Pneumonia: Still the old man’s friend? Arch Intern Med. 2003;163:317-323. 15. Herridge MS, Chu LM, Matte A, et al. The RECOVER program: disability risk groups and 1-year outcome after 7 or more days of mechanical ventilation. Am J Respir Crit Care Med. 2016;194:831-844. 16. Volakli EA, Sdougka M, Drossou-Agakidou V, Emporiadou M, Reizoglou M, Giala M. Short-term and long-term mortality following pediatric intensive care. Pediatr Int. 2012;54:248-255. 17. Pinto NP, Rhinesmith EW, Kim TY, Ladner PH, Pollack MM. Long-term function after pediatric critical illness: results from the survivor outcomes study. Pediatr Crit Care Med. 2017;18:e122-e130. 18. Matsumoto N, Hatachi T, Inata Y, Shimizu Y, Takeuchi M. Longterm mortality and functional outcome after prolonged paediatric intensive care unit stay. Eur J Pediatr. 2019;178:155-160. 19. Knaus WA, Draper EA, Wagner DP, Zimmerman JE. Prognosis in acute organ-system failure. Ann Surg. 1985;202:685-693. 20. Raffin TA. Intensive care unit survival of patients with systemic illness. Am Rev Respir Dis. 1989;140:S28-S35. 21. Leteurtre S, Duhamel A, Salleron J, Grandbastien B, Lacroix J, Leclerc F. PELOD-2: an update of the Pediatric logistic organ dysfunction score. Crit Care Med. 2013;41:1761-1773. 22. Matics TJ, Sanchez-Pinto LN. Adaptation and validation of a pediatric sequential organ failure assessment score and evaluation of the sepsis-3 definitions in critically Ill children. JAMA Pediatr. 2017;171: e172352.

23. Weinstein MC, Stason WB. Foundations of cost-effectiveness analysis for health and medical practices. N Engl J Med. 1977;296:716721. 24. Predicting outcome in ICU patients. 2nd european consensus conference in intensive care Medicine. J Intensive Care Med. 1994; 20:390-397. 25. Yagiela LM, Barbaro RP, Quasney MW, et al. Outcomes and patterns of healthcare utilization after hospitalization for pediatric critical illness due to respiratory failure. Pediatr Crit Care Med. 2019;20:120-127. 26. Oczkowski WJ, Barreca S. The functional independence measure: its use to identify rehabilitation needs in stroke survivors. Arch Phys Med Rehabil. 1993;74:1291-1294. 27. Enright PL, Sherrill DL. Reference equations for the six-minute walk in healthy adults. Am J Respir Crit Care Med. 1998;158:13841387. 28. Pollack MM, Holubkov R, Glass P, et al. Functional status scale: new pediatric outcome measure. Pediatrics. 2009;124:e18-e28. 29. Ridley SA, Wallace PG. Quality of life after intensive care. Anaesthesia. 1990;45:808-813. 30. Tarlov AR, Ware Jr JE, Greenfield S, Nelson EC, Perrin E, Zubkoff M. The medical outcomes study. An application of methods for monitoring the results of medical care. JAMA. 1989;262:925-930. 31. Visser MC, Fletcher AE, Parr G, Simpson A, Bulpitt CJ. A comparison of three quality of life instruments in subjects with angina pectoris: the sickness impact profile, the Nottingham Health Profile, and the Quality of Well Being Scale. J Clin Epidemiol. 1994;47: 157-163. 32. Kaplan RM, Atkins CJ, Timms R. Validity of a quality of well-being scale as an outcome measure in chronic obstructive pulmonary disease. J Chronic Dis. 1984;37:85-95. 33. Chelluri L, Grenvik AN, Silverman M. Intensive care for critically ill elderly: Mortality, costs, and quality of life. Review of the literature. Arch Intern Med. 1995;155:1013-1022. 34. Slatyer MA, James OF, Moore PG, Leeder SR. Costs, severity of illness and outcome in intensive care. Anaesth Intensive Care. 1986;14:381-389. 35. Varni JW, Seid M, Kurtin PS. PedsQL 4.0: reliability and validity of the Pediatric Quality of Life Inventory version 4.0 generic core scales in healthy and patient populations. Medical Care. 2001;39:800-812. 36. Leteurtre S, Martinot A, Duhamel A, et al. Development of a pediatric multiple organ dysfunction score: use of two strategies. Med Decis Making. 1999;19:399-410. 37. Doughty L, Carcillo JA, Kaplan S, Janosky J. Plasma nitrite and nitrate concentrations and multiple organ failure in pediatric sepsis. Crit Care Med. 1998;26:157-62. 38. Leteurtre S, Martinot A, Duhamel A, et al. Validation of the paediatric logistic organ dysfunction (PELOD) score: prospective, observational, multicentre study. Lancet. 2003;362:192-197. 39. Proulx F, Fayon M, Farrell CA, Lacroix J, Gauthier M. Epidemiology of sepsis and multiple organ dysfunction syndrome in children. Chest. 1996;109:1033-1037. 40. Graciano AL, Balko JA, Rahn DS, Ahmad N, Giroir BP. The Pediatric Multiple Organ Dysfunction Score (P-MODS): development and validation of an objective scale to measure the severity of multiple organ dysfunction in critically ill children. Crit Care Med. 2005;33:1484-1491. 41. Typpo KV, Petersen NJ, Hallman DM, Markovitz BP, Mariscalco MM. Day 1 multiple organ dysfunction syndrome is associated with poor functional outcome and mortality in the pediatric intensive care unit. Pediatr Crit Care Med. 2009;10:562-570. 42. Schlapbach LJ, Straney L, Bellomo R, MacLaren G, Pilcher D. Prognostic accuracy of age-adapted SOFA, SIRS, PELOD-2, and qSOFA for in-hospital mortality among children with suspected infection admitted to the intensive care unit. Intensive Care Med. 2018;44:179188.

References

e4





































55. Mdege ND, Brabyn S, Hewitt C, Richardson R, Torgerson DJ. The 2 x 2 cluster randomized controlled factorial trial design is mainly used for efficiency and to explore intervention interactions: a systematic review. J Clin Epidemiol. 2014;67:10831092. 56. Sedgwick P. What is a crossover trial? BMJ. 2014;348:g3191. 57. Piaggio G, Elbourne DR, Altman DG, Pocock SJ, Evans SJ. Reporting of noninferiority and equivalence randomized trials: an extension of the CONSORT statement. JAMA. 2006;295:11521160. 58. US Food and Drug Administration. Step 3: Clinical Research. fda. gov/patients/drug-development-process/step-3-clinical-research/. 59. Hernan MA, Robins JM. Per-Protocol analyses of pragmatic trials. N Engl J Med. 2017;377:1391-1398. 60. Pocock SJ, McMurray JJV, Collier TJ. Statistical controversies in reporting of clinical trials: part 2 of a 4-part series on statistics for clinical trials. J Am Coll Cardiol. 2015;66:2648-2662. 61. Wood AM, White IR, Thompson SG. Are missing outcome data adequately handled? A review of published randomized controlled trials in major medical journals. Clin Trials. 2004;1:368376. 62. Cook RJ, Sackett DL. The number needed to treat: a clinically useful measure of treatment effect. BMJ. 1995;310:452-454. 63. Moore GW, Hutchins GM, Miller RE. Token swap test of significance for serial medical data bases. Am J Med. 1986;80:182-190. 64. Feinstein AR. The unit fragility index: an additional appraisal of “statistical significance” for a contrast of two proportions. J Clin Epidemiol. 1990;43:201-209. 65. Walsh M, Srinathan SK, McAuley DF, et al. The statistical significance of randomized controlled trial results is frequently fragile: a case for a Fragility Index. J Clin Epidemiol. 2014;67:622-628. 66. Matics TJ, Khan N, Jani P, Kane JM. The Fragility of statistically significant findings in pediatric critical care randomized controlled trials. Pediatr Crit Care Med. 2019;20(6):e258-e262. 67. Dervan LA, Watson RS. The Fragility of using p value less than 0.05 as the dichotomous arbiter of truth. Pediatr Crit Care Med. 2019;20: 582-583.

































43. Watson RS, Crow SS, Hartman ME, Lacroix J, Odetola FO. Epidemiology and outcomes of pediatric multiple organ dysfunction syndrome. Pediatr Crit Care Med. 2017;18:S4-S16. 44. Bodet-Contentin L, Frasca D, Tavernier E, Feuillet F, Foucher Y, Giraudeau B. Ventilator-free day outcomes can be misleading. Crit Care Med. 2018;46:425-429. 45. Fiser DH. Assessing the outcome of pediatric intensive care [comment]. J Pediatr. 1992;121:68-74. 46. Pollack MM, Holubkov R, Funai T, et al. Pediatric intensive care outcomes: development of new morbidities during pediatric critical care. Pediatr Crit Care Med. 2014;15:821-827. 47. Merritt C, Menon K, Agus MSD, et al. Beyond survival: pediatric critical care interventional trial outcome measure preferences of families and healthcare professionals. Pediatr Crit Care Med. 2018; 19:e105-e11. 48. Heyland DK, Hopman W, Coo H, Tranmer J, McColl MA. Longterm health-related quality of life in survivors of sepsis. Short form 36: a valid and reliable measure of health-related quality of life. Crit Care Med. 2000;28:3599-3605. 49. Curley MA, Wypij D, Watson RS, et al. Protocolized sedation vs usual care in pediatric patients mechanically ventilated for acute respiratory failure: a randomized clinical trial. JAMA. 2015;313:379-389. 50. Aspesberro F, Mangione-Smith R, Zimmerman JJ. Health-related quality of life following pediatric critical illness. Intensive Care Med. 2015;41:1235-1246. 51. Heneghan C, Goldacre B, Mahtani KR. Why clinical trial outcomes fail to translate into benefits for patients. Trials. 2017;18. 52. Rho JH, Bauman AJ, Boettger HG, Yen TF. A search for porphyrin biomarkers in Nonesuch Shale and extraterrestrial samples. Space Life Sci. 1973;4:69-77. 53. Lassere MN. The Biomarker-Surrogacy Evaluation Schema: a review of the biomarker-surrogate literature and a proposal for a criterionbased, quantitative, multidimensional hierarchical levels of evidence schema for evaluating the status of biomarkers as surrogate endpoints. Stat Methods Med Res. 2008;17:303-340. 54. Pocock SJ, Clayton TC, Stone GW. Challenging issues in clinical trial design: part 4 of a 4-part series on statistics for clinical trials. J Am Coll Cardiol. 2015;66:2886-2898.

12 Prediction of Short-Term Outcomes During Critical Illness in Children JULIA A. HENEGHAN, MICHAEL C. SPAEDER, AND MURRAY M. POLLACK

PEARLS • •









Physiologic instability is a key factor in the prediction of shortterm outcomes in critically ill patients. Prediction tools are central to controlling for severity of illness in studies and unit-based quality assessments for both internal and external benchmarking. Regression analysis is typically the central technique for constructing outcomes prediction tools.

Ongoing efforts to provide high-quality, error-free care require both the evaluation of complex systems and an assessment of the quality of care. Outcomes research is an important aspect of both requirements. Scoring systems add objectivity to these assessments, especially in critical care units. Controlling for population differences, such as differences in severity of illness, enables both the inclusion of different healthcare systems in a single investigative effort and contrasting individual healthcare systems in quality of care assessments. Measuring mortality adjusted for physiologic status and other case mix factors has been the core methodology of adult, pediatric, and neonatal intensive care assessments for decades for both internal and external benchmarking. However, mortality rates in most pediatric intensive care units (PICUs) have decreased since these methods were developed. Medical therapies increasingly focus on reducing morbidity in survivors. Unfortunately, most quantitative outcome assessment methods continue to focus on the dichotomous outcomes of survival and death. Recently, there has been a new appreciation of the importance of other patient outcomes, such as discharge functional status, and better understanding of their determinants. The future will most likely see a diversity of patient outcomes of interest, methods to associate risk factors with these outcomes, and use of these risk factors for outcome prediction.

Historical Perspective The “modern” history of intensive care unit (ICU) scoring systems started with the Clinical Classification Scoring (CCS) system and the Therapeutic Intervention Scoring System (TISS).1 Although 82









Assessment of the validity of prediction tools centers on two statistical measures: discrimination and calibration. Although mortality has historically been the outcome of interest, prediction tools for morbidity have recently been developed, as well as for clinical outcomes such as length of stay and reintubation.

simple, the CSS system established the basis of severity of illness as a concept related to both physiologic instability and amount and intensity of therapy, ranging from routine inpatient care to the need for frequent physician and nursing assessments and/or therapeutic interventions. The TISS was based on the concept that sicker patients receive more therapy, such as mechanical ventilation or vasoactive agent infusions; thus, the number and sophistication of therapies serves as a proxy for severity of illness. Initially, 76 therapies and monitoring techniques were graded from 1 to 4 on the basis of complexity, skill, and cost. The TISS score still exists today, although the number of therapies has been reduced and objectivity has been added to the score.2 The concepts of sequential or multiple organ system failures (MOSFs) were also important in the development of the concepts of severity of illness. Mortality rates increased as the number of failed organ systems increased. The MOSF syndrome was initially described in children in 1986.3 Although there have been numerous minor adjustments to the definition of an organ system failure, it continues to be based on the initial concepts of failure defined as extreme physiologic dysfunction or use of a therapy preventing that dysfunction. Organ system failures have also been proposed as an outcome measure; since death is uncommon in PICUs, it is appealing to postulate that the number of organ failures or the temporal resolution of these organ failures could be a practical outcome. New or progressive multiple-organ dysfunction has been used as an outcome measure for large recently completed and ongoing studies.4,5 Additionally, recent studies have examined the relationship between the number of dysfunctional organ systems and patient



CHAPTER 12

Prediction of Short-Term Outcomes During Critical Illness in Children

outcomes, including in general pediatric critical care patients6 as well as subgroups of patients with severe sepsis7 or bone marrow transplants.8 Physiologic status is the underlying foundational concept for MOSF and the TISS score. Conceptually, severity of illness may be considered a continuous variable with extremes of outcomes (survival, death) occurring at low and high values. The threshold value determining survival or death is unknown and may vary from patient to patient. Physiologic instability has been an exceptionally productive concept expressed in multiple scoring systems in pediatric, neonatal, and adult intensive care with systems such as the Pediatric Risk of Mortality (PRISM) score, Score for Neonatal Acute Physiology (SNAP), Acute Physiology and Chronic Health Evaluation (APACHE), and many others. Recently, the development of new morbidity during critical illness has also been related to physiologic instability, with the morbidity risk rising as the instability increases until, at higher states of instability, high morbidity risk transitions to mortality risk. Interest in and investigation of morbidity have been hindered by the lack of measurement methods that are reliable, relevant, and practical for large studies. The development of the Functional Status Scale and its use in a national study of more than 10,000 critically ill children hold promise that morbidity will be a more important and relevant outcome in critical care assessments.9 Since its publication, the Functional Status Scale has been used to measure outcomes in general PICU patients as well as subgroups of children with traumatic brain injury and other traumatic injuries, those undergoing stem cell transplantation, and those requiring extracorporeal membrane oxygenation.

Methods Conceptual Framework When possible, the severity method should include variables fundamental to the issues being assessed. The fundamental role of pediatric critical care has been to monitor and treat physiologic instability. The development of severity measures has mirrored this role, first as descriptive categories, then as quantification of therapy designed to treat physiologic instability, and, finally, with physiologic instability itself as the foundational concept. Databases have become larger, and the availability of descriptive, categorical, and diagnostic data that they contain has increased. These data can also be associated with severity of illness and are being used for quality measures such as standardized mortality ratios and measures of severity of illness in academic studies. However, variables such as diagnosis and operative status are proxy variables whose risk estimation is, at least in part, one or more steps removed from physiologic status. Therefore, they are only indirect measures of severity that are vulnerable to “gaming” to alter an individual site’s results. Methods based on primarily categorical data often do not perform well across variable critical care environments.

Statistical Issues Regression analysis is typically the central technique for constructing outcome prediction tools. The type of outcome variable (e.g., continuous, dichotomous) is one determinant of the type of regression analysis used. Multiple linear regression analysis is most often used for models that seek to predict outcomes that are continuous variables (e.g., length of stay). Logistic regression analysis

83

is most often used for models that seek to predict outcomes that are categorical variables (e.g., survival/death). As data science applications in medicine have become more sophisticated and datasets have become larger, many areas of analysis have incorporated the use of machine-learning models to understand and predict patient outcomes. These can generally be thought of as a continuum of methods to approach data with different strengths and weaknesses. Traditional statistical analysis typically attempts to assign a relationship between a set of variables within a sample, while machine learning attempts to generate a function or pattern that can be generalized for prediction. In general, the data characteristics assumed in a machine-learning approach will be less restrictive than those for traditional statistical modeling. Finally, machine-learning approaches are especially well suited to large datasets, while traditional statistical modeling becomes more unwieldy with more complex inputs. However, as with traditional statistical tests, machine-learning algorithms each have unique characteristics that impact overall performance. Regardless of how a prediction tool is created, the assessment of its validity centers on two statistical measures: discrimination and calibration.10 Discrimination is the accuracy of a model in differentiating outcome groups and is most often assessed by the area under the receiver operating characteristic curve (AUC), which is equivalent to the C statistic. Broadly, this represents the average sensitivity of the test when modeled over all possible specificities. An AUC 5 1 represents a model with perfect accuracy; an AUC 5 0.5 represents a model with no apparent accuracy. A rough guide for model discriminatory performance is as follows: AUC 5 0.9–1.0 (excellent), 0.8–0.9 (good), 0.7–0.8 (fair), 0.6–0.7 (poor), and 0.5–0.6 (unacceptable). Calibration refers to the ability of a model to assign the correct probability of outcome to patients over the entire range of risk prediction. In practical terms for an outcome such as mortality, calibration assesses whether the model-estimated probability of mortality for patients with a particular covariate pattern agrees with the actual observed mortality rate. The most accepted method for measuring calibration is the Hosmer-Lemeshow goodness-of-fit test. Although the AUC is helpful in determining overall characteristics of the test, it does not allow for comparison between the individual specificity or sensitivity of the test. Additionally, as researchers use large datasets more commonly, an artificial increase in the AUC may be seen due to the large sample sizes, particularly when the model is overfit. The use of positive predictive values, which incorporates the prevalence of the queried condition, may better represent the performance of predictive models. Finally, the AUC may be not fully representative of unbalanced patient samples. This is a particular concern with outcomes such as mortality in pediatric critical care, which occur relatively rarely. It remains important to consider a variety of test characteristics when assessing the suitability of a specific test or model. An important issue in developing and evaluating severity models is the population used to derive and validate the method. The models are based on the populations used to develop them. For example, the Vermont Oxford Neonatal outcome predictor was developed in a large population from inborn nurseries and has been criticized for its lack of applicability to referral centers. The Paediatric Index of Mortality (PIM) and its subsequent updates (PIM2 and PIM3) were developed in predominantly Australian and European populations where the relationship of categorical and physiologic variables to outcome may be different than in the United States or developing countries.

SECTION II



84

Pediatric Critical Care: Tools and Procedures

Current Prediction Tools for Assessment of Mortality Risk Neonatal Intensive Care Unit Prediction Methods Three well-established prediction methods are used for the assessment of severity of illness and mortality risk in neonates: the Clinical Risk Index for Babies II (CRIB II),11 SNAP-II,12 and the Vermont Oxford Network risk adjustment.13 All scores can be calculated during the first 12 hours of life. CRIB II is the second generation of CRIB, which was developed in the United Kingdom from 812 neonates born at less than 31 weeks’ gestation or weighing less than 1500 g.14 CRIB II is a simplified version of CRIB, validated on 3027 neonates born at 32 weeks’ gestation or less. It is a five-item score composed of sex, gestation, birth weight, admission temperature, and worst base excess in first 12 hours of life. SNAP-II is the second generation of SNAP, which was a physiology-based severity of illness score with 34 variables for babies of all birth weights from the United States and Canada.15 SNAP-II simplified SNAP to six physiologic variables: mean blood pressure, lowest temperature, Pao2/Fio2 ratio, lowest serum pH, seizure activity, and urine output. In an effort to improve the predictive capabilities of SNAP-II for mortality, three additional variables were added: birth weight, small for gestation age, and Apgar (appearance, pulse, grimace, activity, and respiration) score below 7 at 5 minutes. The resulting nine-variable score for prediction of mortality risk was named Score for Neonatal Acute Physiology with Perinatal Extension (SNAPPE-II). The Vermont Oxford Network is a network of more than 800 institutions worldwide that maintains databases on interventions and outcomes for infants cared for at member institutions. The basic Vermont Oxford Network risk adjustment model includes variables for gestational age, race, sex, location of birth, multiple birth, 1-minute Apgar score, small for gestational age, major birth defect, and mode of delivery, with additional features included in prediction models for very- and extremely-low-birth-weight infants, those with chronic lung disease, or those with birth defects. Revalidation efforts of these tools employing a variety of data sources have demonstrated largely similar discriminatory abilities among the tools. Using data from the Vermont Oxford Network, Zupancic et al. validated SNAPPE-II on nearly 10,000 infants with similar performance to the Vermont Oxford Network risk adjustment.16 Within this study cohort, the addition of congenital anomalies to SNAPPE-II improved discrimination significantly. Reid et al. compared CRIB-II and SNAPPE-II in a cohort of Australian preterm infants and found similar performance between the tools and good overall discriminatory ability.17

Pediatric Intensive Care Unit Prediction Tools The prediction of mortality in the PICU has centered primarily on the use of two different acuity scoring systems, the PRISM score18 and the PIM3.19 Historically, these systems have been thought to be quite effective in discrimination but to lack robust calibration. PRISM is a fourth-generation physiology-based score for quantifying physiologic status and mortality prediction (Table 12.1). The original tool was developed on 11,165 patients from 32 different PICUs in the United States and includes 21 physiologic variables. The mortality predictions are routinely updated, the last update

being completed on 19,000 patients. Among PRISM’s strengths are its flexibility to extend beyond mortality prediction to provide riskadjusted PICU length-of-stay estimates.20,21 Historically, PRISM mortality risk assessments were made using physiologic data from the initial 12 hours of PICU care. Notably, PRISM quantifies physiologic status and uses categorical variables to facilitate accurate estimation of mortality risk. Recently, the Collaborative Pediatric Critical Care Research Network (CPCCRN) of the National Institute of Child Health and Human Development used data from more than 10,000 patients to improve PRISM by reducing bias and other potential sources of error.22 The new version of PRISM uses only the first PICU admission, and hospital outcome is predicted. Initially, PRISM used PICU outcome and subsequent PICU admissions in the same hospitalizations with additional mortality risk. However, decisions around discharge timing and location are important aspects of quality of care. For example, an inappropriately discharged PICU patient with a subsequent PICU readmission during the same hospitalization was previously credited as a good outcome for the first admission, while the subsequent admission had an additional mortality risk credited to the subsequent PICU admission mortality risk. Therefore, the subsequent PICU admission mortality risk was inflated even though it was associated with the premature or inappropriate discharge. Second, the PRISM observation time period has changed from the sampling period for the first 12 hours of care to a significantly shorter time period (2 hours before admission to 4 hours after admission for laboratory data and the first 4 hours of PICU care for other physiologic variables) since this better represents the patient’s underlying physiology instead of response to therapy.23 Third, admission of cardiovascular surgery patients for “optimizing” therapy or observation before their intervention is now common in many institutions, which necessitated a new definition of the PRISM observation period. An objective method to determine the PRISM observation for cardiovascular patients is now available. Finally, when PRISM was initially developed, the scores for physiologic derangements for each variable were calibrated to mortality odds ratios so that the PRISM score for each variable represented equivalent risk. Due to concerns that these variables may no longer represent equivalent risk, the new PRISM algorithm partitions PRISM into neurologic and nonneurologic components for outcome prediction. PRISM algorithms for mortality prediction and morbidity prediction are publicly available.21,23 The PIM3 mortality prediction model was developed from 53,112 patients from 60 PICUs in Australia, New Zealand, Ireland, and the United Kingdom (Table 12.2). PIM3 requires 10 variables collected from the time of initial patient contact to 1 hour after arrival in the PICU.19 In contrast with PRISM III, PIM3 uses only four physiologic variables but includes six categorical variables that classify patients on the basis of reason for admission, use of mechanical ventilation in the first hour, and diagnostic risk strata. PIM3 has not been extensively tested in the United States. Numerous obvious differences distinguish PRISM III from PIM3 (e.g., interval for data collection, number of physiologic variables, inclusion of nonphysiologic data). The impact that these differences have on mortality prediction in the form of bias must be considered.10,21 Foundationally, PRISM quantifies physiologic instability (PRISM III score) and uses categorical variables to facilitate accurate estimation of mortality risk while PIM estimates mortality risk only. PIM has not performed well in cardiovascular surgical populations in which outcome is strongly associated with



CHAPTER 12

Prediction of Short-Term Outcomes During Critical Illness in Children

85

TABLE Pediatric Risk of Mortality (PRISM) Score IV 12.1 For computation of mortality and morbidity risk, physiologic variables are measured only in the first 4 hours of pediatric intensive care unit (PICU) care and laboratory variables are measured in the time period from 2 hours before PICU admission through the first 4 hours. See references for the appropriate time periods to assess cardiovascular surgical patients younger than 3 months of age. The neurologic PRISM IV consists of the mental status and pupillary reflex parameters. Only the first PICU admission is scored. Check publications for the most up-to-date prediction algorithms.

Measurement

Score Pco2 (mm Hg)

CARDIOVASCULAR AND NEUROLOGIC VITAL SIGNS Systolic blood pressure (mm Hg)

Measurement

Score

50–75

1

.75

3

40–55

3

Total CO2 (mmol/L)

.34

4

,40

7

Pao2 (mm Hg)

42–49.9

3

45–65

3 7

,42

6

,45 55–75

3

CHEMISTRY TESTS

,55

7

Glucose

2

65–85

3

.200 mg/dL or .11 mmol/L

7

Potassium (mmol/L)

.6.9

3

,65 Temperature

,33°C or .40°C

3

Mental status

5

Neonate

.11.9 mg/dL or .4.3 mmol/L

3

Stupor/coma or GCS ,8

All other ages

.14.9 mg/dL or .5.4 mmol/L

3

Neonate

.0.85 mg/dL or .75 mmol/L

2

Infant

.0.90 mg/dL or .80 mmol/L

2

Child

.0.90 mg/dL or .80 mmol/L

2

Adolescent

.1.30 mg/dL or .115 mmol/L

2

Neonate Infant Child Adolescent

Heart rate (beats/min) Neonate Infant Child Adolescent Pupillary reflexes

215–225

3

.225

4

215–225

3

.225

4

185–205

3

.205

4

145–155

3

.155

4

One fixed

7

Both fixed

11

ACID-BASE, BLOOD GASES Acidosis (pH or total CO2) pH

CO2

.7.55

2

7.48–7.55

3

7.0–7.28

2

,7.0

6

5.0–16.9

2

,5

6

Blood urea nitrogen

Creatinine

HEMATOLOGY TESTS White blood cell count (cells/mm3) ,3000

4

Platelet count (3103 cells/mm3) 100–200

2

50–99

4

,50

5

Neonate

PT .22 or PTT .85

3

All other ages

PT .22 or PTT .57

3

PT or PTT (sec)

GCS, Glasgow Coma Scale; PT, prothrombin time; PTT, partial thromboplastin time.

postoperative physiologic status.24 PIM3 uses a 1-hour (vs. 4 hours for PRISM III) PICU observation time, which might imply that it is potentially less affected by PICU therapies. However, the variable observation period before PICU admission could impose significant institution-level bias on the basis of the percent of patients transported to the PICU from other locations and involvement of

the PICU team in the transport or emergency department care. PIM3 includes a therapeutic intervention (mechanical ventilation) as a predictor variable that introduces bias from the prehospital and emergency department settings and introduces a therapy into the score when the use of the score to evaluate quality of care is closely related to the provision of therapy.

SECTION II



86

Pediatric Critical Care: Tools and Procedures

TABLE Paediatric Index of Mortality 3 (PIM3) Variables 12.2 and Model Coefficients Score calculated based on variables collected from the time of initial patient contact to 1 hour after arrival in the pediatric intensive care unit.

Variable Pupils fixed to light? (yes/no) Elective admission (yes/no)

Coefficient 3.8233 20.5378

Mechanical ventilation in the first hour (yes/no)

0.9763

Absolute value of base excess (mmol/L)

0.0671

SBP at admission (mm Hg) 2

20.0431

SBP /1000

0.1716

100 3 Fio2/Pao2 (mm Hg)

0.4214

Recovery postprocedure? Yes, recovery from a bypass cardiac procedure

21.2246

Yes, recovery from a nonbypass cardiac procedure

20.8762

Yes, recovery from a noncardiac procedure

21.5164

Very-high-risk diagnosis (yes/no)

1.6225

Cardiac arrest preceding ICU admission Severe combined immunodeficiency Leukemia or lymphoma after first induction Bone marrow transplant recipient Liver failure is main reason for ICU admission High-risk diagnosis (yes/no)

1.0725

Spontaneous cerebral hemorrhage Cardiomyopathy or myocarditis

Additional Algorithms in the Public Domain

Hypoplastic left heart syndrome Neurodegenerative disease Necrotizing enterocolitis is main reason for ICU admission Low-risk diagnosis (yes/no)

Surgeons and European Association for Cardiothoracic Surgery (STS-EACTS) Congenital Heart Surgery Mortality Score was introduced in 2009. The score was developed on 77,294 procedures entered into the STS and EACTS Congenital Heart Surgery databases and validated on an additional 27,700 procedures.25 Procedure-specific relative risks of in-hospital mortality were estimated for more than 140 congenital heart disease procedures. To combine procedure-specific risks with patient-specific factors, the patient’s age, weight, and preoperative hospital length of stay were added to the model. The STS-EACTS score has good discrimination for in-hospital mortality (AUC 5 0.816) and outperformed both the Risk Adjustment for Congenital Heart Surgery (RACHS-1) and the Aristotle Basic Complexity score in the validation sample. The Pediatric Cardiac Critical Care Consortium (PC4) has recently leveraged its multi-institutional registry to begin developing risk prediction models, incuding for postoperative mortality, for patients with cardiac disease.26 This case mix model (PC4 Post-Surgical Mortality Model) is based on 8543 patients who had cardiac surgery either during or immediately preceding admission to a participating cardiac ICU in the United States. It includes patient-level preprocedural, operative (including STS scoring), and postoperative (within 2 hours) physiologic characteristics and demonstrates good discrimination of mortality (C statistic, 0.92). The inclusion of postoperative characteristics is an attempt to isolate the cardiac ICU performance from the surgical performance. Finally, since morbidity and mortality are associated with physiologic instability, the outcomes of postoperative cardiac patients are well predicted with PRISM. Simultaneous prediction of morbidity and mortality using PRISM also performs well even without inclusion of surgical complexity scoring.27

22.1766

Asthma is main reason for ICU admission Bronchiolitis is main reason for ICU admission Croup is main reason for ICU admission Obstructive sleep apnea is main reason for ICU admission Diabetic ketoacidosis is main reason for ICU admission

It should be clear to any practitioner of critical care that severity of illness and mortality risk are dynamic variables subject to a variety of influences, such as therapies provided, presence of comorbidities, and evolution of the disease process. Although not designed to specifically predict mortality risk, there are severity of illness scores obtained over the course of PICU care that correlate with mortality risk. The Paediatric Logistic Organ Dysfunction score quantifies degree of organ dysfunction among six different organ systems—neurologic, cardiovascular, respiratory, renal, hematologic, and hepatic.28 The score was developed from 594 patients and subsequently validated on 1806 patients demonstrating good discrimination of mortality (AUC 5 0.91).29 Increased Multiple Organ Dysfunction Score (MODS) values, as mentioned previously, have been linked to relatively poor outcomes.

Seizure disorder is main reason for ICU admission Constant

21.7928

ICU, intensive care unit; SBP, systolic blood pressure.

Next Generation: Morbidity and Mortality Prediction—Trichotomous Outcome Morbidity Assessment

Cardiac Intensive Care Unit Prediction Tools Among the subgroup of infants admitted to the PICU or cardiac ICU following repair or palliation of congenital heart disease, efforts have been taken to ascribe risk of mortality based on surgical procedure and patient covariates. The Society for Thoracic

Although mortality adjusted for physiologic status and other case mix factors has been the core methodology of adult, pediatric, and neonatal intensive care assessments for decades, mortality for most pediatric critical illnesses has decreased since these methods were developed. More important, therapies are increasingly focused on reducing morbidity in survivors. However, most quantitative



CHAPTER 12

Prediction of Short-Term Outcomes During Critical Illness in Children

outcome assessment methods continue to focus on the dichotomous outcomes of survival or death. A major challenge of pediatrics is the development of welldefined morbidity measures that are rapid, reliable, and objective; measure the child’s status at the time of testing; and are applicable to a broad range of ages in a variety of environments. In-depth neuropsychologic testing will likely remain the clinical standard, but other methods applicable to all pediatric ages and sufficiently rapid to be used in large samples are necessary. The Glasgow Outcome Scale score was adapted to children in the Pediatric Cerebral Performance Category and Pediatric Overall Performance Category scores (PCPC/POPC), but sufficient interrater reliability was achievable only when neighboring categories were combined.30 Therefore, using these scores in outcome studies risks requiring very large sample sizes to detect significant differences. Outcome studies in adult medicine have demonstrated the value of scales such as the Activities of Daily Living Scale. Thus the foundation of the Functional Status Scale (FSS) was to adapt the concepts of activities of daily living to pediatrics in a manner that met the criteria described earlier.9 The FSS was developed through a formal consensus process involving a variety of pediatric healthcare specialists. It is composed of six domains: respiratory status, feeding, motor functioning, communication, sensory functioning, and mental status. Each domain is assessed objectively from normal to very severe dysfunction. The score was designed to enable assessment from parents’ or caregivers’ reports or from medical records. It was validated on more than 800 patients from seven institutions against an adaptive behavior scale, has very good interrater reliability, and has been used in numerous studies, including a study of more than 10,000 pediatric patients. Comparisons with the POPC and PCPC demonstrated FSS increases with each higher POPC and PCPC category. However, the dispersion of the FSS scores indicated a lack of precision in the POPC/PCPC system when compared with the more objective and granular FSS system.31 A recent multisite study of more than 5000 pediatric ICU patients demonstrated the importance of the development of new functional status morbidities during intensive care.32 The rate of new functional status morbidities (assessed by an increase of 3 points in the FSS score) was 4.8%, twice as high as the rate of hospital deaths. On hospital discharge, the good category decreased from a baseline of 72% to 63%, mild abnormality increased from 10% to 15%, moderate abnormality status increased from 13% to 14%, severe status increased from 4% to 5%, and very severe was unchanged at 1%. The highest new morbidity rates were seen in patients with neurologic diagnoses (7.3%), acquired cardiovascular disease (5.9%), cancer (5.3%), and congenital cardiovascular disease (4.9%). New morbidities occurred in all ages, especially in those younger than 12 months of age. New morbidities involved all FSS domains, with the highest proportions involving respiratory, motor, and feeding dysfunction. Comparing recent data with historical data suggests that pediatric critical care may have exchanged mortality for morbidity over the past several decades. Although the rates cannot be precisely compared over time because of the different research methods, data from the 1990s demonstrated a PICU mortality rate of 4.6% and a PICU morbidity rate of 3.1% (based on a 2 POPC change) while current data reflect a hospital mortality rate of 2.4% and morbidity rate of 4.8%. Thus, the “morbidity and mortality rate” decreased only from 7.7% to 7.2% as the mortality rated decreased and the new morbidity rate increased. Because these rates are not severity or risk adjusted, the changes in

87

admission criteria and other factors that have occurred in the past several decades could have also significantly influenced this comparison. New functional status morbidities associated with PICU stays present on hospital discharge are associated with many of the same factors as mortality, including physiologic status measured by the PRISM III score, age, admission source, and diagnostic factors.33 Importantly, these new morbidities, when measured with the FSS score, can be modeled simultaneously with mortality. Critical care mortality is usually associated with physiologic multiple organ system abnormalities. It appears that new morbidity significantly affecting functional status is often an event along the path toward mortality, as both outcomes are strongly associated with the degree of physiologic alterations. Trichotomous modeling uncovered the phasic association of morbidity risk with physiologic status; morbidity risk increases with physiologic instability but then decreases as patients with potential morbidity die. Recent studies indicate that trichotomous logistic regression can produce a well-performing model for simultaneous prediction of both morbidity and mortality suitable for risk adjustment in research, quality, and other studies.33 The addition of morbidity to outcome prediction has wide implications for research trials and quality programs, especially those currently based on internal or external benchmarking of standardized mortality ratios. Care assessments that focus on morbidity and mortality will have wide appeal and relevance. Potentially, evaluations of, and improvements in, the structure and process of care analogous to those resulting from the investigations of the variability of standardized mortality ratios could result from the inclusion of this important new outcome. Initiatives that monitor standardized mortality ratios could find relevance in the inclusion of standardized morbidity ratios as well.

Application of Prediction Tools in Pediatric Intensive Care Prediction tools can be applied at both the population and individual patient levels. At the population level, prediction tools are used in benchmarking, which is a process in which the performance of entities (e.g., individuals, PICUs, institutions) is observed and then compared with internal or external standards. Clinical scoring systems (e.g., PRISM III) are used to control for severity of illness and other factors, thus allowing for standardized comparisons. The two most common standardized comparisons used in pediatric intensive care are the standardized mortality ratio (observed mortality rate divided by the expected mortality rate) and the standardized length-of-stay ratio (observed length of stay divided by the expected length of stay). Risk adjustment models typically present information regarding performance as standardized mortality ratios (SMR). These are calculated by comparing the number of observed outcomes to the number of expected outcomes based on the clinical characteristics of the patients. If the boundaries of this estimate are such that the lower bound is greater than 1, that center has experienced the outcome more than would be expected based on its case mix. Alternatively, if the upper bound is less than 1, the outcome is less common than would be expected, and if the bounds cross 1, then the observed values are within the range of what would be expected. As researchers move toward evaluating morbidity and other outcomes, the SMR concept can be similarly extended. Internal benchmarking relates to the comparison of performance within an entity. For example, an individual PICU may want to

SECTION II



88

Pediatric Critical Care: Tools and Procedures

compare the impact of a new care protocol on length of stay as compared with the current internal standard of care for a particular illness. Calculating a standardized ICU length-of-stay ratio would allow for the comparison of practices while accounting for differences in patients’ severity of illness between the two groups. External or competitive benchmarking allows for direct comparisons between individual hospitals or PICUs by controlling for differences in case mix. At the individual patient level, the uses of prediction tools for short-term outcomes are varied. Clinical scoring systems are often employed in clinical trials and outcome analyses to control for patient severity of illness or in measuring changes in physiologic status after a novel therapy has been initiated, but this has not yet achieved clinical relevance for individual patients.10

Future Directions: Predictive Analytics and Tools for Decision Support The use of computerized decision support in adult, pediatric, and neonatal ICUs has grown considerably over the past several years. The integration of alerts, reminders, and protocols into computer order entry systems can help in guiding therapy and the reduction of medication errors.34 Children in the ICU have their physiologic parameters, including vital signs and laboratory results, monitored frequently. The dynamic and relatively dense nature of these variables in the electronic health record (EHR) allows for opportunities for real-time prediction and intervention in ways that are not possible in care locations with sparser data, such as the general care ward. There is, however, still a paucity of effective and validated decision support tools to guide the critical care practitioner in making real-time decisions that affect patient outcomes. The use of continuous monitoring data in the ICU to predict clinical deterioration is a recent subject of great interest, as it ideally allows for intervention to occur in a timely fashion and potentially prevent the deterioration. Moorman et al.35 developed a predictive algorithm for clinical deterioration in very-low-birth-weight infants employing heart rate characteristics monitoring and demonstrated a 20% relative reduction of mortality in a randomized clinical trial of 3003 patients. Efforts to develop predictive monitoring algorithms in older children and adults are underway. Predictive monitoring algorithms have been used in adult patients for a variety of conditions, both in and outside of the ICU. Perhaps the most prominent area of work for predictive algorithms relates to the detection of sepsis. Sepsis is relatively common and presents a large burden to the healthcare system. With regulatory and organizational mandates for appropriate sepsis care, the hope is that these models will preemptively identify patients before clinical decompensation. Adult critical care researchers have commonly used the Medical Information Mart for Intensive Care (MIMIC) database, a publicly available database sourced from Beth Israel Deaconess Medical Center, or institutional EHR data to predict sepsis hours to days before clinical diagnosis in adults. In neonates, Masino et al. recently

demonstrated the use of a variety of machine-learning models to predict sepsis 4 hours prior to clinical diagnosis with EHR data from a single institution.36 AUCs for the different models ranging from 0.80 to 0.82 when considering only those with positive cultures and 0.85 to 0.87 when children treated for sepsis despite negative cultures were also included. The increasing availability of large pediatric critical care datasets potentially provides the foundation for the creation of decision support tools. The various pediatric critical care data sources in the United States and worldwide vary greatly as they relate to accessibility/cost, clinical detail, and represented population.37 The expectation of the National Institutes of Health that federally funded investigators make their data widely and freely available to the public will only increase the amount of available data. Currently, datasets generated from research conducted by the CPCCRN and the Pediatric Emergency Care Applied Research Network are freely available to researchers. The Pediatric Data Science and Analytics group of the Pediatric Acute Lung Injury and Sepsis Investigators maintains a regularly updated list of pediatric critical care data sources (https://www. palisi.org/palisi-pedal).

Key References Graciano AL, Balko JA, Rahn DS, Ahmad N, Giroir BP. The pediatric multiple organ dysfunction score (P-MODS): development and validation of an objective scale to measure the severity of multiple organ dysfunction in critically ill children. Crit Care Med. 2005;33:14841491. Leteurtre S, Duhamel A, Salleron J, et al. PELOD-2: an update of the pediatric logistic organ dysfunction score. Crit Care Med. 2013: 41(7):1761-1773. Moorman JR, Carlo WA, Kattwinkel J, et al. Mortality reduction by heart rate characteristic monitoring in very low birth weight neonates: a randomized trial. J Pediatr. 2011;159:900-906. Pollack MM, Holubkov R, Funai T, et al. Simultaneous prediction of new morbidity, mortality, and survival without new morbidity from pediatric intensive care: a new paradigm for outcomes assessment. Crit Care Med. 2015;43:1699-1709. Pollack MM, Holubkov R, Funai T, et al. The Pediatric risk of mortality score: update 2015. Pediatr Crit Care Med. 2016;17(1):2-9. Pollack MM, Holubkov R, Glass P, et al. Functional status scale: new pediatric outcome measure. Pediatrics. 2009;124(1):e18-e28. Reid S, Bajuk B, Lui K, et al. Comparing CRIB-II and SNAPPE-II as mortality predictors for very preterm infants. J Paediatr Child Health. 2015;51(5):524-528. Straney L, Clements A, Parslow RC, et al. Paediatric index of mortality 3: an updated model for predicting mortality in pediatric intensive care. Pediatr Crit Care Med. 2013;14:673-681. Tabbutt S, Schuette J, Zhang W, et al. A novel model demonstrates variation in risk-adjusted mortality across pediatric cardiac ICUs after surgery. Pediatr Crit Care Med. 2019;20(2):136-142.

The full reference list for this chapter is available at ExpertConsult.com.

e1























































































1. Cullen DJ, Civetta JM, Briggs BA, Ferrara LC. Therapeutic intervention scoring system: a method for quantitative comparison of patient care. Crit Care Med. 1974;2(2):57-60. 2. Lefering R, Zart M, Neugebauer EA. Retrospective evaluation of the simplified therapeutic intervention scoring system (TISS-28) in a surgical intensive care unit. Intensive Care Med. 2000;26(12): 1794-1802. 3. Wilkinson JD, Pollack MM, Ruttimann UE, et al. Outcome of pediatric patients with multiple organ system failure. Crit Care Med. 1986;14(4):271-274. 4. Agus MS, Wypij D, Hirshberg EL, et al. Tight glycemic control in critically ill children. N Engl J Med. 2017;376(8):729-741. 5. Tucci M, Lacroix J, Fergusson D, et al. The age of blood in pediatric intensive care units (ABC PICU): study protocol for a randomized controlled trial. Trials. 2018;19(1):404. 6. Graciano AL, Balko JA, Rahn DS, Ahmad N, Giroir BP. The Pediatric Multiple Organ Dysfunction Score (P-MODS): development and validation of an objective scale to measure the severity of multiple organ dysfunction in critically ill children. Crit Care Med. 2005;33:1484-1491. 7. Lin JC, Spinella PC, Fitzgerald JC et al. for the Sepsis Prevalence, Outcomes, and Therapy Study Investigators. New or progressive multiple organ dysfunction syndrome in pediatric severe sepsis: a sepsis phenotype with higher morbidity and mortality. Pediatr Crit Care Med. 2017;18(1):8-16. 8. Lamas A, Otheo E, Ros P, et al. Prognosis of child recipients of hematopoietic stem cell transplantation requiring intensive care. Intensive Care Med. 2003;29(1):91-96. 9. Pollack MM, Holubkov R, Glass P, et al. Functional status scale: new pediatric outcome measure. Pediatrics. 2009;124(1):e18-e28. 10. Marcin JP, Pollack MM. Review of the methodologies and applications of scoring systems in neonatal and pediatric intensive care. Pediatr Crit Care Med. 2000;1:20-27. 11. Parry G, Tucker J, Tarnow-Mordi W. CRIB II: an update of the clinical risk index for babies score. Lancet. 2003;361:1789-1791. 12. Richardson DK, Corcoran JD, Escobar GJ, et al. SNAP-II and SNAPPE-II: simplified newborn illness severity and mortality risk scores. J Pediatr. 2001;138:92-100. 13. Horbar JD, Soll RF, Edwards WH. The Vermont Oxford Network: a community of practice. Clin Perinatol. 2010;37:29-47. 14. Cockburn F, Cooke RWI, Gamsu HR, et al. The CRIB (clinical risk index for babies) score: a tool for assessing initial neonatal risk and comparing performance of neonatal intensive care units. Lancet. 1993;342:193-198. 15. Richardson DK, Gray JE, McCormick MC, et al. Score for neonatal acute physiology: a physiologic severity index for neonatal intensive care. Pediatrics. 1993;91:617-623. 16. Zupancic JAF, Richardson DK, Horbar JD, et al. Revalidation of the score for neonatal acute physiology in the Vermont Oxford Network. Pediatrics. 2007;119:e156-e163. 17. Reid S, Bajuk B, Lui K, et al. Comparing CRIB-II and SNAPPE-II as mortality predictors for very preterm infants. J Paediatr Child Health. 2015;51(5):524-528. 18. Pollack MM, Patel KM, Ruttimann UE. PRISM III: an updated Pediatric Risk of Mortality score. Crit Care Med. 1996;24(5): 743-752.

19. Straney L, Clements A, Parslow RC, et al. Paediatric index of mortality 3: an updated model for predicting mortality in pediatric intensive care. Pediatr Crit Care Med. 2013;14:673-681. 20. Ruttimann UE, Pollack MM. Variability in duration of stay in pediatric intensive care units: a multi-institutional study. J Pediatr. 1996; 128(1):35-44. 21. Tibby SM, Taylor D, Festa M, et al. A comparison of three scoring system for mortality risk among retrieved intensive care patients. Arch Dis Child. 2002;87:421-425. 22. Pollack MM, Holubkov R, Funai T, et al. The pediatric risk of mortality score: update 2015. Pediatr Crit Care Med. 2016;17(1):2-9. 23. Pollack MM, Dean JM, Butler J, et al. The ideal time interval for critical care severity-of-illness assessment. Pediatr Crit Care Med. 2013;14(5):448-453. 24. Czaja AS, Scanlon MC, Kuhn EM, Jeffries HE. Performance of the Pediatric Index of Mortality 2 for pediatric cardiac surgery patients. Pediatr Crit Care Med. 2011;12(2):184-189. 25. O’Brien SM, Clarke DR, Jacobs JP, et al. An empirically based tool for analyzing mortality associated with congenital heart surgery. J Thorac Cardiovasc Surg. 2009;138:1139-1153. 26. Tabbutt S, Schuette J, Zhang W, et al. A novel model demonstrates variation in risk-adjusted mortality across pediatric cardiac ICUs after surgery. Pediatr Crit Care Med. 2019;20(2):136-142. 27. Berger JT, Holubkov R, Reeder R, et al. Morbidity and mortality prediction in pediatric heart surgery: physiological profiles and surgical complexity. J Thorac Cardiovasc Surg. 2017;154(2):620-628.e6. 28. Leteurtre S, Duhamel A, Salleron J, et al. PELOD-2: an update of the pediatric logistic organ dysfunction score. Crit Care Med. 2013:41(7):1761-1773. 29. Leteurtre S, Martinot A, Duhamel A, et al. Validation of the paediatric logistic organ dysfunction (PELOD) score: prospective, observational, multicentre study. Lancet. 2003;362:192-197. 30. Fiser DH. Assessing the outcome of pediatric intensive care. J Pediatric. 1992;121:69-74. 31. Pollack MM, Holubkov R, Funai T, et al. Relationship between the functional status scale and the pediatric overall performance category and pediatric cerebral performance category scales. JAMA Pediatrics. 2014;168(7):671-676. 32. Pollack MM, Holubkov R, Funai T, et al. Pediatric intensive care outcomes: development of new morbidities during pediatric critical care. Pediatr Crit Care Med. 2014;15(9):821-827. 33. Pollack MM, Holubkov R, Funai T, et al. Simultaneous prediction of new morbidity, mortality, and survival without new morbidity from pediatric intensive care: a new paradigm for outcomes assessment. Crit Care Med. 2015;43:1699-1709. 34. Williams CN, Bratton SL, Hirshberg EL. Computerized decision support in adult and pediatric critical care. World J Crit Care Med. 2013;2:21-28. 35. Moorman JR, Carlo WA, Kattwinkel J, et al. Mortality reduction by heart rate characteristic monitoring in very low birth weight neonates: a randomized trial. J Pediatr. 2011;159:900-906. 36. Masino AJ, Harris MC, Forsyth D, et al. Machine learning models for early sepsis recognition in the neonatal intensive care unit using readily available electronic health record data. PLoS One. 2019; 14(2):e0212665. 37. Bennett TD, Spaeder MC, Matos RI, et al. Existing data analysis in pediatric critical care research. Front Pediatr. 2014;2:79.

References

e2

Abstract: Outcomes research in pediatric critical care is an important aspect of providing high-quality, error-free medical care. Scoring systems and predictive models can add objectivity to these assessments. The historical basis for these scoring systems was the therapeutic intensity required by the patient; mortality was the outcome of focus. As the data available from critically ill

patients increase over time and critical care shifts from solely mortality prevention to include morbidity prevention, additional outcomes and analytic techniques have become relevant. Key Words: outcomes, morbidity, mortality, predictive analytics, PRISM score

13 Pediatric Critical Care Transport LAUREN RAKES, REID W.D. FARRIS, AND GEORGE A. (TONY ) WOODWARD

PEARLS •















The goal of interfacility transport is to ensure critical care delivery to the patient, provide acute and ongoing stabilization, and to anticipate disease progression as well as the evolution of respiratory failure and cardiovascular instability in a high-risk environment. For most disease processes, speed of transport should not take precedence over providing quality resuscitation. Transfer by specialized pediatric critical care transport teams may improve patient outcomes. Necessary components of a transport system include a standard communication process, appropriately trained team

As pediatric emergency and critical care centers have become more regionalized, the need for quality interfacility transport increases. More than 89% of pediatric emergency department (ED) visits occur in nonpediatric EDs, where the extent of illness or injury is assessed and initial stabilization is provided.1,2 Many community hospitals do not have the personnel, facilities, provider/staff skill, or equipment to administer critical care to infants or children beyond the period of initial stabilization, necessitating transfer. In transport, children may be subjected to a high-risk environment with limited resources and monitoring capabilities. The goal during transport should be to minimize the risk of deterioration or secondary injury during transport while advancing the care initiated at the receiving facility. For most pediatric critical illness, definitive care involves early and continued administration of standard therapies. Many of these interventions, including timely initiation of resuscitation fluids, inotropes administered via peripheral intravenous line, and antibiotic therapy, can improve outcomes.3,4 If appropriate resuscitation waits until the transport team arrives at the referring facility or the child arrives in the pediatric intensive care unit (PICU), benefits of early action may be lost. Therapies must begin before and continue during transport for the benefits noted in these studies to occur. Significant barriers to realizing this ideal state exist, including the tension that exists between the need to transfer patients as rapidly as possible and the desire to make transfers as safe as possible. This chapter summarizes the physiology relevant to pediatric transport, particularly air transport. It is emphasized that a wellrun interfacility transport team that is focused on the specific needs of children can make a significant difference in patient outcomes. Information about appropriate vehicles, medication,





members, reliable equipment, continuing education, competency assessments, and quality and safety monitoring. The legal responsibility for selection of the interfacility transport process rests mostly with the referring facility. However, the pediatric intensivist who acts as the medical control physician (MCP, also called medical command physician) can play an important role in assessing the situation (medical and logistical), recommending management, anticipating disease progression, and minimizing risk of deterioration of pediatric patients en route to tertiary care.

and equipment for transport is best summarized in the American Academy of Pediatrics (AAP) Guidelines for Air and Ground Transport of Neonatal and Pediatric Patients.5

Pediatric Transport Systems In many regions of the United States, medical transport teams that emphasize adult care, such as emergency medical services (EMS) or regional flight teams (referred to as nonspecialized transport teams here), are scattered throughout the community, close to referring hospitals, whereas specialized pediatric transport teams are often located at the sponsoring tertiary care facility. Because they are so accessible to referring hospitals, nonspecialized teams transport the majority of critically ill children. Training and protocols of these teams may be primarily focused on the major causes of mortality in the adult population, such as myocardial infarction, stroke, and trauma, disease processes for which rapid transfer to definitive care is an important determinant of outcome.6 Pediatric intensivists coordinating interfacility transfers need to be aware of the concerns that lead referring physicians to prefer rapid transport by a nonspecialized team over transfer by a specialized pediatric team originating at the accepting facility. These concerns are summarized in Table 13.1. Two independent studies reported that, as recently as 2003, only 6% of EDs were completely equipped to care for children.7 In an assessment of the compliance by EDs with nationally recognized guidelines on the care of children in EDs, improvements since 2003 were noted, although deficits in equipment and safety procedures remain common.8 Limited pediatric training coupled with infrequent exposure to pediatric patients may hamper the 89

90

SECTION II



Pediatric Critical Care: Tools and Procedures

TABLE Arguments Against and for Use of Specialized Pediatric Transport Teams 13.1

Against Use

For Use

Specialized teams take too long to get to the patient.

Concern for bed flow in small referring EDs and/or facilities without significant critical care staff/resources is real, but when we direct and provide goal-directed ICU care to the patient, the time to initiating time-sensitive therapies may be shortened.

Specialized teams spend too much time on the scene.

Scene time is necessary for stabilization, but the need for additional measures may be underappreciated by the referring hospital. Much of this care can be anticipated, recommended, and directed by the transport medical command physician. If the referring team is guided through and is able to provide the care suggested the transport team can then be more efficient with assessing the care provided and securing the patient for the transport process.

Time to definitive care is longer with When interventions only available at the accepting facility are the most important determination of outcome this may specialized teams. be true; however, in general, with specialized team, total transport time has not been shown to correlate with negative outcomes. Critical care delivery should start at the moment of contact by the referral center to the MCP. It should not wait until team arrival or arrival at the referring center. Adult teams have PALS training and can do the same thing.

Nonspecialized teams often lack experience with children, may be less experienced in assessment of children, and may have difficulty maintaining learned skills with limited frequency of exposures.

Specialized teams are expensive and True, but pediatric critical care transport systems are an integral part of a tiered community response to the needs of resource intensive. ill and injured children, many of whom do not initially present to tertiary care, pediatric EDs, or pediatric critical care–capable hospitals. Having a skilled process to allow for effective pediatric critical care regionalization is important and in the patients’ and communities’ best interest. EDs, Emergency departments; ICU, intensive care unit; MCP, medical control physician; PALS, Pediatric Advanced Life Support.

ability of ED and EMS providers to respond appropriately to pediatric emergencies. Fewer than 10% of all EMS runs nationwide involve infants and children, and a small percentage of these involve advanced life support (ALS) or critical care.9,10 Babl and colleagues demonstrated that, in a program with 50 active ALS providers, each provider is estimated to have one pediatric bagvalve-mask case every 1.7 years, one pediatric intubation every 3.3 years, and one intraosseous cannulation every 6.7 years.11 Without repeated reinforcement, providers’ knowledge and skills deteriorate over time.12,13 Underutilization of these skills may impact abilities as well as drive aversion to performing procedures in children. Multiple investigators have documented a disconcertingly low percentage of successful intubations in children compared with adult patients.14–18 In a retrospective study comparing prehospital intervention of pediatric and adult patients with head injury, paramedics had difficulty with intubation in 69% of children compared with 21% of adults, and they were unable to establish IV access in 34% of children versus 14% of adults.19 These interventions are key components of resuscitation of the critically ill pediatric patient as respiratory insufficiency, seizures, and shock are common reasons for referral to tertiary care. One study identified shock in 37% of children transferred to tertiary centers regardless of the reason for referral.20 It is increasingly recognized that rapid resuscitation is critical to the management of pediatric shock.21 Han and colleagues reported that, when community physicians aggressively resuscitated and successfully reversed shock before a transport team arrived, patients had a ninefold increase in their odds of survival.22 These studies defy the popular notion that pretransport stabilization and management wastes time and delays definitive therapy. The MCP should recognize that the opportunity to provide pediatric critical care starts at the moment of contact from a referring provider and should not wait unit arrival of the transport team or patient arrival at the receiving facility.

Specialized Teams Improve Outcome In 1978, Chance and associates23 demonstrated reduced mortality and more stable physiology in neonates weighing less than 1.5 kg who were transported by a specialized team. Since that time, several investigators have reported a decrease in the number of preventable insults in children transported by a pediatric critical care team compared with a multispecialty team.24–28 In a 2001 study of children transported with head injury, Macnab and colleagues determined that $135,952 in additional costs of care resulted from secondary adverse events occurring during transport by nonspecialized teams.29 In a prospective cohort study in which allocation of teams depended on team availability, not severity of illness, Orr and colleagues showed that use of a specialized team resulted in decreased severity-adjusted mortality (9% vs. 23%) compared with use of a nonspecialized team.30 Similarly, a large retrospective study of unplanned PICU admissions demonstrated that transfer by a specialty team was associated with improved survival in a multivariable analysis controlling for severity of illness.31 Pediatric specialized teams often perform additional stabilization maneuvers at the referring facility prior to transport. In a prospective observational study, pediatric teams initiated sedation 23% of the time, inotropes 44% of the time, and osmolar therapies for intracranial hypertension nearly 50% of the time when the referring facility had failed to do so.32 Transport teams also initiated mechanical ventilation, acquired central venous access, and placed or adjusted tracheal tubes.30 Time at the bedside for specialized transport teams can be relatively long owing to these interventions, but scene time has not been associated with mortality.28 Meaningful stabilization and therapeutic interventions should also continue en route. In a before-and-after intervention trial, introduction of a goal-directed resuscitation protocol decreased



CHAPTER 13 Pediatric Critical Care Transport

the number of interventions required in the ICU and decreased the hospital length of stay for pediatric patients transported with systemic inflammatory response syndrome.33 In a prospective randomized controlled trial, enhanced monitoring of blood pressure during pediatric interfacility transport resulted in more aggressive resuscitation during transfer, shorter ICU stay, and less organ dysfunction.34

Components of a Specialized Interfacility Transport Team Pediatric transport is part of a critical care continuum that includes EMS, the referring site of care, secondary transfer, and the receiving critical care facility. An ideal system would provide excellent communication between the referring and receiving hospitals; give clear, efficient, and experienced advice to support the referring staff’s care; bring high-level critical care to the patient at the referring institution; and continue optimal care through transport and into the PICU. Ideally, physicians and other caregivers from emergency medicine, neonatology, surgery, and intensive care all take an active role in designing each segment of the continuum and maintaining quality assurance. The critically ill child ultimately will be the responsibility of the pediatric intensivist; thus it behooves the intensivist to have significant input into system design and protocols. The specialized transport system has a responsibility to the referral community to make tertiary care widely accessible. Differences in topography, weather patterns, and the distribution of hospitals and population centers mean that the ideal transport system will differ from region to region. Regardless, a transport system should include a communications center, administrative staff, appropriately trained team members, reliable equipment, and a safety and quality improvement program.

Communications Ideally, a communications center should be easily accessible and staffed around the clock by communication specialists who are trained in handling emergency calls and who have no other distracting duties.35,36 The communication specialist should function as both an informational hub and someone who helps facilitate the conversations required to arrange all physical aspects of the transport. These efforts should free the transferring physician, MCP, and transport team members to focus their attention on patient care. Protocols may help to streamline the process and prevent errors. The capability for ongoing communication between team members and the communication center throughout the transport process should be available. A detailed log of transport requests, including times, demographic data, diagnosis, and vehicle issues, should be kept for both administrative review and medical-legal documentation.

Staffing The administrative staff of a transport system should include, at a minimum, a medical director, transport coordinator, and MCPs.35,36 The medical director should be a specialist in pediatric or neonatal critical care or emergency medicine. This individual should be experienced in both air and ground transport (as appropriate to the specialist’s specific system) and should understand patient

91

care capabilities and limitations in the transport environment. The medical director must be actively involved in the development and renewal of transport protocols; quality management; and the hiring, training, competency assessments, and continuing education of all transport personnel. This individual must orient the physicians who provide online medical control to the policies, procedures, and patient care protocols and should act as a liaison to the referral community for teaching and outreach.35,36 The transport coordinator, usually a nurse or paramedic, collaborates with the medical director with regard to training, protocols, scheduling, data collection, quality management, and marketing. The medical director and transport coordinator should participate in patient transport whenever possible to observe and assess the team capabilities and maintain clinical transport skills and perspective. An MCP should participate in every transport and provide advice to the referring physician and transport team as necessary. In many cases, the MCP will also be the receiving physician. The MCP should be experienced in handling transport calls, obtaining focused information, and providing efficient and concise management suggestions for the period before arrival of the transport team. The MCP should be knowledgeable about the availability of resources and have the ability to efficiently accept transferred patients without further consultation, to perform triage, and to activate backup systems when necessary. Team composition and training strategies vary considerably among transport programs. A two-person team composed of a nurse and respiratory therapist is the most common configuration for specialized pediatric teams.37 Transport crew members should be experienced in the care of critically ill pediatric/neonatal patients and be able to manage complex environments and limited resources. They must be highly skilled in airway management, resuscitation, and vascular access. They should have a fundamental knowledge of field priorities and be able to make decisions independently. The team transporting a critically ill pediatric patient should include a team leader who is experienced in managing life-threatening illnesses or injuries in neonates and children, most commonly a transport nurse or advanced practitioner. Routine physician presence on specialized transport teams in the United States was recently estimated to be low.37 All team members should have specific training in transport medicine, which includes methods of functioning in a moving environment, troubleshooting for equipment-related problems, and knowledge of aeromedical physiology as appropriate. Each program should define the cognitive knowledge and technical skills required for each professional group and should include a method to document the acquisition and maintenance of these skills. Instruction typically includes didactic sessions designed to assist personnel to acquire cognitive knowledge, a skill development and maintenance program, and a supervised orientation period. Simulation has been shown to improve adherence to protocols in the training of crisis-response teams and may be a useful adjunct to team member training.38,39

Equipment For a thorough discussion of transport equipment and options, readers are referred to the AAP Guidelines for Air and Ground Transport of Neonatal and Pediatric Patients.5 Equipment taken on transport should be complete and adequate to provide continuing intensive care throughout the trip. Oxygen capacity and reserve should be calculated for each patient transported and

92

SECTION II



Pediatric Critical Care: Tools and Procedures

should be at least twice the amount needed for the expected duration of the trip in case of delays or equipment malfunction. For air medical transport, weight and space restrictions must be considered when selecting equipment and range of medications.

Safety and Quality Improvement Safety should be a high priority in any transport program. Emergency vehicle operation carries substantial risks, not only to the crew and patient but also to others in its vicinity. Accidents that occur during medical transport are rare but can result in significant morbidity and mortality. During ground transport, ambulance drivers should be discouraged from using lights and sirens to circumvent traffic rules, as there is no evidence to support a positive effect on patient outcomes.40 Aeromedical transport involves a unique set of safety issues. Pilots who are under pressure to fly or who are sensitive to competition among aeromedical services within a region may fail to observe minimal weather standards, contributing to accidents. Pilots should be isolated from patient care issues to give them the freedom to make sound decisions based on the flight conditions. The transport team should be adept at survival techniques for their region and should always be prepared to deal with an off-airport landing. Regular sessions to review safety and emergency procedures for each transport mode should be provided for the transport team members. The Commission on Accreditation of Medical Transport Systems (CAMTS) has a long history as a peer review organization that aims to improve quality and safety of medical transport through a voluntary accreditation process that includes an application process, site surveys, and program evaluation.41 Regardless of membership in the CAMTS, a robust quality improvement (QI) program is an essential component to any transport team. Elements of a strong QI program include chart audit and case review, compliance with national metrics, and a process for continuous improvement. Twelve core quality metrics for pediatric and neonatal transport were identified by national transport leaders in 2015.42 In addition, the Ground Air Medical Quality in Transport (GAMUT) QI collaborative was founded in 2013, which established an international, multicenter database to facilitate tracking and benchmarking of transport-specific metrics.43,44

Stresses of the Transport Environment Interfacility transport is always risky for the patient. Movement of vehicles and ambient noise make examination difficult and monitor function less reliable. Each transfer of the stretcher or isolette from hospital to vehicle or between ambulance and aircraft adds potential for disruption of necessary tubes or devices. Fear and anxiety produced by the transport environment can worsen some conditions; for this and other reasons, the authors recommend that a parent travel with the child as able. In spite of the space and environment limitations, technologic advances have allowed some specialized transport teams to make the modern pediatric ICU mobile. Advanced therapies—including inhaled nitric oxide, noninvasive positive pressure ventilatory support, and extracorporeal life support—are now routinely used in the transport environment.45–48 Air transport adds complexity and unique physiologic considerations. Atmospheric pressure changes associated with increasing cabin altitude impact atmospheric—and therefore alveolar— oxygen tension as well as the volume of gas-filled spaces. The United States Federal Aviation Administration regulations mandate cabin altitude less than 8000 feet in fixed-wing flight.

Most fixed-wing medical flights maintain a cabin pressure equivalent to 6000 to 8000 feet.49 Rotor/wing aircraft usually fly relatively close to the ground and do not routinely have the ability to pressurize when they go to a higher altitude (when in an area of high altitude or crossing mountains). Atmospheric pressure drops from 760 at sea level to 565 mm Hg O2 at 8000 feet with a corresponding drop in the partial pressure of oxygen. Predicting Pao2 at altitude is an inexact science; all that can be said conclusively is that patients with a marginal Pao2 at ground level will be worse at cruising altitude. In practice the impact of a cabin pressure in this range can usually be overcome with supplemental oxygen.49 Oxygen should be used prophylactically in patients who are sensitive to alveolar hypoxia, such as those with reactive pulmonary hypertension. Boyle’s law describes the relationship between pressure and volume of a gas at a given temperature. The formula for Boyle’s law is: P1V1 5 P2V2 where P1 is the pressure at altitude 1, V1 is volume of gas at altitude 1, P2 is pressure at altitude 2, and V2 is volume of gas at altitude 2. This relationship becomes clinically significant when considering gas-filled spaces during changes in altitude and the resultant expansion and contraction of those gases. Common examples include gases present in the bowel, sinuses, and pleural space. A change from sea level to 8000 feet will result in a 30% increase in volume of gases. Even at altitudes used by rotorcraft (generally ,2000 feet above ground level), the relatively small volume of air in a tracheal tube cuff may be subject to clinically significant pressure changes. Studies have shown that significant increases in tracheal tube cuff pressure occur in rotor/wing transports even when ground-level cuff pressures are appropriate.50,51 Tracheal tube cuff pressures should be measured and adjusted during flight. Air removed at altitude may need to be replaced during descent. In addition, most ventilators are calibrated for performance at sea level. Tidal volumes delivered by the LTV 1000 (Pulmonetic Systems Inc.), a commonly used transport ventilator, may vary from 5% to 12% at a simulated altitude of 4000 and 8000 feet in volume control mode. At 15,000 feet LTVdelivered tidal volumes may be 30% to 37% greater than set tidal volumes.52 Similar findings have been reported with the Drager Oxylog ventilator (Drägerwerk AG & Co.).53 In addition to the effects on atmospheric pressure, other challenges are encountered in the aeromedical transport environment. Temperature drops significantly with increasing altitude, potentially complicating patient temperature regulation, especially for the smallest patients. The noise and vibration produced by rotorcraft make auscultation and simple procedures nearly impossible. Flight crews must rely on monitoring that does not depend on audible sounds, such as noninvasive blood pressure readings, capnometry, and pulse oximetry to assess patients in flight. The main advantage of rotor-wing transport is the speed at which the transport team can be deployed and return with the patient. Fixed-wing transport is most commonly used for long-distance travel or when weather conditions may not be appropriate for rotor-wing flights. In addition, some fixed-wing aircraft have the capability to pressurize the cabin, thus mitigating the effects of flight altitude on the patient. However, acceleration and deceleration forces during takeoff and landing can cause clinically significant fluid shifts. In most aircraft the stretcher must be positioned head forward, which may cause blood to pool in the legs during acceleration or takeoff and in the head during deceleration of



CHAPTER 13 Pediatric Critical Care Transport

landing.49 In a patient with shock, raising the feet can minimize the decrease in cardiac output owing to decreased venous return associated with takeoff. In a patient with increased intracranial pressure, the head should be raised during landing to minimize the increase in intracranial pressure associated with landing in this position.

Referring Hospital Responsibilities Transfer of patients from one institution to another in the United States is regulated by federal statute. The current legal standard was established by the Consolidated Omnibus Budget Reconciliation Act (COBRA) of 1986 and its amendment in 1989, which created requirements for patient stabilization and transfer, guaranteeing equal access to emergency treatment to all regardless of ability to pay.54,55 The Emergency Medical Treatment and Labor Act (EMTALA) established by the COBRA legislation further delineates rules for interfacility transfer. Appropriate transfers must meet the following criteria: (1) the transferring hospital must provide care and stabilization within its ability; (2) the referring physician certifies that the medical benefits expected from the transfer outweigh the risks; (3) the patient consents to transfer after being informed of the risks of transfer; (4) the receiving facility must have available space and qualified personnel and agree to accept the transfer; (5) copies of medical records and imaging studies should accompany the patient; and (6) the interfacility transport must be made by qualified personnel with the necessary equipment. Under COBRA, the referring hospital bears primary responsibility for the safety of interfacility transfer. Ideally, the decision about choice of team and mode of transport would be made jointly between the referring and receiving physicians.

Summary The potential limited pediatric training and pediatric critical care exposure and experience of nonspecialized transport personnel— coupled with the differences in size, anatomy, physiology, and psychosocial aspects unique to children—will continually challenge these providers. The approach that prioritizes rapidity of transport over stabilization and initiation of care may not result in the best outcomes for children. The need for pediatric-specific

93

intensive care begins long before the patient arrives in the tertiary care center. Children transported by nonspecialized teams are at higher risk of transport-related adverse events and worse outcomes. The primary goals in the majority of pediatric transports should be recognition of severity of illness, timely initiation of appropriate therapies, and providing a high level of clinical care with a focus on safety. The unique perspective of the pediatric intensivist is valuable and can help to change practice in the referring community.

Key References AAP Section on Transport Medicine, Romito J, Insoft RM, Schwartz HP. Guidelines for Air and Ground Transport of Neonatal and Pediatric Patients Manual. 4th ed. Elk Grove Village, IL: American Academy of Pediatrics; 2015. Aspiotes CR, Gothard MQ, Gothard MD, Parrish R, Schwartz HP, Bigham MT. Setting the Benchmark for the Ground and Air Medical Quality in Transport International Quality Improvement Collaborative. Air Med J. 2018;37:244-248. Cheema B, Welzel T, Rossouw B. Noninvasive ventilation during pediatric and neonatal critical care transport: a systematic review. Pediatr Crit Care Med. 2019;20:9-18. Commission on Accreditation of Medical Transport Systems. CAMTS. https://www.camts.org/. Davis AL, Carcillo JA, Aneja RK, et al. American College of Critical Care Medicine Clinical Practice Parameters for Hemodynamic Support of Pediatric and Neonatal Septic Shock. Crit Care Med. 2017;45:10611093. Gausche-Hill M, Ely M, Schmuhl P, et al. A national assessment of pediatric readiness of emergency departments. JAMA Pediatr. 2015;169: 527-534. Ground and Air Medical Quality in Transport. http://gamutqi.org/. Orr RA, Felmet KA, Han Y, et al. Pediatric specialized transport teams are associated with improved outcomes. Pediatrics. 2009;124:40-48. Patel SC, Murphy S, Penfil S, Romeo D, Hertzog JH. Impact of interfacility transport method and specialty teams on outcomes of pediatric trauma patients. Pediatr Emerg Care. 2018;34:467-472. Stroud MH, Sanders Jr RC, Moss MM, et al. Goal-directed resuscitative interventions during pediatric interfacility transport. Crit Care Med. 2015;43:1692-1698.

The full reference list for this chapter is available at ExpertConsult.com.

e1

References













































32.





33.





34.





35.





36.





37.





38.





39.





40.





41.





42.



43.















31.





30.





29.





28.





27.















26.





25.





24.





23.





22.





1. Gausche-Hill M, Schmitz C, Lewis RJ. Pediatric preparedness of US emergency departments: a 2003 survey. Pediatrics. 2007;120:12291237. 2. Whitfill T, Auerbach M, Scherzer DJ, Shi J, Xiang H, Stanley RM. Emergency care for children in the United States: epidemiology and trends over time. J Emerg Med. 2018;55:423-434. 3. de Oliveira CF, de Oliveira DS, Gottschald AF, et al. ACCM/PALS haemodynamic support guidelines for paediatric septic shock: an outcomes comparison with and without monitoring central venous oxygen saturation. Intensive Care Med. 2008;34:1065. 4. Kumar A, Roberts D, Wood KE, et al. Duration of hypotension prior to initiation of effective antimicrobial therapy is the critical determinant of survival in human septic shock. Crit Care Med. 2006;34:1589. 5. AAP Section on Transport Medicine, Romito J, Insoft RM, Schwartz HP. Guidelines for Air and Ground Transport of Neonatal and Pediatric Patients Manual. 4th ed. Elk Grove Village, IL: American Academy of Pediatrics; 2015. 6. Seidel JS, Hornbein M, Yoshiyama K, et al. Emergency medical services and the pediatric patient: are the needs being met? Pediatrics. 1984;73:769-772. 7. American College of Surgeons Committee on Trauma, American College of Emergency Physicians, National Association of EMS Physicians, Pediatric Equipment Guidelines Committee-Emergency Medical Services for Children (EMSC) Partnership for Children Stakeholder Group, American Academy of Pediatrics. Policy statement—Equipment for ambulances. Pediatrics. 2009;124: e166-e171. 8. Gausche-Hill M, Ely M, Schmuhl P, et al. A national assessment of pediatric readiness of emergency departments. JAMA Pediatr. 2015;169:527-534. 9. Glaeser PW, Linzer J, Tunik MG, et al. Survey of nationally registered emergency medical services providers: pediatric education. Ann Emerg Med. 2000;36:33-38. 10. Drayna PC, Browne LR, Guse CE, et al. Prehospital pediatric care: opportunities for training, treatment, and research. Prehosp Emerg Care. 2015;19:441-447. 11. Babl FE, Vinci RJ, Bauchner H, et al. Pediatric prehospital advanced life support care in an urban setting. Pediatr Emerg Care. 2001;17:36-37. 12. Su E, Schmidt TA, Mann NC, et al. A randomized controlled trial to assess decay in acquired knowledge among paramedics completing a pediatric resuscitation course. Acad Emerg Med. 2000;7: 779-786. 13. Youngquist ST, Henderson DP, Gausche-Hill M, et al. Paramedic self-efficacy and skill retention in pediatric airway management. Acad Emerg Med. 2008;15:1295-1303. 14. Aijian P, Tsai A, Knopp R, et al. Endotracheal intubation of pediatric patients by paramedics. Ann Emerg Med. 1989;18:489-494. 15. Losek JD, Bonadio WA, Walsh-Kelly C, et al. Prehospital endotracheal intubation performance review. Pediatr Emerg Care. 1989;5:1. 16. Mishark KJ, Vukov LF, Gudgell SF. Airway management and air medical transport. J Air Med Transp. 1992;11:7-9. 17. Boswell WC, McElveen N, Sharp M, et al. Analysis of prehospital pediatric and adult intubation. Air Med J. 1995;14:125-127. 18. Doran JV, Tortella BJ, Drivet WJ, et al. Factors influencing successful intubation in the prehospital setting. Prehosp Disaster Med. 1995;10:259-264. 19. Bankole S, Asuncion A, Ross S, et al. First responder performance in pediatric trauma: a comparison with an adult cohort. Pediatr Crit Care Med. 2011;12:e166-e170. 20. Carcillo JA, Kuch BA, Han YY, et al. Mortality and functional morbidity after use of PALS/APLS by community physicians. Pediatrics. 2009;124:500-508. 21. Davis AL, Carcillo JA, Aneja RK, et al. American College of Critical Care Medicine Clinical Practice Parameters for Hemodynamic

Support of Pediatric and Neonatal Septic Shock. Crit Care Med. 2017;45:1061-1093. Han YY, Carcillo JA, Dragotta MA, et al. Early reversal of pediatricneonatal septic shock by community physicians is associated with improved outcome. Pediatrics. 2003;12:793-799. Chance GW, Matthew JD, Gash J, et al. Neonatal transport: a controlled study of skilled assistance. J Pediatr. 1978;93:662-666. Patel SC, Murphy S, Penfil S, Romeo D, Hertzog JH. Impact of interfacility transport method and specialty teams on outcomes of pediatric trauma patients. Pediatr Emerg Care. 2018;34:467-472. Bellingan G, Olivier T, Batson S, et al. Comparison of a specialist retrieval team with current United Kingdom practice for the transport of critically ill patients. Intensive Care Med. 2000;26:740-744. Macnab AJ. Optimal escort for interhospital transport of pediatric emergencies. J Trauma. 1991;31:205-209. Edge WE, Kanter RK, Weigle CG, et al. Reduction of morbidity in interhospital transport by specialized pediatric staff. Crit Care Med. 1994;22:1186-1191. Orr R, Venkataraman S, Seidberg N, et al. Pediatric specialty care teams are associated with reduced morbidity during pediatric interfacility transport. Crit Care Med. 1999;27:A30. Macnab AJ, Wensley DF, Sun C. Cost-benefit of trained transport teams: estimates for head-injured children. Prehosp Emerg Care. 2001;5:1-5. Orr RA, Felmet KA, Han Y, et al. Pediatric specialized transport teams are associated with improved outcomes. Pediatrics. 2009;124:40-48. Ramnarayan P, Thiru K, Parslow R, et al. Effect of specialist retrieval teams on outcomes in children admitted to paediatric intensive care units in England and Wales: a retrospective cohort study. Lancet. 2010;376:698-704. Lampariello S, Clement M, Aralihond AP, et al. Stabilisation of critically ill children at the district general hospital prior to intensive care retrieval: a snapshot of current practice. Arch Dis Child. 2010;95:681-685. Stroud MH, Sanders Jr RC, Moss MM, et al. Goal-directed resuscitative interventions during pediatric interfacility transport. Crit Care Med. 2015;43:1692-1698. Stroud MH, Prodhan P, Moss M, et al. Enhanced monitoring improves pediatric transport outcomes: a randomized controlled trial. Pediatrics. 2011;127:42-48. Ehrenwerth J, Hackel A. Air-to-ground communication: a valuable aid in the transport of critically ill patients. Crit Care Med. 1986;14:543-547. Accreditation standards of the Commission on Accreditation of Air Medical Services. 5th ed. Anderson, SC: Commission on Accreditation of Air Medical Services; 2001. Tanem J, Triscari D, Chan M, Meyer MT. Workforce survey of pediatric interfacility transport systems in the United States. Pediatr Emerg Care. 2016;32:364-370. DeVita MA, Schaefer J, Lutz J, et al. Improving medical emergency team (MET) performance using a novel curriculum and a computerized human patient simulator. Qual Saf Health Care. 2005;14:326331. Campbell DM, Dadiz R. Simulation in neonatal transport medicine. Semin Perinatol. 2016;40(7):430-437. Burns B, Hansen ML, Valenzuela S, et al. Unnecessary use of red lights and sirens in pediatric transport. Prehosp Emerg Care. 2016;20:354-361. Commission on Accreditation of Medical Transport Systems. CAMTS. https://www.camts.org/. Schwartz HP, Bigham MT, Schoettker PJ, et al. Quality Metrics in Neonatal and Pediatric Critical Care Transport: A National Delphi Project. Pediatr Crit Care Med. 2015;16:711-717. Aspiotes CR, Gothard MQ, Gothard MD, Parrish R, Schwartz HP, Bigham MT. Setting the benchmark for the ground and air medical quality in Transport International Quality Improvement Collaborative. Air Med J. 2018;37:244-248.

e2



















50. Orsborn J, Graham J, Moss M, Melguizo M, Nick T, Stroud M. Pediatric endotracheal tube cuff pressures during aeromedical transport. Pediatr Emerg Care. 2016;32:20-22. 51. Long MT, Cvijanovich NZ, McCalla GP, Flori HR. Changes in pediatric-sized endotracheal tube cuff pressure with elevation gain: observations in ex vivo simulations and in vivo air medical transport. Pediatr Emerg Care. 2018;34:570-573. 52. Rodriquez Jr D, Branson RD, Dorlac W, et al. Effects of simulated altitude on ventilator performance. J Trauma. 2009;66(suppl4):S172S177. 53. Flynn JG, Singh B. The performance of Dräger Oxylog ventilators at simulated altitude. Anaesth Intensive Care. 2008;36:549-552. 54. Fell MJ. The Emergency Medical Treatment and Active Labor Act of 1986: providing protection from discrimination in access to emergency medical care. Spec Law Dig Health Care Law. 1996;9-42. 55. Omnibus Budget Reconciliation Act of 1989, sec. 6018 42 USC 1395cc (West Supp 1990).



















44. Ground and Air Medical Quality in Transport. http://gamutqi.org/. 45. Cheema B, Welzel T, Rossouw B. Noninvasive ventilation during pediatric and neonatal critical care transport: a systematic review. Pediatr Crit Care Med. 2019;20:9-18. 46. Holt PL, Hodge AB, Ratliff T, Frazier WJ, Ohnesorge D, Gee SW. Pediatric extracorporeal membrane oxygenation transport by EC145 with a custom-built sled. Air Med J. 2016;35:171-175. 47. Lindén V, Palmer K, Reinhard J, et al. Inter-hospital transportation of patients with severe acute respiratory failure on extracorporeal membrane oxygenation—national and international experience. Intensive Care Med. 2001;27:1643-1648. 48. Wilson BJ, Heiman HS, Butler TJ, Negaard KA, DiGeronimo R. A 16-year neonatal/pediatric extracorporeal membrane oxygenation transport experience. Pediatrics. 2002;109:189-193. 49. Martin T, Glanfield M. The physiological effects of altitude. In: Martin T, ed. Aeromedical Transportation: A Clinical Guide. 2nd ed. Aldershot, Hampshire, England: Ashgate Publishing; 2006:39-54.

e3

Abstract: Pediatric subspecialty and critical care medicine is largely regionalized, resulting in need for interfacility transport of patients with a range of severity of illness. The goal during transport should be to minimize the risk of deterioration while advancing the care initiated at the referring facility. Use of specialized pediatric transport teams has been shown to improve patient outcomes. These specialized teams represent a vital part of a continuum

of critical care providing tertiary care to patients who present to nontertiary care facilities and providers. Intentional implementation, administration, oversight, education, and ongoing quality assurance are essential to a successful transport process. Key Words: Transport, flight physiology, team composition, medical control physician, interfacility

14 Pediatric Vascular Access and Centeses LAUREN R. EDWARDS, MATTHEW P. MALONE, PARTHAK PRODHAN, AND STEPHEN M. SCHEXNAYDER



Intraosseous infusion is an essential emergency vascular access, with several mechanical devices available. Vigilant observation of the needle insertion site is essential to recognize extravasation and prevent serious complications. Pericardiocentesis may be required for both diagnostic and therapeutic purposes. Except in life-threatening tamponade, ultrasound imaging should be used to improve success and reduce complications. Umbilical arterial and venous access may be useful in neonates up to 2 weeks of age in the pediatric intensive care unit. Vascular

Intraosseous Infusion Venous access can be one of the most challenging aspects of caring for critically ill infants and children. Peripheral veins can be difficult to cannulate, particularly in the setting of shock, where there is shunting of blood away from the periphery and collapse of small veins. Because of these challenges, intraosseous (IO) infusion has become widely accepted as a quick, reliable means to establish short-term emergency venous access in critically ill children.1,2 The marrow space provides a noncollapsible access point to the vascular system. Marrow sinusoids drain into medullary venous channels that empty into the systemic circulatory system. This allows IO infusions of fluids and medications to be rapidly distributed.

Indications IO infusion is indicated for conditions requiring the rapid acquisition of intravenous (IV) access, where the establishment of conventional peripheral access is difficult or impossible: cardiopulmonary arrest, shock, burns, and status epilepticus. In these situations, limited attempts at standard peripheral access are usually made prior to placing an IO needle. In addition to its use in the hospital, IO access has been used successfully in the prehospital setting as well as in critical care transport.3–5 The success rate for acquiring IO access is high—greater than 95% with experienced practitioners.6 Similar success rates and equivalent pharmacokinetics have been demonstrated when mechanical IO devices are used.7,8 Most fluids and medications that 94















PEARLS access is paramount for the effective management of critically ill and injured children. Pediatric critical care providers must be expert at obtaining access using a number of different techniques and approaches. Because fluid accumulation in serosal cavities can be part of disease processes as well as a response to fluid resuscitation, fluid removal by centesis is frequently needed for both diagnostic and therapeutic purposes.

can be given through a conventional IV line can be administered as an IO infusion with comparable results. In the setting of cardiac arrest or severe shock, IO access is as effective as peripheral venous access in providing volume resuscitation and antibiotics to the central circulation. However, three antibiotics—chloramphenicol, vancomycin, and tobramycin—achieve subtherapeutic levels when administered via an IO line at standard IV doses.9 Additional details regarding laboratory studies obtained from IO versus IV samples are discussed on ExpertConsult.com.

Contraindications IO infusion has few absolute contraindications. A fractured or previously punctured bone should not be used, as infusing fluid will extravasate and potentially cause compartment syndrome. Therefore, if an IO needle penetrates the cortex but is nonfunctional, alternative bone sites must be used for subsequent attempts. Bone diseases such as osteogenesis imperfecta and osteopetrosis have been suggested as contraindications to IO infusion.16 Placing the needle into an area of cellulitis or burn risks seeding infection and causing osteomyelitis. This is a relative contraindication, as limited sites may be available, making these less desirable locations acceptable for use.

Supplies and Equipment The bone marrow space is accessed with the use of one of several different types of needles. Conventional bone marrow needles (e.g., the Jamshidi needle, Becton Dickinson) and IO infusion

94.e1

Comparison of Laboratory Studies Obtained From Intraosseous Versus Intravenous Samples IO access can be used for certain clinical laboratory studies. Specimens should be collected promptly after access is established— ideally, after aspirating 1 to 2 mL of waste blood and prior to any medications being given. Bicarbonate, partial pressure of carbon dioxide (Pco2), and partial pressure of oxygen (Po2) can differ significantly between IO and venous specimens, whereas pH and base excess correlate well,10 as well as lactic acid. 11 Ionized calcium levels may correlate, but the data are inconsistent. 10,12 No significant differences were found with glucose, blood urea nitrogen, creatinine, hemoglobin, and

hematocrit when comparing IO specimens and venous blood samples.10–12 IO sodium concentrations are lower; however, the difference does not appear clinically relevant.11 IO potassium levels are significantly higher than venous specimens and are invalid; treating these levels poses the risk of inducing iatrogenic hypokalemia.10,11 Leukocytes and platelet counts are unreliable from IO specimens.12 In general, when comparing IO and venous or arterial sources, there are correlations and clinical similarities in laboratory values. However, caution should be exercised with their interpretation, as the evidence is relatively weak.13 In contrast, a marrow specimen can be cultured in lieu of a blood culture and can be used for blood type and crossmatching.14,15



CHAPTER 14 Pediatric Vascular Access and Centeses

needles (e.g., the Cook intraosseous infusion needle, Cook Medical) can be used by manual insertion. Usually, a 15- or 18-gauge needle is chosen, with the latter being used in infants. If these are not available, lumbar puncture needles can be used; however, they bend easily.6,17 In neonates, a 19- or 21-gauge butterfly needle can be used.17 Needles with a stylet are preferred to prevent clogging of the needle with bony particulate. Three mechanical devices appropriate for pediatric patients have been introduced: the Bone Injection Gun (BIG, PerSys Medical), New Intraosseous (NIO, PerSys Medical), and the EZIO (Teleflex; Fig. 14.1). The BIG and NIO are spring-loaded devices, whereas the EZ-IO is a small, battery-powered drill. These devices penetrate the bone marrow more quickly and are used more frequently than the manual method in many settings, including prehospital.18,19 Finally, the FASTResponder (Pyng Medical), a specialized sternal injection gun, is used primarily in the adult population but has been cleared for use in adolescents who are 12 years of age and older in the United States, Canada, and most of Europe. Other equipment required for IO needle placement and infusion include antiseptic solution (chlorhexidine or povidoneiodine), sterile gloves and drapes, a syringe with saline or heparinized

​EZ-IO battery-powered drill. (Courtesy Vidacare Corporation.)

saline flush, a T-connector or stopcock, IV fluid and tubing, and IO dressing components (gauze and tape) and/or securement device. Optional supplies include a syringe for laboratory specimen collection, towel or IV bag for extremity stabilization, pressure bag, and materials for local anesthesia (syringe with 25-gauge needle and 1% lidocaine).

Technique The IO needle can be placed into the bone marrow at one of several sites: the proximal tibia, distal femur, distal tibia, proximal humerus, iliac crest, and sternum. In preparation for IO insertion, the selected body site needs to be stabilized. In an extremity, this can be achieved by placing a towel or IV bag underneath it. Additionally, the operator’s nondominant hand is used to stabilize the extremity. However, it is essential to ensure that the hand is clear of the area behind the insertion site to minimize the risk of needlestick injury. The anatomic landmarks and intended insertion site should be identified, and the overlying skin prepped with an antiseptic solution. If the patient is conscious, topical anesthesia is indicated. The proximal tibia is the most commonly used location for IO access. In children, the insertion site is located on the tibial plateau, approximately 1 cm medial and 1 to 2 cm distal to the tibial tuberosity. In young children, this location offers the thinnest cortex while having the highest vascular content in the area and avoids injury to the proximal epiphyseal plate. For adolescents and adults, the insertion site is 2 cm medial and 1 cm proximal to the tibial tuberosity (Fig. 14.2). Accessing the midshaft increases the risk for fracture. To obtain IO access in the distal femur, the needle should be positioned in the midline, approximately 2 to 3 cm proximal to the patella. In the distal tibia, the needle is placed 1 cm proximal to the medial malleolus, midway between the anterior and posterior surfaces. The distal tibia may be easier to access in older children (.6 years), as the proximal tibial cortex has become thicker. Although it is less preferred, the distal fibula can also be accessed 1 cm above the lateral malleolus. For the proximal humerus, the patient’s hand is positioned on the abdomen with the elbow adducted; the insertion site is the greater tubercle, 1 to 2 cm proximal to the surgical neck of the humerus. The anterior superior iliac spine is the insertion site for IO access on the iliac crest. The sternum has historically been an access point used by the military in combat. More recently, sternal IOs have been developed for use in civilians, including adolescents and adults. A

Tibial tuberosity Insert needle into medial flat surface of the anterior tibia.

Growth plate

• Fig. 14.2





• Fig. 14.1

95

​Insertion of the intraosseous needle into the anterior tibia.

96

SECTION II



Pediatric Critical Care: Tools and Procedures

needle is injected a set depth into the manubrium using an injection gun. It can provide effective access, even while cardiopulmonary resuscitation (CPR) is in progress. However, CPR must be paused for placement—ideally, during a pulse and rhythm check. Sternal IO lines should be avoided in infants and children due to the risk for cardiac or major vessel puncture as well as inadequate drug delivery due to the small sternal marrow cavity. IO needles can be placed manually or using mechanical devices. With manual needles, the needle insertion angle is controversial: some suggest inserting the needle at a 60- to 75-degree angle away from the epiphyseal plate of long bones, while others recommend using a 90-degree angle to prevent the needle from sliding along the bone. The needle is advanced using firm pressure and a twisting motion until a “give” (loss of resistance) is felt, indicating entry into the marrow space. The bony cortex, which thickens with age, requires considerable force to penetrate. The stylet is removed, and a syringe is attached to the needle in an attempt to aspirate marrow. Correct placement of the needle should always be confirmed to avoid complications such as extravasation. There are several ways to confirm that the needle is correctly placed: (1) aspiration of bloody fluid; (2) observation that the needle stands upright in the bone without support; (3) lack of resistance when saline solution is infused; and (4) absence of noticeable swelling of the soft tissues or extravasation of fluid. Sometimes marrow cannot be aspirated even if the needle is correctly placed; in that case, one must rely on the other means of confirmation. Placement of IO needles with mechanical devices has success rates equal to or higher than manual methods, with the added benefit of ease of use and less risk to the user.4,10 However, the high cost of this equipment may be a limitation, especially in small facilities or resource-limited settings. In children, prehospital providers prefer the IO drill to the spring-loaded injection gun.20,21 The appropriate mechanical device is selected based on age/weight of the patient and the intended IO insertion site. The EZ-IO is approved for use in patients who weigh greater than or equal to 3 kg; the appropriate needle length is chosen based on weight, site, and presence of excessive tissue. Operators should ensure that at least 5 mm of the catheter remain visible outside of the skin. Longer needles may be considered in obese patients. where the tibial tuberosity is not palpable and body mass index exceeds 43.22 This device is approved for pediatric use in the proximal tibia, distal femur, distal tibia, and proximal humerus. In contrast, the BIG Pediatric and NIO-Pediatric (NIO-P) are intended for use only in the proximal tibia, with the former cleared for use in term neonates to children 12 years of age and the latter in children 3 to 12 years of age. Respective adult versions can be used for patients older than 12 years. Detailed instructions for use of mechanical IO insertion devices are discussed on ExpertConsult.com.

Maintenance Once correct placement of the needle is confirmed and secured, fluids and medication can be administered with a syringe via a stopcock or T-connector, or a standard IV infusion set can be connected to the needle. The site should be observed visually and by palpation for signs of extravasation immediately after placement and every 5 to 10 minutes during use. If evidence of extravasation is observed, the needle should be removed to avoid compartment syndrome. To remove, while holding the hub of the catheter (or needle) itself or attaching a Luer-Lok syringe, traction is applied

while rotating the catheter clockwise, pulling it out of the bone without bending or rocking. Following pressure hemostasis, a dressing is applied using aseptic technique. IO access is intended only for short-term use in emergency resuscitative situations; long-term use increases the risk of extravasation, compartment syndrome, and infection.23,24 Therefore, once IO access is secured, efforts should be directed toward obtaining definitive IV access. Once alternative access is obtained, the IO needle should be removed.

Complications Significant complications of IO insertion and infusion are rare. The most common complication is extravasation of fluid. The causes of extravasation include incomplete penetration of the bony cortex, movement of the needle such that the hole is larger than the needle, dislodgment of the needle, penetration of the posterior cortex, and leakage of fluid through another hole in the bone, such as a previous IO site or fracture. Extravasation of a small amount of fluid is usually not problematic. However, with larger volumes, compartment syndrome can develop, which may require fasciotomy and even amputation. Use of the IO line for prolonged periods or with pressure bags increases the risk for this complication. If extravasation occurs, the needle should be removed and the extremity diligently observed for signs of compartment syndrome. Experience to date suggests that the complications of the new mechanical insertion devices are similar to manual IO needle use. Other rare complications include infection and bone fracture. Osteomyelitis, cellulitis, and sepsis have been reported in conjunction with IO infusion.25,26 Risk for infection is increased when IO access is prolonged, and these devices are used in patients with bacteremia.

Summary IO infusion is a valuable means of obtaining temporary emergency vascular access in the critically ill infant or child, as it has a high success rate. Using appropriate technique and vigilantly monitoring the insertion site for extravasation can usually prevent complications.

Arterial Catheter Placement The dynamic and rapidly evolving nature of pediatric critical illness often requires frequent blood sampling and continuous blood pressure monitoring in order to thoroughly assess acid-base status, oxygenation, and ventilation as well as to plan timely interventions aimed at improving systemic oxygen delivery (Do2). The arterial catheter serves as an invaluable tool to achieve these goals in addition to providing a visible pressure waveform that may contribute additional diagnostic information (see Chapters 26 and 33). Therefore, the ability to place an arterial catheter is a fundamental skill in pediatric critical care medicine.

Indications There are several indications for an arterial catheter: • Need for continuous invasive blood pressure measurements to assess the patient’s hemodynamic status and allow for timely assessment of interventions aimed at improving hemodynamics, such as fluid administration and titration of vasoactive infusions.

96.e1

Instructions for Use of Mechanical Intraosseous Insertion Devices When using the EZ-IO drill, the operator must first ensure that the driver and needle set are securely seated magnetically. After removing the safety cap, the operator positions the driver at the insertion site with the needle set at a 90-degree angle to the bone, pushes the needle set through the skin until it touches bone, and squeezes the trigger, applying moderate, steady downward pressure until the bone cortex is penetrated. The trigger should be released when a sudden “give” is felt upon entry into the medullary space in children to ensure that it does not penetrate the posterior cortex (although it may be advanced 1 to 2 cm into the space for adolescents). If excessive force is used, the driver may stall and not penetrate the bone. After IO placement, the power driver is detached and the stylet removed. After confirming catheter stability and placement, a primed extension set is attached to the catheter hub, and the apparatus is flushed. When using the spring-loaded injection gun, such as the BIG Pediatric or NIO-P, one must dial in the desired needle penetration depth based on the patient’s age. For the BIG Pediatric, the

red barrel is firmly held at the insertion site with the nondominant hand at a 90-degree angle while the dominant hand pulls out the safety latch. After placing two fingers of the dominant hand on the wings and the palm on top and applying consistent and gentle downward pressure, the device is activated with the dominant hand. The device is removed by pulling upward with a slight side-to-side motion, leaving the cannula in place. The trocar is pulled out; then, the safety latch can be slid around the cannula and taped for stabilization. For the NIO-P, hold the red barrel with the nondominant hand and place the designated location arrow on the tibial tuberosity with the arrow pointed toward the knee. Rotating the cap 90 degrees with the dominant hand will unlock the device, which is pressed against the skin with the palm of the dominant hand, while placing two fingers on the trigger wings. Continuing to apply downward pressure with the palm, the trigger wings are pulled upward to activate. Following insertion, the base of the needle stabilizer is maintained while lifting up and disconnecting the device from the cannula and removing the trocar and securing the line.



CHAPTER 14 Pediatric Vascular Access and Centeses







• Need for frequent sampling of arterial blood for laboratory analysis. Access to arterial blood through an indwelling catheter eases the task of obtaining blood samples painlessly. Catheter-derived arterial samples also eliminate skewing of results caused by physiologic changes related to the stress and discomfort of vascular puncture. Most notably, an indwelling arterial catheter allows for frequent assessment of arterial blood gas measurements, thereby providing the most accurate information on a patient’s acid-base status as well as measurement of partial pressure of arterial oxygen. • Need for continuous monitoring of cerebral perfusion pressure in patients with traumatic brain injury or other causes of increased intracranial pressure (see Chapter 60). • Need for arterial access to facilitate therapeutic procedures, such as exchange transfusions and continuous arteriovenous hemodiafiltration.

Contraindications Few absolute contraindications for placement of arterial catheters exist. The skin at the site of arterial access must be intact prior to insertion of a catheter. Evidence of infection of the skin or underlying structures is a contraindication to catheter placement at that site. Other disruptions in skin integrity, such as burns, are relative contraindications. Severe coagulopathy and systemic anticoagulation increase the risk of hemorrhage from unsuccessful arterial punctures during attempted arterial catheter placement and from the site of arterial catheter insertion. These risks must be weighed against the potential benefits of improved monitoring when facing the decision of whether to place an arterial catheter in this patient population. A catheter should not be placed in an extremity with compromised perfusion. Evidence of adequate collateral circulation is desirable prior to placement of an arterial catheter. The traditional means of assessing collateral circulation to the hand is the Allen test. The radial and ulnar arteries are compressed until the distal extremity is blanched. Pressure over one artery is then released; capillary refill should return to the distal extremity within 5 seconds. The test is repeated, releasing pressure from the other contributing artery. A normal Allen test does not guarantee adequate collateral circulation, and an abnormal test does not necessarily indicate that complications will occur.27 Additionally, the Allen test is considered less reliable for patients in shock. It is common for arterial catheters to be placed without assessing collateral circulation in emergent circumstances.

Procedure Supplies and equipment required for arterial catheterization are listed in eBox 14.1.

Technique The initial step in placing an arterial catheter is site selection. The radial, posterior tibial, and dorsalis pedis arteries are optimal sites owing to easy accessibility and typically good collateral circulation. Placement of the catheter in distal arteries of the extremities also allows for ease of site observation and hemorrhage control with direct pressure. Preductal placement in the right radial artery is preferred in infants with ductal-dependent cardiac lesions. Catheters also can be placed in the axillary or femoral arteries if no peripheral sites are suitable. Insertion of a catheter into the

97

axillary artery is more difficult technically than the other sites mentioned and is associated with a risk of brachial plexus injury due to hematoma compressing the neurovascular bundle.28 Many physicians have been reluctant to place arterial catheters for longterm use into the femoral artery—particularly in infants and young children—for fear of complications, most notably severe ischemia of the limb. A retrospective study of 234 pediatric burn patients who underwent 745 femoral artery catheterizations revealed a 1.1% rate of loss of distal pulse; limb ischemia was associated with younger age, smaller patient size, and increased severity of the burn injury.29 Patients who suffered limb ischemia were managed with immediate catheter removal and systemic heparinization. Three underwent thrombectomy, with one requiring amputation of a digit. Traditionally, it is taught that the brachial arteries should not be used for arterial catheters because of the lack of collateral blood flow and risk of distal extremity ischemia. In a review of arterial catheter placements performed at a pediatric cardiac surgical center, 386 brachial artery catheters were placed in infants weighing 20 kg or less with no report of permanent ischemic damage and only three with temporary perfusion loss.30 Despite these results, the complete lack of collateral circulation at the brachial artery requires careful consideration of risks and benefits before placement of a brachial artery catheter. Additionally, the superficial temporal arteries should not be used owing to poor collateral flow and the potential for retrograde flow, which could result in showering of emboli into the cerebral circulation. Next, the selected site must be properly immobilized prior to placement of the indwelling catheter. If placing a radial artery catheter, the wrist is hyperextended 30 degrees to develop a straighter course and more superficial position of the radial artery. Typically, the radial pulse is best palpated in a position just proximal to the wrist crease. The technique for radial arterial catheter placement has been summarized in the “Videos in Clinical Medicine” series in the New England Journal of Medicine.31 The intended site for arterial catheter placement requires preparation with an aseptic solution and draping of sterile towels. Infiltration of lidocaine (1% without epinephrine) should be considered in most patients unless infiltration will obscure landmarks or the patient is deeply sedated. Alternatively, a topical anesthetic cream can be used for local anesthesia. Systemic narcotics or anxiolytics may be administered but demands caution in patients who are not receiving mechanical ventilation. Percutaneous placement of the catheter can be accomplished using one of several techniques. In the over-the-needle technique, similar to placement of a peripheral IV catheter, the needle is inserted through the skin at a 30-degree angle. When a flash of blood is obtained in the hub, advance the needle another 1 to 2 mm. Holding the needle steadily, the catheter is advanced over the needle into the lumen of the vessel. Blood should flow continuously into the catheter hub prior to attempting to advance the catheter. Once the catheter is inserted through the skin to the hub, pressure is applied over the artery proximal to the catheter and a flushed Luer-Lok connector is attached to the hub. Correct placement of the catheter is verified by easily aspirating arterial blood into a syringe. The catheter is then flushed and secured with suture or tape. A chlorhexidine-impregnated patch is usually placed at the site of catheter insertion (to decrease catheter-associated bloodstream infections), and an occlusive dressing with a transparent adhesive film is applied over the catheter as a protective barrier.32 The second percutaneous technique, transfixation, involves using the over-the-needle technique; however, when a flash of

97.e1

• eBOX 14.1 Supplies and Equipment for Arterial

Catheterization • Appropriate size catheter (24 gauge for infants, 22 gauge for toddlers and older) • Sterile gloves • 10% povidone-iodine or chlorhexidine solution • Sterile towels • Syringe with 1% lidocaine and 25-gauge needle for local infiltration • Topical anesthetic cream • Luer-Lok connector with heparinized flush • 3-0 silk suture • Instrument tray with needle holder and scissors • Cloth tape • Plastic, nonocclusive dressing • Connecting tubing • Transducer • Fluids containing heparin (1 U/mL) and papaverine

98

SECTION II



Pediatric Critical Care: Tools and Procedures

blood is seen in the hub, the needle and catheter are advanced further, through the posterior wall of the artery transfixing the vessel to the underlying structures. The needle is removed, leaving the catheter in place. The catheter is then slowly withdrawn until the tip is again intraluminal with blood flowing into the hub. The catheter is then advanced into the artery to its hub. Catheter advancement can be facilitated by attaching a Luer-Lok connecter with a heparinized flush-filled syringe on it and gently flushing as the catheter is advanced. The final and most successful percutaneous method for catheter placement in critically ill patients involves the use of the Seldinger technique.33 A needle is used to pierce the anterior wall of the artery. When arterial blood return is seen, a guidewire is placed through the introducer needle. The wire should meet little to no resistance. If resistance is met, the wire is retracted and pulsatile blood flow is assured at the hub of the needle. The depth or angle of the needle may need to be adjusted if blood is not flowing. If the guidewire glides in easily on insertion, it is advanced into the lumen of the vessel, the needle removed, and the catheter threaded over the guidewire into the arterial lumen. This method can also be used with the over-the-needle catheter technique. Improved success rates and shorter time to insertion characterize pediatric arterial catheters placed using a guidewire compared with no wire guide.34 However, an adult study found no difference in success rate or insertion times between the two groups.35 Ultrasound-assisted placement of arterial catheters is becoming increasingly popular (see Chapter 15). Ultrasound guidance significantly increases the first attempt success rate of radial arterial cannulation and decreases hematoma complication rates compared with the palpation or Doppler technique.36 Studies have also demonstrated decreased time to insertion and decreased number of attempts at catheter placement using ultrasoundguided techniques.36–40 However, in the hands of physicians who are inexperienced in performing arterial cannulations via ultrasound guidance (yet proficient in traditional methods), no difference in success rates was observed between the ultrasound-assisted and palpation-based techniques.37 The study shed light on the importance of experience and frequent practice in the use of ultrasound-guided arterial cannulation to develop competency. A cutdown approach serves as an alternative if percutaneous attempts are unsuccessful. A superficial incision of the skin is made perpendicular to the artery. The subcutaneous tissues are bluntly dissected parallel to the vessel using a hemostat. When the artery is identified, the posterior wall is gently dissected away from the adjacent structures. Two loops are placed around the vessel, one proximal and one distal. These loops are used to elevate the artery during cannulation; they should never be used to tie off the vessel. The artery is then cannulated under direct visualization using the over-the-needle technique. The catheter is secured with a suture through the skin, and the wound is closed with interrupted sutures. If excessive bleeding persists, gentle traction can be applied to the proximal loop in an attempt to control the hemorrhage.

Maintenance of an Arterial Catheter To prolong the patency of an arterial catheter, heparinized fluid is most commonly infused through the catheter. A common practice is to infuse 0.9% sodium chloride containing heparin 1 U/mL at 3 mL/h; slower infusion rates may be used in small infants requiring fluid restriction. There is conflicting literature supporting the use of heparin to positively impact patency and mitigate the risk of thrombus formation in peripheral arterial catheters.41,42 More

recent adult studies call into question the benefit of heparin and spotlight the need for more rigorous clinical investigation.43,44 However, these studies do not take into account the smaller vessel size and prolonged monitoring common in critically ill children. A randomized controlled trial demonstrated that the addition of papaverine (120 mg/L) to routine arterial catheter fluids significantly lowered the rate of catheter failure.45 This study recommended avoiding the use of papaverine in neonates owing to a perceived increase in risk of intraventricular hemorrhage. However, a more recent study of neonates 25 to 36 weeks’ gestational age did not confirm this concern.45 Despite these results, some institutions routinely avoid the use of papaverine in arterial catheter fluids in preterm neonates and patients with traumatic brain injury or other preexisting intracranial hemorrhage. Arterial catheters should always be visible so that any bleeding around the catheter site or inadvertent disconnection of the tubing from the catheter can be immediately identified to avoid significant hemorrhage. Securing the catheter with suture and using Luer-Lok connectors decrease the possibility of accidental detachment. The site of catheter insertion should be closely monitored for signs of infection or compromised perfusion. Mottling of the skin proximal or distal to the catheter may be indicative of intraarterial thrombus formation, and discoloration of fingers or toes distal to a catheter may result from emboli. If these complications occur, the catheter must be removed. Children with femoral artery catheters are at a higher risk of thrombus formation, particularly newborns and children with low body weight, low cardiac output, and elevated hematocrit.46 Transducing an arterial catheter is performed by placing the transducer at the level of the right atrium and zeroing to atmospheric pressure for accurate measurements.47 Studies in animal models have demonstrated that positioning the transducer to be level with the aortic root results in accurate measurement of mean arterial pressure (MAP) regardless of position or catheter site. This is in contrast to the significant error in MAP measurement that occurs when the transducer is level with the catheter tip.48 Arterial fluids and tubing are currently recommended to be changed every 96 hours.49 The overlying dressing is also changed on a scheduled basis and any time it becomes soiled or nonocclusive. Inability to draw blood from a catheter or flattening of the waveform on the monitor is suggestive of either a kinked catheter or thrombus formation at the end of the catheter. If no evidence of compromised perfusion is present distal to the catheter, the catheter may be exchanged over a guidewire. However, strong consideration should be given to removing the existing catheter and placing a new arterial catheter at a different site, as exchanging a catheter over a guidewire has been associated with an increased risk of catheter-related bloodstream infection (CRBSI) in central venous catheters.50

Complications Complications related to arterial catheters include hemorrhage, thrombus formation, emboli, distal ischemia, and infection. Permanent ischemic complications related to radial artery catheters in adult patients are rare.28 A multi-institutional diagnostic code database study demonstrated that 10.3% of patients with arterial catheters also had a code associated with infection or inflammation, and 7.5% had a thrombotic- or embolic-associated complication code. These complications were more common in younger children and longer hospitalizations.51 An uncommon but wellrecognized complication of arterial catheter placement is growth



CHAPTER 14 Pediatric Vascular Access and Centeses

arrest due to physeal injury from extravasation, aneurysm formation, or ischemia.52 Catheter-related infections can be local or systemic. The risk of catheter-related infection was previously thought to be lower for arterial catheters than for central venous catheters. However, a meta-analysis indicated that arterial catheters are an underrecognized source of CRBSI.53 Risks for an arterial catheter infection are related to the duration of catheter use and catheter placement in the femoral artery.53–56 The presence of an arterial catheter has been noted to be a risk for CRBSI, but it has been suggested that a positive culture is more likely to be a surrogate marker for greater illness severity.57 Nevertheless, an arterial catheter should be considered a potential source of sepsis, and strong consideration should be given to removing an arterial catheter when it is no longer needed for optimal care.

Summary Arterial catheters are an important way to monitor hemodynamics and gain valuable laboratory data in order to proactively provide interventions and manage critically ill children. The potential risks and benefits of arterial catheter placement should be weighed carefully prior to performing the procedure. Rigorous studies investigating the complications associated with arterial catheterization are lacking for critically ill children; thus, further study is needed.51

Central Venous Line Placement Central venous catheter (CVC) placement and use are frequently required in caring for critically ill patients. The need for central access should be anticipated so that circumstances surrounding the procedure, such as aseptic technique and patient safety, can be optimized.

Indications and Contraindications













Indications for CVCs include the following: • Need for reliable and durable venous access • Lack of or inadequate peripheral venous access • Administration of vasoactive infusions, total parenteral nutrition, and medications that require central venous delivery • Need for frequent blood sampling • Monitoring of central venous pressure and central venous oxygen saturation • Provision of access for extracorporeal support modalities, such as continuous renal replacement therapy and apheresis Contraindications to central access are not absolute and are primarily related to specific CVC placement sites. In the presence of coagulopathy or systemic anticoagulation, operators should consider avoiding sites where bleeding may be difficult to control (e.g., subclavian vein). Generally, CVC insertion sites with intravascular hardware (e.g., pacemaker, ventriculoatrial shunt, hemodialysis catheter) or adjacent to permanent hardware (e.g., cerebral ventricular shunt catheters subcutaneously tunneled along the neck) should be avoided owing to risks of infection, hardware puncture, and venous stasis. CVCs placed at a time of bacteremia will likely become colonized with the pathogen.58 Catheters should not be inserted through overtly infected skin. In traumatic brain injury management, it is reasonable to abstain from CVC placement in the neck vessels to avoid obstruction of jugular venous drainage from the brain and exacerbating intracranial

99

hypertension.59 The relative risks and benefits of CVC placement should be carefully weighed prior to each procedure.

Technique Critically ill pediatric patients range greatly in size. Being aware of vessel dimensions as well as the proximity and anatomic relation of the respective artery to the vein are important in central vein cannulation. Central vein diameters vary across the pediatric age groups (eTable 14.1); based on this measurement, an appropriately sized catheter should be selected for CVC placement. These catheters are commonly made of a plastic polymer and available in a variety of diameters, lengths, and number of lumens. CVCs are often packaged with the introducer needle, guidewire, and tissue dilator that correspond to the selected catheter diameter and length. Maximal sterile barrier precautions should be used whenever a CVC is placed in the pediatric intensive care unit (PICU). This includes mask, cap, sterile gown and gloves, and sterile full-body drape.60,61 Additionally, the following should be performed: thorough hand hygiene (which may involve the use of surgical antiseptic handwash or scrub brush), skin preparation at the insertion site using chlorhexidine antiseptic solution (covering an extensive area and allowing adequate dry time), and creation of a large sterile field with sterile drapes and towels to cover the patient’s entire body and bed (in order to minimize the risk of inadvertent contamination of sterile equipment and surfaces). Chlorhexidine is superior to povidone-iodine for skin disinfection.62,63 Adequate sedation and analgesia along with local anesthesia should be used to provide patient comfort during the procedure. Patient movement will also be minimized, diminishing the risk of sterile field disruption and allowing the procedure to be performed more easily and safely. Adherence to CVC insertion and maintenance bundles will minimize the risk of CRBSI. Most CVCs employed in the PICU are placed using the Seldinger technique, in which the clinician places an introducer needle into the desired vein while aspirating with a slip tip syringe. When the lumen of the needle is fully within the vein lumen, blood freely flows into the syringe. The needle is held in place with the nondominant hand while the syringe is disconnected with the other hand. Blood should continue to passively flow from the needle hub but not be pulsatile. A J-tipped guidewire is inserted into the open hub of the needle and advanced into the vein with little to no resistance (Fig. 14.3). If resistance is felt, further advancement of the wire should be avoided. Adjusting the needle position by slightly altering its depth or changing the angle of entry may facilitate guidewire insertion. If there continues to be resistance, the wire is carefully withdrawn, and the syringe is reattached to the needle in order to reidentify the vein’s lumen. If the wire is not easily retracted, the needle and wire should be removed as a unit, reducing the risk of breaking the wire. Once the guidewire is well within the lumen of the vein, a small nick in the skin adjacent to the needle is made using a No. 11-blade scalpel, enlarging the puncture site to more easily accommodate the dilator and catheter. The introducer needle is carefully withdrawn along the wire, maintaining control of the wire at all times either by holding it directly or intermittently using a sterile hemostat to clamp the end of the wire to ensure that it is not lost into the patient or onto the floor. A dilator is advanced along the wire and then twisted through the puncture site, dilating the tissue planes that lead to the lumen of the vessel; the vein itself should not be dilated. Following withdrawal of the

99.e1

Approximate Mean Femoral and Internal

eTABLE Jugular Vein Diameters Across the Pediatric 14.1

Age Interval

Mean IJV Diameter (mm)

Mean FV Diameter (mm)

25–27 wk PCAa

2.1

1.5

a

3.3

1.9

a

37–39 wk PCA

4.2

2.3

1 mo

5.5b

4.5c

b

5.4c

Age 31–33 wk PCA

1y

6.2

2 yd

6.7

6.3

d

7.8

7

6 yd

8.9

7.7

4y

8 yd

10

8.5

d

11.1

9.2

d

12.8

10.4

d

16 y

14.5

11.5

19 yd

16.2

12.6

10 y 13 y

a

Data from Tailounie M, McAdams LA, Frost KC, et al. Dimension and overlap of femoral and neck blood vessels in neonates. Pediatr Crit Care Med. 2012;13:312–317. b Data from Alderson PJ, Burrows FA, Stemp LI, Holtby HM. Use of ultrasound to evaluate internal jugular vein anatomy and to facilitate central venous cannulation in paediatric patients. Br J Anaesth. 1993;70:145–148. c Data from Warkentine FH, Clyde Pierce M, Lorenz D, Kim IK. The anatomic relationship of femoral vein to femoral artery in euvolemic pediatric patients by ultrasonography: implications for pediatric femoral central venous access. Acad Emerg Med. 2008;15:426–430. d Data from Steinberg C, Weinstock DJ, Gold JP, et al. Measurements of central blood vessels in infants and children: normal values. Cathet Cardiovasc Diagn. 1992;27:197–201. FV, Femoral vein; IJV, internal jugular vein; PCA, postconception age.

100

SECTION II



Pediatric Critical Care: Tools and Procedures

neck, and the face is turned to the contralateral side. Most commonly, the middle approach is used, in which the introducer needle enters the skin at a 30-degree angle at the apex of the triangle formed by the clavicle and the heads of the sternocleidomastoid muscle and is directed toward the ipsilateral nipple (Fig. 14.4A). For the anterior approach, the introducer needle enters the skin along the anterior margin of the sternocleidomastoid halfway between the mastoid process and the sternum and is directed at the ipsilateral nipple (Fig. 14.4B). Using the posterior approach, the needle enters the skin along the posterior border of the sternocleidomastoid halfway between the mastoid process and the clavicle and is directed toward the suprasternal notch (Fig. 14.4C).64

A

Subclavian Vein Cannulation Following Trendelenburg positioning of the supine patient, a narrow cloth roll is placed beneath the patient, between the scapulae. The introducer needle enters the skin inferior to the junction of the middle and lateral thirds of the clavicle and is directed toward the suprasternal notch. The needle passes along the inferior surface of the clavicle until it enters the subclavian vein (Fig. 14.5).64

Femoral Vein Cannulation Following flat or slight reverse Trendelenburg positioning of the supine patient, a towel is placed under the hips to slightly raise them, to enhance exposure of the inguinal crease insertion site. The leg is abducted and externally rotated. The arterial pulse is palpated just distal to the inguinal ligament, halfway between the anterior iliac crest and the pubic symphysis. The femoral vein is approximately 5 mm medial to the artery in infants and toddlers and 1 cm in adolescents and adults. The introducer needle enters the skin 1 to 2 cm distal to the inguinal ligament at a 30-degree angle in line with the course of the vein and parallel to the axis of the thigh (Fig. 14.6).64

B

C • Fig. 14.3

​ eldinger technique. (A) Guidewire is placed through the introS ducer needle into the lumen of the vein. (B) Catheter is advanced into the vein lumen along the guidewire. (C) Wings of the catheter are secured to the skin with suture.  

dilator, the catheter is advanced into position over the wire (see Fig. 14.3). The guidewire is removed, leaving the catheter in place. Blood should be easily aspirated from each lumen; then, the lumens should be completely cleared of blood by flushing with sterile heparinized saline to reduce the chance of clot formation. Several systems for securing the catheter are commercially available, but it may also be secured with suture. A large loop of suture is placed in the skin, attached through the wings of the catheter, and tied down. The suture should be taut, preventing catheter movement without causing skin necrosis within the loop of suture. A chlorhexidine-impregnated patch can be applied and a transparent adhesive film placed over the catheter, creating an occlusive dressing.

Internal Jugular Vein Cannulation Multiple approaches can be used to cannulate the internal jugular vein. The patient is placed supine in slight Trendelenburg position. A roll of bed linen is placed under the shoulders to extend the

Use of Ultrasound for Central Venous Line Placement Ultrasonography has been increasingly used to facilitate the placement of CVCs in the PICU. Anatomic variation of the central veins is not uncommon, reported in 7% to 18% of pediatric patients, and can make cannulation more difficult using landmarks alone.65,66 Bedside ultrasonography offers direct visualization of the vessel before and during the intervention. Use of ultrasound reduces insertion-related complications in children67–70 and CVC placement success rates are improved with the use of ultrasound.71 As familiarity with real-time ultrasound guidance is improving, novel techniques, including subclavian access, are successfully being implemented in pediatrics.72 Routine use of real-time ultrasound guidance for CVC placement is recommended and reflects best practice, particularly for internal jugular catheterization (see Chapter 15).

Complications CRBSI is the most common complication related to CVCs (see Chapter 109). In children, the location of the insertion site is not related to infection risk.73 The risk of infection is decreased by the use of a bundle of practices during insertion and ongoing maintenance of the CVC. The insertion bundle includes strict maximal sterile barrier precautions and aseptic technique. Dressing changes



CHAPTER 14 Pediatric Vascular Access and Centeses

101

Carotid artery

Sternocleidomastoid muscle

Sternocleidomastoid muscle

Internal jugular vein

Internal jugular vein

Ipsilateral nipple

Ipsilateral nipple

A

B

Sternocleidomastoid muscle Internal jugular vein External jugular vein

Sternal notch

C • Fig. 14.4



​Approaches to the internal jugular vein. The patient is supine, in slight Trendelenburg position, with the neck extended over a shoulder roll and the head rotated to the contralateral side. (A) Middle approach: The introducer needle enters the skin at a 30-degree angle at the apex of the triangle formed by the heads of the sternocleidomastoid muscle and clavicle, directing it toward the ipsilateral nipple.​ (B) Anterior approach: The carotid pulse is palpated and may be slightly retracted medially. The introducer needle enters along the anterior margin of the sternocleidomastoid, about halfway between the suprasternal notch and mastoid process, while being directed toward the ipsilateral nipple. (C) Posterior approach: The introducer needle enters at the posterior margin of the sternocleidomastoid, just superior to where the external jugular vein crosses, and is directed under its heads toward the suprasternal notch.

with chlorhexidine skin prep, minimizing catheter access, and daily assessment of the need of the catheter are all recommended as a part of CVC maintenance.74 Antimicrobial-impregnated catheters may decrease the risk of catheter-related infection, but more pediatric studies are needed.75 Pneumothorax may result if the lung is punctured during jugular or subclavian vein CVC placement. This complication is less likely with careful patient positioning, attention to anatomic landmarks, and real-time use of ultrasonography as the introducer needle is advanced. Chest radiography should be performed after these approaches are attempted to document absence of pneumothorax and verify CVC position.

Thrombosis may occur in the vessel surrounding the catheter and is associated with malignancies and diabetic ketoacidosis.76,77 Ectopy may ensue when the guidewire or catheter, positioned too deeply, stimulates the right heart. Prompt retraction of the guidewire or catheter typically resolves the ectopic arrhythmia. Bleeding at the skin puncture site from an inadvertent arterial puncture is usually controlled by direct pressure. However, hemorrhage can be difficult to control and may be potentially lifethreatening if there is injury to deeper vascular structures or when coagulopathy is present. Veins and arteries may be perforated far from the intended puncture site by the introducer needle, guidewire, dilator, or catheter. Injury to the femoral or iliac vessels may

102

SECTION II



Pediatric Critical Care: Tools and Procedures

to acute blood loss. Cardiac tamponade can occur when the catheter perforates within the pericardial reflection. Undesirable positioning of a central venous line can be detected radiographically and should be corrected as soon as possible.78,79

Peripherally Inserted Central Venous Catheters

Clavicle

Subclavian vein Sternal notch

• Fig. 14.5

​Approach to the subclavian vein. The patient is supine, in slight Trendelenburg position, with a small roll along the spine between the shoulders. The needle enters the skin at the junction of the lateral and middle thirds of the inferior clavicle and is directed toward the suprasternal notch, passing along the inferior edge of the clavicle.  

Inguinal ligament Femoral nerve Femoral artery Femoral vein

Peripherally inserted CVCs (PICCs) are used with increasing frequency in PICU patients. For infants and smaller children, often only a single-lumen catheter can be placed. Multilumen catheters may be placed in older children. Although PICC lines can often be placed by visualization or palpation of veins in the antecubital fossae, ultrasound is frequently used to place these catheters proximal to the antecubital fossa. Success is more frequent when the catheters are inserted in the basilic vein.80 PICC lines are most often constructed of soft silicone or plastic polymer. The catheter length is measured before catheter insertion, and the catheter is trimmed to the appropriate length. Placement of PICC lines is most commonly performed using a modification of the Seldinger technique. A needle or catheter is inserted into the vein; then, a guidewire is placed, followed by a dilator. A soft peel-away introducer is often inserted next, with the catheter inserted through the introducer sheath. The sheath is peeled away after the catheter is in place. Outside of interventional radiology suites, chest radiography remains the primary method for documenting the location of the catheter tip. While PICC lines are popular and associated with a lower risk of placement-related complications, they are subject to the same complications as CVCs, including catheter-associated infection, frequent thrombosis, perforation, embolization, and fracture.81–84 Recent data in adults demonstrate a very high incidence of catheter-related thrombosis when the catheter exceeds 45% of the vessel diameter, although pediatric data are lacking.85 PICC lines are associated with added thrombosis risk when compared with other types of CVCs in children. The associated thrombotic complications can ultimately limit vascular access options in chronically ill children.

Ultrasound-Assisted Peripheral Venous Access

Right leg

• Fig. 14.6 ​Approach to the femoral vein. The patient is flat and supine, with the thigh slightly abducted and externally rotated. The introducer needle enters the skin 1 to 2 cm distal to the inguinal ligament and 0.5 to 1 cm medial to the pulse of the femoral artery.  

result in pelvic or retroperitoneal bleeding. Lacerations of the jugular, subclavian, or innominate veins or superior vena cava may result in hemothorax. Bleeding complications are more severe in the presence of a coagulopathy or thrombocytopenia. If possible, these should be treated before and/or during central vein access attempts.78,79 A CVC positioned where it applies pressure to the wall of a vessel or the heart increases the risk of perforation, which can lead

Ultrasound-assisted peripheral venous access is increasingly used for challenging venous access situations given the availability of point-of-care ultrasound in critical care settings. Ultrasound facilitates visualization of deeper veins that may not be readily found using inspection and palpation and can be used as a dynamic technique to guide catheter placement in real time. This technique requires operator training and experience—like most procedures, its success improves with increasing operator experience. Studies comparing the success of this technique to conventional peripheral venous access have yielded mixed results.

Venous Cutdown With the widespread use of central venous access and IO access during emergencies, venous cutdown is rarely performed. Venous cutdown is indicated when percutaneous access is not achievable and the need for IV access warrants a more invasive approach. Materials needed depend on the technique used for vein cannulation. As with percutaneous CVC placement, the skin should be prepped and draped, and aseptic technique should be used. A skin incision is made perpendicular to the vein. The tissue surrounding the vein is bluntly dissected to completely expose the vein.



CHAPTER 14 Pediatric Vascular Access and Centeses

103

Supplies and Equipment

Distal ligature Venotomy

Proximal ligature



• Fig. 14.7

​Venous cutdown.

Ligatures are passed around the vein, distal and proximal to the intended site of cannulation. A small venotomy is created; using the ligatures to control the vein, a catheter is directly passed into the lumen of the vein. The distal ligature can be tightened to control bleeding, while the proximal ligature helps secure the catheter (Fig. 14.7). Alternatively, an over-the-needle IV catheter can be directly introduced into the exposed vein without venotomy. Finally, the Seldinger technique can be used, in which an introducer needle and guidewire are inserted into the lumen of the exposed vein, followed by catheter placement over the wire. This approach is particularly useful for femoral venous cutdown. After the catheter is in place, it is secured with suture, and the wound is closed around the catheter. The complications of venous cutdown are similar to those seen in other venous access techniques. There is a risk of bleeding from the open wound, especially in patients with coagulopathy or systemic anticoagulation. The open wound also increases the risk of infection. Injury to adjacent structures, such as arteries and nerves, during incision and blunt dissection is another risk with cutdown.64,86,87

Umbilical Arterial Catheter and Umbilical Venous Catheter Placement Umbilical vein cannulation was first described in 1947 for an exchange transfusion in an infant with severe indirect hyperbilirubinemia.88 Umbilical artery cannulation was later described in 1959 for blood gas sampling.88 Since the early 2000s, umbilical arterial catheter (UAC) and umbilical venous catheter (UVC) placement have become routine procedures in the neonatal intensive unit (NICU).89 The UAC is indicated for frequent blood sampling, continuous measurement of blood pressure, and exchange transfusion.89,90 The UVC is used for administration of fluids, parenteral nutrition, and blood products.89,90 However, the advantages of these lines must be carefully balanced against the potential risks.91,92 Several life-threatening complications have been associated with the use of these catheters.91,92 As many neonates admitted to PICUs are older than a few days of age, this procedure is of limited value for most pediatric intensivists but can be very useful in the first few days of life.

Prepackaged umbilical catheter insertion trays are commercially available and contain different sizes of catheters. The 3.5 Fr and 5 Fr are the most frequently used catheters.89,90 Umbilical catheters are typically made of polyvinylchloride and have a single end hole, as side hole catheters have been linked with higher incidence of thrombosis.93,94 Umbilical catheters are available as single or multiple lumens. A single lumen can be used in either vessel, whereas a double- or triple-lumen catheter is used exclusively in the umbilical vein.95 More than one lumen allows the administration of incompatible fluids. A 3.5 Fr catheter is used for infants weighing ,1500 g, and a 5 Fr catheter is used for larger infants.89,90

Technique The infant is kept in a supine position by using soft restraints of the arms and legs or via a swaddling technique. A catheter is prepared for insertion by connecting to a Luer-Lok stopcock and flushing with saline, with or without heparin. The umbilical stump and surrounding skin are thoroughly cleansed with 2% chlorhexidine or povidone-iodine.89,96 The antiseptic agent is allowed to dry and is then removed with sterile saline. The area is draped, sparing the head and chest to allow for appropriate patient monitoring. A cord tie is applied around the umbilical stump. Using a scalpel, the cord is horizontally cut 1 to 2 cm above the umbilical ring.96 The larger, single thin-walled umbilical vein is typically located at the 12 o’clock position, whereas the two thick-walled generally constricted umbilical arteries are identified at the 5 and 7 o’clock positions.96 A single umbilical artery is sometimes isolated and can be a normal variation.89,96

Umbilical Arterial Cannulation Once the umbilical artery is identified, the iris forceps is used to gently dilate the arterial lumen by first inserting the forceps in the closed position and subsequently allowing both prongs to spring open and dilate the lumen.89,90 The catheter is introduced 0.5 cm in the lumen of the vessel.89,90 Thereafter, the umbilical cord is pulled toward the infant’s head before further advancing the catheter. The direction of the catheter advancement is caudal.97 The catheter enters the umbilical artery, passes through the right internal iliac artery, the right common iliac artery, and, finally, the descending aorta.97 Resistance to catheter advancement is occasionally encountered secondary to vasospasm, at the junction of the umbilical artery and fascial plane, or at the level of the bladder. Gentle pressure can be applied. Sometimes, the catheter can cross the wall of the umbilical artery, creating a false lumen. A double-catheter technique can then be attempted.98 The first misdirected catheter follows a path of least resistance. A second catheter is also used to bypass this pathway and then enters the aorta.98 If this technique fails, the second umbilical artery is cannulated. The patency of the catheter is verified by easy blood aspiration and flushing. The catheter is sutured in place, using a purse string stitch cinched tightly to provide hemostasis and wrapping both ends of the suture around the catheter before tying a square knot. The line is further secured using a tape bridge or other available stabilization device.99 The umbilical tie is loosened and kept in place for any needed hemostasis. A transducer can be attached for continuous blood pressure monitoring while still allowing blood sampling.100

104

SECTION II



Pediatric Critical Care: Tools and Procedures

Two lengths of UAC insertion are described in the literature.101–103 Low or high placement of the catheter is based on the vertebral level at which the catheter tip resides in the aorta.97,101–103 Low placement is defined as the catheter tip caudal to the origins of the renal arteries, whereas a high placement is described as the catheter tip in the descending aorta above the diaphragm and below the left subclavian artery.97,101–103 A Cochrane review evaluated the effects of the position of UACs and concluded that high-placed catheters led to fewer clinical vascular complications.104 Although reference charts based on patient morphometrics are available, there are simple formulas that predict the depth of insertion of arterial catheter.101–103 Shukla’s regression equation based on birth weight (BW) predicts UAC insertion length (cm) as (3 3 BW (kg) 1 9) for infants weighing 1500 g or more, whereas Wright’s formula (4 3 BW [kg] 1 7) results in more accurate UAC placement (cm) for infants weighing less than 1500 g.102,103

Umbilical Venous Cannulation Once the umbilical vein is identified, a catheter is introduced carefully in the lumen. The umbilical vein does not always require routine dilation prior to the introduction of the catheter.89,90,96 The umbilical cord is gently pulled toward the feet during placement to straighten the course of the vein. The direction of the catheter advancement is cephalad. A properly placed UVC resides at the inferior vena cava–right atrial junction via the umbilical vein and the ductus venosus.105 Sometimes, the catheter is misdirected into the portal, splenic, or mesenteric vein. If there is resistance to insertion or poor blood return, inappropriate position of the catheter should be suspected. The double-catheter technique, described in the UAC section, can also be used in UVC placement.105 The patency of the catheter is then verified by adequate blood return and flushing. The line is secured using the same technique described for UACs. The length of inserted catheter is determined by the size of the infant and indication for placement.89,102,105 In emergency situations, the catheter is advanced to a depth where rapid blood return is achieved (2–4 cm in most infants). A long-term UVC resides at the junction of the inferior vena cava and right atrium. The two most commonly used methods to predict accurate depth of UVC are nomograms based on the measurement of the shoulder-umbilicus length and regressions equations based on BW.101,102 Shukla’s formula, ([3 3 BW (kg) 1 9]/2) 1 1, is widely used to estimate the length of UVC insertion (cm).102 Verheij suggested that Shukla’s formula leads to overinsertion of catheters and recommended a revised formula, (3 3 BW [kg] 1 9)/2.106

Proper Placement of Umbilical Arterial and Venous Catheters Anteroposterior and lateral views of a thoracoabdominal radiograph are required to confirm proper placement of the catheters. Low UAC placement correlates with the third and fourth lumbar vertebrae on chest film, whereas a high placement correlates with the sixth and tenth thoracic vertebrae.107 UVC tip should be positioned at or just above the diaphragm or between the eighth to tenth vertebrae. Several studies have questioned the optimal diagnostic approach to determine the correct position of the umbilical catheters.107 Questions were raised on the difficulty of relating anatomic structures to the projection of vertebral bodies on radiographs secondary to the variability of these structures in relation

to bony landmarks. Bedside ultrasonography is suggested as a better modality for verifying the position of the umbilical catheters.107 However, the disadvantage of this technique is the constant need for qualified personnel to perform the study at the time of the catheter’s placement. Owing to this limitation, most centers still rely on radiography to assess catheter position.

Maintenance Infants are typically placed in the supine position or on their sides. A dressing should not be applied to the umbilicus so that the catheter insertion site can be easily inspected.89,90,96 The UVC is maintained as part of a closed system to prevent air embolism. A continuous infusion is needed to keep the lumen of the UAC clear, and the catheter is flushed after blood draws to minimize clot formation. Continuous fluid infusion containing heparin is needed in the arterial line.108 The composition of the heparincontaining fluid varies by institution and is influenced by gestational age and electrolyte status. A typical infusion includes 38 to 77 mEq/L sodium chloride or sodium acetate or an isotonic amino acid solution with heparin 1 U/mL.97,109

Removal Umbilical catheters are removed one at a time. Each catheter is pulled to approximately 5 cm, then the catheter is slowly withdrawn in increments of 1 cm/min.97 This process is especially important during the removal of UACs because it allows the artery to spasm and provide hemostasis. If bleeding occurs, pressure is applied by elevating and pinching the skin just above the cord for venous bleeding or below the cord for arterial bleeding. A hemostat can also be used to pinch the lumen of the vessel for persistent bleeding.89,97

Complications Umbilical venous and arterial cannulations are associated with potential complications. These complications are related to placement and malposition of the catheters or prolonged catheter placement in the umbilical vessel.

Umbilical Arterial Cannulation Several complications are linked to UAC placement and catheter tip position. Trauma to the vessel, leading to hemorrhage, can occur during placement. Vasospasm of the umbilical artery with resulting blanching or cyanosis of the toes, feet, or buttocks has been described.110,111 Warming of the unaffected limb may improve perfusion of the other extremity. Otherwise, catheter removal is warranted to prevent ischemic complication.90,110,111 Other complications include peritoneal perforation, bladder injury, catheter fracture, intravascular knots of catheters, or catheterization of the urachus, resulting in urinary ascites.90,110,111 Additional complications can develop with prolonged indwelling of the UAC. McAdams and colleagues investigated the effects of UAC placement in an animal model and concluded that thrombus formation was detected in 80% of aortic sections.112 In addition, the incidence of developing aortic thrombus increases proportionally to the duration of UAC placement and has been reported as 16% within 1 day, 32% within 7 days, and 80% within 21 days of UAC placement. The presentation of emboli ranges from asymptomatic to limb-threatening ischemia or mesenteric artery occlusion with necrotizing enterocolitis or renal



CHAPTER 14 Pediatric Vascular Access and Centeses

artery occlusion with renal failure and hypertension.90,110–112 Furthermore, once the intima of the vessel has been traumatized, the vessel becomes susceptible to infection. In most instances, the microorganisms are coagulase-negative staphylococci. These pathogens produce a biofilm that preferentially adheres to irregular catheter surfaces.113 Catheter-related infection may also cause aortic aneurysm.92 The Centers for Disease Control and Prevention (CDC) reaffirmed in 2011 that UACs should be removed as soon as possible and should not be kept longer than 5 days— sooner if signs of vascular insufficiency occur.113

Umbilical Venous Cannulation A common complication of UVC placement is catheter tip malposition. A low-positioned UVC within the confluence of the portal circulation may precipitate hepatic injury.91 Clinical features of liver complications vary and can be asymptomatic or present as abdominal distension with hepatomegaly, hypotension, worsening respiratory status, or portal thrombosis with chronic portal hypertension. A catheter tip in the right atrium can also result in pericardial effusion and tamponade.114 Complications related to prolonged UVC cannulation include sepsis and thrombosis.115 Multiple interventions are recommended to prevent central line–associated bloodstream infection and include limiting central line access for injecting medications, enforcing hub disinfection before accessing the central line, and replacing UVCs as soon as possible with PICCs.113 The CDC has recommended removal of UVCs as soon as possible when no longer needed, but this could be extended up to 14 days if managed aseptically.113 Interestingly, Butler-O’Hara and colleagues completed a randomized controlled trial that showed similar infection rates with UVCs left in place up to 28 days compared with UVCs replaced by PICCs after 7 to 10 days.116 However, the same authors later published a quality improvement project revealing a greater risk of infection with long-term compared with short-term UVC followed by PICC placement.117 The authors concluded that the substantial decrease in PICC infection rates in their unit altered the risk-benefit ratios between the two strategies (short and long term) of UVC use.

Summary Umbilical lines are commonly used in the care of severely ill neonates. The use of UACs for blood collection and blood pressure monitoring and the use of UVCs for nutrition or medications have become commonplace in NICUs and can also be used in the PICU. Although there are several benefits to their use, umbilical catheters are associated with potential problems. An awareness of the possible complications is important to minimize serious consequences and provide timely interventions.

Pulmonary Artery Catheterization Pulmonary artery catheter (PAC) monitoring was introduced into practice in 1970 by Swan and Ganz (see Chapters 26, 27, and 30). However, because of the invasiveness of the procedure and lack of a proven survival benefit for patient management, other less invasive surrogate techniques have significantly decreased the use of the PAC.118–121 Placement of the catheter can be performed at the bedside, but skill and experience in the placement, management, and data acquisition are required to avoid complications and for proper interpretation of the hemodynamic data. Most catheters are balloon

105

tipped and flow directed. They are able to measure right atrial, pulmonary artery, and pulmonary capillary wedge pressures as well as determine cardiac output and oxygen saturations in the right heart chambers. Single-lumen catheters may be placed directly into the pulmonary artery at the time of cardiac surgery. Both techniques are used in pediatric patients, but the singlelumen catheter is most frequently employed because of the frequency of pulmonary hypertension complicating the postoperative management of pediatric cardiac patients. The flowdirected, balloon-tipped catheter is usually placed in the ICU to assist in determining the etiology of shock, pulmonary edema, and pulmonary hypertension, as well as to help guide fluid and vasoactive-inotropic therapy over time. The PAC should not be used for the routine care of ICU patients, but it may be useful in heart failure patients with persistent symptoms despite standard measures, in patients undergoing heart transplantation evaluation, and for patients with pulmonary hypertension.120,121 Pulmonary hypertension may either be primary or secondary, the latter including pulmonary hypertension in postoperative congenital cardiac patients. These patients are prone to wide swings in pulmonary artery (PA) pressures associated with variations in oxygenation, ventilation, and sedation level. When inhaled nitric oxide is used to manage postoperative pulmonary hypertension, direct measurement of PA pressure helps guide titration of therapy. In patients with severe respiratory failure requiring high positive airway pressure with associated hemodynamic compromise, PACs may facilitate diagnosis of low cardiac output and direct therapy. When Do2 in such patients is significantly limited because of hypoxemia, low cardiac output, or both, measurement of Do2 using variables derived from information provided by the catheter may be useful. In children with severe shock unresponsive to fluid resuscitation and requiring vasoactive-inotropic infusions, the PAC may better define the hemodynamic profile, thus directing more specific therapy. Significant controversy exists regarding the benefits and potential harms caused by this invasive form of hemodynamic monitoring.120–122 An older multicenter observational study reported increased mortality with PACs. Subsequently, several randomized clinical trials failed to demonstrate a benefit to PAC-guided therapy. Some studies reported an association with increased morbidity and mortality,119 whereas others did not find differences with or without PACs.122 No studies in children have demonstrated better outcomes with the use of the PAC monitoring. Multiple barriers exist to PAC use, including patient risk with placement, the ability to measure similar variables via less invasive measures, increased cost, inaccurate measurement leading to misuse of PAC-derived variables, and incorrect interpretation and clinical application. Additionally, with the decreased use of this technology, the skill required to maintain competency in placement and interpretation of the data provided presents a significant challenge to many institutions.

Contraindications There are no specific contraindications to placement of a PAC, but there are several relative contraindications, including bleeding diathesis, which increases the risk for percutaneous access, and severe tricuspid or pulmonary insufficiency, which can make bedside catheter placement prohibitively difficult. Unstable cardiac arrhythmias that are easily triggered by catheter manipulation are also a relative contraindication. Catheter placement for measurement of cardiac output using the thermodilution technique is

106

SECTION II



Pediatric Critical Care: Tools and Procedures

contraindicated in the presence of intracardiac shunts, tricuspid insufficiency, or pulmonary insufficiency, as the thermodilution measurement will be inaccurate.

Procedure and Equipment Balloon-tipped, flow-directed catheters are available with two diameters, 5 Fr and 7 Fr. The 5 Fr diameter catheter is most appropriate for patients weighing less than 15 kg; the 7 Fr diameter catheter is best for patients weighing more than 15 kg. Some PACs employ fiberoptic spectrophotometry for continuous measurement of mixed venous oxygen saturation. Single-lumen PACs are most commonly placed in the operating room at the time of heart surgery. The standard PAC is 1 m long. The PAC is equipped with proximal and distal ports facilitating measurement of intravascular pressures, infusion of vasoactive agents, fluids, and blood sampling. The distance between the proximal and distal lumen ports varies depending on the catheter: standards are 10, 15, 20, and 30 cm. Choosing the catheter with the correct lumen distances is crucial in order to monitor the appropriate pressure. At the tip are a thermistor used to calculate cardiac output and a balloon that may be inflated and deflated as necessary. Some catheters have an additional right ventricular port for temporary pacemaker insertion, and some have the fiberoptic oxygen saturation sensor for continuous measurement of mixed venous oxygen saturation. Other necessary pieces of equipment are a monitor with cardiac output capability or a computer to determine cardiac output using thermodilution and compatible pressure transducers. Carbon dioxide is used in some centers to inflate the balloon to minimize the risk of air embolization, although room air is most commonly used. The catheters are placed through a percutaneous introducer sheath, which is placed with the same technique as described for CVCs. Before placement, the catheter should be flushed and filled with fluid through which intravascular pressures are transmitted to a transducer. The equipment is then zeroed to atmospheric pressure at the level of the patient’s left atrium (midaxillary line, fourth intercostal space) and calibrated. If all air bubbles are not removed from the tubing, they may result in damping of the waveform tracing and, consequently, erroneously low systolic pressure. Thrombus at the tip of the catheter may also alter the waveform (see also Chapter 26). The insertion site is prepared in sterile fashion with chlorhexidine solution and draped with sterile towels. It is important to drape a wide area with sterile sheets (full field barrier drape) in order to avoid exposure of the catheter, because of the length of the PAC. The PAC is inserted through the introducer sheath. A sterile sleeve is placed on the end of the sheath, and the catheter is passed through the sleeve, then through the introducer diaphragm and into the sheath. Anatomically, the preferred sites of insertion are the right internal jugular, left subclavian, right subclavian, and left internal jugular veins. Usually, the placement of the catheter is guided by pressure waveform monitoring, but fluoroscopic visualization will occasionally be needed, particularly if the PAC is placed from a femoral site. Once the catheter tip enters the venous circulation, the balloon is inflated with air. From this point, the catheter should be advanced with the balloon inflated to prevent damage to the myocardium, cardiac valves, or pulmonary artery branches. If the catheter is withdrawn, the balloon must first be deflated to avoid valvular injury.

The catheter is advanced to the right atrium (RA), then across the tricuspid valve into the right ventricle (RV), and across the pulmonary valve into the PA. As the catheter continues to float with the balloon inflated, it will wedge in a branch PA, occluding the blood flow. The pulmonary artery occlusion pressure (PAOP), or pulmonary artery wedge pressure (PAWP), will be recorded from the distal lumen. If the balloon is deflated, a PA pressure tracing will be recorded. If the waveforms are not obtained, the balloon should be deflated, and the catheter pulled back to the RA before attempting placement again. After insertion, a chest radiograph is obtained to ensure proper catheter placement and rule out pneumothorax. The catheter tip should be visualized within West zone III of the lung ideally (see Chapters 26, 42, and 43). The pressure waveforms are characteristic; when the catheter is advanced to the RA, the atrial trace has a respiratory variation that helps to confirm that the catheter is in the thorax. Once in the atrium, the balloon is inflated and advanced to the RV, where the trace is characterized by a rapid upstroke in early systole with an equally rapid downstroke at the end of systole and diastolic pressure near zero. Turning the catheter with a clockwise motion usually helps in advancing the PAC. The catheter is advanced to the PA. The PA trace has the same peak systolic pressure of the RV, but as systole ends, the trace shows a slower fall that continues through diastole, because the diastolic pressure in the PA is higher than the RV diastolic pressure. Once in the PA, the catheter is advanced slightly until a pulmonary wedge trace is seen. This trace is similar to the RA trace, although usually with a higher pressure. PAWP is obtained when the balloon is inflated and the catheter floats into the wedge position. Because the catheter floats to an area of greatest blood flow in the lung, it most likely will be in an area consistent with West zone III, where arterial pressure is higher than both venous and alveolar pressures. Measurement of PAWP is best done at end expiration to minimize the effect of changes in pleural pressure. Once the wedge is measured and the balloon is deflated, the PA trace should return. If the trace does not change, the catheter should be retracted until the PA trace is seen. The catheter should not be left inflated in the wedge position because of the risk of pulmonary infarction. The catheter is appropriately positioned when the PA pressure trace is present when the balloon is not inflated and the pulmonary capillary wedge trace is present when the balloon is inflated. Once it has been confirmed that the PAC is in good position by pressure trace and radiography, the catheter should be secured in the sleeve and taped to the patient. PAC information acquisition and interpretation is detailed on ExpertConsult.com and in Chapter 26.

Maintenance Care of the PAC is similar to that for any CVC. The catheter and sheath should be dressed sterilely at all times, and the dressing changed according to protocol. The catheter is housed in a sterile sleeve that allows for aseptic technique if further manipulation is necessary. Pressure transduction of the distal (PA) and proximal (RA) ports and continuous electrocardiographic (ECG) monitoring are mandatory. This setup continually confirms proper placement of the catheter. Whenever the balloon is inflated to determine PAWP, the balloon is allowed to deflate passively by opening the balloon port and removing the syringe. This step helps prevent balloon rupture. Balloon rupture should be suspected if blood is obtained when aspirating the balloon port. In this situation,

106.e1

Pulmonary Artery Catheter Information Acquisition and Interpretation Much hemodynamic and Do2 information can be obtained from the Swan-Ganz–type PAC. Multiple hemodynamic pressures can be obtained, including RA, RV PA, and PAWP. RA pressure is useful for determining preload of the RV. Pulmonary artery pressure is useful for determining the presence of pulmonary hypertension both at baseline and with manipulation of oxygenation, ventilation, ventilator pressures, inhaled nitric oxide, and other procedures. PAWP reflects left ventricular preload. In most patients with normal cardiac function and anatomy, right atrial or central venous pressure adequately reflects LV preload as well. However, in the presence of certain congenital heart defects, with significant ventricular dysfunction, or with high mechanical ventilatory pressures, a significant discrepancy may exist between right and left ventricular preload. In such circumstances, measurement of PAWP may be useful for guiding fluid and inotropic therapy. Mixed venous oxygen saturation (Svo2) can be determined directly and continuously with a catheter containing the fiberoptic oximeter. In the absence of the oximeter, intermittent blood sampling from the distal port when in place in the PA allows for Svo2 measurement. The thermistor at the tip of the catheter allows for measurement of cardiac output using the thermodilution method. This method uses the Fick principle, based on the law of conservation of thermal energy. A specific amount of known temperature fluid is injected in the proximal port (upstream), and the temperature change downstream (at the thermistor) is recorded. The change in temperature over time allows for measurement of blood flow—in this case, cardiac output. According to Jansen, this measure of cardiac output is accurate if the following conditions are met: (1) no loss of cold occurs between the injection site and the thermistor, (2) mixing of the cold injectate (indicator using Fick terms) and the blood is

complete, and (3) the temperature change caused by the injection of cold fluid is sufficient to be detected by the thermistor. To perform thermodilution cardiac output measurements, the catheter must be connected to the thermodilution computer, which is either freestanding or part of the cardiac monitor. A specific volume of injectate, either room temperature or iced, is injected rapidly into the proximal port of the catheter. The temperature difference over time that is detected at the thermistor is recorded as a curve. The computer then integrates the area under the curve, which is inversely proportional to the cardiac output. The cardiac output is calculated and projected. For children, this number should be divided by their body surface area in square meters, deriving the cardiac index. The injectate can be either iced or room temperature. Although a greater signal-to-noise measurement is obtained, the disadvantages of iced injectate include risk of hypothermia in pediatric patients requiring frequent cardiac output measurements and the poor accuracy of the first injection because of warmer fluid in the catheter. In conditions of high or low cardiac output, less variance occurs with iced injectate compared with room-temperature injectate. However, for convenience and the safety of pediatric patients, room-temperature injectate is generally recommended. Usually, three to five injections yield adequate results. Some error can be introduced by faulty technique. Injecting variable volumes or injecting with variable rates can result in inaccurate measures. Multiple injections and averaging of the results can overcome these problems. The presence of tricuspid or pulmonary insufficiency can lead to an overestimation of cardiac output. Echocardiography may be necessary to rule out the presence of valvular insufficiency. Intracardiac shunts, such as a ventricular septal defect, result in false values for cardiac output. Mechanical ventilation has been shown to alter stroke volume, which can result in a variability of cardiac output measurements. Therefore, one should perform the injection at the same time in the ventilator cycle to standardize the cardiac output measurements.



CHAPTER 14 Pediatric Vascular Access and Centeses

remove and then replace the catheter if it is still clinically indicated. As noted earlier, arrhythmias can occur, particularly if the catheter becomes dislodged. A chest radiograph should be taken daily to assess for catheter position.

Complications PA catheterization is a significantly invasive procedure. Complications can occur during the Seldinger procedure to access the vein, during the passage of the PAC (across two heart valves), or during catheter use. Bleeding, infection, and pneumothorax may occur during venous access. Arrhythmias can be encountered during placement of the PAC or due to dislodgment of the catheter. Arrhythmias include supraventricular tachycardia while the PAC tip is in the RA to premature ventricular beats or even ventricular tachycardia while the PAC tip is in the RV. Usually, the arrhythmias cease when the PAC tip reaches the pulmonary artery. Lidocaine, amiodarone, and defibrillation may be needed on occasion for ventricular arrhythmias; thus, these drugs should be readily available. Once the catheter is in place, pulmonary infarction or hemorrhage is a risk. Rupture of the distal PA, endothelial damage, and valvular damage have been reported, as well as knotting of the catheter requiring fluoroscopic retrieval. The PAC should be removed as soon as possible to minimize the risk of complications.

Summary Since the introduction of the PAC, controversy has surrounded the technology regarding the benefits and potential harms caused by this invasive form of hemodynamic monitoring. In adult clinical trials, the usefulness of the PAC has been challenged because no benefit in patient outcome has been observed, and some retrospective studies have described worse outcomes. Accurate acquisition and interpretation of PAC data are paramount for making appropriate therapeutic decisions.

Thoracentesis Thoracentesis is a procedure used to remove abnormal accumulations of nonphysiologic substances from within the potential space of the pleura, including fluid (hydrothorax), blood (hemothorax), air (pneumothorax), or pus (empyema). Pleural effusions in children are most commonly the result of an infectious process (50%–70% are parapneumonic effusions), with congestive heart failure and malignancy being less common causes.124–126 Volume resuscitation with third spacing after shock is also a cause of pleural effusions in the PICU. There are many other pathologic causes of pleural effusions in children (eBox 14.2).

Indications Thoracentesis may be used diagnostically for new fluid accumulations or therapeutically to relieve cardiopulmonary compromise resulting from large accumulations of fluids or air. New and particularly persistent pleural fluid collections should be investigated with diagnostic thoracentesis. If ongoing evacuation is required, tube thoracostomy should be considered (discussed later in this chapter). Ultrasound is useful to identify fluid accumulations when there is complete opacification of the hemithorax on chest radiograph and helps characterize fluid consistency to determine whether it is a complicated, loculated, or simple, free-flowing

107

effusion. Ultrasound also helps identify optimal locations for successful aspiration and has decreased complication rates.127–131 Ultrasound is almost universally used by interventional radiologists for thoracentesis and tube thoracostomy and is increasingly used by pediatric intensivists. Adult data have shown a decrease in complications with bedside ultrasound for thoracentesis, but similar data for children are lacking.

Contraindications Thoracentesis has few to no absolute contraindications. Relative contraindications include insufficient amount of pleural fluid and severe bleeding diatheses. Uncorrected coagulopathy and thrombocytopenia may predispose to bleeding complications; however, thoracentesis can generally be accomplished in this setting using a small needle and careful technique. Skin infections or wounds at the insertion site can lead to introduction of new infection into the pleural space and thus should be avoided. Presence of positivepressure ventilation may increase the risk of pneumothorax. However, the risk is low and should not serve as a contraindication to a medically necessary procedure.127,129 An uncooperative patient can lead to damage of the underlying vascular structures and lung parenchyma. This risk can be mitigated by appropriate sedation and analgesia.

Preparation Sedation and analgesia are frequently required to safely perform thoracentesis in pediatric patients. Topical anesthetic agents reduce the discomfort associated with infiltration of local anesthetics and should be placed on the predetermined insertion site at least 30 minutes prior to the procedure (depending on the topical agent).

Technique If thoracentesis is being performed for evacuation of a pneumothorax, the patient should be placed in the supine position and needle aspiration should be performed perpendicularly to the chest wall at the second intercostal space (along the superior border of the third rib) in the midclavicular line. For removal of pleural fluid, the patient should be placed in the upright, seated position, while infants and young children may be held in burping position by an assistant. Mechanically ventilated patients should be in partial decubitus position with the fluid-containing side down, in a dependent position. The usual site for fluid aspiration is the seventh intercostal space in the posterior axillary line (near the tip of the scapula). However, realtime bedside ultrasonography should be used to identify pleural fluid, ideal puncture site, and planned trajectory for the needle path. Once determined, the patient should be maintained in the same position, and the site should be marked with a skin indentation from a needle cap or sterile site marker. Aseptic technique should be observed throughout the procedure. The site is prepped with 2% chlorhexidine or 10% povidone-iodine and draped with sterile towels. The skin entry site is then generously infiltrated with a local anesthetic using a 27- to 30-gauge needle to create a wheal of fluid just under the skin. The needle is then advanced through this wheal, perpendicular to the skin to infiltrate the underlying subcutaneous tissues, superior portion of the rib, and periosteum, always aspirating prior to instilling the anesthetic. Care must be taken not to exceed maximal drug dosages for weight to

107.e1

The hemodynamic data obtained or calculated with the PAC should be interpreted to make therapeutic decisions. There are not isolated “good” or “bad” cardiac output values, but appropriate cardiac output is that which permits an adequate Do2. As a global index of adequacy between consumption and Do2, Svo2 is the target of choice for therapeutic decisions. Svo2 should be kept

above a threshold value between 65% and 70%, and all other PAC parameters should be used to choose how to maintain Svo2 above this value. This Svo2 goal can be achieved by fluid administration, blood transfusion, increasing or decreasing inotropic support, or vasopressors.118,123

• eBOX 14.2 Causes of Pleural Effusion in Children Infectious (Exudates)

Collagen Vascular Disease

• • • • •

• • • • •

Bacterial Viral Fungal Mycobacterial (tuberculous and nontuberculous) Parasitic

Rheumatoid pleurisy Lupus pleuritis Churg-Strauss syndrome Sjögren syndrome Granulomatosis with polyangiitis (formerly Wegener granulomatosis)

Cardiovascular (Transudates)

Neoplastic

• • • •

• • • • •

Congestive heart failure Constrictive pericarditis Postcardiac surgery Superior vena cava obstruction (SVC syndrome)

Pulmonary • • • • • •

Pulmonary infarction Atelectasis Pulmonary embolism Pulmonary sequestration Asbestosis Trapped lung

Intraabdominal Disease • • • • • • • • •

Post–abdominal surgery Pancreatitis Hepatitis Peritonitis Subdiaphragmatic abscess Hepatic abscess Splenic abscess Meigs syndrome Cirrhosis with ascites

Iatrogenic • Drug-induced pleuritis • Enteral feeding tube misplacement • Extravascular central venous catheter placement

Lymphoma/leukemia Mesothelioma Chest wall tumors Carcinomas Neuroblastoma

Renal • • • •

Uremia Urinary tract obstruction Nephrotic syndrome Peritoneal dialysis

Miscellaneous • • • • • • • • • • • •

Esophageal rupture Traumatic hemothorax Chylothorax (post–thoracic surgery or congenital) Hydrops fetalis Lymphedema Lymphangiectasia Hypoalbuminemia Hypothyroidism with myxedema Sarcoidosis Post–radiation therapy Immunoblastic lymphadenopathy Familial Mediterranean fever

108

SECTION II



Pediatric Critical Care: Tools and Procedures

avoid iatrogenic complications depending on the local anesthetic used (e.g., maximum of 4 mg/kg of 1% lidocaine without epinephrine or 7 mg/kg lidocaine with epinephrine). The needle is then advanced over the superior border of the rib while gently aspirating until the pleural space is reached. An over-the-needle catheter of sufficient length can be used for aspiration of fluid with a syringe. If infection or viscous exudate is suspected, a larger catheter (16- to 18-gauge) may be required. Aspiration is continued until a sufficient quantity of fluid for planned diagnostic studies is obtained. A three-way stopcock with attached tubing may be placed on the catheter to facilitate this process. If fluid or air is being removed for relief of cardiopulmonary compromise, aspiration is continued until symptoms improve. The catheter is subsequently removed and a sterile dressing is applied over the entry site.

Complications Risk of complications from thoracentesis is low.128 The most common complication of thoracentesis is pneumothorax (6% in meta-analyses).127–129,131 Use of ultrasound decreases risk of this complication and thus should be used whenever possible.130 A platelet count of greater than 50,000/mL and near-normal coagulation studies (international normalized ratio ,2) are ideal, but the procedure can be safely performed with careful technique and avoidance of the neurovascular bundle located on the inferior border of the rib. Additionally, recent literature calls into question the need for correcting coagulopathy and thrombocytopenia prior to the procedure given the low risk of hemorrhagic complications.128,130,132,133 Still, many providers will attempt to correct the hematologic abnormalities before or during the procedure. Soft-tissue infections can be avoided with use of proper sterile technique. Insertion of the needle through existing skin infections or wounds should be absolutely avoided. Reexpansion pulmonary edema (REPE) has long been reported in adult patients with removal of large volumes (.1500 mL) of fluid or air and usually is apparent in the first 2 hours following thoracentesis but can occur up to 48 hours later.134,135 More recent literature reports the occurrence of REPE in children.134–138 Risk factors may include chronic lung collapse (.72 hours), evacuation of a large amount of pleural fluid or air (usually .1500 mL in adults, .20 mL/kg in children), brisk lung reexpansion, and excessively negative pleural pressures (less than 220 mm Hg). While rare, this complication is associated with high risk for morbidity and mortality (reported up to 20%); thus strategies to prevent REPE should be used.

Interpretation Analysis of pleural fluid is separated into two basic diagnostic categories: exudates and transudates. Transudates arise from imbalances of hydrostatic or oncotic pressures, such as seen in congestive heart failure or nephrotic syndrome. Exudates can be caused by a variety of mechanisms, most commonly from pleural and lung inflammation or impaired lymphatic drainage. The criteria used to distinguish between the two have evolved but are rooted in Light’s Criteria Rule139 (Box 14.3). The updated combination of two or more of these criteria increase the diagnostic sensitivity for the rule. More recent diagnostic rules, including the two-test rule and three-test rule, include pleural fluid cholesterol level greater than 45 mg/dL and do not require concomitant serum levels to be obtained.140

• BOX 14.3 Light’s Criteria Rule for Diagnosing

Exudative Effusions If at least one of the following criteria is present, effusion is defined as exudate: • Pleural to serum protein ratio .0.5 • Pleural to serum LDH ratio .0.6 • Pleural fluid LDH more than twice the upper limit of normal serum LDH value LDH, Lactate dehydrogenase.

Additional pleural fluid studies should be sent to aid in diagnosis, especially fluid for culture, cell count with differential, and cytology. Low pleural glucose (,60 mg/dL) and pleural pH (,7.3) with very elevated nucleated cell counts (.50,000/mL) are highly suggestive of empyema. Elevated pleural triglyceride levels (.110 mg/dL) and lymphocyte predominance suggest chylothorax, while triglycerides ,50 mg/dL effectively rule it out. Elevated amylase suggests pancreatitis or esophageal rupture. Advances in polymerase chain reaction (PCR) technology allow for rapid and accurate diagnosis of viruses and bacteria in pleural fluid.141

Summary Thoracentesis, when performed by skilled providers, can be a safe and useful diagnostic procedure for pediatric pleural effusions. It can also be a useful technique to aid in resolving cardiopulmonary compromise resulting from significant thoracic air or fluid accumulation.

Tube Thoracostomy Tube thoracostomy (commonly known as chest tube) placement is a common procedure and may be required for a variety of reasons in critically ill pediatric patients. Pneumothoraces can develop spontaneously, as a result of acute lung injury, or as a sequela of procedures or surgery. Large parapneumonic effusions and empyemas resulting from infectious pulmonary processes are increasing in frequency and often require chest tube placement. Hemothoraces or hemopneumothoraces from trauma frequently require evacuation and continuous drainage. Chylothoraces in postoperative cardiac patients may require tube placement and drainage. There are no absolute guidelines defining the size or type of effusion necessitating continuous drainage; rather, close assessment of patients’ symptoms and clinical status are paramount in decision-making.142 The technique used for placement depends on the nature of the material to be removed (e.g., viscous, thin, transudative, or exudative).

Contraindications As with thoracentesis, tube thoracostomy has few to no absolute contraindications. The relative risks are similar to those discussed earlier regarding thoracentesis, which must be weighed against the potential benefits of the procedure. When emergent or urgent intervention is required, tube thoracostomy should not be delayed.

Supplies and Equipment Supplies required vary depending on the type of thoracostomy tube to be placed. Tube type should be selected based on the predicted



CHAPTER 14 Pediatric Vascular Access and Centeses

etiology of the pathologic condition. Traditional or larger-bore “surgical” thoracostomy tubes may be required for more viscous fluids found in hemothoraces or complicated loculated empyemas.19 Small-bore (,14 Fr) catheters, placed using modified Seldinger technique, are ideal for simple effusions and pneumothoraces. However, recent evidence and guidelines suggest superiority of these smaller catheters even for loculated empyemas with the concomitant use of chemical debridement with fibrinolytics.143 Many institutions have specially designed chest tube trays with sterile instruments; commercially assembled kits are also available. Typical requirements include sterile gauze, sterile towels for draping, syringes and needles for local anesthesia and aspiration, scalpel, curved hemostat or Kelly clamps of various sizes, needle driver, suture, and scissors. Other materials required are an appropriately sized chest tube, chlorhexidine or povidone-iodine solution, sterile gloves, a drainage apparatus (e.g., Pleur-Evac, Teleflex), local anesthetic, and occlusive sterile dressing.

Technique As mentioned earlier, the technique for placement depends on the nature of material expected to be removed from the pleural space and the type of tube chosen. The majority of effusions and pneumothoraces requiring continuous drainage in the pediatric critical care setting can be evacuated with the placement of a small-caliber tube (5 Fr to 9 Fr) via a modified Seldinger technique.144 Placement of small-bore pigtail catheters are less painful than traditional tube thoracostomy; successful treatment of empyema with chemical debridement using fibrinolytic agents (e.g., alteplase or streptokinase) is well described and recommended as first-line therapy.142–146 Bedside ultrasound will facilitate fluid location and enhances the chance of procedural success. Tube thoracostomy is quite painful. Generous infiltration of the skin, subcutaneous tissue, intercostal muscles, underlying rib, and periosteum with local anesthetics can decrease sedation and analgesia requirements. Care should be taken to avoid exceeding maximal local anesthetic dosages for weight. Following administration of sedation and analgesia, the patient is placed in the supine position. Throughout the procedure, aseptic technique should be observed. The overlying skin is prepped with chlorhexidine or povidone-iodine and draped in sterile fashion. A needle attached to a syringe (5 or 10 mL) is inserted in the fourth or fifth intercostal space in the midaxillary line. Continuous aspiration is applied while the needle is advanced until fluid or air is obtained. A guidewire is then placed into the pleural space through the finder needle. A small skin incision with scalpel blade is made and the overlying skin and subcutaneous tissues are dilated with a skin dilator. A small-bore pigtail catheter with multiple side holes is then placed over the wire and advanced into the pleural space. The guidewire is removed and the catheter is attached to a standard chest tube drainage system. The tube is anchored to the skin with suture or a commercially available suture-less skinanchoring device. A variation of this technique allows for placement of largercaliber tubes via commercially available tube-over-obturator systems (e.g., Thal-Quick, Cook Medical). After placement of the guidewire as described earlier, progressively larger skin dilators are used to facilitate placement of a larger-bore chest tube. Caution should be used with placement of these devices in diseases of poor lung compliance or pulmonary hyperinflation, as these conditions may predispose to intraparenchymal tube placement and the development of bronchopleural fistulas.147

109

Traditional techniques for larger-caliber tube thoracostomy may be required for drainage of highly viscous fluids, including cases of hemothorax or empyemas that fail small-caliber chest tube and chemical debridement with fibrinolytics.143 While observing aseptic technique, the skin is prepped with chlorhexidine or povidone-iodine and draped in sterile fashion, as described earlier. A skin incision large enough to allow placement of the chosen tube is made with the scalpel parallel to the axis of the ribs in the fourth or fifth intercostal space in the midaxillary line. A curved hemostat or Kelly clamp is used to bluntly dissect the underlying subcutaneous tissue until the superior border of the rib is reached. The clamp is then used to push through and dilate the intercostal muscle and pleura above the superior border of the rib. In a larger child, the index finger can be used to further dilate the subcutaneous tissues and intercostal muscles. Any intrapleural adhesions can also be manually broken up. The end of the chest tube is then attached to a clamp and inserted into the pleural space. The clamp is opened and the tube advanced to the proper depth, ensuring that all side holes are within the chest cavity. The tube should be advanced anteriorly for pneumothoraces and posteriorly for effusions. The tube is then attached to a pleural drainage system. Many techniques for securing chest tubes have been described, but the most important aspect of the choice of method is that the operator removing the tube knows how the tube was anchored. Some operators prefer a horizontal mattress suture on both sides of the tube, while others prefer a purse-string suture that can be pulled together after the tube is removed. If the latter technique is used, no knot is placed at the skin level, but extra suture is wrapped around the body of the tube and then tied to the tube itself. Upon chest tube removal, the extra suture serves as skin suture for wound closure.

Maintenance Following successful chest tube placement, a chest radiograph is obtained to verify proper position and to evaluate for resolution of pleural fluid or pneumothorax. Tubes should be evaluated for continued air leak by checking for air bubbles in the leak chamber of the drainage apparatus. Persistent air leak with no evidence of pneumothorax on chest radiograph suggests the development of a bronchopleural fistula or airway injury. Alternately, it could indicate potential air entrainment at a loose tubing connector site. Some authorities suggest intermittent external negative pressure to “strip” the tube to maintain patency, although little evidence supports this practice.25 Instillation of fibrinolytics such as alteplase restores patency of tubes clogged by proteinaceous material.146 Timing of chest tube removal depends on the indication for placement. Tubes placed for pneumothoraces may be removed following resolution of the air leak. Our practice is to place the tube to water seal, without suction, for at least several hours and obtain a chest radiograph prior to removal. Literature supports this practice.146,148–150 Tubes placed for drainage of pleural fluid may be safely removed once drainage has decreased to 2 to 3 mL/ kg per day. Removal of large tubes may be painful and requires analgesia or sedation, especially in small children. Tubes can be safely removed during the inspiratory or expiratory phase.151

Complications Thoracostomy tube placement has numerous potential complications, and rates of complication have been estimated as high as 30%. The list of published complications is too extensive for this

110

SECTION II



Pediatric Critical Care: Tools and Procedures

chapter and includes those discussed previously in the Thoracentesis section.152 Malposition is the most common complication of chest tube placement. Any structure within the thorax may be inadvertently penetrated with the use of excessive force. Placement in the lung parenchyma is relatively common, potentially leading to bronchopleural fistulas.153 Vascular injury may occur with high placement, intraabdominal organ injury with low placement, and left-sided placement can lead to thoracic duct injury and development of chylothorax. Deep placement may lead to mediastinal perforation.154 Computed tomography is required to evaluate the exact placement of chest tubes in cases in which inadvertent misplacement or complications are suspected.

Contraindications

Summary

Procedure

Tube thoracostomy is a common procedure in pediatric critical care medicine. With attention to detail in the hands of a skilled provider, this procedure may be accomplished safely and with minimal risk. Small-caliber tubes should be used when possible; chemical debridement with fibrinolytic agents may be used to restore the patency of clogged tubes and are of therapeutic benefit in cases of empyema.

Drainage of a pericardial effusion can be performed either by simple needle aspiration or by insertion of a drainage catheter. If the procedure is for diagnosis only and the effusion is small, then needle drainage is adequate. However, if tamponade is present, the effusion is sizable, or the effusion likely will continue to accumulate, insertion of a catheter for continuous drainage is indicated. Supplies and equipment required for pericardiocentesis are summarized at ExpertConsult.com.

Pericardiocentesis Pericardiocentesis is the aspiration of fluid or air from the pericardial space. Pericardiocentesis can be performed emergently without any imaging guidance techniques. However, it should be undertaken only in dire circumstances because of the risk of the procedure and the higher failure rate when no guidance is used. Interventional cardiologists, when available, may be invaluable resources in performing these high-risk procedures.

Indications Drainage of a pericardial effusion due to any cause is absolutely indicated when cardiac tamponade is present. Drainage is often recommended if the effusion is large, even in the absence of tamponade, for diagnosis and fluid removal.155 For small effusions, pericardiocentesis may be indicated for diagnosis alone. In pediatric patients, pericardial effusions most commonly occur with postviral or idiopathic pericarditis, but they are also seen with postpericardiotomy syndrome, collagen vascular disease, oncologic disease, and, rarely, uremia.155 Purulent pericarditis resulting from Staphylococcus aureus or Streptococcus pneumoniae infection can be seen in cases of concomitant pneumonia with empyema. Although rare in developed countries, tuberculous pericarditis should be considered as a cause in high-burden tuberculosis areas or among immigrants from those areas. Drainage of purulent pericarditis is indicated for relief of tamponade, prevention of constrictive pericarditis, and diagnosis and drainage of infection. With purulent pericarditis, open surgical drainage may be more effective because of the difficulty in draining purulent exudate.156 If using a tube for pericardiocentesis and drainage, instillation of a fibrinolytic agent such as alteplase (recombinant tissue plasminogen factor) may be considered with purulent effusions.157 Traumatic pericardial effusions secondary to penetrating trauma often require surgical drainage of the blood because tamponade is common. Pneumopericardium secondary to pulmonary air leaks in mechanically ventilated patients is usually well tolerated hemodynamically but may require drainage, especially in small infants, because of the development of tamponade.

There is no absolute contraindication to pericardiocentesis in an emergency situation. The presence of aortic dissection or myocardial rupture is considered a major contraindication. The presence of a bleeding diathesis or coagulopathy is another contraindication. Open drainage is preferred over closed drainage when the patient has traumatic tamponade and is in cardiac arrest. When the effusion is loculated in a location not easily reached via the subxiphoid approach, needle pericardiocentesis is contraindicated because the risk of complications increases while the possibility of successful drainage is low.

Monitoring for Pericardiocentesis Critically ill patients require continuous pulse oximetry and ECG monitoring. Blood pressure must be measured very frequently. Before pericardiocentesis, airway patency and respiratory support should be ensured. In patients with tamponade physiology, procedures such as rapid sequence intubation or positive pressure ventilation can precipitate a sudden decrease in preload as an antecedent for impaired cardiac output and cardiac arrest. Volume resuscitation, conversely, can augment preload and temporarily improve hemodynamics. Monitoring the ECG tracing while advancing the pericardiocentesis needle enables the clinician to detect needle contact with the epicardium, which helps prevent cardiac puncture. Echocardiographic guidance is recommended for most pericardiocentesis because it can be done at the bedside and is logistically less complex.158 Echocardiographic scanning is indicated prior to needle drainage or catheter insertion for any reason except tamponade with cardiac arrest. The echocardiogram can visualize the size of the effusion; its distribution around the heart, including any loculations; the presence of fibrin or clots; and evidence of tamponade. Tamponade can be diagnosed using twodimensional imaging when the right atrium collapses during late diastole.

Technique Needle aspiration can be performed blindly in the event of a true emergency, such as traumatic tamponade; however, the technique has higher complication and failure rates. Among the potential chest sites for pericardiocentesis in infants and children, the subxiphoid approach is the safest and most common, although other approaches have been described. The approach is extra-pleural and, in patients with normal anatomy, avoids major vessels such as the internal mammary, coronary, and pericardial arteries.155 The subxiphoid and lower costal margin are prepared with 2% chlorhexidine. The area is draped in a sterile fashion. Lidocaine local anesthesia is infiltrated at the junction of the xiphoid and left

110.e1









4. Appropriate sterile sample tubes should be available for collection of fluid for chemical, cellular, and microbiological analysis. 5. A cardiac monitor is essential for determining arrhythmias during the procedure. 6. Catheter, dilator, and flexible J-wire are necessary when the catheter will be left in place. Placement in the pericardial sac of a 5- to 8-Fr pigtail catheter with multiple side holes is recommended. The size of the patient and viscosity of the fluid determine the size of the catheter. If the fluid is fibrinous in appearance by echocardiography, then a larger-bore catheter should be placed. Several pigtail catheters are manufactured specifically for fluid drainage, often available as kits that also contain an appropriately sized dilator and J-wire guide. If a kit is not available, then a venous dilator of appropriate size with a separate J-wire can be used. A J-wire is used to prevent another puncture of the pericardium or heart using a straight wire. Before the needle is inserted, the wire, dilator, and catheter must be checked to ensure that their sizes are compatible.











Supplies and equipment required for pericardiocentesis include the following: 1. Needles for drainage range in size from 14- to 20-gauge depending on the type of fluid and size of the patient. For thicker consistency fluids, such as pus or blood, a larger-bore needle is used. A steel needle, such as a vascular introducer needle or spinal needle, can be used. However, an IV catheter is often more effective because once the fluid is reached, the steel inner needle can be removed from the IV catheter, leaving the softer, needleless catheter in place while the fluid is aspirated. This process decreases the risk of cardiac puncture. 2. Syringes, three-way stopcock, and short extension tubing are assembled for aspiration. A 5- or 10-mL slip-tip syringe is used when accessing the pericardium for easier manipulation. A larger 20- to 30-mL syringe may be needed for fluid drainage, depending on the predicted volume. The stopcock and short tubing are useful when draining large amounts of fluid. 3. Equipment for insertion includes 2% chlorhexidine solution, sterile gloves and drapes, and 1% lidocaine for local anesthesia.



CHAPTER 14 Pediatric Vascular Access and Centeses

Cardiac tamponade Pericardium

Lower border of lung

111

the pericardial sac, then the needle is appropriately placed in the pericardium. Once the needle is determined to be well positioned in the pericardial sac, if a catheter is to be inserted, then the J-wire can be passed through the needle as with standard Seldinger technique. The needle is removed, and the dilator is passed over the wire to open the tissues outside the pericardium and enlarge the puncture in the pericardium. The dilator is removed, taking care to leave the J-wire in good position. The catheter is passed over the wire into the pericardial sac. Its position can be confirmed echocardiographically. The wire is then removed. The connecting tubing and three-way stopcock are connected and the fluid is aspirated. The tubing can be connected to a drainage bag for removal of fluid that continues to accumulate. A sample of the aspirated fluid should be sent to the laboratory for appropriate analysis and culture. The catheter is secured with suture and covered with an occlusive dressing. A chest radiograph should be taken at the end of the procedure and daily to confirm the catheter position.

Maintenance

• Fig. 14.8





​Insertion of needle for pericardiocentesis at the junction of the xiphoid and the left costal margin, aiming toward the left shoulder. (From Brundage SI, Scott BG, Karmy-Jones R et al. Pericardiocentesis and pericardial window. In: Shoemaker WC, Velmahos GC, Demetriades D, eds. Procedures and Monitoring for the Critically Ill. Philadelphia: Elsevier; 2002.)

costal margin. The needle is inserted at a 30- to 45-degree angle and directed toward the left clavicle (Fig. 14.8). The slip-tip syringe is attached and is aspirated continually while the needle is inserted. Needle advancement is halted when air or fluid is aspirated. During insertion, the needle is guided using two-dimensional echocardiography. The echocardiographic probe is placed on the chest, where the fluid is best seen. The needle tip is identified by ultrasound and followed as the needle is advanced.159 Another technique involves mounting the needle on the echocardiographic probe, which has been placed in a sterile sleeve. The needle is advanced while the operator also handles the probe. This technique allows the use of locations other than the subxiphoid approach for insertion of the introducer needle, with the potential for better fluid visualization.160 After the fluid has been echocardiographically evaluated, the patient is placed supine with the head elevated approximately 30 degrees. If blood is obtained, analysis is necessary to determine whether the blood is of pericardial or intracardiac origin. Several techniques are helpful for this determination. The hematocrit of pericardial fluid will be lower than that of intracardiac blood, which will be equal to the patient’s hematocrit. Dropping a few milliliters of the fluid on gauze sponges determines whether the fluid will clot. Fluid that does not clot is pericardial; fluid that does clot is most likely intracardiac blood. Another technique involves injection of small amounts of saline microbubble contrast (saline in a syringe that has been agitated) through the introducer needle while imaging with echocardiography. If contrast bubbles are seen in the heart, then the tip of the needle is intracardiac or intravascular. If bubbles appear in

The catheter is simple to maintain. The dressing should be changed according to the ICU protocol for CVCs. The fluid in the drainage bag should be measured and the amount recorded on a regular schedule. If fluid is no longer draining, then a small amount (1–2 mL) of heparinized saline can be infused into the pericardium through the stopcock after preparation with antiseptic solution. This process can release any fibrinous material occluding the catheter. If no fluid is forthcoming, then echocardiography can be performed to determine whether residual pericardial fluid remains. If residual fluid is present, the catheter is again flushed in an attempt to open it up. If the fluid had originally been purulent, then instillation of a fibrinolytic agent may allow better drainage. If no fluid remains, the catheter can be removed, depending on the patient’s condition and underlying cause of fluid development.

Complications The most serious and immediate mechanical complications of pericardiocentesis are myocardial puncture or laceration, vascular injury (coronary, intercostal, internal mammary, or intraabdominal), pneumothorax, air embolism, and arrhythmia (ventricular and supraventricular). Coronary laceration occurs rarely, resulting in acute myocardial ischemia. Transperitoneal needle passage can traverse intraabdominal organs. The liver is most commonly involved but is associated with low risk of significant hemorrhage. Perforation of a hollow viscus is theoretically possible but rarely reported. Infection of the indwelling catheter can occur but is rare because the catheter is not in place for more than 3 to 4 days.

Summary Pericardiocentesis, with or without catheter placement, is indicated for relief of cardiac tamponade and for a diagnosis of certain pericardial effusions. It is a lifesaving technique for patients with tamponade. Patients needing emergency pericardiocentesis are monitored with pulse oximetry and ECG monitors. Used in conjunction with guidance techniques such as echocardiography or electrocardiography, pericardiocentesis can be performed safely in patients of all ages.

112

SECTION II



Pediatric Critical Care: Tools and Procedures

Abdominal Paracentesis Abdominal paracentesis is the percutaneous sampling and drainage of peritoneal fluid by needle aspiration through the abdominal wall. Analysis of ascitic fluid, combined with a detailed history and physical examination, frequently confirms the etiology of ascites.

Indications Paracentesis may be used diagnostically for new fluid accumulations or therapeutically to relieve cardiopulmonary or end-organ (e.g., renal) compromise resulting from large accumulations of fluid. Diagnostic paracentesis is indicated in any patient with new-onset ascites, with existing ascites and clinical deterioration, and in cases of suspected bacterial peritonitis.161,162 Therapeutic paracentesis should be used for the relief of abdominal compartment syndrome secondary to massive ascites.163–165

Contraindications Paracentesis has few to no absolute contraindications. Relative indications include small amounts of ascitic fluid and severe bleeding diatheses such as seen in disseminated intravascular coagulation.166,167 Uncorrected coagulopathy and thrombocytopenia may rarely predispose to bleeding complications. However, prophylactic platelet or plasma transfusion is not recommended.161,162 Severe renal dysfunction has the highest association of bleeding complications after paracentesis.168 Skin infections or wounds at the insertion site can lead to introduction of new infection into the peritoneum and thus should be avoided. Massively distended bowel as can be seen in severe ileus or prior surgical scars with bowel adhesions present a risk of bowel perforation with paracentesis. An uncooperative patient can lead to damage of the underlying vascular structures or bowel. This risk may be mitigated by the appropriate use of sedation and analgesia.

Procedure When present, physical examination findings of flank dullness, shifting dullness, and fluid waves support the presence of ascites and guide selection of the needle insertion site, while ultrasound guidance improves the success rate of paracentesis. Additionally, when physical examination findings are insufficient or difficult due to body habitus, ultrasonography can detect smaller amounts of fluid, differentiate free from loculated fluid, and is more sensitive than physical examination in diagnosing ascites.161,162,166,169,170

Technique After the patient’s bladder is emptied fully by voiding or catheterization, the patient is positioned in a semi-recumbent position. The site of needle insertion is selected by physical examination or ultrasound. Bowel floats in ascites and tends to stay in a nondependent, midline position. Therefore, the preferred site for paracentesis is in the dependent, lateral area of the left lower quadrant: two fingerbreadths (2–3 cm) in both the medial and cephalad directions from the anterior superior iliac spine. Additionally, studies have shown that the abdominal wall is thinner and the pool of ascites deeper in the left lower quadrant.171 The right lower quadrant should be avoided owing to the cecum’s proximity to the abdominal wall. Potential insertion sites with surgical scars should also be avoided owing to the risk of underlying bowel

adhesions.161,162,166 The inferior epigastric arteries running cephalad midway between the pubic symphysis and anterior iliac spine in the rectus sheath should be avoided. In patients with portal hypertension, the midline may become vascularized and should be avoided, as well as any visible collateral vessels, to reduce the chance of hemorrhagic complications.161,162,166,172 While continually observing aseptic technique, the entry site is disinfected with chlorhexidine or povidone-iodine and draped in sterile fashion. Always aspirating while advancing, the skin and subcutaneous tissue down to the peritoneum are infiltrated with local anesthetic using a small-gauge needle. If no commercially assembled paracentesis kit is available, spinal needles are ideal for paracentesis fluid sampling (20- to 22-gauge for small children, 16- to 20-gauge for larger children). Prior to needle insertion, the skin at the insertion site should be pulled downward approximately 2 cm to create a nonlinear Z-track. While aspirating with an attached syringe, the clinician advances the needle slowly until free flow of fluid into the syringe is noted, but it is removed immediately if frank blood is aspirated. Approximately 20 to 40 mL of ascitic fluid is sufficient for diagnostic evaluation, but much larger volumes may be removed for therapeutic paracentesis in tense ascites. If fluid return stops or is sluggish, changing the patient’s position may be helpful. Once fluid collection is complete and the needle is removed, direct pressure with gauze is applied and a sterile pressure dressing is placed. When ongoing drainage is needed, a small-caliber pigtail catheter can be placed using the modified Seldinger technique (as described in the Tube Thoracostomy section).

Complications Serious complications of abdominal paracentesis are very rare (1%–3%), but several have been described.162,166 The most commonly described complication is persistent leakage of ascitic fluid, which can be avoided if proper Z-track technique and smallcaliber needles are used.173 Ongoing fluid leaks can be managed by closing the defect with suture or cyanoacrylate skin adhesive.174 While even more rare, abdominal wall hematoma or bleeding, bladder perforation, and intestinal perforation can occur. In patients with severe bleeding complications, 70% had renal dysfunction, 59% had coagulopathy, and 8% had thrombocytopenia.168 The risk of intestinal perforation is increased with previous history of abdominal surgery owing to adhesions and can be minimized by avoiding surgical scars for the needle insertion site. Additionally, intestinal perforation from paracentesis has been reported with marked bowel distension; though rare, decompression should be considered prior to paracentesis.175 Bladder emptying decreases the risk of bladder perforation. Ultrasound guidance can further decrease the risk of organ perforations. Strict adherence to sterile technique and avoidance of areas of skin or soft-tissue infection for needle insertion decrease the risk of infection. Hypotension may result if a large volume of ascites is removed at once (.15–20 mL/kg); in that circumstance, postprocedural albumin administration is sometimes required.

Interpretation Analysis of ascitic fluid in combination with clinical assessment generally will yield a definitive diagnosis. The characteristics of ascitic fluid in various conditions are summarized in eTable 14.2. Fluid studies obtained should include total protein, albumin, Gram stain and culture, and cell count with differential. Optional

112.e1

eTABLE Characteristics of Ascitic Fluid in Various Conditions 14.2

Condition

Clinical Characteristics

Laboratory Findings

Portal hypertension

Straw colored, sterile

SAAG 1.1 g/dL Total protein ,2.5–3.0 g/dL WBCs ,250–500/uL, ,1/3 neutrophils

Spontaneous bacterial peritonitis

Cloudy or turbid Gram positive ,10%, cultures may be negative, single organisma

PMNs 250/uL Total protein ,1 g/dL LDH and glucose similar to serum SAAG usually 1.1 g/dL

Secondary bacterial peritonitis

Gram positive, multiple organisms

PMNs 250/uL Total protein .1 g/dL LDH . serum Glucose ,50 mg/dL SAAG ,1.1 g/dL

Chylous ascites

Milky or yellow, with recent fat ingestion

WBCs 1000–5000/uL, predominately lymphocytes Triglycerides .200 mg/dL, usually .1000 mg/dL Cholesterol .48 mg/dLb

Pancreatic ascites

Turbid, tea colored, or bloody

WBCs and total protein increased Amylase and lipase . serum Amylase levels may be falsely low in young infants, but lipase always highc

Urinary ascites

Protein ,1 g/dL Urea and creatinine . serumd

Malignant ascites

Bloody

Protein and LDH elevated SAAG ,1.1 g/dL

Nephrotic syndrome

Straw colored

SAAG ,1.1 g/dL Total protein ,2.5 g/dLc

Tuberculous ascites

Yellow or rarely bloody, may have fibrin clots

Total protein .2.5 g/dL WBCs .1000/uL, primarily lymphocytes PCR usefule

PMNs: Absolute, corrected number of polymorphonuclear leukocytes for red blood cells, where one PMN is subtracted from the absolute PMN number for every 250 red cells/mm3. a Data from Kandel G, Diamant NE. A clinical view of recent advances in ascites. J Clin Gastroenterol. 1986;8(1):85–99. b Data from McGibbon A, Chen GI, Peltekian KM, et al. An evidence-based manual for abdominal paracentesis. Dig Dis Sci. 2007;52(12):3307–3315. c Data from Glauser JM. Paracentesis. In: Roberts JR, Hedges JR, eds. Clinical Procedures in Emergency Medicine. Philadelphia: WB Saunders; 1991. d Data from Runyon BA. Care of patients with ascites. N Engl J Med. 1994;330(5):337–342. e Data from Uzunkoy A, Harma M, Harma M. Diagnosis of abdominal tuberculosis: experience from 11 cases and review of the literature. World J Gastroenterol. 2004;10(24):3647–3649. LDH, Lactate dehydrogenase; PCR, polymerase chain reaction; SAAG, serum-to-ascites albumin gradient; WBCs, white blood cells.



CHAPTER 14 Pediatric Vascular Access and Centeses

studies to consider are glucose, lactate dehydrogenase, amylase, acid-fast bacilli smear and culture, cytology, and triglycerides. Corresponding serum chemistries should be obtained for comparison. Analysis of the serum-to-ascites albumin gradient (SAAG) is useful for differentiating between ascites resulting from portal hypertension or other etiologies, as it is an indirect measurement of portal pressure. SAAG of 1.1 g/dL or greater correlates with portal hypertension with 97% accuracy.162

Summary Needle aspiration of ascitic fluid is a safe procedure when performed with appropriate precautionary measures. Analysis of ascitic fluid obtained by paracentesis is useful in the evaluation of patients with new-onset ascites, with existing ascites and clinical deterioration, and in cases of suspected bacterial peritonitis. Large-volume therapeutic paracentesis or continuous drainage via a small-caliber catheter placed at the time of paracentesis (such as with tube thoracostomy) improves respiratory dynamics and reduces complications secondary to abdominal compartment syndrome.

Key References Barnett CF, Vaduganathan M, Lan G, Butler J, Gheorghiade M. Critical reappraisal of pulmonary artery catheterization and invasive hemodynamic assessment in acute heart failure. Expert Rev Cardiovasc Ther. 2013;11(4):417-424. Cousins TR, O’Donnell JM. Arterial cannulation: a critical review. AANA J. 2004;72(4):267-271. Froehlich CD, Rigby MR, Rosenberg ES, et al. Ultrasound-guided central venous catheter placement decreases complications and decreases

113

placement attempts compared with the landmark technique in patients in a pediatric intensive care unit. Crit Care Med. 2009;37(3): 1090-1096. Giefer MJ, Murray KF, Colletti RB. Pathophysiology, diagnosis, and management of pediatric ascites. J Pediatr Gastroenterol Nutr. 2011; 52(5):503-513. Goligher EC, Leis JA, Fowler RA, Pinto R, Adhikari NK, Ferguson ND. Utility and safety of draining pleural effusions in mechanically ventilated patients: a systematic review and meta-analysis. Crit Care. 2011;15(1):R46. Noonan PJ, Hanson SJ, Simpson PM, et al. Comparison of complication rates of central venous catheters versus peripherally inserted central venous catheters in pediatric patients. Pediatr Crit Care Med. 2018;19(12):1097-1105. Practice guidelines for central venous access 2020: an updated report by the American Society of Anesthesiologists Task Force on central venous access. Anesthesiology. 2020;132(1):8-43. Shen KR, Bribriesco A, Crabtree T, et al. The American Association for Thoracic Surgery consensus guidelines for the management of empyema. J Thorac Cardiovasc Surg. 2017;153(6):e129-e146. Vayre F, Lardoux H, Pezzano M, Bourdarias JP, Dubourg O. Subxiphoid pericardiocentesis guided by contrast two-dimensional echocardiography in cardiac tamponade: experience of 110 consecutive patients. Eur J Echocardiogr. 2000;1(1):66-71. Voigt J, Waltzman M, Lottenberg L. Intraosseous vascular access for inhospital emergency use: a systematic clinical review of the literature and analysis. Pediatr Emerg Care. 2012;28(2):185-199. Yamaguchi RS, Noritomi DT, Degaspare NV, et al. Peripherally inserted central catheters are associated with lower risk of bloodstream infection compared with central venous catheters in paediatric intensive care patients: a propensity-adjusted analysis. Intensive Care Med. 2017;43(8):1097-1104.

The full reference list for this chapter is available at ExpertConsult.com.

e1

































































































1. de Caen AR, Kleinman ME, Chameides L, et al. Part 10: Paediatric basic and advanced life support: 2010 International Consensus on Cardiopulmonary Resuscitation and Emergency Cardiovascular Care Science with Treatment Recommendations. Resuscitation. 2010; 81(suppl 1):e213-e259. 2. Voigt J, Waltzman M, Lottenberg L. Intraosseous vascular access for in-hospital emergency use: a systematic clinical review of the literature and analysis. Pediatr Emerg Care. 2012;28(2):185-199. 3. Fiorito BA, Mirza F, Doran TM, et al. Intraosseous access in the setting of pediatric critical care transport. Pediatr Crit Care Med. 2005;6(1):50-53. 4. Gazin N, Auger H, Jabre P, et al. Efficacy and safety of the EZ-IO intraosseous device: out-of-hospital implementation of a management algorithm for difficult vascular access. Resuscitation. 2011; 82(1):126-129. 5. Salassi-Scotter M, Fiser DH. Adoption of intraosseous infusion technique for prehospital pediatric emergency care. Pediatr Emerg Care. 1990;6(4):263-265. 6. Orlowski JP. Emergency alternatives to intravenous access. Intraosseous, intratracheal, sublingual, and other-site drug administration. Pediatr Clin North Am. 1994;41(6):1183-1199. 7. Horton MA, Beamer C. Powered intraosseous insertion provides safe and effective vascular access for pediatric emergency patients. Pediatr Emerg Care. 2008;24(6):347-350. 8. Leidel BA, Kirchhoff C, Bogner V, Braunstein V, Biberthaler P, Kanz KG. Comparison of intraosseous versus central venous vascular access in adults under resuscitation in the emergency department with inaccessible peripheral veins. Resuscitation. 2012;83(1): 40-45. 9. Jaimovich DG, Kumar A, Francom S. Evaluation of intraosseous vs intravenous antibiotic levels in a porcine model. Am J Dis Child. 1991;145(8):946-949. 10. Veldhoen ES, de Vooght KM, Slieker MG, Versluys AB, Turner NM. Analysis of bloodgas, electrolytes and glucose from intraosseous samples using an i-STAT((R)) point-of-care analyser. Resuscitation. 2014;85(3):359-363. 11. Tallman CI, Darracq M, Young M. Analysis of intraosseous blood samples using an EPOC point of care analyzer during resuscitation. Am J Emerg Med. 2017;35(3):499-501. 12. Hurren JS. Can blood taken from intraosseous cannulations be used for blood analysis? Burns. 2000;26(8):727-730. 13. Jousi M, Laukkanen-Nevala P, Nurmi J. Analysing blood from intraosseous access: a systematic review. Eur J Emerg Med. 2019; 26(2):77-85. 14. Brickman KR, Krupp K, Rega P, Alexander J, Guinness M. Typing and screening of blood from intraosseous access. Ann Emerg Med. 1992;21(4):414-417. 15. Orlowski JP, Porembka DT, Gallagher JM, Van Lente F. The bone marrow as a source of laboratory studies. Ann Emerg Med. 1989; 18(12):1348-1351. 16. Fiser DH. Intraosseous infusion. N Engl J Med. 1990;322(22): 1579-1581. 17. Dubick MA, Pfeiffer JW, Clifford CB, Runyon DE, Kramer GC. Comparison of intraosseous and intravenous delivery of hypertonic saline/dextran in anesthetized, euvolemic pigs. Ann Emerg Med. 1992;21(5):498-503. 18. Frascone RJ, Jensen J, Wewerka SS, Salzman JG. Use of the pediatric EZ-IO needle by emergency medical services providers. Pediatr Emerg Care. 2009;25(5):329-332. 19. Bielski K, Szarpak L, Smereka J, Ladny JR, Leung S, Ruetzler K. Comparison of four different intraosseous access devices during simulated pediatric resuscitation. A randomized crossover manikin trial. Eur J Pediatr. 2017;176(7):865-871. 20. Gerritse BM, Scheffer GJ, Draaisma JM. Prehospital intraosseus access with the bone injection gun by a helicopter-transported emergency medical team. J Trauma. 2009;66(6):1739-1741.

21. Shavit I, Hoffmann Y, Galbraith R, Waisman Y. Comparison of two mechanical intraosseous infusion devices: a pilot, randomized crossover trial. Resuscitation. 2009;80(9):1029-1033. 22. Kehrl T, Becker BA, Simmons DE, Broderick EK, Jones RA. Intraosseous access in the obese patient: assessing the need for extended needle length. Am J Emerg Med. 2016;34(9):1831-1834. 23. Ribeiro JA, Price CT, Knapp Jr DR. Compartment syndrome of the lower extremity after intraosseous infusion of fluid. A report of two cases. J Bone Joint Surg Am. 1993;75(3):430-433. 24. Simmons CM, Johnson NE, Perkin RM, van Stralen D. Intraosseous extravasation complication reports. Ann Emerg Med. 1994;23(2): 363-366. 25. Rosovsky M, FitzPatrick M, Goldfarb CR, Finestone H. Bilateral osteomyelitis due to intraosseous infusion: case report and review of the English-language literature. Pediatr Radiol. 1994;24(1):72-73. 26. La Fleche FR, Slepin MJ, Vargas J, Milzman DP. Iatrogenic bilateral tibial fractures after intraosseous infusion attempts in a 3-month-old infant. Ann Emerg Med. 1989;18(10):1099-1101. 27. Brzezinski M, Luisetti T, London MJ. Radial artery cannulation: a comprehensive review of recent anatomic and physiologic investigations. Anesth Analg. 2009;109(6):1763-1781. 28. Cousins TR, O’Donnell JM. Arterial cannulation: a critical review. AANA J. 2004;72(4):267-271. 29. Mourot JM, Oliveira HM, Woodson LC, Herndon DN, Chung DH. Complications of femoral artery catheterization in pediatric burn patients. J Burn Care Res. 2009;30(3):432-436. 30. Schindler E, Kowald B, Suess H, Niehaus-Borquez B, Tausch B, Brecher A. Catheterization of the radial or brachial artery in neonates and infants. Paediatr Anaesth. 2005;15(8):677-682. 31. Tegtmeyer K, Brady G, Lai S, Hodo R, Braner D. Videos in clinical medicine. Placement of an arterial line. N Engl J Med. 2006; 354(15):e13. 32. Timsit JF, Schwebel C, Bouadma L, et al. Chlorhexidine-impregnated sponges and less frequent dressing changes for prevention of catheter-related infections in critically ill adults: a randomized controlled trial. JAMA. 2009;301(12):1231-1241. 33. Beards SC, Doedens L, Jackson A, Lipman J. A comparison of arterial lines and insertion techniques in critically ill patients. Anaesthesia. 1994;49(11):968-973. 34. Yildirim V, Ozal E, Cosar A, et al. Direct versus guidewire-assisted pediatric radial artery cannulation technique. J Cardiothorac Vasc Anesth. 2006;20(1):48-50. 35. Ohara Y, Nakayama S, Furukawa H, Satoh Y, Suzuki H, Yanai H. Use of a wire-guided cannula for radial arterial cannulation. J Anesth. 2007;21(1):83-85. 36. Aouad-Maroun M, Raphael CK, Sayyid SK, Farah F, Akl EA. Ultrasound-guided arterial cannulation for paediatrics. Cochrane Database Syst Rev. 2016;9:CD011364. 37. Ganesh A, Kaye R, Cahill AM, et al. Evaluation of ultrasoundguided radial artery cannulation in children. Pediatr Crit Care Med. 2009;10(1):45-48. 38. Gao YB, Yan JH, Gao FQ, Pan L, Wang XZ, Lv CJ. Effects of ultrasound-guided radial artery catheterization: an updated meta-analysis. Am J Emerg Med. 2015;33(1):50-55. 39. Schwemmer U, Arzet HA, Trautner H, Rauch S, Roewer N, Greim CA. Ultrasound-guided arterial cannulation in infants improves success rate. Eur J Anaesthesiol. 2006;23(6):476-480. 40. Shiver S, Blaivas M, Lyon M. A prospective comparison of ultrasound-guided and blindly placed radial arterial catheters. Acad Emerg Med. 2006;13(12):1275-1279. 41. Kordzadeh A, Austin T, Panayiotopoulos Y. Efficacy of normal saline in the maintenance of the arterial lines in comparison to heparin flush: a comprehensive review of the literature. J Vasc Access. 2014;15(2):123-127. 42. Randolph AG, Cook DJ, Gonzales CA, Andrew M. Benefit of heparin in peripheral venous and arterial catheters: systematic review and meta-analysis of randomised controlled trials. BMJ. 1998; 316(7136):969-975.

References

e2

















































64. Dieckmann RA, Fiser DH, Selbst SM. Central venous access. In: Dieckmann RA, FD, Selbst SM, eds. Pediatric Emergency and Critical Care Procedures. Philadelphia, PA, 1997:796. 65. Souza Neto EP, Grousson S, Duflo F, Tahon F, Mottolese C, Dailler F. Ultrasonographic anatomic variations of the major veins in paediatric patients. Br J Anaesth. 2014;112(5):879-884. 66. Alderson PJ, Burrows FA, Stemp LI, Holtby HM. Use of ultrasound to evaluate internal jugular vein anatomy and to facilitate central venous cannulation in paediatric patients. Br J Anaesth. 1993;70(2): 145-148. 67. Chuan WX, Wei W, Yu L. A randomized-controlled study of ultrasound prelocation vs anatomical landmark-guided cannulation of the internal jugular vein in infants and children. Paediatr Anaesth. 2005;15(9):733-738. 68. Froehlich CD, Rigby MR, Rosenberg ES, et al. Ultrasound-guided central venous catheter placement decreases complications and decreases placement attempts compared with the landmark technique in patients in a pediatric intensive care unit. Crit Care Med. 2009;37(3):1090-1096. 69. Keenan SP. Use of ultrasound to place central lines. J Crit Care. 2002;17(2):126-137. 70. Verghese ST, McGill WA, Patel RI, Sell JE, Midgley FM, Ruttimann UE. Ultrasound-guided internal jugular venous cannulation in infants: a prospective comparison with the traditional palpation method. Anesthesiology. 1999;91(1):71-77. 71. Hind D, Calvert N, McWilliams R, et al. Ultrasonic locating devices for central venous cannulation: meta-analysis. BMJ. 2003; 327(7411):361. 72. Rhondali O, Attof R, Combet S, Chassard D, de Queiroz Siqueira M. Ultrasound-guided subclavian vein cannulation in infants: supraclavicular approach. Paediatr Anaesth. 2011;21(11):1136-1141. 73. Reyes JA, Habash ML, Taylor RP. Femoral central venous catheters are not associated with higher rates of infection in the pediatric critical care population. Am J Infect Control. 2012;40(1):43-47. 74. Miller MR, Niedner MF, Huskins WC, et al. Reducing PICU central line-associated bloodstream infections: 3-year results. Pediatrics. 2011;128(5):e1077-e1083. 75. Darouiche RO, Raad, II, Heard SO, et al. A comparison of two antimicrobial-impregnated central venous catheters. Catheter Study Group. N Engl J Med. 1999;340(1):1-8. 76. Revel-Vilk S, Yacobovich J, Tamary H, et al. Risk factors for central venous catheter thrombotic complications in children and adolescents with cancer. Cancer. 2010;116(17):4197-4205. 77. Worly JM, Fortenberry JD, Hansen I, Chambliss CR, Stockwell J. Deep venous thrombosis in children with diabetic ketoacidosis and femoral central venous catheters. Pediatrics. 2004;113(1 Pt 1): e57-e60. 78. Scott WL. Complications associated with central venous catheters. A survey. Chest. 1988;94(6):1221-1224. 79. Smith-Wright DL, Green TP, Lock JE, Egar MI, Fuhrman BP. Complications of vascular catheterization in critically ill children. Crit Care Med. 1984;12(12):1015-1017. 80. Kaye R, Sane SS, Towbin RB. Pediatric intervention: an update— part II. J Vasc Interv Radiol. 2000;11(7):807-822. 81. Allen AW, Megargell JL, Brown DB, et al. Venous thrombosis associated with the placement of peripherally inserted central catheters. J Vasc Interv Radiol. 2000;11(10):1309-1314. 82. Barrier A, Williams DJ, Connelly M, Creech CB. Frequency of peripherally inserted central catheter complications in children. Pediatr Infect Dis J. 2012;31(5):519-521. 83. Chow LM, Friedman JN, Macarthur C, et al. Peripherally inserted central catheter (PICC) fracture and embolization in the pediatric population. J Pediatr. 2003;142(2):141-144. 84. Jumani K, Advani S, Reich NG, Gosey L, Milstone AM. Risk factors for peripherally inserted central venous catheter complications in children. JAMA Pediatr. 2013;167(5):429-435. 85. Sharp R, Cummings M, Fielder A, Mikocka-Walus A, Grech C, Esterman A. The catheter to vein ratio and rates of symptomatic venous thromboembolism in patients with a peripherally inserted

















































43. Del Cotillo M, Grane N, Llavore M, Quintana S. Heparinized solution vs. saline solution in the maintenance of arterial catheters: a double blind randomized clinical trial. Intensive Care Med. 2008; 34(2):339-343. 44. Robertson-Malt S, Malt GN, Farquhar V, Greer W. Heparin versus normal saline for patency of arterial lines. Cochrane Database Syst Rev. 2014(5):CD007364. 45. Heulitt MJ, Farrington EA, O’Shea TM, Stoltzman SM, Srubar NB, Levin DL. Double-blind, randomized, controlled trial of papaverine-containing infusions to prevent failure of arterial catheters in pediatric patients. Crit Care Med. 1993;21(6):825-829. 46. Brotschi B, Hug MI, Latal B, et al. Incidence and predictors of indwelling arterial catheter-related thrombosis in children. J Thromb Haemost. 2011;9(6):1157-1162. 47. Kirchhoff KT, Rebenson-Piano M, Patel MK. Mean arterial pressure readings: variations with positions and transducer level. Nurs Res. 1984;33(6):343-345. 48. McCann UG II, Schiller HJ, Carney DE, et al. Invasive arterial BP monitoring in trauma and critical care: effect of variable transducer level, catheter access, and patient position. Chest. 2001;120(4):1322-1326. 49. Ullman AJ, Cooke ML, Gillies D, et al. Optimal timing for intravascular administration set replacement. Cochrane Database Syst Rev. 2013(9):CD003588. 50. Garnacho-Montero J, Aldabo-Pallas T, Palomar-Martinez M, et al. Risk factors and prognosis of catheter-related bloodstream infection in critically ill patients: a multicenter study. Intensive Care Med. 2008;34(12):2185-2193. 51. King MA, Garrison MM, Vavilala MS, Zimmerman JJ, Rivara FP. Complications associated with arterial catheterization in children. Pediatr Crit Care Med. 2008;9(4):367-371. 52. Macnicol MF, Anagnostopoulos J. Arrest of the growth plate after arterial cannulation in infancy. J Bone Joint Surg Br. 2000;82(2): 172-175. 53. O’Horo JC, Maki DG, Krupp AE, Safdar N. Arterial catheters as a source of bloodstream infection: a systematic review and meta-analysis. Crit Care Med. 2014;42(6):1334-1339. 54. Koh DB, Gowardman JR, Rickard CM, Robertson IK, Brown A. Prospective study of peripheral arterial catheter infection and comparison with concurrently sited central venous catheters. Crit Care Med. 2008;36(2):397-402. 55. Lorente L, Jimenez A, Jimenez JJ, et al. Higher arterial catheter-related infection rates in femoral than in dorsalis pedis access. J Hosp Infect. 2010;74(4):365-369. 56. Khalifa R, Dahyot-Fizelier C, Laksiri L, et al. Indwelling time and risk of colonization of peripheral arterial catheters in critically ill patients. Intensive Care Med. 2008;34(10):1820-1826. 57. Smith MJ. Catheter-related bloodstream infections in children. Am J Infect Control. 2008;36(10):S173.e171-e173. 58. Olson ME, Lam K, Bodey GP, King EG, Costerton JW. Evaluation of strategies for central venous catheter replacement. Crit Care Med. 1992;20(6):797-804. 59. Orliaguet GA, Meyer PG, Baugnon T. Management of critically ill children with traumatic brain injury. Paediatr Anaesth. 2008;18(6): 455-461. 60. Raad, II, Hohn DC, Gilbreath BJ, et al. Prevention of central venous catheter-related infections by using maximal sterile barrier precautions during insertion. Infect Control Hosp Epidemiol. 1994;15(4 Pt 1): 231-238. 61. Carrer S, Bocchi A, Bortolotti M, et al. Effect of different sterile barrier precautions and central venous catheter dressing on the skin colonization around the insertion site. Minerva Anestesiol. 2005;71(5):197-206. 62. Maki DG, Ringer M, Alvarado CJ. Prospective randomised trial of povidone-iodine, alcohol, and chlorhexidine for prevention of infection associated with central venous and arterial catheters. Lancet. 1991;338(8763):339-343. 63. Mimoz O, Villeminey S, Ragot S, et al. Chlorhexidine-based antiseptic solution vs alcohol-based povidone-iodine for central venous catheter care. Arch Intern Med. 2007;167(19):2066-2072.





104.





105.





106.





107.





109.





108.











103.







102.







101.







100.



99.





98.





97.





96.





95.





94.





93.





92.





91.





90.





89.





88.

110. Bryant BG. Drug, fluid, and blood products administered through the umbilical artery catheter: complication experiences from one NICU. Neonatal Netw. 1990;9(1):27-32, 43-26. 111. Oppenheimer DA, Carroll BA, Garth KE. Ultrasonic detection of complications following umbilical arterial catheterization in the neonate. Radiology. 1982;145(3):667-672. 112. McAdams RM, Winter VT, McCurnin DC, Coalson JJ. Complications of umbilical artery catheterization in a model of extreme prematurity. J Perinatol. 2009;29(10):685-692. 113. O’Grady NP, Alexander M, Burns LA, et al. Guidelines for the prevention of intravascular catheter-related infections. Clin Infect Dis. 2011;52(9):e162-e193. 114. Monteiro AJ, Canale LS, Barbosa R, Meier M. Cardiac tamponade caused by central venous catheter in two newborns. Rev Bras Cir Cardiovasc. 2008;23(3):422-424. 115. Landers S, Moise AA, Fraley JK, Smith EO, Baker CJ. Factors associated with umbilical catheter-related sepsis in neonates. Am J Dis Child. 1991;145(6):675-680. 116. Butler-O’Hara M, Buzzard CJ, Reubens L, McDermott MP, DiGrazio W, D’Angio CT. A randomized trial comparing long-term and short-term use of umbilical venous catheters in premature infants with birth weights of less than 1251 grams. Pediatrics. 2006;118(1):e25-e35. 117. Butler-O’Hara M, D’Angio CT, Hoey H, Stevens TP. An evidencebased catheter bundle alters central venous catheter strategy in newborn infants. J Pediatr. 2012;160(6):972-977.e972. 118. Barnett CF, Vaduganathan M, Lan G, Butler J, Gheorghiade M. Critical reappraisal of pulmonary artery catheterization and invasive hemodynamic assessment in acute heart failure. Expert Rev Cardiovasc Ther. 2013;11(4):417-424. 119. Chatterjee K. The Swan-Ganz catheters: past, present, and future. A viewpoint. Circulation. 2009;119(1):147-152. 120. Shah MR, Hasselblad V, Stevenson LW, et al. Impact of the pulmonary artery catheter in critically ill patients: meta-analysis of randomized clinical trials. JAMA. 2005;294(13):1664-1670. 121. Summerhill EM, Baram M. Principles of pulmonary artery catheterization in the critically ill. Lung. 2005;183(3):209-219. 122. Bernard GR, Sopko G, Cerra F, et al. Pulmonary artery catheterization and clinical outcomes: National Heart, Lung, and Blood Institute and Food and Drug Administration Workshop Report. Consensus Statement. JAMA. 2000;283(19):2568-2572. 123. Robin E, Costecalde M, Lebuffe G, Vallet B. Clinical relevance of data from the pulmonary artery catheter. Crit Care. 2006;10(Suppl 3):S3. 124. Alkrinawi S, Chernick V. Pleural infection in children. Semin Respir Infect. 1996;11(3):148-154. 125. Hardie W, Bokulic R, Garcia VF, Reising SF, Christie CD. Pneumococcal pleural empyemas in children. Clin Infect Dis. 1996;22(6): 1057-1063. 126. Utine GE, Ozcelik U, Kiper N, et al. Pediatric pleural effusions: etiological evaluation in 492 patients over 29 years. Turk J Pediatr. 2009;51(3):214-219. 127. Ault MJ, Rosen BT, Scher J, Feinglass J, Barsuk JH. Thoracentesis outcomes: a 12-year experience. Thorax. 2015;70(2):127-132. 128. Cantey EP, Walter JM, Corbridge T, Barsuk JH. Complications of thoracentesis: incidence, risk factors, and strategies for prevention. Curr Opin Pulm Med. 2016;22(4):378-385. 129. Goligher EC, Leis JA, Fowler RA, Pinto R, Adhikari NK, Ferguson ND. Utility and safety of draining pleural effusions in mechanically ventilated patients: a systematic review and meta-analysis. Crit Care. 2011;15(1):R46. 130. Gordon CE, Feller-Kopman D, Balk EM, Smetana GW. Pneumothorax following thoracentesis: a systematic review and meta-analysis. Arch Intern Med. 2010;170(4):332-339. 131. Jones PW, Moyers JP, Rogers JT, Rodriguez RM, Lee YC, Light RW. Ultrasound-guided thoracentesis: is it a safer method? Chest. 2003;123(2):418-423. 132. Hibbert RM, Atwell TD, Lekah A, et al. Safety of ultrasoundguided thoracentesis in patients with abnormal preprocedural coagulation parameters. Chest. 2013;144(2):456-463.



87.

central catheter (PICC): a prospective cohort study. Int J Nurs Stud. 2015;52(3):677-685. Randolph J. Technique for insertion of plastic catheter into saphenous vein. Pediatrics. 1959;24:631-635. Simon RR, Hoffman JR, Smith M. Modified new approaches for rapid intravenous access. Ann Emerg Med. 1987;16(1):44-49. Tiffany KF, Burke BL, Collins-Odoms C, Oelberg DG. Current practice regarding the enteral feeding of high-risk newborns with umbilical catheters in situ. Pediatrics. 2003;112(1 Pt 1):20-23. Green C, Yohannan MD. Umbilical arterial and venous catheters: placement, use, and complications. Neonatal Netw. 1998;17(6):23-28. Nash P. Umbilical catheters, placement, and complication management. J Infus Nurs. 2006;29(6):346-352. Grizelj R, Vukovic J, Bojanic K, et al. Severe liver injury while using umbilical venous catheter: case series and literature review. Am J Perinatol. 2014;31(11):965-974. Mendeloff J, Stallion A, Hutton M, Goldstone J. Aortic aneurysm resulting from umbilical artery catheterization: case report, literature review, and management algorithm. J Vasc Surg. 2001;33(2): 419-424. Barrington KJ. Umbilical artery catheters in the newborn: effects of catheter design (end vs side hole). Cochrane Database Syst Rev. 2000;(2):CD000508. Barrington KJ. Umbilical artery catheters in the newborn: effects of catheter materials. Cochrane Database Syst Rev. 2000;(2): CD000949. Ramachandran P, Cohen RS, Kim EH, Glasscock GF. Experience with double-lumen umbilical venous catheters in the low-birthweight neonate. J Perinatol. 1994;14(4):280-284. Lewis GC, Crapo SA, Williams JG. Critical skills and procedures in emergency medicine: vascular access skills and procedures. Emerg Med Clin North Am. 2013;31(1):59-86. Phelps DL. The umbilical artery catheter: prevention of bleeding after its removal. Pediatrics. 1972;50(1):164. Schreiber MD, Perez CA, Kitterman JA. A double-catheter technique for caudally misdirected umbilical arterial catheters. J Pediatr. 1984;104(5):768-769. Elser HE. Options for securing umbilical catheters. Adv Neonatal Care. 2013;13(6):426-429. Butt WW, Whyte H. Blood pressure monitoring in neonates: comparison of umbilical and peripheral artery catheter measurements. J Pediatr. 1984;105(4):630-632. Dunn PM. Localization of the umbilical catheter by post-mortem measurement. Arch Dis Child. 1966;41(215):69-75. Shukla H, Ferrara A. Rapid estimation of insertional length of umbilical catheters in newborns. Am J Dis Child. 1986;140(8): 786-788. Wright IM, Owers M, Wagner M. The umbilical arterial catheter: a formula for improved positioning in the very low birth weight infant. Pediatr Crit Care Med. 2008;9(5):498-501. Barrington KJ. Umbilical artery catheters in the newborn: effects of position of the catheter tip. Cochrane Database Syst Rev. 2000; (2):CD000505. Mandel D, Mimouni FB, Littner Y, Dollberg S. Double catheter technique for misdirected umbilical vein catheter. J Pediatr. 2001;139(4):591-592. Verheij GH, te Pas AB, Smits-Wintjens VE, Sramek A, Walther FJ, Lopriore E. Revised formula to determine the insertion length of umbilical vein catheters. Eur J Pediatr. 2013;172(8):1011-1015. Michel F, Brevaut-Malaty V, Pasquali R, et al. Comparison of ultrasound and X-ray in determining the position of umbilical venous catheters. Resuscitation. 2012;83(6):705-709. Barrington KJ. Umbilical artery catheters in the newborn: effects of heparin. Cochrane Database Syst Rev. 2000;(2):CD000507. Jackson JK, Biondo DJ, Jones JM, et al. Can an alternative umbilical arterial catheter solution and flush regimen decrease iatrogenic hemolysis while enhancing nutrition? A double-blind, randomized, clinical trial comparing an isotonic amino acid with a hypotonic salt infusion. Pediatrics. 2004;114(2):377-383.





86.































e3

e4





156.









158.





159.



160.







157.













161.





163.





164.



165.









162.





166.





167.





168.















169.





170.







171.





172.





173.





174.



175.











































133. Puchalski J. Thoracentesis and the risks for bleeding: a new era. Curr Opin Pulm Med. 2014;20(4):377-384. 134. Tariq SM, Sadaf T. Images in clinical medicine. Reexpansion pulmonary edema after treatment of pneumothorax. N Engl J Med. 2006;354(19):2046. 135. Tarver RD, Broderick LS, Conces DJ, Jr. Reexpansion pulmonary edema. J Thorac Imaging. 1996;11(3):198-209. 136. Hirsch AW, Nagler J. Reexpansion pulmonary edema in pediatrics. Pediatr Emerg Care. 2018;34(3):216-220. 137. Jardine OS. Reexpansion pulmonary edema. Am J Dis Child. 1991;145(10):1092-1094. 138. Kira S. Reexpansion pulmonary edema: review of pediatric cases. Paediatr Anaesth. 2014;24(3):249-256. 139. Light RW, Macgregor MI, Luchsinger PC, Ball Jr WC. Pleural effusions: the diagnostic separation of transudates and exudates. Ann Intern Med. 1972;77(4):507-513. 140. Heffner JE, Brown LK, Barbieri CA. Diagnostic value of tests that discriminate between exudative and transudative pleural effusions. Primary Study Investigators. Chest. 1997;111(4):970-980. 141. Utine GE, Pinar A, Ozcelik U, et al. Pleural fluid PCR method for detection of Staphylococcus aureus, Streptococcus pneumoniae and Haemophilus influenzae in pediatric parapneumonic effusions. Respiration. 2008;75(4):437-442. 142. Islam S, Calkins CM, Goldin AB, et al. The diagnosis and management of empyema in children: a comprehensive review from the APSA Outcomes and Clinical Trials Committee. J Pediatr Surg. 2012;47(11):2101-2110. 143. Light RW. Pleural controversy: optimal chest tube size for drainage. Respirology. 2011;16(2):244-248. 144. Aziz F, Penupolu S, Flores D. Efficacy of percutaneous pigtail catheters for thoracostomy at bedside. J Thorac Dis. 2012;4(3):292-295. 145. Shen KR, Bribriesco A, Crabtree T, et al. The American Association for Thoracic Surgery consensus guidelines for the management of empyema. J Thorac Cardiovasc Surg. 2017;153(6):e129-e146. 146. Thommi G, Nair CK, Aronow WS, Shehan C, Meyers P, McLeay M. Efficacy and safety of intrapleural instillation of alteplase in the management of complicated pleural effusion or empyema. Am J Ther. 2007;14(4):341-345. 147. Lois M, Noppen M. Bronchopleural fistulas: an overview of the problem with special focus on endoscopic management. Chest. 2005;128(6):3955-3965. 148. Cerfolio RJ, Bass C, Katholi CR. Prospective randomized trial compares suction versus water seal for air leaks. Ann Thorac Surg. 2001;71(5):1613-1617. 149. Pacanowski JP, Waack ML, Daley BJ, et al. Is routine roentgenography needed after closed tube thoracostomy removal? J Trauma. 2000;48(4):684-688. 150. Pacharn P, Heller DN, Kammen BF, et al. Are chest radiographs routinely necessary following thoracostomy tube removal? Pediatr Radiol. 2002;32(2):138-142. 151. Bell RL, Ovadia P, Abdullah F, Spector S, Rabinovici R. Chest tube removal: end-inspiration or end-expiration? J Trauma. 2001;50(4): 674-677. 152. Kwiatt M, Tarbox A, Seamon MJ, et al. Thoracostomy tubes: a comprehensive review of complications and related topics. Int J Crit Illn Inj Sci. 2014;4(2):143-155. 153. Remerand F, Luce V, Badachi Y, Lu Q, Bouhemad B, Rouby JJ. Incidence of chest tube malposition in the critically ill: a prospective computed tomography study. Anesthesiology. 2007;106(6):1112-1119. 154. Rashid MA, Wikstrom T, Ortenwall P. Mediastinal perforation and contralateral hemothorax by a chest tube. Thorac Cardiovasc Surg. 1998;46(6):375-376. 155. Maisch B, Seferovic PM, Ristic AD, et al. Guidelines on the diagnosis and management of pericardial diseases executive summary;

The Task Force on the Diagnosis and Management of Pericardial Diseases of the European Society of Cardiology. Eur Heart J. 2004;25(7):587-610. Sagrista-Sauleda J, Barrabes JA, Permanyer-Miralda G, Soler-Soler J. Purulent pericarditis: review of a 20-year experience in a general hospital. J Am Coll Cardiol. 1993;22(6):1661-1665. Wiyeh AB, Ochodo EA, Wiysonge CS, et al. A systematic review of the efficacy and safety of intrapericardial fibrinolysis in patients with pericardial effusion. Int J Cardiol. 2018;250:223-228. Vayre F, Lardoux H, Pezzano M, Bourdarias JP, Dubourg O. Subxiphoid pericardiocentesis guided by contrast two-dimensional echocardiography in cardiac tamponade: experience of 110 consecutive patients. Eur J Echocardiogr. 2000;1(1):66-71. Silverman N. Postoperative evaluation. In: Silverman N, ed. Pediatric Echocardiography. Baltimore, MD: William and Wilkins; 1993. Maggiolini S, Bozzano A, Russo P, et al. Echocardiography-guided pericardiocentesis with probe-mounted needle: report of 53 cases. J Am Soc Echocardiogr. 2001;14(8):821-824. Giefer MJ, Murray KF, Colletti RB. Pathophysiology, diagnosis, and management of pediatric ascites. J Pediatr Gastroenterol Nutr. 2011;52(5):503-513. Runyon BA, Practice Guidelines Committee AAftSoLD. Management of adult patients with ascites due to cirrhosis. Hepatology. 2004;39(3):841-856. Corcos AC, Sherman HF. Percutaneous treatment of secondary abdominal compartment syndrome. J Trauma. 2001;51(6):10621064. Kirkpatrick AW, Roberts DJ, De Waele J, et al. Intra-abdominal hypertension and the abdominal compartment syndrome: updated consensus definitions and clinical practice guidelines from the World Society of the Abdominal Compartment Syndrome. Intensive Care Med. 2013;39(7):1190-1206. Sharpe RP, Pryor JP, Gandhi RR, Stafford PW, Nance ML. Abdominal compartment syndrome in the pediatric blunt trauma patient treated with paracentesis: report of two cases. J Trauma. 2002;53(2):380-382. Runyon BA. Paracentesis of ascitic fluid. A safe procedure. Arch Intern Med. 1986;146(11):2259-2261. Runyon BA. Care of patients with ascites. N Engl J Med. 1994; 330(5):337-342. Sharzehi K, Jain V, Naveed A, Schreibman I. Hemorrhagic complications of paracentesis: a systematic review of the literature. Gastroenterol Res Pract. 2014;2014:985141. Hickerson SL, CJ, Schutze GE. Diagnostic procedures. In: Dieckmann RA, Fiser DH, Selbst SM, eds. Pediatric Emergency and Critical Care Procedures. St. Louis, MO: Mosby; 1997. Nazeer SR, Dewbre H, Miller AH. Ultrasound-assisted paracentesis performed by emergency physicians vs the traditional technique: a prospective, randomized study. Am J Emerg Med. 2005;23(3): 363-367. Sakai H, Sheer TA, Mendler MH, Runyon BA. Choosing the location for non-image guided abdominal paracentesis. Liver Int. 2005;25(5):984-986. Oelsner DH, Caldwell SH, Coles M, Driscoll CJ. Subumbilical midline vascularity of the abdominal wall in portal hypertension observed at laparoscopy. Gastrointest Endosc. 1998;47(5):388-390. De Gottardi A, Thevenot T, Spahr L, et al. Risk of complications after abdominal paracentesis in cirrhotic patients: a prospective study. Clin Gastroenterol Hepatol. 2009;7(8):906-909. Hale Jr BR, Girzadas Jr DV. Application of 2-octyl-cyanoacrylate controls persistent ascites fluid leak. J Emerg Med. 2001;20(1): 85-86. Mallory A, Schaefer JW. Complications of diagnostic paracentesis in patients with liver disease. JAMA. 1978;239(7):628-630.

e5

Abstract: Pediatric vascular access and centeses are core skills in pediatric critical care. Intraosseous access is essential when intravenous access is not readily attainable, but central venous access remains an essential skill. Arterial access is important for both hemodynamic monitoring and blood sampling. All intravascular devices carry the risk of complications, including infection. Pericardiocentesis is important for both diagnostic and therapeutic purposes, and for treating cardiac tamponade. Thoracentesis and

tube thoracostomy are needed for both the treatment of pneumothorax and pleural effusions or hemothorax. Abdominal paracentesis and catheterization is an important technique for managing abdominal compartment syndrome. Key words: central venous catheterization, intraosseous infusion, arterial cannulation, paracentesis, pulmonary artery catheterization, thoracentesis, tube thoracostomy, pericardiocentesis

15 Ultrasonography in the Pediatric Intensive Care Unit ERIK SU, AKIRA NISHISAKI, AND THOMAS CONLON

Critical care ultrasound has been a topic of exploding interest and impact in the pediatric intensive care unit (PICU), with thousands of citations published yearly on use of ultrasound for diagnostic and procedural applications in the critically ill. Since the 1980s technologic advancements in imaging quality and digital storage have resulted in bedside ultrasound equipment that is both portable and powerful. Its use in the ICU results from recognition of its value for procedure guidance, interaction with cardiac intensivists in specialty ICUs, and shared interest from other clinical specialties, such as emergency medicine. Ultrasound is emerging as standard of care for many ICU procedures and is increasingly recognized as a valuable diagnostic tool. Nonprocedural bedside ultrasound applications performed by intensivists contrasts with traditional diagnostic imaging. It is interpreted at the bedside, occurs synchronously with management decisions, and is repeated as necessary by the bedside clinician for serial assessment. Intensivistoperated examination is more focused on assisting the direct and immediate delivery of care, whereas diagnostic imaging by radiologists and cardiologists emphasizes optimizing study detail. The ability of the intensivist to repeat diagnostic point-of-care studies to evaluate the effect of targeted therapies as a form of monitoring and further guide management decisions emphasizes an additional clinical benefit of this unique technology.

Ultrasound Physics and Basics of Image Optimization The physical principles that make ultrasound possible are well described. Ultrasound energy is transmitted in sound waves through substances at a frequency greater than 20 kHz, beyond the range of human hearing. Probes used for bedside ultrasound 114





Intensivist-operated ultrasound is an important adjunct for vascular procedures at the pediatric intensive care unit bedside and is emerging as an important procedural aid across all percutaneous vascular access applications. Accuracy of intensivist-operated ultrasound examining the heart, lungs, abdomen, and cerebral circulation has also been explored. Cardiac ultrasound examinations by intensivists













PEARLS demonstrate high sensitivity for pathology similar to studies performed by imaging specialists. Practice and familiarity with ultrasound’s capabilities, advantages, and disadvantages contribute to responsible and effective practice. Successful translation of ultrasound from training to clinical practice requires thoughtfully designed processes with regard to protocols, recordkeeping, and quality improvement activities.

typically use frequencies in the range of 1 to 20 MHz or more. Ultrasound impulses are transmitted in a coordinated manner from an array of piezoelectric elements in a transducer aimed at a target of interest. Reflected sound from tissue and interfaces between substances of differing density is received by the same piezoelectric array. Based on known speeds of ultrasound in tissue as well as image processing by the ultrasound device, a two-dimensional image is generated and cycled over time to compose a continuous video or cine. Variability in tissue sound velocity and attenuation properties—including refraction, reflection, absorption, or scattering of returning ultrasound energy—may introduce artifact in images (Fig. 15.1).1 A portion of image quality is naturally machine dependent; a variety of parameters are adjustable for image optimization to reduce diagnostic error. Image size modification tools include depth, zoom, and scan area. Though a wide and deep scan area permits capture of a large field of view, short depth or high zoom factor facilitates imaging of small or superficial structures (Fig. 15.2). Gain is also an important setting for image optimization and can be adjusted in several ways. Gain affects the sensitivity of the machine to returning ultrasound energy. Returning ultrasound energy is attenuated as it passes through tissue. Time to return of reflected ultrasound energy determines depth of visualized structures rendered on the screen. Time-gain compensation settings can be adjusted to emphasize gain at different depths based on when the return signal reaches the transducer. On various machines, these may appear as sliding controls or knobs that control near and far gain. At the bedside, gain is adjusted relative to ambient room lighting so that relevant pathology can be visualized (Fig. 15.3). Frequency is an adjustable control that also affects image quality. Higher ultrasound frequencies allow for higher-resolution imaging as a function of improved spatial discrimination of structures.



CHAPTER 15

Ultrasonography in the Pediatric Intensive Care Unit

115

* A

A

* B B • Fig. 15.1



​Ultrasound artifacts. (A) Reflection artifact across the pleural line (asterisk). (B) Mirror artifact across a diaphragm (asterisk).

C • Fig. 15.2

​Effects of depth on ultrasound image, with (B) demonstrating same image as (A) at greater depth and (C) at shallower depth than (A).  

Higher frequencies also result in poorer imaging of deep structures owing to signal attenuation. Conversely, lower-frequency imaging can identify deeper structures at the expense of image resolution (Fig. 15.4). Doppler ultrasound is used in a variety of procedural and diagnostic applications to derive the speed and direction of moving structures in the body, blood flow being a common target. Using color flow Doppler in vascular imaging, ultrasound machines color code areas on the two-dimensional image where tissue and fluid are moving. Returning reflected ultrasound frequency is either Doppler shifted higher (movement toward the transducer) or lower (movement away from the transducer). In the human body, this can correspond not only to blood but also to the motion of effusion fluid and other potential spaces as well. Pulsed wave Doppler samples a small area of the screen for Doppler data that are then plotted along a velocity/time scale (Fig. 15.5). Doppler is most useful when the direction of blood flow is parallel to the ultrasound beam. When flow is not parallel, angle of incidence correction is possible in many commercially available systems. By convention, angle correction is currently not used in echocardiography because of the difficulty in matching beam angles to nonlaminar flows in the heart.

Power Doppler displays amplitude of echoes from moving cells and is less angle dependent than traditional Doppler, thereby permitting improved visualization of vessel branching. For this reason, power Doppler can be a useful modality for vessel identification in procedural guidance. M-mode ultrasound graphs the received ultrasound signal along a one-dimensional beam over time. This is useful for tracking objects in motion to quantify and characterize their movement (Fig. 15.6). Examples of M-mode applications include assessing respiratory variation of the inferior vena cava (IVC),

116

SECTION II



Pediatric Critical Care: Tools and Procedures

A

A

B

B

• Fig. 15.3

​Effects of gain on ultrasound image with (B) demonstrating same image as (A) at lower gain.  

calculating a left ventricular shortening fraction, and evaluating diaphragmatic excursion. When using M-mode, it is important that the operator’s scanning hand remain motionless because operator movement will be captured on the M-mode tracing as well. Sweep speed is an important machine setting for M-mode and pulsed wave Doppler when data are plotted against time. Sweep speed is defined as the speed at which the data tracing is graphed on the screen. In some instances a faster sweep speed is necessary to reduce the amount of time displayed on the screen, thereby “stretching” the tracing so that finer details of movement are captured. This is helpful in echocardiography with a tachycardic heart so that cardiac motion is more easily discerned. Conversely, slowing sweep speed may be helpful in compressing more data into one screen (Fig. 15.7). Slowing sweep speed might be necessary for visualizing changes in diaphragmatic motion or changes in arterial flow over a respiratory cycle.

Transducers Ultrasound transducers appear in a multitude of configurations that serve different specialized functions. They are largely divided into higher-frequency linear array probes and lower-frequency sector type probes, with occasional exceptions. Linear array transducers are a mainstay of procedural guidance for intensivists and other clinical interventionalists (e.g., interventional radiologists) using ultrasound (Fig. 15.8). They are configured

C • Fig. 15.4

​Effects of frequency on ultrasound image with (B) demonstrating same image as (A) at higher frequency and (C) at lower frequency than (A).  

with an array of parallel-oriented piezoelectric elements designed to image a field of tissue below the probe at close proximity to the skin. These probes also tend to emit higher frequencies (7 MHz or above) to visualize the region of interest at high resolution, with some small structure probes operating above 20 MHz. Linear array transducers are optimal for procedural guidance because needle localization is easier with their wide, shallow, and high-resolution field of view. Linear array transducers are also well suited to diagnostic imaging at shallow depths, where higher resolution is preferred. Examples include thoracic imaging and vascular imaging.

A

Ultrasonography in the Pediatric Intensive Care Unit

117



B • Fig. 15.5

​Doppler modalities in ultrasound; pulsed wave (A) and color flow Doppler (B).

1

2

• Fig. 15.6



CHAPTER 15

Procedural Guidance



​ -mode imaging in ultrasound. 1, Two-dimensional image for M cursor targeting. 2, M-mode tracing.

diagnostic imaging in the abdomen, particularly in infants and young children. Curvilinear transducers operate at the same to slightly higher frequency (1–12 MHz) than phased array transducers and are engineered for improved image resolution at the expense of a slower frame rate. For this reason, they excel at looking at relatively nonmobile structures such as the abdomen and its vasculature. Large curvilinear probes are often too large for children; therefore, smaller-radius microconvex curvilinear probes are preferred. These probes are also used for cranial ultrasound in infants owing to their higher frequency and small footprint. Within the ICU, their higher resolution and footprint that is slightly wider than that of a phased array transducer also make them useful for procedure guidance, particularly for vascular access and regional anesthesia. In addition, they may facilitate visualizing the lung and thoracic structures.

Linear arrays range in size and shape. In addition to conventional arrays, in which the transducer face forms the end of the handpiece, linear transducers also include hockey stick–shaped vascular probes, originally designed with improved ergonomics for carotid imaging but also useful for peripheral vascular procedures in children. Sector-type transducers encompass phased array transducers and curvilinear transducers. The sector term refers to the image generated by the concave array appearing as a wedge-shaped sector on the ultrasound screen. Phased array transducers are lowfrequency devices and have a small footprint on the skin, designed for deeper wide-field imaging from a small surface on the body (Fig. 15.9). Because these probes are largely used in cardiac imaging, the ultrasound hardware is optimized for a fast frame rate to capture motion at the expense of image resolution. Because of the small footprint and low frequency, these probes are also optimally suited for transcranial imaging for Doppler flow measurements of the cerebral arteries. This type of imaging requires a low fundamental frequency of approximately 1 MHz, with most probes ranging between 1 and 6 MHz. These probes can also be used for

Venous Access Long before entering the ICU, ultrasound became a mainstay for procedural guidance in interventional procedures. In the hands of intensivists, ultrasound has proved to be a useful adjunct in vascular access, increasing first-pass success and reducing adverse events during central venous catheter (CVC) insertion in a number of large series and meta-analysis of the pediatric literature.2–7 Though the majority of ultrasound trials for vascular access have been conducted with trainees likely less familiar with landmark methods for CVC insertion, the preponderance of the primarily adult evidence supporting ultrasound guidance for internal jugular vein CVC cannulation has added ultrasound guidance to the standard of care in this procedure. Although it appears that ultrasound guidance does improve safe vascular access, its use is a skill that requires training and practice. Novice providers should practice procedures with skilled operators before attempting them independently. Maneuvers for identification of vessel patency are helpful for targeting veins for cannulation. The primary and most reliable method is observing for compressibility of the venous structure and absence of pulsatility. It is important to recognize that

118

SECTION II



Pediatric Critical Care: Tools and Procedures

A

B • Fig. 15.7

​Effect of sweep speed on time-based modalities. In this example of M-mode imaging, the image in (A) is running at a slower sweep speed than the image in (B).  

A

B

• Fig. 15.8

​Examples of linear array transducers. (A) Conventional linear array probes. (B) Carotid vascular (“hockey stick”) probe.  

• Fig. 15.9



​Examples of phased array transducers.

Ultrasonography in the Pediatric Intensive Care Unit

119

A

B • Fig. 15.10

​Sonographic approaches to vascular access. (A) Transverse plane. (B) Longitudinal plane.  

patients with pulmonary hypertension can demonstrate pulsatile venous flow that confounds differentiation between vein and artery. Though this finding is most pronounced in patients with a primary diagnosis of severe idiopathic pulmonary hypertension, it can also be seen in patients with acquired pulmonary hypertension from increased intrathoracic pressure or cardiogenic shock. In the event that compressibility, Doppler flow, or pulsatility cannot identify a venous structure, this may mean that a vessel may no longer be continuous due to sclerosis or thrombosis. Venous thrombi often occupy space in the vessel lumen, obliterate venous flow, and resist external compression. Doppler modalities for identifying vessel occlusion are well described but have limited utility when thrombi are partially occlusive or in shock states when flow is very low. Various approaches to central vessel cannulation using ultrasound exist. Dynamic (real-time ultrasound visualization) approaches to the central veins include the transverse (Fig. 15.10A) or longitudinal (Fig. 15.10B) planes relative to the vessel. An advantage of the transverse approach, with the view plane perpendicular to the vessel axis, is that it improves lateral steering of the needle to avoid arterial puncture. A disadvantage is that needle tip identification is harder, because only a cross-section of the needle shaft is visualized. Identification of the needle tip is crucial for successful access and prevention of inadvertent injury; it is performed by sliding the probe and staying over the needle tip throughout the procedure. This sliding technique will help avoid inadvertent puncture of deeper structures (i.e., the pleura or carotid artery in internal jugular venous cannulation). The longitudinal approach, with the view plane parallel to the vessel axis, has reciprocal advantages and disadvantages. It facilitates observation of the advancing needle tip throughout the initial puncture, but it does not provide a view of lateral structures. For these reasons, transverse visualization of the vessels is often more intuitive and most frequently taught to novice practitioners. The static approach uses the ultrasound to identify the vessel, followed by marking of the site for needle puncture without direct ultrasound visualization of the needle during the attempted cannulation. This permits use of both hands for the technical performance of the procedure; however, patient positioning or physiology may change prior to venipuncture, rendering the mark inaccurate. The dynamic approach has been shown to be superior to the static approach.5 Though most existing literature focuses on internal jugular venous access, experience with other vessel access using ultrasound guidance has been published. There is limited adult patient experience reported on the placement of CVCs in the subclavian vein.8,9 Ultrasound guidance for subclavian venous access may be stymied by relative probe to patient size issues in pediatric patients. In the longitudinal orientation, the transducer occupies the majority of the skin in the infraclavicular space and limits the area for target insertion. The arc of the subclavian vein through the viewing plane also hampers this method. A number of authors have described a supraclavicular approach to the subclavian or brachiocephalic vein.10–13 This approach uses a longitudinal transducer orientation to the vessel and can be limited by space above the clavicle in patients. Ultrasound guidance for femoral vein CVC placement remains contentious. Existing literature is methodologically limited and includes only small series or series that include femoral cannulation mixed with other catheterization sites.14–16 Though data on the efficacy of ultrasound in femoral cannulation is lacking,



CHAPTER 15

operator comfort with ultrasound, similar technical performance, and similar complication concerns with the practice of internal jugular cannulation suggest meaningful benefits for ultrasound guidance in the femoral area.17–19 The literature includes few descriptions of central axillary vein CVCs,20 but this site can be useful for patients with limited options for central access. When inserted in the axillary vein at a position in or distal to the axilla, the catheter takes a similar course to subclavian CVCs. A longer catheter length is necessary so that it reaches from the axilla to the turn of the brachiocephalic vein into the superior vena cava (SVC). In contrast with other CVCs, the redundancy of axillary tissues complicates venipuncture, in which case ultrasound guidance seems especially beneficial.

Arterial and Peripheral Intravenous Access Outside of central veins, ultrasound has also been useful for peripheral intravenous (IV) access and arterial access. For patients

120

SECTION II



Pediatric Critical Care: Tools and Procedures

with difficult peripheral access, ultrasound facilitates targeting and site selection, leading to successful IV placement.21–24 Ultrasound can identify sites difficult to detect from the surface, particularly collateral vessels that may become engorged owing to obstruction of previously cannulated veins. An important consideration is vessel depth, because deep infiltrates are not as readily recognized as shallow ones, resulting in soft-tissue injury and potential complications.25 IV catheters exceeding an inch in length are useful to access deeper vessels and to maximize length of catheter within the vascular lumen. Their insertion benefits from placement in veins with sufficiently long straight courses. Deeper vessels require a steeper angle of insertion, which increases the risk of penetrating the back wall of the vessel. Increasing interest in ultrasound guidance has seen a concomitant increase in the production of longer-length catheters by manufacturers, some of which exceed 2.25 inches at the time of this writing. Following the threading of the IV catheter into the vein under ultrasound visualization may help ameliorate this effect, as the operator can keep the device off the back wall of the vessel. For these reasons, IVs deeper than 1 cm from the skin surface are still often riskier candidate vessels. A number of series have been published on ultrasound guidance for peripheral arterial access. Though historically the arterial pulse has been relied on for arterial catheterization, both adult26 and pediatric studies27,28 demonstrate that ultrasound guidance using methods similar to venous procedures facilitates arterial access. Recent literature also suggests that ultrasound use mitigates both patient and provider characteristics associated with difficult arterial line placement, demonstrating the technology’s ability to bridge the “experience divide” between those experienced and not with landmark-based techniques.27 Ultrasound can also help identify catheter position after CVC placement. Direct visualization of other CVCs in the IVC in addition to umbilical catheters is possible in patients.29–31 However, identifying upper extremity CVCs is difficult because views of the superior cavoatrial junction are obstructed by lung artifact in older children and adults.32–34 Another technique that has been demonstrated to verify CVC position is use of agitated saline.35,36 Agitated saline is produced by aspirating a small blush of blood (,0.1 mL) into 10 mL of saline and rapidly flushing it between two 10-mL syringes attached to a stopcock connected to the CVC until the saline appears homogenous. At this time, the saline is steadily and promptly flushed into the patient so that the contrast is delivered while an ultrasound probe images the superior or inferior vena cava or the right heart. Agitated saline opacifies vascular structures and indicates continuity of the catheter within the proximal vessel or heart. Failure of the bubbles to appear in the venous space suggests that the catheter is not in the correct vessel. Pitfalls in this technique include slow injection, in which contrast may not reach the heart, and the presence of shunting, which could also diminish return of bubbles to the area being examined with ultrasound. Agitated saline techniques should be used with acceptance that small bubbles might travel into the systemic circulation, where there is a small risk that they could obstruct distal organ perfusion.

Umbilical Access Umbilical access can also be facilitated by ultrasound in neonatal patients who may be in the pediatric ICU for specialized therapies, such as advanced cardiac care or extracorporeal membrane oxygenation.29,30 Umbilical catheters can be placed using sagittal

positioning of a small curvilinear or phased array transducer over the IVC as it enters the right atrium. Confirmation of tip position at the IVC–right ventricular (RV) junction is possible using this method because catheters are echogenic. This technique can be employed in umbilical arterial catheter placement by identifying the catheter in the descending aorta at the level of the diaphragm.

Drainage Procedures Ultrasound is a useful adjunct to thoracentesis and thoracostomy. Pleural fluid collection is highly amenable to ultrasound interrogation. Simple effusions appear as an enclosed dark anechoic space sharply differentiated from surrounding tissues and often with underlying consolidated lung appearing to float within the fluid. Ultrasound is useful for characterizing effusion complexity: proteinaceous septations and debris appear echogenic within the effusion. Effusions can be visualized from several locations. During a pleural examination, using a linear array, effusions are recognizable as a dark anechoic space below the parietal pleura. Effusion can also be seen on echocardiographic views, particularly those capturing the dependent areas of the lower thorax, such as subcostal or apical views (Fig. 15.11A). Pleural effusion can also be seen in abdominal examinations of the right and left upper quadrant if the view captures the posterior aspects of the diaphragm. In these cases, the effusion appears as an anechoic space cephalad to the diaphragm. An advantage of imaging an effusion using a phased array or curvilinear probe is that a large portion of the effusion tends to be visible in the far field of the sonographic sector, allowing identification of a deep effusion pocket, its extent, and its complexity (Fig. 15.11B). Color Doppler ultrasound may assist with effusion identification because its motion within the potential space will be detectable. Pleural fluid collections can be easily targeted for drainage using ultrasound to ensure safe needle insertion above the diaphragm. Though dynamic (i.e., real-time) ultrasound guidance can be used during the drainage of an effusion, the static technique (i.e., marking the location for needle insertion) simplifies the procedure without exposing the patient to increased procedural risk. There is presently no pediatric evidence to suggest clear superiority of one technique over another, though an adult study demonstrated a reduction in complications (e.g., pneumothoraces) when using the dynamic technique compared with the static technique.37 Thoracostomy tubes can also be identified in the effusion space, but they are less visible once fluid is drained. Prior to attachment of a thoracostomy tube to suction, visualization of the effusion space and position of the chest tube within can sometimes help in determining whether the tube travels anterior or posterior to the lung. Abdominal fluid collections are also characterized as simple or complex, and drainage of the peritoneal space is possible using ultrasound guidance. An important consideration in performing paracentesis is locating and avoiding the inferior epigastric arteries traveling from the external iliac arteries cephalad along the inside of the anterior abdominal wall at the lateral edge of the rectus abdominis muscle. These vessels are identifiable on ultrasound and the machine can assist in planning a safe needle insertion. With ultrasound, paracentesis is most easily performed from the lateral approach. After appropriately preparing the patient and identifying fluid for drainage, the arteries are identified. Rather than performing a traditional z-track entry, the needle can be introduced obliquely into the peritoneal cavity with the probe oriented



CHAPTER 15

Ultrasonography in the Pediatric Intensive Care Unit

121

*

* B

A • Fig. 15.11



​Examples of pleural effusions seen with a phased array probe. (A) Simple effusion (asterisk). (B) complex effusion (asterisk).

*

the literature,38,39 though some hypothesize that it is beneficial in patients who have difficult landmarks, such as older and obese patients40 and children with spine abnormalities (e.g., scoliosis). Such patients can benefit from ultrasound-guided lumbar puncture due to difficulty in determining surface landmarks and insertion depth secondary to lumbar muscularity or adipose tissue. The technique can be performed under ultrasound visualization, but owing to the near-perpendicular angle of needle insertion into the skin as well as reduced ability to directly visualize the dura, subarachnoid space, and spinal cord as children age, the procedure is often best performed under static guidance using insertion markings (Fig. 15.13). This process involves visualizing the spine in two planes, both sagittally along the spine to identify the appropriate entry level and axially (transversely) across it to center insertion in the midline. Views of the spine can also be used for measuring angle and depth of needle insertion to reach the dural space so that an appropriate needle can be selected.

Diagnostic Modalities Pulmonary Ultrasound



• Fig. 15.12

​Ultrasound-guided paracentesis. Asterisk indicates the needle.

longitudinally over the needle for dynamic guidance (Fig. 15.12). The oblique track helps reduce the chance of leakage, is easier under ultrasound than the z-track, and allows direct visualization of needle entry into the peritoneum to avoid underlying solid and hollow organ puncture. A temporary catheter can then be placed using the Seldinger technique for either one-time or ongoing drainage of ascites fluid.

Lumbar Puncture Ultrasound-guided lumbar puncture provides a limited advantage over landmark-based techniques in children as demonstrated in

Historically, pulmonary ultrasound has been considered impractical owing to air-filled alveoli impeding ultrasound transmission.41 Yet ultrasound images are produced within the thoracic cavity by the complex interaction of the ultrasound beam with interfaces between air- and water-filled structures. Seminal work done by intensivists examining the lungs of critically ill adults and children led to descriptions of pulmonary artifact patterns that reflect lung pathology.42 Age can affect visualization of the thoracic cavity in the pediatric patient. Infants have excellent windows for thoracic visualization because of shallow imaging depths to reach the pleural space, higher body water content, and limited thoracic cage ossification. As children get older, their imaging windows may become more difficult as a result of development, including increasing ossification and decreasing body water content, though imaging windows remain largely accessible. If areas of the pleura are difficult to visualize owing to ribs, rotating the probe parallel to the ribs within the intercostal space can improve the view. Lung motion is still appreciated as artifacts across the pleural plane.

122

SECTION II



Pediatric Critical Care: Tools and Procedures

*

*

A

*

B • Fig. 15.13

​Views of the lumbar spine. (A) Longitudinal view. (B) Transverse view. Asterisks indicate spinous processes.  

Alternatively, a microconvex or phased array transducer can be used to see the pleural line, though these lower-frequency probes often perform better in evaluating deep structures. Thoracic ultrasound of the pleural space identifies overlying skin and soft tissue nearest to the probe, followed by deeper rib and intercostal structures, and then the pleural space. The visceral pleura slides against the parietal pleura through a thin and often invisible layer of normal pleural fluid. Though the exact physiologic mechanisms for ultrasound lung artifacts is often unclear, their similarity to the artifacts associated with pathology in other organ systems suggest their origins. Reverberation artifacts that replicate the pleural line at regular intervals from proximal to distal field are called A-lines. These can appear regardless of pathology or whether tissue or air is underlying the parietal pleura; therefore, they need only be recognized as artifacts in the image. B-lines and Z-lines are generated radially from interfaces near the pleural line and appear to pierce through the lung parenchyma. B-lines reach the end of the viewable sector and obliterate A-lines, whereas Z-lines are limited reverberatory extensions from the pleural line (Fig. 15.14).

• Fig. 15.14 ​Ultrasound of the thorax using a linear probe. 1, A-line. 2, B-line.  

A pattern of increased B-lines suggests increased interstitial lung water. In the adult population, this is consistent with pulmonary edema.43 It is reasonable to infer that it would bear similar implication in children. However, studies also suggest that B-lines indicate the presence of neonatal respiratory distress syndrome or transient tachypnea of the newborn in the neonatal intensive care setting as well as bronchiolitis in children in the emergency department (ED) setting.44–46 Further elucidation of the significance of B-line patterns in critically ill children is lacking. B-lines in children may also represent lung consolidation. Consolidated lung appears increasingly granular with the appearance of liver-like parenchyma, commonly referred to as hepatization owing to the augmented visibility of consolidated lung parenchyma and vascular architecture appearing similar to hepatic structures. In the case of infectious pneumonia, diffuse geographic areas of consolidation, air, and necrosis may develop, resulting in what some authors term the shred sign. The diagnostic value of these lung consolidation patterns in critically ill children has not been well quantified to date. Identification of pneumothorax with ultrasound has been described as a dissociation of the visceral from parietal pleura by ultrasound-obstructing air, such that the parietal pleura is no longer visible (Fig. 15.15).43 Loss of pleural sliding is indicative of pneumothorax; identification of the limit of a pneumothorax as the point where pleural sliding abuts an area where it is absent is described as the lung point. Identification of the lung point is highly specific for pneumothorax.47 M-mode ultrasound is also useful for identification of pneumothorax. Placement of the Mmode cursor through the pleural surface characterizes movement of the lung parenchyma as a granular-textured echogenic area distal to the pleural line. In contrast with the more horizontal linear pattern of the proximal chest wall, this assumes a seashorelike appearance (Fig. 15.15B). When the M-mode cursor overlays pneumothorax, no lung parenchyma movement is seen, and the entire M-mode tracing over time appears to be a pattern of horizontal lines, otherwise described as a barcode or stratosphere sign (Fig. 15.15D). Bedside ultrasound used to detect pneumothorax among high-risk neonates has been shown to be highly accurate and timely.48 Ultrasound views of diaphragm movement can confirm tracheal placement of an endotracheal tube (ETT) during intubation. This is potentially useful in the management of a ventilated patient requiring rapid confirmation of tracheal placement, such

A

C

B

D



CHAPTER 15

Ultrasonography in the Pediatric Intensive Care Unit

123

• Fig. 15.15



​Identification of pneumothorax via ultrasound. (A) Normal two-dimensional (2D) lung ultrasound image. (B) Corresponding M-mode image to (A). (C) Note lack of B-lines in pneumothorax in 2D image. (D) Corresponding pneumothorax M-mode image to (C).

as in the case of a difficult airway patient for whom removal of the tube and reintubation could be dangerous. In this instance, subcostal views of the diaphragm capturing motion of each leaflet while the patient receives positive-pressure breaths can be performed (Fig. 15.16). Confirmation that at least one leaflet moves with large positive-pressure breaths will help verify that the tube is in the airway. Lack of excursion on one side might suggest main-stem bronchus occlusion from secretions or inadvertent main-stem bronchial intubation of the contralateral lung. This technique has been used to confirm tube placement in the operating theater49 as well as in the pediatric ED.50 In the neonatal population the tube is visualized within the trachea in the sagittal plane from the parasternal view typically used for evaluation of the aortic arch and pulmonary arteries. Dennington and colleagues were able to accurately gauge depth of ETT position in neonates with high accuracy.51 In larger infants and older children, this technique has not been successful owing to thoracic growth and ossification.

Airway ultrasound to visualize tracheal stenosis and appropriate ETT fit have also been described, though this is still an exploratory application since it depends on successful identification of the ETT, cuff, and potential areas of stenosis using ultrasound in the air-filled trachea. The accuracy of this technique is still wanting of larger-scale population studies. Some authors have described use of saline to aid in visualization, though there is the potential risk of damaging the cuff in this application.52 Airway ultrasound is readily performed at the level of the larynx or midtrachea (Fig. 15.17) with fair visualization of laryngeal structures. Emerging applications for this modality, in addition to ETT sizing and assessment of tracheal caliber, include identification of vocal cord paresis, identification of landmarks for cricothyroidotomy, assessment of esophageal intubation, and identification of structures for transcutaneous injection, among others.53 Use of bedside ultrasound to assess diaphragm paresis has also been described.54–58 Assessment of spontaneous diaphragm excursion using ultrasound in the oblique coronal (described previously)

124

SECTION II



Pediatric Critical Care: Tools and Procedures

*

B

A • Fig. 15.16

​Subcostal image of the diaphragm. (A) Asterisk indicates the diaphragm that is identifiable on M-mode imaging (B).  

• Fig.

15.17 ​Laryngeal ultrasound—transverse view at the level of the cricothyroid membrane angled superiorly through the larynx. Asterisk indicates the true vocal cord.  

or sagittal planes accurately demonstrates diaphragm paresis. This technique is also likely useful for patients with diaphragm paralysis from a variety of causes, including protracted neuromuscular blockade or intrinsic neuromuscular dysfunction. Emerging literature also recognizes pediatric diaphragmatic atrophy with intubation and mechanical ventilation similar to changes seen in adult critically ill patients.59 In adults, such changes in diaphragm thickness have been associated with extubation failure and increased mortality; such associations have not been confirmed in pediatric patient populations.

Abdominal Ultrasound Assessment of abdominal pathology is frequently confounded by nonspecific complaints, particularly in sedated patients. Ultrasound as a noninvasive technology has potential for evaluating abdominal pathology without the radiation exposure of computed tomography (CT). However, as is the case with pulmonary pathology, air within the abdominal cavity creates challenging obstacles to ultrasound interrogation. When air causes artifacts,

determining whether the air is within the peritoneum or in the bowel is difficult and is a limitation of abdominal ultrasonography. Air localized within the liver vasculature and bowel wall hypervascularity have been associated with necrotizing enterocolitis in the at-risk neonate.60–62 Similarly, fluid in the abdomen can appear within or outside the intestinal lumen. The normal appearance of air artifact and stool is absent in fluid-filled ileus, resulting in ultrasound visualization of distended, anechoic bowel loops. Peritoneal fluid as a result of ascites, hemorrhage, or peritoneal dialysis fluid is commonly seen in acute care settings. A focused assessment with sonography in trauma (FAST) is performed in four abdominal windows where dependent fluid could appear from a traumatic injury. The probe is usually a curvilinear or phased array transducer. The FAST examination is widely used in adult trauma resuscitation for identification of intraperitoneal fluid, likely either from hemorrhage or ruptured viscus.63–65 Large-scale efficacy studies and meta-analyses have reduced initial enthusiasm for the test, citing inadequate specificity, operator and patient variability, and debatable impact on imaging with abdominal CT or patient outcome as major vulnerabilities. In children, the sensitivity of the test is as poor as 52% in some series66; thus, its influence on changing management has been questioned.67 Meta-analyses published by the Cochrane library suggest similar concerns about the FAST examination for adult patients as well.68,69 When seen, anechoic (dark) fluid in the abdomen may indicate intraperitoneal injury. However, solid-organ injury may result in small to no detectable fluid. It is also important to note that the diagnostic characteristics of the FAST examination in the ED are commonly performed without any elevation of the upper torso. When the torso is elevated in the ICU setting, fluid will likely settle in the more dependent lower quadrants of the abdomen. The FAST examination is commonly referred to as the focused assessment for free fluid when fluid is suspected and assessed using ultrasound in nontrauma patients.

Right Upper Quadrant (Fig. 15.18A) The hepatorenal recess (Morison pouch) is the most dependent part of the peritoneum in the supine patient—which, again, can change with repositioning of the patient. Fluid can be seen between the retroperitoneal kidney and the intraperitoneal liver or above the liver as well. The transducer is placed near or below the

A

B

C

D



CHAPTER 15

Ultrasonography in the Pediatric Intensive Care Unit

• Fig. 15.18 E lower ribs of the thorax in the posterior axillary line to visualize the right kidney, with the indicator directed toward the patient’s head. Fluid in the Morison pouch, inferior pole of the kidney, or the perihepatic space suggests intraperitoneal injury. The diaphragm is also seen in this view, and pleural effusion or intrathoracic hemorrhage in the trauma setting can also be identified or suspected when a classic mirror artifact of the liver due to the diaphragm is not visualized. The probe is usually oriented coronally but can be oriented axially with respect to the patient; thus, the entire organ should be scanned. If views from the posterior axillary line are difficult, the right kidney can be visualized in the anterior sagittal plane by placing the transducer at the lower edge of the costal margin just lateral to the midclavicular line with the

125



​The focused assessment with sonography in trauma (FAST) exam. (A) Right upper quadrant view. (B) Left upper quadrant view. (C) Longitudinal and (D) transverse views of the bladder. (E) Subcostal cardiac view.

indicator pointed toward the patient’s head. In this view, the Morison pouch can be visualized through the liver.

Left Upper Quadrant (Fig. 15.18B) The splenorenal recess can be visualized from the left flank at the level of the posterior axillary line as well. It is visualized similarly to the view in the right, with the probe indicator positioned toward the head for a coronal view of the kidney and surrounding spaces. Anterior windows are usually not feasible because of stomach contents. The left kidney is more cephalad in the abdomen than the right, and views from above the costal margin may be necessary. In this view, pleural effusions can also be visualized.

SECTION II



126

Pediatric Critical Care: Tools and Procedures

Pelvis (Figs. 15.18C and 15.18D) Particularly if a patient has been upright or inclined after trauma, fluid can accumulate in the pelvis in a manner not seen in the upper quadrants. As with visualization of the kidneys, the bladder can serve as an easily identifiable landmark from which to reference regional anatomy and discern pathology. The bladder is imaged in both the axial and sagittal planes. Fluid posterior to the bladder in the male or posterior to the bladder or uterus in the female patient suggests pathology. Subcostal Cardiac (Fig. 15.18E) The heart is visualized in the FAST scan from the subcostal margin below the xiphoid process, with the probe aimed toward the patient’s left shoulder and the indicator oriented toward the right flank if the machine remains set in a radiology convention setting with the screen indicator to the operator’s left. The heart is imaged for pericardial effusion in this view, where dark fluid would appear adjacent to the heart. Using FAST bladder views, verification of the presence of a urinary catheter can also be performed by visualizing the catheter itself or the water-filled balloon of the Foley catheter. A large volume in the bladder in an anuric patient indicates obstruction or malplacement of the urinary catheter. Though several authors have defined methods for calculating bladder volume, its variable geometry precludes easy approximation of volume. However, a practitioner can judge whether there is urine in the bladder despite efforts at diuresis or catheterization. Solid echogenic structures in the distended abdomen suggest that the abdomen is filled with a foreign mass (tumor), enlarged or swollen viscera, or a collection of a solidifying substance such as exudative ascites or clotting blood. Such findings should discourage needle drainage of a space in the evaluation of abdominal distention or intraabdominal hypertension unless there is also a large volume of free fluid. In shock management evaluation of renal perfusion may be informative as a surrogate for shock severity. A marked difference between systolic and diastolic renal arterial flow may suggest hypoperfusion.70,71 However, examining this phenomenon to date has not consistently shown efficacy in evaluating shock states.72–78 Whether this assessment will prove useful in children remains to be determined.

Cardiac Ultrasound Imaging specialists often have a wide selection of phased array transducers for imaging the heart. Large adult-sized transducers, with more sophisticated technology and lower-frequency transmission for adequate penetration, suit adolescents and young adults well. Smaller transducers allow use of slightly higher frequencies for imaging infants and young children, and their smaller faces permit better skin contact when the probe is held at shallow angles to the skin. Therefore, it is important that adequate equipment be available for accurate bedside ultrasound cardiac evaluation of critically ill patients. Imaging of the heart is performed using locations, or windows, on the body where acoustic transmission to the heart is adequate and less encumbered by effects of body position and tissue interference. These include the subcostal window immediately below the xiphoid process, the parasternal window to the left of the patient’s sternum, and the apical window near the patient’s point of maximal cardiac impulse, typically below the left pectoralis major

muscle. These windows form standard echocardiography views (Fig. 15.19). Of note, different groups (i.e., cardiology, emergency medicine, critical care medicine) may use different screen indicator orientations during cardiac ultrasound evaluation. Within the scope of this text, the screen indicator is at the top right of the screen for all cardiac views. Subcostal windows can be used to visualize the base of the heart either longitudinally, such that all four chambers are seen (Fig. 15.19A), or in cross-section, such that only the atria or ventricles are seen. The probe is placed in the subxiphoid region and beam aimed toward the patient’s left shoulder. For a longitudinal view, the probe indicator is directed toward the patient’s left flank, or approximately the 2 o’clock to 3 o’clock position with the top of the clock oriented toward the patient’s head. For a transverse view, the probe indicator is directed at the patient’s head and the view aligned across the chambers of interest. From the transverse view the probe can also be directed directly posteriorly through the inferior vena cava (Fig. 15.19B) as it passes into the right atrium to assess its size variation through the respiratory cycle or through the aorta for evaluation of flow. Any view of the heart should be more than a single image; fanning the transducer beam through the organ can provide the most complete impression of the heart. Septal defects are most easily visualized from the subcostal position using Doppler sonography because flow across them is most parallel to the ultrasound beam. In addition, the window is closest to the base of the heart and provides excellent imaging of effusion, particularly if the patient is slightly inclined head up. This window also has an advantage in small children with multiple dressings or monitoring devices on the chest because the subcostal window may be the only area not obscured. Subcostal views are also important during active resuscitation from cardiac arrest when chest compressions must have priority (see section on Cardiac Arrest). Though subcostal windows benefit from not having intervening lung tissue obscure the heart, they can be hindered by interference from a gas-filled stomach and/or bowel. In the case of internal interference from air-filled viscera, insonating the base of the heart from a position slightly overlying the right lobe of the liver can sometimes improve the view. Subcostal views may also be difficult in patients with substernal chest tubes or ventricular assist devices. Views from the parasternal windows are commonly acquired from the third and fourth intercostal interspaces near the sternum on the patient’s left chest. To acquire the parasternal long-axis view (Fig. 15.19C), the transducer indicator is toward the patient’s right shoulder and aligned along the major axis of the left heart. From this view, the left atrium, mitral valve (MV), left ventricular chamber, and left ventricular outflow tract (LVOT) are readily visible in continuity with the right ventricular outflow tract that appears anterior to the LVOT. This view can be modified to image the right ventricular inflow view by fanning the transducer anteriorly through the right heart. The right atrium and IVC are usually visible and the tricuspid valve opens into the right ventricle anterior and caudad to the right atrium. The SVC may be occasionally visualized in infants, though it is frequently obscured by the lung in an older patient. The parasternal window can also be used for visualizing the heart in a plane perpendicular to its major axis, or the short-axis view (Fig. 15.19D). Short-axis views are performed with the transducer indicator aligned toward the patient’s left shoulder. There are several short-axis views that span the length of the heart from atrium to apex. Imaging the ventricle at the midchamber

1

2 3

* A

B

4

3

2

2

1 1

C

D V

1

3 1

2

4 4

2

3

E

F

C,D,E

F

• Fig. 15.19

​Basic cardiac views. (A) Subcostal long-axis view where the liver (1), right ventricle (2), and left ventricle (3) are visible. (B) Inferior vena cava (asterisk). (C) Parasternal long-axis view where the left atrium (1), left ventricle (2), aorta (3), and right ventricular outflow tract (4) are visible. (D) Parasternal short-axis view at the midpapillary level where the left ventricle (1) and right ventricle (2) are visible. (E) Parasternal short axis at the aortic valve level where the aortic valve (1), tricuspid valve (2), atrial septum (3), and left coronary artery are visible (4). (F) Apical four-chamber view visualizing the left ventricle (1), left atrium (2), right ventricle (3), and right atrium (4).  

A,B

128

SECTION II



Pediatric Critical Care: Tools and Procedures

level is useful for the pediatric intensivist to assess qualitative left ventricular function, as this location is where radial contractility is most pronounced. In this view, the beam transects the ventricle such that the papillary muscles are the primary intraventricular structures visible, with the inferoseptal papillary muscle appearing in the lower left and the anterolateral papillary muscle appearing in the lower right of the image. A short-axis view of the heart can also be performed at the aortic valve level. In this view the aortic leaflets are visible and roughly form a lambda sign in the middle of the appropriately centered image (Fig. 15.19E). With careful targeting, the left or right coronary root is sometimes visible in this view. On the screen, the left atrium is directly posterior to the aortic valve (6 o’clock) and moving clockwise is the right atrium (8 o’clock), tricuspid valve (9 o’clock), right ventricle (12 o’clock), and pulmonary valve (2 o’clock) with the pulmonary artery and its bifurcation sometimes visualized at the 4 o’clock position. Apical views (Fig. 15.19F) are highly dependent on thoracic and abdominal structures. Increased intrathoracic pressure, as in asthma or high airway pressure ventilation, can cause the apex of the heart to move caudad and medially. Hyperinflated lung may obscure the apex. Conversely, high intraabdominal pressure can push the diaphragm cephalad and displace the heart cephalad and laterally. Apical views are insonated near the point of maximal impulse at the caudad aspect of the left pectoralis major muscle. The axis of the probe should be aligned with the major axis of the heart and point toward the center of the mediastinum. For the apical four-chamber view, the indicator is oriented toward the left flank, usually between the 2 o’clock and 3 o’clock positions. From this view the four chambers and both atrioventricular valves of the heart should be visible. In addition to the four chambers, this view allows visualization of both atrioventricular valves and their movement. The left heart appears on the right of the screen and atria appear at the bottom when the probe face appears at the top of the screen. From the four-chamber view the transducer can be fanned anteriorly so that the beam intercepts the LVOT for interrogation of outflow velocity. This is called the apical five-chamber view. From the four-chamber view, the probe can be rotated 60 degrees counterclockwise to obtain a two-chamber view of the left heart (left ventricle and atrium) for assessment of function. Cardiac ultrasound can provide assistance in titrating fluid, inotropes, and vasopressors for persistent shock in children.79,80 Initial pediatric experience has demonstrated that its use is feasible and associated with good outcome, complementing available adult data describing the utility of intensivist-driven cardiac ultrasound for hemodynamic assessment. It is reasonable to surmise that the value of focused cardiac ultrasound noted in adults81 might be similar or better in children, as views of the heart may be better given pediatric body habitus. Most literature related to cardiac ultrasound for hemodynamic assessment in children has focused on volume status assessment. Adult echocardiographers and pediatric nephrologists have used the collapsibility and distensibility of the compliant IVC as a noninvasive surrogate for volume status and estimation of dry weight for dialysis patients, respectively.82,83 Critical care physicians are most often interested in identification of patients who are volume responsive. Volume responsiveness means that a fluid challenge will result in a subsequent increase in stroke volume. Within literature, volume responsiveness is typically described as an increase in stroke volume by greater than 10% to 15% in response to a 10 to 20 mL/kg fluid bolus. Surrogate markers for fluid responsiveness can be categorized as static or dynamic parameters. Static parameters are point-in-time

measurements—such as heart rate, central venous pressure (CVP), and blood pressure—and poorly correlate with fluid responsiveness. Dynamic measures are separated in time and relative to a physiologic perturbation. Most commonly, they are measures that are taken in response to changes in intrathoracic pressure during respiration. Respiratory variation of aortic outflow velocity appears to be a promising measure indicating fluid responsiveness in critically ill pediatric populations.84 The diameter of the IVC is usually assessed in the sagittal view about 2 to 3 cm distal to the inferior cavoatrial junction in adults. At this point the vessel diameter can be measured using M-mode or two-dimensional (2D) methods. Available evidence regarding IVC assessment for volume status is largely related to two physiologic conditions: (1) the patient who is pharmacologically paralyzed and intubated receiving a set tidal volume at near normal airway pressure and (2) the spontaneously breathing patient. IVC respiratory variation exceeding 12% to 18% in the neuromuscularly blocked adult patient85,86 has been described as a threshold for volume responsiveness. Current convention in the adult is the use of maximum diameter observing for changes greater than 12%. Similar criteria have not yet been established in children, and evaluating changes in caliber of smaller pediatric vessels may be prone to more measurement error. It has also been demonstrated that increasing airway pressure decreases respiratory variation and can mask hypovolemia. In the pediatric operating room, titration of positive-pressure ventilation as well as initiation of inhalational anesthesia changes the collapsibility of the IVC.87 Evidence indicates that IVC behavior is not equivalent between patients receiving sedation and positive-pressure ventilation compared with those who are spontaneously breathing. Using a ratio of the size of the IVC to aorta (IVC:Ao) diameter in a transverse view may provide a useful assessment of volume status in children and avoids the challenge that results from the range of sizes in children. An IVC:Ao ratio of 0.8 or less appears to suggest clinically significant dehydration, whereas a ratio greater than 1.2 suggests hypervolemia.82 Fluid responsiveness was not described; this study targeted diagnosis rather than evaluation of physiologic response to therapy. However, in critically ill children, limited data suggest that IVC:Ao and IVC collapsibility are poor surrogates for CVP.88 In addition, despite its use in septic shock algorithms as a surrogate marker for intravascular volume depletion, CVP measurements have come under considerable scrutiny given their poor accuracy.89 Assessment of left heart performance as a surrogate for volume status has also been an area of interest in adults and children. Velocity of blood flow across LVOT has been used as a surrogate of stroke volume and indicator of intravascular volume status. Assessment of the LVOT using Doppler ultrasound is performed from the apical five-chamber view observing velocity of left ventricular ejection (Fig. 15.20). Stroke volume can be approximated by obtaining the Doppler velocity tracing of flow across the LVOT over time, calculating its integral, velocity time integral, and multiplying this by the measured cross-sectional area of the LVOT. The cross-section of the LVOT is usually measured from the parasternal long-axis view using the diameter measured between the aortic valve leaflets in midsystole. Using the estimated stroke volume and multiplying by the heart rate allows approximation of cardiac output. The LVOT Doppler tracing can also provide a beat-to-beat assessment of changes in stroke volume through the respiratory cycle. Respiratory changes in peak LVOT velocity exceeding 14% appear to identify volume responsiveness in intubated pediatric ICU patients.90,91 In assessing



CHAPTER 15

Ultrasonography in the Pediatric Intensive Care Unit

129

*

A

B • Fig. 15.20



​Doppler interrogation of the left ventricular outflow tract (LVOT). (A) Apical five-chamber view with LVOT identified (asterisk). (B) Pulsed-wave Doppler of the LVOT.

variability in the Doppler flow tracing, it is advisable to slow the sweep speed of the machine so that multiple cardiac cycles—in particular, aortic flow in systole—are observed through the respiratory cycle. Variability of 14% is thought to indicate volume responsiveness. Pitfalls of this technique include limited apical views owing to lung inflation and patient habitus. Further concerns particular to pediatric patients include the need for a smaller phased array transducer to maintain skin contact and small LVOT for appropriate placement of the Doppler cursor, made more challenging in the dehydrated child with a hyperkinetic heart. Effusion in the pericardial space appears similar to pleural effusion as a largely dark and anechoic space separating the heart from the reflective pericardium (Fig. 15.21). Effusion may not be concentric and instead collect in dependent areas; therefore, V

subcostal windows are often ideal for visualizing them. Effusions can also be complex as a result of accumulated protein in an empyema, solidification of hemorrhage, or presence of tumor. Tamponade is a clinical diagnosis; however, the presence of an effusion and subsequent changes to cardiac morphology and function suggestive of tamponade physiology are detectable using ultrasound. The most specific indicator of tamponade is IVC engorgement secondary to impeded venous return to the heart. Other ultrasound findings consistent with tamponade may include collapse of the low-pressure right heart chambers during filling periods in the cardiac cycle, namely, late diastole/early systole in the right atrium and early diastole in the right ventricle.92 Respiratory variation in the left ventricular inflow is also associated with tamponade physiology, as demonstrated by noticeably reduced mitral inflow during inspiration. Conversely, inflow across the tricuspid valve may increase during inspiration.

Pericardiocentesis Imaging at the time of pericardiocentesis often facilitates effusion drainage. Use of a phased array probe in the apical position may not allow visualization of the needle but permits monitoring effective drainage of the effusion and wire placement for drain placement. Injecting a small quantity of agitated saline into the pericardiocentesis needle for sonographic contrast can help confirm that a needle is in the pericardial space rather than a vascular space, particularly if an effusion is bloody.93 Dynamic visualization techniques have been described for pericardiocentesis using a linear array transducer placed in the subxiphoid area in children94 or the left parasternal area in adults,95 with the indicator pointed cephalad. These have been limited series; whether these techniques have broad applicability in the pediatric setting remains undetermined.

*

• Fig. 15.21



24cm ​Pericardial effusion. Asterisk indicates effusion.

Left Ventricular Function Although the best metric for characterizing cardiac contractility remains elusive, acute care practitioners can estimate cardiac function using qualitative and quantitative markers with reasonable accuracy when compared with echocardiography specialists.96,97 The motion of the left ventricle (LV) assessed through the cardiac cycle is useful for this assessment. Visualizing the LV across the center of the chamber in multiple views, a sonographer can visually approximate or directly measure the excursion of the ventricular walls. These values are then compared with known standards

130

SECTION II



Pediatric Critical Care: Tools and Procedures

B

A • Fig. 15.22

​E-point septal separation (EPSS). (A) Cursor alignment in two-dimensional imaging for (B) M-mode measurement of EPSS.  

(e.g., a normal left ventricular fractional shortening [FS] measures between 25% to 45% of the LV end-diastolic diameter). This measurement is optimally performed quantitatively in the parasternal short-axis view at the midchamber (papillary muscle) level. Area-based measurements of LV systolic function are also useful. These include changes in the cross-section of the LV seen in the short-axis or apical views (ejection fraction [EF]) by Simpson’s method of discs. Area-based calculations are advantageous to single-dimension assessments such as FS because they reduce the effects of regional wall motion abnormalities on accuracy of EF measurement. These calculations are prone to error from inaccurate inclusion of intracavitary structures such as the papillary muscles and trabeculae as well as various artifacts. Apical views are also challenging in a child without appropriately sized transducers or if the lungs are hyperinflated, which can compromise accurate assessment of the LVEF. Substantial mentored practice is strongly recommended for skill development and accurate results. M-mode modalities, such as E-point septal separation (EPSS) and MV annulus plane of systolic excursion (MAPSE) can also approximate systolic function. EPSS assesses the relative motion of the MV anterior leaflet during diastole (Fig. 15.22). In the

parasternal long-axis view, at early diastole the leaflet is readily visualized as being more proximal to the probe and moving toward the septum both in the early passive phase of ventricular filling and late phase with atrial contraction. In the failing heart, end-systolic volume increases and diminishes the gradient across the MV for flow into the LV during diastole. Therefore the excursion of the anterior MV leaflet is less pronounced and does not come as close to the septum. The distance between the leaflet and septum is measured in the parasternal long-axis view by placing the M-mode cursor across the MV tips of the leaflets. In the adult a normal EPSS is less than 6 mm and an abnormal one exceeds 10 mm. Normal values have been published in pediatric populations.98,99 MAPSE is assessed from the apical view of the MV by placing the M-mode cursor across the lateral end of the MV annulus (Fig. 15.23). In M-mode the vertical excursion of the MV apparatus is quantified as an approximation of the longitudinal contraction of the heart. The septal end of the MV can also be measured, though this tends to be a better indicator of biventricular function. Normal values have been published for pediatric populations.100

* * B

A • Fig. 15.23

​Mitral annulus plane of systolic excursion (MAPSE). (A) Cursor alignment in two-dimensional imaging for (B) M-mode measurement of MAPSE. Vertical excursion of the lateral side of the mitral valve annulus (asterisk) is measured to determine MAPSE.  

Right Ventricular Function In the pediatric ICU, assessment of the RV can provide valuable information about the effects of pulmonary vascular resistance changes and mechanical ventilation on cardiac performance. However, assessment of the RV can be difficult because of its position closer to the sternum and its triangular shape straddling the LV and making characterization of its movement through the cardiac cycle difficult. Subtle signs of RV dysfunction can be seen with dilation and pulsatility in the IVC, although some pulsatility can be normal in both the IVC and subclavian veins. From the apical position or parasternal right ventricular inflow or parasternal short-axis view at the aortic valve level, identification of a tricuspid valve regurgitant jet can potentially be used to evaluate RV systolic pressures using continuous-wave Doppler. A more complete explanation of methods for measuring tricuspid regurgitation can be found in dedicated comprehensive echocardiography texts.101 Other clues to RV dysfunction include leftward interventricular septal deviation. Septal deviation can be visualized in multiple views; however, it is most prominent and appropriately evaluated in the parasternal short-axis view at the mid-chamber (papillary muscle) level. Septal position can reveal RV volume overload from pressure overload as a cause of RV dysfunction (Fig. 15.24). Volume overload is typically characterized by septal deviation occurring in diastole but not systole, resulting in the LV assuming a predominantly circular conformation during systole. It is easy to make the septum look falsely flat by not having the plane of imaging perpendicular to the axis of the ventricle. Therefore, if a flat septum is seen, an effort should be made to sweep through the heart, sliding from base to apex, to ensure that the potential for a false-positive finding is limited. In the progression of RV failure to pressure overload of the ventricle, septal deviation is seen through the entire cardiac cycle. Cardiac Arrest During cardiac arrest, ultrasound may help to identify reversible causes, including critical hypovolemia and pericardial effusion with tamponade physiology.102 In the modern PICU, ultrasound machines are easily deployed to the patient’s bedside to augment ongoing resuscitation efforts. Ultrasound use in cardiac arrest requires



CHAPTER 15

Ultrasonography in the Pediatric Intensive Care Unit

131

particular attention to patient and provider. Ultrasound gel conducts electricity and must be wiped from the skin before defibrillation. The gel makes contact surfaces slippery for clinicians providing chest compressions and may dislodge pads and monitoring. Echocardiography should be performed briefly during pulse checks to minimize interruptions in chest compressions. Primary use of subcostal views minimizes interference with compressors and defibrillator pads on the chest. Apical views may also be possible but are often more difficult to acquire, which may delay identifying reversible causes of arrest. The sonographer should not be the code team leader. If the code leader is the only provider with ultrasound skills, temporarily transfer the code leader role to focus on the ultrasound study. While there are studies that demonstrate feasibility and value of ultrasound during resuscitative efforts,103 several studies have shown that the use of ultrasound may be associated with longer interruptions in chest compressions.104,105 Caution should be exercised so that ultrasound does not negatively impact CPR performance during cardiac arrest. Best practice is to obtain the cardiac views during planned pulse checks or chest compression provider switches.106 Adult emergency medicine providers are exploring the use of transesophageal ultrasound during resuscitation. The potential advantages of this approach are no need for chest compression interruptions, capability to visualize LV outflow obstruction to optimize the hand position of chest compression providers, and ability to monitor quality of compressions performed.107 These benefits are balanced with known risks associated with transesophageal ultrasound and material costs for the technology. Absent cardiac contractility, termed cardiac standstill, has been described as highly indicative of unlikely return of spontaneous circulation103,108,109 in adults. In contrast, in the pediatric setting, recovery of cardiac function after standstill has been described.110 In a study of providers polled on interpretation of potential cardiac standstill videos, the providers demonstrated only moderate agreement in correctly identifying cardiac standstill.111 This can be practically even more difficult when only a few or even just one limited 2D view is obtained during resuscitation. For these reasons, currently there is a lack of supportive evidence for using ultrasound to primarily decide the termination of resuscitation efforts in pediatric practice. While bedside ultrasound is a robust implement for identifying reversible causes, caution should be exercised regarding how ultrasound result should be used to guide resuscitative efforts.

Neurosonology 1 2



• Fig. 15.24 ​Chronic severe right ventricular failure. Note thickened right ventricular wall (1) and interventricular septum that bows into the left ventricle (2).

Neurosonology encompasses sonography of the central nervous system, peripheral nervous systems, and cerebral circulation.112 In the PICU, neurosonology use has most commonly been described in evaluation of cerebral blood flow. More recently, there has been interest in ultrasound of the ocular orbit and optic nerve sheath. Distention of the optic nerve sheath demonstrated via direct ophthalmic ultrasound can potentially provide evidence of increased intracranial pressure.113 The eye is imaged from the front over the closed eyelid to visualize the sheath approximately 3 mm behind the vitreous humor and retina. Careful measurement of the maximum sheath diameter is important to decrease the risk of underestimating pressure. Copious gel, preferably made for ophthalmic use, is advised to reduce eye irritation. There are inconsistent data about normal or abnormal values for children of different ages or different intracranial disorders. A similar view for the posterior retinal space has also been described for retinal hemorrhages in

132

SECTION II



Pediatric Critical Care: Tools and Procedures

child abuse cases and other traumatic and infectious pathologies involving the eye.114 The diagnostic utility of this application remains under investigation. Use of transcranial Doppler for assessing cerebral blood flow has become common for monitoring vasospasm in patients with subarachnoid hemorrhage and other disorders encountered in the adult neurocritical care unit. There is considerable interest in its application in pediatric neurocritical care as a noninvasive assessment tool for cerebral perfusion. Insonating the middle cerebral artery from the temporal window lateral to either eye requires a low-frequency transducer that can operate near 1 MHz. It is also possible to insonate the anterior cerebral artery in patients with an open anterior fontanelle, and the vertebrobasilar system can be insonated from the foramen magnum with some occipital support and care not to disrupt a critical airway or cervical spine. Color Doppler is used to image the cranial vessels; pulsed-wave Doppler can subsequently be used to identify velocities in the vessel of interest. From the peak systolic (SBF) and diastolic flow (DBF), the resistive index (RI) can be calculated as RI 5 (SBF – DBF)/SBF or the pulsatility index as PI 5 (SBF – DBF)/(time-averaged mean velocity). An increasing RI or PI is suggestive of vascular constriction and a greater difference between systolic and diastolic flow.

Translation to Practice In 2011 an international consensus document on training in critical care ultrasound was endorsed and published by 13 critical care societies.115 These societies agreed that general critical care ultrasound, including basic-level echocardiography, should be a required component of training for critical care trainees. The Society of Critical Care Medicine (SCCM) Ultrasound Certification Task Force subsequently published suggestions116 for curricular development and programmatic infrastructure based on previously published and successfully implemented guidelines from the American College of Emergency Physicians.117 Guidelines for ultrasound applications in critical care practice have also been published by the SCCM.118,119 Societal guidelines regarding critical care applications infrequently discuss pediatric considerations. This has allowed pediatric critical care providers the opportunity to design local curricula based on applications to guide treatment in commonly encountered clinical scenarios amenable to ultrasound interrogation. Within pediatric critical care, translating ultrasound education to the care of critically ill children has proven beneficial in the clinical setting, bringing meaningful findings that change management.120 Recent studies based on surveys suggest that most academic pediatric critical care departments in the United States have an ultrasound machine, and many providers have access to ultrasound education and are performing both diagnostic and procedural studies.121,122 These studies suggest standardized infrastructure—institutional credentialing processes, consistent documentation practices, secured image storage, and quality assurance systems—which are frequently underdeveloped across PICUs. These are essential infrastructure elements for successful ultrasound practice implementation in PICUs.

Conclusion Ultrasound use is increasingly common in pediatric critical care medicine for procedural guidance and real-time clinical assessment at the bedside. Optimal ultrasound use in the PICU requires an

infrastructure for provider education and credentialing process, quality assurance, documentation, and image storage. Implementation of critical care ultrasound will have a powerful impact in clinical care: better procedural safety, timely and accurate understanding of patient physiology, and, ultimately, better patient outcomes. More research is needed to determine the role of ultrasound in the ICU, but emerging indications for procedure and diagnosis and more incorporation into daily clinical care are likely.

Key References Aouad MT, Kanazi GE, Abdallah FW, et al. Femoral vein cannulation performed by residents: a comparison between ultrasound-guided and landmark technique in infants and children undergoing cardiac surgery. Anesth Analg. 2010;111:724-728. Barbier C, Loubi res Y, Schmit C, et al. Respiratory changes in inferior vena cava diameter are helpful in predicting fluid responsiveness in ventilated septic patients. Intensive Care Med. 2004;30 (9):1740-1746. Bouferrache K, Amiel J-B, Chimot L, et al. Initial resuscitation guided by the Surviving Sepsis Campaign recommendations and early echocardiographic assessment of hemodynamics in intensive care unit septic patients. Crit Care Med. 2012;40:2821-2827. Brass P, Hellmich M, Kolodziej L, et al. Ultrasound guidance versus anatomical landmarks for internal jugular vein catheterization. Cochrane Database Syst Rev. 2015;(1):CD006962. Brass P, Hellmich M, Kolodziej L, et al. Ultrasound guidance versus anatomical landmarks for subclavian or femoral vein catheterization. Cochrane Database Syst Rev. 2015;(1):CD011447. Breitkreutz R, Price S, Steiger HV, et al. Focused echocardiographic evaluation in life support and peri-resuscitation of emergency patients: a prospective trial. Resuscitation. 2010;81:1527-1533. Byon HJ, Lim CW, Lee JH, et al. Prediction of fluid responsiveness in mechanically ventilated children undergoing neurosurgery. Br J Anaesth. 2013;110:586-591. Czarnik T, Gawda R, Perkowski T, et al. Supraclavicular approach is an easy and safe method of subclavian vein catheterization even in mechanically ventilated patients: analysis of 370 attempts. Anesthesiology. 2009;111:334-339. Dennington D, Vali P, Finer NN, et al. Ultrasound confirmation of endotracheal tube position in neonates. Neonatology. 2012;102:185-189. Durand P, Chevret L, Essouri S, et al. Respiratory variations in aortic blood flow predict fluid responsiveness in ventilated children. Intensive Care Med. 2008;34:888-894. Epelman M, Navarro OM, Daneman A, et al. M-mode sonography of diaphragmatic motion: description of technique and experience in 278 pediatric patients. Pediatr Radiol. 2005;35:661-667. Faingold R, Daneman A, Tomlinson G, et al. Necrotizing enterocolitis: assessment of bowel viability with color Doppler US1. Radiology. 2005;235:587-594. Feissel M, Michard FDR, Faller JP, et al. The respiratory variation in inferior vena cava diameter as a guide to fluid therapy. Intensive Care Med. 2004;30(9):1834-1837. Fields JM, Dean AJ, Todman RW, et al. The effect of vessel depth, diameter, and location on ultrasound-guided peripheral intravenous catheter longevity. Am J Emerg Med. 2012;30(7):1134-1140. Fox JC, Boysen M, Gharahbaghian L, et al. Test characteristics of focused assessment of sonography for trauma for clinically significant abdominal free fluid in pediatric blunt abdominal trauma. Acad Emerg Med. 2011;18:477-482. Froehlich CD, Rigby MR, Rosenberg ES, et al. Ultrasound-guided central venous catheter placement decreases complications and decreases placement attempts compared with the landmark technique in patients in a pediatric intensive care unit. Crit Care Med. 2009;37:1090-1096. Galicinao J, Bush AJ, Godambe SA. Use of bedside ultrasonography for endotracheal tube placement in pediatric patients: a feasibility study. Pediatrics. 2007;120:1297-1303.

Hayes J, Borges B, Armstrong D, et al. Accuracy of manual palpation vs ultrasound for identifying the L3-L4 intervertebral space level in children. Paediatr Anaesth. 2014;24:510-515. Holmes JF, Kelley KM, Wootton-Gorges SL, et al. Effect of abdominal ultrasound on clinical care, outcomes, and resource use among children with blunt torso trauma: a randomized clinical trial. JAMA. 2017;317:2290-2296. Kantor DB, Su E, Milliren CE, et al. Ultrasound guidance and other determinants of successful peripheral artery catheterization in critically ill children. Pediatr Crit Care Med. 2016;17:1124-1130. Kosiak W, Swieton D, Piskunowicz M. Sonographic inferior vena cava/ aorta diameter index, a new approach to the body fluid status assessment in children and young adults in emergency ultrasound—preliminary study. Am J Emerg Med. 2008;26:320-325. Kutty S, Attebery JE, Yeager EM, et al. Transthoracic echocardiography in pediatric intensive care. Pediatr Crit Care Med. 2014;15:329-335. Le A, Hoehn ME, Smith ME, et al. Bedside sonographic measurement of optic nerve sheath diameter as a predictor of increased intracranial pressure in children. Ann Emerg Med. 2009;53:785-791. Lichtenstein DA, Mauriat P. Lung ultrasound in the critically ill neonate. Curr Pediatr Rev. 2012;8:217-223. Lichtenstein DA. Ultrasound examination of the lungs in the intensive care unit. Pediatr Crit Care Med. 2009;10:693-698. Miller SG, Brook MM, Tacy TA. Reliability of two-dimensional echocardiography in the assessment of clinically significant abnormal hemidiaphragm motion in pediatric cardiothoracic patients: comparison with fluoroscopy. Pediatr Crit Care Med. 2006;7:441-444. Neal JT, Chen A. 2090707 The effect of bedside ultrasound (US) assistance on the proportion of successful infant lumbar punctures in a pediatric emergency department: a randomized controlled trial. Ultrasound Med Biol. 2015;41:S60-S61. Ng L, Khine H, Taragin BH, et al. Does bedside sonographic measurement of the inferior vena cava diameter correlate with central venous pressure in the assessment of intravascular volume in children? Pediatr Emerg Care. 2013;29:337-341.



CHAPTER 15

Ultrasonography in the Pediatric Intensive Care Unit

133

Panebianco NL, Fredette JM, Szyld D, et al. What you see (sonographically) is what you get: vein and patient characteristics associated with successful ultrasound-guided peripheral intravenous placement in patients with difficult access. Acad Emerg Med. 2009;16:1298-1303. Pershad J, Myers S, Plouman C, et al. Bedside limited echocardiography by the emergency physician is accurate during evaluation of the critically ill patient. Pediatrics. 2004;114:e667-e671. Ranjit S, Aram G, Kissoon N, et al. Multimodal monitoring for hemodynamic categorization and management of pediatric septic shock. Pediatr Crit Care Med. 2014;15:e17-e26. Riggs BJ, Trimboli-Heidler C, Spaeder MC, et al. The use of ophthalmic ultrasonography to identify retinal injuries associated with abusive head trauma. Ann Emerg Med. 2016;67:620-624. Shime N, Hosokawa K, MacLaren G. Ultrasound imaging reduces failure rates of percutaneous central venous catheterization in children. Pediatr Crit Care Med. 2015;16:718-725. Spurney CF, Sable CA, Berger JT, et al. Use of a hand-carried ultrasound device by critical care physicians for the diagnosis of pericardial effusions, decreased cardiac function, and left ventricular enlargement in pediatric patients. J Am Soc Echocardiogr. 2005;18:313-319. Steffen K, Thompson WR, Pustavoitau A, et al. Return of viable cardiac function following sonographic cardiac standstill in pediatric cardiac arrest. Pediatr Emerg Care. 2017;33(1):58-59. Stengel D, Rademacher G, Ekkernkamp A, et al. Emergency ultrasoundbased algorithms for diagnosing blunt abdominal trauma. Cochrane Database Syst Rev. 2015;65:CD004446. Tsung JW, Blaivas M. Feasibility of correlating the pulse check with focused point-of-care echocardiography during pediatric cardiac arrest: a case series. Resuscitation. 2008;77:264-269. Warkentine FH, Clyde Pierce M, Lorenz D, et al. The anatomic relationship of femoral vein to femoral artery in euvolemic pediatric patients by ultrasonography: implications for pediatric femoral central venous access. Acad Emerg Med. 2008;15:426-430.

The full reference list for this chapter is available at ExpertConsult.com.

e1































































































1. Prabhu SJ, Kanal K, Bhargava P, et al. Ultrasound artifacts: classification, applied physics with illustrations, and imaging appearances. Ultrasound Q. 2014;30:145-157. 2. Brass P, Hellmich M, Kolodziej L, et al. Ultrasound guidance versus anatomical landmarks for internal jugular vein catheterization. Cochrane Database Syst Rev. 2015;1:CD006962. 3. Brass P, Hellmich M, Kolodziej L, et al. Ultrasound guidance versus anatomical landmarks for subclavian or femoral vein catheterization. Cochrane Database Syst Rev. 2015;1:CD011447. 4. Rabindranath KS, Kumar E, Shail R, et al. Ultrasound use for the placement of haemodialysis catheters. Cochrane Database Syst Rev. 2011;27:CD005279. 5. Dodge KL, Lynch CA, Moore CL, et al. Use of ultrasound guidance improves central venous catheter insertion success rates among junior residents. J Ultrasound Med. 2012;31:1519-1526. 6. de Souza TH, Brandão MB, Nadal JAH, et al. Ultrasound guidance for pediatric central venous catheterization: a meta-analysis. Pediatrics. 2018; 142:e2018;1719. 7. Shime N, Hosokawa K, MacLaren G. Ultrasound imaging reduces failure rates of percutaneous central venous catheterization in children. Pediatr Crit Care Med. 2015;16:718-725. 8. Bodenham AR. Ultrasound-guided subclavian vein catheterization: beyond just the jugular vein. Crit Care Med. 2011;39:1819-1820. 9. Gualtieri E, Deppe SA, Sipperly ME, et al. Subclavian venous catheterization: greater success rate for less experienced operators using ultrasound guidance. Crit Care Med. 1995;23:692-697. 10. Czarnik T, Gawda R, Perkowski T, et al. Supraclavicular approach is an easy and safe method of subclavian vein catheterization even in mechanically ventilated patients: analysis of 370 attempts. Anesthesiology. 2009;111:334-339. 11. Kocum A, Sener M, Calıskan E, et al. An alternative central venous route for cardiac surgery: supraclavicular subclavian vein catheterization. J Cardiothorac Vasc Anesth. 2011; 25:1018-1023. 12. Lu W-H, Yao M-L, Hsieh K-S, et al. Supraclavicular versus infraclavicular subclavian vein catheterization in infants. J Chin Med Assoc. 2006;69:153-156. 13. Rhondali O, Attof R, Combet S, et al. Ultrasound-guided subclavian vein cannulation in infants: supraclavicular approach. Paediatr Anaesth. 2011;21:1136-1141. 14. Aouad MT, Kanazi GE, Abdallah FW, et al. Femoral vein cannulation performed by residents: a comparison between ultrasoundguided and landmark technique in infants and children undergoing cardiac surgery. Anesth Analg. 2010;111:724-728. 15. Iwashima S, Ishikawa T, Ohzeki T. Ultrasound-guided versus landmark-guided femoral vein access in pediatric cardiac catheterization. Pediatr Cardiol. 2008;29:339-342. 16. Froehlich CD, Rigby MR, Rosenberg ES, et al. Ultrasound-guided central venous catheter placement decreases complications and decreases placement attempts compared with the landmark technique in patients in a pediatric intensive care unit. Crit Care Med. 2009; 37:1090-1096. 17. Akingbola OA, Nielsen J, Hopkins RL, et al. Femoral vein size in newborns and infants: preliminary investigation. Crit Care. 2000;4:120-123. 18. Hopkins JW, Warkentine F, Gracely E, et al. The anatomic relationship between the common femoral artery and common femoral vein in frog leg position versus straight leg position in pediatric patients. Acad Emerg Med. 2009;16:579-584. 19. Warkentine FH, Clyde Pierce M, Lorenz D, et al. The anatomic relationship of femoral vein to femoral artery in euvolemic pediatric patients by ultrasonography: implications for pediatric femoral central venous access. Acad Emerg Med. 2008;15:426-430. 20. Pittiruti M, Biasucci DG, La Greca A, et al. How to make the axillary vein larger? Effect of 90° abduction of the arm to facilitate ultrasound-guided axillary vein puncture. J Crit Care. 2016;33: 38-41.

21. Hanada S, Van Winkle MT, Subramani S, et al. Dynamic ultrasound-guided short-axis needle tip navigation technique vs. landmark technique for difficult saphenous vein access in children: a randomised study. Anaesthesia. 2017;72:1508-1515. 22. Otani T, Morikawa Y, Hayakawa I, et al. Ultrasound-guided peripheral intravenous access placement for children in the emergency department. Eur J Pediatr. 2018;177:1443-1449. 23. Panebianco NL, Fredette JM, Szyld D, et al. What you see (sonographically) is what you get: vein and patient characteristics associated with successful ultrasound-guided peripheral intravenous placement in patients with difficult access. Acad Emerg Med. 2009; 16:1298-1303. 24. Takeshita J, Yoshida T, Nakajima Y, et al. Superiority of dynamic needle tip positioning for ultrasound-guided peripheral venous catheterization in patients younger than 2 years old: a randomized controlled trial. Pediatr Crit Care Med. 2019; Publish Ahead of 2019;20(9):e410-e414. 25. Fields JM, Dean AJ, Todman RW, et al. The effect of vessel depth, diameter, and location on ultrasound-guided peripheral intravenous catheter longevity. Am J Emerg Med. 2012;30:1134-1140. 26. Shiloh AL, Savel RH, Paulin LM, et al. Ultrasound-guided catheterization of the radial artery: a systematic review and meta-analysis of randomized controlled trials. Chest. 2011; 139:524-529. 27. Kantor DB, Su E, Milliren CE, et al. Ultrasound guidance and other determinants of successful peripheral artery catheterization in critically ill children. Pediatr Crit Care Med. 2016;17: 1124-1130. 28. Nakayama Y, Nakajima Y, Sessler DI, et al. A novel method for ultrasound-guided radial arterial catheterization in pediatric patients. Anesth Analg. 2014;118:1019-1026 29. Fleming SE, Kim JH. Ultrasound-guided umbilical catheter insertion in neonates. J Perinatol. 2011;31:344-349. 30. George L, Waldman JD, Cohen ML, et al. Umbilical vascular catheters: localization by two-dimensional echocardio/aortography. Pediatr Cardiol. 1982;2:237-243. 31. Katheria AC, Fleming SE, Kim JH. A randomized controlled trial of ultrasound-guided peripherally inserted central catheters compared with standard radiograph in neonates. J Perinatol. 2013;33: 791-794. 32. Matsushima K, Frankel HL. Bedside ultrasound can safely eliminate the need for chest radiographs after central venous catheter placement: CVC sono in the surgical ICU (SICU). J Surg Res. 2010;163:155-161. 33. Marano L, Izzo G, Esposito G, et al. Peripherally inserted central catheter tip position: a novel empirical-ultrasonographical index in a modern surgical oncology department. Ann Surg Oncol. 2014;21: 656-661. 34. Vezzani A, Brusasco C, Palermo S, et al. Ultrasound localization of central vein catheter and detection of postprocedural pneumothorax: an alternative to chest radiography. Crit Care Med. 2010;38: 533-538. 35. Horowitz R, Gossett JG, Bailitz J, et al. The FLUSH study—flush the line and ultrasound the heart: ultrasonographic confirmation of central femoral venous line placement. Ann Emerg Med. 2014;63: 678-683. 36. Wen M, Stock K, Heemann U, et al. Agitated saline bubble-enhanced transthoracic echocardiography: a novel method to visualize the position of central venous catheter. Crit Care Med. 2014; 42:e231-3. 37. Helgeson SA, Fritz AV, Tatari MM, et al. Reducing Iatrogenic pneumothoraces: using real-time ultrasound guidance for pleural procedures. Crit Care Med. 2019;47:903-909. 38. Hayes J, Borges B, Armstrong D, et al. Accuracy of manual palpation vs ultrasound for identifying the L3-L4 intervertebral space level in children. Paediatr Anaesth. 2014;24:510-515. 39. Neal JT, Chen A. 2090707 The effect of bedside ultrasound (US) assistance on the proportion of successful infant lumbar punctures in a pediatric emergency department: a randomized controlled trial. Ultrasound Med and Biol. 2015;41:S60-S61.

References

e2















































64. Rozycki GS, Ochsner MG, Schmidt JA, et al. A prospective study of surgeon-performed ultrasound as the primary adjuvant modality for injured patient assessment. J Trauma/ 1995;39:492-498; discussion 498-500. 65. Rozycki GS, Ballard RB, Feliciano DV, et al. Surgeon-performed ultrasound for the assessment of truncal injuries: lessons learned from 1540 patients. Ann Surg. 1998;228:557-567. 66. Fox JC, Boysen M, Gharahbaghian L, et al. Test characteristics of focused assessment of sonography for trauma for clinically significant abdominal free fluid in pediatric blunt abdominal trauma. Acad Emerg Med. 2011;18:477-482. 67. Holmes JF, Kelley KM, Wootton-Gorges SL, et al. Effect of abdominal ultrasound on clinical care, outcomes, and resource use among children with blunt torso trauma: a randomized clinical trial. JAMA. 2017;317:2290-2296. 68. Stengel D, Rademacher G, Ekkernkamp A, et al. Emergency ultrasound-based algorithms for diagnosing blunt abdominal trauma. Cochrane Database Syst Rev. 2015;65:CD004446. 69. Stengel D, Leisterer J, Ferrada P, et al. Point-of-care ultrasonography for diagnosing thoracoabdominal injuries in patients with blunt trauma. Cochrane Database Syst Rev. 2018;12:CD012669. 70. Kong H-Y, Chen F, He Y, et al. Intrarenal resistance index for the assessment of acute renal injury in a rat liver transplantation model. BMC Nephrol. 2013;14:55. 71. Le Dorze M, Bouglé A, Deruddre S, et al. Renal Doppler ultrasound: a new tool to assess renal perfusion in critical illness. Shock. 2012;37:360-365. 72. Dewitte A, Coquin J, Meyssignac B, et al. Doppler resistive index to reflect regulation of renal vascular tone during sepsis and acute kidney injury. Crit Care. 2012;16:R165. 73. Theodoraki K, Theodorakis I, Chatzimichael K, et al. Ultrasonographic evaluation of abdominal organs after cardiac surgery. J Surg Res. 2015;194:351-360. 74. Darmon M, Schortgen F, Vargas F, et al. Diagnostic accuracy of Doppler renal resistive index for reversibility of acute kidney injury in critically ill patients. Intensive Care Med. 2011;37:68-76. 75. Guinot P-G, Bernard E, Abou Arab O, et al. Doppler-based renal resistive index can assess progression of acute kidney injury in patients undergoing cardiac surgery. J Cardiothorac Vasc Anesth. 2013;27:890-896. 76. Bossard G, Bourgoin P, Corbeau JJ, et al. Early detection of postoperative acute kidney injury by Doppler renal resistive index in cardiac surgery with cardiopulmonary bypass. Br J Anaesth. 2011;107:891-898. 77. Yang P-L, Wong DT, Dai S-B, et al. The feasibility of measuring renal blood flow using transesophageal echocardiography in patients undergoing cardiac surgery. Anesth Analg. 2009;108:1418-1424. 78. Marty P, Szatjnic S, Ferre F, et al. Doppler renal resistive index for early detection of acute kidney injury after major orthopaedic surgery: a prospective observational study. Eur J Anaesthesiol. 2015;32:37-43. 79. Kutty S, Attebery JE, Yeager EM, et al. Transthoracic echocardiography in pediatric intensive care: impact on medical and surgical management. Pediatr Crit Care. Med 2014; 15:329-335. 80. Ranjit S, Aram G, Kissoon N, et al. Multimodal monitoring for hemodynamic categorization and management of pediatric septic shock: a pilot observational study. Pediatr Crit Care Med. 2014; 15:e17-e26. 81. Bouferrache K, Amiel J-B, Chimot L, et al. Initial resuscitation guided by the Surviving Sepsis Campaign recommendations and early echocardiographic assessment of hemodynamics in intensive care unit septic patients: a pilot study. Crit Care Med. 2012;40: 2821-2827. 82. Kosiak W, Swieton D, Piskunowicz M. Sonographic inferior vena cava/aorta diameter index, a new approach to the body fluid status assessment in children and young adults in emergency ultrasound-preliminary study. Am J Emerg Med. 2008;26:320-325. 83. Haciomeroglu P, Ozkaya O, Gunal N, et al. Venous collapsibility index changes in children on dialysis. Nephrology (Carlton). 2007;12:135-139.























































40. Peterson MA, Pisupati D, Heyming TW, et al. Ultrasound for routine lumbar puncture. Acad Emerg Med. 2014;21:130-136. 41. Trinavarat P, Riccabona M. Potential of ultrasound in the pediatric chest. Eur J Radiol. 2014;83:1507-1518. 42. Lichtenstein DA, Mauriat P. Lung Ultrasound in the Critically Ill Neonate. Curr Pediatr Rev. 2012;8:217-223. 43. Lichtenstein DA. Ultrasound examination of the lungs in the intensive care unit. Pediatr Crit Care Med. 2009;10:693-698. 44. Basile V, Di Mauro A, Scalini E, et al. Lung ultrasound: a useful tool in diagnosis and management of bronchiolitis. BMC Pediatr. 2015;15:63. 45. Cattarossi L. Lung ultrasound: its role in neonatology and pediatrics. Early Hum Dev. 2013;89(suppl 1):S17-S19. 46. Rodriguez Fanjul J, Balcells C, Aldecoa-Bilbao V, et al. Lung Ultrasound as a Predictor of Mechanical Ventilation in Neonates Older than 32 Weeks. Neonatology. 2016;110:198-203. 47. Lichtenstein D, Mezière G, Biderman P, et al. The “lung point”: an ultrasound sign specific to pneumothorax. Intensive Care Med. 2000;26:1434-1440. 48. Raimondi F, Rodriguez Fanjul J, Aversa S, et al. Lung ultrasound for diagnosing pneumothorax in the critically ill neonate. J Pediatr. 2016;175:74-78.e1. 49. Hsieh K-S, Lee C-L, Lin C-C, et al. Secondary confirmation of endotracheal tube position by ultrasound image. Crit Care Med. 2004;32:S374-7. 50. Galicinao J, Bush AJ, Godambe SA. Use of bedside ultrasonography for endotracheal tube placement in pediatric patients: a feasibility study. Pediatrics. 2007;120:1297-1303. 51. Dennington D, Vali P, Finer NN, Kim JH. Ultrasound confirmation of endotracheal tube position in neonates. 2012;102:185-189. https:// www.karger.com/Article/FullText/338585. 52. Tessaro MO, Salant EP, Arroyo AC, et al. Tracheal rapid ultrasound saline test (T.R.U.S.T.) for confirming correct endotracheal tube depth in children. Resuscitation. 2015;89:8-12. 53. Dalesio NM, Kattail D, Ishman SL, et al. Ultrasound use in the pediatric airway: the time has come. A A Case Rep. 2014;2:23-26. 54. Kunovsky P, Gibson GA, Pollock JC, et al. Management of postoperative paralysis of diaphragm in infants and children. Eur J Cardiothorac Surg. 1993;7:342-346. 55. Miller SG, Brook MM, Tacy TA. Reliability of two-dimensional echocardiography in the assessment of clinically significant abnormal hemidiaphragm motion in pediatric cardiothoracic patients: Comparison with fluoroscopy. Pediatr Crit Care Med. 2006;7:441-444. 56. Sanchez de Toledo J, Munoz R, Landsittel D, et al. Diagnosis of abnormal diaphragm motion after cardiothoracic surgery: ultrasound performed by a cardiac intensivist vs. fluoroscopy. Congenit Heart Dis. 2010; 5:565-572. 57. Gil-Juanmiquel L, Gratacós M, Castilla-Fernández Y, et al. Bedside ultrasound for the diagnosis of abnormal diaphragmatic motion in children after heart surgery. Pediatr Crit Care Med. 2017;18: 159-164. 58. Epelman M, Navarro OM, Daneman A, et al. M-mode sonography of diaphragmatic motion: description of technique and experience in 278 pediatric patients. Pediatr Radiol. 2005;35:661-667. 59. Glau CL, Conlon TW, Himebauch AS, et al. Progressive diaphragm atrophy in pediatric acute respiratory failure. Pediatr Crit Care Med. 2018;19:406-411. 60. Faingold R, Daneman A, Tomlinson G, et al. Necrotizing enterocolitis: assessment of bowel viability with color Doppler US. Radiology. 2005;235:587-594. 61. Kim H-Y, Kim I-O, Kim WS, et al. Bowel sonography in sepsis with pathological correlation: an experimental study. Pediatr Radiol. 2011;41:237-243. 62. Kim W-Y, Kim WS, Kim I-O, et al. Sonographic evaluation of neonates with early-stage necrotizing enterocolitis. Pediatr Radiol. 2005;35:1056-1061. 63. Ma OJ, Mateer JR, Ogata M, et al. Prospective analysis of a rapid trauma ultrasound examination performed by emergency physicians. J Trauma. 1995;38:879-885.

e3





105.





106.



108.



109.



110.



111.



112.



113.



114.



115.



116.

117.

























107.







119.



120.











118.



121.



122.

































84. Gan H, Cannesson M, Chandler JR, et al. Predicting fluid responsiveness in children: a systematic review. Anesth Analg. 2013; 117:1380-1392. 85. Barbier C, Loubières Y, Schmit C, et al. Respiratory changes in inferior vena cava diameter are helpful in predicting fluid responsiveness in ventilated septic patients. Intensive Care Med. 2004;30:1740-1746. 86. Feissel M, Michard F, Faller J-P, et al. The respiratory variation in inferior vena cava diameter as a guide to fluid therapy. Intensive Care Med. 2004;30:1834-1837. 87. Lin EE, Chen AE, Panebianco N, et al. Effect of inhalational anesthetics and positive-pressure ventilation on ultrasound assessment of the great vessels: a prospective study at a children’s hospital. Anesthesiology. 2016;124:870-877. 88. Ng L, Khine H, Taragin BH, et al. Does bedside sonographic measurement of the inferior vena cava diameter correlate with central venous pressure in the assessment of intravascular volume in children? Pediatr Emerg Care. 2013;29:337-341. 89. Marik PE, Cavallazzi R. Does the central venous pressure predict fluid responsiveness? An updated meta-analysis and a plea for some common sense. Crit Care Med. 2013;41:1774-1781. 90. Byon H-J, Lim C-W, Lee J-H, et al. Prediction of fluid responsiveness in mechanically ventilated children undergoing neurosurgery. Br J Anaesth. 2013;110:586-591. 91. Durand P, Chevret L, Essouri S, et al. Respiratory variations in aortic blood flow predict fluid responsiveness in ventilated children. Intensive Care Med. 2008;34:888-894. 92. Singh S, Wann LS, Schuchard GH, et al. Right ventricular and right atrial collapse in patients with cardiac tamponade—a combined echocardiographic and hemodynamic study. Circulation. 1984;70:966-971. 93. Chiang HT, Lin M. Pericardiocentesis guided by two-dimensional contrast echocardiography. Echocardiography. 1993;10:465-469. 94. Law MA, Borasino S, Kalra Y, et al. Novel, long-axis in-plane ultrasound-guided pericardiocentesis for postoperative pericardial effusion drainage. Pediatr Cardiol. 2016;37:1328-1333. 95. Fitch MT, Nicks BA, Pariyadath M, et al. Videos in clinical medicine. Emergency pericardiocentesis. N Engl J Med. 2012; 366:e17. 96. Pershad J, Myers S, Plouman C, et al. Bedside limited echocardiography by the emergency physician is accurate during evaluation of the critically ill patient. Pediatrics. 2004;114:e667-e671. 97. Spurney CF, Sable CA, Berger JT, et al. Use of a hand-carried ultrasound device by critical care physicians for the diagnosis of pericardial effusions, decreased cardiac function, and left ventricular enlargement in pediatric patients. J Am Soc Echocardiogr. 2005;18:313-319. 98. Engle SJ, DiSessa TG, Perloff JK, et al. Mitral valve E point to ventricular septal separation in infants and children. Am J Cardiol. 1983;52:1084-1087. 99. Matzer L, Cortada X, Ferrer P, et al. Widened E point septal separation in a normal pediatric population. Chest. 1985;87:73-75. 100. Koestenberger M, Nagel B, Ravekes W, et al. Left ventricular longaxis function: reference values of the mitral annular plane systolic excursion in 558 healthy children and calculation of z-score values. Am Heart J. 2012;164:125-131. 101. Otto CM. The Practice of Clinical Echocardiography. 6 ed. Elsevier; 2007. 102. Tsung JW, Blaivas M. Feasibility of correlating the pulse check with focused point-of-care echocardiography during pediatric cardiac arrest: a case series. Resuscitation. 2008;77:264-269. 103. Breitkreutz R, Price S, Steiger HV, et al. Focused echocardiographic evaluation in life support and peri-resuscitation of emergency patients: a prospective trial. Resuscitation. 2010;81:1527-1533. 104. Clattenburg EJ, Wroe P, Brown S, et al. Point-of-care ultrasound use in patients with cardiac arrest is associated prolonged cardiopulmonary

resuscitation pauses: a prospective cohort study. Resuscitation 2018; 122:65-68. Huis In ‘t Veld MA, Allison MG, Bostick DS, et al. Ultrasound use during cardiopulmonary resuscitation is associated with delays in chest compressions. Resuscitation. 2017;119:95-98. Cha S, Gottschalk A, Su E, et al. Use of point-of-care ultrasonography in simulation-based advanced cardiac life support scenarios. J Anesth Perioper Med. 2018;5:53-60. Teran F, Dean AJ, Centeno C, et al. Evaluation of out-of-hospital cardiac arrest using transesophageal echocardiography in the emergency department. Resuscitation. 2019;137:140-147. Blyth L, Atkinson P, Gadd K, et al. Bedside focused echocardiography as predictor of survival in cardiac arrest patients: a systematic review. Acad Emerg Med. 2012;19:1119-1126. Salen P, Melniker L, Chooljian C, et al. Does the presence or absence of sonographically identified cardiac activity predict resuscitation outcomes of cardiac arrest patients? Am J Emerg Med. 2005;23:459-462. Steffen K, Thompson WR, Pustavoitau A, et al. Return of viable cardiac function after sonographic cardiac standstill in pediatric cardiac arrest. Pediatr Emerg Care. 2017;33:58-59. Hu K, Gupta N, Teran F, et al. Variability in interpretation of cardiac standstill among physician sonographers. Ann Emerg Med. 2018;71:193-198. Su E, Dalesio N, Pustavoitau A. Point-of-care ultrasound in pediatric anesthesiology and critical care medicine. Can J Anaesth. 2018;65:485-498. Le A, Hoehn ME, Smith ME, et al. Bedside sonographic measurement of optic nerve sheath diameter as a predictor of increased intracranial pressure in children. Ann Emerg Med. 2009;53:785-791. Riggs BJ, Trimboli-Heidler C, Spaeder MC, et al. The use of ophthalmic ultrasonography to identify retinal injuries associated with abusive head trauma. Ann Emerg Med. 2016;67:620-624. Expert Round Table on Ultrasound in ICU: International expert statement on training standards for critical care ultrasonography. Intensive Care Med. 2011;37:1077-1083. From the Ultrasound Certification Task Force on behalf of the Society of Critical Care Medicine, Pustavoitau A, Blaivas M, Brown SM, et al. Recommendations for Achieving and Maintaining Competence and Credentialing in Critical Care Ultrasound with Focused Cardiac Ultrasound and Advanced Critical Care Echocardiography. http://journals.lww.com/ccmjournal/Documents/Critical%20 Care%20Ultrasound.pdf. American College of Emergency Physicians. American College of Emergency Physicians. ACEP emergency ultrasound guidelines-2001. Ann Emerg Med. 2001;38:470-481. Frankel HL, Kirkpatrick AW, Elbarbary M, et al. Guidelines for the appropriate use of bedside general and cardiac ultrasonography in the evaluation of critically ill patients-Part I: general ultrasonography. Crit Care Med. 2015;43:2479-2502. Levitov A, Frankel HL, Blaivas M, et al. Guidelines for the appropriate use of bedside general and cardiac ultrasonography in the evaluation of critically ill patients-Part II: cardiac ultrasonography. Crit Care Med. 2016;44:1206-1227. Conlon TW, Himebauch AS, Fitzgerald JC, et al. Implementation of a pediatric critical care focused bedside ultrasound training program in a large academic PICU. Pediatr Crit Care Med. 2015;16:219-226. Conlon TW, Kantor DB, Su ER, et al. Diagnostic bedside ultrasound program development in pediatric critical care medicine: results of a national survey. Pediatr Crit Care Med. 2018;19:e561-e568. Nguyen J, Amirnovin R, Ramanathan R, et al. The state of pointof-care ultrasonography use and training in neonatal-perinatal medicine and pediatric critical care medicine fellowship programs. J Perinatol. 2016;36:972-976.

e4

Abstract: The chapter presents updates on existing knowledge of procedural applications of ultrasound, including in vascular access and other ultrasound-guided procedures. A wide array of diagnostic applications are covered as well, including pulmonary and airway, abdominal, cardiac assessment, and neurologic

applications. Importantly, the chapter also addresses the evolving practice of ultrasound in the pediatric intensive care unit. Key words: echocardiography, procedures, vascular access, umbilical access, trauma, neurosonology

SECTION

III

Pediatric Critical Care: Psychosocial and Societal 16. Patient- and Family-Centered Care in the Pediatric Intensive Care Unit 136 17. Pediatric Critical Care Ethics 144 18. Ethical Issues Around Death and Dying 154 19. Palliative Care in the Pediatric Intensive Care Unit 158

 



 



 



20. Organ Donation Process and Management of the Organ Donor 163 21. Long-Term Outcomes Following Critical Illness in Children 175 22. Burnout and Resiliency 183  



 







 



135

16 Patient- and Family-Centered Care in the Pediatric Intensive Care Unit JENNY KINGSLEY AND JONNA D. CLARK







“Intensive care unit. Wow, those are scary words for parents to hear! We viewed the intensive care unit as a place people went when their medical situation was desperate or where they were likely to die.”2 Many parents share this parent’s voice when faced with the hospitalization of a critically ill child.2 These families not only face the extreme crisis of having a sick child whose life may be threatened, but they are also forced into a foreign environment and culture that can be intimidating, overwhelming, and perceived as unwelcoming. Families in the intensive care unit (ICU) are at one of the worst moments in their lives and suffer from an extreme level of stress, in which they may feel powerless and out of control. This level of stress creates short- and long-term consequences, as many parents and children face severe emotional distress, which may result in posttraumatic stress disorder (PTSD).3–7 To mitigate the severity of this stressful response and optimize the quality of medical care provided, patient- and family-centered care (PFCC) is a philosophy that highlights the importance of including the patient, when possible, and family in the provision of healthcare. The American Academy of Pediatrics (AAP) acknowledges that the “family is the primary source of a child’s strength and support” and defines PFCC as “an innovative approach to the planning, delivery, and evaluation of healthcare that is grounded in a mutually beneficial partnership among patients, families, and providers.”1 PFCC can be incorporated across the 136

Developing a partnership built on mutual respect among healthcare providers, patients, and families • Empowering patients and families to participate in shared medical decision-making Incorporating PFCC into practice improves patient and family satisfaction, reduces stress and anxiety, fosters the parent-child relationship, and ultimately increases the quality, efficacy, efficiency, and safety of healthcare delivered. Overcoming real and perceived barriers to incorporating PFCC into practice requires an explicit, collaborative, and transparent approach involving all stakeholders to identify creative solutions.





Patient- and family-centered care (PFCC) uses an “innovative approach to the planning, delivery, and evaluation of healthcare that is grounded in a mutually beneficial partnership among patients, families, and providers.”1 Fundamental principles of PFCC include: • Honoring differences and respecting each individual patient and family • Maintaining flexibility in practice, policies, and procedures to fully accommodate the needs of patients and families • Sharing honest and consistent information using collaborative communication • Using a transdisciplinary approach to provide optimal support for the family unit









PEARLS





spectrum of healthcare, from home to outpatient to inpatient, and is essential for the provision of high-quality care in the ICU. Ongoing research in this area demonstrates that parents’ participation in the care of their hospitalized child directly benefits the child by reducing anxiety, improving cooperation, improving activity level, and reducing the length of stay.8,9 Additional data reveal more effective utilization of healthcare resources, reduced healthcare costs, and improved provider satisfaction.1

Definition of “Family” In the delivery of PFCC, patients and families, rather than healthcare providers or the legal system, define their families.10,11 Since family structures are heterogeneous and evolve over time within a variety of cultural contexts,10,11 the term “family” requires a broad definition, referring to “two or more persons who are related in any way—biologically, legally, or emotionally.”10 A wide variety of family structures exists, including blended families, singleparent households, adoptive homes, same-sex couples, and transgenerational models where extended family members—such as grandparents, aunts, uncles, or siblings—serve as the primary caretakers.7,10,11 While healthcare providers and institutions have a legal obligation to respect the law regarding surrogate decisionmaking for minors, they also have an ethical and professional obligation to respect the broader definition of family in practice,



CHAPTER 16

Patient- and Family-Centered Care in the Pediatric Intensive Care Unit

acknowledging that the family is the primary source of strength and support for the child.1,8

Historical Evolution of Patient- and FamilyCentered Care The conceptual aspects of PFCC can be dated back to as early as the 1950s, when studies demonstrated a negative impact of maternal-child separation, leading to a shift toward encouraging parent participation in the care of the hospitalized child.1,8,11,12 In 1983 the Committee on Hospital Care and Pediatric Section of the Society of Critical Care Medicine published “Guidelines for Pediatric Intensive Care Units” highlighting that “Parents should be allowed to stay with the critically ill child as much as possible.”13 These guidelines explained that the presence of parents may lead to faster recovery, stating, “The familiar face and voice of a parent may reach a child who appears comatose but is beginning to respond to stimuli.”13 In the following 2 decades, there was extensive research in the value and benefit of PFCC in healthcare communities. Several national organizations were founded, including Family Voices, an organization that advocates for family-centered, communitybased services for children with special needs, and the Institute for Patient- and Family-Centered Care, an organization that helps foster partnerships between healthcare professionals, patients, and families to advance family-centered care in healthcare settings.1 More recently, several national organizations, including the Institute of Medicine (2001), Maternal and Child Health Bureau (2005), American College of Critical Care Medicine Task Force (2007), and American Academy of Pediatrics (2012) have furthered this approach by developing guidelines and endorsing that PFCC is essential to the provision of high-quality care.1,14

Fundamental Needs of Patients and Families in the Intensive Care Unit Based on the complexity and intensity of the pediatric intensive care unit (PICU), families’ needs are high. Although specific needs of the individual patient and family are important to consider, there is consensus that most families share similar needs, including (1) honest, accurate, and up-to-date information; (2) close proximity to the patient; (3) timely notification of any changes in the patient’s condition; (4) assurance that the healthcare team cares about the child and that the patient is receiving excellent care; (5) access to resources to meet basic physiologic needs; and (6) feeling that there is hope.7,15–18 Research demonstrates that provider perceptions often underestimate the needs of families. Furthermore, when families’ needs are known, they are not adequately addressed in practice. Unfortunately, when families’ needs are not met, there is a negative impact on satisfaction of care in addition to a higher amount of stress and anxiety.6,7,18

Core Principles of Patient- and FamilyCentered Care1,12 The practice of PFCC, in which patients and families are treated as integral partners with the healthcare team, is now recognized as the standard of care in pediatric critical care medicine.1,12 Borrowing from the framework established by the AAP, the following discussion highlights each of the fundamental principles of PFCC within the context of the PICU (Fig. 16.1).11

Maintaining flexibility in provision of healthcare

Honoring and embracing differences

Fostering partnership among healthcare providers, patients, and families

Family presence and engagement

Patient- and family-centered care

Cultural humility

Transdisciplinary care for the whole child and family

Collaborative and empathetic communication Shared medical decision-making



• Fig. 16.1

137

​Core principles of patient- and family-centered care.

138

SECTION III



Pediatric Critical Care: Psychosocial and Societal

Honoring Differences and Respecting Each Child and Family One of the most fundamental aspects of developing a “mutually beneficial partnership” with patients and their families requires a relationship of mutual respect, in which differences among stakeholders are honored.1 The AAP states, “Listen to and respect. . . each child and family’s innate strengths and cultural values. Honor racial, ethnic, cultural, and socioeconomic background and patient and family experiences.”1 Patients and families demonstrate a wide range of responses within their own social construct, using a variety of coping mechanisms to overcome the distressing state. Recognizing that one individual cannot be an expert on all cultures, faiths, ethnicities, spirituality, or religious practices, practicing cultural humility and acknowledging biases and assumptions are crucial.19 In the ICU, where major medical decisions regarding life, death, and quality of life are made quickly, differences in beliefs and value systems may be accentuated among healthcare providers and families. Taking the time to listen to different perspectives and understand how knowledge, past experiences, and value systems impact decisions is essential. In the majority of situations, families and healthcare providers desire what is in the best interest of the child. Acknowledging that values and belief systems affect perceptions of what actions are in the best interest of the child can be helpful when navigating challenging circumstances. For example, parents of a beloved child who suffers from a life-threatening illness with a poor prognosis may choose to prolong life as long as possible, treasuring and valuing each day they have with their child alive, while healthcare providers may view prolonging life as merely prolonging suffering. Developing a collaborative relationship ensures that differences in opinion are respected. For patients and families whose English fluency is minimal, fundamental communication and language barriers exist. In this case, developing a partnership built on trust and understanding may be especially difficult. The use of in-person language interpreters and cultural navigators, who serve as healthcare guides for the family and cultural guides for the healthcare providers, is essential to building this relationship. Optimizing additional means for communication with non-English-speaking families, such as video interpreter service, can also be beneficial.  



Maintaining Flexibility in Practice and Procedures to Deliver Healthcare within the Context of the Family The PICU is a highly complex environment that requires a systematic approach to policies and protocols to ensure the delivery of efficient, effective, and safe healthcare. When developing these policies and protocols, it is essential to recognize how these systems and processes impact patients and families. Hospital systems and organizations need to “ensure flexibility in organizational policies, procedures, and provider practices so services can be tailored to the needs, beliefs, and cultural values of each child and family.”1 Practices and procedures should “facilitate choice. . . about approaches to care.”1 Historically, hospitals were designed as sterile environments that restricted visitation, with fear of spread of infection and the breach of confidentiality.11 However, with the evolution of PFCC, many healthcare organizations and institutions are making the effort to better accommodate patients’ and families’ needs by  



including them in the planning and delivery of care. When a child is hospitalized in the ICU, the needs of the family are extremely high.7,17,18 Addressing many of these needs can be approached considering the following categories: (1) visitation policies and physical accommodations for families, (2) family-centered rounds, (3) family presence during cardiopulmonary resuscitation and other invasive procedures, and (4) collaborative communication.14,20 When assessing the potential benefits and harms of policies, procedures, and delivery of healthcare in these areas, open dialogue and transparency regarding the needs of all stakeholders are essential, allowing for creative solutions when conflicts arise.

Geography of the Intensive Care Unit In planning and designing an ICU the geography of the unit needs to accommodate both healthcare providers and patients and families. For example, a unit that allows for effective and efficient management of multiple critically ill patients is essential for healthcare providers, while patients and families need privacy, space in close proximity to their child, and access to an area where they may find respite from the intensive environment to build strength and repose.13,21 Admission Process and Visiting Hours Parents and family members should be treated as partners rather than visitors within the ICU. They should feel welcomed into the foreign environment through the use of a variety of communication styles, both spoken and written, and remain well informed of processes and procedures. In general, visiting hours for parents and primary caretakers should be nonrestrictive in order to optimize parent and child bonding.14 However, maintaining flexibility in these visiting policies is important as there may be unique circumstances when limiting visitation by family members is necessary to create a calm and healing environment for the child. Creating a “Personalized” Room Patients and families should have the liberty to create an environment that mimics home, with decorations, photographs, favorite toys, and blankets, as long as the modifications do not place the child at risk and do not interfere with the provision of medical care.13,14 Although the ICU is a place of work for the healthcare providers, it is the bedroom for a critically ill child and potentially may be the last place a dying child spends time.22 Creating a home environment is not only beneficial to the child but also helps the healthcare team gain insight into who the child is as a person, enabling healthcare providers to address the needs of the child more appropriately. Sibling Participation Siblings of critically ill children are severely affected and need to be supported according to the values of the family.7,23 Siblings need maintenance of a familiar lifestyle, including family cohesion, distraction from the immediate crisis, hospital visitation, and developmentally appropriate information.7 The transdisciplinary team, including child life specialists and social workers, can help educate parents and caregivers about anticipated questions and emotional responses that may occur, and provide appropriate referral services and support systems.23 Parental Presence During Cardiopulmonary Resuscitation and Invasive Procedures Numerous professional organizations, including the American Heart Association and the American College of Critical Care



CHAPTER 16

Patient- and Family-Centered Care in the Pediatric Intensive Care Unit

Medicine, endorse offering families the option to remain in close proximity to their loved ones during cardiopulmonary resuscitation and invasive procedures.24,25 Allowing family presence has direct benefits for parents and caregivers, including improved satisfaction, better understanding, reduced anxiety, better coping, more emotional stability, and improved adjustment to a child’s death.25–27 Despite these potential benefits, the practice remains controversial. Clinicians raise concerns that parental presence may increase the risk for litigation and may impact technical performance, clinical decision-making, and the ability to teach.25,26 In a large single-center study, using formal practice guidelines and interprofessional education to prepare clinicians for parental presence, “few clinicians reported that parent presence affected their technical performance (4%), therapeutic decision-making (5%), or ability to teach (9%).”26 Moreover, recent data suggest that family presence during intubations does not affect the rate of first attempt success, adverse events, or self-reported team stress level, suggesting that family presence during invasive procedures can be safely implemented.28 By developing protocols, institutional policies, education, and dedicated staff resources to allow parental choice and provide support for parental presence during invasive procedures and cardiopulmonary resuscitation, these concerns can be minimized.25

Transition Points and Follow-up Care, including Bereavement Once adjusted to the ICU, many families ultimately appreciate the high level of physician supervision and nursing presence, making transitions to the acute care floor potentially difficult. Supporting families through these transitions during the course of an illness is necessary.29 For families who spend an especially long period of time in the ICU, providing families with realistic expectations and allowing families to visit the acute care floor prior to transfer can be beneficial. In addition to providing additional support for transitions between different geographic locations in the hospital, developing a framework to help families transition through different approaches to care, from curative to life-prolonging to comfort care, is important.19 Furthermore, using a systematic approach to follow-up with families following the death of child, with condolence letters and formal follow-up meetings, is also necessary to reduce the perception of abandonment and support the bereavement process.17,30–32

Sharing Information Using Collaborative Communication High-quality communication with families is essential in the ICU, where medical information is complex and detailed, highstakes decisions are required, and multiple teams of healthcare providers are involved.33 While there is a spectrum of preferred decision-making roles of families, ranging from an independent and autonomous approach to delegating decisions to clinicians, most families need to be well informed.5 Optimizing communication with families reinforces the development of a partnership with families, acknowledging the important role of parents and caregivers and their unique knowledge of their child. The AAP recommends “sharing complete, honest, and unbiased information with patients and their families on an ongoing basis so that they may effectively participate in care and decision making.”1 Over the past 2 decades, there has been extensive research in optimizing communication between healthcare providers, particularly in the ICU and surrounding end-of-life care

139

decision-making.32,34,35 In the ICU, information is shared using three different verbal models, including individual updates at the bedside, family-centered rounds, and formal care conferences. Written information may also be provided. Often a combination of all of these approaches is necessary to optimize communication with patients and families. Depending on the educational, social, and cultural backgrounds of patients and families, in addition to different provider styles, a variety of approaches to communication may be used.

Elements of High-Quality Communication Sharing medical information is complex and requires a high level of skill among healthcare providers. Parents and providers identify a number of barriers to communication in the PICU, including communication breakdown related to care coordination and provider hand-offs, families feeling undervalued or unheard as the experts on their children, and the inadequacy of family-centered morning rounds.36 While using a variety of means to communicate with patients and families is necessary (as detailed earlier), parents cite specific elements of communication as very important.35,37 High-quality elements of communication include “comprehensive and complete information; clarity of information with the use of clear language; ease of access to caregivers and their explanations through the course of care; pacing of information, soliciting of parents’ emotional responses and addressing their questions; consistency of information; honesty, lack of false hope, empathy as demonstrated by verbal and nonverbal, and affective communication; summary statements and next steps.”23 Other important components of high-quality communication include using open-ended questions, soliciting goals, and increasing time families spend speaking.38 Recent research has found that these elements of high-quality patient- and family-centered communication are associated with higher parent satisfaction when incorporated into family conferences in the PICU.39 A useful tool to help clinicians practice high-quality communication is the VALUE mnemonic, in which family statements are valued, family emotions are acknowledged, the family is listened to, the patient is understood as a person, and family questions are elicited.40 This mnemonic emphasizes the importance of using collaborative communication, or a form of communication in which interpersonal communication and the relationship between the parties are “inexorably entwined.”20 Collaborative communication can accomplish at least five important tasks in PFCC, including (1) establishing a set of goals that guides collaborative efforts; (2) exhibiting mutual respect and compassion for each other; (3) developing sufficient understanding of differing perspectives; (4) ensuring maximum clarity and correctness of what is communicated; and (5) managing intrapersonal and interpersonal processes that affect how information is sent, received, and processed.20 When used consistently in family conferences, evidence suggests that the VALUE mnemonic is associated with lower family symptoms of anxiety, PTSD, and depression.41 Particularly in the setting of the ICU, optimizing these elements of high-quality communication may be difficult owing to the number of healthcare team providers involved and the short time frame in which news must be delivered. Therefore, adequate training for all healthcare providers who interact with families is essential. In addition, when patients spend prolonged periods of time within the ICU or have multiple subspecialty services involved, the identification of a continuity provider whose role is to maintain consistency in the healthcare plan while ensuring adequate communication with the family may be beneficial.

140

SECTION III



Pediatric Critical Care: Psychosocial and Societal

However, current evidence is inadequate to demonstrate clear benefit of or harm from increasing staffing consistency.42 Furthermore, using a team approach with respectful and excellent communication among team members, including physicians and nurses, ensures that clear, consistent, and comprehensive information is shared in an empathetic manner. Collaboration within the healthcare team results in improved overall outcomes in critical care, including increased patient survival, decreased length of stay, and decreased readmission rates, in addition to improved patient and family satisfaction, decreased symptoms of anxiety and depression among family members, and reduced ICU nurse and physician burnout.40

Family-Centered Rounds Rounds in the ICU serve several purposes, including (1) to provide a setting for decision-making related to the management of the child’s care; (2) to ensure adequate communication among healthcare team members; and (3) to allow for teaching of medical students, residents, and other trainees.43 Incorporating the patient and family into these rounds fosters a partnership with the family and aligns with a patient- and family-centered approach. While larger and more extensive studies are necessary to fully determine the benefits and potential barriers to this approach, many professional organizations endorse it. According to several pediatric studies, this approach improves family satisfaction, improves communication between all stakeholders, increases family trust in the medical team, and potentially improves the quality of medical care provided, leading to shorter lengths of stay.44–46 Several potential barriers and healthcare provider concerns regarding this approach include (1) prolongation of rounds, (2) breach of confidentiality of protected health information, (3) reduced opportunities for teaching, and (4) concern for undermining trust of the healthcare team when different opinions are shared or teaching is done.43,45,47 Current evidence suggests that family presence on rounds does increase per patient rounding times.48 However, family-centered rounds exhibit other benefits. Family-centered rounds have demonstrated improved efficiency and communication because family concerns are addressed with clarity, preventing communication gaps and confusion. In addition, family presence usually lends itself to additional teaching opportunities.45,47 Finally, when surveyed, the majority of parents are not concerned about privacy issues. Therefore this concern can easily be discussed with parents on an individual basis.40,42,44 To remain effective and efficient, using a family-centered approach to rounding requires thoughtful planning and following specific guidelines to streamline the process. Highlighting the purpose of rounds to all stakeholders and identifying explicit roles prior to rounds can help prevent misunderstandings.43 Structured Transdisciplinary Care Conferences In critical care, in which illnesses are life threatening and changes occur rapidly, sharing information with frequent updates at the bedside is crucial. Furthermore, in circumstances in which medical decision-making is highly complex, multiple subspecialties and disciplines are involved, or the severity of illness may lead to the demise of the patient, proactive and timely structured transdisciplinary care conferences may also be beneficial.40 Although there is variability in the approach and style of these conferences, the palliative care and adult critical care literature provides a general framework for the facilitation of these conferences and highlights several tools.20,40,41,49,50 In adult critical care, specific aspects of

these conferences are associated with increased quality of care, decreased family psychological symptoms, improved family ratings of communication, and improved outcomes. These aspects include timeliness (occur within 72 hours of ICU admission), private location for the conference, consistent communication by all members of the healthcare team, increased proportion of time spent listening to families speak, the use of empathetic statements, assurance that the loved one will not suffer, and providing explicit support for decisions made by the family.40,41 Current pediatric research suggests that pediatric intensivists fail to incorporate certain aspects that make transdisciplinary conferences effective for furthering PFCC. Often pediatric intensivists dominate conversations in family conferences, focus mostly on the medical aspects of care, and use complex speech, possibly preventing valuable family engagement in these conferences.39 While additional research is needed to detail the benefits and potential negative aspects of this format for communication in the pediatric patient population, using elements of high-quality communication should be considered as potentially useful to optimize communication with families.

Providing Transdisciplinary Support for the Family Unit Fundamental to PFCC is “providing and ensuring formal and informal support for the child and family for each phase of the child’s life.”1 Owing to the complex nature of pediatric critical illness and the high level of emotional distress triggered by hospitalization, a transdisciplinary team approach is necessary to provide full support for the patient and family, including siblings, who are at high risk for feeling abandoned and neglected. Acknowledging that patients and families enter the ICU with varying degrees of internal coping mechanisms in addition to external support systems is critical to optimizing support. Teams from a multitude of disciplines, all with different expertise, should be incorporated into the healthcare team, when appropriate, to help demonstrate the value in providing a holistic approach to care. Nurses at the bedside play a crucial role in recognizing family coping strategies, identifying unmet family needs, and bridging gaps in communication.7 Consultation with subspecialists in palliative care and ethics should be considered early in the course of hospitalization to improve communication, prevent conflict, identify important goals through eliciting patient values and preferences, and ultimately optimize shared medical decisionmaking.5 Physical therapy, occupational therapy, and rehabilitation medicine should be incorporated into the patient’s care using a systematic approach to assist with functional recovery and potentially improve long-term quality of life. Specialists in music therapy, art therapy, pet therapy, psychology, child life, and educational services can improve the quality of life in the ICU and reduce ICU-related morbidities, such as anxiety, depression, delirium, psychosis, and PTSD.4,7 Social workers can provide assistance to reduce both emotional distress and stressors that arise from practical aspects of having a child hospitalized, such as food, transportation, and employment, while chaplains can address the spiritual needs of the patient and family.5,13,51 With the multitudes of providers involved, excellent communication is necessary to avoid additional stressors to families that inconsistent messaging can cause. In addition to incorporating healthcare providers from multiple disciplines, peer-to-peer support can also be highly beneficial.1 Families may choose to serve on patient advocacy committees,



CHAPTER 16

Patient- and Family-Centered Care in the Pediatric Intensive Care Unit

share their experiences with other families through family support groups, and volunteer to serve as consultants to other families who are affected by similar disease processes or experiences. Allowing families who are in a crisis mode to contact and access other families who have shared similar experiences may broaden coping mechanisms and may help guide families through their journey, particularly when they are faced with making very difficult decisions.

Collaborating and Building Partnerships with Patients and Families Developing a partnership with families is essential to fulfilling the fundamental principles of PFCC. A partnership requires an “interpersonal relationship between two or more people who work together to achieve a mutually defined purpose” which, in the PICU, is “providing the best possible care for the child from a holistic perspective.”52 The holistic approach requires inclusion of the family, as the center of strength for the child. When partnering with parents and caregivers, who know the child best and provide invaluable information regarding who the child is as a person, healthcare providers treat the child more effectively and address the child’s needs more appropriately.45 This partnership allows parents and healthcare providers to contribute their own expertise to achieve a common goal, leading to a relationship in which the power differential between healthcare providers and families equalizes.10 While building a partnership with patients and families at the bedside is essential, optimizing PFCC also requires collaboration “at all levels of healthcare: in the delivery of care to the individual child; in professional education, policy making, program development, implementation, and evaluation; and in healthcare facility design.”1 Encouraging families to participate in all dimensions of healthcare through serving on family advisory councils, quality improvement projects, and developing research protocols is critical.1 Furthermore, building this partnership requires a transdisciplinary approach and is “grounded in collaboration among patients, families, physicians, nurses, and other professionals in clinical care.”1 In order to optimize these relationships, clarity and transparency regarding the roles, boundaries, and expertise of each stakeholder is critical.53

Empowering Patients and Families to Facilitate Shared Medical Decision-Making PFCC is optimized through “recognizing and building on the strengths of individual children and families and empowering them to discover their own strengths, build confidence, and participate in making choices and decisions about their healthcare.”1 Empowering the patient and family to participate in shared decision-making is fundamental, for “the perspectives and information provided by families, children, and young adults are essential components of high-quality clinical decision-making.”1 In the setting of the ICU, complex and high-stakes decisions are made in a short time frame. In many circumstances, there is a high degree of uncertainty in outcomes, making these decisions even more difficult. The majority of these clinical decisions are based on the medical expertise, empirical evidence, experience of the healthcare providers and consultants, and—most importantly—values and perspectives of the patient and family. By definition, shared decision-making occurs when both the physician and patient (if possible) and family “share their opinions

141

and jointly reach a decision.”40 Dimensions of shared decisionmaking include (1) providing medical information and eliciting patient values, preferences, and goals; (2) exploring family’s preferred role in decision-making; and (3) deliberation and decisionmaking.40,54 Collaborative communication, as discussed earlier in this chapter, is required to adequately fulfill the first dimension. In regard to dimension two, multiple studies demonstrate that families have varying decision-making preferences, particularly regarding limitation of life support or other aggressive interventions.5,40 Therefore prior to making assumptions, exploring the family’s preferential role in decision-making is important. Furthermore, shared decision-making should not be interpreted as allowing the family to decide without support from the providers or giving families increased responsibility or autonomy,12 as “there is a fine balance between supporting and guiding a family while allowing the family appropriate space to make their own decisions.”23 Most families do not want to feel alone when making difficult decisions. Recommendations and guidance from providers may help alleviate potential burdens associated with making difficult decisions. Finally, shared decision-making does not mean that families should exclusively drive medical care or be empowered to make decisions that are not medically sound. The key aspect of shared decision-making is that the process is collaborative and incorporates the opinions and expertise of all stakeholders. Finding the right balance among stakeholders to achieve goal-oriented patient care through established frameworks for shared decision-making is necessary to provide optimal PFCC.54 There are multiple benefits associated with shared decisionmaking. When patients and families become active participants in their healthcare, there is improved understanding and more motivation to follow through with care. In addition, for parents and caregivers who lose their sense of control with the hospitalization of a critically ill child, participating in decisions can provide a great source of strength. As one mother explains, “I was able to still be her mom.”55 By “encouraging them to continue actively in their parental role by promoting shared decision making and helping the family to retain their responsibilities throughout hospitalization” parents and caregivers retain their identities, which fosters the integrity of the parent-child relationship, maintains cultural and family traditions, and demonstrates respect for and value of the child as a person.52,55 Although further studies are necessary to elucidate the important driving factors for parental decision-making, parents overall want to be “good parents.”56 Understanding what parents value as important factors in “being a good parent” potentially may improve the quality of PFCC.56

Patient- and Family-Centered Care Improves Outcomes for all Stakeholders Over the past 2 decades, PFCC has improved outcomes in the provision of high-quality patient care. According to the AAP, “patient and family-centered care can improve patient and family outcomes, improve the patient’s and family’s experience, increase patient and family satisfaction, build on child and family strengths, increase professional satisfaction, decrease healthcare costs, and lead to more effective use of healthcare resources.”1 According to numerous studies, patients and families directly benefit from the incorporation of the fundamental principles into practice. Patients and families have reduced anxiety, better cooperation, improved satisfaction, reduced emotional distress, better

142

SECTION III



Pediatric Critical Care: Psychosocial and Societal

adjustment to hospitalization, and faster recovery from illness and surgery.1,8 Because the needs of families are addressed more explicitly, families also have improved functioning, increased confidence in providing ongoing care to their child, are more willing to seek help from healthcare providers, develop a greater degree of trust, and demonstrate improved competence in problem-solving and making complicated healthcare decisions.1,8 When families receive clear and consistent communication and actively participate in the care of their child, they experience a greater sense of control, resulting in preservation of the parent-child relationship, with reduced anxiety, posttraumatic stress, and complicated grief.8 Implementing PFCC in PICUs has special implications in the setting of pediatric death. The majority of inpatient pediatric deaths occur in the PICU. Current end-of-life practices in the ICU are suboptimal. Families describe disempowerment, loss of control, and alienation from their dying child in the ICU as a result of both their child’s illness and the cultural and physical environment in the PICU.57 Moreover, bereaved parents are at risk for adverse mental and physical health after their child’s death.58 Compassionate and empathetic PFCC in PICUs may help improve end-of-life care and mitigate bereavement outcomes by helping parents preserve their parental role, engaging families more, maintaining the pediatric patient’s personhood, and fostering meaning making.57 Healthcare providers, institutions, and the healthcare systems also directly benefit from PFCC. When healthcare providers gain important insights into the needs and values of patients and their families, they establish trusting relationships and improve the quality, efficiency, and safety of the care they provide. As a result, providers have improved work satisfaction, which leads to improved job performance, reduced burnout, and decreased staff turnover.1 With the provision of higher-quality care, healthcare institutions and systems benefit with reduced healthcare costs, improved patient and family satisfaction, reduced risk for litigation, and potentially gain a more competitive position in the marketplace.1

Overcoming Barriers and Challenges to Patient- and Family-Centered Care in the Intensive Care Unit While significant progress has been made in incorporating PFCC into the ICU, there remains a gap between current and proposed practice. Ongoing research reveals that families continue to express that their needs are not always met.2,8,15,17,22 While the reason for this gap in practice is likely multifactorial, there are several barriers that are extremely challenging to overcome (Box 16.1). First, there is a fundamental difference in perceiving the ICU as a “bedroom” versus an “office.”21 For families, the ICU needs to serve as a bedroom, where a sick child can experience warmth, comfort, and a healing environment. On the other hand, critical care providers need an office equipped with computers, monitors, alarms, and advanced technology. This dichotomy fundamentally inhibits the creation of a bedroom and reinforces the role of “the visitor” and “the patient,” rather than fostering a collaborative partnership.21 Second, the emotional intensity of life-altering and life-threatening circumstances is unavoidable. The impact of this emotional intensity on patients and families and the complexity of the medical care provided creates an implicit power differential between healthcare providers, patients, and families, further reducing the ability to develop a partnership. Finally, the necessity

• BOX 16.1 Potential Barriers to Patient- and Family-

Centered Care in the Intensive Care Unit • Dichotomy in patient, family, and provider needs of the ICU as a “bedroom” versus an “office” • Emotional intensity of life-altering and life-threatening circumstances • High-stakes nature and complexity of medical decision-making, requiring explicit value-based decisions • Implicit and explicit power differential between healthcare providers, patients, and families • Lack of or minimal formal training in high-quality communication skills for healthcare providers • Enhancement of cultural and language barriers in the fast-paced, emotionally intense, and complex setting • Healthcare provider misperception of increased risk for litigation, distraction, and the inability to teach trainees with greater family involvement • Increased complexity of patient population, leading to medical- and technology-savvy families who have different needs and potentially require different models of care

of making high-stakes decisions wrought with uncertainty exposes fundamental differences in belief and value systems that are potentially challenging to overcome. Even under ideal circumstances with the most advanced communication systems and extensive training in cultural humility, these differences can create intense barriers and conflict rooted in different perceptions of what is in the best interest of the patient. These conflicts further prohibit the development of partnership. Overcoming these fundamental barriers requires acknowledgment, transparency, and further research to identify both the explicit and implicit practices and values within the ICU (Box 16.2).21 Using a model of PFCC, this research requires employing a collaborative approach in which all stakeholders have an equal voice in order to develop creative solutions. In addition to these fundamental barriers, there is an increasing complexity to the patient population that requires the PICU. Accommodating the needs of medically complex children’s parents who are medically and technology savvy may require different models of care.59 PICUs need to be able to fully accommodate for the “skills and expertise” of families of chronically ill and medically complex patients: “there is a clear need for research that identifies what constitutes success to these families, along with the challenges faced by staff who care for them…[in order to] change practice and foster a culture of supportive care inside the PICU.”59 • BOX 16.2 Overcoming Barriers to Patient- and

Family-Centered Care in the Intensive Care Unit • Provide formal training in cultural humility; develop a robust interpreter service and hire cultural navigators. • Provide formal training in collaborative communication and goal-oriented, shared medical decision-making. • Include patient and family input for intensive care unit (ICU) policies, procedures, planning, and delivery of healthcare. • Acknowledge and maintain transparency regarding implicit and explicit values and practices in the ICU. • Support and engage in patient- and family-centered care research to identify and overcome additional barriers. • Develop adequate support systems for healthcare providers who suffer from compassion fatigue and vicarious trauma.



CHAPTER 16

Patient- and Family-Centered Care in the Pediatric Intensive Care Unit

PFCC is not equivalent to family-driven care, in which families exclusively direct the medical care, dominating the relationship. The establishment of professional boundaries and specific roles is critical in the development of mutually respectful and collaborative partnerships. Despite the knowledge that high-quality communication is an essential component of providing comprehensive PFCC, physicians’ communication skills remain suboptimal. Adult and pediatric communication literature has consistently demonstrated that family satisfaction with PFCC is associated with a higher proportion of family speech, empathetic statements, and specific statements of support, as outlined in previous sections of this chapter.39 Yet when participating in care conferences, physicians use complex language, speak for the majority of the time, and miss one-third of families’ emotional cues.38,60 Further research is needed to understand how physicians’ communication skills can be improved and whether training programs can be implemented to improve PFCC.42 Finally, the development of compassion fatigue and secondary traumatization experienced by healthcare providers who are witnesses to extreme suffering of children and families in this complex environment can lead to disengagement from patients and families.61 High clinical workloads and demanding extraneous professional obligations can lead to provider burnout and loss of the ability to maintain an empathetic approach to patients and families. Therefore the needs of healthcare providers should also be explicitly recognized and addressed by institutions and healthcare systems in order to improve the quality of PFCC delivered.

Summary The pediatric critical care unit is a complex environment that creates a high level of stress for patients and families. Incorporating PFCC into practice improves patient and family satisfaction, reduces stress and anxiety, fosters the parent-child relationship, and ultimately increases the quality, efficacy, efficiency, and safety of care delivered. Developing a partnership with patients and families built on mutual respect, using collaborative communication, providing extensive support for the family unit, and encouraging patient and family participation in all aspects of care, including shared medical decision-making, are fundamental principles essential to the practice of PFCC. Overcoming real and

143

perceived barriers to incorporating PFCC into practice requires a collaborative and transparent approach involving all stakeholders to identify creative solutions while providing adequate support to healthcare providers, who are at risk for compassion fatigue, secondary traumatization, and burnout.

Key References Committee on Hospital Care, Institute for Patient- and Family-Centered Care. Patient- and family-centered care and the pediatrician’s role. Pediatrics. 2012;129(2):394-404. Davidson JE, Aslakson RA, Long AC, et al. Guidelines for familycentered care in the neonatal, pediatric, and adult ICU. Crit Care Med. 2017;45(1):103-128. Doorenbos A, Lindhorst T, Starks H, Aisenberg E, Curtis JR, Hays R. Palliative care in the pediatric ICU: challenges and opportunities for family-centered practice. J Soc Work End Life Palliat Care. 2012; 8(4):297-315. Feudtner C. Collaborative communication in pediatric palliative care: a foundation for problem-solving and decision-making. Pediatr Clin North Am. 2007;54(5):583-607, ix. Greenway TL, Rosenthal MS, Murtha TD, Kandil SB, Talento DL, Couloures KG. Barriers to communication in a PICU: a qualitative investigation of family and provider perceptions. Pediatr Crit Care Med. 2019;20:e415-e422. Hill C, Knafl KA, Santacroce SJ. Family-centered care from the perspective of parents of children cared for in a pediatric intensive care unit: an integrative review. J Pediatr Nurs. 2017;S0882-5963(17):3053130536. Macdonald ME, Liben S, Carnevale FA, Cohen SR. An office or a bedroom? Challenges for family-centered care in the pediatric intensive care unit. J Child Health Care. 2012;16(3):237-249. McGraw SA, Truog RD, Solomon MZ, Cohen-Bearak A, Sellers DE, Meyer EC. “I was able to still be her mom”—parenting at end of life in the pediatric intensive care unit. Pediatr Crit Care Med. 2012;13(6): e350-356. Meert KL, Clark J, Eggly S. Family-centered care in the pediatric intensive care unit. Pediatr Clin North Am. 2013;60(3):761-772. October TW, Fisher KR, Feudtner C, Hinds PS. The parent perspective: “being a good parent” when making critical decisions in the PICU. Pediatr Crit Care Med. 2014;15(4):291-298.

The full reference list for this chapter is available at ExpertConsult.com.

e1































































































1. Committee On Hospital C, Institute For P, Family-Centered C. Patient- and family-centered care and the pediatrician’s role. Pediatrics. 2012;129(2):394-404. 2. Merk L, Merk R. A parents’ perspective on the pediatric intensive care unit: our family’s journey. Pediatr Clin North Am. 2013;60(3): 773-780. 3. Atkins E, Colville G, John M. A ‘biopsychosocial’ model for recovery: a grounded theory study of families’ journeys after a paediatric intensive care admission. Intensive Crit Care Nurs. 2012;28(3):133140. 4. Colville G, Darkins J, Hesketh J, Bennett V, Alcock J, Noyes J. The impact on parents of a child’s admission to intensive care: integration of qualitative findings from a cross-sectional study. Intensive Crit Care Nurs. 2009;25(2):72-79. 5. Doorenbos A, Lindhorst T, Starks H, Aisenberg E, Curtis JR, Hays R. Palliative care in the pediatric ICU: challenges and opportunities for family-centered practice. J Soc Work End Life Palliat Care. 2012;8(4):297-315. 6. Nelson LP, Gold JI. Posttraumatic stress disorder in children and their parents following admission to the pediatric intensive care unit: a review. Pediatr Crit Care Med. 2012;13(3):338-347. 7. Shudy M, de Almeida ML, Ly S, et al. Impact of pediatric critical illness and injury on families: a systematic literature review. Pediatrics. 2006;118(suppl 3):S203-218. 8. Just AC. Parent Participation in care: bridging the gap in the pediatric ICU. Newborn Infant Nurs Rev. 2005;5(4):179-187. 9. Hill C, Knafl KA, Santacroce SJ. Family-centered care from the perspective of parents of children cared for in a pediatric intensive care unit: an integrative review. J Pediatr Nurs. 2017;S08825963(17):30531-30536. 10. Institute for Patient- and Family-Centered Care. About PFCC. http://www.ipfcc.org/about/pfcc.html. Accessed 04/09/2019. 11. Frazier ARN, Frazier HRN, Warren NAPRN. A discussion of familycentered care within the pediatric intensive care unit. Crit Care Nurs Q. 2010;33(1):82-86. 12. Kuo DZ, Houtrow AJ, Arango P, Kuhlthau KA, Simmons JM, Neff JM. Family-centered care: current applications and future directions in pediatric health care. Matern Child Health J. 2012; 16(2):297-305. 13. Guidelines for Pediatric Intensive Care Units. Pediatrics. 1983; 72(3):364-372. 14. Meert KL, Clark J, Eggly S. Family-centered care in the pediatric intensive care unit. Pediatr Clin North Am. 2013;60(3):761-772. 15. Coyne I, Cowley S. Challenging the philosophy of partnership with parents: a grounded theory study. Int J Nurs Stud. 2007;44(6):893904. 16. Foster MJ, Whitehead L, Maybee P, Cullens V. The parents’, hospitalized child’s, and health care providers’ perceptions and experiences of family centered care within a pediatric critical care setting: a metasynthesis of qualitative research. J Fam Nurs. 2013;19(4):431-468. 17. Latour JM, van Goudoever JB, Schuurman BE, et al. A qualitative study exploring the experiences of parents of children admitted to seven Dutch pediatric intensive care units. Intensive Care Med. 2011;37(2):319-325. 18. Wong P, Liamputtong P, Koch S, Rawson H. Families’ experiences of their interactions with staff in an Australian intensive care unit (ICU): a qualitative study. Intensive Crit Care Nurs. 2015;31(1):51-63. 19. Juarez JA, Marvel K, Brezinski KL, Glazner C, Towbin MM, Lawton S. Bridging the gap: a curriculum to teach residents cultural humility. Fam Med. 2006;38(2):97-102. 20. Feudtner C. Collaborative communication in pediatric palliative care: a foundation for problem-solving and decision-making. Pediatr Clin North Am. 2007;54(5):583-607, ix.

21. Beck SA, Weis J, Greisen G, Andersen M, Zoffmann V. Room for family-centered care – a qualitative evaluation of a neonatal intensive care unit remodeling project. J Neonatal Nurs. 2009;15(3):88-99. 22. Macdonald ME, Liben S, Carnevale FA, Cohen SR. An office or a bedroom? Challenges for family-centered care in the pediatric intensive care unit. J Child Health Care. 2012;16(3):237-249. 23. Jones BL, Contro N, Koch KD. The duty of the physician to care for the family in pediatric palliative care: context, communication, and caring. Pediatrics. 2014;133(suppl 1):S8-15. 24. Carroll DL. The effect of intensive care unit environments on nurse perceptions of family presence during resuscitation and invasive procedures. Dimens Crit Care Nurs. 2014;33(1):34-39. 25. Dingeman RS, Mitchell EA, Meyer EC, Curley MA. Parent presence during complex invasive procedures and cardiopulmonary resuscitation: a systematic review of the literature. Pediatrics. 2007;120(4):842854. 26. Curley MA, Meyer EC, Scoppettuolo LA, et al. Parent presence during invasive procedures and resuscitation: evaluating a clinical practice change. Am J Respir Crit Care Med. 2012;186(11):11331139. 27. McAlvin SS, Carew-Lyons A. Family presence during resuscitation and invasive procedures in pediatric critical care: a systematic review. Am J Crit Care. 2014;23(6):477-484; quiz 485. 28. Sanders Jr RC, Nett ST, Davis KF, et al. Family presence during pediatric tracheal intubations. JAMA Pediatr. 2016;170(3):e154627. 29. Berube KM, Fothergill-Bourbonnais F, Thomas M, Moreau D. Parents’ experience of the transition with their child from a pediatric intensive care unit (PICU) to the hospital ward: searching for comfort across transitions. J Pediatr Nurs. 2014;29(6):586-595. 30. Davis R. A small kindness. J Hosp Med. 2010;5(9):569-570. 31. Kean S. A Framework for a Physician-parent follow-up meeting after a child’s death in a PICU and why this family-centered care approach should interest us all. Crit Care Med. 2014;42(1):214-216. 32. Meert KL, Eggly S, Berg RA, et al. Feasibility and perceived benefits of a framework for physician-parent follow-up meetings after a child’s death in the PICU. Crit Care Med. 2014;42(1):148-157. 33. Walter JK, Benneyworth BD, Housey M, Davis MM. The factors associated with high-quality communication for critically ill children. Pediatrics. 2013;131(suppl 1):S90-95. 34. de Vos MA, Bos AP, Plotz FB, et al. Talking with parents about endof-life decisions for their children. Pediatrics. 2015;135(2):e465476. 35. Meert KL, Eggly S, Pollack M, et al. Parents’ perspectives on physician-parent communication near the time of a child’s death in the pediatric intensive care unit. Pediatr Crit Care Med. 2008;9(1):2-7. 36. Greenway TL, Rosenthal MS, Murtha TD, Kandil SB, Talento DL, Couloures KG. Barriers to Communication in a PICU: a qualitative investigation of family and provider perceptions. Pediatr Crit Care Med. 2019;20(9):e415-e422. 37. Orioles A, Miller VA, Kersun LS, Ingram M, Morrison WE. “To be a phenomenal doctor you have to be the whole package”: physicians’ interpersonal behaviors during difficult conversations in pediatrics. J Palliat Med. 2013;16(8):929-933. 38. October TW, Dizon ZB, Roter DL. Is it my turn to speak? An analysis of the dialogue in the family-physician intensive care unit conference. Patient Educ Couns. 2018;101(4):647-652. 39. October TW, Hinds PS, Wang J, Dizon ZB, Cheng YI, Roter DL. Parent satisfaction with communication is associated with physician’s patient-centered communication patterns during family conferences. Pediatr Crit Care Med. 2016;17(6):490-497. 40. Curtis JR, White DB. Practical guidance for evidence-based ICU family conferences. Chest. 2008;134(4):835-843. 41. Lautrette A, Darmon M, Megarbane B, et al. A communication strategy and brochure for relatives of patients dying in the ICU. N Engl J Med. 2007;356(5):469-478.

References

e2

















56.





57.





58.





59.





60.



61.











55.





54.





53.















42. Davidson JE, Aslakson RA, Long AC, et al. Guidelines for familycentered care in the neonatal, pediatric, and adult ICU. Crit Care Med. 2017;45(1):103-128. 43. McPherson G, Jefferson R, Kissoon N, Kwong L, Rasmussen K. Toward the inclusion of parents on pediatric critical care unit rounds. Pediatr Crit Care Med. 2011;12(6):e255-261. 44. Drago MJ, Aronson PL, Madrigal V, Yau J, Morrison W. Are family characteristics associated with attendance at family centered rounds in the PICU? Pediatr Crit Care Med. 2013;14(2):e93-97. 45. Ingram TC, Kamat P, Coopersmith CM, Vats A. Intensivist perceptions of family-centered rounds and its impact on physician comfort, staff involvement, teaching, and efficiency. J Crit Care. 2014;29(6):915-918. 46. Walker-Vischer L, Hill C, Mendez SS. The experience of Latino parents of hospitalized children during family-centered rounds. J Nurs Adm. 2015;45(3):152-157. 47. Davidson JE. Family presence on rounds in neonatal, pediatric, and adult intensive care units. Ann Am Thorac Soc. 2013;10(2):152-156. 48. Gupta PR, Perkins RS, Hascall RL, Shelak CF, Demirel S, Buchholz MT. The effect of family presence on rounding duration in the PICU. Hosp Pediatr. 2017;7(2):103-107. 49. McDonagh JR, Elliott TB, Engelberg RA, et al. Family satisfaction with family conferences about end-of-life care in the intensive care unit: increased proportion of family speech is associated with increased satisfaction. Crit Care Med. 2004;32(7):1484-1488. 50. Scheunemann LP, McDevitt M, Carson SS, Hanson LC. Randomized, controlled trials of interventions to improve communication in intensive care: a systematic review. Chest. 2011;139(3):543-554. 51. Majdalani MN, Doumit MA, Rahi AC. The lived experience of parents of children admitted to the pediatric intensive care unit in Lebanon. Int J Nurs Stud. 2014;51(2):217-225. 52. Ames KE, Rennick JE, Baillargeon S. A qualitative interpretive study exploring parents’ perception of the parental role in the paediatric

intensive care unit. Intensive Crit Care Nurs. 2011;27(3): 143-150. Baird J, Davies B, Hinds PS, Baggott C, Rehm RS. What impact do hospital and unit-based rules have upon patient and family-centered care in the pediatric intensive care unit? J Pediatr Nurs. 2015;30(1):133-142. Quill TE, Holloway RG. Evidence, preferences, recommendations-finding the right balance in patient care. N Engl J Med. 2012; 366(18):1653-1655. McGraw SA, Truog RD, Solomon MZ, Cohen-Bearak A, Sellers DE, Meyer EC. “I was able to still be her mom”--parenting at end of life in the pediatric intensive care unit. Pediatr Crit Care Med. 2012;13(6):e350-356. October TW, Fisher KR, Feudtner C, Hinds PS. The parent perspective: “being a good parent” when making critical decisions in the PICU. Pediatr Crit Care Med. 2014;15(4):291-298. Butler AE, Hall H, Willetts G, Copnell B. Family experience and PICU death: a meta-synthesis. Pediatrics. 2015;136(4):e961973. Hendrickson KC. Morbidity, mortality, and parental grief: a review of the literature on the relationship between the death of a child and the subsequent health of parents. Palliat Support Care. 2009; 7(1):109-119. Rennick JE, Childerhose JE. Redefining success in the PICU: new patient populations shift targets of care. Pediatrics. 2015;135(2):e289291. October TW, Dizon ZB, Arnold RM, Rosenberg AR. Characteristics of physician empathetic statements during pediatric intensive care conferences with family members: a qualitative study. JAMA Netw Open. 2018;1(3):e180351. Meadors P, Lamson A. Compassion fatigue and secondary traumatization: provider self care on intensive care units for children. J Pediatr Health Care. 2008;22(1):24-34.

e3

Abstract: Patient- and family-centered care (PFCC) is a model of care delivery that partners with patients and their families to optimize the provision of high-quality healthcare. It is built on mutual respect, collaborative communication, and shared decision-making. This chapter details the benefit of incorporating PFCC into practice, including improving patient and family satisfaction; reducing stress and anxiety; and, ultimately, increasing

the quality, efficacy, efficiency, and safety of care delivered. It also outlines the barriers to implementing PFCC in current practice. Key Words: Patient- and family-centered care, family satisfaction, shared medical decision-making, collaborative communication, family-centered rounds, cultural humility, transdisciplinary care

17 Pediatric Critical Care Ethics MITHYA LEWIS-NEWBY, EMILY BERKMAN, AND DOUGLAS S. DIEKEMA

PEARLS

“Bioethics is the discipline devoted to the identification, analysis, and resolution of value-based problems and competing moral claims that arise in medicine between patients, families, healthcare professionals, healthcare institutions, and society at large.”1

Examples of Ethical Issues in the Pediatric Intensive Care Unit





Critical care clinicians (including physicians, nurses, respiratory therapists, and other staff) face issues every day in the practice of pediatric critical care medicine. Some ethical issues occur daily (everyday ethics) in the pediatric intensive care unit (PICU) but may be subtle and difficult to recognize, such as how rounds are prioritized or how implicit biases are infused into clinician communication and decision-making. Other ethical issues, such as a heated disagreement between the medical team and a family about the best course of action for a critically ill child (crisis ethics), are typically more obvious to everyone involved. The following are examples of ethical issues that may arise in the PICU: • Prioritizing rounds to address the sickest first (prioritarianism and scarce resource allocation) • Advising a family to withdraw life-sustaining therapies in the setting of a severe brain injury (value-based judgment) 144







• Transfusing a hemorrhaging child against the wishes of the child’s parents, who are of the Jehovah’s Witness faith (best interest standard and harm principle) • Treating an air-hungry dying child with morphine to the point of unconsciousness and bradypnea (doctrine of double effect) • Lifting sedation on an 18-year-old on extracorporeal membrane oxygenation (ECMO) to discuss the possible limitation of life support (doctrine of informed consent) • Allocating PICU beds when census reaches capacity (scarce resource allocation)



Defining Bioethics







consistent with the patient’s previously expressed values (“substituted judgment standard”). For patients who have not previously been competent, surrogates must decide what is in the best interest of the patient from the patient’s perspective (“best interest standard”). Except in emergencies, clinicians must obtain legal permission to override parental refusals of recommended medical services. Clinicians must establish that the intervention will benefit the child and that forgoing the intervention places the child at significant risk of serious harm. Mediation and negotiation toward finding a mutually acceptable solution should be attempted before seeking legal intervention. Disputes regarding potentially inappropriate or “futile” services in cases in which there is a lack of consensus about what constitutes accepted medical practice should be resolved through a fair process-based approach.













Ethical issues in the pediatric intensive care unit (PICU) include high-visibility crises as well as subtle everyday ethical issues that stem from values and biases that infuse daily decisions. Critical care clinicians should develop an approach to ethical issues that includes (1) recognition and clarification; (2) information gathering; (3) ethical analysis; (4) communication of recommendations; and (5) support of the patient, family, and medical team. Autonomy to make choices in medical decisions is embodied in the requirements of the doctrine of informed consent: disclosure, understanding, competency, and voluntariness. Adolescents designated as emancipated or mature minors may be considered competent to make independent medical decisions. Clinicians should strongly consider including adolescents in medical decisions even if they do not possess the legal right to do so. Surrogate decision-making is common in the PICU. For previously competent patients, surrogates should make decisions





Domains of Bioethics The practice of bioethics encompasses many different domains. These domains may be present to varying degrees in individual ethical issues.

Value-Based Decision-Making In pluralistic societies (such as the United States) where moral diversity is prevalent, individuals or groups may have competing or conflicting moral claims. These moral claims are based on differing values that are not easily compared. For example, some individuals may place great value on the extension of life even if it entails significant burdens, whereas other individuals may value the quality of life more highly than life extension. The clinician



CHAPTER 17 Pediatric Critical Care Ethics

should make every effort to identify and understand the moral values that underlie the positions of different stakeholders and understand that these values are culturally embedded.

State and National Laws and Legal Precedence The law interacts frequently with bioethics in several important ways. First, the answer to many questions that are framed as ethical issues is based on the law (e.g., age of competency for decisionmaking). Second, at times, legal action may be required for the resolution of ethical issues (e.g., a court-appointed decisionmaker may be required in certain cases of medical neglect). Finally, legal precedent may help inform analogous bioethics cases (e.g., previous cases regarding emergent blood transfusions or the prolongation or withdrawal of life support). Professional Codes and Healthcare Organization Policies and Regulations Most medical disciplines adhere to a professional code of ethics.1,2 These codes should be considered in ethical analysis. Additionally, healthcare organizations issue various policies around ethics (e.g., parental request for potentially inappropriate therapies, allocation of resources, conscientious objection, or disclosure of medical error) with which the ethicist and clinician must be familiar. Finally, national healthcare organizations—such as Medicaid and Medicare, as well as insurance companies—may have regulations that impact ethical practice (such as the Centers for Medicare and Medicaid Services [CMS] requirement that all patients older than 18 years be asked about an advance directive on admission to the hospital). Communication, Negotiation, and Mediation A large portion of ethical issues stems from communication that has broken down. In many cases, what may seem like an intractable dispute can be resolved by repairing communication between parties. Resolving some communication problems may be facilitated by the ethicist, a role in which expert conflict mediation skills prove valuable. For example, staff distress may be addressed with a staff-only meeting to better understand the roots of the distress and allow a venue in which differences of opinion can be aired. Another example includes family members who feel that their goals are unheard or misunderstood and who may benefit from a series of facilitated care conferences with the medical team at which a common understanding of the situation and goals of care might be achieved.

Prevailing Ethical Theories and Norms Certainly, the ethicist should be knowledgeable about prevailing ethical theories and norms and adept at applying them to bioethical dilemmas (see later section for more details about specific ethical theories). There is no overarching ethical theory that can resolve all ethical dilemmas. Instead, ethical dilemmas are often examined under the lens of several theories in order to come to a “best possible” recommendation. It is important to recognize that these ethical theories and norms are not static and that they are society and culture dependent.

Who Should Address Ethical Issues in the Pediatric Intensive Care Unit? A wide variety of ethical issues arise in the PICU. Different ethical issues may require different levels of analysis and resolution. Some issues may be resolved with relatively basic skills, while the resolution of other issues may require significantly advanced skills.

145

Some ethical issues may be resolved easily and quickly by the critical care clinician, others may require the advanced skills of an ethics consultant, and others may require review by a multimember ethics committee.

Critical Care Team Many ethical issues can and should be addressed by the intensivist or PICU team. Just as pediatric intensivists are trained to provide primary cardiology, neurology, nephrology, palliative, and other specialized care with subspecialty consultation in complex cases, the same should be true for bioethics. All pediatric intensivists should be trained in and pursue continuing education in bioethics. Ethical issues are common in the PICU; all intensivists should have a solid understanding of the basic aspects involved in identifying, analyzing, and resolving bioethical dilemmas. This will require an understanding of the basics of the domains mentioned earlier as well as comfort with a basic tool set for approaching ethical issues. Routinely addressing basic ethical issues may resolve simpler bioethical issues quickly, help in deescalating more complex conflicts before they become intractable, and, in some cases, prevent issues from arising in the first place.

Ethics Consultant For more complex bioethical dilemmas, pediatric bioethics specialists may be consulted. Ethicists come from a variety of disciplines, training, and experiences. Ethics consultants should have advanced skills in ethical assessment and analysis, ethical and hospital processes, and interpersonal skills in negotiation, communication, and facilitation. Additionally, ethics consultants should be experienced in advanced moral reasoning and ethical theory, be facile with advanced bioethical concepts, and have a strong knowledge base about how healthcare systems, clinical context, institutional policies, and health law impact ethical decisions.3 The pediatric bioethics consultant should also be knowledgeable about the unique ethical issues that arise in the pediatric setting.

Ethics Committee Some ethical issues may require resolution or final recommendation from a full ethics committee composed of members from a wide variety of disciplines (including nonmedical members of the local community) and representing a diverse set of values, experiences, and perspectives. Ideally, ethics committees should promote a fair process and reduce the risk of arbitrariness.3,4 Examples of issues that are optimal for an ethics committee review include cases involving an intractable conflict of values, potentially high-visibility cases, and institutional ethics issues. Ethics committees may also perform post-hoc review of cases handled by ethics consultants for purposes of quality improvement.

Approach to Bioethics Dilemmas in the Pediatric Intensive Care Unit Critical care clinicians should learn and become competent with a basic organized analytic approach to ethical dilemmas. Several approaches have been published, most containing the same basic elements.5–7 In general, a systematic approach to ethical issues will involve five elements: recognition and clarification, information gathering, analysis of issues, communication of recommendations, and support (Fig. 17.1 and Tables 17.1 and 17. 2).

146

SECTION III



Pediatric Critical Care: Psychosocial and Societal

Clinical reasoning Problem

Clinical ethical reasoning Problem

TABLE CASES: A Step-by-Step Approach to Ethics 17.1 Consultation

C History

Medical facts

Exam

Medical goals

Data

Differential diagnosis, clinical assessment

A

S

Differential diagnosis, ethical assessment

S Treatment plan

•  Fig. 17.1

Best course of action

Comparison between clinical reasoning and clinical ethical reasoning. (From Kaldjian L, Weir R, Duffy T. A clinician’s approach to clinical ethical reasoning. J Gen Intern Med. 2005;20:306–311.)

Determine whether a formal meeting is needed Engage in ethical analysis Identify the ethically appropriate decision-maker Facilitate moral deliberation about ethically justifiable options

Explain the Synthesis • • • •

Further information and dialogue

Consider the types of information needed Identify the appropriate sources of information Gather information systematically from each source Summarize the case and the ethics questions

Synthesize the Information • • • •

E Further diagnostic evaluation

Assemble the Relevant Information • • • •

Patient goals

Context

Clarify the Consultation Request • Characterize the type of consultation request • Obtain preliminary information from the requester • Establish realistic expectations about the consultation process • Formulate the ethics questions

Communicate the synthesis to key participants Provide additional resources Document the consultation in the health record Document the consultation in consultation service records

Support the Consultation Process • • • •

Follow up with participants Evaluate the consultation Adjust the consultation process Identify underlying systems issues



Recognition and Clarification of Ethical Issues The first step in approaching any ethical issue is to recognize that the situation raises a question related to ethics. Some issues may be more appropriately handled by a child protection team (when neglect or abuse is obviously the issue) or a palliative care team (when there is no conflict over the appropriateness of palliative care but help is required in transitioning to a palliative care plan; see also Chapter 16). During this first stage, it is also appropriate to assess and optimize communication. Communication that has broken down is frequently a contributor—if not the basis—of ethical disputes. At a minimum, improving communication commonly will help to diffuse a crisis so that the issue can be addressed productively. At times, improved communication between parties may be sufficient to resolve the issue altogether. Finally, once an ethical issue has been identified, the next step is to try to articulate, as precisely as possible, the nature of the ethical question.

Information Gathering “Good ethics requires good facts” is a common saying in bioethics circles. Sound bioethical recommendations can arise only from a solid understanding of the facts involved in the case. Facts include

From National Center for Ethics in Health Care. Ethics Consultation: Responding to Ethics Questions in Health Care. 2nd ed. Washington, DC: U.S. Department of Veterans Affairs; 2015:viii. https://www.ethics.va.gov/docs/integratedethics/ec_primer_2nd_ed.pdf

medical facts, such as the mortality rate for a procedure or condition both at the institution and nationally, alternate medical options, or an accurate assessment of the child’s medical condition. Facts also include relevant contextual details, such as family culture and religion, family circumstances, available resources and support, and more. A useful tool for gathering facts surrounding an ethical issue is the four categories method (see Table 17.2).7 During this phase, it is important to seek to understand the perspectives, values, and goals of the patient (if possible), the family or other surrogates, and various involved members of the medical team (including primary physicians, consulting physicians, nurses, and other staff). When possible, these perspectives should be obtained firsthand to avoid the inevitable misunderstandings that result from secondhand or thirdhand information. It is important to understand stakeholders’ opinions but also the goals and values behind their opinions. Often, a better understanding of the stakeholders’ underlying values and deeper goals can help to clarify stated requests and open a path to resolution. Finally, identifying or refining the core ethical dilemma or primary conflict may be easier once the appropriate data have been collected and the voices of those with a stake in the situation have been heard. It is an important skill and part of the assessment to be able to identify and clarify the ethical dimensions of the dilemma.



CHAPTER 17 Pediatric Critical Care Ethics

147

TABLE A Case-Based Approach to Ethical Decision-Making 17.2

MEDICAL INDICATION

PATIENT PREFERENCES

Principles of Beneficence and Nonmaleficence

Principles of Respect for Autonomy

• What is the patient’s medical problem? Is the problem acute? Chronic? Critical? Reversible? Emergent? Terminal? • What are the goals of treatment? • In what circumstances are medical treatments not indicated? • What are the probabilities of success of various treatment options? • In sum, how can this patient be benefited by medical and nursing care, and how can harm be avoided?

• Has the patient been informed of benefits and risks, understood this information, and given consent? • Is the patient mentally capable and legally competent, and is there evidence of incapacity? • If mentally capable, what preferences about treatment is the patient stating? • If incapacitated, has the patient expressed prior preferences? • Who is the appropriate surrogate to make decisions for the incapacitated patient? • Is the patient unwilling or unable to cooperate with medical treatment? If so, why?

QUALITY OF LIFE

CONTEXTUAL FEATURES

Principles of Beneficence, Nonmaleficence, and Respect for Autonomy

Principles of Justice and Fairness

• What are the prospects, with or without treatment, for a return to normal life, and what physical, mental, and social deficits might the patient experience even if treatment succeeds? • On what grounds can anyone judge that some quality of life would be undesirable for a patient who cannot make or express such a judgment? • Are there biases that might prejudice the provider’s evaluation of the patient’s quality of life? • What ethical issues arise concerning improving or enhancing a patient’s quality of life? • Do quality-of-life assessments raise any questions regarding changes in treatment plans, such as forgoing life-sustaining treatment? • What are the plans and rationale to forgo life-sustaining treatment? • What is the legal and ethical status of suicide?

• Are there professional, interprofessional, or business interests that might create conflicts of interest in the clinical treatment of patients? • Are there parties other than clinicians and patients, such as family members, who have an interest in clinical decisions? • What are the limits imposed on patient confidentiality by the legitimate interests of third parties? • Are there financial factors that create conflicts of interest in clinical decisions? • Are there problems of allocation of scarce health resources that might affect clinical decisions? • Are there religious issues that might affect clinical decisions? • What are the legal issues that might affect clinical decisions? • Are there considerations of clinical research and education that might affect clinical decisions? • Are there issues of public health and safety that affect clinical decisions? • Are there conflicts of interest within institutions or organizations (e.g., hospitals) that may affect clinical decisions and patient welfare?

Modified from Jonsen AR, Siegler M, Winslade W. Clinical Ethics. 7th ed. New York: McGraw-Hill; 2010. https://depts.washington.edu/bhdept/node/84.

Analysis of Ethical Issues Once the ethical question has been clarified, the facts have been gathered, and the voices of the stakeholders heard, the next step is to begin an ethical analysis of the case. Relevant institutional policies, healthcare regulations, and legal standards should be applied to the case. In value-based ethical dilemmas, a variety of ethical theories may be applied to assist in determining the recommended course of action. It is important to understand that there is no consensus on a predominant ethical theory. All ethical theories have benefits and limitations. They can and should be used in combination to help analyze the situation and assist in coming to the “best possible” set of recommendations. The following is a limited example of ethical theories that may be applied to the analysis of ethical dilemmas.

Consequentialism A consequentialist approach to moral decisions will focus primarily on the predicted outcomes of various choices. Of reasonable options available, the best choice will be the one most likely to provide the most favorable balance of benefit versus burden. Generally, in bedside decision-making, a consequentialist approach is based in the principle of beneficence, focusing the analysis on the benefits and burdens to the patient.8

Deontology A deontologic approach to moral decisions will focus on moral duties, rights of others, and ethical rules and principles, regardless of the ultimate outcome of the decision. A deontologist might, for example, insist on the application of a rule to always tell the truth, rather than attempting to assess whether telling the truth would result in a good or bad outcome.

Principalism Most medical ethicists agree on a small number of principles that should generally guide medical behavior. These include respect for autonomy, beneficence, nonmaleficence, and justice. The principle of respect for autonomy places the desires and decisions of the competent patient as the most important consideration in deciding on a course of action. Because individual values about benefits and harms differ, individual wishes should be respected. The clinician’s primary duty is to ensure that the patient has the information required to make a decision and understands that information. Some have argued for a broader principle of respect for persons on the basis that there are ways of respecting persons that are important even for nonautonomous individuals (such as children). The principle of beneficence requires that clinicians take positive steps to help their patients and that medical interventions should

148

SECTION III



Pediatric Critical Care: Psychosocial and Societal

ultimately benefit the patient. This principle demands that the primary consideration in all therapeutic decisions be the good of the patient. Closely related to the principle of beneficence is the principle of nonmaleficence. Nonmaleficence holds that clinicians have a duty to avoid causing unnecessary harm to patients. In the PICU, many medical therapies (such as intubation) inadvertently cause suffering or are burdensome; the principle of nonmaleficence requires that the benefit of these procedures justify the harm, burden, or suffering that may occur as a result of their use. The principle of double effect (see Chapter 18 for further details) is sometimes used in determining whether an action that causes both benefit and harm can be justified. At its core, this principle requires that clinicians always consider the suffering of the patient when making medical recommendations. Finally, the principle of justice is a complicated principle that seeks fairness when competing claims exist. Among other things, the principle requires that scarce resources be distributed fairly and not based on irrelevant factors. One limitation of principalism is that it lacks guidance on how to prioritize these principles when they are in conflict with one another in an individual case.9

Virtue-Based Ethics Virtue-based theories of ethics emphasize the moral character of the individual clinician. In other words, the clinician who possesses desirable, virtuous characteristics is more likely to make the best ethical choice most consistently. Virtue-based approaches tend to be most helpful when addressing boundary issues, conscientious objection, or other issues related to professionalism and are often less helpful in the more typical conflicts that occur in the patient care setting. Casuistic Ethics A casuistic approach to ethical issues analyzes the current ethical issue by comparing it with prior similar cases and others’ past experiences and previous outcomes to help determine the best decision. It is similar to the reasoning used in legal cases, in which key precedential cases may be used as “anchors” for appropriate resolutions. The analysis then seeks to explore similarities and differences between the current situation and those precedential cases in determining whether similar or different resolutions would be most appropriate. Care Ethics Ethics of care shifts the moral question from “what is just?” to “how should we respond?” Instead of basing decisions on universal standards and impartiality, the ethics of care argues that caring for others and preserving relationships are the foundations of morality. Narrative Ethics Narrative approaches to ethical issues emphasize the importance of understanding cases by taking all their details and complexities into account while seeking to avoid reducing cases to essential elements and then applying a rule or principle. In this approach, details matter, and the resolution of a specific situation may be determined by how each of the possible options will best fit the narrative of the patient’s life (or death). Communitarian Ethics Communitarianism shifts focus from the individual to the family and community. It finds intrinsic value in social obligation and the common good rather than in individual autonomy. What is

in the best interest of the family or society supersedes the best interest of the individual patient. During the process of ethical analysis and consideration of recommendations, it may be helpful to consider whether a solution exists that has not yet been considered by the different stakeholders. Initially, in a conflict, stakeholders may strongly advocate for their specific requests. Through a thorough examination of the case, optimization of communication, and a deeper understanding of the underlying goals and values of the stakeholders, an alternative solution that may satisfy all stakeholders, at least to some degree, may become apparent. Clinical ethicists should always seek to identify these alternate, often creative, solutions.

Communication of Recommendations Once an ethical analysis has been performed, the resulting recommendations must be communicated to the parties with a stake in the outcome. This may include a note or notes in the patient’s chart and may involve discussions with the patient, family, and staff involved in the care of the patient. Care should be taken to fully and transparently disclose the rationale behind the recommendations. Stakeholders should have the opportunity for appeal, a process that may be facilitated by the ethics committee, a patient advocate, or some other institutional mechanism. Importantly, the recommendations of ethics consultants and committees are just that—recommendations to the clinical team. Support for the recommendations and ultimate decision-making may require hospital administration or even the courts in some cases.

Support Even after a decision has been made about how to proceed, patients, families, and staff often require additional support.

Address Staff Distress Medical staff distress is often an important component of ethical issues. Staff may experience moral distress from continuing or not pursuing a particular plan for a patient. Addressing staff distress through individual conversations or staff-only meetings may help to deescalate the conflict. Failure to address staff distress may contribute to burnout, job dissatisfaction, and compassion fatigue.10 Support the Patient and Family Patients and families involved in an ethical dilemma are often under a tremendous amount of stress. This is in addition to the stresses that result simply from having a child admitted to the PICU. Families may perceive a lack of power and support in the PICU environment—even the ethics consultants and committees may be seen as part of the power structure of the hospital. It is important to identify advocates and support mechanisms for patients and families under these conditions.

Ethics of Patient and Surrogate Decision-Making Although the concept of patient autonomy seems obvious to all who practice medicine in this era, it was only a little over 60 years ago when the trend was for doctors to be more paternalistic and directive about medical decisions and for patients to accept these



CHAPTER 17 Pediatric Critical Care Ethics

decisions. In the 1960s, a patient movement began to advocate for more patient autonomy in decision-making. In that era, the predominant ethical cases surrounded the right to die. A landmark case in 1975 involved Karen Ann Quinlan: her parents wanted the right to remove her ventilator, as she was in a persistent vegetative state. The physicians refused, believing that they would be killing her and fearing homicide charges. The beginning of the patient autonomy movement was focused on the right to refuse treatments. Generally speaking, these trends toward autonomy and personal choice became embodied in the doctrine of informed consent. Over the past several decades, the pendulum has continued to swing past “right to die” cases. More recent trends in the patient autonomy movement surround the “right to live” and “right to demand treatments.”

Patient Decision-Making A patient is given a tremendous amount of latitude to accept or refuse offered medical treatments if the patient is deemed competent for decision-making.

Doctrine of Informed Consent The doctrine of informed consent applies to both medical decisions and research. Informed consent must satisfy four requirements that apply when surrogates provide permission as well as when consent is obtained directly from patients. Disclosure means that the clinician should supply the patient with sufficient information that a “reasonable person” would desire to be able to make an informed medical decision. Understanding means that the clinician should assess the patient’s understanding of the proposed course of action, the risks and benefits of that course of action, and any available alternatives along with the risks and benefits associated with them. Understanding may be particularly impaired in the critical care setting, in which the high stakes and time pressures can impact the ability to achieve optimal understanding. Capacity means that the patient must meet legal requirements for competency, be able to understand the medical decision, form a reasonable judgment based on the consequences of the decision, and be able to communicate that decision to others. Legally, children younger than the age of 18 years are not considered competent for medical decision-making with the exception of emancipated and mature minors. Emancipated minors are considered competent on the basis of characteristics that are defined by state law but that may include pregnancy, parenthood, or establishing financial independence. Mature minors represent another category that is defined by state law whereby a minor, usually above a certain age, can be judged competent to make certain medical decisions. Most states require a judge to make these determinations, and the judge may restrict the determination to the medical decisions at hand. Voluntariness means that decisions must be voluntary and not subject to coercion, manipulation, or undue influence. Importantly, physicians should not withhold or deemphasize information in an effort to manipulate patients.11

Emergency Exception to Informed Consent Under specific emergent circumstances, informed consent may be forgone in order to provide necessary lifesaving medical interventions. The emergency exception requires that the medical care in question is required emergently, the patient is incompetent, that no surrogates are readily available, and that medical intervention is needed to save the patient’s life or prevent permanent disability.

149

Advance Directives Advance directives allow competent persons to document their values and medical decision-making preferences before becoming incapacitated patients.12 Living wills document values and desires in writing, and healthcare durable power of attorney designates a surrogate who presumably understands the patient’s values and desires. The Patient Self-Determination Act requires all Medicare/ Medicaid participating institutions to inform patients older than the age of 18 years of their rights to formulate advance directives on admission. Advance directives for children can also be developed by parents or other guardians with input from children as appropriate.13

Child and Adolescent Decision-Making Children younger than the age of 18 years are generally considered not competent to make medical decisions unless they meet criteria for emancipation or mature minor status. However, most agree that the opinions of children and adolescents should not simply be disregarded. Adolescents should be involved in discussions about their healthcare and should be offered the opportunity to voice their feelings, opinions, and concerns. They should also be provided reasonable opportunities to make choices and have those choices respected.14 On the other hand, there is no consensus as to whether any adolescent is truly mature enough to refuse lifesaving treatment in situations in which there is likely to be a good prognosis with a proven intervention. Although adolescents older than the age of 13 years are generally capable of making rational decisions, they are less likely to do so under conditions of high emotion or intense pressure. They are more likely than adults to act impulsively without full consideration of consequences, and they tend to weigh current rewards and harms more strongly than future consequences of a decision. Many factors would be relevant in determining whether an adolescent possesses sufficient maturity to make a life-altering medical decision. Minimally, however, judges and clinicians should require a high level of psychosocial maturity and consider the adolescent’s ability to understand and reason, project meaningfully into the future, express a relatively settled set of values and beliefs, and demonstrate that the adolescent’s decision is driven more by long-term interests than short-term concerns. The chances of a good outcome with treatment and the burden of the proposed interventions are also relevant considerations. In general, it would be unusual to allow an adolescent to refuse interventions in a situation in which the parents would not be allowed to make that decision for the adolescent.15

Shared Decision-Making The patient autonomy movement that began in the 1960s moved medical decision-making away from a predominantly paternalistic approach. There has been some concern, however, that in an effort to prioritize patient autonomy, patients are all too frequently provided with a menu of options without sufficient guidance in decision-making. The middle ground is shared decisionmaking. Shared or collaborative decision-making should combine the clinician’s expert knowledge and experience with the patient’s and family’s values and preferences.16,17 Even though patients are rightfully granted autonomy to give informed consent, clinicians should not abdicate responsibility for recommending a course of action based on the patient’s values and guiding the patient through the decision-making process (see Chapter 16 for additional discussion).

150

SECTION III



Pediatric Critical Care: Psychosocial and Societal

Surrogate Decision-Making In critical care medicine (adult and pediatric), it is not uncommon for patients to lack decision-making capacity. In these situations, a surrogate decision-maker is required to participate in medical decision-making. Surrogate decision-making falls into two categories: decision-making for patients who were previously recognized as competent under the law (e.g., a 22-year-old) and patients who have never been competent (e.g., a 4-month-old). Surrogates must adhere to different standards in each of these two categories.

Surrogate Decision-Making for Previously Competent Patients In the category of the formerly competent patient, the surrogate decision-maker (parent, spouse, etc.) must adhere to the substituted judgment standard. In other words, the surrogate is asked to make decisions most consistent with what the patient would have wanted. This is an attempt to preserve the patient’s autonomy and honor the patient’s values. There may be some cases in the PICU for which this standard applies, such as older adolescents who have been living with chronic disease or young adults who have been admitted to the unit. Surrogate Decision-Making for Never-Competent Patients In the category of the not previously competent patient (which includes the majority of patients in the PICU), surrogates are held to a different standard—the best-interest standard. In theory, most children have not yet developed stable values and beliefs; therefore these values cannot be known and cannot be used for surrogate decision-making. The best-interest standard attempts to maximize the benefit-to-burden ratio for the patient from the patient’s perspective. This, of course, is open to interpretation (and conflict) regarding how to calculate and weigh the various benefits and burdens as they would be experienced by the patient. Quality of life is a phrase that arises commonly in conversations about a patient’s best interest. It is important to keep in mind that quality of life is a subjective and value-based assessment. Clinicians should be aware of their own biases, and caution should be used when applying this concept to the best-interest standard. Parents as Surrogate Decision-Makers Parents (or guardians) are generally empowered to make healthcare decisions on behalf of their children and, with few exceptions, have the legal authority to do so. From an ethical perspective, parents are generally considered the default surrogate decision-makers for their children because they are most likely to understand the unique needs of each child and are presumed to desire what is best for the child. Additionally, some degree of family autonomy is considered an important social value. Finally, in settings of uncertainty, family values and competing family interests may be considered. Limits of Parental Refusals In most situations, parents are granted wide latitude in the decisions they make on behalf of their children; the law has respected those decisions except when they place the child’s health, wellbeing, or life in jeopardy.18,19 Parental authority is not absolute, however—when a parent or guardian fails to adequately guard the interests of a child, the decision may be challenged and the state may intervene. A clinician’s authority to interfere with parental

decision-making is limited. Except in emergency situations in which a child’s life is threatened imminently or a delay would result in significant suffering or risk to the child, the physician cannot do anything to a child without the permission of the child’s parent or guardian. Touching (physical examination, diagnostic testing, or administering a medication) without consent is generally considered a battery under the law. The clinician’s options include either tolerating the parents’ decision (while continuing to try to convince them to act otherwise) or involving a state agency to assume medical decision-making authority on behalf of the child. Only the state can order a parent to comply with medical recommendations. This can take different forms but most frequently either includes involvement of child protective services (under a claim of medical neglect) or a court order. Both options represent a serious challenge to parental authority and will generally be perceived as disrespectful and adversarial by parents. Such action interferes with family autonomy, can adversely affect the family’s future interactions with medical professionals, and can negatively impact the emotional well-being of the child. Neither should be undertaken without serious consideration. Before initiating involvement of state agencies to limit parental authority and override parental refusal, the clinician must establish that (1) the recommended course of action is likely to benefit the child in an important way, (2) the treatment is of proven efficacy with a reasonable likelihood of success, (3) the parent or surrogate’s decision to refuse intervention places the child at significant risk of serious harm in comparison with the recommendations of the healthcare team (applying the harm principle), and that (4) all attempts at mediation and negotiation to find a mutually acceptable solution have been exhausted (Box 17.1).20

Limits of Parental Demands There are also limits to a parent’s ability to demand medical therapies that are not recommended by the medical team. Clinicians need not always accede to parental requests. Healthcare professionals have an independent obligation to apply their knowledge and skills in a way that meets professional standards of care and benefits the patient. For example, clinicians can refuse parental requests for medical therapies for their children that • BOX 17.1

Conditions for Justified State Interference with Parental Decision-Making

• By refusing to consent, are the parents placing their child at significant risk of serious harm? • Is the harm imminent, requiring immediate action to prevent it? • Is the intervention that has been refused necessary to prevent the serious harm? • Is the intervention that has been refused of proven efficacy and therefore likely to prevent the harm? • Does the intervention that has been refused by the parents not also place the child at significant risk of serious harm, and do its projected benefits outweigh its projected burdens significantly more favorably than the option chosen by the parents? • Would any other option prevent serious harm to the child in a way that is less intrusive to parental autonomy and more acceptable to the parents? • Can the state intervention be generalized to all other similar situations? • Would most parents agree that the state intervention was reasonable? From Diekema DS. Parental refusals of medical treatment: the harm principle as threshold for state intervention. Theor Med Bioeth. 2004;25:243–264.



CHAPTER 17 Pediatric Critical Care Ethics

clearly are not medically indicated, such as antifungal medications for a bacterial pneumonia or an appendectomy for acute gastroenteritis. There are other circumstances, however, when parents and the medical team have value-based disagreements about medical therapies. A classic example is that of parents who demand ongoing mechanical ventilation for their child in a persistent vegetative state against the recommendation of the medical team. These cases involving requests or demands for potentially inappropriate or nonbeneficial treatments (previously known as “futility” cases) require special consideration primarily because it is difficult to prioritize conflicting values in our pluralistic society. To resolve these value-based disagreements, most major medical societies recommend the use of what is called a “fair process-based approach.” Once again, optimizing early intrateam and teamfamily communication, as well as attempts at early conflict resolution, may prevent many of these cases from reaching a crisis level.21 (For a complete discussion of this complicated topic, see Chapter 16.)

Other Ethical Issues in the Pediatric Intensive Care Unit Research Ethics In the quest for continual improvement of the delivery of critical care, children admitted to the PICU are often participants in research. Clinicians should be familiar with the basic ethical and regulatory aspects of clinical research in children.22,23 The doctrine of informed consent applies to most research. Child assent is generally required for research involving children unless an institutional review board (IRB) has waived the regulatory requirement for child assent because the child lacks the capacity for assent or the research offers a prospect of direct benefit to the child that is not available outside of the research. In healthy children, only research that poses no more than minimal risk to the child is acceptable. In children with disease, more than minimal risk may be acceptable only if there is a prospect of direct benefit to the individual child or if the research is likely to yield generalizable knowledge that is vital to understanding the child’s condition and the research exposes the child to no more than a minor increase over minimal risk. All research requires approval of an IRB. There are additional ethical considerations regarding research in the PICU. First, patients and families in the PICU are commonly under tremendous duress, are particularly vulnerable due to critical illness, and may have difficulty truly engaging in the informed consent process.24 Second, PICU clinicians and researchers are often one and the same; thus, families may feel conflicted about declining research and may worry about how their lack of research participation may influence the clinical care that their child receives. Researchers should be cautious to avoid undue influence or pressure on families when approaching them about enrollment in PICU research.

Resource Allocation Resource allocation is pertinent in the PICU in several ways. As PICUs are often functioning at or near capacity, the most basic resources (such as beds and nurses) may require thoughtful

151

allocation. Moreover, planning for disasters and other surges in the need for critical care requires careful consideration of how scarce resources will be allocated when multiple demands exceed those resources. Adding to the complexity, one of the greatest ethical dilemmas facing our current generation is the need to control resource consumption and allocate healthcare resources in a sustainable way on a daily basis. As technology advances at breathtaking speeds, there is more and more we can do to prolong life. What is our individual or collective role in controlling the use of technology and practicing stewardship of our resources? National experts have thus far been unable to come to consensus on an ethical approach to fairly allocate resources during disasters or how to consistently and fairly balance cost, quality of care, and quality of life in the critical care setting. Importantly, resource allocation policies should be addressed at the policy level, not at the bedside by individual clinicians. Individual institutions should have clear protocols about allocation of limited resources (e.g., PICU beds, ventilators, ECMO circuits, diversion policies) that may occur during times of surge. Using a predetermined set of standardized criteria can help to mitigate bias in individual triage decisions, thus avoiding potential injustices caused by variability. ICU clinicians should engage at the institutional, regional, and national level to affect public policy regarding the allocation of resources. Finally, any resource allocation strategy should be developed with broad social input and should be transparent25–28 (see also Chapter 9).

Ethical Issues at the End of Life Many ethical dilemmas arise at the end of life. These issues— which include withdrawing and withholding life support, analgesia, and sedation at the end of life, along with the doctrine of double effect, definitions of death, and organ donation—are covered in other chapters (see Chapters 18 to 20).

Limits to Clinician Refusals Generally, clinicians have no legal duty to provide illegal therapies, therapies for which the physician is not qualified or competent to provide, therapies that are considered “unnecessary” or “nonindicated” (such as antifungals for a bacterial pneumonia), or any therapy that is disallowed by institutional or legal decision. However, clinicians may not refuse to provide accepted medical care to patients on the basis of invidious discrimination (e.g., race, gender). There is ongoing debate, however, about the extent to which a clinician can refuse to provide accepted medical care based on personal moral beliefs, also known as conscientious objections.29,30 Generally, these types of refusals should be accommodated only if the refusal will not harm the patient and the refusal will not create undue burdens for colleagues or the institution. Providing a personal exemption under these circumstances typically requires that another clinician take over the care of the patient in a timely manner. Accommodation may not be possible in all cases; in such instances, the clinician would be required to provide the service or face institutional or legal consequences. Importantly, a conscientious objection is not sufficient justification for unilaterally forgoing life-sustaining therapies against the wishes of a patient or surrogates. A conscientious objection

152

SECTION III



Pediatric Critical Care: Psychosocial and Societal

should be used only to request a personal exemption from providing a service (a shield), not to impose the clinician’s moral values onto the patient (a sword). A fair process-based approach should be used to resolve such conflicts, as discussed previously.

Medical Training Many PICUs are located within pediatric teaching hospitals. Nursing students, medical students and residents, and other professional trainees may rotate through PICUs, learning valuable critical care medicine. There are also a fair number of PICUs nationally that have established pediatric critical care medicine fellowships. These training programs are essential to continuing to train the next generation of pediatric intensivists. The balance of benefits to the trainee (and, potentially, patient and family) must be weighed against the potential risks and burdens to the patient and family from receiving care from an inexperienced clinician, particularly in the setting of a critically ill child. To mitigate the risks and burdens to the child and family, other teaching methods (such as simulation) should be optimized, strict standards for supervision should be applied until competency is established, and the level of training should always be disclosed to the family31 (see also Chapter 10).

Use of Unproven Medical Therapies In the often dramatic and emotionally charged fight to save a child’s life, the medical team may consider using medical therapies that are new, have little to no evidence, or can be considered truly experimental outside of a conventional research setting. Typically, clinicians consider these options only when all other conventional options have failed and the outcome without them is near-certain death. The potential benefits to using unproven medical therapies include prolonging the child’s life, pushing the envelope for the development of new therapies and technologies over time, and adding to the fund of knowledge about the therapy. Because childhood critical illness is relatively rare, large randomized controlled trials may be unrealistic. On the other hand, although idealistically applied, use of these unproven therapies or technologies may burden patients, families, staff, and healthcare systems with little gain. An earnest attempt to consider the balance of benefit and burden of unproven therapies must be undertaken before they are offered to families. Clinicians should be aware of any conflict of interest or selfbenefit that they may receive from use of the therapy. The lack of evidence for the therapy should be carefully disclosed to the family.23 Finally, attempts should be made to enroll in ongoing trials of the therapy or, if this is not available, at the very least, details of the case should be carefully documented and published if possible. Finally, clinicians should be aware of when an innovative therapy, device, or procedure is regulated by the US Food and Drug Administration because additional requirements exist in these situations.

Global Health It is increasingly common for pediatric critical care clinicians to be intertwined with global health delivery in resource-limited settings in several ways: (1) clinicians may participate in medical missions or disaster relief efforts, (2) US fellowships may train clinicians from other countries, and (3) pediatric critical care

clinicians may engage in helping to develop PICUs or training programs in other countries. Certainly, much can be gained from sharing knowledge and experience and developing these relationships. As always, clinicians should be careful to not impose US cultural values and norms onto other cultures and societies. Importantly, ethical principles familiar to the US critical care clinician are not necessarily universal and should not be applied indiscriminately to other cultures. US clinicians may be emotionally challenged by practicing under different ethical norms (particularly when caring for critically ill children) and should be prepared to refrain from judging unfamiliar values. Systems developed to provide critical care in other countries should meet the needs of the people who use them and should be developed to be self-sustaining32,33 (see Chapter 8). Finally, ethical dilemmas may arise for patients and families from other countries who require critical care in the United States. It is important to engage in learning about the family’s values and cultural norms when addressing ethical issues in this setting. Incorporating these sometimes unfamiliar values and norms into an ethical analysis may be challenging, particularly when they do not coincide with Western notions of patient autonomy, patient-centered best interests, or the appropriateness of disclosure of medical information to the patient.

Medical Errors With rare exception, medical errors of all types should be disclosed to patients and families.34,35 Truthfulness with patients and surrogates should be considered part of the fiduciary duty of a clinician. Any determination that an error should not be disclosed to a patient or family should be reviewed by a third party or a larger committee. If a clinician witnesses another’s error, the clinician is similarly obligated to take steps to ensure that the error is disclosed.

Relationship Boundaries Clinicians in the PICU form relationships with patients and families at a stressful time in their lives. At times, intense, deep, caring relationships may develop; these can be meaningful and therapeutic for the clinician as well as the patient or family. The long-standing medical culture of training clinicians to maintain emotional distance from patients and families may not only be unrealistic in the emotionally charged PICU but also contribute to career dissatisfaction and burnout. Additionally, families consistently report desiring clinicians who genuinely care about them and their child. Deep, genuine, caring relationships between clinicians and patients and families may be beneficial.36 Care should be taken, however, to avoid nonbeneficial or harmful relationships. Personal attachments that cloud clinical judgment should be avoided. Romantic or overly intimate relationships should be avoided. Caution should be exercised when considering a relationship that extends beyond the walls of the PICU and boundaries of the medical relationship, including social media contact.37

Preventive Ethics Many ethical dilemmas have recurring themes and common triggers and may be predicted before the conflict reaches a state of



CHAPTER 17 Pediatric Critical Care Ethics

crisis. A proactive—as opposed to reactive—approach may address, deescalate, or even prevent crisis ethics situations from arising. This would clearly be of benefit to the patient, family, staff, and institution alike. One approach is to embed an ethicist in the PICU to make rounds and identify ethical issues to be addressed proactively. Another approach is to create PICU systems to address common triggers, such as assigning continuity physicians to chronic patients. However it is accomplished, it is in everyone’s interest to avoid intractable crisis ethical dilemmas.38

Goals for the Ethical Practice of the Intensivist It is essential for critical care clinicians to understand the basis for complex ethical decisions and actions so that intensivists’ patient care remains morally and ethically sound in the setting of high stakes, high pressures, competing needs, great uncertainty, and diverse perspectives and values. There are few absolutes in bioethics. By its nature, it is a continually shifting field to which intensivists must constantly adjust and to which intensivists must actively contribute. Despite changes and advances in this field, critical care clinicians can continually provide the best, most ethical care by understanding the history of ethical standards and current debate. Intensivists must continue to focus on compassionate and empathetic care, which will nearly always identify the right course of action. Critical care team members must value collaboration and constantly work to improve their communication and mediation skills. Critical care clinicians should open their hearts to deep relationships and be willing to explore the values of others and their own. Finally, intensivists should always be open to incorporating the expertise of others who can add to their practice. An intentional focus on these qualities will promote ethically sound care even in a rapidly changing and inherently uncertain environment (Box 17.2).

• BOX 17.2

153

Goals for Ethical Practice of the Intensivist

Focus on Compassionate and Empathetic Care • • • • •

Value collaboration with patients, families, and colleagues Hone relational techniques and communication and mediation skills Continually explore self-values and values of others Develop a basic approach to ethical dilemmas Understand the history and meaning of prevailing ethical norms applicable in the intensive care unit • Stay up to date on current ethical controversies applicable in the intensive care unit • Know when to involve experts in ethics, conflict resolution, and family and staff support

Key References American Academy of Pediatrics, Committee on Bioethics. Guidelines on foregoing life-sustaining medical treatment. Pediatrics. 1994;93: 532-536. Beauchamp TL, Childress JF. Principles of Biomedical Ethics. 5th ed. New York, NY: Oxford University Press; 2001. Bosslet GT, White DB, Au D, et al. An official ATS/AACN/ACCP/ESICM/SCCM policy statement: responding to requests for futile and potentially inappropriate treatments in intensive care units. Am J Respir Crit Care Med. 2015;191:1318-1330. Diekema DS. Parental refusals of medical treatment: the harm principle as threshold for state intervention. Theor Med Bioeth. 2004;25:243-264. Katz AL, Webb SA, Committee on Bioethics, American Academy of Pediatrics. Technical report: informed consent in decision-making in pediatric practice. Pediatrics. 2016;138(2):e20161485. Moon M, and the American Academy of Pediatrics Committee on Bioethics. Policy statement: institutional ethics committees. Pediatrics. 2019;143(5):e20190659. Morrison W, Clark JD, Lewis-Newby M, Konn AA. Titrating clinician directiveness in serious pediatric illness. Pediatrics 2018;142: S178-S186.

The full reference list for this chapter is available at ExpertConsult.com.

e1























































































1. American Medical Association Council on Ethical and Judicial Affairs. Code of Medical Ethics: Current Opinions with Annotations, 2012-2013. Chicago, IL: American Medical Association; 2013. 2. American Nurses Association. Code of Ethics for Nurses with Interpretive Statements. Silver Spring, MD: ANA; 2008. 3. ABSH. ASBH Core Competencies for Health Care Ethics Consultation. 2nd ed. Glenview, IL: American Society for Bioethics and Humanities; 2011. 4. Moon M, and the American Academy of Pediatrics Committee on Bioethics. Policy statement: institutional ethics committees. Pediatrics. 2019;143(5):e20190659. 5. Kaldjian L, Weir R, Duffy T. A clinician’s approach to clinical ethical reasoning. J Gen Intern Med. 2005;20:306-311. 6. Fox E, Berkowitz KA, Chanko BL, Powell T. Ethics Consultation: Responding to Ethics Questions in Health Care. www.ethics.va.gov/ IntegratedEthics. 7. A Case-Based Approach to Ethical Decision-Making. In: Jonsen AR, Siegler M, Winslade W. Clinical Ethics. 7th ed. New York: McGraw-Hill; 2010. 8. Pellegrino ED, Thomasma DC. For the Patient’s Good: The Restoration of Beneficence in Health Care. New York, NY: Oxford University Press; 1988. 9. Beauchamp TL, Childress JF. Principles of Biomedical Ethics. 5th ed. New York, NY: Oxford University Press; 2001. 10. Rushton CH. Defining and addressing moral distress: tools for critical care nursing leaders. AACN Adv Crit Care. 2006;17:161-168. 11. Berg JW, Appelbaum PS, Lidz CW, Parker LS. Informed Consent: Legal Theory and Clinical Practice. 2nd ed. Fair Lawn, NJ: Oxford University Press; 2001. 12. Field MJ, Cassel CK, eds. Institute of Medicine, Committee on Care at the End of Life: Approaching Death: Improving Care at the End of Life. Washington, DC: National Academies Press; 1997. 13. American Academy of Pediatrics, Committee on Bioethics. Guidelines on foregoing life-sustaining medical treatment. Pediatrics. 1994;93:532-536. 14. Katz AL, Webb SA, Committee on Bioethics, American Academy of Pediatrics. Technical report: informed consent in decision-making in pediatric practice. Pediatrics. 2016;138(2):e20161485. 15. Diekema DS. Adolescent refusals of life-saving treatment: are we asking the right questions? Adolesc Med State Art Rev. 2011;22: 213-228. 16. Kon AA, Davidson JE, Morrison W, et al. Shared decision making in ICUs: an American College of Critical Care Medicine and American Thoracic Society Policy Statement. Crit Care Med. 2016; 44(1):188-201. 17. Morrison W, Clark JD, Lewis-Newby M, Konn AA. Titrating clinician directiveness in serious pediatric illness. Pediatrics. 2018;142: S178-S186. 18. Antommaria AHM, Weise KL, and the American Academy of Pediatric Committee on Bioethics. Policy Statement: conflicts between religious or spiritual beliefs and pediatric care: informed refusal, exemptions, and public funding. Pediatrics. 2013;132:962-965. 19. Diekema DS. Religious objections to medical care. In: Jennings B, Eckenwiler L, Kaebnick G, et al, eds. Bioethics. 4th ed. Farmington Hills, MI: MacMillan Reference USA; 2014.

20. Diekema DS. Parental refusals of medical treatment: the harm principle as threshold for state intervention. Theor Med Bioeth. 2004;25: 243-264. 21. Bosslet GT, White DB, Au D, et al. An official ATS/AACN/ACCP/ ESICM/SCCM policy statement: responding to requests for futile and potentially inappropriate treatments in intensive care units. Am J Respir Crit Care Med. 2015;191:1318-1330. 22. Field MJ, Berman RE, eds. Institute of Medicine: Ethical Conduct of Clinical Research Involving Children. Washington, DC: National Academies Press; 2004. 23. Diekema DS. Conducting ethical research in pediatrics: a brief historical overview and review of pediatric regulations. J Pediatr. 2006;149:S3-S11. 24. Hulst JM, Peters JW, van den Bos A, et al. Illness severity and parental permission for clinical research in a pediatric ICU population. Intensive Care Med. 2005;31:880-884. 25. Persad G, Wertheimer A, Emanuel EJ. Principles for allocation of scarce medical interventions. Lancet. 2009;373:423-431. 26. Christian MD, Toltzis P, Kanter RK, et al. Treatment and triage recommendations for pediatric emergency mass critical care. Pediatr Crit Care Med. 2011;12:S109-S119. 27. Sinuff T, Kahnamoui K, Cook DJ, et al. Rationing critical care beds: a systematic review. Crit Care Med. 2004;32:1588-1597. 28. Truog RD, Brock DW, Cook DJ, et al. Rationing in the intensive care unit. Crit Care Med. 2006;34:958-963. 29. Lewis-Newby M, Wicclair M, Pope T, et al. Managing conscientious objections in intensive care medicine: an official policy statement of the American Thoracic Society. Am J Respir Crit Care Med. 2015; 191:219-227. 30. Committee on Bioethics. Policy statement—physician refusal to provide information or treatment on the basis of claims of conscience. Pediatrics. 2009;124:1689-1693. 31. Ziv A, Wolpe PR, Small SD, Glick S. Simulation-based medical education: an ethical imperative. Simul Healthc. 2006;1:252-256. 32. Hyder AA, Pratt B, Ali J, et al. The ethics of health systems research in low- and middle-income countries: a call to action. Glob Public Health. 2014;9:1008-1022. 33. Riviello ED, Letchford S, Achieng L, Newton MW. Critical care in resource-poor settings: lessons learned and future directions. Crit Care Med. 2001;39:860-867. 34. O’Connor E, Coates HM, Yardley IE, Wu AW. Disclosure of patient safety incidents: a comprehensive review. Int J Qual Health Care. 2010;22:371-379. 35. Boyle D, O’Connell D, Platt FW, Albert RK. Disclosing errors and adverse events in the intensive care unit. Crit Care Med. 2006; 34:1532-1537. 36. Remen RN. Practicing a medicine of the whole person: an opportunity for healing. Hematol Oncol Clin North Am. 2008;22:767-773. 37. Committee on Bioethics. Policy statement—pediatrician-familypatient relationships: managing the boundaries. Pediatrics. 2009; 124:1685-1688. 38. US Department of Veterans Affairs. National Center for Ethics in Health Care, Preventive Ethics: Addressing Ethics Quality Gaps on a Systems Level. 2nd ed. Washington, DC: VA; 2014.

References

e2

Abstract: Ethical questions arise every day in the rapidly changing and inherently uncertain environment of the pediatric intensive care unit. It is essential for critical care clinicians to understand the basis for complex ethical decisions and actions to ensure that patient care remains morally and ethically sound in the setting of high stakes, high pressures, competing needs, and diverse perspectives and values.

Key words: Bioethics, shared decision-making, doctrine of informed consent, surrogate decision-making, scarce resource allocation, communication, harm principle, mediation, adolescent decision-making

18 Ethical Issues Around Death and Dying MEREDITH G. VAN DER VELDEN AND JEFFREY P. BURNS



Many of the ethical issues that emerge in the care of the critically ill child do so at the end of life. Although many controversies still exist and new ethical dilemmas continually surface when facing death in the pediatric intensive care unit (PICU), significant progress toward a degree of consensus has been made over the years. This chapter provides an overview of ethical concerns that arise at the end of life in the PICU, including decision-making near the end of life, the ethics of withdrawal and withholding of life-sustaining treatments, issues around death determination, and issues that arise after death has been declared. Much of the discussion of the delivery of end-of-life care can be found in the chapter on palliative care (see Chapter 19).

Decision-Making at the End of Life There is little debate that decision-making authority for infants and children, particularly those in the PICU who are unable or too young to make decisions, rests with the parents.1 In light of the diversity of individual and family values and the complexity of the decisions being made, parents are justifiably provided wide discretion in these healthcare decisions for their children.2 However, there is similar consensus that physicians have an obligation to protect their patients in a way that may involve challenging the wishes of the parents on behalf of a child’s “best interests” when this rare situation arises.3 More than 30 years ago, the President’s Commission for the Study of Ethical Problems in Medicine and Biomedical Research3 produced the original guiding documents for most of these issues. In addition to addressing the determination of “best interests” when 154









Although decision-making at the end of life most commonly rests with a child’s parents, there may be times when a parent requests therapies that are deemed inappropriate by the clinical team. As truly futile treatment is difficult to define, deliberation over possibly inappropriate therapies should focus on intensive communication and negotiation, with hospital processes available for support and deliberation when this fails. Rationing decisions should not be made for an individual patient when evaluating the appropriateness of a treatment at the















PEARLS end of life. Rather, policies on rationing should be made at the institutional level. The majority of deaths that occur in the pediatric intensive care unit do so following a decision to withdraw or withhold life-sustaining treatments. There is no legal or moral distinction between withdrawing and withholding treatment. The doctrine of double effect supports the use of the titration of sedatives and analgesics to ensure comfort at the end of life.

approaching treatment options, the commission also discussed an approach to decision-making with parents. It concluded that although decision-making authority should rest with the parents under most circumstances, there may be times when it is appropriate for the physician to act against the parents’ wishes, specifically when parents choose to forgo clearly beneficial treatment and when parents prefer to provide futile treatment (Table 18.1). Although most physicians agree, and ethics and the law support, the opinion that delivering treatment that is “futile” is inappropriate, the concept of futility has been so controversial that it is rarely an effective support for a physician in overriding a parent’s wishes. Regardless, most major societies—including the American Academy of Pediatrics (AAP), Society of Critical Care Medicine (SCCM), and American Medical Association (AMA)— do support the physician in withholding futile treatments from patients when it can be so determined.1,4,5 The problem still rests in determining when care is “futile.” While much progress has arguably been made in our approach to cases of “futility” as practitioners in the intensive care unit (ICU), situations continue to arise in which parents wish to administer treatments and support that are deemed inappropriate and nonbeneficial by the providers. In these scenarios, how do practitioners balance the interests of the patient, family, society, and themselves? Is determining “futility” in these situations possible or even appropriate? Burns and Truog addressed the concept of futility and these questions by describing historical and generational accounts of the notion of futility. In doing so, they suggest a practical approach to resolving questions of medical futility.6 The first



CHAPTER 18 Ethical Issues Around Death and Dying

Provide treatment (during review process)

Ambiguous/uncertain

Provide treatment

Forgo treatment

Futile

Provide treatment

Forgo treatment

generation describes attempts at defining futility in order to resolve disputes with family members. These have included attempts to quantitatively (e.g., treatment has been useless in the past 100 cases), qualitatively (e.g., “treatment that merely preserves permanent unconsciousness”),7 and physiologically (e.g., treatment unable to achieve its physiologic goal) define the concept. All of these definitional approaches have been largely unsuccessful in actually resolving questions of disputed therapies between families and care providers as a result of flaws inherent in the definitions, along with failure to reach consensus on the definition in the background of a pluralistic society.8 Following recognition that attempts to define futility failed, the second generation described the subsequent period, which attempted to address disputed cases through procedures aimed at resolution. These have taken the form of individual hospital policies outlining processes to be followed in cases in which the appropriateness of a therapy was brought into question and attempts at consensus between the family and clinicians failed. In general, these policies aim to represent all parties involved and most often include an ethics consultation with possible courses of action if resolution of the dispute is not achieved by mere involvement of this third party. The possible actions include further attempts at resolution, transfer of care, or judicial involvement for the purpose of, or hospital endorsement of, unilateral action on behalf of the clinical team. Ultimately, the policies transfer the decision-making authority from the bedside clinicians and family to this third party. With these processes, the major concern that arises focuses on the neutrality of the committee on behalf of the disputed parties.9 A survey on attitudes and practices of pediatric critical care providers showed that despite most hospitals having these policies, providers do not make unilateral decisions to forgo treatment against the wishes of the family but rather provide the requested support until consensus is reached.10 While this reports practice rather than preference, another survey demonstrated that the majority of pediatric intensivists questioned are not in support of limiting therapies against the wishes of families,11 giving further support to the notion that even with policies in place, acting against the wishes of the family is not likely. The final generation focuses on enhancement of early communication and negotiation with families when anticipating and making decisions about the use of life-sustaining treatments. In concert, this involves clinicians supporting each other while they respect the wishes of the family and deliver care with which they may disagree. This final generation, while not the easiest or most straightforward, may represent the approach that is most aligned with the underpinnings of dedicated critical care—to provide support for patients and their families through illness and death.

Requests for Potentially Inappropriate Treatments in the Intensive Care Unit A joint statement by the American Thoracic Society, American Association for Critical Care Nurses, American College of Chest Physicians, European Society for Intensive Care Medicine, and SCCM recently addressed this issue and endorsed the following recommendations, among others6: 1. Institutions should implement strategies to prevent intractable treatment conflicts, including proactive communication and early involvement of expert consultants. 2. The term “potentially inappropriate” should be used, rather than futile, to describe treatments that have at least some chance of accomplishing the effect sought by the patient, but clinicians believe that competing ethical considerations justify not providing them. Clinicians should explain and advocate for the treatment plan they believe is appropriate. Conflicts regarding potentially inappropriate treatments that remain intractable despite intensive communication and negotiation should be managed by a fair process of conflict resolution; this process should include hospital review, attempts to find a willing provider at another institution, and opportunity for external review of decisions. When time pressures make it infeasible to complete all steps of the conflict resolution process and clinicians have a high degree of certainty that the requested treatment is outside accepted practice, they should seek procedural oversight to the extent allowed by the clinical situation and need not provide the requested treatment. 3. Use of the term “futile” should be restricted to the rare situations in which surrogates request interventions that simply cannot accomplish their intended physiologic goal. Clinicians should not provide futile interventions.15

Provide treatment



Clearly beneficial



Parents Prefer to Forgo Treatment



Parents Prefer to Accept Treatment



Physician’s Assessment of Treatment Options

A final issue relevant to the discussion of decision-making around the use of life-sustaining therapies in a critically ill patient is the rationing of medical care. While it can be tempting to consider cost control in these discussions, as it is a pressing consideration in healthcare today, it should be separated from the decision of the appropriateness of a medical treatment for an individual patient. The questions of cost and appropriateness, although both important, are fundamentally different, and the approaches to answering them should reflect this. Furthermore, ICU care is known to be costly, but the limitation of its use at the end of life is not certain to result in significant cost savings.12 Rationing at the bedside can be complicated.13 For this reason, the AAP has supported the separation of rationing decisions and bedside decision-making for any individual patient.14



TABLE Decision-Making in the Pediatric Intensive 18.1 Care Unit

155

Withholding and Withdrawing of Life-Sustaining Treatments While disagreements about the use of life-sustaining treatment do arise on occasion, it is far more common that a family and the medical team reach consensus about a decision to withdraw lifesustaining therapies. Furthermore, a majority of deaths in the PICU occur after the withdrawal of life-sustaining therapies.16,17 While there may be regional, national, and international variability in the number and percentage of patients who have a decision made to withdraw life-sustaining therapies,17 the ethics of the process of withdrawing life-sustaining treatments remain the same.

156

SECTION III



Pediatric Critical Care: Psychosocial and Societal

At this stage, the difference between withdrawing and withholding treatment may come into question. Although the actions of withdrawing and withholding a therapy may feel undeniably different to families and the care team, there is no true moral or legal distinction between the two.3 Any treatment not directed at comfort may be withdrawn or withheld if agreed upon by the family and clinical team, including, but not limited to, mechanical circulatory or ventilatory support, medications supporting the circulation, renal replacement therapy, antibiotic therapy, and hydration and nutrition. Although clinicians and families may make morally and legally defensible decisions to limit such treatments, it is never acceptable to limit care directed at providing comfort and emotional support for the patient and the patient’s family.18 Once the decision is made to focus on comfort rather than life-sustaining therapies, whether these are being withdrawn or withheld going forward, the aggressiveness of and attention to care cannot dissipate. The doctrine of double effect remains the guiding principle when considering therapies to be used at the end of life when the focus has transitioned from sustenance of life to comfort. While a contentious topic, related in part to concerns of oversimplification and misplaced focus on physician intent, it remains relevant and supportive when drawing the line between possibly unacceptable (e.g., euthanasia) and acceptable (e.g., aggressive palliative therapy) treatment courses.18 As alluded to, the doctrine relies on a distinction between what is intended and what is merely foreseen by the clinician. It states that for any action that has two effects, one good and one bad, it is justifiable if the following four conditions are met (Box 18.1): (1) the action itself must be morally good or neutral, (2) the good and not the bad effect must be intended, (3) the good effect must be a result of the action and not by means of the bad effect, and (4) the good effect must proportionally outweigh the bad effect.19 As clinicians address a number of therapies considered when treating pain and suffering at the end of life, this doctrine supports many, such as the use of analgesia and sedation. It falls short in justifying actions such as the administration of neuromuscular blockade to a dying patient, as well as other courses of treatment that might fall under the distinction of active euthanasia. Both the American College of Critical Care Medicine of the SCCM and the American Thoracic Society provide guidelines on the delivery of care at the end of life, with specific recommendations on management of symptoms.18,20 Among other topics, the guidelines provide a framework for consideration of withdrawing, withholding, or administering therapies. In general, whether deciding to continue or stop a therapy (e.g., antibiotics, mechanical ventilation) or considering the initiation of a new therapy for symptom management (e.g., analgesia), the decisions should center around the patient’s needs and should incorporate ongoing patient assessment and consideration of established guidelines.18

• BOX 18.1 Four Conditions of the Doctrine

of Double Effect 1. The action itself must be good or at least morally neutral. 2. The good effect and not the bad effect must be intended. 3. The good effect must be a result of the action itself and not by means of the bad effect. 4. The good effect must proportionally outweigh the bad effect.















Administration of Analgesics and Sedatives in End-of-Life Care Along with the avoidance of painful interventions, it is well accepted that pharmacologic interventions aimed at pain relief are indicated when withdrawing life-sustaining treatments. The goal for a patient who has had life-sustaining treatments withheld or withdrawn is to treat any signs of discomfort or pain. Consistent with the doctrine of double effect, the focus of analgesic medications—including opioids, with their known side effect of respiratory depression—should be directed at this perceived pain and not to directly causing death. This is best accomplished by careful patient assessment and use of objective scales when appropriate and available, as well as attention to the appropriate dosing titrated to effect, based on a patient’s prior exposure to the medications being considered. The medication dosing will vary patient to patient; any guidelines or procedures should reflect this anticipated variability. To reiterate, the goal of administering these therapies is only to treat patient discomfort and not to hasten the dying process or treat concerns of family members with medication for the patient. The latter concern can and should be dealt with in other ways. In addition to emotional and psychologic support for family members, anticipatory guidance on the process can help avoid the situation in which a family may perceive discomfort and request medications without objective signs from the patient.18 Most sedative agents lack analgesic properties, but their role in symptom management at the end of life is no less relevant. In addition to treating symptoms such as anxiety and agitation, they may also have a role in the treatment of refractory pain and suffering not relieved by analgesic medications. This latter indication works by lowering patient consciousness until symptoms are relieved. Sedation used in this manner has been labeled with terms such as palliative sedation, total sedation, and terminal sedation, the last of which has received much criticism.21 While critics of the practice claim a slide toward active euthanasia, the dominant view is that when intended and titrated to relieve symptoms, not to achieve unconsciousness or death, this does not represent euthanasia and is a key component in the delivery of quality end-of-life care.22 As with titration of analgesics, the doctrine of double effect remains the principle that makes intent the central issue and defends the process. In a survey of US physicians, there exists broad support for the acceptance of unconsciousness as a side effect of sedation to treat suffering and refractory pain.23 In its statement on palliative care in children, the AAP endorses the use of adequate analgesia and sedation to treat pain and other symptoms in patients with terminal conditions while explicitly not supporting the practice of physician-assisted suicide or euthanasia.24

Is There a Role for Neuromuscular Blockade in End-of-Life Care? Neuromuscular blocking agents have no analgesic or sedative properties. Furthermore, while the pharmacologic activity of these agents may give the appearance to others (i.e., family members) that a patient is without distress, in doing so they actually mask many of the objective signs of suffering that we use when titrating medications for such symptoms. Thus, there is no role for the initiation of neuromuscular blockade at the end of life.18,25 The more challenging question arises when determining what to do with existing neuromuscular blocking agents when the team



CHAPTER 18 Ethical Issues Around Death and Dying

and family have come to the decision to withdraw mechanical ventilation. With the principal goal being to adequately treat a patient’s discomfort at the end of life when life-sustaining treatments have been withdrawn, attempts should be made to restore neuromuscular function, including delay in withdrawal when reasonable, in order to regain access to many of the signs and symptoms used to titrate medications for pain and discomfort. However, there may be times when the restoration of function cannot be accomplished in a reasonable time period owing to factors such as altered drug metabolism and clearance resulting from organ dysfunction and length of treatment with such agents. In such cases, the benefit of delaying withdrawal of mechanical ventilation may be outweighed by the burden on the family that comes with this delay. In these rare cases, withdrawing mechanical ventilation without restoration of neuromuscular function is justifiable but must be balanced with extra attention to ensuring patient comfort in the absence of typical signs and symptoms of pain or anxiety.25 This is reflected in the SCCM recommendations for end-of-life care in the ICU.18

Artificial Hydration and Nutrition Any treatment that is not directed at comfort may be withdrawn or withheld at the end of life if the family so chooses and the care team is in agreement. This is no less true for the administration of artificial hydration and nutrition. Although withholding or withdrawing these therapies may be more psychologically distressing for caregivers and family, it is important to be clear that ethics and

157

the law do not distinguish between these and other life-sustaining treatments. Guidelines of the AAP and SCCM18,26,27 provide support for such decisions based on the same considerations of benefit and burden as with the withdrawal and withholding of other life-sustaining therapies.1 In recognition of the unique distress that withdrawing or withholding this therapy may present, the AAP strongly recommends the involvement of ethics committees when these decisions are being made.27

Key References Bosslet GT, Pope TM, Rubenfeld GD, et al. An official ATS/AACN/ ACCP/ESICM/SCCM policy statement: responding to requests for potentially inappropriate treatments in intensive care units. Am J Respir Crit Care Med. 2015;191:1318-1330. Burns JP, Truog RD. Futility: a concept in evolution. Chest. 2007;132:1987-1993. Morparia K, Dickerman M, Hoehn KS. Futility: unilateral decision making is not the default for pediatric intensivists. Pediatr Crit Care Med. 2012;13:e311-e315. Papavasiliou ES, Brearley SG, Seymour JE, et al. From sedation to continuous sedation until death: how has the conceptual basis of sedation in end-of-life care changed over time? J Pain Symptom Manage. 2013; 46:691-706. Weise KL, Okun AL, Carter BS, et al. Guidance on forgoing life-sustaining medical treatment. Pediatrics. 2017;140(3):e20171905.

The full reference list for this chapter is available at ExpertConsult.com.

e1

References



























20.





21.





22.





23.



24.









19.





25.





26.



27.



18.











17.





16.























1. Weise KL, Okun AL, Carter BS, et al. Guidance on forgoing lifesustaining medical treatment. Pediatrics. 2017;140(3):e20171905. 2. Katz AL, Macauley RC, Mercurio MR, et al. Informed consent in decision-making in pediatric practice. Pediatrics. 2016;138(2):e20161485. 3. President’s Commission for the Study of Ethical Problems in Medicine and Biomedical and Behavioral Research. Deciding to Forego Life-Sustaining Treatment: A Report on the Ethical, Medical, and Legal Issues in Treatment Decisions. Washington, DC: US Government Printing Office; 1983. 4. Consensus statement of the Society of Critical Care Medicine’s Ethics Committee regarding futile and other possibly inadvisable treatments. Crit Care Med. 1997;25:887-891. 5. American Medical Association. Opinion 5.5 Medically Ineffective Interventions. Code of Medical Ethics. Chicago: American Medical Association; 2016. https://www.ama-assn.org/delivering-care/ethics/ medically-ineffective-interventions. 6. Burns JP, Truog RD. Futility: a concept in evolution. Chest. 2007;132:1987-1993. 7. Schneiderman LJ, Jecker NS, Jonsen AR. Medical futility: its meaning and ethical implications. Ann Intern Med. 1990;112:949-954. 8. Truog RD, Brett AS, Frader J. The problem with futility. N Engl J Med. 1992;326:1560-1564. 9. Truog RD. Tackling medical futility in Texas. N Engl J Med. 2007; 357:1-3. 10. Burns JP, Mitchell C, Griffith JL, Truog RD. End-of-life care in the pediatric intensive care unit: attitudes and practices of pediatric critical care physicians and nurses. Crit Care Med. 2001;29:658-664. 11. Morparia K, Dickerman M, Hoehn KS. Futility: unilateral decision making is not the default for pediatric intensivists. Pediatr Crit Care Med. 2012;13:e311-e315. 12. Luce JM, Rubenfeld GD. Can health care costs be reduced by limiting intensive care at the end of life? Am J Respir Crit Care Med. 2002; 165:750-754. 13. Truog RD, Brock DW, Cook DJ, et al. Rationing in the intensive care unit. Crit Care Med. 2006;34:958-963; quiz 971. 14. Ethics and the care of critically ill infants and children. American Academy of Pediatrics Committee on Bioethics. Pediatrics. 1996; 98:149-152. 15. Bosslet GT, Pope TM, Rubenfeld GD, et al. An official ATS/AACN/ ACCP/ESICM/SCCM policy statement: responding to requests for

potentially inappropriate treatments in intensive care units. Am J Respir Crit Care Med. 2015;191:1318-1330. Burns JP, Sellers DE, Meyer EC, et al. Epidemiology of death in the PICU at five U.S. teaching hospitals. Crit Care Med. 2014;42: 2101-2108. Moore P, Kerridge I, Gillis J, et al. Withdrawal and limitation of life-sustaining treatments in a paediatric intensive care unit and review of the literature. J Paediatr Child Health. 2008;44:404-408. Truog RD, Campbell ML, Curtis JR, et al. Recommendations for end-of-life care in the intensive care unit: a consensus statement by the American College [corrected] of Critical Care Medicine. Crit Care Med. 2008;36:953-963. Beauchamp TL, Childress JF. Principles of Biomedical Ethics. 7th ed. New York, NY: Oxford University Press; 2012. Lanken PN, Terry PB, Delisser HM, et al. An official American Thoracic Society clinical policy statement: palliative care for patients with respiratory diseases and critical illnesses. Am J Respir Crit Care Med. 2008;177:912-927. Papavasiliou ES, Brearley SG, Seymour JE, et al. From sedation to continuous sedation until death: how has the conceptual basis of sedation in end-of-life care changed over time? J Pain Symptom Manage. 2013;46:691-706. ten Have H, Welie JV. Palliative sedation versus euthanasia: an ethical assessment. J Pain Symptom Manage. 2014;47:123-136. Putman MS, Yoon JD, Rasinski KA, Curlin FA. Intentional sedation to unconsciousness at the end of life: findings from a national physician survey. J Pain Symptom Manage. 2013;46:326-334. American Academy of Pediatrics. Committee on Bioethics and Committee on Hospital Care. Palliative care for children. Pediatrics. 2000;106:351-357. Truog RD, Burns JP, Mitchell C, et al. Pharmacologic paralysis and withdrawal of mechanical ventilation at the end of life. N Engl J Med. 2000;342:508-511. Truog RD, Cist AF, Brackett SE, et al. Recommendations for endof-life care in the intensive care unit: the Ethics Committee of the Society of Critical Care Medicine. Crit Care Med. 2001;29: 2332-2348. Diekema DS, Botkin JR, Committee on B. Clinical report—forgoing medically provided nutrition and hydration in children. Pediatrics. 2009;124:813-822.

e2

Abstract: Although decision-making at the end of life most commonly rests with a child’s parents, there may be times when a parent requests therapies that are deemed inappropriate by the clinical team. As truly futile treatment is difficult to define, deliberation over possibly inappropriate therapies should focus on intensive communication and negotiation, with hospital processes available for support and deliberation when this fails. Rationing decisions should not be made for an individual patient when evaluating the appropriateness of a treatment at the end of life. Rather, policies on rationing should be made at the institutional

level. The majority of deaths that occur in the pediatric intensive care unit do so following a decision to withdraw or withhold lifesustaining treatments. There is no legal or moral distinction between withdrawing and withholding treatment. The doctrine of double effect supports the use of the titration of sedatives and analgesics to ensure comfort at the end of life. Key words: Ethics, end-of-life decision-making, treatment, pediatric critical care, end-of-life care, double effect

19 Palliative Care in the Pediatric Intensive Care Unit ALISA VAN CLEAVE, EILEEN RHEE, AND WYNNE MORRISON

Caring for children with life-limiting illnesses is an important role for the pediatric intensivist. Over the past 2 decades the overall mortality rate of pediatric intensive care units (PICUs) in US teaching hospitals has decreased by half, due in part to medical and technologic advancements.1 These advancements have increased the longevity of children with diagnoses that were previously uniformly fatal and have resulted in a growing number of children who live with chronic, lifelimiting conditions, many of whom are technology dependent. Children with complex chronic illnesses represent an increasing proportion of hospitalized pediatric patients, many of whom require frequent care in the PICU.2 In this population of patients, the intensivist must evaluate the child’s illness trajectory, quality of life, symptom burden, and family preferences for care during each admission. To adequately care for children with chronic life-limiting conditions, as well as those who are near the end of life, pediatric intensivists must have a high degree of competency in core palliative care skills, including communication, shared decision-making, appropriate limitation of interventions, and pain and symptom management. This chapter explores the practice of palliative care in the PICU by intensivists, palliative care providers, and the interdisciplinary team.

Palliative Care Consults in the Pediatric Intensive Care Unit The compelling need for patient- and family-centered care (PFCC) and the broad range of pathophysiology that exists in ICUs demand a mixed model of integrative and consultative palliative care, 158









Pediatric intensivists must have a high level of competency in core palliative care skills, including communication, shared decision-making, appropriate limitation of interventions, pain and symptom management, and end-of-life care. Mastery of communication skills is a vital part of critical care training. When in doubt, talk less and listen more. When considering the limitations of interventions, clinicians should elicit a family’s values, goals, and hopes for their child and develop recommendations for care aimed toward achieving those goals.

















PEARLS Compassionate extubation is an important intensive care unit skill that requires meticulous planning and preparation for both family members and staff. When the goals of care shift toward comfort, pain and symptom management must be prioritized, using both pharmacologic and nonpharmacologic interventions. Indications for consultation by a specialty palliative care team include complex decision-making and communication support, advanced symptom management, need for enhanced family support, or transition to hospice.

which allows for a wider distribution of a limited subspecialty resource.3,4 Primary palliative care is an integrative model that focuses on maximizing and standardizing palliative care practices that clinicians routinely incorporate into the care of their patients.5,6 Secondary palliative care uses consultation of a palliative care team for complex, subspecialty-level problems. Secondary palliative care helps ensure that there is adequate assessment and management of symptoms—as well as attention to emotional and psychological distress, practical and financial concerns, and spiritual and cultural needs—as part of comprehensive PFCC.3 Indications for specialty palliative care consultation include complex decision-making and communication support, symptom management, optimization of quality of life, hospice transition, and end-of-life care that extends beyond usual care practices. Parents of critically ill children often face difficult, value-laden decisions amid bewildering amounts of information and an “irreducible amount of uncertainty.”7 Communication expertise around eliciting patient and family preferences and translating those preferences into decision-making are part of the core set of skills for intensivists. Palliative care specialists can provide additional support and guidance in particularly complex situations to help elucidate goals of care for patients and families.8

Communication The importance of communication in the intensive care setting has become abundantly clear over the past 2 decades. Healthcare providers communicate with families in many ways in the ICU,



CHAPTER 19 Palliative Care in the Pediatric Intensive Care Unit

but the hallmark of ICU communication is the family meeting. In a typical family meeting, members of the medical team sit down with the patient’s family away from the bedside to formally discuss the patient’s care. Commonly, these meetings are organized by the medical team to facilitate difficult conversations, including the delivery of bad news, discussions regarding goals of care, and end-of-life care preferences.9–12

Suboptimal Communication in the Intensive Care Unit Excellent bidirectional communication between providers and families is an essential part of providing comprehensive, familycentered care in the ICU.13–15 In fact, some families deem the quality of a physician’s communication skills more important than their clinical skills.16 Failure to provide adequate communication puts patients and families at risk for poor outcomes, including anxiety, depression, and posttraumatic stress disorder.17 Despite its importance, studies continue to reveal that communication between families and providers in the ICU is suboptimal.18–22 In a recent study,23 researchers interviewed parents of children who died in the PICU regarding the communication that occurred around their child’s end-of-life care. More than 70% of parents gave constructive feedback to physicians regarding the way information was conveyed during their child’s terminal illness. The most common issue raised in this study was physician availability and attentiveness to the families’ informational needs. Other concerns included honesty, withholding of information, use of complex language, pacing of information delivery, providing false hope, body language, and affect during bad news delivery.

Families with Limited English Proficiency Families with limited English proficiency (LEP) are at even greater risk of receiving poor communication from healthcare providers despite using trained medical interpreters. Patients with LEP are less satisfied with physician communication than English-speaking patients, including the degree to which physicians listen, answer questions, explain concepts, and provide support.24–28 A recent study compared the quality of communication during interpreted and noninterpreted PICU family meetings.29 Interpreted meetings had fewer elements of shared decision-making and a greater imbalance between physician and family speech. In fact, LEP families spoke for less than 4 minutes during meetings that lasted 43 minutes on average. Though this finding may be cultural to some degree, it is difficult to argue that effective bidirectional communication can occur with so little family participation, especially when discussing such complex issues as the care of a critically ill child.

Family Meeting as an Intensive Care Unit “Procedure” Complex value-laden decisions, such as the decision to withhold or withdraw life-sustaining therapies, are typically made during ICU family meetings. For that reason, conducting a family meeting with clear, compassionate bidirectional communication is a critical ICU “procedure” that must be effectively taught to all trainees.30 Importantly, because ICU family meetings are frequently organized when the medical team wants to discuss limiting or withdrawing lifesustaining measures, it is possible to foster a hidden agenda implying that, when family meetings are conducted well, families choose to limit or withdraw interventions. In actuality, a successful family meeting is one in which the medical team elicits the patient’s and

159

family’s goals and values, and a care plan is crafted that achieves their goals and honors their values.

Communication Pearls Palliative care specialists receive extensive training on conducting difficult conversations; they can be consulted as a resource for both the medical team and family. However, effective communication of difficult and complex information is a core competency for all intensivists, and these skills must be learned early in training and honed throughout one’s career. A number of strategies exist to assist clinicians in conducting these conversations,31 one of which is the SPIKES protocol,32 summarized in Box 19.1.

Phrases to Avoid Several antiquated phrases remain in the vernacular of healthcare providers that must be eliminated, such as “withdrawal of care,” “nothing more can be done,” and “there is no hope.” Care is never withdrawn from a patient in the ICU. There is always more that can be done to help ensure comfort, provide support, maintain dignity, and create meaning. Allowing families to maintain hope is important.33 Providers may worry that preserving hope and truth-telling are mutually exclusive. However, research suggests the opposite: truthful disclosure of prognostic information, even in the setting of poor prognosis, is associated with increased parental hope.34 This may be because hope is not solely defined by a particular medical outcome. Truthful prognostic information, when delivered compassionately, can allow parents to focus on achievable hopes, such as comfort, quality of life, and meaningful relationships. Eliminating these phrases will help ensure that families do not feel abandoned by the medical team.

• BOX 19.1 SPIKES: A Protocol for Delivering Bad

News Setup: Find a private space with adequate seating. Never conduct a conversation of this nature without sitting down. Patient perspective: Begin by allowing the patient or family to share their understanding, concerns, and goals for the meeting. Always listen before you talk. Invitation: Obtain the family’s permission to give them the information you want to share. This can also be an opportunity for a “warning shot,” or a phrase that prepares the family for difficult news. As an example, one could say, “Unfortunately, I have some difficult information to share with you. Is it all right if I talk about that during this meeting?” Knowledge: Deliver the information clearly and compassionately. Go slowly, and allow for silence. Families may only hear the first piece of bad news delivered before their emotional response prevents further comprehension. Resist the temptation to continue delivering information if the family is having an emotional response. Instead, acknowledge, validate, and explore their emotions. Emotions: Though some providers may feel uncomfortable addressing emotions, families consistently report the importance of empathy from healthcare providers. The presence or absence of empathy can leave indelible marks on family members for years to come. If unsure how to respond, listening with empathic statements is always appropriate. Summary: Provide a brief summary of the meeting, ensuring that you have addressed the goals and concerns laid out by the family at the start of the meeting. State the next steps, and plan for future conversations. Modified from Baile WF, Buckman R, Lenzi R, et al. SPIKES—A six-step protocol for delivering bad news. Oncologist. 2000;5(4):302–311.

160

SECTION III



Pediatric Critical Care: Psychosocial and Societal

Though protocols and guidelines can be helpful, all providers will encounter situations in which it is difficult to know what to say or how to proceed. When this occurs, we are reminded by Elaine Meyer of the importance of “being present, not perfect.”35 Connecting with patients and families on a human level by bearing witness to their suffering will facilitate continued collaboration amid even the most difficult circumstances.

Limitation of Interventions One of the most difficult options for a family and the medical team to consider when a child may be dying is whether advanced technologic supports offer any benefit. Such supports can include invasive or noninvasive mechanical ventilation, medical or mechanical support of the circulation, surgical interventions, renal replacement therapy, intravenous (IV) medications, or medically administered nutrition and hydration. Forgoing such interventions requires that the family and medical team agree that such therapies offer little chance of benefit, that the pain or suffering they cause is not worth the hoped-for benefit, that the therapies no longer provide a reasonable quality of life, or that they otherwise do not help to achieve important goals for the child and family.

Do Not Attempt Resuscitation Orders An important tool when determining desired goals is the do not attempt resuscitation (DNAR) order. Such orders historically became necessary when medical care advanced to such a degree that many “intensive care” interventions, such as mechanical ventilation or cardiopulmonary resuscitation (CPR), became the default pathway to prolong life in most circumstances.36,37 The term DNAR is beginning to replace DNR (do not resuscitate), because DNAR does not presuppose that resuscitation attempts will be successful. Some centers have shifted terminology for DNAR orders even further by calling them allow natural death (AND) orders.38,39 The general public may have an inflated perception of the success of CPR based somewhat on media depictions.40 A physician’s willingness to share one’s medical opinion regarding the likelihood of success of CPR, especially if that likelihood is low, may be helpful in a family’s decision-making. Using phrases that focus on what will be provided (e.g., comfort) rather than on what will be withheld (e.g., resuscitation) may help families understand the reasoning behind such choices. How choices are presented, or “framed,” may affect patient and family decisions.41–43 In discussions with the patient and family, it is imperative that the clinician elicit the family’s overall goals rather than presenting a list of all possible interventions and asking for a yes/no answer to each. Once the family members have articulated their goals (e.g., “going home,” “avoiding painful procedures,” or “waiting to see if our child can get back to baseline”), then the clinician can determine what interventions might help achieve those goals. DNAR orders are not “all or none,” and a range of interventions could make sense depending on the clinical circumstances. Clarity in the orders is important, however, especially if multiple transitions in care providers may occur. During these discussions, it is important to avoid phrases such as “Do you want us to do everything for your child?” Such phrases are nonspecific and imply that “doing everything” is the right course of action,44 because the converse is to “do nothing.” In circumstances in which a clinician feels strongly that invasive technologic support will not lead to long-term benefit, it is acceptable to

recommend against CPR or intubation as a way to protect the child from interventions that will not help.45 Making a recommendation is an important part of shared decision-making; recommendations should be based on the goals and values articulated by the patient and family.46 Although the ethical justification for withholding a therapy is exactly the same as for withdrawing a therapy, it may be psychologically more difficult for some families to stop interventions that are already in place than it is to forgo pursuing new ones. Therefore limitations of interventions may often begin as “nonescalation” plans, with consideration of withdrawal of an intervention when the clinical trajectory becomes clear.

Hospice Support in the Home During discussions around goals of care, some families may share that allowing their child to die at home would be a great source of comfort or meaning. In certain clinical scenarios, this option may be feasible and appropriate to offer. Hospice agencies are invaluable partners specifically skilled to coordinate and facilitate such a care plan. Hospices provide comfort-oriented medical care and psychosocial support to patients with life-limiting illnesses and their families. US hospices are independent agencies structured to be compliant with Medicare guidelines.43,44 Importantly, hospices do not provide shift-based home nursing care for patients. Hospice nurses visit patients on a regular basis (from twice weekly to monthly, depending on needs) to help caregivers assess symptoms and manage changes in clinical status. They are also on call 24 hours a day for support by phone and, in special circumstances, can provide continuous in-home care for up to 72 hours for patients who are actively dying. Although hospital-based palliative care teams and community-based hospices are distinct, pediatric palliative care teams work closely with hospices to help ensure a seamless transition from hospital to home.45,47

Compassionate Extubation Discontinuing mechanical ventilation (now called compassionate extubation) is an important skill for all intensivists. It requires meticulous planning and symptom management. Preparing the family is an important first step. This includes determining who should be present, asking whether the family member to be present would like to hold the child during extubation, and distinguishing between expected signs that are part of the dying process (e.g., color change, noisy breathing) versus signs that would be treated with additional medication. Providers should avoid overly precise predictions of how quickly a patient will die following compassionate extubation, as some patients may breathe longer on their own than anticipated. Providing a range of time, such as “minutes to hours” or “days to weeks,” gives families a general idea of expected time course. Preparing medications ahead of time to treat anticipated symptoms is important; having a titration plan in place may help staff assess and respond to distress.48 It is sometimes helpful to decrease ventilatory support shortly before extubation to assess whether the patient develops dyspnea. If the patient appears uncomfortable on lower ventilatory settings, additional doses of medication can be given before extubation. Although medication for dyspnea and agitation is essential, neuromuscular blockade should not be administered if a ventilator is being withdrawn. Neuromuscular blockade may hasten death; it also makes it difficult to assess distress and determine whether additional medications for comfort are needed.49,50 It is possible to discontinue other interventions—such as vasoactive infusions, extracorporeal



CHAPTER 19 Palliative Care in the Pediatric Intensive Care Unit

circulatory support, or supplementary oxygen—while awaiting resolution of neuromuscular blockade before extubation.

Pain and Symptom Management When the goals of care shift to comfort, it is important to pay close attention to medication management and symptom control. Common symptoms at the end of life include pain, dyspnea, anxiety, and agitation. Many medications that treat these symptoms are part of routine ICU care (eTable 19.1; see also Chapter  132). Medication choices may differ significantly depending on how long a child is expected to live following removal of ICU interventions, what sources of pain exist, or how neurologically intact the patient is, although there is likely large variability between different centers.51 eTable 19.1 includes typical starting doses, but doses will need to be at a significantly higher level if a patient has developed tolerance. Medications should be titrated to effect, with the maximum dose dictated only by side effects. For opioids and benzodiazepines, bolus doses and infusion rates may be repeatedly increased by 20% to 50% until symptoms are controlled, which typically occurs before respiratory depression. In patients without IV access, other routes of medication administration (transdermal, sublingual, rectal, subcutaneous) can be considered rather than increasing discomfort by necessitating needle sticks and procedures to maintain venous access.

Medication Management Opioids Several commonly used opioids are listed in eTable 19.1. Opioids treat both pain and dyspnea and may also have sedating effects. They work via central nervous system µ-receptors. Potential side effects include constipation, nausea, pruritus, urinary retention, and respiratory depression.52,53 Side effects should be anticipated and prevented if possible (e.g., with a bowel regimen). Intractable side effects can sometimes be managed by rotating to another agent in the class.54 Some side effects, such as nausea and vomiting, may resolve over time. Distinguishing features of specific opioids are important to mention. Codeine should be avoided because approximately 10% of the general population lacks the hepatic enzyme necessary to convert it to morphine, and up to 35% of children demonstrate inadequate conversion to morphine.55 Meperidine should also be avoided because its metabolite, normeperidine, can accumulate and cause seizures. Fentanyl is commonly used in ICUs because of its rapid onset and titratability, but it can be problematic at the end of life if used for longer than brief periods because tolerance can develop rapidly. However, the transdermal (patch) form is often useful for patients who are unable to tolerate enteral medications and no longer have IV access. Morphine leads to histamine release, which can cause pruritus and hypotension, which may improve with rotation to hydromorphone. At very high doses, morphine has neuroexcitatory effects that cause hyperalgesia, delirium, and myoclonus. Morphine should also be avoided in renal failure, as accumulation of its metabolites causes myoclonus. Methadone Methadone differs from other opioids and therefore bears special mention. It is a µ-receptor agonist, as well as an N-methyl-d-aspartate (NMDA) receptor antagonist. It has a long and highly variable halflife. Its NMDA effects can sometimes improve pain control in patients who have become tolerant to high doses of other opioids, and

161

it may also be effective in treating neuropathic pain. Its long half-life can lead to drug accumulation, which may cause late-onset side effects, such as obtundation. Careful adjustment of dosing schedules is required. Methadone has many drug–drug interactions that require careful review. It can also prolong the QT interval; thus it is prudent to screen patients with an electrocardiogram before its initiation.

Other Pharmacologic Agents Acetaminophen or nonsteroidal antiinflammatory drugs (e.g., ibuprofen, naproxen, ketorolac) may be useful adjuncts to the medications shown in eTable 19.1. However, doses of these medications cannot be escalated because of the risk of toxicity. Combination agents, such as acetaminophen with oxycodone, should be avoided because acetaminophen limits the ability to escalate the opioid component. Ketamine is a dissociative anesthetic that also has NMDA effects. It offers excellent pain control and may have opioid-sparing benefits. However, it can cause disturbing hallucinations and delirium. Similar to methadone, it may have advantages for neuropathic pain. Other important adjunctive pain control methods include regional anesthesia (nerve blocks, epidural, or spinal anesthesia), lidocaine patches, and occasional treatment with steroids.

Symptom Management Pain A multitude of agents are available for the treatment of pain, many of which are discussed in other chapters (see Chapter 132). Neuropathic pain may be treated by methadone, gabapentin, or amitriptyline. Steroids and IV bisphosphonates are useful adjuncts for pain relief due to malignant bone pain. Nonpharmacologic adjuncts to pain control may also be useful. Research suggests that integrative therapies, such as art and music therapy, can be effective adjuncts for pain treatment in children with cancer.55

Dyspnea Opioids are the mainstay of treatment for dyspnea. Nebulized opioids are sometimes used, although they have not shown consistent benefit in controlled trials.47,57 In addition to opioids, dyspnea may be improved with a fan blowing in the patient’s face. Other respiratory support, such as supplementary oxygen and noninvasive positive pressure, can be considered if they enhance comfort, but there is no mandate to use them if they add to distress or prohibit patient disposition to another location (e.g., home or outside of the ICU) that would be preferable to the family. Agitation and Anxiety Benzodiazepines are often useful to treat agitation or anxiety at the end of life and may also help decrease opioid requirement.58 Low doses may be sufficient, but some patients may require escalation to sedating doses. A calm, quiet environment can be helpful but so can distracting or enjoyable activities.

Nausea and Vomiting Several agents are available for the treatment of nausea and vomiting. Ondansetron or metoclopramide are often effective. Benzodiazepines are also useful. Phenothiazines, such as promethazine and prochlorperazine, are efficacious but can be very sedating. They may also cause extrapyramidal side effects, which diphenhydramine can help mitigate. Olanzapine or haloperidol can be used when other agents are ineffective.59

161.e1

eTABLE Medications Commonly Used for Pain and Distress at the End of Lifea 19.1

Medication

Routes

Starting Dose

Notes

Morphine

PO, SL, PR, SQ, IV

0.05–0.1 mg/kg every 3–4 h Infusion: 0.01–0.03 mg/kg/h

Renally excreted; causes histamine release

Hydromorphone

PO, IV, SQ, SL

0.015 mg/kg every 3–4 h Infusion: 0.003 mg/kg/h

Fentanyl

IV, SQ, buccal, nasal, patch

0.5-1 mcg/kg every 30 min Infusion: 1 µg/kg/h

Transdermal patches available in 12.5, 25, 50, 75 and 100 µg/h

Methadone

PO, IV

0.05–0.1 mg/kg every 6–12 h initially, then decrease frequency

Long acting and may accumulate; may be adjunctive for neuropathic pain via NMDA effects; prolongs QT interval; multiple drug interactions

Midazolam

PO, IV, SC

0.05–0.1 mg/kg every 2–4 h Infusion: 0.03–0.1 mg/kg/h

Onset of action within minutes when given IV

Lorazepam

PO, IV, IM

0.025–0.1 mg/kg every 4–8 h

Less hypotension than midazolam, slightly slower onset

Diazepam

PO, PR

0.05–0.2 mg/kg every 6-12 h

IM and IV formulations available but rarely used due to pain/phlebitis; IV form may also be given PO or PR

Opioids

Benzodiazepines

Other Sedatives and Adjuncts Ketamine

PO, IM, IV

0.2–0.5 mg/kg/dose

May be adjunctive for neuropathic pain via NMDA effects

Gabapentin

PO

10 mg/kg/day

For neuropathic pain; increase daily until 30 mg/kg/ day, then reassess

Amitriptyline

PO

5 mg

For neuropathic pain; target dose 0.5–1 mg/kg/day

a All doses are starting doses for patients not previously exposed and may need to be escalated to much higher levels. IM, Intramuscular; IV, intravenous; NMDA, N-methyl-D-aspartate; PO, oral; PR, per rectum; SL, sublingual; SQ, subcutaneous.

162

SECTION III



Pediatric Critical Care: Psychosocial and Societal

Seizures Seizures can also occur at the end of life, for which benzodiazepines are a good first-line agent. Levetiracetam or valproate are sometimes used as prophylaxis against seizures in patients at high risk (e.g., with brain tumors).58 Bowel Obstruction Bowel obstruction is a particularly difficult situation to manage. Decompression with nasogastric drainage may improve symptoms. Relieving constipation is often important. Steroids may be beneficial if the obstruction is due to a mass. Motility agents can be helpful, but they may also increase pain. Octreotide (intravenous or subcutaneous) has been used to decrease intestinal secretions and may improve symptoms such as vomiting.60 Palliative surgery can be considered, but the degree and duration of benefits versus burdens should be carefully weighed.59 Palliative Sedation Rarely, symptoms may remain uncontrolled at the end of life despite maximal medical management. In such circumstances, palliative sedation may be considered. Palliative sedation is “the use of sedative medications to relieve intolerable suffering from refractory symptoms by a reduction in patient consciousness.”61,62 Benzodiazepines, barbiturates, dexmedetomidine, or propofol can be used. Additionally, propofol has advantageous effects against nausea, pruritus, seizures, and myoclonus, while dexmedetomidine is useful because it does not cause respiratory depression. Sedation to unconsciousness can be justified when symptoms cannot be managed by other means and death is considered imminent (e.g., within hours to days). Protocols have been published that guide the implementation of palliative sedation, which include prerequisite consensus by an interdisciplinary team that symptoms are truly refractory and that the patient is imminently dying of a terminal illness.63

Care of Family and Staff after a Child’s Death The death of a child is a tragic event that affects all who are touched by it. Grief support is a crucial part of the ongoing care that bereaved families need after their child dies; these services are typically provided by referral to a community- or hospitalbased bereavement program. Such programs provide ongoing support and frequent assessments to identify complicated grief when it occurs. Bereaved families of chronically ill children often describe a sense of “double loss,” both for their child and for their medical team who cared for them over the course of months or years.64 Staff members in the ICU are also impacted by the death of a child and are at risk for compassion fatigue and burnout due to repeated exposure to secondary trauma.65 The American College

of Critical Care Medicine Task Force guidelines for support of the patient and family in the ICU setting recommend structured support mechanisms for staff, such as debriefing sessions,66 which can be facilitated by social work, spiritual care, or palliative care. For some individuals, these sessions provide a safe forum to discuss their feelings about a particular patient or experience, which can help process grief. Others may benefit from developing personalized ways to process stress or grief, which can include any number of activities, such as exercise, reflective writing, outdoor activities, engaging in a spiritual practice, or meeting with a counselor or therapist regularly. In addition, some staff members choose to send condolence letters or attend memorial services for children as a way to further support the family, honor the memory of the child, and process their own grief.

Key References American Academy of Pediatrics Committee on Bioethics. Guidelines on foregoing life-sustaining medical treatment. Pediatrics. 1994;93:532-536. Boss R, Nelson J, Weissman D, et al. Integrating palliative care into the PICU: a report from the improving palliative care in the ICU Advisory Board. Pediatr Crit Care Med. 2014;15:762-767. Clark JD, Dudzinski DM. The culture of dysthanasia: attempting CPR in terminally ill children. Pediatrics. 2013;131:572-580. Dahlin CM, ed. The National Consensus Project for Quality Palliative Care: Clinical Practice Guidelines for Quality Palliative Care. 3rd ed. Pittsburgh, PA: The National Consensus Project for Quality Palliative Care; 2013. Feudtner C, Morrison W. The darkening veil of “do everything.” Arch Pediatr Adolesc Med. 2012;166:694-695. Guerrero AD, Chen J, Inkelas M, et al. Racial and ethnic disparities in pediatric experiences of family-centered care. Med Care. 2010;48:388393. Hurd CJ, Curtis JR. The intensive care unit family conference. Teaching a critical intensive care unit procedure. Ann Am Thorac Soc. 2015; 12:469-471. Kon AA. The shared decision-making continuum. JAMA. 2010;304:903904. Meyer EC, Ritholz MD, Burns JP, et al. Improving the quality of end-oflife care in the pediatric intensive care unit: parents’ priorities and recommendations. Pediatrics. 2006;117:649-657. Munson D. Withdrawal of mechanical ventilation in pediatric and neonatal intensive care units. Pediatr Clin North Am. 2007;54:773-785. Truog RD, Cist AF, Brackett SE, et al. Recommendations for end-of-life care in the intensive care unit: the Ethics Committee of the Society of Critical Care Medicine. Crit Care Med. 2001;29:2332-2348. Van Cleave AC, Roosen-Runge MU, Miller AB, et al. Quality of communication in interpreted versus noninterpreted PICU family meetings. Crit Care Med. 2014;42:1507-1517.

The full reference list for this chapter is available at ExpertConsult.com.

e1











































































































1. Burns JP, Sellers DE, Meyer EC, et al. Epidemiology of death in the PICU at five US teaching hospitals. Crit Care Med. 2014;42:2101-2108. 2. Davies D, Hartfield D, Wren T. Children who “grow up” in hospital: inpatient stays of six months or longer. Paediatr Child Health. 2014; 19:533-536. 3. von Gunten CF. Secondary and tertiary palliative care in US hospitals. JAMA. 2002;287:875-881. 4. Feudtner C, Kang TI, Hexem KR, et al. Pediatric palliative care patients: a prospective multicenter cohort study. Pediatrics. 2011; 127:1094-1101. 5. Nelson JE, Bassett R, Boss RD, et al. Models for structuring a clinical initiative to enhance palliative care in the intensive care unit: a report from the IPAL-ICU Project (Improving Palliative Care in the ICU). Crit Care Med. 2010;38:1765-1772. 6. Boss R, Nelson J, Weissman D, et al. Integrating palliative care into the PICU: a report from the improving palliative care in the ICU Advisory Board. Pediatr Crit Care Med. 2014;15:762-767. 7. Renjilian CB, Womer JW, Carroll KW, et al. Parental explicit heuristics in decision-making for children with life-threatening illnesses. Pediatrics. 2013;131:e566-e572. 8. Dahlin CM, ed. The National Consensus Project for Quality Palliative Care: Clinical Practice Guidelines for Quality Palliative Care. 3rd ed. Pittsburgh, PA: The National Consensus Project for Quality Palliative Care; 2013. 9. Curtis JR, White DB. Practical guidance for evidence-based ICU family conferences. Chest. 2008;134:835-843. 10. Cypress BS. Family conference in the intensive care unit: a systematic review. Dimens Crit Care Nurs. 2011;30:246-255. 11. Meyer EC, Ritholz MD, Burns JP, et al. Improving the quality of end-of-life care in the pediatric intensive care unit: parents’ priorities and recommendations. Pediatrics. 2006;117:649-657. 12. Hickey M. What are the needs of families of critically ill patients? A review of the literature since 1976. Heart Lung. 1990;19:401-415. 13. Azoulay E, Pochard F, Kentish-Barnes N, et al. Risk of posttraumatic stress symptoms in family members of intensive care unit patients. Am J Respir Crit Care Med. 2005;171:987-994. 14. Abbott KH, Sago JG, Breen CM, et al. Families looking back: one year after discussion of withdrawal or withholding of life-sustaining support. Crit Care Med. 2001;29:197-201. 15. Azoulay E, Chevret S, Leleu G, et al. Half the families of intensive care unit patients experience inadequate communication with physicians. Crit Care Med. 2000;28:3044-3049. 16. Hanson LC, Danis M, Garrett J. What is wrong with end-of-life care? Opinions of bereaved family members. J Am Geriatr Soc. 1997;45:1339-1344. 17. Stricker KH, Kimberger O, Schmidlin K, et al. Family satisfaction in the intensive care unit: what makes the difference? Intensive Care Med. 2009;35:2051-2059. 18. Curtis JR, Engelberg RA, Wenrich MD, et al. Missed opportunities during family conferences about end-of-life care in the intensive care unit. Am J Respir Crit Care Med. 2005;171:844-849. 19. Meert KL, Eggly S, Pollack M, et al. Parents’ perspectives on physician-parent communication near the time of a child’s death in the pediatric intensive care unit. Pediatr Crit Care Med. 2008;9:2-7. 20. Baker DW, Hayes R, Fortier JP. Interpreter use and satisfaction with interpersonal aspects of care for Spanish-speaking patients. Med Care. 1998;36:1461-1470. 21. Carrasquillo O, Orav EJ, Brennan TA, et al. Impact of language barriers on patient satisfaction in an emergency department. J Gen Intern Med. 1999;14:82-87. 22. Guerrero AD, Chen J, Inkelas M, et al. Racial and ethnic disparities in pediatric experiences of family-centered care. Med Care. 2010; 48:388-393. 23. Morales LS, Cunningham WE, Brown JA, et al. Are Latinos less satisfied with communication by health care providers? J Gen Intern Med. 1999;14:409-417.

24. Mosen DM, Carlson MJ, Morales LS, Hanes PP. Satisfaction with provider communication among Spanish-speaking Medicaid enrollees. Ambul Pediatr. 2004;4:500-504. 25. Van Cleave AC, Roosen-Runge MU, Miller AB, et al. Quality of communication in interpreted versus noninterpreted PICU family meetings. Crit Care Med. 2014;42:1507-1517. 26. Hurd CJ, Curtis JR. The intensive care unit family conference. Teaching a critical intensive care unit procedure. Ann Am Thorac Soc. 2015;12:469-471. 27. Marcus JD, Mott FE. Difficult conversations: from diagnosis to death. Ochsner J. 2014;14:712-717. 28. Baile WF, Buckman R, Lenzi R, et al. SPIKES-A six-step protocol for delivering bad news: application to the patient with cancer. Oncologist. 2000;5:302-311. 29. Orioles A, Miller VA, Kersun LS, et al. “To be a phenomenal doctor you have to be the whole package”: physicians’ interpersonal behaviors during difficult conversations in pediatrics. J Palliat Med. 2013;16:929933. 30. Mack JW, Wolfe J, Cook EF, et al. Hope and prognostic disclosure. J Clin Oncol. 2007;25:5636-5642. 31. Meyer EC. July 7, 2014. On Being Present, not Perfect. http://vector. childrenshospital.org/2014/07/communication-and-the-patient-experience-on-being-present-not-perfect/. 32. Burns JP, Edwards J, Johnson J, et al. Do-not-resuscitate order after 25 years. Crit Care Med. 2003;31:1543-1550. 33. Morrison W, Berkowitz I. Do not attempt resuscitation orders in pediatrics. Pediatr Clin North Am. 2007;54:757-771, xi-xii. 34. Schlairet MC, Cohen RW. Allow-natural-death (AND) orders: legal, ethical, and practical considerations. HEC Forum. 2013;25:161171. 35. Venneman SS, Narnor-Harris P, Perish M, et al. “Allow natural death” versus “do not resuscitate”: three words that can change a life. J Med Ethics. 2008;34:2-6. 36. Diem SJ, Lantos JD, Tulsky JA. Cardiopulmonary resuscitation on television. Miracles and misinformation. N Engl J Med. 1996;334: 1578-1582. 37. Barnato AE, Arnold RM. The effect of emotion and physician communication behaviors on surrogates’ life-sustaining treatment decisions: a randomized simulation experiment. Crit Care Med. 2013; 41:1686-1691. 38. Halpern SD, Loewenstein G, Volpp KG, et al. Default options in advance directives influence how patients set goals for end-of-life care. Health Aff. 2013;32:408-417. 39. Halpern SD, Ubel PA, Asch DA. Harnessing the power of default options to improve health care. N Engl J Med. 2007;357:1340-1344. 40. Feudtner C, Morrison W. The darkening veil of “do everything.” Arch Pediatr Adolesc Med. 2012;166:694-695. 41. Clark JD, Dudzinski DM. The culture of dysthanasia: attempting CPR in terminally ill children. Pediatrics. 2013;131:572-580. 42. Kon AA. The shared decision-making continuum. JAMA. 2010; 304:903-904. 43. Connor SR. U.S. hospice benefits. J Pain Symptom Manage. 2009;38: 105-109. 44. Hoyer T. A history of the Medicare hospice benefit. Hosp J. 1998;13:61-69. 45. Feudtner C, Feinstein JA, Satchell M, et al. Shifting place of death among children with complex chronic conditions in the United States, 1989-2003. JAMA. 2007;297:2725-2732. 46. Feudtner C. Epidemiology and the care of children with complex conditions. In: Wolfe J, Hinds P, Sourkes B, eds. Textbook of Interdisciplinary Pediatric Palliative Care. Philadelphia, PA: Elsevier; 2011:7-18. 47. Ullrich CK, Mayer OH. Assessment and management of fatigue and dyspnea in pediatric palliative care. Pediatr Clin North Am. 2007;54:735-756. 48. Munson D. Withdrawal of mechanical ventilation in pediatric and neonatal intensive care units. Pediatr Clin North Am. 2007;54:773785.

References

e2























59. Santucci G, Mack JW. Common gastrointestinal symptoms in pediatric palliative care: nausea, vomiting, constipation, anorexia, cachexia. Pediatr Clin North Am. 2007;54:673-689. 60. Currow DC, Quinn S, Agar M, et al. Double-blind, placebo-controlled, randomized trial of octreotide in malignant bowel obstruction. J Pain Symptom Manage. 2015;49:814-821. 61. Beller EM, van Driel ML, McGregor L, et al. Palliative pharmacological sedation for terminally ill adults. Cochrane Database Syst Rev. 2015;(1):Cd010206. 62. de Graeff A, Dean M. Palliative sedation therapy in the last weeks of life: a literature review and recommendations for standards. J Palliat Med. 2007;10:67-85. 63. Gurschick L, Mayer DK, Hanson LC. Palliative sedation: an analysis of international guidelines and position statements. Am J Hosp Palliat Care. 2015;32(6):660-671. electronically published 7 May 2014. 64. Contro N, Kreicbergs U, Reichard W, Sourkes B. Anticipatory grief and bereavement. In: Wolfe J, Hinds P, Sourkes B, eds. Textbook of Interdisciplinary Pediatric Palliative Care. Philadelphia, PA: Elsevier; 2011:41-54. 65. Robins PM, Meltzer L, Zelikovsky N. The experience of secondary traumatic stress upon care providers working within a children’s hospital. J Pediatr Nurs. 2009;24:270-279. 66. Davidson JE, Powers K, Hedayat KM, et al. Clinical practice guidelines for support of the family in the patient-centered intensive care unit: American College of Critical Care Medicine Task Force 20042005. Crit Care Med. 2007;35:605-622.



























49. Truog RD, Cist AF, Brackett SE, et al. Recommendations for endof-life care in the intensive care unit: the Ethics Committee of the Society of Critical Care Medicine. Crit Care Med. 2001;29:2332-2348. 50. American Academy of Pediatrics Committee on Bioethics. Guidelines on foregoing life-sustaining medical treatment. Pediatrics. 1994;93:532-536. 51. Ragsdale L, Zhong W, Morrison W, et al. Pediatric exposure to opioid and sedation medications during terminal hospitalizations in the United States, 2007-2011. J Pediatr. 2015;166:587-593. 52. Friedrichsdorf SJ, Kang TI. The management of pain in children with life-limiting illnesses. Pediatr Clin North Am. 2007;54:645-672. 53. Zernikow B, Michel E, Craig F, et al. Pediatric palliative care: use of opioids for the management of pain. Paediatr Drugs. 2009;11:129-151. 54. Nalamachu SR. Opioid rotation in clinical practice. Adv Ther. 2012;29:849-863. 55. Williams DG, Patel A, Howard RF. Pharmacogenetics of codeine metabolism in an urban population of children and its implications for analgesic reliability. Br J Anaesth. 2002;89:839-845. 56. Thrane S. Effectiveness of integrative modalities for pain and anxiety in children and adolescents with cancer: a systematic review. J Pediatr Oncol Nurs. 2013;30:320-332. 57. Boyden JY, Connor SR, Otolorin L, et al. Nebulized medications for the treatment of dyspnea: a literature review. J Aerosol Med Pulm Drug Deliv. 2015;28:1-19. 58. Wusthoff CJ, Shellhaas RA, Licht DJ. Management of common neurologic symptoms in pediatric palliative care: seizures, agitation, and spasticity. Pediatr Clin North Am. 2007;54:709-733.

e3

Abstract: Pediatric intensivists must develop a high level of competency in core palliative care skills, such as communication, shared decision-making, appropriate limitation of interventions, compassionate extubation, and symptom management. Intensivists must also know when to consult a palliative care team for secondary palliative care support, such as in the case of complex

decision-making, advanced symptom management, need for enhanced family support, or transitioning a patient to hospice. Key Words: palliative care, communication, shared decisionmaking, compassionate extubation, symptom management, grief support, hospice

20 Organ Donation Process and Management of the Organ Donor TH OMAS A. NAKAGAWA, MUDIT MATHUR, AND ANTHONY A. SOCHET





A significant gap continues to exist between the numbers of organ transplants and transplant recipients on the national waiting list in the United States.1 Children less than 18 years of age account for approximately 1.5% of all patients waiting for a transplant.1 The majority of children and adults have end-stage renal failure and are waiting for a renal transplant, followed by those with hepatic disease waiting for a liver.1 Despite fewer pediatric donors following neurologic death, the total number of pediatric transplants is increasing.1 There is also a continued increase in children receiving organ transplants from pediatric donors.1–3 Unfortunately, children still die waiting for a life-saving transplant. The highest death rate observed occurs in those less than 1 year of age.1–3 At the same time, the proportion of children removed as a transplant candidate from the waitlist and die because their underlying illness progresses continues to increase.1 Taken in sum, there is a growing need for donated organs and improved therapies to preserve existing organ function for those awaiting transplantation. The global importance of organ donation is recognized and supported by the American Academy of Pediatrics (AAP) and other organizations.3,4 The AAP emphasizes education, the need to shape public policy, and a system in which organ procurement, distribution, and costs are fair and equitable to children and adults.4 Each state has laws and regulations for the determination of death that have been modeled after the Uniform Determination of Death Act (UDDA) in most cases.5 The UDDA provides a definition of death, stating that an individual who has sustained

• •













Successful organ recovery for transplantation requires perioperative donor management expertise to correct physiologic derangements associated with neurologic death. A collaborative, multidisciplinary approach to end-of-life care and donation requires early referral to the organ procurement organization, allowing medical professionals to dialogue and improve authorization rates while supporting families with end-of-life care decisions. Determination of neurologic death in children is based on clinical criteria that are consistent across the age spectrum.









PEARLS Donation after circulatory death (DCD) permits donation from patients who have suffered a catastrophic brain injury but do not progress to neurologic death. The sustained practice of pediatric DCD donation accounts for roughly 10% of all DCD donors nationally. Patient and graft survival for organs, especially kidneys, transplanted from DCD donors is comparable to organ transplants from donation after neurologic death. Neonatal donation provides another valuable opportunity to recover organs for transplantation.

either (1) irreversible cessation of circulatory and respiratory functions or (2) irreversible cessation of all functions of the entire brain (including the brainstem) is dead. A determination of death must be made in accordance with accepted medical standards. Criteria for the determination of death have been established in national guidelines for adults and children.6–8 In accordance with the dead donor rule, donation can only occur after death has been declared. Donation cannot result in the death of the patient.9 An ethical discussion about defining and determining death and the dead donor rule are beyond the scope of this chapter (see Chapter 18). Organ donation can occur following neurologic death (donation after brain death [DBD]), following circulatory death (donation after circulatory death [DCD]), or through living donation. The vast majority of organs are recovered following death established by neurologic criteria. Donor organs potentially recovered for transplantation are dependent on the type of donation. Organs recovered from DBD include the heart, lungs, liver, kidneys, pancreas, and intestines. Organs recovered following DCD include lung, liver, kidneys, and pancreas. Additionally, hearts have been recovered from DCD donors.10,11 Tissues including skin, bone, cartilage, heart valves, and corneas can be recovered from both DBD and DCD donors. Most pediatric deaths occur in intensive care units (ICUs) following the unplanned withdrawal of life-sustaining medical therapies,12 making opportunities for pediatric donation a rare event. Missed opportunities for organ donation occur for many 163

164

SECTION III



Pediatric Critical Care: Psychosocial and Societal

reasons, but the majority can be accounted for when a family declines the option of donation. Families may not have been provided the opportunity for donation because the medical team did not recognize donor eligibility. Approaching families about donation at an inappropriate time, failing to develop a shared cognitive model for the concept of brain or circulatory death, unreliable or misleading educational resources, and racial barriers may also contribute to declined authorization rates.3,13–16 Even after authorization, potential organs for transplantation may become unsuitable because of caregivers’ lack of familiarity with appropriate donor management, damage during recovery prior to transplantation, and medical examiner or coroner denials.13 Families who authorize donation are more likely to comprehend neurologic death and report a positive hospital experience compared with families that choose not to authorize donation.17 Unique aspects related to pediatric organ donation and transplantation are listed in eBox 20.1.

Process of Organ Donation Organ donation should be viewed as a process. Like many other aspects in medicine, this process evolves over time and includes (1) identification of a potential donor, (2) determination of neurologic death in a timely manner, (3) authorization for organ donation, (4) perioperative management of the donor, and (5) recovery of organs for transplantation. The donation process begins when a critically ill or injured child is identified as a potential donor, with an early or timely referral to the organ procurement organization (OPO). A federal mandate requires notification of the designated OPO for any impending death. Early involvement and timely notification prior to the determination of death generates a greater amount of time for collaboration with the OPO.18 This best practice permits coordination of the donation process with the medical team.3,4,14,18–20 Ensuring OPO engagement and the intensive care team’s understanding of the entire donation process is essential to eliminate confusion that may disrupt the donation process. Ensuring this best practice not only improves authorization rates but also assists families with understanding and coping with end-of-life care issues. A protective stance with parents or guardians of children because of preconceptions about donor eligibility may have the unintended consequence of denying families the opportunity to help or save the life of another person.21 Prior to approaching the family, the medical team and OPO should discuss potential donor suitability and coordinate the best way to approach a family about organ donation.3,4,14,18,19 Many national and international organizations have emphasized donation as a routine part of end-of-life care and have adopted this collaborative best practice to increase authorization and recovery of organs for transplantation.3,4,18,20,22,23 Discussions about donation with parents or guardians may differ from spouse or relatives of adult donors. Contrary to the traditional approach of the OPO coordinator requesting donation and decoupling the death and authorization process, one study found that the timing of organ donation discussions did not appear to influence donation decisions.24 Parents want sufficient time to discuss end-of-life care issues, including donation, and may prefer discussions regarding donation with the pediatric intensivist or a member of the healthcare team they have come to trust.3,24 Few families of either adults or children appear to suffer psychological harm by having the option of donation presented to them.21 In addition to collaboration with the OPO, successful authorization for pediatric donation may include the engagement of palliative care

specialists. Palliative providers may act as an additional resource to assist the ICU team and provide family support during end-of-life care discussions and organ donation.25,26 The benefits of organ donation extend beyond the transplant recipient. There are psychological and social benefits for the potential donor’s family. Organ donation may assist families in finding solace in the belief that their child’s death was not without meaning and helped or saved the life of other people.3,27 Knowing that their child will be remembered after death is one important way that organ donation helps families heal.

Role of the Pediatric Intensivist and Critical Care Team in the Process of Organ Donation Management of critically ill children and pediatric organ donors is best accomplished at a tertiary or quaternary pediatric facility that provides specialized pediatric critical care expertise required to manage this select group of patients.14,18 Medical management of the potential pediatric organ donor requires knowledge of the physiologic derangements associated with this patient population. Hemodynamic instability, alterations in oxygenation and ventilation, metabolic and endocrine abnormalities, and coagulation disturbances are common. Support and care of the family provided by a team of physicians, nurses, social workers, chaplains, family service providers, organ donation specialists, and other support staff trained in the unique aspects of pediatric medicine are integral to the care of these children and their families.3,14,18 The pediatric intensivist is central in coordinating care to ensure the successful recovery of organs from pediatric donors. The integral involvement of the pediatric intensivist and critical care team in the management of critically ill and injured children has been a foundation of clinical practice in many successful pediatric centers. Involvement of critical care specialists— especially in pediatric donation, in which there is a limited and decreasing number of donors—improves the quality and number of organs recovered.14,18 Caring for the critically ill child and the child’s family through all phases of illness, including end-of-life issues, should be a seamless transition. The continuum of care for the dying patient who progresses to death and becomes a donor requires the expertise of the pediatric intensivist and critical care team to preserve the option of donation.8 Patient management to prevent deterioration of organ systems and subsequent loss of transplantable organs while helping family members deal with the death of their child is fundamental to facilitating the donation process and successful recovery of organs for transplantation. National and international best practices for deceased organ donation emphasize that patient management should preserve the option of donation for potential donors prior to and after declaration of death.3,4,14,17–20,22,23 Preconceptions about eligibility for donation by the critical care team may not be current or accurate. Suitability of donor organs for transplantation is best assessed by the OPO.18 Thresholds for acceptable organ dysfunction can vary according to time of evaluation, transplant program comfort levels, and recipient urgency. For example, serial echocardiograms may demonstrate donor response to effective medical therapy, enabling cardiac recovery for transplantation: a positive blood culture or bacterial meningitis may not preclude organ donation if antibiotic therapy has been administered.18 In the United States health policy changes have impacted donor eligibility. The HIV Organ Policy Equity Act enacted in 2013 allows HIV-positive donors to donate organs for transplantation into HIV-positive recipients. Additionally, organs

164.e1

• eBOX 20.1 Unique Issues With Pediatric Organ

Donation and Transplantation • Parents and guardians must act as surrogate decision makers without benefit of being guided by the donor’s wishes, as is often possible with adults. • There is age-related variation in the timing associated with determination of brain death. • There are technical challenges related to surgical procedures in smaller children and infants. • Acquisition and use of organs recovered from children may be limited by size and weight constraints. • Specialized care required for the management of critically ill children and pediatric organ donors may be lacking at institutions without pediatric expertise.



CHAPTER 20

Organ Donation Process and Management of the Organ Donor

from hepatitis-positive donors are now being transplanted. Although these situations may be uncommon for pediatric patients, it emphasizes the need to be aware of the latest developments in public policy and use the expertise of the OPO as part of highquality end-of-life care.18 Collaboration with investigative teams, medical examiners, and coroners in cases of accidental or nonaccidental death is imperative to determine cause of death and allow the donation process to proceed.13,18 Preservation of evidence and early consultation and discussions with the coroner, medical examiner, or forensic team may result in requests for additional noninvasive imaging needed to supplement the death investigation without precluding donation.

Determination of Neurologic Death Most organ donations occur following neurologic death; therefore, the determination of neurologic death must precede any efforts to recover organs. Timely and efficient determination of neurologic death is important for several reasons: it allows the family to begin the grieving process as they prepare for the loss of a loved one; if donation is planned, organ preservation and preparation for recovery can begin; and if donation is not planned, medical therapies can be stopped, permitting redistribution of scarce ICU resources to other critically ill and injured patients. Determination of neurologic death must never be rushed or take priority over the needs of the patient or the family. Appropriate emotional support for the family should be provided, including adequate time to grieve with their child after death has occurred. Determination of neurologic death in children is a clinical process based on specific criteria consistent across the age spectrum. Criteria for the determination of neurologic death in infants and children in the United States were revised in 2011.6,7 These guidelines outline the minimum criteria to determine neurologic death for infants greater than 37 weeks estimated gestational age to 18 years of age. The guidelines do not challenge the definition but rather provide criteria for the determination of death. Although examination criteria for infants, children, and adults are similar, determination of irreversible injury and neurologic death can be more challenging in younger patients because of physiologic and anatomic differences, along with differences in the mechanism of injury in infants and children.6–8 Hypoxic-ischemic or traumatic brain injuries are the most common causes of neurologic death for infants and children.28 Two chronologically distinct, unbiased, independent neurologic examinations and age-based recommendations are a primary difference between adult and pediatric guidelines to determine neurologic death.6–8 eBox 20.2 lists the recommended observation periods based on the current pediatric guidelines for the determination of neurologic death. A clinical history, known cause of coma, and neurologic injury consistent with the clinical presentation are prerequisites to establish that an irreversible condition has occurred. Neurologic criteria to determine death in infants and children are listed in eBox 20.3. Normal physiologic parameters must be established and maintained before a determination of neurologic death or neurodiagnostic testing can be meaningful. Coma and apnea must coexist. In addition, confounding variables must be corrected before an evaluation of neurologic death. Conditions that may interfere with the neurologic examination or factors capable of causing a comatose state imitating brain death should be considered, corrected, and ruled out.

165

These include hypothermia, hypotension, severe hepatic or renal dysfunction, inborn errors of metabolism, metabolic disturbances including hypoglycemia, and toxic or iatrogenic ingestions. Toxins can interfere with the neurologic examination and should be considered in cases in which a definitive etiology of coma cannot be established. Longer-acting or continuous infusion of sedative agents and recent administration of neuromuscular blocking agents can interfere with the neurologic examination. When determining the appropriate timing of the clinical examination, the half-life of previously administered agents must be considered. End-organ dysfunction and the use of therapeutic hypothermia can delay pharmacologic clearance. Adequate time for clearance of sedatives and neuromuscular blocking agents must be provided. Testing for drug intoxication—including barbiturates, opiates, and alcohol—should be performed as indicated. Clearance of neuromuscular blocking agents can be confirmed by use of a nerve stimulator. There may be situations in which these conditions cannot be corrected and an ancillary study may be required to assist with the determination of neurologic death. For instance, the use of barbiturate coma in traumatic brain injury may require a prolonged observation period to ensure that measured levels are in the lowto mid-therapeutic range before a clinical determination of neurologic death can occur. Barbiturates reduce cerebral blood flow (CBF); however, there is no evidence that high-dose barbiturate therapy completely arrests CBF. Radionuclide CBF study or cerebral arteriography can be used in patients receiving high-dose barbiturate therapy to demonstrate the absence of CBF.6,7 Patients receiving targeted temperature management and hypothermia protocols require adequate time for drug clearance following rewarming prior to initiating testing for neurologic death. Current adult and pediatric guidelines in the United States and Canada recommend a minimum core body temperature of .35°C (95°F).6–9,29 The United States pediatric guidelines suggest waiting at least 24 hours following rewarming before instituting testing for neurologic death.6,7 A period greater than 24 hours following targeted temperature management may be required to determine neurologic prognostication and ensure that diagnostic error does not occur when determining neurologic death.30–33

Testing for Apnea Apnea testing is a critical and essential component of the clinical examination to determine neurologic death. Testing for apnea will result in respiratory acidosis and potential hypoxemia. Therefore testing must be performed safely with special attention to maintaining ideal oxygenation and hemodynamics. Apnea testing should be performed only after the patient has met the clinical criteria for neurologic death.6,7 Apnea testing must allow adequate time for the partial pressure of carbon dioxide (Paco2) to increase to levels that would normally stimulate respiration. Baseline Paco2 should be measured and allowed to rise to 60 mm Hg or greater and 20 mm Hg above the baseline to account for infants and children with chronic respiratory disease or insufficiency who may breathe only in response to supranormal Paco2 levels.6,7 The patient should be continually observed for any spontaneous respiratory movements over a 5- to 10-minute period or longer, with documentation of arterial blood gas(es) reaching the targeted threshold. During apnea, the Paco2 rises approximately 3 to 5 mm Hg/min.6,7 Venous blood gas measurements have not been sufficiently studied to document elevated CO2 during apnea testing.

165.e1

• eBOX 20.2 Recommended Observation Periods to

Determine Brain Death in Infants and Children • Term infants (37 weeks’ estimated gestational age) to 30 days of age: Two examinations and apnea tests separated by at least 24 hoursa • More than 30 days to 18 years of age: Two examinations and apnea tests separated by at least 12 hoursb a The observation period may be decreased if an approved ancillary study is used. A second clinical examination and apnea test must be performed following the ancillary study to declare death. Data from Nakagawa TA, Ashwal S, Mathur M, et al. Guidelines for the determination of brain death in infants and children: an update of the 1987 Task Force recommendations. Crit Care Med. 2011;39(9):2139–2155; and Nakagawa TA, Ashwal S, Mathur M, et al. Clinical report—Guidelines for the determination of brain death in infants and children: an update of the 1987 task force recommendations. Pediatrics. 2011;128(3):e720–740.

• eBOX 20.3 Neurologic Examination Criteria for

Brain Death Determination in Infants and Children 1. Coma. Patient must lack all evidence of responsiveness. Noxious stimuli should not produce a motor response other than spinally mediated reflexes. 2. Apnea. The patient must have the complete absence of documented respiratory effort (if feasible) by formal apnea testing demonstrating a Paco2  60 mm Hg and .20 mm Hg increase above baseline Paco2. 3. Loss of all brainstem reflexes, including: • Midposition or fully dilated pupils that do not respond to light • Absence of movement of bulbar musculature, including facial and oropharyngeal muscles • Absent gag, cough, sucking, and rooting reflexes • Absent corneal reflexes • Absent oculovestibular reflexes 4. Flaccid tone and absence of spontaneous or induced movements, excluding spinal cord events such as reflex withdrawal or spinal myoclonus. 5. Reversible conditions or conditions that can interfere with the neurologic examination must be excluded prior to brain death testing. Data from Nakagawa TA, Ashwal S, Mathur M, et al. Guidelines for the determination of brain death in infants and children: an update of the 1987 Task Force recommendations. Crit Care Med. 2011;39(9):2139–2155; and Nakagawa TA, Ashwal S, Mathur M, et al. Clinical report—Guidelines for the determination of brain death in infants and children: an update of the 1987 task force recommendations. Pediatrics. 2011;128(3):e720–740.

166

SECTION III



Pediatric Critical Care: Psychosocial and Societal

Apnea testing requires preoxygenation with 100% oxygen to prevent hypoxia and enhance the chances of successful completion of the apnea test. Mechanical ventilatory support should be adjusted to normalize Paco2 initially. Mechanical ventilation is removed, permitting Paco2 to rise while observing the patient for spontaneous respiratory effort. During apnea testing, oxygenation can be maintained by using a T-piece circuit connected to the endotracheal tube (ETT) or attaching a self-inflating bag valve system with titration of positive end-expiratory pressure (PEEP). Tracheal insufflation of oxygen using a catheter inserted through the ETT has also been used to provide supplemental oxygen. This technique is not recommended in children, as high gas flow rates may promote CO2 washout preventing adequate Paco2 rise, and catheter insertion too distally can potentiate barotrauma if gas outflow is not optimal or catheter size is too big relative to the ETT.6,7 False reports of spontaneous ventilation have been reported with patients maintained on continuous positive airway pressure for apnea testing despite having the sensitivity of the mechanical ventilator reduced to minimum levels.6,7 Apnea testing is consistent with neurologic death if no respiratory effort is observed during the testing period. The patient is placed back on mechanical ventilator support following apnea testing until death is confirmed with a second clinical examination and apnea test. Apnea testing should be aborted if hemodynamic instability occurs or oxygen saturation decreases to 85% or less. An ancillary study should be pursued to assist with the determination of neurologic death if targeted thresholds for apnea testing cannot be achieved or there is any concern regarding the validity of the apnea test. If an ancillary study is used, a second clinical examination and—if possible—a second apnea test must be performed. Any respiratory effort is inconsistent with neurologic death.

Ancillary Studies Ancillary studies are not necessary or mandatory if a determination of neurologic death can be made based on clinical examination criteria and apnea testing.6,7 Ancillary studies can provide additional supportive information to assist in neurologic death. Importantly, ancillary studies are not a substitute for a complete physical examination. If the clinical examination and apnea test cannot be safely completed, an ancillary study should be used to assist in the determination of death. The need for an ancillary study will be determined by the physician caring for the child based on history, the ability to complete the clinical examination and apnea testing, and state and local requirements.6,7,34 A second neurologic examination and apnea test is required even if an ancillary study is performed to determine neurologic death.6,7 Neurologic examination results must remain consistent with neurologic death throughout the observation and testing period. In circumstances in which an ancillary study is equivocal, the observation period can actually be increased until another study or clinical examination and apnea test are performed to determine neurologic death. A waiting period of 24 hours is recommended before performing another neurologic examination or follow-up ancillary study in situations in which the study is equivocal.6,7 eBox 20.4 lists clinical situations in which ancillary studies may be useful. The most widely available and commonly performed ancillary studies validated in children to assist with the determination of neurologic death are radionuclide CBF study and electroencephalography (EEG).6,7 Evaluation of anterior and posterior cerebral

circulation with four-vessel cerebral angiography is now rarely, if ever, used to evaluate blood flow in the determination of neurologic death in children. This test is difficult to perform in small infants and children, requires transporting a potentially unstable patient to the angiography suite, and necessitates technical expertise that may not be available in every facility. EEG and radionuclide CBF studies are more easily accomplished without the need for extraordinary technical expertise. EEG and radionuclide CBF studies evaluate different aspects of central nervous system (CNS) activity. EEG testing evaluates cortical and cellular function while radionuclide CBF testing evaluates blood flow and uptake into cerebral tissue. Each of these tests requires the expertise of appropriately trained and qualified individuals who understand the limitations of these studies to avoid misinterpretation. Specific criteria for these studies must be met to determine neurologic de ath.6,7,35,36 EEG may be more specific, although less sensitive, than the radionuclide CBF study.6,7 Radionuclide CBF studies have been used extensively with good results. The use of a portable gamma camera for radionuclide angiography has made CBF studies more accessible, allowing for the study to be undertaken at the bedside. This study has become a standard in many institutions, replacing EEG as an ancillary study to assist with the determination of neurologic death in infants and children.6,7,37 Transcranial Doppler sonography and brainstem audio-evoked potentials have not been studied extensively or validated in children.6,7,38,39 As a result, these studies—along with CT angiography, perfusion MRI, magnetic resonance angiography-MRI, (MRA-MRI), and Doppler ultrasonography of the central retinal vessels40 are not currently acceptable ancillary studies to assist with the determination of neurologic death in infants and children.6,7 The sensitivity of EEG and CBF studies are weaker in the neonatal age group.6,7,41,42 Limited experience with ancillary studies performed in newborns younger than 30 days of age indicates that EEG is less sensitive than CBF in confirming the diagnosis of brain death. The younger the child, particularly neonates less than 30 days of age, the more cautious one should be in determining neurologic death. If there is any uncertainty about the examination, apnea testing, or the ancillary study, continued observation is warranted. Additional clinical evaluations and apnea testing or a repeat ancillary study followed by a second clinical examination and apnea test should be performed to make the determination of neurologic death. Technologic advances continue to impact our ability to determine circulatory and neurologic death. In certain circumstances, determination of neurologic death may be complicated by open cerebral trauma or decompressive craniectomy, mechanical support with extracorporeal membrane oxygenation (ECMO), or use of advanced ventilation modalities.43–45 Performing apnea testing for a patient supported with ECMO has been safely accomplished.46,47 The patient is transitioned to a flow-inflating bag valve system with titration of PEEP and hypercapnia induced by reducing the sweep gas or adding exogenous CO2 to the circuit, thus permitting CO2 to rise to an appropriate level to stimulate respiration. The rate of CO2 rise will be variable depending on how much the sweep gas is reduced.48 Adding exogenous CO2 may reduce the duration of the apnea test. Patients supported on advanced mechanical ventilation modes (e.g., airway pressure release ventilation, high-frequency oscillation ventilation) may not tolerate apnea testing due to impairment of oxygenation, ventilation, or hemodynamics. Additionally, apnea testing may be altered by sedation and use of neuromuscular blockade commonly employed with advanced modes of ventilation. Apnea testing

166.e1

• eBOX 20.4 Clinical Situations for Which Ancillary

Studies May Be Useful • When the clinical examination or apnea testing cannot be safely completed due to the underlying medical condition of the patient • When there is uncertainty about the findings of the neurologic examination • If a confounding medication effect may be present • To expedite the determination of neurologic death by reducing the clinical observation period • Social, medical, and legal reasons

Organ Donation Process and Management of the Organ Donor

should be aborted if the patient becomes hemodynamically unstable or oxygen saturations fall to less than 85 mm Hg.6,7 The updated guidelines make no provisions for determining an oxygen saturation threshold for aborting apnea testing in patients with cyanotic heart disease. Patients with open craniocerebral trauma or decompressive craniectomy may not exhibit the increased intracranial pressure that commonly occurs in a closed skull or may retain limited regional circulation. In any situation in which the clinical examination and apnea test cannot be completed, an ancillary study is recommended to assist with the determination of neurologic death. The clinician should be aware that neurologic death cannot be determined if the required clinical examination or ancillary study cannot be completed. Determining neurologic death has great implications with profound consequences. The clinical diagnosis of neurologic death is highly reliable when made by experienced examiners using established criteria.6,7 Appropriate documentation of clinical examination, apnea testing, and any ancillary studies should be recorded when death has been determined. The updated guidelines for the determination of neurologic death in infants and children encourage the use of the incorporated guidelines checklist to assist with standardizing the process and documentation of neurologic death in children.6,7 For detailed information about determining neurologic death, the reader is encouraged to become familiar with the current pediatric guidelines6,7 and supplemental institutional or regional requirements.

Brain Death Physiology Progression to neurologic death results in neuroendocrine dysfunction requiring specific interventions to preserve organ function. Efforts to control cerebral perfusion pressure, hemodynamic manifestations of herniation, and loss of CNS function contribute to the instability that commonly occurs during and after progression to neurologic death. These physiologic changes clearly affect end-organ viability in the prospective organ donor. Understanding the physiologic changes and anticipating associated complications with neurologic death is therefore critical for organ function and recovery. Loss of CNS function causes diffuse vascular regulatory and cellular metabolic injury.48 Neurologic death resulting from cerebral ischemia increases circulating cytokines,49 reduces cortisol production,50 and precipitates massive catecholamine release. The combination of these factors may result in physiologic deterioration and, ultimately, end-organ failure if left untreated. Cerebral blood flow is approximately 50 mL/100 g per minute and accounts for 15% of the cardiac output.51 Without substrate consumption by the brain, glucose needs are reduced and the patient is prone to hyperglycemia. As neurologic death occurs, cerebral metabolism is further decreased and CO2 production falls, resulting in a reduction in Paco2. Hypothermia should be anticipated as a result of hypothalamic failure and loss of thermoregulation. Additionally, impaired adrenergic stimulation results in loss of vascular tone with systemic vasodilation and amplified heat losses. Ischemia of the anterior and posterior pituitary results in neuroendocrine dysfunction and pituitary hormone depletion. If left untreated, this leads to inhibition or loss of hormonal stimulation from the hypothalamus with subsequent fluid and electrolyte disturbances and, eventually, cardiovascular collapse. Hemodynamic deterioration associated with neurologic death is initiated by a massive release of catecholamines, commonly referred to as sympathetic, catecholamine, or autonomic storm.

167

This phenomenon is associated with cerebral ischemia and intracranial hypertension. Clinical manifestations include systemic hypertension and tachycardia.48,52 Autonomic storm exposes organs to extreme sympathetic stimulation from increases in endogenous catecholamines. The local effects of elevated sympathetic stimulation include increased vascular tone, effectively reducing blood flow and potentially causing ischemia to donor organs. Autonomic storm also has direct effects on the myocardium as the surge of catecholamines increases systemic vascular resistance (SVR), myocardial work, and oxygen consumption.53 Ischemic changes occur as a result of an imbalance between myocardial oxygen supply and demand, resulting in subendocardial is chemia.48,54 Myocardial ischemia impairs cardiac output, leading to dysfunction of donor organs. Myocardial dysfunction leads to elevated left ventricular end diastolic pressure and consequent pulmonary edema. This condition may be exacerbated by the displacement of systemic arterial blood into venous and pulmonary circulations due to catecholamine-mediated systemic vasoconstriction. Increased pulmonary vascular resistance and right heart volume overload may displace the ventricular septum into the left ventricle, further impairing cardiac output by impeding left ventricular filling.55 Progression to neurologic death results in a cascade of inflammatory mediator release, causing vasodilation as loss of sympathetic tone and catecholamine depletion occurs.55–57 Additionally, a shift from aerobic to anaerobic metabolism transpires as a result of ischemia and depletion of pituitary hormones, affecting cardiac performance and end-organ function. ­



CHAPTER 20

Pediatric Donor Management Perimortem management of the donor is a continuum of care extending from admission of a critically ill child to the recovery of organs for transplantation. Treatment of the DBD donor and the DCD donor differ and are discussed separately. Following the determination of neurologic death and the decision to proceed with organ donation, efforts to reduce intracranial pressure are abandoned and care shifts toward providing adequate circulation and oxygen delivery to preserve vital organ function for transplantation. Subsequent care will differ from management prior to death. Families and staff must be prepared for the paradigm shift in the goals of therapy from lifesaving to organpreserving. The critical care team should actively manage the potential donor and correct existing physiologic derangements that follow neurologic death to preserve the option of organ donation for the family.18 For example, decreased intravascular volume secondary to efforts aimed at reducing CBF and controlling intracranial hypertension (e.g., volume restriction and diuretic agents) must be repleted. Metabolic derangements should be corrected, such as iatrogenic hypernatremia from hyperosmolar therapy and hyperglycemia associated with catecholamine release and reduced cerebral metabolism. Volume loss from osmotic diuresis associated with hyperglycemia and diabetes insipidus (DI) following neurologic death must be anticipated and addressed to prevent cardiovascular collapse. Hemodynamic management goals are directed at maintaining normal peripheral perfusion and blood pressure for age. Additional donor management goals include preserving lung function, normalization of Paco2, temperature regulation, and metabolic disturbances. Infections present prior to authorization must continue to be treated until organ procurement occurs. Even if no infectious disease concerns exist, prophylactic antibiotics are routinely administered by many OPOs prior to organ recovery.14 Progression from neurologic death to somatic

168

SECTION III



Pediatric Critical Care: Psychosocial and Societal

death and loss of transplantable organs can result if prompt goaldirected care is not implemented.14,55,58 Donor management goals are listed in Table 20.1. In addition to targeting restoration of normal organ physiology, ideal donor management includes ongoing evaluation for organ suitability, serial assessment of organ function, immunologic testing, infectious disease screening, donor organ size matching, organ allocation, and coordination of surgical teams for organ retrieval.18 The goal of donor management therapy is to restore and maintain adequate oxygenation, ventilation, and perfusion to vital organs, thus preserving their function for successful transplantation. This can ultimately result in a higher yield of transplantable organs and improved graft function that may translate to a reduction in hospital length of stay and decreasing acquired morbidity and mortality in the transplant recipient.14,59–64

Treatment of Hemodynamic Instability Cardiac instability is the greatest limiting factor to successful organ recovery. Of all physiologic abnormalities encountered in the prospective organ donor, the cardiovascular system is fraught with the most complexity and variation. Hemodynamic instability and organ dysfunction account for a loss of up to 25% of potential

donors when donor management is not optimized.58 Furthermore, initiation of hormonal replacement therapy (HRT) early in the donation process may assist with stabilization of the donor, improve the quality of organs recovered, and enhance posttransplant graft function.59,64–67 The tremendous physiologic derangements associated with neuroendocrine dysfunction require specific interventions to restore normal physiology. These derangements are detailed later in this chapter and in Chapters 28, 31, and 34.

Hormonal Replacement Therapy Significant volume resuscitation and inotropic support are routinely required to correct severe cardiovascular derangements following neurologic death. Anterior pituitary hormone deficits result in thyroid and cortisol depletion and may contribute to hemodynamic instability.50 HRT restores aerobic metabolism, replaces hormones derived from the hypothalamus and pituitary, augments blood volume, and minimizes the use of inotropic support while optimizing cardiac output. HRT in adult donors is controversial, with correlations of hormone use, cardiac function, and variable clinical outcomes reported.64–67,77–80 One adult study demonstrated a reduced need for vasoactive infusions in 100% of unstable donors and abolished

TABLE Pediatric Donor Management Goals 20.1

Hemodynamic Support • • • •

Normalization of blood pressure Systolic blood pressure appropriate for age (lower systolic blood pressures may be acceptable if biomarkers such as lactate and SVO2 are normal) CVP ,12 mm Hg (if measured) Dopamine ,10 µg/kg/min or use of a single inotropic agent Normal serum lactate

Blood Pressure

Age

Systolic (mm Hg)

Diastolic (mm Hg)

Neonate

60–90

35–60

Infant (6 mo)

80–95

50–65

Toddler (2 y)

85–100

50–65

School Age (7 y)

90–115

60–70

Adolescent (15 y)

110–130

65–80

Oxygenation and Ventilation • • • • •

Maintain Pao2 .100 mm Hg Fio2 0.40 Normalize Paco2 35–45 mm Hg Arterial pH 7.30–7.45 Tidal volumes 8–10 mL/kg and PEEP of 5 cm H2O or tidal volumes 6–8 mL/kg and PEEP of 8–10 cm H2O

Fluids and Electrolytes

Measurement

Range

Serum Na

130–150 (mEq/L)

Serum K

3.0–5.0 (mEq/L)

Serum glucose

60–200 (mg/dL)

Ionized Ca11

0.8–1.2 (mmol/L)

1

1

Thermal Regulation Core body temperature 36–38°C Modified from Nakagawa TA. North American Transplant Coordinators (NATCO) Updated Donor Management and Dosing Guidelines. Lenexa, KS: 2008.

168.e1

The sympathetic storm associated with cerebral ischemia and intracranial hypertension results in intense but transient hypertension. If hypertension is severe and sustained, a cautious approach to treatment can be considered using a single IV dose or continuous infusion of a short-acting antihypertensive agent, such as hydralazine, sodium nitroprusside, esmolol, labetalol, or nicardipine titrated to effect. Profound vasodilation and hypotension following neurologic death occur due to cessation of sympathetic outflow. This should be anticipated and treated to restore normal circulation and perfusion. Profound and abrupt hypotension with release of proinflammatory mediators initiates a cascade of molecular and cellular events with resultant ischemia and reperfusion injury in vital organs.56 Management during this phase should target aggressive restoration of circulating volume, optimizing cardiac output and oxygen delivery to the tissues, and maintaining normal blood pressure for age (see Table 20.1) using catecholamine infusions as necessary.57,68 Isotonic crystalloid solutions—such as normal saline, colloid solutions (e.g., 5% albumin), or blood products (packed red blood cells for the anemic patient or plasma for the patient with a coagulopathy)—can be used for volume replacement. The use of artificial plasma expanders, such as hespan or dextran for volume resuscitation, should be avoided since large volumes of these agents can promote coagulation disturbances and impair renal function.14,68,69–71 Commonly used inotropic agents—such as dopamine, dobutamine, and epinephrine—can be titrated to effect. Catecholamines and dopamine appear to have immunomodulating effects that may help blunt the inflammatory response associated with brain death and improve kidney graft function.72,73 Vasopressors such as norepinephrine, vasopressin, and phenylephrine can be used in situations in which there is profound vasodilation and low SVR, though high doses can reduce perfusion to donor organs, potentially jeopardizing their viability prior to recovery and transplantation. Many OPOs routinely use a combination of inotropic support, volume resuscitation, and hormonal replacement therapy (HRT) to reduce vasoactive infusions that may impair perfusion to potential donor organs. Agents such as thyroid hormone, corticosteroids, vasopressin, and insulin are commonly employed during donor management.14,68,71 HRT can reduce circulatory instability associated with thyroid and cortisol depletion, especially in situations in which significant inotropic support is required.18,64–68 Acidosis, hypoxia, hypercarbia, and electrolyte disturbances can alter myocardial performance and must be corrected. Blood pressure, central venous pressure (CVP), mixed venous oxygen saturation, and serum lactate levels can guide adequate cardiac performance and tissue oxygen delivery. Echocardiography can provide useful information about filling pressures, wall motion abnormalities, and ventricular shortening or ejection fractions.

Serial echocardiograms are routinely employed in donor management and performed to assess cardiac function as treatment of the donor progresses. In many instances, cardiac performance improves with aggressive resuscitation and institution of HRT following neurologic death. An initial echocardiogram showing poor myocardial function should not be used to preclude donation.14,18 Many commonly used clinical indicators of end-organ perfusion become less reliable once brain death has occurred. For example, urine output is traditionally used as a gauge of adequate intravascular volume and renal perfusion but becomes unreliable in the setting of brain death and DI. Similarly, heart rate may not be a reliable sign of intravascular volume status. After death of the brainstem, there is loss of beat-to-beat variation, lack of vagal tone, and, thus, a fixed heart rate is commonly observed.74 Perfusion may be affected by temperature instability and hypothermia, resulting in delayed capillary refill time. Biomarkers such as mixed venous oxygen saturation and serum lactate levels may be more useful to guide cardiovascular management to ensure optimal oxygen delivery to tissues. Elevations in serum lactate and the development of metabolic acidosis provide evidence of tissue ischemia and should prompt immediate attention. Importantly, elevated serum lactate may be present following CNS or multisystem trauma and may persist following neurologic death or in those with profound hepatic dysfunction. Arrhythmias can occur during progression and following neurologic death. The catecholamine storm triggered by adrenergic stimulation results in myocardial ischemia and can cause necrosis of the conduction system, promoting tachydysrhythmias. Following neurologic death, bradyarrhythmias may not be responsive to atropine because of denervation of the heart; epinephrine then becomes the pharmacologic treatment of choice. Other factors contributing to arrhythmias include hypoxemia, hypothermia, cardiac trauma, and the proarrhythmic properties of inotropes. Hypotension from hypovolemia and vasodilation causes poor cardiac output and metabolic acidosis. Metabolic acidosis from inadequate cardiac output and electrolyte disturbances (specifically hypomagnesemia, hypocalcemia, and hypokalemia) that occur with DI may also promote rhythm disturbances. Identification and correction of the underlying cause of the arrhythmia are essential to address and treat rhythm disturbances. Cardiac arrest may be treated as part of active donor management in a decedent following neurologic death.74a,75 Extracorporeal support for the hemodynamically unstable donor has been considered in extreme cases, including hemodialysis for correction of fluid overload and electrolyte disturbances. The use of extracorporeal support to limit warm ischemic time for DCD donors should be avoided because anterograde circulation may be reestablished and negate determination of death.76



CHAPTER 20

Organ Donation Process and Management of the Organ Donor

the need in 53% of such donors.81 Decreased inotropic requirements have also been noted in children who received levothyroxine and vasopressin as part of donor management following neurologic death.82 Although studies are limited, HRT is a reasonable consideration when hemodynamic status does not improve with fluid and inotropic support. Based on a retrospective analysis of 40,124 organ donors over a 10-year period, the optimal HRT combination associated with multiple organ recovery appears to be thyroid hormone, a corticosteroid, vasopressin, and insulin.83 Existing literature supports improved outcomes with HRT and greater procurement rates may be seen with earlier administration of HRT.84 No published studies are available in children; however, one unpublished abstract retrospectively reviewed 1903 pediatric donors and showed that HRT was associated with increased odds of having the liver and at least one kidney and lung transplanted. The greatest benefit of HRT in donor management may, in fact, be improved graft function following transplantation.65,85,86 Given these observations, many OPOs have adopted the use of HRT as a routine part of donor management.14 Commonly used agents and doses for HRT in pediatric donors are listed in Table 20.2. Reduced free triiodothyronine (T3) levels following neurologic death may impair mitochondrial function and deplete energy stores. Animal studies have shown that diminished circulating T3 and thyroxine levels impair oxygen utilization.48 The effects of thyroid hormone on myocardial contractility are complex and can be immediate or delayed. The acute inotropic properties of T3 may occur as a result of beta-adrenoreceptor sensitization or may be completely independent of beta-adrenergic receptors.87–91 Furthermore, T3 administration may play an important role in maintaining aerobic metabolism at the tissue level after neurologic death has occurred.92,93 Beneficial hemodynamic effects in brain-dead patients receiving T3 administration have been variable.88,89 Levothyroxine (Synthroid) and T3 are the two IV thyroid agents available for administration. T3 is used in some centers for HRT; however, the cost of this medication may be prohibitive. No superiority of hemodynamic improvement or cardiac benefits of T3 has been demonstrated when compared with levothyroxine in hemodynamically unstable donors.89 Dosing of thyroid hormone for the pediatric organ donor (see Table 20.2) is weight-based and not well established. One retrospective study showed that younger children received larger bolus and infusion doses than older children and demonstrated enhanced weaning of inotropic support in children who progressed to brain death.82 Corticosteroids such as hydrocortisone are another pharmacologic agent routinely used by many centers for HRT to assist with hemodynamic support. There are little data demonstrating that hydrocortisone provides hemodynamic benefit in the potential pediatric organ donor.56 However, treatment of the donor with high doses of corticosteroids to reduce inflammation associated with neurologic death and modulate immune function may improve donor organ quality and posttransplant graft function.14 Additionally, the potential benefit of hydrocortisone and other steroids may lie in their ability to alter adrenergic receptors and regulate vascular tone by increasing sensitivity to catecholamines.93,94 Steroids have also been shown to stabilize pulmonary function, reduce lung water accumulation, and increase lung recovery from donors.95–98 The combination of thyroid hormone and steroids may be used to reduce vasoactive agent dose requirements in children. Additionally, vasopressin for control of DI can reduce the need for

169

inotropic support. Taken in sum, HRT may provide the greatest benefit to the recipient of transplanted organs by improving donor organ quality.

Management of Pulmonary Issues for the Potential Pediatric Organ Donor Increasing success with lung transplantation for the treatment of patients with end-stage lung and pulmonary vascular disease has placed a premium on the acquisition of lungs from the donor pool. Children waiting for a lung transplant comprise less than 5% of the national pediatric waitlist.1 However, the demand for lungs far exceeds availability because lungs are the organs most likely to be found unsuitable for transplantation. Recovery of lungs for transplantation accounts for 7% to 22% of the multiorgan donor pool.100,101 Low lung transplant recovery rates reflect stringent donor selection criteria and lack of suitable organs for transplantation. A commonly used donor selection criterion is a ratio of Pao2 to fraction of inspired oxygen (Fio2) ratio greater than 300 mm Hg (Pao2 . 300 mm Hg with Fio2 1.0 using a PEEP of 5 cm H2O). Every effort should be made to preserve lung function using lung-protective strategies, as many marginal lungs may become unsuitable for transplantation.101 Many factors contribute to the low rates of acquisition of lungs for transplantation in children. Blunt trauma resulting in pulmonary contusion/hemorrhage and inhalational or thermal injuries may directly damage the lungs and airway structures. Infection can compound the effects of existing lung disease or injury.97 Neurogenic pulmonary edema may develop during the progression or upon completion of neurologic death, resulting in high ventilator settings.55 Sympathetic storm associated with neurologic death causes systemic and pulmonary vasoconstriction. Neurogenic pulmonary edema occurs as pulmonary venous pressure rises, causing pulmonary capillary wall disruption.102 This predictable deterioration of pulmonary function compounds secondary lung injury following neurologic death.101,103 Furthermore, lungs are particularly vulnerable in the face of critical illness, leaving them susceptible to complications such as fat emboli, pulmonary emboli, aspiration pneumonia, ventilator-associated pneumonia, and atelectasis.97 Each of these factors contributes to secondary lung injury, impairs ventilation and oxygenation, and reduces lung suitability for transplantation. Management strategies to protect donor lungs have resulted in improved recovery and successful transplantation of these organs.104,105 These measures include diligent pulmonary secretion clearance with frequent suctioning, patient turning, and airway evaluation with flexible bronchoscopy.97,105 Ventilator management with attention to recruitment maneuvers such as sustained inflations and PEEP have been advocated to avoid the development of atelectasis and treat pulmonary edema associated with the catecholamine storm that occurs with neurologic death.104,106 The benefit associated with these maneuvers must be balanced against the risk of barotrauma and effects on preload that can potentially embarrass cardiac output in the donor with myocardial dysfunction. Cardiovascular effects can be minimized if adequate preload is provided prior to escalation of PEEP. Colloid solutions have been recommended to minimize accumulation of pulmonary edema.106 Albuterol has been shown to enhance clearance of pulmonary edema in an animal model77 and to improve mucociliary clearance.97 However, a randomized controlled trial using highdose albuterol nebulization did not show improvement in donor

170

SECTION III



Pediatric Critical Care: Psychosocial and Societal

oxygenation or lung utilization.107 Corticosteroids are frequently used in the donor and have been shown to reduce lung water accumulation and stabilize pulmonary function.95–98 Another novel therapy involves the use of naloxone to improve gas exchange in donor lungs,108 although a recent study showed no improvement in oxygenation compared with placebo in hypoxemic organ donors.109 The exact mechanism of action of naloxone to enhance pulmonary function is unknown, but free radical scavenging has been suggested. Ventilatory requirements may become minimal in the donor as neurologic death progresses. Respiratory alkalosis is common as the metabolic production of CO2 from the brain ceases and compliance of the chest wall changes. Restoring normocarbia with a Paco2 goal of 35 to 40 mm Hg in the child who has progressed to neurologic death is ideal given the effects of pH on unloading characteristics of oxygen from hemoglobin. Avoiding overdistension of the lungs during mechanical and manual ventilation is crucial to reducing the risk of barotrauma or further pulmonary injury.97,105 Donor management goals include achieving a Pao2 greater than 100 mm Hg and oxygen saturation greater than 95% using the least amount of Fio2 necessary. Adequate alveolar recruitment may be obtained with a tidal volume of 8 to 10 mL/kg, and PEEP of 5 cm H2O or lower tidal volumes of 6 to 8 mL/kg and higher PEEP of 8 to 10 cm H2O. A randomized trial in adult donors demonstrated benefits of using lung-protective ventilation strategy with tidal volumes of 6 to 8 mL/kg, 8 to 10 cm H2O PEEP, a closed circuit for suctioning, continuous positive airway pressure equal to previous PEEP for apnea tests, and recruitment maneuvers after any disconnection from the ventilator. Lung re-

covery rates doubled (54% vs 27%) compared with a conventional ventilator protocol.110 Elevation of the head of the bed and use of a cuffed endotracheal tube with high cuff pressures to reduce aspiration risk are also advocated.97 Additionally, oral care using chlorhexidine may reduce the chances of a ventilatorassociated infection. Frequent turning and chest physiotherapy when appropriate are essential to prevent atelectasis. Prone positioning may be beneficial to optimize ventilation perfusion matching. Serial chest radiographs are commonly obtained to identify correctable issues or evaluate pulmonary pathology. Donor management guidelines for oxygenation and ventilation are summarized in Table 20.1. Diagnostic and therapeutic flexible bronchoscopy can assist with the clearance of mucous plugs or blood clots that may contribute to impaired oxygenation. Many OPOs advocate early bronchoscopic evaluation of the lungs to assess anatomy, address correctable issues, and maximize ventilation strategies to improve lung function.

Fluid and Electrolyte Disturbances Fluid and electrolyte disturbances in the pediatric donor are the result of physiologic abnormalities following neurologic death and the consequences of premortem medical management. Commonly encountered derangements include dehydration; hyperglycemia; and sodium, potassium, and calcium disturbances. If left untreated, these abnormalities can adversely affect donor organ viability. Evaluation and treatment of such disorders in the pediatric organ donor are discussed here and in Chapter 71.

TABLE 20.2 Pharmacologic Agents Used for Hormonal Replacement in Children

Drug

Dose

Route

Comments

Desmopressin (DDAVP)

0.5 mg/h

IV

Terminal half-life of 75 min (range, 0.4–4.0 h). Titrate to control urine output (2–4 mL/kg/h). May be beneficial in patients with an ongoing coagulopathy.

Vasopressin (Pitressin)

0.5 milliunits/kg/h

IV

Half-life of 10–35 min. Titrate to control urine output (2–4 mL/kg/h). Hypertension can occur.

Levothyroxine (Synthroid)

0.8–1.4 mg/kg/h Maximum 20 mg/h

IV

Titrate to effect. Bolus dose 1–5 mg/kg can be administered (maximum dose 20 mg). Smaller infants and children require a higher bolus and infusion dose.

Triiodothyronine

0.05–0.2 mg/kg/h

IV

Titrate to effect.

Methylprednisolone

20–30 mg/kg Maximum dose 2 g or use hydrocortisone infusion below

IV

Dose may be repeated in 8–12 hours. Fluid retention and glucose intolerance can occur.

Hydrocortisone infusion

,25 kg, 1 mg/kg/h 26–35 kg, 50 mg/h 36–45 kg, 75 mg/h .45 kg, 100 mg/h

IV

Maximum dose should not exceed 100 mg/h.

Insulin

0.05–0.1 U/kg/h

IV

Titrate to effect to control blood glucose levels. Monitor for hypoglycemia.

Modified from Nakagawa TA. North American Transplant Coordinators (NATCO) Updated Donor Management and Dosing Guidelines. Lenexa, KS: 2008.

170.e1

Intravascular volume depletion is frequently encountered in the child with traumatic brain injury who has progressed to neurologic death. Fluid restriction is commonly employed in the management of cerebral edema, along with hypertonic solutions and osmotic diuretics. Another contributor to intravascular volume depletion is osmotic diuresis from hyperglycemia secondary

to steroid and catecholamine use and increased availability of glucose from loss of cerebral metabolism. Furthermore, DI exacerbates sodium and water imbalance if not aggressively treated. Intravascular volume and tissue perfusion must be restored and maintained guided by CVP, perfusion, serum electrolyte concentrations, and serial lactate measurements.



CHAPTER 20

Organ Donation Process and Management of the Organ Donor

Diabetes Insipidus Ischemia and ultimate necrosis of the posterior pituitary following neurologic death leads to loss of central antidiuretic hormone secretion and central DI. Uncontrolled urine output with free water losses results in severe hypernatremia, severe dehydration, hypovolemic shock, and eventual cardiovascular collapse. Treatment of DI is detailed here and in Chapter 84.

Oliguria Oliguria can be seen with volume depletion, acute renal insufficiency or failure, and overly aggressive pharmacologic management of DI. If urine output falls to less than 1 mL/kg/h and does not improve after decreasing or discontinuing vasopressin or desmopressin, intravascular volume status must be evaluated and appropriately treated using volume expanders. If urine output does not recover, it may be improved by initiation of inotropic or vasopressor support. Furosemide or mannitol can be used to stimulate urine output in the patient with adequate intravascular volume status. A selective dopamine agonist, such as fenoldopam, can be used to enhance urine output and may provide renal protection in the normotensive or hypertensive patient.118 Treatment of glucose and electrolyte abnormalities that may be encountered in the potential organ donor patient is detailed next.

Coagulation Abnormalities Coagulation abnormalities can arise secondary to the release of tissue thromboplastin and cerebral gangliosides from the injured brain.55 Additionally, the catecholamine surge associated with traumatic brain injury may contribute to coagulation disturbances.119 Treatment of coagulation disturbances seen with brain injury/death is further detailed in Chapters 89 and 90.

Thermoregulatory Instability Hypothermia is common after neurologic death due to loss of hypothalamic-pituitary function that maintains thermoregulation. Vasodilation with an inability to compensate for heat loss by shivering or vasoconstriction is a common cause of thermoregulatory instability in this patient population. Infusion of large volumes of room temperature IV fluids to treat DI and volume depletion contributes to hypothermia. Hypothermia can promote cardiac dysfunction, arrhythmias, coagulopathy, a cold-induced diuresis secondary to decreased renal tubular concentration gradient, and a leftward shift of the oxyhemoglobin dissociation curve, resulting in decreased oxygen delivery to the tissues.122 Radiant warmers, warm blankets, thermal mattresses, warm IV fluids or a blood warmer for infusion of blood products, and environmental warming will help maintain body temperature. Additionally, heating-inspired gases can assist in controlling body temperature. Avoiding hypothermia is essential to prevent deterioration of the potential organ donor.

Medical Examiner and Coroner Issues and Organ Donation for Children Many children who die from head injuries are victims of nonaccidental trauma. The medical examiner or coroner has a legal and social responsibility to determine the manner and cause of death

171

in these cases. When a child’s death is ruled a homicide, great sensitivity is required to balance preserving the integrity of the ongoing criminal investigation while respecting the family’s desire for organ donation to occur. Successful recovery of organs and the prosecution of the perpetrator may still occur in most cases with close cooperation between forensic investigators, treating physicians, transplant team, and OPO.4,14,123–126 Mechanisms have been described, including modification of surgical techniques, to preserve potential evidence and permit the mutual goals of the family and OPO.127 There is no reported case law in which organ donation has resulted in loss of evidence and inability to prosecute a perpetrator involved with the death of a child. Early involvement of the medical examiner or coroner and protocols to facilitate organ recovery in homicide cases may reduce denials for organ donation.124–126,128 Data from the Collaborative Pediatric Critical Care Research Network would suggest that donation is still possible even when cases proceeded with autopsy initiated by the medical examiner or family.129 Involvement of the district attorney during protocol development may also be a consideration, especially in high-profile cases. Efforts to reduce the number of medical examiner denials for donation are supported in the position statement by the National Association of Medical Examiners, which states, “Medical examiners and coroners should permit the recovery of organs and/or tissues from decedents falling under their jurisdiction in virtually all cases, to include cases of suspected child abuse, other homicides, and sudden unexpected deaths in infants.”130 Despite ongoing national efforts, many transplantable organs from potential pediatric donors go unrecovered and represent missed opportunities because of medical examiner/coroner denials.13

Donation after Circulatory Death While the vast majority of recovered organs from donors represent DBD, DCD has become an additional source of valuable organs for transplantation. Formerly known as “non-heart-beating organ donor” or “donation after cardiac death,” DCD is not new. Prior to the development of guidelines to determine neurologic death, cadaveric organs were routinely recovered after death was declared following loss of circulation. DCD allows organ recovery of kidneys, liver, lungs, pancreas, and—in some instances—heart from patients with catastrophic brain injury who do not progress to neurologic death. The discussions and decision to donate organs must be clearly separated from, and can only occur after, the decision to withdraw life-sustaining medical therapy. This avoids a perceived ethical conflict that the patient is being allowed to die primarily to recover organs. Routine humanistic end-of-life care—specifically, comfort measures, including administration of analgesic and sedatives— must be provided as they would for any patient undergoing withdrawal of life-sustaining medical therapies.18 Management of the DCD donor differs from the brain-dead donor because a determination of death is prospective with the planned withdrawal of therapy. Withdrawal of life-sustaining medical therapies can occur in the ICU or operating theater. Logistics regarding patient transport to limit warm ischemic time and to allow family presence until a determination of circulatory death is confirmed are important considerations. Additionally, the medical team should have provisions and prepare the family for ongoing care if donation is not possible. Specific criteria must be met to ensure the viability of recovered DCD organs. Circulatory arrest must usually occur within

171.e1

Hypernatremia (serum sodium .155 mEq/L) can affect organ suitability for transplantation and has been associated with worse graft outcomes following liver transplantation.14,111,112 In addition to hourly maintenance IV fluids, one-quarter or onehalf normal saline can be used to replace urine output in excess of 3 to 4 mL/kg until pharmacologic replacement therapy with vasopressin or desmopressin acetate (DDAVP) is implemented. Glucose should be avoided in renal replacement fluids to prevent further exacerbation of hyperglycemia and osmotic diuresis. Enteral free water supplementation administered through a nasogastric tube can be used for correction of severe hypernatremia. However, enteral free water supplementation may be limited by gastrointestinal ischemia and impaired absorption. Rapid osmotic shifts during correction of hypernatremia are inconsequential since neurologic death has already occurred. Pharmacologic agents such as vasopressin or desmopressin are routinely used in HRT protocols to control excessive urine output and free water loss associated with DI. Pharmacologic treatment is intended to reduce and not completely abolish urine output. Donor management goals while treating DI include maintaining a normal serum sodium level and reducing excessive urine output. Vasopressin is a polypeptide hormone secreted by the hypothalamus and stored in the posterior pituitary. Vasopressin acts on V1 and V2 receptors stimulating contraction of vascular smooth muscle with resultant vasoconstriction. It has a short half-life of 10 to 35 minutes and, unlike desmopressin, has no effect on platelets.14,71,113 Vasopressin can be administered by bolus or continuous IV infusion. The most desirable features of this agent derive from its ease of titration to control urine output. When discontinued, its effects are short-lived. Vasopressin is administered at doses of 0.5 mU/kg per hour and can be titrated to control urine output to 2 to 4 mL/kg per hour.8,71,113 By titrating in

this way, one preserves renal function and avoids volume overload and metabolic abnormalities such as hyponatremia and hyperkalemia. Vasopressin acts synergistically and has a catecholaminesparing effect, making it ideal for the donor with DI and hemodynamic instability, requiring vasopressor support.66,68,71 The infusion may need to be reduced after several hours to avoid complete loss of urine output.14 Excessive dosing of vasopressin should be avoided to preserve end-organ function as high doses, especially when combined with other vasopressors, may potentially reduce splanchnic perfusion, affecting hepatic and pancreatic blood flow. Additionally, vasoconstriction and increased smoothmuscle contractility may affect coronary and pulmonary blood flow.54 Excessive dosing of vasopressin should be avoided to preserve end-organ function. DDAVP is a more potent synthetic polypeptide structurally related to vasopressin. This agent lacks smooth-muscle contractile properties and is more specific for the V2 receptor. Desmopressin enhances platelet aggregation and has a longer half-life of 6 to 20 hours when administered as a single IV dose.115,116 Desmopressin may be a preferred agent for the correction of hypernatremia in hemodynamically stable donors who require no inotropic or vasopressor support or for the donor with ongoing bleeding issues because it enhances platelet aggregation. Desmopressin can be administered by continuous infusion at 0.5 mg/h titrated to control urine output or as a single IV dose.68,71,117 Intramuscular and intranasal administration can result in erratic absorption and should be avoided. The terminal half-life of desmopressin administered by continuous IV infusion is 75 minutes with a range of 0.4 to 4 hours.117 The longer half-life of desmopressin may be less desirable compared with the shorter halflife of vasopressin in potential kidney donors. However, desmopressin therapy may be discontinued 3 to 4 hours prior to organ recovery.68

171.e2

Glucose, Potassium, and Calcium Derangements Neurologic death causes major hormonal alterations resulting in insulin resistance and gluconeogenesis. Hyperglycemia as a result of corticosteroid and catecholamine use and increased availability of glucose due to the loss of cerebral metabolism can lead to an osmotic diuresis, exacerbating an already depleted volume status in the donor. Hyperglycemia can be avoided by frequently monitoring serum glucose concentration and making appropriate adjustments in the dextrose concentration in IV fluids. If these simple maneuvers are unsuccessful in controlling blood glucose levels, an insulin infusion should be administered to maintain glucose levels within a reasonable range (60–200 mg/dL). Although target glucose levels for insulin therapy in the deceased donor continue to be

debated, uncontrolled hyperglycemia should be treated.14 Serum glucose levels should be closely followed to avoid hypoglycemia. Potassium derangements can result from diuresis, acute kidney injury, corticosteroid administration, and acid-base disturbances. Potassium can be supplemented if hypokalemia is significant or if arrhythmias occur. The adverse effects of hyperkalemia are clearly more hazardous than those of hypokalemia. Hypocalcemia occurs commonly secondary to large volume replacement with colloids such as albumin, massive blood transfusions that result in large amounts of citrate reducing free calcium concentrations, and sepsis. Calcium is necessary for myocardial contraction, and hypocalcemia can depress cardiac output, affect SVR, and affect organ perfusion.68 The use of calcium supplementation should be guided by ionized calcium levels.

171.e3

Synthetic plasma expanders such as hespan are not recommended for volume replacement as they can promote or worsen coagulopathy.14,70,71,120 Thrombocytopenia and platelet dysfunction can be induced by commonly used drugs such as heparin, antibiotics, bblockers, calcium channel blockers, histamine H2 receptor antagonists, tromethamine, and hespan.70,121 Patients with hepatic disease or dysfunction have reduced synthesis of vitamin K-dependent clotting factors. A dilutional coagulopathy can occur from massive red cell transfusions if coagulation factors are not replenished. Coagulation abnormalities can also be exacerbated by hypothermia.

Coagulopathy can be treated by using fresh-frozen plasma, platelets, and cryoprecipitate, depending on the underlying abnormality, and by restoring and maintaining normothermia. Coagulation abnormalities should be addressed prior to transport of the donor to the operating suite for organ recovery. A minimum platelet count of 75,000/uL should be obtained prior to recovery of organs in the operating suite. The use of aminocaproic acid (Amicar), an antifibrinolytic agent, and other similar hemostatic agents are not recommended for the treatment of bleeding since microvascular thrombosis may be induced in donor organs.55

172

SECTION III



Pediatric Critical Care: Psychosocial and Societal

60 minutes after withdrawing life support. The time constraint is important since longer periods of hypotension and/or hypoxia prior to circulatory arrest will result in organ ischemia, precluding viable organs for transplantation. Following loss of pulse pressure (mechanical asystole ideally noted on arterial tracings), the patient is observed for 2 to 5 minutes before the recovery of organs can begin. This 2- to 5-minute observation, or “hands-off” period, is crucial to ensure that autoresuscitation (anterograde flow of blood ejected from the heart) with restoration of spontaneous circulation does not occur.75 Prior to and during this observation period, CBF falls below a normal cerebral perfusion pressure threshold. Cerebral activity in DCD patients, measured by EEG, ceases within 15 to 30 seconds after circulatory arrest.131–133 Importantly, it is the loss of circulatory function and not electrical activity (electrical asystole) that is required to determine death.76 Determining when a patient will develop circulatory arrest can be difficult. Scoring tools have been developed in an effort to predict whether death will occur within the specified time period to permit DCD.134–136 These tools should be used in conjunction with the physical examination and in discussion with the critical care team and OPO to determine donor suitability and the likelihood of progressing to death in the allotted time. If circulatory cessation does not occur within 60 minutes, organ recovery is aborted and comfort measures continue in the ICU or another predetermined location. DCD requires close collaboration between the critical care team, operating room staff, and OPO to ensure the successful recovery of organs. The reevaluation of this method of donation was prompted by an increasing need to meet the demands of a growing national transplant waitlist.137,139 There have been significant increases in the number of DCD donors over the past 15 years.1 This sustained practice of organ recovery in children accounts for more than 10% of all DCD donors.1 This method of donation has increased organ recovery from children and enhanced overall organ availability for transplantation.1,137,140–142 Individual numbers of DCD donors may be small at any one pediatric center but the collective impact from all pediatric centers supporting DCD has significantly increased organ availability.1,137,140–143 DCD focuses on recovery of the two most commonly needed organs for children: liver and kidney. Success with transplantation of DCD organs, primarily kidney and liver, is occurring in many centers. Rates of graft survival for DCD kidney and liver appear to be similar to organs recovered from donors following neurologic death.144–149 Results from pediatric DCD renal and liver transplants have been acceptable.2,150,151 Some centers have experienced an increased risk of biliary stenosis, hepatic infections, postoperative complications, and higher repeat transplantation rates with the use of DCD livers.14,152,153 A recent single-center comparison is more encouraging, with 100% 10-year patient and graft survival rates of livers procured from both DCD and neurologic death donors. No ischemic biliary complications with DCD were noted.154 There is increasing experience with lung transplants from DCD donors in adults and children.1,155–158 Neonatal donor opportunities continue to be explored as another source of valuable organs for transplantation.159–163 Although neurologic death is rare in neonates, recovery of DCD organs is becoming more common. Recovery of DCD neonatal kidneys has the potential to increase the number of kidneys available for transplantation.14,150,162,163 En bloc renal transplant from neonatal and pediatric DCD donors is occurring with good success, even in donors as small as 1.9 kg.150,163–165 Additionally,

successful transplantation of three hearts recovered from neonatal DCD donors under an established research protocol10 and the use of ABO-incompatible hearts has occurred.166 Some controversy continues to exist over this method of donation. Ethical concerns focus on the timing of death, whether the potential donor meets criteria for death at the time of organ recovery, and whether antemortem medical therapies result in nonbeneficence or potential maleficence.75,167–171 Although an ethical discussion on DCD donation is beyond the scope of this chapter, many national organizations in the United States and Canada have reviewed and concluded that DCD is ethically acceptable when performed within specific guidelines. The Society of Critical Care Medicine, Institute of Medicine, American Medical Association, AAP, and Canadian and other medical societies support DCD as an acceptable means to recover organs for transplantation.4,172–177 The AAP recognizes ethical concerns and supports institutions in efforts to provide access to DCD while encouraging but not mandating physician participation with this type of donation.178 Ultimately, limiting family members’ desire to have their loved one become a donor is a restriction of autonomy.21 Education regarding this mode of donation is crucial for all healthcare personnel to identify and recover organs from this population of children successfully.3,179,180

Contraindications to Organ Donation There are relatively few medical contraindications to organ donation. The ultimate decision on whether an organ is acceptable for transplantation is determined by the transplant surgeon and medical director of the OPO. This decision is based on the patient’s history, laboratory studies, and inspection of organs at the time of recovery. Overwhelming sepsis is usually a contraindication to donation. In many of these cases, most organs will suffer from inadequate perfusion, rendering organs unacceptable for transplantation. However, patients with meningitis or bacteremia can donate organs if they have been appropriately treated with antibiotics for 24 hours prior to organ recovery.14,18 Other CNS infections, such as encephalitis or disseminated viral infections, may pose too great a risk to the recipient, limiting donation potential in these patients. The advent of direct-acting antiviral therapy for hepatitis has allowed hepatitis C donor organs to be transplanted into recipients with the same virus or hepatitis-negative recipients with reasonable outcomes.181–184 Patients with HIV were once considered ineligible to become donors. However, the HIV Organ Policy Equity (HOPE) Act bill allows transplantation of HIV-positive organs from a donor with the same serotype. With antiretroviral therapy, HIV is no longer considered a fatal disease. In the United States, liver and kidney recovery from a deceased HIV-positive donor were transplanted into an HIV-positive recipient. A living donor with HIV was able to donate a kidney to another HIV-positive recipient.185 Internationally, a unique transplant occurred in which an HIV-positive mother donated a portion of her liver to her HIV-negative child suffering from hepatic failure.186 Active malignancy is a contraindication to donation. However, patients with cancer in remission may be candidates for donation depending on the duration of remission. Patients with CNS tumors that have not metastasized may be eligible to donate organs. However, surgical intervention or medical treatment of these tumors, including shunt placement or craniotomy, may limit eligibility for donation. Concerns regarding donation potential should be discussed with the OPO medical director before a decision regarding donation eligibility can be established.



CHAPTER 20

Organ Donation Process and Management of the Organ Donor

Evolving Areas of Transplantation Improved donor management, operative techniques, postoperative care, organ perfusion technology, and immunosuppressive therapy have advanced transplantation with recovery of more organs and better graft function. These innovations assist in meeting the growing number of potential recipients waiting for a lifesaving transplant. Organs once considered suboptimal are now considered for transplantation as medical care and technologies have enhanced donor organ function and engraftment. Ex vivo and in situ perfusion technologies represent an evolving area of growth and development in the field of transplantation. Regional techniques and organ-specific support systems are designed to improve quality and procurement rates prior to transplantation.187–194 These organ preservation systems specifically designed for kidney, heart, liver, and lungs overcome limitations from cold ischemic preservation that can affect graft performance following transplantation. Ex vivo perfusion can potentially condition and maintain organs in a near-physiologic state (healthy and functioning), allowing for longer preservation time for procured organs prior to transplantation. Ethical concerns exist regarding the use of regional extracorporeal support in the donor to minimize warm ischemic times for DCD abdominal organs. At a minimum, impeding recirculation to the brain or thorax is essential to prevent potential reanimation and restoration of circulation that would negate the determination of circulatory death.76 Infants waiting for a transplant continue to have the highest death rate on the waiting list.1,2 Neonatal donation opportunities are rare but can increase organ availability for transplantation. Novel donor populations, such as anencephalic infants, have the potential to increase organ availability for transplantation.195 Many infants waiting for a heart transplant may die because an organ never becomes available. ABO-incompatible heart transplantation for children less than 2 years and recovery of hearts from neonatal DCD donors may provide needed organs to decrease infant deaths on the national waitlist.160,166,196,197 As mechanical support devices become increasingly available as a bridge to heart transplantation, neonatal donation will require continued advocacy by neonatologists, OPOs, and institutions to limit missed opportunities for donation in the neonatal intensive care unit population. Abdominal organs from neonates are being recovered and used for transplantation. Special processing of liver cells from neonatal donors for liver cell transfusion is now entering into phase II clinical trials.198,199 Donor hepatocytes are infused into infants with urea cycle defects and Crigler-Najjar syndrome as a bridge to transplantation. In addition, en bloc kidney recovery from neonatal donors is occurring and organs are being transplanted with good success.164,165 Intestinal transplantation may be indicated for children with anatomic or surgical short gut (e.g., intestinal atresia, gastroschisis, and volvulus) and those with severe functional or motility disorders (e.g., Hirschsprung disease, microvillus inclusion disease, visceral myopathy, and severe gastrointestinal neuropathy). Allograft types are classified by the inclusion of the liver or stomach, with a majority of recipients receiving the isolated small bowel or the small bowel with the liver. In addition to complications associated with immediate postoperative care and immunosuppression, 5-year graft rejection rates requiring retransplantation occur in 35% to 79% of recipients despite improvement in 5-year survival rates from 47% to 56% to 80% to 93%.200–202

173

Experience with vascularized composite allograft (VCA) transplantation continues to advance. These allografts include the face, hand, lower extremities, abdominal wall, penis, and uterus.203–207 A child who received VCA bilateral upper extremity grafts and a baby born to a uterus transplant recipient are recent advancements in this field.207,208 Vascularized composite allografts are included under the definition of an organ in the Organ Procurement Transplantation Network Final Rule.209

Summary Organ donation is a process that begins when a critically ill or injured child is identified as a potential donor with timely referral to the OPO. Identifying and caring for the pediatric organ donor requires a skilled team of specialists who must not only deal with the deceased child but also the family. Early involvement of the OPO allows coordination with the critical care team and family support services, enhancing the chance for the family to understand and accept the death of their child and authorize organ donation. Timely and accurate determination of neurologic death allows the focus of medical management to transition toward care and preservation of organs for transplantation once authorization is obtained. Management of the pediatric organ donor is a natural extension of care for a critically ill or injured child. Meticulous care of the potential donor results in more transplantable organs with improved graft function. Although adults continue to receive the majority of organs recovered from pediatric donors, more organs are being transplanted into pediatric recipients. The option for organ donation should be made available to every family to preserve their autonomy regarding end-of-life decisions. Families should be emotionally supported and approached about donation opportunities in a professional, compassionate manner that allows for open discussion during the most difficult, agonizing time in their lives. The positive benefits of donation extend beyond the transplant recipient. Donation helps families heal as they deal with the loss of their child. Collaboration between pediatric critical care specialists, the critical care team, and other dedicated professionals providing specialized donor management can affect the lives not only of the donor family but also the many potential recipients and their families through the effects of a lifesaving and life-changing transplant. For more information and additional resources about pediatric organ donation, visit the Organ Donation Toolbox at https:// organdonationalliance.org/organ-donation-toolbox/.

Key References Korte C, Garber JL, Descourouez JL et al. Pharmacist guide to management of the organ donors after brain death. Am J Health-Syst Pharm. 2016;73:1829-1839. Kotloff RM, Blosser S, Fulda GJ, et al. Management of the potential organ donor in the ICU: Society of Critical Care Medicine/American College of Chest Physicians/Association of Organ Procurement Organizations Consensus Statement. Crit Care Med. 2015;43(6):1291-1325. Mallory GB Jr, Schecter MG, Elidemir O. Management of the pediatric organ donor to optimize lung donation. Pediatr Pulmonol. 2009; 44(6):536-546. Martin DE, Nakagawa TA, Siebelink MJ, et al Pediatric deceased donation: a report of the Transplantation Society Meeting in Geneva. Transplantation. 2015;99:1403-1409. Nakagawa TA, Ashwal S, Mathur M, et al. Guidelines for the determination of brain death in infants and children: an update of the

174

SECTION III



Pediatric Critical Care: Psychosocial and Societal

1987 Task Force recommendations. Crit Care Med. 2011;39(9): 2139-2155. Nakagawa TA, Mou SS. Management of the pediatric organ donor. In: LaPointe D, Ohler L, Rudow T, et al., eds. The Clinician’s Guide to Donation and Transplantation. Lenexa, KS: North American Transplant Coordinators Organization (NATCO); 2006:839-845. Nakagawa TA, Shemie SD, Dreyden-Palmer K, et al. Organ donation following neurologic and circulatory determination of death. Pediatr Crit Care Med. 2018:19:S26-S32. Sochet AA, Glazier AK, Nakagawa TA. Diagnosis of brain death and organ donation after circulatory death. In Mastropietro CW, Valentine

KM. Pediatric Critical Care. Current Controversies. Switzerland: Springer Nature; 2019:309-321. The President’s Council on Bioethics. Controversies in the Determination of Death: A White Paper by the President’s Council on Bioethics. Washington, DC: 2008. https://bioethicsarchive.georgetown.edu/ pcbe/reports/death/. Wood KE, Becker BN, McCartney JG, et al. Care of the potential organ donor. N Engl J Med. 2004;351(26):2730-2739.

The full reference list for this chapter is available at ExpertConsult.com.

e1































































































1. OPTN: Organ Procurement and Transplantation Network. https:// optn.transplant.hrsa.gov/data/view-data-reports/national-data/. 2. Workman JK, Myrick CW, Meyers RL, et al. Pediatric organ donation and transplantation. Pediatrics. 2013;131(6):e1723-1730. 3. Martin DE, Nakagawa TA, Siebelink MJ, et al Pediatric deceased donation - a report of the Transplantation Society Meeting in Geneva. Transplantation. 2015;99:1403-1409. 4. Committee on Hospital Care, Section on Surgery, and Section on Critical Care. Policy statement—pediatric organ donation and transplantation. Pediatrics. 2010;125(4):822-828. 5. National Conference of Commissioners on Uniform State Laws. Revised Uniform Anatomical Gift Act (2006). https://www.uniformlaws. org/HigherLogic/System/DownloadDocumentFile.ashx? DocumentFileKey51f54ffba-04e6-c32a-c355-52c1c89f990e& forceDialog50. 6. Nakagawa TA, Ashwal S, Mathur M, et al. Guidelines for the determination of brain death in infants and children: an update of the 1987 Task Force recommendations. Crit Care Med. 2011;39(9): 2139-2155. 7. Nakagawa TA, Ashwal S, Mathur M, et al. Clinical report—guidelines for the determination of brain death in infants and children: an update of the 1987 task force recommendations. Pediatrics. 2011;128(3): e720-740. 8. Wijdicks EF, Varelas PN, Gronseth GS, et al. Evidence-based guideline update: determining brain death in adults: report of the Quality Standards Subcommittee of the American Academy of Neurology. Neurology. 2010;74(23):1911-1918. 9. The President’s Council on Bioethics. Controversies in the Determi nation of Death: A White Paper by the President’s Council on Bioethics. Washington, DC: 2008. https://bioethicsarchive.georgetown. edu/pcbe/reports/death/. 10. Boucek MM, Mashburn C, Dunn SM, et al. Pediatric heart transplantation after declaration of cardiocirculatory death. N Engl J Med. 2008;359(7):709-714. 11. White CW, Messer SJ, Large SR, et al. Transplantation of hearts donated after circulatory death. Font Cardiovas Med. 2018;5-8. 12. Burns JP, Seller DE, Meyer EC, Lewis-Newby M, et al. Epidemiology of death in the pediatric intensive care unit at five US teaching hospitals. Crit Care Med. 2014;42:2101-2108. 13. Webster PA, Markham L. Pediatric organ donation: a national survey examining consent rates and characteristics of donor hospitals. Pediatr Crit Care Med. 2009;10(4):500-504. 14. Kotloff RM, Blosser S, Fulda GJ, et al. Management of the potential organ donor in the ICU: Society of Critical Care Medicine/American College of Chest Physicians/Association of Organ Procurement Organizations Consensus Statement. Crit Care Med. 2015;43(6): 1291-1325. 15. Siminoff LA, Gordon N, Hewlett J, et al. Factors influencing families’ consent for donation of solid organs for transplantation. JAMA. 2001;286(1):71-77. 16. Jones AH, Zoelle B, Dizon BA, October TW. Investigation of Public Perception of Brain Death Using the Internet. CHEST. 2018;154: 286-292. 17. Siminoff LA, Mercer MB, Arnold R. Families’ understanding of brain death. Prog Transplant. 2003;13(3):218-224. 18. Nakagawa TA, Shemie SD, Dreyden-Palmer K, et al. Organ donation following neurologic and circulatory determination of death. Pediatr Crit Care Med. 2018:19:S26-S32. 19. Domínguez-Gil B, Murphy P, Procaccio F. Ten changes that could improve organ donation in the intensive care unit. Intensive Care Med. 2016;42:264-267. 20. Shafer TJ, Wagner D, Chessare J, et al. US organ donation breakthrough collaborative increases organ donation. Crit Care Nurs Q. 2008;31:190-210.

21. Siminoff LA, Molisani AJ, Traino. A comparison of the request process and outcomes in adult and pediatric organ donation. Pediatrics. 2015;136(1):e108-114. 22. European Society of Paedatric & Neonatal Intensive Care End of life Standards. https://espnic-online.org/Education/Standards-andGuidelines. 23. Shemie SD, Robertson A, Beitel J, et al. End-of-life conversations with families of potential donors. Transplantation. 2017;101:S17-S26. 24. Rodrigue JR, Cornell DL, Howard RJ. Pediatric organ donation: what factors most influence parents’ donation decisions? Pediatr Crit Care Med. 2008;9(2):180-185. 25. Owens DA. The role of palliative care in organ donation. J Hosp Palliat Nurs. 2006;8(2):75-76. 26. Boss R, Nelson J, Weissman D, et al. Integrating palliative care into the PICU: a report from the Improving Palliative Care in the ICU Advisory Board. Pediatr Crit Care Med. 2014;15(8):762-767. 27. Merchant SJ, Yoshida EM, Lee TK, et al. Exploring the psychological effects of deceased organ donation on the families of the organ donors. Clin Transplant. 2008;22(3):341-347. 28. Kirschen MP, Francoeur C, Murphy M, et al. Epidemiology of brain death in pediatric intensive care units in the United States. JAMA Pediatr. 2019;173:469-476. 29. Shemie SD, Doig C, Dickens B, et al. Severe brain injury to neurological determination of death: Canadian forum recommendations. Epidemioogy of brain death in pediatric intensive care units in the United States. 2006;174(6):S1-13. 30. Mulder M, Gibbs HG, Smith SW, et al. Awakening and withdrawal of life-sustaining treatment in cardiac arrest survivors treated with therapeutic hypothermia. Crit Care Med. 2014;42(12):2493-2499. 31. Gold B, Puertas L, Davis SP, et al. Awakening after cardiac arrest and post resuscitation hypothermia: are we pulling the plug too early? Resuscitation. 2014;85(2):211-214. 32. Grossestreuer AV, Abella BS, Leary M, et al. Time to awakening and neurologic outcome in therapeutic hypothermia-treated cardiac arrest patients. Resuscitation. 2013;84(12):1741-1746. 33. Souter MJ, Blissett PA, Blosser S, et al. Recommendations for the Critical Care Management of Devastating Brain Injury: Prognostication, Psychosocial, and Ethical Management: a Position Statement for Healthcare Professionals from the Neurocritical Care Society. Neurocrit Care. 2015;23(1):4-13. 34. Lewis A, Adams, N, Chopra A, Kirschen MP. Use of ancillary tests when determining brain death in pediatric patients in the United States. J Child Neurol. 2017;32(12):975-980. 35. Stecker M, Sabau D, Das R, et al. American Electrophysiology Society Guideline 6: Minimum technical standards for EEG recording in suspected cerebral death. J Clin Neurophysiol. 2016;33: 324-327. 36. Donohoe KJ, Agrawal G, Frey KA, et al. SNM practice guideline for brain death scintigraphy 2.0. J Nucl Med Technol. 2012;40: 198-203. 37. Mathur M, Petersen L, Stadtler M, et al. Variability in pediatric brain death determination and documentation in southern California. Pediatrics. 2008;121(5):988-993. 38. Ruiz-López MJ, Martínez de Azagra A, Serrano A, et al. Brain death and evoked potentials in pediatric patients. Crit Care Med. 1999; 27(2):412-416. 39. Rodriguez RA, Cornel G, Alghofaili F, et al. Transcranial doppler during suspected brain death in children: potential limitation in patients with cardiac “shunt”. Pediatr Crit Care Med. 2002;3(2):153-157. 40. Riggs BJ, Cohen JS, Shivakumar B, et al. Doppler ultrasonography of the central retinal vessels in children with brain death. Pediatr Crit Care Med. 2017;18(3):258-264. 41. Ruiz-García M, Gonzalez-Astiazarán A, Collado-Corona MA, et al. Brain death in children: clinical, neurophysiological and radioisotopic angiography findings in 125 patients. Childs Nerv Syst. 2000;16(1):40-45.

References

e2













































64. Rosendale JD, Kauffman HM, McBride MA, et al. Aggressive pharmacologic donor management results in more transplanted organs. Transplantation. 2003;75(4):482-487. 65. Rosendale JD, Kauffman HM, McBride MA, et al. Hormonal resuscitation yields more transplanted hearts, with improved early function. Transplantation. 2003;75(8):1336-1341. 66. Salim A, Martin M, Brown C, et al. The effect of a protocol of aggressive donor management: implications for the national organ donor shortage. J Trauma. 2006;61(2):429-433. 67. Salim A, Martin M, Brown C, et al. Using thyroid hormone in brain-dead donors to maximize the number of organs available for transplantation. Clin Transplant. 2007;21(3):405-409. 68. Nakagawa TA, Mou SS. Management of the pediatric organ donor. In: LaPointe D, Ohler L, Rudow T, et al., eds. The Clinician’s Guide to Donation and Transplantation. Lenexa, KS: North American Transplant Coordinators Organization (NATCO); 2006:839-845. 69. Cittanova ML, Leblanc I, Legendre C, et al. Effect of hydroxyethylstarch in brain-dead kidney donors on renal function in kidney-transplant recipients. Lancet. 1996;348(9042):1620-1622. 70. Zarychanski R, Abou-Setta AM, Turgeon AF, et al. Association of hydroxyethyl starch administration with mortality and acute kidney injury in critically ill patients requiring volume resuscitation: a systematic review and meta-analysis. JAMA. 2013;309(7):678-688. 71. Korte C, Garber JL, Descourouez JL, et al. Pharmacist guide to management of the organ donors after brain death. Am J HealthSyst Pharm. 2016;73:1829-1839. 72. Schnuelle P, Berger S, de Boer J, et al. Effects of catecholamine application to brain-dead donors on graft survival in solid organ transplantation. Transplantation. 2001;72(3):455-463. 73. Schnuelle P, Gottmann U, Hoeger S, et al. Effects of donor pretreatment with dopamine on graft function after kidney transplantation: a randomized controlled trial. JAMA. 2009;302(10):10671075. 74. Piantino JA, Lin A, Crowder D. Early heart rate variability and electroencephalographic abnormalities in acutely brain-injured children who progress to brain death. Pediatr Crit Care Med. 2019;20(1):38-46. 74a. Dalle Ave AL, Gardiner D, Shaw DM. Cardio-pulmonary resuscitation of brain dead organ donors: a literature review and suggestions for practice. Transpl Int. 2016;29:12-19. 75. Sochet AA, Glazier AK, Nakagawa TA. Diagnosis of brain death and organ donation after circulatory death. In Mastropietro CW, Valentine KM, eds. Pediatric Critical Care. Current Controversies. Switzerland: Springer Nature;2019:309-321. 76. Bernat JL, Capron AM, Bleck T, et al. The circulatory-respiratory determination of death in organ donation. Crit Care Med. 2010; 38:972-979. 77. Wood KE, Becker BN, McCartney JG, et al. Care of the potential organ donor. N Engl J Med. 2004;351(26):2730-2739. 78. Taniguchi S, Kitamura S, Kawachi K, et al. Effects of hormonal supplements on the maintenance of cardiac function in potential donor patients after cerebral death. Eur J Cardiothorac Surg. 1992;6(2):96-101. 79. Venkateswaran RV, Steeds RP, Quinn DW, et al. The haemodynamic effects of adjunctive hormone therapy in potential heart donors: a prospective randomized double-blind factorially designed controlled trial. Eur Heart J. 2009;30(14):1771-1780. 80. Venkateswaran RV, Dronavalli V, Lambert PA, et al. The proinflammatory environment in potential heart and lung donors: prevalence and impact of donor management and hormonal therapy. Transplantation. 2009;88(4):582-588. 81. Salim A, Vassiliu P, Velmahos GC, et al. The role of thyroid hormone administration in potential organ donors. Arch Surg. 2001;136(12):1377-1380. 82. Zuppa AF, Nadkarni V, Davis L, et al. The effect of a thyroid hormone infusion on vasopressor support in critically ill children with cessation of neurologic function. Crit Care Med. 2004;32(11):23182322.





















































42. Ashwal S. Clinical diagnosis and confirmatory testing of brain death in children. In: Wijdick E, ed. Brain Death. Philadelphia: Lippincott Williams & Wilkins; 2001:91-114. 43. Muralidharan R, Mateen FJ, Shinohara RT, et al. The challenges with brain death determination in adult patients on extracorporeal membrane oxygenation. Neurocrit Care. 2011;14(3):423-426. 44. Frisardi F, Stefanini M, Natoli S, et al. Decompressive craniectomy may cause diagnostic challenges to assess brain death by computed tomography angiography. Minerva Anestesiol. 2014;80:113-118. 45. Gillson N, Weisleder P, Ream MA. A brain death dilemma: apnea testing while on high-frequency oscillatory ventilation. Pediatrics. 2015;135(1):e5-6. 46. Jarrah RJ, Ajizian SJ, Agarwal S, et al. Developing a standard method for apnea testing in the determination of brain death for patients on venoarterial extracorporeal membrane oxygenation: a pediatric case series. Pediatr Crit Care Med. 2014;15(2):e38-43. 47. Harrar DB, Kukreti V, Dean NP, et al. Clinical determination of brain death in children supported by extracorporeal membrane oxygenation. Neurocrit Care. 2019;31(2):304-311. 48. Power BM, Van Heerden PV. The physiological changes associated with brain death—current concepts and implications for treatment of the brain dead organ donor. Anaesth Intensive Care. 1995;23(1): 26-36. 49. Amado JA, López-Espadas F, Vázquez-Barquero A, et al. Blood levels of cytokines in brain-dead patients: relationship with circulating hormones and acute-phase reactants. Metabolism. 1995;44(6): 812-816. 50. Dimopoulou I, Tsagarakis S, Anthi A, et al. High prevalence of decreased cortisol reserve in brain-dead potential organ donors. Crit Care Med. 2003;31(4):1113-1117. 51. Udomphorn Y, Armstead WM, Vavilala MS. Cerebral blood flow and autoregulation after pediatric traumatic brain injury. Pediatr Neurol. 2008;38(4):225-234. 52. Razek T, Olthoff K, Reilly PM. Issues in potential organ donor management. Surg Clin North Am. 2000;80(3):1021-1032. 53. Pérez López S, Otero Hernández J, Vázquez Moreno N, et al. Brain death effects on catecholamine levels and subsequent cardiac damage assessed in organ donors. J Heart Lung Transplant. 2009;28(8): 815-820. 54. Scheinkestel CD, Tuxen DV, Cooper DJ, Butt W. Medical management of the (potential) organ donor. Anaesth Intensive Care. 1995;23(1):51-9. 55. Lutz-Dettinger N, de Jaeger A, Kerremans I. Care of the potential pediatric organ donor. Pediatr Clin North Am. 2001;48(3):715-749. 56. Pratschke J, Tullius SG, Neuhaus P. Brain death associated ischemia/ reperfusion injury. Ann Transplant. 2004;9(1):78-80. 57. Novitzky D. Donor management: state of the art. Transplant Proc. 1997;29(8):3773-3775. 58. Grossman MD, Reilly PM, McMahon DJ, et al. Loss of potential organ donors due to medical failure. Crit Care Med. 1996; 24:A76. 59. Salim A, Velmahos GC, Brown C, et al. Aggressive organ donor management significantly increases the number of organs available for transplantation. J Trauma. 2005;58(5):991-994. 60. Patel MS, Zatarain J, De La Cruz S, et al. The impact of meeting donor management goals on the number of organs transplanted per expanded criteria donor: a prospective study from the UNOS Region 5 Donor Management Goals Workgroup. JAMA Surg. 2014;149(9):969-975. 61. Souter MJ, Eidbo E, Findlay JY, et al. Organ donor management: Part 1. Toward a consensus to guide anesthesia services during donation after brain death. Semin Cardiothorac Vasc Anesth. 2018;22(2): 211-222. 62. Patel MS, De La Cruz S, Sally MB, et al. Active donor management during the hospital phase of care is associated with more organs transplanted per donor. J Am Coll Surg. 2017:335(4):525-531. 63. Ream RS, Armbrecht ES. Survey of U.S. organ procurement organizations regarding pediatric organ donor management. Pediatr Crit Care Med. 2016;17(10):3459-e468.

e3









106.







107.







108.







109.







110.







111.







112.







114. 115.





116.





117.











113.





119.





120.





121.











118.





123.



124.







122.











125.





126.









127.











































83. Mi Z, Novitzky D, Collins JF, Cooper DK. The optimal hormonal replacement modality selection for multiple organ procurement from brain-dead organ donors. Clin Epidemiol. 2014;7:17-27. 84. Turco L, Glorsky S, Winfield R. Hormone replacement therapy in brain-dead organ donors: a comprehensive review with an emphasis on traumatic brain injury. J Trauma Acute Care Surg. 2019;86(4): 702-709. 85. Katz K, Lawler J, Wax J, et al. Vasopressin pressor effects in critically ill children during evaluation for brain death and organ recovery. Resuscitation. 2000;47(1):33-40. 86. Finfer S, Bohn D, Colpitts D, et al. Intensive care management of paediatric organ donors and its effect on post-transplant organ function. Intensive Care Med. 1996;22(12):1424-1432. 87. Goarin JP, Cohen S, Riou B, et al. The effects of triiodothyronine on hemodynamic status and cardiac function in potential heart donors. Anesth Analg. 1996;83(1):41-47. 88. Orlowski JP, Spees EK. Improved cardiac transplant survival with thyroxine treatment of hemodynamically unstable donors: 95.2% graft survival at 6 and 30 months. Transplant Proc. 1993;25(1 Pt 2):1535. 89. Dhar R, Stahlschmidt E, Yan Y, et al. A randomized trial comparing triidothyronine (T3) with thyroxine (T4) for hemodynamically unstable brain-dead organ donors. Clin Transplant. 2019;33(3): e13486. 90. Cooper DK, Novitzky D, Wicomb WN, et al. A review of studies relating to thyroid hormone therapy in brain-dead organ donors. Front Biosci (Landmark Ed). 2009;14:3750-3770. 91. Polikar R, Burger AG, Scherrer U, et al. The thyroid and the heart. Circulation. 1993;87(5):1435-1441. 92. Ririe DG, Butterworth JF IV, Royster RL, et al. Triiodothyronine increases contractility independent of beta-adrenergic receptors or stimulation of cyclic-3’,5’-adenosine monophosphate. Anesthesiology. 1995;82(4):1004-1012. 93. Novitzky D, Cooper DK, Morrell D, et al. Change from aerobic to anaerobic metabolism after brain death, and reversal following triiodothyronine therapy. Transplantation. 1988;45(1):32-36. 94. Marik PE, Zaloga GP. Adrenal insufficiency in the critically ill: a new look at an old problem. Chest. 2002;122(5):1784-1796. 95. Ullian ME. The role of corticosteriods in the regulation of vascular tone. Cardiovasc Res. 1999;41(1):55-64. 96. Follette DM, Rudich SM, Babcock WD. Improved oxygenation and increased lung donor recovery with high-dose steroid administration after brain death. J Heart Lung Transplant. 1998;17:423-429. 97. Mallory Jr GB, Schecter MG, Elidemir O. Management of the pediatric organ donor to optimize lung donation. Pediatr Pulmonol. 2009;44(6):536-546. 98. Venkateswaran RV, Patchell VB, Wilson IC, et al. Early donor management increases the retrieval rate of lungs for transplantation. Ann Thorac Surg. 2008;85(1):278-286. 99. Deleted in review. 100. Salim A, Martin M, Brown C, et al. Complications of brain death: frequency and impact on organ retrieval. Am Surg. 2006;72(5): 377-381. 101. Mascia L, Bosma K, Pasero D, et al. Ventilatory and hemodynamic management of potential organ donors: an observational survey. Crit Care Med. 2006;34(2):321-327. 102. Cooper DK, Novitzky D, Wicomb WN. The pathophysiological effects of brain death on potential donor organs, with particular reference to the heart. Ann R Coll Surg Engl. 1989;71(4):261-266. 103. López-Aguilar J, Villagrá A, Bernabé F, et al. Massive brain injury enhances lung damage in an isolated lung model of ventilatorinduced lung injury. Crit Care Med. 2005;33(5):1077-1083. 104. Angel LF, Levine DJ, Restrepo MI, et al. Impact of a lung transplantation donor-management protocol on lung donation and recipient outcomes. Am J Respir Crit Care Med. 2006;174(6): 710-716. 105. Straznicka M, Follette DM, Eisner MD, et al. Aggressive management of lung donors classified as unacceptable: excellent recipient survival one

year after transplantation. J Thorac Cardiovasc Surg. 2002;124(2): 250-258. Rosengard BR, Feng S, Alfrey EJ, et al. Report of the Crystal City meeting to maximize the use of organs recovered from the cadaver donor. Am J Transplant. 2002;2(8):701-711. Ware LB, Landeck M, Koyama T, et al. A randomized trial of the effects of nebulized albuterol on pulmonary edema in brain-dead organ donors. Am J Transplant. 2014;14(3):621-628. Eagan C, Keller CA, Baz MA, et al. Effects of administration of intravenous naloxone on gas exchange in brain-dead lung donors. Prog Transplant. 2009;19(3):267-271. Dhar R, Stahlschmidt E, Paramesh A, et al. A randomized controlled trial of naloxone for optimization of hypoxemia in lung donors after brain death. Transplantation. 2018;103(7):1433-1438. Mascia L, Pasero D, Slutsky AS, et al. Effect of a lung protective strategy for organ donors on eligibility and availability of lungs for transplantation: a randomized controlled trial. JAMA. 2010;304: 2620-2627. Totsuka E, Fung U, Hakamada K, et al. Analysis of clinical variables of donors and recipients with respect to short-term graft outcome in human liver transplantation. Transplant Proc. 2004; 36(8):2215-2218. Kaseje N, McLin V, Toso C, et al. Donor hypernatremia before procurement and early outcomes following pediatric liver trans plantation. Liver Transpl. 2015;21(8):1076-1081. Takemoto CK, Hodding JH, Kraus DM. Pediatric Dosage Handbook. 12th ed. Hudson, OH: Lexi-Comp; 2008:1772-1774. Deleted in review. Agersø H, Seiding Larsen L, Riis A, et al. Pharmacokinetics and renal excretion of desmopressin after intravenous administration to healthy subjects and renally impaired patients. Br J Clin Pharmacol. 2004;58(4):352-358. Köhler M, Harris A. Pharmacokinetics and haematological effects of desmopressin. Eur J Clin Pharmacol. 1988;35(3):281-285. Takemoto CK, Hodding JH, Kraus DM. Pediatric Dosage Handbook. 12th ed. Hudson, OH: Lexi-Comp; 2008:512-514. Murphy MB, Murray C, Shorten GD. Fenoldopam: a selective peripheral dopamine-receptor agonist for the treatment of severe hypertension. N Engl J Med. 2001;345(21):1548-1557. Kearney TJ, Bentt L, Grode M, et al. Coagulopathy and catecholamines in severe head injury. J Trauma. 1992;32(5): 608-611. Barron ME, Wilkes MM, Navickis RJ. A systematic review of the comparative safety of colloids. Arch Surg. 2004;139(5):552-563. Ansell JE. Acquired bleeding disorders. In: Rippe JM, Alper JS, eds. Intensive Care Medicine. 2nd ed. Boston: Little, Brown & Co.; 1991:1013-1023. Danzl DF, Pozos RS. Accidental hypothermia. N Engl J Med. 1994;331(26):1756-1760. Graham M. The role of the medical examiner in fatal child abuse. In: Monteleone JA, Brodeur AE, eds. Child Maltreatment: A Clinical Guide and Reference. 2nd ed. St. Louis, MO: G.W. Medical Publishing, Inc.; 1998:531-558. Shafer TJ, Schkade LL, Evans RW, et al. Vital role of medical examiners and coroners in organ transplantation. Am J Transplant. 2004;4(2):160-168. Goldstein B, Shafer T, Greer D, et al. Medical examiner/coroner denial for organ donation in brain-dead victims of child abuse: controversies and solutions. Clin Intensive Care. 1997;8(3): 136-141. Shafer TJ, Schkade LL, Siminoff LA, et al. Ethical analysis of organ recovery denials by medical examiners, coroners, and justices of the peace. J Transpl Coord. 1999;9(4):232-249. Wolf DA, Derrick SM, Wood RP. Preservation of evidence during pediatric organ donation: a modified thoracotomy procedure designed to increase consent in medical examiner cases. Prog Transplant. 2011;21(1):67-70.

e4















































149. Saidi RF, Elias N, Kawai T, et al. Outcome of kidney transplantation using expanded criteria donors and donation after cardiac death kidneys: realities and costs. Am J Transplant. 2007;7(12):27692774. 150. Troppman D, Santhanakrishnan C, Fananapazir G, et al. Pediatric en bloc kidney transplantation from very small (,10 kg) donation after circulatory death (versus brain death) donors: single center matched-pair analysis of 130 transplants. Am J Transplant. 2018; 18(11):2811-2817. 151. Hwang CS Levea SL, Parekh JR, et al. Should more donation after cardiac death livers be used in pediatric transplantation? Pediatr Transplant. 2019;23(1):e13323. 152. Maheshwari A, Maley W, Li Z, et al. Biliary complications and outcomes of liver transplantation from donors after cardiac death. Liver Transpl. 2007;13(12):1645-1653. 153. Vivalda S, Zhengbin H, Xiong Y, et al. Vascular and biliary complications following deceased donor liver transplantation: a meta-analysis. Transplant Proc. 2019;51(3):823-832. 154. Hong JC, Venick R, Yersiz H, et al. Liver transplantation in children using organ donation after circulatory death: a casecontrol outcomes analysis of a 20-year experience in a single center. JAMA Surg. 2014;149(1):77-82. 155. Oto T. Lung transplantation from donation after cardiac death (non-heart-beating) donors. Gen Thorac Cardiovasc Surg. 2008; 56(11):533-538. 156. Krutsinger D, Reed RM, Blevins A, et al. Lung transplantation from donation after cardiocirculatory death: a systematic review and meta-analysis. J Heart Lung Transplant. 2015;34(5): 675-684. 157. van Suylen V, Luijk B, Hoek RAS, et al. A multicenter study on long-term outcomes after lung transplantation comparing donation after circulatory death and donation after brain death. Am J Transplant. 2017;17(10):2679-2686. 158. Sweet SC. Pediatric Lung Transplantation. Resp Care. 2017;62(6): 776-798. 159. Labrecque M, Parad R, Gupta M, et al. Donation after cardiac death: the potential contribution of an infant organ donor population. J Pediatr. 2011;158(1):31-36. 160. Mathur M, Castleberry D, Job L. Identifying potential heart donors among newborns undergoing circulatory determination of death. J Heart Lung Transplant. 2011;30(4):389-394. 161. Stiers J, Aguayo C, Siatta A, et al. Potential and actual neonatal organ and tissue donation after circulatory determination of death. JAMA Pediatr. 2015;169(7);639-645. 162. Hanley H, Kim S, Willey E, et al. Identifying potential kidney donors among newborns undergoing circulatory determination of death. Pediatrics. 2014;133(1):e82-87. 163. Perez R, Santhanakrishnan C, Demattos A, et al. The neonatal intensive care unit (NICU) as a source of deceased donor kidneys for transplantation: initial experience with 20 cases (abstract #2289). Am J Transplant. 2014;14(suppl 3):134. 164. Bhayana S, Kuo YF, Madan P, et al. Pediatric en bloc kidney transplantation to adult recipients: more than suboptimal? Transplantation. 2010;90(3):248-254. 165. Lau KK, Berg GM, Schjoneman YG, Perez RV, Butani L. Pediatric en bloc kidney transplantation into pediatric recipients. Pediatr Transplant. 2010;14(1):100-104. 166. Chen CY, Warner P, Albers EL, et al. Donor-specific anti-HLA antibody production following pediatric ABO-incompatible heart transplantation. Pediatr Transplant. 2019;23(2):e13332. 167. Joffe AR, Carcillo J, Anton N, et al. Donation after cardiocirculatory death: a call for a moratorium pending full public disclosure and fully informed consent. Philos Ethics Humanit Med .2011;6:17. 168. Nakagawa TA, Rigby MR, Bratton S, et al. A call for full public disclosure for donation after circulatory determination of death in children. Pediatr Crit Care Med. 2011;12(3):375-377.













































128. Twamley H, Haigh A, Williment C, Hudson C, Whitney J, Neuberger J. Prospective audit to evaluate the potential of the coronial system to increase solid organ donation. BMJ Open. 2016;6(7): e010231. 129. Meert KL, Keele L, Morrison W, et al. Eunice kennedy shriver National Institute of Child Health and Human Development Collaborative Pediatric Critical Care Research Network. End-of-life practices among tertiary care PICUs in the United States: a multicenter study. Pediatr Crit Care Med. 2015;16(7):e231-238. 130. Pinckard JK, Geiselhart RJ, Moffatt E, et al. National Association of Medical Examiners position paper: medical examiner release of organs and tissue for transplantation. Acad Forensic Pathol. 2013; 4:497-504. 131. Pana R, Hornby L, Shemie SD, et al. Time to loss of brain function and activity during circulatory arrest. J Crit Care. 2016;34:77-83. 132. Rady MY, Verheijde JL. Neuroscience and awareness in the dying human brain: implications for organ donation. J Crit Care. 2016; 34:121-123. 133. Norton L, Gibson RM, Gofton T, et al. Electroencephalographic recording during withdrawal of life-sustaining therapy until 30 minutes after declaration of death. Can J Neurol Sci. 201744: 139-145. 134. Shore PM, Huang R, Roy L, et al. Development of a bedside tool to predict time to death after withdrawal of life-sustaining therapies in infants and children. Pediatr Crit Care Med. 2012;13(4):415-422. 135. Rabinstein AA, Yee AH, Mandrekar J, et al. Prediction of potential for organ donation after cardiac death in patients in neurocritical state: a prospective observational study. Lancet Neurol. 2012;11(5): 414-419. 136. Lewis J, Peltier J, Nelson H, et al. Development of the University of Wisconsin donation after cardiac death evaluation tool. Prog Transplant. 2003;13(4):265-273. 137. Mazor R, Baden HP. Trends in pediatric organ donation after cardiac death. Pediatrics. 2007;120(4):e960-6. 138. Deleted in review. 139. Morrissey PE, Monaco AP. Donation after circulatory death: current practices, ongoing challenges, and potential improvements. Transplantation. 2014;97(3):258-264. 140. Durall AL, Laussen PC, Randolph AG. Potential for donation after cardiac death in a children’s hospital. Pediatrics. 2007;119(1): e219-224. 141. Pleacher KM, Roach ES, Van der Werf W, et al. Impact of a pediatric donation after cardiac death program. Pediatr Crit Care Med. 2009;10(2):166-170. 142. Naim MY, Hoehn KS, Hasz RD, et al. The Children’s Hospital of Philadelphia’s experience with donation after cardiac death. Crit Care Med. 2008;36(6):1729-33. 143. Bennet EE, Sweney J, Aguayo C, et al. Pediatric organ donation potential at a children’s hospital. Pediatr Crit Care Med. 2015; 16(9):814-820. 144. Weber M, Dindo D, Demartines N, et al. Kidney transplantation from donors without a heartbeat. N Engl J Med. 2002;347(4):248255. 145. Zhu D, McCague K, Lin W, et al. Outcome of kidney transplantation from donor after cardiac death: Reanalysis of the US mycophenolic renal transplant registry. Transplant Proc. 2018;50(5):1258-1263. 146. Fridell JA, Mangus RS, Thomas CM, et al. Donation after circulatory arrest in pancreas transplantation: A report of 10 cases. Transplant Proc. 2017:49(10):2310-2314. 147. Summers DM, Watson CJ, Pettigrew GJ, et al. Kidney donation after circulatory death (DCD): state of the art. Kidney Int. 2015; 88(2):241-249. 148. Pitarch Martinez M, Sanchez Perez B, Leon Diaz FJ, et al. Donation after cardiac death in liver transplantation: an additional source of organs with similar results to donation after brain death. Transplan Proc. 2019;51(1):4-8.

e5

















































187. Beuth J, Falter F, Pinto Ribeiro RV, et al. New strategies to expand and optimize heart donor pool: Ex vivo heart perfusion and donation after circulatory death: a review of current research and future trends. Anesth Analg. 2019;123;406-413. 188. Miñambres E, Suberviola B, Dominguez-Gil B, et al. Improving the Outcomes of Organs Obtained From Controlled Donation After Circulatory Death Donors Using Abdominal Normothermic Regional Perfusion. Am J Transplant. 2017;17(8):2165-2172. 189. De Carlis L, De Carlis R, Lauterio A, Di Sandro S, Ferla F, Zanierato M. Sequential use of normothermic regional perfusion and hypothermic machine perfusion in donation after cardiac death liver transplantation with extended warm ischemia time. Transplantation. 2016;100(10):e101-2. 190. Andreasson AS, Dark JH, Fisher AJ. Ex vivo lung perfusion in clinical lung transplantation—state of the art. Eur J Cardiothorac Surg. 2014;46(5):779-88. 191. De Carlis R, Di Sandro S, Lauterio A, Ferla F, Dell’Acqua A, Zanierato M, De Carlis L. Successful donation after cardiac death liver transplants with prolonged warm ischemia time using normothermic regional perfusion. Liver Transpl. 2017;23(2):166-173. 192. Ardehali A, Esmailian F, Deng M, et al. Ex-vivo perfusion of donor hearts for human heart transplantation (PROCEED II): a prospective, open-label, multicentre, randomised non-inferiority trial. Lancet. 2015;325:2577-2584. 193. Yeung JC, Cypel M, Keshavjee S. Ex-vivo lung perfusion: the model for the organ reconditioning hub. Curren Opin Organ Transplant. 2017;22:287-289. 194. D’Cunha HC, Rojas M. Ex vivo lung perfusion: Past, present, and future. ASAIO J. 2018;64(2):135-139. 195. Akagawa TA, Zollinger C, Chao J, Hill R, Angle S, Pilot M. Anencephalic infants as organ donors. Transplantation. 2017.101;8(suppl 2):S60. 196. John M, Bailey LL. Neonatal heart transplantation. Ann Cardiothorac Surg. 2018;7:118-125. 197. Urschel S. West LJ. ABO-incompatible heart transplantation. Curr Opin Pediatr. 2016;28(5):613-619. 198. Meyburg J, Opladen T, Spiekerkötter U, et al. Human heterologous liver cells transiently improve hyperammonemia and ureagenesis in individuals with severe urea cycle disorders. J Inherit Metab Dis. 2018;41(1):81-89. 199. Smets F, Dobbelaere D, McKiernan P, et al. Phase I/II trial of liver derived mesenchymal stem cells in pediatric liver based metabolic disorders: a prospective, open label, multicenter, partially randomized, safety study of one cycle of heterologous human adult liver-derived progenitor cells (HepaStem®) in urea cycle disorders and crigler-najjar syndrome patients. Transplantation. 2019;103(9): 1903-1915. 200. Grant D, Abu-Elmagd K, Mazariegos G, et al. Intestinal transplant registry report: global activity and trends. Am J Transpl. 2015; 15:210-219. 201. Abu-Elmagd KM, Kosmach-Park B, Costa G, et al. Long-term survival, nutritional anatomy, and quality of life after intestinal and multivisceral transplantation. Ann Surg. 2012;256(3):494-508. 202. Abu-Elmagd KM, Costa G, Bond GJ, et al. Five hundred intestinal and multivisceral transplantations at a single center: major advances with new challenges. Ann Surg. 2009;250(4)56-581. 203. Wainright JL, Wholley CL, Rosendale J, et al. VCA deceased donors in the United States. Transplantation. 2018;103(5):990997. 204. McDiarmid SV. Vascularized composite allotransplantation in children: what we can learn from sold organ transplant. Curr Opin Organ Transplant. 2018;23:605-614. 205. Westvik TS, Dermietzel A, Pomahac B. Facial restoration by transplantation: the Brigham and Women’s face transplant experience. Ann Plast Surg. 2015;74(suppl 1):S2-8. 206. Sosin M, Rodriquez ED. The face transplantation update: 2016. Plast Reconstr Surg. 2016;137:1841-1850.













































169. Carcillo JA, Orr R, Bell M, et al. A call for full public disclosure and moratorium on donation after cardiac death in children. Pediatr Crit Care Med. 2010;11(5):641-643. 170. Truog RD, Miller FG. Counterpoint: are donors after circulatory death really dead, and does it matter? No and not really. Chest. 2010;138(1):16-18. 171. Halpern SD, Truog RD. Organ donors after circulatory determination of death: not necessarily dead, and it does not necessarily matter. Crit Care Med. 2010;38(3):1011-1012. 172. Institute of Medicine. Non-Heart-Beating Organ Transplantation: Medical and Ethical Issues in Procurement. Washington, DC: National Academy Press; 1997. https://www.nap.edu/read/6036/ chapter/1. 173. Institute of Medicine. Non-Heart-Beating Organ Transplantation: Practice and Protocols. https://www.nap.edu/read/9700/ chapter/1. 174. American Medical Association. Opinion 2.157 - Organ Donation After Cardiac Death. https://journalofethics.ama-assn.org/sites/ journalofethics.ama-assn.org/files/2018-05/coet1-1602_0.pdf. 175. Ethics Committee, American College of Critical Care Medicine; Society of Critical Care Medicine. Recommendations for nonheartbeating organ donation. A position paper by the Ethics Committee, American College of Critical Care Medicine, Society of Critical Care Medicine. Crit Care Med. 2001;29(9): 1826-1831. 176. Gries CJ, White DB, Truog RD, et al. An official American Thoracic Society/International Society for Heart and Lung Transplantation/Society of Critical Care Medicine/Association of Organ and Procurement Organizations/United Network of Organ Sharing Statement: ethical and policy considerations in organ donation after circulatory determination of death. Am J Respir Crit Care Med. 2013;188(1):103-109. 177. Weiss MJ, Hornby L, Rochwerg B, et al., and the members of the pDCD Guideline Development Working Groups. Canadian Guidelines for Controlled Pediatric Donation after Circulatory Determination of Death. Pediatric Critical Care Medicine. 2017;18:1035-1046. 178. Committee on Bioethics. Ethical controversies in organ donation after circulatory death. Pediatrics. 2013;131(5):1021-1026. 179. Curley MA, Harrison CH, Craig N, et al. Pediatric staff perspectives on organ donation after cardiac death in children. Pediatr Crit Care Med. 2007;8(3):212-219. 180. Mathur M, Taylor S, Tiras K, et al. Pediatric critical care nurses’ perceptions, knowledge, and attitudes regarding organ donation after cardiac death. Pediatr Crit Care Med. 2008; 9(3):261-269. 181. McLean RC, Reese PP, Acker M, et al. Transplanting hepatitis C virus-infected hearts into uninfected recipients: a single-arm trial. Am J Transplant. 2019;19:2533-2542. 182. Cotter TG, Paul S, Sandikci B, et al. Increasing utilization and excellent initial outcomes following liver transplant of hepatitis C virus (HCV)-viremic donors into HCV-negative recipients: outcomes following liver transplant of HCV-viremic donors. Hepatology. 2019;69(6):2381-2395. 183. Sise ME, Chute DF, Gustafson JL, et al. Transplantation of hepatitis C virus infected kidneys into hepatitis C virus uninfected recipients. Hemodial Int. 2018;22(suppl1):S71-S80. 184. Woolley AE, Singh SK, Goldberg JH, et al. Heart and lung transplants from HCV-infected donors to uninfected recipients. N Engl J Med. 2019;380:1606-1617. 185. https://www.nebcnew.com/health/health-news/us-begins-organtransplants-living-donors-who-have-hiv-n988386. 186. Botha J, Conradie F, Etheredge H, et al. Living donor liver transplant from an HIV-positive mother to her HIV-negative child: opening up new therapeutic options. AIDS. 2018;32(16): F13-F19.

e6





209. Guidance for Organ Procurement Programs (OPOs) for VCA Deceased Donor Authorization. https://optn.transplant.hrsa.gov/resources/ guidance/opo-guidance-on-vca-deceased-donor-authorization/





207. Shores JT, Brandacher G, Lee WP. Hand and upper extremity transplantation: an update of outcomes in the worldwide experience. Plast Reconstr Surg. 2015;135(2):351e-360e. 208. Maurer MM, Sauer IM, Pratschke J, et al. First healthy baby after deceased donor uterus transplantation: birth to a new era? Transplantation. 2019;103:652-653.

e7

Abstract: Organ transplantation is the accepted therapy for endstage organ failure. However, the national waitlist continues to increase because of a shortage of organ donors. Organ donation is a process that includes (1) identification of a potential donor, (2) timely determination of neurologic death, (3) authorization for donation, (4) perioperative donor management, and (5) recovery of organs for transplantation. Successful organ recovery requires a collaborative approach involving the critical care team,

organ procurement organization, and other medical specialists. Evolving areas in transplantation include vascularized composite allografts and use of donor organs from neonates, human immunodeficiency virus, and hepatitis-positive donors. Key Words: pediatric, organ donation, brain death, circulatory death, transplantation

21 Long-Term Outcomes Following Critical Illness in Children ELIZABETH Y. KILLIEN, JERRY J. ZIMMERMAN, FRANÇOIS ASPESBERRO, AND R. SCOTT WATSON









Mortality reduction represented the first frontier for critical care medicine and historically has been the most commonly used outcome measure in interventional trials enrolling critically ill patients. Fortunately, as the subspecialty of pediatric critical care has matured, overall mortality rates for critically ill children have substantially declined to current rates of 2% to 3% of all pediatric intensive care unit (PICU) admissions.1–3 Many children surviving critical illness, however, struggle to regain their prehospitalization health status, experiencing persistent deficits in functional status, health-related quality of life (HRQL), cognitive and school performance, and mental health. They also experience high rates of hospital readmission, ongoing healthcare use, and late mortality. Patients’ families also experience emotional, financial, and social strains, which, in turn, can make it more difficult for them to support the recovery of their children. As increasing numbers of children are now surviving severe illnesses, pediatric critical care may be exchanging mortality for enduring morbidity.4,5 The next frontier for pediatric critical care medicine is to characterize the scope of long-term postdischarge morbidity, better identify the antecedents of morbidity and the survivors most at risk, and define potential targets for intervention to help optimize recovery from pediatric critical illness.

Post–Intensive Care Syndrome Children surviving critical illness and intensive care are vulnerable to ongoing problems in all domains of life (Table 21.1). Physical, emotional, cognitive, social, and family functioning may all be







Development of specialized pediatric intensive care has contributed to substantially reduced mortality for critically ill children. Research has identified physical, cognitive, health-related quality of life, and mental health domains as areas of persistent impairment among children surviving critical illness. Postintensive care syndrome is the development of new or worsening impairments in physical, cognitive, or mental health







PEARLS arising after critical illness and persisting beyond acute care hospitalization. Morbidity measures are increasingly being incorporated as primary and secondary endpoints in pediatric critical care interventional trials. There is an urgent need for additional research to better characterize postintensive care morbidity and its risk factors, with the goal of minimizing adverse sequelae associated with critical illness.

affected to the detriment of overall HRQL and a family’s socioeconomic status. In adult survivors of critical illness, an increasing appreciation for new morbidity persisting after discharge in multiple domains led to the development of the concept of post– intensive care syndrome (PICS) and post–intensive care syndrome family (PICS-F).6 More recently, this concept has been applied to children as pediatric PICS (PICS-p), with the notable difference from the PICS concept in adults being that childhood development and family are integral to a child’s recovery from illness (Fig. 21.1).7,8

Health-Related Quality of Life HRQL is increasingly being used as a comprehensive measure of health outcomes9 and has been identified by both families and healthcare professionals as the most important outcome to assess among PICU survivors.10 Quality of life is defined as an individual’s perception of his or her position in life in relation to the individual’s goals, expectations, standards, and concerns.11,12 HRQL is defined as quality of life in which a dimension of personal judgment over one’s health and disease is added13,14; it encompasses the impact of health status on physical, mental, emotional, and social functioning.15,16 HRQL in children is influenced by factors such as the ability to participate in peer groups, keep up with developmentally appropriate activities, and succeed in school. It provides a broad view of child health, encompassing aspects of perceived health, health behavior, and well-being.17–19 175

176

SECTION III



Pediatric Critical Care: Psychosocial and Societal

TABLE Domains of Health Outcomes and Examples of 21.1 Morbidity Following Pediatric ICU Discharge

Domain

Morbidity examples

Health-related quality of life

Unable to participate in activities Difficulty with self-care Low energy level Worrying or feeling sad Difficulty getting along with peers Memory or attention deficits Difficulty with schoolwork or missing school

Functional/physical

Cognitive

Mental health

Family

Social

Economic

Baseline status

Pediatric intensive care experience

Motor dysfunction Impaired level of consciousness Feeding or respiratory support Hearing or vision deficits Neuromuscular weakness ICU-associated polyneuropathy Pain

Physical health

Executive function Processing speed Memory Attention Academic performance

ICU, Intensive care unit.

Baseline HRQL is determined by genetics20; parent, family, and home characteristics21; and chronic, comorbid conditions (Fig. 21.2).22,23 In assessing HRQL among a generalized sample of US children, lower HRQL was noted for children in lower socioeconomic status groups, those with healthcare access barriers, adolescents compared with children, and individuals with chronic medical conditions.19 These same variables likely impact HRQL recovery following critical illness in addition to the effects of acute illness and associated treatments. Individual characteristics influencing HRQL include personality traits, chronic comorbid conditions, and genetics; environmental characteristics include parental stress, family dynamics, and home demographics; and clinical variables specific to critical illness include the intensity and duration of organ system dysfunction and exposure to ICU therapies.24 HRQL instruments should provide reliable and valid measures that can quickly and easily be used to quantify morbidity or disability

Emotional health

Social health

Trajectory of recovery

Family stress Mental health of family members Relationship conflict and divorce Disruption of siblings’ routines

Costs of acute care Loss of income due to missed work or job loss Ongoing care needs

Cognitive health

• Family • Parents • Siblings

Developmental impact

Anxiety Depression Acute stress disorder Posttraumatic stress disorder Disturbed sleep patterns and nightmares

Difficulty interacting with peers Missed school Family isolation

Child

Days to decades

• Fig.

21.1 ​Post–intensive care syndrome in pediatrics (PICS-p). The PICS-p conceptual framework incorporates the child’s baseline functioning, physical and psychosocial development, interdependence of family, integral aspects of social health, and potential trajectories of lifetime recovery. (From Manning JC, Pinto NP, Rennick JE, Colville G, and Curley MAQ. Conceptualizing postintensive care syndrome in children—The PICS-p framework. Pediatr Crit Care Med. 2018;19[4]:239–300.)  



after a child’s critical illness or injury. These tools must be multidimensional and include physical, mental, and social health domains. Preferably, HRQL questionnaires should be completed by the critically ill children if they are 6 to 8 years of age and older. Parent-proxy reporting is often necessary in a critically ill population, however, and is also a valid approach.25 eTable 21.2 provides a list of HRQL measures that have been used in pediatric critical care studies and summarizes important psychometric properties of these tools.9 The most comprehensive instruments for assessing HRQL that are currently available are the PedsQL 4.0 Generic Core Scales, KIDSCREEN-27, the 28-item Child Health Questionnaire parent form (CHQ-PF28), and KINDL. These tools are offered as both self-reports and proxy reports, cover a wide age range of children, are brief with a low response burden, are multidimensional, have been shown to have internal consistency and test-retest reliability, demonstrate sensitivity to change over time, and have content and construct validity.



CHAPTER 21

Long-Term Outcomes Following Critical Illness in Children

Characteristics of the Individual

Biological Variables • Genetics • Chronic Illness

Characteristics of the Environment and Family

Characteristics of Healthcare • Treatment • System (Access) • Organization (Staffing)

Critical Illness

Acute Insult • Pneumonia • Traumatic brain injury

177

Course of illness Organ function HRQL ICU Insult • Side effects of standard care • Iatrogenic complications Growth and development

• Fig. 21.2  



​Components that contribute to post–pediatric intensive care unit health-related quality of life (HRQL). HRQL after pediatric critical illness is affected by multiple factors. In addition to a child’s psychologic, biological, and environmental characteristics, characteristics of the healthcare system and a child’s capacity for growth and development interact with the acute illness or injury and its evolution in a multidirectional manner. For example, chronic illness can affect HRQL and a child’s family/environment, which, in turn, can influence the risk of subsequent episodes of critical illness. (From Watson RS, Crow SS, Hartman ME, Lacroix J, Odetola FO. Epidemiology and outcomes of pediatric multiple organ dysfunction syndrome [MODS]. Pediatr Crit Care Med. 2017;18[3 Suppl 1]:S4–S16.)

Assessing Change From Baseline A common method of evaluating HRQL, functional status, and other outcomes is to compare patient scores to published population norms for the instrument. There are limitations to this approach, however, as children’s baseline status may be either above or below the population mean. Thus, a significant change in their health status from their own individual baseline may not be reflected in a comparison to population norms. Therefore, the impact of critical illness will likely be underestimated for patients whose baseline status was above the population mean and experienced declines following their hospitalization and may be overestimated for patients whose baseline was below the population mean but did not decline further (Fig. 21.3).26 In one study of HRQL outcomes among children with sepsis, only 69% of patients who had a clinically significant decline from their individual baseline HRQL score at the time of follow-up would have been identified as being below the population mean, while 34% of the patients who were significantly below the population mean at follow-up were not significantly below their individual baseline score.27 Additionally, the general population of children sampled to determine population norms may not be representative of the typical population of children experiencing critical illness. Over half of the children admitted to US PICUs have complex chronic health conditions.28,29 These children experience higher illness

severity, longer lengths of stay, higher inpatient mortality, and high frequency of morbidity following illness.22,28,30–37 One population-based sample of PICU patients found that 6 years after an ICU admission, nearly 95% of patients with low HRQL at follow-up had chronic conditions—most commonly, neurologic diseases such as cerebral palsy and epilepsy, chromosomal abnormalities, and malignancies.38 Comparing their HRQL to the general pediatric population may not adequately reflect the trajectory of their HRQL after hospitalization, as their baseline HRQL scores may be substantially lower than the population mean. Children with cerebral palsy, for example, have a mean baseline PedsQL score of 51.3 points,39 two standard deviations below the population mean.40 There is also an important subset of children requiring ICU care who experience improvement in health status after a hospitalization. Patients with congenital heart disease and those who receive solid-organ transplants have higher rates of functional outcomes improvement than their healthy counterparts, most likely due to the resolution of chronic organ dysfunction following operative cardiac repair or organ transplantation. Nearly half of children with elective PICU admissions experience HRQL improvement after discharge.41 Some children dependent on respiratory or feeding technology at baseline also experience improvement in functional status following an ICU stay.42 Many

177.e1

eTABLE Health-Related Quality of Life (HRQL) Assessment Tools 21.2

Time (min)

Report

Reliability

Validity

Physical

Emotional

Social/ Behavioral

School

6

45

Self

ICC Test-retest

Content Construct Criterion

x

x

x

x

76 (proxy) 45 (self)

5

20

Proxy Self

ICC Test-retest

Content Construct Criterion

x

x

x

x

10–18

87

13

20

Proxy Self (101)

ICC Test-retest

Content Construct Criterion

x

x

x

x

x

Child Health Questionnaire— Parent Form (US)

5–18

50

13

20

Proxy Self (51)

ICC Test-retest

Content Construct Criterion

x

x

x

x

x

CHQ-PF28

Child Health Questionnaire— Parent Form (US)

5–18

28

13

5–10

Proxy Self (51)

ICC Test-retest

Content Construct Criterion

x

x

x

x

x

COOP

Dartmouth COOP Charts (US)

8–18

6

6

DISABKIDS

DISABKIDS (Europe)

4–16

37

6

Self

Test-retest

Construct

x

x

x

x

Proxy Self

ICC Test-retest

Content Construct

x

x

x

EQ–5D

EuroQOL (Europe)

5–18

5

Proxy Self

Test-retest

Construct

x

x

FIM/WEEFIM

Functional Independence Measure

0–18

18

Proxy

Test-retest

Content Construct

x

FS IIR

Functional Status II Revised (US)

0–16

14

8

10

Proxy

ICC Test-retest

Content Construct

x

x

HUI 3

Health Utilities Index 3 (Canada)

2–18

45

8

,5

Proxy Self

Test-retest

Construct

x

x

ITQOL

Infant & Toddler QOL Questionnaire

2 mo– 5y

103

10

Proxy

ICC Test-retest

Content Construct Criterion

x

x

Instrument

Name/Origin

Age (y)

No. Items

CHIP-AE

Child Health & Illness Profile—Adolescent Edition (US)

11–17

183

CHIP-CE

Child Health & Illness Profile—Child Edition (US)

6–11

CHQ-CF87

Child Health Questionnaire— Child Form (US)

CHQ-PF50

No. Domains

10

Sensitivity to Change

x x x

x

KIDSCR-52

Kidscreen (International)

8–18

52

10

15–20

Proxy Self

ICC

Content Construct

x

x

x

x

x

KIDSCR-27

Kidscreen (International)

8–18

27

5

10–15

Proxy Self

ICC Test-retest

Content Construct

x

x

x

x

x

KINDL

Kidscreen (Germany)

8–16 4–7

24 12

6 4

5–10

Proxy Self

ICC

Content Construct

x

x

x

x

x

PEDI

Pediatric Evaluation & Disability Inventory

0–8

237

Proxy

ICC Test-retest

Content Construct Criterion

x

x

PedsQL 4.0

Pediatric Quality of Life Inventory (US)

2–18

23

Proxy Self (51)

ICC Test-retest

Content Construct Predictive

x

x

QWD

Quality of Well-Being Scale

4–18

29

Proxy Self (111)

Test-retest

Construct

x

TACQOL

TNO-AZL Child QOL (Netherlands)

6–15

108

7

10

Proxy Self (81)

ICC

Content Construct

x

x

x

TA PQOL

TNO-AZL Parent QOL (Netherlands)

1–5

43

4

10

Proxy

x

x

x

YQOL-R

Youth Quality of Life Instrument— Research Version (US)

11–18

56

4

10–15

Self

ICC Test-retest

x

x

x

VSP-A

Vecu et Sante Percue de l’Adolescent (France)

11–17

37

9

Self

ICC Test-retest

x

x

x

4

5–10

Content Construct

x

x

x

x

x

x

ICC, internal consistency.

177.e2

178

SECTION III

100

Patient 1

90

Patient 2



Pediatric Critical Care: Psychosocial and Societal

Population mean

80 70 Patient 3 HRQL score

60 50

Patient 4

40 30 20 10 0

Baseline

Admission

Discharge

6 months

1 year

3 years

5 years

Time of assessment

• Fig. 21.3

​Hypothetical health-related quality of life (HRQL) trajectory of sample patients following critical illness compared with population mean. Patient 1: Previously healthy child who sustains a traumatic brain injury and has persistent clinically significant HRQL deterioration from baseline but does not fall significantly below the population mean. Patient 2: Previously healthy child admitted with sepsis who recovers to baseline within 6 to 12 months of discharge. Patient 3: Child with congenital heart disease who undergoes corrective surgery and has significant improvement in HRQL from baseline. Patient 4: Child with cerebral palsy admitted with respiratory illness who recovers back to baseline but remains significantly below the population mean.  



children experience HRQL improvement following hospitalization for sepsis, especially among those with preexisting chronic illnesses and immune compromise, possibly due to improved control of underlying conditions during hospitalization.27 Collectively, these data demonstrate the importance of assessment of baseline health status whenever possible to appropriately interpret post-hospitalization outcomes.

Summary of Outcomes in General Pediatric Intensive Care Unit Populations Hospital Readmission and Late Mortality Multiple studies have demonstrated that children surviving critical illness remain at high risk for hospital readmission and death in the years following their initial PICU hospitalization.43–49 Nearly one-quarter of PICU survivors are readmitted to the hospital in the year after discharge, and half of all readmissions are nonelective; one-third of nonelective admissions include a repeat PICU stay. This is particularly pronounced among patients with prolonged ICU stays, with 36% of children with an index PICU stay of at least 2 weeks being readmitted within the year. Importantly, 14% of children with an index

stay of only 1 day are also readmitted.47 Mortality in the year after PICU discharge is 10 times higher than in the general US pediatric population,47 with late mortality particularly high among patients with prolonged ICU stays50 and patients with complex chronic conditions.51

Health-Related Quality of Life Impaired HRQL has been demonstrated in many children following critical illness.9,11,12,15,30,41,52–58 While estimates vary depending on the instrument used and time to follow-up, approximately one-third of PICU survivors experience significant impairments in HRQL when assessed between 6 months and 2 years after hospital discharge,30,41,54 and one study demonstrated that 16% had an unfavorable quality of life when assessed up to 6 years after discharge.56 HRQL outcomes for patients who required prolonged PICU stays are worse; two separate studies both found that 43% to 47% of survivors of PICU stays longer than 28 days had impaired HRQL at least 2 years after discharge.15,55 Higher severity of illness tends to correlate with worse HRQL after discharge,30,53,57 but this is not consistently demonstrated across all studies.52,59 Children with chronic comorbidities have been repeatedly found to have worse HRQL after critical illness.15,53,54,58



CHAPTER 21

Functional Status Between 10% and 36% of PICU survivors experience some degree of functional impairment at discharge, with persistent functional impairment after more than 2 years in 10% to 13%.60 New substantial functional morbidity, defined by an increase in Functional Status Scale (FSS)61 score of 3 or greater, was present in 4.8% of PICU patients at discharge and encompassed all functional domains, with the highest proportion of new morbidity present in respiratory, motor, and feeding domains.4 Importantly, new morbidity as defined by FSS increased to 6.5% at 6 months and 10.4% at 3 years postdischarge.62 When measured by Pediatric Overall Performance Category (POPC) score, 37% of survivors of urgent PICU admissions reported a score of 3 or worse (at least moderated disability) 1 month after discharge, with mean POPC scores significantly worse than baseline, especially among patients with prolonged ICU stays and oncologic diagnoses.53 While over 80% of patients with organ dysfunction in the ICU were found to experience functional deterioration with critical illness, two-thirds demonstrated recovery at 6-month follow-up, with faster recovery among younger children.63 Among patients with prolonged PICU stays (i.e., 28 days), however, over half had moderate to severe disability at a median of 4 years after discharge.55

Neurocognitive Status Decline in neurocognitive status occurs in up to one-quarter of PICU patients.64 PICU survivors had significantly lower neurocognitive scores than the general population 18 months after discharge, with lower scores associated with longer PICU stays; lower global cognitive function was associated with increased behavioral problems and worse executive functioning.65 Among patients urgently admitted to the PICU, 28% had poor adaptive behavior functioning after discharge.53

Mental Health Children surviving an ICU stay are at high risk for impaired mental health and development of posttraumatic stress disorder (PTSD) following their hospitalization. Multiple studies identified an inverse relationship between PTSD and HRQL following critical illness.18,52,66–77 One study found that at a median of 5 months after discharge, 20% of PICU survivors were at risk for psychiatric disorders, 34% were at risk for PTSD, 38% were at risk for fatigue disorder, and 80% were at risk for a sleep disorder.78 Children admitted to the PICU were significantly more likely to develop PTSD than children admitted to the general hospital ward.74 Younger children and those who were more severely ill and endured more invasive procedures had significantly more medical fears, a lower sense of control of their health, and ongoing posttraumatic stress responses for 6 months following PICU discharge.75

Family Functioning Families of critically ill children may experience high levels of stress and uncertainty, relationship conflict, and financial burdens during and following their child’s hospitalization.79,80 Parents of PICU survivors may also have psychologic sequelae affecting their own quality of life.12 Several studies found that over one-quarter of parents of PICU patients screened positive for PTSD 6 to 12 months after discharge.68,74 Importantly, PTSD in families is not well correlated with objective factors

Long-Term Outcomes Following Critical Illness in Children

179

such as severity of illness but rather is correlated to subjective experiences; many families experience delayed reactions, with PTSD scores increasing over time.68

Outcomes for Common Pediatric Intensive Care Unit Illness Categories Respiratory Failure The Randomized Evaluation of Sedation Titration for Respiratory Failure (RESTORE) trial of long-term outcomes among mechanically ventilated pediatric patients found that 20% of patients experienced a decline in functional status from baseline to 6-month postdischarge follow-up and 30% were at risk for PTSD. Of those with normal preadmission function, 19% had impaired HRQL.81,82 Older age, history of prematurity or cancer, inadequate pain and sedation management, use of clonidine and methadone, and ventilator duration were among the demographic and clinical factors associated with poor outcomes.81 Among children surviving acute respiratory distress syndrome (ARDS), nearly one-quarter experience substantial new morbidity as defined by an increase in FSS of 3 or greater, and 8% die in the 3 years following discharge.83 Among a limited sample of ARDS survivors seen in follow-up 1 year after discharge, one-third had persistent pulmonary function deficits, and quality of life scores were significantly lower than those of a comparison group of patients with chronic asthma.84

Sepsis Sepsis and other severe infections are associated with high morbidity across multiple domains of health outcomes. Children surviving hospitalization for severe sepsis have mortality rates 2 to 50 times greater than that of the general population depending on age and gender, and high rates of hospital readmission.43 HRQL is impaired in sepsis survivors, with 38.5% of PICU patients with sepsis failing to recover to their baseline HRQL at a median of 31 days postdischarge; older age, immune compromise, septic shock, and longer length of stay were all associated with failure to recover.27 Children surviving meningococcemia also exhibit impaired HRQL up to 3 years after discharge from a PICU.31,85 Decline in functional status is common after severe sepsis, with 34% of survivors experiencing a decline in functional status 28 days after sepsis onset. Clinical factors associated with increased risk of poor outcome included central nervous system or intraabdominal infections, a history of recent trauma, requirement for cardiopulmonary resuscitation, and high PRISM III score.86 Neurocognitive status and development are also affected. Children admitted to the PICU with meningoencephalitis and sepsis experience more severe deficits in neuropsychologic and school performance than other forms of critical illness.78,87 Patients with sepsisassociated encephalopathy had delayed neurodevelopment, lower IQ scores, and declines in school performance and behavior at follow-up,88 and infants surviving pertussis demonstrated neurodevelopmental deficits 1 year after discharge.89

Trauma Traumatic injury is the leading cause of preventable morbidity in children. Pediatric trauma mortality has been steadily decreasing over the past several decades, leaving an increasing number of children at risk of experiencing long-term impairments.90,91 Children surviving traumatic injury experience high

180

SECTION III



Pediatric Critical Care: Psychosocial and Societal

rates of hospital readmission45,46 and ongoing healthcare use92 and are at risk for impaired HRQL,67,93–98 decreased physical functioning,93,95,98–104 PTSD,105,106 depression,107 and family strain.98,108 While patients exhibit improvements over time, some continue to have deficits across a variety of health domains up to 2 years following injury.18 Importantly, one of the factors most strongly associated with poor HRQL after trauma is the presence of PTSD.18,105 Studies focused on patients with traumatic brain injury (TBI) have similarly demonstrated deficits across a range of health domains.109–117 New functional morbidity is present in 37% of TBI survivors at hospital discharge, including over half of patients with severe TBI.109 Functional impairments may persist for years following TBI.112,114,116 Nearly half of survivors of severe pediatric TBI with elevated intracranial pressure demonstrate residual neurologic and neurocognitive deficits more than 2 years after injury, and 16% remained dependent on caregivers.110 Impaired HRQL occurred in 40% of TBI survivors 1 year after injury111 and can persist for at least 2 years.114,117 TBI survivors frequently experience family dysfunction113 and have high rates of ongoing healthcare use and unmet healthcare needs.115 Higher severity of head injury is consistently associated with worse outcome.109–111,113,114,117

Extracorporeal Life Support Several studies evaluated outcomes among populations of children requiring extracorporeal life support (ECLS). A study of neonatal and pediatric ECLS survivors found significantly lower HRQL scores compared to population norms at a median of 5 years after discharge.118 FSS was moderately-to-severely abnormal among 29% of neonatal and pediatric ECLS survivors at hospital discharge, but one-third of survivors had a good FSS.119 At 18 months after discharge, 87% of ECLS survivors were found to have a normal neurologic status; follow-up up to 20 years later found that 90% of survivors had no disability and had normal quality of life, with only 5% demonstrating cognitive impairment.120 Cardiac ECLS survivors were found to have significantly lower HRQL scores than population norms at a median 6-year follow-up, with 18% reporting significant physical limitations or fair/poor health.121 In contrast, a study of children surviving extracorporeal CPR found good mean quality of life and family functioning scores at median 3-year follow-up.122

Examples of Postdischarge Outcomes in Pediatric Interventional Trials Survival is still the most commonly used primary outcome in pediatric critical care trials.10,123–125 After survival, families of critically ill children and critical care practitioners see long-term functional status and HRQL as the most important outcomes for an interventional trial.10 The first large pediatric investigation that included a morbidity measure as a secondary outcome was the interventional trial examining bactericidal, permeability-increasing protein (rBPI21) as adjunctive therapy for meningococcemia sepsis.126 Although mortality did not differ between children treated with placebo versus study drug, those who received rBPI21 demonstrated superior gross functional status at day 28. Subsequently, the RESOLVE trial of activated protein C as adjunctive therapy for pediatric septic shock found no differences in mortality or time to composite resolution of organ dysfunction,127 but 34% of participants

experienced a decline in their functional status 28 days after enrollment and 18% demonstrated poor functional outcome.86 For the Therapeutic Hypothermia After Cardiac Arrest trials, the primary efficacy outcome was survival at 12 months with a Vineland Adaptive Behavior Scales (VABS-II) score of 70 or higher among patients with a VABS-II score of at least 70 before cardiac arrest.128,129 Of 629 total patients randomized, 517 (82%) had results for the primary outcome, which did not significantly differ between the groups. In a single-center study of tight glycemic control involving 700 critically ill children, 569 (81%) subjects underwent neurocognitive testing 3.8 to 4.1 years after study randomization, with no differences found by treatment arm or associated with in-hospital hypoglycemia.130 In the RESTORE cluster-randomized trial of 2449 critically ill children that examined nurse-implemented, goal-directed sedation versus usual care for acute respiratory failure, the primary outcome measure—duration of mechanical ventilation—did not differ between groups.131 However, children randomized to the intervention arm were exposed to fewer sedative and analgesic medications; accordingly, they were generally more awake while intubated. At 6 months post-PICU discharge, a stratified, random sample of subjects was evaluated, with no significant differences found by treatment arm in HRQL, decline in functional status from baseline, or PTSD.82 An ongoing study, RESTORE-cognition, is examining the relationships between sedative exposure during pediatric critical illness with long-term neurocognitive outcomes 2.5 to 5 years following hospital discharge among a subset of participants in the original RESTORE study.132 The Approaches and Decisions for Acute Pediatric TBI (ADAPT) trial is an international, observational comparative effectiveness study that enrolled more than 1000 children with severe TBI to examine the utility of six common interventions.133 ADAPT investigators conducted telephone surveys at 6 months and comprehensive neuropsychologic testing at 12 months to assess HRQL, intellectual ability, speech, memory, executive function, attention/processing speed, and motor skills. At this writing, two large pivotal interventional trials using nonmortality outcomes have initiated enrollment. PROSPECT (Prone and Oscillation Pediatric Clinical Trial, NCT03896763) will employ a two-by-two factorial, response-adaptive design to examine combinations of supine and prone positioning and conventional and high-frequency oscillatory ventilation for supporting up to 1000 children with severe ARDS. Although mechanical ventilation-free days truncated at 28 days represents the primary outcome measure, the investigators will also serially assess functional status and HRQL at 1, 3, 6, and 12 months as exploratory outcomes. SHIPSS (Stress Hydrocortisone In Pediatric Septic Shock, NCT03401398) will examine hydrocortisone as adjunctive therapy for pediatric septic shock. As the primary endpoint, SHIPSS investigators will employ a composite measure of death or persistent, severe HRQL disability compared to baseline. Functional status and HRQL will be assessed at baseline, 28 days, and 90 days. Design of SHIPPS was informed by the prospective, descriptive cohort outcome investigation, Life After Pediatric Sepsis Evaluation (LAPSE, R01HD073362), which ascertained the logistical and biological plausibility of this composite mortality/morbidity outcome.134 LAPSE reported that at 1, 3, 6, and 12 months following admission to the ICU for septic shock, 8%, 11%, 12%, and 13% of patients had died, while 50%, 37%, 30%, and 35% of surviving patients had not yet regained their baseline HRQL.135 At 1 month, 35% of patients had died or survived with persistent, severe HRQL disability; various measures



CHAPTER 21

Long-Term Outcomes Following Critical Illness in Children

181

TABLE Comparison of Estimated Sample Sizes for Interventional Trial for Septic Shock Using Mortality vs Composite 21.3 Poor Outcome (Death or Severe HRQL Impairment at 28 Days) as Primary Outcome Measure

Outcome Measure All-cause mortality Poor outcome

Baseline Occurrence (%)

25% Treatment Effect

Treatment Occurrence (%)

Subjects per Treatment Group

8.0

2.0% reduction

6.0

3419

35.0

8.5% reduction

26.5

618

Two-sided test, a 5 .05 and power 5 .90, outcomes assessed at 28 days. HRQL, Health-related quality of life.

of organ dysfunction were highly associated with this composite poor outcome.136 Pediatric critical care clinical trialists clearly recognize that mortality no longer tells the complete story of outcomes for critically ill children. In addition, mortality is increasingly not logistically feasible as an endpoint for interventional trials from the standpoint of study power. For example, current 28-day mortality for pediatric septic shock is approximately 8%, but death or persistent, severe HRQL disability (poor outcome) occurs in roughly 35% of patients. As summarized in Table 21.3, a marked difference in patient enrollment numbers would be required for a hypothetical sepsis interventional trial employing these two patient-centered clinically meaningful endpoints.

Strategies to Assess Long-Term Outcomes Strategies to Improve Follow-up For longitudinal studies, use of a variety of strategies results in the greatest subject retention. Accordingly, a Cohort Retention Toolbox has been developed and is available free of charge online.137 This website provides several relevant resources, such as participant contact information forms, follow-up protocols, strategies for locating participants, retention strategies, communication templates and manuals, and materials for follow-up staff training. Including monetary incentives, keeping questionnaires short, and contacting people before questionnaires are sent represent key practices for maintaining research subject retention and participation.138 A systematic review and metaanalysis addressing this question demonstrated the importance of monetary incentives after collection of the data.139 A recent article summarized methods for collecting follow-up information from parents 12 months following their child’s emergency admission to the PICU.140 Of the 218 parents participating in this study, 47% chose to complete questionnaires online (77% completion rate), 42% chose to complete postal questionnaires (48% completion rate), and 11% chose to complete questionnaires by telephone interview (44% completion rate). The authors emphasized the importance of a priori identifying parental preferences for longitudinal contact and study participation. A final strategy for maintaining patient and family engagement in long-term follow-up is to include a qualitative study aim. This approach generates insight into the patient and family’s personal experience of the onset and treatment, as well as recovery, and lasting effects of critical illness.141 Qualitative data extend far beyond any validated survey information, add considerable richness to outcome results, and probably should be a required aspect of any clinical trial.

Follow-up Programs for Intensive Care Unit Survivors Aside from recommendations from the Pediatric Acute Lung Injury Consensus Conference for the care of children who have recovered from pediatric ARDS,142 there is currently no other specific guidance in place for follow-up of survivors of pediatric critical illness. A recent survey of pediatric critical care medicine physicians found that very few programs routinely measure longterm functional or HRQL outcomes among PICU survivors, but most physicians believe it should occur.143 One example of routine outcome measurement is the Outcomes Assessment Program at Seattle Children’s Hospital, which was developed in 2010 to measure patient- and family-centered outcomes for hospital inpatients. Families and patients were asked at the time of hospital admission to report baseline HRQL and HRQL status at the time of admission; they were then resurveyed 4 to 12 weeks after hospital discharge. These data have been used in multiple research studies assessing change in HRQL for both ward and PICU patients.27,58,144 Another approach to assessing and potentially treating longterm impairments after critical care is to conduct in-person assessments with patients and families in follow-up clinics. In 2011, the Indiana University School of Medicine opened one of the first collaborative care clinics in the United States aimed at meeting the recovery needs of adult ICU survivors, with a mission to maximize the cognitive, physical, and psychologic recovery of ICU survivors via patient and caregiver needs assessment and a follow-up visit that includes a family conference.145 Intensive care follow-up clinics have become increasingly common in adult critical care146,147 and neonatal intensive care.148 Interest in the development of pediatric-specific critical care follow-up clinics is increasing, though their effectiveness has not yet been determined, and to date, most clinics are focused on follow-up after pediatric neurocritical care.149–151

Other Initiatives The Society of Critical Care Medicine’s THRIVE initiative provides resources and education for patients and families surviving critical illness who may be affected by various aspects of PICS.6,152 The THRIVE Collaborative program offers education, resources, and community to assist patients and families after intensive care and hospitalization. Currently, THRIVE focuses on peer support groups153 and postintensive care recovery clinics.154 Through the Collaborative Pediatric Critical Care Research Network, the National Institute of Child Health and Human Development is currently supporting a consensus effort to provide recommendations for a set of core outcome measures to be used to study

182

SECTION III



Pediatric Critical Care: Psychosocial and Societal

postdischarge outcomes after pediatric critical illness, which will be informed by a scoping review of studies of postdischarge outcomes of pediatric critical illness being conducted by the Pediatric Acute Lung Injury and Sepsis Investigators Long-Term Outcomes Subgroup.

Potential Targets for Interventions Many adverse sequelae of critical illness and critical care may be amenable to intervention. Acute loss of skeletal muscle mass contributes to ongoing physical disability common among survivors of critical illness.155,156 Early and more aggressive physical therapy and passive range of motion introduced in the ICU may reduce ICU neuropathy and other musculoskeletal complications, but the ability to stratify risks and tailor programs to individual needs requires further study. There is substantial interest in developing PICU-based rehabilitation programs, though pediatric critical care providers have concerns regarding barriers to implementation.143 Early mobilization programs, such as “PICU Up!,” are now being developed and have been found to be feasible to implement.157 Larger-scale studies are evaluating the impact of such programs. Another active area of research is how improvement in sleep quality and avoidance of delirium may improve outcomes among survivors. As sleep is known to be severely disturbed in the hospital, interventions that improve the quality of sleep may reduce cognitive and psychiatric sequelae.158,159 Delusional memories are reported by almost a third of children surviving critical illness. These can be exacerbated by exposure to opiates and benzodiazepines, and they are associated with an increased risk of PTSD.66 Interventions such as ICU diaries and journals kept by family members may facilitate reintegration into life after critical illness and are well received by families in the PICU.160–165 Psychologic impairments such as depression are prominent and important features of PICS-p and affect both patients and families.166 Accordingly, psychologic support may improve outcomes in both children and parents. Further research is essential to establish the optimal timing, extent, and type of psychologic support for these children and families.

Conclusion Critical care practitioners are classically trained in resuscitation science that is detailed throughout this textbook. This expertise has led to a marked decrease in mortality associated with childhood critical injury and illness. There is now a greater recognition that critical care begins and ends outside the walls of the ICU. Intensivists and their care-provider colleagues are increasingly aware of the impact of genetics, family dynamics, home environment, and preexisting comorbidities on the intensity, duration, and recovery from critical illness. With maturation of the field,

pediatric critical care practitioners now have the opportunity as well as the obligation to look beyond PICU discharge to minimize not only mortality but also the burden of long-term morbidity in accordance with patients’ and families’ values. Widespread agreement exists in the pediatric critical care community that research is urgently needed to better characterize postdischarge morbidity and understand how to optimize longterm outcomes.7,8,125,167,168 Pediatric intensivists must lead the effort to better understand and optimize long-term outcomes through clinical practice, research, and advocacy. All pediatric critical care practitioners should actively acknowledge and address the long-term physical, cognitive, and mental health impact of critical care on their patients. Ultimately, maximizing long-term functional status and HRQL should be the most important goals of critical care medicine.

Key References Choong K, Fraser D, Al-Harbi S, et al. Functional recovery in critically ill children, the “WeeCover” multicenter study. Pediatr Crit Care Med. 2018;19(2):145-154. Colville G, Kerry S, Pierce C. Children’s factual and delusional memories of intensive care. Am J Respir Crit Care Med. 2008;177(9):976-982. Colville G, Pierce C. Patterns of post-traumatic stress symptoms in families after paediatric intensive care. Intensive Care Med. 2012;38(9): 1523-1531. Czaja AS, Zimmerman JJ, Nathens AB. Readmission and late mortality after pediatric severe sepsis. Pediatrics. 2009;123(3):849-857. Manning JC, Pinto NP, Rennick JE, Colville G, Curley MAQ. Conceptualizing post intensive care syndrome in children-the PICS-p framework. Pediatr Crit Care Med. 2018;19(4):298-300. Merritt C, Menon K, Agus MSD, et al. Beyond survival: pediatric critical care interventional trial outcome measure preferences of families and healthcare professionals. Pediatr Crit Care Med. 2018;19(2): e105-e111. Pollack MM, Holubkov R, Glass P, et al. Functional status scale: new pediatric outcome measure. Pediatrics. 2009;124(1):e18-28. Quasney MW, López-Fernández YM, Santschi M, Watson RS, for the Pediatric Acute Lung Injury Consensus Conference Group. The outcomes of children with pediatric acute respiratory distress syndrome: proceedings from the Pediatric Acute Lung Injury Consensus Conference. Ped Crit Care Med. 2015;16:S23-S40. Rennick JE, Johnston CC, Dougherty G, Platt R, Ritchie JA. Children’s psychological responses after critical illness and exposure to invasive technology. J Dev Behav Pediatr. 2002;23(3):133-144. Watson RS, Asaro LA, Hutchins L, et al. Risk factors for functional decline and impaired quality of life after pediatric respiratory failure. Am J Respir Crit Care Med. 2019;200(7):900-909.

The full reference list for this chapter is available atExpertConsult.com.

e1

References

































30.





31.





32.





33.





34.





35.





36.





37.





38.





39.





40.



41.































29.





28.





27.





26.





25.













24.





23.





1. Epstein D, Wong CF, Khemani RG, et al. Race/Ethnicity is not associated with mortality in the PICU. Pediatrics. 2011;127(3): e588-597. 2. Namachivayam P, Shann F, Shekerdemian L, et al. Three decades of pediatric intensive care: who was admitted, what happened in intensive care, and what happened afterward. Pediatr Crit Care Med. 2010;11(5):549-555. 3. Visser IH, Hazelzet JA, Albers MJ, et al. Mortality prediction models for pediatric intensive care: comparison of overall and subgroup specific performance. Intensive Care Med. 2013;39(5):942-950. 4. Pollack MM, Holubkov R, Funai T, et al. Pediatric intensive care outcomes: development of new morbidities during pediatric critical care. Pediatr Crit Care Med. 2014;15(9):821-827. 5. Rennick JE, Childerhose JE. Redefining success in the PICU: new patient populations shift targets of care. Pediatrics. 2015;135(2): e289-291. 6. Needham DM, Davidson J, Cohen H, et al. Improving long-term outcomes after discharge from intensive care unit: report from a stakeholders’ conference. Crit Care Med. 2012;40(2):502-509. 7. Manning JC, Pinto NP, Rennick JE, Colville G, Curley MAQ. Conceptualizing post intensive care syndrome in children-the PICSp framework. Pediatr Crit Care Med. 2018;19(4):298-300. 8. Watson RS, Choong K, Colville G, et al. Life after critical illness in children-toward an understanding of pediatric post-intensive care syndrome. J Pediatr. 2018;198:16-24. 9. Aspesberro F, Mangione-Smith R, Zimmerman JJ. Health-related quality of life following pediatric critical illness. Intensive Care Med. 2015;41(7):1235-1246. 10. Merritt C, Menon K, Agus MSD, et al. Beyond survival: pediatric critical care interventional trial outcome measure preferences of families and healthcare professionals. Pediatr Crit Care Med. 2018; 19(2):e105-e111. 11. Knoester H, Bronner MB, Bos AP, Grootenhuis MA. Quality of life in children three and nine months after discharge from a paediatric intensive care unit: a prospective cohort study. Health Qual Life Outcomes. 2008;6:21. 12. Knoester H, Grootenhuis MA, Bos AP. Outcome of paediatric intensive care survivors. Eur J Pediatr. 2007;166(11):1119-1128. 13. Eiser C, Morse R. Quality-of-life measures in chronic diseases of childhood. Health Technol Assess. 2001;5(4):1-157. 14. WHO. The World Health Organization Quality of Life assessment (WHOQOL): position paper from the World Health Organization. Soc Sci Med. 1995;41(10):1403-1409. 15. Conlon NP, Breatnach C, O’Hare BP, Mannion DW, Lyons BJ. Health-related quality of life after prolonged pediatric intensive care unit stay. Pediatr Crit Care Med. 2009;10(1):41-44. 16. Organization WH. Constitution of the World Health Organization Basic Document. Geneva, Switzerland: World Health Organization; 1948. 17. Janssens L, Gorter JW, Ketelaar M, Kramer WL, Holtslag HR. Health-related quality-of-life measures for long-term follow-up in children after major trauma. Qual Life Res. 2008;17(5):701-713. 18. Martin-Herz SP, Zatzick DF, McMahon RJ. Health-related quality of life in children and adolescents following traumatic injury: a review. Clin Child Fam Psychol Rev. 2012;15(3):192-214. 19. Simon AE, Chan KS, Forrest CB. Assessment of children’s healthrelated quality of life in the United States with a multidimensional index. Pediatrics. 2008;121(1):e118-126. 20. Sorensen TI, Nielsen GG, Andersen PK, Teasdale TW. Genetic and environmental influences on premature death in adult adoptees. N Engl J Med. 1988;318(12):727-732. 21. Varni JW, Sherman SA, Burwinkle TM, Dickinson PE, Dixon P. The PedsQL family impact module: preliminary reliability and validity. Health Qual Life Outcomes. 2004;2:55. 22. Feudtner C, Christakis DA, Connell FA. Pediatric deaths attributable to complex chronic conditions: a population-based study of

Washington State, 1980-1997. Pediatrics. 2000;106(1 Pt 2): 205-209. Mestrovic J, Kardum G, Sustic A, et al. Neurodevelopmental disabilities and quality of life after intensive care treatment. J Paediatr Child Health. 2007;43(10):673-676. Wilson IB, Cleary PD. Linking clinical variables with health-related quality of life. A conceptual model of patient outcomes. JAMA. 1995;273(1):59-65. Varni JW, Limbers CA, Burwinkle TM. Parent proxy-report of their children’s health-related quality of life: an analysis of 13,878 parents’ reliability and validity across age subgroups using the PedsQL 4.0 Generic Core Scales. Health Qual Life Outcomes. 2007;5:2. Killien EY, Watson RS, Zimmerman JJ. Inherent value of baseline measures when assessing the trajectory of health-related quality of life among children surviving critical illness. Intensive Care Med. 2018;44(11):1979-1981. Killien EY, Farris RWD, Watson RS, Dervan LA, Zimmerman JJ. Health-related quality of life among survivors of pediatric sepsis. Pediatr Crit Care Med. 2019;20(6):501-509 Edwards JD, Houtrow AJ, Vasilevskis EE, et al. Chronic conditions among children admitted to U.S. pediatric intensive care units: their prevalence and impact on risk for mortality and prolonged length of stay. Crit Care Med. 2012;40(7):2196-2203. Typpo KV, Petersen NJ, Petersen LA, Mariscalco MM. Children with chronic illness return to their baseline functional status after organ dysfunction on the first day of admission in the pediatric intensive care unit. J Pediatr. 2010;157(1):108-113.e101. Polic B, Mestrovic J, Markic J, et al. Long-term quality of life of patients treated in paediatric intensive care unit. Eur J Pediatr. 2013;172(1):85-90. Buysse CM, Raat H, Hazelzet JA, Hop WC, Maliepaard M, Joosten KF. Surviving meningococcal septic shock: health consequences and quality of life in children and their parents up to 2 years after pediatric intensive care unit discharge. Crit Care Med. 2008; 36(2):596-602. Boyle CA, Boulet S, Schieve LA, et al. Trends in the prevalence of developmental disabilities in US children, 1997-2008. Pediatrics. 2011;127(6):1034-1042. Burns KH, Casey PH, Lyle RE, Bird TM, Fussell JJ, Robbins JM. Increasing prevalence of medically complex children in US hospitals. Pediatrics. 2010;126(4):638-646. Dosa NP, Boeing NM, Ms N, Kanter RK. Excess risk of severe acute illness in children with chronic health conditions. Pediatrics. 2001; 107(3):499-504. Esper AM, Martin GS. The impact of comorbid [corrected] conditions on critical illness. Crit Care Med. 2011;39(12):2728-2735. Odetola FO, Gebremariam A, Davis MM. Comorbid illnesses among critically ill hospitalized children: impact on hospital resource use and mortality, 1997-2006. Pediatr Crit Care Med. 2010;11(4):457-463. Oeyen SG, Vandijck DM, Benoit DD, Annemans L, Decruyenaere JM. Quality of life after intensive care: a systematic review of the literature. Crit Care Med. 2010;38(12):2386-2400. Kyosti E, Ala-Kokko TI, Ohtonen P, et al. Factors associated with health-related quality of life 6 years after ICU discharge in a finnish paediatric population: a cohort study. Intensive Care Med. 2018; 44(9):1378-1387. Varni JW, Limbers CA, Burwinkle TM. Impaired health-related quality of life in children and adolescents with chronic conditions: a comparative analysis of 10 disease clusters and 33 disease categories/ severities utilizing the PedsQL 4.0 Generic Core Scales. Health Qual Life Outcomes. 2007;5:43. Varni JW, Burwinkle TM, Seid M, Skarr D. The PedsQL 4.0 as a pediatric population health measure: feasibility, reliability, and validity. Ambul Pediatr. 2003;3(6):329-341. Cunha F, Almeida-Santos L, Teixeira-Pinto A, Neves F, Barata D, Costa-Pereira A. Health-related quality of life of pediatric intensive care survivors. J Pediatr (Rio J). 2012;88(1):25-32.

e2

















































64. Caprarola SD, Kudchadkar SR, Bembea MM. Neurologic outcomes following care in the pediatric intensive care unit. Curr Treat Options Pediatr. 2017;3(3):193-207. 65. Meyburg J, Ries M, Zielonka M, et al. Cognitive and behavioral consequences of pediatric delirium: a pilot study. Pediatr Crit Care Med. 2018;19(10):e531-e537. 66. Colville G, Kerry S, Pierce C. Children’s factual and delusional memories of intensive care. Am J Respir Crit Care Med. 2008;177(9): 976-982. 67. Holbrook TL, Hoyt DB, Coimbra R, et al. Trauma in adolescents causes long-term marked deficits in quality of life: adolescent children do not recover preinjury quality of life or function up to two years postinjury compared to national norms. J Trauma. 2007;62(3): 577-583; discussion 583. 68. Colville G, Pierce C. Patterns of post-traumatic stress symptoms in families after paediatric intensive care. Intensive Care Med. 2012; 38(9):1523-1531. 69. Dow B KJ, Long D. Beyond survival recall and aspects of care and environment in children’s psychological distress following PICU admission. Pediatr Crit Care Med. 2011:680. 70. Holbrook TL, Hoyt DB, Coimbra R, Potenza B, Sise M, Anderson JP. Long-term posttraumatic stress disorder persists after major trauma in adolescents: new data on risk factors and functional outcome. J Trauma. 2005;58(4):764-769; discussion 769-771. 71. Horner MD, Hamner MB. Neurocognitive functioning in posttraumatic stress disorder. Neuropsychol Rev. 2002;12(1):15-30. 72. Landolt MA, Vollrath ME, Gnehm HE, Sennhauser FH. Posttraumatic stress impacts on quality of life in children after road traffic accidents: prospective study. Aust N Z J Psychiatry. 2009;43(8): 746-753. 73. Qureshi SU, Long ME, Bradshaw MR, et al. Does PTSD impair cognition beyond the effect of trauma? J Neuropsychiatry Clin Neurosci. 2011;23(1):16-28. 74. Rees G, Gledhill J, Garralda ME, Nadel S. Psychiatric outcome following paediatric intensive care unit (PICU) admission: a cohort study. Intensive Care Med. 2004;30(8):1607-1614. 75. Rennick JE, Johnston CC, Dougherty G, Platt R, Ritchie JA. Children’s psychological responses after critical illness and exposure to invasive technology. J Dev Behav Pediatr. 2002;23(3):133-144. 76. Vollrath M, Landolt MA. Personality predicts quality of life in pediatric patients with unintentional injuries: a 1-year follow-up study. J Pediatr Psychol. 2005;30(6):481-491. 77. Zatzick DF, Jurkovich GJ, Fan MY, et al. Association between posttraumatic stress and depressive symptoms and functional outcomes in adolescents followed up longitudinally after injury hospitalization. Arch Pediatr Adolesc Med. 2008;162(7):642-648. 78. Als LC, Picouto MD, Hau SM, et al. Mental and physical well-being following admission to pediatric intensive care. Pediatr Crit Care Med. 2015;16(5):e141-149. 79. Davidson JE, Jones C, Bienvenu OJ. Family response to critical illness: postintensive care syndrome-family. Crit Care Med. 2012; 40(2):618-624. 80. Shudy M, de Almeida ML, Ly S, et al. Impact of pediatric critical illness and injury on families: a systematic literature review. Pediatrics. 2006;118(Suppl 3):S203-218. 81. Watson RS, Asaro LA, Hutchins L, et al. Risk factors for functional decline and impaired quality of life after pediatric respiratory failure. Am J Respir Crit Care Med. 2019;200(7):900-909. 82. Watson RS, Asaro LA, Hertzog JH, et al. Long-term outcomes after protocolized sedation versus usual care in ventilated pediatric patients. Am J Respir Crit Care Med. 2018;197(11):1457-1467. 83. Keim G, Watson RS, Thomas NJ, Yehya N. New morbidity and discharge disposition of pediatric acute respiratory distress syndrome survivors. Crit Care Med. 2018;46(11):1731-1738. 84. Ward SL, Turpin A, Spicer AC, Treadwell MJ, Church GD, Flori HR. Long-term pulmonary function and quality of life in children after acute respiratory distress syndrome: a feasibility investigation. Pediatr Crit Care Med. 2017;18(1):e48-e55.



















































42. Heneghan JA, Reeder RW, Dean JM, et al. Characteristics and outcomes of critical illness in children with feeding and respiratory technology dependence. Pediatr Crit Care Med. 2019;20(5):417-425. 43. Czaja AS, Zimmerman JJ, Nathens AB. Readmission and late mortality after pediatric severe sepsis. Pediatrics. 2009;123(3):849-857. 44. Maddux AB, Bennett TD. Mortality after pediatric critical illness: made it home, still vulnerable. Pediatr Crit Care Med. 2018;19(3): 272-273. 45. Maddux AB, DeWitt PE, Mourani PM, Bennett TD. Hospital readmissions after pediatric trauma. Pediatr Crit Care Med. 2018; 19(1):e31-e40. 46. Naseem HU, Dorman RM, Bass KD, Rothstein DH. Intensive care unit admission predicts hospital readmission in pediatric trauma. J Surg Res. 2016;205(2):456-463. 47. Hartman ME, Saeed MJ, Bennett T, Typpo K, Matos R, Olsen MA. Readmission and late mortality after critical illness in childhood. Pediatr Crit Care Med. 2017;18(3):e112-e121. 48. Hessey E, Morissette G, Lacroix J, et al. Long-term Mortality After acute kidney injury in the pediatric ICU. Hosp Pediatr. 2018; 8(5):260-268. 49. Hessey E, Morissette G, Lacroix J, et al. Healthcare Utilization after acute kidney injury in the pediatric intensive care unit. Clin J Am Soc Nephrol. 2018;13(5):685-692. 50. Namachivayam SP, d’Udekem Y, Millar J, Cheung MM, Butt W. Survival status and functional outcome of children who required prolonged intensive care after cardiac surgery. J Thorac Cardiovasc Surg. 2016;152(4):1104-1112.e1103. 51. Kalzen H, Larsson B, Eksborg S, Lindberg L, Edberg KE, Frostell C. Survival after PICU admission: the impact of multiple admissions and complex chronic conditions. PLoS One. 2018;13(4):e0193294. 52. Colville GA, Pierce CM. Children’s self-reported quality of life after intensive care treatment. Pediatr Crit Care Med. 2013;14(2):e85-92. 53. Ebrahim S, Singh S, Hutchison JS, et al. Adaptive behavior, functional outcomes, and quality of life outcomes of children requiring urgent ICU admission. Pediatr Crit Care Med. 2013;14(1):10-18. 54. Morrison AL, Gillis J, O’Connell AJ, Schell DN, Dossetor DR, Mellis C. Quality of life of survivors of pediatric intensive care. Pediatr Crit Care Med. 2002;3(1):1-5. 55. Namachivayam P, Taylor A, Montague T, et al. Long-stay children in intensive care: long-term functional outcome and quality of life from a 20-yr institutional study. Pediatr Crit Care Med. 2012;13(5): 520-528. 56. Taylor A, Butt W, Ciardulli M. The functional outcome and quality of life of children after admission to an intensive care unit. Intensive Care Med. 2003;29(5):795-800. 57. Jones S, Rantell K, Stevens K, et al. Outcome at 6 months after admission for pediatric intensive care: a report of a national study of pediatric intensive care units in the United Kingdom. Pediatrics. 2006;118(5):2101-2108. 58. Aspesberro F, Fesinmeyer MD, Zhou C, Zimmerman JJ, MangioneSmith R. Construct validity and responsiveness of the pediatric quality of life inventory 4.0 generic core scales and infant scales in the PICU. Pediatr Crit Care Med. 2016;17(6):e272-279. 59. Cunha F, Mota T, Teixeira-Pinto A, et al. Factors associated with health-related quality of life changes in survivors to pediatric intensive care. Pediatr Crit Care Med. 2013;14(1):e8-15. 60. Ong C, Lee JH, Leow MK, Puthucheary ZA. Functional outcomes and physical impairments in pediatric critical care survivors: a scoping review. Pediatr Crit Care Med. 2016;17(5):e247-259. 61. Pollack MM, Holubkov R, Glass P, et al. Functional status scale: new pediatric outcome measure. Pediatrics. 2009;124(1):e18-28. 62. Pinto NP, Rhinesmith EW, Kim TY, Ladner PH, Pollack MM. Long-term function after pediatric critical illness: results from the survivor outcomes study. Pediatr Crit Care Med. 2017;18(3): e122-e130. 63. Choong K, Fraser D, Al-Harbi S, et al. Functional recovery in critically Ill children, the “WeeCover” multicenter study. Pediatr Crit Care Med. 2018;19(2):145-154.

e3









106.







107.





108.







109.







110.







111.







112.





114.





115.



116.









113.











117.





119.





120.





121.





122.



123.













118.





125.





124.











































85. Buysse CM, Vermunt LC, Raat H, et al. Surviving meningococcal septic shock in childhood: long-term overall outcome and the effect on health-related quality of life. Critical care (London, England). 2010;14(3):R124. 86. Farris RW, Weiss NS, Zimmerman JJ. Functional outcomes in pediatric severe sepsis: further analysis of the researching severe sepsis and organ dysfunction in children: a global perspective trial. Pediatr Crit Care Med. 2013;14(9):835-842. 87. Als LC, Nadel S, Cooper M, Pierce CM, Sahakian BJ, Garralda ME. Neuropsychologic function three to six months following admission to the PICU with meningoencephalitis, sepsis, and other disorders: a prospective study of school-aged children. Crit Care Med. 2013;41(4):1094-1103. 88. Kaur J, Singhi P, Singhi S, Malhi P, Saini AG. Neurodevelopmental and behavioral outcomes in children with sepsis-associated encephalopathy admitted to pediatric intensive care unit: a prospective case control study. J Child Neurol. 2016;31(6): 683-690. 89. Berger JT, Villalobos ME, Clark AE, et al. Cognitive development one year after infantile critical pertussis. Pediatr Crit Care Med. 2018;19(2):89-97. 90. Myers SR, Branas CC, French B, Nance ML, Carr BG. A national analysis of pediatric trauma care utilization and outcomes in the United States. Pediat emerg care. 2019;35(1):1-7. 91. Oliver J, Avraham J, Frangos S, Tomita S, DiMaggio C. The epidemiology of inpatient pediatric trauma in United States hospitals 2000 to 2011. J Pediatr Surg. 2018;53(4):758-764. 92. Van de Voorde P, Sabbe M, Tsonaka R, et al. The long-term outcome after severe trauma of children in Flanders (Belgium): a population-based cohort study using the International Classification of Functioning–related outcome score. Eur J Pediatr. 2011;170(1): 65-73. 93. Aitken ME, Tilford JM, Barrett KW, et al. Health status of children after admission for injury. Pediatrics. 2002;110(2 Pt 1):337-342. 94. Davey TM, Aitken LM, Kassulke D, et al. Long-term outcomes of seriously injured children: a study using the Child Health Questionnaire. J Paediatr Child Health. 2005;41(5-6):278-283. 95. Gabbe BJ, Simpson PM, Sutherland AM, et al. Functional and health-related quality of life outcomes after pediatric trauma. J Trauma. 2011;70(6):1532-1538. 96. Schneeberg A, Ishikawa T, Kruse S, et al. A longitudinal study on quality of life after injury in children. Health Qual Life Outcomes. 2016;14(1):120. 97. Schweer LH, Cook BS, Bivens K, Van Kuiken D, Garcia VF, Falcone Jr RA. Family perception: quality of life following a child’s traumatic injury. J Trauma Nurs. 2006;13(1):6-14; quiz 15-16. 98. Winthrop AL, Brasel KJ, Stahovic L, Paulson J, Schneeberger B, Kuhn EM. Quality of life and functional outcome after pediatric trauma. J Trauma. 2005;58(3):468-473; discussion 473-464. 99. Aitken ME, Jaffe KM, DiScala C, Rivara FP. Functional outcome in children with multiple trauma without significant head injury. Arch Phys Med Rehabil. 1999;80(8):889-895. 100. Kendrick D, Vinogradova Y, Coupland C, et al. Recovery from injury: the UK burden of injury multicentre longitudinal study. Inj Prev. 2013;19(6):370-381. 101. Macpherson AK, Rothman L, McKeag AM, Howard A. Mechanism of injury affects 6-month functional outcome in children hospitalized because of severe injuries. J Trauma. 2003;55(3):454-458. 102. Polinder S, Meerding WJ, Toet H, Mulder S, Essink-Bot ML, van Beeck EF. Prevalence and prognostic factors of disability after childhood injury. Pediatrics. 2005;116(6):e810-817. 103. Valadka S, Poenaru D, Dueck A. Long-term disability after trauma in children. J Pediatr Surg. 2000;35(5):684-687. 104. Janssens L, Gorter JW, Ketelaar M, Kramer WL, Holtslag HR. Long-term health condition in major pediatric trauma: a pilot study. J Pediatr Surg. 2009;44(8):1591-1600. 105. Holbrook TL, Hoyt DB, Coimbra R, Potenza B, Sise M, Anderson JP. Long-term posttraumatic stress disorder persists after major

trauma in adolescents: new data on risk factors and functional outcome. J Trauma. 2005;58(4):764-771. Kassam-Adams N, Marsac ML, Hildenbrand A, Winston F. Posttraumatic stress following pediatric injury: update on diagnosis, risk factors, and intervention. JAMA Pediatr. 2013; 167(12):1158-1165. Han PP, Holbrook TL, Sise MJ, et al. Postinjury depression is a serious complication in adolescents after major trauma: injury severity and injury-event factors predict depression and long-term quality of life deficits. J Trauma. 2011;70(4):923-930. Osberg JS, Kahn P, Rowe K, Brooke MM. Pediatric trauma: impact on work and family finances. Pediatrics. 1996;98(5):890-897. Bennett TD, Dixon RR, Kartchner C, et al. Functional status scale in children with traumatic brain injury: a prospective cohort study. Pediatr Crit Care Med. 2016;17(12):1147-1156. Jagannathan J, Okonkwo DO, Yeoh HK, et al. Long-term outcomes and prognostic factors in pediatric patients with severe traumatic brain injury and elevated intracranial pressure. J Neurosurg Pediatr. 2008;2(4):240-249. McCarthy ML, MacKenzie EJ, Durbin DR, et al. Healthrelated quality of life during the first year after traumatic brain injury. Arch Pediatr Adolesc Med. 2006;160(3):252-260. Mhanna MJ, Mallah WE, Verrees M, Shah R, Super DM. Outcome of children with severe traumatic brain injury who are treated with decompressive craniectomy. J Neurosurg Pediatr. 2015;16(5): 508-514. Rashid M, Goez HR, Mabood N, et al. The impact of pediatric traumatic brain injury (TBI) on family functioning: a systematic review. J Pediatr Rehabil Med. 2014;7(3):241-254. Rivara FP, Koepsell TD, Wang J, et al. Disability 3, 12, and 24 months after traumatic brain injury among children and adolescents. Pediatrics. 2011;128(5):e1129-1138. Slomine BS, McCarthy ML, Ding R, et al. Health care utilization and needs after pediatric traumatic brain injury. Pediatrics. 2006; 117(4):e663-674. Aaro Jonsson CC, Emanuelson IM, Charlotte Smedler A. Variability in quality of life 13 years after traumatic brain injury in childhood. Int J Rehabil Res. 2014;37(4):317-322. Brown EA, Kenardy J, Chandler B, Anderson V, McKinlay L, Le Brocque R. Parent-reported health-related quality of life in children with traumatic brain injury: a prospective study. J Pediatr Psychol. 2016;41(2):244-255. Yu YR, Carpenter JL, DeMello AS, et al. Evaluating quality of life of extracorporeal membrane oxygenation survivors using the pediatric quality of life inventory survey. J Pediatr Surg. 2018; 53(5):1060-1064. Cashen K, Reeder R, Dalton HJ, et al. Functional status of neonatal and pediatric patients after extracorporeal membrane oxygenation. Pediatr Crit Care Med. 2017;18(6):561-570. Di Leo V, Biban P, Mercolini F, et al. The quality of life in extracorporeal life support survivors: single-center experience of a long-term follow-up. Childs Nerv Syst. 2019;35(2):227-235. Elias MD, Achuff BJ, Ittenbach RF, et al. Long-term outcomes of pediatric cardiac patients supported by extracorporeal membrane oxygenation. Pediatr Crit Care Med. 2017;18(8):787-794. Torres-Andres F, Fink EL, Bell MJ, Sharma MS, Yablonsky EJ, Sanchez-de-Toledo J. Survival and long-term functional outcomes for children with cardiac arrest treated with extracorporeal cardiopulmonary resuscitation. Pediatr Crit Care Med. 2018;19(5):451458. Menon K, McNally JD, Zimmerman JJ, et al. Primary outcome measures in pediatric septic shock trials: a systematic review. Pediatr Crit Care Med. 2017;18(3):e146-e154. Yehya N, Thomas NJ. Relevant Outcomes in pediatric acute respiratory distress syndrome studies. Front Pediatr. 2016;4:51. Heneghan JA, Pollack MM. Morbidity: changing the outcome paradigm for pediatric critical care. Pediatr Clin North Am. 2017; 64(5):1147-1165.

e4















































145. Khan BA, Lasiter S, Boustani MA. CE: critical care recovery center: an innovative collaborative care model for ICU survivors. Am J Nurs. 2015;115(3):24-31; quiz 34, 46. 146. Lasiter S, Oles SK, Mundell J, London S, Khan B. Critical care follow-up clinics: a scoping review of interventions and outcomes. Clin Nurse Spec. 2016;30(4):227-237. 147. Schofield-Robinson OJ, Lewis SR, Smith AF, McPeake J, Alderson P. Follow-up services for improving long-term outcomes in intensive care unit (ICU) survivors. Cochrane Database Syst Rev. 2018;11:Cd012701. 148. Kuppala VS, Tabangin M, Haberman B, Steichen J, Yolton K. Current state of high-risk infant follow-up care in the United States: results of a national survey of academic follow-up programs. J Perinatol. 2012;32(4):293-298. 149. Dodd JN, Hall TA, Guilliams K, et al. Optimizing neurocritical care follow-up through the integration of neuropsychology. Pediatr Neurol. 2018;89:58-62. 150. Wainwright MS, Grimason M, Goldstein J, et al. Building a pediatric neurocritical care program: a multidisciplinary approach to clinical practice and education from the intensive care unit to the outpatient clinic. Semin Pediatr Neurol. 2014;21(4): 248-254. 151. Williams CN, Kirby A, Piantino J. If you build it, they will come: initial experience with a multi-disciplinary pediatric neurocritical care follow-up clinic. Children (Basel, Switzerland). 2017;4(9):83. 152. Elliott D, Davidson JE, Harvey MA, et al. Exploring the scope of post-intensive care syndrome therapy and care: engagement of non-critical care providers and survivors in a second stakeholders meeting. Crit Care Med. 2014;42(12):2518-2526. 153. Mikkelsen ME, Jackson JC, Hopkins RO, et al. Peer support as a novel strategy to mitigate post-intensive care syndrome. AACN Adv Crit Care. 2016;27(2):221-229. 154. Kuehn BM. Clinics aim to improve post-icu recovery. JAMA. 2019. 155. Herridge MS, Cheung AM, Tansey CM, et al. One-year outcomes in survivors of the acute respiratory distress syndrome. N Engl J Med. 2003;348(8):683-693. 156. Puthucheary ZA, Rawal J, McPhail M, et al. Acute skeletal muscle wasting in critical illness. Jama. 2013;310(15):1591-1600. 157. Wieczorek B, Ascenzi J, Kim Y, et al. PICU Up!: Impact of a quality improvement intervention to promote early mobilization in critically ill children. Pediatr Crit Care Med. 2016;17(12): e559-e566. 158. Beebe DW. Cognitive, behavioral, and functional consequences of inadequate sleep in children and adolescents. Pediatr Clin North Am. 2011;58(3):649-665. 159. Kamdar BB, King LM, Collop NA, et al. The effect of a quality improvement intervention on perceived sleep quality and cognition in a medical ICU. Crit Care Med. 2013;41(3):800-809. 160. Egerod I, Christensen D. A comparative study of ICU patient diaries vs. hospital charts. Qual Health Res. 2010;20(10):1446-1456. 161. Egerod I, Storli SL, Akerman E. Intensive care patient diaries in Scandinavia: a comparative study of emergence and evolution. Nurs Inq. 2011;18(3):235-246. 162. Garrouste-Orgeas M, Coquet I, Perier A, et al. Impact of an intensive care unit diary on psychological distress in patients and relatives*. Crit Care Med. 2012;40(7):2033-2040. 163. Knowles RE, Tarrier N. Evaluation of the effect of prospective patient diaries on emotional well-being in intensive care unit survivors: a randomized controlled trial. Crit Care Med. 2009;37(1): 184-191. 164. Herrup EA, Wieczorek B, Kudchadkar SR. Feasibility and perceptions of PICU diaries. Pediatr Crit Care Med. 2019;20(2): e83-e90.



















































126. Levin M, Quint PA, Goldstein B, et al. Recombinant bactericidal/ permeability-increasing protein (rBPI21) as adjunctive treatment for children with severe meningococcal sepsis: a randomised trial. rBPI21 Meningococcal Sepsis Study Group. Lancet. 2000;356 (9234):961-967. 127. Nadel S, Goldstein B, Williams MD, et al. Drotrecogin alfa (activated) in children with severe sepsis: a multicentre phase III randomised controlled trial. Lancet. 2007;369(9564):836-843. 128. Moler FW, Silverstein FS, Holubkov R, et al. Therapeutic hypothermia after out-of-hospital cardiac arrest in children. N Engl J Med. 2015;372(20):1898-1908. 129. Moler FW, Silverstein FS, Holubkov R, et al. Therapeutic hypothermia after in-hospital cardiac arrest in children. N Engl J Med. 2017;376(4):318-329. 130. Mesotten D, Gielen M, Sterken C, et al. Neurocognitive development of children 4 years after critical illness and treatment with tight glucose control: a randomized controlled trial. JAMA. 2012;308(16):1641-1650. 131. Curley MA, Wypij D, Watson RS, et al. Protocolized sedation vs usual care in pediatric patients mechanically ventilated for acute respiratory failure: a randomized clinical trial. JAMA. 2015; 313(4):379-389. 132. Curley MAQ, Watson RS, Cassidy AM, et al. Design and rationale of the “Sedation strategy and cognitive outcome after critical illness in early childhood” study. Contemp Clin Trials. 2018;72:8-15. 133. Bell MJ, Wisniewski SR. Severe traumatic brain injury in children: a vision for the future. Intensive Care Med. 2016;42(10):1618-1620. 134. Pearson GA. Mathematical morbidity in paediatric intensive care. Lancet. 2003;362(9379):180-181. 135. Zimmerman JJ, Banks R, Berg RA, et al. Trajectory of mortality and health-related quality of life morbidity following communityacquired pediatric septic shock. Crit Care Med. 2020;48(3): 329-337. 136. Zimmerman JJ, Banks R, Berg RA, et al. Critical illness factors associated with long-term mortality and health-related quality of life morbidity following community-acquired pediatric septic shock. Crit Care Med. 2020;48(3):319-328. 137. Needham DM. Cohort Retention Tools. 2019. https://www.improvelto. com/cohort-retention-tools/. 138. Edwards PJ, Roberts I, Clarke MJ, et al. Methods to increase response to postal and electronic questionnaires. Cochrane Database Syst Rev. 2009(3):MR000008. 139. Brueton VC, Tierney JF, Stenning S, et al. Strategies to improve retention in randomised trials: a cochrane systematic review and meta-analysis. BMJ Open. 2014;4(2):e003821. 140. Pulham RA, Wray J, Feinstein Y, et al. Feasibility and acceptability of methods to collect follow-up information from parents 12 months after their child’s emergency admission to pediatric Intensive Care. Pediatr Crit Care Med. 2019;20(4):e199-e207. 141. Gallop KH, Kerr CE, Nixon A, Verdian L, Barney JB, Beale RJ. A qualitative investigation of patients’ and caregivers’ experiences of severe sepsis*. Crit Care Med. 2015;43(2):296-307. 142. Quasney MW L-FY, Santschi M, Watson RS, for the Pediatric Acute Lung Injury Consensus Conference Group. The outcomes of children with pediatric acute respiratory distress syndrome: proceedings from the Pediatric Acute Lung Injury Consensus Conference. Ped Crit Care Med. 2015;16:S23-S40. 143. Treble-Barna A, Beers SR, Houtrow AJ, et al. PICU-based rehabilitation and outcomes assessment: a survey of pediatric critical care physicians. Pediatr Crit Care Med. 2019;20(6):e274-e282. 144. Desai AD, Zhou C, Stanford S, Haaland W, Varni JW, MangioneSmith RM. Validity and responsiveness of the pediatric quality of life inventory (PedsQL) 4.0 generic core scales in the pediatric inpatient setting. JAMA Pediatr. 2014;168(12):1114-1121.

e5









167. Peters MJ, Argent A, Festa M, et al. The intensive care medicine clinical research agenda in paediatrics. Intensive Care Med. 2017;43(9):1210-1224. 168. Tamburro RF, Jenkins TL, Kochanek PM. Strategic planning for research in pediatric critical care. Pediatr Crit Care Med. 2016; 17(11):e539-e542.





165. Mikkelsen G. The meaning of personal diaries to children and families in the paediatric intensive care unit: a qualitative study. Intensive Crit Care Nurs. 2018;45:25-30. 166. Hopkins RO, Girard TD. Medical and economic implications of cognitive and psychiatric disability of survivorship. Semin Respir Crit Care Med. 2012;33(4):348-356.

e6

Abstract: As pediatric intensive care has become progressively more effective at reducing mortality from critical illness, a wide range of long-term sequelae among survivors is becoming apparent. A comprehensive understanding of the ongoing impact of critical illness and critical care therapies on survivors is essential in developing interventions to improve care and enhance the recovery of patients who experience life-threatening illness and injury.

Incorporation of morbidity measures as important outcomes in pediatric critical care clinical trials is becoming increasingly common and is identified as a priority by patients and families. Key Words: health-related quality of life, functional status, longterm follow-up, morbidity, neurodevelopment, postintensive care syndrome, survivors

22 Burnout and Resiliency RUTH KLEINPELL AND JASON M. KANE

PEARLS •







Both individual and organizational measures can be used to mitigate burnout and build resilience. Organizational strategies to promote a healthy work environment include team debriefings after critical events, enabling self-scheduling and time off, limiting the number of

Burnout in healthcare professionals can result from prolonged chronic emotional and interpersonal stressors. It has been defined as a constellation of three dimensions: emotional exhaustion, depersonalization, and diminished feelings of personal accomplishment.1 A number of recent national initiatives and ongoing research highlight the value of proactive strategies for building resilience and promoting a healthy work environment to mitigate burnout in critical care clinicians. The National Academy of Medicine’s Action Collaborative on Clinician Well-Being and Resilience has raised the awareness of the importance of addressing clinician well-being to prevent burnout.2 National surveys continue to demonstrate the high prevalence of burnout among healthcare providers. In a recent Medscape survey on physician burnout, with more than 15,500 physicians surveyed (61% were men and 39% were women), 44% of physicians reported feeling burned out and over 26 specialties had a burnout rate of 33% or higher.3 Among factors reported to impact burnout negatively, physicians most frequently identified having too many administrative responsibilities, long work hours, required documentation in the electronic health record, lack of respect, and insufficient compensation.3 The statistics are similar for many members of the interdisciplinary critical care team, including critical care nurses,4 pharmacists,5 advanced practice nurses, and physician assistants,6 revealing that burnout among critical care providers is significant.

Burnout and Compassion Fatigue in Pediatric Critical Care Providers Several studies have addressed burnout and psychologic distress among pediatric critical care physicians. In a recent national survey7 of 253 practitioners, 49% reported high burnout scores in at least one of the three subscales of the Maslach Burnout Inventory8 and 21% reported severe burnout. The risk of burnout was almost two times more in women physicians (odds ratio, 1.97; 95% confidence interval, 1.2–3.4).7 No other associations between





consecutive shifts worked, and offering team communication training. Individual measures to mitigate burnout include self-care measures such as taking vacation time, ensuring adequate sleep and rest, exercise, healthy eating, and resiliency training.

personal or practice characteristics and burnout were found; however, regular exercise appeared to be protective.7 With respect to psychologic distress, 30.5% of all participants and 69% of those who experienced severe burnout screened positive. Almost 90% of the physicians who reported severe burnout had considered leaving their practice.7 The results of the study demonstrate that a large proportion of pediatric critical care physicians experience burnout and, of those who do, the vast majority have contemplated leaving the profession. As such, physician burnout poses a significant risk to the future of the pediatric critical care physician workforce. The term compassion fatigue is frequently used interchangeably with burnout, but they are not the same thing. Compassion fatigue results when clinicians are exposed to repeated interactions requiring high levels of empathy with distressed patients, which can be a significant contributing factor to caregiver burnout. Compassion fatigue can lead to physical, emotional, and workrelated symptoms that can affect patient care and personal relationships. Broadly, compassion fatigue results in a reduced capacity and interest in being empathetic for those who are suffering and can be framed as the clinician’s emotional and physical cost of caring.9 In a recent national survey of 609 pediatric critical care fellows and attending physicians, the prevalence of compassion fatigue was found to be 25.7%, and the prevalence of burnout was 23.2%.10 Burnout score, emotional depletion, and distress about a patient and/or the physical work environment were each significant determinants of higher compassion fatigue scores, while the compassion fatigue score, distress about administrative issues and/or coworkers, and the belief that self-care is not a priority were each significant determinants of higher burnout scores.10 Not surprisingly, the investigators found that chronic exposure to patient and family distress placed pediatric critical care physicians at risk for compassion fatigue. Several factors specific to pediatric critical care practice can increase the risk of burnout. These factors include providing 183

184

SECTION III



Pediatric Critical Care: Psychosocial and Societal

• BOX 22.1 Risk Factors for Burnout in Pediatric

Critical Care Clinicians • • • • • • • • • • •

Stressful work environment Workload Work hours/demands Managing critical care pediatric patients Ethical and end-of-life issues in a pediatric population Dealing with families of pediatric patients in distress or at end of life Emotional exhaustion Depersonalization Lack of personal accomplishment Compassion fatigue Lack of fulfillment in work

care to children with complex, potentially life-threatening medical conditions; exposure to critically ill children who are perceived as suffering; interacting with families during crisis situations; and engaging in difficult conversations related to transitioning the pediatric patient from curative care to palliative care around end of life (Box 22.1). Both pediatric and adult critical care nurses identify that moral distress is a significant factor impacting burnout in the intensive care unit (ICU), along with providing end-of-life care.4 While the death of any patient is traumatic, the death of a child is often more distressing for pediatric critical care clinicians.11 A recent task force report from the Academic Leaders in Critical Care Medicine Task Force of the Society of Critical Care Medicine identified that the common risk factors for burnout among ICU physicians and nurses are as follows: (1) personal characteristics, such as younger age of nurses and female gender among intensivists; (2) organizational factors, such as the volume and timing of clinical work, including number of nights and consecutive work days; (3) quality of working relationships, such as interpersonal conflicts; and (4) end-of-life issues, such as providing care to the dying patient.12 Similarly, burnout has been found to increase as pediatric critical care providers spend more nights per month in the hospital.13 Taken together, these findings suggest that the current demands of the ICU physical environment along with models of both physician and nurse staffing are contributing to the high rate of burnout in pediatric critical care.

Critical Care Societies Work to Address Burnout

critical care healthcare professionals should be taught how to recognize the risk factors for burnout, mitigate them if able, and taught how to seek assistance when professional support is needed. Finally, the call to action stresses that organizations need to prioritize and promote workplace wellness and encourage healthcare professionals to take individual accountability for maintaining their own emotional and physical health and resiliency.14 Building on the call to action, the CCSC recently sponsored a national summit on the management and prevention of burnout and engaged 55 invited experts in the fields of psychology, sociology, employee assistance, integrative medicine, psychiatry, suicide prevention, occupational medicine, nursing, social work, employee assistance, sleep medicine, bereavement support, among others, to offer expertise and formulate approaches that accelerate actions to address and prevent burnout.15 Through a series of breakout groups, the summit focused on (1) factors influencing burnout among ICU professionals, (2) identifying burnout in ICU professionals, (3) the value of organizational interventions in addressing burnout, (4) the value of individual interventions in addressing burnout, and (5) advancing the research agenda. The national summit addressed the importance of raising awareness among critical care clinicians and key stakeholders, advocating for workplace changes to promote healthy work environments, and promoting research to explore practical strategies further to address, mitigate, and prevent burnout.15 As a follow-up to the national summit, the CCSC conducted a national survey of 680 CCSC members and identified several initiatives currently being implemented both at the hospital and unit levels to build clinician resilience and address burnout prevention. These include the development of unit-based wellness committees, support groups, ICU team-building training, physical exercise, yoga or meditation, and organizational measures such as healthy food choices on campus, on-site exercise facilities, and mindfulness-based stress-reduction courses.15 The literature also reflects a growing number of novel initiatives, such as the creation of Chief Wellness Officer positions. This officer is charged with addressing clinician resilience on a broad basis, initiating community-building activities implemented by clinicians, such as huddles, social outings, group dinners, and peer support groups (Box 22.2).16,17

Strategies for Building Resilience Resilience has been defined as the ability to adapt coping strategies to minimize distress.18 Resilience has been identified as helping individuals mitigate moral distress and burnout.19 Resilience involves external activities such as: • Developing problem-solving skills • Engaging in physical exercise • Instituting stress-reduction measures • Adopting ways of thinking that lessen the impact of stress • Ensuring adequate sleep and rest • Promoting work-life balance or integration Importantly, resilience can be cultivated through self-efficacy, building a sense of hope, and through developing coping skills.20 This has implications for critical care practitioners, as focused measures to build resilience should be taught to practitioners at all levels of training and to actively practicing clinicians in order to better equip them with skills needed to adapt and cope with the increasingly stressful work that exists in the field of critical care medicine.

The Critical Care Societies Collaborative (CCSC), which is composed of the four major critical care–focused US professional and scientific societies—the American Association of Critical-Care Nurses, American College of Chest Physicians, American Thoracic Society, and the Society of Critical Care Medicine— recognized the importance of addressing burnout among critical care professionals and published a call to action in 2016 that reviewed relevant research and addressed potential interventions for mitigating burnout.14 The call to action highlighted the benefit of destigmatizing burnout and linking the consequence of burnout to the root cause of working in a high-stress profession rather than as a character flaw or weakness, a common misperception cited in the literature. Additionally, the publication recommends that













CHAPTER 22

Burnout in ICU Clinicians Organizational Interventions Ensure adequate staffing. Implement flexible scheduling or self-scheduling. Promote a healthy work environment (see Fig. 22.1). Provide communication training. Encourage development of support groups. Suggest cognitive behavioral therapy.

Team-Based Interventions • Hold team debriefings after stressful events. • Use structured communication tools. • Hold team-building and interpersonal skills training.

Individual-Focused Interventions • Limit number of consecutive days of work. • Encourage self-care measures: adequate rest, exercise, healthy eating habits, relaxation techniques, time management, meditation. • Hold assertiveness training and stress-reduction training. • Encourage work-life balance measures: hobbies, family, and social activities.

The American Association of Critical-Care Nurses Healthy Work Environment framework has been adopted to guide ICU practices in many critical care settings. The framework includes six standards: skilled communication, true collaboration, appropriate staffing, meaningful recognition, effective decisionmaking, and authentic leadership (Fig. 22.1).21 Integrating the healthy work environment framework into critical care practice has been identified as a strategy to ensure that the critical care work setting maximizes interprofessional care and teamwork. Research using the healthy work environment standards has identified that the Healthy Work Environment components can predict burnout risk and that meaningful recognition and authentic leadership can predict compassion satisfaction.22 Additional commonly recognized principles of a healthy ICU environment include “avoiding or managing conflicts” and “improving end-of-life care.” A healthy work environment may be enhanced by using such strategies as team debriefings, structured communication, and collaborating with team members on critical decisions.14 Several resources are available for promoting a healthy work culture and addressing clinician resiliency in critical care (Box 22.3).

Optimal patient outcomes

Skilled communication

Clinical excellence

Authentic leadership

Meaningful

Healthy recognition work environment True collaboration Effective decision making

• Fig. 22.1

Appropriate staffing

​Healthy work environment framework. (Modified from American Association of Critical-Care Nurses. AACN Standards for Establishing and Sustaining Healthy Work Environments: A Journey to Excellence. 2nd ed. Aliso Viejo, CA: 2016.)  



185

Promoting a Healthy Work Environment

• BOX 22.2 Interventions to Prevent or Mitigate

• • • • • •

Burnout and Resiliency

186

SECTION III



Pediatric Critical Care: Psychosocial and Societal

• BOX 22.3 Resources for Promoting a Healthy Work

Culture and Addressing Clinician Resiliency in Critical Care • American Association of Critical-Care Nurses. Is your work environment healthy? Aliso Viejo, CA. https://www.aacn.org/nursing-excellence/healthywork-environments. • Critical Care Societies Collaborative Burnout Summit Session recordings. http://ccsconline.org/optimizing-the-workforce/burnout • Podcast. American Board of Internal Medicine. Burnout and Practicing Today’s Medicine. http://blog.abim.org/podcast-burnout-and-practicingtodays-medicine/. • An Official Critical Care Societies Collaborative Statement: Burnout Syndrome in Critical Care Healthcare Professionals A Call for Action. https://pdfs. semanticscholar.org/f647/8105e21acc3c1616e26b0806f018620c538e.pdf • National Academy of Medicine. Communications Toolkit. Clinician Well- Being Knowledge Hub. Washington, DC: National Academy of Medicine; 2018. https://nam.edu/resource-toolkit-clinician-well-being-knowledgehub/. • American Medical Association Steps Forward. Professional Well-Being. Physician Burnout: Improve Physician Satisfaction and Patient Outcomes. https://edhub.ama-assn.org/steps-forward/module/2702509. • American Nurses Association. Healthy Nurse Healthy Nation Grand Challenge. Silver Spring, MD: American Nurses Association; 2018. http://www. healthynursehealthynation.org/. • Accreditation Council for Graduate Medical Education. Improving physician well-being, restoring meaning in medicine. Chicago, IL: Accreditation Council for Graduate Medical Education; 2017. http://www.acgme.org/What-WeDo/Initiatives/Physician-Well-Being.

Summary Pediatric critical care providers are at increased risk of experiencing burnout. Raising awareness of the importance of recognizing risk factors and implementing strategies to mitigate burnout in the pediatric ICU remains a priority area of care. In acknowledging that burnout can result from administering pediatric critical care

and in attempting to build a healthy work environment, pediatric critical care providers can help to promote a better work environment that fosters resiliency. In addition to directly benefitting critical care clinicians, addressing burnout and promoting a healthy work environment can positively impact patient- and family-focused care outcomes.

Key References American Board of Internal Medicine. PODCAST: Burnout and Practicing Today’s Medicine. 2018. Available at: http://blog.abim.org/ podcast-burnout-and-practicing-todays-medicine/. Critical Care Societies Collaborative website with recordings from national summit on addressing burnout in critical care providers http:// ccsconline.org/optimizing-the-workforce/burnout. Critical Care Societies Collaborative. National Summit on Prevention and Management of Burnout in the ICU. Critical Care Societies Collaborative. Available at: http://ccsconline.org/optimizing-the-workforce/burnout. Leckie JD. I will not cry. Ann Am Thorac Soc. 2018;7:786. Available at: https://www.atsjournals.org/doi/full/10.1513/AnnalsATS.201803159IP#readcube-epdf. Linzer M, Guzman-Corrales L, Poplau S. Preventing Physician Burnout. Chicago, IL: American Medical Association; 2017. Available at: https://www.stepsforward.org/modules/physician-burnout. Moss M, Good VS, Gozal D, Kleinpell R, Sessler CN. An official Critical Care Societies Collaborative statement: burnout syndrome in critical care health care professionals: a call for action. Crit Care Med. 2016; 44(7):1414-1421. National Academy of Medicine. Resource Toolkit for the Clinician WellBeing Knowledge Hub. Washington, DC: National Academy of Medicine; 2018. Available at: https://nam.edu/resource-toolkit-clinician-wellbeing-knowledge-hub/. Nguyen NS, Métraux EL, Morris-Singer AF. Combating clinician burnout with community-building. NEJM Catalyst. catalyst.nejm.org/ doi/full/10.1056/CAT.18.0124/.

The full reference list for this chapter is available at ExpertConsult.com.

e1

References









15.











14.







16.





17.





18.



19.







20.



22.





21.









13.





















1. Maslach C, Schaufeli WB, Leiter MP. Job burnout. Annu Rev Psychol. 2001;52:397-422. 2. National Academy of Medicine’s Action Collaborative on Clinician Well-Being and Resilience https://nam.edu/initiatives/clinicianresilience-and-well-being/. 3. Medscape National Physician Burnout, Depression & Suicide Report 2019. https://www.medscape.com/slideshow/2019-lifestyle-burnoutdepression-6011056. 4. Rushton CH, Batcheller J, Schroeder K, Donohue P. Burnout and Resilience among nurses practicing in high-intensity settings. Am J Crit Care. 2015;5:412-420. 5. Bridgeman PJ, Bridgeman MB, Barone J. Burnout syndrome among healthcare professionals. Am J Health Syst Pharm. 2018;75(4): 147-152. 6. Hoff T, Carabetta S, Collinson GE. Satisfaction, Burnout, and turnover among nurse practitioners and physician assistants: a review of the empirical literature. Med Care Res Rev. 2017;76:3-31. 7. Shenoi AN, Kalyanaraman M, Pillai A, et al. Burnout and psychological distress among pediatric critical are physicians in the United States. Crit Care Med. 2018;46:116-122. 8. Maslach C, Jackson SE. MBI: Maslach Burnout Inventory; Manual Research Edition. Palo Alto, CA: Consulting Psychologists Press; 1986. 9. Meadors P, Lamson A, Swanson M, White, M, Sira N. Secondary traumatization in pediatric healthcare providers: compassion fatigue, burnout, and secondary traumatic stress. OMEGA (Westport). 2010;60(2): 103-128. 10. Gribben JL, Kase SM, Waldman E. Weintraub AS. A cross-sectional analysis of compassion fatigue, burnout, and compassion satisfaction in pediatric critical care physicians in the united states. Pediatr Crit Care Med. 2019;20:213-222. 11. Crowe S, Sullivant S, Miller-Smith L, et al. Grief and burnout in the PICU. Pediatrics. 2017;139:e20164041. 12. Pastores SM, Kvetan V, Coopersmith CM, et al. for the Academic Leaders in Critical Care Medicine (ALCCM) Task Force of the Society of the Critical Care Medicine. Workforce, workload, and burnout among

intensivists and advanced practice providers: a narrative review. Crit Care Med. 2019;47:550-557. Rehder KJ, Cheifetz IM, Markovitz BP, Turner DA; Pediatric Acute Lung Injury and Sepsis Investigators Network. Survey of in-house coverage by pediatric intensivists: characterization of 24/7 in-hospital pediatric critical care faculty coverage. Pediatr Crit Care Med. 2014;15:97-104. Moss M, Good VS, Gozal D, Kleinpell R, Sessler CN. An official Critical Care Societies Collaborative statement: burnout syndrome in critical care health care professionals: a call for action. Crit Care Med. 2016;44(7):1414-1421. Kleinpell R, Moss M, Good V, Gozel D, Sessler C. The critical nature of addressing burnout prevention: results from the critical care societies collaborative’s national summit and survey on prevention and management of burnout in the ICU. Crit Care Med. 2019; 47(12):1692-1698. Shanafelt TD, Noseworthy JH. Executive leadership and physician well-being: nine organizational strategies to promote engagement and reduce burnout. Mayo Clin Proc. 2017;92(1):129-146. Nguyen NS, Metraux MA, Morris-Singer AF. Combating clinician burnout with community-building. NEJM Catal. 2018. https:// catalyst.nejm.org/doi/abs/10.1056/CAT.18.0124 Connor KM. Assessment of resilience in the aftermath of trauma. J Clin Psychiatry. 2006;67(suppl 2):46-49. Mallak LA. Measuring resilience in health care provider organizations. Health Manpow Manage. 1998;24(4-5):148-152. Gillespie BM, Chaboyer W, Wallis M. Development of a theoretically derived model of resilience through concept analysis. Contemp Nurse. 2007;25(1-2):124-135. American Association of Critical Care Nurses. Healthy Work Environments Standards. https://www.aacn.org/nursing-excellence/healthy-workenvironments?tab5Patient%20Care. Kelly L, Todd M. Compassion fatigue and the healthy work environment. AACN Adv Crit Care. 2017;28:351-358.

e2

Abstract: It is acknowledged that critical care providers, including those working in pediatric critical care, are at risk for burnout. As burnout often results in high staff turnover, workplace dissatisfaction, and clinician distress, implementing proactive measures to promote healthy work environments, build individual resilience, and mitigate clinician burnout has become a priority in critical care. This chapter presents an overview of strategies to identify

and prevent burnout in pediatric critical care providers and to promote clinician resilience. Tactics to advance and integrate individual and organizational measures to build clinician resiliency and promote a healthy work environment are highlighted. Key Words: burnout, compassion fatigue, resilience, pediatric critical care, healthy work environment, intensive care unit

SECTION

IV

Pediatric Critical Care: Cardiovascular 23. 24. 25. 26.







Structure and Function of the Heart, 188 Regional Peripheral Circulation, 203 Endothelium and Endotheliopathy, 218 Principles of Invasive Cardiovascular Monitoring, 227 Assessment of Cardiovascular Function, 239 Cardiac Failure and Ventricular Assist Devices, 248 Echocardiographic Imaging, 270 Diagnostic and Therapeutic Cardiac Catheterization, 289 Pharmacology of the Cardiovascular System, 300

 

27.

 

28.

 

29. 30.



 

31.

32. 33. 34. 35. 36.

Cardiopulmonary Interactions, 320 Disorders of Cardiac Rhythm, 329 Shock States, 352 Pediatric Cardiopulmonary Bypass, 363 Critical Care After Surgery for Congenital Cardiac Disease, 380 37. Cardiac Transplantation, 411 38. Physiologic Foundations of Cardiopulmonary Resuscitation, 420 39. Performance of Cardiopulmonary Resuscitation in Infants and Children, 444  







 



 

 

 

187

23 Structure and Function of the Heart LUCIANA RODRIGUEZ GUERINEAU, JAYANI ABEYSEKERA, V. BEN SIVARAJAN, AND STEVEN M. SCHWARTZ

Anatomic Development and Structure Segmental Anatomy The normal heart can be divided into three major segments—the atria, ventricles, and arterial trunks—which are connected through atrioventricular (AV) and ventriculoarterial valves, respectively. Each segment has defining components that differentiate the right- and left-sided structures. The atria have five structural components: (1) the venous return (typically, inferior vena cava, superior vena cava, and coronary sinus to the right atrium and pulmonary veins to the left); (2) the appendage and its extent of pectinate muscles (the constant feature of a right vs. left atrium); (3) the septum; (4) the body; and (5) the vestibule continuing with the AV valve. The normal orientation of the ventricles with respect to each other can be described as D-looped or with right-hand topology depending on the nomenclature system that is used, and have three components: the inlet portion, the trabecular zone, and the outlet. In terms of the AV valves, the tricuspid valve (with septophilic attachments and discontinuity with the semilunar valve) is always associated with the morphologic right ventricle and the mitral valve (septophobic) with the morphologic left ventricle. In a structurally normal heart, the ventriculoarterial valves will connect the pulmonary artery to the right ventricle through the leftward and anterior pulmonary valve and the aorta to the left ventricle through the posterior rightward aortic valve. The identification of each segment and connections is the basis of the segmental approach in classification of congenital heart diseases.1 188









The basic form of the human heart and great vessels is complete 8 weeks after conception, after which the structures grow and mature. Immediately after birth, there is a large increase in total body oxygen consumption and cardiac output to approximately twice its values later in life. The determinants of cardiac output—heart rate, loading conditions (preload and afterload), and contractility—influence each other as demonstrated by the Frank-Starling mechanism, force-frequency relationship, end-systolic pressure-volume relationship, and preload reserve.

















PEARLS Although large arteries are regarded as conduits and capillaries as vessels allowing transport of substances to and from the tissues, many substances can move across arterial walls. Standard echocardiographic assessments (ejection and shortening fraction) reflect myocardial performance (loaddependent measures) as opposed to true contractility. Assessments of adequacy of ventricular-vascular coupling (adequacy of contractile status with a given preload given the afterload conditions) can be assessed by noninvasive or invasive methods.

Abnormalities in cardiac segmentation are often accompanied by ipsilateral changes in respiratory and gastrointestinal organ sidedness (i.e., isomerism). The conduction system is formed by superspecialized myocardium that has an enhanced ability to generate and disseminate the cardiac impulse. The normal electrical impulse originates in the sinus node (high right atrium); disseminates through the atrial myocardium; reaches the AV node, where it is delayed; and then rapidly spreads through the bundle of His to bifurcate in its right and left branches and activate the ventricular myocardium. The delay in the AV node gives time for the atrium to contract before initiating ventricular contraction. The AV node is normally the only electrical connection between the atria and ventricles, as the fibrofatty tissues of the AV grooves provide electrical insulation between atrial and ventricular masses. The right and left coronary artery systems provide arterial supply to the myocardium. They emerge from the aorta at two of the three respective coronary sinuses. The left coronary artery typically divides into the circumflex and anterior descending branches. The circumflex provides arterial blood supply to the lateral and posterior wall of the left ventricle and left anterior descending branch to the anterior wall of the left ventricle and anterior part of the interventricular septum. The right coronary artery supplies the right ventricular (RV) free wall and inferior part of the interventricular septum, where it gives rise to the posterior descending artery (90% of the population, termed right coronary arterial dominance). The venous return from the heart is collected into the major veins, which drain in the coronary sinus. Minor cardiac veins from the anterior surface of the right ventricle drain directly

into the RV cavity in the same way that small veins from the atrial walls (the Thebesian veins) open directly into the respective atria.2

Innervation of the Heart The heart has sympathetic and parasympathetic innervations that are tonically active and responsible for physiologic changes in heart rate. Adrenergic (sympathetic), muscarinic (parasympathetic), and other receptors appear early and are functional even before innervation. Parasympathetic innervation precedes sympathetic innervation in all species.3–5 Innervation is present in the earliest viable human premature infants but may not be fully mature. Cardiac sympathetic nerve fibers arise from cervical sympathetic and stellate ganglia. Vagal nerve fibers descending from medullary centers supply both atria and ventricles and the proximal portion of the bundle of His. The distal part of the bundle of His has only sympathetic nerve supply. Sympathetic and vagal afferents leaving the heart carry information from baroreceptors that respond to high pressures in the ventricles and to lower pressures in the atria, cavae, and pulmonary veins as well as from chemoreceptors that respond to locally produced substances, such as bradykinin and prostaglandin.6

Ductus Arteriosus The ductus arteriosus forms from the embryonic left sixth aortic arch and joins the main pulmonary artery. The ductus is kept open by a balance between prostaglandin E2 (PGE2) and endothelin-1 (ET-1), both of which are formed in its wall and circulate from other sites. Initially, the ductus is sensitive to the dilating action of PGE2. However, later in gestation, it becomes less sensitive to dilator and more sensitive to constrictor prostaglandins.7–9 After birth, oxygen reacts with a cytochrome P-450 and causes release of ET-1 (the most powerful ductus constrictor).10 A switch from dilator to constrictor prostaglandins occurs. In addition, oxygen modulates the function of mitochondrial electron chain transport, causing a net influx of calcium and, ultimately, ductal constriction.11–13 These constrictor effects overpower the dilating effect of nitric oxide, which is released from the ductus when oxygen tension rises.14 The ductus constricts, usually within the first 24 hours and almost invariably within 3 weeks. The lumen then becomes permanently occluded by fibrosis.9,15



CHAPTER 23

Structure and Function of the Heart

189

afterload on mural thickness. The ventricular septum is flattened in the fetus. After birth, it bulges into the right ventricle and functions like part of the left ventricle. Muscle fibers in the ventricles form a complex helical array. Fibers in the left ventricular (LV) midwall are circumferential, parallel to the AV groove. From this position, the fibers twist gradually as they move toward each surface so that at the epicardial surface they are 75 degrees and at the endocardial surface 60 degrees from the circumferential fibers (analogous to the motion of wringing of a towel).17 This disposition of fibers is the basis of the twist or torsional LV deformation during systole. Some investigators believe that the muscle fiber layers form one continuous sheet that is wrapped around itself like a turban.18 When the ventricle is dilated, the fiber angles change and become less effective in ejecting blood.19

Microscopic Anatomy The heart wall is made of a thick myocardium consisting of muscle fibers intermingled by fibroblasts that contribute to the formation of the extracellular matrix, the internal endocardium, which creates a nonthrombotic surface and external epicardium covering the heart. The myocardium works as a syncytium made of branching fibers, each consisting of bundles of myocytes in series. The myocytes are joined to adjacent myocytes by a specialized junction: the intercalated disc that provides mechanical connection by the adherens junctions (desmosomes and other membrane proteins) and electrical connection by the gap junction (connexins and N-cadherin).20–22

Cardiomyocyte Cardiomyocytes are specialized cardiac cells that can be divided in two cell types: those forming the conduction system and those performing the contractile work. The major components of the myocyte are the sarcomeres, which contain the myofibrillar contractile apparatus; the mitochondria, which house enzymes for energy production; the sarcoplasmic reticulum; and the cytosol. The sarcolemma is the cell membrane, which has extensions into the cytoplasm. The numerous proteins in these structures not only play a role in normal function but, if abnormal for genetic or extraneous reasons, contribute to myocardial dysfunction.23

Development of the Human Heart

Contractile Apparatus

The basic form of the human heart and great vessels is complete 8 weeks after conception, after which the structures grow and mature. In the embryo, coronary arteries form from an aortic peritruncal plexus and join the aorta to supply flow to the thickening heart muscle, which can no longer get enough blood from sinusoids from the ventricular cavity. The ventricular mass enlarges by cellular hyperplasia (cell division) and hypertrophy (cell enlargement). Hyperplasia is the major mechanism for which ventricular mass increases in the fetal heart, transitioning to hypertrophy as the dominant mechanism in postnatal life.16 Ventricular growth is believed to depend on flow, as conditions that divert flow from a ventricle are usually associated with hypoplasia of that ventricle and its associated great artery. Before birth, the left and right ventricles have equal wall thickness. After birth and clamping of the umbilical cord, there is a rise in systemic vascular resistance and a decrease in pulmonary vascular resistance. As a result, the left ventricle becomes thicker than the right ventricle, highlighting the impact of workload and

The cardiomyocytes are filled with aligned contractile myofibrils. Each myofibril is composed of thin (mainly actin) and thick (myosin) filaments organized into contractile functional units called sarcomeres. The sarcomere is defined as the structure between two transverse Z lines.24–26 On each side of the Z line is a light zone, the I (isotropic) band, and in the center of the sarcomere are two dark zones, the A (anisotropic) bands, separated by a light H band in the middle of which is a dark, thin M band. The I bands contain paired thin filaments of actin coiled in a helix and attached to the Z lines. Two long tropomyosin filaments lie in the grooves between each pair of actin filaments (Fig. 23.1).27 Every 400 Å, near the crossover points of two actin filaments, is a troponin complex with the following three distinct troponins: (1) troponin T, which binds troponin to tropomyosin; (2) troponin I, which inhibits actin-myosin interaction; and (3) troponin C, which is a high-affinity calcium receptor. The thin actin filaments overlap with thick myosin filaments at the A bands. These myosin filaments are composed of light and heavy chains. The

190

SECTION IV



Pediatric Critical Care: Cardiovascular

Strain

Plasma membrane

Integrin

Ca2+ Troponin T

Troponin C Troponin I

Actin

Thin filament α-Tropomyosin Myosin-binding β-Myosin protein C heavy chain Thick filament

PEVK region Calsarcin 1 T-cap Calcineurin

Titin

MLP cGMP PKG

PDE5

α-Actinin Z-disk

• Fig. 23.1

​Integration of myofibril contraction (actin-myosin complex formation) to attachments to the Z lines and cytoskeleton via various ultrastructural proteins. Ca21, Calcium ion; cGMP, cyclic guanosine monophosphate; MLP, muscle LIM protein; PDE5, phosphodiesterase 5; PKG, protein kinase G; T-cap, titin cap. (Modified from Mudd JO, Kass DA. Tackling heart failure in the twenty-first century. Nature. 2008;451:919–928.)  



light chains coil around each other to form the long core of the myosin molecule. The heavy chains form globular myosin heads that project from the sides of the thick filament toward the actin molecules (see Fig. 23.1). A collar of cardiac myosin-binding protein C encircles the thick filaments. Mutations of this protein are a common cause of hypertrophic cardiomyopathy.28 Between two A bands, there is usually a thin, lighter band, the H band, which has myosin but no actin filaments.26,27 Titin is a giant protein and is the third most abundant fibrillar protein. It extends from the Z band to the M band, has two isoforms, and is the main protein responsible for the elastic behavior of the myocyte.29 It is essential for sarcomere assembly and for sensing sarcomere length since it is speculated to be the major regulator of thick filament length30,31 and, with myomesin (not shown in Fig. 23.1), supports the actomyosin filaments (see Fig. 23.1). Titin mutations, at present, are one of the most common etiologies of inherited dilated cardiomyopathy.32,33

Sarcolemma and Sarcoplasmic Reticulum The cell membrane contains receptors, ion channels, pumps, and exchangers. It has indentations overlying the Z bands; from these indentations, small tubules termed T tubules (for transverse) penetrate the cell. Abutting against the T tubules are dilated expansions of the sarcoplasmic reticulum (junctional reticulum or cisternae), which join the free sarcoplasmic reticulum, a network

of longitudinal tubules inside the cell that surround the thick (myosin) filaments. These tubular systems modulate the entry of calcium to, or its exclusion from, the cytoplasm.3,24 The cisternae contain the calcium-binding protein calsequestrin, whereas the longitudinal tubules contain phospholamban and the adenosine triphosphate (ATP)-dependent calcium pump.3,34 Phospholamban inhibits the affinity of the sarcoplasmic reticulum Ca21-adenosine triphosphatase (SERCA) pump for calcium, and phospholamban phosphorylation relieves the inhibition and increases calcium entry, with a resulting increase in inotropy.35–38 In heart failure, phospholamban phosphorylation is decreased,39 leading to decreased SERCA activity.40,41 A similar decrease in SERCA has been found in sepsis42 and in some forms of dilated cardiomyopathy.43 Cisternae store and release activator calcium, whereas longitudinal tubules remove calcium from the cytosol. Calcium release is primarily via the calcium-activated calcium release channel termed the ryanodine receptor. Mutations in the ryanodine receptor and calsequestrin genes are implicated in catecholaminergic polymorphic ventricular tachycardia.44 Both T tubules and sarcoplasmic reticulum are sparse, undifferentiated, and disorganized early in gestation but increase and differentiate markedly late in gestation and after birth in mammals. Therefore, the immature heart depends mainly on extracellular sources for activator calcium,3,27 partly explaining its marked calcium sensitivity.



CHAPTER 23

191

Structure and Function of the Heart

Cytoplasm During development, the proportion of mitochondria in the myocyte increases, particularly at the time of birth, and mitochondria become larger and develop more complex cristae.3 In the adult, approximately 30% to 40% of the muscle mass is made up of mitochondria. The cytosol contains other calcium-binding proteins3,45 and other major proteins, such as tubulin and desmin.

adhesion proteins, stimulated by growth factors from the myocyte, are present in greatest amount in the neonate, decrease with postnatal age, and increase again during hypertrophy.62,63 Other elements in the extracellular matrix (e.g., laminin, fibronectin, and tenascin) play a major role during morphogenesis and during contraction64 and are important mediators in hypertrophy.

Cytoskeleton and Extracellular Matrix For contractile proteins to shorten the entire myocyte, they must be linked to both the cell membrane and extracellular matrix. Longitudinal connections are made via the Z lines, representing disks that contain proteins such as a-actinin and filamin, which connect the actin and titin filaments of adjacent myocytes.46,47 More lateral connections are made by the extrasarcomeric skeleton. There is an intermyofibrillar cytoskeleton with intermediate filaments, microfilaments, and microtubules.46,48–50 Desmin intermediate filaments provide a three-dimensional scaffold throughout the extrasarcomeric cytoskeleton and connect longitudinally to adjacent Z disks and laterally to subsarcolemmal costameres.48,50 Costameres are subsarcolemmal domains that contain different adhesion complexes connecting the cytoskeletal actin filaments with transmembrane proteins.51–53 These proteins help to fix sarcomeres to the lateral sarcolemma, stabilize the T-tubular system, and connect the sarcolemma to the extracellular matrix. In many of the genetic dilated cardiomyopathies, these proteins are abnormal,54–56 which impacts muscle function. Extracellular collagen plays a major role in cell-cell and cellvessel interactions and in ventricular stiffness.57–60 The relationship between sarcomeres and cytoskeleton changes with maturation, perhaps accounting for maturational differences in the resting sarcomere’s mean length in myocytes.61 Additionally, cell

Physiologic Development and Function Myocardial Mechanics: Cardiac Sarcomere Function Excitation-Contraction Coupling As the electrical impulse propagates through the cardiac muscle, myocyte membranes depolarize. Extracellular calcium at the sarcolemmal membrane and T tubules enters the intracellular space rapidly. Spread of electrical excitation into the myocyte via the T tubules also causes release of intracellular calcium from the sarcoplasmic reticulum.24,65,66 Cytosolic calcium increases from a concentration of 1027 M in diastole to 1025 M in systole. When calcium then binds to troponin C, the inhibitory effect of troponin I is antagonized, and a conformational change of troponin and tropomyosin exposes the actin-myosin binding sites.24,27,65,66 These sites interact with the myosin heads to form cross-bridges (Fig. 23.2). The myosin heads rotate, generate force, and move the actin filaments, just as oars move a boat through the water. Interaction between actin and myosin pulls the two Z lines toward each other, generating force and shortening the muscle. Increasing intracellular calcium results in greater cross-bridge formation and a greater generated force. Isoforms of the troponins and tropomyosin change during devel-

Calcium Influx and Phosphorylation Ca2+

Contraction Cycle

Calmodulin Ca2+

ADP

Pi

Channel

Myosin kinase

IP3 Phospholipase C

ADP ATP

Pi

Myosin Pi

ADP

Myosin phosphatase (dephosphorylation)

Pi

Binds actin

Ca-calmodulin

SR

Pi

Pi

ADP

Myosin phosphatase

ADP

Pi

P

ATP

• Fig. 23.2

Pi

ATP Pi

​Cardiomyocyte calcium cycling and adenosine-triphosphate (ATP) utilization during actinmyosin complex formation. ADP, Adenosine diphosphate; Ca21, calcium ion; Pi, phosphate ion.  

Receptor

Latch State

"Latch state"

192

SECTION IV



Pediatric Critical Care: Cardiovascular

opment, but the functional effects of these changes are unknown.3,67 Troponin I is less sensitive to a fall in pH in the fetus than in the adult, which could be protective in perinatal acidosis. The myosin head contains an adenosine triphosphatase (ATPase) that liberates energy from ATP. The activity of the ATPase determines the velocity of shortening of unloaded muscle by affecting the rate of attachment and detachment of the crossbridges.3,68

the muscle must work against after contraction has been initiated, or afterload, is created using weights added to the lever after initial length is set. Stretching relaxed muscle produces an exponentiallike increase in passive tension (Fig. 23.3B). This elasticity results mainly from titin.29,69–71 At very low sarcomere lengths, the actin filaments from each Z line overlap each other (1.6 and 1.8 mm in Fig. 23.3C). As the sarcomere lengthens, the Z lines move farther apart, and a gap appears between the two sets of actin filaments (2.5 mm in Fig. 23.3C). When the sarcomere reaches a length of approximately 2.2 mm, there is a maximal overlap between actin and myosin filaments26,66,68 (2.2 mm in Fig. 23.3C). At longer muscle and sarcomere lengths, actin and myosin filaments overlap less. The maximal sarcomere length is 3.0 mm. Further elongation of the muscle occurs by slippage of fibers and not by further sarcomere lengthening.26,66,68

Sarcomere Length-Tension Relationships Sarcomere length-tension relationships have been investigated using isolated cardiac muscle strips. Most commonly, a papillary muscle is connected between a lever and a force transducer (Fig. 23.3A). Loading before contraction, or preload, is adjusted using weights attached to the other end of the lever. Loading that Micrometer lever stop

100

Length transducer

Increased contractility Tension (percent of maximum)

Lever

Electrode plates Papillary muscle Oxygen supply

0 80

60

B Z

1.0 µm Thin filament

1.5 µm bridges

100 (Lmax)

Percent of muscle length 1.0 µm

Z

Thick filament

3.5 µm

2.5 µm

H zone

    

A

Active tension

Resting tension

Afterload Preload

Force transducer

Control

2.2 µm Optimum "overlap"

2.0 µm 1.8 µm 1.6 µm

C • Fig. 23.3

Double overlap

​(A) Isolated muscle strip in a water bath and attached to transducers for measuring force and length. Preload is set by the lever stop. (B) Relationship between muscle length and resting tension or active tension at three different contractile levels. (C) Relationships of sarcomere length, positions of the actin and myosin filaments, and contractile force. (A, From Parmley WW, Tyberg JV. Determinants of myocardial oxygen consumption. In: Yu PN, Goodwin JF, eds. Progress in Cardiology. Philadelphia: Lea & Febiger; 1976. B and C, Modified from Sonnenblick EH. Myocardial ultrastructure in the normal and failing heart. In: Braunwald E, ed. The Myocardium: Failure and Infarction. New York: HP Publishing; 1974.)  

Myocardial Mechanics: Myocardial Receptors and Responses to Drugs Cardiac myocytes express a- and b-adrenoceptors that mediate the responses to endogenous and exogenous catecholamines and therefore are involved in the rapid regulation of myocardial function. They can also be inhibited by adrenergic-blocking agents such as b-blockers and are a target in treating heart failure and arrhythmias. a1-Adrenoceptors appear early in gestation and in many species reach their highest density in the neonate.3,4,72 In contrast, b-adrenoceptors increase progressively with age. b1, b2, and b3 are present on myocytes, but b1-adrenoceptors are described as the predominant receptor subtype in cardiac myocytes.73,74 In addition, histamine H2, vasoactive intestinal polypeptide (VIP), adenosine A1, acetylcholine M2, and somatostatin receptors have been identified. They act on the myocyte’s contractile apparatus through one of two main pathways. The major pathway involves membrane-bound receptor–G protein–adenylate cyclase complexes. G proteins include the Gs (stimulatory) and Gi (inhibitory) proteins.75 When agonists stimulate b-adrenergic receptors, the G proteins undergo a conformational change. The changes induce the Gs protein to exchange its guanosine diphosphate (GDP) for guanine triphosphate (GTP). The Gs-a-GTP complex interacts with adenylate cyclase to convert ATP to cyclic adenine monophosphate (cAMP), which activates a variety of protein kinases to phosphorylate proteins, including voltage-dependent calcium channels, phospholamban, and troponin I. Consequently, calcium entry during depolarization and during uptake of calcium into the sarcoplasmic reticulum storage pool is increased, thus increasing contractility. The Gs-a-GTP complex has intrinsic GTPase activity that converts GTP to GDP. In this way, as long as receptors are occupied by the agonist, the Gs cycle produces increasingly more cAMP, amplifying the stimulatory signal. The Gi protein complex undergoes a similar cycle when adenosine or acetylcholine receptors are stimulated. However, activating Gi protein reduces cAMP formation and decreases contractility. b2-adrenergic receptors also couple to Gi in addition to Gs.76 Gi in this context is thought to oppose the effects of Gs to some degree, including limitation of the acute positive inotropic response to adrenergic stimulation and offering some protection from apoptosis.77–79 In heart failure, the number of b1-adrenergic receptors are downregulated, and b2-adrenergic receptors are uncoupled from G proteins.35,73,74,80,81 These changes make the myocardium less responsive to circulating or locally released catecholamines and play a role in the reduced contractility observed in heart failure. Treating heart failure with b-adrenergic blocking agents has been shown to reverse the receptor changes and has also been associated with improved function of muscle strips in adult patients.82–85 b3-adrenoceptors, the minor isoform in the heart, can activate different signaling pathways; their role in heart failure therapy is a recent topic of study.86,87 Milrinone is an agent that stimulates contractility by inhibiting phosphodiesterases and increasing cAMP. Although previously thought to be ineffective in the newborn,88 a multicenter randomized trial89 and subsequent widespread use has confirmed its efficacy in the neonatal population.90,91 Because contractile mechanisms are almost fully developed at birth, the majority of mechanisms controlling contractility (except for changes in the source of calcium) are in place at birth.



CHAPTER 23

Structure and Function of the Heart

193

Myocardial Mechanics: Integrated Muscle Function Relationship Between Muscle Strips and Intact Ventricles With preload, stretching a muscle strip is equivalent to end-diastolic fiber length of the intact ventricle. This length can be measured by various devices in animals; however, in the intact human ventricle, it is best related to end-diastolic diameter or volume. Frequently, end-diastolic pressure has been used interchangeably with enddiastolic volume as an index of preload, but this usage can be misleading if the distensibility of the ventricle changes or if pressure outside the heart (pericardial or intrathoracic) rises.92–95 Afterload is more complicated in the intact ventricle, commonly defined as the pressure or load against which the heart contracts to eject blood. Often, aortic systolic pressure is equated with afterload. However, in the muscle strip, afterload represents the force exerted by the muscle during contraction; in physical terms, force 5 pressure 3 area.71,96–98 Therefore, afterload accounts for the distribution of pressure over the surface to which the force is applied. When the force is applied to a thick spherical chamber, it is more accurately described by the Laplace wall stress relationship: Wall Stress ( ) 

Pr 2h

where P is the transmural pressure across the wall of the chamber, r is radius of curvature, and h is wall thickness. Because the left ventricle is not a regular sphere, particularly in systole, the Laplace formula is an oversimplification.99 A fairly simple and accurate formula was developed by Grossman and colleagues100: Wall Stress (  ) 

Pr h  2h  1   2r  

Note that if the left ventricle dilates acutely, wall stress rises markedly because r gets bigger and h gets smaller. The major findings from studies of muscle strips have been confirmed in intact ventricles. Increasing preload increases the pressure generated by an isolated ventricle that is not allowed to eject, as observed in the 20th century by Otto Frank. If the ventricle is allowed to eject, then increased preload allows the heart to eject the same stroke volume against an increased afterload or to eject a greater stroke volume against a constant afterload. This is the Starling component of the Frank-Starling law.101,102 The mechanism of this response is twofold: (1) lengthening the sarcomere places the myosin and actin fibrils closer together for stronger interaction and (2) increased calcium sensitivity is mediated in some way by titin stretching.30 Therefore, stretching the sarcomere to its optimal length (see Fig. 23.3C) will increase contractility. Beyond that length, further stretching (or preload) can be detrimental as the end-diastolic pressure increases without a significant change in the stroke volume. There is evidence that in failing hearts, this relationship between preload and developed tension may be absent.103 The force-frequency relationship is a property of the cardiomyocyte whereby heart rate modifies contractility. Heart rates up to an optimal heart rate increase the force of contractility beyond which force decreases. The force-frequency relationship can be determined in intact hearts104,105 by examining the response of the

194

SECTION IV



Pediatric Critical Care: Cardiovascular

maximal rate of change of pressure (dP/dt max) in the ventricles after premature beats. The results in intact ventricles and muscle strips are similar. Subsequently, Seed and colleagues106 applied this technique to humans with normal or abnormal LV function and found an optimal R-R interval of 800 ms. This response is mediated by an increase in the intracellular calcium in normal hearts but is limited in the setting of ventricular dysfunction. As with preload, the force-frequency relationship may be abnormal in failing hearts.107,108

Pressure

I 3

1

V0

A

D

1

2

3

Volume I

Pressure

Pressure-Volume Loops If LV pressure and volume are measured simultaneously, the resulting pressure-volume loop gives information about ventricular performance and can be used to assess myocardial contractility in the intact heart. The modern approach to analyzing these loops is based on the elastance concept of Suga and Sagawa.109–111 Elastance is the ratio of pressure change to volume change (the reciprocal of compliance). Consider an isolated ventricle that can be filled to different volumes. At each volume, the ventricle is stimulated to contract and generates a peak systolic pressure (Fig. 23.4A). As volumes increase (1 n 2 n 3), so do the peak systolic pressures generated, and the relationship is linear (Frank’s law). The line joining the peak pressures intercepts the volume axis at a positive value, termed V0, that indicates the unstressed volume of the ventricle. The equation for this line is as follows: Pes  E es (Ves  V0 )

2

2

3

1

D EDV

V0

B

1

2

3

4

Pressure

Volume

2

3

1

EDV EDV

V0

C

1

2

3 I

II

III

Volume

• Fig. 23.4 ​Concept of ventricular elastance. (A) Isolated ventricle contracting at volumes 1, 2, and 3, generating corresponding pressures. Purple line I indicates results at increased contractility. Blue line D indicates results at decreased contractility. (B) Ventricular pressure-volume loops achieving end-systolic pressures of 1, 2, and 3 at corresponding volumes. Purple line I indicates results at increased contractility, with greater endsystolic pressures at each volume. Blue line D indicates results at decreased contractility. From a given end-diastolic volume, either the same ejection fraction is delivered at a lower end-systolic pressure (dotted line 1) or the same end-systolic pressure is achieved but at a much smaller stroke volume and ejection fraction (line 4). (C) As a consequence of afterload increase, the ventricular end-diastolic volumes (EDV) increase, then stroke volume can be maintained, even though ejection fraction decreases. If contractility is decreased (blue line), then stroke volume can be maintained only with increasing end-diastolic pressures. 1, 2, 3, Endsystolic volumes and pressures at normal contractility; I, II, III, end-systolic volumes at decreased contractility; V0, resting (unstressed) volume.  

where Pes is end-systolic pressure, Ees is slope of the line, the endsystolic elastance or the maximum elastance (Emax), Ves is endsystolic volume, and V0 is unstressed volume. If contractility increases (more calcium enters the cells), the ventricle can generate greater pressures at any given volume, thereby generating a steeper pressure-volume line (higher value of Ees; purple line I in Fig. 23.4A). If contractility decreases, the ventricle generates lower pressures at any given volume, and the pressure-volume line is less steep (lower value of Ees; blue line D in Fig. 23.4A). The typical pressure-volume loop shown in Fig. 23.4B is characterized by four phases marked by the opening and closing of the AV and arterial valves: diastole starts when the aortic and pulmonary valve close and the ventricular pressures fall owing to muscle relaxation; initially, with the AV valves closed, the isovolumic relaxation phase occurs since both inlet and outlet valves are closed with no change in ventricular volume. This phase is followed by opening of the AV valve when atrial pressures are higher than ventricular pressures and initiation of ventricular filling. During diastolic filling, volume increases and diastolic pressure rises slightly because of the increase in passive tension. At the end of diastole, when the ventricular pressures surpass atrial pressure, AV valves close and isovolumic contraction starts. In this phase, ventricular pressure rises with no change in volume. When ventricular pressure exceeds aortic pressure, the aortic valve opens, blood is ejected, and ventricular volume decreases. Ejection ends, and pressure falls to diastolic levels as isovolumic relaxation occurs and the cardiac cycle restarts. The decrease in volume during ejection is the stroke volume, which, divided by the end-diastolic volume, gives the ejection fraction; normally, ejection fraction is greater than 65. If afterload is suddenly increased by raising aortic pressure, the normal heart responds as shown in Fig. 23.4B. In the first beat after the increase, the ventricle must generate a higher pressure before the aortic valve opens (loop 2; orange line). It then ejects but

cannot eject the same stroke volume. In fact, the end-systolic volume is that which is appropriate for the higher pressure (compare Fig. 23.4B, end-systolic volume at 1 and 2 in the orange line). If different afterloads are used, the end-systolic pressure-volume points define a sloping line that is the same as the line obtained in the isolated heart at those same volumes. This is the maximal ventricular elastance (Ees) or end-systolic elastance (Ees) line. If ventricular contractility increases, then the ventricle can attain higher ejection pressures at any given volume, and the end-systolic pressure-volume points lie on a steeper line that lies above and to the left of the normal line (purple line I in Fig. 23.4B). If ventricular contractility decreases, then the end-systolic pressure-volume line lies below and to the right of the normal line (blue line in Fig. 23.4B). Note from Fig. 23.4B that, from a given end-diastolic volume, the ventricle with impaired contractility can either eject the original stroke volume at much reduced pressures or eject at a normal pressure only by reducing its stroke volume drastically (loop 4).

In beats that follow a sudden increase in afterload, the ventricles adjust (Fig 23.4C). Because of the reduced stroke volume in the first beat following afterload increase, the end-systolic volume is larger than normal. During diastole, however, a normal stroke volume enters the ventricle so that end-diastolic volume increases (loop 2 in Fig. 23.4C). In normal ventricles, the increased enddiastolic fiber length causes little increase in diastolic pressure. The pressures during ejection and the end-systolic pressurevolume point are unchanged, but stroke volume and ejection fraction increase. After a few more cycles, a new equilibrium is established (loop 3) in which the ventricle ejects a normal stroke volume at the higher afterload. However, the ejection fraction is subnormal because, although the stroke volume is normal, the end-diastolic volume is increased. The ventricle has adapted to the higher afterload by increasing end-diastolic fiber length, a phenomenon described by Starling and discussed by Ross96,97 under the term preload reserve. If the ventricle has decreased contractility (dashed loops, blue line in Fig 23.4C), the same pattern of response occurs but with some important differences. With decreased contractility, the ventricle cannot eject a normal stroke volume from a normal end-diastolic volume. Compensation results in a larger than normal increase in end-diastolic volume, even at normal afterloads. Any increase in afterload causes a further increase in end-diastolic volume; this increase causes diastolic pressures to rise to high values that cause pulmonary congestion. The normal preload reserve has been used up in the attempt to eject a reasonable stroke volume against a modestly increased afterload. In more depressed hearts, even normal afterloads cannot be handled by the ventricle without a pathologically raised diastolic pressure in the ventricles or a drastic decrease in stroke volume. Note that in these hearts, because of the relatively flat slope of the maximal ventricular elastance line, a slight reduction of afterload produces a relatively large increase in stroke volume and a relatively large decrease in ventricular end-diastolic volume and pressure. This is one of the mechanisms for cardiac improvement with afterload reduction in the setting of systolic dysfunction. The normal RV pressure-volume curve is triangular, unlike the more rectangular LV pressure-volume curve described earlier.112 This difference is accounted for by a relative lack of isovolumic contraction and relaxation times in the right ventricle. The normally low afterload of the right ventricle and the high compliance of the outflow portion of the ventricle allow ejection to begin almost instantaneously after the onset of contraction and proceed through pressure decline so that there is near complete emptying of the ventricle by the end of systole and the ejection time of the right ventricle thus spans the entire period of systole. An important consequence of this relationship is that even small increases in RV afterload begin to make the RV pressure-volume curve resemble the normal LV pressure-volume curve, with isovolumic contraction and relaxation times becoming more prominent.113 Ejection fraction is reduced, although stroke volume may be maintained due to RV dilation,114 and the thin-walled right ventricle may handle this new physiology quite poorly.

Assessing Myocardial Contractility: Systolic Ventricular Function An index of contractility must reflect the ability of the ventricle to perform work independent of changes in preload and afterload. Contractility can be defined as the alterations in cardiac function that occur secondary to changes in cytosolic calcium availability or sarcomere sensitivity to calcium. Thus, b-adrenergic agonists or phosphodiesterase inhibitors, which increase cytosolic calcium,



CHAPTER 23

Structure and Function of the Heart

195

are positive inotropic agents. However, quantifying contractility in the intact heart is difficult115 because all indices of contractility are indices of overall performance of cardiac function and are not independent of loading conditions and heart rate. The relationship between loading conditions and contractility is complex since the handling of calcium within the myocyte is influenced by (1) the myocardial fiber length, which, in turn, depends on the preload (Frank-Starling mechanism)116; and (2) afterload, as it has been demonstrated that contractility increases in response to a rise in the afterload.117 Despite these limitations, the methods we currently have to assess contractility can be divided into those occurring in early systole during isovolumic contraction (isovolumic phase indices) and those that occur later, during ejection (ejection phase indices). Isovolumic Phase Indices

The maximal rate of change of ventricular pressure (dP/dt max) is achieved during the isovolumic contraction, before the aortic valve opens, and is relatively unaffected by changes in preload. It can be measured by invasive pressure tracing or, indirectly, from a continuous-wave Doppler tracing of a mitral regurgitation jet by echocardiography. However, the index is markedly affected by changes in afterload. Thus, it must be used with caution when afterloads are very different. This method is more useful for measuring acute changes in contractility than for assessing absolute contractility. Ejection Phase Indices

The index of contractility most commonly used today is the maximal (end-systolic) ventricular elastance of Suga and Sagawa, which is defined by the slope of the end-systolic pressure-volume relationships. Measurements must be obtained at several different levels of afterload, and either ventricular volumes must be measured or echocardiographic dimensions must be used as substitutes for volumes.71,98 Several studies have shown that the maximal elastance line often is not linear, as previously stated,118,119 but the values of elastance in the midrange of pressures are accurate enough to use the slope of the end-systolic pressure-volume relationship as a parameter of left ventricular contractility.119 The LV end-systolic wall stress-velocity of fiber shortening relation as a single beat index of contractility has also been used. This index is not exempted from limitations, which arise from the need to adjust for changes in afterload,120 and single-point determinations are of little use since the relationship is not linear (Fig 23.5).117 Echocardiographic measurements of ventricular function are most commonly used in clinical situations. These are load-dependent measurements of contractility. These techniques measure global and regional function. They can be subdivided into those that are based on dimensional and volume changes (i.e., shortening and ejection fraction, RV fractional area change) and those that are Doppler based (such as dP/dt max).121 M-mode–generated ejection fraction is a popular method used in children to assess LV function noninvasively. Even though it is useful, it is a loaddependent measurement and is less accurate in the setting of mitral regurgitation, dysynchrony, regional wall motion abnormalities, and LV dilation (see Chapter 31 for more details). M-mode–derived tricuspid annular plane systolic excursion (TAPSE) is also an easily derived measurement of RV systolic function. Cardiac magnetic resonance imaging can be considered as an additional imaging modality if more accurate ventricular volumes and measurements of systolic function are required.

196

SECTION IV



Pediatric Critical Care: Cardiovascular

1.6

Vcfc = −0.0044 σes + 1.23 r = −0.84 n = 118

Rate corrected Vcf (circ/s)

1.4 +2SD 1.2

Mean −2SD

1.0

0.8

0.6 0

20

40

60

80

100

120

LV end-systolic wall stress (g/cm2)

A

* Low afterload

B

*

End-systolic wall stress

1*

* Low afterload

Normal afterload High afterload

Normal afterload High afterload

Mean Vcfc

2

*

Mean Vcfc

Mean Vcfc

1

End-systolic wall stress

2

*

End-systolic wall stress

• Fig. 23.5

​(A) Relationship between rate-corrected mean velocity of fiber shortening (Vcf) and left ventricular (LV) end-systolic wall stress. (B) Possibility of misinterpreting the relationship between mean velocity of fiber shortening and end-systolic wall stress. Left, Data point 1 is more than two standard deviations (SD) above normal relation (taken from left panel), suggesting increased contractility. Data point 2 is within the normal range, suggesting normal contractility. Middle, Alternative explanation for point 1 is that contractile state is normal, but points obtained at very low afterloads follow a hyperbolic, not a linear, relationship. Right, Alternative explanation for point 2 is that contractility is decreased. However, because of the hyperbolic relationship and the low afterload, it appears within the “normal” linear range. (A, From Colan SD, Borow KM, Neumann A. Left ventricular end-systolic wall stress-velocity of fiber shortening relation: a load-independent index of myocardial contractility. J Am Coll Cardiol. 1984;4:715–724. B, Modified from Banerjee A, et al. Nonlinearity of the left ventricular end-systolic wall stress-velocity of fiber shortening relation in young pigs: a potential pitfall in its use as a single-beat index of contractility. J Am Coll Cardiol. 1994;23:514–524.)  

Assessing Myocardial Relaxation: Diastolic Ventricular Function Diastolic function refers to the rate and extent of ventricular relaxation.93,122 Many forms of heart disease manifest abnormalities of both systolic and diastolic function, but one or the other form of dysfunction may predominate and impact optimal therapy.123 Diastolic dysfunction results in increased ventricular diastolic pressure at normal or even low ventricular volume122 either from an increase in passive stiffness of the ventricles from chronic infiltrates (e.g., amyloid), myocardial scars, constrictive pericarditis, or diffuse myocardial fibrosis, or from impaired relaxation or AV dyssynchrony. Diastolic relaxation of ventricular muscle associated with rapid release of calcium from troponin and its subsequent uptake by the sarcoplasmic reticulum allows actin-myosin cross-bridges to dissociate and the sarcomeres to lengthen, permitting the ventricle to

relax. Impairment of calcium removal due to abnormalities in major contractile proteins or transport processes decreases the rate and extent of relaxation.69,124 Many heart diseases, including ischemia, impair calcium metabolism and diastolic ventricular function.40,43,105,125 The most common methods to assess diastolic function are invasive measurement of end-diastolic LV pressure in the catheter laboratory or noninvasively by measurements of tissue Doppler indices, ventricular inflow, and systemic and pulmonary venous Doppler profiles. Assessing diastolic function noninvasively remains challenging in children.126,127 One can measure diastolic function more accurately using micromanometer-tipped catheters to assess the time constant of diastolic relaxation (Tau), which has been associated with clinically relevant events such as duration of intensive care and hospital stay after the Fontan operation.128 However, this primarily remains a research tool owing to complexity and expense.

Pericardial Function The parietal pericardium is a fibrous membrane that surrounds the heart and is separated from the epicardium by a thin layer of fluid. Owing to its nonelastic properties, the pericardium enhances mechanical interactions of the cardiac chambers and limits acute cardiac dilation.129 Thus, if the ventricles enlarge because of sudden volume load or myocardial depression, the pericardium becomes tense and restrains further enlargement of the ventricles.94,95 This is seen with acute myocardial ischemia, where LV diastolic pressure can increase without much change in ventricular volume because of tension from the pericardium. In this setting, changes in the diastolic pressure-volume relationship reflect both myocardial stiffness and pericardial constraint.92,93,130–132 Transmural pressure is the difference between intracavitary and extracavitary pressure (intracardiac pressure–pericardial pressure). Pericardial pressure reflects intrapleural pressure during the respiratory cycle,133 which oscillates around –5 cmH2O in a spontaneously breathing patient in the absence of pericardial disease. With each inspiration, the pleural pressure becomes more negative and the pericardial pressure drops, which increases cardiac transmural pressure and facilitates filling of the right heart. Mechanical ventilation increases pleural and pericardial pressure, reduces transmural pressure and impedes filling of the right heart.134 This reduction in transmural pressure is detrimental for right heart filling but decreases both ventricular and aortic transmural pressure, decreasing wall stress/afterload and therefore improving stroke volume. In volume-replete patients, increased intrathoracic pressure will primarily assist the left ventricle. However, this benefit may be lost or it may even be detrimental to use positive-pressure ventilation in the volume-depleted patient. This afterload reduction and a decrease in metabolic demands due to less respiratory effort are the basis of using positive pressure, either noninvasive or invasive, ventilation in the setting of heart failure.135 Furthermore, in those with significant work of breathing, such as patients with pulmonary edema, the inspiratory pleural pressure is likely to be much more negative than in healthy people. This serves to significantly increase LV transmural pleural pressure and potentially make the use of positive-pressure ventilation more beneficial. Ventricular Interactions Ventricular interactions independent of humoral, neural, or circulatory effects are called ventricular interdependence and can be divided into diastolic and systolic ventricular interactions.131,136 They occur because of anatomic associations between the ventricles, interventricular septum as a shared septal wall, and enclosure within the pericardium. In normal circulation, diastolic ventricular interactions are responsible for changes in the pulse pressure during spontaneous ventilation. During inspiration, RV filling and volume increase, and the septum moves slightly toward the left, increasing LV enddiastolic pressure and decreasing LV end-diastolic volume and therefore decreasing stroke volume. These effects reverse during exhalation, increasing stroke volume. This mechanism is the basis of pulsus paradoxus in conditions that accentuate ventricularventricular interactions such as cardiac tamponade or asthma. In the failing right ventricle, dilation of the right ventricle pushes the septum to the left, decreasing LV volume and preload and shifting the LV diastolic pressure-volume relationship upward137 and decreasing cardiac output.94,138,139 Increasing LV afterload by manipulation of systemic vascular resistance can potentially counteract septal displacement, improving ventricular-ventricular



CHAPTER 23

Structure and Function of the Heart

197

interactions and overall cardiac output.140,141 In systole, due to the shared interventricular septum, the left ventricle generates 20% to 40% of RV contractility,142 whereas only 4% to 10% of LV systolic pressure is generated by the right ventricle. The decrease in cardiac output that occurs with acute RV failure with an intact pericardium is at least partially attributable to a decrease in systolic LV performance.143

Neural Control of the Heart The autonomic nervous system is the major determinant of heart rate in the normal heart, through the balance of sympathetic and parasympathetic tone.144 Sympathetic fibers innervate the atria, ventricle muscles, and the conduction system. Sympathetic activation releases norepinephrine through both the autonomic nervous system and humoral adrenaline from the adrenal glands. Catecholamines bind to b1-adrenoceptors activating Gs proteins, which results in increased heart rate (chronotropic effect), more rapid AV conduction (dromotropic effect), enhanced contractility (inotropic effect), and faster relaxation (lusitropic effect).145 The parasympathetic acetylcholine fibers mainly innervate the sinus and AV nodes. Vagal effects on the heart are mostly demonstrated by changes in heart rate but minimal direct effect on myocardial contractility. Parasympathetic stimulation may have an indirect effect on inotropy by reducing effects of circulating catecholamines or sympathetic nerve stimulation. Conversely, blockade of muscarinic receptors can intensify the myocardial contractile response to sympathetic stimulation. In conscious animals, resting sympathetic tone is low, and parasympathetic tone is high. Therefore, sympathetic blockade has little effect on heart rate and myocardial contractility, whereas parasympathetic blockade causes marked tachycardia. On the other hand, many anesthetics depress the sympathetic nervous system, leading to acutely decreased contractility and bradycardia. The carotid and aortic baroreceptors respond to changes in arterial blood pressure. Since basal sympathetic tone is usually low, inhibiting sympathetic tone by raising aortic pressure has little effect on myocardial contractility, whereas a decrease in arterial pressure causes a reflex increase in sympathetic tone, with increases in heart rate and contractility. Carotid and aortic chemoreceptors are stimulated by low partial pressure of arterial oxygen, high partial pressure of arterial carbon dioxide, and low pH, but only when changes are significant, and even then the increase in myocardial contractility is modest. The fetus seems to be less sensitive than the adult to chemoreceptor stimulation.146 Innervation is not necessary for cardiac function, as evidenced by those who have undergone cardiac transplantation. The response to exercise in the denervated heart is limited and mediated by increases in circulating catecholamines and a rise in body temperature. In intact animals and humans, b-adrenoreceptor blockade blunts the heart rate increase with exercise and abolishes inotropic response.144 Cardiac Output Cardiac output (CO) is the volume of blood ejected by the heart over one minute; therefore, CO 5 stroke volume (volume ejected per contraction) 3 heart rate (contractions per minute). Cardiac output in the fetus is determined mainly by heart rate because of a limited capacity to increase stroke volume that results mainly from decreased diastolic distensibility. Consequently, fetal bradycardia is detrimental to blood flow and oxygen delivery. However, the fetal heart can respond to increased preload (Starling’s law)

198

SECTION IV



Pediatric Critical Care: Cardiovascular

with increased stroke volume provided that there is no concomitant increase in afterload.147 Usually, infusion of fluid into an animal causes arterial pressure to rise, and the increased afterload tends to inhibit the increase in stroke volume that would otherwise occur.147–149 Immediately after birth, there is a significant increase in total body oxygen consumption and cardiac output to about twice its later values (per unit body size).150 This increase has been related to an increase in adrenergic receptors stimulated by fetal thyroid hormones.151 In addition, because approximately 80% of the infant’s hemoglobin is in the form of fetal hemoglobin at birth, the reduced ability of this hemoglobin to unload oxygen at the tissue level compels the infant to have a higher cardiac output than the infant will have 4 to 6 weeks later.150 Therefore, the neonate has limited cardiac output reserve and the heart has near-maximal contractility.152,153 These features make the neonate unusually susceptible to diseases that impair cardiac function. However, the Frank-Starling mechanism is intact at this time.154 Evidence indicates that b-adrenoceptor stimulation helps the neonatal ventricle adapt to volume loads.155 Thus b-adrenoceptor blockade might be expected to be much more harmful in the neonate than in the older person with minimal sympathetic tone.

Myocardial Metabolism: Normal Myocardial Energy Metabolism Basic Metabolic Processes Basal metabolic processes can be studied by measuring oxygen uptake, production of heat, or utilization of high-energy phosphates. Isolated papillary muscle and whole-heart preparations reveal that most oxygen consumed generates force (internal work). Approximately 15% is used for shortening (external work), 20% for basal metabolic processes (protein synthesis, sarcolemmal sodium-potassium transport), and 10% for activity of sodiumpotassium adenosine triphosphatase and calcium-adenosine triphosphatase.156–159 The myocardium consumes approximately 8 to 10 mL oxygen/100 g muscle per minute under basal conditions. Potassium-induced cardioplegia reduces myocardial oxygen consumption, but “resting” cardiac muscle still consumes more than five times as much oxygen as does resting skeletal muscle. During maximal exercise, the myocardium may consume as much as 60 to 80 mL oxygen/100 g muscle per minute.160 Cardiac energy generated by oxidizing substrates to carbon dioxide and water is both used and stored with most of the stored energy in the form of ATP. When needed, ATP breaks down to adenosine diphosphate or adenosine monophosphate and releases energy for contractile or transport processes.161 Substrates for energy production can be glucose, lactate, or fatty acids,162 with the b-oxidation of long-chain fatty acids being the preferred substrate except in the neonatal myocardium. l-Carnitine is essential for fatty acid transport across the mitochondrial membrane. After fatty acids enter the cell, they are activated to fatty acid (or acyl) coenzyme A (CoA) compounds by palmitoyl-CoA synthetase, then linked by carnitine palmitoyl transferase I to carnitine to form acylcarnitines, thus releasing CoA. Acylcarnitines cross the mitochondrial membrane and, at the inner surface of the membrane, carnitine palmitoyl transferase II transfers the fatty acids back to CoA. The fatty acids can then undergo b-oxidation to produce ATP. Fetuses and neonates have decreased activity of carnitine palmitoyl transferase and palmitoyl-CoA synthetase. Thus, glucose, lactate, and short-chain fatty acids are the preferred myocardial energy substrates.3,163 Ischemia

of heart or skeletal muscle depletes carnitine, as does chronic congestive heart failure.164,165 Carnitine supplementation in these states may be appropriate. Increases in plasma fatty acid concentration in fasting or sympathetic stimulation suppress oxidation of carbohydrates by the heart.64,162 Therefore, lactate consumption or extraction cannot be used as an accurate guide to cardiac metabolism unless the concentration of fatty acids is also evaluated.166 ATP is usually generated by oxidative phosphorylation; however, when oxygen supply is restricted, ATP can be generated by anaerobic glycolysis. Accumulation of the byproducts of glycolysis inhibit key enzymes and interfere with further ATP production. Therefore, the myocardium is unable to build an oxygen debt without further depressing energy production and contractility. More than 30% of the myocardial mass is mitochondria, highlighting the importance of oxidative metabolism to the heart.156 Studies of the fetal heart in ovine models reveal that fetal ventricles and adult left ventricles have similar oxygen consumption. Because fetal oxygen content is lower, myocardial blood flow per unit mass is about twice as high in the fetus as in the adult.167,168 Oxidative capacity is lower and glycogen stores and glycolytic flux are higher in the fetal heart. This may explain why the immature heart is more resistant to hypoxemia, provided that an adequate supply of glucose is available for glycolysis. The main substrates used by the fetal heart are glucose, lactate, and pyruvate, although ketones, amino acids, and short- and medium-chain fatty acids also can provide energy.169 For these reasons, prolonged severe hypoglycemia can seriously depress cardiac function in the neonate but is unlikely to do so in the older person.

Determinants of Myocardial Oxygen Consumption In 1958, Sarnoff and Mitchell170 found that the area under the LV pressure curve in systole (termed the tension-time index) correlates with LV oxygen consumption; later work has found peak wall tension (or stress) to be an even better predictor171–174 because of the importance of wall thickness and ventricular dimensions to wall stress. Increases in contractility or heart rate increase myocardial oxygen consumption. However, because they decrease ventricular size and thus wall stress, effects on oxygen consumption are mitigated.175 Stroke volume is also a predictor of myocardial oxygen consumption.25,176–179 This relationship can be assessed using the area within the pressure-volume loop. This approach has been extended by Suga and colleagues111,180–185 to note that inclusion of the area representing end-systolic pressure energy (Fig. 23.6) leads to a more accurate model. Subtracting contributions of basal myocardial metabolism shows that the oxygen consumption–pressure-volume area (PVA) relationship is independent of contractile state. Further studies showed that PVA-independent oxygen consumption is a function of contractility, defined by Emax. Certain interventions—for example, acidosis— made the slope of this relation between PVA-independent oxygen consumption to Emax steeper, reflecting decreased efficiency of the system. Myocardial Oxygen Demand-Supply Relationship Myocardial oxygen demand is roughly proportional to ventricular systolic pressure and the duration of systole, which can be represented by the area under the real-time pressure curve of the ventricle in systole: the systolic pressure-time index (SPTI).186,187 The SPTI is dramatically influenced by cardiac afterload and the correlation between the SPTI and myocardial oxygen demand is

Volume

PVA Control Emax

B

V0

O2 consumption

O2 consumption

A

ES

Pressure PE

ED

ED

C

Volume

PVA Increased Emax E-C coupling

Basal

Basal

Pressure-volume area

F

E

D

P VA

ED

E-C coupling

199

V0

Volume

PVA-independent O2 consumption

EW

V0

Structure and Function of the Heart

ES

Pressure

Pressure

ES



CHAPTER 23

Emax

• Fig. 23.6



​Relationship of myocardial oxygen consumption to the pressure volume area (PVA). (A) Ventricular pressure-volume loop with pressure plotted on the ordinate and volume on the abscissa. Arrow shows the direction of inscription of the loop. (B) Shaded area to the left of the pressure-volume loop in the pressure-volume diagram represents potential energy (PE). (C) Total area (PVA) is the sum of the external mechanical work area (EW) and the potential energy area (PE). (D) PVA is linearly proportional to oxygen consumption, but some oxygen consumption is independent of PVA. The PVA-independent oxygen consumption shown below the upper horizontal line results from excitation-contraction (E-C) coupling and basal oxygen consumption. (E) When contractility is increased, as indicated by the increased value for maximum elastance (Emax), the relationship between PVA and oxygen consumption is unchanged, but PVA-independent oxygen consumption increases. (F) Relationship between Emax and PVA-independent oxygen consumption is linear. With myocardial depression, the slope of this relationship is steeper (dashed line). Thus, for any value of Emax, PVA-independent oxygen consumption is increased so that myocardial efficiency is reduced. ED, End-diastolic pressure-volume line; ES, line of end-systolic pressure-volume points (end-systolic elastance); EW, area representing external mechanical work; V0, unstressed ventricular volume.

SPTI

• Fig. 23.7

DPTI

​The systolic pressure time index (SPTI) reflects myocardial work and oxygen demand. The diastolic pressure time index (DPTI) reflects myocardial blood flow. (Modified from Fuhrman BP. Regional circulation. In: Fuhrman BP, Shoemaker WC, eds. Critical Care: State of the Art, vol 10. Fullerton, CA: Society of Critical Care Medicine; 1989.)  

imperfect because it does not account for wall stress.188 Because LV myocardial perfusion is restricted to diastole (see Chapter 24), myocardial oxygen supply is proportional to both duration of diastole and myocardial perfusion pressure in diastole. In general, diastolic myocardial perfusion pressure can be represented graphically as the difference between superimposed aortic and LV pressure curves. The area between these curves, from the instant of aortic valve closure in diastole to reopening of the aortic valve in systole, has been termed the diastolic pressure-time index (DPTI) and is proportional to subendocardial blood flow. When multiplied by arterial oxygen content, this index correlates with subendocardial oxygen supply.189 The DPTI 3 arterial oxygen content/SPTI ratio (Fig. 23.7) is a fair indicator of myocardial oxygen balance. At critical levels, subendocardial ischemia occurs.186,187 This ratio is worsened by tachycardia, which shortens diastole and the duration of myocardial perfusion, by elevation of end-diastolic pressures in the ventricles or by elevation of coronary sinus pressure. It is adversely affected by low aortic diastolic pressure (as in shock, aortic valve insufficiency, or other large diastolic runoff lesions) and by elevated ventricular systolic pressure (as in aortic stenosis, systemic hypertension, or pulmonary hypertension). The ratio is favorably affected by balloon aortic counterpulsation, which elevates aortic diastolic pressure and reduces systolic afterload. Given the imperfect

200

SECTION IV



Pediatric Critical Care: Cardiovascular

nature of this ratio, too much emphasis should not be placed on any given value, but two points are clear: (1) a fall in the ratio moves toward a supply-to-demand imbalance, and (2) any ratio less than the 8.9 that typifies normal subjects likely indicates myocardial ischemia.190

Effects of Myocardial Ischemia on Cardiac Function and Metabolism Ischemia indicates inadequate flow to supply the demand for oxygen by an organ or tissue and reduced clearance of metabolites,191,192 which distinguishes ischemia from hypoxemia, in which there is a normal flow with decreased oxygen delivery. Because the heart cannot sustain an oxygen debt, inadequate oxygen supply rapidly decreases energy supply to the myocytes, which cease to contract normally. If a branch of the left coronary artery is severely narrowed or occluded acutely, within 5 to 15 seconds the myocardium supplied by that branch stops contracting, turns blue, bulges, and thins during each systole. In global ischemia, the subendocardial muscle is affected first because it has the lowest coronary flow reserve.193–195 Temporary imbalance of supply and demand leads to two patterns of response depending on the duration of the ischemia. If a branch coronary artery is occluded for 15 to 30 minutes and then the occlusion is removed, flow returns to normal rapidly, but the muscle may not contract normally for many hours. This phenomenon is known as reperfusion injury or stunning.196–199 It should be distinguished from the “no reflow” phenomenon in which, after a longer occlusion, release of the occlusion is followed by incomplete restoration of flow because of myocardial edema, cell swelling, plugging by neutrophils, and endothelial damage. Stunning may occur after prolonged cardiopulmonary bypass surgery with cardioplegia and may account for some of the cardiac depression that is observed in the early postoperative recovery period.200,201 Chronic ischemia of moderate severity causes myocardial hibernation, an adaptive response that leads to metabolic downregulation and reduction of flow without extensive cell death.202–205 Regional function is reduced, but restoration of flow leads to functional recovery. This phenomenon is best known from studies of coronary artery disease but can be present in some children with normal coronary arteries and subendocardial ischemia. Chronic imbalance of oxygen supply and demand leads to death of the affected muscle cells, producing either a localized infarct or diffuse, perhaps patchy, subendocardial fibrosis as occurs commonly with severe aortic stenosis, cyanotic heart disease, or dilated cardiomyopathy.

Systemic Vasculature General Anatomy The large arteries are elastic, with the media containing concentric lamellae of perforated elastic tubes crosslinked by transverse collagen (type III) and smooth muscle.57,206 When smooth muscle contracts, the wall becomes stiffer. Smaller arteries have fewer lamellae. The media are bounded by the external and internal elastic laminae, beyond which are the adventitia with nerves and vasa vasorum and the intima with sparse fibrous tissue and a metabolically active endothelium, respectively. Arterioles have no lamellae and only a thin media with circular or spiral smooth muscle; the only elastic tissue is in the inner and outer elastic laminae. Capillaries are thin walled and nonmuscular, ideal for transport of materials into and from the tissues. However, they contain pericytes that have myosin, actin, and tropomyosin and

might have some contractile function. Veins have medial muscle but thinner walls relative to lumen diameter than do arteries. Their endothelium may have different properties. The numerous extracellular matrix components are reviewed by Buga and Ignarro.207 The developmental aspects of blood vessels are reviewed by Stenmark and Weiser.208

Physiologic Mechanisms General Features

Although large arteries are regarded as conduits and capillaries as vessels allowing transport of substances to and from the tissues, many substances can move across arterial walls. Oxygen and carbon dioxide can diffuse across arteriolar walls, and lipoproteins can penetrate the walls of large arteries. Whether atheromatous deposits form in arteries depends on the balance of lipoprotein that enters and leaves the arterial wall. This balance depends on the concentration and chemical nature of lipoproteins and the action of components of the wall, such as glycosaminoglycans, in binding altered lipoprotein molecules and preventing their transit through the wall. Arteriolar tone controls peripheral resistance and, with cardiac output, determines blood pressure and regional flow. Regions of the circulation may differ markedly in their patterns of vascular regulation. A potent stimulus for increased vascular resistance in one region of the circulation may have a different effect in another. For example, during hemorrhagic shock, flow is maintained to the heart and brain but is reduced to muscle, kidneys, and the gut. Venous and venular tone, together with diuretic and antidiuretic factors, determine blood volume and venous pressure. The two active components of the systemic circulation are the medial smooth muscle and the endothelium. They both have receptors for innumerable agonists and antagonists that diffuse from autonomic nerve endings, circulate from remote regions, or are produced locally. The smooth muscle is responsible for vasoconstriction or vasodilation. The vascular endothelium is one of the metabolic powerhouses of the body. Endothelial cells have several major functions: 1. They play important roles in the response to injury by causing leukocyte adhesion and extravasation.209,210 2. They are intimately bound up with coagulation208,210 by virtue of the production of procoagulant (e.g., platelet-activating factor [PAF], von Willebrand factor, fibronectin, and factors V and X) and anticoagulant factors (e.g., heparin, dermatan sulfate, thrombomodulin, ectonucleotidase) and by the production of nitric oxide and PGI2, which inhibit platelet aggregation and degranulation. 3. They regulate capillary permeability by producing ET-1 (increase) or PGE1 (decrease) and respond with increased plasma leakage to substances such as bradykinin, histamine, thrombin, oxygen radicals, and PAF.211 4. They regulate smooth muscle contraction in response to shear stress in keeping with an overriding principle that shear rate must be kept constant within narrow limits to prevent endothelial damage.212















Control of Vascular Tone

In general, regional circulations regulate their flow to obtain the required amounts of oxygen and nutrients. Vasomotor tone is strongly influenced by several mechanisms: (1) innervation and neural processes, (2) circulating endocrine and neuroendocrine mediators, (3) blood gas composition, (4) local metabolic products, (5) endothelial-derived factors, and (6) myogenic processes.

Receptors responsive to neural products (norepinephrine, acetylcholine, neuropeptides) are found throughout the circulation. Nevertheless, innervation and receptor distributions are organ specific, which allows rapid, patterned, coordinated redistribution of blood flow and an orchestrated response to hypoxia, postural changes, and hemorrhage. Although these receptors respond to circulating agonists (including angiotensin II and adrenal epinephrine) and to those liberated locally, they are generally associated with innervation by autonomic nerves. In general, presynaptic a-adrenergic stimulation causes norepinephrine release and vasoconstriction. b-Adrenergic stimulation generally causes vasodilation. Cholinergic stimulation generally causes vasodilation. In all organs, sensory and efferent nerve endings contain nonadrenergic, noncholinergic peptides, for example, neuropeptide Y, VIP, calcitonin gene-related peptide, and substance P.213–225 Neuropeptide Y is co-localized and released with norepinephrine,226 and VIP is co-localized with acetylcholine and released upon stimulation of vagal nerve endings. Most of these peptides, except neuropeptide Y, are vasodilatory and they help modulate blood pressure and regional flows. Humoral regulators of vascular tone and blood volume include angiotensin, adrenomedullin, aldosterone, arginine vasopressin (AVP), bradykinin, histamine, serotonin, thyroxine, natriuretic peptides, and various reproductive hormones. Most of these regulators have both direct effects and secondary effects, which tend to be organ specific or regional. They often have altered concentrations in hypertension, congestive heart failure, or shock, and their antagonists are used in therapy. Some agents—such as histamine, serotonin, and thyroxine—probably affect peripheral resistance only in abnormal states and are not physiologic regulators. Angiotensin plays a special role in the homeostasis of blood pressure. Its concentration increases in hemorrhagic or hypovolemic shock, following increased renal production of renin that produces angiotensin I from angiotensinogen. Angiotensin I is converted to active angiotensin II by angiotensin-converting enzyme (ACE) in the endothelium, especially in the pulmonary vessels. However, angiotensin II is also produced locally in the heart and vessel walls by renin that enters from the blood and perhaps by other local proteases.227,228 It causes generalized vasoconstriction in both systemic and pulmonary circulations, but locally it stimulates the release of vasodilating prostaglandins in the lung and kidney. Angiotensin II, via angiotensin I receptors, plays a role in cardiac and smooth muscle cell hypertrophy. In excess, it results in cardiac inflammation, fibrosis, and apoptosis.229–232 Adrenomedullin, originally found in pheochromocytomas, is produced in many normal cell types, including endothelium. Among its many actions are long-lasting vasodilation and diuresis.233,234 Its release may be stimulated by ET-1. It may play a role in treating heart failure.235 Aldosterone, known primarily for its effect on sodium absorption and potassium excretion, has indirect central effects on blood pressure.236–238 Its concentration increases when renin release is stimulated. In patients with congestive heart failure, its decreased breakdown in the liver accounts for high blood concentrations, which are harmful to the heart and blood vessels. Inhibition of aldosterone by spironolactone may have great clinical value.237,239,240 AVP, which is released from the axonal terminals of magnocellular neurons in the hypothalamus, causes vasoconstriction by stimulating VP1 receptors. However, at low concentrations, AVP



CHAPTER 23

Structure and Function of the Heart

201

dilates coronary, cerebral, and pulmonary vessels. It is an antidiuretic hormone that acts on VP2 receptors in the renal collecting ducts.241 Its concentration is low in septic shock, with ventricular arrhythmias, and after cardiac surgery242 but is increased in myocardial and hemorrhagic shock, congestive heart failure, and liver cirrhosis.102,241,243 Selective AVP antagonists promote free water excretion without concomitant electrolyte excretion.243–246 Bradykinin is a potent pulmonary and systemic vasodilator released locally by the action of proteolytic enzymes on kallikrein after tissue injury.247–250 Bradykinin is metabolized by kininase II, which is the same as ACE. Thus, ACE inhibitors not only reduce angiotensin II production but also increase bradykinin concentrations. Bradykinin also causes endothelial cell release of tissue-type plasminogen activator.251 The natriuretic peptides are released from the heart when it is distended in congestive heart failure. They cause vasodilation and increased diuresis. A-natriopeptide (mainly from atria) and Bnatriopeptide (from ventricles) are released from myocardial cells, and C-natriopeptide is released from cardiac endothelium.252–256 These natriopeptides and the kinins are broken down by neutral endopeptidase. Inhibition of this breakdown combined with inhibition of ACE by vasopeptidase inhibitors (e.g., omapatrilat) greatly augments vasodilation.257–262 Tissue levels of oxygen and carbon dioxide reflect adequacy of perfusion and oxygen delivery. These blood gases are potent determinants of regional blood flow and have effects that differ among regions of the circulation. They also have a more general effect mediated by carotid chemoreceptors. Local metabolic regulation of vasomotor tone provides an ideal homeostatic mechanism whereby metabolic demand can directly influence perfusion. For instance, adenosine, which accumulates locally when tissue metabolism is high and tissue oxygenation is marginal, causes pronounced vasodilation in the coronary, striated muscle, splanchnic, and cerebral circulations. Cerebral autoregulation has been suggested to take advantage of local metabolite production as an indicator of adequacy of blood flow. The perivascular concentration of these metabolites is restored to normal as flow rises, washing out the metabolites. Potassium is released from muscle in response to increased work, ischemia, and hypoxia.262 Hypokalemia causes vasoconstriction.263,264 Hyperkalemia, within the physiologic range, causes vasodilation by stimulating Kir channels.265–267 Many of the agents previously discussed are produced locally and are effective as circulating hormones. The endothelial lining of blood vessels plays a prominent role in the regulation of vascular tone.268 Endothelium-derived relaxing factor (EDRF) has been identified as nitric oxide.269 Nitric oxide is a potent vasodilator released from endothelium after stimulation and accounts for some or all of the activity generally ascribed to other agonists. Nitric oxide is released from endothelium when flow increases, an example of positive feedback. Nitric oxide increases smooth muscle soluble guanylate cyclase activity, raises muscle cyclic GMP, and thereby relaxes vascular smooth muscle. In addition to EDRF, endothelial-derived hyperpolarizing factors, which are probably epoxyeicosatrienoic acids and hydrogen peroxide, are now thought to play major roles. The hydrogen peroxide is produced by the action of superoxide dismutase on superoxide anions that are generated by the metabolism of ATP.270 The vascular endothelium elaborates the endothelins (ET-1, ET-2, ET-3), a family of compounds that are vasoactive, structurally related peptides. ET-1 is the most potent vasoconstrictor known. It also promotes mitogenesis and stimulates the renin-angiotensin-aldosterone

202

SECTION IV



Pediatric Critical Care: Cardiovascular

system and the release of vasopressin and atrial natriuretic peptide.237,271–276 Endothelin antagonists, such as bosentan, are being used, specifically in the setting of pulmonary arterial hypertension.277,278 Myogenic responses of vessels are changes in smooth muscle tone in response to stretch or increased transmural pressure. An increase in inflow pressure causes a rise in vessel wall tension and transmural pressure279 that causes localized vasoconstriction. The reverse occurs when inflow pressure falls. The mechanisms of this response are complex. As expected, a complex interplay exists among myogenic, flowmediated, and metabolic regulation of vessel tone.280 The relative importance of these mechanisms likely varies in different vascular beds.

Autoregulation In all organs, when inflow pressure is suddenly raised or lowered while oxygen consumption remains constant, flow rises or falls transiently but then returns to the earlier value. The phenomenon is termed autoregulation. Myogenic tonic response is partly responsible for this phenomenon, but it is not the only mechanism. Some investigators believe that tissues have oxygen sensors that respond to transient increases or decreases in oxygen supply.281–283 Others believe that the process is mediated by greater or lesser release of nitric oxide carried to the tissues by hemoglobin in the form of S-nitrosohemoglobin or by ATP release by red blood cells.283–288 Carbon monoxide produced by the action of hemoxygenase in endothelium and smooth muscle may play a regulatory role.289–296

Key References Anderson PAW. Immature myocardium. In: Moller JH, Neal WA, eds. Fetal, Neonatal, and Infant Cardiac Disease. Norwalk, CT: Appleton & Lange; 1992.

Buga GM, Ignarro LJ. Vascular endothelium and smooth muscle function. In: Gluckman PD, Heymann MA, eds. Pediatrics and Perinatology. The Scientific Basis. London: Edward Arnold; 1996. Colan SD, Borow KM, Neumann A. Left ventricular end-systolic wall stress-velocity of fiber shortening relation: a load-independent index of myocardial contractility. J Am Coll Cardiol. 1984;4:715-724. Friedberg MK, Redington AN. Right versus left ventricular failure: differences, similarities, and interactions. Circulation. 2014;129:1033-1044. Hoffinan JI, Buckberg GD. The myocardial supply: demand ratio—a critical review. Am J Cardiol. 1978;41:327. Hoffman TM, Wernovsky G, Atz AM, et al. Efficacy and safety of milrinone in preventing low cardiac output syndrome in infants and children after corrective surgery for congenital heart disease. Circulation. 2003;107:996-1002. Katz AM. Contractile proteins in normal and failing myocardium. In: Braunwald E, ed. The Myocardium: Failure and Infarction. New York: HP Publishing; 1974. Ross Jr J. Mechanisms of cardiac contraction. What roles for preload, afterload and inotropic state in heart failure? Eur Heart J. 1983;4(suppl A):19. Sagawa K. The end-systolic pressure-volume relation of the ventricle: definition, modifications and clinical use. Circulation. 1981;63:1223. Santamore WP, Dell’Italia LJ. Ventricular interdependence: significant left ventricular contributions to right ventricular systolic function. Prog Cardiovasc Dis. 1998;40:289-308. Smith VE, Zile MR. Relaxation and diastolic properties of the heart. In: Fozzard HA, et al., eds. The Heart and Cardiovascular System: Scientific Foundations. New York: Raven Press; 1991. Strauer BE. Myocardial oxygen consumption in chronic heart disease: role of wall stress, hypertrophy and coronary reserve. Am J Cardiol. 1979;44:730-740. Suga H. Ventricular energetics. Physiol Rev. 1990;70:247. Tyberg JV, Grant DA, Kingma I, et al. Effects of positive intrathoracic pressure on pulmonary and systemic hemodynamics. Respir Physiol. 2000;119:171-179. Tynan MJ, Becker AE, Macartney FJ, Jiménez MQ, Shinebourne EA, Anderson RH. Nomenclature and classification of congenital heart disease. Br Heart J. 1979;41:544-553.

The full reference list for this chapter is available at ExpertConsult.com.

e1







































































































1. Tynan MJ, Becker AE, Macartney FJ, Jiménez MQ, Shinebourne EA, Anderson RH. Nomenclature and classification of congenital heart disease. Br Heart J. 1979;41:544-553. 2. von Lüdinghausen M, Ohmachi N, Besch S, Mettenleiter A. Atrial veins of the human heart. Clin Anat. 1995;8:169-189. 3. Anderson PAW. Immature myocardium. In: Moller JH, Neal WA, eds. Fetal, Neonatal, and Infant Cardiac Disease. Norwalk, CT: Appleton & Lange; 1992. 4. Birk E. Myocardial receptors. In: Gluckman PD, Heymann MA, eds. Pediatrics and Perinatology. The Scientific Basis. London: Edward Arnold; 1996. 5. Friedman WF, et al. The intrinsic physiologic properties of the developing heart. In: Friedman WF, ed. Neonatal Heart Disease. New York: Grune & Stratton; 1973. 6. Coleridge JCG, Coleridge HM. Chemoreflex regulation of the heart. In: Berne RM, Sperelakis N, eds. Handbook of Physiology, Section 2: The Cardiovascular System. Vol 1. The Heart. Bethesda, MD: American Physiological Society; 2001. 7. Clyman RI. Developmental physiology of the ductus arteriosus. In: Polin Long WA, ed. Fetal and Neonatal Cardiology. New York: WB Saunders; 1989. 8. Clyman RI. Medical treatment of the ductus arteriosus in premature infants. In: Polin Long WA, ed. Fetal and Neonatal Cardiology. New York: WB Saunders; 1989. 9. Clyman RI. Patent ductus arteriosus in the premature infant. In: Avery ME, ed. Diseases of the Newborn. Philadelphia: WB Saunders; 1998. 10. Coceani F, Kelsey L. Endothelin-1 release from lamb ductus arteriosus: relevance to postnatal closure of the vessel. Can J Physiol Pharmacol. 1991;69:218-221. 11. Michelakis ED, Rebeyka I, Wu X, et al. O2 sensing in the human ductus arteriosus: regulation of voltage-gated K1 channels in smooth muscle cells by a mitochondrial redox sensor. Circ Res. 2002;91:478-486. 12. Reeve HL, Tolarova S, Nelson DP, Archer S, Weir EK. Redox control of oxygen sensing in the rabbit ductus arteriosus. J Physiol. 2001;533:253-261. 13. Tristani-Firouzi M, Reeve HL, Tolarova S, Weir EK, Archer SL. Oxygen-induced constriction of rabbit ductus arteriosus occurs via inhibition of a 4-aminopyridine-, voltage-sensitive potassium channel. J Clin Invest. 1996;98:1959-1965. 14. Clyman RI, Waleh N, Black SM, Riemer RK, Mauray F, Chen YQ. Regulation of ductus arteriosus patency by nitric oxide in fetal lambs: the role of gestation, oxygen tension, and vasa vasorum. Pediatr Res. 1998;43:633-644. 15. Clyman RI, Chan CY, Mauray F, et al. Permanent anatomic closure of the ductus arteriosus in newborn baboons: the roles of postnatal constriction, hypoxia, and gestation. Pediatr Res. 1999;45:19-29. 16. Korecky B, Rakusan K. Normal and hypertrophic growth of the rat heart: changes in cell dimensions and number. Am J Physiol. 1978; 234:H123-H128. 17. Streeter Jr DD, Hanna WT. Engineering mechanics for successive states in canine left ventricular myocardium. 11. Fiber angle and sarcomere length. Circ Res. 1973;33:656-664. 18. Torrent-Guasp F, Buckberg GD, Clemente C, Cox JL, Coghlan HC, Gharib M. The structure and function of the helical heart and its buttress wrapping. I. The normal macroscopic structure of the heart. Semin Thorac Cardiovasc Surg. 2001;13:301-319. 19. Athanasuleas CL, Buckberg GD, Menicanti L, Gharib M, RESTORE Group. Optimizing ventricular shape in anterior restoration. Semin Thorac Cardiovasc Surg. 2001;13:459-467. 20. Kanno S, Saffitz JE. The role of myocardial gap junctions in electrical conduction and arrhythmogenesis. Cardiovasc Pathol. 2001;10: 169-177. 21. van Veen AA, van Rijen HV, Opthof T. Cardiac gap junction channels: modulation of expression and channel properties. Cardiovasc Res. 2001;51:217-229.

22. Yeager M, Nicholson BJ. Structure of gap junction intercellular channels. Curr Opin Struct Biol. 1996;6:183-192. 23. Hein S, Kostin S, Heling A, Maeno Y, Schaper J. The role of the cytoskeleton in heart failure. Cardiovasc Res. 2000;45:273-278. 24. Chidsey CA. Calcium metabolism in the normal and failing heart. In: Braunwald E, ed. The Myocardium: Failure and Infarction. New York: HP Publishing; 1974. 25. Shroff SG, et al. Mechanical and energetic behavior of the intact left ventricle. In: Fozzard HA, et al., eds. The Heart and Cardiovascular System: Scientific Foundations. New York: Raven Press; 1991. 26. Sonnenblick EH, Spiro D, Spotnitz HM. The ultrastructural basis of Starling’s law of the heart. The role of the sarcomere in determining ventricular size and stroke volume. Am Heart J. 1964;68:336-346. 27. Katz AM. Contractile proteins in normal and failing myocardium. In: Braunwald E, ed. The Myocardium: Failure and Infarction. New York: HP Publishing; 1974. 28. Moolman-Smook J, Flashman E, de Lange W, et al. Identification of novel interactions between domains of myosin binding protein-C that are modulated by hypertrophic cardiomyopathy missense mutations. Circ Res. 2002;91:704-711. 29. Granzier H, Labeit S. Cardiac titin: an adjustable multi-functional spring. J Physiol. 2002;541:335-342. 30. Le Guennec JY, Cazorla O, Lacampagne A, Vassort G. Is titin the length sensor in cardiac muscle? Physiological and physiopathological perspectives. Adv Exp Med Biol. 2000;481:337-348. 31. Tonino P, Kiss B, Strom J, et al. The giant protein titin regulates the length of the striated muscle thick filament. Nat Commun. 2017;8:1041. 32. Herman DS, Lam L, Taylor MRG, et al. Truncations of titin causing dilated cardiomyopathy. N Engl J Med. 2012;366:619-628. 33. LeWinter MM, Granzier HL. Cardiac titin and heart disease. J Cardiovasc Pharmacol. 2014;63:207-212. 34. Muller FU, Kirchhefer U, Begrow F, Reinke U, Neumann J, Schmitz W. Junctional sarcoplasmic reticulum transmembrane proteins in the heart. Basic Res Cardiol. 2002;97(suppl 1):I52-I55. 35. Bristow M. Of phospholamban, mice, and humans with heart failure. Circulation. 2001;103:787-788. 36. Chu G, Kranias EG. Functional interplay between dual site phospholamban phosphorylation: insights from genetically altered mouse models. Basic Res Cardiol. 2002;97(suppl 1):I43-I48. 37. Frank K, Kranias EG. Phospholamban and cardiac contractility. Ann Med. 2000;32:572-578. 38. Schmidt AG, Edes I, Kranias EG. Phospholamban: a promising therapeutic target in heart failure? Cardiovasc Drugs Ther. 2001;15:387-396. 39. Munch G, Bölck B, Karczewski P, Schwinger RH. Evidence for calcineurin-mediated regulation of SERCA 2a activity in human myocardium. J Mol Cell Cardiol. 2002;34:321-334. 40. Frank KF, Bölck B, Brixius K, Kranias EG, Schwinger RH. Modulation of SERCA: implications for the failing human heart. Basic Res Cardiol. 2002;97(suppl 1):I72-I78. 41. Pieske B, Maier LS, Schmidt-Schweda S. Sarcoplasmic reticulum Ca21 load in human heart failure. Basic Res Cardiol. 2002;97(suppl 1):I63-I71. 42. Wu LL, Tang C, Dong LW, Liu MS. Altered phospholamban-calcium ATPase interaction in cardiac sarcoplasmic reticulum during the progression of sepsis. Shock. 2002;17:389-393. 43. Lennon NJ, Ohlendieck K. Impaired Ca21-sequestration in dilated cardiomyopathy (review). Int J Mol Med. 2001;7:131-141. 44. Napolitano C, Priori SG, Bloise R. Catecholaminergic polymorphic ventricular tachycardia. In: Adam MP, Ardinger HH, Pagon RA, et al., eds. SourceGeneReviews® [Internet]. Seattle, WA: University of Washington, Seattle; 2016. 45. Anderson PAW. Developmental cardiac physiology and myocardial function. In: Moller JH, Hoffman JIE, eds. Pediatric Cardiovascular Medicine. New York: Churchill Livingstone; 2000. 46. Gregorio CC, Antin PB. To the heart of myofibril assembly. Trends Cell Biol. 2000;10:355-362.

References

e2















































72. McCormack J, Gelband H, Villafañe J, Xu H, Stolfi A, Pickoff AS. In vivo demonstration of maturational changes of the chronotropic response to alpha-adrenergic stimulation. Pediatr Res. 1988;24:50-54. 73. Bristow MR, Ginsburg R, Umans V, et al. Beta 1- and beta 2-adrenergic-receptor subpopulations in nonfailing and failing human ventricular myocardium: coupling of both receptor subtypes to muscle contraction and selective beta 1-receptor down-regulation in heart failure. Circ Res. 1986;59:297-309. 74. Heilbrunn SM, Shah P, Bristow MR, Valantine HA, Ginsburg R, Fowler MB. Increased beta-receptor density and improved hemodynamic response to catecholamine stimulation during long-term metoprolol therapy in heart failure from dilated cardiomyopathy. Circulation. 1989;79:483-490. 75. Stryer L, Bourne HR. G proteins: a family of signal transducers. Annu Rev Cell Biol. 1986;2:391-419. 76. Xiao RP, Avdonin P, Zhou YY, et al. Coupling of beta2-adrenoceptor to Gi proteins and its physiological relevance in murine cardiac myocytes. Circ Res. 1999;84:43-52. 77. Communal C, Singh K, Sawyer DB, Colucci WS. Opposing effects of beta(1)- and beta(2)-adrenergic receptors on cardiac myocyte apoptosis: role of a pertussis toxin-sensitive G protein. Circulation. 1999;100:2210-2212. 78. Chesley A, Lundberg MS, Asai T, et al. The beta(2)-adrenergic receptor delivers an antiapoptotic signal to cardiac myocytes through G(i)-dependent coupling to phosphatidylinositol 39-kinase. Circ Res. 2000;87:1172-1179. 79. Zaugg M, Xu W, Lucchinetti E, Shafiq SA, Jamali NZ, Siddiqui MA. Beta-adrenergic receptor subtypes differentially affect apoptosis in adult rat ventricular myocytes. Circulation. 2000;102:344-350. 80. Bristow MR. Mechanistic and clinical rationales for using betablockers in heart failure. J Card Fail. 2000;6:8-14. 81. Bristow MR, Roden RL, Lowes BD, Gilbert EM, Eichhorn EJ. The role of third-generation beta-blocking agents in chronic heart failure. Clin Cardiol. 1998;21:I3-I13. 82. Bruns LA, Canter CE. Should beta-blockers be used for the treatment of pediatric patients with chronic heart failure? Paediatr Drugs. 2002;4:771-778. 83. Fowler M. Beta-adrenergic blocking drugs in severe heart failure. Rev Cardiovasc Med. 2002;3(suppl 3):S20-S26. 84. Kukin ML. Beta-blockers in chronic heart failure: considerations for selecting an agent. Mayo Clin Proc. 2002;77:1199-1206. 85. Packer M, Fowler MB, Roecker EB, et al. Effect of carvedilol on the morbidity of patients with severe chronic heart failure: results of the carvedilol prospective randomized cumulative survival (COPERNICUS) study. Circulation. 2002;106:2194-2199. 86. Belge C, Hammond J, Dubois-Deruy E, et al. Enhanced expression of b3-adrenoceptors in cardiac myocytes attenuates neurohormoneinduced hypertrophic remodeling through nitric oxide synthase. Circulation. 2014;129:451-462. 87. Cannavo A, Koch WJ. Targeting b3-Adrenergic Receptors in the Heart: Selective Agonism and b-Blockade. J Cardiovasc Pharmacol. 2017;69:71-78. 88. Artman M, Kithas PA, Wike JS, Strada SJ. Inotropic responses change during postnatal maturation in rabbit. Am J Physiol. 1988;255:H335-H342. 89. Hoffman TM, Wernovsky G, Atz AM, et al. Efficacy and safety of milrinone in preventing low cardiac output syndrome in infants and children after corrective surgery for congenital heart disease. Circulation. 2003;107:996-1002. 90. Bailey JM, Hoffman TM, Wessel DL, et al. A population pharmacokinetic analysis of milrinone in pediatric patients after cardiac surgery. J Pharmacokinet Pharmacodyn. 2004;31:43-59. 91. Zuppa AF, Nicolson SC, Adamson PC, et al. Population pharmacokinetics of milrinone in neonates with hypoplastic left heart syndrome undergoing stage I reconstruction. Anesth Analg. 2006;102:1062-1069. 92. Glantz SA, Misbach GA, Moores WY, et al. The pericardium substantially affects the left ventricular diastolic pressure-volume relationship in the dog. Circ Res. 1978;42:433-441.

























































47. Squire JM. Architecture and function in the muscle sarcomere. Curr Opin Struct Biol. 1997;7:247-257. 48. Capetanaki Y, Milner DJ. Desmin cytoskeleton in muscle integrity and function. Subcell Biochem. 1998;31:463-495. 49. Capetanaki Y, Milner DJ, Weitzer G. Desmin in muscle formation and maintenance: knockouts and consequences. Cell Struct Funct. 1997;22:103-116. 50. Stewart M. Intermediate filament structure and assembly. Curr Opin Cell Biol. 1993;5:3-11. 51. Barth AI, Näthke IS, Nelson WJ. Cadherins, catenins and APC protein: interplay between cytoskeletal complexes and signaling pathways. Curr Opin Cell Biol. 1997;9:683-690. 52. Ben-Ze’ev A, Geiger B. Differential molecular interactions of betacatenin and plakoglobin in adhesion, signaling and cancer. Curr Opin Cell Biol. 1998;10:629-639. 53. Burridge K, Chrzanowska-Wodnicka M. Focal adhesions, contractility, and signaling. Annu Rev Cell Dev Biol. 1996;12:463-518. 54. Towbin JA. The role of cytoskeletal proteins in cardiomyopathies. Curr Opin Cell Biol. 1998;10:131-139. 55. Towbin JA, Bowles NE. Molecular genetics of left ventricular dysfunction. Curr Mol Med. 2001;1:81-90. 56. Towbin JA, Bowles NE. The failing heart. Nature. 2002;415:227-233. 57. Borg TK, Ranson WF, Moslehy FA, Caulfield JB. Structural basis of ventricular stiffness. Lab Invest. 1981;44:49-54. 58. Robinson TF, Cohen-Gould L, Factor SM, Eghbali M, Blumenfeld OO. Structure and function of connective tissue in cardiac muscle: collagen types I and III in endomysial struts and pericellular fibers. Scanning Microsc. 1988;2:1005-1015. 59. Robinson TF, Geraci MA, Sonnenblick EH, Factor SM. Coiled perimysial fibers of papillary muscle in rat heart: morphology, distribution, and changes in configuration. Circ Res. 1988;63:577-592. 60. Zhao MJ, Zhang H, Robinson TF, Factor SM, Sonnenblick EH, Eng C. Profound structural alterations of the extracellular collagen matrix in postischemic dysfunctional (“stunned”) but viable myocardium. J Am Coll Cardiol. 1987;10:1322-1334. 61. Nassar R, Reedy MC, Anderson PA. Developmental changes in the ultrastructure and sarcomere shortening of the isolated rabbit ventricular myocyte. Circ Res. 1987;61:465-483. 62. Terracio L, Gullberg D, Rubin K, Craig S, Borg TK. Expression of collagen adhesion proteins and their association with the cytoskeleton in cardiac myocytes. Anat Rec. 1989;223:62-71. 63. Terracio L, Rubin K, Gullberg D, et al. Expression of collagen binding integrins during cardiac development and hypertrophy. Circ Res. 1991;68:734-744. 64. Bristow JD. Cardiac and myocardial structure and myocardial cellular and molecular function. In: Gluckman PD, Heymann MA, eds. Pediatrics and Perinatology. The Scientific Basis. London: Edward Arnold; 1996. 65. Fabiato A. Calcium release in skinned cardiac cells: variations with species, tissues, and development. Fed Proc. 1982;41:2238-2244. 66. Lakatta EG. Length modulation of myocardial performance: FrankStarling law of the heart. In: Fozzard HA, et al., eds. The Heart and Cardiovascular System: Scientific Foundations. New York: Raven Press; 1991. 67. Mahony L. Development of myocardial structure and function. In: Emmanouilides GC, et al., eds. Heart Disease in Infants, Children, and Adolescents, Including the Fetus and Young Adult. Baltimore, MD: Williams & Wilkins; 1998. 68. Sonnenblick EH. Myocardial ultrastructure in the normal and the failing heart. In: Braunwald E, ed. The Myocardium: Failure and Infarction. New York: HP Publishing; 1974. 69. Smith VE, Zile MR. Relaxation and diastolic properties of the heart. In: Fozzard HA, et al., eds. The Heart and Cardiovascular System: Scientific Foundations. New York: Raven Press; 1991. 70. Granzier HL, Irving TC. Passive tension in cardiac muscle: contribution of collagen, titin, microtubules, and intermediate filaments. Biophys J. 1995;68:1027-1044. 71. Strobeck JE, Sonnenblick EH. Myocardial contractile properties and ventricular performance. In: Fozzard HA, et al., eds. The Heart and Cardiovascular System: Scientific Foundations. New York: Raven Press; 1991.

e3











































116. Lakatta EG, Jewell BR. Length-dependent activation: its effect on the length-tension relation in cat ventricular muscle. Circ Res. 1977;40:251-257. 117. Banerjee A, Brook MM, Klautz RJ, Teitel DF. Nonlinearity of the left ventricular end-systolic wall stress-velocity of fiber shortening relation in young pigs: a potential pitfall in its use as a single-beat index of contractility. J Am Coll Cardiol. 1994;23:514-524. 118. Kass DA, Beyar R, Lankford E, Heard M, Maughan WL, Sagawa K. Influence of contractile state on curvilinearity of in situ endsystolic pressure-volume relations. Circulation. 1989;79:167-178. 119. Krosl P, Abel FL. Problems with use of the end systolic pressurevolume slope as an indicator of left ventricular contractility: an alternate method. Shock. 1998;10:285-291. 120. Colan SD, Borow KM, Neumann A. Left ventricular end-systolic wall stress-velocity of fiber shortening relation: a load-independent index of myocardial contractility. J Am Coll Cardiol. 1984;4: 715-724. 121. Mertens L, Friedberg MK. Systolic ventricular function. In: Lai WW, Mertens LL, Cohen MS, Geva T, eds. Echocardiography in Pediatric and Congenital Heart Disease: From Fetus to Adult. 2nd ed. West Sussex, UK: Wiley Blackwell; 2016:96-131. 122. Gaasch WH, et al. Diastolic properties of the left ventricle. In: Levine HJ, Gaasch WH, eds. The Ventricle: Basic and Clinical Aspects. Boston: Martinus Nijhoff; 1985. 123. Kirk R, Dipchand AI, Rosenthal DN, et al. The International Society for Heart and Lung Transplantation Guidelines for the management of pediatric heart failure: Executive summary. J Heart Lung Transplant. 2014;33:888-909. 124. Bers DM. Cardiac excitation-contraction coupling. Nature. 2002;415:198-205. 125. Frais MA, Bergman DW, Kingma I, Smiseth OA, Smith ER, Tyberg JV. The dependence of the time constant of left ventricular isovolumic relaxation (tau) on pericardial pressure. Circulation. 1990;81:1071-1080. 126. Mawad W, Friedberg MK. The continuing challenge of evaluating diastolic function by echocardiography in children: developing concepts and newer modalities. Curr Opin Cardiol. 2017;32: 93-100. 127. Dragulescu A, Mertens L, Friedberg MK. Interpretation of left ventricular diastolic dysfunction in children with cardiomyopathy by echocardiography: problems and limitations. Circ Cardiovasc Imaging. 2013;6:254-261. 128. Border WL, Syed AU, Michelfelder EC, et al. Impaired systemic ventricular relaxation affects postoperative short-term outcome in Fontan patients. J Thorac Cardiovasc Surg. 2003;126:1760-1764. 129. Smiseth OA, Frais MA, Junemann M, et al. Left and right ventricular diastolic function during acute pericardial tamponade. Clin Physiol. 1991;11:61-71. 130. Tyberg JV, Taichman GC, Smith ER, Douglas NW, Smiseth OA, Keon WJ. The relationship between pericardial pressure and right atrial pressure: an intraoperative study. Circulation. 1986;73: 428-432. 131. Slinker BK, Glantz SA. End-systolic and end-diastolic ventricular interaction. Am J Physiol. 1986;251:H1062-H1075. 132. Smiseth OA, Frais MA, Kingma I, et al. Assessment of pericardial constraint: the relation between right ventricular filling pressure and pericardial pressure measured after pericardiocentesis. J Am Coll Cardiol. 1986;7:307-314. 133. Morgan BC, Guntheroth WG, Dillard DH. Relationship of pericardial to pleural pressure during quiet respiration and cardiac tamponade. Circ Res. 1965;16:493-498. 134. Tyberg JV, Grant DA, Kingma I, et al. Effects of positive intrathoracic pressure on pulmonary and systemic hemodynamics. Respir Physiol. 2000;119:171-179. 135. Naughton MT, Rahman MA, Hara K, Floras JS, Bradley TD. Effect of continuous positive airway pressure on intrathoracic and left ventricular transmural pressures in patients with congestive heart failure. Circulation. 1995;91:1725-1731.

















































93. Misky I. Factors which affect the diastolic pressure-volume curve. Circ Res. 1979;44:592-595. 94. Tyberg JV, Belenkie I, Manyari DE, Smith ER. Ventricular interaction and venous capacitance modulate left ventricular preload. Can J Cardiol. 1996;12:1058-1064. 95. Tyberg JV, Misbach GA, Glantz SA, Moores WY, Parmley WW. A mechanism for shifts in the diastolic, left ventricular, pressure-volume curve: the role of the pericardium. Eur J Cardiol. 1978;7(suppl):163-175. 96. Ross Jr J. Afterload mismatch and preload reserve: a conceptual framework for the analysis of ventricular function. Prog Cardiovasc Dis. 1976;18:255-264. 97. Ross Jr J. Mechanisms of cardiac contraction. What roles for preload, afterload and inotropic state in heart failure? Eur Heart J. 1983;4(suppl A):19-28. 98. Carabello BA, Spann JF. The uses and limitations of end-systolic indexes of left ventricular function. Circulation. 1984;69:1058-1064. 99. McHale PA, Greenfield Jr JC. Evaluation of several geometric models for estimation of left ventricular circumferential wall stress. Circ Res. 1973;33:303-312. 100. Grossman W, Jones D, McLaurin LP. Wall stress and patterns of hypertrophy in the human left ventricle. J Clin Invest. 1975;56:56-64. 101. Glower DD, Spratt JA, Snow ND, et al. Linearity of the FrankStarling relationship in the intact heart: the concept of preload recruitable stroke work. Circulation. 1985;71:994-1009. 102. Goldsmith SR. Congestive heart failure: potential role of arginine vasopressin antagonists in the therapy of heart failure. Congest Heart Fail. 2002;8:251-256. 103. Schwinger RH, Böhm M, Koch A, et al. The failing human heart is unable to use the Frank-Starling mechanism. Circ Res. 1994;74:959-969. 104. Anderson PA, Rankin JS, Arentzen CE, Anderson RW, Johnson EA. Evaluation of the force-frequency relationship as a descriptor of the inotropic state of canine left ventricular myocardium. Circ Res. 1976;39:832-839. 105. Pidgeon J, Lab M, Seed A, et al. The contractile state of cat and dog heart in relation to the interval between beats. Circ Res. 1980;47: 559-567. 106. Seed WA, Noble MI, Walker JM, et al. Relationships between beatto-beat interval and the strength of contraction in the healthy and diseased human heart. Circulation. 1984;70:799-805. 107. Mulieri LA, Hasenfuss G, Leavitt B, Allen PD, Alpert NR. Altered myocardial force-frequency relation in human heart failure. Circulation. 1992;85:1743-1750. 108. Hasenfuss G, Holubarsch C, Hermann HP, Astheimer K, Pieske B, Just H. Influence of the force-frequency relationship on haemodynamics and left ventricular function in patients with non-failing hearts and in patients with dilated cardiomyopathy. Eur Heart J. 1994;15:164-170. 109. Sagawa K. The ventricular pressure-volume diagram revisited. Circ Res. 1978;43:677-687. 110. Sagawa K. The end-systolic pressure-volume relation of the ventricle: definition, modifications and clinical use. Circulation. 1981;63: 1223-1227. 111. Suga H. Ventricular energetics. Physiol Rev. 1990;70:247-277. 112. Redington AN, Gray HH, Hodson ME, Rigby ML, Oldershaw PJ. Characterisation of the normal right ventricular pressure-volume relation by biplane angiography and simultaneous micromanometer pressure measurements. Br Heart J. 1988;59:23-30. 113. Redington AN, Rigby ML, Shinebourne EA, Oldershaw PJ. Changes in the pressure-volume relation of the right ventricle when its loading conditions are modified. Br Heart J. 1990;63:45-49. 114. Matthay RA, Arroliga AC, Wiedemann HP, Schulman DS, Mahler DA. Right ventricular function at rest and during exercise in chronic obstructive pulmonary disease. Chest. 1992;101(suppl 5):255S-262S. 115. Parmley WW, Chuck L, Yeatman L. Comparative evaluation of the specificity and sensitivity of isometric indices of contractility. Am J Physiol. 1975;228:506-510.

e4





















































158. Gibbs CJ, Chapman JB. Cardiac energetics. In: Berne RM, Sperelakis N, eds. Handbook of Physiology, Section 2, the Cardiovascular System. The Heart. Vol 1. Bethesda, MD: American Physiological Society; 2001. 159. Hoffman JIE. Coronary physiology. In: Garfield OB, ed. Current Concepts in Cardiovascular Physiology. New York: Academic Press; 1990. 160. Kitamura K, Jorgensen CR, Gobel FL, Taylor HL, Wang Y. Hemodynamic correlates of myocardial oxygen consumption during upright exercise. J Appl Physiol. 1972;32:516-522. 161. Opie LH. High energy phosphate compounds. In: Drake-Holland AJ, Noble MTM, eds. Cardiac Metabolism. Chichester, United Kingdom: John Wiley & Sons; 1983. 162. Opie LH. Fuels: carbohydrates and lipids. In: Opie LH, ed. The Heart: Physiology and Metabolism. New York: Raven Press; 1991. 163. Rebouche CJ, Paulson DJ. Carnitine metabolism and function in humans. Annu Rev Nutr. 1986;6:41-66. 164. Retter AS. Carnitine and its role in cardiovascular disease. Heart Dis. 1999;1:108-113. 165. Sole MJ, Jeejeebhoy KN. Conditioned nutritional requirements: therapeutic relevance to heart failure. Herz. 2002;27:174-178. 166. Gertz EW, Wisneski JA, Neese R, Houser A, Korte R, Bristow JD. Myocardial lactate extraction: multi-determined metabolic function. Circulation. 1980;61:256-261. 167. Fisher DJ. Cardiac output and regional blood flows during hypoxaemia in unanaesthetized newborn lambs. J Dev Physiol. 1984;6: 485-494. 168. Fisher DJ. Increased regional myocardial blood flows and oxygen deliveries during hypoxemia in lambs. Pediatr Res. 1984;18:602606. 169. Clark JB, Clark Jr CM. The growth and metabolism of the developing heart. In: Jones CT, ed. The Biochemical Development of the Fetus and Neonate. Amsterdam: Elsevier Biomedical; 1982. 170. Sarnoff SJ, Mitchell JH. The control of the function of the heart. In: Hamilton WE, Dow P, eds. Handbook of Physiology. Section 2: Circulation. Vol 1. Washington, DC: American Physiological Society; 2001. 171. McDonald Jr RH, Taylor RR, Cingolani HE. Measurement of myocardial developed tension and its relation to oxygen consumption. Am J Physiol. 1966;211:667-673. 172. Parmley WW, Tyberg JV. Determinants of myocardial oxygen demand. In: Yu P, Goodwin JF, eds. Progress in Cardiology. Philadelphia: Lea & Febiger; 1988. 173. Strauer BE. Myocardial oxygen consumption in chronic heart disease: role of wall stress, hypertrophy and coronary reserve. Am J Cardiol. 1979;44:730-740. 174. Taylor RR. Myocardial oxygen consumption, left ventricular fibre shortening and wall tension. Cardiovasc Res. 1967;1:219. 175. Graham Jr TP, Covell JW, Sonnenblick EH, Ross Jr J, Braunwald E. Control of myocardial oxygen consumption: relative influence of contractile state and tension development. J Clin Invest. 1968;47:375-385. 176. Rooke GA, Feigl EO. Work as a correlate of canine left ventricular oxygen consumption, and the problem of catecholamine oxygen wasting. Circ Res. 1982;50:273-286. 177. Vinten-Johansen J, Duncan HW, Finkenberg JG, et al. Prediction of myocardial O2 requirements by indirect indices. Am J Physiol. 1982;243:H862-H868. 178. Weber KT, Janicki JS. Myocardial oxygen consumption: the role of wall force and shortening. Am J Physiol. 1977;233:H421-H430. 179. Weber KT, Janicki JS. Interdependence of cardiac function, coronary flow, and oxygen extraction. Am J Physiol. 1978;235:H784-H793. 180. Hata K, Goto Y, Kawaguchi O, et al. Hypercapnic acidosis increases oxygen cost of contractility in the dog left ventricle. Am J Physiol. 1994;266:H730-H740. 181. Hata K, Takasago T, Saeki A, Nishioka T, Goto Y. Stunned myocardium after rapid correction of acidosis. Increased oxygen cost of contractility and the role of the Na(1)-H1 exchange system. Circ Res. 1994;74:794-805.















































136. Santamore WP, Dell’Italia LJ. Ventricular interdependence: significant left ventricular contributions to right ventricular systolic function. Prog Cardiovasc Dis. 1998;40:289-308. 137. Weber KT, Janicki JS, Shroff S, Fishman AP. Contractile mechanics and interaction of the right and left ventricles. Am J Cardiol. 1981;47:686-695. 138. Belenkie I, Dani R, Smith ER, Tyberg JV. The importance of pericardial constraint in experimental pulmonary embolism and volume loading. Am Heart J. 1992;123:733-742. 139. Belenkie I, Smith ER, Tyberg JV. Ventricular interaction: from bench to bedside. Ann Med. 2001;33:236-241. 140. Penny DJ, Redington AN. Function of the left and right ventricles and the interactions between them. Pediatr Crit Care Med. 2016;17:S112-S118. 141. Friedberg MK, Redington AN. Right versus left ventricular failure: differences, similarities, and interactions. Circulation. 2014;129:1033-1044. 142. Damiano Jr RJ, La Follette Jr P, Cox JL, et al. Significant left ventricular contribution to right ventricular systolic function. Am J Physiol. 1991;261:H1514-H1524. 143. Brookes C, Ravn H, White P, Moeldrup U, Oldershaw P, Redington A. Acute right ventricular dilatation in response to ischemia significantly impairs left ventricular systolic performance. Circulation. 1999;100:761-767. 144. Vatner SF. Sympathetic mechanisms regulating myocardial contractility in conscious animals. In: Fozzard HA, et al., eds. The Heart and Cardiovascular System: Scientific Foundations. New York: Raven Press; 1991. 145. Mangoni ME, Nargeot J. Genesis and regulation of the heart automaticity. Physiol Rev. 2008;88:919-982. 146. Acker H. Chemoreceptor and baroreceptor control of perinatal oxygen supply in different organs. In: Jones CT, ed. Research in Perinatal Medicine. Fetal and Neonatal Development. Vol 7. Ithaca, NY: Perinatology Press; 1988. 147. Hawkins J, Van Hare GF, Schmidt KG, Rudolph AM. Effects of increasing afterload on left ventricular output in fetal lambs. Circ Res. 1989;65:127-134. 148. Gilbert RD. Effects of afterload and baroreceptors on cardiac function in fetal sheep. J Dev Physiol. 1982;4:299-309. 149. Gilbert RD. Control and distribution of cardiac output in the fetus. In: Jones CT, ed. Research in Perinatal Medicine. Fetal and Neonatal Development. Vol 7. Ithaca, NY: Perinatology Press; 1988. 150. Lister G, Walter TK, Versmold HT, Dallman PR, Rudolph AM. Oxygen delivery in lambs: cardiovascular and hematologic development. Am J Physiol. 1979;237:H668-H675. 151. Breall JA, Rudolph AM, Heymann MA. Role of thyroid hormone in postnatal circulatory and metabolic adjustments. J Clin Invest. 1984;73:1418-1424. 152. Klopfenstein HS, Rudolph AM. Postnatal changes in the circulation and responses to volume loading in sheep. Circ Res. 1978;42:839-845. 153. Teitel DF, Sidi D, Chin T, Brett C, Heymann MA, Rudolph AM. Developmental changes in myocardial contractile reserve in the lamb. Pediatr Res. 1985;19:948-955. 154. Van Hare GF, Hawkins JA, Schmidt KG, Rudolph AM. The effects of increasing mean arterial pressure on left ventricular output in newborn lambs. Circ Res. 1990;67:78-83. 155. Clyman RI, Teitel D, Padbury J, Roman C, Mauray F. The role of beta-adrenoreceptor stimulation and contractile state in the preterm lamb’s response to altered ductus arteriosus patency. Pediatr Res. 1988;23:316-322. 156. Chapman JB. Heat production. In: Drake-Holland A, Noble MIM, eds. Cardiac Metabolism. Chichester, United Kingdom: John Wiley & Sons; 1983. 157. Elzinga G. Cardiac oxygen consumption and the production of heat and work. In: Drake-Holland AJ, Noble MIM, eds. Cardiac Metabolism. Chichester, United Kingdom: John Wiley & Sons; 1983.

e5























































207. Buga GM, Ignarro LJ. Vascular endothelium and smooth muscle function. In: Gluckman PD, Heymann MA, eds. Pediatrics and Perinatology. The Scientific Basis. London: Edward Arnold; 1996. 208. Stenmark KR, Weiser MC. Vascular development and function. In: Gluckman PD, Heymann MA, eds. Pediatrics and Perinatology. The Scientific Basis. London: Edward Arnold; 1996. 209. Springer TA. Traffic signals on endothelium for lymphocyte recirculation and leukocyte emigration. Annu Rev Physiol. 1995;57:827-872. 210. Andrews M, Massicotte P. Hemostasis. In: Gluckman PD, Heymann MA, eds. Pediatrics and Perinatology. The Scientific Basis. London: Edward Arnold; 1996. 211. Friedman MH, Fry DL. Arterial permeability dynamics and vascular disease. Atherosclerosis. 1993;104:189-194. 212. Fry DL. Acute vascular endothelial changes associated with increased blood velocity gradients. Circ Res. 1968;22:165-197. 213. Franco-Cereceda A, Gennari C, Nami R, et al. Cardiovascular effects of calcitonin gene-related peptides I and II in man. Circ Res. 1987;60: 393-397. 214. Beaulieu P, Lambert C. Peptidic regulation of heart rate and interactions with the autonomic nervous system. Cardiovasc Res. 1998; 37:578-585. 215. DiPette DJ, Schwarzenberger K, Kerr N, Holland OB. Systemic and regional hemodynamic effects of calcitonin gene-related peptide. Hypertension. 1987;9:III142-III146. 216. Frase LL, Gaffney FA, Lane LD, et al. Cardiovascular effects of vasoactive intestinal peptide in healthy subjects. Am J Cardiol. 1987; 60:1356-1361. 217. Gangula PR, Zhao H, Supowit SC, et al. Increased blood pressure in alpha-calcitonin gene-related peptide/calcitonin gene knockout mice. Hypertension. 2000;35:470-475. 218. Hagner S, Stahl U, Knoblauch B, McGregor GP, Lang RE. Calcitonin receptor-like receptor: identification and distribution in human peripheral tissues. Cell Tissue Res. 2002;310:41-50. 219. Henning RJ, Sawmiller DR. Vasoactive intestinal peptide: cardiovascular effects. Cardiovasc Res. 2001;49:27-37. 220. Mione MC, Ralevic V, Burnstock G. Peptides and vasomotor mechanisms. Pharmacol Ther. 1990;46:429-468. 221. Onuoha GN, Alpar EK, Chukwulobelu R, Nicholls DP. Distributions of VIP, substance P, neurokinin A and neurotensin in rat heart: an immunocytochemical study. Neuropeptides. 1999;33:19-25. 222. Preibisz JJ. Calcitonin gene-related peptide and regulation of human cardiovascular homeostasis. Am J Hypertens. 1993;6:434-450. 223. Tamburino G, Malatino LS. The vasoactive peptidergic system. Ann Ital Med Int. 1994;9:88-91. 224. Wang BC, Bie P, Leadley Jr RJ, Goetz KL. Cardiovascular effects of calcitonin gene-related peptide in conscious dogs. Am J Physiol. 1989;257:R726-R731. 225. Wharton J, Gulbenkian S. Peptides in the mammalian cardiovascular system. Experientia. 1987;43:821-832. 226. Furness JB, Costa M, Papka RE, Della NG, Murphy R. Neuropeptides contained in peripheral cardiovascular nerves. Clin Exp Hypertens A. 1984;6:91-106. 227. Ganong WF. Origin of the angiotensin I1 secreted by cells. Proc Soc Exp Biol Med. 1994;205:213-219. 228. Muller DN, Luft FC. The renin-angiotensin system in the vessel wall. Basic Res Cardiol. 1998;93:7-14. 229. Burlew BS, Weber KT. Connective tissue and the heart. Functional significance and regulatory mechanisms. Cardiol Clin. 2000;18:435-442. 230. Dzau VJ, Bernstein K, Celermajer D, et al. Pathophysiologic and therapeutic importance of tissue ACE: a consensus report. Cardiovasc Drugs Ther. 2002;16:149-160. 231. Unger T. The role of the renin-angiotensin system in the development of cardiovascular disease. Am J Cardiol. 2002;89(2A):3A-9A. 232. Usui M, Egashira K, Tomita H, et al. Important role of local angiotensin II activity mediated via type 1 receptor in the pathogenesis of cardiovascular inflammatory changes induced by chronic blockade of nitric oxide synthesis in rats. Circulation. 2000;101: 305-310.





















































182. Suga H. Paul Dudley White International Lecture: cardiac performance as viewed through the pressure-volume window. Jpn Heart J. 1994;35:263-280. 183. Suga H, Goto Y, Yasumura Y, et al. O2 consumption of dog heart under decreased coronary perfusion and propranolol. Am J Physiol. 1988;254:H292-H303. 184. Suga H, Hayashi T, Shirahata M, Suehiro S, Hisano R. Regression of cardiac oxygen consumption on ventricular pressure-volume area in dog. Am J Physiol. 1981;240:H320-H325. 185. Suga H, Hisano R, Goto Y, Yamada O, Igarashi Y. Effect of positive inotropic agents on the relation between oxygen consumption and systolic pressure volume area in canine left ventricle. Circ Res. 1983;53:306-318. 186. Hoffman JIE. Transmural myocardial perfusion. Prog Cardiovasc Dis. 1987;29:429. 187. Hoffinan JI, Buckberg GD. The myocardial supply:demand ratio— a critical review. Am J Cardiol. 1978;41:327-332. 188. Brazier J, Cooper N, Buckberg G. The adequacy of subendocardial oxygen delivery: the interaction of determinants of flow, arterial oxygen content and myocardial oxygen need. Circulation. 1974; 49:968-977. 189. Barnard RJ, MacAlpin R, Kattus AA, Buckberg GD. Ischemic response to sudden strenuous exercise in healthy men. Circulation. 1973;48:936-942. 190. Hearse DJ. Myocardial ischaemia: can we agree on a definition for the 21st century? Cardiovasc Res. 1994;28:1737-1744. 191. Reimer KA, Jennings RB, et al. Myocardial ischemia, hypoxia, and infarction. In: Fozzard HA, ed. The Heart and Cardiovascular System: Scientific Foundations. New York: Raven Press; 1991. 192. Canty Jr JM. Coronary pressure-function and steady-state pressure-flow relations during autoregulation in the unanesthetized dog. Circ Res. 1988;63:821-836. 193. Gallagher KP, Kumada T, Koziol JA, et al. Significance of regional wall thickening abnormalities relative to transmural myocardial perfusion in anesthetized dogs. Circulation. 1980;62:1266-1274. 194. Vatner SF. Correlation between acute reductions in myocardial blood flow and function in conscious dogs. Circ Res. 1980;47: 201-207. 195. Cooper HA, Braunwald E. Clinical importance of stunned and hibernating myocardium. Coron Artery Dis. 2001;12:387-392. 196. Kern KB. Postresuscitation myocardial dysfunction. Cardiol Clin. 2002;20:89-101. 197. Kloner RA, Arimie RB, Kay GL, et al. Evidence for stunned myocardium in humans: a 2001 update. Coron Artery Dis. 2001;12: 349-356. 198. Lüss H, Schäfers M, Neumann J, et al. Biochemical mechanisms of hibernation and stunning in the human heart. Cardiovasc Res. 2002;56:411-421. 199. Kato R, Foex P. Myocardial protection by anesthetic agents against ischemia-reperfusion injury: an update for anesthesiologists. Can J Anaesth. 2002;49:777-791. 200. Tennyson H, Kern KB, Hilwig RW, Berg RA, Ewy GA. Treatment of post resuscitation myocardial dysfunction: aortic counterpulsation versus dobutamine. Resuscitation. 2002;54:69-75. 201. Canty Jr JM, Fallavollita JA. Lessons from experimental models of hibernating myocardium. Coron Artery Dis. 2001;12:371-380. 202. Heusch G, Schulz R. The biology of myocardial hibernation. Trends Cardiovasc Med. 2000;10:108-114. 203. Heusch G, Schulz R. Hibernating myocardium: new answers, still more questions! Circ Res. 2002;91:863-865. 204. Mari C, Strauss WH. Detection and characterization of hibernating myocardium. Nucl Med Commun. 2002;23:311-322. 205. Barlow CH, Harken AH, Chance B. Evaluation of cardiac ischemia by NADH fluoroescence photography. Ann Surg. 1977;186: 737-740. 206. Wolinsky H, Glagov S. Comparison of abdominal and thoracic aortic medial structure in mammals. Deviation of man from the usual pattern. Circ Res. 1969;25:677-686.

e6

























































257. Floras JS. Vasopeptidase inhibition: a novel approach to cardiovascular therapy. Can J Cardiol. 2002;18:177-182. 258. Lapointe N, Rouleau JL. Cardioprotective effects of vasopeptidase inhibitors. Can J Cardiol. 2002;18:415-420. 259. Nawarskas J, Rajan V, Frishman WH. Vasopeptidase inhibitors, neutral endopeptidase inhibitors, and dual inhibitors of angiotensin-converting enzyme and neutral endopeptidase. Heart Dis. 2001;3:378-385. 260. Frishman WH, Grewall P. Serotonin and the heart. Ann Med. 2000;32:195-209. 261. Frishman WH, Huberfeld S, Okin S, Wang YH, Kumar A, Shareef B. Serotonin and serotonin antagonism in cardiovascular and noncardiovascular disease. J Clin Pharmacol. 1995;35:541-572. 262. Feigl EO. Coronary physiology. Physiol Rev. 1983;63:1-205. 263. Brace RA. Time course and mechanisms of the acute effects of hypokalemia and hyperkalemia on vascular resistance. Proc Soc Exp Biol Med. 1974;145:1389-1394. 264. Brace RA, Anderson DK, Chen WT, Scott JB, Haddy FJ. Local effects of hypokalemia on coronary resistance and myocardial contractile force. Am J Physiol. 1974;227:590-597. 265. Murray PA, Belloni FL, Sparks HV. The role of potassium in the metabolic control of coronary vascular resistance of the dog. Circ Res. 1979;44:767-780. 266. Murray PA, Sparks HV. The mechanism of K1-induced vasodilation of the coronary vascular bed of the dog. Circ Res. 1978;42:35-42. 267. Brayden JE. Functional roles of KATP channels in vascular smooth muscle. Clin Exp Pharmacol Physiol. 2002;29:312-316. 268. Furchgott RF. Role of endothelium in responses of vascular smooth muscle. Circ Res. 1983;53:557-573. 269. Ignarro LJ, Byrns RE, Buga GM, Wood KS. Endothelium-derived relaxing factor from pulmonary artery and vein possesses pharmacologic and chemical properties identical to those of nitric oxide radical. Circ Res. 1987;61:866-879. 270. Liu Y, Gutterman DD. Vascular control in humans: focus on the coronary microcirculation. Basic Res Cardiol. 2009;104:211-227. 271. Agapitov AV, Haynes WG. Role of endothelin in cardiovascular disease. J Renin Angiotensin Aldosterone Syst. 2002;3:1-15. 272. Evans JJ, Youssef AH, Yandle TG, Lewis LK, Nicholls MG. Effects of endothelin-1 on release of adrenomedullin and C-type natriuretic peptide from individual human vascular endothelial cells. J Endocrinol. 2002;175:225-232. 273. Goraca A. New views on the role of endothelin (minireview). Endocr Regul. 2002;36:161-167. 274. Parris RJ, Webb DJ. The endothelin system in cardiovascular physiology and pathophysiology. Vasc Med. 1997;2:31-43. 275. Minkes RK, Kadowitz PJ. Differential effects of rat endothelin on regional blood flow in the cat. Eur J Pharmacol. 1989;165:161-164. 276. Yang ZH, Richard V, von Segesser L, et al. Threshold concentrations of endothelin-1 potentiate contractions to norepinephrine and serotonin in human arteries. A new mechanism of vasospasm? Circulation. 1990;82:188-195. 277. Ono K, Matsumori A. Endothelin antagonism with bosentan: current status and future perspectives. Cardiovasc Drug Rev. 2002; 20:1-18. 278. Vinten-Johansen J, Duncan HW, Finkenberg JG, et al. Prediction of myocardial O2 requirements by indirect indices. Am J Physiol. 1982;243:H862-H868. 279. Johnson PC. The myogenic response. In: Bohr DF, et al., eds. Handbook of Physiology. The Cardiovascular System. Vol 2. Bethesda, MD: American Physiological Society; 2001. 280. Cornelissen AJ, Dankelman J, VanBavel E, Spaan JA. Balance between myogenic, flow-dependent, and metabolic flow control in coronary arterial tree: a model study. Am J Physiol Heart Circ Physiol. 2002;282:H2224-H2237. 281. Acker H. Cellular oxygen sensors. Ann N Y Acad Sci. 1994;718:3-10. 282. Budinger GR, Duranteau J, Chandel NS, Schumacker PT. Hibernation during hypoxia in cardiomyocytes. Role of mitochondria as the O2 sensor. J Biol Chem. 1998;273:3320-3326.



















































233. Eto T. A review of the biological properties and clinical implications of adrenomedullin and proadrenomedullin N-terminal 20 peptide (PAW), hypotensive and vasodilating peptides. Peptides. 2001;22:1693-1711. 234. Lah JJ, Frishman WH. Adrenomedullin: a vasoactive and natriuretic peptide with therapeutic potential. Heart Dis. 2000;2:259265. 235. Rademaker MT, Charles CJ, Espiner EA, Nicholls MG, Richards AM. Long-term adrenomedullin administration in experimental heart failure. Hypertension. 2002;40:667-672. 236. Rocha R, Funder JW. The pathophysiology of aldosterone in the cardiovascular system. Ann N Y Acad Sci. 2002;970:89-100. 237. Rossi GP, Cavallin M, Nussdorfer GG, Pessina AC. The endothelin-aldosterone axis and cardiovascular diseases. J Cardiovasc Pharmacol. 2001;38(suppl 2):S49-S52. 238. Young MJ, Funder JW. Mineralocorticoid receptors and pathophysiological roles for aldosterone in the cardiovascular system. J Hypertens. 2002;20:1465-1468. 239. Soberman J, Chafin CC, Weber KT. Aldosterone antagonists in congestive heart failure. Curr Opin Investig Drugs. 2002;3:1024-1028. 240. Weber MA. Clinical implications of aldosterone blockade. Am Heart J. 2002;144:S12-S18. 241. Holmes CL, Patel BM, Russell JA, Walley KR. Physiology of vasopressin relevant to management of septic shock. Chest. 2001;120:989-1002. 242. Zimmerman MA, Albright TN, Raeburn CD, Selzman CH. Vasopressin in cardiovascular patients: therapeutic implications. Expert Opin Pharmacother. 2002;3:505-512. 243. Verbalis JG. Vasopressin V2 receptor antagonists. J Mol Endocrinol. 2002;29:1-9. 244. Serradeil-Le Gal C. An overview of SRl21463, a selective nonpeptide vasopressin V2 receptor antagonist. Cardiovasc Drug Rev. 2001;19:201-214. 245. Serradeil-Le Gal C, Wagnon J, Valette G, et al. Nonpeptide vasopressin receptor antagonists: development of selective and orally active V1a, V2 and V1b receptor ligands. Prog Brain Res. 2002;139:197-210. 246. Thibonnier M, Coles P, Thibonnier A, Shoham M. Molecular pharmacology and modeling of vasopressin receptors. Prog Brain Res. 2002;139:179-196. 247. Campbell DJ. The kallikrein-kinin system in humans. Clin Exp Pharmacol Physiol. 2001;28:1060-1065. 248. Erdos EG. Kinins, the long march—a personal view. Cardiovasc Res. 2002;54:485. 249. Erdös EG, Deddish PA. The kinin system: suggestions to broaden some prevailing concepts. Int Immunopharmacol. 2002;2:1741-1746. 250. Kaplan AP, Joseph K, Silverberg M. Pathways for bradykinin formation and inflammatory disease. J Allergy Clin Immunol. 2002;109:195-209. 251. Witherow FN, Dawson P, Ludlam CA, Fox KA, Newby DE. Marked bradykinin-induced tissue plasminogen activator release in patients with heart failure maintained on long-term angiotensinconverting enzyme inhibitor therapy. J Am Coll Cardiol. 2002;40:961-966. 252. van Hinsbergh VW, van Nieuw Arnerongen GP. Intracellular signalling involved in modulating human endothelial barrier function. J Anat. 2002;200:549-560. 253. Burger AJ, Horton DP, LeJemtel T, et al. Effect of nesiritide (B-type natriuretic peptide) and dobutamine on ventricular arrhythmias in the treatment of patients with acutely decompensated congestive heart failure: the PRECEDENT shady. Am Heart J. 2002;144:1102-1108. 254. Elkayam U, Akhter MW, Tummala P, Khan S, Singh H. Nesiritide: a new drug for the treatment of decompensated heart failure. J Cardiovasc Pharmacol Ther. 2002;7:181-194. 255. Hansson M. Natriuretic peptides in relation to the cardiac innervation and conduction system. Microsc Res Tech. 2002;58:378-386. 256. Keating GM, Goa KL. Nesiritide: a review of its use in acute decompensated heart failure. Drugs. 2003;63:47-70.

e7

















290. Johnson FK, Teran FJ, Prieto-Carrasquero M, Johnson RA. Vascular effects of a heme oxygenase inhibitor are enhanced in the absence of nitric oxide. Am J Hypertens. 2002;15:1074-1080. 291. Morse D, Sethi J, Choi AM. Carbon monoxide-dependent signaling. Crit Care Med. 2001;30:S12-S17. 292. Zhang F, Kaide JI, Rodriguez-Mulero F, Abraham NG, Nasjletti A. Vasoregulatory function of the heme-heme oxygenase-carbon monoxide system. Am J Hypertens. 2001;14:62S-67S. 293. Chilian WM, Layne SM, Klausner EC, Eastham CL, Marcus ML. Redistribution of coronary microvascular resistance produced by dipyridamole. Am J Physiol. 1989;256:H383-H390. 294. Webb DJ, Strachan FE. Clinical experience with endothelin antagonists. Am J Hypertens. 1998;11:71S-79S. 295. Smart EJ, Graf GA, McNiven MA, et al. Caveolins, liquid-ordered domains, and signal transduction. Mol Cell Biol. 1999;19:72897304. 296. Gonzalez-Alonso J, Olsen DB, Saltin B. Erythrocyte and the regulation of human skeletal muscle blood flow and oxygen delivery: role of circulating ATP. Circ Res. 2002;91:1046-55.

















283. Chandel NS, Schumacker PT. Cellular oxygen sensing by mitochondria: old questions, new insight. J Appl Physiol. 2000;88:18801889. 284. Cannon RO III, Schechter AN, Panza JA, et al. Effects of inhaled nitric oxide on regional blood flow are consistent with intravascular nitric oxide delivery. J Clin Invest. 2001;108:279-287. 285. Gladwin MT, Shelhamer JH, Schechter AN, et al. Role of circulating nitrite and S-nitrosohemoglobin in the regulation of regional blood flow in humans. Proc Natl Acad Sci U S A. 2000;97:1148211487. 286. Jagger JE, Bateman RM, Ellsworth ML, et al. Role of erythrocyte in regulating local O2 delivery mediated by hemoglobin oxygenation. Am J Physiol Heart Circ Physiol. 2001;280:H2833-H2839. 287. Pollack GH. Maximum velocity as an index of contractility in cardiac muscle. A critical evaluation. Circ Res. 1970;26:111-127. 288. Stamler JS, Jia L, Eu JP, et al. Blood flow regulation by S-nitrosohemoglobin in the physiological oxygen gradient. Science. 1997;276:2034-2037. 289. Coceani F. Carbon monoxide in vasoregulation: the promise and the challenge. Circ Res. 2000;86:1184-1186.

e8

Abstract: Myocardial (macroscopic and microscopic) structure and function are described in this chapter to help the reader understand the physiologic principles (e.g., Frank-Starling mechanism, force-frequency relationship, end-systolic pressure-volume relationship) that govern the normal heart. Regulation of stroke volume, myocardial metabolism, and the effect of ischemia on cardiac function are also explored. Finally, the systemic vasculature and control of vascular tone are reviewed. After reading this

chapter, the reader will be able to understand the determinants of cardiac output and how the heart adapts to meet changing metabolic demands.

Key words: Segmental anatomy, myocyte, sarcomeres, cardiac output, systemic vasculature, cardiac function

24 Regional Peripheral Circulation PETER OISHI, JULIEN I. HOFFMAN, BRADLEY P. FUHRMAN, AND JEFFREY R. FINEMAN

PEARLS •















When delivering critical care, one must understand the specific properties that characterize the various regional circulations because therapies that benefit one region may be detrimental to another. Vascular tone is influenced by (1) innervation and neural processes, (2) circulating endocrine and neuroendocrine mediators, (3) local metabolic products, (4) blood gas composition, (5) endothelial-derived factors, and (6) myogenic processes. The transition from the fetal pulmonary circulation to the postnatal pulmonary circulation is marked by a dramatic fall in pulmonary vascular resistance and rise in pulmonary blood flow. The failure to successfully make this transition is integral to a number of neonatal and infant diseases. An important feature unique to the cerebral circulation is the presence of a blood-brain barrier. As a result, the cerebral













vasculature responds differently from other vascular beds to humoral stimuli. Regulation of myocardial perfusion is tailored to match regional myocardial oxygen supply to demand over the widest possible range of cardiac workload. Increases in myocardial oxygen demand must be met by increases in myocardial blood flow. Critically ill patients are at risk for impaired splanchnic blood flow that can impair the two chief functions of the gastrointestinal system: (1) digestion and absorption of nutrients and (2) maintenance of a barrier to the translocation of enteric antigens. Splanchnic ischemia is associated with increased morbidity and mortality in critically ill patients. Although renal blood flow remains constant over a wide range of renal artery perfusion pressures, urinary flow rate varies as a function of renal perfusion pressure.

General Features

Basic Physiology

General Anatomy

Blood flow to a regional vascular bed is determined primarily by inflow pressure, vascular resistance, and outflow pressure. Inflow pressure is usually systemic arterial pressure. Outflow pressure approximates venous pressure but may exceed venous pressure at times if vascular tone is great enough to close the circulation above venous pressure or if external pressure impinges on the vasculature. In a model to explain the relation of arterial pressure to flow, the circulation is represented by two capacitance vessels separated by a resistance. A standpipe full of blood is allowed to discharge its contents into the arterial vasculature (the proximal capacitance). Blood flows across the resistance site (the arterioles), traverses the venous vasculature (the distal capacitance), and drains to a reservoir at some outflow pressure (Po; Fig. 24.1). The pressure head of the system (Pi) is generated by the weight of the column of blood in the standpipe and is proportional to its height (Pi 5 blood column height in cm H2O). As the standpipe discharges, the height decreases and Pi decreases. This, in turn, decreases the rate of flow (Q) through the vasculature. Q decreases almost linearly with Pi until the column is quite low. Ultimately, flow will cease while there is still pressure in the standpipe.

Blood vessels comprise several distinct layers. Moving from the innermost layer outward are the metabolically active endothelium, intima (with nerves and vasa vasorum), media, and adventitia. Some vessels have fewer layers depending on the position and function of the vessel within the circulation. The large arteries are elastic. Their media contain concentric lamellae of perforated elastic tubes crosslinked by transverse collagen and smooth muscle. When smooth muscle contracts, the wall becomes stiffer. Smaller arteries have fewer lamellae. The media are bounded by the internal and external elastic laminae. Arterioles are less elastic, have no lamellae, and have a thin medium with circular or spiral smooth muscle and inner and outer elastic laminae. Capillaries are thin walled and nonmuscular, ideal for transport of materials to and from the tissues. Veins have medial muscle but thinner walls relative to lumen diameter compared with arteries. The vascular endothelium has important metabolic characteristics, which may differ between vessel types (i.e., arteries vs. veins) and different regions.1,2

203

SECTION IV



204

Pediatric Critical Care: Cardiovascular

Standpipe Pump

Pi

RA

AO

Reservoir A V

Rm V

V

V

•  Fig. 24.1

Ve ins Cv Pc

Po

​Model for facilitating interpretation of vascular pressure-flow relations. When valves (V) are properly positioned, fluid filling the standpipe to height Pi discharges across the circulation to the reservoir at outflow pressure Po. A, Artery; Rm, microvascular resistance; V, vein.  

es eri Art Ca Pi



Rm

•  Fig. 24.3

​Model for facilitating interpretation of the relation of venous return to right atrial pressure. The heart is replaced by a mechanical roller pump. The right atrium (RA) is drained by the pump, and blood is infused into the aorta (AO). Blood then traverses the arteries, which have a capacitance (Ca) at an inflow pressure (Pi) determined by flow rate and microvascular resistance (Rm). Blood then returns through veins having capacitance (Cv) and critical closing pressure (Pc) to the right atrium.  

P





Pi

Pc Po Q

•  Fig. 24.2

​As the standpipe in Fig. 24.1 discharges, Pi falls. Flow consequently slows and ultimately stops when Pi 5 Pc, the critical closing pressure of the circulation. Pi reaches outflow pressure Po only if Po  Pc.  



The pressure at which flow ceases is the critical closing pressure of the circulation (Pc).3 Pi below Pc is insufficient to maintain vessel patency and permit continued flow (Fig. 24.2). Incremental resistance to flow is generally defined as the change in pressure per unit change in flow (dPi/dQ). At pressures well above critical closing pressure, this is nearly identical to the vascular resistance (R) defined clinically as R  ( Pi  Po ) Q

When Pi does not greatly exceed Pc but Pc does greatly exceed Po, however, incremental resistance can differ substantially from this clinical estimate. Thus, an increase in Pc can be confused with a true increase in incremental resistance. For example, a diagnosis of intrinsic pulmonary vascular disease (e.g., pulmonary arterial hypertension) based on measured pulmonary artery pressures in a patient receiving mechanical ventilation with high airway pressures (that raise Pc) may be spurious.

Venous Return and Cardiac Output A second model illustrates the relationship of cardiac output to intrinsic mechanical properties of the systemic vasculature. In this model, the heart acts as a roller pump, creating a circulation much like that achieved during venoarterial extracorporeal support (Fig. 24.3). The roller pump displaces blood from the veins to the arteries and then across the resistance imposed by the arterioles. As Q increases, more blood resides in the arteries and less in the veins. This partitioning of blood depends on arterial capacitance (Ca), venous capacitance (Cv), and resistance to flow. At maximal Q, Pi and arterial blood volume are high and venous pressure (Pv) is low. When Pv reaches Pc, the roller pump cannot be increased further because the veins will collapse when Pv is less than Pc. As Q is reduced, by turning down the roller pump, arterial pressure (Pi) and volume fall and Pv increases. As Q approaches zero, venous pressure approaches the mean circulatory pressure (Pm; Fig. 24.4). The importance of this model is that it can be used to illustrate the role of venous return as an independent determinant of cardiac output. The Starling curve that describes the relationship between preload and contractility (and, hence, cardiac output) is illustrated in Fig. 24.5. Over the steep portion of the curve, optimization of myocardial preload increases ventricular stroke volume. This curve (see Chapter 23) can be superimposed on the venous return curve. Cardiac output occurs at the theoretical intersection of these two curves, representing a given state of cardiac function (Starling curve) and simultaneous set of vascular characteristics (venous return curve; see Fig. 24.5). It is important to recognize that the venous return curve is influenced by changes in blood volume and vascular tone. Transfusion elevates the maximal venous return and thus cardiac output that can be achieved before the system reaches Pc. Hemorrhage has the opposite effect. Because neither transfusion nor hemorrhage directly alters vascular tone, the slope of the curve is not altered (Fig. 24.6). Both interventions alter mean circulatory pressure because they change blood volume.



CHAPTER 24

Regional Peripheral Circulation

205

Qmax a b

Qpump

c

n sio sfu an Tr on a ti Dil l n sa io e Ba ct ag tri rrh ns mo Co He

Q

d e

0 Pc

• Fig. 24.4

PRA

Pm





​Venous return curve. As pump flow (Qpump) varies, right atrial pressure (Pra) is altered by redistribution of blood between arteries and veins. Qpump cannot be increased above Qmax because Pra would fall below critical closing pressure (Pc) of the venous circulation. Pm, Mean circulatory pressure of the vasculature at no flow. I Pm

Pc

curve

II Pm

PRA

•  Fig. 24.6

​Effects of changing blood volume and microvascular resistance on the venous return curve. Curves a, c, and e are parallel but have different mean circulatory pressure (Pm) at zero flow. Curves b, c, and d are nonparallel but have the same Pm. Pc, Critical closing pressure; PRA, right atrial pressure.

r tu re us no Ve

Q





rling Sta

Pm

n cu

e rv

PRA





•  Fig. 24.5 ​Theoretical superimposition of venous return and Starling curves. For any state of the heart and vasculature, these curves intersect at a point that characterizes right atrial pressure (Pra) and cardiac output (Q).

In patients with sepsis, intravascular volume, venous capacitance, vascular resistance, and the inotropic state of the heart can all profoundly influence cardiac output. Descriptions of shock as “warm” or “cold” relate directly to the interaction of these factors. For example, patients with warm shock often demonstrate adequate contractility with low vascular tone, whereas patients with cold shock may have poor contractility with increased or decreased vascular tone. As such, assessments of intravascular volume are important to help guide management.

Changes in vascular tone may alter the maximum venous return (and thus cardiac output) attainable before venous collapse (Pc) at any given intravascular volume. At zero flow, the mean pressure in the system would relate most directly to the volume of blood within the vessels. Thus, changes in vascular tone change the slope of the venous return curve (see Fig. 24.6). In clinical practice, it is unusual for any of these changes in vascular mechanics to occur in isolation. For instance, arteriolar dilation and dilation of other capacitance vessels often occur together. Arteriolar dilation and dilation of capacitance vessels have opposite effects on the venous return curve and, consequently, different effects on cardiac output. It is for this reason that vascular volume expansion is often required in combination with nitroprusside or milrinone infusions in order to ensure adequacy of cardiac output despite a reduction in afterload.

Critical Closing Pressure In many organs as inflow pressure is lowered, Q decreases and ceases at a pressure—the critical closing pressure (Pc)—that is higher than venous pressure. The probable mechanism is the vascular waterfall or Starling resistor. In 1910, Jerusalem and Starling2 described a device designed to control afterload to the left ventricle and that made possible the study of cardiac contractility.4 The device consisted of a collapsible rubber tube traversing a pressurized glass chamber (Fig. 24.7). When pressure surrounding the rubber tube exceeded the outflow pressure set by the reservoir, surrounding pressure opposed the flow of blood and became the true outflow pressure of the device. The physiologic counterpart of this occurs in small vessels surrounded by tissue pressure. In the heart, for example, a Starling resistor effect occurs in extramyocardial coronary veins, although there is also evidence for critical closure of small arterioles. No one has yet demonstrated vessel closure directly. However, this might not be necessary because, as

SECTION IV



206

Pediatric Critical Care: Cardiovascular

where R is resistance, l is tube length, r is the internal radius of the tube, and h is the fluid viscosity. Blood is not a Newtonian fluid, but this fact does not affect the accuracy of calculated vascular resistance much. However, vascular beds do contain many “tubes” in parallel. Thus, for vascular systems, a factor k is added that represents the number of vessels. The equation then becomes

Ps Pi

Q

•  Fig. 24.7

Po

​The Starling resistor is a compressible conduit exposed to surrounding pressure (Ps). When the Ps is less than the outflow pressure (Po), Ps does not oppose blood flow. When the Ps is between inflow (Pi) and outflow pressures, it opposes blood flow. No flow is possible when Ps exceeds Pi.  



small vessels narrow when they are compressed, the wall becomes convoluted and blood cells might become obstructed by the folds even when externally the vessel does not appear to be closed.

Autoregulation In all organs, when inflow pressure is suddenly raised or lowered while oxygen (O2) consumption remains constant, flow rises or falls transiently but then returns to its former value; the phenomenon is termed autoregulation. Several mechanisms, particularly related to O2 sensing, have been implicated in this response, but the precise mechanisms are likely complex and multifactorial. For example, studies have described a role for nitric oxide (NO) carried to the tissues by hemoglobin in the form of S-nitrosohemoglobin.5–7 Other locally produced gases, such as hydrogen sulfide8 and carbon monoxide,9,10 may also play a role. Importantly, some autoregulatory mechanisms are specific to individual microcirculations (e.g., macula densa signaling in the renal circulation). Distensibility and Compliance The distensibility of a vessel is defined as the change in volume as a proportion of the initial volume for a given change in pressure: Distensibility 

V P



1 V

where V is volume and P is pressure. Veins are much thinner than arteries and are about eight times more distensible. Multiplying distensibility by volume yields DV/DP, which is the definition of compliance. Because venous volume is usually more than three times arterial volume, venous compliance is about 20- to 30-fold greater than arterial compliance. As a result, whenever fluids are infused, the veins accommodate the bulk of the fluid volume.

Vascular Resistance Under normal circumstances, vascular resistance is the major control of organ flow and can be understood by considering the resistance of a Newtonian liquid passing through a rigid tube as defined by the Hagen-Poiseuille equation:  8 1 R    4    r 

 8 1  R    4       kr  Because the length and number of vessels and blood viscosity are relatively constant at any one time, change in vessel radius is the major factor responsible for a dynamic change in vascular resistance. Because of the fourth power factor, small changes in radius cause large changes in resistance. Vessel radius is influenced by vascular elasticity and transmural pressure but is mainly regulated by changes in vessel wall smooth muscle tone.

Vascular Impedance Resistance is strictly a steady-state concept. In a pulsatile system, the factors affecting the relationship of pressure dissipation to flow are resistance due to friction and viscosity, fluid inertia, and vessel wall compliance, which combine to produce an impedance to flow that varies with frequency. At zero frequency, steady-state resistance is approximated by the change in mean pressure overflow, but there are substantial contributions made by the first three harmonics that are ignored by this calculation.

Local Regulatory Mechanisms Regions of the circulation may differ markedly in their patterns of vascular regulation. A regulatory stimulus can have multiple effects that differ from one location to another. An agent that potently regulates vascular resistance in one region of the circulation may have no effect in another. For example, during hemorrhagic shock, flow is preserved to the heart and brain at the expense of muscle, the kidneys, and the gut. Vascular tone is strongly influenced by several mechanisms: (1) innervation and neural processes, (2) circulating endocrine and neuroendocrine mediators, (3) local metabolic products, (4) blood gas composition, (5) endothelial-derived factors, and (6) myogenic processes. Innervation and Neural Processes

Receptors responsive to neural products (e.g., norepinephrine and acetylcholine) are found throughout the circulation. Nevertheless, innervation and receptor distribution are organ specific, allowing rapid patterned, coordinated redistribution of blood flow and an orchestrated response to events, such as hypoxia, changes in posture, and hemorrhage. Although these receptors respond to circulating agonists (including adrenal epinephrine), as well as to those liberated locally, they are generally associated with innervation by autonomic nerves. In general, presynaptic a-adrenergic stimulation causes norepinephrine reuptake, whereas postsynaptic a-adrenergic stimulation causes norepinephrine release and vasoconstriction. b-Adrenergic stimulation generally causes vasodilation. Cholinergic stimulation (whether sympathetic or parasympathetic) generally causes vasodilation (see Chapters 31 and 123).

Circulating Endocrine and Neuroendocrine Mediators

Humoral regulators of vascular tone include angiotensin, arginine vasopressin, bradykinin, histamine, and serotonin. Of less certain

significance are aldosterone, thyroxine, antinatriuretic peptide, and various reproductive hormones. Most of these have both direct effects and secondary effects, which tend to be organ specific or regional in nature. Angiotensin plays a special role in the homeostasis of blood pressure and is produced in hemorrhagic or hypovolemic shock. It causes generalized vasoconstriction in both systemic and pulmonary circulations; however, locally, it stimulates the release of vasodilating prostaglandins in the lungs and kidney. Bradykinin is a potent pulmonary and systemic vasodilator released locally by the action of proteolytic enzymes on kallikrein after tissue injury. Histamine is released by mast cells in response to injury and is also a potent vasodilator in most regions of the circulation, but it causes vasoconstriction in the lung. Local Metabolic Products

Local metabolic regulation of vasomotor tone provides an ideal homeostatic mechanism whereby metabolic demand can directly influence perfusion. The precise mechanisms underlying the coupling of blood flow with metabolic activity remain unclear. One theory holds that as the metabolic rate increases, so too does the formation of some vasodilating substance. Thus, the regional vasculature relaxes, allowing more O2 to be delivered in support of this work. As flow rises, the metabolites are washed out, restoring their concentration to normal. Adenosine, for instance, which accumulates locally when tissue metabolism is high and tissue oxygenation is marginal, causes pronounced vasodilation in the coronary, striated muscle, splanchnic, and cerebral circulations. Another example is potassium, which is released from muscle in response to increased work, ischemia, and hypoxia. Hypokalemia causes vasoconstriction; hyperkalemia, within the physiologic range, causes vasodilation. An increasing amount of data demonstrate the importance of the local redox state on the regulation of blood flow through the microcirculation. Reactive oxygen species, such as superoxide, hydrogen peroxide, and peroxynitrite, have been shown to influence normal regulatory processes and participate in the pathophysiology of a wide array of cardiovascular disorders. For example, the rapid reaction of NO with the superoxide anion results in the formation of peroxynitrite, a potent oxidant. Although peroxynitrite is known to have cytotoxic properties, under normal conditions peroxynitrite inhibits leukocyte adherence and platelet aggregation without evidence of cellular injury.11 In disease states, however, peroxynitrite can lead to protein nitration and DNA damage. In addition, elevated levels of superoxide may decrease the bioavailability of NO, leading to abnormal vasomotion.12

Blood Gas Composition Tissue levels of O2 and carbon dioxide have been shown to reflect adequacy of perfusion and O2 delivery.13 These blood gases are potent determinants of regional blood flow and have effects that differ from one region of the circulation to another. They also have a more general effect mediated by carotid chemoreceptors. Endothelial-Derived Factors

The vascular endothelial cells are capable of producing a variety of vasoactive substances, which participate in the regulation of normal vascular tone. These substances, such as NO, carbon monoxide (CO), hydrogen sulfide, and endothelin-1 (ET-1), are capable of producing vascular relaxation and/or constriction, modulating the propensity of the blood to clot, and inducing and/or inhibiting smooth muscle migration and replication14 (Fig. 24.8). Understanding the role of the vascular endothelium and the factors



CHAPTER 24

Regional Peripheral Circulation

207

that it produces in regulating blood flow in health and disease has resulted in several treatment strategies that target, mimic, or augment endothelial processes. Therapies that have been used with variable success include inhaled NO for pulmonary hypertension; L-arginine supplementation for coronary artery disease and the pulmonary vasculopathy of sickle cell disease; phosphodiesterase inhibitors, which prevent the breakdown of cyclic guanosine monophosphate (cGMP) for pulmonary hypertensive disorders; endothelin receptor antagonists for pulmonary hypertensive disorders and vasospasm following subarachnoid hemorrhage; and NO inhibitors for refractory hypotension secondary to sepsis.15–20 Indeed, many older therapies used to promote vascular relaxation, such as nitrovasodilators, affect endothelial function. NO is a labile humoral factor produced by NO synthase from L-arginine in the vascular endothelial cell. NO diffuses into the smooth muscle cell and produces vascular relaxation by increasing concentrations of cGMP via the activation of soluble guanylate cyclase. NO is released in response to a variety of factors, including shear stress (flow) and the binding of certain endothelium-dependent vasodilators (such as acetylcholine, adenosine triphosphate [ATP], and bradykinin) to receptors on the endothelial cell. Basal NO release is an important mediator of both resting pulmonary and systemic vascular tone in the fetus, newborn, and adult, as well as a mediator of the fall in pulmonary vascular resistance normally occurring at the time of birth.21 Dynamic changes in NO release are fundamental to the regulation of all vascular beds. CO is a labile humoral factor produced by the action of hemoxygenase on heme in many tissues, including endothelial cells. Hemoxygenase-1 is constitutive and hemoxygenase-2 is inducible. CO interacts with NO, is an independent stimulator of cGMP, relaxes smooth muscle, inhibits its replication, and has powerful antithrombotic and antiinflammatory effects. It is beginning to enter the field of clinical medicine.10,22 Hydrogen sulfide is produced in most tissues by a variety of mechanisms and may be the ultimate sensor that is stimulated by O2 deficit or excess.8 ET-1 is a 21 amino acid polypeptide also produced by vascular endothelial cells.23 The vasoactive properties of ET-1 are complex; studies have shown varying hemodynamic effects on different vascular beds. However, its most striking property is its sustained hypertensive action. In fact, ET-1 is the most potent vasoconstricting agent discovered, with a potency 10 times that of angiotensin II. The hemodynamic effects of ET-1 are mediated by at least two distinctive receptor populations, ETA and ETB. The ETA receptors are located on vascular smooth muscle cells and mediate vasoconstriction, whereas the ETB receptors may be located on endothelial cells and mediate both vasodilation and vasoconstriction. Individual endothelins occur in low levels in the plasma, generally below their vasoactive thresholds. This suggests that they are primarily effective at the local site of release. Even at these levels, they may potentiate the effects of other vasoconstrictors, such as norepinephrine and serotonin.24 The role of endogenous ET-1 in the regulation of normal vascular tone is presently unclear. Nevertheless, alterations in endothelin-1 have been implicated in the pathophysiology of a number of disease states.25 Endothelial-derived hyperpolarizing factor (EDHF), a diffusible substance that causes vascular relaxation by hyperpolarizing the smooth muscle cell, is another important endothelial factor. EDHF has not yet been identified, but current evidence suggests that the action of EDHF is dependent on potassium (K1) channels (see Fig. 24.8). Activation of K1 channels in the vascular smooth muscle results in cell membrane hyperpolarization,

SECTION IV



208

Pediatric Critical Care: Cardiovascular

Dilation

Constriction

PLA2 AA COX1 COX2

PGI2 EDHF

EC

L-Arg

L-Cit

TXA2

BigET ECE

NOS ETB

ET-1

NO

K+

sGC

AC K+

ATP cAMP PKA

SMC

GTP cGMP Ca2+

PKG

PreProET

ETA ETB PLC

IP3 DAG

Ca2+

Ca2+

•  Fig. 24.8

​Schematic of some endogenous vasoactive agents produced by the vascular endothelium AA, Arachidonic acid; AC, adenylate cyclase; ATP, adenosine triphosphate; cAMP, cyclic adenosine monophosphate; cGMP, cyclic guanosine monophosphate; COX, cyclooxygenase; DAG, diacylglycerol; EC, endothelial cell; ECE, endothelin-converting enzyme; EDHF, endothelial-derived hyperpolarizing factor; ET-1, endothelin-1; GTP, guanosine triphosphate; IP3, inositol 1,4,5, triphosphate; L-arg, L-arginine; L-cit, L-citrulline; NO, nitric oxide; NOS, nitric oxide synthase; PGI2, prostacyclin; PKA, protein kinase A; PKC, protein kinase C; PLA, phospholipase A2; sGC, soluble guanylate cyclase; SMC, smooth muscle cell; TXA2, thromboxane A2.  

closure of voltage-dependent calcium (Ca21) channels, and, ultimately, vasodilation. K1 channels are also present in endothelial cells. Activation within the endothelium results in changes in calcium flux and may be important in the release of NO, prostacyclin, and EDHF. K1 channel subtypes include ATP-sensitive K1 channels, Ca21-dependent K1 channels, voltage-dependent K1 channels, and inward-rectifier K1 channels. The breakdown of phospholipids within vascular endothelial cells results in the production of the important by-products of arachidonic acid, including prostacyclin (PGI2) and thromboxane (TXA2). PGI2 activates adenylate cyclase, resulting in increased cyclic adenosine monophosphate (cAMP) production and subsequent vasodilation, whereas TXA2 results in vasoconstriction via phospholipase C signaling (see Fig. 24.8). Other prostaglandins and leukotrienes also have potent vasoactive properties. Myogenic Processes

In 1902, Bayliss described an intrinsic increase in vascular tone in response to elevated intravascular pressure.26 This myogenic response results in alterations in vascular tone following changes in transmural pressure or stretch. This response is especially important at the arteriolar level and is thought to participate in regional autoregulation. Increases in intravascular pressure and/or stretch result in an increase in arteriolar smooth muscle tone, while decreasing pressures have the reverse effect. The precise mechanisms mediating this response are unclear, but a role for dynamic

.

changes in intracellular Ca21 and myosin light-chain phosphorylation has been documented.27 More recent work has focused on the role of tyrosine phosphorylation pathways, ENaC, transient receptor potential (TRP) channels, K1 channels, and alterations in Ca21 sensitivity in this response.28–32 Moreover, the myogenic response varies between the regional circulations and vessels within a given circulation.33

Regional Circulations Pulmonary Circulation Maldevelopment and/or maladaptation of the pulmonary vascular bed are important components of several neonatal and infant disease states (i.e., chronic lung disease, persistent pulmonary hypertension of the newborn, and congenital heart disease). In addition, strategies aimed at altering postnatal pulmonary vascular resistance are commonly used in the management of these patients. Therefore, an understanding of the regulation of postnatal pulmonary vascular tone is important. The morphologic development of the pulmonary circulation affects the physiologic changes that occur in the perinatal period. In the fetus and neonate, small pulmonary arteries have thicker muscular coats than similarly located arteries in the adult. This muscularity is responsible, in large part, for the pulmonary vascular reactivity and high resistance found in the fetus. Within the

Pulmonary arterial mean pressure (mm Hg)

60 50 40 30 20 10

Pulmonary blood flow (mL/kg/min)

first several weeks after birth, the medial smooth muscle involutes, and the thickness of the media of the small pulmonary arteries decreases rapidly and progressively.34 Following this perinatal transition, the medial layers of the proximal pulmonary vascular bed are completely encircled by smooth muscle. Moving distally, muscularization becomes incomplete (arranged in a spiral or helix) and disappears completely from the most peripheral arterioles.35 In these arterioles, an incomplete pericyte layer is found within the endothelial basement membrane. Smooth muscle precursor cells reside in the nonmuscular portions of the partially muscular pulmonary arteries. Under certain conditions, such as hypoxia, these cells may rapidly differentiate into mature smooth muscle cells. Subsequently, from infancy to adolescence, the arteries undergo progressive peripheral muscularization. In the adult, complete circumferential muscularization extends peripherally such that the majority of small pulmonary arteries are completely muscularized.

160 120 80 40

Pulmonary vascular resistance (mm Hg/mL/min/kg)

1.8 1.6 1.4 1.2 1.0 0.8 0.6 0.4 0.2

Normal Fetal Circulation

Changes in the Pulmonary Circulation at Birth After birth, with initiation of ventilation by the lungs and the subsequent increase in pulmonary and systemic arterial blood O2 tensions, pulmonary vascular resistance decreases and pulmonary blood flow increases by 8- to 10-fold to match systemic blood flow. This large increase in pulmonary blood flow increases pulmonary venous return to the left atrium, increasing left atrial pressure. Then, the valve of the foramen ovale closes, preventing any significant atrial right-to-left shunting of blood. In addition, the ductus arteriosus constricts and closes functionally within several hours after birth, effectively separating the pulmonary and systemic circulations. Mean pulmonary arterial pressure decreases and, by 24 hours of age, is approximately 50% of mean systemic arterial pressure. Adult values are reached 2 to 6 weeks after birth39,40 (Fig. 24.9). The decrease in pulmonary vascular resistance with ventilation and oxygenation at birth is regulated by a complex and incompletely understood interplay between metabolic and mechanical factors, which, in turn, are triggered by the ventilatory and circulatory changes that occur at birth. Physical expansion of the fetal lamb lung without changing O2 tension increases fetal pulmonary

Regional Peripheral Circulation

209

Birth -7

-5

-3

-1

1

3

5

7

Weeks

•  Fig.

24.9 ​Changes in mean pulmonary arterial pressure, pulmonary blood flow, and pulmonary vascular resistance at birth (Data from Morin FC III, Egan E. Pulmonary hemodynamics in fetal lambs during development at normal and increased oxygen tension. J Appl Physiol. 1993;73:213–218; and Soifer SJ, Morin FC III, Kaslow DC, et al. The developmental effects of prostaglandin D2 on the pulmonary and systemic circulations in the newborn lamb. J Dev Physiol. 1983;5:237–250.) .



In the fetus, normal gas exchange occurs in the placenta and pulmonary blood flow is low, supplying only nutritional requirements for lung growth and performing some metabolic functions. Pulmonary blood flow in near-term lambs is between 8% and 10% of total output of the heart.36 Pulmonary blood flow is low despite the dominance of the right ventricle, which, in the fetus, ejects about two-thirds of total cardiac output. Most of the right ventricular output is diverted away from the lungs through the widely patent ductus arteriosus to the descending thoracic aorta, from which a large proportion reaches the placenta through the umbilical circulation for oxygenation. Fetal pulmonary arterial pressure increases with advancing gestation. At term, mean pulmonary arterial pressure is about 50 mm Hg, generally exceeding mean descending aortic pressure by 1 to 2 mm Hg.37 Pulmonary vascular resistance early in gestation is extremely high relative to that in the infant and adult, probably due to the low number of small arteries. Pulmonary vascular resistance falls progressively during the last half of gestation, new arteries develop, and crosssectional area increases. However, baseline pulmonary vascular resistance is still much higher than after birth.37,38



CHAPTER 24

blood flow and decreases pulmonary vascular resistance, but not to newborn values.41 Mechanical factors include the replacement of fluid in the alveoli with gas, which allows unkinking of the small pulmonary arteries, and radial traction on extra-alveolar vessels that maintain their patency.42 Physical expansion of the lung also releases vasoactive substances, such as PGI2. Ventilation of the fetus without oxygenation produces partial pulmonary vasodilation, while ventilation with air or oxygen produces complete pulmonary vasodilation. The exact mechanisms of oxygen-induced pulmonary vasodilation during the transitional circulation remain unclear. The increase in alveolar or arterial O2 tension may decrease pulmonary vascular resistance by either directly dilating the small pulmonary arteries or indirectly stimulating the production of vasodilator substances such as PGI2 or NO. Therefore, there are at least two components to the decrease in pulmonary vascular resistance with the initiation of ventilation and oxygenation. Both components are necessary for the successful transition to extrauterine life. Control of the perinatal pulmonary circulation reflects a balance between factors producing pulmonary vasoconstriction (low O2, leukotrienes, and other vasoconstricting substances) and those producing pulmonary vasodilation (high O2, PGI2, NO, and other vasodilating substances). The dramatic increase in pulmonary blood flow with the initiation of ventilation and oxygenation at birth reflects a shift from active pulmonary vasoconstriction in the fetus to active pulmonary vasodilation in the newborn. Failure of the pulmonary circulation to undergo this normal fall in pulmonary vascular resistance at birth (persistent pulmonary hypertension of the newborn) is associated with a variety of conditions, including aspiration syndromes, sepsis, in utero stress

SECTION IV



210

Pediatric Critical Care: Cardiovascular

events, and certain congenital defects (e.g., congenital diaphragmatic hernia and congenital heart disease).

Regulation of Postnatal Pulmonary Vascular Resistance After the immediate postnatal state, the pulmonary circulation is maintained in a dilated, low-resistance state. Because the inflow pressure of the pulmonary circulation is quite low, there is a vertical gradation to the distribution of blood flow in the lung. Hydrostatic pressure must be adjusted for vertical height above the left atrium, both at the inflow and outflow of every alveolar capillary unit. For example, given a pulmonary artery mean pressure of 20 cm H2O (zeroed at the level of the left atrium), an alveolar-capillary unit 12 cm above the left atrium will face an inflow pressure of only 8 cm H2O. A left atrial pressure of 5 cm H2O would generate no opposing outflow pressure to alveolar capillary units more than 5 cm above the left atrium. Therefore, critical closing pressure of postcapillary vessels would set outflow pressure for a unit 10 cm above the left atrium. Were intrinsic vascular resistance identical throughout the lung, flow at any vertical height would be determined by hydrostatic driving pressure (inflowoutflow) and would be greatest at the base and least at the apex of the lung. West et al. reported that this phenomenon partitions the lung into three vertical regions (Fig. 24.10).44 Zone I vessels are higher above the left atrium than pulmonary artery pressure (expressed in cm H2O) and are not perfused by the pulmonary artery. Zone II vessels lie above the height defined by the hydrostatic left atrial pressure but below the height of pulmonary artery pressure. These units are perfused in proportion to the driving pressure across them, which is approximately pulmonary artery pressure less vertical height (or critical closing pressure, whichever

Zone I

Ppa

Zone II

PLA

LA

• Fig. 24.10

Zone III

Reference height of LA

​The lung is divided vertically into three regions. Zone I alveolar capillary units are unperfused because they see no functional inflow pressure. Zone II units are perfused in proportion to their height above the left atrium (LA). Zone III vasculature is more uniformly perfused because gravity has comparable effects on inflow and outflow pressures. (Modified from Fuhrman BP. Regional circulation. In: Fuhrman BP, Shoemaker WC, eds. Critical Care: State of the Art, vol 10. Fullerton, CA: Society of Critical Care Medicine; 1989.)  





is higher). Zone III vessels lie at a vertical height less than outflow pressure expressed in cm H2O. Driving pressure across these units is independent of height because inflow and outflow pressures are comparably influenced by gravity. Of note, in a supine neonate, there is likely no zone I. Small pulmonary arteries course along with the branching airways, and small pulmonary vessels are intimately related with alveoli. Therefore, airway pressure can directly modulate pulmonary blood flow.45 Alveolar pressure can be loosely translated into surrounding pressure for alveolar vessels. Positive airway pressure applied to the lung can impinge on alveolar vessels whenever alveolar pressure exceeds the other determinants of outflow pressure. During positive-pressure ventilation, outflow pressure of the pulmonary circulation may be determined predominantly by the mechanics of ventilation. The lung is partitioned into zones, but the distribution of flow becomes a complex function of alveolar pressure as well as left atrial and critical closing pressures. To further complicate this view of the pulmonary circulation, lung volume and alveolar pressure both change during positivepressure ventilation. During inspiration, extra-alveolar vessels are dilated by radial traction, reducing their resistance to flow, whereas alveolar vessels narrow and elongate.45 During lung inflation, alveolar surface tension rises, diminishing the transmission of alveolar pressure to alveolar vessels. There is also evidence that lung stretch may directly augment pulmonary vascular tone in a manner that is dependent on calcium flux and subject to calcium channel blockade using verapamil. In fact, it is clear that mechanical ventilation can have profound direct effects on the intact pulmonary circulation that depends on the waveform of airway pressure applied, not on mean airway pressure alone. In heterogeneous lung disease, the application of positive airway pressure can modulate and redistribute blood flow away from ventilated and toward unventilated regions of the lung by directly increasing the pulmonary vascular resistance of lung segments exposed to elevated airway pressure, that is, segments not protected by consolidation or airway obstruction.45 Two of the most important factors affecting pulmonary vascular resistance in the postnatal period are oxygen concentration and pH. Decreasing oxygen tension or pH elicits pulmonary vasoconstriction of the resting pulmonary circulation.46 Alveolar hypoxia constricts pulmonary arterioles, diverting blood flow away from hypoxic lung segments and toward well-oxygenated segments.47 This enhances ventilation-perfusion matching. This is a unique pulmonary vascular response to hypoxia, which is probably greater in newborns compared with adults. The mechanism of alveolar hypoxic pulmonary vasoconstriction remains to be defined and is the subject of several extensive reviews. Acidosis potentiates hypoxic pulmonary vasoconstriction and alkalosis reduces it.46 The exact mechanism of pH-mediated pulmonary vasoactive responses also remains incompletely understood but appears to be independent of partial pressure of arterial carbon dioxide (Paco2). Alveolar hyperoxia and alkalosis are often used to relax pulmonary vascular tone because they generally relieve pulmonary vasoconstriction while having little apparent effect on the systemic circulation as a whole. However, detrimental effects of hypocarbia or respiratory alkalosis on cerebral and myocardial blood flow may occur. The lung is innervated, but neural effects on pulmonary vascular resistance appear to be of little consequence on basal tone. However, pulmonary neurohumoral receptors are sensitive to aadrenergic, b-adrenergic, and dopaminergic agonists. Therefore, vasoactive agents that stimulate these receptors will affect the

211

AUTOREGULATION OF CEREBRAL BLOOD FLOW 100 90 80 70 60 50 40 30 20 10 0 0

25

50

75

100

125

150

175

200

Mean cerebral perfusion pressure (mm Hg)

•  Fig. 24.11

​Cerebral blood flow (CBF) autoregulates at perfusion pressures between 50 and 160 mm Hg. Below 50 mm Hg, CBF falls. Above 160 mm Hg, CBF rises. (Modified from Fuhrman BP. Regional circulation. In: Fuhrman BP, Shoemaker WC, eds. Critical Care: State of the Art, vol 10. Fullerton, CA: Society of Critical Care Medicine; 1989.) 

The brain makes up 2% of body mass, receiving approximately 14% of the cardiac output while accounting for close to 20% of the body’s O2 consumption in adults, up to 30% of cardiac output, 50% or more of total O2 usage, and up to 98% usage of produced hepatic glucose in neonates. Other organ systems receive a larger percentage of the total cardiac output (e.g., the lung) and use greater amounts of O2 (e.g., skeletal muscle), but the brain is unique in its intolerance for diminished blood flow. In fact, although some favorable outcomes have been reported, severe, if not irreversible, damage occurs often after just minutes of circulatory arrest under normal conditions.49 The cranium has three compartments: tissue, cerebral spinal fluid (CSF), and blood. The Monro-Kellie doctrine states that these compartments occupy a relatively fixed space and that an increase in one compartment can only occur at the expense of another. For example, with brain swelling, CSF and cerebral venous blood must be displaced if intracranial pressure (ICP) is to remain unchanged. As the limits of CSF and blood evacuation are approached, ICP rises. Raised ICP and/or venous obstruction can impede cerebral blood flow (CBF). Cerebral perfusion pressure (CPP), defined as the difference between the mean arterial pressure and the ICP, is thus a more accurate descriptor of cerebral inflow pressure. CBF will decline when the CPP falls below the lower limit of the autoregulatory curve. In the setting of raised ICP, this will occur even in the face of elevated systemic arterial pressures. At rest, cerebral O2 consumption is surprisingly high. Glucose is the primary energy substrate, although ketones can be used during periods of starvation. The brain has no functional capacity to store energy; thus, it is completely dependent on a steady supply of O2, as up to 92% of its ATP production results from the oxidative metabolism of glucose.49 An important feature unique to the cerebral circulation is the presence of a blood-brain barrier (BBB). The vascular endothelium of brain capillaries forms a continuous sheet, with adjacent cells joined by tight junctions. Unlike the endothelium of nonneural capillaries, there are no intercellular clefts through which water-soluble particles can traverse, and there is markedly diminished pinocytosis. However, lipid-soluble substances, carbon dioxide (CO2) and O2, can freely diffuse across the endothelium. Metabolically important components—such as glucose, lactate, and amino acids—depend on specific carrier proteins to facilitate



Cerebral Circulation

Regional Peripheral Circulation

their diffusion into the brain. Furthermore, the BBB has a biochemical component, with high levels of degradative enzymes that protect the vascular smooth muscle and extracellular fluid from the effects of circulating vasoactive substances, such as catecholamines. Thus, as a result of the BBB, the cerebral vasculature responds differently from other vascular beds to humoral stimuli. However, humoral stimuli can significantly alter the vascular tone of large cerebral arteries and can affect blood flow to parts of the brain that lack a complete BBB, such as the choroid plexus, median eminence, and area postrema.50 It has been recognized for nearly 80 years that CBF remains constant over a wide range of mean systemic arterial pressures (Fig. 24.11).51 Constant CBF is maintained in the face of increasing inflow pressures by compensatory vasoconstriction. Conversely, in the setting of low systemic arterial pressures (i.e., low inflow pressures) the cerebral vasculature dilates in order to maintain steady CBF. At systemic arterial pressures outside the autoregulatory range, further dilation or constriction can no longer maintain blood flow. At high pressures, disruption of the BBB ensues, with subsequent edema and even hemorrhage from ruptured cerebral vessels. At low pressures, CBF begins to fall—with continued decreases leading to ischemia and, ultimately, brain death.52 Importantly, normal cerebral autoregulation can be impaired in the setting of disease. Traumatic brain injury, subarachnoid hemorrhage, and stroke, for example, can all abolish or impair the normal autoregulatory response.53,54 The brain’s ability to autoregulate flow is well established, but the mechanisms underlying it are not completely understood. A myogenic response appears to be especially important in the setting of raised CPP. Large- and medium-sized cerebral arteries, including the internal carotid artery (ICA), have been shown to constrict both in vitro and in vivo in response to elevated transmural pressures. Although small arteries and arterioles primarily modulate cerebral resistance during normotension, at higher pressures the large cranial vessels dominate. Thus, at high perfusion pressures, smaller, more delicate vessels are protected by changes in upstream resistance.

Cerebral blood flow (mL/100 g/min)

vascular tone of both the pulmonary and systemic circulations. The degree of pulmonary to systemic alterations induced by these agents is variable and often dictated by the relative tone of each vascular bed. Therefore, the response of these agents is difficult to predict in an individual critically ill patient. A selective pulmonary vasodilator was long sought for treatment of pulmonary hypertension because, with the exception of O2, the response of the pulmonary circulation to humoral vasoactive agents is generally similar to that of the systemic circulation. To date, inhaled NO is the most commonly used agent for selective pulmonary vascular dilation. It is noteworthy that its selectivity is not based on a differential effect in the pulmonary and systemic circulations. Rather, when delivered as an inhaled gas, NO is rapidly bound to hemoglobin and inactivated, thus, limiting its effects on the pulmonary circulation. Inhaled prostacyclin is another agent that offers relative pulmonary vascular selectivity owing to its rapid metabolism.48



CHAPTER 24

SECTION IV



212

Pediatric Critical Care: Cardiovascular

In marked contrast to other vascular beds, neural stimuli have relatively little effect on basal CBF. Cerebral vessels display extensive perivascular innervation, especially by the sympathetic nerves arising from the superior cervical sympathetic ganglia, but the brain is well protected from circulating catecholamines by the BBB. Thus, many of the vasoactive agents used in the critical care setting (a- and b-adrenergic agonists) have minimal effects of resting cerebral vascular tone. Mild to moderate electrical stimulation, as well as surgical resection of both the sympathetic and parasympathetic nervous systems, does not alter cerebral vascular tone under resting conditions. However, vigorous sympathetic stimulation, as would occur with strenuous exercise or hypertension, does result in vasoconstriction of large- and medium-sized cerebral vessels. Thus, while a neurogenic mechanism may not mediate cerebral vascular resistance under normal conditions, it does provide protection during times of stress.55 Indeed, patients with chronic hypertension have been shown to have a rightward shift of the autoregulatory curve. As in other vascular beds, it appears that CBF is coupled to changes in metabolism.56 For example, hypothermia decreases the cerebral metabolic rate of oxygen (CMRO2) and therefore CBF, in both animal and human studies.57–59 Seizure activity and fever both increase the CMRO2 and CBF, which explains the deleterious consequences of both conditions for patients with raised ICP.60 The mechanisms underlying this coupling of blood flow and metabolism are still unclear. A number of substances have been shown to affect cerebrovascular tone. These include CO2, O2, hydrogen ions, lactic acid, histamine, potassium ions, prostaglandin, ET-1, NO, and adenosine. CO2 plays a critical role in the regulation of CBF. In fact, a linear increase in CBF is seen with increasing Paco2, making CO2 one of the most potent known cerebral vasodilators.61 CO2 exerts its effect via a reduction of the perivascular pH. Whereas arterial H1 cannot cross the BBB, CO2 can easily diffuse into the brain. Carbonic anhydrase facilitates the reaction between CO2 and H2O, forming carbonic acid with subsequent dissociation producing H1 ions. Perivascular acidosis dilates the cerebral vasculature, while alkalosis leads to vasoconstriction.62 In this way, the cerebral vasculature is distinct in that respiratory acidosis and alkalosis will alter tone and CBF, while metabolic acidosis and alkalosis will not.61,63 Interestingly, abnormal CO2 reactivity has been associated with several disease processes, including traumatic brain injury, subarachnoid hemorrhage, stroke, carotid stenosis, and congestive heart failure.53,54 Indeed, abnormal CO2 vasoreactivity has been used as a means to prognosticate in some disease states.54 Several studies have demonstrated that the cerebral vasculature adapts in the setting of chronically elevated Paco2 with changes in the pH of the brain extracellular fluid. This has obvious implications for the clinician attempting to treat raised ICP with chronic hyperventilation. Partial pressure of arterial oxygen (Pao2) also participates in the regulation of CBF. Arterial hypoxia dilates cerebral vessels at Pao2 below 40 to 50 mm Hg. The relation between CBF and arterial oxygen content is almost linear, and cerebral O2 delivery can be maintained unless arterial O2 content falls below 4 vol%. Hyperoxia does not appear to be a potent stimulus for vasoconstriction, however. The mechanisms of hypoxic vasodilation are not completely understood, but it is known that adenosine and both Ca21-activated and ATP-activated K1 channels are particularly important. Adenosine, which leads to vasodilation through an increase in cAMP, has been found to increase by more than fivefold with hypoxia.64

A large body of evidence, both in animals and humans, implicates NO in a number of important processes within the cerebral circulation.65–67 Vasodilation in response to acetylcholine, oxytocin, substance P, histamine, ET-1, ADP, ATP, and prostaglandin has been shown to be NO dependent in all cases. Clinically, it is noteworthy that nitroprusside and other NO-donor compounds can dilate cerebral vessels.68 This greatly complicates the management of hypertension in patients with increased ICP. In that setting, nitroprusside, for example, may reduce arterial pressure but raise both cerebral blood flow and blood volume, causing herniation to occur. In addition, impaired NO signaling is important in the pathophysiology of subarachnoid hemorrhage in which endothelial dysfunction has been well documented, leading to the important clinical problem of vasospasm. ET-1 also mediates cerebrovascular tone.69 Both ETA and ETB receptors have been identified in the cerebral vasculature. When given in high concentrations, ET-1 constricts cerebral vessels, probably via ETA receptor activation. In low concentrations, however, ET-1 relaxes cerebral vessels via endothelial cell ETB receptor activation, a response that is NO dependent. Sarafotoxin 6c (a selective ETB agonist) causes cerebral vasodilation. However, ETA and combined receptor antagonists do not alter basal cerebrovascular tone. Recently, ET-1 has been identified as an important mediator of vasospasm following subarachnoid hemorrhage. ET-1 levels are increased following subarachnoid hemorrhage. Associated with this increase, ETA receptor levels, smooth muscle cell ETB receptor levels (which mediate vasoconstriction), and endothelin-converting enzyme activity are increased. The potential clinical use of ET receptor antagonists following subarachnoid hemorrhage is under investigation, with promising preliminary results.16

Coronary Circulation Right and left coronary arteries arise from sinuses of Valsalva and course over the surface of the heart. Nutrient branches penetrate the myocardium to supply both superficial (epicardial) and deep (subendocardial) layers of the muscle. Venous blood drains primarily to the coronary sinus, although some returns by way of anterior coronary veins to the right atrium or via sinusoids directly to the ventricles. Myocardial workload (which sets myocardial oxygen demand) is determined by not only the needs of the heart but also the demands of the body. Furthermore, the heart is required to generate its own perfusion pressure. Accordingly, regulation of myocardial perfusion is tailored to match regional myocardial oxygen supply to demand over the widest possible range of cardiac workload and under conditions fashioned not so much for maximal cardiac efficiency but rather for benefit of the body. Myocardial perfusion over a cardiac cycle is approximately the same per gram of tissue in the outer (subepicardial), mid-, and inner (subendocardial) layers of the left ventricle. However, the dynamics during the cardiac cycle are complicated. At the end of diastole, when the ventricle is relaxed and tissue pressures are generally less than 10 mm Hg in any layer of the left ventricle, pressures in the intramural arteries are similar to each other and to aortic pressure. At the beginning of systole, tissue pressure rises to equal intracavitary pressure in the subendocardium but then falls off linearly across the wall to about 10 mm Hg in the subepicardium. These pressures are added to those inside the vessels for an instant because the vessels walls are not rigid. As a result, intravascular pressures in subendocardial arteries exceed aortic pressures, but aortic pressures are higher than pressures in subepi-

Aortic pressure (mm Hg)

120 100 80 100

Phasic coronary blood flow (mL/min)

80 60 40 20

Left coronary artery

0

15 10 5

Right coronary artery

0

0.6

1

Time (s)

•  Fig. 24.12 





​Myocardial blood flow is modulated by ventricular wall tension. Most of the perfusion of the left ventricular myocardium occurs in diastole. (Modified from Berne RM, Levy MN. Cardiovascular Physiology. 7th ed. St. Louis: Mosby; 1997.)

cardial arteries. These pressure gradients and the greater shortening of subendocardial than subepicardial muscle fibers during systole compress the subendocardial vessels and squeeze blood out of them both forward into the coronary sinus and backward toward the epicardium. In fact, narrowing of the subendocardial vessels facilitates thickening and shortening of the myocytes.70 This backflow enters the subepicardial arteries to supply their systolic flow. In systole, there is indeed some forward flow into the orifices of the coronary arteries, but this does not perfuse the myocardium; it merely fills the extramyocardial arteries.71 In fact, there is often reverse flow in the epicardial coronary arteries. In early diastole, blood flows first into the subepicardial vessels that have not been compressed but takes longer to refill the narrowed subendocardial vessels. Given enough time and perfusing pressure, all the myocardium will be perfused. However, if diastole is too short or perfusion pressure too low, subendocardial ischemia occurs. Right ventricular myocardium, on the other hand, is normally perfused in both systole and diastole (Fig. 24.12) because of lower tissue pressures. We would expect perfusion of the hypertrophied right ventricle of severe pulmonic stenosis or tetralogy of Fallot to resemble that of the left ventricle.72

Myocardial Oxygen Demand-Supply Relationship The left ventricle extracts most of the O2 from the blood passing through the myocardium; coronary sinus O2 saturation is normally about 30%. Therefore, increases in myocardial O2 demand must be met by increases in myocardial blood flow. At rest, left ventricular myocardial blood flow is about 80 to 100 mL/100 g per minute, and with maximal exertion, left ventricular O2



CHAPTER 24

Regional Peripheral Circulation

213

consumption increases about fourfold, as does left ventricular blood flow in normal people and animals.73 If coronary perfusion pressure does not change during exertion, the increased flow has to be achieved by a decrease in coronary vascular resistance. The response is termed metabolic regulation. Coronary vascular resistance has three components: a basal low resistance in the arrested heart with maximally dilated vessels, an added resistance when vessels have tone, and a phasic resistance added whenever the ventricle contracts.74 In the beating heart with vessels maximally dilated by a pharmacologic dilator, the second of these resistances is absent. Perfusion of the left ventricular myocardium then produces a steep pressure-flow relation that is linear at higher flows but usually curvilinear at low pressures and flows (Fig. 24.13A). Because the vessels are maximally dilated, flow is uncoupled from metabolism and depends only on driving pressure and resistance. If heart rate is increased, maximal flow at any perfusion pressure decreases because the heart is in a relaxed state for a smaller proportion of each minute. If tone is allowed to return to the coronary vessels, then the pressure-flow relationship can be assessed at different perfusion pressures after cannulating the left coronary artery. It is necessary to do this because, when cardiac metabolism and blood flow are coupled, increasing aortic blood pressure will increase coronary flow not only by increasing perfusion pressure but also by increasing myocardial O2 demand. Under normal conditions, coronary blood flow is autoregulated such that if perfusion pressure is raised or lowered from its normal value, there is a range over which there is almost no change in flow; a rise in pressure has caused vasoconstriction, and a fall in pressure has caused vasodilation. At perfusion pressures above some upper limit, flow increases, probably because the pressure overcomes the constriction. More importantly, at pressures below about 40 mm Hg (but varying, as discussed later), flow decreases predominantly in the deep subendocardial muscle (see Fig. 24.13A), indicating that some vessels have reached maximal vasodilation and can no longer decrease resistance to compensate for the decreased perfusion pressure. In these vessels, flow and pressure are directly related. If this pressure dependency occurs, then further decrease in perfusion pressure decreases local blood flow below its required amount, or if myocardial O2 demands increase at the same low perfusion pressure (as will occur if the ventricle becomes dilated), the requisite increase in flow will not occur. These two conditions cause subendocardial ischemia. At any given pressure, the difference between autoregulated and maximal flows is termed coronary flow reserve.75–77 Coronary flow reserve can be measured in units of mL/min but can also be assessed by a dimensionless flow reserve ratio derived by dividing maximal flow by resting flow. Flow reserve depends on perfusion pressure because of the steepness of the pressure-flow relation in maximally dilated vessels. Coronary flow reserve indicates how much extra flow the myocardium can get at a given pressure to meet increased demands for O2. If reserve is much reduced, then flow cannot increase sufficiently to meet demands and myocardial ischemia will occur. What the figure does not show is that coronary flow reserve is normally lower in the subendocardium than in the subepicardium and that decreases in coronary flow reserve are always more profound in the subendocardium than in the subepicardium. If autoregulated flow is normal but maximal flow is decreased, as indicated by the decreased slope of the pressure-flow relation during maximal dilation (Fig. 24.13B), then coronary flow reserve will be reduced. Such a change can occur with marked

SECTION IV



214

Pediatric Critical Care: Cardiovascular

Normal maximal flow

600

Normal maximal flow

Normal maximal flow

Flow (mL/min)

500 400 300

R1

R2

R1 200

R2

Normal autoregulated flow

0 0

40

Increased autoregulated flow R1

R2

100

A

Reduced maximal flow

80

120

160

Perfusion pressure (mm Hg)

0

B

40

80

Normal autoregulated flow

Normal autoregulated flow 120

0

160

Perfusion pressure (mm Hg)

C

40

80

120

160

Perfusion pressure (mm Hg)

• Fig. 24.13

​(A) Normal pressure-flow relations in the left coronary artery during normal autoregulated flow and maximal vasodilation. Values are appropriate for a left ventricle weighing approximately 100 g. R1, R2, coronary flow reserve measurements at two different coronary perfusing pressures. (B) Effect on coronary flow reserve of a reduced maximal flow. At the same coronary perfusing pressure, flow reserve is reduced from the normal R1 to R2. (C) Effect on coronary flow reserve of an increased autoregulated flow. Reserve is reduced from R1 to R2.  

tachycardia; a decrease in the number of coronary vessels due to small-vessel disease, as in some collagen vascular diseases, especially systemic lupus erythematosus; increased resistance to flow in one or more large coronary vessels because of embolism, thrombosis, atheroma, or spasm; impaired myocardial relaxation due to ischemia; myocardial edema; a marked increase in left ventricular diastolic pressure; marked increase in left ventricular systolic pressure if coronary perfusion pressure is not also increased, as in aortic stenosis or incompetence; and an increase in blood viscosity, most commonly seen with hematocrits over 65%. Coronary flow reserve can also be reduced if maximal flows are normal, but autoregulated flows increase (Fig. 24.13C). Increased myocardial flows above normal values can occur with exercise, tachycardia, anemia, CO poisoning, leftward shift of the hemoglobin O2 dissociation curve (as in infants with a high proportion of fetal hemoglobin), hypoxemia, thyrotoxicosis, acute ventricular dilation (because of increased wall stress), inotropic stimulation by catecholamines, and acquired ventricular hypertrophy. When hypertrophy occurs a few months after birth, ventricular muscle mass increases without a concomitant increase in conducting coronary blood vessels. Ventricular hypertrophy returns wall stress to normal, and myocardial flow per minute per gram of muscle is approximately normal. Therefore, total left ventricular flow is increased in proportion to ventricular mass, but because maximal flow per ventricle is usually unchanged, the coronary flow reserve is diminished. Often, autoregulated flow is increased and maximal flows are reduced at the same time (e.g., with severe tachycardia or cyanotic heart disease with hypoxemia, ventricular hypertrophy, and polycythemia). Under these circumstances, coronary flow reserve can be drastically reduced. A third mechanism that reduces coronary flow reserve is a shift to the right of the pressure-flow line. If with maximally dilated vessels, diastolic coronary flow is measured at different mean diastolic perfusion pressures, a pressure-flow line is obtained that is linear at higher pressures but curved in the low pressure-flow region.78,79 Zero flow occurs at a pressure of about 8 to 12 mm

Hg; this is the critical closing pressure that is above right atrial pressure.80,81 The whole pressure-flow line can be shifted to the right by several factors, most important of which are pericardial tamponade, a rise in right or left ventricular diastolic pressures, and a-adrenergic stimulation. Such a rightward shift decreases flow reserve. It is important to note that because the line of maximal pressure-flow relations slopes up and to the right, any decrease in that slope (Fig. 24.13B), any increase in autoregulated flow (Fig. 24.13C), or any rightward shift of the slope raises the pressure at which autoregulation fails to compensate for decreased perfusing pressure. It is also important to reemphasize that any decrease in coronary flow reserve affects the subendocardium predominantly. Thus, autoregulation will fail first and ischemia will occur in the subendocardium before these changes occur in the subepicardium.82 The predominant reduction in subendocardial flow and reserve is particularly marked when left ventricular diastolic pressure is high. The interactions between myocardial blood flow and ventricular function are of particular importance when there is ventricular hypertrophy. Myocardial wall stress is regulated within a fairly narrow range, with or without myocardial hypertrophy. Consequently, myocardial blood flow per unit mass is fairly constant at about 1 mL/min per gram of left ventricle at rest.83–88 Strauer has shown a close relationship between peak wall stress in systole and the ratio of left ventricular mass to volume.85–87,89,90 If there is no hypertrophy, coronary flow reserve is normal, but it is reduced if the left ventricular mass is increased. Should the heart dilate acutely, then the mass-tovolume ratio decreases, wall stress and myocardial O2 consumption increase, and coronary flow reserve falls. If ventricular dilation is marked, there can be subendocardial ischemia. Decreasing ventricular dilation by afterload and preload reduction reverses these unfavorable events and is another reason for the resulting improvement in ventricular function. Right ventricular myocardial blood flow follows the general principles regarding coronary blood flow, but there are differences

related to the low right ventricular systolic pressure and to the fact that alterations in aortic pressure change coronary perfusing pressure without altering right ventricular pressure. If the normal right ventricle is acutely distended by pulmonary embolism, for example, there will eventually be right ventricular failure; the increased wall stress increases its O2 consumption, but the raised systolic pressure reduces the coronary flow. Therefore, when supply cannot match demand, there will be right ventricular myocardial ischemia.91 Raising aortic perfusing pressure mechanically or with a-adrenergic agonists increases right ventricular myocardial blood flow, relieves ischemia, and restores right ventricular function to normal. Improved coronary flow is not the only mechanism of this improvement; the increased left ventricular afterload moves the ventricular septum toward the right ventricle and improves left ventricular performance.92 If right ventricular pressure is chronically elevated so that there is right ventricular hypertrophy—as in pulmonic stenosis, many forms of cyanotic congenital heart disease, and some chronic lung diseases—then right ventricular myocardial blood flow behaves in the same way as left ventricular blood flow, with one exception.93–95 If aortic pressure is lowered, left ventricular pressure also decreases, as do left ventricular work and O2 consumption. In the right ventricle, however, the workload may not be reduced (if there is no ventricular septal defect). Thus, an imbalance between myocardial O2 supply and demand may occur. The worst imbalance occurs when aortic systolic pressure is maintained but coronary perfusing pressure decreases. This can occur in a child with tetralogy of Fallot who has too large an aortopulmonary anastomosis. The high aortic and left ventricular systolic pressures mandate an equally high right ventricular systolic pressure, but the low diastolic aortic pressure reduces coronary perfusion pressure in diastole and can cause both left and right ventricular ischemia and failure.96

Gastrointestinal Circulation The maintenance of adequate splanchnic blood flow in critically ill patients is important. In a globally compromised circulation, the gastrointestinal system is particularly prone to injury, impairing its two chief functions: the digestion and absorption of nutrients and the maintenance of a barrier to the translocation of enteric antigens.97–99 Moreover, splanchnic ischemia has been associated with multiple-organ failure and increased morbidity and mortality in these patients.100,101 The gastrointestinal circulation has multiple levels of regulation. Broadly, these can be divided into intrinsic and extrinsic mechanisms. Intrinsic mechanisms include local metabolic processes, locally produced vasoactive substances, and myogenic reflexes. Extrinsic factors include circulating vasoactive substances, neural innervation, and general hemodynamic forces.102 Blood flow to the intestinal circulation, like other vascular beds, is autoregulated such that O2 delivery remains fairly constant, with inflow pressures varying from 30 to 120 mm Hg.102 O2, CO2, H1 ions, and adenosine are important local metabolic mediators of this process. Other important vasoactive mediators of intestinal blood flow include serotonin, histamine, bradykinin, and prostaglandin, although their role in autoregulation is unclear.103,104 Finally, various gastrointestinal hormones and peptides released from the intestinal mucosa and intestinal glands—including gastrin, vasoactive intestinal polypeptide, cholecystokinin, secretin, glucagon, enkephalins, somatostatin, and kallidin—are known to have vasoactive properties. A phenomenon unique to the gastrointestinal circulation is the increase in flow following the consumption of nutrients.



CHAPTER 24

Regional Peripheral Circulation

215

This postprandial intestinal hyperemia appears to involve multiple factors. However, the composition of the chyme is particularly important. In fact, luminal distention, mechanical stimulation, and extrinsic neural stimulation are not necessary for the response to occur. Lipids in combination with bile salts are the most potent triggers for postprandial hyperemia. Glucose is the most potent single stimulus for this response. Blood flow to skin and skeletal muscle decreases and cardiac output increases during postprandial hyperemia. Furthermore, nutrients that induce the largest increase in blood flow elicit the largest O2 debt within the intestinal villi. Interestingly, this postprandial hyperemia may be protective in some instances of low blood flow to the intestinal mucosa. Glucose has been shown to ameliorate mucosal ischemia in models of septic and hemorrhagic shock; early enteral feeding has been advocated in human studies as well.106 Conversely, enteral feeding has also been associated with bowel ischemia and injury in some patients, such as premature infants, complicating decisions around enteral feeding during or following low-flow states.107 Increasing data demonstrate a large role for NO in the regulation of gastrointestinal blood flow. NO, at least in part, mediates basal mesenteric and hepatic blood flow. A number of studies implicate endothelial dysfunction, and aberrations in NO signaling in particular, in portal hypertension and cirrhosis.108–110 NO also participates in the maintenance of mucosal barrier function; it is further protective by virtue of its inhibitory effect on platelet and leukocyte adhesion.111 Furthermore, postprandial hyperemia has been shown to involve adenosine-mediated NO release.112 Finally, neuronal NO synthase and inducible NO synthase are important in both normal and abnormal gastrointestinal motility and gastrointestinal inflammatory disorders, respectively.113–115 ET-1 is another important mediator of intestinal blood flow. The intestinal vasculature displays increased vasoconstriction in response to ET-1 compared with other vascular beds. This has particular importance for gastrointestinal blood flow in critically ill patients, as ET-1 levels have been found to be elevated following surgery and in association with a number of disease states, including hypoxia, pancreatitis, and sepsis. ET receptor antagonism ameliorates ischemic injury to the bowel in several models of low-flow states.116,117 Finally, it should be remembered that drugs used to augment systolic blood pressure and/or to enhance cardiac output could have various effects on the gastrointestinal circulation. Fenoldopam, a dopamine-1 receptor agonist, has been shown to improve intestinal perfusion during hemorrhage.118 However, findings on more common agents—such as norepinephrine, dopamine, and vasopressin—have had mixed results depending on the doses used and the models or clinical situations studied.119–121 Investigations continue to target the determination of an optimum strategy to improve overall cardiac output and O2 delivery without compromising flow to specific organs, such as the bowel.

Renal Circulation Blood flow to the kidneys greatly exceeds the metabolic needs of the organs themselves. In a 70-kg adult, combined renal blood flow is approximately 1200 mL/min, accounting for just over 20% of total cardiac output supplying organs that represent under 0.5% of total body weight.122 This high renal blood flow is necessary in order to support glomerular filtration, which maintains solute and fluid homeostasis. Blood is supplied to the kidneys by the renal arteries, which branch to form the interlobar, arcuate, and interlobular arteries.

SECTION IV



216

Pediatric Critical Care: Cardiovascular

Interlobular arteries progress to form the afferent arterioles, which lead to the glomerular capillaries within the glomerulus, the site of fluid and solute filtration. The distal glomerular capillaries reform into the efferent arterioles, which then lead to a second capillary system, the peritubular capillaries. An elevated hydrostatic pressure within the glomerular capillaries supports filtration, whereas a much lower pressure within the peritubular capillary system supports absorption.122 Alterations in the resistance of the afferent and efferent arterioles regulate these pressures and allow for dynamic changes in renal function in response to overall fluid and solute needs. Renal blood flow is determined by the difference between renal artery pressure (which is generally equivalent to systemic arterial pressure) and renal vein pressure over the renal vascular resistance. In general, three vascular segments limit renal vascular resistance: the interlobular arteries, afferent arterioles, and efferent arterioles. Regulation of renal vascular resistance can be broadly divided into extrinsic mechanisms and intrinsic mechanisms. Extrinsic mechanisms, which include the sympathoadrenal system, atrial natriuretic system, and renin-angiotensin-aldosterone axis, modulate renal blood flow by alterations in intrarenal vascular tone, mesangial tone, intravascular volume, and systemic vascular resistance.122 Intrinsic mechanisms, which primarily alter afferent arteriolar resistance, are responsible for the autoregulation of renal blood flow in response to changes in renal perfusion pressure. The juxtaglomerular apparatus, which includes the afferent and efferent arterioles, macula densa, and glomerular mesangium, is an important site in the regulation of renal perfusion and glomerular filtration. Glomerular filtration is largely a function of glomerular filtration pressure, which, in turn, is dependent on renal perfusion pressure and, importantly, the balance between afferent arteriolar and efferent arteriolar tone. Increased efferent arteriolar tone increases glomerular filtration by increasing glomerular pressure, whereas increased afferent arteriolar tone has the opposite effect. Endogenous epinephrine and norepinephrine derived from sympathetic neural input have various effects on renal perfusion and glomerular filtration. Mild sympathetic output preferentially constricts the efferent arterioles, thereby increasing glomerular pressure and filtration.123 However, intense sympathetic discharge results in afferent arteriolar constriction, which decreases glomerular filtration. Furthermore, sympathetic stimulation of afferent arterioles results in renin release, which leads to increased sodium reabsorption and fluid retention. Sympathetic stimulation can affect renal blood flow more generally by alterations in systemic arterial pressure. Clinically, norepinephrine has been shown to increase renal perfusion and renal function (measured by changes in creatinine clearance) in patients with septic shock. Angiotensin II, produced by cleavage of angiotensin I by the enzyme angiotensin-converting enzyme, also has important effects on renal perfusion. Like catecholamine stimulation, the effects of angiotensin II are dose related. At low levels, angiotensin II results in efferent arteriolar constriction, whereas at high levels both the afferent and efferent arterioles are constricted.123 Angiotensin II alters renal blood flow further by alterations in intravascular volume through aldosterone and arginine vasopressin and by increasing systemic vascular resistance. Arginine vasopressin (AVP) is synthesized in the anterior hypothalamus and released from the posterior pituitary gland. It plays a critical role in maintaining serum osmolality within a

narrow range. Both V1 and V2 receptors have been identified. V2 receptors are located on the renal collecting ducts; stimulation results in increased reabsorption of water. Activation of V1 receptors on systemic vessels results in vasoconstriction. Interestingly, V1 receptor activation in the pulmonary vasculature results in vasodilation, at least in part via NO production. Triggers for AVP release include changes in serum osmolality, hypovolemia, and hypotension. Patients with septic shock have been shown to have decreased levels of AVP, which has led to the clinical use of AVP supplementation. Unlike catecholamines and angiotensin II, high levels of AVP appear to preferentially constrict efferent arterioles, which preserves glomerular filtration. ET-1 has diverse effects on the kidney.124 In general, endothelin results in vasoconstriction, decreased renal perfusion, and decreased glomerular filtration. ET-1 constricts both the afferent and efferent arterioles. ET-1 has also been shown to stimulate cell proliferation within the kidney. Conversely, ET-1 may also promote natriuresis through ETB-receptor activation. Furthermore, alterations in ET-1 signaling have been implicated in a host of renal diseases, including acute and chronic renal failure, essential hypertension, glomerulonephritis, renal fibrosis, and renal transplant rejection.124 Important vasodilators within the renal circulation include prostaglandins and atrial natriuretic peptide (ANP). The vasodilating prostaglandins (D2, E2, and I2) are synthesized from arachidonic acid by the enzyme phospholipase A2. Most of the important vasoconstricting factors—such as catecholamines, angiotensin II, and AVP—stimulate the release of prostaglandins, promoting increased renal perfusion and glomerular filtration. ANP is produced within the atrial myocytes and is released in response to increased atrial stretch. Through cGMP signaling, ANP results in afferent arteriolar dilation and increased renal perfusion and glomerular filtration. ANP also antagonizes the actions of endogenous catecholamines, angiotensin II, and AVP. Like other organ systems, renal blood flow is autoregulated via mechanisms intrinsic to the renal vasculature.122 Early studies demonstrate that renal blood flow and glomerular filtration remain constant at renal artery perfusion pressures of between 80 and 180 mm Hg.125 Importantly, urinary flow rate is not constant within the autoregulatory range but rather changes as a function of renal perfusion pressure. The precise mechanisms underpinning this autoregulation are unclear. However, recent evidence indicates that the mechanisms are likely complex, involving interactions between tubuloglomerular feedback and myogenic processes that protect the kidney from damage in the setting of hypertension and regulate renal function.126–128 A number of disease states that affect critically ill patients result in the loss of renal autoregulation. Acute tubular necrosis, septic shock, hepatic failure, and cardiopulmonary bypass have all been associated with renal dysfunction and a loss of renal autoregulation.

Conflicting Needs of Regional Circulations The cardiovascular system is composed of the heart and the regional circulations. In order to maintain homeostasis in health and disease, the circulations operate in concert while preserving O2 delivery to the individual organs that they supply. In certain disease states, however, the regional circulations

or the needs of different organ systems may conflict. Critical care providers must be aware of these conflicts in order to mitigate the consequences. For example, a neonate with a patent ductus arteriosus may suffer from inadequate systemic perfusion (e.g., kidneys and gut) when pulmonary vascular resistance falls, increasing systemic to pulmonary shunting. A given ventilation strategy would be expected to have different consequences for the brain and lungs in a patient suffering from head trauma with raised ICP and severe lung injury (i.e., with standard goals being mild hyperventilation or normal ventilation for brain injury and permissive hypercapnia in lung injury). In a patient with single-ventricle physiology and a superior cavopulmonary anastomosis (Glenn), the pulmonary and cerebral vascular resistance both impact pulmonary blood flow, but CO2 affects each in an opposite manner (i.e., hypercarbia increases pulmonary and decreases cerebral vascular resistance). Vasoactive medications in the setting of pulmonary hypertension are variably effective, in large part depending on the degree to which they impact the pulmonary to systemic vascular resistance ratio. These examples illustrate that unsuccessful arbitration among regional circulations may contribute to the genesis of the syndrome of multiple-organ system failure (see Chapter 111). It is hoped that an increasing understanding of the mechanisms that regulate regional microcirculatory blood flow will lead to new and improved treatments that optimize blood flow and allow the intensivist to successfully arbitrate the regional blood flow “conflict of interests” and improve outcomes for critically ill children.



CHAPTER 24

Regional Peripheral Circulation

217

Key References Aird WC. Phenotypic heterogeneity of the endothelium: I. Structure, function, and mechanisms. Circ Res. 2007;100:158-173. Aird WC. Phenotypic heterogeneity of the endothelium: II. Representative vascular beds. Circ Res. 2007;100:174-190. Cai H, Harrison DG. Endothelial dysfunction in cardiovascular diseases: the role of oxidant stress. Circ Res. 2000;87:840-844. Cannon RO III, Schechter AN, Panza JA, et al. Effects of inhaled nitric oxide on regional blood flow are consistent with intravascular nitric oxide delivery. J Clin Invest. 2001;108:279-287. Chin BY, Otterbein LE. Carbon monoxide is a poison … to microbes! CO as a bactericidal molecule. Curr Opin Pharmacol. 2009;9:490-500. Humpl T, Reyes JT, Holtby H, et al. Beneficial effect of oral sildenafil therapy on childhood pulmonary arterial hypertension: twelve-month clinical trial of a single-drug, open-label, pilot study. Circulation. 2005;111:3274-3280. Moncada S, Higgs A. The L-arginine-nitric oxide pathway. N Engl J Med. 1993;329:2002-2012. Olson KR. Hydrogen sulfide and oxygen sensing in the cardiovascular system. Antioxid Redox Signal. 2009;12:1219-1234. Ryter SW, Alam J, Choi AM. Heme oxygenase-1/carbon monoxide: from basic science to therapeutic applications. Physiol Rev. 2006;86:583-650. Stamler JS, Jia L, Eu JP, et al. Blood flow regulation by S-nitrosohemoglobin in the physiological oxygen gradient. Science. 1997;276:20342037. Sylvester JT, Gilbert RD, Traystman RJ, Permutt S. Effects of hypoxia on the closing pressure of the canine systemic arterial circulation. Circ Res. 1981;49:980-987.

The full reference list for this chapter is available at ExpertConsult.com.

e1



















































































































1. Aird WC. Phenotypic heterogeneity of the endothelium: II. Representative vascular beds. Circ Res. 2007;100:174-190. 2. Aird WC. Phenotypic heterogeneity of the endothelium: I. Structure, function, and mechanisms. Circ Res. 2007;100:158-173. 3. Sylvester JT, Gilbert RD, Traystman RJ, Permutt S. Effects of hypoxia on the closing pressure of the canine systemic arterial circulation. Circ Res. 1981;49:980-987. 4. Jerusalem E, Starling EH. On the significance of carbon dioxide for the heart beat. J Physiol. 1910;40:279. 5. Stamler JS, Jia L, Eu JP, et al. Blood flow regulation by S-nitrosohemoglobin in the physiological oxygen gradient. Science. 1997;276: 2034-2037. 6. Gladwin MT, Shelhamer JH, Schechter AN, et al. Role of circulating nitrite and S-nitrosohemoglobin in the regulation of regional blood flow in humans. Proc Natl Acad Sci U S A. 2000;97:11482-11487. 7. Cannon RO III, Schechter AN, Panza JA, et al. Effects of inhaled nitric oxide on regional blood flow are consistent with intravascular nitric oxide delivery. J Clin Invest. 2001;108:279-287. 8. Olson KR. Hydrogen sulfide and oxygen sensing in the cardiovascular system. Antioxid Redox Signal. 2009;12:1219-1234. 9. Chin BY, Otterbein LE. Carbon monoxide is a poison … to microbes! CO as a bactericidal molecule. Curr Opin Pharmacol. 2009; 9:490-500. 10. Ryter SW, Alam J, Choi AM. Heme oxygenase-1/carbon monoxide: from basic science to therapeutic applications. Physiol Rev. 2006;86: 583-650. 11. Beckman JS, Koppenol WH. Nitric oxide, superoxide, and peroxynitrite: the good, the bad, and ugly. Am J Physiol. 1996;271:C1424-C1437. 12. Cai H, Harrison DG. Endothelial dysfunction in cardiovascular diseases: the role of oxidant stress. Circ Res. 2000;87:840-844. 13. Sparks HV. Effect of local metabolic factors on vascular smooth muscle. In: Bohr DF, Somlyo AP, Sparks HV, eds. Handbook of Physiology. The Cardiovascular System. Bethesda, MD: American Physiological Society; 1980:2. 14. Moncada S, Higgs A. The L-arginine-nitric oxide pathway. N Engl J Med. 1993;329:2002-2012. 15. Humpl T, Reyes JT, Holtby H, et al. Beneficial effect of oral sildenafil therapy on childhood pulmonary arterial hypertension: twelvemonth clinical trial of a single-drug, open-label, pilot study. Circulation. 2005;111:3274-3280. 16. MacDonald RL, Kassell NF, Mayer S, et al. Clazosentan to overcome neurological ischemia and infarction occurring after subarachnoid hemorrhage (CONSCIOUS-1): randomized, double-blind, placebocontrolled phase 2 dose-finding trial. Stroke. 2008;39:3015-3021. 17. Morris CR, Morris SM Jr, Hagar W, et al. Arginine therapy: a new treatment for pulmonary hypertension in sickle cell disease? Am J Respir Crit Care Med. 2003;168:63-69. 18. Rubin LJ, Badesch DB, Barst RJ, et al. Bosentan therapy for pulmonary arterial hypertension. N Engl J Med. 2002;346:896-903. 19. Pickkers P, Dorresteijn MJ, Bouw MP, et al. In vivo evidence for nitric oxide-mediated calcium-activated potassium-channel activation during human endotoxemia. Circulation. 2006;114:414-421. 20. Roberts Jr JD, Fineman JR, Morin FC III, et al. Inhaled nitric oxide and persistent pulmonary hypertension of the newborn. The Inhaled Nitric Oxide Study Group. N Engl J Med. 1997;336:605-610. 21. Palmer RM, Ashton DS, Moncada S. Vascular endothelial cells synthesize nitric oxide from L-arginine. Nature. 1988;333:664-666. 22. Hess DR. Inhaled carbon monoxide: from toxin to therapy. Respir Care. 2017;62(10):1333-1342. 23. Yanagisawa M, Kurihara H, Kimura S, et al. A novel potent vasoconstrictor peptide produced by vascular endothelial cells. Nature. 1988;332:411-415. 24. Yang ZH, Richard V, von Segesser L, et al. Threshold concentrations of endothelin-1 potentiate contractions to norepinephrine and serotonin in human arteries. A new mechanism of vasospasm? Circulation. 1990;82:188-195.

25. Vane JR, Anggard EE, Botting RM. Regulatory functions of the vascular endothelium. N Engl J Med. 1990;323:27-36. 26. Bayliss W. On the local reactions of the arterial wall to changes of internal pressure. J Physiol. 1902;28:220-231. 27. Davis MJ, Hill MA. Signaling mechanisms underlying the vascular myogenic response. Physiol Rev. 1999;79:387-423. 28. Murphy TV, Spurrell BE, Hill MA. Cellular signalling in arteriolar myogenic constriction: involvement of tyrosine phosphorylation pathways. Clin Exp Pharmacol Physiol. 2002;29:612-619. 29. Cole WC, Chen TT, Clement-Chomienne O. Myogenic regulation of arterial diameter: role of potassium channels with a focus on delayed rectifier potassium current. Can J Physiol Pharmacol. 2005;83: 755-765. 30. Drummond HA, Grifoni SC, Jernigan NL. A new trick for an old dogma: ENaC proteins as mechanotransducers in vascular smooth muscle. Physiology (Bethesda). 2008;23:23-31. 31. Schubert R, Lidington D, Bolz SS. The emerging role of Ca21 sensitivity regulation in promoting myogenic vasoconstriction. Cardiovasc Res. 2008;77:8-18. 32. Sharif-Naeini R, Dedman A, Folgering JH, et al. TRP channels and mechanosensory transduction: insights into the arterial myogenic response. Pflugers Arch. 2008;456:529-540. 33. Lidington D1, Schubert R, Bolz SS. Capitalizing on diversity: an integrative approach towards the multiplicity of cellular mechanisms underlying myogenic responsiveness. Cardiovasc Res. 2013;97(3): 404-412. 34. Hislop A, Reid L. Pulmonary arterial development during childhood: branching pattern and structure. Thorax. 1973;28:129-135. 35. Reid LM. Structure and function in pulmonary hypertension. New perceptions. Chest. 1986;89:279-288. 36. Rudolph AM, Heymann MA. Circulatory changes during growth in the fetal lamb. Circ Res. 1970;26:289-299. 37. Rudolph AM. Fetal and neonatal pulmonary circulation. Annu Rev Physiol. 1979;41:383-395. 38. Heyman MA, Soifer S. Control of the fetal and neonatal pulmonary circulation. In: Weir EK, Reeves JT, eds. Pulmonary Vascular Physiology and Pathophysiology. New York: Marcel Dekker; 1989. 39. Iwamoto HS, Teitel D, Rudolph AM. Effects of birth-related events on blood flow distribution. Pediatr Res. 1987;22:634-640. 40. Rudolph AM. Distribution and regulation of blood flow in the fetal and neonatal lamb. Circ Res. 1985;57:811-821. 41. Dawes GS, Mott JC, Widdicombe JG, Wyatt DG. Changes in the lungs of the new-born lamb. J Physiol. 1953;121:141-162. 42. Enhorning G, Adams FH, Norman A. Effect of lung expansion on the fetal lamb circulation. Acta Paediatr Scand. 1966;55: 441-451. 43. Deleted in review. 44. Riley RL. Effects of lung inflation on the pulmonary vascular bed. In: Adams WW, Veith I, eds. Pulmonary Circulation. New York: Grune and Stratton; 1959. 45. Lopez-Muniz R, Stephens NL, Bromberger-Barnea B, et al. Critical closure of pulmonary vessels analyzed in terms of Starling resistor model. J Appl Physiol. 1968;24:625-635. 46. Rudolph AM, Yuan S. Response of the pulmonary vasculature to hypoxia and H1 ion concentration changes. J Clin Invest. 1966;45:399-411. 47. Marshall C, Marshall B. Site and sensitivity for stimulation of hypoxic pulmonary vasoconstriction. J Appl Physiol. 1983;55: 711-716. 48. Hoeper MM, Schwarze M, Ehlerding S, et al. Long-term treatment of primary pulmonary hypertension with aerosolized iloprost, a prostacyclin analogue. N Engl J Med. 2000;342:1866-1870. 49. Vavilala MS, Lee LA, Lam AM. Cerebral blood flow and vascular physiology. Anesthesiol Clin North Am. 2002;20:247-264. 50. Paulson OB. Blood-brain barrier, brain metabolism and cerebral blood flow. Eur Neuropsychopharmacol. 2002;12:495-501. 51. Fog M. Cerebral circulation. The reaction of pial arteries to a fall of blood pressure. Arch Neurol Psychiatry. 1937;37:351-364.

References

e2

























































77. Hoffman JI. Problems of coronary flow reserve. Ann Biomed Eng. 2000;28:884-896. 78. Bellamy RF. Diastolic coronary pressure-flow relations in the dog. Circ Res. 1978;43:92-101. 79. Hoffman JIE, Spaan JAE. Pressure-flow relations in coronary circulation. Physiol Rev. 1990;70:331-390. 80. Scharf SM, Bromberger-Barnea B, Permutt S. Distribution of coronary venous flow. J Appl Physiol. 1971;30:657-662. 81. Uhlig PN, Baer RW, Vlahakes GJ, et al. Arterial and venous coronary pressure-flow relations in anesthetized dogs. Evidence for a vascular waterfall in epicardial coronary veins. Circ Res. 1984;55: 238-248. 82. Coggins DL, Flynn AE, Austin RE Jr, et al. Nonuniform loss of regional flow reserve during myocardial ischemia in dogs. Circ Res. 1990;67:253-264. 83. Strauer BE. Left ventricular hypertrophy, myocardial blood flow and coronary flow reserve. Cardiology. 1992;81:274-282. 84. Strauer BE. Ventricular function and coronary hemodynamics in hypertensive heart disease. Am J Cardiol. 1979;44:999-1006. 85. Strauer BE. Myocardial oxygen consumption in chronic heart disease: role of wall stress, hypertrophy and coronary reserve. Am J Cardiol. 1979;44:730-740. 86. Strauer BE. The coronary circulation in hypertensive heart disease. Hypertension. 1984;6(suppl III):74-80. 87. Strauer BE. The significance of coronary reserve in clinical heart disease. J Am Coll Cardiol. 1990;15:775-783. 88. Hoffman JIE, Grattan MT, Hanley FL, Messina LM. Total and transmural perfusion of the hypertrophied heart. Cardiac Left Ventric Hypertr. 1983;131-151. 89. Vogt M, Motz W, Strauer BE. Coronary haemodynamics in hypertensive heart disease. Eur Heart J. 1992;13(suppl D):44-49. 90. Strauer BE. Significance of coronary circulation in hypertensive heart disease for development and prevention of heart failure. Am J Cardiol. 1990;65:34G-41G. 91. Vlahakes GJ, Turley K, Hoffman JIE. The pathophysiology of failure in acute right ventricular hypertension: hemodynamic and biochemical correlations. Circulation. 1981;63:87-95. 92. Belenkie I, Horne SG, Dani R, et al. Effects of aortic constriction during experimental acute right ventricular pressure loading. Further insights into diastolic and systolic ventricular interaction. Circulation. 1995;92:546-554. 93. Archie JP, Brown R. Effect of preload on the transmural distribution of diastolic coronary blood flow. J Surg Res. 1974;16:215-223. 94. Manohar M, Bisgard GE, Bullard V, Rankin JHG. Blood flow in the hypertrophied right ventricular myocardium of unanesthetized ponies. Am J Physiol. 1981;240(Heart Circ Physiol 9):H881-H888. 95. Murray PA, Vatner SF. Carotid sinus baroreceptor control of right coronary circulation in normal, hypertrophied, and failing right ventricles of conscious dogs. Circ Res. 1981;49:1339-1349. 96. Cooper N, Brazier J, Buckberg G. Effects of systemic-pulmonary shunts on regional myocardial blood flow in experimental pulmonary stenosis. J Thorac Cardiovasc Surg. 1975;70:166-176. 97. Mayers I, Johnson D. The nonspecific inflammatory response to injury. Can J Anaesth. 1998;45:871-879. 98. Baker JW, Deitch EA, Li M, et al. Hemorrhagic shock induces bacterial translocation from the gut. J Trauma. 1988;28:896-906. 99. Ohri SK, Bowles CW, Mathie RT, et al. Effect of cardiopulmonary bypass perfusion protocols on gut tissue oxygenation and blood flow. Ann Thorac Surg. 1997;64:163-170. 100. Moore FA, Sauaia A, Moore EE, et al. Postinjury multiple organ failure: a bimodal phenomenon. J Trauma. 1996;40:501-510, discussion 510-512. 101. Kirton OC, Windsor J, Wedderburn R, et al. Failure of splanchnic resuscitation in the acutely injured trauma patient correlates with multiple organ system failure and length of stay in the ICU. Chest. 1998;113:1064-1069. 102. Matheson PJ, Wilson MA, Garrison RN. Regulation of intestinal blood flow. J Surg Res. 2000;93:182-196.



























































52. Panerai RB, Dawson SL, Eames PJ, Potter JF. Cerebral blood flow velocity response to induced and spontaneous sudden changes in arterial blood pressure. Am J Physiol Heart Circ Physiol. 2001;280: H2162-H2174. 53. Georgiadis D, Sievert M, Cencetti S, et al. Cerebrovascular reactivity is impaired in patients with cardiac failure. Eur Heart J. 2000;21:407-413. 54. Lam JM, Smielewski P, al-Rawi P, et al. Prediction of cerebral ischaemia during carotid endarterectomy with preoperative CO2-reactivity studies and angiography. Br J Neurosurg. 2000;14:441-448. 55. Heistad DD, Marcus ML. Effect of sympathetic stimulation on permeability of the blood-brain barrier to albumin during acute hypertension in cats. Circ Res. 1979;45:331-338. 56. Mintun MA, Lundstrom BN, Snyder AZ, et al. Blood flow and oxygen delivery to human brain during functional activity: theoretical modeling and experimental data. Proc Natl Acad Sci U S A. 2001;98: 6859-6864. 57. Ehrlich MP, McCullough JN, Zhang N, et al. Effect of hypothermia on cerebral blood flow and metabolism in the pig. Ann Thorac Surg. 2002;73:191-197. 58. McCullough JN, Zhang N, Reich DL, et al. Cerebral metabolic suppression during hypothermic circulatory arrest in humans. Ann Thorac Surg. 1999;67:1895-1899, discussion 1919-1921. 59. Cheng W, Hartmann JF, Cameron DE, et al. Cerebral blood flow during cardiopulmonary bypass: influence of temperature and pH management strategy. Ann Thorac Surg. 1995;59:880-886. 60. Brodersen P, Paulson OB, Bolwig TG, et al. Cerebral hyperemia in electrically induced epileptic seizures. Arch Neurol. 1973;28:334-338. 61. Severinghaus JW, Lassen N. Step hypocapnia to separate arterial from tissue PCO2 in the regulation of cerebral blood flow. Circ Res. 1967;20:272-278. 62. Harper AM, Bell RA. The effect of metabolic acidosis and alkalosis on the blood flow through the cerebral cortex. J Neurol Neurosurg Psychiatry. 1963;26:341-344. 63. Kontos HA, Raper AJ, Patterson JL. Analysis of vasoactivity of local pH, PCO2 and bicarbonate on pial vessels. Stroke. 1977;8:358-360. 64. DiGeronimo RJ, Gegg CA, Zuckerman SL. Adenosine depletion alters postictal hypoxic cerebral vasodilation in the newborn pig. Am J Physiol. 1998;274:H1495-H1501. 65. Berger C, von Kummer R. Does NO regulate the cerebral blood flow response in hypoxia? Acta Neurol Scand. 1998;97:118-125. 66. Lavi S, Egbarya R, Lavi R, Jacob G. Role of nitric oxide in the regulation of cerebral blood flow in humans: chemoregulation versus mechanoregulation. Circulation. 2003;107:1901-1905. 67. Lindauer U, Megow D, Matsuda H, Dirnagl U. Nitric oxide: a modulator, but not a mediator, of neurovascular coupling in rat somatosensory cortex. Am J Physiol. 1999;277:H799-H811. 68. Ivankovich AD, Miletich DJ, Albrecht RF, Zahed B. Sodium nitroprusside and cerebral blood flow in the anesthetized and unanesthetized goat. Anesthesiology. 1976;44:21-26. 69. Yoshimoto S, Ishizaki Y, Kurihara H, et al. Cerebral microvessel endothelium is producing endothelin. Brain Res. 1990;508:283-285. 70. Willemsen MJ, Duncker DJ, Krams R, et al. Decrease in coronary vascular volume in systole augments cardiac contraction. Am J Physiol Heart Circ Physiol. 2001;281:H731-H737. 71. Gregg DE, Khouri EM, Rayford CR. Systemic and coronary energetics in the resting unanesthetized dog. Circ Res. 1965;16:102-113. 72. Lowensohn HS, Khouri EM, Gregg DE, et al. Phasic right coronary artery flow in conscious dogs with normal and elevated right ventricular pressures. Circ Res. 1976;39:760-766. 73. Kitamura K, Jorgensen CR, Gobel FL, et al. Hemodynamic correlates of myocardial oxygen consumption during upright exercise. J Appl Physiol. 1972;32:516-522. 74. Klocke FJ. Coronary blood flow in man. Prog Cardiovasc Dis. 1976;19:117-166. 75. Hoffman JIE. Maximal coronary flow and the concept of coronary vascular reserve. Circulation. 1984;70:153-159. 76. Hoffman JIE. A critical view of coronary reserve. Circulation. 1987;75(suppl I):6-11.

e3



























117. Ozel SK, Yuksel M, Haklar G, et al. Nitric oxide and endothelin relationship in intestinal ischemia/reperfusion injury (II). Prostaglandins Leukot Essent Fatty Acids. 2001;64:253-257. 118. Guzman JA, Rosado AE, Kruse JA. Dopamine-1 receptor stimulation attenuates the vasoconstrictive response to gut ischemia. J Appl Physiol. 2001;91:596-602. 119. Jakob SM, Ruokonen E, Takala J. Effects of dopamine on systemic and regional blood flow and metabolism in septic and cardiac surgery patients. Shock. 2002;18:8-13. 120. Clemmesen JO, Galatius S, Skak C, et al. The effect of increasing blood pressure with dopamine on systemic, splanchnic, and lower extremity hemodynamics in patients with acute liver failure. Scand J Gastroenterol. 1999;34:921-927. 121. LeDoux D, Astiz ME, Carpati CM, Rackow EC. Effects of perfusion pressure on tissue perfusion in septic shock. Crit Care Med. 2000;28:2729-2732. 122. Dworkin LD, Sun AM, Brenner BM. The renal circulations. In: Brenner BM, ed. The Kidney. 6th ed. Philadelphia: WB Saunders Company; 2000:277-318. 123. Myers BD, Deen WM, Brenner BM. Effects of norepinephrine and angiotensin II on the determinants of glomerular ultrafiltration and proximal tubule fluid reabsorption in the rat. Circ Res. 1975;37:101110. 124. Naicker S, Bhoola KD. Endothelins: vasoactive modulators of renal function in health and disease. Pharmacol Ther. 2001;90:61-88. 125. Shipley RE, Study RS. Changes in renal blood flow, extraction of inulin, glomerular filtration rate, tissue pressure and urine flow with acute alterations of renal artery blood pressure. Am J Physiol. 1951;167:676-688. 126. Bidani AK, Griffin KA, Williamson G, et al. Protective importance of the myogenic response in the renal circulation. Hypertension. 2009;54:393-398. 127. Cupples WA. Interactions contributing to kidney blood flow autoregulation. Curr Opin Nephrol Hypertens. 2007;16:39-45. 128. Loutzenhiser R, Griffin K, Williamson G, Bidani A. Renal autoregulation: new perspectives regarding the protective and regulatory roles of the underlying mechanisms. Am J Physiol Regul Integr Comp Physiol. 2006;290:R1153-R1167.































103. Meyer T, Brinck U. Differential distribution of serotonin and tryptophan hydroxylase in the human gastrointestinal tract. Digestion. 1999;60:63-68. 104. Fan L, Iseki S. Immunohistochemical localization of vascular endothelial growth factor in the globule leukocyte/mucosal mast cell of the rat respiratory and digestive tracts. Histochem Cell Biol. 1999;111:13-21. 105. Deleted in review. 106. Revelly JP, Tappy L, Berger MM, et al. Early metabolic and splanchnic responses to enteral nutrition in postoperative cardiac surgery patients with circulatory compromise. Intensive Care Med. 2001;27:540-547. 107. Kliegman RM, Pittard WB, Fanaroff AA. Necrotizing enterocolitis in neonates fed human milk. J Pediatr. 1979;95:450-453. 108. Cahill P, Redmond E, Sitzmann JV. Endothelial dysfunction in cirrhosis and portal hypertension. Pharmacol Ther. 2001;89:273-293. 109. Pannen BH, Bauer M, Noldge-Schomburg GF, et al. Regulation of hepatic blood flow during resuscitation from hemorrhagic shock: role of NO and endothelins. Am J Physiol. 1997;272:H2736-H2745. 110. Clemens MG. Nitric oxide in liver injury. Hepatology. 1999;30:1-5. 111. Shah V, Lyford G, Gores G, Farrugia G. Nitric oxide in gastrointestinal health and disease. Gastroenterology. 2004;126:903-913. 112. Matheson PJ, Spain DA, Harris PD, et al. Glucose and glutamine gavage increase portal vein nitric oxide metabolite levels via adenosine A2b activation. J Surg Res. 1999;84:57-63. 113. Watkins CC, Sawa A, Jaffrey S, et al. Insulin restores neuronal nitric oxide synthase expression and function that is lost in diabetic gastropathy. J Clin Invest. 2000;106:373-384. 114. Russo A, Fraser R, Adachi K, et al. Evidence that nitric oxide mechanisms regulate small intestinal motility in humans. Gut. 1999;44:72-76. 115. Perner A, Andresen L, Normark M, et al. Expression of nitric oxide synthases and effects of L-arginine and L-NMMA on nitric oxide production and fluid transport in collagenous colitis. Gut. 2001;49:387-394. 116. Burgener D, Laesser M, Treggiari-Venzi M, et al. Endothelin-1 blockade corrects mesenteric hypoperfusion in a porcine low cardiac output model. Crit Care Med. 2001;29:1615-1620.

e4

Abstract: When delivering critical care, one must understand the specific properties that characterize the various regional circulations because therapies that benefit one region may be detrimental to another. Vascular tone is influenced by (1) innervation and neural processes, (2) circulating endocrine and neuroendocrine mediators, (3) local metabolic products, (4) blood gas composition, (5) endothelial-derived factors, and (6) myogenic processes. The cerebral circulation is characterized by a blood-brain barrier. Regulation of myocardial perfusion is tailored to match regional myocardial oxygen supply to demand over the widest possible

range of cardiac workload. Critically ill patients are at risk for impaired splanchnic blood flow that can impair the two chief functions of the gastrointestinal system: (1) digestion and absorption of nutrients and (2) maintenance of a barrier to the translocation of enteric antigens. Renal blood flow remains constant over a wide range of renal artery perfusion pressures, but urinary flow rate is a function of renal perfusion pressure. Key words: vascular tone, endothelium, cerebral circulation, myocardial blood flow, renal blood flow, splanchnic circulation

25 Endothelium and Endotheliopathy YVES OUELLETTE





Until recently, scientists and clinicians considered the endothelium, the cell layer that lines the blood vessels, as an inert barrier separating the various components of blood and the surrounding tissues. The vascular endothelium is now recognized as a highly specialized and metabolically active organ performing a number of critical physiologic, immunologic, and synthetic functions. These functions include regulation of vascular permeability, fluid and solute exchange between the blood and interstitial space, vascular tone, cell adhesion, homeostasis, and vasculogenesis.1 The normal vascular endothelium is only one cell layer thick, separating the blood and vascular smooth muscle. The endothelium responds to physical and biochemical stimuli by releasing regulatory substances affecting vascular tone and growth, thrombosis and thrombolysis, and platelet and leukocyte interactions with the endothelium. Because normal endothelial function plays a central role in vascular homeostasis, it is logical to conclude that endothelial dysfunction contributes to disease states characterized by vasomotor dysfunction, abnormal thrombosis, or abnormal vascular proliferation. The endothelium lies between the lumen and vascular smooth muscle, where it is uniquely positioned to “sense” changes in hemodynamic forces or blood-borne signals by membrane receptor mechanisms. The endothelial cells can respond to physical and chemical stimuli by synthesis or release of a variety of vasoactive and thromboregulatory molecules and growth factors. The vascular endothelium possesses numerous enzymes, receptors, and transduction molecules, and interacts with other vessel wall constituents and circulating blood cells. In addition to these universal functions, the endothelium may have organ-specific roles that are differentiated for various parts of the body, such as gas exchange in the lungs, control of myocardial function in the heart, or phagocytosis in the liver and spleen. 218





Because of their location, endothelial cells have the ability to interact with blood components, such as flow, soluble factors, and other cells. Endothelial cells integrate these signals into a cohesive regulation of vascular responses. The endothelium controls the vascular tone of the underlying smooth muscle cells through the production of vasodilator and vasoconstrictor mediators.









PEARLS Endothelial cell activation in response to inflammation changes endothelial cellular physiology and alters vascular function. A large number of endothelial cell–active molecules are potential biomarkers for the early diagnosis of sepsis.

Normal Endothelial Function Endothelial Cell Heterogeneity Many vascular diseases appear to be restricted to specific vascular beds. For example, thrombotic events are often localized to single vessels. It is also common for certain vasculitides to specifically affect certain arteries, veins, or capillaries or to affect certain organs. Tumor cells often metastasize more commonly within particular vascular beds. The basis for this variability in vascular disease is poorly understood but may be explained by the heterogeneity of endothelial cells. There has been a greater understanding of how endothelial cell heterogeneity may contribute both to the maintenance of organ-specific function and to the development of disorders restricted to specific vascular beds.1–3 The cell biology of capillary endothelium from different vascular beds may explain differences in tissue function. For example, the brain microcirculation is lined by endothelial cells connected by tight junctions that maintain the blood-brain barrier. By contrast, sinusoids found in the liver, spleen, and bone marrow are lined by endothelial cells that allow transcellular trafficking between intercellular gaps. Similarly, fenestrated endothelial cells found in the intestinal villi, endocrine glands, and kidneys facilitate selective permeability, which is required for efficient absorption, secretion, and filtering.4 Another example of endothelial cell heterogeneity lies in the expression of cell surface receptors involved in cell-to-cell signaling and cell trafficking. For example, in the mouse, lung-specific endothelial cell adhesion molecules are exclusively expressed by pulmonary postcapillary endothelial cells and some splenic venules. Similarly, specific mucosal cell adhesion molecules are expressed primarily on endothelial venules in the Peyer patches of



CHAPTER 25

the small intestine.5,6 Tumor cells may show clear preferential adhesion to the endothelium of specific organs paralleling their in vivo metastatic propensities.7

Endothelial Progenitor Cells A significant amount of literature has shown that maintenance and repair of vasculature in ischemic diseases may be at least partially mediated through recruitment of endothelial progenitor cells (EPCs) from the bone marrow to areas of vascular injury. An EPC is a specific subtype of hematopoietic stem cell that has been isolated from circulating mononuclear cells, bone marrow, and cord blood. EPCs migrate from the bone marrow to the peripheral circulation, where they contribute to vascular repair.8,9 When injected into animal models of ischemia, EPCs are incorporated into sites of neovascularization8,10,11 and have contributed to improved outcomes in patients with ischemic vascular disorders.12 In addition, there has been accumulating evidence for the function of EPCs in critical illnesses such as sepsis. Recruitment of EPCs to areas of endothelial and vascular damage may have prognostic implications and could be associated with clinical outcome. The pathophysiologic changes associated with critical illness—notably, sepsis and sepsis-related organ dysfunction—may lead to apoptosis and necrosis of endothelial cells from the vasculature and recruitment of EPCs from the bone marrow. In various models of vascular injury and organ dysfunction, only a few studies have emerged regarding EPCs in particular as a therapeutic strategy. Transplanted EPCs have been shown to improve survival of mice following liver injury.13 Infusion of EPCs also restored blood flow in a mouse model of hind limb ischemia.9 A prospective randomized trial compared the effects of EPC transplantation in patients with idiopathic pulmonary arterial hypertension versus conventional therapy and showed that after 12 weeks, patients who had received EPCs had a significant improvement in their 6-minute walk test, mean pulmonary artery pressure, pulmonary vascular resistance, and cardiac output.14

Procoagulant Mechanisms The expression and release of tissue factor is the pivotal step in transforming the endothelium from an anticoagulant to a procoagulant surface.21,22 Tissue factor accelerates factor VIIadependent activation of factors X and IX (see Fig. 25.1). The synthesis of tissue factor is induced by a number of agonists— including thrombin, endotoxin, several cytokines, shear stress,

Tissue factor Factor VII

Anticoagulant Mechanisms

TF

Factor VIIIa

Factor X

Factor Xa

Prothrombin

Thrombin

Antithrombin

• Fig. 25.1

Tissue factor inhibitor

Factor VIIa

Fibrinogen

Fibrin

Thrombomodulin

Endothelium Factor VIII

Factor Va

Activated protein C Protein C

​Endothelium control of the coagulation cascade. An inflammatory stimulus upregulates the interaction of tissue factor (TF) with factor VII, which generates activated factor VII (factor VIIa). The TF–factor VIIa complex then leads to the conversion of factor X to factor Xa. The interaction of factors Xa and Va results in the conversion of prothrombin to thrombin and the conversion of fibrinogen to fibrin. Three key anticoagulant pathways can inhibit this process. Protein C is activated through its interaction with cell-surface thrombomodulin and inhibits the activities of factors Va and VIIIa. Antithrombin blocks the activation of multiple factors, including factor X and thrombin. Tissue factor pathway inhibitor interferes directly with the tissue factor–factor VIIa complex.  

The endothelium has anticoagulant, antiplatelet, and fibrinolytic properties.16 Endothelial cells are the major sites for anticoagulant reactions involving thrombin. Thrombin plays a key role in coagulation, including the activation of platelets, activation of several coagulation enzymes and cofactors, and stimulation of procoagulation pathways on the endothelial cell surface. In the normal state, there is little thrombin enzyme activity. The surrounding endothelial cell matrix contains heparin sulfate and related glycosaminoglycans that activate antithrombin III. In addition, the subendothelial cell matrix contains dermatan sulfate, which promotes the antithrombin activity of heparin cofactor II. Furthermore, microvascular endothelial cells release a tissue factor pathway inhibitor that inhibits the factor VIIa/tissue factor complex and further contributes to anticoagulation (Fig. 25.1).

219

Thrombin activity is also modulated by endothelial cell synthesis of thrombomodulin.17,18 The binding of thrombin to thrombomodulin facilitates the enzyme’s activation of the anticoagulant protein C. Activated protein C (APC) activity is enhanced by cofactor C, also called protein S, which is synthesized by endothelial cells as well as by other cells (see Fig. 25.1). APC inhibits factor Va and factor VIIIa. Thrombomodulin (TM) also inhibits prothrombinase activity indirectly by binding factor Xa (Fig. 25.2). Protein C has a special receptor on the endothelial cells: endothelial cell protein C receptor (EPCR). EPCR augments protein C activation approximately 20-fold in vivo by binding protein C and presenting it to the thrombin-TM activation complex. Both EPCR and TM can be found in plasma as soluble proteins. APC retains its ability to bind EPCR; this complex appears to be involved in some of the cellular signaling mechanisms that downregulate inflammatory cytokine formation (tumor necrosis factor [TNF], interleukin-6). In addition, platelet adhesion to endothelial cells is markedly inhibited by endothelium-derived prostacyclin.19 The interactions between platelets and endothelium regulate platelet function, coagulation cascades, and local vascular tone. Microvascular endothelial cells may secrete tissue plasminogen activator (t-PA), the powerful thrombolytic agent in frequent clinical use for treatment of coronary thrombotic occlusion.20 t-PA release is stimulated in vivo by norepinephrine, vasopressin, or stasis within the vessel lumen. Thrombin may also stimulate t-PA release, providing a further endothelium-mediated safeguard against uncontrolled coagulation.

Coagulation and Fibrinolysis A normal physiologic function of the endothelium is to provide an antithrombotic surface inhibiting platelet adhesion and clotting, facilitating normal blood flow. Under pathophysiologic conditions, the endothelium transforms into a prothrombotic surface. A dynamic equilibrium exists between both states that permits a rapid response to an insult and a rapid recovery.15

Endothelium and Endotheliopathy

SECTION IV



220

Pediatric Critical Care: Cardiovascular

PROTEIN C PATHWAY C4bp

Endotoxins Cytokines Coagulation Inhibition of fVa and fVIIIa

L-arg

+

PC

APC sTM

Prot C receptor

Thrombin

TM

TAFI

Endothelial cell Membrane

Inflammation • ↓ Cytokine production • ↓ Leukocyte adherence • ↓ Apoptosis, etc.

Ca2+

+

iNOS cNOS NO + Cit

+PS

sEPCR

Shearing forces CaM

+ Ca2+

Vasoactive agents +

R

Endothelial cell NO + GTP

GC

cGMP-PK cGMP

cGMP-P

Smooth muscle cell

•  Fig. 25.3

​Nitric oxide (NO) is generated from L-arginine (L-arg) by the action of nitric oxide synthase (NOS). In the resting state, constitutive NOS (cNOS) is modulated by intracellular Ca21 and calmodulin (CaM). Stored Ca21 is released in response to vasoactive agents (e.g., acetylcholine and bradykinin) and other external stimuli, such as shearing forces. Activation of endothelial cells by cytokines and endotoxins increases expression of inducible NOS (iNOS). Citrulline (Cit) is a byproduct of NO production. NO has a half-life of only a few seconds in vivo and quickly diffuses to surrounding cells, such as smooth muscle cells. NO stimulates the production of the intracellular mediator cyclic GMP (cGMP). Increased cGMP activates a series of cGMP-dependent protein kinases (cGMP-PKs) and cGMP-dependent phosphatases (cGMP-Ps).  

Fibrinolysis Inhibition of PAI-1

• Fig. 25.2 ​The interaction of the protein C system with the endothelium: Thrombin bound to thrombomodulin (TM) modifies protein C bound to the endothelial protein C (Prot C) receptor on the cell surface to generate activated protein C (APC). APC acts as a natural anticoagulant by inactivating activated factors V (fVa) and VIII (fVIIIa), modulating inflammation by downregulating the synthesis of proinflammatory cytokines, leukocyte adherence, and apoptosis and enhancing fibrinolysis by inhibiting thrombin-activatable fibrinolysis inhibitor (TAFI) and plasminogen activator inhibitor type-1 (PAI-1). C4Bbp, C4b binding protein (binds protein S); 1PS, in the presence of protein S; sEPCR, soluble endothelial cell protein C receptor; sTM, soluble thrombomodulin. (Modified from Hazelzet J. Pathophysiology of pediatric sepsis. In: Nadel S. Infectious Diseases in the Pediatric Intensive Care Unit. London: Springer; 2008.)  



hypoxia, oxidized lipoproteins, and other endothelial insults. Once endothelial cells expressing tissue factor are exposed to plasma, prothrombinase activity is generated and fibrin is formed on the surface of the cells. Tissue factor can also be found in plasma as a soluble protein. Its role there is not well understood, but it probably plays a role in the initiation of coagulation.

Endothelium-Derived Vasodilators The important role that the endothelium plays in controlling vascular tone has only recently been appreciated. Clinicians and researchers have come to appreciate that the endothelium controls underlying smooth muscle tone in response to certain pharmacologic and physiologic stimuli. This response involves a number of luminal membrane receptors and complex intracellular pathways and the synthesis and release of a variety of relaxing and constricting substances.

Nitric Oxide Furchgott and Zawadzki first postulated the existence of an endothelium-derived relaxing factor (EDRF) in 1980, when they noticed that the presence of endothelium was essential for rabbit aortic rings to relax in response to acetylcholine.21 Later, it was determined that the biologic effects of EDRF are mediated by nitric oxide (NO).23 NO is generated from the conversion of L-arginine to NO and L-citrulline by the enzyme nitric oxide synthase (NOS).24 There are two general forms of NOS: constitutive and inducible. In the unstimulated state, NO is continuously produced by constitutive NO synthase (cNOS). The activity of cNOS is modulated by calcium that is released from endoplasmic stores in response to the

activation of certain receptors. Substances such as acetylcholine, bradykinin, histamine, insulin, and substance P stimulate NO production through this mechanism. Similarly, shearing forces acting on the endothelium are another important mechanism regulating the release of NO. The inducible form of NOS (iNOS) is not calcium dependent but instead is stimulated by the actions of cytokines (e.g., TNF-a, interleukins) or bacterial endotoxins (e.g., lipopolysaccharide). iNOS occurs over several hours and results in NO production that may be more than a thousandfold greater than that produced by cNOS. This is an important mechanism in the pathogenesis of inflammation (Fig. 25.3). Inhibition of NOS using competitive analogs of L-arginine drastically reduces endothelium-dependent relaxation in vitro, particularly in large conduit arteries, thereby evoking vasoconstriction. Chronic treatment of animals with NOS inhibitors or suppression of the cNOS gene is reported to induce hypertension.25–27 Once NO is formed by an endothelial cell, it readily diffuses out of the cell and into adjacent smooth muscle cells where it binds and activates the soluble form of guanylyl cyclase, resulting in the production of cyclic guanosine monophosphate (cGMP) from guanosine triphosphate.28 cGMP, in turn, activates a number of cGMP-modulated enzymes (see Fig. 25.3). Increased cGMP activates a kinase that subsequently leads to the inhibition of calcium influx into the smooth muscle cell and decreased calcium-calmodulin stimulation of myosin light chain kinase. This, in turn, decreases the phosphorylation of myosin light chains, decreasing smooth muscle tension development and causing vasodilation. There is also some evidence that increases in cGMP can lead to myosin light chain dephosphorylation by activating the phosphatase. In addition, cGMP-dependent protein kinase phosphorylates potassium ion (K1) channels to induce hyperpolarization, thereby inhibiting vasoconstriction.29,30 Interestingly, NO inhibition of platelet aggregation is also related to the increase in cGMP. Drugs that inhibit the breakdown of cGMP, such as inhibitors of cGMP-dependent phosphodiesterase (e.g., sildenafil), potentiate the effects of NO-mediated actions on the target cell.



CHAPTER 25

Endothelium and Endotheliopathy

221

Therefore, NO contributes to the balance between vasodilator and vasoconstrictor influences that determine vascular tone.31

Endothelium-Derived Vasoconstrictors

Prostacyclin Another major endothelium-derived vasodilator is the prostaglandin prostacyclin (PGI2), a derivative of arachidonic acid synthesized through the action of the enzyme cyclooxygenase. Endothelium cells are capable of producing a variety of vasoactive substances that are products of arachidonic acid metabolism. Among these are prostaglandins, PGI2, leukotrienes, and thromboxanes. These substances act as either vasodilators or vasoconstrictors, among their other biological activities. PGI2 is a potent vasodilator and is active in both the pulmonary and systemic circulations. In addition to its vasodilatory effects, prostacyclin also has antithrombotic and antiplatelet activity. Its release may be stimulated by bradykinin and adenine nucleotides. Like NO, it is chemically unstable, with a short half-life.32 However, unlike NO, PGI2 activity in arterial beds depends on its ability to bind to specific receptors in vascular smooth muscle. Therefore, its vasodilator activity is determined by the expression of such receptors. PGI2 receptors are coupled to adenylate cyclase to elevate cyclic adenosine monophosphate (cAMP) levels in vascular smooth muscle.33 The increase in cAMP results in (1) stimulation of adenosine triphosphate (ATP)-sensitive K1 channels, leading to hyperpolarization of the cell membrane and inhibition of the development of contraction; and (2) increased efflux of calcium ions (Ca21) from the smooth muscle cell and inhibition of the contractile machinery. In addition, PGI2 facilitates the release of NO by endothelial cells, and NO potentiates the action of PGI2 in vascular smooth muscle. Interestingly, NO may also potentiate the effects of prostacyclin. The NO-mediated increase in cGMP in smooth muscle cells inhibits a phosphodiesterase that breaks down cAMP and therefore indirectly prolongs the half-life of the second messenger of PGI2.34

Endothelin is a 21-amino-acid peptide and is one of the most potent vasoconstrictors identified to date. Endothelial cells synthesize the prohormone big endothelin and express endothelinconverting enzymes to generate endothelin. There are three isoforms of endothelin, but only one (ET-1) has been shown to be released from human endothelial cells. ET-1 is synthesized in the endothelial cells. Its release is mediated by a variety of stimuli. ET-1 release is stimulated by angiotensin II, antidiuretic hormone, thrombin, cytokines, reactive oxygen species, and shearing forces acting on the vascular endothelium. ET-1 release is inhibited by NO as well as by PGI2 and atrial natriuretic peptide.36,37 ET-1 has a short half-life, suggesting that, similarly to NO, ET-1 is mainly a locally active vasoregulator. Once released by endothelial cells, ET-1 binds to a membrane receptor (ETA) found on adjacent vascular smooth muscle cells. This binding leads to calcium mobilization and smooth muscle contraction. The ETA receptor is coupled with a G-protein linked to phospholipase-C, resulting in the formation of IP3. Interestingly, ET-1 can also bind to an ETB receptor located on the vascular endothelium, which stimulates the formation of NO by the endothelium. This release of NO appears to modulate the ETA receptor–mediated contraction of the vascular smooth muscle. Its physiologic role includes maintenance of basal vascular resistance, and it is present in healthy subjects in low concentrations.

Endothelium-Derived Hyperpolarizing Factor Endothelium stimulation by acetylcholine also produces hyperpolarization of the underlying smooth muscle and thereby induces vasorelaxation. This process is not mediated by NO but is instead mediated by another endothelium-derived factor. This factor increases K1-channel conductance in smooth muscle cells, resulting in smooth muscle cell relaxation. The resulting vasodilation is not inhibited by NG-methyl-L-arginine (L-NMMA), the specific antagonist of NO, but rather is inhibited by ouabain, a sodiumpotassium adenosine triphosphatase inhibitor. In addition, in most medium- to resistance-sized arteries, electrophysiologic studies have established that endothelium-dependent hyperpolarization of vascular smooth muscle is resistant to the combined inhibition of both NOs and cyclooxygenases. Accordingly, a component of the endothelium-dependent relaxation in these arteries is mediated by a substance different from NO and PGI2. This component of endothelium-dependent vasodilation has been attributed to an as yet unidentified diffusible endotheliumderived hyperpolarizing factor (EDHF).35 Of significant clinical importance is the fact that the EDHFmediated effect increases as the arterial diameter decreases, such as in resistance arteries. EDHF likely plays a significant role in the regulation of peripheral vascular resistance and local hemodynamics. Unfortunately, in the absence of selective inhibitors of the EDHF pathway, it is not possible to evaluate the relevance of EDHF in vivo.35

Endothelins (Endothelium-Derived Contracting Factors)

Reactive Oxygen Species Endothelial cells secrete oxygen-derived free radicals and hydrogen peroxide in response to shear stress and endothelial agonists such as bradykinin. Such reactive oxygen species are reported to inactivate NO, resulting in vasoconstriction. Reactive oxygen species may also facilitate the mobilization of cytosolic Ca21 in vascular smooth muscle cells and promote Ca21 sensitization of the contractile elements. Under conditions of hyperoxia, endotheliumderived superoxide anion may combine with NO with diffusionlimited kinetics to generate peroxynitrite, negating NO-mediated vasodilation, an effect inhibited by superoxide dismutase, which metabolizes superoxide anion to hydrogen peroxide.38 Vasoconstrictor Prostaglandins The metabolism of arachidonic acid by cyclooxygenase in endothelial cells may lead to the secretion of precursors of thromboxanes and leukotrienes. These prostaglandins act on receptors in vascular smooth muscle to induce vasoconstriction. PGI2, however, is the major endothelial metabolite of arachidonic acid that is generated through the cyclooxygenase pathway. Thus, under normal circumstances, the influence of the small amounts of vasoconstrictor prostanoids released by endothelial cells is masked by the production of PGI2, NO, and EDHF.39,40

Endothelium and Blood Cell Interactions In addition to the interactions of the endothelium with blood coagulation factors, endothelial cells also express cell-surface molecules known as the endothelial glycocalyx (EG). Over the past decade, we have learned about the role of the glycocalyx in vascular physiology and pathology, including mechanotransduction, hemostasis, signaling, and blood cell–vessel wall interactions. The glycocalyx is a gel-like layer enriched with carbohydrates. Its dimensions vary according to the type of vasculature, ranging from

SECTION IV



222

Pediatric Critical Care: Cardiovascular

Neutrophils

Intact glycocalyx

Integrin Lack of permeability

Transmigration Rolling and Firm Tethering integrin adhesion activation

Selectin receptor

Lack of leukocyte attachment

E-selectin

Glycocalyx

Selectin expression Endothelial cell

ICAM Inflammatory stimulus

Injured glycocalyx

Extracellular matrix

• Fig. 25.5

​Leukocyte recruitment process and transmigration. The multistep model for leukocyte recruitment at sites of inflammation begins with the activation of neutrophils and endothelial cells. Once activated, endothelial cells express selectins, whose binding to neutrophils initiates rolling and adhesion of neutrophils to the endothelium. Activated integrins on the surface of neutrophils bind to endothelial cell intercellular adhesion molecules (ICAMs), facilitating a firm adhesion. Transmigration through the endothelium further involves interactions with other molecules, such as platelet endothelial cell adhesion molecules and cadherins on the surface of endothelial cells.  

Increased permeability

Leukocyte translocation

• Fig. 25.4

​Physiologic role of the intact glycocalyx. The endothelial glycocalyx is an important determinant of vascular permeability, influences blood cell–vessel wall interactions, acts as a mechanotransducer, and binds to plasma proteins. The injured glycocalyx leads to impaired mechanotransduction, increased leukocyte egress, loss of coagulation control, and increased permeability.  

0.2 mm to more than 2 mm. The EG consists of various types of glycosaminoglycans covalently attached to plasma membrane– bound core proteoglycans. Heparan sulfate comprises 50% to 90% of endothelial glycosaminoglycans. The remainder is a mixture of hyaluronic acid, dermatan, keratan, and chondroitin sulfates.41 In healthy vessels, the EG determines vascular permeability, attenuates blood cell–vessel wall interactions, mediates shear stress sensing, enables balanced signaling, and fulfills a vasculoprotective role. When the EG is disrupted or modified, these properties are lost (Fig. 25.4). Evidence is emerging that damage to the glycocalyx plays a pivotal role in several vascular pathologies.

Interactions of Leukocytes With the Vessel Wall It is now well established that flowing leukocytes may adhere to specific regions of the endothelium in response to tissue injury or infection. These multicellular interactions are essential precursors of physiologic inflammation. Leukocytes interact with vessel surfaces through a multistep process that includes (1) initial formation of usually reversible attachments; (2) activation of the attached cells; (3) development of stronger, shear-resistant adhesion; and (4) spreading, emigration, and other sequelae36 (Fig. 25.5). Selectins are key molecules in the interaction of leukocytes and endothelial cells. They are transmembrane glycoproteins that recognize cell-surface carbohydrate ligands found on leukocytes and initiate and mediate tethering and rolling of leukocytes on the endothelial cell surface. Selectins constitute a family of three known molecules. L-selectin is expressed on most leukocytes and

binds to ligands constitutively expressed on endothelial cells found in venules of lymphoid tissues. Its expression is induced on endothelium at sites of inflammation. E-selectin is expressed on activated endothelial cells and leukocytes. P-selectin is rapidly redistributed from secretory granules to the surface of platelets and endothelial cells stimulated with thrombin. Both endothelial cell E-selectin and P-selectin bind to ligands on leukocytes.37 With stimulation, leukocytes usually attach to the glycocalyx of endothelial cells, where shear stresses are lowest. Leukocytes adherent to the endothelium can make contact with flowing leukocytes through the L-selectin molecule, resulting in amplification of leukocyte recruitment to sites of inflammation. It is generally understood that selectins initiate inflammatory, immune, and hemostatic responses by promoting transient multicellular interactions.42 Proinflammatory molecules presented on the surface of the endothelium proceed to activate a second family of adhesion molecules, the integrins, and cause cells to firmly adhere. After the initial tethering of leukocytes to endothelial cells, leukocytes then must roll prior to transmigrating through the endothelium. Inhibition of leukocyte adhesion does not reduce leukocyte rolling, suggesting that rolling and adhesion are distinct molecular events. In addition, inhibiting rolling reduces adhesion, suggesting that rolling is a prerequisite of leukocyte adhesion/recruitment and, ultimately, the inflammatory response. Leukocytes subsequently migrate between endothelial cells into tissues by mechanisms that are not completely understood but that we know are affected by gradients of chemokines, integrin activation states, and interactions with PECAM-1, an Ig-like receptor. This migration requires disruption of endothelial-cellto-endothelial-cell interaction of cadherins at tight junctions. Leukocyte recruitment to lymphoid tissues or inflammatory sites requires the coordinated expression of specific combinations of adhesion and signaling molecules. Diversity at each step of the cascade ensures that the appropriate leukocytes accumulate for a restricted period in response to a specific challenge.4,42



CHAPTER 25

Platelet Adhesion Endothelial cells and circulating platelets normally do not interact with each other due to the release of PGI2, release of NO, and expression of CD39 on the surface of endothelial cells.43 During vascular injury and inflammation, platelets adhere to exposed subendothelial components and are rapidly activated. Circulating platelets interact with the adherent platelets, producing a hemostatic plug that promotes thrombin generation and development of a stable fibrin clot. High shear stress, as seen in arteries, increases platelet adherence to the subendothelium where unactivated platelets attach to the subendothelium through interactions of platelet glycoproteins with immobilized von Willebrand Factor (vWF), a large multimeric protein with binding sites for several other molecules, including subendothelial collagen. Flowing platelets attach transiently to vWF, resulting in continuous movement of the cells along the surface. Under the lower shear stresses found in veins, unactivated platelets interact with integrins to attach to and immediately arrest on immobilized fibrinogen.44 Once platelets adhere to either vWF or fibrinogen, they are activated by secreted products, such as adenosine diphosphate or epinephrine or by surface molecules such as collagen that crosslink the integrins and other platelet receptors. The activated platelets spread and adhere more avidly to the subendothelial surface, which recruits additional platelets into aggregates. Shear-resistant adhesion may be further enhanced by interactions of other integrins or receptors with laminin, fibronectin, and thrombospondin. As thrombin is generated, converting bound fibrinogen to fibrin, the aggregated platelets contract to strengthen the clot.44

Endothelial Cell Dysfunction Ischemia-Reperfusion Injury Reperfusion of previously ischemic tissues can place the organs at risk for further cellular injury, limiting the recovery of function. The microvasculature, particularly the endothelial cells, is vulnerable to the deleterious consequences of ischemia and reperfusion

223

Cell junctions intact

Lumen β-Catenin α-Catenin p120-catenin VE-cadherin

Subendothelial space

Cell junctions disassembly

Vascular leak

β-Catenin

α-Catenin

VE-caderin

Endothelial Permeability

p120-catenin

Interstitial edema

• Fig 25.6

​Inflammatory mediators induce gaps between endothelial cells by disassembly of intercellular junctions and disturbing the cytoskeleton. The creation of gaps can result in microvascular leak and tissue edema. Key in this process is the dissociation of p120-catenin from VE-cadherin in response to inflammatory mediators.  

Maintenance of the integrity of the vascular endothelium is crucial for several physiologic functions, such as normal tissue fluid homeostasis, vessel tone, and host defense. The vascular integrity and permeability barrier function is crucially supported by intercellular junctions between endothelial cells. There are two major subtypes of endothelial intercellular junctions: tight junctions (or zona occludens) and adherens junctions (or zona adherens). In normal conditions, the barrier function of the vascular endothelium is properly regulated and vascular permeability is limited. In vascular pathology, such as sepsis, proinflammatory signals activate endothelial cells, resulting in disruption and destabilization of the endothelial barrier.45,46 VE-cadherin is the major component of adherens junctions and is a tightly regulated protein complex that joins adjacent endothelial cells and prevents vascular leak (Fig. 25.6). The displacement of VE-cadherin from the cell membrane to the interior of the cell induces gaps between endothelial cells, leading to increased permeability. The disruption of the VE-adherens complex is prevented by another protein, p120catenin, which binds to and stabilizes VE-cadherin at the membrane. Inflammatory mediators are known to cause p120-catenin and VE-cadherin to dissociate, resulting to internalization of VEcadherin.

Endothelium and Endotheliopathy

(I-R). I-R is now recognized as a potentially serious problem encountered during a variety of standard medical and surgical procedures, such as thrombolytic therapy, organ transplantation, and cardiopulmonary bypass.47 Hypoxia and inflammation are intimately linked on many levels and have functional roles in many human diseases. Indeed, a wide range of clinical conditions are characterized by hypoxiaor ischemia-driven inflammation or by inflammation-associated hypoxia. The molecular and biochemical changes in the vascular wall during I-R are characteristic of an acute inflammatory response (Fig. 25.7). The intensity of this inflammatory response can be so severe that the injury response to reperfusion is also manifested in susceptible organs, such as the lungs and cardiovascular system. The resulting systemic inflammatory response syndrome (SIRS) and multiple-organ dysfunction syndrome (MODS) are both associated with significant increases in mortality and morbidity.48 Microvascular dysfunction associated with I-R is manifested as impaired endothelium-dependent dilation in arterioles, enhanced

SECTION IV

Sepsis



224

Pediatric Critical Care: Cardiovascular

Ischemia/ reperfusion

Immune complexes

Systemic mediator release

Neutrophil activation and adhesion molecule expression

Endothelial activation and adhesion molecule expression

Neutrophil-endothelial cell interaction: Tethering, rolling, adhesion, transmigration

Endothelial cell dysfunction

Neutrophil-mediated remote organ vascular dysfunction

Endothelial cell–mediated vascular dysfunction

Tissue injury

•  Fig. 25.7

​Mechanisms that underlie the development of local and remote organ injury following an initial inflammatory event. The activation of endothelial cells and circulating neutrophils leads to the expression and activation of adhesion molecules that facilitate neutrophil invasion of vascular beds, resulting in local and remote organ dysfunction.  

fluid filtration, leukocyte plugging in capillaries, and the trafficking of leukocytes and plasma protein extravasation in postcapillary venules. During the initial period following reperfusion, activated endothelial cells in the microcirculation produce more oxygen radicals and less NO. The resulting imbalance between superoxide and NO in endothelial cells leads to the production and release of inflammatory mediators (e.g., platelet-activating factor, TNF-a) and enhances the biosynthesis of adhesion molecules that mediate leukocyte–endothelial cell adhesion.48 Since its discovery in the early 1990s, hypoxia-inducible factor 1 (HIF1) has been increasingly recognized for its key role in transcriptional control of more than 100 genes that regulate a wide spectrum of cellular functional events, including angiogenesis, vasomotor control, glucose and energy metabolism, erythropoiesis, iron homeostasis, pH regulation, cell proliferation, and viability. Animal studies have provided compelling data to demonstrate a pivotal role for the HIF pathway in the pathogenesis of ischemic injury. For example, HIF1a has been shown to play a role in mediating cardioprotection.49 The inflammatory mediators released as a consequence of reperfusion also appear to activate endothelial cells in remote organs that are not exposed to the initial ischemic insult. Oxidants and activated leukocytes have been implicated as mediators of remote organ injury in I-R. This distant response to I-R can result in leukocyte-dependent microvascular injury that is characteristic of SIRS and MODS. The pulmonary damage associated with MODS can range from mild dysfunction to severe failure, as in acute respiratory distress syndrome (ARDS). The pulmonary injuries associated with ARDS include increased pulmonary microvascular permeability and the accumulation of neutrophil-rich

alveolar fluid. Respiratory failure often is associated with cardiovascular, hepatic, gastrointestinal, and renal dysfunction as well as central nervous system involvement. MODS is associated with dysfunction of the coagulation cascade and immune system, resulting in thrombosis, disseminated intravascular coagulation, and immunocompromise. The initiation of MODS may also lead to further tissue ischemia, resulting in additional insult.50

Sepsis Although the pathophysiologic process of multiple organ dysfunction during sepsis is multifactorial, one common feature is the dysfunction of the microcirculation, including the resistance of arteries, capillaries, and postcapillary venules. The microcirculation cannot be considered a simple passive conduit. Rather, it is a functionally active system of interactions among the vascular wall; circulating and tissue-associated cells, such as leukocytes, platelets, and mast cells; and extracellular mediators that contribute to the regulation of local, downstream, and upstream vascular tone. Sepsis is particularly associated with microvascular endothelial cell dysfunction leading to (1) the breakdown of endothelial barrier function, leading to tissue edema and uncontrolled inflammatory cell infiltration; (2) vasomotor dysfunction, leading to the formation of arteriovenous shunts in association with loss of peripheral resistance; and (3) disturbance of oxygen transport and utilization by tissue cells.51 Another major mechanism during sepsis is the change from anticoagulant to procoagulant and the contribution of microthrombi to the disturbance of the microcirculation. Lipopolysaccharide, an important pathogen product, is recognized by pathogen recognition receptors such as toll-like receptors on cells of the innate immune system. This will lead to intracellular signaling and to the production of cytokines and other potent chemokines. Septic shock is often associated with the loss of fluid from the intravascular into the extravascular space with the potential progressive loss of circulating blood, eventually leading to a depression of cardiac output. Similarly, loss of fluid into the extravascular space can lead to life-threatening edema in the lungs, kidney, and brain of septic patients. The loss of fluid is not believed to be associated with changes in hydrostatic or osmotic pressures within the vascular compartment but rather to the breakdown of endothelial barrier function. The permeability of the vascular barrier can be modified in response to specific stimuli acting on endothelial cells. Many inflammatory agonists mediate endothelial hyperpermeability via a calcium-dependent mechanism. Multiple cascades of intracellular signaling reactions are initiated when an inflammatory agonist binds to its respective receptor expressed on the endothelial surface (e.g., thrombin binds the protease-activated receptor-1, histamine binds its receptor H1, and vascular endothelial growth factor binds its vascular endothelial growth factor receptor 2 [VEGFR-2]). This breakdown allows migration of water and macromolecules, including proteins, into the extravascular space. The pathophysiologic mechanisms proposed include the separation of tight junctions between endothelial cells, as well as cytoskeleton contraction, rather than destructive changes of endothelial cells leading to defects in the endothelium.44,52 Studies on the microcirculation of the gut have shown the development of a gap between microvascular and venous oxygen tension, suggesting enhanced shunting of the microcirculation. Defects in distributing blood to regional vascular beds or the microcirculation could be responsible for tissue hypoxia and limited oxygen extraction. Clinical evidence of decreased microvessel



CHAPTER 25

Endothelium and Endotheliopathy

225

density in the sublingual microcirculation of fluid-resuscitated septic patients is consistent with findings of decreased functional capillary flow in the gut, liver, and skeletal muscle microcirculation in animal models of sepsis. This clinical finding raises the possibility that abnormal microvascular oxygen transport develops in multiple organs despite fluid resuscitation, leading to heterogeneous microvascular dysfunction and local tissue hypoxia in severe cases of sepsis.

in the interactions with endothelial cells belong to three major families: selectins, sialomucins, and integrins. Interestingly, it is recognized that these interactions participate in tissue specificity in various vasculitic conditions. For instance, specific selectin interactions mediate cutaneous tropism in several inflammatory disorders, including graft-versus-host disease and dermatomyositis.55

Hemolytic-Uremic Syndrome

There is a strong rationale for targeting markers of endothelial cell activation as clinically informative biomarkers to improve diagnosis, prognostic evaluation, or risk stratification of critically ill patients. During sepsis, a large number of endothelial cell–active molecules are potential biomarkers for the early diagnosis. These include regulators of endothelial activation, such as vascular endothelial growth factor (VEGF), the angiopoietin pathway (Ang-1/2), adhesion molecules (intercellular adhesion molecule [ICAM], vascular cell adhesion molecule [VCAM], and E-selectin), mediators of permeability and vasomotor tone (ET-1), and mediators of coagulation (e.g., vWF).56 Elevated endothelin levels have been found in systemic and pulmonary hypertension, coronary artery disease, and heart failure, although the role of ET-1 in the pathophysiology of these conditions has been postulated but not proved.57,58 Cellular markers such as EPCs and endothelial microparticles (EMPs) are also gaining interest as biomarkers. There is a positive correlation between EPC number and survival in sepsis.59 In addition, there is a functional impairment of EPCs with decreased proliferative and migratory capacities of EPCs. These findings have led to therapeutic strategies (e.g., statins) that focus on improving EPC number and function during sepsis.60 In contrast, an elevation of EMPs is considered a marker of endothelial dysfunction in cardiovascular disease. However, the number of EMPs is positively related to survival and inversely correlated with the Sequential Organ Failure Assessment (SOFA) score in patients with sepsis. Because it is becoming increasingly clear that microparticles are more than simple markers of endothelial damage or activation, their interpretation as markers of endothelial dysfunction is less ambiguous.61 Interestingly, EPCs show a biphasic response after traumatic brain injury; after an initial decrease, they peak 7 days after the insult. Furthermore, they have been associated with an improved outcome after traumatic brain injury.62 However, the clinical utility of these biomarkers is limited by a lack of assay standardization, unknown receiver operating characteristics, and lack of validation. In addition, evidence is lacking that these biomarkers are associated with endothelial cell and vascular tone dysfunction. It remains speculative whether the use of endothelial cell biomarkers will guide therapy during critical care illness in the near future.

Thrombotic thrombocytopenic purpura (TTP) and the hemolytic uremic syndrome (HUS) are related disorders characterized clinically by microangiopathic hemolytic anemia and thrombocytopenia. Pathologically, both conditions include the development of platelet microthrombi that occlude small arterioles and capillaries. Endothelial dysfunction plays a prominent role in the pathogenesis of both disorders. HUS commonly occurs in early childhood (≈90% of cases). It often follows an episode of bloody diarrhea caused by enteropathic strains of Escherichia coli that release an exotoxin, verotoxin-1 (VT-1; see also Chapter 75). VT-1 binds with high affinity to receptors expressed in high density on renal glomerular endothelial cells. VT-1 is directly cytotoxic to endothelial cells, where it promotes neutrophil-mediated endothelial cell injury. VT-1 induces the production of TNF-a by monocytes and cells within the kidney. In turn, TNF-a, in synergy with interleukin-1, increases VT-1 receptor expression and exacerbates the sensitivity of the endothelium to toxin-mediated and antibody-mediated cytotoxicity. It also promotes vWF release and impairs fibrinolytic activity.53 There is considerable evidence to suggest that endothelial cell injury plays a role in the pathogenesis of TTP. Platelet microthrombi in TTP contain abundant vWF but little fibrinogen in contrast to those seen in disseminated intravascular coagulopathy. A subgroup of patients has been identified who suffer from chronic, relapsing TTP and whose plasma continues to contain elevated levels of unusually large vWF multimers (ULvWFs) between relapses. ULvWFs may exacerbate microvascular thrombosis through their ability to aggregate platelets at high levels of shear stress. The secretion of ULvWF by cultured endothelial cells is stimulated by many agonists, including Shiga toxin. However, elevated levels of vWF occur in other thrombotic microangiopathies; their exact role in TTP/HUS requires further study. Endothelial damage plays a pivotal role in the pathogenesis of the disease. The events that initiate TTP remain unknown. More recently, plasma from patients with TTP and HUS has been reported to induce apoptosis in microvascular endothelial cells. Interestingly, cells from dermal, renal, and cerebral origin were most susceptible, whereas pulmonary and coronary arterial cells were less susceptible.54

Vasculitic Disorders Vasculitis is a disease that targets all levels of the arterial tree, from aorta to capillaries. It also affects venules, with leukocyte infiltration and necrosis. Different forms of vasculitis attack different vessels and are classified accordingly. The inflammatory process may target vessels of any type throughout the vascular system, although distinct clinicopathologic entities preferentially involve vessels of particular sizes and locations. Small vessels anywhere in the body may be affected by focal necrotizing lesions where extravasation of leukocytes drives the inflammatory responses, resulting in vasculitis. Leukocyte adhesion molecules participating

Biomarkers of Endothelial Activation

Conclusions The endothelium can no longer be viewed as a static physical barrier that simply separates the blood from tissue. Rather, the endothelium coordinates key functions of different tissues in normal and pathophysiologic conditions. This is accomplished by the interaction of endothelial cells with circulating factors and cells and its ability to transmit biochemical and biophysical signals to surrounding tissues. A greater understanding of endothelial physiology will lead to novel therapeutic approaches in complex clinical conditions.

SECTION IV



226

Pediatric Critical Care: Cardiovascular

Key References Aird WC. Endothelial cell heterogeneity. Crit Care Med. 2003;31(suppl 4):S221-S230. Asahara T, Masuda H, Takahashi T, et al. Bone marrow origin of endothelial progenitor cells responsible for postnatal vasculogenesis in physiological and pathological neovascularization. Circ Res. 1999;85: 221-228. Asahara T, Murohara T, Sullivan A, et al. Isolation of putative progenitor endothelial cells for angiogenesis. Science. 1997;275:964-967. Chistiakov DA, Orekhov AN, Bobryshev YV. Endothelial barrier and its abnormalities in cardiovascular disease. Front Physiol. 2015;6:365. Dejana E. Endothelial adherens junctions: implications in the control of vascular permeability and angiogenesis. J Clin Invest. 1996;98: 1949-1953. Kalka C, Masuda H, Takahashi T, et al. Transplantation of ex vivo expanded endothelial progenitor cells for therapeutic neovascularization. Proc Natl Acad Sci U S A. 2000;97:3422-3427.

Kumar S, West DC, Ager A. Heterogeneity in endothelial cells from large vessels and microvessels. Differentiation. 1987;36:57-70. McCarthy SA, Kuzu I, Gatter KC, Bicknell R. Heterogeneity of the endothelial cell and its role in organ preference of tumour metastasis. Trends Pharmacol Sci. 1991;12:462-467. Reitsma S, Slaaf DW, Vink H, van Zandvoort MA, oude Egbrink MG. The endothelial glycocalyx: composition, functions, and visualization. Pflugers Arch. 2007;454:345-359. Rubanyi GM. The role of endothelium in cardiovascular homeostasis and diseases. J Cardiovasc Pharmacol. 1993;22(suppl 4):S1-S14. Sukriti S, Tauseef M, Yazbeck P, Mehta D. Mechanisms regulating endothelial permeability. Pulm Circ. 2014;4:535-551.

The full reference list for this chapter is available at ExpertConsult.com.

e1



















































































































1. Rubanyi GM. The role of endothelium in cardiovascular homeostasis and diseases. J Cardiovasc Pharmacol. 1993;22(suppl 4):S1-S14. 2. Aird WC. Endothelial cell heterogeneity. Crit Care Med. 2003;31(suppl 4): S221-S230. 3. Kumar S, West DC, Ager A. Heterogeneity in endothelial cells from large vessels and microvessels. Differentiation. 1987;36:57-70. 4. Dejana E. Endothelial adherens junctions: implications in the control of vascular permeability and angiogenesis. J Clin Invest. 1996;98:1949-1953. 5. Butcher EC, Picker LJ. Lymphocyte homing and homeostasis. Science. 1996;272:60-66. 6. Zhu DZ, Cheng CF, Pauli BU. Mediation of lung metastasis of murine melanomas by a lung-specific endothelial cell adhesion molecule. Proc Natl Acad Sci U S A. 1991;88:9568-9572. 7. McCarthy SA, Kuzu I, Gatter KC, Bicknell R. Heterogeneity of the endothelial cell and its role in organ preference of tumour metastasis. Trends Pharmacol Sci. 1991;12:462-467. 8. Asahara T, Murohara T, Sullivan A, et al. Isolation of putative progenitor endothelial cells for angiogenesis. Science. 1997;275:964-967. 9. Kalka C, Masuda H, Takahashi T, et al. Transplantation of ex vivo expanded endothelial progenitor cells for therapeutic neovascularization. Proc Natl Acad Sci U S A. 2000;97:3422-3427. 10. Asahara T, Masuda H, Takahashi T, et al. Bone marrow origin of endothelial progenitor cells responsible for postnatal vasculogenesis in physiological and pathological neovascularization. Circ Res. 1999; 85:221-228. 11. Asahara T, Takahashi T, Masuda H, et al. VEGF contributes to postnatal neovascularization by mobilizing bone marrow-derived endothelial progenitor cells. EMBO J. 1999;18:3964-3972. 12. Schachinger V, Erbs S, Elsässer A, et al. Intracoronary bone marrowderived progenitor cells in acute myocardial infarction. N Engl J Med. 2006;355:1210-1221. 13. Taniguchi E, Kin M, Torimura T, et al. Endothelial progenitor cell transplantation improves the survival following liver injury in mice. Gastroenterology. 2006;130:521-531. 14. Wang XX, Zhang FR, Shang YP, et al. Transplantation of autologous endothelial progenitor cells may be beneficial in patients with idiopathic pulmonary arterial hypertension: a pilot randomized controlled trial. J Am Coll Cardiol. 2007;49:1566-1571. 15. Bombeli T, Mueller M, Haeberli A. Anticoagulant properties of the vascular endothelium. Thromb Haemost. 1997;77:408-423. 16. Rosenberg RD, Rosenberg JS. Natural anticoagulant mechanisms. J Clin Invest. 1984;74:1-6. 17. Esmon CT, Fukudome K. Cellular regulation of the protein C pathway. Semin Cell Biol. 1995;6:259-268. 18. Esmon NL. Thrombomodulin. Semin Thromb Hemost. 1987;13:454-463. 19. Majerus PW. Arachidonate metabolism in vascular disorders. J Clin Invest. 1983;72:1521-1525. 20. Levin EG, Santell L, Osborn KG. The expression of endothelial tissue plasminogen activator in vivo: a function defined by vessel size and anatomic location. J Cell Sci. 1997;110(Pt 2):139-148. 21. Furchgott RF, Zawadzki JV. The obligatory role of endothelial cells in the relaxation of arterial smooth muscle by acetylcholine. Nature. 1980;288:373-376. 22. Rapaport SI, Rao LV. The tissue factor pathway: how it has become a “prima ballerina.” Thromb Haemost. 1995;74:7-17. 23. Garland CJ, Plane F, Kemp BK, Cocks TM. Endothelium-dependent hyperpolarization: a role in the control of vascular tone. Trends Pharmacol Sci. 1995;16:23-30. 24. Stamler JS, Singel DJ, Loscalzo J. Biochemistry of nitric oxide and its redox-activated forms. Science. 1992;258:1898-1902. 25. Forstermann U. Biochemistry and molecular biology of nitric oxide synthases. Arzneimittelforschung. 1994;44:402-407. 26. Forstermann U, Closs EI, Pollock JS, et al. Nitric oxide synthase isozymes. Characterization, purification, molecular cloning, and functions. Hypertension. 1994;23(6 Pt 2):1121-1131.

27. Moncada S, Palmer RM, Higgs EA. Nitric oxide: physiology, pathophysiology, and pharmacology. Pharmacol Rev. 1991;43:109-142. 28. Rapoport RM, Murad F. Agonist-induced endothelium-dependent relaxation in rat thoracic aorta may be mediated through cGMP. Circ Res. 1983;52:352-357. 29. Bolotina VM, Najibi S, Palacino JJ, Pagano PJ, Cohen RA. Nitric oxide directly activates calcium-dependent potassium channels in vascular smooth muscle. Nature. 1994;368:850-853. 30. Lincoln TM, Komalavilas P, Cornwell TL. Pleiotropic regulation of vascular smooth muscle tone by cyclic GMP-dependent protein kinase. Hypertension. 1994;23(6 Pt 2):1141-1147. 31. Stamler JS, Loh E, Roddy MA, Currie KE, Creager MA. Nitric oxide regulates basal systemic and pulmonary vascular resistance in healthy humans. Circulation. 1994;89:2035-2040. 32. Moncada S, Vane JR. The role of prostacyclin in vascular tissue. Fed Proc. 1979;38:66-71. 33. Kukovetz WR, Holzmann S, Wurm A, Pöch G. Prostacyclin increases cAMP in coronary arteries. J Cyclic Nucleotide Res. 1979;5: 469-476. 34. Delpy E, Coste H, Gouville AC. Effects of cyclic GMP elevation on isoprenaline-induced increase in cyclic AMP and relaxation in rat aortic smooth muscle: role of phosphodiesterase 3. Br J Pharmacol. 1996;119:471-478. 35. Feletou M, Vanhoutte PM. Endothelium-derived hyperpolarizing factor. Clin Exp Pharmacol Physiol. 1996;23:1082-1090. 36. Kubes P, Kerfoot SM. Leukocyte recruitment in the microcirculation: the rolling paradigm revisited. News Physiol Sci. 2001;16:76-80. 37. McEver RP, Moore KL, Cummings RD. Leukocyte trafficking mediated by selectin-carbohydrate interactions. J Biol Chem. 1995;270: 11025-11028. 38. Rubanyi GM, Vanhoutte PM. Superoxide anions and hyperoxia inactivate endothelium-derived relaxing factor. Am J Physiol. 1986; 250(5 Pt 2):H822-H827. 39. Coleman RA, Smith WL, Narumiya S. International Union of Pharmacology classification of prostanoid receptors: properties, distribution, and structure of the receptors and their subtypes. Pharmacol Rev. 1994;46:205-229. 40. Halushka PV, Mais DE, Mayeux PR, Morinelli TA. Thromboxane, prostaglandin and leukotriene receptors. Annu Rev Pharmacol Toxicol. 1989;29:213-239. 41. Reitsma S, Slaaf DW, Vink H, van Zandvoort MA, oude Egbrink MG. The endothelial glycocalyx: composition, functions, and visualization. Pflugers Arch. 2007;454:345-359. 42. Springer TA. Traffic signals on endothelium for lymphocyte recirculation and leukocyte emigration. Annu Rev Physiol. 1995;57:827-872. 43. Schafer AI. Vascular endothelium: in defense of blood fluidity. J Clin Invest. 1997;99:1143-1144. 44. Roth GJ. Platelets and blood vessels: the adhesion event. Immunol Today. 1992;13:100-105. 45. Sukriti S, Tauseef M, Yazbeck P, Mehta D. Mechanisms regulating endothelial permeability. Pulm Circ. 2014;4:535-551. 46. Chistiakov DA, Orekhov AN, Bobryshev YV. Endothelial barrier and its abnormalities in cardiovascular disease. Front Physiol. 2015; 6:365. 47. Grace PA, Mathie RT. Ischemia-Reperfusion Injury. London: Blackwell Science; 1999. 48. Neary P, Redmond HP. Ischemia-reperfusion injury and the systemic inflammatory response syndrome. In: Grace PA, Mathie RT, eds. Ischemia-Reperfusion Injury. London: Blackwell Science; 1999:123-136. 49. Orfanos SE, Mavrommati I, Korovesi I, Roussos C. Pulmonary endothelium in acute lung injury: from basic science to the critically ill. Intensive Care Med. 2004;30:1702-1714. 50. Lehr HA, Arfors KE. Mechanisms of tissue damage by leukocytes. Curr Opin Hematol. 1994;1:92-99. 51. DÍaz NL, Finol HJ, Torres SH, Zambrano CI, Adjounian H. Histochemical and ultrastructural study of skeletal muscle in patients with sepsis and multiple organ failure syndrome (MOFS). Histol Histopathol. 1998;13:121-128.

References

e2







59.





60.





61.



62.























52. Li H, Forstermann U. Nitric oxide in the pathogenesis of vascular disease. J Pathol. 2000;190:244-254. 53. Moake JL. Haemolytic-uraemic syndrome: basic science. Lancet. 1994;343:393-397. 54. Mitra D, Jaffe EA, Weksler B, Hajjar KA, Soderland C, Laurence J. Thrombotic thrombocytopenic purpura and sporadic hemolyticuremic syndrome plasmas induce apoptosis in restricted lineages of human microvascular endothelial cells. Blood. 1997;89:1224-1234. 55. Savage CO. The evolving pathogenesis of systemic vasculitis. Clin Med (Lond). 2002;2:458-464. 56. Xing K, Murthy S, Liles WC, Singh JM. Clinical utility of biomarkers of endothelial activation in sepsis—a systematic review. Crit Care. 2012;16:R7. 57. Masaki T. The discovery, the present state, and the future prospects of endothelin. J Cardiovasc Pharmacol. 1989;13(suppl 5):S1-S4. 58. Yanagisawa M, Kurihara H, Kimura S, Goto K, Masaki T. A novel peptide vasoconstrictor, endothelin, is produced by vascular endothelium

and modulates smooth muscle Ca21 channels. J Hypertens Suppl. 1988;6:S188-S191. Rafat N, Hanusch C, Brinkkoetter PT, et al. Increased circulating endothelial progenitor cells in septic patients: correlation with survival. Crit Care Med. 2007;35:1677-1684. Darwish NI, Liles WC. Emerging therapeutic strategies to prevent infection-related microvascular endothelial activation and dysfunction. Virulence. 2013;4:572-582. Soriano AO, Jy W, Chirinos JA, et al. Levels of endothelial and platelet microparticles and their interactions with leukocytes negatively correlate with organ dysfunction and predict mortality in severe sepsis. Crit Care Med. 2005;33:2540-2546. Liu L, Wei H, Chen F, Wang J, Dong JF, Zhang J. Endothelial progenitor cells correlate with clinical outcome of traumatic brain injury. Crit Care Med. 2011;39:1760-1765.

e3

Abstract: Because of their location, endothelial cells have the ability to interact with blood components, such as flow, soluble factors, and other cells. Endothelial cells integrate these signals into a cohesive regulation of vascular responses. The endothelium controls the vascular tone of the underlying smooth muscle cells through the production of vasodilator and vasoconstrictor mediators.

Endothelial cell activation in response to inflammation changes endothelial cellular physiology and alters vascular function. Key words: Endothelial cells, smooth muscle cells, vascular tone, coagulation, vasodilators, vasoconstrictors, leukocytes, vascular permeability, inflammation

10 26

Chapter Title Principles of Invasive Cardiovascular Monitoring

CHAPTER AUTHOR

MATTHEW R. TIMLIN AND KENNETH A. SCHENKMAN

PEARLS • •

To gain basic knowledge of the development of the eye. To develop essential understanding how abnormalities at P Evarious A R L Sstages of development can arrest or hamper normal formation of the ocular structures and visual pathways. • Hemodynamic monitoring refers to measurement of the functional characteristics of the heart and circulatory system that affect the perfusion of tissues with oxygenated blood. • Hemodynamic monitoring can be performed invasively or noninvasively and can be used for diagnosis, surveillance, or titration of therapy. • The central venous waveform is composed of three waves (a, c, and v) and two wave descents (x and y).



Role of Invasive Hemodynamic Monitoring

and pulmonary artery catheters (PACs). Invasive hemodynamic monitoring can provide the skilled intensivist with a plethora of valuable information but should still be integrated with all patient data rather than viewed in isolation. Successful use of invasive hemodynamic measurements necessitates skills to obtain these measures safely with attention to the risks imposed on the patient. As with any technology, the use of invasive hemodynamic monitoring is in evolution, and it is incumbent on the clinician to be familiar with developments as they arise. This chapter aims to be a practical guide to the use of hemodynamic monitoring in the PICU. It reviews general principles of measurement and discusses the three main types of invasive hemodynamic monitoring: CVC, arterial catheter, and PAC. It addresses the indications and controversies, interpretation of waveforms, and potential complications and also reviews cardiac output (CO) monitoring and calculation of oxygen consumption and delivery. New techniques coupling invasive monitoring with noninvasive devices are also discussed. The specific techniques for gaining access to make these measurements are detailed in Chapter 14, which covers invasive procedures.









• •









Since William Harvey’s observation in the early 1600s that the heart pumps blood in a continuous circuit, the function of the circulatory system has been the subject of intense scrutiny. Hemodynamic monitoring refers to measurement of the functional characteristics of the heart and circulatory system that affect the perfusion of tissues with oxygenated blood in order to maintain homeostasis and to remove byproducts of metabolism. Several different types of invasive hemodynamic monitoring can be used concurrently to guide management. The goal of hemodynamic monitoring is to provide accurate diagnoses and to guide additional interventions to deliver improved care to the critically ill patient. In his 1733 report “Statical essays: containing haemastaticks; or, an account of some hydraulick and hydrostatical experiments made on the blood and blood-vessels of animals,” Hales1 described early experiments in horses in which he used tubular devices inserted directly into arteries to measure intravascular pressures. Fig. 26.1 depicts Hales and an assistant in the process of these early experiments. This figure also illustrates a simple method for inferring arterial versus venous placement of a vascular catheter, which can also give a quick bedside estimate of central venous pressure. Frequently in the pediatric intensive care unit (PICU), noninvasive assessments of hemodynamics are supplemented by invasive hemodynamic measures that require entrance into the intravascular space. Such invasive hemodynamic measurements include placement of central venous catheters (CVCs), arterial catheters,

• •





To acquire adequate information about normal anatomy of the eye and related structures and develop a strong foundation for the understanding of common ocular problems and their consequences. The arterial waveform has three components: rapid upstroke, dicrotic notch, and runoff. Pulse pressure variation has excellent specificity as an indicator of fluid responsiveness in many critically ill patients. Cardiac output can be calculated using the Fick method or measured directly via thermodilution. A pulmonary artery catheter can be used to measure cardiac output and indices of oxygen delivery and extraction.

Indications for Invasive Hemodynamic Measurements The three main indications for invasive hemodynamic monitoring are diagnosis, surveillance, and titration of therapy. Diagnosis may include the differentiation of septic shock (through assessment of 227

228

SECTION IV



Pediatric Critical Care: Cardiovascular

Harmonic amplitudes

1-Fundamental

2

3

4

6

5

Pressure

1+2+3+4+5+6

Average pressure

•  Fig. 26.2

Carotid artery pressure Time

​Fourier series representation of an arterial pressure tracing. Bottom, High-fidelity carotid artery pressure tracing and the sum of the first six harmonics of its Fourier series representation. Despite the few terms used in the synthesis, the close fit of the two curves is evident. Top, Individual harmonic components labeled with their harmonic number. (From Cobbold RSC. Transducers for Biomedical Measurements: Principles and Applications. New York: John Wiley & Sons; 1974.)

•  Fig. 26.1

​ linician and an assistant measuring the blood pressure of a C horse. (From Pickering G. Systemic arterial hypertension. In: Fishman AP, Dickinson WR, eds. Circulation of the Blood: Men and Ideas. New York: Oxford University Press; 1964.)  

factors such as diminished right heart filling pressures or preload and decreased systemic vascular resistance) from cardiogenic shock (characterized by elevated left heart pressures and afterload). Surveillance implies observation over time. The purpose of surveillance may be to assess the stability of a patient at risk for adverse changes or to determine the response to therapy. Invasive measurements performed for diagnostic purposes often are continued for surveillance. Titration of therapy is often based on information gleaned from invasive measurements.

Principles of Measurement Intensive care clinicians rely on a wide variety of measurement systems to assess patient clinical status and response to therapy. However, not all clinicians have a good understanding of how physiologic variables are measured and, consequently, may not be able to troubleshoot monitoring systems or recognize when information obtained is inaccurate. A detailed discussion of monitoring is beyond the scope of this chapter, but a basic understanding of the principles of measurement is helpful in deciding which measurements to trust and how to assess a monitoring system for accuracy. Detailed descriptions of monitoring systems are provided elsewhere.2–4



Signal Analysis Measurements generally are made directly by comparison with known standards or indirectly by use of a calibration system. Determination of length or weight usually is made by direct comparison with a standard ruler or standard mass. Most invasive measurements in the ICU are made indirectly, thereby requiring use of a calibration system. Thus, understanding the basis for calibration of a system is important to determine the validity of the measurement. Measurement systems detect and transform signals so that they can be presented in an interpretable way to the user. Signals can be characterized as static or dynamic. Slowly changing signals, such as body temperature, can be thought of as static. Hemodynamic measurements change from moment to moment and thus are dynamic. Physiologic signals may be periodic; for example, arterial pressure is periodic because it varies with the cardiac cycle. Complex periodic signals, such as an arterial pressure waveform, can be described mathematically as the sum of a series of simpler waveforms, called a Fourier series. Alternatively, the arterial tracing can be thought of as a sum of simpler waveforms, sine waves, and cosine waves. Fig. 26.2 depicts an arterial pressure waveform as the sum of the first six terms in the Fourier series. The sum of the first six terms in the series forms a waveform similar to the original tracing. Adding terms from the Fourier series, or higher harmonics, results in an increasingly better representation of the actual waveform. In general, to reproduce a pressure tracing without loss of significant characteristics for clinical use, the measurement system must have an accurate frequency response to approximately 10 times the fundamental frequency (first 10 harmonics).



CHAPTER 26 Principles of Invasive Cardiovascular Monitoring

229

The sampling rate of a measurement system determines how often a physiologic value is measured. For body temperature, sampling every few minutes might be sufficient, but for arterial pressure measurement, a higher rate is necessary. This principle may seem obvious; however, as an example of the importance of sampling rate, consider the number of points needed to define a circle. If three equidistant points are placed on a circle, a triangle is described, not a circle. Similarly, four points describe a square. If the number of points (sampling rate) is increased, the circle is described more completely. For a sine wave, the minimum frequency of sampling needed to preserve the waveform is twice the frequency. This mathematical minimum is known as the Nyquist frequency.4 For complex waveforms, such as arterial pressure tracings, the sampling rate must be at least twice the highest frequency component in the waveform.

assumption allows a system to be calibrated under two conditions, with the rest of the values falling on the line defined by those two points. Actual nonlinearity of the system adversely affects the measurements. Calibration is a process in which the reading, or output of a device, is adjusted to match a known input value. For example, an electronic pressure transducer may be calibrated against a mercury manometer. If the input to the device is zero, the output should be adjusted so that the reading also is set to zero. This “zeroing” reduces any baseline offset, thus reducing systematic errors in subsequent readings. The system then is calibrated to a nonzero value, for example, 100 mm Hg pressure, and the system gain is adjusted to read this value as well.

Measurement Systems

The ability of a measurement system to accurately measure an oscillating signal, such as arterial blood pressure, is dependent on the system’s frequency response. The system can either overestimate or underestimate the true amplitude of a signal. If the system is overdamped, the value reported underestimates the amplitude, and waveform characteristics may be lost. Resonance in the system may result in overestimation of the amplitude. Measurement of arterial systolic pressure—the amplitude of the arterial waveform—may be inaccurate because of overdamping, and important waveform characteristics may be lost if the frequency response of the measurement system is poor.

Hemodynamic monitoring in the clinical setting usually uses a fluid-coupled system in which changes in pressure are transmitted via a column of (ideally incompressible) fluid in an (ideally incompressible) tube to a mechanical transducer. The mechanical transducer, usually a displaceable screen diaphragm, converts a change in pressure to an electrical signal, which can be processed and displayed. In laboratory settings, vascular pressures can be measured by a transducer at the point of interest rather than remotely, as in the clinical setting. Measuring pressure at the point of interest—directly in the aorta, for example—decreases loss of signal integrity because of the measurement system. Most clinical pressure measuring systems have sufficient fidelity for clinical purposes. However, compliance, resistance, or impedance in the pressure tubing can result in damping or alteration of the recorded signal. Care should be taken to ensure that the length of the tubing is not overly long, as this can lead to resonance of the physiologic signal. Similarly, the narrower the tubing used in the system, the more resonant it will be. The presence of bubbles in the fluid can further damp the recorded signal.

Errors in Measurement The ideal measurement system determines the actual or “true” value for the measured variable. However, determination of a true value may be difficult. Every measurement system is subject to various errors. Errors in measurement can be classified as either systematic or random. Systematic errors occur in a predictable manner and are reproduced with repeated measures. Bias in a measurement system—for example, a baseline offset—results in a systematic error. Random errors are unpredictable and do not recur predictably with repeated measures. Accuracy of a measurement is defined by the difference between the measured and true values, divided by the true value. Precision is defined by the reproducibility of the measurement; thus, a more precise system yields more similar values for repeated measures under the same conditions than does a less precise system. Imprecision can be thought of as a representation of random errors, whereas bias can be thought of as a representation of systematic errors.

Frequency Response

Impedance Impedance is the ratio of the change in blood flow along a vessel to the change in the pressure in the vessel. Impedance has both resistive and reactive components. In a pulsatile system such as the cardiovascular system, resistance alone does not fully describe the impediment or impedance to forward flow of blood. The caliber, length, and arrangement of the blood vessels and the mechanical properties of the blood (such as its rheology and viscosity) determine resistance in the blood vessels. Reactance includes compliance of the vessels and inertia of the blood and thus is a dynamic component of impedance. This is important because the pulsatile nature of the cardiovascular system is dynamic. When blood is propelled through a vessel at a branch point, a reflected pressure wave back toward the heart increases the impedance of the system. The major sites of wave reflection from vessel branching are from vessels approximately 1 mm in diameter.2 Thus, these small vessels contribute significantly to overall impedance. Fig. 26.3 shows the relationships between pressure and flow velocity with distance along the length of the aorta. Because blood pressure increases with distance from the heart and flow velocity decreases with distance, the impedance increases toward the peripheral vasculature. Hemodynamic measuring systems are essentially physical extensions of the vascular system; thus, the configuration and characteristics of the tubing and transducer system can alter the overall effect of impedance.

Invasive Techniques

Calibration

Central Venous Catheters

Many measurement systems are linear, that is, based on an assumption that the relationship between the inputs and outputs from a measurement device can be fitted to a straight line. This

Indications Indications for CVC placement in pediatric patients include assessment of central venous pressure (CVP), monitoring of large

230

SECTION IV



Pediatric Critical Care: Cardiovascular

fluid shifts between the intravascular and extravascular spaces, infusion of vasoactive substances, monitoring central venous oxygen saturation, and infusion of hyperosmolar fluids and/or irritants.5–7

Velocity (cm/sec)

Pressure (cm H2O)

Interpretation of Waveforms CVP is ideally a measure of right atrial pressure, although it may be measured in the inferior or superior vena cava (SVC). It is a measure of preload—the force or load on the right ventricle during relaxation or filling. CVP is measured at the end of diastole, just prior to ejection. Final filling of the right ventricle occurs at the end of atrial contraction. When the tricuspid valve is open during diastole, the right atrium and right ventricle form a continuous column; therefore, right atrial pressure reflects right ventricular end-diastolic pressure. CVP is used to measure filling pressure or preload and is an indicator of volume status; however, it is influenced by a variety of physiologic phenomena. It is commonly used in patients with hypovolemic or septic shock in whom volume resuscitation is desirable prior to institution of vasopressor therapy. In patients with decreased right ventricular function or pulmonary hypertension, an increased CVP well beyond normal limits may be observed and further fluid resuscitation may contribute to the development of congestive heart failure. Increases in positive end-expiratory pressure can decrease preload despite an increased CVP. Finally, increases in extrathoracic pressure, such as that caused by increased abdominal distension, can increase CVP.

It is important to keep in mind that CVP as an absolute number does not necessarily indicate fluid status due to its many physiologic determinants (stressed venous volume, venous compliance, venous resistance, right heart function, and sympathetic tone, among others). At the extremes (i.e., ,6–8 or .12–15 mm Hg), CVP has satisfactory performance as an indicator of fluid responsiveness.8 Additionally, the change in CVP (or its staying the same) during therapeutic interventions or diagnostic maneuvers has been shown to perform better9 than the absolute number. As with most widely available measurements, the widespread availability of CVP at the bedside has many limitations but, when integrated into other data, frequently provides important information. The CVP waveform is divided into three components: a, c, and v waves (Fig. 26.4). Each component can be correlated with a specific portion of the electrocardiogram (ECG) tracing. The a wave occurs with atrial contraction and is seen after the P wave of the electrocardiogram during the PR interval. Thus, the mean value of the a wave approximates right ventricular end-diastolic pressure. Cannon a waves (Fig. 26.5), which are enlarged a waves seen when the right atrium is ejecting against a closed tricuspid valve, may be seen when atrioventricular discordance occurs (i.e., during junctional ectopic tachycardia, ventricular tachycardia, or heart

260 220 180 140

Ascending Thoracic Abdominal Abdominal aorta aorta aorta aorta middle distal Flow velocity

90 50

Femoral A

X

•  Fig. 26.4

0 30

• Fig. 26.3

​Pressure pulses and flow velocity at various points in the systemic arterial circulation. Data were obtained from dogs and are similar to measurements made in humans. The data indicate that both peak and pulse pressure increase with distance from the heart, whereas oscillation in flow velocity shows a progressive decrease. Consequently, impedance (discussed in the text) must increase toward the periphery.  

• Fig. 26.5

V

C

Y

A C

V X

Y

​Central venous pressure (A) tracing with corresponding electrocardiogram (ECG). The a wave is produced by atrial contraction and occurs after the P wave of the ECG during the PR interval. The c wave (C) is produced by closure of the tricuspid valve and takes place early in systole at the end of the QRS complex in the RST junction. The v wave (V) is caused by rapid filling of the right atrium late in systole before opening of the tricuspid valve and is seen between the T and P waves of the ECG. The x descent (X) reflects the decrease in pressure in the right atrium after the a wave as the tricuspid valve is pulled away from the right atrium by the right ventricle as it contracts during systole. The y descent (Y) is the decrease in right atrial pressure that occurs after the v wave as the tricuspid valve opens and blood moves from the right atrium into the right ventricle.  

​Cannon a waves are enlarged a waves seen when the right atrium is ejecting against a closed tricuspid valve. These waves are typically seen when atrioventricular discordance occurs, such as during junctional ectopic or ventricular tachycardia or heart block.  



CHAPTER 26 Principles of Invasive Cardiovascular Monitoring

block). The c wave occurs in early systole with closure of the tricuspid valve and is seen at the end of the QRS complex in the RST junction. The v wave occurs during filling of the right atrium in late systole before opening of the tricuspid valve and is seen between the T and P waves of the ECG. The v wave is increased in the setting of tricuspid regurgitation. The x descent is the decrease in pressure after the a wave, reflecting atrial relaxation. The y descent is the decrease in pressure that occurs after the v wave as the tricuspid valve opens and passive filling of the right ventricle occurs.

Mixed Venous Oxygen Saturation Mixed venous oxygen saturation (Svo2) can be measured intermittently by blood sampling from a CVC or continuously by using a specially designed CVC. Such catheters typically have two to three lumens and have the same capabilities of standard CVCs, with the additional potential for spectrophotometric monitoring. The Svo2 catheters use reflection spectrophotometry and are able to read hemoglobin oxygen saturation continuously. The reflected light is dependent on the oxygenated and deoxygenated hemoglobin concentration in the circulating blood.10 Svo2 measurement may be used to inform the practitioner of the relationship between oxygen delivery and consumption and is often used as a surrogate for cardiac index (CI). Rivers et al.11 showed that when continuous Svo2 monitoring was used to guide resuscitation and hemodynamic support in patients with severe sepsis and septic shock, survival rates improved. Guidelines set forth by the American College of Critical Care Medicine/Pediatric Advanced Life Support have recommended goal-directed therapy with a target Svo2 of 70% or more in children and adolescents who are in septic shock.7 Rivers’ findings have subsequently not been replicated, and continuous monitoring of Svo2 has not been shown to improve patient outcomes in large wellconducted studies.12–14 However, since Rivers’ initial report on goal-directed therapy, there have been sustained improvements in sepsis mortality15 that have coincided with an increase in awareness of the importance of monitoring end-organ perfusion. This has led to the development of many new noninvasive techniques for perfusion monitoring.16 When placing CVCs for monitoring, the catheter should be placed such that the tip taking measurements is at the cavoatrial junction. Svo2 measurements obtained from the inferior vena cava exhibit greater variability because of fluctuations in splanchnic oxygen utilization and thus are less reliable. Svo2 measurements from the right atrium contain coronary sinus blood and are more desaturated because of the high oxygen extraction rate of the myocardium. Studies in critically ill children have evaluated Svo2 measurements obtained in the pulmonary artery and SVC. Concordance analysis showed appropriate agreement in the measurements between these two sampling sites.17 This finding has clinical importance because the use of PACs has declined and CVC use has been increasing.18,19

Arterial Pressure Catheters Indications The transition to direct monitoring of arterial blood pressure dates back to the mid-1950s when two separate studies compared invasive arterial measurements and noninvasive or cuff measurements in healthy adults.20,21 Van Bergen et al.21 noted a frequent difference between direct and indirect measurements, with indirect measurements increasingly lower than direct measurements as

231

the systemic blood pressure increased. The greatest disparity was found in young hypertensive patients. Similarly, Cohn and Luria22 observed that invasive arterial pressures were significantly greater than cuff pressures and emphasized the importance of direct measurements of systemic arterial pressure when caring for patients with hypotension and shock. Continuous direct monitoring of arterial blood pressure should be considered when treating patients who require more than minimal vasopressor therapy. Indications for arterial catheterization include continuous monitoring of systemic arterial blood pressure, frequent blood sampling, and withdrawal of blood during exchange transfusions.23 The procedural information regarding site selection and procedural techniques for placing arterial lines is covered elsewhere in this text.

Interpretation of Waveforms The arterial waveform has three main components: (1) a rapid upstroke and downslope that correlates with systolic ejection, (2) a dicrotic notch that correlates with closure of the aortic valve, and (3) a smooth runoff that correlates with diastole. The dicrotic notch or incisura is decreased in situations of hyperdynamic CO in which left ventricular output and stroke volume (SV) are increased, pulse pressure is widened, and diastolic blood pressure (DBP) is increased (e.g., surgical systemic-to-pulmonary shunts, patent ductus arteriosus, aortic regurgitation, anemia, fever, sepsis, hypovolemia, exercise). Conversely, cardiac tamponade and severe aortic stenosis can narrow the pulse pressure and are associated with a deflection (anacrotic notch) on the ascending limb of the waveform.24 Systolic pressures measured in the periphery typically are greater than those measured more centrally because of pulse amplification of pressure waves reflected back from arterial branch points24,25 (see Fig. 26.3). More peripheral sites, such as the radial artery, have greater systolic blood pressure (SBP) and lower DBP than more central sites and thus taller and narrower waveforms with greater pulse pressures (difference between SBP and DBP). Important to note is that the mean arterial pressure (MAP, in Eq. 26.1) represents the area under the waveform curve. It has traditionally been felt that the overall magnitude of the reading remains the same regardless of the location of the tracing. However, some evidence exists that, among critically ill adults, MAP readings from radial artery catheters are consistently lower than those from femoral artery catheters by up to approximately 5 mm Hg.26–29 

MAP 2 DBP 1 (SBP 2 DBP)/3

Eq. 26.1

The appearance of the arterial waveform also provides clinical information to the observer. Pulsus alternans (Fig. 26.6A) is observed when regular variations occur in the amplitude of the peak systolic pressure during sinus rhythm. This phenomenon can be seen in patients with severe left ventricular failure. Pulsus paradoxus (Fig. 26.6B) demonstrates an exaggerated decrease in the systolic pressure (.10 mm Hg) during the inspiratory phase of the respiratory cycle. This phenomenon can be observed in patients with pericarditis, pulmonary hyperinflation, and decreased intravascular volume. The physiology that leads to pulsus paradoxus in pathologic states may also be used to assess for fluid responsiveness in intubated patients by measuring their pulse pressure variation (PPV). A number of studies have shown that, in intubated patients without spontaneous breathing, substantial (.12%–13%) PPV between cardiac cycles during insufflation and expiration have excellent performance in predicting fluid responsiveness.30–33 Many modern

232

SECTION IV



Pediatric Critical Care: Cardiovascular

monitors can now analyze PPV automatically and report the number to the clinician directly.34 The physiology of cardiopulmonary interactions that underlie this is discussed at length in Chapter 32. Briefly, during insufflation of a sufficiently large tidal volume (at least 8 mL/kg ideal body weight) right ventricular preload falls dramatically, resulting in a decreased left ventricular preload after a delay of a few cardiac cycles (Fig. 26.7).

Pulmonary Artery Catheters History and Controversy In 1847, Claude Bernard described a method for measuring intracardiac pressures in animals by inserting a glass tube in the Pulsus alternans

A Pulsus paradoxus

Inspiration

B

•  Fig. 26.6

​(A) Pulsus alternans occurs with left ventricular failure and is characterized by regular variations in the peak amplitude of systolic pressure during sinus rhythm. (B) Pulsus paradoxus is characterized by an exaggerated decrease in the systolic blood pressure during inhalation. It is commonly seen in conditions marked by great swings in intrathoracic pressure, such as in status asthmaticus, or when there are changes in cardiac function, as in pericarditis. In severe hypovolemia, pulsus paradoxus also can be observed as a result of a decrease in preload. (Modified from McGee S. Evidence-Based Physical Diagnosis. Philadelphia: Elsevier; 2018.)  



RV Preload Pleural pressure

RV Afterload

Transpulmonary pressure

LV Afterload LV Preload

RV Ejection

heart.35 However, the true pioneers of cardiac catheterization were two other Frenchmen: Jean Baptiste Auguste Chaveau, at that time a veterinarian interested in the relationship between the dynamic motion of the heart and heart sounds, and Etienne-Jules Marey, a physician interested in the physiology of the circulation. In the early 1860s, using techniques adapted from Bernard’s work, Chaveau and Marey inserted a double-lumen catheter into the right atrium of a horse to record phasic changes in intracardiac pressures as they simultaneously recorded the apical impulse.35–38 Right heart catheterization was not considered a safe practice in humans until the early 20th century. In 1929, Werner Forssman, a German surgeon, secretly performed a right heart catheterization on himself. In direct contradiction to his supervisor’s instructions, Forssman inserted a urinary catheter into his own left antecubital vein and then the remainder of the way to his right atrium under fluoroscopic guidance with the aid of a mirror. Forssman performed right heart catheterizations on himself a total of nine additional times without adverse consequences and expanded his findings by demonstrating the feasibility of injecting contrast dye during the procedure.39,40 In the early 1940s, Andres Cournand and Dickinson Richards, working at Bellevue Hospital in New York, continued Forssman’s work. They performed right heart catheterization in healthy humans and in those with cardiac failure.40–43 In 1956, Forssman, Cournand, and Richards won the Nobel Prize in Physiology or Medicine for their discoveries relating to heart catheterization and pathologic changes in the circulatory system. They were the first investigators to measure pulmonary capillary wedge pressures using cardiac catheterization.44,45 In 1953, Lategola and Rahn46 performed experiments in dogs in which they were the first to use a self-guiding balloon-tipped catheter to measure pressures in the pulmonary circulation. Seventeen years later, Swan et al.47 at the University of California, Los Angeles, used this technique to assess right heart pressures in

LV Preload

LV Ejection

LV Ejection

PPMax PPMin

•  Fig 26.7

​Mechanisms of heart-lung interactions that lead to pulse pressure variation. Top line, Airway pressure tracing. Bottom line, Arterial pressure tracing. LV, Left ventricle; PP, pulse pressure; RV, right ventricle. (Figure based on data from Teboul JL, Monnet X, Chemla D, Michard F. Arterial pulse pressure variation with mechanical ventilation. Am J Respir Crit Care Med. 2019;199:22–31.)  



CHAPTER 26 Principles of Invasive Cardiovascular Monitoring

humans. In doing so, they brought this methodology to the bedside, where it is still used today. In the 50 years since Swan and Ganz added balloon flotation and thermodilution to the PAC, its use has been controversial. The literature is rife with studies reporting salutary effects of placing them, studies showing no effect, and studies showing harm.48–53 Due to this controversy and the significant risks attendant to its use, the PAC has fallen out of favor; its routine use today is rare outside of specific clinical scenarios discussed later. With regard to pediatric patients, the Pulmonary Artery Catheter Consensus Conference, based on a consensus of expert opinions, concluded that the PAC was useful for clarifying cardiopulmonary physiology in critically ill infants and children with pulmonary hypertension; shock refractory to fluid resuscitation and/or low-to-moderate doses of vasoactive medications; severe respiratory failure requiring high mean airway pressures; and, on rare occasions, multiple organ failure. They found no data indicating that PAC use increases mortality in children; however, they also failed to find any controlled trials that demonstrated a benefit of PAC use. The panel recommended PAC use for selected patients and called for randomized controlled trials, a registry of PAC use, and studies to assess the impact of PAC use on cost and duration of ICU/hospital stay.54 A further review of current studies55 demonstrated level B and level C evidence for most indications.

Indications Although controversial, current indications for PAC use in children include septic shock unresponsive to fluid resuscitation and vasopressor support,56–58 refractory shock following severe burn injuries,59 congenital heart disease (CHD),58 pulmonary hypertension,60,61 multiple organ failure,62 liver transplantation,63 and respiratory failure requiring high mean airway pressures.58,64 Capabilities of PACs include determination of CVP, pulmonary artery pressure, and pulmonary artery occlusion pressure (PAOP), also referred to as pulmonary capillary wedge pressure. PAOP is a measurement of left atrial pressure and left ventricular end-diastolic pressure (when the mitral valve is open). PACs are also used to assess CO, Svo2, oxygen delivery (Do2) and consumption (Vo2), and pulmonary vascular resistance (PVR) and systemic vascular resistance (SVR). The PAC is the only bedside tool that can examine the function of the right and left ventricles separately aside from echocardiography, which provides significantly less precise assessments. PACs are used to establish diagnoses, guide response to therapy, and assess the determinants of oxygen delivery. PACs are especially helpful in cases of discordant ventricular function. One of the most common uses of the PAC in infants and children is monitoring pulmonary pressures during and after repair of CHD. In addition to flow-directed, balloon-tipped PACs, transthoracic left atrial catheters are often used in these patients.65 Use of PACs has altered the management of children with CHD by identifying residual anatomic defects and diagnosing pulmonary hypertensive crisis.66,67 The ability to monitor PAP provides the means to titrate response to inhaled nitric oxide and other pulmonary vasodilators.68,69 The lack of response to inhaled nitric oxide may suggest a residual structural anomaly in postoperative patients and indicate the need for interventional cardiac catheterization and/or surgical repair.69 In addition to monitoring for pulmonary hypertensive crisis, PACs can be used to assess the effects of changes in concentration of inspired CO2 on mean pulmonary artery pressure (MPAP), pulmonary vascular resistance index (PVRI), and CI.70

233

Monitoring Techniques with the Pulmonary Artery Catheter The functional features of the PAC are several. Its use as a method of indicator thermodilution, measurement of cardiac output, monitoring, and blood sampling is well documented. Fig. 26.8 demonstrates the expected waveforms as the catheter passes through the cardiovascular system. Technical concepts are important for understanding the calculations needed for optimum use. See Table 26.1 for a summary of the hemodynamic parameters that can be derived from a PAC.

Catheter Placement PACs typically contain the following ports (see Fig. 26.8). The proximal port is located 15 cm from the tip in 5 Fr catheters and 30 cm from the tip in larger catheters. It opens into or near the right atrium. The proximal port provides access for infusion of fluid or drugs, injection of cold saline solution as indicator (thermodilution method), CVP monitoring, and blood sampling. In infants or small children, PAC placement may result in improper location of the proximal port before the right atrium such that the port lies inside the sheath or outside the body. Therefore, it is essential to verify not only the placement of the distal tip in the pulmonary artery but also the location of the proximal port. The distal port opens at the tip of the catheter. It is used for monitoring PAP and PAOP, blood sampling of mixed venous blood gases, and infusion of fluids. By monitoring pressure continuously through this port during catheter placement, the location of the tip can be determined from the characteristic pressure tracings shown in Fig. 26.8. After placement, PAP should be monitored continuously in order to identify inadvertent migration into the pulmonary capillary bed or “wedged” position. It is important to allow the catheter tip to “float” into the wedged position only when actively measuring PAOP in order to minimize risk of pulmonary artery infarct or rupture. The balloon inflation port inflates the balloon, which is located 1 cm proximal to the catheter tip. The balloon is inflated for flowdirected catheter placement and PAOP monitoring. The thermistor is located just proximal to the balloon and connects to a bedside computer to measure changes in the temperature of pulmonary artery blood. The oximeter uses a fiberoptic-based sensor to continuously measure the Svo2. Larger catheters also may have cardiac pacing ports. An adultsized catheter is available for continuous CO determination when coupled with an appropriate bedside computer.

Indirectly Measured Variables Measurements from PACs include directly and indirectly measured or derived variables. Directly measured variables include CVP, MAP, MPAP, PAOP, CO, arterial oxygen saturation (Sao2), and Svo2. Derived parameters include CI, PVR, SVR, PVRI, and systemic vascular resistance index (SVRI), as well as SV (in mL/ beat) and stroke volume index (SVI; in mL/beat per m2). Stroke index (SI), or SVI, normally is 30 to 60 mL/m2.71,72 

SV 5 CO/HR

Eq. 26.2

 SVI 5 SV/BSA

Eq. 26.3

234

SECTION IV



Pediatric Critical Care: Cardiovascular

SvO2 % Proximal port

100 80 60

Distal port

40 20 0 Balloon inflation port

Thermistor connector

0

15

30

45

60

Time min

Oximeter connector

ge ed

W

e en tric l tv

rtery

Ri

gh

ry a

30 mm Hg

ion sit po

ig

R

40

ona Pulm

um

tri

a ht

20 10 0

• Fig. 26.8

​Components and functional features of a thermodilution flow-directed pulmonary artery catheter. The flexible multilumen catheter with the balloon at the distal tip inflated is in the wedge position. The proximal ends of the five lumens are labeled. The distal port is connected to a pressure measurement system for catheter insertion and subsequent monitoring. When the distal tip is within the central venous circulation, the balloon is inflated to enhance flow direction of the tip through the right atrium into the right ventricle and then to the pulmonary artery. Recorded pressures (bottom) correspond to these locations, confirming the course of the catheter. The last tracing on the right corresponds to the “wedge” position, commonly reflecting pressure transmitted from the left atrium via the pulmonary veins and capillaries. Upper right panel shows an example of a continuous Svo2 (venous oxygen saturation) tracing from the fiberoptic monitor available on adult-size catheters. (Modified from Daily EK, Tilkian AG. Hemodynamic monitoring. In: Tilkian AG, Daily EK, eds. Cardiovascular Procedures, Diagnostic Techniques and Therapeutic Procedures. St. Louis: Mosby; 1986.)  



Left ventricular stroke work index (LVSWI) and right ventricular stroke work index (RVSWI) normally are 56 6 6 and 0.5 6 0.06 gm-m/m2, respectively.71,72 Note that all values are for pediatric patients unless otherwise indicated. 

LVSWI 5 SI 3 MAP 3 0.0136

Eq. 26.4



RVSWI 5 SI 3 MAP 3 0.0136

Eq. 26.5

Measurement of Cardiac Output CO is the volume of blood pumped by the heart each minute, or SV multiplied by the number of ejections per minute or HR (CO 5 HR 3 SV) and often is expressed as CI, which is CO divided by the body surface area (BSA) in square meters. The normal range for infants and children is approximately 3.3 to 6 L/min/m2.71,72 Two methods for calculating CO are discussed here: the Fick method and thermodilution.

Fick Method In 1870, Adolph Fick was the first to study the relationship between blood flow and gas exchange in the lungs using a mathematic model.73 Fick hypothesized that the amount of oxygen extracted by the body from the blood must equal the amount of oxygen taken up by the lungs during breathing. Fick also reasoned that the flow of blood through the lungs must equal the CO to the remainder of the body in the absence of a shunt. If the amount of oxygen consumed by the body and the amount of oxygen extracted by the body from the blood can be determined, then the CO can be determined. In Fick’s time, oxygen consumption was measured using a basal metabolism spirometer, and the oxygen content in arterial and venous blood was measured using a rudimentary method.73 Although Fick’s method remains the gold standard, it is rarely used in the ICU because it is less practical than the more commonly used thermodilution method described in the next section. However, Fick’s method is commonly used in the cardiac catheterization laboratory because the required data are easily measured in this setting, although oxygen consumption is often estimated.



CHAPTER 26 Principles of Invasive Cardiovascular Monitoring

235

TABLE Hemodynamic Parameters 26.1

Parameter

Formula

Normal Range

Units

CI 5 CO/BSA

3.5–5.5

L/min/m2

SI 5 CI/heart rate 3 1000

30–60

mL/m2

Arterial-mixed venous O2 content difference

avDo2 5 Cao2 – Cvo2

30–55

mL/L

O2 delivery

Do2 5 CI 3 O2

O2 consumption

Cardiac index Stroke index

620 6 50

mL/min/m2

Vo2 5 CI 3 avDo2

120–200

mL/min/m2

O2 extraction ratio

ERO2 5 avDo2/Cao2

0.26 6 0.02

Arterial oxygen content

(1.34 3 Hb 3 Sao2) 1 (Pao2 3 0.003)

mL/L

Venous oxygen content

(1.34 3 Hb 3 Svo2) 1 (Pvo2 3 0.003)

mL/L

Fick principle

VO2 5 CO 3 (Cao2 – Cvo2)

Systemic vascular resistance index

SVRI 5 80 3 (MAP – CVP)/CI

Pulmonary vascular resistance index

PVRI 5 80 3 (MPAP – PAOP)/CI

LV stroke work index

LVSWI 5 SI 3 MAP 3 0.0136

56 6 6

gm-m/m2

RV stroke work index

RVSWI 5 SI 3 MPAP 3 0.0136

0.5 6 0.06

gm-m/m2

dyne • s/cm5/m2

80–200

dyne • s/cm5/m2  







800–1600

avDo2, Arterial-mixed venous content difference; BSA, body surface area in m2; Cao2, O2 content of systemic arterial blood in mL/L; CI, cardiac index; CO, cardiac output; Cvo2, O2 content of mixed venous blood in mL/L; CVP, central venous pressure in mm Hg; DO2, oxygen delivery; ERO2, O2 extraction ratio; Hb, hemoglobin; LVSWI, left ventricular stroke index; MAP, mean systemic arterial pressure in mm Hg; 80 is the conversion factor used for the units in the table; MPAP, mean pulmonary arterial pressure in mm Hg; PAWP, pulmonary artery wedge pressure in mm Hg, which is approximately equal to the left atrial pressure under many circumstances; Pvo,2, partial oxygen pressure in mixed venous blood; PVRI, pulmonary vascular resistance index; RVSWI, right ventricular stroke work index; SI, stroke index; Svo2, venous oxygen saturation; SVRI, systemic vascular resistance index; Vo2, oxygen consumption. Modified from Katz RW, Pollack MM, Weibley RE. Pulmonary artery catheterization in pediatric intensive care. In: L.A. Barness, ed. Advances in Pediatrics. Chicago: Year–Book; 1984.

As noted previously, Fick’s equation is based on the assumption that the amount of oxygen extracted by the body from the blood equals the amount of oxygen taken up from the lungs during breathing. The oxygen content of blood is generally expressed in milliliters of O2 per deciliter of blood. The difference in oxygen content of arterial blood (Cao2) and venous blood (Cvo2) is termed the arterial-mixed venous oxygen content difference (avDo2). By multiplying the avDo2 by the amount of blood pumped through the lungs or body (CO) we can calculate the oxygen consumption. Note that CO is generally expressed in L/min. Therefore, we must multiply the avDo2 by 10 to convert it to milliliters of O2 per liter of blood if we are to perform the calculation using CO in L/min. The oxygen content of the blood is a function of the hemoglobin (Hb) concentration of blood in g/dL, the Sao2 or Svo2 expressed in decimal form, and the arterial or venous partial pressure of arterial oxygen (Pao2 or Pvo2) expressed in mm Hg. The oxygen-carrying capacity of adult Hb is 1.34 mL O2/g Hb, and the Bunsen solubility coefficient of O2 in plasma at 37°C equals 0.003 mL/mm Hg per dL. A true Svo2 is measured in the pulmonary artery; however, in the presence of an intracardiac leftto-right Svo2 shunt, Svo2 should be measured in the SVC.



 2 5 (1.34 3 Hb 3 Sao2) 1 (Pao2 3 0.003) Cao

Eq. 26.6

 2 5 (1.34 3 Hb 3 Svo2) 1 (Pvo2 3 0.003) Cvo

Eq. 26.7

avDo2 5 Cao2 – Cvo2

Eq. 26.8

The avDo2 is the difference between Cao2 and Cvo2 and normally ranges from 2.8 to 7.8 mL/dL in children.72

As noted earlier, the amount of oxygen extracted (consumed) by the body from the blood equals avDo2 multiplied by the amount of blood that flows through the lungs (QP). Assuming QP equals the flow of blood through the systemic circulation (QS), then QP is a measure of CO. (Note that pulmonary and systemic blood flows cannot be assumed to be identical in children with CHD with single-ventricle physiology or those with anatomic shunts.)  2 extraction 5 10 3 (Cao2 – Cvo2) 3 CO O

Eq. 26.9

The amount of oxygen taken up by the lungs equals the amount of oxygen consumed by the body. According to Fick, the amount of oxygen extracted by the body from the blood (Eq. 26.10) equals oxygen consumption (Vo2).  3 (Cao2 – Cvo2) 3 CO 5 VO2 in L/min 10

Eq. 26.10

 CO 5 VO2/[10 3 (Cao2 – Cvo2)]

Eq. 26.11

As noted in Eqs. 26.6 and 26.7, the amount of dissolved oxygen in blood (Pao2 or Pvo2) contributes an almost negligible amount to the oxygen content and can be left out (unless very high) for ease of computation. By rearranging Eq. 26.11, a rough estimate of CO can be calculated rather easily at the bedside without use of a PAC:  CO 5 VO2/(1.34 3 Hb 3 (Sao2 – Svo2) 3 10)

Eq. 26.12

Oxygen consumption can be measured using the metabolic cart or taken from standardized tables.17 Hb concentration can be

236

SECTION IV



Pediatric Critical Care: Cardiovascular

measured directly. Sao2 can be taken from the pulse oximeter. Svo2 can be measured by the oximeter at the distal end of the PAC or determined from a venous blood gas sample from a catheter in the internal jugular or subclavian vein. These data also can be used to calculate the intrapulmonary shunt fraction, which is the fraction of blood that passes through unventilated areas of lung:  Qs/Qt 5 (Cpvo2 2 Cao2)/(Cpvo2 2 Cvo2)

Eq. 26.13

where Cao2 is systemic arterial oxygen content and Cvo2 is mixed venous oxygen content. Cpvo2 is the theoretical oxygen content in a normal pulmonary vein and can be estimated using the alveolar gas equation:  Cpvo 2 5 1.34 3 Hb 3 Spvo2 1 Ppvo2 3 0.003

Eq. 26.14

where Spvo2 is pulmonary vein O2 saturation and Ppvo2 is pulmonary vein po2. For the normal lung, Ppvo2 can be estimated from the alveolar air equation (Eq. 26.15), and Spvo2 is presumed to be 1.0:  Ppvo 2 5 Pao2 5 (Pio2 2 Pwp) 2 Paco2/R

Eq. 26.15

where Pao2 is alveolar partial pressure of oxygen, Pio2 is inspiratory pO2, Pwp is vapor pressure of water (47 mm Hg at 37°C), Paco2 is arterial CO2, and R is respiratory quotient, which is normally assumed to be 0.8. The normal shunt fraction is 3% to 7%.

Thermodilution Method In 1921, Stewart74 first described an indicator-dilution method for measuring CO. Flow was calculated by measuring the change in concentration of an indicator over time. The “ideal” indicator is “stable, nontoxic, uniformly distributed, and does not leave the system between sites of injection and detection. However, it should be rapidly cleared in a single circulation time to prevent recirculation interfering with measurement.”75 In 1953, Fegler76,77 demonstrated that a change in the heat content of blood could be used as an indicator for CO measurement. A bolus of cold liquid of a known temperature is injected into or proximal to the right atrium. A thermistor near the PAC tip in the pulmonary artery or a pulmonary artery branch measures a change in the temperature of the blood as the bolus passes by the end of the catheter. A computer calculates the flow by integrating the change in temperature at the thermistor. The first law of thermodynamics, the conservation of heat, is the fundamental principle underlying thermodilution. Thermodilution makes several assumptions: physiologic conditions must remain constant during the period of observation; all heat exchange occurs between the indicator and the blood without heat loss to the surrounding tissues; mixing of the injectate and blood is complete upstream of the temperature measurement; and the temperature sensor is sufficiently sensitive, accurate, and rapidly responsive to depict accurately the change in temperature over time. Measurement of CO using the thermodilution method can be understood by examining a modified version of the StewartHamilton equation.75 V1 is injectate volume (in mL); Tb is temperature of the pulmonary artery at baseline (in degrees Celsius); Ti is temperature of the injectate (in degrees Celsius); K1 is the

density factor that equals the specific heat of the injectate multiplied by the specific gravity of the injectate, divided by the product of the specific heat and specific gravity of blood; and K2 is a constant that figures in the dead space of the catheter and the loss of heat from the injectate as it moves through the catheter. The denominator of the equation is the integral of the change in the temperature of the blood (Tb) over time (t):  CO 5 V1(Tb 2 Ti)K1K2/∫DTb(t)dt

Eq. 26.16

The computer generates a CO curve with the area under the curve inversely related to the magnitude of the CO. In settings of low CO, less warm blood flows with the injectate, and the injectate stays cooler. The difference between the injectate temperature and that of the blood remains large, and the CO curve has a high domed shape, with a slow return to baseline temperature. In situations of high CO, more pulmonary artery blood flows with the injectate, and the temperature of the injectate approaches or equals that of the blood more rapidly. In these situations, because the difference between the final temperature of the injectate and that of the blood is small, the CO curve rapidly returns to baseline following a sharp spike from the cold injectate. In extreme lowflow states, the change in temperature of the injectate resulting from handling alone, before the injectate even enters the catheter from the proximal port, may be greater than the change caused by warming of the injectate by the flow of blood. A correction factor is added to the equation to account for warming of the injectate because of handling alone. However, the correction factor may be inaccurate if the injection is too slow or the syringe is held in the injector’s hands too long. Therefore, CO readings should be made as quickly as possible and should be repeated until three successive readings are within 15% of each other. Other sources of error include a falsely elevated CO because of inadvertent warming of the thermistor when it is up against the wall of the pulmonary artery. The thermodilution method generally should not be used in patients with an intracardiac shunt. However, if the shunt fraction is less than 10%, the error likely is negligible.24 Measurements in the cardiac catheterization lab are frequently considered the gold standard for CO determination, but the PAC has clinical advantages for intermittent measurements at the bedside. It is important to keep in mind the shortcomings of the PAC, including accuracy of measurements and risks to the patient. Stetz et al.78 evaluated PAC thermodilutional determination of CO and found that a difference of 15% or less across three measurements suggested acceptable precision. More recent follow-up studies have found that this level of precision is infrequently achieved; for example, Dhingra et al.79 evaluated thermodilutional PAC measurements against direct Fick calculations and found a percentage of error of 62%. An animal model study80 found that PACs were only able to consistently detect CO changes of at least 30%.

Calculation of Oxygen Delivery and Consumption Metabolic derangements, such as fever, sepsis, and shock, interfere with DO2 to and VO2 by the tissues. Svo2 is a measure of the oxygenation of blood returning to the heart. Svo2 can be measured continuously by a fiberoptic oximeter (see description of PAC ports in the catheter placement section) and normally ranges



CHAPTER 26 Principles of Invasive Cardiovascular Monitoring

Do2 also can be expressed as the product of CI and Cao2 and Vo2 as the product of CI and avDo2. The normal value for Do2 is 620 6 50 mL/min per square meter. Vo2 typically ranges from 120 to 200 mL/min per square meter.71,72 

Do2 5 CI 3 Cao2

Eq. 26.18



Vo2 5 CI 3 avDo2

Eq. 26.19

Interpretation of Waveforms The waveforms corresponding to the right atrium and systemic arterial blood pressure were discussed in previous sections. The pressure in the right atrium ranges from approximately 3 to 12 mm Hg. As the PAC passes into the right ventricle, the diastolic pressure drops to 0 to 10 mm Hg and the systolic pressure increases to 13 to 42 mm Hg. As the catheter enters the pulmonary artery, the diastolic pressure increases to 3 to 21 mm Hg while the systolic pressure remains relatively similar to that of the right ventricle, 11 to 36 mm Hg. Once the catheter tip advances into the pulmonary capillary bed and the pulmonary artery is occluded by the inflated balloon, the measured pressure decreases to 2 to 14 mm Hg.24 By recognizing the changes in the various tracings, the movement of the catheter tip can be followed through the chambers of the right heart and into the pulmonary circulation without simultaneous imaging. The waveforms are affected by the components of the respiratory cycle. As expected, the effects of respiration differ during unsupported breathing (negative pressure) versus mechanical ventilation (positive pressure). During normal unsupported ventilation, PAP decreases during inhalation and increases during exhalation. In contrast, during mechanical ventilation, PAP increases during inhalation and decreases during exhalation. The cyclical changes induced by the respiratory cycle cause the tracings to take on a sinusoidal pattern once the tip of the catheter enters the thorax. The effects of respiration on PAC determinations can be minimized by measuring pressures at the end of expiration, when pleural pressures are closest to zero. Because CVP is a measure of preload or filling of the right ventricle, it reflects changes in volume status, right ventricular function, and pulmonary vascular tone. Similarly, PAOP measures filling pressures of the left atrium and ventricle. When the pulmonary artery is occluded, the pressure from the left atrium is transmitted back to the catheter tip. During diastole, when the mitral valve is open and the aortic valve is closed, a continuous fluid-filled column is formed from the catheter tip to the left ventricle and PAOP is equivalent to the left ventricular enddiastolic pressure. In patients with cardiogenic shock, an elevated PAOP may reflect decreased function of the left ventricle. In this situation, rather than providing further fluid resuscitation or preload, increasing contractility or decreasing afterload may be preferable. Afterload is the load that the heart must eject blood against.

T 5 P 3 r/2t



Eq. 26.20

For a given pressure, wall stress is increased by an increase in radius (ventricular dilation); therefore, volume administration may increase ventricular diameter and, consequently, wall stress. Similarly, during spontaneous breathing, the transluminal pressure and, consequently, the wall stress increase, whereas during mechanical ventilation (positive pressure), the transluminal pressure and wall stress both decrease. Ventricular hypertrophy increases wall thickness and therefore decreases wall stress.

Resistance To understand resistance, returning to Ohm’s law is helpful: voltage (V) varies directly with resistance (R) and current (I): V 5 IR



Eq. 26.21

Rearranging Eq. 26.21 by substituting pressure for voltage and flow for current gives Eq. 26.22: R 5 (Pin 2 Pout)/Q



Eq. 26.22

where R is resistance, Pin is pressure going into a vessel, Pout is pressure exiting the vessel, and Q is flow. According to Poiseuille’s law, the resistance of flow through a tube varies directly with the viscosity of the fluid and the length of the tube and is inversely proportional to the radius to the fourth power multiplied by pi (π): R 5 8hl/pr4



Eq. 26.23

where h is viscosity, l is length, and r is radius. Unfortunately, Poiseuille’s law assumes uniform viscosity, length, and radius, none of which holds true in the case of pulmonary or systemic circulation; however, the principles behind the law are valuable in understanding the major determinants of resistance. By substituting the appropriate values into Eq. 26.20, the formulas for SVR and PVR can be derived. CO is substituted for Qs and Qp in the absence of a right-to-left or left-to-right shunt or singleventricle physiology. In the case of the equation for PVR (Eq. 26.25), PAOP is substituted for pulmonary vein pressure in determining Pout: 

SVR 5 (MAP 2 CVP)/CO

Eq. 26.24



PVR 5 (MPAP 2 PAOP)/CO

Eq. 26.25

SVR and PVR are measured in mm Hg 3 minute 3 L21 (or mm Hg/L per min). These units also are referred to as hybrid resistance units or Wood units after the cardiologist Paul Wood.21 By multiplying by 80, hybrid resistance units or Wood units can be converted to the centimeter-gram-seconds (cgs) system, where resistance is measured as dyne • s/cm5, also known as absolute resistance units. PVR and SVR often are indexed for BSA (in m2). The SVRI and PVRI are measured as dyne • s/m2 per cm5:  

Eq. 26.17



ERO2 5 avDo2/Cao2





According to Laplace’s law, ventricular wall stress (T) is proportional to ventricular transluminal pressure (P 5 intraluminal pressure – extraluminal pressure) and radius (r) and is inversely related to twice the wall thickness (t):



from 65% to 75%. The oxygen extraction ratio (ERO2) is avDo2 (see Eq. 26.8) divided by Cao 2 (Eq. 26.11) and usually is approximately 25%71,72:

237



SVRI 5 80 3 (MAP 2 CVP)/Cl

Eq. 26.26



PVRI 5 80 3 (MPAP 2 PAOP)/Cl

Eq. 26.27

238

SECTION IV



Pediatric Critical Care: Cardiovascular

SVRI usually is 800 to 1600 dyne • s/m2 per cm5 in children72,73 and 2180 6 210 in adults.81  



Calculation of Intracardiac Shunt If the oxygen saturations throughout the cardiopulmonary circulation are known, derivation of the values for the ratio of pulmonary to systemic blood flow or intracardiac shunt (Qp/Qs) is possible:  5 Vo2/(1.34 3 10 3 Hb[Spvo2 2 Spao2]) Qp

Eq. 26.28

 5 Vo2/(1.34 3 10 3 Hb[Sao2 2 Svo2]) Qs

Eq. 26.29

 Qp/Qs 5 (Sao2 2 Svo2)/(Spvo2 2 Spao2)

Eq. 26.30

where Spvo2 is oxygen saturation in the pulmonary vein and Spao2 is oxygen saturation in the pulmonary artery. In the absence of severe intrapulmonary shunt, Spvo2 approaches 98% to 100%. In a complete mixing lesion, Spao2 and Sao2 should be equal by definition, enabling Sao2 to be substituted for Spao2.

Novel Monitoring Strategies As PAC use has fallen dramatically and familiarity with it has waned, providers have been replacing it with many innovative techniques, often used in concert with one another, to obtain similar information. Many of these investigations can be performed noninvasively. Pulse wave Doppler (PWD) has shown promise82 as an assessment of left ventricular function, as has esophageal Doppler,83 although the latter requires a transesophageal echocardiogram probe. Thermodilution from PACs may be supplanted by lithium dilution and/or transpulmonary thermodilution methods.84,85 Although the long-term utilization of such techniques is not yet routine in children, the relative ease of use or placement, often using in situ CVCs and arterial lines, in comparison with the PAC, and additional monitoring capabilities, make them attractive.

Conclusions Invasive hemodynamic monitoring provides the intensivist with valuable information regarding the condition of critically ill children. Correct interpretation of this information is important to aid in the management of these patients, but these data must be integrated with the rest of the patients’ assessments. When pieces

of data conflict with one another, the invasive origin of one does not necessarily suggest its superiority. New noninvasive monitoring modalities are emerging that may eventually supplant the need for these invasive measurements. Thus far, however, invasive monitoring remains a cornerstone of pediatric critical care medicine.

Key References Baloglu O, Aluquin VP, Tamburro RF, et al. Assessing pulmonary arterial hypertension in infants with severe chronic lung disease of infancy: a role for a pulmonary artery catheter? Pediatr Cardiol. 2013;34: 1330-1334. Cohn JN, Luria MH. Studies in clinical shock and hypotension; the value of bedside hemodynamic observations. JAMA. 1964;190: 891-896. Eskesen TG, Wetterslev M, Perner A. Systematic review including reanalyses of 1148 individual data sets of central venous pressure as a predictor of fluid responsiveness. Intensive Care Med. 2016;42: 324-332. Marik PE, Cavallazzi R, Vasu T, Hirani A. Dynamic changes in arterial waveform derived variables and fluid responsiveness in mechanically ventilated patients: a systematic review of the literature. Crit Care Med. 2009;37:2642-2647. Marik PE. Noninvasive cardiac output monitors: a state-of the-art review. J Cardiothorac Vasc Anesth. 2013;27:121-134. Mercier JC, Beaufils F, Hartmann JF, Azema D. Hemodynamic patterns of meningococcal shock in children. Crit Care Med. 1988;16:27-33. Pagnamenta A, Lador F, Azzola A, Beghetti M. Modern invasive hemodynamic assessment of pulmonary hypertension. Respiration. 2018; 95:201-211. Perez AC, Eulmesekian PG, Minces PG, Schnitzler EJ. Adequate agreement between venous oxygen saturation in right atrium and pulmonary artery in critically ill children. Pediatr Crit Care Med. 2009;10: 76-79. Ruth A, McCracken CE, Fortenberry JD, Hall M, Simon HK, Hebbar KB. Pediatric severe sepsis: current trends and outcomes from the Pediatric Health Information Systems database. Pediatr Crit Care Med. 2014;15:828-838. Walkey, AJ, Wiener RS, Lindenauer, PK. Utilization patterns and outcomes associated with central venous catheter in septic shock – a population-based study. Crit Care Med. 2013;41:1450-1457. Yang X, Du B. Does pulse pressure variation predict fluid responsiveness in critically ill patients? A systematic review and meta-analysis. Crit Care. 2014;18:650.

The full reference list for this chapter is available at ExpertConsult.com.

e1











































































































1. Hales S. Statical Essays: Containing Haemastaticks; or, an Account of Some Hydraulick and Hydrostatical Experiments Made on the Blood and Blood-Vessels of Animals. London: Innis, Manby, and Woodward; 1733. 2. Tobin M. Principles and Practice of Intensive Care Monitoring. New York: McGraw-Hill; 1998. 3. Webster J. Medical Instrumentation. Boston: Houghton Mifflin Company; 1978. 4. Bronzino J. The Biomedical Engineering Handbook. Boca Raton, FL: CRC Press; 1995. 5. Lowrie L, Difiore J, Martin R. Monitoring in the pediatric and neonatal intensive care units. In: Tobin JMM, ed. Principles and Practice of Intensive Care Monitoring. San Francisco, CA: McGrawHill; 1998. 6. Heitmiller EWR. Hemodynamic monitoring considerations in pediatric critical care. In: Rogers M, ed. Textbook Pediatric Intensive Care. Baltimore: William & Wilkins; 1996. 7. Carcillo JA, Fields AI. Clinical practice parameters for hemodynamic support of pediatric and neonatal patients in septic shock. Crit Care Med. 2002;30:1365-1378. 8. Eskesen TG, Wetterslev M, Perner A. Systematic review including re-analyses of 1148 individual data sets of central venous pressure as a predictor of fluid responsiveness. Intensive Care Med. 2016;42: 324-332. 9. Cecconi M, Aya HD, Geisen M, et al. Changes in the mean systemic filling pressure during a fluid challenge in postsurgical intensive care patients. Intensive Care Med. 2013;39:1299-1305 10. Edwards Lifesciences. PediaSat Oximetry Catheter. 2008. http:// ht.edwards.com/resourcegallery/products/presep/pdfs/ar03176.pdf. 11. Rivers E, Nguyen B, Havstad S, et al. Early goal-directed therapy in the treatment of severe sepsis and septic shock. N Engl J Med. 2001;345: 1368-1377. 12. Yealy DM, Kellum JA, Huang DT, et al. A randomized trial of protocol-based care for early septic shock. N Engl J Med. 2014; 370:1683-1693. 13. Peake SL, Delaney A, Bailey M, et al. Goal-directed resuscitation for patients with early septic shock. N Engl J Med. 2014;371: 1496-1506. 14. Mouncey PR, Osborn TM, Power GS, et al. Trial of early, goaldirected resuscitation for septic shock. N Engl J Med. 2015;372: 1301-1311. 15. Ruth A, McCracken CE, Fortenberry JD, Hall M, Simon HK, Hebbar KB. Pediatric severe sepsis: current trends and outcomes from the Pediatric Health Information Systems database. Pediatr Crit Care Med. 2014;15:828-838. 16. Marik PE. Noninvasive cardiac output monitors: a state-of the-art review. J Cardiothorac Vasc Anesth. 2013;27:121-134. 17. Perez AC, Eulmesekian PG, Minces PG, Schnitzler EJ. Adequate agreement between venous oxygen saturation in right atrium and pulmonary artery in critically ill children. Pediatr Crit Care Med. 2009;10:76-79. 18. Berthiaume LR, Peets AD, Schmidt U, et al. Time series analysis of use patterns for common invasive technologies in critically ill patients. J Crit Care. 2009;24:471.e9-e14. 19. Walkey, AJ, Wiener RS, Lindenauer, PK. Utilization patterns and outcomes associated with central venous catheter in septic shock – a population-based study. Crit Care Med. 2013;41:1450-1457. 20. Roberts LN, Smiley JR, Manning GW. A comparison of direct and indirect blood-pressure determinations. Circulation. 1953;8: 232-242. 21. Van Bergen FH, Weatherhead DS, Treloar AE, et al. Comparison of indirect and direct methods of measuring arterial blood pressure. Circulation. 1954;10:481-490. 22. Cohn JN, Luria MH. Studies in clinical shock and hypotension; the value of bedside hemodynamic observations. JAMA. 1964;190: 891-896.

23. Kaye W. Invasive monitoring techniques: arterial cannulation, bedside pulmonary artery catheterization, and arterial puncture. Heart Lung. 1983;12:395-427. 24. Vargo T. Cardiac catheterization: hemodynamic measurements. In: Garson A, Fisher D, Neish S, eds. Science and Practice of Pediatric Cardiology. Baltimore, MD: Williams & Wilkins; 1998. 25. Murgo JP, Westerhof N, Giolma JP, Altobelli SA. Aortic input impedance in normal man: relationship to pressure wave forms. Circulation. 1980;62:105-116. 26. Mignini M, Piacentini E, Dubin A. Peripheral arterial blood pressure monitoring adequately tracks central arterial blood pressure in critically ill patients: an observational study. Crit Care. 2006; 10(2):R43. 27. Compton F, Zukunft B, Hoffmann C, Zidek W, Schaefer J. Performance of a minimally invasive uncalibrated cardiac output monitoring system (Flotrac/Vigileo) in haemodynamically unstable patients. Br J Anaesth. 2008;100(4):451-456. 28. Galluccio S, Chapman M, Finnis M. Femoral-radial arterial pressure gradients in critically ill patients. Crit Care Resusc. 2009;11(1):34-38. 29. Kim W, Jun J, Huh J, Hong S, Lim C, Koh Y. Radial to femoral arterial blood pressure differences in septic shock patients receiving high-dose norepinephrine therapy. Shock. 2013;40(6): 527-531. 30. Magder S, Guerard B. Heart-lung interactions and pulmonary buffering: lessons from a computational modeling study. Respir Physiol Neurobiol. 2012;182:60-70. 31. Marik PE, Cavallazzi R, Vasu T, Hirani A. Dynamic changes in arterial waveform derived variables and fluid responsiveness in mechanically ventilated patients: a systematic review of the literature. Crit Care Med. 2009;37:2642-2647. 32. Michard F, Boussat S, Chemla D, et al. Relation between respiratory changes in arterial pulse pressure and fluid responsiveness in septic patients with acute circulatory failure. Am J Respir Crit Care Med. 2000;162:134-138. 33. Yang X, Du B. Does pulse pressure variation predict fluid responsiveness in critically ill patients? A systematic review and meta-analysis. Crit Care. 2014;18:650. 34. Auler Jr JO, Galas F, Hajjar L, Santos L, Carvalho T, Michard F. Online monitoring of pulse pressure variation to guide fluid therapy after cardiac surgery. Anesth Analg. 2008;106:1201-1206. 35. Fye WB. Jean-Baptiste Auguste Chauveau. Clin Cardiol. 2003;26: 351-353. 36. Chaveau J, Ej M. Determination graphique des rappats de la pulsation cardiaque avee les mouvements de l’ oriellette et du ventrucule, obtenu au moyen d’un appareil enregistecur. CR Mear Soc Biol. 1861;3:3-11. 37. Chaveau J, Ej M. De la force deployee par la contraction de differentes cavites du coeur. CR Mear Soc Biol. 1862;3:151-154. 38. Chaveau A, Marey E. Apparells at Experiences cardiographiques Demonsration Nouvelle du mechanisme des mouvement du mechanisme des mouvements du Coeur par l’Emploi des Instruments Enregistreurs a Indications Continuees. Paris: JB Balliere; 1863. 39. Forssman W. Die Sondierung des rechten herzens. Klin Wochenschr. 1929;8:2085-2087. 40. Fontenot C, O’Leary JP. Dr. Werner Forssman’s self-experimentation. Am Surg. 1996;62:514-515. 41. Cournand A, Ranges H. Catheterization of the right auricle in man. Proc Soc Exp Biol Med. 1941;46:462-466. 42. Richards D, Cournand A, Darling R, et al. Pressure of blood in the right auricle in animals and man: under normal conditions and in right heart failure. Am J Physiol. 1941;136:115-123. 43. Cournand A. Measurement of the cardiac output in man using the right heart catheterization. Fed Proc. 1945;4:207-212. 44. Hellems H, Haynes F, Dexter L, et al. Pulmonary capillary pressure in animals estimated by venous arterial catheterization. Am J Physiol. 1948;155:98-105. 45. Mukhopadhyay M. A biographical sketch of Lewis Dexter. Tex Heart Inst J. 2001;28:133-138.

References

e2















































66. Damen J, Wever JE. The use of balloon-tipped pulmonary artery catheters in children undergoing cardiac surgery. Intensive Care Med. 1987;13:266-272. 67. Hopkins RA, Bull C, Haworth SG, et al. Pulmonary hypertensive crises following surgery for congenital heart defects in young children. Eur J Cardiothorac Surg. 1991;5:628-634. 68. Atz AM, Adatia I, Jonas RA, Wessel DL. Inhaled nitric oxide in children with pulmonary hypertension and congenital mitral stenosis. Am J Cardiol. 1996;77:316-319. 69. Adatia I, Atz AM, Jonas RA, Wessel DL. Diagnostic use of inhaled nitric oxide after neonatal cardiac operations. J Thorac Cardiovasc Surg. 1996;112:1403-1405. 70. Morray JP, Lynn AM, Mansfield PB. Effect of pH and PCO2 on pulmonary and systemic hemodynamics after surgery in children with congenital heart disease and pulmonary hypertension. J Pediatr. 1988;113:474-479. 71. Cayler GG, Rudolph AM, Nadas AS. Systemic blood flow in infants and children with and without heart disease. Pediatrics. 1963;32:186-201. 72. Krovetz LJ, McLoughlin TG, Mitchell MB, Schiebler GL. Hemodynamic findings in normal children. Pediatr Res. 1967;1:122-130. 73. Gottschall CA. The greatest medical discovery of the millennium (Fundamental steps to the understanding of cardiac performance). Arq Bras Cardiol. 1999;73:320-330. 74. Stewart G. The output of the heart in dogs. Am J Physiol. 1921;57: 27-50. 75. Moise SF, Sinclair CJ, Scott DH. Pulmonary artery blood temperature and the measurement of cardiac output by thermodilution. Anaesthesia. 2002;57:562-566. 76. Fegler G. A thermocouple method of determination of heart output in anaesthetized dogs. Montreal: XIX International Physiological Congress Proceedings p. 341;1953. 77. Fegler G. Measurement of cardiac output in anesthetized animals by a thermodilution method. Q J Exp Physiol. 1954;39:153-164. 78. Stetz CW, Miller RG, Kelly GE, Raffin TA: Reliability of the thermodilution method in the determination of cardiac output in clinical practice. Am Rev Respir Dis. 1982;126:1001-1004. 79. Dhingra VK, Fenwick JC, Walley KR, Chittock DR, Ronco JJ. Lack of agreement between thermodilution and Fick cardiac output in critically ill patients. Chest. 2002;122:990-997. 80. Phillips RA, Hood SG, Jacobson BM, West MJ, Wan L, May CN. Pulmonary artery catheter (PAC) accuracy and efficacy compared with flow probe and transcutaneous Doppler (USCOM): an ovine cardiac output validation. Crit Care Res Pract. 2012;62:1494. 81. Shoemaker W, Chang P, Bland R, Al E. Cardiorespiratory monitoring in postoperative patients: Pulmonary Artery Catheter Consensus Conference: consensus statement. Crit Care Med. 1979;7:243-249. 82. Teboul JL, Saugel B, Cecconi M, et al. Less invasive hemodynamic monitoring in critically ill patients. Intensive Care Med. 2016;42: 1350-1359. 83. Huttemann E, Schelenz C, Kara F, Chatzinikolaou K, Reinhart K. The use and safety of transoesophageal echocardiography in the general ICU—a minireview. Acta Anaesthesiol Scand. 2004;48:827-836. 84. Monnet X, Teboul JL. Transpulmonary thermodilution: advantages and limits. Crit Care. 2017;21:147. 85. Monnet X, Anguel N, Osman D, Hamzaoui O, Richard C, Teboul JL. Assessing pulmonary permeability by transpulmonary thermodilution allows differentiation of hydrostatic pulmonary edema from ALI/ARDS. Intensive Care Med. 2007;33:448-453.

















































46. Lategola M, Rahn H. A self-guiding catheter for cardiac and pulmonary arterial catheterization and occlusion. Proc Soc Exp Biol Med. 1953;84:667-668. 47. Swan HJ, Ganz W, Forrester J, et al. Catheterization of the heart in man with use of a flow-directed balloon-tipped catheter. N Engl J Med. 1970;283:447-451. 48. Connors AF Jr, Speroff T, Dawson NV, et al. The effectiveness of right heart catheterization in the initial care of critically ill patients. SUPPORT Investigators. JAMA. 1996;276:889-897. 49. Sandham JD, Hull RD, Brant RF, et al. A randomized, controlled trial of the use of pulmonary-artery catheters in high-risk surgical patients. N Engl J Med. 2003;348:5-14. 50. Richard C, Warszawski J, Anguel N, et al. Early use of the pulmonary artery catheter and outcomes in patients with shock and acute respiratory distress syndrome: a randomized controlled trial. JAMA. 2003;290:2713-2720. 51. Hadian M, Pinsky MR. Evidence-based review of the use of the pulmonary artery catheter: impact data and complications. Crit Care. 2006;10:S8. 52. Shah MR, Hasselblad V, Stevenson LW, et al. Impact of the pulmonary artery catheter in critically ill patients: meta-analysis of randomized clinical trials. JAMA. 2005;294:1664-1670. 53. Friese RS, Shafi S, Gentilello LM. Pulmonary artery catheter use is associated with reduced mortality in severely injured patients: a National Trauma Data Bank analysis of 53,312 patients. Crit Care Med. 2006;34:1597-1601. 54. Taylor R, Ahrens T, Yaakov B, PACCCP. Pulmonary artery catheter consensus conference: consensus statement. Crit Care Med. 1997; 25:910. 55. Perkin RM, Anas N. Pulmonary artery catheters. Pediatr Crit Care Med. 2011;12:S12-S20. 56. Carcillo JA, Davis AL, Zaritsky A. Role of early fluid resuscitation in pediatric septic shock. JAMA. 1991;266:1242-1245. 57. Mercier JC, Beaufils F, Hartmann JF, Azema D. Hemodynamic patterns of meningococcal shock in children. Crit Care Med. 1988;16:27-33. 58. Pollack MM, Reed TP, Holbrook PR, Fields AI. Bedside pulmonary artery catheterization in pediatrics. J Pediatr. 1980;96: 274-276. 59. Reynolds EM, Ryan DP, Sheridan RL, Doody DP. Left ventricular failure complicating severe pediatric burn injuries. J Pediatr Surg. 1995;30:264-269; discussion 269-270. 60. Baloglu O, Aluquin VP, Tamburro RF, et al. Assessing pulmonary arterial hypertension in infants with severe chronic lung disease of infancy: a role for a pulmonary artery catheter? Pediatr Cardiol. 2013;34:1330-1334. 61. Pagnamenta A, Lador F, Azzola A, Beghetti M. Modern invasive hemodynamic assessment of pulmonary hypertension. Respiration. 2018;95:201-211. 62. Thompson AE. Pulmonary artery catheterization in children. New Horiz. 1997;5:244-250. 63. Singh S, Nasa V, Tandon M. Perioperative monitoring in liver transplant patients. J Clin Exp Hepatol. 2012;2:271-278. 64. DeBruin W, Notterman DA, Magid M, et al. Acute hypoxemic respiratory failure in infants and children: clinical and pathologic characteristics. Crit Care Med. 1992;20:1223-1234. 65. Wheedon D, Shore DF, Lincoln C. Continuous monitoring of pulmonary artery pressure after cardiac surgery in infants and children. J Cardiovasc Surg (Torino). 1981;22:307-311.

e3

Abstract: Hemodynamic monitoring refers to measurement of the functional characteristics of the heart and circulatory system that affect the perfusion of tissues with oxygenated blood. Hemodynamic monitoring can be performed invasively or noninvasively and can be used for diagnosis, surveillance, or titration of therapy. The central venous waveform is composed of three waves (a, c, and v) and two wave descents (x and y). The arterial waveform has three components: rapid upstroke, dicrotic notch, and runoff. Pulse

pressure variation has excellent specificity as an indicator of fluid responsiveness in many critically ill patients. Cardiac output can be calculated using the Fick method or measured directly via thermodilution. A pulmonary artery catheter can be used to measure cardiac output and indices of oxygen delivery and extraction. Key words: pulmonary artery catheter, Swan-Ganz, arterial line, central venous catheter, pulse pressure, hemodynamics

10 27

Chapter Titleof Cardiovascular Function Assessment

CHAPTER AUTHOR NATHANIEL R. SZNYCER-TAUB, THOMAS J. KULIK, JOHN R. CHARPIE, AND MELVIN C. AL MODOVAR

PEARLS Again R L Sbasic knowledge of the development of the eye. •P ETo

••

















To develop essential understanding how abnormalities at inCardiovascular assessment and monitoring in the pediatric various stages of development can arrest or hamper normal tensive care unit require careful integration of physical findings, formation the ocular structures data and visual pathways. laboratory of studies, and electronic to make appropriate therapeutic decisions. Noninvasive monitoring includes physical examination, chest radiography, echocardiography, blood pressure monitoring, and pulse oximetry. Invasive monitoring includes intravascular and intracardiac monitoring, cardiac output measurements (thermodilution or Fick method), and laboratory studies. It is important to appreciate the quantity of therapy needed to achieve and sustain adequate systemic oxygen delivery and





To acquire adequate information about normal anatomy of the eye and related ifstructures and is develop a strong the foundation perfusion pressure the clinician to understand pafor the understanding of common ocular problems and tient’s overall condition, discern the patient’s trajectory,their and consequences. anticipate associated consequences of current management choices. Management of patients with single-ventricle physiology (such as the neonate with hypoplastic left heart syndrome) poses several unique challenges to the cardiac intensivist, including optimization of pulmonary-to-systemic blood flow ratios for best systemic oxygen delivery.

Pediatric patients undergoing surgical treatment for congenital heart disease (CHD) or those with severe systemic illnesses such as sepsis and other causes of multiple-organ system failure commonly have impaired cardiovascular function.1,2 In addition to treating the primary disease process, the pediatric intensivist should use strategies to reliably assess and monitor cardiovascular function, which specifically involve assessing adequacy of oxygen delivery (Do2) and systemic perfusion pressure, the primary determinants of tissue oxygenation.

O2, the ease with which these demands can be increased by small environmental changes, and the apparently limited reserve for augmenting cardiac output (CO) or O2 extraction acutely.7,8 Thus, it is crucial that cardiovascular assessment and monitoring in the pediatric intensive care unit (PICU) involve continuous and reliable evaluation of the adequacy of systemic perfusion and Do2 to select appropriate hemodynamic support strategies.

Cardiovascular Function

If one considers hemodynamic monitoring not only in terms of Do2 and perfusion pressure but also in terms of what therapy is required to produce a given level of tissue oxygenation, one gains a much better understanding of the overall condition of the patient. It is therefore important to monitor not only Do2 and perfusion pressure but also the quantity of therapy (QOT) needed to procure and maintain adequate tissue oxygenation. Consider two hypothetical 6-month-old infants, Destiny and Dakota, 2 hours after repair of tetralogy of Fallot. They have identical (and adequate) Do2 and perfusion pressure, but Destiny has a left atrial (LA) pressure of 6 mm Hg and a right atrial (RA) pressure of 8 mm Hg, whereas Dakota has received volume infusion to achieve an LA pressure of 6 mm Hg and an RA pressure of 15 mm Hg. Assuming that the levels of intravascular volume provided are exactly those needed to achieve the identical Do2 values and perfusion pressures, it is clear that the physiologies of these two patients are different. The clinician who has learned what to expect relative to the QOT in any given set of circumstances will find Dakota’s sufficient tissue oxygenation only mildly reassuring and asks: is there substantial residual right ventricular (RV) outflow

The function of the heart and vasculature is to deliver oxygen (O2) and other nutrients to various tissues in order to meet the metabolic demands of the organism. Mild to moderate depression of Do2 is normally compensated by augmented O2 extraction at the tissue level, thereby maintaining a stable level of oxygen consumption (Vo2). When Do2 falls below some critical level, this compensatory mechanism fails, and a state of O2 supply dependency exists3 such that any further drop in Do2 leads to a parallel fall in Vo2.4–6 Under a state of supply-dependent Vo2, affected tissues and organs attempt to maintain homeostasis partly through anaerobic metabolism. Several studies suggest that the initial metabolic response to hypoxemia or decreased Do2 differs between the newborn and older ages and varies among different vascular beds. In adults at rest, Do2 is in great excess of Vo2. This “O2 surplus” means that moderate reductions of O2 transport are generally well tolerated without compromise of Vo2. In contrast to the adult, the metabolism of the newborn may be particularly susceptible to modest alterations in O2 transport because of the high resting demands for

Quantity of Therapy

239

240

SECTION IV



Pediatric Critical Care: Cardiovascular

tract obstruction or another problem or problems that I need to know about? Will Dakota have sufficient Do2 in 10 hours when postbypass myocardial depression is at its worst? Might additional therapy secure adequate Do2 at a lower filling pressure, thereby minimizing adverse effects of systemic venous hypertension? As this example illustrates, the QOT concept is useful for three reasons: 1. The QOT is, in part, a function of the patient’s overall condition and can reflect anatomic or physiologic problems that require further exploration. 2. Physiologic trajectory is key, especially early in the course of certain illnesses or after cardiopulmonary bypass, when Do2 predictably declines over the first 12 hours.2 Taking into account the QOT relative to tissue oxygenation at any point in time helps one better estimate the likelihood of the need for augmented support (e.g., mechanical support of the circulation) as time passes. 3. Some therapies (e.g., fluid infusion to obtain high filling pressure or high airway pressure), while helpful at a point in time, can be pernicious over the longer run: high venous pressure, especially in infants, causes third spacing of fluid, and the effect of ventilator-associated lung injury on lung function can be devastating. By using high central venous or airway pressures, the intensivist is, in effect, incurring a debt to secure short-term perfusion that will have to be repaid later. Experienced clinicians always take the QOT into account in their work, which influences their level of concern about a patient and guides subsequent timing and choice in adjusting therapy. Almost any form of therapy might be included in the QOT concept. However, this chapter focuses on medical therapies that have the most important effect on hemodynamics, including Do2 and perfusion pressure. These therapies include inotropic/vasoactive agents, volume infusion, and airway pressures used during mechanical ventilation. The amount of inotropic and vasoactive drugs administered, assuming that they are used appropriately, seems to be a crude indicator of patient illness.2 Volume infusion to achieve adequate filling pressure is required even for the normal heart, but the QOT concept applies when higher-than-normal filling pressure is needed to maintain adequate CO. The consequences of high filling pressures are body edema (especially in infants), pleural and other cavity space effusions, and pulmonary edema. If high venous pressure is coupled with systemic hypotension (e.g., with a failing Fontan circulation), there may be critically reduced transtissue perfusion pressure with a potentially negative impact on cerebral and splanchnic perfusion. With respect to mechanical ventilation, the need for high mean airway pressure (Paw) is most commonly a reflection of lung disease, but pulmonary edema on a hydrodynamic basis may occasion the use of high Paw for optimal lung recruitment. High Paw can reduce venous return to the heart, increase pulmonary vascular resistance (PVR), and contribute to ventilator-associated lung injury.











Variables That Determine Tissue Oxygenation Tissue oxygenation is directly related to both Do2 and systemic arterial blood pressure (SAP). Do2, the quantity of O2 delivered to the tissues per minute, is the product of systemic blood flow (SBF), which equals CO except in patients with certain cardiac malformations, and arterial O2 content:

DO 2 ( mL min )  10  CO ( L min )  CaO 2 ( mL 100 mL blood )

where CO is cardiac output or SBF (in liters per minute or liters per minute per square meter) and Cao2 is quantity of O2 bound to hemoglobin plus the quantity of O2 dissolved in the plasma in arterial blood. The O2 content of arterial blood (mL O2/dL blood) equals:

(

)

CaO 2  SaO 2  Hbg [ g dL ]  1.36  ( PaO 2  0.003)

where Sao2 is arterial O2 saturation, Hgb is hemoglobin concentration (in grams per deciliter), 1.36 (constant) is the amount of O2 bound per gram of hemoglobin (mL) at 1 atm of pressure, Pao2 is arterial partial pressure of O2, and 0.003 (constant) multiplied by the Pao2 equals amount of O2 dissolved in plasma at 1 atm. The quantity of dissolved O2 is generally considered to be negligible in the normal range of Pao2. Hypoxia, because of poor gas exchange within the lungs (i.e., intrapulmonary shunt), or in the setting of CHD with right-to-left shunting, is an important determinant of blood O2 content. CO is the product of stroke volume (quantity of blood ejected per beat) and heart rate; SAP is determined by CO and systemic vascular resistance (SVR). The four primary determinants of cardiac function are preload (which determines the precontractile lengths of the myofibrils); end-systolic wall stress (function of systemic blood pressure and physical characteristics of the arterial system, ventricular wall thickness, and chamber dimension); myocardial contractility; and heart rate. These determinants of ventricular function can be altered by many factors in the intensive care setting. Preload, or end-diastolic volume, is affected by ventricular compliance (rate and extent of cardiomyocyte relaxation and cardiac connective tissue), intravascular volume, and intrathoracic pressure. Expansion of the heart resulting from transmural filling pressure, rather than the LA pressure per se, determines the force of contraction. Therefore, intrathoracic (or intrapericardial) pressure is a key determinant of preload. Ventricular hypertrophy, vasodilator and diuretic therapies, and positive pressure mechanical ventilation all adversely affect preload. Similarly, cardiac function is inversely related to afterload, or endsystolic wall stress. Anatomic obstructions and systemic or pulmonary hypertension may negatively affect ventricular systolic and diastolic function. Excessively fast or slow heart rates and inappropriately timed atrial contraction (relative to ventricular systole) may negatively affect ventricular filling or function. Finally, myocardial contractility is often negatively affected by the following factors: hypoxemia, acidosis, hypomagnesemia, hypocalcemia, hypoglycemia, hyperkalemia, cardiac surgery, sepsis, and cardiomyopathies.

Monitoring Tissue Oxygenation CO can be assessed qualitatively by physical examination and other modalities and quantitatively by a variety of techniques using invasive and noninvasive devices, laboratory data, and other clinical indicators. Do2 is easily derived if systemic blood flow can be measured, but it is only indirectly inferred if this information is lacking.

Qualitative Assessment of Cardiac Output Physical Examination The physical examination is often the initial and most common technique used to assess and monitor cardiovascular function. Significantly diminished CO may manifest as diminished peripheral pulses, cool or mottled extremities, and delayed capillary



CHAPTER 27 Assessment of Cardiovascular Function

refill. However, certain clinical signs of low CO may be unreliable depending on the particular diagnosis. For example, in the context of cardiac lesions associated with a large arterial pulse pressure (e.g., severe aortic insufficiency and aortopulmonary shunts), peripheral pulses may be increased despite low CO and reduced systemic Do2. Patients in septic shock are often peripherally vasodilated and warm despite hypotension and reduced tissue Do2. Central cyanosis from either cardiac or respiratory causes results from arterial O2 desaturation. In contrast, peripheral cyanosis results from vasoconstriction or low blood flow at the microcirculatory level. In some patients, cyanosis is a relatively subtle physical finding, particularly if the patient is anemic or has a dark complexion. Hydration status can be assessed by skin turgor, dryness of mucous membranes, and fullness of the anterior fontanel (in infants), but these manifestations of hydration status relate mostly to interstitial fluid and may poorly reflect intravascular volume, which must be directly measured. Cardiac auscultation for abnormal heart sounds—including valve clicks, rubs, gallops, and murmurs—may provide the first indication of a significant functional or structural cardiac abnormality, although these sounds do not directly reflect Do2. Unfortunately, the lack of a heart murmur, especially in low CO states, does not necessarily rule out a significant residual cardiac lesion. The presence of crackles on pulmonary auscultation, particularly in the older pediatric patient, may signify pulmonary edema. However, crackles are nonspecific and may be caused by lung disease or fluid overload in addition to disorders of cardiac structure or function. Finally, jugular venous distension and hepatomegaly are indicative of high right-sided filling pressures often associated with RV dysfunction.

Chest Radiography Although the chest radiograph is of little value in assessing a patient’s hemodynamic profile per se, it may be helpful for assessing certain aspects related to cardiovascular status. Provided that the chest radiograph is technically adequate, the clinician can assess heart size, contour and configuration, pulmonary vascularity, pleural effusions, lung parenchyma, and abdominal situs. Some of these findings, when abnormal, may help determine the etiology of cardiovascular dysfunction. Chronically increased pulmonary arterial pressure may be indicated by enlarged pulmonary arteries in the hila that radiate toward the periphery of the lung. Conditions that increase pulmonary blood flow (PBF, or Qp) at least twice normal also increase the size of the pulmonary arteries. Increased pulmonary capillary pressure may be inferred by the presence of pulmonary edema. The edema may present as a “fluffy” hilum but may have a more diffuse granular appearance in neonates. Pleural effusions may accompany pulmonary edema, particularly in conditions associated with poorly compensated congestive heart failure. Increased pulmonary venous markings are indicative of elevated pulmonary venous pressures of any cause, although usually from decreased left ventricular compliance or obstruction in the pulmonary veins or mitral stenosis. In the early postoperative period, the most important information obtained from the chest radiograph is (1) the positions of the endotracheal tube, chest tubes, and intracardiac lines; (2) the presence of extrapulmonary fluid or air; and (3) the presence of pulmonary edema. The cardiothoracic ratio gives a quantitative estimate of cardiac size, which is obtained by dividing the transverse measurement of the cardiac shadow in the posteroanterior view by the width of the thoracic cavity. Cardiomegaly is present if this value is greater than 0.5 in adults and 0.6 in infants.

241

Although useful for assessing left ventricular (LV) enlargement, the cardiothoracic ratio is not as sensitive to RV enlargement. RV enlargement results in lateral and upward displacement of the cardiac apex on the posteroanterior view and filling of the retrosternal space on the lateral view. It is perhaps more important to know what the chest radiograph may not reveal; significant cardiac problems, such as constrictive pericarditis, acute fulminant myocarditis, and even acute pericardial tamponade are often associated with a normal-sized heart on a chest radiograph.

Quantitative Assessment of Cardiac Output Quantitative measures of CO in the ICU can be obtained by a variety of techniques, including the Fick method, thermodilution, dye dilution techniques (now rarely used), and Doppler echocardiography. Each of the first three methods applies a similar principle of dilution of an indicator: O2, cold, or indocyanine green dye, respectively. The change in concentration of a substance is proportional to the volume of blood in which it is being diluted. In general, thermodilution is the method most widely used in the intensive care setting. However, for conditions of low CO, the Fick method is more reliable than the thermodilution or dye dilution techniques. Conversely, the Fick method is less accurate in conditions of high systemic blood flow because of difficulty in measuring narrow arteriovenous O2 differences in the blood.

Thermodilution Technique The thermodilution technique requires use of a specialized pulmonary artery (PA) catheter. CO is calculated by injecting a known volume of iced water or saline solution into the right atrium (proximal catheter port) and measuring the temperature change at the catheter tip in the PA. CO is calculated by the following equation: CO  1.08  Vi ( Tb  Ti ) Tb ( t ) dt

where Vi is injectate volume (mL), Tb is temperature of blood, Ti is injectate temperature, and Tb(t)dt is area under the curve. In general, thermodilution CO measures are performed using a completely automated system, and the calculations are performed by a computer. This method (as with any indicator dilution method) requires complete mixing; thus, it is most accurate in situations in which a mixing chamber is located proximal to the thermistor. It is generally used only in patients who do not have intracardiac or great vessel–level shunts or an insufficient valve between the injection site and sampling site. The injection must be made rapidly because a slow injection will give a falsely elevated CO. Possible sources of error with this method include inaccurate measurement of the volume of injectate or temperature of the blood or injectate, close approximation of the thermistor to a vessel wall, and inadequate mixing, as is sometimes seen in venous systems with low flow.

Fick Method According to the Fick principle, CO equals O2 consumption divided by the arteriovenous O2 content difference:

CO  VO 2 ( CaO 2  CvO 2 )

where Vo2 is O2 consumption (in milliliters per minute) and Cao2 and Cvo2 are arterial and venous O2 content (mL O2/100 mL blood), respectively. Care must be taken to select the appropriate sampling site for a true mixed venous blood sample. With a normal heart, the best site to obtain a mixed venous sample is within

242

SECTION IV



Pediatric Critical Care: Cardiovascular

the PA. If a left-to-right shunt is present, however, the mixed venous site should be the cardiac chamber proximal to the site of the shunt. When a site other than the PA is used for the mixed venous site, the resultant value for arteriovenous O2 difference is a less reliable reflection of the absolute CO, but it can be used for serial observations and for monitoring response to therapy over time. Because measuring Vo2 in the intensive care setting requires special equipment and is somewhat cumbersome, the arteriovenous O2 difference is often used as an indirect measure of CO. A wide arteriovenous O2 difference generally reflects a low CO and indicates a large O2 extraction by the tissues, whereas a narrow arteriovenous O2 difference usually reflects a high CO. (Note that it is the arteriovenous content difference, not O2 saturation difference, that matters; with anemia, the O2 saturation difference may be wider than normal despite normal CO.) Unfortunately, studies suggest that Vo2 is quite variable for any individual patient in an intensive care setting.9 Furthermore, mixed venous O2 saturation (and, hence, arteriovenous O2 difference) may be misleading in patients with decreased tissue O2 extraction, as can be seen with septic shock.10,11

Doppler Echocardiography Doppler techniques can be used to measure CO using the mean velocity of systolic flow, heart rate at the time of measurement, and cross-sectional area of the artery in which measurements are being made (usually the ascending aorta): CO    V  HR where A is the area of the orifice, V is integrated flow velocity, and HR is heart rate. To determine the integrated flow velocity, the area under the Doppler curve must be measured. The area of the aortic orifice is commonly obtained by measuring the aortic diameter from the two-dimensional image, where A 5 0.785 ∞ d2. This technique requires special care for accurate Doppler interrogation of blood flow and is seldom used in the critical care unit.

Pulse Oximetry Pulse oximetry measures the quantity of hemoglobin saturated with O2 in peripheral arterial blood. It depends on two principles: (1) oxygenated and reduced hemoglobin have different absorption spectra; and (2) at constant light intensity and hemoglobin concentration, O2 saturation of hemoglobin is a logarithmic function of the intensity of transmitted light (Beer-Lambert law). Two wavelengths of light that have different absorption spectra for reduced hemoglobin and oxyhemoglobin are transmitted from the light-emitting diodes through the arterial bed. Light absorption at the two wavelengths is compared, yielding the ratio of oxyhemoglobin to reduced hemoglobin, or the O2 saturation. Pulse oximeters have a high potential for error at saturations below 80%.12 Furthermore, the O2 dissociation curve flattens out at the high range so that at saturations greater than 95%, large changes in Pao2 accompany small changes in saturation. This phenomenon should be kept in mind when monitoring premature infants, for whom it is important to avoid hyperoxia.

Other Measures of Oxygen Delivery Acid-Base Status When tissue hypoxia occurs and affected tissues and organs resort, in part, to anaerobic metabolism, increased production of lactate, carbon dioxide (CO2), and hydrogen ions occurs. The anion gap, the difference in unmeasured serum anions and unmeasured

serum cations, can yield information regarding the cause of metabolic acidosis. If the anion gap is normal (8–16 mEq/L), loss of bicarbonate has occurred, usually via the kidneys or gastrointestinal tract, or rapid dilution of the extracellular fluid has occurred.

Blood Lactate Blood lactate concentration is a laboratory measure that indirectly reflects perfusion.13,14 Blood lactate measurements are extensively used for monitoring and evaluating response to therapy. It has been demonstrated that initial absolute blood lactate levels are less important than the temporal trend in lactate concentrations for predicting morbidity and mortality in postoperative cardiac patients.15,16 Unfortunately, the specificity of blood lactate is imperfect and may lack sensitivity for detecting supply-dependent O2 consumption, particularly if it is only regional. In addition, blood lactate depends on hepatic metabolism and the rate of production and clearance. It may also be elevated in cases of severe hypoxic ischemic brain injury or in advanced bowel ischemia. Serum Biomarkers In recent years, there has been an increased interest in the measurement and use of serum biomarkers as an assessment of cardiac, renal, and neurologic function in critical illness. The most commonly used cardiac-specific serum biomarkers in the assessment of cardiovascular health are troponin and B-type natriuretic peptide (BNP).17 Troponin release is specific to myocardial injury. therefore, it may be elevated in patients with myocarditis, pericarditis, coronary injury or occlusion, and sepsis. After cardiac surgery, troponin levels can be elevated due to myocardial injury as a result of cardiopulmonary bypass and aortic cross-clamping. Therefore, its utility in this circumstance is unclear unless there is a specific concern about coronary blood flow. BNP is released in response to ventricular wall stress due to volume or pressure overload. High levels of circulating BNP have been correlated with congestive heart failure states—a trend over time is likely the most helpful for the clinician. Other biomarkers to measure the health of the kidneys and brain also hold promise as potentially important tools to assess these systems noninvasively. Gastric Tonometry Gastric tonometry, a technique available for clinical use in adult and some pediatric ICUs, allows indirect assessment of perfusion by measuring gut intramucosal pH or partial pressure of carbon dioxide (Pco2).18,19 It may have an advantage over blood lactate concentration in that it can uncover regional hypoxia and hypoperfusion involving the gut and can be adapted for continuous online measurement.19 Nonetheless, this technique assumes that a critical reduction in O2 transport manifests in the splanchnic circulation before it can be detected systemically (probably a reasonable assumption), and tonometric methods are not entirely noninvasive. Urine Output Urine output generally reflects CO, but oliguria may occur in the first 24 hours after open heart surgery, especially in neonates, even in the context of good CO and blood pressure. Thus, it is important to consider urine output in the context of other indicators of organ perfusion and not as an isolated variable. It should also be noted that the kidneys are quite sensitive to perfusion pressure and that good systemic blood flow coupled with low systemic arterial pressure (due to low SVR) may adversely affect urine output more than other measures of tissue perfusion.



CHAPTER 27 Assessment of Cardiovascular Function

Near-Infrared Spectroscopy Near-infrared spectroscopy (NIRS), a noninvasive technique that has been applied to assess systemic and regional O2 transport in several clinical and laboratory studies,20–28 is now commonly used, particularly in patients with CHD, as a means of trending regional Do2 or as a surrogate for mixed venous O2 saturation or systemic Do2.29 Retrospective studies in patients with CHD have shown associations with lower NIRS measurements and acute kidney injury,30,31 risk of necrotizing enterocolitis,32 and postoperative mortality in infants with single-ventricle physiology.33 Abdominal site NIRS has been shown to correlate with simultaneous intramucosal pH measurements by gastric tonometry in neonates and infants with CHD undergoing catheter-based or surgical intervention.34 In an experimental setting in which Do2 is controlled, NIRS has been used to correlate cytochrome aa3 (the terminal link in the electron transport chain responsible for mitochondrial respiration), Vo2, and lactate flux.25 Thus, NIRS has the potential to identify a critical regional reduction in O2 transport at the cellular level.

Systemic Arterial Blood Pressure Invasive Blood Pressure Monitoring Intravascular pressure monitoring is often essential in the management of critically ill neonates and infants in the ICU. Typically, an end-hole catheter is inserted into a vessel (or cardiac chamber) and connected to a pressure transducer by a coupling system composed of fluid-filled extension tubing, a stopcock for withdrawing blood and balancing the transducer to atmospheric pressure, and a continuous infusion device to flush out blood and air. The transducer translates pressure into an electrical signal that can be processed through a preamplifier into a waveform and numerical display on a monitor. The pressure transducer must be properly calibrated, dampened, and positioned (at mid-chest level). Inaccurate measurements can occur for a variety of reasons. In the pediatric population, blood pressure is age dependent and is a relatively insensitive marker of CO and Do2. Because blood pressure is the product of CO and SVR, hypotension may result from diminished CO and/or decreased SVR.35 Because the treatment options are different, distinguishing low CO from low SVR is important.

Noninvasive Blood Pressure Monitoring The auscultatory method of blood pressure measurement with a cuff and pressure gauge is difficult if access to the patient is limited, if the patient is small or uncooperative, and when frequent recordings are required. Therefore, two techniques, Doppler and oscillometric measurements, have been developed. The Doppler technique uses a Doppler ultrasound probe that is applied to the radial or brachial artery. A cuff wrapped around the upper arm is inflated until the audible Doppler signal is obliterated and then deflated until the signal first becomes audible again (systolic blood pressure). This method has been validated in low-flow states and in small children.36 The oscillometric method has the advantage of being readily automated. The device for indirect noninvasive mean arterial pressure (Dinamap, GE) is based on the principle that blood flow through a vessel produces oscillation of the arterial wall that may be transmitted to an inflatable cuff encircling the extremity. As cuff pressure decreases, a characteristic change occurs in the magnitude of oscillation at the levels at which systolic, diastolic, and mean pressures are registered. Accuracy of Dinamap blood pressures has been validated in children, and it

243

correlates well with direct intravascular radial artery pressures.37 The accuracy of these two techniques relates to the cuff size. If the cuff is too narrow, the pressure recorded may be erroneously high; if the cuff is too wide, the pressure recorded may be underestimated. Both techniques are unreliable and inadequate in patients with low CO, hypotension, dysrhythmias, significant edema, or systemic vasoconstriction.

Central Venous or Intracardiac Pressure Monitoring Pressures can also be measured in the cardiac chambers or in the pulmonary vasculature. However, the necessity for intravascular or intracardiac lines should always be carefully considered and should be removed as soon as the clinical condition permits. The placement of relatively large catheters in small vessels for prolonged periods carries a risk of thrombosis and systemic thromboembolism. Central venous access affords the opportunity to measure central venous pressure (CVP), deliver drugs or high-osmolarity nutritional solutions, and repeatedly sample blood to monitor venous O2 saturations and for other laboratory studies. Intraarterial lines offer the opportunity to continuously monitor arterial pressure and for intermittent blood gas analysis. Intravascular pressures provide information about ventricular preload and afterload. RV preload is assessed by the CVP. The CVP is determined by a variety of factors, including patient age, preoperative status (i.e., a patient with RV hypertrophy and increased RA pressure), cardiac performance, intrathoracic pressure, blood volume, vasopressor therapy, and status of the pericardium. The CVP a wave reflects atrial contraction; the v wave reflects atrial filling. Serial measurements of CVP are frequently used to evaluate the response to fluid administration. RV afterload can be assessed using a PA catheter. This catheter is particularly important for monitoring PA pressure and therapeutic response to vasodilators in patients with elevated PVR. The PA wedge pressure reflects LA pressure (in the absence of pulmonary vein stenosis). In the postoperative cardiac patient, a direct LA line can be placed to directly assess LV preload. LV afterload is assessed by measurement of SAP, provided that no LV outflow tract obstruction is present.

High-Frequency Physiologic Data Capture and Streaming Analytics Given the large amount of data that intensivists are presented with on a moment-to-moment basis, there has been recent interest in the use of software and computer algorithms to collect and display the relevant data in real time. In addition, there is ongoing and promising research into the use of streaming analytics and machine learning to help clinicians predict which patients may be at higher risk of certain clinical outcomes, thus enhancing opportunities for timely decisions and therapeutic or preventive interventions.38,39

Assessing Variables That Affect the Quantity of Therapy If it is important to take into account the QOT needed to secure adequate tissue perfusion, it follows that one would like to assess the variables that affect the QOT. Table 27.1 summarizes the effect of abnormalities of cardiovascular and pulmonary function on QOT. What follows is a brief description of how these cardiovascular variables may be assessed in the critical care unit.

244

SECTION IV



Pediatric Critical Care: Cardiovascular

TABLE Cardiovascular Function and Quantity 27.1 of Therapy (QOT)

Cardiac System Variable

Impact on QOT

Cardiac Function g Ventricular systolic function

h Filling pressure, h inotropic/pressor support

g Ventricular diastolic function

h Filling pressure, h pressor support

Abnormal rhythm

h Filling pressure, h inotropic/pressor support

Intracardiac structural lesions (g efficiency)

h Filling pressure, h inotropic/pressor support

Single ventricle with A-P shunt (g efficiency)

h Filling pressure, h inotropic/pressor support

Peripheral Vasculature h SVR

h Ventricular work n h filling pressure, h inotropic/pressor support

g SVR

h Filling pressure, h inotropic/pressor support

h PVR 2 ventricles

h Ventricular work n h filling pressure, h inotropic/pressor support

Aortopulmonary shunt

gPBF n g O2 n h filling pressure, h inotropic/pressor support

Bidirectional Glenn

h SVC pressure

Fontan

h Filling pressure, h inotropic/pressor support

Vascular Function “Leaky” vascular bed n edema n volume infusion n edema

h Filling pressure, h inotropic/pressor support

Pulmonary Function g Lung compliance, g gas exchange

h Airway pressure n g venous return n h systemic venous pressure, barotrauma

A-P, Aortopulmonary; O2, oxygen, PBF, pulmonary blood flow; PVR, pulmonary vascular resistance; SVR, systemic vascular resistance.

Ventricular Systolic Function Precise measurement of ventricular systolic function is difficult,40 especially in the critical care setting. A commonly used surrogate is the echocardiographic demonstration of ventricular wall excursion/ shortening. LV shortening and ejection fractions can be measured using echocardiography with reasonable accuracy, but these variables are influenced by preload and afterload and by inotropic conditions. RV morphology makes echocardiographic measurement of systolic function even more problematic despite a variety of described techniques to assess this variable.41 For either ventricle, most often one resorts to a qualitative assessment of ventricular wall excursion as a crude estimate of systolic function. Although cardiac magnetic resonance angiography/magnetic resonance imaging (MRI) can accurately measure LV and RV ejection fraction, it is often of limited practical utility in the critically ill pediatric patient.

Ventricular Diastolic Function Ventricular diastolic function is also difficult to measure precisely. In the critical care unit, diastolic dysfunction is usually manifested as a need for increased filling pressures for a given magnitude of ventricular output. Echocardiography sometimes will demonstrate an apparently underfilled ventricle despite adequate or high filling pressures, but most often a lack of compliance is inferred from high filling pressures alone. Echocardiographic measures of ventricular compliance exist, but many clinicians have found them to be of limited practical value. It is important to emphasize that transmural filling pressure, not atrial pressure per se, is what determines diastolic filling; elevated pericardial or intrathoracic pressure will reduce ventricular filling for any given atrial pressure.42

Rhythm Disturbance A variety of abnormal rhythms can decrease systemic perfusion and therefore lead to increased QOT. A surface electrocardiogram may be sufficient to delineate the type and mechanism of an arrhythmia; however, especially with tachycardia, all too often one cannot clearly discriminate P waves from the T and QRS deflections. The use of atrial leads in conjunction with limb leads can be exceedingly helpful for both diagnosing and treating arrhythmias in postoperative patients. Alternatively, esophageal electrodes sometimes can be helpful, although they are somewhat cumbersome to use (and cannot always affect atrial capture). It is important to frequently and carefully reassess the rhythm because significant changes (e.g., from sinus rhythm to junctional ectopic tachycardia) may escape casual detection.

Abnormal Systemic Vascular Resistance SVR is determined by the following equation: SVR  ( SAP  CVP ) CO

where SAP is mean systemic arterial pressure (mm Hg), CVP is mean central venous pressure (mm Hg), and CO is cardiac output, usually indexed to surface area (in liters per minute per square meter). Increased SVR can be useful when CO is insufficient for adequate systemic perfusion pressure with normal SVR. On the other hand, SVR increased beyond that needed for adequate SAP increases systemic ventricular afterload and therefore may negatively affect CO.43 For reasons discussed in the following section on single-ventricle physiology, increased SVR may also result in excess PBF in patients with an aortopulmonary shunt. Finally, increased SAP in a newly postoperative patient may contribute to excessive bleeding. In contrast, low SVR can cause systemic hypotension despite adequate or supranormal CO. Anecdotal observations and some published information indicate that low SVR may occur after cardiac surgery as well as with other systemic illnesses (e.g., sepsis). As previously noted, because CO is infrequently measured in PICUs, SVR is most commonly inferred from observation of cutaneous perfusion and SAP. Indeed, it is important to evaluate systemic hypotension in the context of cutaneous perfusion (brisk capillary refill suggests low SVR) because rational therapy for decreased SVR with adequate CO (vasopressor support) is quite different from that useful for hypotension due to inadequate CO.



CHAPTER 27 Assessment of Cardiovascular Function

Increased Pulmonary Vascular Resistance The clinical consequences of increased PVR are directly related to the specific cardiac anatomy and physiology. With two separate ventricles, high PVR can reduce systolic and diastolic function of the pulmonary ventricle and limit its output. In patients with a physiologically large aortopulmonary shunt, increased PVR can be useful up to a point because it reduces what would otherwise be excessive PBF. On the other hand, if PVR is too high, or in the setting of an excessively restrictive aortopulmonary shunt, inadequate PBF results. With a bidirectional Glenn circulation, elevated PVR may result in upper body congestion and hypoxemia. With a Fontan circulation, high systemic venous pressure, low CO, edema, high chest tube output, and hypoxemia (if a fenestration is present) may occur. In patients with a structurally normal cardiovascular system, measuring PVR is analogous to measurement of SVR and is subject to the same practical difficulties. Echocardiographic estimation of RV pressure is a useful surrogate, using a tricuspid regurgitant jet, pulmonary regurgitant jet, or interventricular septal position. Unfortunately, in the absence of significant tricuspid regurgitation (or another defect that allows a pressure gradient to be measured between the right ventricle and a chamber of known pressure), echocardiographic estimation of RV pressure is crude. For patients with an aortopulmonary shunt, PVR is rarely measured in the ICU; determining PBF requires measuring Vo2 (an assumed Vo2 is questionable because it is so variable9), pulmonary arterial O2 saturation, and pulmonary venous O2 saturation. Because pulmonary venous catheters are rarely used, pulmonary venous O2 saturations are usually assumed. However, this is a potential source of significant error because these saturations are variable and unpredictable.44 Most important, pulmonary artery pressure is essentially never measured in the critical care unit in patients with shunts. For these patients, even estimating PVR is problematic because many variables (e.g., PVR, systemic blood pressure, PBF and CO, hematocrit, and Vo2) influence the most obvious manifestation of increased PVR, low systemic arterial O2 saturation. Circumstantial data may be used to infer that PVR is not elevated in hypoxemic patients with an aortopulmonary shunt; for example, echocardiographic demonstration of narrowing of the shunt suggests that increased PVR is likely not the cause of the hypoxemia. Alternatively, an increase in systemic arterial O2 saturation with inhaled nitric oxide suggests that baseline PVR is increased. Measuring PBF in patients with a cavopulmonary palliation (e.g., bidirectional Glenn, hemi-Fontan, and Fontan) can also be done using the Fick method (thermodilution cannot be used to measure PBF because of inadequate mixing of the cold indicator in the systemic venous pathway). Measurement of the ratio of pulmonary to systemic blood flow (Qp:Qs) can be calculated as the arterial saturation minus the inferior vena cava saturation divided by the pulmonary venous saturation minus the inferior vena cava saturation.45 From a practical standpoint, increased PVR in this setting is often inferred from high systemic venous pathway pressures (superior vena caval pressures in a bidirectional Glenn patient), taking into consideration possible anatomic obstruction in the superior vena cava, pulmonary arteries, or pulmonary veins or increased systemic ventricular end-diastolic pressures.

structural lesions are (1) increased ventricular afterload (e.g., RV or LV outflow tract obstruction); (2) increased ventricular volume load (e.g., ventricular septal defect, excessive PBF through a surgical shunt or patent ductus, and atrioventricular valve regurgitation); (3) impaired ventricular filling (e.g., atrioventricular valve stenosis); (4) reduced PBF with shunting into the systemic circulation; (5) mixing of pulmonary and systemic venous blood; and (6) D-transposition of the great arteries physiology (i.e., pulmonary venous return predominantly directed to the PA and systemic venous return predominantly directed to the aorta). Impaired coronary perfusion resulting in ventricular ischemia is perhaps not, conceptually, a problem of efficiency but a structural lesion that needs to be considered. Many patients have some combination of these lesions. It is beyond the scope of this chapter to discuss the evaluation of cardiac patients relative to structural lesions and their impact on Do2, systemic perfusion pressure, and QOT. Suffice it to say that it is exceedingly important that the anatomy and associated physiology of patients with cardiac malformations be well defined. Echocardiography is the single most useful modality for delineating cardiac structure (and often even physiology) in the critical care unit. Cardiac catheterization and angiography remain important diagnostic and therapeutic tools, and MRI and computed tomography are sometimes helpful.

Vascular Integrity By vascular integrity, we refer to the ability of the vascular (mostly microvascular) bed to keep fluid in the intravascular space. Leaky blood vessels result in organ, chest wall, and peripheral edema, as well as fluid accumulation in the thoracic and abdominal cavities. This situation is exacerbated by high CVP, particularly when ventricular diastolic dysfunction exists, and tends to be selfperpetuating, especially in infants (Fig. 27.1). It is important to assess third spacing, particularly in infants, where opening the chest may help minimize the hemodynamic effects of chest wall edema in post–cardiac surgery patients.

↑ Tissue edema

Volume infusion

↑ CVP (persistent or transient)

↑ Chest wall stiffness

Hypotension

↑ Intrathoracic pressure ↓ Decreased venous return

Inefficient Circulation • Fig. 27.1 pressure.





The most important fundamental ways that structural defects can result in an inefficient circulation in patients with cardiac

245

​Effect of compromised vascular integrity. CVP, Central venous

246

SECTION IV



Pediatric Critical Care: Cardiovascular

Similarly, assessment for abdominal compartment syndrome due to ascites may allow surgical or catheter-based evacuation with improved pulmonary mechanics and urine output. Also, because progressive edema, even if relatively benign early on, is likely to eventually become a significant problem, this finding should figure prominently in the clinician’s overall assessment of the patient’s condition. There is currently rapidly evolving interest in the role that lymphatic abnormalities may play in chronic effusions, protein losing enteropathy, and plastic bronchitis in Fontan (and other) patients.46,47 MRI-guided, invasive interventions on the large lymphatic ducts can provide relief of such manifestations, and it seems likely that such interventions (and possibly modification of lymphatic function by medications) will play an important role in cardiac critical care in the future.

DO2 = CaO2 × SBF

CaO2 = CPVO2

1) PVO2 sat 2) Hgb

Pulmonary Function Pulmonary dysfunction due to edema or acute or chronic lung injury can be a major physiologic liability for the obvious reasons related to impaired gas exchange. In addition, insofar as increased Paw is required for adequate gas exchange, venous return to the heart may be impaired. This can be especially important in special circumstances, such as the patient after cavopulmonary palliation. Williams and colleagues48 nicely showed the unfavorable impact of small increments of positive end-expiratory pressure in patients after Fontan palliation, which, as careful inspection of their data reveals, was mostly due to decreased venous return. Increased Paw, especially if it causes overinflation of the lungs, also can increase PVR. Finally, increased Paw, if applied for sufficient duration, can take a long-term toll by chronic reduction in lung function. It is beyond the scope of this chapter to describe all available techniques for evaluating lung function. However, high Paw is an important component of QOT and should lead the intensivist to consider alternatives (e.g., permissive hypercapnia or extracorporeal life support).

3) SBF = CO

• Fig. 27.2

​Variables that determine systemic oxygen delivery (Do2) with a normal heart. Blue dots depict desaturated (systemic venous) blood; red dots depict fully saturated (pulmonary venous) blood. Dissolved O2 in the blood is ignored. Cao2, Systemic arterial blood O2 content; CO, cardiac output; Cpvo2, pulmonary venous blood O2 content; Hgb, blood hemoglobin concentration; Pvo2 sat, pulmonary venous O2 saturation; SBF, systemic blood flow. DO2 = CaO2 × SBF CaO2

CPVO2

CMVO2

Physiology of the Patient With a Single Ventricle Patients with single-ventricle physiology differ from children with two functioning ventricles in many ways. They pose several unique challenges to the pediatric intensivist. For many patients with a single ventricle, initial palliation in infancy may involve placement of an aortopulmonary (e.g., modified Blalock-Taussig) or right ventricle to PA (Sano) connection as a source of PBF. The total output of the single-ventricle circulation is thus the sum of the pulmonary (Qp) and systemic (Qs) blood flows. The relative percentage of blood flow to the pulmonary and systemic circulations depends, in part, on the resistance in each vascular bed. Because PVR is usually substantially lower than SVR soon after birth, the size (diameter, length, and vessel of origin) of the aortopulmonary shunt is also a major contributor to total resistance to flow in the pulmonary circuit and, hence, an important determinant of Qp/ Qs. SVR is often the single most important variable influencing Qp,49 and postoperative afterload reduction has been shown to improve outcomes and markers of Do2.50,51 In patients with a single ventricle who have complete admixture of systemic and pulmonary venous blood, arterial O2 saturation is influenced by not only lung function (e.g., pulmonary venous O2 saturation) but also Qp and myocardial function (which influences Qs and, hence, mixed venous O2 saturation; Figs. 27.2 and 27.3).





3) VO2 DO2 sat

1) PVO2 sat 2) Hgb

4) Qp : Qs Rp : Rs

SBF = 5) CO – PBF Qp : Qs

• Fig. 27.3

​Variables that determine systemic oxygen delivery (Do2) with a cardiac malformation resulting in complete mixing of systemic and pulmonary venous blood. Blue dots depict desaturated (systemic venous) blood; red dots depict fully saturated (pulmonary venous) blood. Dissolved O2 in the blood is ignored. Cao2, Systemic arterial blood O2 content; Cmvo2, systemic venous O2 content; CO, cardiac output; Cpvo2, pulmonary venous blood O2 content; Hgb, blood hemoglobin concentration; Pvo2 sat, pulmonary venous O2 saturation; PBF, pulmonary blood flow; Qp:Qs, ratio of pulmonary to systemic blood flow; Rp:Rs, ratio of pulmonary to systemic vascular resistance; SBF, systemic blood flow; Vo2, total body O2 consumption.  





CHAPTER 27 Assessment of Cardiovascular Function

Computer modeling of shunt-dependent single-ventricle physiology49,52 suggests that a Qp/Qs ,1.0 is ideal for optimizing systemic O2 availability for a given pulmonary venous O2 saturation, CO, and Vo2. The arterial O2 saturation, considered in isolation, is a poor indicator of Qp/Qs because a low mixed venous O2 saturation will depress arterial O2 saturation, even in patients with a high Qp/Qs. In a patient with left-to-right atrial shunting, accurately measuring Qp/Qs, which equals systemic arterial O2 saturation minus systemic venous O2 saturation divided by pulmonary venous O2 saturation minus pulmonary arterial O2 saturation, requires determining O2 saturation in blood in the proximal superior vena cava (SVC), as distinct from the RA or inferior vena cava, and is subject to the previously noted variability and unpredictability of pulmonary venous O2 saturations. That said, estimating Qp/Qs using the SVC O2 saturation (as mixed venous) and the arterial O2 saturation (which is also the pulmonary artery O2 saturation) and assuming the pulmonary venous O2 saturation is helpful in estimating whether a patient with a single ventricle and an aortopulmonary shunt has appropriate (i.e., associated with optimal systemic Do2), increased, or decreased Qp/Qs. However, it is important to note that Qp/Qs is merely a number when considered in isolation. It must be placed into the context of the patient’s overall status considering the parameters previously outlined for assessing systemic Do2. In particular, in patients following singleventricle palliation, a progressive decline in the serum lactate concentration (regardless of the initial postoperative concentration) is a fairly sensitive and specific marker for early survival. In contrast, rising lactate levels are generally a robust predictor of early postoperative cardiovascular collapse or need for mechanical support unless therapy can improve the hemodynamic picture.15,16 It should be noted that the physiology is somewhat different in patients with two ventricles and an aortopulmonary shunt (e.g., tetralogy of Fallot with severe RV outflow tract obstruction and a modified Blalock-Taussig shunt). Because PBF, is usually made up of both systemic arterial blood and systemic venous blood, arterial O2 saturation is higher for a given amount of PBF, and Qp:Qs cannot be calculated using systemic arterial O2 saturation as the pulmonary artery O2 saturation. The cardiopulmonary physiology of patients with bidirectional Glenn palliation differs somewhat from that of others as there is a paradoxical relationship between alveolar ventilation and arterial O2 saturation53,54 whereby PBF (and therefore arterial O2 saturation) are primarily affected by the quantity of cerebral blood flow (CBF). Hypoventilation with associated increases in Pco2 leads to increases in CBF, PBF, and arterial O2 saturation (as long as the hypoventilation is not associated with severe acidosis or atelectasis and therefore decreased pulmonary venous saturations). Two other additional points are worth noting. First, because arterial O2 saturations often increase significantly during the first several postoperative hours after Glenn palliation, lower than desired but acceptable O2 saturations early on do not necessarily imply inadequate palliation. Second, although some degree of upper body edema and duskiness is not unusual soon after an operation, marked upper body congestion suggests the possibility of obstruction of the SVC to PA pathway, which requires prompt evaluation. Echocardiography may be sufficient to interrogate this pathway, although angiography may be required.

247

Evaluation of postoperative Fontan patients is much the same as for other postoperative cardiac patients. However, because Fontan patients are particularly sensitive to factors that impede transit of blood across the lungs and into the ventricle, it is important to identify any such factors, especially remediable ones, early on in the struggling patient. Anatomic abnormalities that might have little clinical importance in other circumstances (e.g., partial obstruction of one or more pulmonary veins) can have a marked impact on the early postoperative Fontan patient. The same goes for modestly increased PVR and mildly decreased ventricular compliance. Catheters in the central systemic veins and left atrium are useful for estimating PVR and ventricular compliance, and the previously noted measures of systemic perfusion are helpful. The sick postoperative Fontan patient may pose the perfect storm of marginal SAP, acutely elevated CVP (relative to preoperative CVP), and hypoxemia (from right-to-left shunting of highly desaturated systemic venous blood through a fenestration), all complicated by the use of inotropic agents (which increase myocardial and total body Vo2). The experienced intensivist will consider these multiple variables, including the QOT, in the aggregate when evaluating the patient.

Key References Appelbaum A, Blackstone EH, Kouchoukos NT, et al. Afterload reduction and cardiac output in infants early after intracardiac surgery. Am J Cardiol. 1977;39:445-451. Bradley SM, Simsic JM, Mulvihill DM. Hypoventilation improves oxygenation after bidirectional superior cavopulmonary connection. J Thorac Cardiovasc Surg. 2003;126:1033-1039. Chang AC, Kulik TJ, Hickey PR, et al. Real-time gas-exchange measurement of oxygen consumption in neonates and infants after cardiac surgery. Crit Care Med. 1993;21:1369-1375. Hoffman, GM, Tweddell JS, Ghanayem NS, et al. Alteration of the critical arteriovenous oxygen saturation relationship by sustained afterload reduction after the Norwood procedure. J Thorac Cardiovasc Surg. 2004;127(3):738-745. Migliavacca F, Pennati G, Dubini G, et al. Modeling of the Norwood circulation: effects of shunt size, vascular resistances, and heart rate. Am J Physiol Heart Circ Physiol. 2001;280:H2076-H2086. Park MK, Menard SM. Accuracy of blood pressure measurement by the Dinamap monitor in infants and children. Pediatrics. 1987;79: 907-914. Scheinman MM, Brown MA, Rapaport E. Critical assessment of use of central venous oxygen saturation as a mirror of mixed venous oxygen in severely ill cardiac patients. Circulation. 1969;40:165-172. Schumacher KR, Reichel RA, Vlasic JR, et al. Rate of increase in serum lactate level risk-stratifies infants after surgery for congenital heart disease. J Thorac Cardiovasc Surg. 2014;148(2):589-595. Schwartz S, Frantz RA, Shoemaker WC. Sequential hemodynamic and oxygen transport responses in hypovolemia, anemia, and hypoxia. Am J Physiol. 1981;241:H864-H871. Tortoriello TA, Stayer SA, Mott AR, et al. A noninvasive estimation of mixed venous oxygen saturation using near-infrared spectroscopy by cerebral oximetry in pediatric cardiac surgery patients. Paediatr Anaesth. 2005;15:495-503.

The full reference list for this chapter is available at ExpertConsult.com.

e1







































































































1. Court O, Kumar A, Parrillo J, et al. Clinical review: myocardial depression in sepsis and septic shock. Crit Care. 2002;6:500-508. 2. Wernovsky G, Wypij D, Jonas RA, et al. Postoperative course and hemodynamic profile after the arterial switch operation in neonates and infants: a comparison of low-flow cardiopulmonary bypass and circulatory arrest. Circulation. 1995;92:2226-2235. 3. Schlictig R. Oxygen delivery and consumption in critical illness. In: Civetta J, Taylor R, Kirby RR, eds. Critical Care. 2nd ed. Philadelphia, PA: Lippincott-Raven; 1997. 4. Adams RP, Dieleman LA, Cain SM. A critical value for O2 transport in the rat. J Appl Physiol Respir Environ Exerc Physiol. 1982;53: 660-664. 5. Cain SM. Oxygen delivery and uptake in dogs during anemic and hypoxic hypoxia. J Appl Physiol Respir Environ Exerc Physiol. 1977;42: 228-234. 6. Schwartz S, Frantz RA, Shoemaker WC. Sequential hemodynamic and oxygen transport responses in hypovolemia, anemia, and hypoxia. Am J Physiol. 1981;241:H864-H871. 7. Lister G, Walter TK, Versmold HT, et al. Oxygen delivery in lambs: cardiovascular and hematologic development. Am J Physiol. 1979;237: H668-H675. 8. Klopfenstein HS, Rudolph AM. Postnatal changes in the circulation and responses to volume loading in sheep. Circ Res. 1978;42: 839-845. 9. Jain A, Shroff SG, Janicki JS, et al. Relation between mixed venous oxygen saturation and cardiac index: nonlinearity and normalization for oxygen uptake and hemoglobin. Chest. 1991; 99:1403-1409. 10. Scheinman MM, Brown MA, Rapaport E. Critical assessment of use of central venous oxygen saturation as a mirror of mixed venous oxygen in severely ill cardiac patients. Circulation. 1969;40: 165-172. 11. Chang AC, Kulik TJ, Hickey PR, et al. Real-time gas-exchange measurement of oxygen consumption in neonates and infants after cardiac surgery. Crit Care Med. 1993;21:1369-1375. 12. Webb RK, Ralston AC, Runciman WB. Potential errors in pulse oximetry. II. Effects of changes in saturation and signal quality. Anaesthesia. 1991;46:207-212. 13. Kruse JA. Blood lactate and oxygen transport. Intensive Care World. 1987;4:121-125. 14. Kruse JA, Haupt MT, Puri VK, et al. Lactate levels as predictors of the relationship between oxygen delivery and consumption in ARDS. Chest. 1990;98:959-962. 15. Charpie JR, Dekeon MK, Goldberg CS, et al. Serial blood lactate measurements predict early outcome after neonatal repair or palliation for complex congenital heart disease. J Thorac Cardiovasc Surg. 2000; 120:73-80. 16. Schumacher KR, Reichel RA, Vlasic JR, et al. Rate of increase in serum lactate level risk-stratifies infants after surgery for congenital heart disease. J Thorac Cardiovasc Surg. 2014;148(2):589-595. 17. Domico M and Allen M. Biomarkers in Pediatric Cardiac Critical Care. Pediatr Crit Care Med. 2016;17:S215-S221. 18. Guzman JA, Lacoma FJ, Kruse JA. Relationship between systemic oxygen supply dependency and gastric intramucosal PCO2 during progressive hemorrhage. J Trauma. 1998;44:696-700. 19. Guzman JA, Kruse JA. Development and validation of a technique for continuous monitoring of gastric intramucosal pH. Am J Respir Crit Care Med. 1996;153:694-700. 20. Slavin KV, Dujovny M, Ausman JI, et al. Clinical experience with transcranial cerebral oximetry. Surg Neurol. 1994;42:531. 21. Wyatt JS, Cope M, Delpy DT, et al. Quantitation of cerebral blood volume in human infants by near-infrared spectroscopy. J Appl Physiol. 1990;68:1086-1091. 22. Edwards AD, Wyatt JS, Richardson C, et al. Cotside measurement of cerebral blood flow in ill newborn infants by near infrared spectroscopy. Lancet. 1988;2:770-771.

23. Tateishi A, Maekawa T, Soejima Y, et al. Qualitative comparison of carbon dioxide-induced change in cerebral near-infrared spectroscopy versus jugular venous oxygen saturation in adults with acute brain disease. Crit Care Med. 1995;23:1734-1738. 24. Lewis SB, Myburgh JA, Thornton EL, et al. Cerebral oxygenation monitoring by near-infrared spectroscopy is not clinically useful in patients with severe closed-head injury: a comparison with jugular venous bulb oximetry. Crit Care Med. 1996;24:1334-1338. 25. Guery BP, Mangalaboyi J, Menager P, et al. Redox status of cytochrome a, a3: a noninvasive indicator of dysoxia in regional hypoxic or ischemic hypoxia. Crit Care Med. 1999;27:576-582. 26. Hampson NB, Piantadosi CA. Near infrared monitoring of human skeletal muscle oxygenation during forearm ischemia. J Appl Physiol. 1988;64:2449-2457. 27. Tashiro H, Suzuki S, Kanashiro M, et al. A new method for determining graft function after liver transplantation by near-infrared spectroscopy. Transplantation. 1993;56:1261-1263. 28. Noriyuki T, Ohdan H, Yoshioka S, et al. Near-infrared spectroscopic method for assessing the tissue oxygenation state of living lung. Am J Respir Crit Care Med. 1997;156:1656-1661. 29. Tortoriello TA, Stayer SA, Mott AR, et al. A noninvasive estimation of mixed venous oxygen saturation using near-infrared spectroscopy by cerebral oximetry in pediatric cardiac surgery patients. Paediatr Anaesth. 2005;15:495-503. 30. Owens GE, King K, Gurney JG, et al. Low renal oximetry correlates with acute kidney injury after infant cardiac surgery. Pediatr Cardiol. 2011;32(2):183-188. 31. Adams PS, Vargas D, and Baust T, et al. Associations of perioperative renal oximetry via near-infrared spectroscopy, urinary biomarkers, and postoperative acute kidney injury in infants after congenital heart surgery: should creatinine continue to be the gold standard? Pediatr Crit Care Med. 2019;20(1):27-37. 32. DeWitt AG, Charpie JR, Donohue JE, et al. Splanchnic nearinfrared spectroscopy and risk of necrotizing enterocolitis after neonatal heart surgery. Pediatr Cardiol. 2014;35(7);1286-1294. 33. Hoffman FM, Ghanayem NS, Scott JP, et al. Postoperative cerebral and somatic near-infrared Spectroscopy Saturations and outcome in hypoplastic left heart syndrome. Ann Thorac Surg. 2017;103(5): 1527-1535. 34. Kaufman JM, Almodovar MC, Zuk JP, et al. Correlation of abdominal site near-infrared spectroscopy with gastric tonometry in infants following surgery for congenital heart disease. Pediatr Crit Care Med. 2008;9:62-68. 35. Argenziano M, Chen JM, Choudhri AF, et al. Management of vasodilatory shock after cardiac surgery: identification of predisposing factors and use of a novel pressor agent. J Thorac Cardiovasc Surg. 1998;116:973-980. 36. Waltemath CL, Preuss DD. Determination of blood pressure in lowflow states by the Doppler technique. Anesthesiology. 1971;34:77-79. 37. Park MK, Menard SM. Accuracy of blood pressure measurement by the Dinamap monitor in infants and children. Pediatrics. 1987;79: 907-914. 38. Baronov D, McManus M, Butler, E, et al. Next generation patient monitor powered by in-silico physiology. Conf Proc IEEE Eng Med Biol Soc. 2015;2015:4447-4453. 39. Rusin CG, Acosta SI, Shekerdemian LS, et al. Prediction of imminent, severe deterioration of children with parallel circulations using real-time processing of physiologic data. J Thorac Cardiovasc Surg. 2016;152(1):171-177. 40. Suga H. Paul Dudley White Memorial Lecture: cardiac performance viewed through the pressure-volume window. Jpn Heart J. 1994; 35:263. 41. Cacciapuoti F. Echocardiographic evaluation of right heart function and pulmonary vascular bed. Int J Cardiovasc Imaging. 2009;25:689-697. 42. Daughters GT, Frist WH, Alderman EL, et al. Effects of the pericardium on left ventricular diastolic filling and systolic performance early after cardiac operations. J Thorac Cardiovasc Surg. 1992;104: 1084-1091.

References

e2



















49. Migliavacca F, Pennati G, Dubini G, et al. Modeling of the Norwood circulation: effects of shunt size, vascular resistances, and heart rate. Am J Physiol Heart Circ Physiol. 2001;280:H2076-H2086. 50. Mills KI, Kaza AK, Walsh BK, et al. Phosphodiesterase inhibitorbased vasodilation improves oxygen delivery and clinical outcomes following stage 1 palliation. J Am Heart Assoc. 2016;5(11):e003554. 51. Hoffman, GM, Tweddell JS, Ghanayem NS, et al. Alteration of the critical arteriovenous oxygen saturation relationship by sustained afterload reduction after the Norwood procedure. J Thorac Cardiovasc Surg. 2004;127(3):738-745. 52. Barnea O, Austin EH, Richman B, et al. Balancing the circulation: theoretic optimization of pulmonary/systemic flow ratio in hypoplastic left heart syndrome. J Am Coll Cardiol. 1994;25(5):13761381. 53. Bradley SM, Simsic JM, Mulvihill DM. Hyperventilation impairs oxygenation after bidirectional superior cavopulmonary connection. Circulation. 1998;98(suppl 19):II372. 54. Bradley SM, Simsic JM, Mulvihill DM. Hypoventilation improves oxygenation after bidirectional superior cavopulmonary connection. J Thorac Cardiovasc Surg. 2003;126:1033-1039.



















43. Appelbaum A, Blackstone EH, Kouchoukos NT, et al. Afterload reduction and cardiac output in infants early after intracardiac surgery. Am J Cardiol. 1977;39:445-451. 44. Taeed R, Schwartz SM, Pearl JM, et al. Unrecognized pulmonary venous desaturation early after Norwood palliation confounds Gp:Gs assessment and compromises oxygen delivery. Circulation. 2001;103:2699-2704. 45. Salim M, Case C, Sade R, et al. Pulmonary/systemic flow ratio in children after cavopulmonary anastomosis. J Am Coll Cardiol. 1995;25(3): 735-738. 46. Dori Y, Keller M, Rome J, et al. Percutaneous lymphatic embolization of abnormal pulmonary lymphatic flow as treatment of plastic bronchitis in patients with congenital heart disease. Circulation. 2016;133(12):1160-1170. 47. Savla J, Itkin M, Rossano J, et al. Post-Operative Chylothorax in Patients with Congenital Heart Disease. J Am Coll Cardiol. 2017; 69(19):2410-2422. 48. Williams DB, Kiernan PD, Metke MP, et al. Hemodynamic response to positive end-expiratory pressure following right atriumpulmonary artery bypass (Fontan procedure). J Thorac Cardiovasc Surg. 1984;87:856-861.

e3

Abstract: Cardiovascular assessment and monitoring in the pediatric intensive care unit require careful integration of physical findings, laboratory studies, and electronic data to make appropriate therapeutic decisions. Multiple noninvasive and invasive monitoring techniques are available for clinicians. For a given clinical scenario, an appreciation of the quantity of therapy required to achieve and sustain adequate systemic oxygen delivery and perfusion

pressure is useful for the clinician to understand the patient’s overall condition, discern the patient’s trajectory, and anticipate associated consequences of current management choices. Key words: cardiac output, oxygen delivery, cardiovascular assessment, invasive and noninvasive monitoring, quantity of therapy, streaming analytics

28 Cardiac Failure and Ventricular Assist Devices ANA LIA GRACIANO, JOSEPH PHILIP, AND KEITH C. KOCIS



Pediatric Heart Failure Heart failure (HF) is the final pathway of many pathophysiologic states affecting cardiovascular performance leading to inadequate cardiac output (CO) and decreased end-organ perfusion with diminished oxygen delivery to the vital organs. These include states of altered preload, afterload, contractility, and abnormal heart rate (HR) or rhythm. The pathophysiologic syndrome of HF is the result of a complex interplay among circulatory, neurohormonal, and molecular abnormalities. The etiology of HF in children differs from that of the adult and includes both primary cardiac and noncardiac causes, with the largest disease burden being due to congenital heart malformations and cardiomyopathies.1–3 HF in children leads to characteristic signs and symptoms—such as poor growth, feeding difficulties, respiratory distress, exercise intolerance, and fatigue. The pathophysiology of HF includes volume overload (e.g., left-to-right shunting, semilunar valve regurgitation), pressure overload (e.g., congenital aortic stenosis), or a combination of both as seen in patients with complex congenital heart disease. The primary goal in the management of HF is to ensure adequate tissue oxygen delivery, as an imbalance between delivery and consumption results in organ dysfunction, failure, and cardiogenic shock. Acute decompensated HF (ADHF) is a common final pathway for children with congenital or acquired heart disease. They can present with failed palliation of congenital heart disease, acquired cardiomyopathies, or acute exacerbations of chronic HF. Initial therapies to treat ADHF target 248



• •



Advances in medical management, surgical techniques, and mechanical circulatory support (MCS) for pediatric patients with congestive heart failure continue to improve patient outcomes. Indications for the use of MCS continue to evolve. Patient size, expected duration of support, and goals of support (i.e., bridge to recovery vs. bridge to transplant) must be considered in the choice of an MCS device. MCS is a lifesaving therapeutic option for patients with advanced heart failure. Device options in children are expanding but remain limited due to size constraints.







• •







PEARLS In children, most short-term MCS continues to be achieved with extracorporeal life support. Small patients (,3 kg), renal failure, and the need for biventricular support greatly increase the risk of death during support. A multidisciplinary team approach to the selection of the appropriate mechanical circulatory device is critical to a successful outcome.

providing respiratory support; decreasing metabolic demands (i.e., work of breathing); optimizing preload, afterload, and contractility; and optimizing HR and rhythm. Despite advances in pharmacologic therapies beyond inotropes, diuretics, and antiarrhythmic agents, a group of patients will continue to deteriorate or require excessive cardiopulmonary support to maintain adequate CO. This group should be evaluated early for mechanical circulatory support (MCS) before additional endorgan dysfunction or irreversible damage occurs. Several innovative types of MCS are being created and brought into clinical care while extended indications and management strategies are currently being developed for older devices. Simultaneously, extracorporeal life support (ECLS) continues to improve and remains the most commonly deployed form of MCS for infants and children.4–6

Low Cardiac Output Syndrome Low cardiac output syndrome (LCOS) describes a clinical state with a specific profile of biochemical markers in which there is inadequate systemic oxygen delivery (Do2) to meet the metabolic demands (oxygen consumption [Vo2]) of the patient. The condition has been recognized since the 1960s; numerous studies have documented the predicted changes in physiologic parameters.7,8 LCOS has been an active area of research, and multiple reviews of current therapies are available to guide the intensivist.9 LCOS is frequently seen in myocarditis, cardiomyopathies, with prolonged bradyarrhythmias or tachyarrhythmias, and commonly after



CHAPTER 28

complex pediatric cardiac surgery. Postoperative physiologic changes secondary to cardiopulmonary bypass, myocardial ischemia during aortic cross-clamping, cardioplegia, residual uncorrected lesions (i.e., aortic arch obstruction), ventriculotomy, changes in the loading conditions to the myocardium, and dysrhythmias may all contribute to the development of LCOS. A variety of proinflammatory triggers are activated during cardiopulmonary bypass as a result of blood contact with foreign surfaces, ischemia, reperfusion, oxygen free radicals, tissue trauma, and temperature fluctuations. These complex inflammatory responses include complement activation, cytokine release, leukocyte and platelet activation, and the expression of adhesion molecules.10–12 LCOS has been reported to affect up to 25% of infants and children following cardiac surgery, typically occurs between 6 and 18 hours after cardiac surgery, and results in a longer intensive care and hospital stay with increased morbidity and mortality. It is associated with elevated systemic vascular resistance and pulmonary vascular resistance (PVR), impaired myocardial function, dysrhythmias, and capillary leak. When unrecognized or inadequately treated, LCOS can result in irreversible end-organ failure, cardiac arrest, and death. Prevention, early recognition, and optimal treatment are essential to ameliorate or reverse its course.13–15

Definitions The concepts of Do2 and Vo2 are of fundamental importance in managing critically ill children. Systemic oxygen delivery (Do2) is defined as the amount of O2 delivered to peripheral tissues each minute and is determined by CO and the oxygen content of arterial blood (Cao2). CO is defined as the product of HR and stroke volume (SV), and the arterial oxygen content (Cao2) is determined by hemoglobin (Hb) concentration, the arterial oxygen saturation (Sao2), and the partial pressure of oxygen in the arterial blood (Pao2). Lastly, Vo2 can be measured by indirect calorimetry, or calculated using the Fick equation as the arterial venous difference in oxygen content (Cao2 2 Cvo2) multiplied by the CO: Do2 5 CO 3 Cao2 CO 5 HR 3 SV Cao2 5 (Hb 3 1.36 3 Sao2) 1 (Pao2 3 0.003) Vo2 5 CO 3 (Cao2 – Cvo2) Achieving a positive balance between O2 supply and demand is essential and can be accomplished by decreasing Vo2 or increasing Do2. Vo2 is determined by tissue metabolism and is increased during periods of increased muscular activity (i.e., seizures, exercise), infection, fever, or with increased levels of circulating catecholamines. Normally, the ratio between Do2 and Vo2 is 4:1; this ratio is high enough that cellular respiration is not supply dependent and Vo2 is mainly a function of tissue O2 demands. When Vo2 increases based on increased metabolic demands, Do2 increases accordingly. Therefore, consumption drives delivery. Inability to maintain the normal Do2:Vo2 ratio is initially compensated by increased O2 extraction. However, when the rate of Vo2 exceeds Do2, anaerobic metabolism begins. Early studies in neonates with transposition of the great arteries (TGA) who underwent the arterial switch procedure documented the development of low CO 6 to 12 hours after surgery.16 These findings led to a placebo-controlled intervention study whereby milrinone was randomly administered (bolus followed by continuous infusion) in a low (0.25 mg/kg per minute) versus

Cardiac Failure and Ventricular Assist Devices

249

high (0.75 mg/kg per minute) dose to infants and children after cardiopulmonary bypass (CPB), in addition to the required inotropic agents needed to separate from CPB. In those patients who received high-dose prophylactic milrinone, the development of LCOS in the first 36 hours after surgery was decreased to 12% compared with 26% in the placebo group. Recent Cochrane reviews have failed to demonstrate a consistent improvement in the treatment of cardiogenic shock in infants and children after cardiac surgery with either milrinone or levosimendan, a calciumsensitizing agent.17,18

Assessment Although a complete assessment of Do2 and Vo2 in critically ill infants and children is extremely challenging, numerous hemodynamic and biochemical biomarkers can be readily obtained to help guide the bedside clinician. Mixed venous oxygen saturation (SvO2) or central venous oxygen saturation (Scvo2), arterial lactate, and near-infrared spectroscopy (NIRS) are important clinical parameters that can be serially/continuously measured in patients at risk for LCOS. Svo2 and Scvo2 are a reflection of the total body Do2/Vo2 ratio and can be important metrics to follow in critically ill patients.19,20 An elevated lactate level on admission (.4.5 mmol/L) and rising at 0.9 mmol/L per hour postoperatively is associated with major adverse events, including death, in infants after cardiac surgery.21–24 Furthermore, lower Svo2 may increase the predictive power of elevated arterial lactate levels for mortality after pediatric cardiac surgery.21,25 It is not uncommon to observe a disparity between direct measurements of Do2 and CO and estimates based on physical examination and interpretation of conventional laboratory and hemodynamic parameters.26–28 Compensatory increases in systemic vascular resistance maintain arterial blood pressure as CO decreases and central venous pressures may not correlate well with ventricular filling (preload).29,30 Femoral venous catheters are routinely placed in neonates and children, providing an additional factor (intraabdominal pressure) in the accurate interpretation of these continuous variables. The effects of positive-pressure ventilation on these measurements are well described.31,32 Hemodynamic stability in the early postoperative period after Norwood palliation depends on an adequate total CO from the single ventricle and a balanced pulmonary-to-systemic blood flow ratio (Qp:Qs). Systemic oxygen saturation (Sao2) alone is a poor predictor of Qp:Qs because of variability in systemic venous oxygen saturation (Svo2) and pulmonary vein saturation (Spvo2) after the Norwood operation. Monitoring and optimizing Svo2 have been shown to improve outcomes in pediatric patients at risk for developing shock, including hypoplastic left heart syndrome.33,34 Intermittent Svo2 monitoring can be obtained to assess oxygen transport balance, but the presence of intracardiac shunts (i.e., left-to-right shunting of pulmonary venous blood across the atrial septum) near the site of sampling and the need for repeated blood sampling limits its universal use. It has been reported that when Svo2 monitoring is used in the postoperative care of critically ill neonates, significantly fewer adverse events are encountered. Significant decreases in Svo2 can occur without appreciable changes in Sao2, blood pressure, or HR. When low Svo2 is recognized, increased inodilator support and measures that decrease metabolic demands (i.e., sedation, neuromuscular blockade) often successfully return this metric toward the normal range. In contrast, ventilator and inspired gas adjustments have less effect

250

SECTION IV



Pediatric Critical Care: Cardiovascular

on correcting Svo2. The critical O2 extraction ratio is defined by the onset of shock and ranges from 50% to 60%.22,35 NIRS is a noninvasive technique used to monitor tissue oxygenation and perfusion. NIRS was initially used in the operating room to monitor brain oxygenation during cardiopulmonary bypass but has expanded as a useful and frequent monitoring technique in the intensive care unit (ICU). Cerebral NIRS noninvasively assesses cerebral tissue oxygenation and relies on the relative lucency of biological tissue to near-infrared light where oxy- and deoxyhemoglobin have distinct absorption spectra.34,36–38 The oximeter monitors the nonpulsatile signal reflecting the microcirculation where 75% to 85% of the blood volume is venous. Thus, the NIRS-derived oxygen saturation is an indicator of oxygen extraction for the region of brain beneath the optode. There is a good correlation between cerebral oxygen saturations, jugular bulb, and superior vena cava saturations.39,40 Cerebral oxygen saturations are closely and inversely correlated with the oxygen extraction ratio in neonates following the Norwood procedure. Strategies measuring NIRS in both cerebral and somatic regions provide better estimates of outcome.24,38,41 The pulse contour CO (Pulse index Continuous Cardiac Output [PiCCO]) measurement system allows continuous hemodynamic monitoring using a large (femoral or axillary) artery catheter and a central venous catheter. This technology uses intermittent transpulmonary thermodilution and pulse contour analysis. With the use of specific algorithms, various parameters such as CO, extravascular lung water (EVLW), global end-diastolic volume (GEDV), pulse pressure variation (PPV), and stroke volume variation (SVV) can be obtained. PiCCO can assist in preload assessment in a volumetric manner. The PiCCO system may give inaccurate measurements in patients with arrhythmias, rapid temperature changes, intra/extracardiac shunts, aortic aneurysm, aortic stenosis, pneumonectomy, pulmonary embolism, and extracorporeal circulation.35,42,43 The USCOM ultrasonic CO monitor (USCOM Pty Ltd.) is a noninvasive device that determines intermittent CO by continuous-wave Doppler ultrasound and estimates of the normal aortic valve annulus based on the weight or height of the patient. Systemic vascular resistance can then be calculated when an invasive or noninvasive blood pressure is obtained simultaneously. Although approved by the US Food and Drug Administration (FDA) for use in children, pediatric data are still limited.44–47

Specific Treatments to Improve Cardiac Function HF is a clinical and pathophysiologic syndrome that results from ventricular dysfunction and volume or pressure overload, either alone or in combination. In contrast to adult patients, tremendous heterogeneity exists between the etiology and pathophysiology of HF in pediatric patients. There are significant barriers in applying adult HF data to children owing to factors related to developmental cardiac physiology. Despite these differences, treatment strategies for pediatric patients have often followed the recommendations from large, randomized multicenter trials in adult patients or “consensus” opinions based on single “best” institution clinical practices due to a lack of randomized, multicenter pediatric clinical trials. With the expanded authority and mandates of the FDA Pediatric Advisory Committee and the origins of the National Institutes of Health Collaborative Pediatric Critical Care Research Network and the new Pediatric Cardiac Critical Care Consortium (PC4), the future is brighter for high-quality clinical trials in children.48,49 Management of chronic congestive HF with digitalis formed the basis of early therapy (1970s) despite its narrow therapeutic index for safety. Volume overload is common, and loop and/or

thiazide diuretics  the addition of the potassium-sparing antimineralocorticoid, spironolactone are still also commonly used today, although drug compounding for children and potassium monitoring is problematic. Patients with advanced pulmonary hypertension, right ventricular (RV) failure, or restrictive left ventricle (LV) physiology present a unique challenge, as CO may worsen in face of the rapid reduction in LVSV that can occur with aggressive diuresis.50 Although angiotensin-converting enzyme (ACE) inhibitors exert favorable effects on cardiac remodeling and survival in adults with congestive HF, their role in children is less clear as randomized placebo-controlled trials in children evaluating the effect of ACE inhibitors in single-ventricle patients have failed to demonstrate a beneficial effect.51–54 Selective b-blockers, such as metoprolol, are a mainstay in adult cardiac patients, with some utility in children.55,56 The nonselective b-blocker/a1-blocker, carvedilol, has been used commonly in pediatric HF treatment in recent years. Most pediatric uses of these drugs have been extrapolated from the positive effects seen in adult trials that demonstrated a reduction in mortality and risk of hospitalization. Single-center trials have demonstrated both improved ejection fraction with the use of carvedilol in children awaiting heart transplantation with dilated cardiomyopathy and a delayed time to transplantation or death.54,57 However, the most recent and largest multicenter, randomized, double-blind placebocontrolled trial of carvedilol in children and adolescents with symptomatic systolic HF did not demonstrate an improved survival benefit.58 A recent Cochrane systematic review of randomized controlled trials investigating the effect of b-blockers in pediatric HF concluded that there are not enough data to recommend or discourage the use of these drugs in pediatric HF.52,55 A current review from the PC4 database found that 6% of pediatric patients admitted to a cardiac intensive care unit (CICU) were diagnosed and treated for ADHF and that these patients had multiple comorbidities, high mortality rates, and frequent readmissions, particularly those with congenital heart disease (CHD). In this multicenter data collection, the median age at admission was 0.93 years and 57% of the cohort had CHD. A total of 88% received vasoactive infusions while 59% required mechanical ventilation. Common complications were arrhythmias (19%), cardiac arrest (10%), sepsis (7%), and acute renal failure requiring dialysis (3%). The median length of stay was 7.9 days and the readmission rate was 22%. Overall, CICU mortality was 15% but 19% in those with CHD, compared with 11% in those without CHD. Independent risk factors associated with CICU mortality included age less than 30 days, CHD, vasoactive infusions, ventricular tachycardia, mechanical ventilation, sepsis, pulmonary hypertension, extracorporeal membrane oxygenation (ECMO), and cardiac arrest.59 For hospitalized children with ADHF and/or LCOS, traditional first-line management is to use continuous infusions of catecholamines or their analogs (i.e., dopamine, epinephrine, norepinephrine, or dobutamine) to increase cardiac contractility (b1). However, when used in high doses these agents can be deleterious to global and myocardial Vo2, induce myocardial cell apoptosis, and lead to increased mortality. In these patients, it is important to integrate diuretic, inotropic, and vasodilating therapy in conjunction with careful monitoring of hemodynamic parameters and end-organ perfusion. Milrinone, a phospodiesterase-3 inhibitor, has emerged as an important inodilating agent, which is now widely used in children following open-heart surgery after a landmark study showed that the prophylactic use of milrinone was associated with a decreased likelihood of LCOS in children following open-heart surgery in a dose-dependent fashion.60 This benefit is thought to result from



CHAPTER 28

both improved myocardial contractility and pulmonary and systemic vasodilatory effects. Milrinone reduces RV and LV afterload through systemic and pulmonary vasodilation and improves diastolic relaxation (lusitropy) of the myocardium through its enhanced cyclic adenosine monophosphate–dependent diastolic reuptake of calcium.61–64 Sildenafil, an oral or intravenous drug that inhibits cyclic guanosine monophosphate–specific phosphodiesterase type 5 causing smooth muscle relaxation, has proven to be beneficial in neonates with severe pulmonary hypertension and as an adjunct in weaning these patients from inhaled nitric oxide.65–67 Levosimendan, a newer drug, is a calcium sensitizer that enhances the contractility of the ventricle by binding to cardiac troponin C. In addition, the drug acts as a vasodilator by opening ATPsensitive potassium channels in the vascular smooth muscle, resulting in decreases in systemic and pulmonary vascular resistance. The improvement in myocardial performance is accomplished without an increase in intracellular calcium, thus providing a cardioprotective effect. Finally, an active metabolite with a half-life of 3 days prolongs the duration of action for this continuously infused medication.68,69 A recent prospective observational study in 110 patients with a median age of 346.5 days (4 days–19.6 years) undergoing cardiac surgery reported the safety of levosimendan in all age groups and categories of congenital heart disease, demonstrating optimization of CO with a low incidence of arrhythmias. Levosimendan was started at the initiation of the rewarming phase of bypass and continued for 48 hours.70 Despite these encouraging studies, a Cochrane review in 2017 concluded that the “current level of evidence is insufficient to judge whether prophylactic levosimendan prevents LCOS and mortality in pediatric patients undergoing surgery for congenital heart disease.” The authors concluded that “no

Cardiac Failure and Ventricular Assist Devices

significant differences have been detected between levosimendan and standard inotrope treatments in this setting.”71 Fenoldopam, a selective dopamine-1 receptor partial agonist, causes systemic vasodilation and increased renal blood flow with improved renal function in adults. Many studies have examined fenoldopam as a possible therapeutic agent capable of preventing the onset of postoperative acute kidney injury (AKI). Two most recent meta-analyses concluded that although fenoldopam decreases the incidence of postoperative AKI, it does not reduce the need for renal replacement therapy or mortality.72,73 Data on the use of fenoldopam in pediatric patients are sparse, with one retrospective study demonstrating a significant improvement in diuresis in neonates after CPB with the addition of fenoldopam to conventional diuretic therapy, whereas another prospective trial failed to demonstrate a beneficial effect. Ricci et al. demonstrated in a small randomized controlled trial that high-dose fenoldopam decreased urine neutrophil-gelatinase-associated lipocalin (NGAL) and cystatin C, suggesting a renoprotective effect.74 At this time, there is no strong evidence for the routine use of fenoldopam to prevent AKI after pediatric cardiac surgery.50

Broad Treatment Strategies Supportive care for ADHF begins with strategies aimed at improving the specific components of Do2 and Vo2 listed previously (Fig. 28.1). This includes standard critical care therapies primarily focused on noncardiac support, such as intubation and mechanical support for respiratory insufficiency, temperature control, red cell transfusion for significant anemia, and control of pain and agitation with multimodal analgesia and anxiolysis. Mechanical respiratory support decreases metabolic demands (Vo2) by

Clinical symptoms Poor perfusion, hepatic congestion, nausea, vomiting, or respiratory distress

Monitoring Decreased urine output, dysrhythmia, hypotension, elevated RAP, elevated PAP, low or elevated LAP, elevated lactate level, SVO2 acidosis, echocardiography/cardiac catheterization

Treatment

Decrease metabolic needs Optimize sedation and analgesia Control temperature Optimize ventilation

Rate control Sinus rhythm Control temperature Optimize preload Consider transfusion of packed red blood cells

• Fig. 28.1

Augment stroke volume 1. Increase contractility epinephrine, dopamine, or dobutamine 2. Decrease afterload systemic: milrinone or pulmonary: inhaled NO

​Management of heart failure in children. LAP, Left atrial pressure; NO; nitric oxide; PAP, pulmonary artery pressure; RAP, right atrial pressure; Svo2, mixed venous oxygen saturation.  

251

252

SECTION IV



Pediatric Critical Care: Cardiovascular

decreasing the work of breathing, reduces LV afterload (wall stress), and can improve Sao2 by the administration of O2 and positive end-expiratory pressure (PEEP), thus increasing Cao2. However, endotracheal intubation can be risky in decompensated patients with HF, as the induction agents, laryngoscopy, and conversion to positive-pressure ventilation can induce cardiac arrest. The benefits of noninvasive positive-pressure ventilation (i.e., bilevel positive airway pressure) have not been proven, and the risks remain significant, particularly with regard to delay in timing of intubation and the need for additional sedation in children. Cao2 can also be increased by transfusing packed red blood cells to increase hemoglobin concentrations in significant anemia. Research in the PICU has not demonstrated a negative impact on patient outcomes when a restrictive transfusion practice was embraced, though the study did not examine complex or high-risk congenital cardiac patients or those with unrepaired or palliated cyanotic cardiac defects where a relative polycythemia is normally maintained to compensate for the decrease in Sao2.75 Clearly, numerous significant deleterious effects on the recipient occur after allogenic blood transfusion, and the risk of donor-directed blood can be greater. In patients who are euvolemic, hypervolemia with worsening pulmonary edema must be avoided when the decision to transfuse red cells is made.76,77 CPB results in numerous significant neurohormonal perturbations in neonates, infants, and children after cardiac surgery. A pattern of sick euthyroid syndrome has been identified in pediatric patients after CPB. Several studies have demonstrated acute salutary effects of preoperative thyroid prophylaxis78 or postoperative therapy with triiodothyronine resulting in increases in specific hemodynamic parameters, improved diuresis, and a decreased need for additional cardiopulmonary support without significant adverse effects79,80 although better outcomes have yet to be shown.81 Critical illness–related corticosteroid insufficiency (CIRCI) has been demonstrated imprecisely in clinical trials in neonates, infants, and children after cardiac surgery. Stress-dose hydrocortisone replacement similarly has resulted in positive acute hemodynamic improvements and, yet again, this has not translated into improved survival or decrease in morbidities in these patients. All of these studies are confounded by the use of differing adrenal corticosteroids in varying doses used in the preoperative/intraoperative anesthetic/perfusion protocols.82–84 The STRESS trial is an ongoing randomized double-blind placebo-controlled trial evaluating the safety and efficacy of single-dose intraoperative methylprednisolone 30 mg/kg versus placebo in infants (,1 year of age) undergoing cardiac surgery with CPB.85 A multicenter randomized controlled study evaluating two levels of glycemic control was stopped prematurely by the data safety monitoring committee owing to futility in reaching significance and apparent harm (hypoglycemia and hospital-acquired infections) in the low-glucose (80–110 mg/dL) compared with the high-glucose target (150–180 mg/dL) groups.86 Interestingly, pediatric cardiac surgery patients were excluded from enrollment. Despite the results of this large multicenter pediatric trial, the physiology and pathophysiology of hyperglycemia in the stress response to critical illness remains significant, and other well-controlled studies point to differing conclusions based on the details of the management strategy.87,88 Finally, a Pediatric Cardiac Intensive Care Society’s consensus statement could not recommend any hormonal replacement/monitor strategies (thyroid, corticosteroid, or insulin) owing to the lack of robust double-blinded randomized clinical trials.89 Terlipressin, a synthetic long-acting analog of vasopressin with a higher affinity for vascular V1-receptors than vasopressin, has been used in adult patients to treat extremely low CO, but its ap-

plication in children is limited. In postoperative pediatric cardiac patients, terlipressin demonstrated an improvement in respiratory, hemodynamic, and renal indices in refractory LCOS.90 Although these results are encouraging, further investigation with prospective randomized trials is needed to provide data regarding the efficacy and safety of terlipressin in infants and children.91 Inhaled nitric oxide (iNO) is a potent short-acting vasodilator resulting in numerous beneficial cardiopulmonary effects on the pulmonary, systemic, and coronary circulations. Its use is standard for severe, reversible pulmonary arterial hypertension (PAH) in the neonate and in children with congenital heart disease. It is also useful in the treatment of RV failure. A variety of other drugs (e.g., nitroprusside) and amino acids (arginine, citrulline) are precursors to NO and thus increase its circulating levels. Pediatric patients with severe PAH can also be treated with intravenous prostacyclin analogues, such as epoprostenol or treprostinil,92,93 or continuously inhaled epoprostenol. In studies in neonates with persistent pulmonary hypertension88,94 and critically ill noncardiac pediatric patients,95,96 this therapy has proven to be equally efficacious as iNO and, in extreme cases, they can be used synergistically in a child.97

Mechanical Circulatory Support in Pediatric Patients Medical therapy for HF has improved survival and quality of life, although a large number of patients still require advance treatment with intravenous inotropic support, mechanical ventilation, and/or heart transplantation. MCS should be considered for children with decompensated HF who cannot be stabilized with medical therapy alone. Rapid advances in the field of MCS have dramatically changed the management of children with end-stage HF with emphasis on timely evaluation of ventricular assist devices (VADs) to preserve or recover end-organ function. When medical treatment is maximized or ineffective, patients should be considered for MCS for temporary support until heart function recovers, as a bridge to heart transplantation, or as a destination therapy (life with permanent VAD). MCS through a variety of devices can be used to treat right, left, or biventricular failure. This is accomplished with devices that are either extracorporeal (outside the human body), paracorporeal (partial within and outside the human body), or intracorporeal or implantable (residing completely within the human body). MCS devices can provide either pulsatile flow or continuous flow depending on the specific design. MCS can also be provided to the left ventricle alone with an intraaortic balloon pump or it can replace the entire function of the heart with a total artificial heart. Finally, cardiac and pulmonary support can be provided with the addition of a membrane oxygenator in specific types of ECLS devices. Venoarterial (VA) ECLS provides cardiopulmonary support, whereas venovenous (VV) ECLS provides only pulmonary support for severe respiratory failure. Despite improvements in operative techniques, management of CPB, and myocardial protection, myocardial dysfunction and failure can occur after surgery for CHD with involvement of the left and/or right ventricles. Approximately 5% of children undergoing cardiac surgery require MCS. The patient’s underlying pathophysiology, size, and expected length of support dictate which technique is most suitable. In children, most MCS continues to be achieved with ECLS. Since ECLS and VADs have advantages and disadvantages, determining the most appropriate modality for the individual patient requires significant experience and expertise.



CHAPTER 28

ECLS is the most utilized form of short-term MCS for pediatric patients with decompensated HF unresponsive to medical therapy. Adaptations and simplification of the traditional CPB circuit have resulted in the standard VA ECLS circuit (Fig. 28.2) through either extrathoracic (i.e., carotid artery/jugular vein or femoral vessels) or transthoracic (right atrium/ascending aorta) placement after median sternotomy. The venous cannula allows for drainage of blood from the patient by a negative pressure created by the servo-regulated centrifugal pump or, previously, a roller-head that propels blood through the remainder of the system at high pressure. Gas exchange (O2 addition and CO2 removal) occurs next through an artificial lung (oxygenator comprised of hollow fibers) where countercurrent sweep gas composed of O2/air mixture titrated to a specific fraction of inspired oxygen (Fio2) passes external to the blood phase. Blood temperature is controlled by a heat exchanger before blood is returned to the body through a major artery. Additional components include ports for infusion of medications, arterial and venous pressure monitors, Svo2 and/or blood gas analyzers, and flow and bubble detectors. There can be placement of a hemofilter for fluid control or a dialysis circuit (venovenous, arteriovenous, arterial-arterial) and, frequently, a bridge connecting the venous and arterial sides of the circuit is used during trials off ECLS or in case of a circuit emergency. Individual centers customize their ECMO circuits to serve their patient population by creating a less complicated circuit that is easier to manage with fewer connectors to reduce the number of potential sites for blood stasis.98,99 VA ECLS provides biventricular and pulmonary support. It is important to note that complete cardiac bypass cannot be provided by VA ECLS owing to incomplete capture and drainage of the cardiac systemic venous return by the venous cannula. Routine cannulation for VA ECLS occurs via one of three vascular

• Fig. 28.3

​Chest radiograph showing venoarterial extracorporeal membrane oxygenation cannulas.

ECMO SYSTEM

O2 blender Membrane oxygenator

Premembrane pressure monitor

Warmed H2O input

Postmembrane pressure monitor

Pump

Heat exchanger

F l u i d s

H e p a r i n

RV

LV

Venous reservoir

• Fig. 28.2

​Extracorporeal membrane oxygenation circuit. ECMO, Extracorporeal membrane oxygenation; H2O, water; LV, left ventricle; O2, oxygen; RV, right ventricle.  

253

access points: (1) The transcervical approach places the venous cannula in the right atrium (RA) via the right internal jugular vein and the arterial cannula in the transverse aortic arch via the right carotid artery. (2) The transthoracic approach results in direct cannulation of the RA and ascending aorta through a median sternotomy for patients who either cannot be weaned from cardiopulmonary bypass or require MCS in the immediate postoperative period. (3) Femoral artery and vein cannulation for adolescents and adults uses longer cannulae to reach the RA and descending aorta. A combination of these approaches with more than one venous cannulation site can be employed when very high flows are required. The transcervical and transthoracic approaches are the preferred methods for small children. Fig. 28.3 is a chest radiograph



Extracorporeal Life Support

Cardiac Failure and Ventricular Assist Devices

254

SECTION IV



Pediatric Critical Care: Cardiovascular

A

•  Fig. 28.4

​(A) Venovenous extracorporeal membrane oxygenation double-lumen cannula. (B) Chest radiograph showing position of double-lumen cannula.  

B obtained for evaluation of cannula placement in a patient supported by VA ECLS. VV ECLS provides pulmonary support without cardiac support. In VV ECLS, a single double-lumen cannula (Fig. 28.4) is placed in the RA through a transcervical approach. This cannula provides both inflow and outflow for the circuit. VV ECLS can improve RV function as a result of oxygenated, pH-balanced blood flowing to the lungs, thus decreasing PVR and right heart afterload. Although some patients experience improvement in overall cardiac function with VV ECLS, it might not be sufficient or sustained. Therefore, when significant myocardial dysfunction exists, VA ECLS is the preferred modality.

Extracorporeal Membrane Oxygenation Indications and Contraindications It is widely accepted that all patients considered for MCS should have either a reversible physiologic process or should be a candidate for bridge to transplant or destination therapy. The severity of organ dysfunction and estimated time frame for recovery of cardiac and other organ failure both are used in determining optimal device selection. Several studies emphasize the importance of early institution of ECLS before a prolonged period of low CO results in multiorgan dysfunction. Appropriate patient selection is vital to maximize survival.100–102



CHAPTER 28

Myocarditis and Extracorporeal Life Support The clinical course of myocarditis is variable, with some patients presenting with subclinical disease, others with an indolent course progressing to a dilated cardiomyopathy, and a distinct subset of patients with fulminant disease. Without MCS, patients with rapidly progressive disease had expected survival rates of only 25% to 50%. Patients with acute fulminant myocarditis have worse short- and long-term outcomes when there are decreased LV ejection, ventricular arrhythmias, and/or LCOS.103 With aggressive utilization of ECLS as a bridge to transplantation or recovery, survival rates for patients are now reported to be as high as 90%.104–107 It has been shown that the institution of MCS can normalize ventricular geometry, cellular composition, metabolism, and, ultimately, function—a phenomenon referred to as reverse remodeling. This process is thought to improve ventricular dysfunction because of favorable influences on the neurohormonal cardiovascular milieu and ventricular unloading.108–110 For patients with end-stage dilated cardiomyopathy secondary to myocarditis, the use of MCS without transplantation has resulted in survival rates as high as 67% to 80%.64

Postcardiopulmonary Bypass Failure to wean from CPB occurs in approximately 1% to 3.2% of pediatric congenital cardiac surgery cases.111–113 Individual institutions have reported survival rates to hospital discharge between 32% and 54% for pediatric patients who require MCS after cardiac surgery. The use of MCS is currently widely accepted to support vital organs while allowing for myocardial recovery. It is imperative that these patients be evaluated for the presence of residual cardiac defects that may be causing or worsening cardiovascular collapse. Intraoperative transesophageal echocardiography (TEE), transthoracic echocardiography (TTE), and diagnostic cardiac catheterization may identify the need for reoperation or interventional cardiac catheterization to improve the patient’s hemodynamic status.114 In one large center, 2% of cardiac surgeries resulted in an early postoperative catheterization, 50% diagnostic and 50% interventional, and there was no procedural mortality. Importantly, 30% of those undergoing catheterization required reoperation.115 Balloon valvuloplasty, angioplasty of aortic arch obstructions, device closure of residual septal defects, coil occlusion of aortopulmonary collateral vessels, or atrial septostomy all may be crucial interventions to improve the patient’s hemodynamic state and allow for separation from MCS. Untreated and significant residual cardiac defects have been shown to be almost universally fatal for patients requiring MCS.116 The use of MCS for postcardiotomy support in neonates and infants with single-ventricle physiology is technically more complex with worse outcomes.117,118 Thus, previously this could have been considered a relative contraindication to MCS.119,120 The 2020 Extracorporeal Life Support Organization (ELSO) Registry (2015–20) reports a 44% survival rate for ECLS use in neonates with hypoplastic left heart (eTables 28.1 and 28.2).

Extracorporeal Cardiopulmonary Resuscitation Survival from pediatric cardiac arrest has improved tremendously over the last 20 years. However, half of the children who suffered in-hospital cardiac arrest do not survive to hospital discharge and those who do survive have significant morbidity.121 Although neurocognitive outcome has improved for survivors of cardiac

Cardiac Failure and Ventricular Assist Devices

255

arrest, the duration of cardiopulmonary resuscitation (CPR) remains inversely proportional to survival. The American Heart Association guidelines for in-hospital pediatric cardiac arrest now recommend consideration of extracorporeal CPR (ECPR) during CPR if the conditions leading to the arrest are likely to be reversible or amenable to heart transplantation.122 One of the most important aspects of the decision to institute ECPR is adequate patient selection. There is sufficient data to support the use of ECPR in children with cardiac disease who suffer an in-hospital cardiac arrest.123–126 However, wide variability in patient selection and the ability to institute ECLS in a timely fashion (ideally, ,60 minutes) has led to variable success, with survival rates ranging from 0% to 100%.118,127–129 Thus, some large centers have created systems for rapid-deployment ECLS using a team that is immediately available to cannulate with a preprimed circuit.6,130,131 Children with CHD have increased risk for an in-hospital cardiac arrest. Their survival to discharge after ECPR has been reported to be up to 50%, yet single-ventricle patients suffer more cardiac arrests and have the worst outcome.117,130,132,133 Reduced survival of ECPR has been associated with lower postcannulation pH, higher lactic acid, and end-organ injury. Post–cardiac arrest management (temperature control, target flow rates, target perfusion pressure) is beyond the scope of this chapter but constitutes an extremely important factor that can impact outcome. The January 2020 ELSO Registry for neonatal and pediatric patients supported with ECLS following cardiac arrest (ECPR) reported a survival to hospital discharge of 46% for neonates and 37% for children.

Bridge to Transplantation Although heart transplantation is the treatment of choice for endstage myocardial failure, many children die every year waiting for a suitable organ to become available. As a result, MCS is now increasingly used as a bridge to heart transplantation.127,134 The most common indications for MCS as a bridge to transplant include cardiac failure due to CHD, cardiomyopathy, and graft rejection after heart transplantation. Children with myocardial failure secondary to fulminant myocarditis also demonstrate increased shortterm survival when treated with MCS and transplantation.104,135–138 Complications associated with ECLS generally limit the duration of support to approximately 2 weeks, but as long as complications precluding transplant have not developed, no arbitrary cutoff for duration of ECLS has been determined. An analysis of the United States Scientific Registry of Transplant Recipients demonstrated that waitlist mortality varied as much as 10-fold based on recipient factors. Over the last decade, the proportion of pediatric heart transplant candidates with CHD increased from 48% in 2008 to 62.2% in 2018. From the pediatric heart transplantations performed in 2018, 7.8% were in children younger than 10 years. The overall percentage of candidates with VADs at the time of listing increased from 11.8% in 2008 to 32.6% in 2018. Recipient characteristics associated with waitlist mortality included ECMO, mechanical ventilation, listing status 1A, CHD, dialysis support, and non-white race. Waitlist mortality for infants was significantly higher than for older patients (Fig. 28.5). Although waitlist mortality remains high, VAD support has been an important factor in reducing waitlist mortality in small children.139 An analysis of the United Network of Organ Sharing (UNOS) database comparing the pre-VAD (1999–2004) and post-VAD (2005–15) eras reported more than 50% reduction in waitlist mortality in the post-VAD era,140 supporting the importance of centers of excellence that can provide these state-of-the-art therapies.

255.e1



eTABLE Neonatal Cardiac Runs by Diagnosis (ELSO Registry, January 2020) 28.1

Total Runs Congenital heart defect

Average Run Time (h)

Longest Run Time (h)

Survived

% Survived

963

144

1463

464

48

Cardiac arrest

14

133

309

5

35

Cardiogenic shock

70

172

1746

39

55

Cardiomyopathy

15

363

2109

8

53

Myocarditis

13

210

478

8

61

539

175

3566

298

55

Other

eTABLE Neonatal Cardiac Runs by Congenital 28.2 Heart Defect (ELSO Registry, January 2020)

Left-to-right shunt

Total Runs

Average Run Time (h)

Survived (n)

Survived (%)

74

161

37

50

Left-sided obstructive

89

135

41

46

Hypoplastic left heart

427

136

192

44

Right-sided obstructive

50

133

24

44

Cyanotic increased pulmonary blood flow

64

177

25

39

Cyanotic decreased pulmonary blood flow

299

1271

39

58

256

SECTION IV



Pediatric Critical Care: Cardiovascular

differing expectations for quality of life if survival is achieved. Technical limitations are an inability to obtain vascular access secondary to thrombosis, abnormal anatomy, or prior surgery.

100

Percent

80 60

Critical Care Management During Extracorporeal Life Support

Still waiting Removed from list Died DD transplant

40 20 0 0

12

24

26

Months post listing

• Fig. 28.5

​Waitlist mortality statistics from the Scientific Registry of Transplant Recipients (SRTR). DD, Deceased donor.  

A study merging UNOS and Pediatric Health Information Systems databases reported that children with CHD, particularly single-ventricle conditions, require substantially greater hospital resource utilization and have significantly worse outcomes during the first year after heart transplantation compared with other indications, emphasizing the importance of modifying patient risk factors with interventions such as pretransplantation conditioning with VAD support and cardiac rehabilitation.141

Malignant Dysrhythmias MCS can be lifesaving for pediatric patients with malignant dysrhythmias unresponsive to pharmacotherapy. A subset of patients with acute fulminant myocarditis will present with refractory LCOS and ventricular tachycardia or high-degree heart block that further compromises end-organ perfusion. In this scenario, MCS may be used to bridge patients to recovery following initiation of antiarrhythmic agents and/or electrophysiologic mapping with ablation therapy or transplantation. Several potentially lethal arrhythmias—such as supraventricular tachycardia, junctional ectopic tachycardia, ventricular tachycardia, or torsades de pointes—can occur congenitally, can be acquired in the perioperative period, or manifest as a result of the ingestion of toxic substances or medications.142–144 Again, ECLS can allow time for resolution of the dysrhythmia with aggressive medical treatment and the subsequent recovery of cardiac function.145

Contraindications Although individual institutions may have variations to this list, the number of absolute contraindications to ECLS continues to decrease.146–149 Irreversible cardiac failure without the option for transplantation, irreversible lung failure, severe neurologic dysfunction, grade III/IV intracranial hemorrhage, uncontrolled bleeding, and lethal congenital anomalies/genetic syndromes are considered contraindications. Each center should evaluate carefully the decision to provide ECLS to these patients. Relative contraindications include prematurity (,34 weeks), weight less than 2 kg, and grade II intraventricular hemorrhage.149–151 Cardiac arrest is not a contraindication if rapid effective CPR was initiated, the underlying etiology is deemed reversible, and time to cannulation is less than 60 minutes. The challenge remains determining what is irreversible, shortened life span, poor prognosis, or severe, as these definitions have changed over time and with

Patients who require MCS are critically ill by definition, often with multiple end-organ dysfunction. Thus it is not surprising that complications occur with a greater frequency for this group than for other critically ill patients. Basic management principles are discussed in this section.

Cardiac Output When assessing the hemodynamic state of a patient, one must consider to what degree conventional support (mechanical ventilation and vasoactive agents) is contributing to cardiac and pulmonary function versus the ECLS system. The amount of flow provided by the ECLS system is measured through ultrasonic flow probes on the high-pressure side of the circuit beyond where any shunts (i.e., high-pressure to low-pressure connection through a hemofilter) may occur. An approximation of flow generated by the pump can be calculated from the product of the revolutions per minute (RPMs) and circuit tubing diameter. For most cardiac ECLS patients, flow is initiated at 120 to 150 mL/kg per minute. However, it is important to note that flow should be adjusted to fit the physiologic needs of each patient. Patients with septic shock may require higher flows, approaching 200 mL/kg per minute, to support their metabolic needs assuming that venous drainage is sufficient to yield such flows. A precise measurement of the patient’s intrinsic CO (i.e., the amount of blood flow that is not passed through the ECLS circuit) is unobtainable. However, indirect evaluation of the patient’s CO is possible through an assessment of arterial systolic blood pressure, pulse pressure, HR, organ perfusion, SvO2, and lactic acid levels on a given flow rate. Comparisons of these variables over time allow the clinician to make decisions regarding the adequacy of circulatory support or the readiness to wean from support. An echocardiogram can also provide valuable information about cardiac filling and function and guide therapy, particularly when weaning from ECLS or during a trial off ECLS. Svo2 is measured in the venous return portion of the circuit, with a goal of 65% to 80%. However, if a left-to-right shunt exists, such as a left atrial vent, the Svo2 will be falsely elevated, particularly if the patient is receiving high Fio2 and PEEP. Serial lactate measurements are often helpful to aid in the assessment of global end-organ perfusion. Elevated lactate levels may occur in patients with ongoing hepatic dysfunction, sepsis, low CO, and end-organ hypoperfusion. A rough approximation of the relative contributions of the patient and circuit pulmonary parameters is possible by analyzing serial patient and circuit blood gas assays (pH, partial pressure of arterial carbon dioxide, Pao2, Sao2) while taking into consideration the mechanical ventilation settings (including Fio2 and PEEP), circuit flow rate, gas sweep rate, and circuit O2 concentration. Finally, shunts within the patient or the circuit must be taken into consideration. Examples of right-to-left patient shunts include those with a patent ductus arteriosus, atrial septal defect, or ventricular septal defect in the setting of severe PAH. Circuit left-toright shunts are generally limited to a left atrial vent, open bridge, the arteriovenous hemodialysis filter, and in vivo continuous arterial blood gas devices. Total blood flow for the patient on MCS requires the addition of ECLS circuit flow with the patient’s native CO minus any shunt within the system as a whole.



CHAPTER 28

Troubleshooting Hemodynamic compromise often continues despite MCS. Lowdose inotropes or inodilators can aid cardiac contractility to augment native CO and reduce afterload to both the right and left ventricles. The most commonly used agents are dopamine (3–5 mg/kg per minute), epinephrine (0.03–0.05 µg/kg/min), dobutamine (5 µg/kg per minute), or milrinone (0.25–0.75 mg/kg per minute). Recall that while these are the typical modest range of doses in children, the presence of ECLS reduces the plasma level of these agents owing to the marked increase in the volume of distribution and circulating blood volume while on ECLS. The use of catecholamines in high doses is detrimental to cardiac recovery and should be avoided by increasing circuit flow to provide adequate CO. Likewise, the complete removal of any inotropic support is typically not suggested while on ECLS either, particularly when cardiac stun (lack of opening of the aortic valve during systole, which leaves the arterial line flat from only ECLS flow) is present. Tachydysrhythmias and pulseless electrical activity requiring cardiopulmonary resuscitation occurs in 3% of neonatal and pediatric ECLS runs. Pharmacologic/electrical cardioversion of any dysrhythmia should be attempted emergently even while on ECLS to prevent further deterioration in cardiac perfusion and function. Cardiac pacing can also be used to optimize CO. Hypovolemia Hypovolemia is a common occurrence during MCS for a variety of reasons. Inadequate venous drainage secondary to cannula malposition, cardiac tamponade, tension pneumothorax, or hemothorax may occur and generally results in hypotension requiring immediate correction. Ongoing evaporative losses from the oxygenator and bleeding secondary to coagulopathy can contribute to hypovolemia. Initiation of ECLS activates a host of inflammatory mediators, resulting in capillary leak and hypovolemia.152 Finally, attempts at mobilizing the large amount of “third-spaced” fluid with either diuretics, hemofiltration, or through drains in pleural/peritoneal cavities can quickly cause either inadequate CO from the patient or inadequate venous drainage to the ECLS circuit. Hypertension Hypertension secondary to neurohormonal dysregulation or pain/ agitation is one of the most common and unavoidable cardiovascular complications of ECLS. Although hypertension has not been demonstrated to negatively impact patient survival, its presence can worsen bleeding or further impair cardiac function and thus should be promptly addressed.153

Cardiac Stun Cardiac stun, a term that describes reversible global dyskinesia of the ventricle, was coined by Braunwauld and Kloner in 1982.154 Reversible cardiac dysfunction that results in the lack of antegrade LV ejection during systole resembles electromechanical dissociation, which has been observed frequently in patients following the initiation of VA ECLS. Excluding conditions of physiologic tamponade from thoracic issues, blood, or air, an infant on ECLS should have sufficient cardiac function to generate a minimal pulse pressure of 10 mm Hg. Evaluations of patients who experience cardiac stun upon initiation of ECLS defined this condition as the absence of aortic valve opening during systole, equalizing of the patient preductal and postmembrane circuit Pao2, and absence of pulse pressure in the aorta. The etiology of cardiac stun is multifactorial and hypothesized to be the sequelae of acute ischemia followed by reperfusion or severe

Cardiac Failure and Ventricular Assist Devices

257

electrolyte disturbances. The incidence of stun on ECLS is 5% to 12% in neonates and, when present and prolonged (.24 hours), results in a significant increase in mortality for these patients. Stun typically occurs during the initiation of bypass in patients who were hypoxic, hypercarbic, acidotic, and suffered a cardiac arrest prior to ECLS. Important factors in trying to minimize cardiac stun upon the initiation of ECLS include correcting the pH and ionized calcium levels in the circuit and infusing calcium chloride to the patient upon commencing ECLS, followed by close monitoring of arterial blood gases and electrolytes with rapid correction of abnormalities.145

Echocardiography and Cardiac Catheterization Initial diagnostic and hemodynamic assessment should be attempted by transthoracic echocardiography, but imaging windows may be severely limited resulting in insufficient information. TEE may improve diagnostic accuracy but may not be possible secondary to bleeding risks or patient size. Even if adequate imaging is obtained, the specific hemodynamic state of the patient on ECLS must be taken into account when interpreting these studies. Invariably, the loading conditions of the heart are markedly altered during ECLS. The echocardiographer should actively interface with the ECLS team (to potentially modify flows, add volume, modify mechanical ventilator support, etc.) in order to obtain the most comprehensive assessment of cardiac function to inform clinical decisions regarding the continuation or removal of further ECLS. Cardiac catheterization can be a useful tool for select patients who fail to wean from ECLS. The specific loading conditions of the heart must be considered in the context of the acquired data in order to make sound clinical decisions. Therapeutic interventions that might be performed in the catheterization lab include balloon or blade atrial septostomy to alleviate left atrial hypertension, balloon valvuloplasty or angioplasty of vascular obstructions, device closure of residual atrial or ventricular shunts, and/or coil embolization of aortopulmonary shunts.116 Correction of these types of residual defects in the catheterization laboratory or surgical correction in the operating room may be required to allow for separation from MCS.114 For patients with significant LV failure, it may be necessary to decompress the left heart to prevent or reverse pulmonary edema or hemorrhage, decrease mitral regurgitation and, importantly, improve coronary perfusion to increase the chances of myocardial recovery. In this scenario, venting of the LV occurs through surgical placement of a cannula in the left atrial appendage connected to the venous drainage to the ECLS circuit. This is accomplished in the operating room/ICU during or after transthoracic ECLS cannulation or in the catheterization laboratory through creation of an interatrial connection via balloon or blade septostomy. This procedure can markedly improve LV function and increase the chances of survival.155,156 Single Ventricle Lower survival rates are universally found in this subset of patients, which may be attributed to an imbalance of the systemic and pulmonary circulations, volume burden to the single ventricle after complex palliative surgery, compromised single-ventricle function (particularly with RV morphology), and/or impaired coronary perfusion to the systemic ventricle when a systemic-topulmonary artery (PA) connection (i.e., modified Blalock-Taussig [BT] shunt) results in diastolic runoff from the aorta. Despite these challenges, larger centers continue to report improved outcomes with the accumulation of experience and application of

258

SECTION IV



Pediatric Critical Care: Cardiovascular

innovative strategies. For example, initial efforts to balance the systemic and pulmonary circulations on ECLS included either completely or partially occluding the aortopulmonary shunt, which has now been demonstrated to increase mortality. The use of smaller-size BT shunts or the use of the Sano modification (RV to PA nonvalved conduit for stage one hypoplastic left heart palliation) has contributed to decreasing the recirculation that would otherwise occur. Of note, higher ECLS with flows approaching 200 mL/kg per minute may be required to provide adequate systemic and pulmonary support when the shunt is left open.157,158

Anticoagulation Strategies ECLS requires meticulous management of hemostasis to limit patient morbidities. Hemorrhagic and thrombotic complications are a major concern for patients during ECLS, particularly after cardiac surgery. Bleeding can manifest at surgical sites (arterial/ venous cannulation sites, surgical repair sites [atriotomy, ventriculotomy, aortotomy sites], sternal incision, indwelling catheter sites, etc.) or may be masked in areas such as the thorax, intracranial vault, or the gastrointestinal tract. A meta-analysis of observational studies, including 1763 patients on VA ECMO, reported a 33% incidence of bleeding that was mostly correlated to the heparin monitoring strategy.159 Prevention strategies that target reduction of hematologic complications focus on maintenance of the hemostatic regulatory mechanisms as close to normal as possible.160–163 Patients placed on ECLS following cardiac surgery represent a unique population at risk for hemorrhagic management since they often have multiple surgical sites, dilutional coagulopathy, and abnormal coagulation patterns. Apart from single-center experience, no well-defined consensus or protocol is available for pediatric and neonatal ECLS. The patient’s age, diagnosis, clinical status in conjunction with the specific details of the ECLS device, and, finally, the flow through the circuit will dictate which anticoagulation strategy should be employed. The most commonly used agent for anticoagulation on ECLS is continuously infused unfractionated heparin (UH). Most centers measure platelet counts, hematocrit, prothrombin time (PT), partial thromboplastin time (PTT), fibrinogen, specific factor levels (i.e., anti-factor Xa, antithrombin III [ATIII]), quantitative heparin levels, and activated clotting time (ACT). The ACT is the most commonly measured test for coagulation, as it can be performed quickly at the bedside, though other coagulation tests are available at the point of care (i.e., PTT).164,165 Despite this advantage, ACT results vary markedly based on the technique used and are a nonspecific measure of coagulation because they measure the total time for a clot to form after ex vivo activation. The ACT is a global composite of the different individual components of coagulation; thus, more specific tests (listed earlier) must be used in concert to ascertain which specific component in the coagulation cascade is affected. No single test is currently used to guide the anticoagulation regimen; rather, a panel of tests must be obtained and analyzed in concert. Increasingly, many centers have reported on the effective use of thromboelastography (TEG) in determining patient coagulation status.166 When “adequately anticoagulated on a continuous unfractionated heparin infusion,” the target ACT should be between 180 and 220 seconds when the ECLS pump is flowing at full flow (i.e., .150 mL/kg per minute). This goal can be decreased to 160 seconds on full flow if significant bleeding is present, particularly in the immediate postoperative period. UH is metabolized via two mechanisms: at low doses, via a saturable mechanism representing clearance by the reticuloendothelial system and endothelial cells to which heparin binds with

high affinity, and at high doses, via a nonsaturable mechanism represented by renal excretion. Thus, close monitoring during ECLS must be followed when a patient’s urinary output is oscillating between oliguria and polyuria. Newer treatment strategies include the use of additional agents such as ATIII in either bolus or infusion form to correct abnormalities in the clotting cascade. ATIII is an a2-glycoprotein serine protease inhibitor that inactivates a number of enzymes from the coagulation system, including the activated forms of factors II, VII, IX, X, XI, and XII. Replacement of ATIII is controversial, with some centers replacing it only when the level is low (,30%) and heparin infusion rates are increasing without a concomitant increase in ACT or PTT,167,168 while other centers maintain levels near 100%. Bivalirudin is a direct thrombin inhibitor that works independent of antithrombin on both circulating and clot-bound thrombin. It is currently approved for use during percutaneous coronary intervention and heparin-induced thrombocytopenia. It has a quick onset of action and a short half-life. Bivalirudin infusions have been reported in a small number of pediatric patients on ECLS.169–171 At our institution, we have successfully used bivalirudin in patients with suspected heparin-induced thrombocytopenia, heparin resistance, risk of major bleeding, and/or evidence of thrombosis while on heparin. Ranucci et al. demonstrated that the use of bivalirudin in ECMO was associated with less total blood loss and fewer transfusion needs in postcardiotomy patients requiring ECMO.172 There is wide variability on the dosing strategies, but at our institution we start the infusion at 0.15 to 0.20 mg/kg per hour without the administration of a bolus and titrate to goal PTT 60 to 80 seconds. Anticoagulation with Coumadin, clopidogrel, low-molecular-weight heparin, and aspirin individually or in combination can be considered with certain MCS devices other than ECLS.173 Heparin-induced thrombocytopenia (HIT) is a relatively rare but serious complication of heparin administration caused by antibodies binding to a complex of heparin and platelet factor 4 that leads to large-vessel thrombosis and increased mortality. A drop in platelet count by more than 50% of the highest previous value should raise suspicion of HIT and trigger investigation. Treatment includes discontinuing heparin administration and initiating direct thrombin inhibitor therapy (i.e., argatroban) if continued anticoagulation is needed.160,174 The lysine analogs tranexamic acid and ε-aminocaproic acid are antifibrinolytic agents that have been shown to reduce bleeding in ECLS patients undergoing surgical procedures. However, prophylactic administration failed to reduce the incidence of intracranial hemorrhage in neonates.162,175 Profound abnormalities in many components of the coagulation cascade commonly occur in postoperative cardiac patients on ECLS. With this in mind, an attempt to normalize components of coagulation not impacted by heparin is important. Platelets are consumed at surgical bleeding sites, sequestered by the membrane oxygenator; thus, transfusions are required to maintain counts greater than 100,000/mm3 in bleeding patients or patients at high risk. However, lower transfusion thresholds can be set to avoid excessive exposure to blood products in the nonbleeding patient. Administration of fresh-frozen plasma to broadly increase multiple coagulation factors activity is also common. When hypofibrinogenemia occurs, cryoprecipitate infusion can be used owing to the high concentrations of fibrinogen in the low volume of the cryoprecipitate unit. The availability of plasma protein concentrate allows administration of highly concentrated factors in a



CHAPTER 28

substantially reduced volume, decreasing the volume burden associated with conventional therapy. Although rare, the application of heparin-free ECLS has been reported.176,177

,

Ventilation Strategies Ventilator management during ECLS remains controversial. Although data exist to guide clinicians regarding prevention of barotrauma, volutrauma, and O2 toxicity for mechanically ventilated patients with acute respiratory distress syndrome in general, there is a lack of controlled clinical trials and consensus for pediatric patients on ECLS.178–180 Lung collapse strategies used for respiratory support on ECLS are not used by most cardiac centers. Goals continue to target the prevention of atelectasis with utilization of appropriate PEEP in order to maximize oxygenation of nonbypassed blood returning to the left atrium, which then is ejected by the LV to perfuse the coronary arteries. In addition, providing modest levels of ventilator support can be achieved with either a pressure- or volume-limited mode of ventilation targeting a delivered tidal volume of 6 mL/kg. Respiratory rates between 10 and 25 are set depending on the age and rest strategy being employed and the degree that the patient’s lungs are required for gas exchange. Optimizing Pao2 to the patient and circuit by blending Fio2 to keep the Fio2 below 0.6 reduces free radical formation and O2 toxicity, providing adequate oxygenation to decrease pulmonary hypertension and optimize myocardial Do2. Chest radiographs are routinely performed to assess and guide strategies to optimize lung volume so that volutrauma and barotrauma are avoided. Fluid, Nutrition, and Renal Fluid overload and electrolyte disturbances such as high or low serum levels of potassium, calcium, magnesium, and phosphorus are common and need correction. Most cardiac patients on ECLS receive total parenteral nutrition owing to the increased risk of gastrointestinal complications in patients with cardiac defects, umbilical artery catheters, poor perfusion, or other bowel abnormalities. A select subset may tolerate trophic enteral feedings; whenever possible, even a small amount of enteral feeds should be provided. Diuretics—commonly, furosemide—are often employed to provide optimal fluid balance in patients with significant capillary leak and fluid retention. Optimization of fluid status is essential to weaning and eventual separation from ECLS. For anuric or oliguric patients, early placement of an in-line hemofilter into the ECLS circuit with or without countercurrent dialysate or a complete continuous renal replacement therapy device is recommended. Still, the management decisions that balance the use and timing of diuretics versus hemofiltration are center specific. The indications for initiation of dialysis are the same as for other critically ill patients with renal failure. Multiple retrospective reviews of patients supported by ECLS have found that fluid overload and AKI are risk factors for increased mortality.181–183

Analgesia and Sedation Adequate analgesia and sedation are essential for both safety and comfort and to decrease metabolic demands in patients with circulatory compromise in the early postoperative recovery period. While an opiate and a benzodiazepine class drug have been used historically, a multimodal approach is increasingly preferred, which could include other agents such as dexmedetomidine, ketamine, or propofol. Current oxygenators can bind many drugs, including opioids, depending on the lipophilic and proteinbinding qualities of the specific drug. Thus dosing amounts are

Cardiac Failure and Ventricular Assist Devices

259

usually significantly higher while on ECLS.178,184 In this study, ECMO circuits were set up using Quadrox-iD pediatric oxygenators primed with whole blood to represent a 5-kg patient. Hydromorphone and fentanyl were injected (also, mycophenolate and tacrolimus) and serial blood samples were taken over 12 hours. In this ex vivo model, hydromorphone hydrochloride was not as significantly sequestered compared with fentanyl (mycophenolic acid and tacrolimus serum concentrations were stable). Benzodiazepine infusions may be used with cautious monitoring, as toxicity from propylene glycol (lorazepam), other solvents (midazolam), or long-acting toxic metabolites (diazepam) have been reported in critically ill neonates, infants, and children.185 Recent studies are finding an increase in delirium when benzodiazepines are used in critically ill pediatric patients; thus, some centers are beginning to limit their administration. Neuromuscular blockade should largely be avoided to limit the development of critical care neuromyopathy, allow for regular evaluation of the central nervous system, and limit soft-tissue fluid accumulation. Central nervous system infarcts, hemorrhage, or seizures are all known complications of ECLS. For infants with an open fontanel, a daily head ultrasound should be performed early in the course of treatment and with any change in clinical neurologic status. For older patients, a significant change in their neurologic status has a high likelihood of heralding major intracranial pathology, which needs to be promptly diagnosed by computed tomography in order to guide treatment and determine patient viability. Several centers have reported a new approach in which older patients are supported without the need of continuous analgesia and sedation and without the need of mechanical ventilation. This awake ECLS modality has been used as bridge to recovery, bridge to VAD, or bridge to transplantation.186,187

Infection It is not surprising that patients on ECLS are at a high risk of developing nosocomial bloodstream infections (BSIs). Identified risk factors include the duration of ECLS,188 open versus closed chest cannulation, the presence of central venous lines, and undergoing a major procedure prior to or while on ECLS. It appears that older patients on ECLS for respiratory failure may be at higher risk of healthcare-associated infection than neonates or cardiac patients. BSIs during ECLS for both pediatric cardiac patients post-CPB and neonates with cardiac or respiratory failure have been associated with a poor outcome. The diagnosis of sepsis is difficult in patients supported with ECLS. Although variable degrees of leukopenia have been documented for neonates supported with ECLS, an increase in phagocytosis and intracellular killing by neutrophils also occurs. Temperature is controlled by the circuit’s heat exchanger; thus, infection is generally not manifested by fever in these patients. Hypotension or thrombocytopenia may occur for a variety of reasons. In view of this observation, the standard of care in many ECLS centers has been to perform routine surveillance cultures and provide prophylactic antibiotics. However, management strategies to limit infectious risks continue to evolve. Owing to lack of a proven benefit and concerns regarding the long-term impact of broad-spectrum antibiotics on local bacterial resistance profiles, an increasing number of centers now perform daily blood cultures without the routine use of prophylactic antibiotics. Additional retrospective data may suggest that routine surveillance cultures may not be warranted.189 Further investigation is needed to determine the impact of prophylactic antibiotic use on the incidence of BSI, local antimicrobial flora, length of stay, survival, and cost.190

260

SECTION IV



Pediatric Critical Care: Cardiovascular

Intrahospital Transport Crucial situations exist for patients supported with ECLS that require intrahospital transport. This can include mobilization from the intensive care unit to a variety of locations, such as the catheterization laboratory, radiology suite, or the operating theater. Reluctance to perform diagnostic or therapeutic interventions is often driven by fear of potentially disastrous complications during transport, yet these fears are largely unfounded. Guidelines designed to promote the establishment of an organized, efficient transport process supported by appropriate equipment and personnel have been recognized and are increasingly used in hospitals. Intrahospital transport for patients on ECLS is a labor-intensive process that should be approached in a coordinated effort with specific focus on the preparatory phase, the transfer phase, and the posttransport phase. Simulation practice with the various teams is now standard practice, resulting in safe intrahospital transport for patients on ECLS.191–193

Ventricular Assist Devices The two main types of VADs are pulsatile pumps and continuousflow pumps. Although the initial types of VADs were pulsatile, continuous-flow pumps have been gaining popularity worldwide both for adult and pediatric patients (eTable 28.3) because of their decreased incidence of thromboembolic complications, smaller size, and ability to discharge patients home. Most VADs share similar basic principles, and based on the pump-patient interface, devices can be classified as intracorporeal or paracorporeal. Cannulation depends on the type of support required. The right atrium (systemic venous drainage) and PA (arterial return) are cannulated for a right ventricular assist device (RVAD), and the LV (pulmonary venous drainage) and aorta (arterial return) are cannulated for a left ventricular assist device (LVAD). A combination of both right and left ventricular assist devices is termed a biventricular assist device (BiVAD). For children with complex CHD, including single-ventricle physiology, placement of the inflow and outflow cannulas can be varied and complex. The pump is connected to a controller and power supply and other monitoring sensors. A comparison of ECLS and VADs is listed in eTable 28.4.

Pulsatile Ventricular Assist Devices Pulsatile VADs function on the principle of positive displacement by trapping a fixed amount of blood and then forcing (displacing) that trapped volume into an exit cannula. Since the early report of pneumatically driven, pulsatile, paracorporeal VADs in children, the pediatric experience with pulsatile devices has continued to grow.194,195 Advantages of these devices include the ability to support infants and toddlers, provide long-term support (weeks to months), provide biventricular support without an oxygenator, increase patient mobility, and provide pulsatile flow to the body that more closely mimics the normal output of the heart. The external position of the pump allows device exchange in case of malfunction or thrombus formation. Disadvantages include a propensity for thromboembolic complications and the need for exteriorization of the cannulae. Infection is a serious complication, though immobilization of the cannulas close to the exit site can decrease its incidence. These devices have a special silicone system that connects the blood pump to the body.196,197 Most times, LVAD insertion may be sufficient for bridging patients to heart transplantation, even in the presence of significant preoperative RV dysfunction. Adequate unloading of the LV reduces the LV end-diastolic pressure and, in

turn, reduces RV afterload, which may improve RV systolic function. If there is a significant burden of tachyarrhythmia, severe RV dysfunction at baseline, or failure of the RV to recover despite LVAD placement, BiVAD support should be considered. Some of the early devices include the Thoratec Ventricular Assist System (Thoratec Corp.), which is a pneumatically driven, polyurethane sac enclosed in a plastic housing designed for intermediate and long-term use (weeks to years). The Thoratec VAD has a stroke volume of 65 mL and can be operated at rates up to 100 beats per minute, providing blood flow rates of almost 7 L/ minute. There are two Thoratec VAD systems, one paracorporeal (PVAD) and one implantable (IVAD). The Thoratec Paracorporeal Pneumatic VAD (PVAD) is indicated as a bridge to transplantation or bridge to recovery system. It can provide acute or intermediate uni- or biventricular support. Furthermore, it can be used in smaller patients who are poor candidates for implantable devices, with the smallest reported patient weighing 17 kg (body surface area [BSA] 0.73 m2). The Thoratec IVAD is a smaller, intracorporeal device with the same features as the Thoratec PVAD except that it is used when longer support is anticipated. The device has been FDA approved since 2004 for circulatory assistance as a bridge to transplant or bridge to recovery.198,199 The long-term pulsatile device that has had the best outcomes in comparison with ECMO is the Berlin Heart EXCOR. It is currently the most commonly used and accepted VAD and is the only labeled VAD available for long-term support of neonates and infants. This device, first approved for use in Europe in the late 1990s, is a paracorporeal pulsatile pneumatic pump with polyurethane valves and a transparent chamber through which the adequacy of filling and emptying can be assessed and evaluated for thrombus formation. This pump can be used to support left, right, or both ventricles. The blood-contacting surfaces of the pump are heparin coated, reducing anticoagulation requirements. Because it is available in a wide range of sizes, the Berlin Heart EXCOR VAD provides circulatory support options for pediatric patients ranging from 2.5 kg to adolescents. The Berlin Heart EXCOR was specially designed and developed for pediatric patients. The pump sizes vary between 10 mL and 60 mL. The 10-mL and 15-mL pumps are suitable for neonates, infants, and young toddlers. The 25-mL and 30-mL pumps can be used in children usually younger than 10 years (BSA 1 m2). Adult-size pumps (50 mL, 60 mL) can be implanted in older children. Sizes are adopted based on the ability to provide cardiac index of greater than 2.5L/min per m2 with the VAD rate preferably below 120 beats/min for non–singleventricle physiology support. All Berlin Heart EXCOR cannulae exit the body through the upper abdominal wall (Fig. 28.6). Survival with the Berlin EXCOR in the pediatric population has been reported across multiple centers internationally, with survival to discharge at 60% to 80% in children from 2 weeks up to age 16 years. Survival is significantly better in patients with myocarditis and dilated cardiomyopathy than for patients with CHD. Neurologic complications vary between 25% and 30%.136,200–202 The Berlin Heart EXCOR pediatric VAD multicenter trial was published in 2012, which led to its approval by the FDA. Although data from the trial suggest that 90% of children can be bridged to transplantation with the EXCOR, with a stroke risk of 29%, the primary cohort captured only one-fourth of all US children implanted with the EXCOR during the 3-year study period and did not include patients in whom the device was implanted as compassionate use. In a later publication, Almond et al. examined EXCOR outcomes in all 204 children implanted during the study period.200 Overall survival in this unselected

260.e1

eTABLE PediMACS Approved Devices (Age ,19 Years) 28.3

Device Class

Device Brand

Paracorporeal continuous

Thoratec Centrimag Thoratec Pedimag Maquet Rotaflow Sorin Revolution

Implantable continuous

HeartWare HVAD HeartMate II LVAD

Percutaneous

Abiomed Impella 2.5/5.0 Tandem Heart Abiomed Impella CP

Total Artificial Heart

Syncardia Total Artificial Heart

LVAD, Left ventricular assist device.

eTABLE Extracorporeal Life Support vs Ventricular Assist Devices 28.4

Pulsatile VAD

Oxygenator

Yes

No, but can be added

No

Anticoagulation

ACT 180-200

Heparin 1 aspirin Long term: LMWH/warfarin 1 aspirin

Heparin 1 aspirin dipyridamole Clopidogrel Long term: LMWH/warfarin 1 aspirin dipyridamole clopidogrel

Support

Cardiac 1 respiratory

Cardiac

Cardiac

Type of ventricular support

Biventricular

Uni- or biventricular

Uni- or biventricular

Ventricular decompression

May need LA “venting”

Via direct drainage cannula in LA or LV apex

Via direct drainage cannula in LV apex

Risk of air embolus

Low

Yes (especially with LVAD)

Yes (especially with LVAD)

Length of support

Short term

Short/long term

Long term

Cannulation site

Transthoracic or transcervical

Transthoracic

Transthoracic

6

6

6

Continuous-flow VAD

6

ECLS

ACT, Activated clotting time; ECLS, extracorporeal life support; LA, left atrial; LMWH, low-molecular-weight heparin; LV, left ventricular; LVAD, left ventricular assist device; VAD, ventricular assist device.



CHAPTER 28

2

4

Cardiac Failure and Ventricular Assist Devices

261

3

1

B

A • Fig. 28.6



​(A) Berlin Heart biventricular assist device showing apical cannulation, which allows better left ventricular unloading and decreased afterload to the right ventricle. 1, Deoxygenated blood flows from the body into the right atrium. Because the right ventricle is unable to pump blood into the lungs, the blood goes into the device. 2, The blood is pumped out from the device into the pulmonary artery. 3, Once the blood is oxygenated in the lungs, it flows into the left atrium. Because the left ventricle is unable to pump, the blood goes into the device. 4, The Berlin Heart pumps blood into the aorta and systemic circulation. (B) Set of cannulas for the Berlin Heart assist device. (B, Courtesy Berlin Heart, Inc., The Woodlands, TX.)

cohort was lower at 75%, and the risk of neurologic dysfunction was similar at 29%. Children were more likely to die on EXCOR support if they had significant renal or hepatic impairment at implantation or RV failure requiring BiVAD support. In addition, children weighing less than 5 kg did significantly worse. These findings demonstrate that EXCOR survival varies considerably and depends heavily on patient characteristics at implantation, underscoring the need for careful patient selection. Specifically, excessive delay in implanting a device until renal, hepatic, or RV function has failed increases the risk of death on EXCOR support, whereas implanting too early may also escalate mortality by unnecessarily exposing children to the risks of support (i.e., stroke, death). Most of the neurologic events are secondary to left atrial cannulation; hence, many centers favor ventricular apical cannulation. It is encouraging to note that even within the short 3-year time frame of this study, pediatric centers that have refined their patient characteristics over time are observing significantly lower patient mortality.175,203–205

Continuous-Flow Ventricular Assist Devices Short- to Medium-Term Ventricular Assist Device Support Centrifugal pumps have been available since the late 1970s to support postoperative cardiac failure.206 They have been an effective support for infants and children with myocardial failure from

a variety of etiologies (e.g., acute myocarditis, dilated cardiomyopathy, acute transplant rejection, anomalous left coronary artery from the PA).207,208 Centrifugal pumps are used for short-term support and are, in essence, mini-ECLS circuits without an oxygenator. The CentriMag VAD (Thoratec Corp.) belongs to a class of magnetically levitated devices operating in a bearingless rotor that floats within a rotating magnetic field. Because of its contact-free environment and absence of seals or valves, this device minimizes blood trauma, thrombus formation, and hemolysis. It is capable of operating over a range of speeds up to 5500 RPM, generating flows up to 10 L/minute. The Thoratec PediMag blood pump, a miniaturized version of the CentriMag, is an extracorporeal circulatory support device providing hemodynamics stabilization for small patients (,10 kg) in need of cardiopulmonary assistance. It is cleared for clinical use up to 6 hours; thus, it can be used as a short-term solution to support the circulation while longer-term options are considered. The device is capable of flowing up to 1.5 L/min. These devices are generally used for short- or intermediateterm support for recovery, conversion to a long-term VAD, or as a bridge to transplant.209,210 Other temporary continuous-flow devices include Jostra Rotaflow (MAQUET Cardiovascular). It is a centrifugal extracorporeal pump (50 mm in diameter, made of polycarbonate material). It has one point bearing with three magnetic fields powered via an

262

SECTION IV



Pediatric Critical Care: Cardiovascular

electromagnetic mechanism. This levitated mechanism reduces mechanical friction, hemolysis, and overall wear.211 Another system of use in pediatrics is the Tandem Heart (CardiacAssist) VAD. It is a centrifugal pump, like the devices already mentioned, that is currently approved by the FDA for clinical use in adults.212 Kulat et al. modified the system to include a variable restrictive recirculation shunt to allow lower flows for support in smaller patients.213,214 Advantages of these short-term VAD systems compared with traditional ECLS include lower priming volumes, adequate left (or systemic) ventricular decompression, lower heparin requirements, less hemolysis, ease of transport, and relatively low costs.215 In addition, central cannulation spares the neck vessels. These short-term devices are now used also for longer duration of support, up to 3 to 6 weeks.216 Centrifugal LVADs require a functional RV to supply preload to the LV drained by the pump. Complications of the centrifugal VAD system include thrombus formation in the circuit, hemolysis, bleeding (including acquired von Willebrand disease217 and infection218). Yarlagadda et al. reported the outcome of children listed for heart transplantation between 2011 and 2015 supported with temporary circulatory support (TCS) devices compared with a match control of children supported with ECMO during the same period. The number of implanted TCS devices increased from 3 or fewer before 2011 to 50 in 2015. A total of 93 children were implanted (59% LVAD, 23% RVAD, and 18% BiVAD). The most commonly used device was the CentriMag-PediMag system (65%), followed by TandemHeart (18%) and Rotaflow (6%). The support duration was longer for the device cohort (median, 19 days vs. 6 days; P , .001) and was longest for CentriMag-PediMag, with 27% supported for more than 60 days. The TCS device cohort had longer overall survival (hazard ratio, 0.61; 95% confidence interval, 0.39–0.96), with 90-day mortality before transplant that was modestly reduced (from 45% with ECMO to 39% with TCS).219 Successful use of the TCS as a bridge to implantable VAD and then to heart transplantation in a patient with single-ventricle physiology has recently been reported.220

Long-Term Ventricular Assist Device Support After the first-generation, long-term pulsatile VADs were noted to have significant complications, the use of axial continuous-flow devices increased with the hope to decrease device-related complications. The initial long-term axial continuous-flow devices included the Heart Assist 5 MicroMed DeBakey VAD (Fig. 28.7) and Jarvik 2000. The MicroMed DeBakey VAD was approved for use in pediatric patients in 2004. It supported pediatric patients as small as 18 kg, providing a wide degree of cardiac support, with blood flows from 1 to 10 L/min. This was an implantable LVAD, focusing on bridge to transplantation. It represented the first truly miniaturized device. The advantages of this device were its small size, relative ease of implant and explant, decreased infection risk, and continuous flow. It had a significant risk of thrombosis in addition to the risk for hemolysis.221,222 The Jarvik 2000 was an adult device with a relatively small surface area, measuring 1.8 cm in diameter by 5 cm in length. The device usually is implanted into the LV apex via a left thoracotomy, and the outflow graft is placed into the descending aorta. Blood flow ranges from 2 to 7 L/min and is determined by impeller speed and systemic vascular resistance, with the usual setting at 9000 RPMs (range, 8000–12,000 RPMs). An infant Jarvik 2000 was under investigation but, owing to issues with hemolysis, had been modified to the Infant Jarvik 2015, which is being tested in the PumpKIN (Pumps for Kids, Infants, and Neonates) trial.223,224

•  Fig. 28.7

​HeartMate II ventricular assist device. (Courtesy MicroMed Technology, Houston, TX.)  

In the context of axial continuous-flow devices, the HeartMate II (Thoratec Corp.) is the most commonly used implantable axialflow rotary pump using blood-immersed mechanical bearings with textured blood-contacting surfaces. The percutaneous version is totally implantable. The HeartMate II (Fig. 28.8) is smaller than the first-generation devices, principally owing to the elimination of the sac or reservoir necessary in pulsatile pumps. This



CHAPTER 28

Cardiac Failure and Ventricular Assist Devices

263

Flow inducer/impeller Flow straightener

Flow diffuser (1) Pre-sealed outflow graft

(2) Exclusive flow probe Front bearing

A

Flow housing

Magnet pieces

Rear bearing

Motor stator

B

(3) Near silent pump

(4) Rigid titanium inflow cannula

Actual size: 71mm × 30mm, 92 grams

C •  Fig. 28.8



​HeartAssist 5 pediatric ventricular assist device (modern DeBakey ventricular assist device). (A) DeBakey ventricular assist device in a cutaway view. (B) HeartAssist 5 with its position in the heart.​ (C) HeartAssist 5 shown with a US quarter for size comparison. (Courtesy Abbott Laboratories, Abbott Park, IL.)

device has two cannulas (inflow and outflow) without valves. It has smooth surfaces in the inlet and outlet stators but still requires anticoagulation. Clinical experience shows that the HeartMate II LVAD provides excellent support, with significant improvement in functional capacity. HeartMate II is approved for both destination therapy and bridge to heart transplantation. In a retrospective review of the Interagency Registry for Mechanically Assisted Circulatory Support (INTERMACS) data from 2008 to 2011, there were 28 pediatric patients, with 7 (25%) undergoing device placement in a pediatric hospital. At 6-month follow-up, the composite of survival to transplantation, ongoing support, or recovery was 96% for the pediatric group, which was not significantly different from the young adult group. Bleeding complications requiring surgical intervention were more common in the pediatric group.225 The FDA has already approved the HeartMate II as a bridge to transplant and for destination therapy. The device has low thrombogenicity and low thromboembolic risk, making it a good device option for destination therapy. The newer generations of continuous-flow devices over the past 10 years include centrifugal continuous-flow pumps with an impeller or rotor suspended in the blood flow path using a noncontact bearing design, which uses either magnetic or hydrodynamic levitation. The levitation systems suspend the moving impeller within the blood field without any mechanical contact, eliminating frictional wear and reducing heat generation. This feature hopes to promise longer durability and higher reliability with a low incidence of device failure and need for replacement. Usually, magnetic levitation devices are larger owing to the need for a complex position sensing and control system. Examples of thirdgeneration devices are the DuraHeart (Terumo, Inc.), HVAD (HeartWare Corp.), HeartMate III (Abbot Inc.) and EVAHEART LVAS (Sun Medical Technology Research Corp.).226,227

The HeartWare HVAD system (Medtronic) has gained significant popularity in the pediatric world for its use in older children and adolescents given its smaller size. Its ability to be intracorporeally implanted allows the patient to be discharged home on VAD support. The HVAD system was approved by the FDA in 2012 for adults through the ADVANCE (Evaluation of the HeartWare LVAD System for the Treatment of Advanced Heart Failure) trial, and its application in pediatrics has increased since the first reported use as a bridge to heart transplantation in an adolescent.228 As the implantable continuous-flow devices continued to increase in pediatrics so did the use of the HVAD. VanderPluym et al. reported the outcome of children supported with an intracorporeal continuous-flow LVAD who were entered in the Pediatric Interagency Registry for Mechanical Circulatory Support (PediMACS), the pediatric portion of INTERMACS. They identified 192 children from 2012 to 2017 with a median weight of 51.5 kg and a median support of 2.8 months. Twelve children weighed less than 20 kg at time of implant, and the majority (58.3%) had CHD compared with 11.7% in children who weighed 20 kg or more. Serious adverse events in children included infection (27%), major bleeding (23%), device malfunction or pump thrombosis (11%), and stroke (10%). Interestingly, children who weighed less than 20 kg at time of implant had lower major bleeding complications, infections, and stroke compared with the older counterparts. Almost half of the children who weighed 20 kg or more were discharged home as they waited for a transplant as opposed to only 33% of children who weighed less than 20 kg.229 The HeartMate III, as opposed to the HeartMate II (axial continuous-flow device) is a centrifugal flow, fully levitated, selfcentering rotor pump with intrinsic pulsatility. The HeartMate III was approved by the FDA in 2018 for destination therapy following the MOMENTUM trial.230 The study showed that although

264

SECTION IV



Pediatric Critical Care: Cardiovascular

the disabling thromboembolic complications were similar, the rates of overall stroke and pump malfunction were lower in the HeartMate III than in the HeartMate II.227,231 The use of this device in the pediatric population continues to increase not only in children with dilated cardiomyopathy but also in those with CHD.225,232

Total Artificial Heart Total artificial heart (TAH) devices completely replace the patient’s native ventricles and all four cardiac valves. The SynCardiaSystems TAH is the modern version of the Jarvik 7 implanted in 1982. It is the only FDA-approved TAH in the United States. It is used as destination therapy and, more recently, as a bridge to heart transplant. As the device has evolved, so too have its indications and its name, from Symbion TAH to CardioWest TAH, to the SynCardia TAH most recently. The 70-mL SynCardia-t TAH is suitable for patients with BSA of 1.7 m2 or greater. The SynCardia-t TAH is approved for outpatient use, allowing many patients to be discharged home. The development of the 50 mL SynCardia TAH addressed the need for a smaller pump for use in small adults and adolescents down to a BSA of 1.2 m2. Indications for TAH implantation are numerous, with the most common being biventricular failure. In patients younger than 19 years, cardiomyopathy comprises 61% of the indications, with dilated cardiomyopathy being the most common.233 Although initially designed for use as destination therapy, most of the implants worldwide have been used as a bridge to transplantation. Progression to transplant occurs at a reported rate of 60% to 80% in the adult population.234,235 Morales et al.236 reported the worldwide experience for all patients supported with a TAH between 2005 and 2015 (n 5 43). The most common diagnoses included dilated cardiomyopathy (42%), followed by transplant rejection (19%) and CHD (16%). Successful bridge to transplantation varied by diagnosis; 75% of the patients with dilated cardiomyopathy were bridged to transplant, whereas only 25% of the patients with graft rejection were transplanted. Children and adolescents supported with a TAH had survival rates similar to adults with the TAH or biventricular devices. Preclinical in vivo trials are underway for continuous flow (CF) TAH devices. The Cleveland Clinic valveless, sensorless, pulsatile pediatric continuous-flow TAH (p-CFTAH) promises its use in small children. Anatomic fit studies correlating BSA with vertebral-sternal distance projects that the p-CFTAH could be suitable for children with BSA as small as 0.3 m2.237

Device Selection Three factors drive device selection: type of support (cardiopulmonary or cardiac), planned duration of support, and BSA. Short-term support (,14 days) is used for acute myocarditis, graft dysfunction after cardiac transplant, and postcardiotomy. Venoarterial ECMO/ECLS has been successfully used and is the most widely available form of short-term biventricular support. ECLS is commonly employed when there is failure to wean from CPB. When compared with temporary VADs, it has the advantage of peripheral cannulation, which can be done in the intensive care setting. Temporary VADs, on the other hand, spare cervical vessel loss, allow for better decompression of the LV, and minimize the inflammatory response given the absence of an oxygenator. Compared with ECLS, it can provide longer support time (2–4 weeks) as a bridge to decision for a long-term VAD. Temporary VADs are

•  Fig.

28.9 ​Pediatric Jarvik 2015 shown with a paperclip for size comparison.  

better suited to semi-elective cases before the onset of cardiac failure has resulted in secondary lung injury.238,239 Long-term support is often used in patients with cardiomyopathy and in some patients with CHD who fail to improve on a short-term device. The Berlin EXCOR continues to be the device of choice for infants and toddlers, while the HVAD is predominantly implanted in pediatric patients with a BSA greater than 0.60 to 0.65 m2. The HeartMate II is an excellent option for adolescents with BSA approximately 1.3 m2 or larger. According to the PediMACS registry, continuous-flow VADs account for more than 60% of long-term device implantations in the United States. However, the continuous-flow devices used are designed for adults, leading to a patient-device size mismatch, especially in small children. The PumpKIN trial, initially a two-arm randomized study (Jarvik 2015 vs. Berlin Heart EXCOR) is now being reevaluated with a potential transition to single-arm testing the Infant Jarvik 2015 (Fig. 28.9), which is the first continuous-flow VAD specifically designed for small children.224

Indications and Management In general, the use of MCS in pediatric patients with acute or chronic end-stage HF is indicated when conventional medical therapy has failed. Newer considerations include inability to wean mechanical ventilator support with aggressive use of inotropic support, which prevents appropriate nutritional and physical rehabilitation. Indication for VAD implant in pediatrics has now evolved from the traditional bridge to transplant to include bridge to recovery as well as bridge to decision. The use of MCS as destination therapy has been a traditional option in the adult population. Although limited in pediatrics, it is garnering more interest240 and has been recently reported in patients with Duchenne muscular dystrophy.241 Contraindications to the use of VADs in pediatric patients include irreversible end-organ dysfunction; risk of intracranial bleed, as with congenital AV malformations (e.g., moyamoya disease); underlying risk of increased bleeding, such as coagulation factor deficiencies; or, conversely, thrombotic disorders, such as factor V Leiden deficiency. Also, active malignancy and morbid



CHAPTER 28

obesity are considered contraindications.242 Other considerations include thickness of the ventricular wall, semilunar valve regurgitation, and intracardiac shunts. Thick ventricles (such as those in hypertrophic cardiomyopathy) can prevent proper filling of the VAD; these patients should be considered on a case-by-case basis.243 Significant aortic insufficiency (or pulmonary insufficiency) will not permit adequate emptying of the ventricle and often necessitates closure of the aortic valve at the time of VAD implantation. Closure of intracardiac defects will prevent embolization of thrombus or air.244 Determining the most ideal time for VAD implantation remains a challenge. The majority of pediatric implants are performed for INTERMACS level 2. However, patients who are critically ill at the time of implant (INTERMACS 1 and 2) have significantly increased 30-day mortality when compared with those at INTERMACS levels 3 and 4. Determining risk/benefit ratio of VAD implantation at more stable INTERMACS levels is a difficult but important multidisciplinary decision. Standardizing intraoperative and postoperative management, management of right HF and pulmonary hypertension, and anticoagulation protocols have the potential to optimize outcomes. It is important and often challenging to avoid initiating mechanical support too late. Moderate end-organ dysfunction often improves with MCS, especially renal and hepatic dysfunction, but can be irreversible when aggressive treatment is not implemented in a timely manner.242,245

Basic Management of Ventricular Assist Device Patients Anticoagulation and Antiplatelets Patients assisted with VADs are not anticoagulated for the first 24 to 48 hours to decrease risk of bleeding. The traditional first line of anticoagulation has been heparin. UH is usually started when chest tube output is #2 mL/kg per hour, initial infusion dose is 10 U/kg per hour (range, 10–20 U/kg per hour) titrated to goal aPTT 60 to 80 seconds and/or anti-Xa targets of 0.35 to 0.50 U/mL. Underlying coagulopathy or thrombocytopenia should be corrected as clinically indicated. Bivalirudin, a direct thrombin inhibitor, has been used successfully in pediatric patients with VADs, with argatroban being the other alternative.169 Bivalirudin use has significantly decreased the number of pump changes and thromboembolic complications. Initial dose of bivalirudin is 0.15 mg/kg per hour with a target PTT of 60 to 90 seconds.229,246 As in the initial EXCOR protocol, these infusions can be transitioned at 1 to 2 weeks to warfarin in the older patients and to enoxaparin in the younger patients. Once the initial agent has achieved therapeutic levels in the context of good hemostasis, antiplatelet therapy is started. The Edmonton protocol163 involves a three-drug regimen that includes aspirin, dipyridamole, and either warfarin or enoxaparin and long-term agents depending on the patient’s age. This protocol was used as a guideline in the Berlin EXCOR Investigational Device Exemption (IDE) prospective trial.200 Despite the guideline, the prevalence of major bleeding was significant: 42% in patients with a BSA less than 0.7 m2 and 50% in patients with a BSA of 0.7 to 1.5 m2. Unfortunately, there was also a significant prevalence of thrombosis with a high incidence of stroke (29% in both cohorts). With the high thrombotic complications noted with the Edmonton protocol, a more aggressive antiplatelet regimen was introduced by the Stanford group. The usual agents

Cardiac Failure and Ventricular Assist Devices

265

include aspirin and dipyridamole, with the addition of a second antiplatelet agent, clopidogrel.247 These agents are often initiated in sequence. Initial doses of aspirin include 5 mg/kg per day divided into two doses, with maximal therapy at 30 mg/kg per day. Initial doses of dipyridamole include 2.5 mg/kg divided 3 times per day with a maximal therapy of 15 mg/kg divided 3 times per day. TEG and platelet function assays had been used as a guide to antiplatelet therapy but more recently have fallen out of favor given a high coefficient of variation of the assay as well as confounding technical issues.248 Implementation of each of these protocols is institution specific. Understanding optimal anticoagulation and antiplatelet therapy in children on VAD support remains a significant challenge. With continuous-flow devices such as the HeartMate II or III or HeartWare VAD, aspirin is usually the only antiplatelet therapy needed. Acquired von Willebrand syndrome is common in patients with VADs; the loss of large von Willebrand factor (vWF) multimers is caused by excessive cleavage of vWF under the constant high shear forces associated with VADs. These alterations in vWF structure and adhesive activity recover rapidly after VAD explantation.249 Bleeding due to acquired vWF syndrome is usually managed with cryoprecipitate or purified vWF concentrate.

Antibiotic Prophylaxis Patients who are scheduled for VAD implantation require an antiseptic bath with an agent such as chlorhexidine 1 day prior to surgery, preferably the day of VAD implant. Infection prophylaxis using broad-spectrum antibiotics and antifungal drugs is continued for 48 hours after the implantation. Antifungal therapy is especially considered if the patient is on ECMO support prior to VAD implant for more than 5 to 7 days.

Care of the Drive Line and Cannulas To prevent infection of the cannula sites, dressing changes must be done in a sterile and consistent fashion. The frequency of dressing changes is increased if there are signs of infection. Following the use of sterile precautions, an antiseptic swab or scrub, such as chlorhexidine, is used to clean the exit site. Primary dressing material that is silver impregnated and nonadherent is used, such as Telfa. In the case of the Berlin EXCOR VAD, abdominal wound pads are placed to secure the cannulas. For implantable intracorporeal devices such as the HVAD or HeartMate II, place the adhesive dressing (e.g., Covaderm) over the site, ensuring that the dressing is completely occluded on all four sides.

Ventricular Assist Device—Congenital Heart Disease and Single-Ventricle Physiology Outcomes for pediatric patients with dilated cardiomyopathy/ myocarditis on a VAD have always been significantly better than those for patients with CHD.250 Over the years, the outcome for biventricular CHD has continued to evolve but one of the biggest challenges has been the support of patients with single-ventricle physiology. Two general categories of patients with single-ventricle physiology may require MCS: those prior to palliative surgery but without complete separation of the systemic and pulmonary circulation and those who are post-Fontan in whom all systemic venous return flows passively into the pulmonary circulation. In

266

SECTION IV



Pediatric Critical Care: Cardiovascular

the first group, inflow is provided from the single ventricle with outflow to the ascending aorta, and pulmonary blood flow is maintained via a systemic-to-pulmonary artery shunt or a bidirectional Glenn shunt. In patients with a failing Fontan, cardiac failure may result from primary ventricular dysfunction, producing high Fontan pressures or from failing Fontan physiology (preserved ventricular function but moderate to severe elevation of pulmonary vascular resistance), yielding chronic elevation of pressures in the Fontan pathway and sequelae of HF. In patients with primary ventricular dysfunction and normal pulmonary vascular resistance, care for the failing physiology has been increasingly successful. Some centers have used the pulsatile Berlin EXCOR for support, and others have used continuousflow devices such as the HeartWare VAD or HeartMate III.251,252 A subanalysis of the EXCOR IDE database identified that 26 out of 281 patients supported with VAD had single-ventricle physiology; fewer single-ventricle patients were bridged to transplant or recovery when compared with those with biventricular physiology, 1 out of 9 supported after stage I palliation survived, and 3 out of 5 patients with total cavopulmonary anastomosis were successfully bridged to transplant.247–253 In the setting of failing Fontan physiology, cannulation choices are more challenging. If elevated pulmonary vascular resistance leads to systemic venous hypertension, then a second VAD may be required to support flow through the pulmonary circulation.114,254 As experience continues to grow, the outcome of failing Fontan circulation patients supported with VADs continues to improve.252 In contrast, the care of single-ventricle infants prior to and after stage I palliation continues to be challenging. Postcardiotomy salvage VAD (postcardiotomy ECLS to durable VAD) in congenital heart surgery has dismal outcomes.235 Conway et al. evaluated the Berlin EXCOR VAD in children weighing 10 kg or less. Patients with HF in the context of CHD had worse outcomes, and none of the patients below 5 kg with CHD who required ECLS support before VAD implantation survived.204 Some centers do not offer VADs to patients with single-ventricle CHD and opt to support these children with ECLS or a combination

of advanced inotropic support and mechanical ventilation. Nevertheless, there are several case reports describing the use of VAD support for single-ventricle HF. At our center, we have palliated patients with failing single-ventricle physiology in the context of hypoplastic left heart, with a combination of hybrid palliation for pulmonary blood flow and placing the single VAD with the outflow in the PA and inflow from the right atrium.202 We have used this strategy in close to 10 patients with approximately 60% survival to discharge. Gazit et al. reported a case series of seven patients with single-ventricle physiology after stage I palliation supported with MCS using a centrifugal extracorporeal pump. The pump was attached to the cardiac chambers using the cannulas for a durable VAD. The median support duration was 64 days. Complications included a neurologic event in one patient and reoperation from bleeding in two; 43% of the supported patients were successfully discharged home.255 As more centers continue to support an increasing number of infants with singleventricle physiology, their combined experience continues to grow, with the final frontier being an implantable continuousflow VAD in these patients.

Outcomes PediMACS, the pediatric component of INTERMACS, presented all pediatric data (patients ,19 years) from September 2012 to December 2017.256 Thirty hospitals implanted 508 devices in 423 patients aged younger than 19 years (Figs. 28.10and 28.11). Diagnoses included cardiomyopathy (62%), myocarditis (11%), and CHD (20%). Notably, 52 of these 86 patients with CHD had single-ventricle physiology. LVAD support predominated in 81% and BiVADs in 15%; 80% of the patients were alive on device or bridged to transplant at 6 months. The patients with an implantable continuous-flow (IC) pump were older, age at implant, 13.4 3.8 years with only 12% having CHD. They were significantly different from the paracorporeal continuousflow (PC) pump cohort (n 5 79; age, 3.9 5.2 years; 86% intubated at implant and 38% with CHD) and the paracorporeal 6

6

PediMACS prospective implants: September 19, 2012 - December 31, 2017 cumulative hospital, patient, and device enrollment counts 500

Cumulative frequency

Devices (n = 508) Patients (n = 423)

400

Hospitals (n = 30) 300 200 100 0 Jul 2012

Jan 2013

Jul

Jan 2014

Jul

Jan 2015

Jul

Jan 2016

Jul

Jan 2017

Jul

Jan 2018

Implant date

•  Fig. 28.10

​PediMACS enrollment. (From Morales D, Rossano J, VanderPluym C, et al. Third Annual Pediatric Interagency Registry for Mechanical Circulatory Support (Pedimacs) Report: Preimplant characteristics and outcomes. Ann Thorac Surg. 2019;107:993–1004.)  



Cardiac Failure and Ventricular Assist Devices



CHAPTER 28

Kaplan-Meier survival on a device by PediMACS overall (n = 423) Coverage: September 19, 2012 - December 31, 2017 100% 90%

PediMACS (n = 423, Deaths = 86)

80% % Survival

70% 60% 50% 40% 30% 20% 10% 0%

At Risk: 423 0

96

1

2

3

4

5

6

39 7

8

9

10

11

12

Months after device implant Shaded areas indicate 70% confidence limits P value (log-rank) = N/A Event: Death (censored at transplant or recovery)

A Kaplan-Meier survival on a device by age group (n = 423) Coverage: September 19, 2012 - December 31, 2017 100%

11–19 years (n = 184, Deaths = 26)

90% 80%

6–11 years (n = 71, Deaths = 18

% Survival

70% 60%

1–6 years (n = 79, Deaths = 15)

50%

< 1 year (n = 89, Deaths = 27)

40% 30%

At Risk: 184 71 79 89

20% 10% 0%

51 23 13 9

1

0

2

3

4

5

29 8 2

6

7

8

9

10

11

12

Months after device implant Shaded areas indicate 70% confidence limits P value (log-rank) = < .0001 Event: Death (censored at transplant or recovery)

B Kaplan-Meier survival on a device by device class (n = 397) Coverage: September 19, 2012 - December 31, 2017 100%

Implantable continuous (n = 197, Deaths = 23)

90%

Paracorporeal pulsatile (n = 121, Deaths = 29)

80% % Survival

70% 60%

Paracorporeal continuous (n = 79, Deaths = 23)

50% 40% 30% 20% 10% 0%

At Risk: 198 79 120 0

1

68 3 23 2

3

4

5

6

36 1 1 7

8

9

10

11

12

Months after device implant Shaded areas indicate 70% confidence limits P value (log-rank) = < .0001 Event: Death (censored at transplant or recovery)

C • Fig. 28.11



​(A) Kaplan-Meier survival curve for overall PediMACS group of 423 patients receiving implants September 2012 to December 2017. (B) Age at implant. (C) Device type.

267

268

SECTION IV



Pediatric Critical Care: Cardiovascular

Kaplan-Meier survival on a device by patient profile (n = 421) Coverage: September 19, 2012 - December 31, 2017 100%

4–7. Resting symptoms or less sick (n = 7, Deaths = 0) 2. Progressive decline (n = 232, Deaths = 33)

90% 80%

3. Stable but inotrope dependent (n = 44, Deaths = 7)

% Survival

70% 60% 50% 40% 30% 20% 10% 0%

1. Critical cardiogenic shock (n = 138, Deaths = 45) At Risk: 7 44 323 138 0

1

5 15 55 21 2

3

4

5

6

2 9 24 4 7

8

9

10

11

12

Months after device implant Shaded areas indicate 70% confidence limits P value (log-rank) = PA

PA PA

PV PA

PA > PA > PV

PA PV

PA PA

• Fig. 32.8

PV > PA

​Pulmonary capillaries are, in essence, surrounded by gas-filled alveoli. The influence of alveolar pressure (PA ) on regional lung blood flow is similar to the influence of surrounding pressure on flow through a Starling resistor. At ambient values of PA, pulmonary vein pressure (PV) opposes inflow. As PA rises, it begins to oppose inflow only after PA . PV. It modulates inflow until PA reaches hydrostatic inflow pressure (PA), at which point flow ceases.  

capillaries is a local phenomenon. It can divert pulmonary blood flow away from normal lung segments toward consolidated or atelectatic lung segments whose airways do not effectively transmit airway pressure to the alveolus.26 The application of high PEEP in the presence of lobar pneumonia may increase blood flow through unventilated lung and worsen hypoxemia by this mechanism. From a beneficial perspective, PEEP may relieve atelectasis and improve ventilation, thereby relieving alveolar hypoxic vasoconstriction. Whether PEEP benefits or impairs pulmonary blood flow depends in part on the balance of its effect on atelectasis (lung recruitment) and its effect on alveolar capillaries (alveolar overdistension).

Regulation of Pulmonary Vascular Resistance The resistance to flow through a vessel is described by the following: 

R  81  r 4

Eq. 32.5



where h is viscosity, l is length, and r is radius. It follows that PVR can be effectively controlled by active changes in vessel radius. Mechanical ventilation may alter carbon dioxide clearance/ blood pH and PAO2, both of which influence vessel tone and radius.27,28 Hypoxic pulmonary vasoconstriction is a powerful mechanism for sustaining systemic oxygenation in the face of lung disease.29–32 Relief of atelectasis and restoration of segmental ventilation not only increases the fraction of the lung that is ventilated but also restores blood flow to those segments by several mechanisms. Segmental alveolar hypoxia is relieved. Segmental volume is restored, which returns segmental vascular resistance to its volume-dependent nadir. Gas exchange is also improved, thereby reducing global PVR.

Direct Effects of Airway Pressure on Pulmonary Vascular Tone Pulmonary vessels are stretched by lung inflation. The lung of infant lambs responds to abrupt changes in airway pressure with changes in vascular tone. Abrupt distension of one lung of the intact infant lamb increases the PVR of that lung alone.33 The resistance change is sensitive to the waveform of the lung distension34 and persists for some time after relief of distending pressure and return of lung volume to baseline.35 This effect is calcium channel dependent36 and resembles a myogenic reflex whereby direct vessel stretch causes constriction.

Respiration and Left Ventricular Preload Positive airway pressure can reduce LV filling via several mechanisms as described earlier: limiting systemic venous return and RV filling and/or increasing RV afterload and decreasing RV ejection. Positive airway pressure may also decrease LV filling by adversely impacting LV compliance as a result of ventricular interdependence.

Ventricular Interdependence The RV and LV share a common muscle mass and pericardial space. It follows that compliance of either ventricle will be influenced by volume and pressure of the other chamber (Fig. 32.9). Increased venous return to the right heart, as occurs during spontaneous inspiration and especially during execution of a Müller maneuver, shifts the interventricular septum to the left, reducing compliance of the LV.37,38 Similarly, excessive compression of the



CHAPTER 32 Cardiopulmonary Interactions

• Fig. 32.9



​Increasing the diastolic volume of the right ventricle (RV) reduces compliance of the left ventricle (LV). FRC, functional residual capacity.

pulmonary circulation by positive airway pressure may impede RV ejection, causing the RV to dilate and encroach on the left. Reduced LV compliance tends to diminish stroke volume by diminishing LV muscle stretch and ejection force (by the FrankStarling mechanism).

Respiration and Left Ventricular Afterload The LV ejects blood from within the thorax to the extrathoracic arterial system. Most of the resistance to this forward flow resides in the arterioles. From a practical point of view, the pressure in the extrathoracic arteries can be described by the following equations: Eq. 32.6



where Q is CO, Partery is Pi before the arteriole, and Pms is Po after the arteriole as defined for the venous return curve. When the LV contracts, it creates internal pressure against the closed aortic valve by generating tension in the myocardium that encircles the ventricular chamber. This “wall tension” causes ventricular pressure to rise until it reaches aortic diastolic pressure, opening the aortic valve and ejecting the stroke volume. Creation of wall tension and subsequent shortening of myocardial fibers perform the external mechanical work of the heart. When the heart squeezes, it creates a pressure difference between the ventricle and juxtacardiac space. In effect, the myocardium creates a Ptm to produce a ventricular pressure sufficient to open the aortic valve. Aortic diastolic pressure and external (juxtacardiac) pressure determine the myocardial wall tension needed



Eq. 32.7



or Partery  Q  R arteriole  Pms





The expected effects of a rise in airway pressure are as follows: 1. Decreased filling of the RV acts to decrease RV stroke volume. 2. Pulmonary vascular compression increases RV afterload and acts to decrease RV ejection and stroke volume. 3. These effects may either increase or decrease RV size, depending on which effect predominates; if RV volume increases, ventricular interdependence adversely impacts LV compliance. 4. Both diastolic displacement of the interventricular septum toward the LV (with resultant crowding of the LV) and crowding of the juxtacardiac space by the expanding lungs reduce LV compliance. Both factors act to decrease LV filling and stroke volume. 5. The fall in LV afterload that results from increased juxtacardiac pressure acts to increase LV stroke volume. These effects of positive airway pressure may have conflicting effects on stroke volume; thus, their aggregate effect on CO is not entirely predictable. In general, positive airway pressure has its most pronounced effect on the right heart; therefore, positive

)

 Pms  Q  R arteriole

Preload Dependence Versus Afterload Dependence



artery

Studies of the effects of positive airway pressure on LV contractility have yielded conflicting results. Certainly, ventilator-induced changes in preload and afterload have secondary effects on stroke volume, but independent effects of positive airway pressure on LV contractility have not been consistently demonstrated. Negative inotropic effects modulated by reflexes, mediators, or alterations in coronary blood flow have been described,41–46 but most animal and human studies fail to show that positive airway pressure has any primary effect on myocardial contractility.47–50 It has been suggested that high levels of PEEP may compress coronary vessels, cause myocardial ischemia, and thereby impair ventricular function.51–53 LV myocardium is perfused predominantly in diastole. To the extent that juxtacardiac pressure exceeds diastolic pressure in the coronary sinus, such an effect is plausible. This assertion appears more compelling for patients in shock and for those with intrinsic coronary blood flow limitations than for otherwise normal individuals.



(P





Cardiac Contractility



Spontaneous inspiration crowds the LV



LV RV



FRC

to open the aortic valve. Both pressures represent afterloads to LV ejection.39,40 A vasoconstrictor of arterial resistance vessels increases the pressure required to open the aortic valve and LV afterload. This increases the wall tension that the heart must generate to eject blood. The Müller maneuver, forced inspiration against a closed glottis, does the same thing. It reduces juxtacardiac pressure, thereby raising the Ptm required to open the aortic valve. Thus, the Müller maneuver also increases LV afterload. A vasodilator of arterial resistance vessels lowers aortic pressure, reducing the pressure required to open the aortic valve and LV afterload. PPV or application of continuous positive airway pressure (CPAP) increases juxtacardiac pressure to such an extent that the wall tension required to open the aortic valve and LV afterload are diminished. The net effect of PPV is often augmentation of LV stroke volume and CO. Arterial pressure is commonly observed to rise during positive-pressure inspiration, whereas it falls during spontaneous inspiration. These are largely effects of afterload on stroke volume.



LV

325

326

SECTION IV



Pediatric Critical Care: Cardiovascular

airway pressure reduces CO in most patients. This effect is greatest in patients who are hypovolemic because the driving pressure for systemic venous return (Pms 2 Pra) is more sensitive to change in Pra when Pms is low. In addition, pulmonary venous pressure is low and PPV increases the proportion of lung units where Ps (alveolar) . Pi, increasing RV afterload and decreasing RV ejection. In adults, the Pra threshold (at zero PEEP) below which increasing airway pressure reduces CO is approximately 12 mm Hg.54 Another definition of preload dependence is responsiveness to vascular volume infusion. By this definition, when the dominant effect of positive airway pressure is to impede right heart filling, vascular volume infusion increases stroke volume and CO. Four measurable parameters predict responsiveness to vascular volume infusion: (1) variation of pulse pressure over the respiratory cycle (maximum 2 minimum), (2) arterial systolic pressure, (3) Pra, and (4) pulmonary artery wedge pressure.55 Of these parameters, the inspiratory increase in arterial pulse pressure is the most sensitive and specific predictor of “preload dependence” (by receptor operating characteristic curve). Greater than 15% inspiratory rise in pulse pressure appears to identify adults with preload dependence during PPV.56 One might expect the converse also to apply. Reduced magnitude of the effects of PPV on RV filling or augmented effects on LV ejection may make the patient “afterload dependent.” Patients who have high blood volume, such as those in congestive cardiac failure or those with chronic anemia, should have high Pms and decreased sensitivity to changes in Pra.57 Moreover, the patient with poor LV contractility may greatly benefit from the afterload reduction of PPV.58 If positive airway pressure enhances the ejection of blood into the systemic circulation, this may directly reduce left atrial pressure. From these considerations, improved CO reduces Pra, enhancing systemic venous return (see Fig. 32.1B). When favorable effects on LV ejection act to reduce right and left atrial pressures, CO improves. Such a patient might be thought of as “afterload dependent.”

Fluid Responsiveness During Positive Pressure Ventilation In the critical care unit, whether volume infusion will augment CO or merely contribute to vascular volume overload is often a vital issue. In adult patients receiving PPV, about half of all hemodynamically unstable critical care patients are not volume responsive.59 Traditional guides to volume resuscitation have focused on measurement of cardiac filling pressures and responses to fluid challenges. In critically ill patients and in normal subjects, the stroke volume response to vascular volume infusion is poorly predicted by measurement of either right atrial or pulmonary artery occlusion pressure.60 Much recent interest has focused on minimally invasive estimation of the likelihood of responding to fluid infusion. The cardiopulmonary interactions described previously can be used to predict response to fluid challenge. The change in stroke volume over the course of the positive pressure respiratory cycle strongly predicts fluid responsiveness; the greater the percentage change in stroke volume, the greater the response to volume infusion.61 Similar relations have been shown between fluid responsiveness and respiratory variation in aortic blood flow velocity62 and arterial pulse pressure.54 Inspiratory collapsibility of the inferior63 and superior64 vena cavae has also been shown to predict volume responsiveness. Changes in CO induced by either fluid resuscitation or by application of PEEP appear to be

predicted by measurement of pulse pressure variability.55 In general, in these patients, the greater the pulse pressure variability, the more preload dependent the patient is to PEEP and the more responsive to fluid resuscitation.

Pulsus Paradoxus in Respiratory Distress Arterial pressure normally falls during spontaneous inspiration, which is best explained as a result of increasing LV afterload at a time in the respiratory cycle when ventricular interdependence restricts LV filling. It is well known that pericardial tamponade causes accentuation of the normal inspiratory decrease in systemic blood pressure, a phenomenon known as pulsus paradoxus. This phenomenon has been attributed to accentuation of normal ventricular interdependence in the face of restricted biventricular diastolic volume of the heart. During loaded spontaneous inspiration, as in the Müller maneuver or in the presence of inspiratory airway obstruction (e.g., croup), the inspiratory fall in juxtacardiac pressure is exaggerated and LV afterload is accentuated. Again, the result is pulsus paradoxus, an accentuated drop in blood pressure during inspiration. The increase in blood pressure that occurs during positive pressure inspiration has been termed reverse pulsus paradoxus. This finding has been attributed to LV afterload reduction by the rise in juxtacardiac pressure that occurs during positive pressure inspiration. Reverse pulsus paradoxus is a normal finding during PPV but may be accentuated in afterload-dependent states (e.g., LV systolic dysfunction), as discussed earlier in the Preload Dependence Versus Afterload Dependence section).

Positive-Pressure Ventilation and Right Ventricular Output in Acute Respiratory Distress Syndrome In acute respiratory distress syndrome (ARDS), PPV may induce significant heart-lung interactions that adversely impact RV loading conditions and output, impairing gas exchange, CO, and ultimately systemic DO2, potentially offsetting gains in oxygenation resulting from increases in airway pressure. As reviewed earlier, PPV may decrease systemic venous return, RV filling, and output. Despite the use of a lung-protective strategy, PPV may also increase PVR, impairing RV systolic ejection. While the increase in air-blood barrier permeability is evenly distributed and extravascular lung water (EVLW) is diffusely increased, a vertical, gravitational gradient exists for lung densities and EVLW formation.65 The increase in lung weight exaggerates the normal compressive gravitational forces present in lung parenchyma, leading to the formation of nonaerated tissue in gravity-dependent regions of the lung.66 Lung tissue in nongravity-dependent regions of the lung is well aerated with nearnormal mechanical characteristics and thus receives a disproportionate amount of the airway pressure, resulting in alveolar overdistension, while blood flow in this region is limited due to the effects of gravity on pulmonary perfusion. PVR increases to the extent that zone I conditions are created (PA . PA; see Fig. 32.8) and regions under zone II conditions are increased (PA . PA . PV; see Fig. 32.8). A limitation of preload and increase in afterload may combine to decrease RV output. With an increase in afterload, a compensatory increase in preload is needed to maintain stroke volume, which may be limited for the reasons discussed earlier. VieillardBaron and colleagues described this finding in their study of adult patients with ARDS using echocardiography with Doppler to evaluate beat-to-beat inflow and outflow and ventricular



CHAPTER 32 Cardiopulmonary Interactions

dimensions throughout the respiratory cycle.66 They demonstrated inspiratory reductions in RV fractional area contraction associated with a significant increase in RV end systolic dimensions, while RV end diastolic area remained unchanged. They concluded that this likely reflects a relative decrease in RV preload because an increase in afterload and end systolic volume should produce a corresponding (compensatory) increase in preload. ARDS is frequently associated with RV systolic impairment, the most severe form of which is cor pulmonale, which occurs in upward of 20% of adult patients and has been shown to be independently associated with a higher risk of death in these patients.67,68 Indicators of a limited CO state—such as an increased oxygen extraction ratio, elevated lactate level, or thermodilutionmeasured low CO—merit an echocardiogram to delineate the mechanism(s) responsible.69 Echocardiography provides several hemodynamic parameters—including RV systolic pressure, dimensions, and function—establishing whether right heart impairment is present and whether it is due to inadequate preload, elevated afterload, or a combination of the two.

Effects of Cardiovascular Function on Respiration Shock States and Respiratory Function Shock of any cause diminishes perfusion of respiratory muscles and can lead to respiratory failure and respiratory arrest. It also causes metabolic acidosis, which constricts pulmonary vessels and opposes lung blood flow.70,71 Acidosis is a potent stimulus of respiratory effort and contributes to tachypnea and respiratory distress, which, in turn, worsen the demand on the heart. Shock is injurious to both heart and lungs; one final common pathway to recovery is the initiation of mechanical ventilation, which benefits both organ systems. Hypovolemic shock can create extreme preload dependency.72 In hypovolemic shock, diastolic blood pressure may fall during positive pressure inspiration, impairing coronary perfusion. This may cause myocardial ischemia and worsen cardiac function. Cardiogenic shock elevates percent tissue oxygen extraction from the blood by reducing DO2. The resultant decline in venous oxygen tension has a paradoxical effect. It increases the efficiency of pulmonary blood flow by allowing greater oxygen uptake per unit of pulmonary blood flow. That is, the more desaturated the blood that enters the pulmonary circulation, the more oxygen can be uploaded. This process requires that alveolar pO2 not limit the amount of oxygen available for uptake and is one reason to administer oxygen to patients experiencing cardiorespiratory failure.

Elevated Work of Breathing and the Circulation During quiet respiration, the heart has no difficulty satisfying the metabolic demand of the respiratory muscles. However, with a significant increase in respiratory muscle oxygen demand and a limitation of CO, respiratory muscle DO2 may be inadequate. Unlike cardiac muscle, respiratory muscles can accumulate a limited oxygen debt, and persistent hypoperfusion may interfere with their ability to perform the requisite work of breathing. In addition, in a low CO state, a competition among viscera for blood flow is created. The brain, myocardium, and respiratory pump lack adrenergic receptors and, under intense neurohormonal activation, do not experience an increase in resistance to flow; thus, they compete for a limited CO. Roussos and

327

colleagues73–75 demonstrated in animal models of cardiogenic and septic shock that mechanical ventilation and the unloading of the respiratory pump lead to a significant redistribution of blood flow from muscles of respiration to other vital organs, including the brain. Because it unloads the respiratory pump and decreases LV afterload, mechanical ventilation should be considered an essential tool in the armamentarium for treating heart failure and shock.

Congestive Heart Failure/Critical Heart Failure and Shock All that has been said about shock is equally true of congestive heart failure (CHF). In fact, there is a continuum from CHF to cardiogenic shock. CHF elicits physiologic responses that attempt to restore and maintain an adequate CO. As these homeostatic responses are exhausted, CO becomes inadequate, and the patient develops obvious manifestations of cardiogenic shock and respiratory failure. In addition to the impact of shock on respiratory function and reserve, CHF generally causes fluid retention and pulmonary edema. Treatment of cardiogenic shock by vascular volume expansion may, by augmenting cardiac filling pressures, improve CO at the expense of pulmonary edema, which leads to elevated airway resistance, atelectasis, intrapulmonary shunt, and impaired oxygenation. Impaired respiratory mechanics lead to exaggerated negative-pressure breathing and increased LV afterload while increasing circulatory demands by increasing respiratory muscle oxygen demand. Further, as the work of breathing increases, neurohormonal pathways are stimulated. These pathways increase systemic vascular resistance and blood pressure and, in doing so, contribute significantly to increases in LV afterload, as described earlier. PPV with PEEP improves lung function and gas exchange and eliminates exaggerated negative swings in ITP, eliminating respiratory muscle oxygen demand while releasing sympathetic nervous system activation. LV afterload is reduced, increasing stroke volume and CO. PPV decreases respiratory and cardiac muscle oxygen demand while improving CO and DO2. The net effect is a significant improvement in the myocardial and global oxygen supply-demand relationship in patients with cardiogenic shock. Pulmonary venous hypertension renders the pulmonary circulation less sensitive to positive airway pressure, as a disproportionate number of lung units have Pv . Ps (alveolar pressure; zone III conditions). The patient is afterload-dependent and should respond favorably to the afterloadreducing effects of PPV. When treating the patient with CHF during PPV, there is a risk of worsening pulmonary edema during fluid resuscitation. It is wise to assess pulse pressure, as well as systolic and diastolic pressures, to predict volume responsiveness before administering volume.76 These parameters may also prove useful in assessing the impact of changes in ITP on CO.55

Cardiomyopathies and Congenital Heart Disease Hypertrophic cardiomyopathy is characterized by intact LV systolic function but has varying degrees of diastolic dysfunction that may compromise output due to inadequate ventricular filling.77 Thus, the impact of increases in ITP on systemic venous return may not be tolerated. If pulmonary venous pressure is elevated secondary to impaired LV compliance, the impact of PPV and lung volumes on PVR will be mitigated as the proportion of lung units with Pv . Ps increases. Hypertrophic cardiomyopathy may also obstruct LV outflow, either at rest or with exertion. PPV may

328

SECTION IV



Pediatric Critical Care: Cardiovascular

not be tolerated, as a decrease in preload and afterload leads to a decrease in ventricular operating volumes, exacerbating the obstruction.78 Restrictive cardiomyopathies invariably have severe abnormalities of ventricular diastolic dysfunction. One would also expect these patients not to tolerate PPV owing to its impact on systemic venous return. The impact of ITP on ventricular diastolic disease is exemplified in some patients following repair of tetralogy of Fallot, where varying degrees of RV diastolic dysfunction are common. In a subgroup of these patients, the degree of impairment is severe and has been termed restrictive physiology. In these patients, atrial systole causes RV diastolic pressure to rise above pulmonary arterial diastolic pressure, generating pulmonary arterial flow during ventricular diastole. LV systolic function is, in most cases, normal; thus, the primary impact of changes in ITP is on the right heart. Further, pulmonary venous pressure should not be elevated. Therefore, PPV may be expected to increase the proportion of lung units with Ps (alveolar) . Pi, increasing RV afterload, which may not be tolerated. Shekerdemian and colleagues79 demonstrated a significant increase in CO when patients were converted from PPV to negative pressure ventilation using a cuirass.

Glenn and Fontan Procedures Hearts that cannot support two separate circulations after repair (univentricular hearts and hearts having one hypoplastic ventricle) are often palliated and then repaired by routing systemic venous return directly to the lung without an intervening subpulmonic pumping chamber (Glenn and Fontan procedures). In the Glenn circulation, pulmonary blood flow is derived from venous drainage from the upper extremities and brain by way of the superior vena cava and is sensitive to changes in ITP and its impact on the superior vena cava–pulmonary artery confluence, as well as lung volume and its impact on PVR. One would expect pulmonary blood flow and oxygenation to improve during spontaneous respiration. In the Glenn circulation, the inferior vena cava drains directly to the single ventricle. While systemic venous return is adversely affected by increases in ITP, it is not impacted by the lack of a subpulmonic pumping chamber; thus, CO is generally well maintained following the Glenn procedure. In the Fontan circulation, venous return from the lower body is diverted directly to the pulmonary arteries; thus, all systemic venous return must traverse the pulmonary circulation without a subpulmonic pumping chamber to maintain adequate

ventricular filling. The Pms drives systemic venous return from the venous reservoirs to the central venous structures as in a normal circulation but is also responsible for driving systemic venous return across the pulmonary circulation to the single ventricle.80 Decreases in the common atrial pressure during the cardiac cycle contribute to the pressure gradient driving pulmonary venous return.81 Because the pulmonary circulation and single ventricle reside entirely within the chest, changes in ITP do not contribute to driving pulmonary blood flow other than by altering the pressure gradient for systemic venous return to the vena cava–pulmonary artery confluence, as well as by altering the effective compliance of the ventricle as in the normal circulation. Shekerdemian and colleagues82 demonstrated the importance of ITP in the Fontan circulation. Immediately following the Fontan procedure, negative-pressure ventilation using a cuirass significantly improved CO compared with PPV. Williams and colleagues83 demonstrated the sensitivity of the Fontan circulation to PPV by exposing the inverse relationship between pulmonary vascular resistance and cardiac output with progressive increases in PEEP.

Key References Aubier M, Trippenbach T, Roussos C. Respiratory muscle fatigue during cardiogenic shock. Appl Physiol. 1981;51:499-508. Funk DJ, Jacobsohn E, Kumar A. The role of venous return in critical illness and shock—part I: physiology. Crit Care Med. 2013;41:255262. Gattinoni L, Chiumello D, Valenza C, F. Bench-to-bedside review: chest wall elastance in acute lung injury/acute respiratory distress syndrome patients. Critical Care. 2004;8:350-355. Gattinoni L, Marini JJ, Pesenti A, et al. The “baby lung” became an adult. Intensive Care Med. 2016;42:663-673. Michard F, Chemla D, Richard C, et al. Clinical use of respiratory changes in arterial pulse pressure to monitor the hemodynamic effects of PEEP in patients with acute lung injury. Am J Respir Crit Care Med. 1999;159:935-939. Vieillard-Baron A, Loubieres Y, Schmitt J-M, et al. Cyclic changes in right ventricular output impedance during mechanical ventilation. J Appl Physiol. 1999;87:1644-1650. Vieillard-Baron A, Matthay M, Teboul JL, et al. Experts’ opinion on management of hemodynamics in ARDS patients: focus on the effects of mechanical ventilation. Intensive Care Med. 2016;42:739-749.

The full reference list for this chapter is available at ExpertConsult.com.

e1





















































25. West JB, Dollery CT. Distribution of blood flow and the pressure flow relations of the whole lung. J Appl Physiol. 1965;20: 175-183. 26. Mink SN, Light RB, Cooligan T, et al. Effect of PEEP on gas exchange and pulmonary perfusion in canine lobar pneumonia. J Appl Physiol. 1981;50:517-523. 27. Schreiber MD, Heymann MA, Soifer SJ. Increased arterial pH, not decreased PaCO2 attenuates hypoxia-induced pulmonary vasoconstriction in newborn lambs. Pediatr Res. 1986;20:113-117. 28. Rudolph AM, Yuan S. Response of the pulmonary vasculature to hypoxia and H1 ion concentration changes. J Clin Invest. 1966; 45:399-411. 29. Fishman AP. Vasomotor regulation of the pulmonary circulation. Ann Rev Physiol. 1980;42:211-220. 30. Doyle JST, Wilson JS, Warren JV. The pulmonary vascular responses to short-term hypoxia in human subjects. Circulation. 1952;5: 263-270. 31. Wagner WW. Pulmonary circulation: control through hypoxic vasoconstriction. Semin Respir Med. 1985;7:124-135. 32. Peake MD, Harabin AL, Brennan NJ, et al. Steady-state vascular responses to graded hypoxia in isolated lungs of five species. J Appl Physiol. 1981;51:1214-1219. 33. Fuhrman BP, Everitt J, Lock JE. Cardiopulmonary effects of unilateral airway pressure changes in intact infant lambs. J Appl Physiol. 1984;56:1439-1448. 34. Fuhrman BP, Smith-Wright DL, Kulik TJ, et al. Effects of static and fluctuating airway pressure on intact pulmonary circulation. J Appl Physiol. 1986;60:114-122. 35. Fuhrman BP, Smith-Wright DL, Venkataraman S, Howland DF. Pulmonary vascular resistance after cessation of positive end-expiratory pressure. J Appl Physiol. 1989;66:660-668. 36. Venkataraman ST, Fuhrman BP, Howland DF. PEEP-induced calcium channel-mediated rise in PVR in neonatal lambs. Crit Care Med. 1993;21:1066-1076. 37. Taylor RR, Covell JW, Sonnenblick EH, et al. Dependence of ventricular distensibility on filling of the opposite ventricle. Am J Physiol. 1967;213:711-718. 38. Mitchell JR, Whitelaw WA, Sas R. RV filling modulates LV function by direct ventricular interaction during mechanical ventilation. Am J Physiol Heart Circ Physiol. 2005;289:H549-H557. 39. Buda AJ, Pinsky MR, Ingels Jr NB, et al. Effect of intrathoracic pressure on left ventricular performance. N Engl J Med. 1979;301: 453-459. 40. Pinsky MR, Summer WR, Wise RA, et al. Augmentation of cardiac function by elevation of intrathoracic pressure. J Appl Physiol. 1983; 54:950-955. 41. Patten MT, Liebman PR, Hechtman HB. Humorally mediated decreases in cardiac output associated with positive end expiratory pressure. Microvasc Res. 1997;13:137-139. 42. Patten MT, Liebman PR, Manny J, et al. Humorally mediated alterations in cardiac performance as a consequence of positive endexpiratory pressure. Surgery. 1978;84:201-205. 43. Grindlinger GA, Manny J, Justice R, et al. Presence of negative inotropic agents in canine plasma during positive end-expiratory pressure. Circ Res. 1979;45:460-467. 44. Dunham BM, Grindlinger GA, Utsunomiya T, et al. Role of prostaglandins in positive end-expiratory pressure-induced negative inotropism. Am J Physiol. 1981;241:783-788. 45. Manny J, Grindlinger G, Mathe AA, et al. Positive end-expiratory pressure, lung stretch and decreased myocardial contractility. Surgery. 1978;84:127-133. 46. Glick G, Wechsler AS, Epstein SE. Reflex cardiovascular depression produced by stimulation of pulmonary stretch receptors in the dog. J Clin Invest. 1969;48:467-473. 47. Calvin JE, Driedger AA, Sibbald WJ. Positive end-expiratory pressure (PEEP) does not depress left ventricular function in patients with pulmonary edema. Am Rev Respir Dis. 1981;124:121-128.























































1. Cournand A, Motley HL, Werko L, et al. Physiologic studies of the effects of intermittent positive pressure breathing on cardiac output in man. Am J Physiol. 1948;152:162-174. 2. Funk DJ, Jacobsohn E, Kumar A. The role of venous return in critical illness and shock—part I: physiology. Crit Care Med. 2013;41:255-262. 3. Guyton AC. Determination of cardiac output by equating venous return curves with cardiac output curves. Physiol Rev. 1955;35: 123-129. 4. Guyton AC, Lindsey AW, Kaufmann BN, Richardson T. Venous return at various right atrial pressure and the normal venous return curve. Am J Physiol. 1957;189:609-615. 5. Fessler HE, Brower RG, Wise RA, Permutt S. Effects of positive end-expiratory pressure on the canine venous return curve. Am Rev Respir Dis. 1992;146:4-10. 6. Cassidy SS, Robertson Jr CH, Pierce AK, et al. Cardiovascular effects of positive end-expiratory pressure in dogs. J Appl Physiol. 1978; 44:743-750. 7. Brookhart JM, Boyd TE. Local differences in intrathoracic pressure and their relation to cardiac filling pressure in the dog. Am J Physiol. 1947;148:434-444. 8. Marini JJ, O’Quin R, Culver BH, et al. Estimation of transmural cardiac pressure during ventilation with PEEP. J Appl Physiol. 1982;53:384-391. 9. Brecher GA, Hubay CA. Pulmonary blood flow and venous return during spontaneous respiration. Circ Res. 1955;3:210-214. 10. Pinsky MR. Instantaneous venous return curves in an intact canine preparation. J Appl Physiol. 1984;56:765-771. 11. Pinsky MR. Determinants of pulmonary arterial flow variation during respiration. J Appl Physiol. 1984;56:1237-1245. 12. Morgan BC, Abel FL, Mullins GL, et al. Flow patterns in cavae, pulmonary artery, pulmonary vein and aorta in intact dogs. Am J Physiol. 1969;210:297-305. 13. Fewell JE, Abendschein DR, Carlson J, et al. Mechanism of decreased right and left ventricular end-diastolic volumes during continuous positive-pressure ventilation in dogs. Circ Res. 1980;47: 467-472. 14. Takata M, Wise RA, Robotham JL. Effects of abdominal pressure on venous return: abdominal vascular zone conditions. J Appl Physiol. 1990;69:1961-1972. 15. Takata M, Robotham JL. Effects of inspiratory diaphragmatic descent on inferior vena caval venous return. J Appl Physiol. 1992; 72:597-607. 16. Teboul JL, Pinsky MR, Mercat A, et al. Estimating cardiac filling pressure in mechanically ventilated patients with hyperinflation. Crit Care Med. 2000;28:3631-3636. 17. Pinsky MR. Recent advances in the clinical application of heart-lung interactions. Curr Opin Crit Care. 2002;8:26-31. 18. Gattinoni L, Chiumello D, Valenza CF. Bench-to-bedside review: chest wall elastance in acute lung injury/acute respiratory distress syndrome patients. Crit Care. 2004;8:350-355. 19. Burton AC, Patel DJ. Effect on pulmonary vascular resistance of inflation of the rabbit lungs. J Appl Physiol. 1958;12:239-246. 20. Barer GR, Howard P, McCurrie JR, et al. Changes in the pulmonary circulation after bronchial occlusion in anesthetized dogs and cats. Circ Res. 1969;25:747-764. 21. Benumof JL. Mechanism of decreased blood flow to atelectatic lung. J Appl Physiol. 1979;46:1047-1048. 22. Whittenberger JL, McGregor M, Berglund E, et al. Influence of state of inflation of the lung on pulmonary vascular resistance. J Appl Physiol. 1960;15:878-882. 23. Lopez-Muniz R, Stephens NC, Bromberger-Barnea B, et al. Critical closure of pulmonary vessels analyzed in terms of Starling resistor model. J Appl Physiol. 1968;24:625-635. 24. West JB, Dollery CT, Naimark A. Distribution of blood flow in isolated lung: relation to vascular and alveolar pressures. J Appl Physiol. 1964;19:713-724.



References

e2











































66. Vieillard-Baron A, Loubieres Y, Schmitt J-M, et al. Cyclic changes in right ventricular output impedance during mechanical ventilation. J Appl Physiol. 1999;87:1644-1650. 67. Boissier F, Katsahian S, Razazi K, et al. Prevalence and prognosis of cor pulmonale during protective ventilation for acute respiratory distress syndrome. Intensive Care Med. 2013;39:1725-1733. 68. Monchi M, Bellenfant F, Cariou A, et al. Early predictive factors of survival in the acute respiratory distress syndrome. A multivariate analysis. Am J Resp Crit Care Med. 1998;158:1076-1081. 69. Vieillard-Baron A, Matthay M, Teboul JL, et al. Experts’ opinion on management of hemodynamics in ARDS patients: focus on the effects of mechanical ventilation. Intensive Care Med. 2016;42: 739-749. 70. Enson Y, Giuntini C, Lewis ML, et al. The influence of hydrogen ion concentration and hypoxia on the pulmonary circulation. J Clin Invest. 1964;43:1146-1162. 71. Rudolph AM, Yuan S. Response of the pulmonary vasculature to hypoxia and H1 ion concentration changes. J Clin Invest. 1966;45:339-411. 72. Pepe PE, Lurie KG, Wigginton JG, et al. Detrimental hemodynamic effects of assisted ventilation in hemorrhagic states. Crit Care Med. 2004;32(suppl):S414-S420. 73. Aubier M, Trippenbach T, Roussos C. Respiratory muscle fatigue during cardiogenic shock. J Appl Physiol Respir Environ Exerc Physiol. 1981;51:499-508. 74. Hussain SNA, Roussos C. Distribution of respiratory muscle and organ blood flow during endotoxic shock in dogs. J Appl Physiol. 1985;59:1802-1808. 75. Viires N, Aubier SM, Rassidakis A, et al. Regional blood flow distribution in dog during induced hypotension and low cardiac output. J Clin Invest. 1983;72:935-947. 76. Pinsky MR. Using ventilation-induced aortic pressure and flow variation to diagnose preload responsiveness. Intensive Care Med. 2004;30:1008-1010. 77. Fifer MA, Vlahakes GJ. Management of symptoms in hypertrophic cardiomyopathy. Circulation. 2008;117:429-439. 78. Braunwald E, Oldham Jr H, Ross J, et al. The circulatory response of patients with idiopathic hypertrophic subaortic stenosis to nitroglycerin and to the Valsalva maneuver. Circulation. 1964;29: 422-431. 79. Shekerdemian LS, Bush A, Shore DF, et al. Cardiorespiratory responses to negative pressure ventilation after tetralogy of Fallot repair: a hemodynamic tool for patients with a low-output state. J Am Coll Cardiol. 1999;33:549-555. 80. Mace L, Dervanian P, Bourriez A, et al. Changes in venous return parameters associated with univentricular Fontan circulations. Am J Physiol Heart Circ Physiol. 2000;279:H2335-H2343. 81. Fogel MA, Weinberg PM, Hoydu A, et al. The nature of flow in the systemic venous pathway measured by magnetic resonance blood tagging in patients having the Fontan operation. J Thorac Cardiovasc Surg. 1997;114:11032-11041. 82. Shekerdemian LS, Bush A, Shore DF, et al. Cardiopulmonary interactions after the Fontan operation. Augmentation of cardiac output using negative pressure ventilation. Circulation. 1997;96:3934-3942. 83. Williams DB, Kiernan PD, Metke MP, et al. Hemodynamic response to positive end-expiratory pressure following right atriumpulmonary artery bypass (Fontan procedure). J Thorac Cardiovasc Surg. 1984;87:856-861.











































48. Dhainaut JF, Bricard C, Monsallier FJ, et al. Left ventricular contractility using isovolumic phase indices during PEEP in ARDS patients. Crit Care Med. 1982;10:631-635. 49. Johnston WE, Vinten-Johansen I, Santamore WP, et al. Mechanism of reduced cardiac output during positive end-expiratory pressure in the dog. Am Rev Respir Dis. 1989;140:1257-1264. 50. Rankin JS, Olsen CO, Arentzen CE, et al. The effects of airway pressure on cardiac function in intact dogs and man. Circulation. 1982;66:108-120. 51. Schulman DS, Biondi JW, Matthay RA, et al. Effect of positive endexpiratory pressure on right ventricular performance: importance of baseline right ventricular function. Am J Med. 1988;84:57-67. 52. Schulman DS, Biondi JW, Zohgbi S, et al. Coronary flow limits right ventricular performance during positive end-expiratory pressure. Am Rev Respir Dis. 1990;141:1531-1537. 53. Fessler HE, Brower RG, Wise R, et al. Positive pleural pressure decreases coronary perfusion. Am J Physiol. 1990;258:H814-H820. 54. Jellinek H, Krafft P, Fitzgerald RD, et al. Right atrial pressure predicts hemodynamic response to apneic positive airway pressure. Crit Care Med. 2000;28:672-678. 55. Michard F, Chemla D, Richard C, et al. Clinical use of respiratory changes in arterial pulse pressure to monitor the hemodynamic effects of PEEP in patients with acute lung injury. Am J Respir Crit Care Med. 1999;159:935-939. 56. Michard F, Boussat S, Chemla D, et al. Relation between respiratory changes in arterial pulse pressure and fluid responsiveness in septic patients with acute circulatory failure. Am J Respir Crit Care Med. 2000;162:134-138. 57. Van Den Berg P, Jansen JR, Pinsky MR. Effect of positive pressure on venous return in volume-loaded cardiac surgical patients. J Appl Physiol. 2002;92:1223-1231. 58. Pinsky MR, Matuschak GM, Klain M. Determinants of cardiac augmentation by increase in intrathoracic pressure. J Appl Physiol. 1985;58:1189-1198. 59. Michard F, Teboul JL. Predicting fluid responsiveness in ICU patients: a critical analysis of the evidence. Chest. 2002;121: 2000-2008. 60. Kumar A, Anel R, Bunnell E. Pulmonary artery occlusion pressure and central venous pressure fail to predict ventricular filling volume, cardiac performance or the response to volume infusion in normal subjects. Crit Care Med. 2004;32:691-699. 61. Reuter DA, Kirchner A, Felbinger TW. Usefulness of left ventricular stroke volume variation to assess fluid responsiveness in patients with reduced cardiac function. Crit Care Med. 2003;31:1399-1404. 62. Slama M, Masson H, Teboul JL. Monitoring of respiratory variations of aortic blood flow velocity using esophageal Doppler. Intensive Care Med. 2004;30:1182-1187. 63. Feissel M, Michard F, Faller JP, Teboul JL. The respiratory variation in inferior vena cava diameter as a guide to fluid therapy. Intensive Care Med. 2004;30:1834-1837. 64. Vieillard-Baron A, Chergui K, Rabiller A. Superior vena caval collapsibility as a gauge of value status in ventilated septic patients. Intensive Care Med. 2004;30:1734-1739. Abel JG, Salerno TA, Panos A, et al. Cardiovascular effects of positive pressure ventilation in humans. Ann Thorac Surg. 1987;43:198-206. 65. Gattinoni L, Marini JJ, Pesenti A, et al. The “baby lung” became an adult. Intensive Care Med. 2016;42:663-673.

e3

Abstract: Positive-pressure ventilation (PPV) alters ventricular loading conditions and compliance. In patients who are hypovolemic, the effects of positive airway pressure on the right heart predominate, whereas in patients who have systemic ventricular systolic dysfunction, the effects of PPV on left ventricular afterload predominate. Large changes in arterial pulse pressure over the respiratory cycle help to identify mechanically ventilated patients who may have a favorable response to the administration of fluid or who may not tolerate high levels of positive end-expiratory pressure

without fluid administration. PPV raises juxtacardiac pressure, thereby reducing left ventricular afterload. Respiratory effort imposes critical loads on the heart, and respiratory muscle failure from inadequate oxygen delivery is a final common pathway to death from shock. Key Words: positive-pressure ventilation, ventricular loading conditions, preload, afterload, juxtacardiac pressure

10 33

Chapter Title Disorders of Cardiac Rhythm

CHAPTER FRANK A. AUTHOR FISH AND PRINCE J. KANNANKERIL

PEARLS • • •















To gain basicmay knowledge of the development ofask, the“What’s eye. the Arrhythmias result from ongoing therapies; To develop essential understanding how abnormalities DEAL?” (drugs and drips, electrolytes, airway and acid-base,atlines). various stages of development can arrest or hamper normalarAppropriate diagnosis is key. Always attempt to document formation the ocular structures and visual therapy. pathways. rhythmia inofmultiple leads before instituting For ventricular fibrillation or pulseless ventricular tachycardia, begin cardiopulmonary resuscitation and defibrillate immediately. Involve a cardiologist before initiating (chronic) antiarrhythmic drug therapy.

Cardiac arrhythmias are frequently encountered in the intensive care unit (ICU) setting. This chapter reviews diagnosis and management of arrhythmias representing a primary disease process and those that occur secondary to other conditions or therapies (Table 33.1).1,2 Prompt restoration of hemodynamic stability concurrent with appropriate identification of the arrhythmia mechanism and predisposing factors is emphasized while providing a broader overview of arrhythmia mechanisms and their associated presentations in pediatric patients.

Classification of Arrhythmias Arrhythmias can be classified according to rate, electrocardiographic features, and, when possible, the underlying electrophysiologic mechanisms. Electrocardiographically, arrhythmias can be characterized as bradycardias, extrasystoles (ectopy), or tachycardias. Bradycardias are further subdivided by the level of dysfunction (e.g., sinus node dysfunction vs. atrioventricular [AV] block) and by the ensuing rhythm (sinus, atrial, junctional, or idioventricular). Extrasystoles and tachycardias are categorized as atrial, junctional, or ventricular in origin. Tachycardias are initially characterized by the level of origin (supraventricular vs. ventricular), by electrocardiographic pattern, and functional mechanism: reentry, automaticity, or triggered activity. Whereas most treatment algorithms (e.g., Pediatric Advanced Life Support) assume a reentrant mechanism, abnormal triggering and automaticity may be particularly important in the ICU setting. Although differentiating between these mechanisms is sometimes difficult, it may be essential in guiding appropriate therapy, especially when initial therapies prove ineffective.

• •









To acquire possible, adequateuse information anatomy of rate Whenever available about meansnormal to document atrial the eye and related structures and develop a strong foundation to discern correct ventricular-atrial relationship. for the understanding of common ocular problems and their Supraventricular tachycardia is a nonspecific electrocardioconsequences. graphic pattern. Multiple types of supraventricular tachycardia exist; appropriate therapy depends on appropriate diagnosis. Whenever possible, opt for therapies that maintain atrioventricular synchrony.

Bradycardias Appropriate Versus Normal Heart Rate Since normal heart rate ranges vary tremendously during childhood as a function of age and autonomic tone, “appropriate” heart rate is a more useful concept than “normal” heart rate. Thus, any inappropriately fast or slow rate for a given clinical circumstance warrants evaluation for factors affecting the sinus rate, such as pain, agitation, respiratory insufficiency, oversedation, anemia, or acidosis, as well as the potential for other arrhythmias resembling sinus.

Sinus Bradycardia and Sinus Pauses Causes of sinus bradycardia include high vagal tone, hypothermia, acidosis, increased intracranial pressure, drug toxicities, or direct surgical trauma to the sinoatrial (SA) node. Primary sinus node dysfunction in childhood is rare but has been described.3 Transiently profound sinus bradycardia or prolonged sinus pauses of several seconds’ duration may be caused by intense vagal episodes, such as those occurring during neurocardiogenic syncope, apnea, or endotracheal suctioning. When clearly correlated with a vagal stimulus, pacing can usually be deferred. However, the hemodynamically tenuous patient with persistent or recurring bradycardias may warrant vagolytic, sympathomimetic, or pacing therapies. Atrial premature beats that fail to conduct over the AV node can be mistaken as sinus pauses (when isolated) or sinus bradycardia (when in a bigeminal pattern). The blocked P wave may be obscured when buried in the preceding T wave; searching for changes in T-wave morphology or for intermittently conducted P waves may reveal the diagnosis. In cardiac patients, sinus node dysfunction may be the result of surgical injury or heterotaxy 329

330

SECTION IV



Pediatric Critical Care: Cardiovascular

TABLE Classification of Arrhythmias by Type and Basis 33.1

Arrhythmia Ventricular premature beats and supraventricular premature beats Sinus bradycardia, sick sinus syndrome

Primary

Secondary

111

111

11

11

11

111

Incomplete AV block Mobitz type I Mobitz type II Congenital third-degree AV block

11 111

Acquired third-degree AV block Paroxysmal SVT (AV reentrant tachycardia, AV nodal reentrant tachycardia)

11 11

111

Ectopic atrial tachycardia

11

Atrial flutter and intraatrial reentry

11

111

1

111

Atrial fibrillation Chaotic atrial tachycardia Junctional ectopic tachycardia

11 1

111

11

11

Torsades des pointes

1

1

Ventricular fibrillation

1

11

11

1

Monomorphic ventricular tachycardia

Bidirectional ventricular tachycardia

AV, atrioventricular; SVT, supraventricular tachycardia; 111, typical; 11, occasional; 1, rare.

AV block. The PR and QRS duration are usually prolonged in type II block. Type II block may be more ominous in that there may be abrupt loss of conduction of multiple consecutive atrial impulses, resulting in ventricular asystole. As such, Mobitz type II and may require more aggressive and preemptive intervention with pacing. Higher grades of second-degree AV block are best characterized by the ratio of atrial to ventricular depolarizations (2:1, 3:1, 4:1, and so on), as the level of block cannot be inferred. Highgrade AV block during sinus rhythm is usually pathologic, whereas it can represent a normal physiologic response within the AV node in the setting of a rapid atrial tachyarrhythmia, such as atrial flutter. Importantly, vagally mediated AV block may result in transient high-grade block, usually with concurrent slowing of the sinus rate. This should not be misconstrued as Mobitz type II AV block, as pacing would not usually be warranted. Complete or third-degree AV block represents complete loss of AV conduction, usually with a junctional or idioventricular escape rhythm. In complete AV block, the escape rhythm is usually very regular, whereas AV block with variation in the RR interval usually indicates intermittent conduction during second-degree block (Fig. 33.1) or a junctional rhythm with slower atrial rate and intermittent AV conduction (sinus capture complexes). Bundle branch block patterns occur when impaired conduction in the specialized intraventricular conduction system results in delayed right or left ventricular depolarization, resulting in an aberrant widened QRS complex. Bundle branch block and AV block sometimes represent normal physiologic responses to abrupt shortening of the cycle length (as with premature atrial systoles or supraventricular tachycardia initiation) or may result from drug effects, surgical injury, or primary disease within the specialized conduction tissue.

Escape Rhythms and Accelerated Rhythms syndromes. Sinus bradycardia can also be an initial manifestation of long QT syndrome in infancy and of channelopathies.4

Atrioventricular Block AV block is characterized as first-degree, second-degree, and third-degree, according to whether there is conduction delay, intermittent block, or complete loss of AV conduction, respectively. As with sinus bradycardia, AV conduction delay may be the result of intense vagal tone, metabolic derangements, drug toxicity, or direct injury to the AV node. When transient AV block is the result of vagal tone, there is usually concurrent slowing of the sinus rate. Second-degree block can be further characterized as Mobitz type I or Mobitz type II. In Mobitz I (Wenckebach conduction), there is progressive prolongation of the PR before block. It may be best recognized by comparing the last PR interval before block with the next conducted PR. Mobitz type I usually represents block in the AV node and is less likely to progress suddenly to high-grade block. The periodicity is the result of progressive beatto-beat delay within the AV node until conduction fails, allowing the AV node to recover on the ensuing cardiac cycle. In some settings, such as in well-conditioned athletes, Mobitz type I block may be benign. Mobitz type II AV block is characterized by abrupt failure to conduct without prior lengthening of the PR interval. It usually is due to block below the AV (within the His bundle) and may portend a greater potential for sudden progression to complete

In the presence of sinus bradycardia or AV block, slower escape rhythms typically emerge from the atrium, AV node, HisPurkinje system, or ventricular myocardium. When slower than the appropriate sinus rate, they are referred to as escape rhythms (atrial, junctional, or idioventricular). Similar rhythms may emerge and compete with an appropriate sinus rhythm, in which case they are referred to as an accelerated (junctional or idioventricular) rhythm. Such accelerated rhythms can result from increased adrenergic tone, intrinsic mechanisms (such as idioventricular rhythm), or sympathomimetic drug infusions. It is important to distinguish these accelerated subsidiary rhythms from escape rhythms resulting from AV block. Only rarely does an accelerated junctional or ventricular rhythm result in significant symptoms in a healthy child.5 However, in the critically ill patient, loss of AV synchrony may be poorly tolerated and atrial pacing at a slightly faster rate to reestablish AV synchrony may be beneficial. The rate defining an accelerated junctional or ventricular rhythm from a corresponding pathologic tachycardia can sometimes be arbitrary but is usually determined by the similarity in rates and gradual transitions between the accelerated rhythm with the concurrent sinus rhythm.

Tachycardias Tachycardia Mechanisms While sinus tachycardia is the most common tachycardia among ill patients, pathologic tachycardias can occur due to three basic



CHAPTER 33

Disorders of Cardiac Rhythm

I

aVR

V1

V4

II

aVL

V2

V5

III

aVF

V3

V6

331

VI

• Fig. 33.1  



​Complete atrioventricular (AV) block, presumably congenital, in an asymptomatic 9-year-old with slow resting heart rate. Note the regular RR interval, which confirms complete rather than incomplete (second-degree) AV block.

mechanisms: reentry, automaticity, or triggered. Reentry accounts for most forms of supraventricular tachycardia (SVT), including atrial flutter and most sustained ventricular tachycardias (VTs). Reentrant tachycardias display an abrupt onset and termination, usually maintaining a relatively fixed rate. In contrast, automatic tachycardias arise from ectopic foci within the atrium, AV node, or ventricles and tend to display more gradual changes in rate with warm up and cool down at onset and offset. Triggered tachycardias result from abnormal secondary depolarizations (afterdepolarizations). They tend to occur as repetitive bursts or salvos of tachycardia and may be recognized by their dependence on underlying heart rate for initiation. Triggered activity is especially important in several specific situations, such as digoxin toxicity (delayed afterdepolarizations), congenital and drug-induced long QT syndromes (early afterdepolarizations), and other hereditary arrhythmia syndromes, such as catecholaminergic polymorphic ventricular tachycardia (CPVT). Many early postoperative atrial arrhythmias and chaotic (multifocal) atrial tachycardias display behavior suggestive of a triggered mechanism. The dependence of triggered activity on underlying heart rate can sometimes be exploited therapeutically because either raising or lowering the underlying rate may suppress the tachycardia. Though it can be difficult to discern among the various tachycardia mechanisms, it is important to recognize that automatic and triggered arrhythmias will not respond favorably to electrical cardioversion.

Supraventricular Tachycardias Supraventricular tachycardia is often used to describe the typical electrocardiographic phenotype of a regular, narrow QRS tachycardia without discernible P waves that starts and stops abruptly

(also described as paroxysmal atrial tachycardia [PAT] or paroxysmal supraventricular tachycardia [PSVT]). However, SVTs include a more diverse array of tachycardia mechanisms, including atrial flutter and fibrillation and junctional tachycardias, many of which may, at times, display a typical SVT phenotype.6 As such, depending on the specific SVT mechanism, they may display regular, irregular, or wide QRS patterns. These mechanisms may respond very differently to treatments with adenosine, pacing, cardioversion, or other antiarrhythmic drugs. Therefore, in approaching a patient with SVT, it is best to understand the broader differential diagnosis and to identify the most likely mechanism, to the degree possible, in order to choose the most appropriate therapy.

Atrioventricular Reciprocating Tachycardias (AV Reentry) AV reciprocating tachycardias represent the most common SVT in infants and young children.7 They result from reentry between the atria and ventricles, using an accessory AV connection (pathway) and the AV node. As such, they must display a fixed 1:1 AV relationship, since any break in this relationship will terminate the tachycardia. Usually, antegrade conduction from atria to the ventricles is over the AV node–His conduction system and retrograde conduction is via the accessory connection, referred to as orthodromic reciprocating tachycardia (ORT).6,7 Atrial activation closely follows ventricular activation, such that P waves (if discernible) immediately follow each QRS complex in the ST segment. Permanent junctional reciprocating tachycardia (PJRT) is a unique variant of ORT in which the accessory pathway displays slow, decremental (AV node-like) conduction.8 The result is typically a slower, more incessant long RP tachycardia displaying a short or normal PR interval. In a single-lead rhythm strip, PJRT

332

SECTION IV



Pediatric Critical Care: Cardiovascular

VI

II

V5

• Fig. 33.2

​Supraventricular tachycardia resulting from the permanent form of junctional reciprocating tachycardia. Note the slow rate, long RP and short PR interval, and inverted P waves in electrocardiogram leads II and III. At rest, this rhythm often shows incessantly repetitive termination (with retrograde block), followed by immediate reinitiation (after single, isolated sinus complexes). With exercise, this rhythm is often faster and sustained, rendering it electrocardiographically indistinguishable from the atypical form of nodal reentry.  



may resemble sinus tachycardia, although a 12-lead electrocardiogram (ECG) reveals atypical P-wave morphology with P-wave inversion in the inferior limb leads. Repetitively spontaneous termination and prompt reinitiation are common features aiding in recognition of PJRT (Fig. 33.2). In many patients with ORT, the accessory pathway can only conduct retrograde and thus is clinically evident only during tachycardia or during ventricular pacing; the QRS is normal during sinus rhythm. However, if the accessory connection conducts antegradely during sinus rhythm, preexcitation results in a delta wave: shortened PR interval and slurring of the QRS upstroke, the hallmark of Wolff-Parkinson-White (WPW) syndrome. Many patients with congenital heart disease may display a slurred QRS upstroke due to intraventricular conduction delay, which can be readily distinguished from WPW if the PR is normal or prolonged. Antidromic reciprocating tachycardia (ART) is a much less common form of AV reentry in which the circuit is reversed—antegrade conduction from atrium to ventricle is over the accessory connection, resulting in a very wide (maximally preexcited) QRS. ART can be difficult to distinguish from VT, and because it is a potentially more dangerous rhythm than ORT, it is most appropriately managed as VT in the acute setting. However, patients experiencing ART will display the WPW pattern upon restoration of sinus rhythm. A specific form of antidromic AV reentry uses an atriofascicular connection as the antegrade limb of the tachycardia and AV node (or a second accessory connection) as the retrograde limb. The atriofascicular connection (essentially an accessory AV node) traverses the lateral tricuspid valve annulus and inserts into the right ventricular fascicular system. It results in wide QRS tachycardia resembling a typical left bundle branch block pattern and is particularly common among patients with Ebstein anomaly. Owing to the decremental nature of the fiber, overt preexcitation is often subtle or altogether absent during sinus rhythm. Though usually responsive acutely to maneuvers interrupting AV node conduction, electrophysiologic study is required to confirm the diagnosis and distinguish it from VT or other forms of SVT with left bundle branch block aberrancy.

Atrioventricular Nodal Reentrant Tachycardia AV nodal reentrant tachycardia (AVNRT) is the second most common cause of SVT in older children and young adults without

WPW syndrome or structural heart disease. It is seen less commonly in infants.6,7 This tachycardia is attributed to so-called dual AV nodal physiology in which two or more separate inputs into the AV node (slow and fast pathways) conduct into and out of the AV node, providing the substrate for reentry. Classically, two electrocardiographically distinct forms of AVNRT may occur. In typical AV node reentry, antegrade and retrograde conduction occurs over the slow and fast AV node inputs, respectively; retrograde P waves are obscured by the preceding QRS complex. In the atypical form of AV node reentry, the circuit is reversed, resulting in a long RP and short PR with an inverted P wave. Thus, typical AVNRT closely resembles ORT, whereas atypical AVNRT resembles PJRT (see previous section). Like PJRT, atypical AVNRT can also be mistaken for sinus tachycardia as well as other atrial tachycardias (see next section). AVNRT with 2:1 conduction and other complex variations can be defined only with an intracardiac electrophysiologic study.

Primary Atrial Tachycardias Atrial tachycardias present with diverse ECG patterns and behaviors and may be the result of reentrant, automatic, and likely triggered mechanisms. ECG patterns may include discrete and regular P waves (ectopic atrial tachycardia, intraatrial reentry), sawtooth flutter waves (atrial flutter), or disorganized atrial activity (atrial fibrillation, chaotic atrial tachycardia).9 Conduction to the ventricles can be variable, depending on atrial rate and AV conduction. Thus, the resultant ventricular rate, rather than the atrial rate, determines the severity of symptoms. Irregularity in the ventricular rate and loss of AV synchrony also contribute to symptoms. Sometimes, it may be difficult to discern atrial tachycardias from sinus, particularly when 1:1 conduction is present and if the P-wave axis is normal. Likewise, a 2:1 conduction pattern can be difficult to recognize, but an unusually short or long PR during tachycardia should raise suspicion of a second hidden P wave. Observing for transient variations in the conduction pattern or using adenosine or vagal maneuvers to create transient AV block may reveal the faster, ongoing atrial rhythm. Direct recordings of atrial activity (via esophageal, epicardial, or intraatrial recordings) are also useful, particularly in revealing 2:1 AV conduction. Interrogation of a permanent pacemaker will also allow determination of the atrial rhythm. Despite the potential electrocardiographic

similarities of the various primary atrial tachycardias, the varying mechanisms confer important differences in clinical behavior and management. Many atrial tachycardias encountered in the pediatric intensive care setting are due to atrial reentry and are commonly referred to as atrial flutter regardless of whether the classic sawtoothed pattern (as in neonatal atrial flutter) is observed. In older patients with congenital heart disease (CHD), the term intraatrial reentrant tachycardia (IART) is often used to describe the slower, scar-dependent atrial reentry, which can result in more discrete and sometimes normalappearing P waves (rather than classic flutter waves). Because of the slower atrial rate, 1:1 AV conduction may be especially common, further confounding the diagnosis. Likewise, administration of an AV node–blocking agent may result in 2:1 AV conduction, leading to the erroneous conclusion that the tachycardia has been converted to sinus rhythm with first-degree AV block. Thus, this arrhythmia should be considered in any patient with postoperative CHD presenting with a monotonous and inappropriate tachycardia, otherwise suggestive of sinus tachycardia on ECG. Early after congenital heart surgery, atrial tachycardias often occur as repetitive, self-limited bursts. Many of these are likely due to automatic or triggered behavior. While they may require therapy in the short term, they may not portend ongoing susceptibility to long-term IART. Atrial fibrillation, a common arrhythmia among the elderly, is far less common in pediatric patients. It may occur as the result of atrial myocarditis,10 secondary to an underlying reentrant SVT,11 or mechanical stimulation by central venous catheters (Fig. 33.3). In patients with WPW syndrome, rapid conduction over the accessory pathway during atrial fibrillation can be life-threatening. When atrial fibrillation occurs in a young patient, the underlying



CHAPTER 33

Disorders of Cardiac Rhythm

basis should be sought because the long-term implications and treatment measures are much different from those in adult patients. A significant proportion of teens and adolescents presenting with lone atrial fibrillation are found to have an underlying SVT when taken for electrophysiology study. Thus the long-term therapy of atrial fibrillation in this age group is much different from that administered to the elderly. Ectopic atrial tachycardia (EAT) is an automatic arrhythmia that typically presents as an incessant and chronically elevated atrial rhythm that may be mistaken for sinus tachycardia, especially when the automatic focus is in the right atrium. First-degree AV block may be seen as a physiologic response to the inappropriately accelerated atrial rate. Patients with EAT often experience no overt palpitations but present instead with ventricular dysfunction and sometimes frank congestive heart failure (CHF). Because adenosine may transiently inhibit the automatic focus, termination with adenosine is nondiagnostic in distinguishing EAT from sinus tachycardia. Instead, observation of the heart rate behavior over periods of several hours and careful scrutiny of the P-wave morphology on 12-lead ECG, especially with transitions between the EAT and sinus, are necessary to establish the diagnosis. Chaotic (multifocal) atrial tachycardia is an uncommon arrhythmia that is usually observed in infants and toddlers, often in association with a viral respiratory illness.12 The hallmark features are a rapid and irregular atrial rate, often exceeding 300 beats/ min, and presence of multiple P-wave morphologies. The resulting ventricular response is irregularly irregular, simulating atrial fibrillation. However, this rhythm is probably the result of multiple triggered foci within the atria. Thus, acute termination measures (i.e., direct current cardioversion, adenosine, or pacing) are of little benefit. Usually, this arrhythmia resolves within weeks or

I

aVR

V1

V4

II

aVL

V2

V5

III

aVF

V3

V6

V1

• Fig. 33.3



​Sustained atrial fibrillation in a previously healthy 17-year-old placed on extracorporeal membrane oxygenation for pneumonia and acute respiratory distress syndrome. Review of stored telemetry revealed atrial ectopy soon after cannulation, eventually initiating atrial fibrillation. Following direct current cardioversion to sinus rhythm, frequent ectopy persisted until the venous cannula was repositioned. Several months later, the patient remained free of further arrhythmias.  

333

334

SECTION IV



Pediatric Critical Care: Cardiovascular

months of presentation. Treatment is based on the severity of symptoms, which may range from negligible to life-threatening.

Junctional Ectopic Tachycardia Junctional ectopic tachycardia (JET) probably arises from an abnormal automatic focus or a protected microreentrant circuit in the region of the AV node or proximal His bundle. Antegrade conduction is usually over the normal His-Purkinje system with a narrow QRS. Commonly, there is retrograde (ventriculoatrial [VA]) block with complete VA dissociation; the resulting sinuscapture complexes aid in the recognition of this mechanism. Sometimes, antegrade AV block coexists with JET. Variants include the common transient postsurgical JET, a congenital chronic JET, and paroxysmal JET described primarily in adults.13,14 As in postoperative atrial tachycardias, direct atrial recordings aid the diagnosis. In some cases, JET is associated with 1:1 VA conduction, in which case additional pacing or pharmacologic maneuvers may be necessary to distinguish it from other mechanisms of SVT. Ventricular Tachycardias VTs arise exclusively within the ventricle(s); by definition, the QRS duration is always aberrant and prolonged for a given age and heart rate. The QRS morphology may be either uniform or changing (bidirectional, polymorphic). Classically, VTs are associated with VA dissociation (atrial rhythm at a slower rate). Thus, VA dissociation, when present, is helpful, but when absent (or uncertain) does not exclude VT as the underlying mechanism. The presence of periodic fusion complexes (QRS morphology intermediate between tachycardia morphology and sinus morphology) is diagnostic since it implies VA dissociation. In infants and young children, recognition of VT may sometimes be difficult. Acutely, VT may be mistaken for SVT due to the relatively narrower QRS complexes and 1:1 VA conduction due to robust AV node conduction. However, the QRS should be different from the baseline QRS during sinus. JET with aberrancy, like VT, is often characterized by sinus-capture complexes but without QRS fusion. The QRS with sinus capture or with sustained AV conduction during overdrive atrial pacing should remain unchanged in JET. Like SVTs, VTs are diverse in pattern, mechanism, and severity, and they may result from each of the tachycardia mechanisms previously discussed (reentry, automaticity, and triggered activity) with important therapeutic implications. The clinical setting, electrocardiographic pattern, and severity of symptoms dictate acute treatment approaches.

• Fig. 33.4

Approach to Diagnosis Monitoring and General Assessment In the intensive care setting, there may be a trade-off between precision of diagnosis and urgency of therapy. Even so, appropriate diagnosis remains key to establishing appropriate ongoing therapy. When an arrhythmia develops, factors such as level of consciousness, ventilation, tissue perfusion, and acid-base status (including mixed venous saturation and lactate) govern the acuity with which treatment is required and the extent to which additional diagnostic measures can be employed before initiating therapy. Minimal initial diagnostic evaluation should always include permanently recorded ECG rhythm strips along with a rapid review of drugs being administered, potential toxic exposures, respiratory and acid-base status, and known associated illnesses that might be arrhythmogenic. Electrolytes, including calcium and magnesium, should be obtained along with drug screening in the patient presenting with altered mental status. The details of recent cardiac surgical procedures and any recent trauma (chest and cranial) should be quickly reviewed. Indwelling catheter position should be noted on radiographs for potential intracardiac location. Concurrent with this brief survey, a differential diagnosis of the rhythm disturbance should be established quickly, followed by the most appropriate emergency therapy. If the patient is sufficiently stable, therapy may be deferred until a 12-lead ECG is obtained and other diagnostic measures taken for more precisely characterization.

Surface Electrocardiogram and Bedside Monitoring The surface ECG remains the cornerstone of arrhythmia diagnosis. Certainly, in patients with known cardiac abnormalities and, arguably, in all patients admitted to the ICU, a baseline ECG should be obtained at admission. This ECG may provide a valuable baseline for later comparison in the event of a new arrhythmia or other changes, such as cardiac ischemia, which may predispose to arrhythmias. For sustained tachycardias, a full 12-lead ECG should be obtained whenever possible because diagnostic details (such as QRS aberrancy, atrial rate, and P-wave morphology) or hidden features (such as flutter waves) may be evident only in selected leads (Fig. 33.4). Because most contemporary ICU monitors provide a full disclosure feature, a strip should usually be available to assess additional details such as the onset of an arrhythmia as well as variations in rate, beat-to-beat regularity, QRS morphology and

​Ventricular tachycardia following cardiac surgery. Note similarity between QRS complexes during sinus rhythm and ventricular tachycardia in two of three recorded leads.  



duration, and AV relationship (Table 33.2), which collectively should provide an initial differential diagnosis (Fig. 33.5). Ideally, multiple ECG monitoring leads should be inspected. Rate trends should be reviewed for abruptness of onset to help discriminate between sinus and nonsinus tachycardias in the critically ill patient. TABLE Electrocardiographic Patterns 33.2

Pattern

Description

AV Block Mobitz type I

Shortened PR of first conducted beat after block

Mobitz type II

No change in PR before/after block Periods of high-grade block

Third-degree

Fixed, rather than variable, RR interval

Supraventricular Tachycardias AV reentrant supraventricular tachycardia

P waves obscured or buried in ST segment

AV nodal reentrant supraventricular tachycardia

P waves obscured by terminal QRS Pseudo R9 in lead V1 during tachycardia, not sinus

Junctional ectopic tachycardia

Narrow QRS tachycardia, VA dissociation RR periodically shortened because of sinus capture complexesa

Atrial ectopic tachycardia

Monotonous rate, inappropriately fast Abnormal P-wave morphology (may be subtle)

Intraatrial reentrant tachycardia

Inappropriately fast rate, discrete P waves, variable AV conduction in postoperative patient with congenital heart disease

Atrial flutter

Variable RR interval Rapid, sawtooth flutter waves (.280 beats/min)

Atrial fibrillation

Irregular ventricular rate Coarse baseline with no discernible P waves

Chaotic atrial tachycardia

3 P-wave morphologies, irregular atrial rate, variable AV conduction (periods of atrial flutter or fibrillation common)

Ventricular Tachycardias Monomorphic

Wide QRS for age, different from baseline Slurred upstroke of QRS Variable VA conductionb Sinus capture complexesb

Idiopathic types

Left bundle branch block, inferior axis (right ventricular outflow tract origin) Right bundle branch block, left superior axis (left ventricular septal origin)

Bidirectional

Alternating QRS axis (beat-to-beat)

Torsades des pointes

Initiation with short-long-short sequence QT-interval prolongation before onset Twisting of QRS axis

AV, Atrioventricular; VA, ventriculoatrial. a Junctional ectopic tachycardia may be associated with third-degree AV block. b Helpful when seen; absent if 1:1 VA relationship.



CHAPTER 33

Disorders of Cardiac Rhythm

335

While the surface ECG usually is sufficient to characterize most bradycardias, additional diagnostic maneuvers, sometimes coupled with direct recording of atrial activity, may be required to accurately characterize tachycardias. Changes in ventricular rate and regularity, QRS duration and morphology, and atrial-toventricular relationship must be actively sought. When available, temporary atrial pacing wires or an esophageal ECG can facilitate the diagnosis (Fig. 33.6). Likewise, observing the response of an arrhythmia following perturbations such as premature atrial contractions (PACs) or premature ventricular contractions (PVCs) may be informative. Repetitive patterns in ventricular activation referred to as grouped beating always provide important clues to the diagnosis.

Bradycardias Perhaps the most important diagnostic issues in bradycardias are determination of whether AV conduction occurs and recognizing the presence and rate of any underlying intrinsic escape rhythms. This should usually be straightforward when the atrial rate exceeds the ventricular rate during second-degree or third-degree AV block. In complete AV block, the resultant escape rhythm is usually regular; in second-degree AV block, the ventricular intervals vary (see Fig. 33.1). This is especially helpful when sinus node disease and AV block coexist; variation of the RR interval typically implies some degree of AV conduction. When atrial pacing is feasible (as in patients with recent cardiac surgery), AV conduction can be more directly characterized. The distinction between bradycardia resulting from AV block and sinus node dysfunction may have important therapeutic implications, particularly with respect to pacing. If AV nodal conduction is fully intact, it is usually desirable to pace the atrium only (see AAI mode) rather than perform dual-chamber pacing. In contrast, isolated AV block is best managed by sensing and tracking the intrinsic atrial rate (see DDD mode) and pacing the ventricle. Other variations in pacing modes are discussed elsewhere in this chapter.

Extrasystoles Extrasystoles, or premature beats, are defined as supraventricular (supraventricular premature beats or premature atrial or junctional complexes) or ventricular (premature ventricular complexes, ventricular premature beats [VPBs]) in origin. True junctional extrasystoles are uncommon. Isolated premature QRS complexes with prolonged QRS duration may represent either ventricular extrasystoles or aberrantly conducted atrial extrasystoles. Distinguishing the two may be difficult from a single rhythm strip. When the extrasystole results in an early QRS with normal morphology and duration, a supraventricular extrasystole may be presumed. Usually, an early P wave can be discerned, but it may be obscured by the preceding T wave in certain leads. The ensuing sinus beat is usually advanced by the atrial extrasystole, but entrance block can often result in a full compensatory pause, which is usually more characteristic of ventricular extrasystoles. It is important to view multiple leads, since a ventricular extrasystole may resemble the normal QRS in one lead but appear totally dissimilar and broader in others. The ECG can also help discern unifocal versus multifocal PVCs and also help localize the site of PVC origin. The ECG features favoring ventricular extrasystoles over aberrantly conducted atrial extrasystoles include (1) wide QRS morphology, (2) a full compensatory pause prior to the ensuing sinus beat, (3) presence of fusion beats, and (4) absence of a discernible premature P wave. A full compensatory pause indicates failure of

336

SECTION IV



Pediatric Critical Care: Cardiovascular

• Fig. 33.5

​Ventricular tachycardia in an 8-year-old with previous muscular ventricular septal defect repair. Note transient ventriculoatrial block (arrows), excluding a supraventricular mechanism with aberrant conduction.  



aVR aVL aVF

A

B

150– 100– 50

• Fig. 33.6

​(A) Narrow QRS tachycardia in an infant following a stage I Norwood operation. Possible atrioventricular (AV) dissociation is suggested, but P waves are not easily discerned on the surface electrocardiogram. (B) Atrial recording from the same patient (after rate increased) using an epicardial atrial pacing wire. AV dissociation with faster junctional rate is demonstrated, typical of junctional ectopic tachycardia. Absence of clearly shortened RR intervals because of sinus capture might indicate associated AV block.  



the sinus node to be reset by the ventricular depolarization, though a very premature beat may occasionally reset the sinus node because of retrograde (VA) conduction over the AV node to the atrium. Fusion indicates a QRS morphology intermediate between a fully anomalous QRS and the normal QRS. Fusion is also seen in patients with ventricular preexcitation (WPW) and in whom PACs often result in a widened QRS due to selective delay in conduction of the premature impulse through the AV node but not the accessory pathway. The distinction between atrial and ventricular extrasystoles may be somewhat academic in otherwise asymptomatic individuals because neither generally warrants therapy. However, either might be a harbinger for myocardial irritability and should prompt a search for underlying causes. Occasionally, measures to suppress ectopy may appear to improve cardiac output by regularizing filling time in an otherwise tenuous patient. The relative advantages and risks of any such measure, whether achieved with medications or temporary pacing, need to be considered individually.

Tachycardias With Normal QRS In otherwise healthy infants, children, and adolescents, SVT usually represents AV reentrant tachycardia or AVNRT (Fig. 33.7).

Further distinction between these two mechanisms has little impact on acute management. However, in the ICU setting, primary atrial tachycardias (including sinus tachycardia) and junctional tachycardias are considerably more prevalent (particularly following cardiac surgery). The finding of abnormal P-wave morphology (determined by 12-lead ECG), a PR interval greater than 50% of the RR interval, or completely obscured P waves favors a nonsinus mechanism. Finally, ensuring normal QRS duration for age, rather than simply relying on adult standards, is essential in discriminating from VTs. Distinguishing sinus tachycardia from various types of SVT may be difficult. In young patients, intraarterial reentry, atrial flutter, and atrial fibrillation are usually seen following surgical treatment for congenital heart defects involving the atrium (atrial septal defects, atrial repair of transposition of the great arteries, or the Fontan operation).15 The term intraatrial reentrant tachycardia is often used in this setting when a reentrant tachycardia displays discrete P waves rather than the usual sawtooth flutter waves. Because the atrial rate is often relatively slow in comparison with typical atrial flutter, a high index of suspicion is essential in distinguishing this rhythm from sinus rhythm or sinus tachycardia, especially when fixed 1:1 conduction or 2:1 conduction (with blocked P waves obscured by the QRS or T wave). Again, direct atrial recordings using transesophageal electrocardiography or temporary epicardial atrial pacing wires usually facilitate the diagnosis and characterize the AV relationship (see Fig. 33.6B), as can vagal maneuvers, administration of adenosine to temporally interrupt AV conduction, or—in ambulatory patients—simply changing position from supine to standing to assess for changes in the ventricular response.

Tachycardias with Prolonged QRS Tachycardia with prolonged QRS can represent VT, an SVT with aberrancy, or, occasionally, a preexcited tachycardia in a patient with ventricular preexcitation (WPW, Mahaim fiber). VA dissociation, the hallmark and most specific ECG feature of VT, may not be seen in childhood because of rapid retrograde conduction over the AV node (see Fig. 33.5). The distinction between VT and SVT with aberrant conduction can be difficult and may ultimately require invasive electrophysiologic study. Other features favoring VT are the presence of fusion complexes (implying AV dissociation), a superior QRS axis, or a concordant QRS axis across the precordium (i.e., pure R waves or pure S waves). However, these criteria may be unreliable in patients with CHD where normal conduction axes may be abnormal at baseline.



CHAPTER 33

Disorders of Cardiac Rhythm

I

aVR

V1

V4

II

aVL

V2

V5

III

aVF

V3

V6

337

V4R

• Fig. 33.7





​Supraventricular tachycardia resulting from atrioventricular nodal reentry in an infant. Although considerably less common than orthodromic reciprocating tachycardia in this age group, the P wave on the terminal portion of the QRS complex results in a pseudo rSr9 pattern. During sinus rhythm, this terminal deflection on the QRS was absent; the transesophageal recording confirmed the mechanism.

Generally, tachycardias with prolonged QRS should be presumed to be VTs until or unless evidence of an alternative diagnosis is clearly demonstrated. Prolonged attempts to differentiate SVT from VT by noninvasive means may simply delay treatment, and the wrong conclusion may prove disastrous: acute treatment based on a presumed diagnosis of VT is rarely deleterious even when the mechanism subsequently proves to be supraventricular. However, an erroneous presumption of SVT with aberrant conduction may result in rapid clinical deterioration. When feasible, a full 12-lead ECG may aid in the diagnosis, particularly when a baseline ECG in normal rhythm is available for comparison. Apparent hemodynamic stability should not be mistaken as evidence of SVT over VT whether in an otherwise healthy child or in a patient with known cardiac disease.

Assessment of Atrial Activation When the AV relationship during a tachycardia is unclear, sometimes it can be inferred indirectly by other available monitoring. Invasive arterial and venous pressure waveforms can help define atrial contractile action in some situations. For example, cannon A waves are commonly noted in patients with atrial flutter or JET. Direct recording of atrial activity may clarify the AV relationship when it cannot be determined from the surface ECG or other means. Patients recovering from cardiac surgery may have temporary atrial epicardial pacing wires that can be used to record atrial electrograms directly while simultaneously recording the surface ECG (see Fig. 33.6B). Attachment of the atrial wires to a unipolar precordial lead (V lead) on the monitor is an easy way to

observe atrial activation. Alternatively, the atrial wire can simply be placed beneath a limb lead, producing a larger/sharper atrial signal of atrial activation (which can be further accentuated by placing the bedside monitor to detect pacing events). When necessary equipment is available, a bipolar esophageal catheter inserted in the esophagus behind the left atrium can also demonstrate atrial activation.

Diagnostic Uses of Adenosine Although most widely used as an acute therapy for terminating SVT that involves the AV node, adenosine administration also may yield important diagnostic clues to the underlying arrhythmia mechanism.16 By producing transient block in the AV node during tachycardia, it is often possible to distinguish AV reentrant tachycardias and AVNRTs (either of which should terminate) from atrial tachycardias and VTs. However, adenosine’s effects are not always confined to the AV node. Ectopic (automatic) atrial and junctional tachycardias, intraatrial reentry, and certain VTs may also terminate with adenosine. Extreme caution should be taken when administering adenosine during wide QRS tachycardia. Adenosine produces vasodilation, which theoretically can result in hemodynamic deterioration and tachycardia acceleration, or even fibrillation, if tachycardia fails to terminate. Ventricular fibrillation has been rarely observed when adenosine is administered in the setting of WPW syndrome, probably as a result of atrial fibrillation that is then conducted rapidly to the ventricles. Cardiac defibrillation capability should always be readily at hand when administering adenosine for diagnostic or therapeutic purposes.

338

SECTION IV



Pediatric Critical Care: Cardiovascular

Treatment of Rhythm Disturbances The approach to treatment of cardiac arrhythmias is influenced by the clinical setting, but several important considerations help guide therapy in any given situation. The first and most important concern is the degree of hemodynamic compromise associated with a particular arrhythmia. At one extreme, minor rhythm disturbances may be more readily recognized in the intensive care setting than in other situations simply because of the level of monitoring, which may prompt undue attention and unnecessary treatment. At the other extreme, otherwise life-threatening arrhythmias ordinarily requiring acute therapy may be of little acute consequence in the setting of extracorporeal life support (ECLS) or mechanical ventricular assist devices. Indeed, ECLS may serve as adjunctive therapy for refractory arrhythmias. Even ventricular fibrillation in a patient with a ventricular assist device (VAD) rarely results in immediate decompensation. In contrast, arrhythmias that might ordinarily be well tolerated may be acutely destabilizing in an already critically ill patient and require immediate intervention. A second important consideration in critically ill patients, particularly in those after cardiac surgery, is to favor therapies that maintain appropriate AV synchrony whenever feasible. In the setting of marginal hemodynamics, the practice of medically slowing the ventricular rate during arrhythmias such as atrial tachycardias and fibrillation, junctional tachycardia, or AV block may be inadequate to preserve cardiac output (Fig. 33.8). A third consideration in the management of arrhythmias in the ICU is the recognition that many arrhythmias in this setting are iatrogenic. Even minor arrhythmias may herald more serious issues, such as electrolyte disturbances, acidosis, subendocardial ischemia, excessive catecholamine infusions, or increased intracranial pressure. It is important to identify and correct any such underlying causes because therapies directed at the rhythm itself may not protect from more serious rhythm decompensation. Finally, whenever feasible, acute and short-term measures with limited potential to impair hemodynamics generally should be favored over chronic therapies. Thus, nonpharmacologic therapies (such as pacing or cardioversion) or ultra-short-acting drugs (such as adenosine or esmolol) may be preferable to chronic antiarrhythmic therapy. Whether chronic therapy is warranted for a given arrhythmia is determined more by its underlying mechanism, clinical setting, and frequency than by the severity of the arrhythmias encountered in the intensive care setting. Before beginning chronic antiarrhythmic therapy, consultation with a mm/sec

PACING ON P P P P P P P P P P PP P P P P

20 0

10 75

55

• Fig. 33.8

30

85

65

​Junctional ectopic tachycardia with hypotension immediately improved with faster atrial pacing—in this case, resulting in 2:1 atrioventricular (AV) block and AV synchrony.  



cardiologist versed in the spectrum of arrhythmias seen in childhood is advisable. The impact of acute measures on chronic arrhythmia management becomes increasingly crucial with the emergence of amiodarone use in the ICU and the increasing availability of nonpharmacologic therapies such as radiofrequency catheter ablation and implantable defibrillators for a broader spectrum of arrhythmias and patient populations.17–19

Bradycardia Therapies Whenever treatment is instituted for a rhythm disturbance, an underlying cause should be sought and corrected. This is especially important for bradycardias that occur in the intensive care setting, where airway compromise and respiratory insufficiency probably are the most common causes of acute bradycardias. Increased intracranial pressure, hypothermia, or iatrogenic causes also may produce bradycardias that require specific interventions beyond those outlined here. Emergency interventions for AV nodal and sinus nodal dysfunction are essentially identical. Chest compressions should be administered if there is no effective underlying rhythm.

Pharmacologic Treatment of Bradycardias After appropriate confirmation or restoration of airway integrity and ventilatory function, initial treatment of symptomatic bradycardias is usually pharmacologic, whether the cause is sinus node slowing or AV nodal block. Atropine (0.01–0.04 mg/kg intravenously [IV] or, if necessary, intramuscularly or via endotracheal tube) may transiently ameliorate bradycardia caused by hypoxia (or other vagal stimulants), digoxin, intracranial hypertension, or AV block as a result of Lyme disease. Atropine is less likely to reverse bradycardic effects of b-blocking agents or other antiarrhythmic drugs, particularly in the setting of underlying sinus node disease. Epinephrine (0.1 µg/kg) can be administered by various routes to accelerate the heart rate. Continuous infusions of epinephrine (0.05–2 mg/kg per minute) or isoproterenol (0.02–0.2 mg/kg per minute) may be instituted. In general, highdose epinephrine or isoproterenol infusions should be replaced by temporary pacing as soon as feasible. Even if lower doses of these agents prove adequate, temporary pacing should be available as a backup. Occasionally, methylxanthines are useful as an alternative to pacing for nonlethal bradycardias. Glycopyrrolate and ketamine may help augment rates in bradycardic patients, requiring sedation or anesthesia. Temporary and Permanent Pacing for Bradycardias Pacing is an essential adjunct to medical management of arrhythmias in the ICU. Several reviews of pacing in children are available.20,21 Pacing can be accomplished using permanently implanted pacemaker and lead systems, temporary epicardial leads attached to the heart at the time of cardiac surgery, transvenous placement of a temporary pacing lead, or transcutaneous patches. Most pacing is performed for bradyarrhythmias, although temporary pacing may be used to terminate reentrant tachyarrhythmias.

Principles of Pacing All pacing requires a complete circuit with at least one lead on or near each chamber that is to be paced. Often, two leads are placed on each chamber (bipolar leads), although sometimes only the cathode is attached to the heart (unipolar leads), with a subcutaneous electrode acting as the anode. The metal can of a permanent

pacemaker can also serve as the anode. The intensity of the pacing stimulus is related to the stimulus duration (pulse width) and its amplitude, which can be expressed as either current (mA) or voltage (V). Energy is proportional to the pulse width and the square of the amplitude. Most temporary pacemakers provide a fixed pulse width, with an adjustable current output (mA). Permanent pacemakers generally have both an adjustable pulse width and amplitude. Sensing intrinsic activity of the chamber being paced is important to prevent asynchronous pacing that may inadvertently induce an atrial or ventricular tachycardia. The sensitivity of permanent and temporary pacemakers is adjustable. The sensitivity setting (mV) refers to a sensing threshold for detection of spontaneous cardiac activity. The spontaneous activity must exceed that threshold to be detected by the pacemaker. Thus, a lower numeric sensitivity setting makes the pacemaker more sensitive to both spontaneous activity of the atrium or ventricle (appropriate sensing) and other electrical signals (oversensing). The programmed mode and timing circuits of the pacemaker determine when the pacemaker paces and its response to sensed events. A simplified pacing code uses three letters to describe pacing modes.22 The first two letters refer to the chamber(s) paced and chamber(s) sensed, respectively (A, atrium; V, ventricle; D, dual). The third letter refers to the response to sensed events (I, inhibit; T, trigger or track; D, dual). Thus, a single-chamber atrial or ventricular pacing demand mode is AAI or VVI mode, whereas dual-chamber pacing is generally DDD mode. Corresponding asynchronous modes are AOO, VOO, or DOO and may be important to prevent inadvertent inhibition of pacing due to electrical interference, such as with electrocautery. Other pacing modes may be employed with permanent pacing systems. Recognizing the peculiarities of these modes is important when distinguishing observed pacing behaviors as appropriate or dysfunctional. Timing intervals most readily adjusted include low rate, AV delay, upper tracking rate (UTR), and postventricular atrial refractory period (PVARP). The sum of the AV delay and the PVARP in milliseconds determines the minimum atrial cycle length (60,000 divided by heart rate) and thus the maximum rate, which can be tracked and paced in the ventricle (UTR).

Temporary Pacing In the pediatric ICU, temporary pacing is most commonly used in patients after surgical treatment of CHD. Temporary epicardial pacing wires are usually placed on the atria and ventricles, allowing pacing of either chamber. As previously noted, direct atrial recording (by attaching the wire to an ECG lead) may also aid in the diagnosis of tachyarrhythmias. Atrial burst pacing can be used to terminate reentrant supraventricular arrhythmias such as IART, AVNRT, and ORT. Pace termination of ORT is shown in Fig. 33.9.

• Fig.





33.9 ​Pace termination of orthodromic reciprocating tachycardia with a burst of atrial pacing.



CHAPTER 33

Disorders of Cardiac Rhythm

339

Although JET generally cannot be terminated with burst pacing, atrial pacing at a rate faster than the JET rate often improves hemodynamics by allowing AV synchrony until the JET resolves or is pharmacologically controlled (see Fig. 33.8). In bradycardic patients who do not have temporary epicardial pacing wires, transcutaneous pacing can be performed acutely. It is important that electrodes and output are appropriate for patient size, and positioning may be critical to maintaining capture. In general, transcutaneous pacing is used only for a short time while a temporary transvenous pacing lead is placed. At the bedside, placement of a balloon-tipped temporary transvenous lead is best accomplished via the internal jugular or subclavian vein because the catheter often can be directed to the ventricle blindly. When fluoroscopy is available, a temporary active fixation may be preferred, allowing more secure positioning and the choice of pacing the atrium, ventricle, or both (in dual-chamber mode).

Setting Temporary Pacing Parameters For AAI or VVI demand pacing, the pacemaker is usually initially set at a rate higher than the patient’s intrinsic atrial or ventricular rate. The pacing threshold (lowest output that captures the chamber being paced) should be determined by decreasing the pacing current (mA) just until capture is lost and then setting output to at least twice threshold. Similarly, sensing threshold is determined by first lowering to a rate below the intrinsic rate of the chamber being paced, then adjusting the sensitivity to a higher numeric setting (less sensitive) until the pacemaker stops sensing the intrinsic activity as indicated by loss of blinking markers on the device and/or inappropriate pacing on the monitor. The sensitivity is then adjusted to a lower numeric value (more sensitive) to determine the highest numeric value at which the pacemaker senses appropriately (sensing threshold). Ideally, the sensitivity is programmed to one-half the sensing threshold, but this is not always possible, especially for temporary atrial leads. Failure to sense can result in inappropriate pacing, which can induce tachyarrhythmias. Fig. 33.10 shows a single inappropriate atrial stimulus initiating sustained ORT. Dual-chamber pacing is more complex. The atrial and ventricular sensitivity and output are set using basically the same process described for single-chamber pacing. The other timing parameters discussed earlier must also be set, including the UTR, AV delay, and PVARP. Atrial events occurring during the PVARP will be ignored by the pacemaker and will not result in a ventricular paced event. The pacemaker is also refractory to spontaneous atrial events during the AV interval. Thus, the total atrial refractory period (TARP) includes both the AV interval and the PVARP and limits the UTR (which cannot exceed the TARP). Thus, increasing the UTR often requires shortening the AV interval and/or the PVARP, thus decreasing the TARP. The TARP also determines high-rate behavior when atrial rates exceed the UTR. If the spontaneous atrial cycle length (60,000 ms/min divided by the spontaneous atrial rate in beats/min) is less than the TARP, only half of the spontaneous atrial beats will result in pacing. The point at which this occurs is known as the 2:1 block rate. In contrast, if the intrinsic atrial cycle length is greater than the sum of the AV delay and PVARP, atrial events exceeding the UTR will still be noted by the pacemaker. The resulting pattern of ventricular pacing resembles Wenckebach AV conduction and is referred to as pacemaker Wenckebach. Thus, it is desirable to program the AV delay and PVARP such that the resulting sum (TARP) is less than the upper tracking limit in milliseconds (60,000 divided by upper programmed rate) to favor pacemaker

340

SECTION IV



Pediatric Critical Care: Cardiovascular

I

• Fig. 33.10

​Initiation of orthodromic reciprocating tachycardia with a single atrial paced beat that falls at a vulnerable time. After several narrow-complex beats, bundle branch block in tachycardia results in a wide-complex rhythm.  



Wenckebach behavior and avoid an acute drop in pacing rate by 50% when the UTR is reached. However, it is also important to distinguish appropriate pacemaker Wenckebach from failure to sense the atrium or failure to pace the ventricle. When initiating dual-chamber pacing, it is important to be sure that the atrial lead senses appropriately and that the rates, intervals, and refractory periods are set appropriately to allow the pacemaker to track the spontaneous atrial rate. If the intrinsic atrial activity is not appropriately sensed and the dual-chamber temporary pacemaker’s low rate is lower than the patient’s intrinsic atrial rate, you may falsely assume that dual-chamber pacing is occurring when it is not. The presence of cannon A waves on central venous pressure tracing may provide a clue that AV synchrony is not occurring. Reversing the atrial leads, lowering the numeric atrial sensitivity, or adjusting the AV interval, UTR, or PVARP may remedy this situation. If the atrium still cannot be sensed appropriately, setting the pacemaker’s low rate higher than the patient’s own atrial rate will allow AV synchrony. In effect, this is pacing in DVI mode: pacing atrium and ventricle, sensing the ventricle, and inhibiting pacemaker output when spontaneous ventricular beats occur, avoiding asynchronous pacing by maintaining a higher atrial-paced rate. Occasionally, a reentrant arrhythmia known as pacemakermediated tachycardia (PMT) may be seen, in which a DDD pacemaker (temporary or permanent) senses the atrium and paces the ventricle, with the patient’s own AV node conducting the impulse up from the ventricle to the atrium and the cycle repeating again. Thus, the pacemaker acts as the antegrade limb of the reentrant circuit, while the patient’s own AV node acts as the retrograde limb. Usually, PMT can be avoided with careful adjustment of the pacemaker’s AV interval and PVARP. In a permanent pacing system, PMT can usually be terminated acutely by placing a magnet over the pacemaker; the resultant asynchronous pacing effectively interrupts the antegrade limb of the reentrant circuit. Fig. 33.11 shows initiation of PMT by loss of atrial capture with termination of the tachycardia by a spontaneous ventricular beat. The selection of optimal temporary pacing mode and rate must be individualized for each patient. Usually, atrial pacing (AAI) is preferred over dual-chamber pacing when AV conduction is intact. However, marked first-degree AV block may unfavorably affect ventricular filling such that dual-chamber pacing provides

better hemodynamics. Periodic reassessment of pacing and sensing thresholds, as well as underlying rhythm, should be performed at least twice daily, and hemodynamic responses to changes in mode or timing parameters should be performed as needed with changes in clinical status.

Permanent Pacing—Indications and Selection The two most common indications for permanent pacing are high-grade AV block and sinus node dysfunction. AV block may be congenital or acquired, with surgical damage to the conduction system the most common cause of acquired AV block. In patients with surgical AV block, there may be recovery of normal conduction with resolution of edema. In general, permanent pacing is not recommended unless there has been no recovery for 7 to 14 days.23 Occasionally, if the surgeon is confident that the conduction system has been permanently damaged or if temporary pacing is not reliable, permanent pacemakers may be implanted within 7 days after the initial injury. Elective pacing for congenital AV block in the first decade of life is usually prompted by symptoms, low ventricular rates, or ventricular ectopy. Elective pacing is commonly recommended in the second decade of life even for asymptomatic patients with congenital heart block on the basis of studies showing that the first symptom in teenagers and adults may be catastrophic.24 Pacing for sinus node dysfunction is most commonly performed in patients with structural heart disease, usually following extensive atrial surgery such as the Fontan operation or atrial repair of transposition of the great arteries. In older children and adults, transvenous pacing is preferred due to lower morbidity of implantation, optimized lead placement, and lower susceptibility to lead fractures as compared with epicardial leads, unless intracardiac shunting is present. In younger children, however, concern about venous occlusion in a patient who will require many decades of pacing often favors placement of epicardial pacemaker leads. The development of epicardial leads that elute a small amount of dexamethasone appears to improve epicardial lead performance,25 although epicardial lead fractures remain a problem that is cause for concern. In patients requiring a lifetime of pacing, various approaches to allow atrial and ventricular pacing are commonly needed. Lower-profile lumenless pacing leads provide an alternative to epicardial pacing in

A P

A P

B V

A P

B V

A S

A S

B V

B V

0.40 ms

A P

B V

B V

A S

A S

B V

A P

B V

A S

B V

A R

B V

A S

B V

A P

A S

B V

A P

V S

341

Disorders of Cardiac Rhythm



CHAPTER 33

A S

B V

A S

B V

A P

B V

B V

A P

B V

A P

B V

• Fig. 33.11  



​Initiation and termination of pacemaker-mediated tachycardia. Top, Two atrial pacing stimuli fail to capture, followed by ventriculoatrial conduction of a ventricular paced beat. The resulting atrial beat (AS) is sensed and triggers another ventricular beat, and the process repeats. Had the retrograde atrial beat occurred during the postventricular atrial refractory period, it would not have triggered a ventricular paced beat. Bottom, Spontaneous ventricular beat inhibits the ventricular pacing, terminating the process. Atrial electrogram tracings are shown at the bottom of each panel. AP, Atrial pace; AR, atrial refractory; BV, biventricular pace; VS, ventricular sense.

smaller patients and appear to perform comparably to more traditional pacing leads over time.25a The recent development of leadless pacemakers represents an exciting prospect for future pacing options. However, in their current design, they are not suitable for younger patients and are available only for single-chamber right ventricular pacing.26 Ongoing advances, including leadless designs for dual-chamber or left ventricular pacing, may prove much more useful in patients with abnormal anatomy requiring dualchamber or biventricular pacing in whom a standard transvenous option is not available.

Other Indications for Pacing Considerable data have shown that a prolonged QRS duration (either due to conduction delay or chronic pacing) results in mechanical dyssynchrony, which further impairs ventricular function. Biventricular pacing, with independent stimulation of the right and left ventricles, can reverse this dyssynchrony and improve ventricular function in some patients. Limited large-scale data are available in the pediatric population on this approach to heart failure treatment (often referred to as cardiac resynchronization therapy), but small studies suggest benefit.27 Certainly, if pacing is otherwise required in a patient with impaired ventricular function, a biventricular pacing system should be considered. Alternatively, with epicardial pacing, placement of the ventricular lead on the left ventricular apex rather than right ventricular apex may achieve similar benefit. However, identifying patients most likely to benefit from this modality and the best technique for optimizing the timing of activation between the ventricles remains unresolved.

Tachycardia Therapies Vagal Maneuvers Vagal maneuvers were once the most commonly used intervention for terminating SVTs. They occasionally terminate VTs as well. Mechanical maneuvers, such as the Valsalva maneuver or carotid sinus massage, usually produce effective vagal stimulation beyond infancy. In infants, a similar reflex vagal response can be elicited sometimes by applying firm, steady abdominal pressure or by applying an ice pack to the face. These maneuvers should be attempted for 15 to 30 seconds. Endotracheal suctioning may also terminate tachycardias by this mechanism.

Acute Pharmacologic Therapies Adenosine

An endogenous nucleoside with profound effects on SA node and AV node conduction, adenosine has become a mainstay in the acute treatment of SVT with normal QRS duration.16 Administered as a rapid bolus, it produces transient but profound depression of AV nodal conduction and should reliably terminate reciprocating tachycardias (AV nodal reentry, AV reentry). Given the prevalence of AV reentry and AV nodal reentry among otherwise healthy young patients, adenosine is often advocated for wide QRS tachycardias as a therapeutic and/or diagnostic maneuver. It usually causes transient AV block without terminating most primary atrial tachycardias but may transiently suppress atrial automatic tachycardias (ectopic atrial or junctional) and occasionally may terminate atrial reentrant tachycardias. Certain VTs may be adenosine sensitive, particularly those originating because of abnormal triggering in the right ventricular outflow tract (RVOT).

342

SECTION IV



Pediatric Critical Care: Cardiovascular

Adenosine (100–300 mg/kg) must be administered rapidly because of rapid metabolism by erythrocytes. If tachycardia is not terminated, determination must be made regarding whether a larger dose is warranted, the dose was given too slowly, or VA or AV conduction was altered without terminating tachycardia (see discussion on diagnosis). Therefore, it is important to record an ECG strip during adenosine administration so that important diagnostic or therapeutic clues are not missed. Because of its brief effect (half-life of 8–10 seconds), tachycardias sometimes immediately reinitiate following successful termination. If they reinitiate, readministration of the same dose should be attempted rather than increasing the dose further. In addition to effects on the AV node, adenosine can produce sinus arrest, which may be prolonged in the setting of intrinsic SA nodal dysfunction after heart transplant; in the presence of drugs that interfere with its metabolism, such as dipyridamole and diazepam; or with drugs that may exaggerate its effects, such as class I, II, or III antiarrhythmic drugs. High doses should not be used indiscriminately in these situations. The use of adenosine in patients with reactive airway disease may be problematic because adenosine occasionally triggers severe bronchospasm. Conversely, its effects may be antagonized by aminophylline and other methylxanthines that the patient may be receiving. Adenosine produces dramatic but transient chest pain, along with systemic vasodilation, both of which tend to increase sympathetic tone. As a result, adenosine may paradoxically accelerate tachycardias if termination is unsuccessful or, in the case of primary atrial tachycardias (atrial flutter or fibrillation), may produce more rapid conduction over the AV node (after initially slowing the ventricular rate). Various secondary arrhythmias may occur following administration of adenosine—particularly, ventricular ectopy, atrial fibrillation, or, rarely, ventricular fibrillation. Although these effects are usually transient, emergency external cardioversion should always be available whenever adenosine is administered. The appropriateness of adenosine has been questioned in patients with known ventricular preexcitation syndromes (WPW syndrome) or suspected VTs. Nevertheless, its thoughtful and careful use remains invaluable for both diagnosis and treatment of many tachycardias. Antiarrhythmic Agents

The addition of pharmacologic agents following adenosine administration should be guided by the clinical situation, known or suspected tachycardia mechanism, and response to adenosine administration. In some cases (such as in patients with wide QRS tachycardia or in hemodynamically compromised patients), it may be most appropriate to proceed directly to pacing termination or cardioversion if adenosine is unsuccessful in restoring sinus rhythm. In other instances, acute antiarrhythmic drug therapy may be warranted.29 The Vaughan Williams classification divides drugs according to their surface ECG effects, which often correlate closely with their cellular electrophysiologic effects: those that block cardiac sodium channels (class I); block b-adrenoreceptors (class II); prolong repolarization (class III); and block calcium channels (class IV). Digoxin and adenosine, which are not included in this classification scheme, exert their primary antiarrhythmic effects on the AV node. Magnesium also has depressant effects on the AV node and suppresses early and late afterdepolarizations (triggered activity). Many of the available drugs manifest properties of more than one class, which contribute collectively to their antiarrhythmic action.30 In general, class I drugs (particularly IA and IC) slow conduction in atrial, ventricular, or accessory pathway tissue and class III

drugs prolong refractoriness in these same tissues. Class IA drugs usually accomplish both effects. b-Adrenergic antagonists, calcium channel antagonists, digoxin, and adenosine act primarily by slowing AV nodal conduction or inhibiting abnormal automaticity. Thus, the latter group of drugs is primarily used for reciprocating tachycardias using the AV node (ART, ORT, AVNRT) or to induce second-degree AV block during a primary atrial tachycardia. In contrast, classes IA, IC, and III drugs may be more effective in terminating or directly suppressing primary atrial tachycardias and may be effective for reciprocating tachycardias.6,29 Despite the various antiarrhythmic agents available for chronic therapy, relatively few are suitable for acute administration to the critically ill patient either because the drugs are not available in IV formulation or they have significant negative inotropic effects when administered IV (Table 33.3). This discussion is limited to agents suitable for acute and short-term parenteral administration. All antiarrhythmic agents have the potential for producing bradycardia, particularly when administered acutely, and most have negative inotropic and/or hypotensive effects. Careful observation is required during initial administration and subsequent infusion of all IV antiarrhythmic agents. Although many are contraindicated in cases of heart failure or hypotension, therapy may be necessary if the arrhythmia is contributing significantly to the patient’s hemodynamic compromise. Procainamide.  Procainamide is useful for various SVTs and VTs in the intensive care setting. Its broad electrophysiologic effects include both conduction slowing and increased refractoriness in atrial tissue, ventricular tissue, and accessory AV connections. Unlike quinidine, procainamide can be administered IV. Effective plasma concentrations (6–10 mg/dL) can be readily achieved with a total loading dose of 15 mg/kg over 15 minutes (or in smallbolus increments at a similar rate). Careful and repeated blood pressure monitoring is required because of potential negative inotropic and direct vasodilator effects. If hypotension complicates infusion, administration should be momentarily interrupted until blood pressure returns to normal. In primary atrial tachycardias, a vagolytic effect may increase the ventricular response over the AV node. Occasionally, atrial tachycardia that is conducting 2:1 to the ventricle slows sufficiently to allow 1:1 conduction, converting a hemodynamically stable rhythm to an unstable rhythm. Thus, one should always be prepared to use cardioversion if necessary. Procainamide, like other class IA and class III drugs, is contraindicated in patients with the congenital or acquired long QT syndromes. Regular monitoring of plasma concentration every 6 to 12 hours is necessary during IV administration to maintain levels between 5 and 10 mg/dL. The active metabolite N-acetylprocainamide contributes to the antiarrhythmic action; higher levels of the parent drug may be necessary in patients lacking the enzyme to produce this metabolite. Lidocaine.  Intravenously administered lidocaine is useful for suppressing and sometimes terminating VTs in children. Although somewhat less likely to acutely terminate VTs than procainamide or amiodarone, lidocaine’s lack of significant negative inotropic effect makes it attractive for this indication. The usual loading dose is 1 to 2 mg/kg acutely or 3 mg/kg over 20 to 30 minutes, followed by a 20 to 50 µg/kg per minute infusion. Lidocaine levels should be monitored to prevent central nervous system (CNS) toxicity. With chronic use (4–7 days), accumulation of the metabolite glycine xylide may impair drug efficacy by interfering with the parent drug effect at the sodium channel. Despite traditional recommendations for its use in ventricular fibrillation, lidocaine increases defibrillation energy requirements.



CHAPTER 33

Disorders of Cardiac Rhythm

343

TABLE Treatment of Bradycardias, Supraventricular Tachycardias, and Ventricular Tachycardias 33.3

Primary Therapies

Secondary Therapies

Long-Term Therapies

Sinus bradycardia

Atropine 0.01 mg/kg Epinephrine 0.1 mg/kg Transcutaneous pacemaker

Temporary pacemaker Isoproterenol infusion

Permanent pacemaker (AAIR, DDDR)

AV block (high grade)

Transcutaneous pacemaker

Temporary pacemaker

Permanent pacemaker (DDDR)

Sinus tachycardia

Identify cause(s)

Sedation, pain control Adjust catecholamines Respiratory support

b-Blockers, if chronic Consider nonsinus mechanism

Paroxysmal supraventricular tachycardia, AV nodal reentrant tachycardia

Vagal maneuvers Adenosine Transesophageal termination Procainamide

Esmolol Verapamil Procainamide (IV) Amiodarone (IV) Class I, class III

b-Blockers, class I, class III Amiodarone Radiofrequency ablation Radiofrequency ablation

AET and other incessant supraventricular tachycardia

Amiodarone Esmolol Avoid cardioversion

b-blockers Amiodarone

Atrial flutter ,24 h

Rate control (diltiazem IV) Procainamide Pace termination with pacemaker DC cardioversion Ibutilide (transesophageal echocardiography if duration unknown of .24 h to rule out thrombus)

Pace termination (Transesophageal, intracardiac)

Radiofrequency ablation Antitachycardia pacemaker

Atrial fibrillation ,24 h

Same as above, except pace termination not feasible)

Chaotic atrial tachycardia

Procainamide, amiodarone

b-Blocker (rate control)

Propafenone, amiodarone

Monomorphic (conscious, stable)

Procainamide/lidocaine DC cardioversion Pace termination if PM, ICD

Procainamide/lidocaine Amiodarone

Defined by substrate

Known heart disease

Same as above

Amiodarone

ICD, radiofrequency ablation Amiodarone

Known idiopathic

Consider IV verapamil Avoid cardioversion

b-Blocker

Calcium channel blocker b-Blocker Radiofrequency ablation

Pulseless (monomorphic, polymorphic)

DC cardioversion b-Blocker Amiodarone

Amiodarone (unless long QT) b-Blocker Magnesium

ICD

Ventricular fibrillation

Defibrillation

Epinephrine Vasopressin

ICD

Bradycardias

Supraventricular Tachycardias

Ventricular Tachycardias

AET, Atrial ectopic tachycardia; AV, atrioventricular; ICD, implantable cardioverter-defibrillator; PM, pacemaker.

b-Blocking agents.  A limited number of b-blocking agents are useful for IV treatment of tachycardias. Acutely, their role is generally limited to incessant tachycardias, which seem to be dependent on sympathetic tone, and VTs related to myocarditis, ischemia/reperfusion injury, or congenital long QT syndromes. In hemodynamically unstable patients, b-blocking agents should be used cautiously because of hypotension and potential sinus bradycardia once tachycardia terminates. All

may produce bronchospasm, hypotension, or bradycardia or may depress ventricular function. Esmolol, a short-acting, nonselective b-blocker with a half-life of 2 to 5 minutes, can be administered as a continuous infusion. A loading dose of 250 to 500 mg over 1 to 2 minutes followed by an infusion of 50 to 300 mg/kg per minute. The infusion can be titrated upward by doubling every 3 to 5 minutes up to 500 mg/kg per minute. Repeat loading doses may be useful as the infusion is

344

SECTION IV



Pediatric Critical Care: Cardiovascular

increased. Its very short half-life is excellent for short-term use, but extended efficacy is limited by tachyphylaxis. For longer-term IV administration, metoprolol (0.05–0.10 mg/kg) is administered by slow IV infusion every 4 to 6 hours, while carefully observing for hypotension (or bradycardia). Amiodarone.  Amiodarone is arguably the single most potent antiarrhythmic drug available both in the acute IV setting and when administered chronically. At sufficient doses, it is often effective in controlling various tachycardias refractory to other antiarrhythmic agents.31 Although typically regarded as a class III agent, its effects are considerably more diverse. It not only prolongs repolarization (by blocking potassium channels) but, to varying degrees, it blocks some sodium channel (class I effect), calcium channel (class IV effect), and b-receptors (class II effect). Amiodarone is administered as a total loading dose of 5 mg/kg divided into 1-mg/kg aliquots given at 5- to 10-minute intervals. The loading can be truncated if arrhythmia control is achieved. If hypotension ensues, volume expansion or calcium chloride (10–30 mg/kg) should be administered. If arrhythmia control is not achieved, a second loading dose can be administered 30 to 60 minutes later. A continuous infusion of 3 to 6 mg/kg/min can be initiated (5–10 mg/day). Other QT Prolonging (Class III) Antiarrhythmic Drugs Intravenous sotalol.  Sotalol is a class III (i.e., QT prolonging)

antiarrhythmic drug with significant b-blocking properties used extensively in its oral form for a variety of supraventricular and ventricular arrhythmias. It is now available in IV form and has been used effectively in children for termination of a variety of atrial and ventricular arrhythmias.32,33 It can also be used to continue maintenance antiarrhythmic therapy in patients unable to take the oral form. It is infused as a 1-mg/kg bolus (maximum 80 mg) over 1 hour, closely monitoring blood pressure and rhythm. The dose can be repeated if termination does not result with the first bolus. It should be noted that mean times to termination have varied widely, from 33 minutes to up to 12 hours. It is important to ensure normal serum levels of potassium and magnesium before administration. Hypotension is the main acute side effect, and it should be used in caution in the setting of depressed ventricular function. Ibutilide.  Ibutilide is a class III antiarrhythmic drug available only intravenously due to extensive first-pass metabolism of the oral form. It is cleared rapidly (elimination time 3–6 hours), and pharmacokinetics are not altered by age, sex, or hepatic or renal dysfunction.34 Ibutilide is used for termination of atrial fibrillation or atrial flutter, with acute success rates as high as 76%.35 In children and patients with congenital heart disease, ibutilide has been reported to successfully terminate atrial flutter or fibrillation in 71%, with rare episodes of torsades de pointes and nonsustained ventricular tachycardia.36 Prior to administration, serum potassium and magnesium should be checked and repleted if low, and preexisting QT prolongation is a relative contraindication. The standard dose is 0.01 mg/kg for patients less than 60 kg and 1 mg for patients heavier than 60 kg infused over 10 minutes, with a repeat dose if necessary. Calcium Channel–Blocking Agents

Verapamil and diltiazem have proved useful for terminating SVT involving the AV node (AV reentry, AV nodal reentry). However, their acute efficacy is no greater than that of adenosine, and both may cause hypotension or cardiovascular collapse in young infants or patients with poor ventricular function.37 Either drug can be

useful as an alternative to adenosine when tachycardias have repeatedly reinitiated following termination with adenosine. Both agents may help slow the ventricular response over the AV node during atrial flutter or fibrillation. Verapamil is administered as a bolus of 0.15 mg/kg. Diltiazem can be administered as a bolus of 0.15 to 0.35 mg/kg and can be infused continuously at 0.05 to 0.2 mg/kg per hour if ongoing effect is necessary. In addition to vasodilation and negative inotropic effects, both can accelerate antegrade conduction over accessory pathways in patients with WPW syndrome. Therefore, they are contraindicated for preexcited atrial fibrillation and, generally, they should not be administered during uncharacterized wide QRS tachycardias. Likewise, oral calcium channel blockers generally should not be used as maintenance therapy for patients with WPW syndrome. If hemodynamic compromise develops, IV calcium gluconate should be administered immediately. Magnesium Sulfate

Magnesium (administered as 25–50 mg/kg magnesium sulfate) has proved useful in the treatment of certain ventricular and supraventricular arrhythmias. Its actions appear to be mediated through depression of early and late afterdepolarizations; depressant effects on AV nodal conduction; and, at high doses, indirect inhibition of sodium-potassium adenosine triphosphatase. It is most effective in the acute treatment of torsades de pointes and as a temporizing measure in the treatment of arrhythmias associated with digoxin toxicity.38 Therapeutic efficacy is not restricted to situations in which hypomagnesemia is present. Although magnesium has efficacy comparable with that of adenosine in the acute termination of SVT resulting from AV reentry and AV nodal reentry, it has more severe and lasting adverse effects. It has little demonstrable effect in the acute treatment of monomorphic VTs or polymorphic VTs not associated with QT prolongation.

Digoxin

Digoxin has been used for various supraventricular arrhythmias, including AV reentry, AV nodal reentry, and primary atrial tachycardias. In a randomized controlled trial of infants younger than 4 months with SVT, recurrence rates were similar between digoxin and propranolol.39 Digoxin may increase the risk of rapid antegrade conduction during atrial fibrillation in older patients and possibly infants with preexcitation. Like calcium channel– blocking agents, it should be avoided altogether in the treatment of patients with ventricular preexcitation (WPW syndrome). Its use is further confounded by potentially dangerous interactions with other medications, including quinidine, verapamil, amiodarone, flecainide, phenytoin, and warfarin. At toxic dosages, its direct cellular effects may predispose to dangerous tachycardias and bradycardias.

Dexmedetomidine

Dexmedetomidine is a selective a2-adrenergic receptor agonist that provides sedation, anxiolysis, and analgesia with minimal to no respiratory depression. As a result, it has become widely used in a variety of settings, including the pediatric ICU. Dexmedetomidine acts as a peripheral parasympathomimetic and a central sympatholytic, with electrophysiologic effects, including sinus and atrioventricular node depression.40 Studies have suggested that dexmedetomidine is effective for acute termination of SVT in children, with fewer side effects compared with adenosine.41 Serious adverse events—including sudden pauses, asystole, and

loss of pacemaker capture—have also been reported with dexmedetomidine infusion.42,43 Ivabradine

Ivabradine is a novel antiarrhythmic drug that works by inhibiting cardiac pacemaker current (If) in normal sinus node as well as in AV node and bundle of His.44 Unlike b-blockers, it exerts its negative chronotropic action independent of the neurohumeral system. Its most common usage is in the treatment of inappropriate sinus tachycardia. However, its use has been reported in treatment of both congenital and postoperative junctional ectopic tachycardias.45,46 Adverse events have not been reported in this setting.

Cardioversion and Defibrillation Cardiovascular collapse or failure of mechanical and pharmacologic interventions for tachycardias may warrant cardioversion. For tachycardias with discrete QRS complexes, synchronization with the QRS should be confirmed (the default mode for most defibrillators is nonsynchronized, and most revert to nonsynchronized shocks after each shock is delivered). Proper synchronization may require changing the ECG lead configuration to achieve an upright QRS complex. Several factors may determine the success of cardioversion and defibrillation. Energy requirements may vary from 0.25 to 1 J/kg for SVTs to greater than 2 J/kg for VTs. Newer defibrillators with a biphasic rather than monophasic waveform have reduced defibrillation energy requirements. Electrode (paddle) location is an important variable. If conversion is not achieved with low- or moderate-energy levels, consideration should be given to changing electrode position before using higher-energy levels. Automatic tachycardias are characteristically refractory to cardioversion and may account for treatment failure. Finally, some antiarrhythmic drugs, particularly sodium channel–blocking drugs (see later discussion), increase defibrillation energy requirements and pacing thresholds, whereas other drugs (QT-prolonging drugs) appear to have a favorable effect.47

Approach to Therapy Extrasystoles In general, isolated extrasystoles do not require treatment unless they are sufficiently frequent to impair hemodynamics or they serve as frequent initiating events for tachycardias. In otherwise healthy children and adolescents, extrasystoles are a benign finding. In other settings, complex ventricular extrasystoles may identify patients at increased risk for cardiac arrest. Even in such situations, prophylaxis may not decrease and may actually increase the risk. Effort should instead be directed at identifying possible causes and correcting any predisposing factors, which include ischemia, electrolyte disorders, acidosis, pericarditis, or direct trauma from recent cardiac surgery, blunt or penetrating chest trauma, and intracardiac catheter-induced irritation. Numerous drugs—including digoxin, catecholamines, or drugs associated with the acquired long QT syndrome—may produce extrasystoles.

Sustained Tachycardias Most sustained tachycardias observed in the intensive care setting warrant immediate attention and intervention. Sinus tachycardia may indicate the need for additional sedation and analgesia or may reflect hemodynamic compromise as a consequence of



CHAPTER 33

Disorders of Cardiac Rhythm

345

anemia, hypovolemia, or impaired myocardial function. Sinus tachycardia as a consequence of hyperthermia may be poorly tolerated in children already critically ill, especially following cardiac surgery. Sinus tachycardia may reflect an underlying neuroendocrine process, such as hyperthyroidism or pheochromocytoma, requiring acute medical intervention (b-blocker) while instituting therapy for the underlying disorder. Nonsinus tachycardias in patients with primary rhythm disturbances may warrant therapy to prevent life-threatening events, prevent the development of myocardial dysfunction as a consequence of chronic (incessant) tachycardia, or simply alleviate acute tachycardia-related symptoms. The acuity of the situation dictates the approach to therapy. Tachycardias occurring secondary to other abnormalities (structural heart disease, metabolic derangements, drug toxicity) should always be regarded as high risk for serious hemodynamic deterioration.

Unstable Patients The approach to patients with tachycardia is determined largely by the degree of hemodynamic compromise (see Table 33.3). Patients who are hemodynamically unstable or in cardiovascular collapse resulting from sustained tachycardia almost always warrant prompt cardioversion or defibrillation. Antiarrhythmic medications (and other supportive measures) should replace cardioversion in the unstable patient only when tachycardia is known to be incessant or is unresponsive to cardioversion (e.g., JET, atrial ectopic tachycardia, chaotic tachycardia, and PJRT). Cardiopulmonary resuscitation should always be instituted in the absence of a pulse or blood pressure, as is typically the case for polymorphic VT and ventricular fibrillation. Although hemodynamic and ventilatory support should be initiated immediately and maintained following tachycardia termination as needed, cardioversion (with bag and mask ventilation initially) should take precedence over other interventions.48 Underlying factors contributing to the tachycardia should be sought, including hypoxia, infection (cardiac or systemic), drug toxicities (see later discussion), and electrolyte derangements. Once tachycardia is terminated, acute therapies may focus on either suppressing recurrences or terminating them when they recur. Although antiarrhythmic medications eventually may be necessary to suppress recurrences, most have negative inotropic or vasodilating effects, particularly when administered intravenously. Often, it is preferable to delay specific therapy after initial termination until ventricular function improves. Most recurrences of SVTs can be safely treated with temporary pacing or adenosine rather than with repeated cardioversions. In the event of frequent recurrences, a transesophageal catheter may be left in place for this purpose, or a transvenous atrial pacing catheter may be warranted in selected patients. Similarly, temporary ventricular pacing may be useful in some circumstances for recurrent VT. Although adenosine can be administered repeatedly because of its short halflife, the resulting vasodilation may be poorly tolerated in patients with tachycardia mechanisms unresponsive to adenosine.

Treatment Failure When seemingly appropriate electrical and pharmacologic interventions fail to terminate tachycardias, three possibilities should be considered: erroneous diagnosis, unrecognized tachycardia termination and reinitiation, or a technical error in the termination technique.

346

SECTION IV



Pediatric Critical Care: Cardiovascular

Errors in Diagnosis As noted earlier, automatic atrial tachycardias, JET, and occasionally chaotic atrial tachycardia might be mistaken for tachycardias with reentrant mechanism (ORT, AVNRT, and primary atrial reentry). Each of these conditions is usually refractory to electrical termination (either pace termination or cardioversion), yet the diagnosis may be subtle if atrial activity is obscured. Confusion between VTs and SVTs with prolonged QRS probably remains the most frequent diagnostic error. Occasionally, the presumption of VT may lead to ineffective treatments. For example, following cardiac surgery, incessant, monomorphic wide QRS tachycardia that is refractory to cardioversion may actually be JET with postsurgical bundle branch block. Brief atrial pacing at a faster rate may be necessary to confirm the diagnosis. Certain tachycardias, such as torsades de pointes related to long QT syndromes (congenital or drug-induced) or bidirectional VT resulting from digoxin toxicity, must be recognized to provide more appropriate and specific therapies to prevent recurrences following cardioversion. Unrecognized Termination and Reinitiation Unrecognized reinitiation may occur following medical termination, pace termination, or cardioversion. In some tachycardias (as in PJRT or other incessant forms of SVT), reinitiation is expected. However, it also may occur inadvertently as the result of continued pacing beyond the point of termination or may be facilitated by sinus pauses, junctional beats, or ectopic beats following adenosine or cardioversion. Again, measures to decrease the factors favoring reinitiation (e.g., shorter pacing bursts, antibradycardia pacing, and coadministration of an antiarrhythmic drug) should be used rather than further increases in energy or dose of the terminating therapy. With frequent terminations and reinitiation of tachycardia, multiple repeated cardioversions are likely ineffective and may cause myocardial injury. Improper Technique Appropriate administration of adenosine is accomplished through an IV catheter (peripheral or central) by rapid push followed immediately with ample flush. Because of rapid metabolism by erythrocytes, arterial administration may be ineffective in terminating tachycardia (yet still produce vasodilation). Errors in cardioversion or pacing technique are generally attributable to insufficient energy or improper electrode (or paddle) placement. For pace termination, the stimulator must be capable of sufficient output for the pacing modality being used (see section on temporary pacing). ICU personnel should be familiar with the defibrillation devices available in the ICU, including adjustment of electrocardiographic gain and lead selection to allow synchronous cardioversion when appropriate. However, in ventricular fibrillation or polymorphic tachycardia, asynchronous countershock is necessary. Use of excessive energy may damage the myocardium and, when repeated, may lead to preterminal bradycardias, which are refractory to all pacing modalities and progress to complete electromechanical dissociation if hypoxia and acidosis are not corrected.

Specific Arrhythmias Primary Arrhythmias Some arrhythmias require unique therapeutic approaches or are seen with sufficient frequency to warrant a brief review (Box 33.1).

• BOX 33.1 Rhythm Disturbances Primary Rhythm Disturbances Paroxysmal supraventricular tachycardias (atrioventricular nodal reentrant tachycardia) • Congenital AV block • Congenital long QT syndrome, Brugada syndrome • Other genetic arrhythmias • Ventricular tachycardias resulting from Purkinje hamartoma • Verapamil-sensitive ventricular tachycardias • Accelerated ventricular rhythm











Secondary Rhythm Disturbances Early Postoperative Arrhythmias JET • Postsurgical AV block • Early primary atrial tachycardia



Late Postoperative Arrhythmias Ventricular arrhythmias (postoperative tetralogy of Fallot) • Sick sinus syndrome

Metabolic Derangements Electrolyte disturbances • Endocrine derangements (thyroid) • CNS injury • Hypothermia, hyperthermia • Acute hypoxia (newborns) • Acute myocardial infarction









Drug Toxicity, Proarrhythmia Digoxin Cocaine Tricyclic antidepressants Antiarrhythmic drugs Quinidine/sotalol Flecainide/encainide Organophosphates

Infectious Lyme disease • Myocarditis, endocarditis

AV, Atrioventricular; CNS, central nervous system; JET, junctional ectopic tachycardia.

Orthodromic Reciprocating Tachycardia in Infancy Infants with ORT can present with tachycardia in utero, at birth, or within the first weeks to months of life. Postnatally, tachycardia sustained beyond a few hours may result in CHF that may progress to shock, acidosis, and complete cardiovascular collapse.49 In the latter situation, ORT may be terminated during resuscitation efforts such that its causative role remains unrecognized. Thus, SVT should be considered in the differential diagnosis of neonatal shock, along with other conditions such as sepsis, aortic coarctation, and congenital adrenal hyperplasia, in which sinus tachycardia associated with cardiovascular collapse would be expected. Initial conversion can typically be achieved with either vagal stimulation such as ice-to-face, IV adenosine or, in dire circumstances, DC cardioversion. With successful conversion, initial oral therapy with digoxin or a b-blocker appear similarly effective in infants.39 However, when initial conversion is followed by immediate reinitiation, repetitive doses of adenosine tend to promote incessant tachycardia. IV agents useful for initial control of incessant tachycardia include esmolol, procainamide, sotalol, or amiodarone. Second-line oral agents include sotalol, flecainide, propafenone, and amiodarone.



CHAPTER 33

Disorders of Cardiac Rhythm

I

aVR

V1

V4

V3R

II

aVL

V2

V5

V4R

III

aVF

V3

V6

V7

347

V4R

• Fig. 33.12





​Chaotic atrial tachycardia. Note two discrete P-wave morphologies before conversion to sinus rhythm.

Tachycardia-Induced Cardiac Dysfunction Although most SVTs are paroxysmal or episodic, chronic SVTs pose a unique problem. Many are minimally symptomatic and are recognized only by the inappropriately fast rate. However, with time, varying degrees of CHF become evident and ventricular dysfunction may be severe. Even then, the diagnosis may not be immediately evident. Consequently, chronic tachycardia must be considered in any patient presenting with gradually progressive CHF. In one series, chronic atrial tachycardia was present in 37% of patients initially diagnosed with idiopathic cardiomyopathy and listed for heart transplantation.50 In patients with structurally normal hearts, the most common incessant SVTs are PJRT and ectopic atrial tachycardia. These conditions may occur throughout infancy, childhood, and adolescence. The rates (often ,200 beats/min) and normal PR interval during tachycardia may lead to an erroneous diagnosis of sinus tachycardia secondary to the hemodynamic compromise (see earlier section, Approach to Diagnosis). An abnormal P-wave axis on 12-lead ECG and Holter monitoring to look for interruptions in the tachycardia with changes in P-wave morphology are helpful. Electrophysiologic study may still be necessary to establish the diagnosis. In infants, incessant VTs and the rare congenital form of JET also are seen. In each of these entities, it is important first to recognize the primary role of the tachycardia in producing secondary congestive symptoms and to recognize the futility of acute therapies such as adenosine, pace termination, and cardioversion. Most are catecholamine dependent so that inotropic agents may aggravate the situation and compromise the efficacy of antiarrhythmic regimens, whereas b-blocking agents may be useful despite the presence of heart failure. Once the diagnosis is established, chronic antiarrhythmic therapy is instituted to control or limit the tachycardia. Uncontrolled, severe cardiac symptoms may result, but ventricular dysfunction improves once tachycardia is suppressed

medically or treated by catheter ablation.51 Despite the severity of heart failure, antiarrhythmic medications that depress ventricular function are usually well tolerated.

Chaotic Atrial Tachycardia Chaotic atrial tachycardia is a primary atrial tachycardia characterized by three or more different P-wave morphologies and irregular, rapid atrial rates (Fig. 33.12). Although atrial flutter may be associated with it, episodes are usually self-limited and cardioversion is neither indicated nor effective. Asymptomatic patients with slow or intermittent tachycardia may not require treatment. Occasionally, digoxin is used to limit AV conduction when atrial rates are excessive or to enhance contractility in the setting of tachycardia-induced cardiomyopathy.52 An association with respiratory syncytial virus has been described in some patients.12 Various agents have been used in symptomatic cases; amiodarone and propafenone are the most effective.52,53 Long QT Syndromes The long QT syndromes (LQTSs) are a diverse group of disorders, both congenital and acquired, in which individuals are at risk for torsades de pointes and sudden death because of abnormalities in ventricular repolarization. In both congenital and acquired forms, the rate-corrected QT intervals usually exceed 0.46 second and, more typically, are greater than 0.48 to 0.50 second. Associated anomalies of T-wave morphology—including T-wave alternans, bifid T waves, and prominent U waves—are common and T-wave morphology may offer clues to genotype in congenital LQTSs. It can sometimes be difficult to establish the diagnosis of congenital long QT syndrome because QT prolongation may sometimes be modest in affected individuals and there may be considerable overlap in QTc ranges with the normal, healthy population.54 However, congenital LQTS should be strongly considered in all patients with any degree of QT prolongation and a history of

348

SECTION IV



Pediatric Critical Care: Cardiovascular

• Fig. 33.13

​Congenital long QT syndrome presenting with 2:1 atrioventricular block. The corrected QT interval is greater than 560 ms, resulting in functional block of every other sinus beat in the His-Purkinje system.  



• Fig. 33.14

sustained, and even prolonged episodes may terminate spontaneously. Torsades de pointes that degenerates to ventricular fibrillation requires defibrillation. Because the adrenergic response to defibrillation may trigger recurrent arrhythmias, defibrillation should be performed in an unconscious or sedated patient. Treatment of immediate recurrence of torsades de pointes is challenging and includes magnesium sulfate, increasing the heart rate with temporary pacing or isoproterenol, and sedation.56 For most acquired long QT syndromes (and some congenital forms), increasing the heart rate using isoproterenol or by pacing shortens the QT interval, though isoproterenol may exacerbate some forms of the congenital long QT syndrome.57 Therefore, isoproterenol should be used only when there is underlying bradycardia and cardiac pacing cannot be started immediately. Correcting hypokalemia, hypomagnesemia, or hypocalcemia and removing potentially causative agents may be important in the ICU setting.

syncope, polymorphic VT, cardiac arrest, or a family history of unexplained sudden death. The diagnosis should also be considered in any child presenting with unexplained seizures, particularly if experienced during physical exertion or other conditions of adrenergic stress. Conversely, incidental QT prolongation in an asymptomatic child (with a negative family history) warrants further scrutiny and evaluation for other factors contributing to QT prolongation. Finally, infants with LQTS may present with functional 2:1 AV block due to prolonged cardiac repolarization (Fig. 33.13).4 Patients with symptoms or arrhythmias associated with QT prolongation require careful evaluation for secondary causes, which include CNS injury, hypocalcemia, hypokalemia, and drugs that prolong the QT interval. The list of drugs that prolong the QT interval is extensive and includes antiarrhythmic and noncardiac drugs, most of which block IKr, the rapid component of the delayed rectifier potassium current.55 Continually updated lists of these drugs are available at www.crediblemeds.org. Torsade de pointes is the specific arrhythmia associated with long QT syndromes and is responsible for the symptoms (Fig. 33.14). This characteristic arrhythmia is recognized by progressive undulation in the QRS axis, resulting in a twisting appearance, and is usually associated with a specific initiation with a VPB following a pause (often following a previous VPB). Many episodes are not

Idiopathic Ventricular Tachycardias in Healthy Patients Accelerated ventricular rhythm is observed occasionally in neonates in the first few days of life at rates only slightly faster than the appropriate sinus rates. The rhythm competes with the sinus mechanism, and alternation between sinus and ventricular rhythm with fusion beats is common. The rhythm is self-limited, does not usually result in hemodynamic compromise, and carries a good prognosis. No specific therapy is necessary unless rates are excessive.58 A similar rhythm is seen in older children and usually has a similarly benign course.59 Two other characteristic VTs may be seen in otherwise healthy children and adolescents. One arises from the right or left ventricular outflow tracts, resulting in a left bundle branch block pattern with inferior QRS axis. This pattern of arrhythmia often displays incessant, self-limited salvos, in which case the term repetitive monomorphic VT has been used. A more paroxysmal pattern is seen less often. Patients are often asymptomatic, though palpitations, chest pain, and occasionally syncope can result. Treatment is dictated largely by symptoms. The other idiopathic VT arises from the posterior fascicle of the left-sided conduction system, producing a right bundle branch block pattern with leftward QRS axis (Fig. 33.15). This tachycardia has been called fascicular or verapamil-sensitive VT. Interestingly, the acute response to verapamil in the past sometimes resulted in the misclassification as SVT with aberrant conduction. Clinically, this arrhythmia can be managed much like SVT in that life-threatening decompensation or progression to ventricular fibrillation is uncommon.

​Torsades des pointes in a teenage patient with long QT syndrome. This arrhythmia is associated with no pulse and results in syncope. It often terminates spontaneously but otherwise rapidly degenerates to ventricular fibrillation.  





CHAPTER 33

Disorders of Cardiac Rhythm

I

aVR

V1

V4

II

aVL

V2

V5

III

aVF

V3

V6

349

• Fig. 33.15  



​Idiopathic ventricular tachycardia in an otherwise healthy 12-year-old. Note right bundle branch block pattern with superior axis, typical of origin within the left posterior fascicle region of the left ventricular septum.

Although the mechanisms underlying outflow tract VTs (triggered activity) and posterior fascicular VT (micro-reentry) are dissimilar, both may respond acutely and chronically to calcium channel blockers. Chronic therapies can include b-blockers or (for RVOT) flecainide. However, ultimately, catheter ablation is usually the treatment of choice in symptomatic individuals. Ventricular tachycardias not displaying the characteristic features of outflow tract VT or posterior fascicular VT should prompt an aggressive search for occult heart conditions, including cardiomyopathies, myocarditis, cardiac tumors (rhabdomyomas, fibromas, hamartomas) or arrhythmogenic right ventricular cardiomyopathy.60 Evaluation should include magnetic resonance imaging with contrast and, often, electrophysiology study.

Bidirectional Polymorphic Ventricular Tachycardias CPVT is an inherited condition that bears some similarity to the long QT syndrome. Individuals may present with syncope or cardiac arrest during exertion or emotional stress. However, at baseline, the ECG may be entirely normal and the condition may become evident only when challenged with exercise or catecholamine infusion. The characteristic arrhythmia is bidirectional VT, in which there is an alternating QRS axis during salvos of VT. Often, patients are entirely asymptomatic during sometimes long, incessant runs of bidirectional VT, and acute therapies, particularly cardioversion, are not warranted. Patients may also experience concurrent atrial arrhythmias. Defects in multiple genes that alter calcium release from the sarcoplasmic reticulum can underlie CPVT, but most patients

carry mutations in the cardiac ryanodine receptor (RYR2). Once the diagnosis has been established, chronic therapies generally include a b-blocker, sometimes in combination with flecainide. Cervical sympathetic denervation may also enhance chronic arrhythmia suppression.

Secondary Rhythm Disturbances Certain arrhythmias characteristically follow operative treatment of CHD. Among those observed in the early postoperative period are complete heart block, JET, and primary atrial tachycardias. Late postoperative arrhythmias include ventricular arrhythmias following tetralogy of Fallot repair and atrial arrhythmias following the Mustard/Senning and Fontan procedures.

Postoperative Arrhythmias Postsurgical Atrioventricular Block

Inadvertent damage to the AV conduction system may occur with cardiac surgery, especially after closure of ventricular septal defects (particularly associated with l-loop ventricles), during resection of septal tissue, or after insertion of prosthetic valves in the tricuspid, aortic, or mitral position. Bradycardia from AV block can be initially managed using isoproterenol to accelerate the ventricular rate or with epicardial temporary pacing wires placed at surgery. Although temporary pacing frequently is necessary for rate support, permanent pacemaker implantation should usually be delayed 7 to 14 days to allow for potential recovery of AV conduction. Most patients who recover AV conduction do so within 9 days of surgery.61

350

SECTION IV



Pediatric Critical Care: Cardiovascular

Junctional Ectopic Tachycardia

JET immediately following cardiac surgery may be mistaken for third-degree AV block; however, on rewarming, ventricular rates approach or exceed 200 beats/min. Atrial wires or esophageal electrography confirms the key diagnostic features: AV dissociation with normal QRS and the regular ventricular rate faster than the atrial rate. Appropriately timed atrial systoles conducted to the ventricle result in advancement of the tachycardia (without a change in QRS morphology or subsequent pause). If the QRS is normal but the RR interval does not shorten with appropriately timed atrial systoles, JET with retrograde (VA) conduction or third-degree AV block with JET as the escape rhythm should be suspected. Infants with JET usually are often severely ill, and b-adrenergic agonists, fever, and endogenous catecholamines accelerate the tachycardia. Initial treatment of postoperative JET includes sedation and analgesia, withdrawal of adrenergic stimulants (to the extent possible), and cooling. The tachycardia may be suppressed by temporary overdrive (atrial) pacing, by pacing at a rate sufficient to produce 2:1 AV block, or by AVT mode pacing to provide AV synchrony.62 A variety of medications have been used for postoperative JET, including procainamide, lidocaine, dexmedetomidine, and, recently, ivabradine. IV amiodarone has been used with perhaps the greatest efficacy.63 Although emergency radiofrequency ablation has been performed in rare instances, aggressive temporizing measures, including extracorporeal membrane oxygenation, appear warranted given the transient nature of this arrhythmia.

Late Postoperative Arrhythmias Atrial tachycardia and bradycardia are common late sequelae following the Senning and Mustard operations for d-transposition of the great arteries, atrial septal defect closure, and the Fontan procedure for tricuspid atresia and single ventricle.15 Patients with these arrhythmias appear to be at increased risk for sudden death, although whether death is the result of atrial tachycardia itself, associated bradycardia, degeneration to VT, or even nondysrhythmic events remains unclear. Likewise, in patients with repaired or palliated CHD, ventricular arrhythmias that are associated with risk for sudden death may develop. There seems to be little justification for empirical drug therapy to suppress asymptomatic ventricular arrhythmias in these patients. Earlier repair is believed to decrease the incidence of serious problems. Still, the relative contributions of postoperative hemodynamic abnormalities, natural history of the unrepaired lesions, and surgical technique to the development of late arrhythmias remain uncertain. Similarly, the roles of pacemaker/defibrillator therapy, prospective electrophysiologic study, and antiarrhythmic drug testing are not well established. Because both bradycardias and tachycardias develop in many patients, the correlation of symptoms with electrophysiologic abnormality is important in guiding therapy. Metabolic Derangements Electrolyte Disturbances

Hyperkalemia causes characteristically tall (peaked or tented) T waves with a narrow base with progressive changes at higher concentrations, including decreased P-wave amplitude, QRS prolongation, SA nodal and AV nodal block, and, ultimately, ventricular fibrillation. Mild to moderate hypokalemia may cause prominent U waves, diminished T-wave amplitude, T-wave inversion, and fusion of the T wave and U wave, along with increased spontaneous ventricular ectopy and inducible ventricular arrhythmias. Arrhythmias caused by hypokalemia are potentiated by catecholamines, and hypokalemia itself potentiates the toxic effects of digoxin and the proarrhythmic effects of drugs associated

with drug-induced long QT syndrome.64 Severe hypokalemia is associated with ventricular fibrillation. Hypercalcemia produces T-wave inversion and shortens the QT interval. Hypocalcemia prolongs the time to the peak of the T wave but not the QT interval itself. Isolated calcium abnormalities are uncommon, and arrhythmias caused by such abnormalities are rare, although hypercalcemia may aggravate digitalis toxicity.2 Endocrine Disorders (Thyroid)

Hyperthyroidism exerts both sympathetic-like and direct cardiovascular actions that produce sinus tachycardia and atrial fibrillation, but ventricular arrhythmias are uncommon. These arrhythmias respond to b-blockers and resolve when the euthyroid state is restored. Combination treatment with digoxin potentiates AV nodal block while minimizing negative inotropic effects. Hypothyroidism causes sinus bradycardia and AV conduction disturbances; QT interval prolongation is common but rarely associated with torsades de pointes.65

Central Nervous System Injury

The most common ECG change associated with CNS trauma and increased intracranial pressure is sinus bradycardia, usually with associated hypertension. These bradycardias appear to be vagally mediated and usually respond to atropine. However, potentially serious arrhythmias may occur within 24 hours following blunt trauma to the head, subdural hematoma, and subarachnoid hemorrhage. QT-interval prolongation is common and, in combination with bradycardia and hypokalemia, may provoke torsades de pointes.

Hypothermia and Hyperthermia

Mild hypothermia can cause a range of reversible ECG changes, including sinus bradycardia; prolongation of the PR, QRS, and QT intervals; and a characteristic secondary deflection on the terminal portion of the QRS (Osborn wave).66 Severe hypothermia may cause more significant bradycardias, including AV block and asystole or VTs and ventricular fibrillation. Therapeutic hypothermia is associated with QT prolongation, and torsades de pointes has been reported.67 In contrast, hyperthermia causes sinus tachycardia and may enhance other tachycardias, such as PSVT, ectopic atrial arrhythmias, and especially JET in susceptible patients.

Acute Myocardial Infarction

Acute myocardial infarction is uncommon in young patients but may occur in cases of anomalous origin of the left coronary artery, when there is perinatal stress, following Kawasaki disease, with blunt chest wall trauma, and following cardiac transplantation and the arterial switch procedure. It can occur after air embolism in cyanotic CHD or after open-heart operations. The diagnosis may be overlooked in infants and children because of the inconsistency of symptoms and relatively poor (60%) clinical recognition by electrocardiography.68 Nevertheless, acute infarction may result in various rhythm disturbances, including sinus bradycardia (as a result of the Bezold-Jarisch reflex), AV conduction disturbances, intraventricular block, and asystole.

Arrhythmias Resulting From Drug Toxicity Digoxin

Digoxin toxicity may cause various arrhythmias and should be suspected in any patient in whom a new arrhythmia develops during digoxin therapy. Likewise, digoxin ingestion should be considered in patients with acute arrhythmias, particularly those associated

494 544

611 516 477 444 444 455 444 450 444 411 1538

455

• Fig. 33.16  



​Bidirectional ventricular tachycardia. This unusual arrhythmia is seen only in patients with digoxin toxicity and two rare genetic arrhythmia syndromes: Andersen-Tawil syndrome (periodic paralysis and ventricular arrhythmias) and catecholaminergic polymorphic ventricular tachycardia. The patient is asymptomatic during this arrhythmia, but patients appear to be at risk for ventricular fibrillation.

with CNS and gastrointestinal symptoms. Accelerated junctional rhythm may be the first arrhythmia seen. Progressive AV block is common. Sinus bradycardia resulting from either SA node exit block or sinus arrest may occur, as can atrial fibrillation (but usually not atrial flutter). Ectopic atrial arrhythmias may occur. Nearly any ventricular arrhythmia may occur, including multiform ventricular extrasystoles, bigeminy, VT (particularly bidirectional VT, otherwise only seen in rare genetic arrhythmia syndromes; Fig. 33.16), and ventricular fibrillation.69 In general, digoxin concentrations less than 2 ng/mL are considered nontoxic. Neonates usually tolerate levels as high as 3.5 ng/mL. Nevertheless, neonates and other ICU patients may be more susceptible to digoxin toxicity because of renal dysfunction, electrolyte imbalances, and hypoxia. Hypokalemia, excessive calcium infusions, and rapid sinus rates exacerbate digitalisrelated arrhythmias. Purified digoxin-specific Fab antibody fragment, which binds the drug and is eliminated in the urine, is used to treat digoxin toxicity. Prophylactic treatment with this preparation should be gauged according to the quantity ingested, time since ingestion, and serum digoxin level. Magnesium sulfate is a useful temporizing treatment while specific antibody treatment is being implemented. Cardioversion should be reserved for life-threatening tachycardias or those unresponsive to these therapies. Cocaine

Life-threatening ventricular arrhythmias, cardiac arrest, and myocardial infarction can occur in healthy individuals with normal coronary arteries following cocaine ingestion and in prenatally exposed neonates.70,71 Cocaine produces myocardial ischemia and infarction by inducing severe local coronary vasoconstriction, increasing myocardial-metabolic demand through its potent chronotropic effects, and increasing afterload. In infarct models, cocaine directly potentiates arrhythmias induced by catecholamines.72 These factors favor the use of b-adrenergic antagonists as first-line treatment for cocaine-related arrhythmias. Additionally, cocaine blocks fast inward sodium channels, similar to class I antiarrhythmic agents.73 QT prolongation and torsades de pointes have been observed. Tricyclic Antidepressants and Phenothiazine

Phenothiazines and tricyclic antidepressants produce electrophysiologic (and potentially antiarrhythmic) effects similar to quinidine and procainamide. They slow conduction velocity in atrial and ventricular tissue, prolong repolarization, and exert anticholinergic effects, accounting for the observed ECG changes of conduction disturbances, prolonged QT intervals and QRS



CHAPTER 33

Disorders of Cardiac Rhythm

351

duration, and various tachycardias and bradycardias.74 Sinus tachycardia, atrial tachycardias and VTs, and AV conduction disturbances distal to the AV node occur occasionally during normal therapeutic administration and may reflect individual susceptibility to QT-prolonging agents. Arrhythmias commonly follow intentional overdose, resulting in hypotension (due to a-blocking effects), severe anticholinergic effects (neuromuscular and mucosal), seizures, and coma. Quinidine and procainamide are contraindicated for tachycardias because of these agents. In patients manifesting early cardiotoxicity, arrhythmias may develop 3 to 7 days following ingestion, apparently because of release of tissue stores. Therefore, ECG monitoring should be continued for at least 24 to 48 hours after apparent ECG and rhythm normalization and longer if severe arrhythmias are observed.

Infections Myocarditis may cause atrial tachycardias and VTs or acquired heart block. Lyme disease may produce high-grade acute AV block. Although AV conduction usually normalizes with appropriate antibiotic therapy, temporary pacing may be required.75 Antibiotic treatment should be instituted on the basis of the history and electrocardiographic findings alone while awaiting confirmatory serology. Bacterial endocarditis can cause AV conduction disturbances, particularly when the aortic valve is involved. Unstable or persisting conduction abnormalities (.7 days) carry a high risk of mortality (43%–80%) and are indications for early valve replacement.76 Myocarditis may be responsible for some cases of VT in otherwise healthy individuals and may range from chronic ventricular ectopy or tachycardia to fulminant and refractory arrhythmias leading to electromechanical dissociation. Chaotic atrial tachycardia may occur in the setting of infection with respiratory syncytial virus, although the cause of this association is unclear. Finally, paroxysmal tachycardias of any etiology may be exacerbated by acute infections that cause fever, dehydration, and increased sympathetic tone. Short-term modifications of chronic therapy may be necessary, particularly when oral administration becomes impractical.

Key References Gikonyo BM, Dunnigan A, Benson Jr DW. Cardiovascular collapse in infants: association with paroxysmal atrial tachycardia. Pediatrics. 1985;76:922-926. Kang KT, Potts JE, Radbill AE, et al. Permanent junctional reciprocating tachycardia in children: a multicenter experience. Heart Rhythm. 2014;11:1426-1432. Moore JP, Patel PA, Shannon KM. Predictors of myocardial recovery in pediatric tachycardia-induced cardiomyopathy. Heart Rhythm. 2014;11:1163-1169. Pinto DS. Cardiac manifestations of Lyme disease. Med Clin North Am. 2002;86:285-296. Valsangiacomo E, Schmid ER, Schupbach RW, et al. Early postoperative arrhythmias after cardiac operation in children. Ann Thorac Surg. 2002;4:792-796. Walsh EP, Cecchin F. Recent advances in pacemaker and implantable defibrillator therapy for young patients. Curr Opin Cardiol. 2004; 19:91-96. Weindling SN, Saul JP, Walsh EP. Efficacy and risks of medical therapy for supraventricular tachycardia in neonates and infants. Am Heart J. 1996;131:66-72.

The full reference list for this chapter is available at ExpertConsult.com.

e1





































































































1. Valsangiacomo E, Schmid ER, Schupbach RW, et al. Early postoperative arrhythmias after cardiac operation in children. Ann Thorac Surg. 2002;4:792-796. 2. Ramaswamy K, Hamdan MH. Ischemia, metabolic disturbances, and arrhythmogenesis: mechanisms and management. Crit Care Med. 2000;28:N151-N157. 3. Benson DW, Wand DW, Dyment M, et al. Congenital sick sinus syndrome caused by recessive mutation in the cardiac sodium channel gene (SCN5A). J Clin Invest. 2003;12:1019-1028. 4. Lupoglazoff JM, Denjoy I, Villain E, et al. Long QT syndrome in neonates: conduction disorders associated with HERG mutations and sinus bradycardia with KCNQ1 mutations. J Am Coll Cardiol. 2004;43:826-830. 5. Crosson JE, Callans DJ, Bradley DJ, et al. PACES/HRS expert consensus statement on the evaluation and management of ventricular arrhythmias in the child with a structurally normal heart. Heart Rhythm. 2014;11(9):e55-e78. 6. Paul T, Bertram H, Bokenkamp R. Supraventricular tachycardia in infants, children and adolescents: diagnosis, and pharmacological and interventional therapy. Paediatr Drugs. 2000;2:171-181. 7. Ko JK, Deal BJ, Strasburger JF, Benson Jr DW. Supraventricular tachycardia mechanisms and their age distribution in pediatric patients. Am J Cardiol. 1992;69(12):1028-1032. 8. Kang KT, Potts JE, Radbill AE, et al. Permanent junctional reciprocating tachycardia in children: a multicenter experience. Heart Rhythm. 2014;11:1426-1432. 9. Benditt DG, Benson Jr DW, Dunnigan A, et al. Atrial flutter, atrial fibrillation, and other primary atrial tachycardias. Med Clin North Am. 1984;68:895-918. 10. Frustaci A, Chimenti C, Bellocci F, et al. Histological substrate of atrial biopsies in patients with lone atrial fibrillation. Circulation. 1997;96:1180-1184. 11. Harahsheh A, Du W, Singh H, et al. Risk factors for atrioventricular tachycardia degenerating to atrial flutter/fibrillation in the young with Wolff-Parkinson-White. Pacing Clin Electrophysiol. 2008;31: 1307-1312. 12. Donnerstein RL, Berg RA, Shehab Z, et al. Complex atrial tachycardias and respiratory syncytial virus infections in infants. J Pediatr. 1994;125:23-28. 13. Collins KK, Van Hare GF, Kertesz NJ, et al. Pediatric nonpost-operative junctional ectopic tachycardia medical management and interventional therapies. J Am Coll Cardiol. 2009;53:690-697. 14. Lan YT, Lee JCR, Wetzel G. Postoperative arrhythmia. Curr Opin Cardiol. 2003;18:73-78. 15. Khairy P, Van Hare GF, Balaji S, et al. PACES/HRS expert consensus statement on the recognition and management of arrhythmias in adult congenital heart disease. Heart Rhythm. 2014;11:e102-e165. 16. Pinter A, Dorian P. Intravenous antiarrhythmic agents. Curr Opin Cardiol. 2001;16:17-22. 17. McKee MR. Amiodarone: an “old” drug with new recommendations. Curr Opin Pediatr. 2003;15:193-199. 18. Campbell RM, Strieper MJ, Frias PA. The role of radiofrequency ablation for pediatric supraventricular tachycardia. Minerva Pediatr. 2004;56:63-72. 19. Walsh EP, Cecchin F. Recent advances in pacemaker and implantable defibrillator therapy for young patients. Curr Opin Cardiol. 2004;19:91-96. 20. Takeuchi D, Tomizawa Y. Pacing device therapy in infants and children: a review. J Artif Organs. 2013;16(1):23-33. 21. Sliz Jr NB, Johns JA. Cardiac pacing in infants and children. Cardiol Rev. 2000;8:223-239. 22. Bernstein AD, Daubert JC, Fletcher RD, et al. The revised NASPE/ BPEG generic code for antibradycardia, adaptive-rate, and multisite pacing. North American Society of Pacing and Electrophysiology/ British Pacing and Electrophysiology Group. Pacing Clin Electrophysiol. 2002;25:260-264.

23. Gregoratos G, Abrams J, Epstein AE, et al. Guideline update for implantation of cardiac pacemakers and antiarrhythmia devices— summary article: a report of the American College of Cardiology/ American Heart Association Task Force on Practice Guidelines (ACC/AHA/NASPE Committee to Update the 1998 Pacemaker Guidelines). J Am Coll Cardiol. 2002;40:1703-1719. 24. Michaelsson M, Riesenfeld T, Jonzon A. Natural history of congenital complete atrioventricular block. Pacing Clin Electrophysiol. 1997;20: 2098-2101. 25. Horenstein MS, Walters H III, Karpawich PP. Chronic performance of steroid-eluting epicardial leads in a growing pediatric population: a 10-year comparison. Pacing Clin Electrophysiol. 2003;26:1467-1471. 25a. Moak JP, LaPage MJ, Fish F, et al. Comparison of the Medtronic SelectSure and conventional pacing leads: long-term follow-up in a multicenter pediatric and congenital cohort. Pacing Clin Elctrophys. 2019;42(3):356–365. 26. Reddy VY, Knops RE, Sperzel J, et al. Permanent leadless cardiac pacing: results of the LEADLESS trial. Circulation. 2014;129:1466-1471. 27. Mah DY, Alexander ME, Banka P, et al. The role of cardiac resynchronization therapy for arterial switch operations complicated by complete heart block. Ann Thorac Surg. 2013;96:904-909. 29. Bink-Boelkens MT. Pharmacologic management of arrhythmias. Pediatr Cardiol. 2000;21:508-515. 30. Roden DM. Antiarrhythmic drugs: from mechanisms to clinical practice. Br Heart J. 2000;84:339-346. 31. Vassallo P, Trohman RG. Prescribing amiodarone: an evidence-based review of clinical indications. JAMA. 2000;298:1312-1322. 32. Valdés SO, Miyake CY, Niu MC, et al. Early experience with intravenous sotalol in children with and without congenital heart disease. Heart Rhythm. 2018;15(12):1862-1869. 33. Li X, Zhang Y, Liu H, Jiang H, Ge H, Zhang Y. Efficacy of intravenous sotalol for treatment of incessant tachyarrhythmias in children. Am J Cardiol. 2017;119(9):1366-1370. 34. Doggrell SA. Hancox JC. Ibutilide—recent molecular insights and accumulating evidence for use in atrial flutter and fibrillation. Expert Opin Investig Drugs. 2005;14(5):655-669. 35. Volgman AS, Carberry PA, Stambler B, et al. Conversion efficacy and safety of intravenous ibutilide compared with intravenous procainamide in patients with atrial flutter or fibrillation. J Am Coll Cardiol. 1998;31(6):1414-1419. 36. Hoyer AW, Balaji S. The safety and efficacy of ibutilide in children and in patients with congenital heart disease. Pacing Clin Electrophysiol. 2007;30(8):1003-1008. 37. Weindling SN, Saul JP, Walsh EP. Efficacy and risks of medical therapy for supraventricular tachycardia in neonates and infants. Am Heart J. 1996;131:66-72. 38. Redman J, Worthley LI. Antiarrhythmic and haemodynamic effects of the commonly used intravenous electrolytes. Crit Care Resusc. 2001;3:22-34. 39. Sanatani S, Potts JE, Reed JH, et al. The study of antiarrhythmic medications in infancy (SAMIS): a multicenter, randomized controlled trial comparing the efficacy and safety of digoxin versus propranolol for prophylaxis of supraventricular tachycardia in infants. Circ Arrhythm Electrophysiol. 2012;5:984-991. 40. Hammer GB, Drover DR, Cao H, et al. The effects of dexmedetomidine on cardiac electrophysiology in children. Anesth Analg. 2008; 106:79-83. 41. Chrysostomou C, Morell VO, Wearden P, et al. Dexmedetomidine: therapeutic use for the termination of reentrant supraventricular tachycardia. Congenit Heart Dis. 2013;8:48-56. 42. Shepard SM, Tejman-Yarden S, Khanna S, et al. Dexmedetomidinerelated atrial standstill and loss of capture in a pediatric patient after congenital heart surgery. Crit Care Med. 2011;39:187-189. 43. Webb CA, Weyker PD, Flynn BC. Asystole after orthotopic lung transplantation: examining the interaction of cardiac denervation and dexmedetomidine. Case Rep Anesthesiol. 2012;2012:203240. 44. Koruth JS, Lala A, Pinney S, Reddy VY, Dukkipati SR. The clinical use of ivabradine. J Am Coll Cardiol. 2017;70(14):1777-1784.

References

e2







































60. Stratemann S, Dzurik Y, Fish F, et al. Left ventricular cardiac fibroma in a child presenting with ventricular tachycardia. Pediatr Cardiol. 2008;29:223-226. 61. Weindling SN, Saul JP, Gamble WJ, et al. Duration of complete atrioventricular block after congenital heart disease surgery. Am J Cardiol. 1998;82:525-527. 62. Janousek J, Vojtovic P, Gebauer RA. Use of a modified, commercially available temporary pacemaker for R wave synchronized atrial pacing in postoperative junctional ectopic tachycardia. Pacing Clin Electrophysiol. 2003;26:579-586. 63. Laird WP, Snyder CS, Kertesz NJ, et al. Use of intravenous amiodarone for postoperative junctional ectopic tachycardia in children. Pediatr Cardiol. 2003;24:133-137. 64. Yang T, Roden DM. Extracellular potassium modulation of drug block of IKr. Implications for torsades de pointes and reverse usedependence. Circulation. 1996;93:407-411. 65. Klein I, Ojamaa K. Thyroid hormone and the cardiovascular system. N Engl J Med. 2001;344:501-509. 66. Mattu A, Brady WJ, Perron AD. Electrocardiographic manifestations of hypothermia. Am J Emerg Med. 2002;20:314-326. 67. Huang CH, Tsai MS, Hsu CY, et al. Images in cardiovascular medicine. Therapeutic hypothermia-related torsades de pointes. Circulation. 2006;114:e521-e522. 68. Towbin JA, Bricker JT, Garson Jr A. Electrocardiographic criteria for diagnosis of acute myocardial infarction in childhood. Am J Cardiol. 1992;69:1545-1548. 69. Hastreiter AR, van der Horst RL, Chow-Tung E. Digitalis toxicity in infants and children. Pediatr Cardiol. 1984;5:131-148. 70. Kloner RA, Rezkalla SH. Cocaine and the heart. N Engl J Med. 2003;348:487-488. 71. Frassica JJ, Orav EJ, Walsh EP, et al. Arrhythmias in children prenatally exposed to cocaine. Arch Pediatr Adolesc Med. 1994;148:11631169. 72. Inoue H, Zipes DP. Cocaine-induced supersensitivity and arrhythmogenesis. J Am Coll Cardiol. 1988;11:867-874. 73. Chakko S. Arrhythmias associated with cocaine abuse. Card Electrophysiol Rev. 2002;6:168-169. 74. Witchel HJ, Hancox JC, Nutt DJ. Psychotropic drugs, cardiac arrhythmia, and sudden death. J Clin Psychopharmacol. 2003;23: 58-77. 75. Pinto DS. Cardiac manifestations of Lyme disease. Med Clin North Am. 2002;86:285-296. 76. DiNubile MJ, Calderwood SB, Steinhaus DM, et al. Cardiac conduction abnormalities complicating native valve active infective endocarditis. Am J Cardiol. 1986;58:1213-1217.





































45. Al-Ghamdi S, Al-Fayyadh MI, Hamilton RM. Potential new indication for ivabradine: treatment of a patient with congenital junctional ectopic tachycardia. J Cardiovasc Electrophysiol. 2013;24(7):822-824. 46. Janson CM, Tan RB, Iyer VR, Vogel RL, Vetter VL, Shah MJ. Ivabradine for treatment of tachyarrhythmias in children and young adults. HeartRhythm Case Rep. 2019;5(6):333-337. 47. Fish FA. Ventricular fibrillation: basic concepts. Pediatr Clin North Am. 2004;51:1211-1221. 48. The American Heart Association in collaboration with the International Liaison Committee on Resuscitation. Guidelines 2000 for cardiopulmonary resuscitation and emergency cardiovascular care. Part 10: pediatric advanced life support. Circulation. 2000;102:I291I342. 49. Gikonyo BM, Dunnigan A, Benson Jr DW. Cardiovascular collapse in infants: association with paroxysmal atrial tachycardia. Pediatrics. 1985;76:922-926. 50. Zimmerman FJ, Pahl E, Rocchini A. High incidence of incessant supraventricular tachycardia in pediatric patients referred for cardiac transplantation. Pacing Clin Electrophysiol. 1996;19:663. 51. Moore JP, Patel PA, Shannon KM. Predictors of myocardial recovery in pediatric tachycardia-induced cardiomyopathy. Heart Rhythm. 2014;11:1163-1169. 52. Bradley DJ, Fischbach PS, Law IH, et al. The clinical course of multifocal atrial tachycardia in infants and children. J Am Coll Cardiol. 2001;38:401-408. 53. Fish FA, Mehta AV, Johns J. Characteristics and management of chaotic atrial tachycardia of infancy. Am J Cardiol. 1996;78:10521055. 54. Priori SG, Napolitano C, Schwartz P. Low penetrance in the longQT syndrome: clinical impact. Circulation. 1999;99:529-533. 55. Yang T, Snyders D, Roden D. Drug block of I(kr): model systems and relevance to human arrhythmias. J Cardiovasc Pharmacol. 2001;38:737-744. 56. Khan IA, Gowda RM. Novel therapeutics for treatment of long-QT syndrome and torsades de pointes. Int J Cardiol. 2004;95:1-6. 57. Roden DM, Lazzara R, Rosen M, et al. Multiple mechanisms in the long-QT syndrome. Current knowledge, gaps, and future directions. The SADS Foundation Task Force on LQTS. Circulation. 1996;94:1996-2012. 58. Rehsia SS, Pepelassis D, Buffo-Sequeira I. Accelerated ventricular rhythm in healthy neonates. Paediatr Child Health. 2007;12:777779. 59. Reynolds JL, Pickoff AS. Accelerated ventricular rhythm in children: a review and report of a case with congenital heart disease. Pediatr Cardiol. 2001;22:23-28.

e3

Abstract: Arrhythmias are frequently encountered in the intensive care setting, either as the primary problem requiring care or in the setting of other cardiac or noncardiac disease processes. The acute approach to diagnosis, treatments, and identification of underlying etiologies is crucial to appropriate stabilization of the patient and direction of ongoing management. Likewise, recognition of specific arrhythmia mechanisms is often important in the selection of treatment choices. This chapter attempts to provide an

overview of the approach to treatment of an array of arrhythmias in the intensive care unit setting. Key words: AV block, supraventricular tachycardia, junctional ectopic tachycardia, atrial flutter, ventricular tachycardia, adenosine, temporary pacemakers, permanent pacemakers, antiarrhythmic drugs

34 Shock States LINCOLN SMITH, ALICIA ALCAMO, JOSEPH A. CARCILLO, AND RAJESH ANEJA

The clinical syndrome of shock is one of the most dramatic, dynamic, and life-threatening problems faced by the physician in the critical care setting. Although untreated shock is universally lethal, mortality may be considerably reduced by rapid, proper recognition, diagnosis, monitoring, and treatment.

Definition and Physiology Shock is an acute, complex state of circulatory or metabolic dysfunction that results in failure to deliver or use sufficient amounts of oxygen and/or other nutrients to meet tissue metabolic demands. If prolonged, it leads to multiple-organ failure and death. Therefore, shock can be viewed as a state of acute cellular energy deficiency. Shock can be caused by any serious disease or injury. However, whatever the causative factors, it is always a problem of inadequate cellular sustenance. It is the final common pathway to death. Delivery of oxygen (Do2) is a direct function of cardiac output (CO) and arterial concentration of O2 (Cao2): Delivery of oxygen: Do2 5 CO 3 Cao2 Cardiac output: CO 5 Heart rate 3 Stroke volume Oxygen content: Cao2 5 (Hgb 3 1.34 3 Sao2) 1 (0.003 3 Pao2) where Hgb is hemoglobin, Sao2 is arterial oxygen saturation, and Pao2 is arterial partial pressure of oxygen. Stroke volume is a function of preload, afterload, contractility, and diastolic relaxation. Therefore, optimizing heart rate, contractility, diastolic relaxation, preload, and afterload improves cardiac output. Oxygencarrying capacity can be increased by raising hemoglobin and optimizing its saturation with oxygen. Oxygen delivery can be improved by manipulation of all these factors. Calculation of global oxygen delivery may not reflect regional hypoperfusion and localized ischemia. Inadequate oxygen delivery can result from either limitation or maldistribution of blood flow. Reduced oxygen content (anemia, low Sao2) necessitates higher cardiac output to maintain oxygen delivery. In certain situations (fever, sepsis, trauma), metabolic demands may exceed normal oxygen delivery. Impairment of the extraction or utilization of oxygen by cells and mitochondria creates a functional 352

arteriovenous shunt and may be the harbinger of multiorgan dysfunction syndrome (MODS).1,2

Functional Classification and Common Underlying Etiologies Shock states can be classified into seven functional categories (Box 34.1). Such a tidy classification implies a degree of precision that will be misleading when approaching an individual patient. Vicious cycles play a prominent role in most shock syndromes. Any given patient may display features of any functional category or features of multiple categories over time. Hemodynamic profiles of these categories are summarized in Table 34.1. Early recognition of shock begins with a careful history and physical examination.

Hypovolemic Shock Hypovolemia is the most common cause of shock in infants and children. Etiologies include hemorrhage, fluid and electrolyte loss, endocrine disease, and plasma loss (Box 34.2). Acute losses of 10% to 15% of the circulatory blood volume may be well tolerated in healthy children who have intact compensatory mechanisms. However, an acute loss of 25% or more of the circulating blood volume frequently results in hypovolemic shock that requires immediate management.

Cardiogenic Shock or Congestive Heart Failure Cardiogenic shock or congestive heart failure (CHF) during infancy and childhood is a diagnostic and therapeutic challenge because of its myriad etiologies (Box 34.3). The common denominator in all forms of cardiogenic shock is depressed cardiac output. In many instances, the underlying mechanism is systolic dysfunction or “pump failure.” Cardiogenic shock can also be caused by diastolic dysfunction, as seen in postoperative patients, ischemic heart disease (anomalous left coronary artery from pulmonary artery [ALCAPA]), or disorders associated with ventricular hypertrophy.3,4 Lack of myocardial relaxation increases ventricular end-diastolic pressure and eventually reduces enddiastolic volume. Elevated right ventricular (RV) end-diastolic pressures can cause hepatic and systemic venous congestion and may reduce end-organ perfusion pressures. Elevated left ventricular (LV) end-diastolic pressures result in pulmonary edema and



CHAPTER 34 Shock States

• BOX 34.1 Shock States

• BOX 34.2 Etiologies of Hypovolemic Shock Whole Blood Loss Absolute loss: hemorrhage • External bleeding • Internal bleeding • Gastrointestinal • Intraabdominal (spleen, liver) • Major vessel injury • Intracranial (in infants) • Fractures Relative loss • Pharmacologic (barbiturates, vasodilators) • Positive-pressure ventilation • Spinal cord injury • Sepsis • Anaphylaxis

HEMODYNAMIC PATTERN: BLOOD PRESSURE OR SYSTEMIC VASCULAR RESISTANCE



TABLE Hemodynamic Patterns in Shock States 34.1 and Therapeutic Receptor Targets















Septic Endocrine Cytopathic

Hypovolemic Cardiogenic Obstructive Distributive

353

Shock State

Normal

Decreased

Elevated

Septic Shock None

a1 or V1

None

Stroke index g

b1

a1 and b1

b1 1 b2, or PDE

Myocardial dysfunction (complicating critical illness)a

b1 and/or b2

a1 and b1

b1 1 b2 and/or PDE

Congestive heart failure

b1 and/or b2

b1

b1 1 b2, and/or PDE

PGE1

b1 and PGE1

PGE1

None

a1

None

Cardiogenic Shock





Stroke index h 6

Plasma Loss Burns Capillary leak syndromes • Inflammation, sepsis • Anaphylaxis Protein-losing syndromes

Fluid and Electrolyte Loss







Vomiting and diarrhea Excessive diuretic use Endocrine • Adrenal insufficiency • Diabetes insipidus • Diabetes mellitus

Obstructive Shock Ductal-dependent lesion

Distributive Shock Neurogenic shock

PDE, Phosphodiesterase inhibitor; PGE1, prostaglandin. a For example, acute respiratory distress syndrome or anthracycline therapy.

decreased myocardial perfusion pressure leads to subendocardial ischemia. Abnormalities of the heart rate and rhythm can also cause cardiogenic shock. While bradyarrhythmias cause low cardiac output due to decreased heart rate, atrioventricular dyssynchrony and tachyarrhythmias cause low cardiac output owing to inadequate diastolic filling. Tachyarrhythmias also increase myocardial oxygen consumption and compromise myocardial perfusion. Finally, myocardial dysfunction is frequently a late manifestation of shock of any etiology. It is important to understand the underlying pathophysiology of cardiogenic shock, as therapy designed to improve certain conditions may also adversely affect prognosis in other conditions.4

Obstructive Shock Obstructive shock is caused by the inability to produce adequate cardiac output despite normal intravascular volume and myocardial function. Causative factors may be located within the pulmonary or systemic circulation or may be associated with the heart itself. Examples of obstructive shock include acute pericardial tamponade, tension pneumothorax, pulmonary or systemic

hypertension, and congenital or acquired outflow obstructions. Recognition of the characteristic features of these syndromes is essential because most of the causes can be treated provided that the diagnosis is made early. Cardiac tamponade is defined as hemodynamically significant cardiac compression resulting from accumulating pericardial contents that evoke and defeat compensatory mechanisms. The pericardial space may contain effusion fluid, purulent fluid, blood, or gas. Clinical manifestations of tamponade may be insidious, especially when it occurs in conditions such as malignancy, connective tissue disorders, renal failure, or pericarditis. As cardiac output becomes restricted, the overall picture resembles CHF; however, the lungs are usually clear. Findings on physical examination that suggest cardiac tamponade include pulsus paradoxus, narrowed pulse pressure, pericardial rub, and jugular venous distension. Echocardiography is of particular value in detecting the presence of pericardial effusion and can provide clues about the presence of tamponade physiology before a patient is symptomatic. The normal effects of respiration are accentuated in cardiac tamponade. Echocardiography is useful in demonstrating the exaggerated phasic variation in cardiac volumes and flows caused by tamponade. Respiratory variation in tricuspid and pulmonary flow is more dramatic than mitral and aortic flow: with inspiration, RV early diastolic filling is augmented (.25%), while LV diastolic filling diminishes (.15%). The stroke volume in the pulmonary artery increases with inspiration, while the aortic stroke volume decreases (.10%). The free walls of the right atrium and/or right ventricle collapse in diastole due to compression of these relatively low-pressure chambers by the higher-pressure

354

SECTION IV



Pediatric Critical Care: Cardiovascular

• BOX 34.3 Etiologies of Cardiogenic Shock Heart Rate Abnormalities Supraventricular tachycardia Ventricular dysrhythmias Bradycardia

Metabolic • Hypothyroid, hyperthyroid • Hypoglycemia • Pheochromocytoma • Glycogen storage disease • Mucopolysaccharidoses • Carnitine deficiency • Disorders of fatty acid metabolism • Acidosis • Hypothermia • Hypocalcemia Connective tissue disorders • Systemic lupus erythematosus • Juvenile rheumatoid arthritis • Polyarteritis nodosa • Kawasaki disease • Acute rheumatic fever Neuromuscular disorders • Duchenne muscular dystrophy • Myotonic dystrophy • Limb girdle (Erb) • Spinal muscular dystrophy • Friedreich ataxia • Multiple lentiginosis Toxic reactions • Sulfonamides • Penicillins • Anthracyclines Other • Idiopathic dilated cardiomyopathy







Congenital Heart Defects Lesions with ductal-dependent systemic blood flow in neonates (CoA, critical AS, IAA, HLHS) Left-to-right shunt lesions Single-ventricle dysfunction Systemic ventricular dysfunction (L-TGA) Ischemic cardiomyopathies (ALCAPA)















Cardiomyopathy, Carditis Hypoxic-ischemic events • Postcardiac arrest • Prolonged shock • Head injury • Anomalous coronary artery • Excessive catecholamine state • Cardiopulmonary bypass Infectious • Viral • Bacterial • Fungal • Protozoal • Rickettsial • Sepsis



















































ALCAPA, Anomalous left coronary artery from pulmonary artery; AS, aortic stenosis; CoA, acetylcoenzyme A; HLHS, hypoplastic left heart syndrome; IAA, interrupted aortic arch; L-TGA, looped transposition of the great arteries.

pericardial effusion. This collapse is exaggerated during expiration when right heart filling is reduced.5

Distributive Shock Distributive shock results from maldistribution of blood flow to the tissues and can be considered relative hypovolemia. Abnormalities in distribution of blood flow may result in profound inadequacies in tissue oxygenation even in the face of a normal or high cardiac output. Such maldistribution of flow generally results from widespread abnormalities in vasomotor tone. Distributive shock may be seen with anaphylaxis, spinal or epidural anesthesia, disruption of the spinal cord, or inappropriate administration of vasodilatory medication. Treatment generally includes reversal of the underlying etiology and vigorous fluid administration. In severe cases of distributive shock that is unresponsive to fluids, vasopressor infusions may be necessary.

Septic Shock Septic shock is often a combination of multiple problems, including infection; relative or absolute hypovolemia; maldistribution of blood flow; myocardial depression; and various metabolic, endocrine, and hematologic problems.6 The most common presentation (80%) in children is low cardiac index with or without

abnormalities of vascular tone.7 These children have tachycardia, mental status changes, diminished peripheral pulses, mottled cold extremities, and prolonged capillary refill (.2 seconds). In many pediatric patients with septic shock, oxygen consumption is dependent on oxygen delivery.8 This is similar to the physiologic relationship seen in pediatric patients with cardiogenic shock, suggesting that these two groups could be resuscitated with the same physiologic principles. Adults and some children (20%) present in a hyperdynamic state characterized by an elevated (or normal) cardiac output and decreased systemic vascular resistance.7,9 These patients appear plethoric with warm extremities. They have tachycardia, bounding (or collapsing) pulses, a widened pulse pressure, high fever, mental confusion, and hyperventilation. There may be a rapid progression from high to low cardiac output state. As tissue perfusion worsens, anaerobic metabolism ensues and lactic acid accumulates. The hemodynamic profile changes over time owing to evolution of the shock state and response to therapies.7 All patients with septic shock present with an absolute or functional hypovolemia. Increased microvascular permeability, arteriolar and venular dilation with peripheral pooling of intravascular volume, inappropriate polyuria, and poor oral intake all combine to result in reduced effective blood volume. Volume loss secondary to fever, diarrhea, vomiting, or sequestered third-space fluid also contributes to hypovolemia.



CHAPTER 34 Shock States

Progressive deterioration in oxygen consumption and oxygen extraction portends a poor prognosis. Prior to the onset of cellular hypoxia,2,10,11 changes in glycolysis and gluconeogenesis are early metabolic manifestations of sepsis.12 Insulin responsiveness,13 intracellular calcium stores,14 glucose distribution,15 and adrenergic effects16 have all been implicated.

Endocrine Endocrinologic causes of shock are often omitted as a formal category because they manifest as cardiac or distributive shock. However, since the underlying pathobiology is due to dysfunction of the endocrine system, it is addressed specifically here. Severe hypothyroidism results in bradycardia and hypotension, whereas thyrotoxicosis is a cause of tachydysrhythmias and cardiomyopathy.17 Hypothyroidism has long been associated with other causes of shock and critical illness. Although this “sick euthyroid syndrome” has been thought to be adaptive rather than pathologic, recent data associating low circulating thyroid hormones with worse clinical outcomes suggests that this conventional belief should be reconsidered.18 Adrenal insufficiency may be congenital or acquired and results in a life-threatening inability to mount a stress response. Relative adrenal insufficiency associated with critically ill patients has been well described. However, diagnosis and treatment of critical illness–related corticosteroid insufficiency remain controversial, and there is a current clinical trial to determine the risks and benefits of adjunctive hydrocortisone in septic children.19,20

Mitochondrial Resolution of all of the shock states discussed earlier depend on intact mitochondrial function to restore oxidative phosphorylation of adenosine diphosphate. However, inherited and acquired mitochondrial dysfunction result in an inability to use nutrients delivered to tissues and cells.21–23 Cytopathic hypoxia is an acquired mitochondrial failure observed in some septic patients.2,24 These patients characteristically present with normal to high cardiac output, high arterial and venous oxygen content, and persistent organ dysfunction and lactic acidosis. Diagnosis and treatment of inherited inborn errors of metabolism and mitochondriopathies require genetic and metabolic subspecialty collaboration.22

Multisystem Effects of Shock

355

oxygen is essential in all children with shock. Early tracheal intubation protects the airway, provides relief from respiratory muscle fatigue, facilitates provision of positive airway pressure, redistributes blood flow from the muscles of respiration to core organs, afterload-reduces the left ventricle, and reduces oxygen demands of respiratory muscles. Patients should be ventilated with a lung-protective strategy (see Chapter 48).

Renal Renal failure may develop in association with any of the shock syndromes. Shock-related renal failure is a continuum of acute prerenal azotemia through classic acute tubular necrosis to cortical necrosis. Although low-dose dopamine (3–5 mg/kg per minute) improves renal blood flow,30,31 it also impairs renal oxygen kinetics, inhibits protective feedback loops with the kidney, may worsen tubular injury, and has failed to show benefit in preventing or altering the course of acute renal failure in adults.32,33 Acute anuric renal failure may require treatment with peritoneal dialysis, ultrafiltration, continuous hemofiltration or hemodiafiltration, or hemodialysis (see Chapter 75). Populations for whom early renal replacement therapies result in decreased mortality have not been consistently identified, but there is evidence that fluid overload is associated with mortality in critically ill children with renal dysfunction.34 If renal dysfunction exists, all medications and therapies should be adjusted for creatinine clearance. Highoutput renal failure may occur in shock states without previous oliguria. The polyuria associated with this condition may falsely suggest adequate renal perfusion and adequate vascular volume at a time when the patient’s intravascular volume is, in fact, depleted. Restoration of renal perfusion pressure remains the standard of care.

Coagulation Coagulation abnormalities (e.g., disseminated intravascular coagulation) probably occur to some extent in all forms of shock. Monitoring of prothrombin time, partial thromboplastin time, and platelet count and observation for abnormal bleeding are essential. Replacement therapies of absent clotting factors seem to be the most advantageous treatments. Use of vitamin K, freshfrozen plasma, cryoprecipitate, and platelet transfusions should correct most coagulopathies. If general replacement therapy is ineffective and the patient is at risk for complications, specific factor therapy may be indicated (see section on septic shock).

Management of the multisystem deterioration that occurs in shock states is as important as treating the underlying condition. Respiratory, gastrointestinal, central nervous system, renal, and hematologic abnormalities must be anticipated. Multiple organ dysfunction syndrome (MODS) is the derangement of two or more organs after an insult.25 The severity of MODS has been associated with increased mortality in pediatric intensive care unit (PICU) patients.26–29

Hepatic

Respiratory

Gastrointestinal

Respiratory failure frequently accompanies shock states. It may result from failure of the ventilator pump (i.e., respiratory muscle fatigue) and/or deterioration of lung function (i.e., acute respiratory distress syndrome). Therefore, providing supplemental

Acute nonocclusive mesenteric ischemia is a devastating condition characterized by intense, prolonged splanchnic vasoconstriction, intestinal mucosal hypoxia, and acidosis. Mesenteric ischemia eventually leads to transmural necrosis of the bowel, bacterial

The degree of hepatic dysfunction may determine a patient’s ultimate outcome in severe shock states. Maintaining adequate circulation helps maintain liver function and prevents further hepatocellular damage. Liver function tests should be performed early and followed frequently. If dysfunction exists, drugs requiring hepatic metabolism must be carefully titrated.

356

SECTION IV



Pediatric Critical Care: Cardiovascular

translocation, sepsis, and multisystem organ dysfunction.35–37 Morbidity and mortality for this condition are high because the signs and symptoms are nonspecific. Prevention of gut ischemia through adequate oxygen delivery may prevent bacterial translocation. Some clinicians advocate the use of selective gut decontamination and early enteral nutrition.38,39 Most children with shock will tolerate postpyloric enteral feeding, although gastrointestinal feeding complications are more common than in critically ill patients without shock.40,41 Other gastrointestinal disturbances after hypoperfusion and stress include bleeding, ileus, and bacterial translocation. Ileus may result from electrolyte abnormalities, administration of narcotic medications, or from shock itself. Abdominal distension from ileus or ascites may cause respiratory compromise, especially in infants. The substantial morbidity and mortality of upper gastrointestinal bleeding due to “stress-related mucosal damage” has led to widespread prophylactic use of medications to suppress gastric acid production, but the benefit of this practice remains unproven.42,43

Endocrine Multiple endocrine problems involving fluid, electrolytes, and mineral balance may arise and complicate the management of children in shock. Severe abnormalities of calcium homeostasis can occur during the course of acute hemodynamic deterioration. Patients who have been administered corticosteroids within 6 months preceding the onset of shock should be considered for stress doses of glucocorticoids. Patients in shock because of head or abdominal trauma may have disruption of the hypothalamicanterior pituitary-adrenal axis. Adrenal hemorrhage has been demonstrated as a manifestation of severe sepsis; however, more commonly, patients may develop a relative or functional adrenal insufficiency. Dopamine may also inhibit secretion of prolactin, growth hormone, and thyrotropin in critically ill children.40

Monitoring Consumption of oxygen or any other nutrient can be calculated or measured in a variety of ways. Direct calorimetry is used to measure metabolic activity and was first used in the 18th century by Antoine Lavoisier and Pierre-Simon Laplace to measure the heat generated by an animal.44 Direct calorimetry is a measure of all metabolic activity (aerobic 1 anaerobic). Early studies led to understanding the relationships between oxygen consumption, carbon dioxide production, and metabolic activity. Indirect calorimetry involves the measurement of oxygen consumption and carbon dioxide production to estimate a heat equivalent. These early direct and indirect calorimetry studies led Adolf Fick to determine a relationship between cardiac output, oxygen consumption, and the arterial and venous concentrations of oxygen (Cvo2).45

Fick Equation for Oxygen Consumption Vo2 5 (CO 3 Cao2) – (CO 3 Cvo2) Since the first description of lactate as a prognostic tool in 1964,46 serum lactate levels have been used as a surrogate for illness severity and to monitor response to therapeutic interventions.47 Although elevated lactate levels are best described in the context of septic shock, all forms of shock or tissue hypoperfusion

will result in elevated serum lactate. In aerobic conditions, oxidation of glucose generates pyruvate that undergoes oxidative decarboxylation and is transformed into carbon dioxide and water. Conversely, in anaerobic conditions, there is an accumulation of pyruvate that is associated with an elevated lactate/pyruvate ratio, higher glucose utilization, and low energy production, resulting in elevated serum lactate.48 Conditions that are associated with an elevation of lactate include trauma, hypoxemia, severe anemia, and shock.49 In critically ill children, elevated serum lactate concentration (.2 mmol/L) on admission is associated with higher mortality risk.50–52 Elevated lactate in pediatric trauma patients is strongly correlated with the severity of the injury, length of stay, and mortality.53 Similarly, elevated blood lactate levels at admission in pediatric septic shock patients are predictive of death.54,55 Owing to the technical challenges of obtaining an arterial blood sample in children, venous lactate levels have been demonstrated to be a surrogate for arterial lactate levels (#2 mmol/L). However, the agreement between arterial and venous lactate levels in septic patients is poor for venous lactate levels 2 mmol/L.56 Therefore, an arterial sample must be drawn to confirm an initial measurement for serum lactate 2.0 mmol/L. In response to a decrease in cardiac output and the subsequent reduction in oxygen delivery, there is a compensatory increase in oxygen extraction to prevent anaerobic metabolism. The Fick equation states that CO 5 Vo2/(Cao2 2 Cvo2) where Vo2 is oxygen consumption and Cvo2 is venous oxygen content. The equation can be rearranged and simplified: CO 3 Cao2 2 Vo2 5 CO 3 Cvo2 Dividing both sides by Do2 (which equals CO 3 Cao2) yields: 12 ERO2 5 (CO 3 Cvo2)/(CO 3 Cao2) or Svo2 5 (12 ERO2) 3 Sao2, where ERO2 5 Vo2/Do2 where ERO2 is the oxygen extraction ratio, Svo2 is systemic mixed venous oxygen saturation, and Scvo2 is central venous oxygen saturation. In clinical practice, Sao2 is often kept quite constant and often greater than 0.9; therefore57: Svo2 5 12 ERO2 Scvo2 greater than or equal to 70% remains a target in the American College of Critical Care Medicine Clinical Guidelines for Hemodynamic Support of Neonates and Children with Septic Shock.58 Examination of the correlation between CO and Svo2 measurement reveals that Svo2 measurement performs poorly in high cardiac output states but typically performs well in clinically relevant situations when oxygen delivery is inadequate.57 At present, when pulmonary artery catheters are not being placed, the Scvo2 is used as a surrogate for Svo2. It is noteworthy that the Scvo2 can be 2% to 8% higher than the Svo2, and the relationship between these two variables changes with catheter placement, flow states, and relative changes in the flow in the venae cavae and coronary sinus.59 Brierley and Peters reported that children with community-acquired septic shock were more likely to have low cardiac output shock. In contrast, children with hospital-acquired (catheter) sepsis were more likely to



CHAPTER 34 Shock States

Early identification and aggressive, timely treatment improves the outcome in pediatric shock.87,88 The American Heart Association’s Pediatric Advanced Life Support Guidelines provide a systematic approach to assess shock, focusing the primary goal of management on optimizing and balancing oxygen delivery with oxygen consumption.58 The initial approach for stabilization in a patient with undifferentiated shock is highlighted in Box 34.4. Treatment begins with an assessment of the patient’s airway and breathing to provide adequate oxygen delivery during resuscitation. Ensuring sufficient cardiac output through assessment of circulation and end-organ perfusion is imperative, allowing for titration of fluid resuscitation and vasoactive administration. During the first hour of resuscitation, attention must be directed to the underlying etiology of the shock state.58 Efforts to reduce oxygen requirements when oxygen delivery is compromised are essential. Even routine nursing procedures can increase oxygen consumption by up to 20% to 30% in healthy adults.89 Management should be guided by both the clinical examination and monitoring techniques discussed previously in this chapter and titrated to the desired effect.

Intubation and Mechanical Ventilation Intubation and mechanical ventilation may be necessary in shock states for several reasons: overt respiratory failure, lack of airway protection, and control of energy expenditure. These interventions may reduce oxygen consumption by minimizing respiratory work in addition to improving oxygen delivery.90 Use of sedation and neuromuscular blockade further helps to reduce oxygen consumption related to increased metabolic demand from respiratory work.91 However, clinicians must remain acutely aware of the cardiopulmonary interactions that occur during this intervention to • BOX 34.4 Systematic Approach for Initial

Management of Undifferentiated Shock Airway and Breathing Supplemental oxygen Assess need for endotracheal intubation: airway compromise, respiratory failure

Circulation Fluid resuscitation Vasoactive infusions

Other













Identify specific treatments for underlying etiologies • Hemorrhagic shock: blood transfusion • Cardiogenic shock: limit fluids, vasoactive support, mechanical support if needed • Obstructive shock • Tension pneumothorax: needle decompression • Cardiac tamponade: pericardiocentesis • Pulmonary embolism: thrombolytics, surgical intervention • Ductal-dependent lesion: prostaglandin infusion • Anaphylactic shock: epinephrine • Septic shock: antimicrobials Evaluate for electrolyte disturbances

The American College of Critical Care Medicine Clinical Practice Parameters for Hemodynamic Support of Pediatric and Neonatal Septic Shock recommend titration of therapy to a cardiac output goal of 3.3 to 6.0 L/min per square meter in patients with persistent catecholamine-resistant shock.58 Historically, pulmonary catheter-directed treatment was considered the gold standard for assessing cardiac function and optimizing oxygen delivery in the hemodynamically unstable patient.81 However, the use of the pulmonary catheter has significantly decreased as trials noted the lack of benefit with the use of pulmonary artery catheters in adult patients admitted to the ICU.82,83 For example, in the Study to Understand Prognoses and Preferences for Outcomes and Risks of Treatments (SUPPORT) trial, the use of right heart catheterization in more than 5700 patients was associated with increased mortality and increased utilization of resources.82 Similarly, a meta-analysis also noted a lack of benefit with the use of pulmonary artery catheters in critically ill patients.84 With the decrease in the use of the pulmonary artery catheter, the search continues for noninvasive cardiac output monitoring devices. The ideal attributes of such a device would be that it is reliable, noninvasive, cost-effective, and provides continuous cardiac output monitoring. In addition to CI, other variables that help titrate therapy include stroke volume index, indexed systemic vascular resistance, Cao2, and Do2. Several noninvasive CO monitoring devices are available; a review of the various technology platforms that support these devices is beyond the scope of this chapter. A detailed review of noninvasive hemodynamic monitoring in the ICU can be found elsewhere in the literature.85,86

General Principles



Contemporary Cardiac Output Monitoring in Pediatric Shock

Treatment



manifest high cardiac output shock.60 Furthermore, they noted that low Scvo2 occurred in children with a low cardiac index (CI), but they also found that children with a high CI could have a low Scvo2.60 Similar findings of low Scvo2 in shock have been reported by other investigators.61,62 It is important to emphasize that the clinical context should always be considered when Scvo2 measurements are interpreted—patients in extreme vasodilatory shock or following mitochondrial poisoning demonstrate elevated Scvo2 (.70%), as the oxygen extraction is severely impaired owing to mitochondrial dysfunction.63,64 Despite an adequate hemodynamic status as quantified by Scvo2, microcirculatory splanchnic hypoperfusion may be present in patients with shock, resulting in significant morbidity and mortality.65 The cause of microcirculatory failure is multifactorial and includes physiologic shunting, maldistributed flow, increased microvascular permeability, and microvascular thrombosis.65 Thus, an increased understanding of microcirculatory aberrations and cellular hypoxia has stimulated a search for a minimally invasive means of sampling regional circulations.66–68 Gastric tonometry,69,70 near-infrared spectroscopy,71,72 rectal tonometry,73 sublingual capnometry,74,75 muscle oxygenation,76 tissue microdialysis,77,78 and orthogonal polarization spectral imaging79,80 are investigational methods to evaluate regional circulation, but their clinical utility remains unproven at this time. Repeated evaluations and monitoring of the patient in shock by a competent clinician, with appropriate, timely interventions, remains the most effective and sensitive physiologic monitor available.

357

358

SECTION IV



Pediatric Critical Care: Cardiovascular

understand the impact on cardiac output (see Chapter 32 for further information). Mechanical ventilation can substantially improve oxygen delivery to vital organs, and suboptimal patient-ventilator interactions may result in increased oxygen consumption.90,92 Positive-pressure ventilation decreases preload and increases afterload on the right ventricle. The reduction of preload can often be overcome by treating the patient with a rapid fluid bolus prior to or during intubation. Positive-pressure ventilation also reduces afterload to the left ventricle, which may improve stroke volume.

Fluid Resuscitation Regardless of the underlying insult, most patients in shock are hypovolemic, causing a decrease in preload and subsequent decrement of stroke volume and cardiac output. Early fluid resuscitation is the cornerstone of therapy.58 Based on the Frank-Starling curve, fluid administration will improve ventricular preload and thus increase ventricular stroke volume. Studies of septic pediatric and adult patients showed that early fluid resuscitation was associated with improved patient outcomes; these principles are incorporated in current treatment guidelines.93–95 However, excessive fluid resuscitation in patients with cardiogenic shock and some forms of obstructive shock (massive pulmonary embolism or severe pulmonary hypertension) may rapidly push the patient over the FrankStarling curve into CHF. There is considerable debate about the type of fluid therapy that should be administered during acute resuscitation. The use of colloids in place of crystalloids to increase oncotic pressure and restore intravascular volume has been studied for the management of different shock states, including severe sepsis and postcardiac surgery.96–98 Studies in severe sepsis have shown that the use of albumin replacements either in conjunction with or in place of crystalloids does not improve survival.96,97 In a retrospective pediatric cohort following cardiac surgery, the use of 5% albumin for resuscitation resulted in fluid overload without appreciable clinical benefit.98 The use of normal saline for resuscitation may result in hyperchloremia and metabolic acidosis, both of which have been associated with worse outcomes and mortality in both children and adults with septic shock.99–104 However, a randomized control trial in critically ill adults showed that resuscitation with balanced crystalloids was associated with lower rates of death, persistent renal dysfunction, and need for renal replacement therapy.105 In a retrospective review of pediatric septic patients, there was no difference in outcomes of mortality, acute kidney injury, or need for renal replacement therapy between balanced solutions and normal saline.106 Due to lack of published clinical experience, it remains to be determined whether the use of balanced crystalloids is associated with significant improvements in outcome in critically ill children. Massive transfusion in pediatrics has been defined as a transfusion of 40 mL/kg of any blood product given at any time in the first 24 hours.107 Massive transfusion protocol has been adopted by most pediatric trauma centers to correct hemorrhagic shock with a balanced transfusion ratio of plasma, platelets, and packed red blood cells (PRBCs). The use of such protocols makes physiologic sense and is supported by adult data that demonstrate improved patient outcomes and better hemostasis.108,109 Similar studies in children have not shown the perceived benefit of a balanced transfusion approach in pediatric trauma victims.110–113 For example, in a retrospective review of 6675 pediatric trauma patients in which 105 were massively transfused (.50% of total

blood volume in 24 hours), higher plasma/PRBC and platelet/ PRBC ratios were not associated with increased survival.111 Similarly, in a retrospective review of 435 patients in Canada, an occurrence of massive transfusion was noted in only 3% of patients, which was associated with coagulopathy and poor outcomes.113 Thus, further research needs to be conducted to identify optimal ratios of blood product transfusion that would lead to an improved outcome in children. Careful attention must be paid to the physiologic responses to fluid resuscitation by the clinician to prevent fluid overload. Evidence associating positive fluid balance with increased mortality in critically ill adult and pediatric patients114–117 suggests a need for better predictors of fluid responsiveness and end points of fluid resuscitation.118,119 When sufficient preload has been achieved through fluid resuscitation but the patient remains in shock, other supportive therapies are indicated.

Vasoactive Infusions Infusions to increase cardiac output and improve peripheral vascular tone are indicated when patients have been adequately fluid-resuscitated but hemodynamics remain unstable. Infusions of catecholamines (dopamine, dobutamine, epinephrine, norepinephrine), phosphodiesterase inhibitors (milrinone), and vasopressin are most commonly used. The choice of vasoactive infusion is dependent on the physiologic derangement (see Table 34.1). Catecholamines work through stimulation of a1-, a2-, b1-, b2-, and dopaminergic receptors to increase intracellular cyclic guanosine monophosphate (cGMP; Table 34.2). Phosphodiesterase inhibitors increase cGMP by preventing its degradation within the cell (see Chapter 31). Vasopressin causes vasoconstriction by direct stimulation of vascular smooth muscle cell V1-receptors. Vasopressin also potentiates systemic adrenergic effects. Vasopressin and terlipressin (a synthetic analog of vasopressin with a similar pharmacodynamic profile but with a significantly longer half-life) have also shown some utility in the treatment of catecholamine-resistant shock.120–122 Fenoldopam, a dopamine-receptor agonist, has been used to augment diuresis and improve hemodynamics after cardiac surgery and in septic shock.123,124 Assessment of the patient’s clinical presentation is imperative for the appropriate vasoactive choice. Therapies should be tailored to the unique patient situation, and patients should be closely monitored for expected therapeutic response. Limited data exist regarding the superiority of these different medications. However, there has been recent pediatric data that has shown epinephrine as being more effective and associated with increased survival when compared with dopamine in treatment of septic shock.125,126 Similarly, data from critically ill septic adults suggest the use of norepinephrine over dopamine for firstline treatment of septic shock due to improvement in mortality and lower risk of adverse events.127,128 In adults, nonadrenergic vasopressors, such as vasopressin, have been shown to decrease the amount of catecholamine requirements and reduce mortality when compared with norepinephrine in states of vasodilatory shock.129,130 In addition, clinicians should consider how the use of these vasoactive medications may impact the patient’s physiology and other elements of management. For example, epinephrine has been shown to increase serum lactate levels secondary to b2 stimulation.131 Norepinephrine and epinephrine have also been shown to decrease regional blood flow to organs such as the gastrointestinal



CHAPTER 34 Shock States

359

TABLE Vasoactive Medications and Mechanism by Receptor Target 34.2 RECEPTOR

Vasoactive

a1

b1

b2

D1

Dopamine

Vasoconstriction

Inotropy, chronotropy

Vasodilation

Renal vasodilation

Norepinephrine

Vasoconstriction

Inotropy

Epinephrineb

Vasoconstriction

Inotropy, chronotropy

Vasodilation

Inotropy

Vasodilation

a

Dobutamine Vasopressin

Potentiates

Potentiates

Fenoldopam Inamrinone, milrinone

V1

Vasoconstriction Renal vasodilation

Non–receptor-mediated inotropy, lusitropy, and vasodilation

a Dose related: At low infusion rates, D1-receptor effects predominate; at intermediate rates, b1- and b2-receptor effects predominate; at high rates, a1-receptor effects predominate on peripheral vasculature. b Dose related: At low infusion rates, b-receptor effects predominate; at high rates, a-receptor effects predominate on peripheral vasculature.

tract.132,133 Renal clearance of milrinone may limit its utility in patients with significant renal injury or failure.

Age-Related Therapy Concerns The transition of fetal to neonatal physiology offers a distinct challenge for physicians treating neonatal sepsis. The presentation of septic shock in children is different compared with adult patients, presenting the bedside clinician with a diagnostic and therapeutic challenge.134 The differential for neonates presenting with shock in the first month of life must include undiagnosed congenital heart disease with a ductal-dependent lesion. In addition to the shock treatment listed earlier, prostaglandin E1 (PGE1) should be initiated immediately when there is any concern for a potential ductal-dependent systemic lesion.58 PGE1 dosing is 0.05 to 0.10 mg/kg per minute, adjusted until adequate systemic perfusion is obtained. An echocardiogram should be obtained to confirm diagnosis and determine further surgical intervention needs. Shock in infants is frequently complicated by pulmonary hypertension. Goals for treatment include the primary principles of shock treatment: maintenance of normothermia, optimization of electrolytes and glucose, and ensuring adequate intravascular volume. Vasoactive medications should be used to support cardiac output. Mechanical ventilation goals focus on maintaining oxygenation and optimizing lung volumes to maintain functional residual volume and limit pulmonary vascular resistance. Some infants may benefit from high-frequency ventilation. The addition of pulmonary vasodilators such as inhaled nitric oxide and/ or sildenafil may be of benefit. If medical management fails, cannulation to extracorporeal membrane oxygenation support may be required.135 Neonates may experience transient hypocalcemia in the first few days to weeks of life for a variety of reasons.136 In the setting of shock, neonates are at increased risk for hypocalcemia and myocardial dysfunction. Neonatal myocardium differs from older children and adults, as it has fewer contractile elements and requires higher extracellular calcium concentrations for contractility.137–139 The finding of hypocalcemia in infants who

present in shock should raise the suspicion of left ventricular dysfunction, which is reversible with calcium replacement. If hypocalcemia persists following resolution of shock in a neonate, further evaluation for etiology of hypocalcemia is warranted. Due to low glycogen stores, infants and very young children may quickly develop hypoglycemia during periods of shock due to increased metabolic requirements. Thus, close monitoring of blood glucose levels and appropriate supplementation to maintain euglycemia is imperative.58 In addition, infants and older children with metabolic and mitochondrial disorders may also be highly sensitive to reductions in energy substrates. The goals of therapy in these disease states during periods of stress are to provide enough substrate to prevent catabolism, to ensure adequate hydration, and to correct metabolic derangements. This may include the use of total parenteral nutrition, intravenous lipids, and concurrent use of insulin therapy with high dextrose delivery.140

Specific Shock State Therapy Considerations Hypovolemic Shock Once the airway is ensured or established, measures to restore an effective circulating blood volume should begin immediately. Placement of an adequate intravenous or intraosseous catheter and rapid volume replacement are the most important therapeutic maneuvers to reestablish the circulation in hypovolemic shock. The choice of fluid depends on the nature of the loss. Hemorrhagic shock should be treated with transfusions of blood components, including emergency transfusion of uncrossmatched blood.141,142 Hematocrit is a poor early indicator of the severity of hemorrhage. Depending on the source, surgical intervention may be indicated to control the source of bleeding.143 If the etiology for hypovolemic shock involves ongoing fluid losses from chest tubes, biliary drains, bowel, capillary leak, or other bodily fluids, then resuscitation should be with isotonic or balanced crystalloid solutions.

360

SECTION IV



Pediatric Critical Care: Cardiovascular

Uncomplicated, promptly treated hypovolemic shock usually does not lead to a significant capillary injury and leak. However, severe, prolonged hypovolemic shock, traumatic shock with extensive soft-tissue injury, burn shock, or hypovolemic shock complicated by sepsis may seriously impair capillary integrity. Therefore, once adequate hemodynamics have been restored, fluid administration should be reduced unless there are ongoing fluid losses. Continued assessment of hemodynamic status and vascular volume is essential to guide further therapy. If the patient does not show improvement after several fluid boluses, more monitoring and reevaluation of the diagnosis may be required. Causes of ongoing vascular depletion and other causes of refractory shock should be determined, including unrecognized pneumothorax or pericardial effusion, intestinal ischemia (volvulus, intussusception, necrotizing enterocolitis), sepsis, myocardial dysfunction, adrenocortical insufficiency, and pulmonary hypertension. Arterial blood gases, hematocrit, serum electrolytes, glucose, and calcium should be reevaluated. Correction of acidosis, hypoxemia, or metabolic derangements is essential. Aerobic and anaerobic cultures should be obtained from blood and other appropriate sites and broad-spectrum parenteral antibiotic coverage should be started if sepsis is suspected.

Cardiogenic Shock The general supportive and pharmacologic measures used in the treatment of severe congestive heart failure or cardiogenic shock are listed in Box 34.5. The initial therapy for cardiogenic shock is supplemental oxygen and mechanical ventilation to reduce myocardial demand. Preload should be optimized to allow the patient to take advantage of Starling mechanisms. Correction of metabolic derangements (e.g., pH, glucose, calcium, magnesium) may

• BOX 34.5 General Principles in Management

of Cardiogenic Shock or Severe Congestive Heart Failure Minimize Myocardial Oxygenation Demands • • • •









Intubate and use mechanical ventilation Maintain normal core temperature Provide sedation Improve oxygen-carrying capacity by correcting anemia

Maximize Myocardial Performance • • • •

Correct dysrhythmias Administer prostaglandins if ductal-dependent lesion is suspected Optimize preload: fluid bolusesa Improve contractility: provide oxygen, mechanical ventilation, correct acidosis and other metabolic abnormalities, inotropic and lusitropic drugs • Reduce afterload: provide sedation and pain relief, correct hypothermia, positive pressure ventilation for left ventricular afterload reduction, appropriate use of vasodilators









Mechanical Circulatory Support • Extracorporeal membrane oxygenation • Ventricular assist devices



Heart Transplantation a However, if evidence of congestive heart failure on clinical exam, appropriate salt and water restriction as well as appropriate use of venodilators and/or diuretics are indicated.

enhance cardiac function, and pharmacologic interventions are usually necessary to improve cardiac function (see Tables 34.1 and 34.2). In addition to inotropic effects, catecholamines also possess chronotropic properties and have complex effects on vascular beds of the various organs of the body. Consequently, the choice of an agent depends both on the desired myocardial and peripheral vascular effects. Vasodilators should be considered in shock states in which cardiac dysfunction is associated with elevated ventricular filling pressures, elevated systemic vascular resistance, and normal or near-normal systemic arterial blood pressure. Occasionally, the combination of vasodilator and inotropic therapy results in hemodynamic improvement not attainable with either approach alone. Vasodilators improve cardiac performance and lessen clinical symptoms via arterial and venous smooth muscle relaxation. Arterial relaxation should increase ejection fraction, increase stroke volume, and decrease end-systolic left ventricular volume. Some evidence suggests that vasodilator drugs increase left ventricular compliance, which should improve diastolic function.4 Venous relaxation should shift blood into the periphery and reduce right and left ventricular diastolic volume, having beneficial effects on both pulmonary and systemic capillary pressure. This should be reflected in decreased edema, reduced myocardial wall stress, and improved diastolic perfusion of the myocardium. Intravenous vasodilators with rapid onsets of action and short half-lives are preferred for treatment of cardiogenic shock. Selection of a vasodilator agent would depend on its principal hemodynamic effects and the patient’s specific hemodynamic abnormalities. Factors that increase systemic vascular resistance—such as hypothermia, acidosis, hypoxia, pain, and anxiety—should be treated before vasodilator drugs are considered. Inamrinone, milrinone, and enoximone belong to a class of nonglycoside, nonsympathomimetic inotropic agents that act via potent and selective inhibition of phosphodiesterase.144 Inamrinone and milrinone are particularly useful in the treatment of cardiogenic shock because they improve diastolic function (lusitropy), increase contractility, and reduce afterload by peripheral vasodilation without a consistent increase in heart rate or myocardial oxygen consumption. Both drugs have relatively long half-lives; they should be used cautiously in the presence of hypovolemia and/or hypotension. Milrinone is preferred over inamrinone because of inamrinone’s tendency to cause thrombocytopenia. The use of milrinone has been shown to be effective in decreasing the risk of low cardiac output syndrome in postoperative cardiac patients.145 Afterload reduction of the failing right ventricle is also an important management concern in cardiopulmonary disorders associated with right heart strain and/or failure, including congenital heart disease, acute respiratory distress syndrome, bronchopulmonary dysplasia, and other chronic pulmonary disorders. The ability of the right ventricle to respond to the increased pulmonary vascular resistance seen in these situations often determines outcome. Therefore, measures to decrease pulmonary vascular resistance have become more common in the treatment of many seriously ill pediatric patients, including supplemental oxygen, hyperventilation, metabolic and respiratory alkalosis, inhaled nitric oxide, PGE1, prostacyclin, analgesia, and sedation.146–148 If medical management fails to improve shock states secondary to cardiac failure, use of mechanical support—such as a ventricular assist device or extracorporeal membrane oxygenation (ECMO)— may be indicated. Cardiac transplantation has become an important



CHAPTER 34 Shock States

361

tool for treating patients with severe myocardial dysfunction who otherwise would die of their heart disease.

and myxedema coma. Appropriate replacement with hydrocortisone and thyroid hormone, respectively, is imperative.

Obstructive Shock

Septic Shock

Identification of the cause of obstructive shock will guide therapy. The treatment for the following obstructive shock states will be highlighted: cardiac tamponade, tension pneumothorax, ductaldependent heart lesions, and pulmonary embolism (PE). The definitive treatment for cardiac tamponade is the removal of pericardial fluid or air by pericardiocentesis, which may need surgical drainage by either thoracotomy or a subxiphoid limited approach. The removal of even a small volume of fluid can lead to a rapid improvement in blood pressure and cardiac output. Medical management is not a substitute for drainage but may avert a catastrophe until pericardiocentesis or surgical drainage can be safely performed. The principles of medical management include blood volume expansion to maintain venoatrial pressure gradients and inotropic agents. Treatment of a tension pneumothorax includes immediate needle decompression as a temporizing measure to reduce intrapleural pressure and restore venous return to the right side of the heart. Thoracostomy should then be performed to ensure continued decompression. As with cardiac tamponade, medical management with fluid boluses to maintain a venous return to the heart can be used to maintain systemic perfusion during decompression and thoracostomy. As noted earlier, infants with ductal-dependent cardiac lesions (such as critical aortic stenosis, aortic arch interruption, or juxtaductal coarctation of the aorta) depend on patency of the ductus arteriosus to provide adequate lower body perfusion. A high index of suspicion must be maintained for infants who present in shock in the first month of life; continuous PGE1 infusion should be initiated as the diagnostic evaluation is performed. Treatment of obstructive shock secondary to PE will depend on stability following initial resuscitation. Initial resuscitation is like all other forms of shock: intubation/mechanical ventilation, fluid resuscitation, and vasoactive support. If the patient is hemodynamically stable after this initial resuscitation, then immediate anticoagulation should be initiated with unfractionated heparin in cases with serious clinical concern for PE (given no contraindications to therapy). In patients who remain hemodynamically unstable after initial resuscitation, the use of thrombolytic therapy should be initiated or, if contraindicated, emergent embolectomy should be considered.

Evidence-based algorithms for the resuscitation of septic children and neonates are easy to use and have been shown to improve outcomes across diverse patient populations.58,149 The emphasis during the first hour of resuscitation is directed to age-appropriate goals of heart rate, blood pressure, perfusion pressure (mean arterial pressure, central venous pressure), and capillary refill time of 2 seconds or less. PRBCs may be used if the hematocrit is less than 30% because RBC transfusion increases oxygen delivery to the tissues. However, the expansion of oxygen-carrying capacity may not improve oxygen consumption.150 Transfusion of PRBCs as part of a strategy to increase mixed Svo2 to greater than 70% resulted in improved outcome in pediatric septic shock.62 Timely administration of broad-spectrum antibiotic therapy is a crucial component in the septic shock bundle for the treatment of septic shock. Whenever possible, blood, urine, and samples from other potentially infected sites should be sent for culture and susceptibility testing before initiation of antibiotic therapy. However, obtaining these cultures should never delay appropriate empiric antimicrobial therapy. Delay in treating with appropriate antibiotics is associated with increased mortality and prolonged organ failure. Source control is imperative; therefore, removal or control of microorganisms by surgical debridement and drainage may need to be considered in some cases.

Distributive Shock Treatment of anaphylaxis includes the administration of intramuscular or subcutaneous epinephrine, albuterol for wheezing, steroids, H2-blockers and removal of the offending agent. Fluid resuscitation is imperative for anaphylactic shock. Vasoactive medications may be necessary in patients unresponsive to fluid resuscitation. In the setting of spinal cord trauma or other causes of neurogenic shock, treatment is focused on immobilization of the spinal cord, neurosurgical consultation, and providing hemodynamic stability through fluid administration and initiation of a peripheral vasoconstrictor (phenylephrine is generally preferred given the predominant a1 effects). Other potential presentations of distributive shock include adrenal crisis

Other Therapies Adrenal insufficiency should be suspected in patients with refractory shock resulting from trauma (head or abdominal) or sepsis, who received etomidate, or who have a history of steroid use within the past 6 months.151,152 Direct damage to the hypothalamus, anterior pituitary, or adrenals may result in cortisol deficiency.153,154 Guidelines have been developed for critical illness– related corticosteroid insufficiency diagnosis and management in adults; however, no such data exists for pediatrics.155 Currently, hydrocortisone administration is recommended for the treatment of catecholamine-refractory pediatric septic shock in patients at risk for absolute adrenal insufficiency.58 Extracorporeal life support has been used to support patients of all ages with shock. Using the keyword “shock,” we searched the Extracorporeal Life Support Organization database to determine the use of ECMO in patients with refractory pediatric shock from 1985 through December 2018. Of the 3813 pediatric patients who underwent ECMO for refractory shock, 56% were 1 year old or younger. The overall mortality was 55%, suggesting that ECMO is a potentially lifesaving tool in refractory septic shock. Therefore, patients in refractory shock may benefit from a timely transfer to an ECMO referral center for further evaluation. Thrombocytopenia-associated multiple-organ failure is among a spectrum of syndromes associated with disseminated microvascular thromboses that include disseminated intravascular coagulopathy, thrombotic thrombocytopenic purpura (TTP), and hemolytic uremic syndrome (HUS).156 Therapeutic plasma exchange (TPE) is a standard therapy for patients with TTP/HUS because it replaces a disintegrin and metalloprotease with thrombospondin motifs 1, type 13 (ADAMTS-13), and removes unusually large molecular weight multimers of von Willibrand factor

362

SECTION IV



Pediatric Critical Care: Cardiovascular

and ADAMTS-13 inhibitors.157,158 In a prospective observational study in 9 PICUs, the use of therapeutic plasma exchange in sepsis-induced TAMOF was associated with a decrease in organ dysfunction and a 55% lower adjusted relative risk of 28-day mortality as compared with those not receiving TPE.159 TPE could potentially have improved sepsis-induced organ dysfunction by removing inflammatory mediators, reducing antifibrinolytic molecules, replenishing anticoagulant proteins, and restoring ADAMTS-13 activity to mitigate the dysregulated inflammation, coagulation, and fibrinolytic pathways of sepsis.159

Summary Shock is a life-threatening condition that has a myriad of causes. In order to survive shock, recognition and resuscitative efforts must be achieved early, the etiology elucidated, and ongoing monitoring and therapy instituted. The astute clinician who recognizes shock, promptly establishes therapy, and continuously assesses response during treatment offers the child the best chance for a quality survival.

Key References Arikan AA, Zappitelli M, Goldstein SL, Naipaul A, Jefferson LS, Loftis LL. Fluid overload is associated with impaired oxygenation and morbidity in critically ill children. Pediatr Crit Care Med. 2012;13(3):253-258. Carcillo JA, Kuch BA, Han YY, et al. Mortality and functional morbidity after use of PALS/APLS by community physicians. Pediatrics. 2009;124(2):500-508. Davis AL, Carcillo JA, Aneja RK, et al. The American College of Critical Care Medicine clinical practice parameters for hemodynamic support of pediatric and neonatal septic shock: executive summary. Pediatr Crit Care Med. 2017;18(9):884-890. Rhodes A, Evans LE, Alhazzani W, et al. Surviving Sepsis Campaign: international guidelines for management of sepsis and septic shock: 2016. Crit Care Med. 2016. Epub ahead of print. Weiss SL, Peters MJ, Alhazzani W, et al. Surviving Sepsis Campaign international guidelines for the management of septic shock and sepsisassociated organ dysfunction in children. Pediatr Crit Care Med. 2020;21(2):e52–e106.

The full reference list for this chapter is available at ExpertConsult.com.

e1







































































































1. Weiss SL, Selak MA, Tuluc F, et al. Mitochondrial dysfunction in peripheral blood mononuclear cells in pediatric septic shock. Pediatr Crit Care Med. 2015;16(1):E4-E12. 2. Fink MP. Bench-to-bedside review: cytopathic hypoxia. Crit Care (London, England). 2002;6(6):491-499. 3. Arques S, Ambrosi P, Gelisse R, Roux E, Lambert M, Habib G. Prevalence of angiographic coronary artery disease in patients hospitalized for acute diastolic heart failure without clinical and electrocardiographic evidence of myocardial ischemia on admission. Am J Cardiol. 2004;94(1):133-135. 4. Gaasch WH. Diagnosis and treatment of heart failure based on left ventricular systolic or diastolic dysfunction. JAMA. 1994;271(16): 1276-1280. 5. Ginghina C, Beladan CC, Iancu M, Calin A, Popescu BA. Respiratory maneuvers in echocardiography: a review of clinical applications. Cardiovasc Ultrasound. 2009;7:42. 6. Cohen J. The immunopathogenesis of sepsis. Nature. 2002;420(6917): 885-891. 7. Ceneviva G, Paschall JA, Maffei F, Carcillo JA. Hemodynamic support in fluid-refractory pediatric septic shock. Pediatrics. 1998; 102(2):E19. 8. Carcillo JA, Pollack MM, Ruttimann UE, Fields AI. Sequential physiologic interactions in pediatric cardiogenic and septic shock. Crit Care Med. 1989;17(1):12-16. 9. Parrillo JE, Parker MM, Natanson C, et al. Septic shock in humans. Ann Intern Med. 1990;113(3):227-242. 10. Cairns CB. Rude unhinging of the machinery of life: metabolic approaches to hemorrhagic shock. Curr Opin Crit Care. 2001;7(6): 437-443. 11. Fink MP. Cytopathic hypoxia: is oxygen use impaired in sepsis as a result of an acquired intrinsic derangement in cellular respiration? Crit Care Clin. 2002;18(1):165-175. 12. Bruins MJ, Deutz NEP, Soeters PB. Aspects of organ protein, amino acid and glucose metabolism in a porcine model of hypermetabolic sepsis. Clin Sci. 2003;104(2):127-141. 13. Maitra SR, Wojnar MM, Lang CH. Alterations in tissue glucose uptake during the hyperglycemic and hypoglycemic phases of sepsis. Shock. 2000;13(5):379-385. 14. Crouser ED, Dorinsky PM. Metabolic consequences of sepsis. Clin Chest Med. 1996;17(2):249-261. 15. Chinkes D, Demelo E, Zhang X-J, Traber DL, Wolfe RR. Increased plasma glucose clearance in sepsis is due to increased exchange between plasma and interstitial fluid. Shock. 1995;4(5):356-360. 16. Reinelt H, Radermacher P, Fischer G, et al. Effects of a dobutamineinduced increase in splanchnic blood flow on hepatic metabolic activity in patients with septic shock. Anesthesiology. 1997;86(4): 818-824. 17. Jabbar A, Pingitore A, Pearce SHS, Zaman A, Iervasi G, Razvi S. Thyroid hormones and cardiovascular disease. Nat Rev Cardiol. 2017; 14(1):39-55. 18. Radman M, Portman MA. Thyroid hormone in the pediatric intensive care unit. J Pediatr Intensive Care. 2016;5(4):154-161. 19. Annane D, Pastores SM, Rochwerg B, et al. Guidelines for the diagnosis and management of Critical Illness-Related Corticosteroid Insufficiency (CIRCI) in critically Ill patients (Part I): Society Of Critical Care Medicine (SCCM) and European Society Of Intensive Care Medicine (ESICM) 2017. Intensive Care Med. 2017;43(12):1751-1763. 20. Marik PE. Steroids for sepsis: yes, no or maybe. J Thorac Dis. 2018;10(suppl 9):S1070-S1073. 21. Finsterer J. Mitochondriopathies. Eur J Neurol. 2004;11(3):163-186. 22. Bozi LHM, Campos JC, Zambelli VO, Ferreira ND, Ferreira JCB. Mitochondrially-targeted treatment strategies. Mol Aspects Med. 2020;71:100836. 23. Saudubray J-M, Garcia-Cazorla À. Inborn errors of metabolism overview: pathophysiology, manifestations, evaluation, and management. Pediatr Clin North Am. 2018;65(2):179-208.

24. Hüttemann M, Helling S, Sanderson TH, et al. Regulation of mitochondrial respiration and apoptosis through cell signaling: cytochrome C oxidase and cytochrome C in ischemia/reperfusion injury and inflammation. Biochim Biophys Acta. 2012;1817(4):598-609. 25. Proulx F, Joyal JS, Mariscalco MM, Leteurtre S, Leclerc F, Lacroix J. The pediatric multiple organ dysfunction syndrome. Pediatr Crit Care Med. 2009;10(1):12-22. 26. Graciano AL, Balko JA, Rahn DS, Ahmad N, Giroir BP. The Pediatric Multiple Organ Dysfunction Score (P-MODS): development and validation of an objective scale to measure the severity of multiple organ dysfunction in critically ill children. Crit Care Med. 2005;33(7):1484-1491. 27. Leclerc F, Leteurtre S, Duhamel A, et al. Cumulative influence of organ dysfunctions and septic state on mortality of critically ill children. Am J Respir Crit Care Med. 2005;171(4):348-353. 28. Kraft R, Herndon DN, Finnerty CC, Shahrokhi S, Jeschke MG. Occurrence of multiorgan dysfunction in pediatric burn patients. Ann Surg. 2014;259(2):381-387. 29. Lacroix J, Cotting J, Network Paliasip. Severity of illness and organ dysfunction scoring in children. Pediatr Crit Care Med. 2005;6(suppl 3):S126-S134. 30. Flancbaum L, Choban PS, Dasta JF. Quantitative effects of low-dose dopamine on urine output in oliguric surgical intensive care unit patients. Surv Anesthesiol. 1994;38(5):276-277. 31. Myles PS, Buckland MR, Schenk NJ, et al. Effect of “renal-dose” dopamine on renal function following cardiac surgery. Surv Anesthesiol. 1994;38(2):97. 32. Jones D, Bellomo R. Renal-dose dopamine: from hypothesis to paradigm to dogma to myth and, finally, superstition? J Intensive Care Med. 2005;20(4):199-211. 33. Schenarts PJ, Sagraves SG, Bard MR, et al. Low-dose dopamine: a physiologically based review. Curr Surg. 2006;63(3):219-225. 34. Sutherland SM, Zappitelli M, Alexander SR, et al. Fluid overload and mortality in children receiving continuous renal replacement therapy: the prospective pediatric continuous renal replacement therapy registry. Am J Kidney Dis. 2010;55(2):316-325. 35. Deitch EA. The role of intestinal barrier failure and bacterial translocation in the development of systemic infection and multiple organ failure. Arch Surg. 1990;125(3):403-404. 36. Fink MP. Gastrointestinal mucosal injury in experimental models of shock, trauma, and sepsis. Crit Care Med. 1991;19(5):627-641. 37. Reilly PM, Bulkley GB. Vasoactive mediators and splanchnic perfusion. Crit Care Med. 1993;21(suppl):S55-S68. 38. Cerra FB. Selective gut decontamination reduces nosocomial infections and length of stay but not mortality or organ failure in surgical intensive care unit patients. Arch Surg. 1992;127(2):163-167. 39. Cockerill FR, Muller SR, Anhalt JP, et al. Prevention of infection in critically ill patients by selective decontamination of the digestive tract. Ann Intern Med. 1992;117(7):545-553. 40. López-Herce J, Menca S, Sánchez C, Santiago MJ, Bustinza A, Vigil D. Postpyloric enteral nutrition in the critically ill child with shock: a prospective observational study. Nutr J. 2008;7(1):6. 41. López-Herce J, Santiago MJ, Sánchez C, Menca S, Carrillo A, Vigil D. Risk factors for gastrointestinal complications in critically ill children with transpyloric enteral nutrition. Eur J Clin Nutr. 2007;62(3):395-400. 42. Plummer MP, Blaser AR, Deane AM. Stress ulceration: prevalence, pathology and association with adverse outcomes. Crit Care (London, England). 2014;18(2):213. 43. Marik PE, Vasu T, Hirani A, Pachinburavan M. Stress ulcer prophylaxis in the new millennium: a systematic review and meta-analysis. Crit Care Med. 2010;38(11):2222-2228. 44. Kenny GP, Notley SR, Gagnon D. Direct calorimetry: a brief historical review of its use in the study of human metabolism and thermoregulation. Eur J Appl Physiol. 2017;117(9):1765-1785. 45. Fick AE. úber die messung des blutquantums in der herzventrikeln. In: Verhandlungen Der Physikalisch-Medizinische Gesellschaft Zu Würzburg. Verlag Der Physik-Med. Gesellschaft; 1870: XVI–XVII.

References

e2



















































68. Top APC, Tasker RC, Ince C. The microcirculation of the critically ill pediatric patient. Crit Care (London, England). 2011;15(2):213. 69. Hernandez G, Regueira T, Bruhn A, et al. Relationship of systemic, hepatosplanchnic, and microcirculatory perfusion parameters with 6-hour lactate clearance in hyperdynamic septic shock patients: an acute, clinical-physiological, pilot study. Ann Intensive Care. 2012;2(1):44. 70. Van Haren FMP, Sleigh JW, Pickkers P, Van Der Hoeven JG: Gastrointestinal perfusion in septic shock. Anaesth Intensive Care. 2007;35(5):679-694. 71. Sood BG, Mclaughlin K, Cortez J. Near-infrared spectroscopy: applications in neonates. Semin Fetal Neonatal Med. 2015;20(3): 164-172. 72. Ghanayem NS, Wernovsky G, Hoffman GM. Near-infrared spectroscopy as a hemodynamic monitor in critical illness. Pediatr Crit Care Med. 2011;12(suppl 4):S27-S32. 73. Chendrasekhar A, Pillai S, Fagerli JC, Barringer LS, Dulaney J, Timberlake GA. Rectal Ph measurement in tracking cardiac performance in a hemorrhagic shock model. J Trauma. 1996;40(6): 963-967. 74. Chung KK, Ryan KL, Rickards CA, et al. Progressive reduction in central blood volume is not detected by sublingual capnography. Shock (Augusta, GA). 2012;37(6):586-591. 75. Marik PE. Sublingual capnometry: a non-invasive measure of microcirculatory dysfunction and tissue hypoxia. Physiol Meas. 2006;27(7): R37-R47. 76. Arakaki LSL, Schenkman KA, Ciesielski WA, Shaver JM. Muscle oxygenation measurement in humans by noninvasive optical spectroscopy and locally weighted regression. Anal Chim Acta. 2013; 785:27-33. 77. Kopterides P, Theodorakopoulou M, Ilias I, et al. Interrelationship between blood and tissue lactate in a general intensive care unit: a subcutaneous adipose tissue microdialysis study on 162 critically ill patients. J Crit Care. 2012;27(6):742.e9-742.e18. 78. Burša F, Pleva L, Máca J, Sklienka P, Ševčík P. Tissue ischemia microdialysis assessments following severe traumatic haemorrhagic shock: lactate/pyruvate ratio as a new resuscitation end point? BMC Anesthesiol. 2014;14(1):118. 79. Nadeau RG, Groner W, Winkelman JW, et al. Orthogonal polarization spectral imaging: a new method for study of the microcirculation nature medicine. Nat Med. 1999;5(10):1209-1212. 80. Top APC, Ince C, De Meij N, Van Dijk M, Tibboel D. Persistent low microcirculatory vessel density in nonsurvivors of sepsis in pediatric intensive care. Crit Care Med. 2011;39(1):8-13. 81. Branthwaite MA, Bradley RD. Measurement of cardiac output by thermal dilution in man. J Appl Physiol. 1968;24(3):434-438. 82. Connors AF, Speroff T, Dawson NV, et al. The effectiveness of right heart catheterization in the initial care of critically III patients. JAMA. 1996;276(11):889-897. 83. Connors Jr AF, Dawson NV, Shaw PK, Montenegro HD, Nara AR, Martin L. Hemodynamic status in critically ill patients with and without acute heart disease. Chest. 1990;98(5):1200-1206. 84. Shah MR, Hasselblad V, Stevenson LW, et al. Impact of the pulmonary artery catheter in critically ill patients: meta-analysis of randomized clinical trials. JAMA. 2005;294(13):1664-1670. 85. Marik PE. Noninvasive cardiac output monitors: a state-of the-art review. J Cardiothorac Vasc Anesth. 2013; 27(1):121-134. 86. Marik PE, Baram M. Noninvasive hemodynamic monitoring in the intensive care unit. Crit Care Clin. 2007;23(3):383-400. 87. Han YY, Carcillo JA, Dragotta MA, et al. Early reversal of pediatricneonatal septic shock by community physicians is associated with improved outcome. Pediatrics. 2003;112(4):793-799. 88. Carcillo JA, Kuch BA, Han YY, et al. Mortality and functional morbidity after use of PALS/APLS by community physicians. Pediatrics. 2009;124(2):500-508. 89. Verderber A, Gallagher KJ. Effects of bathing, passive range-of-motion exercises, and turning on oxygen consumption in healthy men and women. Am J Crit Care. 1994;3(5):374-381.



















































46. Broder G, Weil MH. Excess lactate: an index of reversibility of shock in human patients. Science. 1964;143(3613):1457-1459. 47. Andersen LW, Mackenhauer J, Roberts JC, Berg KM, Cocchi MN, Donnino MW. Etiology and therapeutic approach to elevated lactate levels. Mayo Clin Proc. 2013;88(10):1127-1140. 48. Ducrocq N, Kimmoun A, Levy B. Lactate or Scvo2 as an endpoint in resuscitation of shock states? Minerva Anestesiol. 2013;79(9):10491058. 49. Batra P, Dwivedi AK, Thakur N. Bedside ABG, electrolytes, lactate and procalcitonin in emergency pediatrics. Int J Crit Illn Inj Sci. 2014;4(3):247-252. 50. Hatherill M, McIntyre AG, Wattie M, Murdoch IA. Early hyperlactataemia in critically ill children. Intensive Care Med. 2000;26(3): 314-318. 51. Scott HF, Donoghue AJ, Gaieski DF, Marchese RF, Mistry RD. The utility of early lactate testing in undifferentiated pediatric systemic inflammatory response syndrome. Acad Emerg Med. 2012;19(11): 1276-1280. 52. Abdalla MO, Aneja R, Dean D, et al. Synthesis and characterization of noscapine loaded magnetic polymeric nanoparticles. J Magn Magn Mater. 2010;322(2):190-196. 53. Lawton L, Crouch R, Voegeli D. Is lactate an effective clinical marker of outcome for children with major trauma? A literature review. Int Emerg Nurs. 2016;28:39-45. 54. Jat KR, Jhamb U, Gupta VK. Serum lactate levels as the predictor of outcome in pediatric septic shock. Indian J Crit Care Med. 2011; 15(2):102-107. 55. Duke TD, Butt W, South M. Predictors of mortality and multiple organ failure in children with sepsis. Intensive Care Med. 1997;23(6):684-692. 56. Bloom B, Pott J, Freund Y, Grundlingh J, Harris T: The agreement between abnormal venous lactate and arterial lactate in the ED: a retrospective chart review. Am J Emerg Med. 2014, 32(6):596-600. 57. Walley KR. Use of central venous oxygen saturation to guide therapy. Am J Respir Crit Care Med. 2011;184(5):514-520. 58. Davis AL, Carcillo JA, Aneja RK, et al. The American College of Critical Care Medicine clinical practice parameters for hemodynamic support of pediatric and neonatal septic shock: executive summary. Pediatr Crit Care Med. 2017;18(9):884-890. 59. Chawla LS, Zia H, Gutierrez G, Katz NM, Seneff MG, Shah M. Lack of equivalence between central and mixed venous oxygen saturation. Chest. 2004;126(6):1891-1896. 60. Brierley J, Peters MJ. Distinct hemodynamic patterns of septic shock at presentation to pediatric intensive care. Pediatrics. 2008;122(4): 752-759. 61. Deep A, Goonasekera CD, Wang Y, Brierley J. Evolution of haemodynamics and outcome of fluid-refractory septic shock in children. Intensive Care Med. 2013;39(9):1602-1609. 62. De Oliveira CF, De Oliveira DS, Gottschald AF, et al. ACCM/PALS haemodynamic support guidelines for paediatric septic shock: an outcomes comparison with and without monitoring central venous oxygen saturation. Intensive Care Med. 2008;34(6):1065-1075. 63. Fink MP. Cytopathic hypoxia. Is oxygen use impaired in sepsis as a result of an acquired intrinsic derangement in cellular respiration? Crit Care Clin. 2002;18(1):165-175. 64. Fink MP. Cytopathic hypoxia. Mitochondrial dysfunction as mechanism contributing to organ dysfunction in sepsis. Crit Care Clin. 2001;17(1):219-237. 65. Pope JV, Jones AE, Gaieski DF, Arnold RC, Trzeciak S, Shapiro NI. Multicenter study of central venous oxygen saturation (Scvo(2)) as a predictor of mortality in patients with sepsis. Ann Emerg Med. 2010;55(1):40-46.e1. 66. Donati A, Tibboel D, Ince C. Towards integrative physiological monitoring of the critically ill: from cardiovascular to microcirculatory and cellular function monitoring at the bedside. Crit Care (London, England). 2013;17(suppl 1):S5. 67. Cestero RF, Dent DL. Endpoints of resuscitation. Surg Clin North Am. 2015;95(2):319-336.

e3











































108. Holcomb JB, Del Junco DJ, Fox EE, et al. The prospective, observational, multicenter, major trauma transfusion (PROMMTT) study: comparative effectiveness of a time-varying treatment with competing risks. JAMA Surg. 2013;148(2):127-136. 109. Holcomb JB, Tilley BC, Baraniuk S, et al. Transfusion of plasma, platelets, and red blood cells in A 1:1:1 Vs A 1:1:2 ratio and mortality in patients with severe trauma: the PROPPR randomized clinical trial. JAMA. 2015;313(5):471-482. 110. Edwards MJ, Lustik MB, Clark ME, Creamer KM, Tuggle D. The effects of balanced blood component resuscitation and crystalloid administration in pediatric trauma patients requiring transfusion in Afghanistan and Iraq 2002 to 2012. J Trauma Acute Care Surg. 2015;78(2):330-335. 111. Nosanov L, Inaba K, Okoye O, et al. The impact of blood product ratios in massively transfused pediatric trauma patients. Am J Surg. 2013;206(5):655-660. 112. Cannon JW, Johnson MA, Caskey RC, Borgman MA, Neff LP. High ratio plasma resuscitation does not improve survival in pediatric trauma patients. J Trauma Acute Care Surg. 2017;83(2):211-217. 113. Livingston MH, Singh S, Merritt NH. Massive transfusion in paediatric and adolescent trauma patients: incidence, patient profile, and outcomes prior to a massive transfusion protocol. Injury. 2014;45(9):1301-1306. 114. Acheampong A, Vincent J-L. A positive fluid balance is an independent prognostic factor in patients with sepsis. Crit Care. 2015; 19(1):251. 115. Arikan AA, Zappitelli M, Goldstein SL, Naipaul A, Jefferson LS, Loftis LL. Fluid overload is associated with impaired oxygenation and morbidity in critically ill children. Pediatr Crit Care Med. 2012;13(3):253-258. 116. Besen BAMP, Gobatto ALN, Melro LMG, Maciel AT, Park M. Fluid and electrolyte overload in critically ill patients: an overview. World J Crit Care Med. 2015;4(2):116-129. 117. Flori HR, Church G, Liu KD, Gildengorin G, Matthay MA. Positive fluid balance is associated with higher mortality and prolonged mechanical ventilation in pediatric patients with acute lung injury. Crit Care Res Pract. 2011;2011:854142. 118. Gan H, Cannesson M, Chandler JR, Ansermino JM. Predicting fluid responsiveness in children. Anesth Analg. 2013;117(6):13801392. 119. Weber T, Wagner T, Neumann K, Deusch E. Low predictability of three different noninvasive methods to determine fluid responsiveness in critically ill children. Pediatr Crit Care Med. 2015;16(3):E89-E94. 120. Choong K, Bohn D, Fraser DD, et al. Vasopressin in pediatric vasodilatory shock. Am J Respir Crit Care Med. 2009;180(7):632-639. 121. Russell JA, Walley KR, Singer J, et al. Vasopressin versus norepinephrine infusion in patients with septic shock. N Engl J Med. 2008;358(9):877-887. 122. Polito A, Parisini E, Ricci Z, Picardo S, Annane D. Vasopressin for treatment of vasodilatory shock: an ESICM systematic review and meta-analysis. Intensive Care Med. 2011;38(1):9-19. 123. Moffett BS, Orellana RN. Use of fenoldopam to increase urine output in a patient with renal insufficiency secondary to septic shock: a case report. Pediatr Crit Care Med. 2006;7(6):600-602. 124. Ricci Z, Stazi GV, Di Chiara L, et al. Fenoldopam in newborn patients undergoing cardiopulmonary bypass: controlled clinical trial. Interact Cardiovasc Thorac Surg. 2008;7(6):1049-1053. 125. Ramaswamy KN, Singhi S, Jayashree M, Bansal A, Nallasamy K. Double-blind randomized clinical trial comparing dopamine and epinephrine in pediatric fluid-refractory hypotensive septic shock. Pediatr Crit Care Med. 2016;17(11):E502-E512. 126. Ventura AM, Shieh HH, Bousso A, et al. Double-blind prospective randomized controlled trial of dopamine versus epinephrine as first-line vasoactive drugs in pediatric septic shock. Crit Care Med. 2015;43(11):2292-2302. 127. Avni T, Lador A, Lev S, Leibovici L, Paul M, Grossman A. Vasopressors for the treatment of septic shock: systematic review and meta-analysis. Plos One. 2015;10(8):E0129305.









































90. El-Khatib MF, Chatburn RL, Potts DL, Blumer JL, Smith PG. Mechanical ventilators optimized for pediatric use decrease work of breathing and oxygen consumption during pressure-support ventilation. Crit Care Med. 1994;22(12):1942-1948. 91. Manthous CA, Hall JB, Kushner R, Schmidt GA, Russo G, Wood LD. The effect of mechanical ventilation on oxygen consumption in critically Ill patients. Am J Respir Crit Care Med. 1995;151(1): 210-214. 92. Thiagarajan RR, Coleman DM, Bratton SL, Watson RS, Martin LD. Inspiratory work of breathing is not decreased by flowtriggered sensing during spontaneous breathing in children receiving mechanical ventilation: a preliminary report. Pediatr Crit Care Med. 2004;5(4):375-378. 93. Carcillo JA. Role of early fluid resuscitation in pediatric septic shock. JAMA. 1991;266(9):1242-1245. 94. Weiss SL, Peters MJ, Alhazzani W, et al. Surviving Sepsis Campaign international guidelines for the management of septic shock and sepsis-associated organ dysfunction in children. Pediatr Crit Care Med. 2020;21(2):e52–e106. 94a. Rhodes A, Evans LE, Alhazzani W, et al. Surviving Sepsis Campaign: international guidelines for management of sepsis and septic shock: 2016. Crit Care Med. 2017;45(3):486-552. 95. Rivers E, Nguyen B, Havstad S, et al. Early goal-directed therapy in the treatment of severe sepsis and septic shock. N Engl J Med. 2001;345(19):1368-1377. 96. Annane D, Siami S, Jaber S, et al. Effects of fluid resuscitation with colloids Vs crystalloids on mortality in critically ill patients presenting with hypovolemic shock: the CRISTAL randomized trial. JAMA. 2013;310(17):1809-1817. 97. Caironi P, Tognoni G, Masson S, et al. Albumin replacement in patients with severe sepsis or septic shock. N Engl J Med. 2014; 370(15):1412-1421. 98. Dingankar AR, Cave DA, Anand V, et al. Albumin 5% versus crystalloids for fluid resuscitation in children after cardiac surgery. Pediatr Crit Care Med. 2018;19(9):846-853. 99. Stenson EK, Cvijanovich NZ, Anas N, et al. Hyperchloremia is associated with complicated course and mortality in pediatric patients with septic shock. Pediatr Crit Care Med. 2018;19(2): 155-160. 100. Barhight MF, Lusk J, Brinton J, et al. Hyperchloremia is independently associated with mortality in critically ill children who ultimately require continuous renal replacement Therapy. Pediatr Nephrol. 2018;33(6):1079-1085. 101. Krajewski ML, Raghunathan K, Paluszkiewicz SM, Schermer CR, Shaw AD. Meta-analysis of high-versus low-chloride content in perioperative and critical care fluid resuscitation. Br J Surg. 2015;102(1):24-36. 102. Neyra JA, Canepa-Escaro F, Li X, et al. Acute kidney injury in critical illness study G: association of hyperchloremia with hospital mortality in critically ill septic patients. Crit Care Med. 2015;43(9):1938-1944. 103. Noritomi DT, Soriano FG, Kellum JA, et al. Metabolic acidosis in patients with severe sepsis and septic shock: a longitudinal quantitative study. Crit Care Med. 2009;37(10):2733-2739. 104. Suetrong B, Pisitsak C, Boyd JH, Russell JA, Walley KR. Hyperchloremia and moderate increase in serum chloride are associated with acute kidney injury in severe sepsis and septic shock patients. Crit Care. 2016;20(1):315. 105. Semler MW, Self WH, Wanderer JP, et al. Balanced crystalloids versus saline in critically ill adults. N Engl J Med. 2018;378(9): 829-839. 106. Weiss SL, Keele L, Balamuth F, et al. Crystalloid fluid choice and clinical outcomes in pediatric sepsis: a matched retrospective cohort study. J Pediatr. 2017;182:304-310.e10. 107. Neff LP, Cannon JW, Morrison JJ, Edwards MJ, Spinella PC, Borgman MA. Clearly defining pediatric massive transfusion: cutting through the fog and friction with combat data. J Trauma Acute Care Surg. 2015;78(1):22-28; discussion 28-29.

e4

































146. Kinsella JP, Toews WH, Henry D, Abman SH. Selective and sustained pulmonary vasodilation with inhalational nitric oxide therapy in a child with idiopathic pulmonary hypertension. J Pediatr. 1993;122(5 Pt 1):803-806. 147. Pacher R, Globits S, Wutte M, et al. Beneficial hemodynamic effects of prostaglandin E1 infusion in catecholamine-dependent heart failure. Crit Care Med. 1994;22(7):1084-1090. 148. Rich GF, Murphy GD, Roos CM, Johns RA. Inhaled nitric oxide. Anesthesiology. 1993;78(6):1028-1035. 149. Brierley J, Carcillo JA, Choong K, et al. Clinical practice parameters for hemodynamic support of pediatric and neonatal septic shock: 2007 update from the American College of Critical Care Medicine. Crit Care Med. 2009;37(2):666-688. 150. Mink RB, Pollack MM. Effect of blood transfusion on oxygen consumption in pediatric septic shock. Crit Care Med. 1990; 18(10):1087-1091. 151. Chan CM, Mitchell AL, Shorr AF. Etomidate is associated with mortality and adrenal insufficiency in sepsis: a meta-analysis. Crit Care Med. 2012;40(11):2945-2953. 152. Menon K, Ward RE, Lawson ML, et al. A prospective multicenter study of adrenal function in critically ill children. Am J Respir Crit Care Med. 2010;182(2):246-251. 153. Tanriverdi F, Schneider HJ, Aimaretti G, Masel BE, Casanueva FF, Kelestimur F. Pituitary dysfunction after traumatic brain injury: a clinical and pathophysiological approach. Endocr Rev. 2015;36(3): 305-342. 154. Alsayali MM, Atkin C, Rahim R, Niggemeyer LE, Doody O, Varma D. Traumatic adrenal gland injury: epidemiology and outcomes in a major Australian trauma center. Eur J Trauma Emerg Surg. 2010;36(6):567-572. 155. Annane D, Pastores SM, Rochwerg B, et al. Guidelines for the diagnosis and management of Critical illness-Related Corticosteroid Insufficiency (CIRCI) in critically ill patients (Part I): Society of Critical Care Medicine (SCCM) and European Society Of Intensive Care Medicine (ESICM) 2017. Crit Care Med. 2017;45(12): 2078-2088. 156. Nguyen TC, Carcillo JA. Bench-to-bedside review: thrombocytopeniaassociated multiple organ failure—a newly appreciated syndrome in the critically ill. Crit Care. 2006;10(6):235. 157. Bell WR, Braine HG, Ness PM, Kickler TS. Improved survival in thrombotic thrombocytopenic purpura-hemolytic uremic syndrome. Clinical experience in 108 patients. N Engl J Med. 1991; 325(6):398-403. 158. Veyradier A, Obert B, Houllier A, Meyer D, Girma JP. Specific von Willebrand factor-cleaving protease in thrombotic microangiopathies: a study of 111 cases. Blood. 2001;98(6):1765-1772. 159. Fortenberry JD, Nguyen T, Grunwell JR, et al. Therapeutic plasma exchange in children with thrombocytopenia-associated multiple organ failure: the thrombocytopenia-associated multiple organ failure network prospective experience. Crit Care Med. 2019;47(3): E173-E181.







































128. Vasu TS, Cavallazzi R, Hirani A, Kaplan G, Leiby B, Marik PE. Norepinephrine or dopamine for septic shock: systematic review of randomized clinical trials. J Intensive Care Med. 2012;27(3):172-178. 129. Belletti A, Musu M, Silvetti S, et al. Non-adrenergic vasopressors in patients with or at risk for vasodilatory shock. a systematic review and meta-analysis of randomized trials. Plos One. 2015; 10(11):E0142605. 130. Serpa Neto A, Nassar AP, Cardoso SO, et al. Vasopressin and terlipressin in adult vasodilatory shock: a systematic review and metaanalysis of nine randomized controlled trials. Crit Care. 2012; 16(4):R154. 131. James JH, Luchette FA, Mccarter FD, Fischer JE. Lactate is an unreliable indicator of tissue hypoxia in injury or sepsis. Lancet. 1999;354(9177):505-508. 132. Krejci V, Hiltebrand LB, Sigurdsson GH. Effects of epinephrine, norepinephrine, and phenylephrine on microcirculatory blood flow in the gastrointestinal tract in sepsis. Crit Care Med. 2006;34(5): 1456-1463. 133. Meier-Hellmann A, Reinhart K, Bredle DL, Specht M, Spies CD, Hannemann L. Epinephrine impairs splanchnic perfusion in septic shock. Crit Care Med. 1997;25(3):399-404. 134. Aneja RK, Carcillo JA. Differences between adult and pediatric septic shock. Minerva Anestesiol. 2011;77(10):986-992. 135. Steinhorn RH. Neonatal pulmonary hypertension. Pediatr Crit Care Med. 2010;11(suppl 2):S79-S84. 136. Thomas TC, Smith JM, White PC, Adhikari S. Transient neonatal hypocalcemia: presentation and outcomes. Pediatrics. 2012;129(6): E1461-E1467. 137. Chin TK, Friedman WF, Klitzner TS. Developmental changes in cardiac myocyte calcium regulation. Circ Res. 1990;67(3):574-579. 138. Baum VC, Palmisano BW. The immature heart and anesthesia. Anesthesiology. 1997;87(6):1529-1548. 139. Anderson PA. The heart and development. Semin Perinatol. 1996; 20(6):482-509. 140. Parikh S, Saneto R, Falk MJ, et al. A modern approach to the treatment of mitochondrial disease. Curr Treat Options Neurol. 2009; 11(6):414-430. 141. Kwan I, Bunn F, Roberts I, Committee WP-HTCS. Timing and volume of fluid administration for patients with bleeding. Cochrane Database Syst Rev. 2003;(3):CD002245. 142. Spinella PC, Holcomb JB. Resuscitation and transfusion principles for traumatic hemorrhagic shock. Blood Rev. 2009;23:231-240. 143. Beekley AC. Damage control resuscitation: a sensible approach to the exsanguinating surgical patient. Crit Care Med. 2008;36(suppl 7): S267-S274. 144. Ringe HIG, Varnholt V, Gaedicke G. Cardiac rescue with enoximone in volume and catecholamine refractory septic shock. Pediatr Crit Care Med. 2003;4(4):471-475. 145. Hoffman TM, Wernovsky G, Atz AM, et al. Efficacy and safety of milrinone in preventing low cardiac output syndrome in infants and children after corrective surgery for congenital heart disease. Circulation. 2003;107(7):996-1002.

e5

Abstract: Shock is an acute state of circulatory or metabolic dysfunction that results in failure to deliver or use sufficient amounts of oxygen and/or other nutrients to meet tissue metabolic demands. If prolonged, it leads to multiple-organ failure and death. Shock can be caused by any serious disease or injury. However, whatever the causative factors, it is always a problem of inadequate cellular sustenance. Shock states can be classified into categories;

however, any given patient may display features of multiple categories over time. Fluid resuscitation, improvement of oxygen delivery, and minimization of oxygen consumption are the cornerstones of treatment of patients in shock. Key Words: shock, oxygen delivery, oxygen consumption, cardiac output, fluid resuscitation, lactate

10 35

Chapter Title Pediatric Cardiopulmonary Bypass

CHAPTER AUTHOR RICHARD M. GINTHER JR AND JOSEPH M. FORBESS

PEARLS • •













To gain basic knowledge of thewhich development ofin the Cardiopulmonary bypass (CPB), originated theeye. midTo developcentury, essential understanding howfor abnormalities twentieth was designed to allow the repair of at congenvarious of Its development can arrest hamper normal ital heartstages defects. history has since been or characterized by performation of the ocular structures and pathways. petual technological advancements thatvisual have been instrumental in sustaining the momentum of clinical progress of this field. Because of the morbidity associated with the “time on pump,” many early surgeries were performed at profoundly hypothermic temperatures by using circulatory arrest. The current philosophy underpinning the use of pediatric CPB is to meet the metabolic demands of the patient throughout

Background History Surgery for congenital heart disease has evolved into a relatively safe intervention considering its brief history and countless hurdles. This historical journey is, of course, filled with triumphs and tragic failures, telling a story of progressive intuition and challenges steadily surmounted. This has culminated in the generally successful model that is used today (Table 35.1). The early years of cardiac surgery spawned many novel techniques for operations that did not rely on cardiopulmonary bypass (CPB) as used today. Surgeons initiated their efforts in cardiovascular surgery with attempts to repair extracardiac vascular anomalies such as patent ductus arteriosus and coarctation of the aorta. On August 26, 1938, at the Boston Children’s Hospital, Dr. Robert Gross performed the world’s first successful patent ductus arteriosus closure on a 7-year-old girl.1 Soon, exposing the heart and attempting to correct life-threatening cardiac defects became a reality. In the early 1950s, surgeons began to explore several different approaches to repairing intracardiac defects. One technique, popularized by Dr. F. John Lewis, used total body hypothermia and vena cava inflow occlusion to achieve direct visualization of atrial septal defects.2 Although this technique proved to be fairly safe for simple atrial septal defects, failure was often the result when more complex defects were attempted.3,4 Surgeons needed a way to safely perfuse the patient’s circulatory system and extend the “safe” surgical time. In the late 1930s, Dr. John Gibbon and his wife Mary, a nurse and research assistant, began developing a

• •









To adequate information about of normal anatomy of theacquire repair while minimizing the impact associated nonphysithe eyeeffects. and related structures and develop a strong foundation ologic for aspects the understanding common ocular and their All of CPB haveofexperienced majorproblems technological imconsequences. provements. Circuits are miniaturized and cause less blood trauma, blood component therapy is highly directed, and onpump patient monitoring techniques have advanced. The progress of pediatric CPB has played a major role in the steady reduction of morbidity and mortality associated with cardiac surgery in children. Pediatric mortality rates are now comparable to those in adult patients.

heart-lung machine to do just this. By the early 1950s, Dr. Gibbon, in an interesting collaboration with International Business Machines Corporation (IBM), reported promising success in the laboratory using a heart-lung machine on cats and dogs.5–7 After a previous fatal attempt to repair an atrial septal defect (ASD) in a 15-month-old child in February 1952, Dr. Gibbon successfully closed an ASD in an 18-year-old patient using his heart-lung machine on May 6, 1953.8 Unfortunately, Dr. Gibbon was not able to repeat the same success with the heart-lung machine on subsequent cases, and his next four patients died. Other surgical teams devised their own versions of CPB but were unable to replicate laboratory successes, and no other human survivors were reported. It was theorized that perhaps these hearts were too sick to be repaired and that it was unrealistic to expect that these hearts could recover. CPB became a widespread disappointment, and most investigators abandoned the technique. While others were reporting their attempts using the heart-lung machine, however,9–12 Dr. C. Walton Lillehei and his colleagues at the University of Minnesota introduced a new approach for supporting patients during surgery: controlled cross-circulation. During cross-circulation, the patient’s parent was used as the “heart-lung machine” and supported the patient during the operation (Fig. 35.1). Considering the potential for a 200% operative mortality, this was a highly controversial technique. However, using this method, Dr. Lillehei was able to effectively close an ASD on March 26, 1954.13 Dr. Lillehei and his colleagues14 continued a remarkable series of successes using cross-circulation by performing 45 operations for anomalies that included ventricular septal defect, atrioventricular canal, and tetralogy of Fallot, with an 363

364

SECTION IV



Pediatric Critical Care: Cardiovascular

TABLE Successful Congenital Cardiac Surgery 35.1 Milestones

Year

Event

Surgeon

1938

Patent ductus arteriosus ligation

Gross

1944

Coarctation repair

Crafoord

1944

Blalock-Taussig shunt

Blalock, Taussig

1946

Potts shunt

Potts

1947

Closed pulmonary valvotomy

Sellors

1948

Atrial septectomy

Blalock, Hanlon

1951

Pulmonary artery band

Muller, Dammann

1952

Atrial septal defect closure using atrial well

Gross

1952

Atrial septal defect closure using hypothermia

Lewis

1953

Atrial septal defect closure using cardiopulmonary bypass

Gibbon

1954

Ventricular septal defect closure using cross-circulation

Lillehei

1958

Superior cavopulmonary shunt (Glenn shunt)

Glenn

1958

Senning operation for transposition of great arteries

Senning

1963

Mustard operation for transposition of great arteries

Mustard

1968

Fontan procedure for tricuspid atresia

Fontan

1975

Arterial switch for transposition of great arteries

Jantene

1981

Norwood procedure for hypoplastic left heart syndrome

Norwood

1985

Neonatal heart transplantation

Bailey

operative mortality of only 38%. This progress with more complex lesions prompted investigators to rethink their options for supporting, repairing, and recovering these patients. Two surgical camps ignited the resurgence of the artificial heart-lung machine: Dr. Lillehei and his colleagues at the University of Minnesota and Dr. John Kirklin and his colleagues at the nearby Mayo Clinic. Dr. Kirklin and colleagues15 reported a 50% mortality among eight patients using a modification of the Gibbon-IBM pump oxygenator in the spring of 1955. Months later, Lillehei and colleagues16 reported a 29% mortality among seven patients using their own heart-lung machine and the groundbreaking DeWall Bubble Oxygenator. These two groups demonstrated that surgical repair of complex congenital defects could be performed in a more controlled environment than cross-circulation or inflow occlusion, with promising results. What followed were many groups initiating open-heart programs primarily addressing congenital heart disease. Despite significant improvements in survival rates, congenital cardiac repairs remained a daunting undertaking with significant risk. Bypass circuits were enormous when compared with the patient blood volume, the systemic response was an extreme shock, and the understanding of the physiologic response to this “nonphysiologic” extracorporeal circulation was quite

limited. Investigators sought to use CPB but limit the actual cumulative time that nonphysiologic blood flow is provided to the patient—with its attendant risk. The bypass circuit could be used to cool the patient down to profound hypothermia after a lengthy period of topical cooling. The circulation of the patient could then be safely terminated for lengthy periods of time, allowing for complex cardiac repairs. At the conclusion of the repair, the heartlung machine could be used to fully warm the patient. These hypothermic circulatory arrest techniques with limited periods of extracorporeal circulation were popularized in the early 1970s by Dr. Barratt-Boyes and proved to dramatically extend the “safe” period of support.17 Surgeons began to perform increasingly complex congenital heart repairs. Pediatric cardiac surgical care was further refined over the subsequent several decades. The development of smaller, more efficient, and customizable heart-lung machine hardware and components, as well as improvements in myocardial protection, have allowed surgical teams to move away from the concept of limited CPB and toward a more “full-flow” philosophy wherein the metabolic demands of the body are continuously met while the patient is on the heart-lung machine. This chapter explores the concepts that form the basis of this philosophy and the techniques that surgical teams currently use to support pediatric patients during cardiovascular surgery.

Surgical Team The surgical team consists of highly trained specialists, each of whom plays a vital role in the safety and success of the surgical procedure. This specialized team is led by the cardiac surgeon and typically includes an assistant surgeon or physician assistant, anesthesiologist, perfusionist, and several nurses, surgical scrub technologists, anesthesia assistants, and perioperative surgical assistants. A perfusionist is a healthcare professional who specializes in all aspects of extracorporeal circulation. The primary focus of a perfusionist is to support the cardiac surgical patient during CPB. Because of this, the perfusionist’s clinical expertise is a critical component of operative success. Perhaps the first perfusionist was Mary Gibbon, Dr. Gibbon’s wife. In addition to helping design the Gibbon-IBM heart-lung machine, she assembled and operated it as well. The term perfusionist did not emerge until the early 1970s; in the early days of cardiac surgery, surgical groups would typically use any locally available combination of physiologists, biochemists, cardiologists, or surgical residents to help operate the heart-lung machine. Now, cardiovascular perfusionists are highly trained, nationally certified (Certified Clinical Perfusionist), statelicensed allied health professionals. The common scope of practice for a perfusionist consists of CPB, extracorporeal membrane oxygenation (ECMO), isolated limb/organ chemoperfusion, ventricular assist devices, autotransfusion, and intraaortic balloon counterpulsation.

Equipment and Preparation for Cardiopulmonary Bypass Heart-Lung Machine Console and Pumps The CPB machine, commonly referred to as the heart-lung machine, is the mechanical hardware that a perfusionist uses to support the patient during surgery. Until the late 1950s, the CPB hardware and circuitry were typically handmade, and many of the components had to be handwashed and sterilized for reuse.



CHAPTER 35

Pediatric Cardiopulmonary Bypass

365

Sigmamotor pump

Patient

Donor

Defect

• Fig. 35.1





​Controlled cross-circulation. (From Stoney WS. Evolution of cardiopulmonary bypass. Circulation. 2009;119:2844–2853.)

The hardware components were designed at that time with two objectives: to pump blood through the patient’s cardiovascular system and to successfully perform respiratory gas exchange, hence, the term heart-lung machine. Unfortunately, this heartlung apparatus was large, difficult to move, had no safety features, and was not available to other institutions eager to operate. Surgeons interested in these handcrafted devices would often visit the surgical groups at the University of Minnesota and Mayo Clinic, but few could replicate their expensive and intricate systems. Eventually, industry developers began to commercially release heart-lung machines with hardware components consolidated onto a wheel-mounted console. Interestingly, although cardiac surgery began with the pediatric patient population, heart-lung machines were developed as one-size-fits-all units and were not customizable for smaller patients. Modern heart-lung machine consoles are mobile, offer many pump configuration options, are loaded with safety features, and seamlessly send intraoperative CPB data to the electronic medical record. These design improvements allow for better configuration options for the pediatric surgical population. An ideal heart-lung machine for pediatric CPB is customizable for circuit miniaturization and offers safety devices and hardware that accommodate both smaller tubing sizes and circuitry. Customizations such as mast mounting pumps in various configurations and incorporating mini-roller pumps with shorter raceway lengths are two popular heart-lung machine configurations.18,19 Several different types of mechanical pumps have been used to substitute the function of the heart; interestingly, the roller pump has remained a standard pump mechanism since the beginning of

CPB. A roller pump functions by positive fluid displacement. Tubing is placed in a curved raceway; as occlusive rollers rotate over the compressible tubing, blood is pushed forward, creating a continuous nonpulsatile flow. The flow output is controlled by changing the revolutions per minute (RPMs) of the pump. Roller pumps are the most commonly used arterial (heart) pump in pediatrics (Fig. 35.2).20 While roller pumps are used as the arterial pump, the heart-lung machine console also holds several other roller pumps used for cardiotomy field suction, venting the heart, and cardioplegia delivery. The centrifugal pump is another type of arterial pump that has gained significant popularity since the mid-1970s. A centrifugal pump uses an impeller cone and rotational kinetic energy to propel the blood. Because it is nonocclusive, it is thought to be safer and cause less hemolysis than roller pumps. Centrifugal blood flow is controlled by the impeller cone RPMs and is also dependent on preload and sensitive to resistance distal to the pump. Because the pump is not occlusive, any resistance or occlusion will result in a reduction or cessation of flow. These pumps require the use of a flow probe to measure actual flow; the nonocclusive property is considered a safety feature in the event of cannula obstruction or accidental arterial line occlusion. The use of centrifugal pumps during ECMO has become increasingly popular owing to the suggested hemolytic and safety benefits; however, these benefits have often been refuted.21–24 Roller pumps remain the main arterial pump type in pediatric CPB because they are simple, inexpensive, and, importantly, require a much smaller prime volume than centrifugal pumps.

366

SECTION IV



Pediatric Critical Care: Cardiovascular

Cardiopulmonary Bypass Circuit

• Fig. 35.2



​Roller pump with ¼-inch tubing placed in the raceway. Venous line; gravity drainage

SVC & IVC bicaval cannulation

mm Hg

Temp 4°C

Bubble sensor

Oxygenator; heat exchanger; integrated arterial filter

mm Hg

Water heater/cooler

Gas filter Field suction

Field suction

Vent; cardioplegia; mini roller pumps

Arterial pump Isoflurane

Arterial line

1:4

del Nido cardioplegia

Cardiotomy; venous reservoir

Hemoconcentrator

Cardiotomy field suction

Level sensor



The handmade circuits used on children in the mid-1950s were elaborate, and the large blood volume required to prime them was a burden on the blood bank. Perfusionists would have to spend the evening of surgery assembling the circuit and then tackle the tedious task of dismantling, rewashing, and sterilizing the same circuitry after surgery. Fortunately, manufacturers now offer a wide variety of disposable circuit components that are fairly simple to assemble. The modern CPB circuit is a series of components consisting of cannulas, tubing, venous reservoir, filters, oxygenator, heat exchanger, hemoconcentrator, suction, and cardioplegia delivery system. Deoxygenated blood from the superior vena cava (SVC) and inferior vena cava (IVC) travels down a venous line, usually pulled by simple gravitational siphon effect, and into a venous reservoir. The deoxygenated blood in the reservoir is pumped through the oxygenator and then back to the patient’s aorta (or other major artery) via the arterial line (Fig. 35.3). This blood pathway diverts blood away from the heart and lungs, creating a bloodless operative field. In the adult patient population, where the circuit prime volume is typically no greater than 25% of the patient’s blood volume, a single circuit size can be used for almost all patient sizes. The small circuit prime-to-patient blood volume ratio helps

Gas flow meter

Blender Air O2 CO2

• Fig. 35.3

​Schematic of the cardiopulmonary bypass circuit at Children’s Health Dallas. CO2, Carbon dioxide; IVC, inferior vena cava; O2, oxygen; SVC, superior vena cava; temp, temperature.  



• Fig. 35.4  



​Sorin S5 heart-lung machine with mast-mounted arterial pump and Terumo Baby FX reservoir and oxygenator at Children’s Health Dallas.

to minimize patient hemodilution during CPB, ultimately reducing the likelihood of donor blood exposure. The same adult circuit, with a prime volume of about 1 L, would be approximately 500% of the circulating blood volume in a neonate. This discrepancy would seem outrageous considering current circuit options, but the prime-to-blood volume ratio was even higher before manufacturers began to release pediatric oxygenators in the mid1980s. Since the oxygenator is one of the largest volume components of the CPB circuit, any significant reduction in size would result in large prime volume reductions. A circuit miniaturization movement began—the new clinical challenge in pediatric CPB was to reduce both circuit prime volume and surface area. The goal of circuit prime and surface area reduction is to minimize hemodilution and the deleterious effects of foreign surface blood contact activation. Strategies such as using smaller diameter and shorter tubing lengths and incorporating neonatal and pediatric CPB components have allowed clinicians to reach this goal. At Children’s Health Dallas, the perfusionists have made many circuit modifications to achieve a static prime volume of approximately 165 mL in our neonatal circuit. This prime volume lowers the circuit size to approximately 45% of the blood volume of a 3-kg patient (Fig. 35.4).

Oxygenators An oxygenator, the artificial lung of the CPB circuit, might be considered the most important component of the circuit. It is responsible for oxygen (O2) and carbon dioxide (CO2) gas exchange, as well as volatile anesthetic administration. A heat exchanger, used for cooling and warming the perfusate—and, hence, the patient— is housed inside the oxygenator. Certain newer models now integrate the arterial filter, to reduce particulate matter, into the oxygenator. A venous reservoir, which includes both venous line and cardiotomy suction filters and various ports for drug and fluid administration, is typically packaged with an oxygenator. Currently, hollow fiber membrane oxygenators, which fully separate the blood flow from gas flow by a thin polymer membrane, are used during CPB. A brief history of oxygenator development reveals much about some of the important engineering solutions that have allowed for cardiac surgery to be performed more safely in progressively smaller patients. The first oxygenators used in the early days of cardiac surgery were hardware units that either used rotating discs or large mesh screens. These oxygenators worked by creating a large surface area film of blood, either over rotating discs in a pool of venous blood



CHAPTER 35

Pediatric Cardiopulmonary Bypass

367

or trickling over large mesh screens and exposing the film of blood to an oxygenated atmosphere.25,26 Though these units were successful at oxygenating blood, they required extremely large priming volumes; were not disposable; were difficult to assemble, operate, and clean; and lost significant efficiency during hemodilution. In addition to these disadvantages, these oxygenators were not commercially available to clinicians looking to operate beyond the University of Minnesota and Mayo Clinic. The University of Minnesota team dramatically changed this landscape in the late 1950s by releasing the simple, disposable, inexpensive, and commercially available DeWall-Lillehei bubble oxygenator.27 Though the safety of actively adding bubbles to the blood was debated, the commercial availability of this device contributed to a rapid global expansion of cardiac surgery. The bubble oxygenator was a distinct improvement over the previous unwieldy direct blood contact oxygenators, yet it was still limited in that the direct blood-air interface could produce significant blood trauma. This trauma accrues over time; thus, the safety margin for longer pump runs was diminished for longer, complex cases. The next generation of oxygenators, membrane oxygenators, better mimicked the function of the lungs. These microporous, gas-permeable membranes eliminated direct contact between gas and blood, thus, reducing blood trauma.26 The concept of a microporous membrane separating the gas and blood was sound, but it took decades of research to find a suitable membrane material before these oxygenators could replace bubble oxygenators commercially. Initial success with silicone membranes was observed with long-term support during ECMO. However, in the operating room, these membranes proved to be less efficient and prone to plasma leakage and thrombus formation.26,28 The development and release of polypropylene microporous membranes allowed for efficient gas exchange over a wide range of temperatures and pump flow rates, replacing the bubble oxygenator during CPB in the mid-1980s. In these oxygenators, CO2 and O2 flow meters and a gas blender control gas and volatile anesthetic flow through the inside of the hollow polypropylene fibers. The gas within the hollow fibers passively diffuses into the blood flowing on the outside of the fibers. In 1985, Cobe released the popular Variable Prime Cobe Membrane Lung (VPCML) designed for the pediatric market. This oxygenator was divided into separate compartments and gave clinicians three maximum blood flow options, 1.3, 2.6, and 4.0 L per minute (LPMs) depending on which compartments were opened.29 The VPCML also tried a new concept with the heat exchanger. Once a separate external CPB component, the stainless-steel heat exchanger was placed inside the venous reservoir. The stainless-steel coil wrapped around the inside of the reservoir was not efficient unless a large amount of volume was held in the reservoir. This was counter to the efforts to reduce the overall circuit prime volume. Despite this shortcoming in the VPCML model, the move toward integration and consolidation of functionalities continued. These heat exchangers are now integrated within the oxygenator housing. Considering that pediatric cardiac surgery is more likely than adult surgery to use moderate to deep hypothermia, these heat exchangers need to be extremely efficient with a small surface area. As technology relentlessly improved, membrane hollow fibers were wrapped into tighter configurations. This eventually allowed for a priming volume low enough to release a dedicated neonatal oxygenator. In 2006, Dideco released the first neonatal oxygenator with a prime volume of 31 mL and maximum rated flow of 700 mL/min.30 The new generation of neonatal and pediatric

368

SECTION IV



Pediatric Critical Care: Cardiovascular

TABLE Tubing Specifications and Maximum Blood Flow 35.2 Ranges Tested at Children’s Health Dallas TUBING SPECIFICATIONS

Internal Diameter (in) ⁄

2.5



3.7



5



9.65

5 32 3 16 14

3.5

,450

5

,750

7

,1300

500–650

13

,3000

1300–1500

   





18

,5500

2000–2200



21.71

27

.5000

4000–4500



28.5

38

5000–5500

12



38.61

45

.5000



55.77

65

7 16

58



Maximum Gravity Drainage (mL/min)

13.5

38



mL/rev

Maximum Arterial Flow (mL/min)



5 16

• Fig. 35.5

mL/ft

18

CHILDREN’S HEALTH DALLAS PROTOCOL

​Terumo Baby FX05 pediatric oxygenator.

oxygenators achieves much higher maximum flow rates while keeping prime volumes appropriate for neonates. This has allowed clinical teams to achieve consistent physiologic outcomes after pump runs in neonates and small infants. A modern pediatric device such as the Terumo Baby FX oxygenator with integrated arterial filter (Terumo Cardiovascular Group) offers a low total prime volume and a high maximum blood flow (Fig. 35.5). With arterial filter integration, this oxygenator has a total prime volume of 43 mL and a maximum rated blood flow of 1.5 LPMs. This low prime oxygenator is suitable for neonates but also accommodates patients up to approximately 15 kg. This wide range of blood flow and low prime improves the likelihood of bloodless surgery— wherein an asanguineous prime is used—for the larger patients in range for this device. The Maquet Quadrox-i Neonatal oxygenator (Maquet Holding) is another oxygenator with an integrated arterial filter that has a 40-mL total prime volume and 1.5-LPMs maximum flow. When considering the additional volume of an external arterial line filter, this high-efficiency oxygenator with an integrated filter offers the lowest total prime volume unit on the market today.31 Current trends in oxygenator design and development include integration of the arterial line filter, biocompatible surface coatings for circuit tubing, decreasing flow resistance, and more efficient heat exchange.

in pediatrics. Changing the internal diameter of tubing affects blood flow resistance and must not impede venous drainage or arterial blood flow. At our institution, we select arteriovenous line sizes that accommodate gravity venous drainage and do not exceed an arterial line pressure of 350 mm Hg (Table 35.2). Large reductions in tubing length have been made possible by positioning the smaller new-generation pump consoles close to the patient and using mast mounted pumps to bring components closer together. Also, smaller-diameter venous line tubing may be used to further reduce the prime volume, but vacuum-assisted venous drainage (VAVD) must be used to augment the gravity siphon. The bioreactivity of blood coming into contact with artificial surfaces, such as tubing, is known to exacerbate the systemic inflammatory response and disrupt hemostasis. A major advancement has been the development of surface coatings that attempt to mimic the endothelial surface of blood vessels. These coatings have been shown to attenuate the increase of cytokines and inflammatory markers and preserve platelets.32,33 When selecting tubing for the pediatric circuit, the goal is to safely achieve maximum blood flows, decrease prime volume, and attenuate blood trauma.

Tubing

A hemoconcentrator is an ultrafiltration device that consists of semipermeable membrane fibers that remove plasma water and solutes. They function similarly to hemodialysis units but are simpler in that they do not require a dialysate solution. Blood flows through microporous membrane fibers, and since the hydrostatic pressure is higher inside the membrane fibers, effluent fluid permeates the membrane and can be removed. The membrane pore sizes are typically less than 55,000 Da, which preserve plasma proteins such as albumin (65,000 Da) and maintain the colloid oncotic pressure. The ultrafiltration rate of a hemoconcentrator is dependent on the hydrostatic pressure gradient across the membrane, blood flow rate through the membrane fibers, membrane pore size, and the hematocrit. Ultrafiltration is useful for

The tubing used to connect the various components of the CPB circuit to the patient is made of a medical-grade polyvinyl chloride. Tubing length and diameter are the two main factors to consider when designing a circuit. Shorter tubing with the smallest internal diameter will reduce prime volume, but the tubing must also be large enough to safely manage required blood flows and line pressures for a given patient. In the past, ¼-, 3⁄8-, and ½-inch tubing were the only tubing options, which made circuit miniaturization a difficult task. Currently, a wide range and selection of pediatric tubing and connector sizes are available. Tubing sizes such as 1⁄8, 3⁄16, and ¼ inch have become the new standards

Hemoconcentrators



CHAPTER 35

Pediatric Cardiopulmonary Bypass

369

increasing hematocrit, reducing high potassium levels after cardioplegia delivery, and removing harmful inflammatory mediators. Hemodilution during pediatric CPB is difficult to avoid; a 2004 survey of pediatric cardiac surgery centers reports that 98% of perfusionists routinely use a hemoconcentrator during CPB.20

solution should be “physiologic” and attempt to attenuate the adverse response to artificially supporting a patient with an extracorporeal circuit.

Circuit Prime

Due to the foreign surface contact and resultant intrinsic activation of the coagulation cascade, the patient must be anticoagulated before CPB. Heparin is the most widely used anticoagulant during CPB. It acts by super-activating antithrombin III (ATIII), which then inactivates thrombin and other proteases involved in coagulation. Heparin is used because it is fast-acting, and anticoagulation reversal can easily be achieved by administering protamine. Anticoagulation helps prevent circuit thrombus formation and avoid the devastating effects of potential arterial thromboembolism. Heparin was the anticoagulant used during Dr. Gibbon’s first successful cardiac surgery in 1953; its use during CPB has continued for more than 60 years. Before heparin administration and dosing protocols were available, anticoagulation methods were cumbersome and unsafe. The dosing was empiric, and the only methods for testing heparinization were lengthy laboratory heparin concentration tests. Fortunately, the activated clotting time (ACT) test was introduced in 1966—this bedside whole blood test became the foundation of how heparinization is monitored in the cardiac operating room today.35 Traditional laboratory tests such as partial thromboplastin time (PTT) and prothrombin time (PT) are sensitive to low doses of heparin and therefore are not useful during CPB. The ACT test is a point-of-care test that measures the time (in seconds) needed for activated whole blood to form thrombin. In 1975, Bull et al.36 reported a heparin management approach using the ACT test, and the technique quickly became universally accepted. The report describes the heparin dose-response curve technique and suggests an optimal ACT range of 480 seconds during CPB. In this technique, ACT tests are run on various whole blood samples containing different heparin concentrations and results are plotted versus the heparin concentration. The heparin dose-response curve, commonly referred to as the Bull curve, demonstrates the individualized ACT response to different levels of heparinization and is a useful tool in estimating the concentration of heparin necessary to achieve an ACT of 480 seconds (Fig. 35.6). Maintaining ACT results of at least 480 seconds during CPB remains the standard of care today. Though most clinicians will agree that 480 seconds is acceptable during CPB, there is debate regarding whether that value should be universally applied considering that not all ACT analyzers operate and activate blood in the same manner. Pediatric patients undergoing cardiac repair suffer disproportionate postoperative bleeding complications after CPB, likely because of their size and immature coagulation system. Contributing factors to postoperative bleeding are dilution of coagulation factors during CPB, induction of the systemic inflammatory response, hematologic changes in cyanotic patients, hypothermia, and numerous coagulation factor deficiencies. All these situations can inhibit adequate anticoagulation with heparin and ultimately lead to the generation of thrombin. It has been shown that prolonged ACT results of pediatric patients poorly correlate with the plasma levels of heparin during CPB.37 Reports have shown that pediatric patients require higher plasma heparin concentrations than adults because they metabolize heparin faster, have a larger blood volumeto-body weight ratio, and have lower ATIII levels.38,39 Therefore, weight-based heparin doses and ACT monitoring used with adult patients are not recommended for use in pediatric patients.

The CPB circuit is primed with a crystalloid replacement fluid. Common solutions include Plasma-Lyte A, lactated Ringer, and Normosol-R.34 Lactated Ringer is a replacement fluid that contains 29 mEq/L of lactate but lacks magnesium. Plasma-Lyte A and Normosol-R both closely mimic human physiologic plasma electrolyte concentrations, osmolality, and pH. However, these two solutions do not contain calcium. At Children’s Health Dallas, the perfusionists use Plasma-Lyte A because it does not contain lactate or calcium. This allows the perfusionists to lower CPB perfusate calcium levels, which is desirable, as is discussed later. Once the CPB circuit is primed with a crystalloid solution and cleared of any air, the total prime volume of the circuit is estimated. The perfusionist must then choose between initiating CPB with or without adding heterologous blood. Unlike the adult patient population, blood products are often added to the neonatal and pediatric CPB circuits due to the small patient blood volume-to-circuit prime volume ratio. The dilutional effect of the crystalloid prime is determined by calculating the patient resultant hematocrit (HCTr). The HCTr formula, HCTr 5 (Patient blood volume 3 HCT)/ (Patient blood volume 1 Circuit prime volume), is calculated once the patient hematocrit value is measured in the operating room before surgery. The institutional protocol at Children’s Health Dallas is to maintain a CPB HCTr above 30%. If that value cannot be reached, then packed red blood cells (PRBCs) are added to the circuit. The institutional protocol also directs that a half unit of fresh-frozen plasma, approximately 100 mL, will be added to the circuit prime for all patients less than 6 kg. The pre-CPB circuit prime drug additives at our institution include heparin (1000 U/mL), 8.4% sodium bicarbonate, 20% mannitol, furosemide (10 mg/mL), methylprednisolone, tranexamic acid, and 25% albumin (Table 35.3). The ideal prime TABLE Cardiopulmonary Bypass Circuit Prime Drugs 35.3

Drug

Action

Prime Dose

Heparin

Anticoagulant

Calculated by Medtronic HMS and varies per patient

Sodium bicarbonate

Buffer

Achieve pH 7.40

Mannitol

Osmotic diuretic; oxygen radical scavenger

0.5 mg/kg; 12.5 g maximum dose

Furosemide

Loop diuretic

0.25 mg/kg; 20 mg maximum dose

Methylprednisolone

Corticosteroid

30 mg/kg; 1 g maximum dose

25% Albumin

Plasma protein

10% circuit prime volume

Tranexamic acid

Antifibrinolytic

20 mg/kg; 20 g maximum dose

Anticoagulation

370

SECTION IV



Pediatric Critical Care: Cardiovascular

from an arterial or intravenous line is used to run an ACT and HPT. If the ACT reaches 480 seconds or the HPT confirms an adequate heparin concentration, cardiotomy pump suction may be used, and CPB may be initiated when the surgeon inserts the arterial and venous cannulas. ACT and HPT tests are run every 30 minutes during CPB, and heparin is administered if the ACT falls below 480 seconds or the heparin concentration falls below the maintenance value calculated by the HDR. If a parameter is low, the Hepcon HMS PLUS uses a formula based on the HDR, blood volume of the patient, and circuit prime volume to calculate the amount of heparin needed to adequately raise the ACT or HPT.

600 D

500 C

ACT (sec)

400 B 300 200

D

A

100

C 0 0

100

200 300 Heparin dose (U/kg)

400

• Fig. 35.6

​ n example of a heparin dose-response curve wherein the A patient’s baseline activated clotting time (ACT) is shown at point A. An initial heparin dose of 200 U/kg resulted in an ACT shown at point B. A linear extension of points A and B is drawn with an intersection at 400 (point C) and 480 seconds (point D). These target intersects can be used to estimate further heparin doses to administer to the patient. (From Bull BS, Huse WM, Brauer FS, Korpman RA. Heparin therapy during extracorporeal circulation. II. The use of a dose-response curve to individualize heparin and protamine dosage. J Thorac Cardiovasc Surg. 1975;69:685– 689.)  



The potential variability of a pediatric patient’s response to heparin necessitates an individual dosing regimen and the use of different coagulation tests. A useful bedside hemostasis management tool in pediatric cardiac surgery is the Hepcon HMS PLUS (Medtronic Inc.). The Hepcon HMS PLUS is fully automated and is used to run the following tests: ACT, heparin dose response (HDR) to identify individual heparin needs, and heparin-protamine titration (HPT) to verify heparin concentration. A baseline sample is collected from the arterial line before heparinization and is used to test the HDR. The HDR test determines the baseline ACT and patient response to increasing amounts of heparin. Results are used to identify heparin-resistant or heparin-sensitive patients and determine the patient heparin concentration needed to achieve appropriate anticoagulation. To test the blood heparin concentration with the HPT test, blood is added to tubes containing different mg/mL concentrations of protamine. Heparin and protamine bind in a 1:1 ratio; thus, the tube that produces a clot can be used to determine the unit/mL heparin concentration. The HPT test is used frequently during CPB to maintain heparin concentrations suggested by the HDR and is run post-CPB to verify proper heparin reversal after protamine administration. Without heparinization, hemodilution and the degree of hypothermia alone could extend the ACT beyond 480 seconds; this effect is amplified in the pediatric patient. Administering heparin to maintain a patient heparin concentration calculated by the HDR, despite an ACT greater than 480 seconds, will help to reduce consumptive coagulopathy, thrombin generation, fibrinolysis, neutrophil activation, and the need for transfusions.40,41 Once the patient is ready to be cannulated for CPB, a heparin bolus is administered to the patient by the anesthesiologist into an intravenous line or by the surgeon directly into the right atrium. In general, the patient receives a 400 U/kg dose of heparin minutes before arterial cannulation. Once the heparin has circulated within the patient for approximately 5 minutes, a blood sample

Cannulation Cannulation refers to the process in which the surgeon attaches the venous limb of the CPB circuit to the systemic venous circulation of the patient while attaching the arterial limb to the systemic arterial system of the patient. This is most commonly accomplished by placing an arterial cannula in the distal ascending aorta and venous cannulas in the SVC and IVC, respectively. The cannulas are inserted through appropriately sized purse-string sutures and secured with tourniquets. This bicaval configuration allows for the achievement of “total” CPB; the vast majority of cardiac repairs can be accomplished using this technique. In pediatric cardiac programs, patients ranging in weight from approximately 1000 g up to adulthood are placed on CPB. Therefore, a wide range of cannula sizes must be kept in stock. Arterial cannulas range from as small as 8 Fr (2.67 mm) in diameter up to over 20 Fr. Venous cannulas for CPB are available in straight and angled varieties and range down to as small as 10 Fr. Inserting these cannulas into the diminutive aorta and venae cavae of neonates is a taxing technical exercise that must be accomplished without complication in order to appropriately support the patient during the repair and leave the patient with undamaged vessels at the cannulation sites postoperatively. Exceptions to standard bicaval cannulation are frequently seen in pediatric practice. First, patients can have anomalies of systemic venous return, such as bilateral SVCs, ipsilateral hepatic veins, or an interrupted IVC with azygous continuation to the SVC. All these anomalies have to be assessed, and an appropriate venous cannulation strategy must be devised. Occasionally, if these anomalies are prohibitive for selective cannulation or the overall patient size is so small that the venae cavae are too small to cannulate, the right atrial appendage is cannulated in isolation and periods of circulatory arrest, wherein venous return is not required, are used to accomplish intracardiac portions of the repair. Alternatives to standard ascending aortic cannulation are also used. In order to accomplish aortic arch reconstructions without resorting to circulatory arrest, a small prosthetic vascular tube graft is anastomosed to the innominate artery and the arterial cannula is inserted into this “chimney” graft. Alternatively, if the patient is large enough, the innominate artery can be cannulated directly. These innominate artery cannulation techniques allow the brain to be perfused up the right carotid artery while the aortic arch is being repaired. Reoperations are common in congenital heart surgery. A number of these patients have pulmonary outflow conduits that are densely adherent to the sternum. Patients with transposition of the great arteries have an abnormally anteriorly located ascending aorta that can also be adherent to the chest wall in the midline. Peripheral cannulation via a femoral artery is sometimes necessary

in these instances. Peripheral arterial cannulation in children should be performed only when absolutely necessary and converted to the ascending aorta as soon as possible. The obturation of the femoral artery by the cannula almost always causes hypoperfusion of the lower extremity. With longer cannulation times, the leg can be at significant risk for ischemic complications.

Cardiopulmonary Bypass Pediatric vs. Adult Considerations Although many of the management techniques governing pediatric and adult CPB are similar, several differences do exist (Table 35.4). The small size of the pediatric patient and nature of the surgical repair often expose these patients to moderate or deep hypothermic temperatures, wide ranges of perfusion flow rates, and hemodilution. These management techniques represent extreme shifts from normal physiologic parameters, and the harmful effects are potentially more pronounced in these small patients. Low-flow perfusion or circulatory arrest at deep hypothermia (15–20°C) is often required because of the complexity of the repair, significant aortopulmonary collateral blood flow returning to the operative field from the pulmonary veins, or simply because position of the perfusion cannulas interferes with access to the surgical site. Compared with adults, hemodilution of the pediatric patient during CPB has a larger impact on the concentration of blood components and the blood-to-foreign surface area exposure is TABLE Differences Between Adult and Pediatric 35.4 Cardiopulmonary Bypass

Parameter

Adult

Pediatric

Hypothermic temperature

Rarely below 25°C–32°C

Commonly 15°C–20°C

Use of circulatory arrest

Rare

Common

25%–33%

150%–300%

Pump Prime Dilution effects on blood volume Additional additives

Blood, albumin

Perfusion pressures

50–80 mm Hg

25–50 mm Hg

Influence of a-stat versus pH-stat management strategy

Minimal at moderate hypothermia

Marked at deep hypothermia

Measured Paco2 differences

30–45 mm Hg

20–80 mm Hg

Hypoglycemia

Rare—requires significant hepatic injury

Common— reduced hepatic glycogen stores

Hyperglycemia

Frequent— generally easily controlled with insulin

Less common— rebound hypoglycemia may occur

Glucose Regulation

Paco2, Partial pressure of arterial carbon dioxide. From Greely WJ, Cripe CC, Nathan AT. Anesthesia for pediatric cardiac surgery. In: Miller RD, Cohen NH, Eriksson LI, et al. Miller’s Anesthesia. 8th ed. Philadelphia: Elsevier; 2015:2820.



CHAPTER 35

Pediatric Cardiopulmonary Bypass

371

much greater. This relatively greater exposure to nonendothelialized surfaces can lead to an increased inflammatory response and damage the formed elements of blood. Another important contrast between pediatric and adult support involves calcium management. The immature myocardium is susceptible to exacerbated postischemic injury due to overly rapid calcium loading at reperfusion.42,43 Because of this, at Children’s Health Dallas, a perfusate that is relatively depleted of calcium is used until well after cross-clamp removal. Calcium is restored in a stepwise fashion before weaning from the circuit. The coagulation system of neonates and infants also differs from adults in that they have quantitative deficiencies of coagulation factors at baseline. These deficiencies of the immature coagulation system coupled with hemodilution discussed earlier result in significant postoperative coagulopathies and anemia that must be addressed with blood products much more commonly than in adult cases.

Initiation of Cardiopulmonary Bypass Before starting a planned operation, the surgical team will discuss the procedure and form a detailed management plan for each team member. The perfusionist must understand the type and complexity of the surgery and discuss the proper cannula section, degree of hypothermia, myocardial protection technique, and any other unique patient variables that might affect perfusion management. The fundamental concepts of managing CPB for congenital patients are similar to adults, but anatomic variations and physiologic extremes complicate the approach. Once the surgical plan is established, the patient is prepped and draped for the skin incision and sternotomy. With the chest open, the surgeon exposes the heart and major vessels and then directs the anesthesiologist to administer heparin before cannulation. Alternatively, the surgeon can administer the heparin directly to the right atrium. Next, the arterial and venous cannulas are inserted, but before it is safe to initiate CPB, it is important to confirm an adequate anticoagulation level by obtaining an ACT and heparin concentration and that the arterial cannula is unobstructed. Congenital patients, especially cyanotic patients, often demonstrate variable dose responses to heparin. In addition, it cannot be assumed that the CPB dose of heparin is circulating in the patient, as intravenous line malfunction can occur. Initiating CPB on a pediatric patient with an obstructed aortic cannula could quickly exsanguinate the patient, as venous drainage commences without return of this blood to the patient and could cause severe hypotension. Once these two safety checks are complete, the patient is ready for CPB. The venous line is unclamped to gravity siphon deoxygenated blood from the patient, and the arterial flow is slowly increased as the heart begins to empty. Bicaval cannulation is often used in congenital surgery, but it is common to initiate CPB with only one cannula. This is done to verify adequate drainage from one cannula, as it would be difficult diagnose poor drainage from a single cannula if both were open. Once both cannulas are open, adequate venous drainage is confirmed when the central venous pressure (CVP) falls to zero and the SVC, IVC, and right atrium are collapsed. Total (also termed full or complete) CPB is achieved when all the systemic venous blood is being diverted to the heartlung machine and full arterial flow can be achieved. In the rare occurrence that venous anomalies prohibit inserting appropriately sized venous cannulas, VAVD may be used to enhance return to the venous reservoir and achieve full-flow CPB. However, VAVD has been reported to induce blood trauma, exacerbate gaseous

372

SECTION IV



Pediatric Critical Care: Cardiovascular

microemboli, and cause retrograde flow in the venous line if the vacuum apparatus malfunctions or becomes occluded.44,45 Considerations for safe VAVD operation include using a pressure monitor to maintain a venous line pressure between 0 to 240 mm Hg (gravity siphon will typically provide 25 to 215 mm Hg), adding both positive and negative pressure relief valves to the sealed venous reservoir, being vigilant to recognize and ask the surgeon to correct sources of venous line air, including a moisture trap in the vacuum kit, and only using the minimal amount of vacuum to achieve full arterial flow.46–49 When on total CPB, the mean arterial pressure (MAP) and CVP should confirm that the heart is empty by showing a flat tracing. However, the many anatomic variations of the congenital patient can lead to blood returning to the heart despite adequate drainage. Variations such as a patent ductus arteriosus, major aortopulmonary collateral arteries, an unrecognized left SVC draining to the coronary sinus, and aortic insufficiency can return blood to the heart and should be considered when evaluating venous drainage. Assessment of adequate venous drainage and perfusion flow rate is critical before proceeding to the surgical repair. A poorly positioned venous or arterial cannula can restrict optimal perfusion flow; addressing this issue during the aortic cross-clamp period would waste unnecessary myocardial ischemic time. Once cannula placement is deemed acceptable, full perfusion flow is achieved, and oxygenation from the oxygenator is confirmed, the anesthesiologist can turn off the ventilator.

Determining and Monitoring Effective Perfusion Flow Rate The fundamental goals of bypass are to meet the metabolic demands of all tissues and to attenuate the deleterious pathophysiologic effects of artificially supporting a patient. Once the patient is transitioned to CPB, several management techniques are used to safely optimize the level of support. Perfusion flow rate, which represents the cardiac output during CPB, is altered to meet the O2 consumption needs of the patient. Global adequacy of flow is estimated in real time by the display of O2 saturation by a sensor in the venous return line. Assessing regional O2 consumption to the brain, kidneys, or bowel, for instance, is a challenge. Additionally, owing to age-related differences of body surface area (BSA)-to-blood volume ratios, flow rate indexes are higher in neonates than adults. The optimal effective flow rate or cardiac index for any size patient remains unclear. Considering that the perfusion flow rate is not fixed during the different phases of CPB, several variables are helpful in determining a safe minimal rate. Initial normothermic target rates are calculated for CPB initiation by weight and BSA. At Children’s Health Dallas, in addition to the CPB initiation flow rates calculated by weight, the perfusionist calculates several cardiac indexes ranging from 2.4 to 3.0 L/min per m2. Factors such as the degree of hypothermia, acid-base balance, depth of anesthesia and neuromuscular blockade, hematocrit, venous saturation, lactate level, urine output, and near infrared saturation trends are used to guide perfusion flow rates. Patient temperature is the greatest factor affecting perfusion flow; rates as low as 40 to 50 mL/kg are routinely used at core temperatures in the 20°C range. A growing area of concern is the avoidance and early detection of acute kidney injury (AKI) in congenital heart disease patients. While traditional diagnostic approaches have not been thoroughly validated in children, it has been widely reported that there is a 40% to 50% incidence of AKI in congenital heart

disease patients, and 64% incidence in neonatal patients.50–52 The kidneys perceive nonpulsatile flow or a decrease in arterial flow as hypovolemia, and the resultant neurohormonal cascade is thought to trigger the AKI complex. Since most perioperative risk factors, such as younger age and the incidence of higher surgical complexity, are nonmodifiable, therapeutic strategies have focused on optimally managing perfusion flow rate, arterial pressure, and hematocrit.53

Arterial Pressure The MAP will slowly lose its pulsatile trace and flatten out as the heart empties on CPB. Though the MAP is calculated by factoring the systolic and diastolic pressures, the value of the flat tracing is referred to as the MAP and perfusion pressure during CPB. The transition to CPB often leads to hypotension; in contrast to adult cases, vasopressors (e.g., phenylephrine) are not typically administered in the early phase of CPB in young patients. The goal in the early phase of CPB is to cool the patient and reduce the metabolic demands. Low perfusion pressure, 20 to 30 mm Hg, is accepted during the cooling phase, and vasodilators (e.g., phentolamine) are used to reduce arterial tone and increase uniformity of perfusion and improve cooling. Vasodilation has been shown to improve temperature distribution and reduce lactate production in pediatric deep hypothermic CPB.54 Hemodilution, hypocalcemia, and the inflammatory response are also factors that cause hypotension at the onset of CPB. Hemodilution will lower the perfusion pressure because of the viscosity reduction, and hemoconcentration performed by the perfusionist with the hemoconcentrator can easily increase perfusion pressure by raising the hematocrit. The systemic inflammatory response is triggered by the foreign surface contact of blood and bypass circuitry. This response releases many vasoactive mediators, which can quickly drop the perfusion pressure, and highlights the importance of minimizing circuit surface area for the pediatric patient. The decrease in pressure in an adult patient with coronary or carotid stenosis would likely be treated with a vasopressor, while increasing perfusion flow is the preferred method in young patients. Arterial and Venous Oxygen Saturation Most O2 saturation monitoring techniques are noninvasive, inexpensive, and can be used in real time. Changing perfusion flow rate and O2 delivery will have immediate and direct effects on O2 saturation levels. Pulse oximetry is a clinical mainstay used to monitor O2 delivery to the extremities during the preoperative and postoperative periods. However, due to the nonpulsatile flow pattern generated during CPB, this technology is ineffective. As mentioned earlier, a mainstay monitoring technique during CPB is to track the O2 saturation of the venous line blood draining into the venous reservoir. As a general guideline, perfusion flow rate is adjusted to maintain this mixed venous O2 saturation (Svo2) greater than 70%. While this guideline is helpful during “normal” physiologic conditions, the many nonphysiologic variables of CPB cause shifts in the oxyhemoglobin dissociation curve (Fig. 35.7); these venous O2 saturations may fail to represent satisfactory O2 delivery to the tissues. Leftward shifts in the curve prevent O2 from being released from hemoglobin, which could deceivingly demonstrate an acceptable Svo2 in the presence of tissue hypoxia. As the patient is cooled during CPB, regional deoxygenation has been shown to occur despite a normal or rising Svo2 without increasing perfusion flow rate.55 Hypothermia not only strengthens the hemoglobin O2 affinity but also creates an alkaline blood pH and causes a further leftward oxyhemoglobin

100

Oxygen saturation (%)

80 70 60

40

Increase perfusion flow rate Increase hematocrit pH-stat blood gas strategy Decrease temperature Increase mean arterial pressure; vasopressor Verify adequate superior vena cava drainage

30 20 10

0

10

20

30

40 50 60 PO2 (mm Hg)

70

80

90

100

• Fig. 35.7



​The oxyhemoglobin dissociation curve. 2,3-DPG, 2,3-Diphosphoglycerate; Po2, partial pressure of oxygen.  

373

Infrared Spectroscopy Values

Right shift: Increased 2,3-DPG Hyperthermia Hypercarbia Acidosis

50

Pediatric Cardiopulmonary Bypass

• BOX 35.1 Methods to Improve Cerebral Near-

Left shift: Decreased 2,3-DPG Fetal hemoglobin Hypothermia Hypocarbia Alkalosis

90



CHAPTER 35

shift. In this situation, it is important to cool and warm patients methodically to minimize the temperature gradient between the blood and tissues and maintain perfusion flow so that the Svo2 does not trend downward. As tissue temperature decreases, venous line Svo2 can be used to help guide the reduction of perfusion flow rate. In addition to hypothermia, other factors—such as third spacing, myocardial dysfunction, and hemodilution—may reduce O2 delivery and lead to cellular hypoxia. Therefore, it is common practice to augment O2 delivery using higher fraction of inspired O2 settings and maintaining a partial arterial oxygen pressure (Pao2) above normal physiologic values, with the goal of increasing the O2 gradient between capillaries and tissue beds. However, the O2 tension strategy must be carefully managed to avoid the generation of systemic reactive O2 species, systemic inflammation, O2 radicals, and end-organ injury. This paradox is especially pronounced in patients with cyanotic defects owing to their physiologic sensitivity to oxidative stress.56 While the literature does not clearly delineate what Pao2 value is considered high or hyperoxic, it is suggested that CPB should be initiated by controlling oxygenation in a slow, graded manner and to not exceed Pao2 values of 350 mm Hg.57,58 It is especially critical to meet the metabolic needs of brain tissue, and global Svo2 values may misrepresent regional O2 consumption. It has been shown that considerable regional differences exist and Svo2 can overestimate regional saturations from the brain.59 The majority of venous blood analyzed by the venous line Svo2 comes from IVC cannula, and IVC saturation is notoriously misleading as an estimate of O2 consumption owing to “contamination” by highly saturated renal venous blood. Though the Svo2 does have its place in guiding perfusion flow rate, more specific, regional oxygenation assessment is currently recommended to ensure adequate perfusion flow distribution, particularly for the brain.

Near-Infrared Spectroscopy Near-infrared spectroscopy (NIRS) is a noninvasive optical sensor that can measure cerebral and somatic tissue oxygenation. NIRS

monitoring is gaining considerable popularity because sensor pads may be placed over various regional tissue beds, particularly both cerebral hemispheres, and display real-time results. Somatic monitoring sites—such as flank, abdominal, and muscle—are suggested to help broaden the assessment of systemic hypoperfusion. The technology works by bouncing various wavelength arcs of near-infrared light from a sensor emitter and detector. These photodetectors allow for selective measurement of tissue oxygenation. This technology is widely used in the operating room and intensive care unit, although interpreting the results has been a topic of debate. A validation study performed at Children’s Health Dallas demonstrated that cerebral NIRS values accurately predicted the O2 saturation in the SVC on CPB and that flank NIRS values were significantly associated with IVC saturation.60 As increasing evidence validates tissue oximetry against invasive measurements, NIRS monitoring has shown its value in quickly detecting regional low flow.61–64 At Children’s Health Dallas, the perfusionists use NIRS trends and values to guide perfusion flow rate, hematocrit, blood gas strategy, temperature, and vasomotor tone (Box 35.1). It is important to note that NIRS helps to guide rather than dictate perfusion management. The upper limits and critical lower values reported by NIRS are poorly defined. Considering the regional O2 oxygen saturation variations of the neonatal and infant cardiac surgical patients, NIRS provides valuable information and early detection of poor perfusion in critical organs. NIRS has been particularly useful as a real-time monitor on patients with hypoplastic left heart syndrome, for example. Cerebral O2 saturation measured after stage I palliation has been shown to strongly correlate with hemodynamic parameters and help to identify early postoperative complications.62,65 Hoffman et al.66 found that avoiding cerebral hypoxia with the use of NIRS monitoring was the most significant factor in improving childhood neurodevelopmental outcomes. Additionally, Sood et al.67 demonstrated that perioperative NIRS monitoring was useful in predicting neurodevelopmental outcomes, especially when evaluating the percent decrease of cerebral O2 saturation from baseline values during the intraoperative period.

Methods to Optimize Physiologic Management Target Hematocrit and Ultrafiltration Despite the progress of reducing the pediatric circuit prime volume, hemodilution during CPB remains difficult to avoid.68 Hemodilution can cause edema, coagulopathy, blood and colloid osmotic pressure reduction, and the need to transfuse blood products. Blood product transfusion is the most straightforward solution to address these complications; however, the risk-benefit assessment of blood transfusion must be considered. Transfusionrelated complications include increased postoperative morbidity and mortality, prolonged mechanical ventilation and hospital stay,

374

SECTION IV



Pediatric Critical Care: Cardiovascular

exacerbation of the inflammatory response, and infection.69–72 Modern blood bank testing and donor screening have significantly reduced infectious complications, but noninfectious risk remains a major concern. In addition, smaller patients are exposed to a higher transfusion risk because the transfusion effects may be more pronounced than in adult patients. With a goal of minimizing hemodilution and donor blood exposure, the cardiac team must implement a transfusion algorithm and define a target hematocrit during CPB. Perioperative and developmental outcomes data reported from clinical trials at the Boston Children’s Hospital demonstrate that, when compared with target CPB hematocrit values of 30%, hematocrit values at or below 20% are associated with adverse outcomes and that the benefits of hematocrit values higher than 25% should be further investigated.73,74 The protocol at Children’s Health Dallas is to maintain a hematocrit of at least 30% during CPB. Reducing circuit prime volume and incorporating an ultrafiltration device can significantly reduce donor blood exposure. Originally, a hemoconcentrator added to the CPB circuit was used primarily to remove plasma water and raise the hematocrit. This process is referred to as conventional ultrafiltration (CUF) and its initial use was met with a few limitations. Perfusionists were accustomed to using diuretics to help increase the hematocrit; however, this strategy offered little control and required adequate kidney perfusion during CPB. When ultrafiltration emerged as a CPB technique in the mid-1980s, hemoconcentrators were viewed as an expensive option for removing excess circuit volume. Also, while fluid is removed from the circuit during CUF, the volume in the venous reservoir level diminishes and CUF must be stopped before the reservoir is emptied, which would have catastrophic consequences. Thus, the amount of fluid removed during CUF is dependent on available volume in the venous reservoir, which limits the ability to effectively raise the hematocrit. In 1991, a report described a modified ultrafiltration (MUF) technique performed after weaning the patient from CPB that enabled the perfusionist to concentrate the entire circuit and return most of that volume back to the patient.75 During this era, pediatric circuit prime volume was still rather high; various MUF techniques proved to be a valuable resource for reducing total body water post-CPB. As the prime volumes of CPB circuits have become increasingly smaller relative to the blood volume of the patient, perfusionists have begun to abandon the cumbersome MUF technique in favor of preventing hemodilution rather than reversing hemodilution.76–77 The early popularity of MUF led investigators to explore the potential reduction of proinflammatory mediators during ultrafiltration. An additional ultrafiltration technique, zero balance ultrafiltration (ZBUF), emerged as an alternate method to attenuate the inflammatory response. ZBUF is performed by removing the ultrafiltration effluent from the circuit during CPB while administering a replacement solution (e.g., Plasma-Lyte A) to the venous reservoir in a 1:1 ratio. The highvolume filtration of fluid that is able to be exchanged allows the perfusionist to control electrolyte and glucose levels (e.g., high potassium levels after delivering cardioplegia) and more effectively remove inflammatory mediators. The increasing acceptance of ultrafiltration during CPB led to developing many technique variations during the preoperative, perioperative, and postoperative phases, which can be categorized into two groups: blood concentration and blood filtration. In preparation for neonatal CPB at Children’s Health Dallas, for example, the perfusionist adds approximately 300 mL of PRBCs to the venous reservoir after priming the circuit with

Plasma-Lyte A. The circuit volume is recirculated through the hemoconcentrator, and volume is removed until the reservoir is almost empty. The circuit prime is then “washed” by adding approximately 500 mL of Plasma-Lyte A and then removing that volume. This process is known as prebypass ultrafiltration (PreBUF). In addition to concentrating the circuit, Pre-BUF has been shown to reduce high potassium, glucose, lactate, citrate, and bradykinin levels found in PRBCs. CUF is then performed during the early phase of CPB to remove any excess circuit volume and maintain a hematocrit of 30%. During the warming phase of CPB, the perfusionist performs CUF and ZBUF. This combined ultrafiltration strategy, coupled with a miniaturized circuit, allows the perfusionist to filter the blood and exceed or meet baseline hematocrit values before weaning from CPB.

Hypothermia Deep Hypothermic Circulatory Arrest vs Antegrade Cerebral Perfusion

The therapeutic potential of hypothermia has been known for centuries and has been routinely used in cardiac surgery since its inception. This concept relies on the fundamental physiologic relationship between O2 consumption and temperature. In 1950, Bigelow and colleagues78 compared the use of normothermia and topical cooling on dogs and reported superior ischemic tolerance by surface cooling after 15 minutes of circulatory arrest. The first clinical application in cardiac surgery was reported in 1953 by Lewis and Taufic, who described the successful repair of an ASD in a 5-year-old girl using topical cooling and total body hypothermia.2 To achieve this, patients were submerged in an ice bath to reduce their temperature to approximately 28°C. Then, the defect was closed with the aid of inflow occlusion. In 1958, Sealy and colleagues79 successfully reported the use of hypothermia in conjunction with CPB. The use of CPB with various degrees of hypothermia or deep hypothermic circulatory arrest (DHCA) dramatically increased the “safe” period of support, which enabled surgeons to repair increasingly complex anomalies and allowed cardiac surgery to flourish. Hypothermia suppresses metabolic activity, preserves highenergy phosphate stores, and reduces the reaction rate of biochemical reactions. Several factors are used in determining the type and degree of hypothermia during CPB. The most significant factor is the degree of surgical difficulty and anticipated CPB support time. Complex surgical repairs requiring lengthy support times would benefit from more pronounced hypothermia. The degree of hypothermia varies greatly and is typically classified as mild, moderate, deep, and profound (Table 35.5). Deep hypothermia might be seen as desirable when low flow (#50 mL/kg per minute) or DHCA is desired, as is the case in operations involving complete aortic arch reconstruction. Circulatory arrest is a process in which the perfusion flow is turned off and the TABLE Hypothermia Classifications in Cardiac Surgery 35.5

Category

Core Temperature

Mild

32°C–34°C

Moderate

25°C–32°C

Deep

15°C–25°C

Profound

#15°C

Oxygen consumption (mL· min–1· m–2)

150 37 °C

100 30 °C 25 °C

50

20 °C 15 °C

0 0.0

0.5 1.0 1.5 2.0 Perfusion flow rate (L · min–1· m–2)

2.5

• Fig. 35.8





​Nomogram relating oxygen consumption to perfusion flow rate and temperature. (From Kirklin JW, Barratt-Boyes BG. Hypothermia, circulatory arrest, and cardiopulmonary bypass. In: Kirklin JW, Barratt-Boyes BG, eds. Cardiac Surgery. 2nd ed. New York: Churchill-Livingstone; 1993:91.)

patient’s blood volume is allowed to drain into the venous reservoir. This dramatic application provides an asanguineous and completely motionless surgical field, facilitating complex repairs. In very small patients, the venous cannula may be obstructive and DHCA is required in order to remove the cannula and access the surgical site. Hypothermia also facilitates exposure of the surgical field by allowing decreased perfusion flow rates, which reduces the amount of collateral blood returning to the heart via the pulmonary veins (Fig. 35.8). Patients with pulmonary blood flow restrictions (e.g., tetralogy of Fallot, pulmonary atresia) can develop major aortopulmonary collateral arteries; these collaterals can flood the heart during the aortic cross-clamp period if CPB flow is maintained. This excessive blood return not only obscures the surgical site but may also warm the cold arrested myocardium or wash out cardioplegia from the coronary arteries. Hypothermia and perfusion flow rate reduction can attenuate this collateral flow while maintaining adequate oxygenation to the patient. The rate of cooling varies greatly between different tissue beds; thus, multiple measurement sites are recommended to ensure uniform cooling distribution (Fig. 35.9). The optimal temperature measurement site is controversial, but choosing sites that closely reflect tissue temperatures of vital organs, particularly the brain, is widely accepted. At Children’s Health Dallas, the patient’s nasopharyngeal, rectal, and bladder temperatures are monitored during surgery. Nasopharyngeal temperature closely correlates with brain temperature; however, it may underestimate the global core temperature considering the slower cooling rates of other tissue beds. For this reason, rectal and bladder temperature monitoring sites with slower rates of cooling are typically used to guide cooling end points. The concept of a “safe” circulatory arrest time is controversial, and most guidelines are met with a degree of uncertainty. A nomogram focused on neurologic protection has been devised that estimates safe circulatory arrest times, but values should not be used as absolutes (Fig. 35.10). The historic incidence of perioperative cerebral injury during DHCA has led investigators to explore alternative techniques to protect the brain during complex repairs. Antegrade cerebral perfusion (ACP, also known as selective cerebral perfusion) uses a cannulation technique that directs perfusion flow to only the brain with the theoretic advantage of



CHAPTER 35

Pediatric Cardiopulmonary Bypass

375

protecting it from hypoxic ischemic injury. Though this technique has been adopted among many surgical centers, investigations comparing DHCA and ACP have failed to definitively demonstrate superiority of either technique,80–82 and not all of this work is focused only on neurologic issues. Recent literature has suggested that during ACP, the resultant partial perfusion from collateral vessels provides better protection to abdominal organs than DHCA.83,84 In addition, the ideal temperature during ACP remains unknown. However, recent reports have demonstrated superior results using moderate to mild hypothermia, in the 25°C to 30°C range, rather than deep levels of hypothermia.85,86 Considering the coagulopathy, inflammatory response, and the vascular and organ dysfunction associated with deep hypothermia, investigators have been prompted to explore warmer or normothermic high-flow CPB. Recent studies have shown that high-flow normothermic CPB is as safe as hypothermic CPB during low-risk procedures, and may reduce postoperative inotropic and respiratory support, shorten length of stay, and reduce intraoperative blood transfusion.87,88

pH and Partial Pressure of Arterial Carbon Dioxide Strategy CO2 concentration and pH are primary determinants of cerebral blood flow. As body temperature decreases, the solubility of CO2 increases, resulting in a decreased partial pressure of arterial CO2 (Paco2) and increased pH. Manipulating the acid-base management during hypothermic bypass can be classified by two mechanisms of control: a-stat or pH-stat. Both mechanisms have been studied intensely in reptiles (ectotherms) and hibernating mammals (endotherms), looking at their adaptive blood pH alterations that allow them to withstand extreme temperature fluctuations. pH-stat acid-base management is practiced by hibernating mammals, accomplished by decreasing ventilatory rate and raising Paco2 while maintaining a constant pH during hypothermic conditions. Maintaining pH while varying temperature during hibernation is thought to preserve O2 stores by decreasing metabolic activity. Alternatively, reptiles use a-stat and allow their pH to enter an alkaline state by reducing Paco2. The perfusionist can maintain a pH-stat or “temperature-corrected” acid-base strategy during CPB by allowing CO2 to passively rise or actually adding CO2 to the CPB circuit. Cerebral blood flow has been shown to decrease during hypothermia using the a-stat strategy and demonstrates a linear relationship to the increased Paco2 when a pH-stat strategy is used.89 In addition to preserving cerebral blood flow during hypothermia, a pH-stat strategy induces a rightward shift of the oxyhemoglobin dissociation curve (see Fig. 35.7) potentially allowing for increased O2 off-loading from hemoglobin at the capillary level. Whether pH-stat or a-stat acid-base management during hypothermic CPB demonstrates clear benefits on clinical outcomes has been difficult to demonstrate. However, it is suggested that a pH-stat strategy is optimal for pediatric patients, and a-stat is the optimal strategy for adult patients.90

Myocardial Protection Myocardial protection refers to the strategies and techniques employed to allow the surgeon to work on the heart in a bloodless and motionless field yet recover the best possible postischemic myocardial function and cardiac output for the patient. Strategies for both adults and children attempt to reduce cardiac workload and minimize the metabolic demands and consequences of O2

376

SECTION IV

Pediatric Critical Care: Cardiovascular



Cooling

Rewarming

41 40 39 38 37 36 35 34 33 32 31 30 29 28 Temperature (± SEM)

° Centigrade

27 26

Arterial cannula

25

Myocardial

24

Brain

23

Nasopharyngeal

22 21

Rectal

20

P < .05

19 18 17 16 15 14 13 12 11 10 9 8 7 6 5 0

10

20

30

40

10

20 30 40 Time (minutes)

50

60

70

80

90

• Fig. 35.9

​Relationships of temperatures measured at various sites over time during cooling and warming from cardiopulmonary bypass. (From Stefaniszyn HJ, Novick RJ, Keith FM, et al. Is the brain adequately cooled during deep hypothermic cardiopulmonary bypass? Curr Surg. 1983;40:294–297.)  



deprivation during ischemia. Reducing afterload and emptying the heart by initiating CPB is a myocardial protection technique during beating heart procedures (e.g., palliative shunts, bidirectional Glenn procedure). When the heart needs to be stopped and opened for intracardiac repairs, the aorta is cross-clamped and cardioplegia is delivered to the coronary circulation to cause prolonged asystole. Cardioplegia is a myocardial arrest-producing solution that is formulated to prolong myocardial tolerance to ischemia. There are many techniques to protect the myocardium,

but cardioplegia strategies are often the focus when discussing protection techniques. In North America, high-potassium depolarizing solutions are the most common type of cardioplegia used.34 Universal agreement on an optimal myocardial protective technique is widely debated; most strategies are guided by surgeon or institutional preference. After the first successful cardiac surgical correction using CPB in 1953, surgeons explored a variety of techniques to support the patient during the repair. It did not take long for them to realize

1.0 0.9 0.8 0.7 0.6 0.5

37°C

0.4

28°C

18°C

0.3 0.2 0.1 0.0



Pediatric Cardiopulmonary Bypass

377

of cardioplegia recipes. By the late 1970s, these crystalloid cardioplegia solutions became the dominant form of myocardial protection. A major shift occurred in 1978 when Dr. Buckberg and his group at the University of California, Los Angeles, suggested that blood was an optimal cardioplegia vehicle.93 Blood cardioplegia was shown to be superior to a crystalloid cardioplegia solution because blood provides better O2 delivery, effective buf fering, free radical scavenging, and ideal oncotic pressure.94 Effective myocardial protection has allowed surgeons to approach increasingly complex surgical corrections. The motivation to optimize cardioplegia techniques has led to an incredible variation of myocardial protection techniques. Despite numerous advances and an extensive body of research, myocardial protection techniques continue to be largely program based because hard evidence, from prospective randomized trials, for instance, is lacking.95 The wide variation in protection techniques, however, makes randomized comparisons impractical. A multi-institutional North American survey by Kotani et al.96 suggests that an observational study correlating markers of postoperative myocardial performance with myocardial preservation strategies is warranted. A depolarizing cardioplegia solution uses a high dose of potassium, delivered to the coronary circulation in isolation, to decrease the cardiac membrane resting potential and arrest the heart in diastole. Delivering a cold dose of depolarizing cardioplegia does add a degree of protection, but simply arresting and cooling the heart does not offer optimal protection. An effective cardioplegia solution should (1) achieve quick arrest and minimize ATP depletion, (2) delay the onset of irreversible damage and limit reperfusion injury, (3) be reversible with prompt resumption of cardiac function upon washout, and (4) be nontoxic.97 Efforts to optimize an effective cardioplegia solution have focused on myocyte calcium management and pH buffering. Modified depolarizing cardioplegia, a solution that combines a depolarizing agent with additional membrane-stabilizing additives (e.g., magnesium or lidocaine), has gained interest in attempting to optimize cardioplegia. A modified depolarizing solution that is gaining popularity in congenital surgery is del Nido cardioplegia (Baxter Healthcare Inc.) (Table 35.6).96 Unlike most cardioplegia solutions that require frequent maintenance dosing to achieve effective protection, del Nido cardioplegia is known to provide excellent myocardial protection without the need to frequently redose.98 This reported advantage allows the surgeon to operate uninterrupted while reducing the aortic cross-clamp time. Custodiol HTK (Essential Pharmaceuticals, LLC) is an intracellular cardioplegia that achieves electrical and mechanical arrest by equilibrating the intracellular and extracellular ion concentrations. This solution is also gaining popularity in the congenital patient population because it does not require frequent maintenance doses. ­

Probability of “safe” circulatory arrest

CHAPTER 35

0

10 20 30 40 50 60 70 80 Duration of total circulatory arrest (minutes)

90

• Fig. 35.10





​Probability of a “safe” (absence of structural or functional damage) circulatory arrest according to duration. Estimate at nasopharyngeal temperatures of 37°C, 28°C, and 18°C. (From Kirklin JW, BarrattBoyes BG. Hypothermia, circulatory arrest, and cardiopulmonary bypass. In: Kirklin JW, Barratt-Boyes BG, eds. Cardiac Surgery. 2nd ed. New York: Churchill-Livingstone; 1993:74.)

­

that the poor visibility in an open beating heart needed to be addressed. It was difficult to operate on a beating heart, and lethal air embolism (i.e., air ejected out the aortic valve) claimed the lives of many patients. In 1955, Dr. Melrose and his colleagues91 described a technique of stopping the heart to address these dangerous operative conditions. The report outlines how potassium citrate is used to achieve elective cardiac arrest. This sparked the interest of other investigators. In 1958, Dr. Sealy and his colleagues at Duke reported an additive modification to the Melrose method and coined the term cardioplegia.92 It is interesting to note that improving operative visibility, not protecting the heart, was the goal of arresting the heart. Further investigation began to show that these techniques caused damage to the myocardium and cardioplegia was abandoned. In the 1960s and early 1970s, a number of investigators tried to find an alternative protection technique. Reports of using direct coronary perfusion with intermittent aortic occlusion, topical hypothermia, and normothermic ischemia with aortic occlusion failed to achieve consistent results, presented major time limitations for surgeons, and myocardial damage remained an issue. The observation of “stone heart” by Dr. Cooley and his colleagues92 highlighted the need to prioritize the protection of the myocardium. They then proposed that depleting myocardium adenosine triphosphate (ATP) stores would leave the heart frozen in systole. Fortunately, several European researchers continued to explore cardioplegia solutions throughout the 1960s. They discovered that the chelating action of the citrate ion of the potassium-citrate solution was responsible for myocardial damage because it interferes with cellular calcium and magnesium traffic. This finding resparked interest into pharmacologic cardiac arrest around the world, with a new focus on protecting the myocardium by membrane stabilization and managing calcium and other ion shifts. During the 1970s, a sequence of landmark reports hailed the worldwide return of cardioplegia and its use in protecting the myocardium. These publications identified potassium chloride as a safe arresting agent and, moreover, showed that cold cardioplegia significantly extends the safe is chemic period. The use of cardioplegia additives—such as magnesium, procaine, lidocaine, mannitol, buffering agents, glucose, glutamate, and aspartate—has led to much debate and a plethora

TABLE Crystalloid Formula of del Nido Solution 35.6

Plasma-Lyte A

1000 mL

Potassium chloride

26 mEq

Sodium bicarbonate 8.4%

13 mEq

Mannitol 25%

3.25 g

Lidocaine 2%

130 mg

Magnesium sulfate 50%

2g

378

SECTION IV



Pediatric Critical Care: Cardiovascular

At Children’s Health Dallas, the surgical team uses del Nido cardioplegia with a customized pediatric cardioplegia circuit.99 The cardioplegia is mixed at a 1:4 blood-to-crystalloid ratio; upon aortic cross-clamp placement, a single cold (4–6°C), 20 mL/kg antegrade dose is administered via the aortic root. Although there is no cardioplegia specifically designed for neonatal or congenital patients, several considerations are made to optimize protection. As previously discussed, tolerance of intracellular calcium shifts into the immature myocardium is impaired. The additives lidocaine and magnesium in del Nido cardioplegia both help to control intracellular calcium accumulation. Lidocaine prevents sodium shifts by blocking fast voltage sodium channels, which, in turn, limits calcium shifting into the myocyte. Magnesium, a calcium antagonist, inhibits calcium channel pumps and competes with calcium binding to troponin. These additives help the impaired immature myocardium to maintain effective electrical and mechanical arrest. Hypertrophic ventricles, as seen in tetralogy of Fallot, may not be adequately protected with standard cardioplegia doses. They may require a higher cardioplegia dose and longer delivery time to properly cool and perfuse the hypertrophied myocardium. Cyanotic patients with inadequate pulmonary blood flow often have increased bronchial collateral flow. High collateral flow to the lungs ultimately returns to the heart via the pulmonary veins and is counterproductive while the aorta is cross-clamped. Collateral flow can fill the heart, warm the myocardium, and wash out the cardioplegia from the myocardium. Lowering the systemic temperature not only helps to offset myocardial warming but also allows the perfusionist to lower the perfusion flow and pressure, which, in turn, reduces this collateral flow. Also, adequate left ventricle venting helps to minimize the effect of volume returning to the heart. High sensitivity to the inflammatory response in the immature myocardium can cause edema and reduce ventricular compliance. Mannitol is an additive in del Nido cardioplegia that helps reduce intracellular water accumulation. Care must be taken to not perfuse the myocardium at too high a pressure. This could injure fragile coronary vessels and create myocardial edema. A pressure of 30 to 50 mm Hg has been shown to be adequate.100

Inflammatory Response to Cardiopulmonary Bypass The systemic inflammatory response syndrome (SIRS) instigated by CPB is well documented; limiting this response is associated with improved outcomes.101–103 Multiple factors—including blood exposure to nonendothelialized surfaces (e.g., CPB circuit, air), surgical trauma (e.g., intubation, sternotomy), ischemia-reperfusion, hypothermia, and allogenic transfusion—have been linked to this inflammatory activation. This complex response includes cascade activations of complement, cytokine, coagulation-fibrinolytic, and various cellular activations. Perioperative and postoperative consequences include multiple-organ failure (e.g., myocardium, renal, pulmonary, neurologic, hepatic), coagulopathy, edema, elaboration of injurious oxygen-free radicals, and hypotension.103 SIRS is more pronounced in neonates. The degree of hemodilution, need for longer CPB and ischemic times, and more prevalent use of profound hypothermia are all known to exacerbate SIRS and are contributors to the increased postoperative morbidity seen in neonates compared with older infants and children.104–106 A number of contemporary pharmacologic strategies and CPB techniques are designed to attenuate the inflammatory reactions and remove mediators during CPB. Corticosteroids have been

used during CPB for many years and have been shown to mitigate many inflammatory processes, such as increased capillary permeability, edema, and leukocyte migration. Various timing and dosing protocols have been suggested, with conflicting results.107,108 At Children’s Health Dallas, a 30 mg/kg (1 g maximum dose) perioperative dose of methylprednisolone is administered to the CPB circuit prime. Additionally, 10 mg/kg of methylprednisolone is administered 8 to 12 hours before the initiation of CPB in neonates. A 2018 literature review by Fudulu et al.109 found that while the use of corticosteroids may attenuate markers of inflammation, this reduction does not correlate with improved clinical outcomes. Additionally, Dreher et al.110 demonstrated that eliminating the routine administration of methylprednisolone to the CPB prime did not alter clinical outcomes and was associated with a significant reduction in the incidence of wound infection. The conflicting data and lack of large multicenter, randomized controlled studies that follow a consistent dosing regimen warrants further research and an individualized approach to steroid therapy. Common CPB-based techniques to attenuate SIRS include ultrafiltration, circuit miniaturization, and the use of biocompatible circuit coatings. Ultrafiltration during CPB has been shown to reduce inflammatory mediators, pulmonary injury and edema, postoperative mechanical ventilation support time, and the perioperative need for blood transfusion.111–114 CPB circuit miniaturization and the use of biocompatible surface coatings provide two main benefits in attenuating the inflammatory response. First, the smaller surface area of a miniaturized circuit and biocompatible coating reduces the bioreactivity of blood coming into contact with foreign surfaces.32,33 Second, smaller circuits reduce overall hemodilution and the need to transfuse donor blood, both of which have been shown to exacerbate the inflammatory response.69,70 Although the inflammatory response to CPB cannot be avoided, a multimodal approach can significantly reduce CPBrelated SIRS.

Termination of Cardiopulmonary Bypass Once the surgeon has completed the cardiac repair, the entry sites into the heart are sutured closed. The venous return drainage is retarded, which fills the right heart. The anesthesiologist inflates the lungs, which facilitates the movement of this blood across the pulmonary vasculature and back to the left heart. Any active left heart venting is paused at this point, and the surgeon massages the filling heart to expel blood with any entrained air out of a vent hole in the aortic root. Once there is no longer any air emanating from this site, the patient is placed into the Trendelenburg position, and the cross-clamp is removed from the aorta. The perfusionist then returns to full venous drainage, and the left heart vent can be placed to gentle suction again. The coronary circulation is now reperfused, washing out the cardioplegia. The heart is observed for any return of electrical activity. Temporary epicardial pacing wires are routinely placed, and pacing is initiated if the native rhythm does not promptly return. The left heart vent is removed before initiating ventilation to avoid entrainment air. Transesophageal echocardiography is then used to evaluate for the presence of air in the left heart as ventilation is restarted. If air is present, the vent site in the ascending aorta is kept open to evacuate it. Once the heart is confidently de-aired and the patient has returned to normothermia, the team agrees to wean from CPB. The perfusionist steadily reduces pump flow and venous return until the pump is fully off and the venous line is fully clamped.

The patient’s heart and lungs return to their native support roles at this point. Transesophageal—or, in some cases, epicardial— echocardiography is used to assess the adequacy of the repair. If the repair is deemed successful, a protamine dose, calculated by a heparin-protamine titration (discussed earlier in the Anticoagulation section), is administered. The cannulas are removed and the purse strings are tied down. Adequate reversal of heparin is verified by measuring the ACT and heparin-protamine titration.

Key References Algra SO, Jansen NJ, van der Tweel I, et al. Neurological injury after neonatal cardiac surgery: a randomized, controlled trial of 2 perfusion techniques. Circulation. 2014;129:224-233. Allan CK, Newburger JW, McGrath E, et al. The relationship between inflammatory activation and clinical outcome after infant cardiopulmonary bypass. Anesth Analg. 2010;111:1244-1251. Blinder JJ, Goldstein SL, Lee VV, et al. Congenital heart surgery in infants: effects of acute kidney injury on outcomes. J Thorac Cardiovasc Surg. 2012;143(2):368-374. Fudulu DP, Gibbison B, Upton T, et al. Corticosteroids in pediatric heart surgery: myth or reality. Front Pediatr. 2018;6:112. Ginther Jr RM, Gorney R, Forbess JM. Use of del Nido cardioplegia solution and a low-prime recirculating cardioplegia circuit in pediatrics. J Extra Corpor Technol. 2013;45:46-50.



CHAPTER 35

Pediatric Cardiopulmonary Bypass

379

Kilic A, Whitman GJ. Blood transfusions in cardiac surgery: indications, risks, and conservation strategies. Ann Thorac Surg. 2014;97:726-734. Mokhtari A, Lewis M. Normoxic and hyperoxic cardiopulmonary bypass in congenital heart disease. BioMed Res Int. 2014;2014:678268. Newburger JW, Jonas RA, Soul J, et al. Randomized trial of hematocrit 25% versus 35% during hypothermic cardiopulmonary bypass in infant heart surgery. J Thorac Cardiovasc Surg. 2008;135:347-354, 354.e1-e4. Sood ED, Benzaquen JS, Davies RR, Woodford E, Pizarro C. Predictive value of perioperative near-infrared spectroscopy for neurodevelopmental outcomes after cardiac surgery in infancy. J Thorac Cardiovasc Surg. 2013;145(2):438-445.e431; discussion 444-435. Sturmer D, Beaty C, Clingan S, Jenkins E, Peters W, Si MS. Recent innovations in perfusion and cardiopulmonary bypass for neonatal and infant cardiac surgery. Transl Pediatr. 2018;7(2):139-150. Taylor RL, Borger MA, Weisel RD, Fedorko L, Feindel CM. Cerebral microemboli during cardiopulmonary bypass: increased emboli during perfusionist interventions. Ann Thorac Surg. 1999;68(1):89-93. Xiong Y, Sun Y, Ji B, Liu J, Wang G, Zheng Z. Systematic review and meta-analysis of benefits and risks between normothermia and hypothermia during cardiopulmonary bypass in pediatric cardiac surgery. Paediatr Anaesth. 2015;25(2):135-142.

The full reference list for this chapter is available at ExpertConsult.com.

e1

































































































1. Gross RE, Hubbard JP. Surgical ligation of a patent ductus arteriosus: report of first successful case. JAMA. 1939;112:729-731. 2. Lewis FJ, Taufic M. Closure of atrial septal defects with the aid of hypothermia; experimental accomplishments and the report of one successful case. Surgery. 1953;33:52-59. 3. Lewis FJ, Varco RL, Taufic M. Repair of atrial septal defects in man under direct vision with the aid of hypothermia. Surgery. 1954;36: 538-556. 4. Lewis FJ, Taufic M, Varco RL, Niazi S. The surgical anatomy of atrial septal defects: experiences with repair under direct vision. Ann Surg. 1955;142:401-415. 5. Gibbon Jr JH. Artificial maintenance of circulation during experimental occlusion of pulmonary artery. Arch Surg. 1937;34:1105-1131. 6. Gibbon Jr JH, Miller BJ, Dobell AR, et al. The closure of interventricular septal defects in dogs during open cardiotomy with the maintenance of the cardiorespiratory functions by a pump-oxygenator. J Thorac Surg. 1954;28:235-240. 7. Miller BJ, Gibbon JH, Fineberg C. An improved mechanical heart and lung apparatus; its use during open cardiotomy in experimental animals. Med Clin North Am. 1953;1:1603-1624. 8. Gibbon Jr JH. Application of a mechanical heart and lung apparatus to cardiac surgery. Minn Med. 1954;37:171-185; passim. 9. Dennis C, Spreng Jr DS, Nelson GE, et al. Development of a pumpoxygenator to replace the heart and lungs; an apparatus applicable to human patients, and application to one case. Ann Surg. 1951;134: 709-721. 10. Dodrill FD, Hill E, Gerisch RA, Johnson A. Pulmonary valvuloplasty under direct vision using the mechanical heart for a complete by-pass of the right heart in a patient with congenital pulmonary stenosis. J Thorac Surg. 1953;26:584-594; discussion 195-197. 11. Helmsworth JA, Clark Jr LC, Kaplan S, Sherman RT. An oxygenator-pump for use in total by-pass of heart and lungs; laboratory evaluation and clinical use. J Thorac Surg. 1953;26:617-631; discussion 631-632. 12. Mustard WT. Heart pump and oxygenator. Pediatrics. 1957;19:11241128. 13. Lillehei CW, Cohen M, Warden HE, et al. The results of direct vision closure of ventricular septal defects in eight patients by means of controlled cross circulation. Surg Gynecol Obstet. 1955;101:446-466. 14. Lillehei CW. Controlled cross circulation for direct-vision intracardiac surgery; correction of ventricular septal defects, atrioventricularis communis, and tetralogy of Fallot. Postgrad Med. 1955;17:388396. 15. Kirklin JW, Dushane JW, Patrick RT, et al. Intracardiac surgery with the aid of a mechanical pump-oxygenator system (Gibbon type): report of eight cases. Proc Staff Meet Mayo Clin. 1955;30:201-206. 16. Lillehei CW, Dewall RA, Read RC, et al. Direct vision intracardiac surgery in man using a simple, disposable artificial oxygenator. Dis Chest. 1956;29:1-8. 17. Barratt-Boyes BG, Simpson M, Neutze JM. Intracardiac surgery in neonates and infants using deep hypothermia with surface cooling and limited cardiopulmonary bypass. Circulation. 1971;43(suppl 5): I25-I30. 18. McRobb CM, Mejak BL, Ellis WC, et al. Recent advances in pediatric cardiopulmonary bypass. Semin Cardiothorac Vasc Anesth. 2014;18:153-160. 19. Groom RC, Froebe S, Martin J, et al. Update on pediatric perfusion practice in North America: 2005 survey. J Extra Corpor Technol. 2005; 37:343-350. 20. Sturmer D, Beaty C, Clingan S, Jenkins E, Peters W, Si MS. Recent innovations in perfusion and cardiopulmonary bypass for neonatal and infant cardiac surgery. Translational pediatrics. 2018;7(2):139150. 21. Barrett CS, Jaggers JJ, Cook EF, et al. Pediatric ECMO outcomes: comparison of centrifugal versus roller blood pumps using propensity score matching. ASAIO J. 2013;59:145-151.

22. Barrett CS, Thiagarajan RR. Centrifugal pump circuits for neonatal extracorporeal membrane oxygenation. Pediatr Crit Care Med. 2012; 13:492-493. 23. Lawson S, Ellis C, Butler K, et al. Neonatal extracorporeal membrane oxygenation devices, techniques and team roles: 2011 survey results of the United States’ Extracorporeal Life Support Organization centers. J Extra Corpor Technol. 2011;43:236-244. 24. Meyer AD, Wiles AA, Rivera O, et al. Hemolytic and thrombocytopathic characteristics of extracorporeal membrane oxygenation systems at simulated flow rate for neonates. Pediatr Crit Care Med. 2012;13:e255-e261. 25. Hewitt RL, Creech Jr O. History of the pump oxygenator. Arch Surg. 1966;93:680-696. 26. Lim MW. The history of extracorporeal oxygenators. Anaesthesia. 2006;61:984-995. 27. Gott VL, Dewall RA, Paneth M, et al. A self-contained, disposable oxygenator of plastic sheet for intracardiac surgery; experimental development and clinical application. Thorax. 1957;12:1-9. 28. Gaylor JD. Membrane oxygenators: current developments in design and application. J Biomed Eng. 1988;10:541-547. 29. Crockett J, Grey P. The Variable Prime Cobe Membrane Lung: first impressions. Perfusion. 1987;2:205-212. 30. Lawson DS, Smigla GR, McRobb CM, et al. A clinical evaluation of the Dideco Kids D100 neonatal oxygenator. Perfusion. 2008;23:39-42. 31. Ginther Jr RM, Gorney R, Cruz R. A clinical evaluation of the Maquet Quadrox-i Neonatal oxygenator with integrated arterial filter. Perfusion. 2013;28:194-199. 32. Albes JM, Stöhr IM, Kaluza M, et al. Physiological coagulation can be maintained in extracorporeal circulation by means of shed blood separation and coating. J Thorac Cardiovasc Surg. 2003;126: 1504-1512. 33. Suzuki Y, Daitoku K, Minakawa M, et al. Poly-2-methoxyethylacrylate-coated bypass circuits reduce activation of coagulation system and inflammatory response in congenital cardiac surgery. J Artif Organs. 2008;11:111-116. 34. Harvey B, Shann KG, Fitzgerald D, et al. International pediatric perfusion practice: 2011 survey results. J Extra Corpor Technol. 2012;44:186-193. 35. Hattersley PG. Activated coagulation time of whole blood. JAMA. 1966;196:436-440. 36. Bull BS, Huse WM, Brauer FS, Korpman RA. Heparin therapy during extracorporeal circulation. II. The use of a dose-response curve to individualize heparin and protamine dosage. J Thorac Cardiovasc Surg. 1975;69:685-689. 37. Martindale SJ, Shayevitz JR, D’Errico C. The activated coagulation time: suitability for monitoring heparin effect and neutralization during pediatric cardiac surgery. J Cardiothorac Vasc Anesth. 1996;10:458-463. 38. D’Errico C, Shayevitz JR, Martindale SJ. Age-related differences in heparin sensitivity and heparin-protamine interactions in cardiac surgery patients. J Cardiothorac Vasc Anesth. 1996;10:451-457. 39. Gruenwald CE, Manlhiot C, Crawford-Lean L, et al. Management and monitoring of anticoagulation for children undergoing cardiopulmonary bypass in cardiac surgery. J Extra Corpor Technol. 2010;42:9-19. 40. Codispoti M, Ludlam CA, Simpson D, Mankad PS. Individualized heparin and protamine management in infants and children undergoing cardiac operations. Ann Thorac Surg. 2001;71:922-927; discussion 927-928. 41. Koster A, Fischer T, Praus M, et al. Hemostatic activation and inflammatory response during cardiopulmonary bypass: impact of heparin management. Anesthesiology. 2002;97:837-841. 42. Bolling K, Kronon M, Allen BS, et al. Myocardial protection in normal and hypoxically stressed neonatal hearts: the superiority of hypocalcemic versus normocalcemic blood cardioplegia. J Thorac Cardiovasc Surg. 1996;112:1193-1200; discussion 200-201. 43. Imura H, Lin H, Griffiths EJ, Suleiman MS. Controlled hyperkalemic reperfusion with magnesium rescues ischemic juvenile hearts by reducing calcium loading. J Thorac Cardiovasc Surg. 2011;141:1529-1537.

References

e2









































75.





76.





77.





78.





79.





80.





81.





82.





83.





84.



85.













74.





73.





72.





71.





70.





69.





68.













67.





66.











44. Burch TM, Locke AQ. Air lock and embolism upon attempted initiation of cardiopulmonary bypass while using vacuum-assisted venous drainage. Journal of cardiothoracic and vascular anesthesia. 2012;26(3):468-470. 45. Davila RM, Rawles T, Mack MJ. Venoarterial air embolus: a complication of vacuum-assisted venous drainage. The Annals of thoracic surgery. 2001;71(4):1369-1371. 46. Durandy Y. Vacuum-assisted venous drainage, angel or demon: PRO? The Journal of extra-corporeal technology. 2013;45(2):122-127. 47. Gambino R, Searles B, Darling EM. Vacuum-Assisted Venous Drainage: A 2014 Safety Survey. The Journal of extra-corporeal technology. 2015;47(3):160-166. 48. Jones TJ, Deal DD, Vernon JC, Blackburn N, Stump DA. Does vacuum-assisted venous drainage increase gaseous microemboli during cardiopulmonary bypass? The Annals of thoracic surgery. 2002;74(6):2132-2137. 49. Taylor RL, Borger MA, Weisel RD, Fedorko L, Feindel CM. Cerebral microemboli during cardiopulmonary bypass: increased emboli during perfusionist interventions. The Annals of thoracic surgery. 1999;68(1):89-93. 50. Blinder JJ, Goldstein SL, Lee VV, et al. Congenital heart surgery in infants: effects of acute kidney injury on outcomes. The Journal of thoracic and cardiovascular surgery. 2012;143(2):368-374. 51. Jefferies JL, Devarajan P. Early detection of acute kidney injury after pediatric cardiac surgery. Progress in pediatric cardiology. 2016;41:9-16. 52. Morgan CJ, Zappitelli M, Robertson CM, et al. Risk factors for and outcomes of acute kidney injury in neonates undergoing complex cardiac surgery. The Journal of pediatrics. 2013;162(1):120-127.e121. 53. Riley A, Gebhard DJ, Akcan-Arikan A. Acute Kidney Injury in Pediatric Heart Failure. Current cardiology reviews. 2016;12(2):121-131. 54. Koner O, Tekin S, Koner A, et al. Effects of phentolamine on tissue perfusion in pediatric cardiac surgery. J Cardiothorac Vasc Anesth. 1999;13:191-197. 55. Lindholm L, Hansdottir V, Lundqvist M, Jeppsson A. The relationship between mixed venous and regional venous oxygen saturation during cardiopulmonary bypass. Perfusion. 2002;17:133-139. 56. Morita K. Surgical reoxygenation injury of the myocardium in cyanotic patients: clinical relevance and therapeutic strategies by normoxic management during cardiopulmonary bypass. General thoracic and cardiovascular surgery. 2012;60(9):549-556. 57. Caputo M, Mokhtari A, Miceli A, et al. Controlled reoxygenation during cardiopulmonary bypass decreases markers of organ damage, inflammation, and oxidative stress in single-ventricle patients undergoing pediatric heart surgery. The Journal of thoracic and cardiovascular surgery. 2014;148(3):792-801.e798; discussion 800-791. 58. Mokhtari A, Lewis M. Normoxic and hyperoxic cardiopulmonary bypass in congenital heart disease. BioMed research international. 2014;2014:678268. 59. Redlin M, Koster A, Huebler M, et al. Regional differences in tissue oxygenation during cardiopulmonary bypass for correction of congenital heart disease in neonates and small infants: relevance of nearinfrared spectroscopy. J Thorac Cardiovasc Surg. 2008;136:962-967. 60. Ginther R, Sebastian VA, Huang R, et al. Cerebral near-infrared spectroscopy during cardiopulmonary bypass predicts superior vena cava oxygen saturation. J Thorac Cardiovasc Surg. 2011;142:359-365. 61. Kirshbom PM, Forbess JM, Kogon BE, et al. Cerebral near infrared spectroscopy is a reliable marker of systemic perfusion in awake single ventricle children. Pediatr Cardiol. 2007;28:42-45. 62. Mittnacht AJ. Near infrared spectroscopy in children at high risk of low perfusion. Curr Opin Anaesthesiol. 2010;23:342-347. 63. Tortoriello TA, Stayer SA, Mott AR, et al. A noninvasive estimation of mixed venous oxygen saturation using near-infrared spectroscopy by cerebral oximetry in pediatric cardiac surgery patients. Paediatr Anaesth. 2005;15:495-503. 64. Yoshitani K, Ohnishi Y. The clinical validity of the absolute value of near infrared spectroscopy. J Anesth. 2008;22:502-504. 65. Li J, Zhang G, Holtby H, et al. The influence of systemic hemodynamics and oxygen transport on cerebral oxygen saturation in neo-

nates after the Norwood procedure. J Thorac Cardiovasc Surg. 2008;135:83-90, 90.e1-e2. Hoffman GM, Brosig CL, Mussatto KA, et al. Perioperative cerebral oxygen saturation in neonates with hypoplastic left heart syndrome and childhood neurodevelopmental outcome. J Thorac Cardiovasc Surg. 2013;146:1153-1164. Sood ED, Benzaquen JS, Davies RR, Woodford E, Pizarro C. Predictive value of perioperative near-infrared spectroscopy for neurodevelopmental outcomes after cardiac surgery in infancy. The Journal of thoracic and cardiovascular surgery. 2013;145(2):438-445.e431; discussion 444-435. Richmond ME, Charette K, Chen JM, et al. The effect of cardiopulmonary bypass prime volume on the need for blood transfusion after pediatric cardiac surgery. J Thorac Cardiovasc Surg. 2013;145:1058-1064. Kilic A, Whitman GJ. Blood transfusions in cardiac surgery: indications, risks, and conservation strategies. Ann Thorac Surg. 2014;97: 726-734. Kipps AK, Wypij D, Thiagarajan RR, et al. Blood transfusion is associated with prolonged duration of mechanical ventilation in infants undergoing reparative cardiac surgery. Pediatr Crit Care Med. 2011;12:52-56. Lavoie J. Blood transfusion risks and alternative strategies in pediatric patients. Paediatr Anaesth. 2011;21:14-24. Whitney G, Daves S, Hughes A, et al. Implementation of a transfusion algorithm to reduce blood product utilization in pediatric cardiac surgery. Paediatr Anaesth. 2013;23:639-646. Jonas RA, Wypij D, Roth SJ, et al. The influence of hemodilution on outcome after hypothermic cardiopulmonary bypass: results of a randomized trial in infants. J Thorac Cardiovasc Surg. 2003;126:1765-1774. Newburger JW, Jonas RA, Soul J, et al. Randomized trial of hematocrit 25% versus 35% during hypothermic cardiopulmonary bypass in infant heart surgery. J Thorac Cardiovasc Surg. 2008;135:347-354, 354.e1-e4. Naik SK, Knight A, Elliott MJ. A successful modification of ultrafiltration for cardiopulmonary bypass in children. Perfusion. 1991;6:41-50. McRobb CM, Ing RJ, Lawson DS, Jaggers J, Twite M. Retrospective analysis of eliminating modified ultrafiltration after pediatric cardiopulmonary bypass. Perfusion. 2017;32(2):97-109. Mejak BL, Lawson DS, Ing RJ. Con: Modified Ultrafiltration in Pediatric Cardiac Surgery Is No Longer Necessary. Journal of cardiothoracic and vascular anesthesia. 2019;33(3):870-872. Bigelow WG, Callaghan JC, Hopps JA. General hypothermia for experimental intracardiac surgery: the use of electrophrenic respirations, an artificial pacemaker for cardiac standstill, and radio-frequency rewarming in general hypothermia. Ann Surg. 1950;132:531-537. Sealy WC, Brown Jr IW, Young Jr WG. A report on the use of both extracorporeal circulation and hypothermia for open heart surgery. Ann Surg. 1958;147:603-613. Algra SO, Jansen NJ, van der Tweel I, et al. Neurological injury after neonatal cardiac surgery: a randomized, controlled trial of 2 perfusion techniques. Circulation. 2014;129:224-233. Algra SO, Kornmann VN, van der Tweel I, et al. Increasing duration of circulatory arrest, but not antegrade cerebral perfusion, prolongs postoperative recovery after neonatal cardiac surgery. J Thorac Cardiovasc Surg. 2012;143:375-382. Misfeld M, Leontyev S, Borger MA, et al. What is the best strategy for brain protection in patients undergoing aortic arch surgery? A single center experience of 636 patients. Ann Thorac Surg. 2012;93:1502-1508. Algra SO, Schouten AN, van Oeveren W, et al. Low-flow antegrade cerebral perfusion attenuates early renal and intestinal injury during neonatal aortic arch reconstruction. J Thorac Cardiovasc Surg. 2012;144:1323-1328, 1328.e1-e2. Wang J, Ginther RM, Riegel M, et al. The impact of temperature and pump flow rate during selective cerebral perfusion on regional blood flow in piglets. J Thorac Cardiovasc Surg. 2013;145:188-194; discussion 194-195. Tsai JY, Pan W, Lemaire SA, et al. Moderate hypothermia during aortic arch surgery is associated with reduced risk of early mortality. J Thorac Cardiovasc Surg. 2013;146:662-667.

e3































101. Butler J, Rocker GM, Westaby S. Inflammatory response to cardiopulmonary bypass. Ann Thorac Surg. 1993;55:552-559. 102. Levy JH, Tanaka KA. Inflammatory response to cardiopulmonary bypass. Ann Thorac Surg. 2003;75:S715-S720. 103. Wan S, LeClerc JL, Vincent JL. Inflammatory response to cardiopulmonary bypass: mechanisms involved and possible therapeutic strategies. Chest. 1997;112:676-692. 104. Allan CK, Newburger JW, McGrath E, et al. The relationship between inflammatory activation and clinical outcome after infant cardiopulmonary bypass. Anesth Analg. 2010;111:1244-1251. 105. Hovels-Gurich HH, Vazquez-Jimenez JF, Silvestri A, et al. Production of proinflammatory cytokines and myocardial dysfunction after arterial switch operation in neonates with transposition of the great arteries. J Thorac Cardiovasc Surg. 2002;124:811-820. 106. Appachi E, Mossad E, Mee RB, Bokesch P. Perioperative serum interleukins in neonates with hypoplastic left-heart syndrome and transposition of the great arteries. J Cardiothorac Vasc Anesth. 2007;21:184-190. 107. Graham EM, Atz AM, McHugh KE, et al. Preoperative steroid treatment does not improve markers of inflammation after cardiac surgery in neonates: results from a randomized trial. J Thorac Cardiovasc Surg. 2014;147:902-908. 108. Scrascia G, Rotunno C, Guida P, et al. Perioperative steroids administration in pediatric cardiac surgery: a meta-analysis of randomized controlled trials. Pediatr Crit Care Med. 2014;15: 435-442. 109. Fudulu DP, Gibbison B, Upton T, et al. Corticosteroids in Pediatric Heart Surgery: Myth or Reality. Frontiers in pediatrics. 2018;6:112. 110. Dreher M, Glatz AC, Kennedy A, Rosenthal T, Gaynor JW. A Single-Center Analysis of Methylprednisolone Use during Pediatric Cardiopulmonary Bypass. The Journal of extra-corporeal technology. 2015;47(3):155-159. 111. Darling E, Searles B, Nasrallah F, et al. High-volume, zero balanced ultrafiltration improves pulmonary function in a model of postpump syndrome. J Extra Corpor Technol. 2002;34:254-259. 112. Huang H, Yao T, Wang W, et al. Continuous ultrafiltration attenuates the pulmonary injury that follows open heart surgery with cardiopulmonary bypass. Ann Thorac Surg. 2003;76:136-140. 113. Sever K, Tansel T, Basaran M, et al. The benefits of continuous ultrafiltration in pediatric cardiac surgery. Scand Cardiovasc J. 2004; 38:307-311. 114. Song LO, Yinglong LI, Jinping LI. Effects of zero-balanced ultrafiltration on procalcitonin and respiratory function after cardiopulmonary bypass. Perfusion. 2007;22:339.

































86. Zierer A, El-Sayed Ahmad A, Papadopoulos N, et al. Selective antegrade cerebral perfusion and mild (28°C–30°C) systemic hypothermic circulatory arrest for aortic arch replacement: results from 1002 patients. J Thorac Cardiovasc Surg. 2012;144:1042-1049. 87. Corno AF, Bostock C, Chiles SD, et al. Comparison of Early Outcomes for Normothermic and Hypothermic Cardiopulmonary Bypass in Children Undergoing Congenital Heart Surgery. Frontiers in pediatrics. 2018;6:219. 88. Xiong Y, Sun Y, Ji B, Liu J, Wang G, Zheng Z. Systematic Review and Meta-Analysis of benefits and risks between normothermia and hypothermia during cardiopulmonary bypass in pediatric cardiac surgery. Paediatric anaesthesia. 2015;25(2):135-142. 89. Murkin JM, Farrar JK, Tweed WA, et al. Cerebral autoregulation and flow/metabolism coupling during cardiopulmonary bypass: the influence of PaCO2. Anesth Analg. 1987;66:825-832. 90. Abdul Aziz KA, Meduoye A. Is pH-stat or alpha-stat the best technique to follow in patients undergoing deep hypothermic circulatory arrest? Interact Cardiovasc Thorac Surg. 2010;10:271-282. 91. Melrose DG, Dreyer B, Bentall HH, Baker JB. Elective cardiac arrest. Lancet. 1955;269:21-22. 92. Shiroishi MS. Myocardial protection: the rebirth of potassiumbased cardioplegia. Tex Heart Inst J. 1999;26:71-86. 93. Follette DM, Mulder DG, Maloney JV, Buckberg GD. Advantages of blood cardioplegia over continuous coronary perfusion or intermittent ischemia. Experimental and clinical study. J Thorac Cardiovasc Surg. 1978;76:604-619. 94. Barner HB. Blood cardioplegia: a review and comparison with crystalloid cardioplegia. Ann Thorac Surg. 1991;52:1354-1367. 95. Bartels C, Gerdes A, Babin-Ebell J, et al. Cardiopulmonary bypass: evidence or experience based? J Thorac Cardiovasc Surg. 2002;124:20-27. 96. Kotani Y, Tweddell J, Gruber P, et al. Current cardioplegia practice in pediatric cardiac surgery: a North American multiinstitutional survey. Ann Thorac Surg. 2013;96:923-929. 97. Chambers DJ, Fallouh HB. Cardioplegia and cardiac surgery: pharmacological arrest and cardioprotection during global ischemia and reperfusion. Pharmacol Ther. 2010;127:41-52. 98. Matte GS, del Nido PJ. History and use of del Nido cardioplegia solution at Boston Children’s Hospital. J Extra Corpor Technol. 2012;44:98-103. 99. Ginther RM Jr, Gorney R, Forbess JM. Use of del Nido cardioplegia solution and a low-prime recirculating cardioplegia circuit in pediatrics. J Extra Corpor Technol. 2013;45:46-50. 100. Allen BS, Barth MJ, Ilbawi MN. Pediatric myocardial protection: an overview. Semin Thorac Cardiovasc Surg. 2001;13:56-72.

e4

Abstract: Cardiopulmonary bypass (CPB), which originated in the mid-twentieth century, was designed to allow for the repair of congenital heart defects. Its history has since been characterized by perpetual technological advancements that have been instrumental in sustaining the momentum of clinical progress of this field. The current guidelines for use of CPB to treat congenital heart defects are designed to meet the metabolic demands of the patient throughout the repair while minimizing the impact of associated nonphysiologic effects. The progress of CPB in repair of

congenital heart defects has played a major role in the steady reduction of morbidity and mortality associated with cardiac surgery in children. Pediatric mortality rates are now comparable to those in adult patients. Key words: Cardiopulmonary bypass, perfusionist, congenital heart defect, oxygenation, anticoagulation, ultrafiltration, hypothermia, myocardial protection, systemic inflammatory response

36 Critical Care After Surgery for Congenital Cardiac Disease PAULA HOLINSKI, JENNIFER TURI, VEERAJALANDHAR ALLAREDDY, V. BEN SIVARAJAN, AND ALEXANDRE T. ROTTA





Congenital anomalies account for the largest diagnostic category among causes of infant mortality in the United States.1 Structural heart disease leads the list of congenital malformations. Of the more than 4 million children born each year in the United States, nearly 40,000 have some form of congenital heart disease (CHD). Approximately half of these children appear for therapeutic intervention within the first year of life; the majority require critical care expertise in pediatric intensive care units (PICUs). We recognize that many centers have developed separate specialized pediatric cardiac intensive care units to care for these patients, while some continue to cohort cardiac patients within a general PICU. In this chapter, the PICU designation is used interchangeably to denote the unit caring for critically ill patients necessitating care following surgery or procedures to treat congenital cardiac conditions. These patients now represent a major diagnostic category for admissions in large PICUs across the country, accounting for 30% to 40% or more of PICU admissions in many centers. In addition 380







The neonatal myocardium is less compliant than that of the older child, less tolerant of increases in afterload, and less responsive to increases in preload. A predictable decrease in cardiac index typically occurs 6 to 12 hours after separation from cardiopulmonary bypass, but milrinone administration during the early postoperative period may attenuate this phenomenon. Patients with postoperative low cardiac output (CO) require careful evaluation for unanticipated residual lesions. Patients with restrictive physiology from hypertrophy and diastolic dysfunction of the right ventricle may require high right-sided filling pressures to achieve adequate cardiac output, making them prone to hepatic congestion, anasarca, pleural effusions, and ascites. Inhaled nitric oxide plays an important role in the management of postoperative pulmonary hypertension in the cardiac intensive care unit. Hypoxemia after bidirectional cavopulmonary anastomosis generally is a sign of decreased pulmonary blood flow related to reduced cardiac output.





















PEARLS Liberation from positive-pressure mechanical ventilation should be accomplished as soon as feasible, particularly in patients after a cavopulmonary anastomosis (bidirectional Glenn) or Fontan operation because spontaneous breathing improves pulmonary blood flow, arterial oxygen saturation, and ventricular preload. Ventricular ectopy and elevated atrial pressures after the arterial switch operation should raise suspicion of myocardial ischemia from insufficient coronary blood flow. Postoperative care of the patient with hypoplastic left heart syndrome after stage I palliation (Norwood procedure) may require delicate balancing of the pulmonary and systemic blood flows. A high arterial oxygen saturation denotes excessive pulmonary blood flow and in patients with impaired ventricular output is generally accompanied by inadequate systemic blood flow, acidosis, and end-organ dysfunction.

to the traditional pediatric-age patients, many PICUs now also care for young adult survivors of congenital heart disease, since these patients now outnumber children with congenital heart disease in the general population.2

Neonatal Considerations Care of the critically ill neonate requires an appreciation of the special structural and functional features of immature organs, the interactions of the transitional neonatal circulation, and the secondary effects of the congenital heart lesion on other organ systems.3–5 The neonate responds more quickly and profoundly to physiologically stressful circumstances, such as rapid changes in pH, lactic acid, blood glucose, and temperature. Neonates have diminished fat and carbohydrate reserves compared with older children; however, they have a higher metabolic rate. Immaturity of the liver and kidney may be associated with reduced protein



CHAPTER 36 Critical Care After Surgery for Congenital Cardiac Disease

• BOX 36.1 Advantage of Neonatal Repair Early elimination of cyanosis Early elimination of congestive heart failure Optimal circulation for growth and development Reduced anatomic distortion from palliative procedures Reduced hospital admissions while awaiting repair Reduced parental anxiety while awaiting repair

lesions as truncus arteriosus, complete atrioventricular (AV) canal defects, and transposition of the great arteries (TGA) with ventricular septal defects (VSDs). Finally, cognitive and psychomotor abnormalities associated with months of hypoxemia or abnormal hemodynamics may be diminished or eliminated by early repair. However, if early reparative surgery results in more exposures to CPB (e.g., repeated conduit changes) and associated adverse effects on cognitive or motor function, then the risk-to-benefit assessment must be modified accordingly.

Preoperative Care















Optimal preoperative care involves (1) initial stabilization, airway management, and establishment of adequate vascular access; (2) complete and thorough noninvasive delineation of the anatomic defect(s); (3) initiation/termination of prostaglandin therapy, as appropriate; (4) evaluation and treatment of secondary organ dysfunction, particularly the brain, kidneys, and liver; and (5) cardiac catheterization if necessary, typically for (a) physiologic assessment (e.g., vascular response to oxygen or another pulmonary vasodilator), (b) interventions such as balloon atrial septostomy or valvotomy, or (c) anatomic definition not possible by echocardiography (e.g., coronary artery distribution in pulmonary atresia with intact ventricular septum or delineation of aorticopulmonary collaterals in TOF with pulmonary atresia). Magnetic resonance imaging (MRI) and magnetic resonance angiography have emerged as important adjuvant diagnostic modalities in the evaluation of the cardiovascular system, including qualitative assessments of valve and ventricular function, and quantification of flow, ventricular volume, mass, and ejection fraction.12,13 Congenital heart defects can be complex and difficult to categorize or conceptualize. Rather than trying to determine the management for each individual anatomic defect, a physiologic approach can be taken. The following questions should be asked: 1. How does the systemic venous return reach the systemic arterial circulation to maintain cardiac output? 2. What, if any, intracardiac mixing, shunting, or outflow obstruction exists? 3. Are the pulmonary and systemic circulations in series or parallel? 4. Are the defects amenable to a two-ventricle or single-ventricle repair? 5. Is pulmonary blood flow increased or decreased? 6. Is there a volume load or pressure load on the ventricles? Appropriate organization of preoperative data, patient preparation, and decisions about monitoring, anesthetic agents, and postoperative care are best accomplished by focusing on a few major pathophysiologic problems, beginning with whether the patient is cyanotic, is in CHF, or both. Most pathophysiologic mechanisms that are pertinent to optimal patient preparation and to the perioperative plan focus on one of the following major problems: severe hypoxemia, excessive pulmonary blood flow, CHF, obstruction of blood flow from the left heart, and poor ventricular function. Although some patients with congenital heart disease present with only one of these problems, many have multiple interrelated issues.

synthesis and glomerular filtration such that drug metabolism is altered and hepatic synthetic function is reduced. These issues may be compounded by the normal increased total body water of the neonate compared with the older patient, along with the propensity for capillary leakage. This is especially prominent in the immature lung of the neonate, in which the pulmonary vascular bed is nearly fully recruited at rest, and the lymphatic recruitment required to handle elevated mean capillary pressures associated with increases in pulmonary blood flow may be suboptimal.5 The neonatal myocardium is less compliant than that of the older child, less tolerant of increases in afterload, and less responsive to increases in preload. Younger age also predisposes the myocardium to the adverse effects of cardiopulmonary bypass (CPB) and hypothermic ischemia implicit in support techniques used during cardiac surgery. These factors do not preclude intervention in the neonate but rather simply dictate that extraordinary vigilance be applied to the care of these children and that intensive care management plans account for the immature physiology. The observed benefits of neonatal reparative operations in patients with two ventricles are numerous (Box 36.1). Elimination of cyanosis and congestive heart failure (CHF) early in life optimizes conditions for normal growth and development. Palliative procedures such as pulmonary artery banding and creation of systemic-to-pulmonary artery shunts do not fully address cyanosis or CHF and may introduce their own set of physiologic and anatomic complications. Examples of improved outcomes with a single reparative operation rather than staged palliation as a newborn are well known and evoke little controversy. Approaches that have been abandoned include banding the pulmonary arteries in truncus arteriosus,6 staging repair of type B interrupted aortic arch (IAA),7 and staging repair of transposition of the great arteries with IAA.8 In other conditions (e.g., severely cyanotic newborn with tetralogy of Fallot [TOF]), the risks and benefits of neonatal repair versus a palliative shunt are debatable.9 Whereas the neonate may be more labile than the older child, there is ample evidence that this age group is more resilient in its response to various forms of stress, including metabolic or ischemic injury. Tolerance of hypoxemia in the neonate is characteristic of many species,10 and the plasticity of the neurologic system in the neonate is well known.11 It is the rule rather than the exception that neonates presenting with shock secondary to obstructive left heart lesions can be effectively resuscitated without persistent endorgan impairment. The pliability and mobility of vascular structures in the neonate improve the technical aspects of surgery. Reparative operations in neonates take advantage of normal postnatal changes, allowing more normal growth and development in crucial areas such as myocardial muscle, pulmonary parenchyma, and coronary and pulmonary angiogenesis. Postoperative pulmonary hypertensive events are more common in the infant who has been exposed to weeks or months of high pulmonary pressure and flow.6 This is especially true for such

381

Severe Hypoxemia In the first few days of life, many of the cyanotic forms of CHD present with severe hypoxemia (Pao2 ,50 mm Hg) in the absence of respiratory distress. Infusion of prostaglandin E1 (PGE1) in

382

SECTION IV



Pediatric Critical Care: Cardiovascular

patients with decreased pulmonary blood flow maintains or reestablishes pulmonary flow through the ductus arteriosus. This may also improve mixing of venous and arterial blood at the atrial level in patients with transposition of the great arteries.14 Consequently, neonates rarely require surgery while severely hypoxemic. PGE1 dilates the ductus arteriosus of the neonate with lifethreatening ductus-dependent cardiac lesions and improves the patient’s condition before surgery. PGE1 can reopen a functionally closed ductus arteriosus even several days after birth, or it can be used to maintain patency of the ductus arteriosus for several months postnatally.14,15 The common side effects of PGE1 infusion—apnea, hypotension, fever, central nervous system (CNS) excitation—are easily managed in the neonate when normal therapeutic doses of the drug (0.01–0.05 mg/kg per minute) are used.16 However, PGE1 is a potent vasodilator; thus, intravascular volume may require augmentation at higher doses. Patients with intermittent apnea resulting from administration of PGE1 may require mechanical ventilation preoperatively, although apnea can resolve with the concomitant administration of aminophylline or caffeine.17 PGE1 usually improves the arterial oxygenation of hypoxemic neonates who have poor pulmonary perfusion as a result of obstructed pulmonary flow (critical pulmonic stenosis or pulmonary atresia) by providing pulmonary blood flow from the aorta via the ductus arteriosus. The improved oxygenation reverses the lactic acidosis that often develops during episodes of severe hypoxia, and clinical improvement is seen in a matter of minutes to hours.18

Excessive Pulmonary Blood Flow Excessive pulmonary blood flow is frequently the primary problem of patients with CHD. The intensivist must carefully evaluate the hemodynamic and respiratory impact of left-to-right shunts and the extent to which it contributes to the perioperative course in the ICU. Children with left-to-right shunts may have chronic low-grade pulmonary infection and congestion that cannot be eliminated despite optimal preoperative preparation. If so, surgery should not be postponed further. Respiratory syncytial viral infections are particularly prevalent in this population, but advances in intensive care have markedly improved outcomes with this and other viral pneumonias.19 Aside from the respiratory impairment caused by increased pulmonary blood flow, the left heart must dilate to accept pulmonary venous return that might be several times normal. If the body requires more systemic blood flow, the neonatal heart responds inefficiently, as most of the increment in cardiac output is recirculated to the lungs. Eventually, symptoms of CHF appear. Medical management with inotropes, systemic vasodilators, and diuretics may improve the patient’s condition, but the diuretics may induce profound hypochloremic alkalosis and potassium depletion that often persist after surgery.

Obstruction of Left Heart Outflow Patients who require surgery to relieve obstruction to outflow from the left heart are among the most critically ill children in the ICU. These lesions include interruption of the aortic arch, critical coarctation of the aorta, aortic stenosis (AS), and mitral stenosis or atresia as part of the hypoplastic left heart syndrome (HLHS) spectrum. These neonates present with inadequate systemic perfusion and profound metabolic acidosis. The initial pH may be below 7 despite a low partial pressure of arterial carbon dioxide

(Paco2). Systemic blood flow is largely or completely dependent on blood flow into the aorta from the ductus arteriosus; thus, its closure causes a dramatic worsening of the patient’s condition. The patient suddenly becomes critically ill, and survival requires PGE1 infusion to allow blood flow into the aorta from the pulmonary artery.18,20 PGE1 infusion improves perfusion and metabolism in neonates with acidosis, metabolic derangements, and renal failure because of inadequate systemic perfusion so that surgery generally can be deferred until stability is achieved. Ventilatory and inotropic support and correction of metabolic acidosis— along with calcium, glucose, and electrolyte abnormalities— should occur preoperatively. Adequacy of stabilization, rather than severity of illness at presentation, appears to influence postoperative outcome the most.21

Ventricular Dysfunction Ideally, the intensivist should participate in the preoperative care of all patients who are expected to recover in the ICU after surgery. Understanding the extent of ventricular dysfunction preoperatively provides considerable insight into intraoperative and postoperative events. Although patients with large shunts may have only mild-to-moderate hypoxemia as a result of excessive pulmonary blood flow, the price paid for near-normal arterial oxygen saturation is chronic ventricular dilation and dysfunction and pulmonary vascular obstructive disease. Older patients with CHD and poor ventricular function as a result of chronic ventricular volume overload (aortic or mitral valve regurgitation or longstanding systemic-to-pulmonary arterial shunts) present a different challenge, mitigated to some extent by afterload reduction. However, great care must be exercised in hearts with chronic volume overload as there is a propensity for ventricular fibrillation during sedation, anesthesia, or intubation of the airway. For patients with significantly increased pulmonary-to-systemic flow ratio (Qp/Qs), systemic blood flow should be optimized without further augmenting pulmonary flow during induction of anesthesia in the ICU or in the operating room.

Postoperative Care Assessment When the clinical course of patients after cardiac surgery deviates from the usual expectation of uncomplicated recovery, our first responsibility is to verify the accuracy of the preoperative diagnosis and the adequacy of surgical repair. For example, a young infant who is acidotic, hypotensive, and cyanotic after surgical repair of TOF may tempt us to ascribe these findings to the vagaries of ischemia/reperfusion injury caused by CPB or transient, postoperative stiffness of the right ventricle. However, the real culprit may be an additional VSD undetected preoperatively and therefore not closed, a significant surgical patch leak, or residual right ventricle (RV) outflow obstruction. Correct postoperative assessment is imperative, and treatment follows accordingly. Evaluation of the postoperative patient relies on examination, monitoring, interpretation of vital signs, and imaging. Only when the accuracy of the diagnosis and adequacy of the repair are established can a CPB-related low–cardiac output state be presumed and treatment optimized. Treating low–cardiac output states and preventing cardiovascular collapse often are the central features of pediatric cardiac intensive care and are the focus of this chapter



CHAPTER 36 Critical Care After Surgery for Congenital Cardiac Disease

• BOX 36.2 Ten Intensive Care Strategies to Diagnose

and Support Low–Cardiac Output States



























1. Know in detail the cardiac anatomy and its physiologic consequences. 2. Understand the specialized considerations of the newborn and implications of reparative rather than palliative surgery. 3. Diversify personnel to include experts in neonatal and adult congenital heart disease. 4. Monitor, measure, and image the heart to rule out residual disease as a cause of postoperative hemodynamic instability or low cardiac output. 5. Maintain aortic perfusion and improve the contractile state. 6. Optimize preload (including atrial shunting). 7. Reduce afterload. 8. Control heart rate, rhythm, and synchrony. 9. Optimize heart-lung interactions. 10. Provide mechanical support when needed.

(Box 36.2). The details of the specific considerations for selected lesions are presented in their respective sections. The initial assessment following cardiac surgery begins with a review of the operative findings. This includes details of the operative repair and CPB, particularly total CPB time, myocardial ischemia (aortic cross-clamp), and circulatory arrest or antegrade perfusion times; concerns about myocardial protection; recovery of myocardial contractility; typical postoperative systemic arterial and central venous pressures; findings from intraoperative transesophageal or epicardial echocardiography, if performed; vasoactive medication requirements; and hemostatic management. This information guides subsequent examination, which should focus on the quality of the repair or palliation plus a clinical assessment of cardiac output (Box 38.3). In addition to a complete cardiovascular examination immediately upon arrival to the PICU, a routine set of laboratory tests should be obtained, including a chest radiograph, 12- or 15-lead electrocardiography (ECG), blood gas analysis, serum electrolytes and glucose, ionized calcium and lactate measurements, complete blood count, and coagulation profile. This information, together with an understanding of the preoperative and postoperative loading conditions of the heart, are • BOX 36.3 Signs of Heart Failure or Low–Cardiac

Output States Signs Cool extremities/poor perfusion Oliguria and other end-organ failure Tachycardia Hypotension Acidosis Cardiomegaly Pleural effusions

Monitor and Assess Heart rate, blood pressure, intracardiac pressure Extremity temperature, central temperature Urine output Mixed venous oxygen saturation Arterial blood gas pH and lactate Laboratory measures of end-organ function Echocardiography

383

essential for clinical management in the immediate postoperative period. Optimizing preload involves more than just giving volume to a hypotensive patient. There are numerous considerations to fluid balance involving types of isotonic fluid, ultrafiltration in the operating room, optimal hematocrit, and the use of diuretics or vasopressors. Fluid itself can be detrimental if excess extravascular water results in interstitial edema and end-organ dysfunction of vital organs such as the heart, lungs, and brain. Occasionally, permitting a right-to-left shunt at the atrial level can optimize preload to the left ventricle in some conditions (discussed later). Maintaining aortic perfusion after CPB and improving the contractile state of the heart with higher doses of catecholamines are reasonable goals, but they may have particularly deleterious consequences in the newborn myocardium after hypothermic CPB. The benefits of afterload reduction are well known but, if excessive, may result in hypotension, coronary insufficiency, and cardiovascular collapse. Pacing the heart can stabilize the rhythm and hemodynamics, but it also may contribute to dyssynchronous, inefficient cardiac contraction or may induce other arrhythmias. Although lifesaving in many instances, mechanical support of the failing myocardium in the form of extracorporeal life support (ECLS) or ventricular assist devices has its own set of limitations and morbidities. Almost every treatment approach has its own set of adverse effects. Supporting cardiac output in the postoperative patient is a balance between the promise and poison of therapy.

Monitoring The goal of postoperative monitoring following cardiac surgery primarily focuses on assessing the adequacy of circulatory status and oxygen delivery. The level of vigilance in the immediate postoperative period can be optimized by the combination of laboratory values and physical examination findings with data obtained via noninvasive devices and invasive monitoring to assess intracardiac pressures and oxygen saturations. Near-infrared spectroscopy (NIRS) provides a noninvasive, continuous estimate of regional oxygen supply and demand that serves as a surrogate for hemodynamics and cerebral and somatic oxygenation. It is based on the differential absorption of varying wavelengths of light by hemoglobin as it associates with oxygen to measure oxygen content in a localized tissue bed.22 While data demonstrating a definitive impact on overall outcome are lacking, there are reports that correlate decreased cerebral and/or somatic NIRS with increased mortality, prolonged length of stay (LOS), and worsening neurologic outcomes.23,24 Invasive monitoring of central venous pressure is routine for most patients following cardiac surgery. Intracardiac or transthoracic left atrial (LA) catheters are often used to monitor patients after complex reparative procedures. Pulmonary arterial (PA) catheters now are seldom used but may be particularly useful if one anticipates postoperative pulmonary hypertension, allowing rapid detection of pressure changes and assessment of the response to interventions. LA catheters are especially helpful in the management of patients with ventricular dysfunction, coronary artery perfusion abnormalities, or mitral valve disease. The mean LA pressure typically is 1 to 2 mm Hg greater than mean right atrial (RA) pressure, which generally varies between 1- and 6-mm Hg in nonpostoperative pediatric patients undergoing cardiac catheterization. In postoperative patients, mean LA and RA pressures are often greater than 6 to 8 mm Hg. However, they generally should be less than 15 mm Hg. The compliance of the right atrium is

384

SECTION IV



Pediatric Critical Care: Cardiovascular

• BOX 36.4 Common Causes of Elevated Left Atrial

Pressure After Cardiopulmonary Bypass Decreased ventricular systolic or diastolic function Left atrioventricular valve disease Large left-to-right intracardiac shunt Chamber hypoplasia Intravascular or ventricular volume overload Cardiac tamponade Arrhythmia

greater than that of the left atrium except in the newborn; thus, pressure elevations in the right atrium of older patients with two ventricles typically are less pronounced. Possible causes of abnormally elevated LA pressure are listed in Box 38.4. In addition to pressure data, intracardiac catheters in the right atrium (or a percutaneously placed central venous catheter), left atrium, and pulmonary artery can be used to monitor the oxygen saturation of systemic or pulmonary venous blood and indicate the presence or absence of atrioventricular synchrony. Following reparative surgery, patients with no intracardiac shunts and adequate cardiac output may have a mild reduction in RA oxygen saturation to approximately 60%. Lower RA oxygen saturation does not necessarily indicate low cardiac output. If a patient has arterial desaturation (complete mixing lessons, lung diseases, and so on), the arteriovenous oxygen saturation difference is normally ,30%. Hence, even a low RA saturation may be in keeping with appropriate oxygen delivery and extraction. Elevated RA oxygen saturation often is the result of left-to-right shunting at the atrial level (e.g., from the left atrium, anomalous pulmonary vein, or left ventricular [LV]-to-RA shunt). Blood in the LA normally is fully saturated with oxygen (i.e., approximately 100%). The two chief causes of reduced LA oxygen saturation are an atrial-level right-to-left shunt and pulmonary venous desaturation from abnormal gas exchange. In the absence of left-to-right shunts, PA oxygen saturation is the best representation of the “true” mixed venous oxygen saturation because all sources of systemic venous blood should be thoroughly mixed as they are ejected from the RV. When elevated, this saturation is useful in identifying residual significant left-to-right shunts following repair of a VSD. The absolute value of the PA oxygen saturation is a predictor of significant postoperative residual shunt. In patients following TOF or VSD repair, PA oxygen saturation greater than 80% within 48 hours of surgery with a fractional inspired oxygen concentration (Fio2) less than 0.5 is a sensitive indicator of significant left-to-right shunt (Qp/Qs .1.5) 1 year after surgery.25

Low Cardiac Output Syndrome Although low cardiac output after CPB is often attributable to residual or undiagnosed structural lesions, progressive low–cardiac output states do occur. A number of factors have been implicated in the development of myocardial dysfunction following CPB, including (1) the inflammatory response associated with CPB, (2) myocardial ischemia from aortic cross-clamping, (3) hypothermia, (4) reperfusion injury, (5) inadequate myocardial protection, and (6) ventriculotomy (when performed). The typical decrease in cardiac index has been well characterized in newborns following an arterial switch operation (ASO).26 In a group of 122 newborns, the

median maximal decrease in cardiac index that typically occurred 6 to 12 hours after separation from CPB was 32%, and 1 in 4 of these newborns reached a nadir of the cardiac index lower than 2 L/min per square meter.26 Anticipation of this low cardiac output syndrome (LCOS) and appropriate intervention can do much to avert morbidity or the need for mechanical support. Mixed venous oxygen saturation, whole-blood pH, and lactate are laboratory measures commonly used to evaluate the adequacy of tissue perfusion and, hence, cardiac output.

Volume Adjustments After CPB, the factors that influence cardiac output—such as preload, afterload, myocardial contractility, heart rate, and rhythm—must be continuously assessed and manipulated as needed. Volume expansion (increased preload) is commonly necessary, followed by appropriate use of vasoactive agents. Atrial pressure and the ventricular response to changes in atrial pressure must be evaluated. Ventricular response is judged by observing systemic arterial pressure and waveform, heart rate, skin color, peripheral extremity temperature, peripheral pulse magnitude, urine output, core body temperature, and acid-base balance. Preserving and Creating Right-to-Left Shunts Selected children with low cardiac output may benefit from strategies that allow right-to-left shunting at the atrial level in the face of expected postoperative RV dysfunction. A typical example is early repair of TOF, when the hypertrophied and poorly compliant right ventricle may be further compromised by increased volume load from pulmonary regurgitation secondary to a transannular patch on the RV outflow tract. These children will benefit from leaving the foramen ovale patent to permit right-to-left shunting of blood, thus preserving cardiac output and oxygen delivery despite the attendant transient cyanosis. When the foramen ovale is not patent or is surgically closed, RV dysfunction can lead to reduced LV filling, low cardiac output, and, ultimately, LV dysfunction. In infants and neonates with repaired truncus arteriosus, the same concerns apply and may even be exaggerated if RV afterload is elevated because of pulmonary hypertension. Other Strategies Additional strategies to support low cardiac output associated with cardiac surgery in children include the use of atrioventricular pacing for patients with complete heart block or prolonged interventricular conduction delays and asynchronous contraction.27 Appreciation of the hemodynamic effects of positive and negative pressure ventilation may be used to assist cardiac output. Avoidance of elevated body temperatures and even inducing hypothermia along with appropriate sedation and even paralysis may provide end-organ protection during periods of low cardiac output and aid in the management of postoperative arrhythmias, such as junctional ectopic tachycardia.

Mechanical Cardiac Support Perioperative mechanical circulatory support (MCS) can be lifesaving for critically ill children and young adults with CHDs.28,29 The most common form of pediatric MCS is extracorporeal membrane oxygenation (ECMO). ECMO can be used as rescue during extracorporeal cardiopulmonary resuscitation (ECPR), in failure to wean from CPB, or in patients who develop low–cardiac output states postoperatively despite high levels of pharmacologic support.30,31 it is presently less commonly used as a bridge to heart



CHAPTER 36 Critical Care After Surgery for Congenital Cardiac Disease

transplantation—it is more often used as a prelude to long-term mechanical assist devices or simply to allow time for decisionmaking.29,32 An analysis of 96,596 operations from 80 centers reporting to the Society of Thoracic Surgeons Congenital Heart Surgery Database showed that MCS was used in 2.4% of cases.33 Children who underwent Norwood procedures (17%) or complex biventricular repairs (14%) were more likely to receive MCS. Substantial variation exists in MCS rates across both high- and low-volume centers. Overall, 53% of those children who received MCS did not survive to hospital discharge, with mortality greater than 70% for certain operative lesions (truncus arteriosus repair, Ross-Konno operation).33 Despite this high mortality, it is important to recognize that survival would have been virtually zero for those children without MCS. In a recent report from the Pediatric Cardiac Critical Care Consortium Registry,34 of the 14,526 eligible medical as well as surgical cardiac ICU hospitalizations, 449 (3.1%) had at least one ECMO run. Of these, 329 (3.5%) were surgical and 120 (2.4%) were medical hospitalizations. Of the surgical group, 33 (10%) included preoperative ECMO only and 296 (90%) included postoperative ECMO. Overall, in-hospital mortality was 48.9% in the surgical group and 63.3% in the medical group; mortality rates for hospitalizations including ECPR were 82.7% (surgical) and 50% (medical).34 Due to improved technology, reliability, and mechanical durability of devices, higher rates of patient survival, reduced adverse events, and limited availability of organs for transplantation, intracorporeal MCS has become an accepted long-term inpatient and outpatient therapy for those with advanced heart failure awaiting heart transplantation.35 Ventricular assist devices (VADs) can support function of the left ventricle (left ventricular assist device [LVAD]), right ventricle (right ventricular assist device [RVAD]), or both (biventricular assist device [BiVAD]). A total artificial heart (TAH) replaces the heart itself. VADs have two different mechanisms of blood flow: pulsatile or continuous. Continuous flow devices contain an impeller that rotates at high speed to propel blood. These include axial flow impellers (e.g., HeartMate II [Thoratec Corp.]), or centrifugal pumps (e.g., HeartWare HVAD [HeartWare Corp.]). Paracorporeal pulsatile devices (e.g., Thoratec PVAD-BiVAD [Thoratec Corp.], Berlin Heart EXCOR BiVAD [Berlin Heart Corp.]) are also used in selected cases. Guidelines exist with regard to CPR in children with MCS.35 A more detailed discussion of MCS can be found in Chapter 28.

Right Ventriculotomy and Restrictive Physiology Right ventricular restrictive physiology has been demonstrated by echocardiography as persistent anterograde diastolic blood flow into the pulmonary circulation following reconstruction of the RV outflow in infants and children. This occurs in the setting of elevated RV end-diastolic pressure and RV hypertrophy when the right ventricle demonstrates diastolic dysfunction with an inability to properly relax and fill during diastole. The poorly compliant RV usually is not dilated in this circumstance, and pulmonary valve regurgitation is limited because of the elevated RV diastolic pressure.36,37 The term restrictive RV physiology is also commonly used in the immediate postoperative period in patients who have a stiff, poorly compliant, and sometimes hypertrophied RV. The elevated ventricular end-diastolic pressure restricts filling during diastole, causing an increase in RA filling pressure and, ultimately, systemic venous hypertension. Because of the phenomenon of ventricular

385

interdependence, changes in RV diastolic function and septal position in turn affect LV compliance and function. Factors contributing to diastolic dysfunction include lung and myocardial edema following CPB, inadequate myocardial protection of the hypertrophied ventricle during aortic cross-clamp, coronary artery injury, residual outflow tract obstruction, volume load on the ventricle from a residual VSD or pulmonary regurgitation, and dysrhythmias. In many centers, a residual atrial communication is left to mitigate the perioperative sequelae associated with restrictive RV physiology, namely, a low–cardiac output state. In such a scenario, patients may be desaturated following surgery (typically in the 75% to 85% range) because of this right-to-left shunting, but they maintain systemic cardiac output while avoiding significant systemic venous hypertension. As RV compliance and function improve (usually within 2 to 3 postoperative days), the amount of shunt decreases and both anterograde pulmonary blood flow and Sao2 increase. If significant restrictive RV physiology develops in the absence of an unrestrictive atrial communication, a low–cardiac output state with increased right-sided filling pressure (usually .12 mm Hg) ensues. Such patients often have cool extremities, oliguria, and metabolic acidosis. As a result of the elevated RA pressure, hepatic congestion, ascites, increased chest tube output, and pleural effusions may be evident. These patients may be tachycardic and hypotensive, with a narrow pulse pressure. Preload must be maintained despite an already elevated RA pressure. Significant inotropic support often is required (typically, epinephrine 0.05–0.1 µg/kg per minute). A phosphodiesterase inhibitor, such as milrinone, can be beneficial because of its lusitropic properties; however, one must be cautious in the use of these agents with renal impairment.38 Sedation and paralysis often are necessary for the first 24 to 48 hours to minimize energy expenditure and associated myocardial work. Factors that further impair ventricular diastolic filling—such as loss of AV synchrony, accumulation of pleural fluid or ascites, and high tidal volume ventilation with air trapping—should be mitigated early in the postoperative course. Mechanical ventilation, either hypoinflation or hyperinflation of the lung, hypothermia, and acidosis can contribute to increased afterload on the right ventricle and pulmonary regurgitation. Synchronized intermittent positive-pressure ventilation with the lowest possible mean airway pressure should be the aim, as discussed previously.

Diastolic Dysfunction Occasionally, there is an alteration of ventricular relaxation, an active energy-dependent process, which reduces ventricular compliance. This is particularly problematic in patients with a hypertrophied ventricle undergoing surgical repair, such as TOF or Fontan surgery, and following CPB in some neonates when myocardial edema may significantly restrict diastolic function.37,38 The poorly compliant ventricle with impaired diastolic relaxation has a reduced end-diastolic volume and stroke volume. b-Adrenergic antagonists and calcium channel blockers add little to the treatment of this condition. In fact, hypotension or myocardial depression produced by these agents often outweighs any gain from slowing the heart rate. Calcium channel blockers are relatively contraindicated in neonates and small infants because of their dependence on transsarcolemmal flux of calcium to both initiate and sustain contraction. A gradual increase in intravascular volume to augment ventricular capacity, in addition to the use of low doses of inotropic

386

SECTION IV



Pediatric Critical Care: Cardiovascular

agents, is of modest benefit in patients with diastolic dysfunction. Tachycardia must be avoided and AV synchrony maintained to optimize diastolic filling time and decrease myocardial oxygen demands. If low cardiac output persists despite treatment, vasodilators can be carefully attempted to alter systolic wall tension (afterload) and thus decrease the impediment to ventricular ejection. Because the capacity of the vascular bed increases after vasodilation, simultaneous volume replacement is often necessary. A noncatecholamine inodilator with vasodilating and lusitropic (improved diastolic state) properties, such as milrinone, is useful under these circumstances in contrast with other inotropic agents.39

Pharmacologic Support General principles of pharmacologic support in the neonatal and pediatric patient center on the recognition of the developmental limitations of the neonatal myocardium and the well-described reductions in cardiac output 6 to 12 hours after separation from CPB.26 Despite ongoing development and maturity of adrenergic receptors and L-type calcium channels, catecholamine-based inotropic agents and vasodilators are efficacious in this population. Other nonvasoactive agents serve as adjuncts to optimizing postoperative hemodynamics and fluid balance. The combination of a low-dose inotrope and an afterload reducing agent—or, more commonly, a phosphodiesterase inhibitor—has been shown to decrease the occurrence of postoperative LCOS following CPB.39 It should be understood that the need for vasoactive and inotropic support varies greatly among patients recovering from cardiac surgery and even over time for an individual patient progressing through the postoperative care continuum. The intensity of pharmacologic support employed must be constantly evaluated. One must not embrace a false sense of security when caring for a patient with adequate cardiac output when this requires disproportionate pharmacologic support. The current approach is to employ the lowest level of inotropic and vasoactive support necessary for the achievement of hemodynamic goals. Due to advances in preoperative stabilization, anesthetic strategies, surgical technique, myocardial protection, and CPB, it is not uncommon for a patient to return to the PICU requiring only low doses of milrinone or epinephrine or no pharmacologic support at all. A detailed discussion of cardiovascular pharmacology is beyond the scope of this chapter but can be found in Chapter 31.

surgically. In these cases, the use of pulmonary vasodilator strategies augments only residual or undiagnosed shunts and increases the volume load on the heart. Several factors peculiar to CPB may raise PVR: pulmonary vascular endothelial dysfunction, microemboli, pulmonary leukostasis, excess thromboxane production, atelectasis, hypoxic pulmonary vasoconstriction, and adrenergic events all have been suggested to play a role in postoperative pulmonary hypertension. Postoperative pulmonary vascular reactivity has been related not only to the presence of preoperative pulmonary hypertension and left-to-right shunts but also to the duration of total CPB. The threat of postoperative pulmonary hypertensive crises can be partially addressed by providing surgery at earlier ages, pharmacologic interventions, and other postoperative management strategies (Table 36.1).

Pulmonary Vasodilators Nitric oxide (NO) is the mainstay of therapy in patients with pulmonary hypertension requiring critical care. NO is a vasodilator formed by the endothelium from l-arginine and molecular oxygen in a reaction catalyzed by NO synthase. NO then diffuses to the adjacent vascular smooth muscle cells, where it induces vasodilation through a cyclic guanosine monophosphate-dependent pathway.42,43 Because NO exists as a gas, it can be delivered by inhalation to the alveoli and, hence, to the adjacent blood vessels. Once it diffuses across the wall of the pulmonary blood vessels, NO enters the vascular lumen. There, it is rapidly inactivated by hemoglobin, resulting in selective pulmonary vasodilation. Inhaled NO (iNO) has advantages over intravenously administered vasodilators that may cause systemic hypotension and increase intrapulmonary shunting. Inhaled NO lowers pulmonary artery pressure in a number of diseases without the unwanted effect of systemic hypotension. This effect is especially dramatic in children with cardiovascular disorders and postoperative patients with pulmonary hypertensive crises.41,44,45 Therapeutic uses of iNO in children with CHD abound in the ICU. Newborns with total anomalous pulmonary venous connection (TAPVC) frequently have obstruction of the pulmonary venous pathway where it connects anomalously to the systemic

TABLE Critical Care Strategies for Postoperative 36.1 Treatment of Pulmonary Hypertension

Managing Acute Pulmonary Hypertension in the Intensive Care Unit

Encourage

Avoid

Anatomic investigation

Residual anatomic disease

Children with many forms of CHD are prone to perioperative elevations in pulmonary vascular resistance (PVR).40 This situation complicates the postoperative course when transient myocardial dysfunction is further challenged by increased RV afterload.41 Although postoperative patients with pulmonary hypertension often are presumed to have active and reversible pulmonary vasoconstriction as the source of their pathophysiology, the intensivist is obligated to explore anatomic causes of mechanical obstruction that impose a barrier to pulmonary blood flow. Elevated LA pressure, pulmonary venous obstruction, branch pulmonary artery stenosis, or surgically induced loss of the vascular tree all raise RV pressure and impose an unnecessary burden on the right heart. Similarly, a residual or undiagnosed left-to-right shunt raises pulmonary artery pressure postoperatively and must be addressed

Opportunities for right-to-left shunt as pop-off

Intact atrial septum in right heart failure

Sedation/anesthesia

Agitation/pain

Moderate hyperventilation

Respiratory acidosis

Moderate alkalosis

Metabolic acidosis

Adequate inspired oxygen

Alveolar hypoxia

Normal lung volumes

Atelectasis or overdistension

Optimal hematocrit

Excessive hematocrit

Inotropic support

Low output and coronary perfusion

Vasodilators

Vasoconstrictors/increased afterload



CHAPTER 36 Critical Care After Surgery for Congenital Cardiac Disease

venous circulation. When pulmonary venous return is obstructed preoperatively, pulmonary hypertension is severe and requires urgent surgical relief. Use of iNO in this or other settings of suspected pulmonary venous obstruction is contraindicated. On the other hand, increased neonatal pulmonary vasoreactivity, endothelial injury induced by CPB, and remodeling of the pulmonary vascular bed in this disease contribute to postoperative pulmonary hypertension. Postoperatively in the patient with TAPVC after adequate surgical relief of obstruction, iNO dramatically reduces pulmonary hypertension without adverse changes in heart rate, systemic blood pressure, or vascular resistance. Postoperative patients with TAPVC, congenital mitral stenosis, and other pulmonary venous hypertensive disorders associated with low cardiac output are among the most responsive to iNO. These infants are born with significantly increased amounts of smooth muscle in their pulmonary arterioles and venules. Histologic evidence of muscularized pulmonary veins and pulmonary arteries suggests the presence of vascular tone and capacity for change in resistance at both the arterial and venous sites. The increased responsiveness to iNO seen in younger patients with pulmonary venous hypertension may result from pulmonary vasorelaxation at a combination of precapillary and postcapillary vessels. Resolving the primary venous obstruction is of utmost importance before using iNO in these lesions. Several groups have reported successful use of iNO in a variety of other congenital heart defects following cardiac surgery. Inhaled NO is especially helpful when administered during a pulmonary hypertensive crisis.46 Successful iNO use has been described after the Fontan procedure, following late VSD repair, and with a variety of other anatomic lesions for which patients are at risk of developing postoperative pulmonary hypertensive crises.45–47 Oxygen saturation in response to iNO generally does not improve in very young infants who are excessively cyanotic after a bidirectional Glenn anastomosis.48 In these cases, increasing cardiac output and cerebral blood flow will have a much greater impact on arterial oxygenation because elevated pulmonary vascular tone is seldom the limiting factor in the hypoxemic patient after the bidirectional Glenn operation.49 Inhaled NO can be used diagnostically in neonates with RV hypertension after cardiac surgery to discern those with reversible vasoconstriction. In patients with Ebstein anomaly, a clinical response to iNO can accurately differentiate between functional and anatomic pulmonary atresia.50 In addition, the use of iNO in such patients can facilitate anterograde pulmonary blood flow and hemodynamic stabilization. Failure of the postoperative newborn to respond to iNO should be regarded as strong evidence of anatomic and possibly surgically remediable obstruction.50 If iNO must be discontinued before the pathologic process has been resolved, hemodynamic instability can be expected. The withdrawal response to iNO can be attenuated by pretreatment with the type V phosphodiesterase inhibitor sildenafil.51 Sildenafil inhibits the inactivation of cyclic guanosine monophosphate within the vascular smooth muscle cell and has the potential to augment the effects of either endogenous or exogenously administered NO to affect vascular smooth muscle relaxation. Sildenafil can be administered in an oral or intravenous (IV) form and has a somewhat selective pulmonary vasodilating capacity while lowering LA pressure and providing a modest degree of systemic afterload reduction in some postoperative children. Chronic oral administration of sildenafil to adults with primary pulmonary hypertension improves exercise capacity. This phenomenon has

387

also been demonstrated in pediatric patients with a Fontan circulation, perhaps suggesting a broad therapeutic application on older patients after operation for CHD. Many other vasodilators have been used with variable success in patients with pulmonary hypertensive disorders requiring critical care. IV vasodilators—such as tolazoline, phenoxybenzamine, nitroprusside, and isoproterenol—have little biological basis for selectivity or enhanced activity in the pulmonary vascular bed.52 However, if myocardial contractility is depressed and the afterload reducing effect on the left ventricle is beneficial to myocardial function and cardiac output, then these drugs may be of some value. In addition to drug-specific side effects, intravenous vasodilators all have the potential to produce profound systemic hypotension, critically lowering coronary perfusion pressure while simultaneously increasing intrapulmonary shunt, thus limiting their usefulness in the management of acute postoperative pulmonary hypertension. In fact, in patients with idiopathic pulmonary hypertension who have adequate LV contractility, the use of a vasopressor may help the RV coronary perfusion pressure, LV preload and systolic interventricular dependence, thus preventing a pulmonary hypertensive crisis.

Management of Postoperative Bleeding Infants and children undergoing cardiac surgery are at high risk for hemorrhage and need for transfusion. CPB is a major thrombogenic stimulus that causes a multifactorial coagulopathy due to dilution and consumption of clotting factors and inactivation of platelets. This is further exacerbated by an immature coagulation system and the presence of hypoxia and hypothermia. While upward of 79% of children undergoing cardiac surgery will require transfusion,53 the need for transfusion is associated with increased morbidity.54,55 Transfusion criteria for packed red blood cells is determined by balancing the inherent risks associated with transfusion and the need to optimize oxygen delivery in the face of hemodynamic instability. However, there are increasing data suggesting that the use of more restrictive transfusion parameters results in less transfusion with no difference in clinical outcome.56 To minimize the need for excessive blood products, careful attention to ongoing bleeding and coagulopathy is necessary. Baseline complete blood count and coagulation profile should be measured on return from the operating room. Chest tube outputs of 10 mL/kg in the first hour or 5 mL/kg per hour for subsequent hours should prompt aggressive repletion of abnormal clotting factors, initially with platelets and fresh-frozen plasma (FFP), assessment of adequate reversal of anticoagulants, and a discussion with the surgeon. When bleeding is resistant to therapy, transfusion of factor concentrates should be considered. These include fibrinogen concentrate, prothrombin complex concentrates, or even activated factor VII in selected patients. Significant repletion of red blood cells also necessitates the concurrent transfusion of platelets and FFP to avoid further dilution of clotting components. Ongoing chest tube output that does not abate despite normalization of factors suggests the possibility of surgical bleeding that could necessitate reexploration. Further, while control of postoperative bleeding is the goal, the abrupt cessation of chest tube output—particularly when accompanied by increasing CVP, tachycardia, and hypotension—suggests evolving tamponade. Ensuring chest tube patency may be sufficient to reverse the process. If not, reexploration of the mediastinum may be necessary.

388

SECTION IV



Pediatric Critical Care: Cardiovascular

Cardiac Tamponade The infant’s crowded mediastinum makes compression of the heart and cardiac tamponade an ever-present possibility after chest closure, despite patent drainage tubes and surgical resection of the anterior pericardium. The warning signs of tamponade frequently are subtle in small children, even minutes before cardiovascular collapse. Any significant deterioration in hemodynamics after chest closure should first be attributed to tamponade if ventilation and cardiac rhythm are adequate. The signs of tamponade include tachycardia, hypotension, narrow pulse pressure, and high filling pressures on both the left and right sides of the heart. Acute myocardial perforation with tamponade occasionally occurs during interventional cardiac catheterization procedures. Prompt support of the circulation with volume infusions and pressor support, along with immediate catheter drainage of the pericardial space, are essential in the event of this complication. Hemopericardium after ventricular puncture usually is selflimited, as the muscular ventricle seals the perforation after the responsible wire or catheter is removed. However, laceration of the thin-walled atrium may require suture repair under direct vision in the operating room. Other causes of cardiac tamponade are seen in patients with CHD; treatment frequently requires the assistance of an intensivist for either pericardiocentesis or sedation and monitoring for that definitive procedure. Postoperative tamponade from bleeding immediately after operation, as discussed earlier, is best handled by facilitation of chest tube drainage or reopening the sternotomy. Some children develop pericardial effusions during later phases of their illness because of hydrostatic influences (e.g., patients with modified Fontan operations) or postpericardiotomy syndrome. Fluid in the pericardial space may accumulate under considerable pressure and to the point at which filling of the heart is impaired. If this problem is left unattended, the transmural pressure in the atria diminishes as intraatrial pressures rise, and diastolic collapse of the atria can be observed echocardiographically. Patients become symptomatic with a narrow pulse pressure, pulsus paradoxus, tachycardia, respiratory distress, decreased urine output, hyperkalemia, metabolic acidosis, and hypotension with tremendous endogenous catecholamine response.

Diaphragmatic Dysfunction, Effusions, and Pulmonary Issues Diaphragmatic paresis (reduced motion) or paralysis (paradoxical movement) may precipitate and promote respiratory failure, particularly in the neonate or young infant who largely relies on diaphragmatic function for breathing; older infants and children can recruit accessory and intercostal muscles if diaphragmatic function proves inadequate. Injury to the phrenic nerve may occur during operations that require dissection of the branch pulmonary arteries well out to the hilum (e.g., TOF repair, ASO), arch reconstruction from the midline (e.g., Norwood operation), manipulation of the superior vena cava (SVC; Glenn shunt), takedown of a systemic-to-pulmonary shunt, or after attempted percutaneous central venous access. Phrenic nerve injury occurs more frequently at reoperation, when adhesions and scarring may obscure anatomic landmarks. Extensive thymectomy during neonatal operations to improve exposure also can result in phrenic nerve injury. Topical cooling with ice during deep hypothermia

may cause transient phrenic palsy. Increased work of breathing on low ventilator settings, increased Paco2, and a chest radiograph revealing an elevated hemidiaphragm suggest diaphragmatic dysfunction. However, the chest radiograph may be misleading if it is obtained at the end of inspiration during positive-pressure ventilation when lung volume is at its highest. Ultrasonography is most useful for identifying reduced diaphragmatic motion or paradoxical excursion. Diaphragmatic dysfunction may be transient and resolve over time. However, a patient who fails repeated extubation attempts despite optimizing cardiovascular and nutritional status, and in whom diaphragmatic dysfunction persists with lung volume loss in the affected side, necessitates surgical plication of the diaphragm.57 Although only a temporary effect is gained from plication, the prevention of collapse and volume loss in the affected lung from paradoxical movement of the diaphragm often provides the critical advantage needed for liberation from positive-pressure ventilation. Pleural effusions and ascites may occur in patients after any type of cardiothoracic surgical procedures, especially following the Fontan operation or repairs involving a right ventriculotomy (e.g., TOF, truncus arteriosus) with transient RV dysfunction. Especially in young patients, pleural effusions and increased interstitial lung water may be a manifestation of right heart failure. This seems logically related to raised systemic venous pressure impeding lymphatic return to the venous circulation. Pleural or peritoneal fluid and intestinal distension compete with intrapulmonary gas for thoracic space. Evacuation of the pleural space, drainage of ascites, and bowel decompression facilitate restoration of lung volume. Pulmonary edema, pneumonia, and atelectasis are common causes of abnormal postoperative gas exchange and hypoxemia. If a bacterial pathogen is identified in the respiratory secretions, antibiotics should be initiated promptly. If pulmonary edema is responsible for the gas exchange abnormality, therapy is aimed at lowering the LA pressure through diuresis and pharmacologic means to reduce afterload and improve the lusitropic state of the heart. For infants, fluid restriction frequently is incompatible with adequate nutrition; therefore, an aggressive diuretic regimen is preferable to restriction of caloric intake. Adjustment of endexpiratory pressure and mechanical ventilation serve as supportive therapies until the alveoli and pulmonary interstitium are cleared of the fluid that interferes with gas exchange.

Chylothorax Chylothorax develops in 0.25% to 9.2% of children after cardiac surgery and is associated with negative outcomes, including longer LOS, higher hospitalization costs, and increased risk of in-hospital mortality.58–60 The etiologies and pathophysiology of lymphatic dynamic disorders in these children are poorly understood, but new insights are emerging.61 The thoracic duct ascends to the right of the vertebral column, crosses over to the left hemithorax at the fifth thoracic vertebral body, and drains into the venous circulation at the region of the left subclavian and left jugular veins. In general, chylothorax can be classified as traumatic or nontraumatic. Direct injury to the thoracic duct or its tributaries causes traumatic chylothorax, whereas processes that elevate the central venous pressure (e.g., RV diastolic dysfunction, Fontan physiology, thrombosis or obstruction of the subclavian or internal jugular veins) may lead to nontraumatic chylothorax from alterations in the Starling forces.62



CHAPTER 36 Critical Care After Surgery for Congenital Cardiac Disease

Multiple diagnostic and therapeutic algorithms have been reported in the literature.63,64 Initial investigation includes a chest radiograph or ultrasound to confirm the presence of an effusion, followed by diagnostic and/or therapeutic thoracentesis with pleural fluid analysis and an echocardiogram. Vascular ultrasound imaging or cardiac catheterization may be needed to delineate the etiology further in select cases. Typically, chylothorax should be suspected when a “milky” exudate or unilateral effusion is noted in the postoperative period, classically after enteral feeding is resumed. However, a milky appearance alone is insufficient to diagnose chylothorax.65 Fluid triglyceride levels and cell count with differential are required to further establish the diagnosis. Fluid triglycerides greater than 110 mg/dL or less than 50 mg/dL essentially confirm or exclude the diagnosis, respectively; uncertain cases with values 50 to 110 mg/dL may require additional testing, such as lipoprotein analysis for the demonstration of chylomicrons. In children who are not on enteral feeds or are malnourished, a lipoprotein analysis is suggested even with triglycerides less than 50 mg/dL. Typical pleural fluid in chylothorax has a white cell count greater than 1000/mL with lymphocytic predominance (.80%), and a low lactate dehydrogenase level.65 In addition, chyle has a high protein (.20 g/L) and immunoglobulin content.66,67 Atypical fluid characteristics—such as transudates, neutrophil-predominance, or high lactate dehydrogenase measurement—signal another etiology, such as heart, liver, or kidney dysfunction, or infection. Prolonged chylothorax increases the risk of infection, poor wound healing, malnutrition, fluid and electrolyte imbalances, and delayed separation from respiratory support, all of which may lead to worse outcomes.58,66 Management of postoperative chylothorax can be challenging and includes both conservative and interventional treatments, with considerable institutional practice variation.63,64 In general, treatment begins with the insertion of a chest tube to drain the effusion, confirm the diagnosis, and provide symptomatic relief. Postoperative chylothorax can be divided into low volume (#20 mL/kg per day) or high volume (.20 mL/kg per day) output. Children with low-volume chylothorax are generally started on a high medium-chain triglyceride (MCT), low long-chain triglyceride diet for 7 days. The high MCT diet is continued for 6 weeks in those patients who respond with a decrease in output to less than 10 mL/kg per day. Those who fail this initial dietary modification or have high-volume chylothorax are generally treated with enteral fasting and parenteral nutrition for 7 to 10  days, with consideration for concomitant initiation of somatostatin or its synthetic analog octreotide, administered intravenously or subcutaneously.68,69 Absence of response to this strategy after 2 weeks should prompt consideration of surgical exploration to identify and repair the lymphatic injury or ligate the thoracic duct.63,70 Most high-output chylothorax resolves or significantly improves after surgical intervention. For those that do not, an additional week of enteral fasting and octreotide should be attempted before considering pleurodesis or placement of a pleuroperitoneal shunt.63,71 A recent analysis of the Pediatric Health Information Systems (PHIS) database reported that thoracic duct ligation or pleurodesis was performed at a median of 18 days after the cardiac surgery, and patients were discharged from the hospital at a median of 22 days after surgical treatment of chylothorax.58 More recently, percutaneous thoracic duct embolization has emerged as a less invasive alternative for the treatment of chylothorax.72–74 Newer studies, such as dynamic contrast-enhanced magnetic lymphangiography and intranodal lymphangiography, have provided further insight and therapeutic options for this complex

389

disorder.72–76 Other individualized supportive therapies include administration of 25% albumin for patients with serum albumin less than 2.5g/dL, intravenous immunoglobulin (IVIG) for those with low IgG levels, and multivitamins.

Separating from Mechanical Ventilation Early tracheal extubation of children following congenital heart surgery is not a new concept but has received renewed attention with the evolution of fast-track management for cardiac surgical patients. Early extubation generally refers to tracheal extubation in the operating room or within a few hours (i.e., 4–8 hours) after surgery, although in practice, it means the avoidance of routine overnight mechanical ventilation. Factors to consider when planning early extubation are given in Table 36.2. A number of published reports have described successful tracheal extubation in neonates and older children following congenital heart surgery either in the operating room or soon after in the cardiac ICU.77 This has been possible without adversely affecting patient care and with a low incidence of reintubation or hemodynamic instability. Such a process can reduce complications such as ventilator-associated events but does not obviate meticulous attention to postoperative analgesia and sedation. The judicious use of this practice has streamlined care and highlights the advances in perioperative care of infants and older children after repair of congenital heart defects.78

TABLE Considerations for Planned Early Extubation 36.2 After Congenital Heart Surgery

Factor

Consideration

Patient

Limited cardiorespiratory reserve of the neonate and infant Pathophysiology of specific congenital heart defects Timing of surgery and preoperative management

Anesthesia

Premedication Hemodynamic stability and reserve Drug distribution and maintenance of anesthesia on bypass Postoperative analgesia

Surgery

Extent and complexity of surgery Residual defects Risks for bleeding and protection of suture lines

Conduct of bypass

Degree of hypothermia Level of hemodilution Myocardial protection Modulation of the inflammatory response and reperfusion injury

Postoperative management

Myocardial function Cardiorespiratory interactions Neurologic recovery Analgesia management

390

SECTION IV



Pediatric Critical Care: Cardiovascular

Separation from mechanical ventilation has the potential to cause important physiologic changes (e.g., increased RV preload, increased LV afterload; see Chapter 32); thus, these must be taken into consideration when planning extubation timing. Extubation ideally should occur at the intersect between patient readiness and healthcare team capacity. Although PICUs should strive to provide the same level of care and coverage 24 hours per day every day, it should be recognized that patients with higher complexity and risk often will require a level of undivided attention during and following separation from mechanical ventilation that may compete for attention with concurrent issues affecting other patients in the unit. The decision to extubate ultimately must take these factors into account, and, when necessary, the procedure might benefit from being delayed so that it can be performed under elective conditions and with redundant staffing coverage.

Central Nervous System The dramatic reduction in surgical mortality has been accompanied by a growing recognition of neurologic morbidity in many survivors. In the first months of life, this can manifest in altered tone, abnormal behavior, weak cry, and impaired feeding coordination.79 Later, these deficits are manifested by cognitive and speech and language dysfunction, impaired visual-motor coordination, learning disorders, and problems with executive functioning. All of these contribute to decreased quality of life and increased cost to society.80 Neurologic outcomes in patients with critical CHD appear to be multifactorial, involving the interplay of genetic, prenatal, perioperative, and postoperative factors. Treatment in the ICU can impact a number of these factors. Prenatally, the intrauterine circulation for many critical cardiac lesions results in the delivery of less oxygenated blood to the brain, which alters growth and cerebral vascular resistance.81 The brains of many children with critical CHD demonstrate greater immaturity on brain MRI and have a higher incidence of periventricular leukomalacia (PVL).82 PVL, much like that seen in premature infants, is associated with increased vulnerability of immature oligodendrocytes to hypoxia and ischemia.83 Indeed, preoperative hypoxia and diastolic hypotension and postoperative hypotension are all associated with a greater degree of postoperative PVL.84–86 Intraoperatively, a number of support techniques used during neonatal and infant cardiac surgery (e.g., CPB, profound hypothermia, circulatory arrest) have been implicated as potential causes of brain injury.87 These include (1) the total duration of CPB, (2) extreme hemodilution during CPB to hematocrits less than 20, (3) the duration and rate of core cooling, (4) pH management during core cooling, (5) duration of circulatory arrest, (6) position and function of cannulae, and (7) depth of hypothermia. However, the impact of each of these factors is not consistently seen, suggesting the multifactorial nature of CNS injury following CPB. In the postoperative period, the primary factor that most consistently impacts neurodevelopmental outcomes is duration of hospital stay.88 LOS not only serves as a surrogate for complexity, it also correlates with greater number of medical errors, increased parental stress, and the development of additional morbidities. LOS is also greatly impacted by sedation, prolonged ventilation, and the presence of delirium. Infants and children undergoing cardiac surgery will require analgesia, sedation, and sometimes paralysis to manage pain, anxiety, oxygen delivery, and hemodynamic instability. However, such agents must be optimally chosen and titrated to avoid

under- and overtreatment. Undertreatment can result in an increased stress response, with hemodynamic instability, delayed healing, and the development of posttraumatic stress response.89 Overtreatment can lead to hypotension, prolonged mechanical ventilation, tolerance, withdrawal, and delayed recovery. This balance can be challenging in infants and young children and those on mechanical ventilation who are unable to communicate. However, the use of validated pain and sedation scores for intubated and nonintubated infants and children that use clinical signs— such as alertness, agitation, muscle tone, facial expression, and response—has been shown to provide more objective measures of adequate pain and sedation management.90–92 The use of such scoring systems allows one to tailor therapy to effectively treat pain and minimize oxygen consumption in the most critically ill while also facilitating appropriate state control, early extubation, early mobility, and increased parental involvement. The necessity of more targeted therapy has become increasingly evident with the growing knowledge of the impact of anesthetics, sedatives, and narcotics on the development of delirium and neurologic dysfunction in critically ill infants and children.93 Delirium has been increasingly recognized within pediatrics as a driver of prolonged LOS and has been associated with increased mortality and neurologic dysfunction.94–96 It is thought to be the consequence of underlying medical illness combined with unwanted side effects of treatment and the stressful environment of the ICU. Possible mechanisms include neuronal injury due to inflammation, microemboli, and global or cellular hypoxia, all of which can be further exacerbated in the presence of critical illness and cardiac surgery with prolonged cardiopulmonary bypass.97 The incidence of delirium in the pediatric intensive care population has been estimated to be as high as 30%. Those children who are younger than 2 years, require mechanical ventilation, receive benzodiazepines, or require mechanical restraints are at highest risk. In the general critically ill pediatric population, those patients with increased LOS (8 days vs. 4 days) are at greater risk, as are those with inflammatory-mediated disease processes.98 This may explain the higher incidence and earlier onset of delirium that is seen in the postcardiac surgery population as well as the increased association with duration of CPB.99 This suggests that the best approach for a postoperative cardiac patient is to: 1. Routinely assess pain, sedation, and delirium using validated scoring systems to more effectively target therapy.100,101 2. Minimize narcotic and sedative use. This can be achieved by the use of a standardized method of treatment of pain that routinely uses scheduled, nonopioid analgesics such as acetaminophen or nonsteroidal antiinflammatory drugs in the immediate postoperative period together with opioid agents. Additionally, the use of agents such as dexmedetomidine, a highly selective a2 agonist that provides both sedation and analgesia, may have a narcotic and benzodiazepine-sparing effect.102 Further, dexmedetomidine has been demonstrated to have neuroprotective effects, though the mechanisms remain unclear.103 While its effect on decreasing heart rate can limit its use, it also has been shown to prevent and treat perioperative arrhythmias. 3. Optimize the environment by minimizing stressful factors and augmenting parental involvement. Cycling of lights, controlling extraneous or excess noise, promoting healthy and consistent sleep, and encouraging parental presence and involvement can both decrease the need for sedation and minimize the development of delirium.













CHAPTER 36 Critical Care After Surgery for Congenital Cardiac Disease

Renal Function and Postoperative Fluid Management Risk factors for postoperative renal failure include preoperative renal dysfunction, prolonged bypass time, hemolysis, low cardiac output, and cardiac arrest. In addition to relative ischemia and nonpulsatile blood flow on CPB, angiotensin II–mediated renal vasoconstriction and delayed healing of renal tubular epithelium have been proposed as mechanisms for renal failure. Postoperative sepsis and nephrotoxic drugs may further contribute to injury. Serum creatinine is the most widely used test and the current gold standard for diagnosing acute kidney injury (AKI). Using creatinine measurements to diagnose AKI in children has several shortcomings, including—but not limited to—variable normal levels based on age, gender, race, muscle mass, volume status, comorbidities, and use of certain medications. In addition, creatinine assesses only glomerular filtration and functional changes, and levels typically have a delayed rise over days after more than 50% of kidney function is lost in AKI, making it a poor gold standard.104–107 These limitations have provided the impetus to search for new biomarkers for the early detection of AKI prior to the functional change heralded by an increase in serum creatinine. Promising new biomarkers include neutrophil gelatinase-associated lipocalin (NGAL), kidney injury molecule-1 (KIM-1), cystatin C, urinary interleukin-18 (IL-18), liver-type fatty acid-binding protein (L-FABP), cell cycle marker insulin-like growth factor binding protein 7 (IGFBP7) and tissue inhibitor of metalloproteinases-2 (TIMP-2).107–109 A landmark study of 71 children undergoing CPB showed that the concentrations in serum and urine of NGAL were sensitive, specific, and highly predictive of early AKI after cardiac surgery.4 In another prospective uncontrolled cohort study, plasma NGAL was shown to be an early predictive biomarker of AKI, morbidity, and mortality after pediatric CPB.110 Because of the inflammatory response to bypass and significant increase in total body water, judicious fluid management in the immediate postoperative period is critical. Capillary leak and interstitial fluid accumulation may continue for the first 24 to 48 hours following surgery, necessitating ongoing intravascular volume replacement with colloid or blood products. A fall in cardiac output and increased antidiuretic hormone secretion contribute to delayed water clearance and potential prerenal dysfunction, which could progress to acute tubular necrosis and renal failure if a low–cardiac output state persists. During CPB, optimizing the circuit prime, hematocrit, and oncotic pressure; attenuating the inflammatory response with steroids; and use of modified ultrafiltration techniques have been recommended to limit interstitial fluid accumulation.111 During the first 24 hours following surgery, fluids should be restricted to 50% to 66% of full predicted maintenance and volume replacement titrated to appropriate filling pressures and hemodynamic response. Oliguria in the first 24 hours after complex surgery under CPB is common until cardiac output recovers and neurohumoral mechanisms abate. Although diuretics are commonly prescribed in the immediate postoperative period, neurohumoral influences on urine output are powerful and often limit diuretic response. Time after CPB and enhancement of cardiac output through volume and pharmacologic adjustments are the most important factors that will promote diuresis. Peritoneal dialysis, hemodialysis, and continuous venovenous hemofiltration provide alternate renal support in patients with severe oliguria and AKI. Besides enabling water and solute clearance, maintenance fluids can be increased to ensure adequate

391

nutrition. The indications for renal support vary but include pronounced uremia, life-threatening electrolyte imbalance (such as severe hyperkalemia), ongoing metabolic acidosis, fluid restrictions limiting nutrition, and increased mechanical ventilation requirements secondary to persistent pulmonary edema or ascites. A peritoneal dialysis catheter may be placed preemptively at the completion of surgery for selected cases or as a bedside procedure later in the ICU, when necessary. Indications include the need for renal support or for reducing intraabdominal pressure from ascites that may compromise mechanical ventilation and splanchnic perfusion. Drainage may be voluminous in the immediate postoperative period as third space fluid losses continue. Replacement of these losses with albumin or FFP may be necessary to treat hypovolemia and hypoproteinemia.

Gastrointestinal Issues Adequate nutrition is important following cardiac surgery in neonates and children. These patients often have decreased caloric intake and increased energy demand after surgery; the neonate, in particular, has limited metabolic and fat reserves. Total parenteral nutrition can provide adequate nutrition in the hypercatabolic phase of the early postoperative period. However, achieving proper caloric intake may be challenging in critically ill patients for whom limited fluid intake and an aggressive fluid removal strategy are a priority (e.g., to facilitate chest closure). For these patients, delaying initiation of parenteral nutrition might be advantageous.112 Gastritis, ulcer formation, and upper gastrointestinal bleeding may occur following the stress of cardiac surgery in children and adults. There are limited reports of the efficacy of proton pump inhibitors, histamine H2 receptor blockers, sucralfate, or oral antacids in pediatric cardiac patients, although their use is common in most PICUs. Hepatic failure may occur after cardiac surgery, particularly after the Fontan operation, and typically is characterized by elevated liver enzymes, hyperammonemia, and coagulopathy. Necrotizing enterocolitis, although typically a disease of premature infants, is seen with increased frequency in neonates with CHD. Risk factors include (1) left-sided obstructive lesions, (2) umbilical or femoral arterial catheterization/angiography, (3)  hypoxemia, and (4) lesions with wide pulse pressures (e.g., systemic-to-pulmonary shunts, severe aortic regurgitation) resulting in diastolic runoff in the mesenteric vessels. Frequently, multiple risk factors exist in the same patient, making a specific etiology difficult to establish. Treatment includes intestinal decompression through continuous nasogastric suction, parenteral nutrition, and broadspectrum antibiotics. Bowel exploration or resection may be necessary in severe cases with impending or established perforation.

Infection Low-grade (,38.5°C) fever is common during the immediate postoperative period and may be present for up to 3 to 4 days, even without a demonstrable infectious etiology. However, one ought not to simply disregard the occurrence of fever in the days following surgery, as it might signal an infection, especially in the multiply-instrumented patient. CPB activates complement and other inflammatory mediators but also can lead to derangements of the immune system that increase the likelihood of infection. Sepsis and nosocomial infection after cardiac surgery contribute substantially to overall morbidity. Despite the increased use of

392

SECTION IV



Pediatric Critical Care: Cardiovascular

broad antibiotic coverage with third-generation cephalosporins, these agents do not seem to be more effective in decreasing postoperative infections. Most centers use prophylactic coverage with a first-generation cephalosporin (i.e., cefazolin) with the first dose administered in the operating room and continued for the first 24 hours. Type and duration of prophylactic antibiotic coverage may be altered depending on contributing factors (e.g., chest reexploration, transthoracic ECMO cannulation, delayed sternal closure), but these decisions are best made as part of clinical protocols and a robust antibiotic stewardship program to decrease variability and minimize unnecessary exposure. Meticulous catheter insertion and daily care routines, along with early removal of indwelling catheters in the postoperative patient, are important in reducing the incidence of sepsis.113 Optimal head positioning, mouth care, sedation management, and consideration of an early-extubation strategy can reduce the rates of ventilator-associated events. Mediastinitis occurs in up to 2% of patients undergoing cardiac surgery. Risk factors include delayed sternal closure, particularly beyond 6 days, early reexploration for bleeding, or reoperation.114 Mediastinitis is characterized by persistent fever, redness, dehiscence, and purulent drainage from the sternotomy wound, instability of the sternum, and leukocytosis. Staphylococcus is the most common offending organism. Treatment usually involves debridement and irrigation, along with parenteral antibiotic therapy. The duration of therapy depends on the organism and severity of the infection and is generally between 2 and 4 weeks.

Hyperglycemia Hyperglycemia is a frequent occurrence in the PICU.115,116 As many as 97% and 78% of patients exhibit at least one blood glucose measurement above 125 mg/dL and 200 mg/dL, respectively, following surgical repair of congenital cardiac defects.115,116 The duration of postoperative hyperglycemia in these patients has been strongly and independently correlated with increased morbidity and mortality rates.115,116 Correlation does not signify causation; however, strict glycemic control with insulin administration has been shown to reduce morbidity and mortality rates significantly for adult patients admitted to a surgical ICU,117 and in one small pediatric study,118 two large randomized controlled pediatric trials of glycemic control failed to show improvement in meaningful primary outcomes (number of days alive and free from mechanical ventilation,119 rate of healthcare-associated infection120). In addition, patients assigned to strict glycemic control targeting fasting euglycemia experienced a significant increase in the occurrence of iatrogenic hypoglycemia,119,120 which is just as deleterious, if not more so, than hyperglycemia.121,122 Therefore, strict glycemic control with insulin infusion aimed at fasting euglycemic targets cannot be routinely recommended following cardiac surgery. It may be reasonable to administer an insulin infusion to address severe and persistent postoperative hyperglycemia while targeting the more permissive range, such as the one used in the control arm of the pediatric glycemic trials (150–180 mg/dL).120

Critical Care Management of Selected Specific Lesions Single-Ventricle Anatomy and Physiology For a variety of anatomic lesions, the pulmonary and systemic circulations are in parallel with complete mixing, with a single

ventricle effectively supplying both systemic and pulmonary blood flow. The proportion of ventricular output to either the pulmonary or systemic vascular bed is determined by the relative resistance to flow in the two circuits. The pulmonary arterial and aortic oxygen saturations are equal. Assuming equal mixing, normal cardiac output, and full pulmonary venous saturation, Sao2 of 80% to 85%, with MVo2 of 60% to 65%, indicates Qp/Qs ≈1 and, hence, a balance between systemic and pulmonary flow. Although “balanced,” the single ventricle still must receive and eject twice the normal amount of blood: one part to the pulmonary circulation and one part to the systemic circulation. A Qp/Qs greater than 1 implies a volume burden on the heart that may have a clinical impact depending on the degree, duration, and myocardial reserve. Though lesion-specific considerations are important in the various types of single-ventricle physiology, common management principles to balance flow and augment systemic perfusion do apply.

Neonatal Preoperative Management Changes in PVR have a significant impact on systemic perfusion and circulatory stability, especially preoperatively when the ductus arteriosus is widely patent. In preparation for surgery, it is important that systemic and pulmonary blood flow be as well balanced as possible, especially in the patient who may have accompanying systemic ventricular dysfunction. For example, a newborn with HLHS who has an arterial oxygen saturation greater than 90%, a wide pulse pressure, oliguria, cool extremities, hepatomegaly, and metabolic acidosis has severely limited systemic blood flow. Even though ventricular output is increased, the blood flow that is inefficiently partitioned back to the lungs is unavailable to the other vital organs. Immediate interventions are necessary to prevent imminent circulatory collapse and end-organ injury. In this “overcirculated” state, PVR is falling as it should in the normal postnatal state, and the ductus arteriosus is maintained widely patent to mitigate outflow obstruction from the RV to the systemic circulation. Blood flow manipulation by mechanical ventilation and inotropic support may temporarily stabilize the patient; this should accelerate the timeline for surgical intervention. Similarly, in a patient with pulmonary atresia and an intact ventricular septum, LV-dependent pulmonary circulation occurs. Ductal patency is necessary for pulmonary blood flow. As PVR falls, pulmonary blood flow will be excessive and eventually will steal from the systemic circulation. Preoperative management should focus on the adequacy of systemic oxygen delivery. This is best achieved by thorough and continuous reevaluation of the clinical examination for cardiac output state and perfusion; evaluation of chest radiograph for cardiac size and pulmonary congestion; review of laboratory data for alterations in gas exchange, acid-base status, and end-organ function; and echocardiographic imaging to assess ventricular function and AV valve competence. In a patient with a good systemic ventricular function, even high pulmonary blood flow (as manifested by higher saturations) is well tolerated for a few days. However, in the patient without good systemic ventricular function, an assessment of the balance between pulmonary (Qp) and systemic flow (Qs) becomes important. Qp/Qs is equal to the systemic arteriovenous saturation difference (systemic saturation – central venous saturation) divided by the pulmonary venoarterial saturation difference (pulmonary venous saturation [usually estimated] – pulmonary arterial saturation). In all single-ventricle physiologies, the systemic arterial saturations and pulmonary arterial saturations are equal by definition. If this ratio is greater than



CHAPTER 36 Critical Care After Surgery for Congenital Cardiac Disease

2:1 in a patient with clinical or biochemical evidence of insufficient oxygen delivery, additional interventions and/or surgery is necessary. Initial resuscitation involves maintaining patency of the ductus arteriosus with a PGE1 infusion at a rate of 0.01 to 0.05 µg/kg per minute. Intubation and mechanical ventilation are not necessary for all patients. Patients usually are tachypneic, but provided that the work of breathing is not excessive and systemic perfusion is maintained without metabolic acidosis, spontaneous ventilation is often preferable in order to achieve adequate systemic perfusion and balance of Qp and Qs. If the initial presentation involved circulatory collapse and end-organ dysfunction, then a period of days may be required to establish stability and allow for the return of vital organ function prior to surgery. Patients may require intubation and mechanical ventilation because of apnea secondary to PGE1, presence of a low–cardiac output state, or for manipulation of gas exchange to assist balancing Qp/Qs. An Sao2 greater than 90% indicates pulmonary overcirculation—that is, Qp/Qs greater than 1. PVR can be increased with controlled mechanical hypoventilation to induce respiratory acidosis, often necessitating sedation and neuromuscular blockade, and with a minimization of Fio2 to avoid hyperoxic pulmonary vasodilation. Although these maneuvers often are successful in maintaining a relatively high PVR and reducing pulmonary blood flow, it is important to remember that these patients have a limited oxygen reserve and may desaturate suddenly and precipitously. Controlled hypoventilation reduces functional residual capacity (FRC) and, therefore, also decreases the oxygen reserve. Patients who have continued pulmonary overcirculation with high Sao2 and reduced systemic perfusion despite these maneuvers require early surgical intervention to control pulmonary blood flow. At the time of surgery, a temporary snare can be placed around either branch of the pulmonary artery to limit pulmonary blood flow effectively. Alternatively, if there are important comorbidities that preclude a standard palliative operation (prematurity, intracranial hemorrhage), bilateral pulmonary arterial bands can be placed through a median sternotomy to optimize systemic blood flow. Decreased pulmonary blood flow in preoperative patients with a parallel circulation is reflected by hypoxemia with Sao2 less than 75%. This may result from restricted flow across a small ductus arteriosus, increased PVR secondary to parenchymal lung disease, or increased pulmonary venous pressure secondary to obstructed pulmonary venous drainage or a restrictive atrial septal defect (ASD). In patients with a later postnatal presentation, blood flow through a restrictive ductus may be augmented by the administration of high-dose PGE1 (0.1–0.2 µg/kg per minute). Patients at this level of prostaglandin delivery should have their airway secured and may benefit from vasopressor therapy to both offset the vasodilating effects of PGE1 and increase the systemic vascular resistance (SVR) to augment pulmonary blood flow. Sedation, paralysis, and optimization of mechanical ventilation to maintain an alkalosis may be effective if PVR is elevated. Inhaled nitric oxide (iNO) also may be useful in selected cases. Systemic oxygen delivery can be optimized by augmenting cardiac output and increasing hematocrit level greater than 40%. Among some newborns with HLHS, pulmonary blood flow may be insufficient because mitral valve hypoplasia, in combination with a restrictive or nearly intact atrial septum, severely restricts pulmonary venous return to the heart. The newborn is intensely cyanotic and may have a pulmonary venous congestion pattern on chest radiograph. Urgent interventional cardiac catheterization with balloon septos-

393

tomy or dilation (or stent placement) of a restrictive ASD may be necessary.123,124 Immediate surgical intervention and palliation are preferred in some centers. Increasingly, these patients are being identified prenatally with an option of fetal catheter-based intervention. Despite such advances, survival in this subgroup is reduced (between 48% and 69%).124–128 Systemic perfusion is maintained with the use of volume and vasoactive agents. Inotropic support is occasionally necessary because of ventricular dysfunction secondary to the increased volume load. This may be of particular concern in the neonate who presents as a postnatal diagnosis. Systemic afterload reduction with agents such as phosphodiesterase inhibitors may improve systemic perfusion, although the reduction in systemic vascular resistance may worsen hypoxemia if this is the primary problem. Oliguria and a rising serum creatinine level may reflect renal insufficiency from a low cardiac output. Necrotizing enterocolitis is a risk secondary to splanchnic hypoperfusion; we prefer not to enterally feed newborns with a wide pulse width and low diastolic pressure (usually ,30 mm Hg) prior to surgery. It is important to evaluate end-organ perfusion and function continuously.

Postoperative Management The postoperative management of patients with single-ventricle anatomy and physiology will be discussed later in this chapter, in the section detailing postoperative care of newborns with HLHS following stage I palliation.

Bidirectional Cavopulmonary Anastomosis In this procedure, also known as a bidirectional Glenn (BDG) shunt, the SVC is transected and connected end-to-side to the right pulmonary artery, while the pulmonary arteries remain in continuity. Therefore, flow from the SVC is bidirectional into both left and right pulmonary arteries. In most situations, the SVC becomes the only source of pulmonary blood flow, and inferior vena cava (IVC) blood returns to the common atrium. Performed between age 3 to 6 months, the BDG has proved to be an important early staging procedure in single-ventricle physiology for relieving volume and pressure overload, pulmonary artery distortion, and coronary hypoperfusion associated with an aortopulmonary shunt. However, the BDG circulation is not a stable source of pulmonary blood flow in the first few months of life when the PVR is too high to accommodate sufficient passive pulmonary blood flow for tolerable oxygenation. The BDG usually is performed on CPB using mild hypothermia with a beating heart. Therefore, the complications related to CPB and aortic cross-clamping are minimal, and patients can be weaned and extubated in the early postoperative period.129 In selected cases, the BDG anastomosis can be accomplished without CPB. Systemic hypertension is common following a BDG. The etiology remains to be determined, but possible factors include improved contractility and stroke volume after the volume load on the ventricle is reduced, and brainstem-mediated mechanisms secondary to the increased systemic and cerebral venous pressure. Treatment with vasodilators may be necessary during the early postoperative period. Following the BDG anastomosis, arterial oxygen saturation should be in the 80% to 85% range; however, stabilization to this level can take a number of days. In addition, positive-pressure ventilation in these patients reduces passive pulmonary blood flow; thus, these patients are generally excellent candidates for early extubation. Oxygen saturation frequently improves after

394

SECTION IV



Pediatric Critical Care: Cardiovascular

Factors Contributing to a Lower-Than-

TABLE Anticipated Oxygen Saturation in Patients 36.3

With Single-Ventricle Physiology

Etiology

Considerations

Low Fio2

Low delivered oxygen concentration Failure of oxygen delivery device

Pulmonary vein desaturation

Ventilation-perfusion defects • Alveolar process (e.g., edema/infection/ atelectasis)

• Restrictive process (e.g., effusion/ bronchospasm)

Intrapulmonary shunt • Severe RDS

• Pulmonary AVM

• PA-to-PV collateral vessel(s)

g Pulmonary blood flow

Anatomic RV outflow obstruction Anatomic pulmonary artery stenosis Increased PVR Atrial level right-to-left shunt Ventricular level right-to-left shunt

g Oxygen content

Low mixed venous oxygen level • Increased oxygen extraction: hypermetabolic state

• Decreased oxygen delivery: low–cardiac output state

Anemia AVM, Arteriovenous malformation; Fio2, fractional inspired concentration of oxygen; PA, pulmonary artery; PV, pulmonary vein; PVR, pulmonary vascular resistance; RDS, respiratory distress syndrome; RV, right ventricle.

extubation. Persistent hypoxemia (Sao2 ,70%) can be secondary to a low–cardiac output state (low mixed venous oxygen saturation [Svo2]), low pulmonary blood flow, or lung disease (Table 36.3). Treatment is directed at improving contractility, reducing afterload, and ensuring that the patient has a normal rhythm and hematocrit. Increased PVR is an uncommon cause, and iNO is rarely beneficial in these patients.48 This finding is not surprising because PA pressure and resistance and vascular tone are not high enough following this surgery to see a demonstrable benefit from iNO. Persistent profound hypoxemia should be investigated in the catheterization laboratory to evaluate hemodynamics, to look for residual anatomic defects that might limit pulmonary flow, such as SVC or PA stenosis or a restrictive ASD, and to coil any significant venous decompressing collaterals if present (e.g., SVC to azygous vein).

Fontan Procedure Since the original description in 1971130 the Fontan procedure and its subsequent modifications have been successfully used to treat a wide range of single-ventricle congenital heart defects.131 As a child grows after a BDG, the proportion of IVC blood

returning to the heart increases, causing systemic oxygen saturations to fall. This often occurs around 2 to 3 years of age, prompting Fontan completion. This is functionally accomplished by connecting the IVC blood flow directly to the PA. The surgical reconstruction is “physiologic” in that the systemic and pulmonary circulations are in series and cyanosis is corrected. Since the early 1990s the technique of Fontan completion has changed from the lateral tunnel to an extracardiac conduit. Over the past 25 years, the creation of a fenestration (a small opening between the Fontan pathway and the atrium) has gained broader acceptance to reduce postoperative complications. However, given current long-term outcome data, it is important to remember that the operation is still palliative rather than curative.130,132 The mortality and morbidity associated with this surgery have declined substantially over the years, and many patients with stable singleventricle physiology can lead reasonably normal lives.133 Postoperative considerations in managing Fontan physiology include targeting a systemic venous pressure of 15 to 20 mm Hg and LA pressure of 5 to 10 mm Hg—that is, a transpulmonary pressure drop of 5 to 10 mm Hg. Intravascular volume must be maintained and hypovolemia must be treated promptly. Changes in mean intrathoracic pressure and PVR have a significant effect on pulmonary blood flow. Pulmonary blood flow has been shown to be biphasic following the Fontan procedure; earlier resumption of spontaneous ventilation is recommended to avoid the detrimental effects of positive-pressure ventilation.134,135 Doppler analysis demonstrates that pulmonary blood flow predominantly occurs during inspiration in a spontaneously breathing patient—that is, when the mean intrathoracic pressure is subatmospheric. In many centers, patients after a standard-risk Fontan completion are identified for early extubation (in the operating room or within 6 hours of ICU admission). With proper selection criteria, optimal CPB and anesthesia management, and early extubation, most patients have an uncomplicated course after Fontan completion and can be discharged from the ICU environment within 1 or 2 days. In Fontan patients who do not meet early-extubation criteria, the method of mechanical ventilation requires thoughtful consideration. A tidal volume of 8 to 10 mL/kg with the lowest possible mean airway pressure is optimal. If appropriate selection criteria are followed, patients undergoing a modified Fontan procedure will have a low PVR without labile pulmonary vascular resistance. Therefore, vigorous hyperventilation and induction of a respiratory or metabolic alkalosis are generally of little benefit in this group. A normal pH and Paco2 of 40 mm Hg should be the goal and, depending on the amount of right-to-left shunt across the fenestration, the arterial oxygen saturation usually is in the 80% to 90 % range. The fenestration functions as a “pop-off” when forward flow to the pulmonary circulation is impaired, thereby sending blood into the systemic atrium and ventricle. As a result, systemic hypoxemia (Sao2) from the right-to-left shunt is the price paid for maintenance of adequate systemic cardiac output. The use of positive end-expiratory pressure (PEEP) requires thoughtful consideration based on patient-specific postoperative circumstances. The beneficial effects of a PEEP-related increase in FRC, maintenance of lung volume, and redistribution of lung water must be carefully balanced against the possible detrimental effect of an increase in mean intrathoracic pressure on passive pulmonary blood flow. A PEEP of 3 to 5 cm H2O, however, rarely has either hemodynamic consequence or substantial effect on effective pulmonary blood flow. Alternative methods of mechanical ventilation have been used in these patients. High-frequency ventilation has been employed



CHAPTER 36 Critical Care After Surgery for Congenital Cardiac Disease

successfully in selected cases, although the potential hemodynamic consequences of the raised mean intrathoracic pressure must be continually evaluated. Airway pressure-release ventilation has been shown to be superior in preserving systemic cardiac output when compared with standard pressure/volume control ventilation in patients post-Fontan completion.136 Negative-pressure ventilation can be beneficial by augmenting pulmonary blood flow, but application is cumbersome in the patient with surgical site dressings, and chest tubes typically present after a midline sternotomy.137 Afterload stress is poorly tolerated after a modified Fontan procedure because of the increase in myocardial wall tension and end-diastolic pressure. A phosphodiesterase inhibitor such as milrinone may be particularly beneficial. Besides being a weak inotrope with pulmonary and systemic vasodilating properties, its lusitropic action assists by improving diastolic relaxation and lowering ventricular end-diastolic pressure, thereby improving effective pulmonary blood flow and cardiac output.

Complications After the Fontan Procedure Pleuropericardial Effusions

The incidence of recurrent pleural effusions and ascites has decreased since the introduction of the fenestrated baffle technique. Nevertheless, for some patients, chylous drainage remains a major problem, with associated respiratory compromise, hypovolemia, and possible hypoproteinemia. These effusions can occur secondary to injury to the thoracic duct, persistent elevation of systemic venous pressure, or development of extensive lymphatic collaterals. Depending on the clinical significance of the drainage, more extensive evaluation may be required, as discussed earlier.

Rhythm Disturbances

Junctional bradycardia after Fontan completion is commonly observed postoperatively and rarely affects cardiac output significantly. In the rare circumstance in which that occurs, the use of AAI pacing rapidly addresses the situation. Atrial flutter or fibrillation, heart block, and, less commonly, ventricular dysrhythmia may have a significant impact on immediate recovery and on long-term outcome.138 Sudden loss of sinus rhythm initially causes an increase in LA and ventricular end-diastolic pressure and a fall in cardiac output. The SVC or PA pressure must be increased, usually with volume replacement, to maintain the transpulmonary gradient. Prompt treatment with antiarrhythmic drugs, pacing, or cardioversion is necessary. Premature Closure of the Fenestration

Not all patients require a fenestration for a successful, uncomplicated Fontan operation. Those with ideal preoperative hemodynamics often maintain adequate pulmonary blood flow and cardiac output without requiring a right-to-left shunt across the baffle. Similarly, not all Fontan patients who received a fenestration have a right-to-left shunt in the immediate postoperative period. These patients are fully saturated following surgery and may have an elevated right-sided filling pressure but nevertheless maintain an adequate cardiac output. The challenge is predicting which patients are at risk for low cardiac output after a Fontan procedure and who will benefit from the placement of a fenestration. Even patients with ideal preoperative hemodynamics may manifest a significant low–cardiac output state after surgery. In a review of 2747 Fontan completions from 68 centers contributing to the Society of Thoracic Surgeons database, 65% received a surgical fenestration at the time of initial operation.139 Premature closure of the fenestration may occur in the immediate postoperative

395

period, leading to a low–cardiac output state with progressive metabolic acidosis and large chest drain losses from systemic venous hypertension. Patients may respond to volume replacement, inotrope support, and vasodilation. However, if hypotension and acidosis persist, cardiac catheterization and removal of thrombus or dilation of the fenestration may be urgently needed. Persistent Hypoxemia

Arterial oxygen saturation levels may vary substantially following a modified Fontan procedure. Common causes of persistent arterial oxygen desaturation less than 75% include a poor cardiac output with a low Svo2, a large right-to-left shunt across the fenestration, and additional “leak” in the baffle pathway producing more shunting. Persistent hypoxemia can also be caused by an intrapulmonary shunt or venous admixture from decompressing vessels draining from the systemic venous baffle to the pulmonary venous system. Reevaluation with conventional or bubble contrast echocardiography and cardiac catheterization may be necessary. Low Cardiac Output State

An elevated LA pressure after a modified Fontan procedure may reflect poor ventricular function from decreased contractility or increased afterload stress, atrioventricular valve regurgitation, or loss of sinus rhythm (Table 36.4). Treatment consists of maintaining the high right-sided filling pressures (to maintain the transpulmonary gradient) and initiating inotropes and vasodilators. If a severely low–cardiac output state with acidosis persists, takedown of the Fontan operation and conversion to a BDG anastomosis or other palliative procedure might be lifesaving. Central venoarterial extracorporeal membrane oxygenation (VA ECMO) support in this instance may be an effective bridging strategy to urgent reoperation. Emergent cannulation to VA ECMO late after BDG and Fontan is associated with high morbidity and mortality and is generally contraindicated.140,141

Patent Ductus Arteriosus Pathophysiology The ductus arteriosus is a fetal vascular communication between the main pulmonary artery at its bifurcation and the descending aorta below the origin of the left subclavian artery. When patent, it provides a simple shunt between the systemic and pulmonary arteries. The magnitude and direction of flow between the systemic and pulmonary vessels are determined by the relative resistance to flow in the two vascular beds and the resistance of the ductus itself. With a large, nonrestrictive ductus and low PVR, the pulmonary blood flow is excessive and the volume load of the left heart is large. Systolic and diastolic flow away from the aorta may steal blood from vital organs and compromise end-organ function at many sites.142 In addition, overcirculated lungs and elevated LA pressure increase the work of breathing.143,144

Critical Care Management Although the patent ductus arteriosus (PDA) of premature infants can often be closed medically with indomethacin, contraindications to use of this agent (e.g., intracranial hemorrhage, renal dysfunction, and hyperbilirubinemia) may require surgical closure of the defect.145 Thoracotomy and surgical ligation of the ductus arteriosus are standard in term and preterm infants who are medically unstable. Beyond the newborn period, most centers now occlude the ductus with a percutaneously inserted vascular umbrella or by using coils for smaller PDAs. In stable patients who are not

396

SECTION IV



Pediatric Critical Care: Cardiovascular

TABLE Etiology and Treatment Strategies for Patients With Low Cardiac Output Immediately Following 36.4 the Fontan Procedure

Low Cardiac Output

Etiology

Treatment

Inadequate pulmonary blood flow and preload to left atrium Increased PVR Pulmonary artery stenosis

Volume replacement Reduce PVR Correct acidosis Inotropic support

Pulmonary vein stenosis Premature fenestration closure

Systemic vasodilation Catheter or surgical intervention

Ventricular failure Systolic dysfunction Diastolic dysfunction AVV regurgitation or stenosis

Maintain preload Inotrope support Systemic vasodilation Establish sinus rhythm or atrioventricular synchrony

Loss of sinus rhythm h Afterload stress

Correct acidosis Mechanical support Surgical intervention, including takedown to BDG and transplantation

Increased TPG Baffle .20 mm Hg LAp ,10 mm Hg h TPG ..10 mm Hg

Clinical State High Sao2/low Svo2 Hypotension/tachycardia Core temperature high Poor peripheral perfusion SVC syndrome with pleural effusions and increased chest tube drainage Ascites/hepatomegaly Metabolic acidosis

Normal TPG Baffle .20 mm Hg LAp .15 mm Hg TPG normal 5–10 mm Hg

Clinical State Low Sao2/low Svo2 Hypotension/tachycardia Poor peripheral perfusion Metabolic acidosis

AVV, Atrioventricular valve; BDG, bidirectional Glenn anastomosis; LAp, left atrial pressure; PVR, pulmonary vascular resistance; Sao2, systemic arterial oxygen saturation; SVC, superior vena cava; Svo2, SVC oxygen saturation; TPG, transpulmonary gradient.

candidates for an interventional cardiology approach (by nature of the length of the PDA), video-assisted thoracoscopic surgery (VATS) can be used.146 Advantages of VATS compared with open thoracotomy include decreased postoperative pain, shorter hospital stay, and decreased incidence of chest wall deformity.147 Healthy asymptomatic patients undergoing surgery can be extubated in the operating room, allowing many options for anesthetic management. However, the fragile premature infant with severe lung disease may require mechanical ventilation for protracted periods after ligation of the ductus arteriosus. Fentanyl, pancuronium, oxygen, and air constitute a common anesthetic regimen for this procedure.148 Many centers will bring the operative room environment to this patient population, performing the surgical ligation in the neonatal intensive care unit. Management of the premature infant in the operating room requires special considerations of gas exchange, hemodynamic performance, temperature regulation, metabolism and glucose management, and drug and oxygen toxicity. Thoracotomy and lung retraction usually decrease lung compliance and increase oxygen and ventilatory requirements. A transient rise in systemic blood pressure with ligation of the ductus arteriosus may increase LV afterload or elevate cerebral perfusion pressure to the detriment of a premature patient. Inadvertent ligation of the left pulmonary artery or descending aorta has occurred because the ductus arteriosus often is the same size as the descending aorta. The ductus is located near the recurrent laryngeal nerve (RLN), which may be damaged during the procedure. In addition to the close relationship of the RLN to the PDA and descending

aorta, the RLN has a variable course that may be difficult to identify during dissection. Prior reports of PDA ligation performed by open thoracotomy indicate that the incidence of RLN injury is 1.2% to 8.8%.149,150 RLN paralysis causes hoarseness and is not detected until the endotracheal tube is removed. The incidence may be reduced by location of the RLN within the thorax prior to ligation or clip placement using direct intraoperative stimulation of the RLN and evoked electromyogram monitoring.151 Ligation of an isolated ductus arteriosus generally results in normal cardiovascular function and reserve several months postoperatively.152

Atrial Septal Defect Pathophysiology There are three anatomic varieties of ASD. The most common, ASD secundum, is a defect in the septum primum, which ordinarily covers the region of the foramen ovale. ASD primum is a defect of the inferior portion of the atrial septum (endocardial cushion), usually accompanied by a cleft in the anterior leaflet of the mitral valve. Sinus venous defects are located near the junction of the right atrium and the SVC or IVC. They frequently are associated with a partial anomalous pulmonary venous connection. Left-to-right shunting (simple) occurs at the atrial level, causing RV volume overload. The degree of atrial-level shunting is a function of the difference between right and left ventricular compliance as opposed to atrial pressure differential. Pulmonary blood



CHAPTER 36 Critical Care After Surgery for Congenital Cardiac Disease

flow is increased, but generally not enough to make these patients symptomatic during early childhood. However, later in life, as the LV becomes less compliant and the LA pressures increase, the left-to-right shunt and volume load increase, and symptoms of CHF may occur. In rare patients, the longstanding increase in pulmonary blood flow causes pulmonary vascular obstructive disease.153 Other problems associated with longstanding volume load from an ASD include atrial fibrillation.

Critical Care Management The defect can be closed primarily with sutures or, if it is sufficiently large, with a synthetic patch. Sinus venosus defects associated with partial anomalous pulmonary venous connection require a more extensive patch that also directs the partial anomalous pulmonary venous return into the left atrium. These patients are among the healthiest encountered in the cardiac ICU. Their anesthesia can be managed in many ways, but early tracheal extubation, either in the operating room or in the immediate postoperative period, is the norm. Atrial arrhythmias, including atrial flutter and atrial fibrillation, are rarely seen during the postoperative period. Mitral regurgitation may occur in patients who have undergone repair of an ASD primum. Residual ASDs are uncommon, but occasionally failure to recognize partial anomalous pulmonary venous return results in a residual left-to-right shunt. With the exceptions mentioned, these patients usually have nearly normal cardiovascular function and reserve after repair.

Ventricular Septal Defect Pathophysiology Defects in the ventricular septum occur at several locations in the muscular partition dividing the ventricles. Simple shunting occurs across the ventricular septum. The magnitude of pulmonary blood flow is determined by the size of the VSD and the PVR.154 With a nonrestrictive defect, high LV flows and pressures are transmitted to the pulmonary artery. Therefore, surgical repair is indicated within the first 2 years of life to prevent the progression of pulmonary vascular obstructive disease.155 In patients with established pulmonary vascular disease, the pulmonary arteriolar changes may not recede when the defect is closed. In such cases, there may be progressive PVR elevation.156,157 The growth and development of the pulmonary vascular bed are significant factors in the patient’s ability to normalize pulmonary vascular hemodynamics after surgery.157 When PVR approaches or exceeds systemic vascular resistance, right-to-left shunting occurs through the VSD and the patients develop progressive hypoxemia (Eisenmenger syndrome). Closing the VSD in this circumstance may be contraindicated, as it would result in acute right heart failure without other therapies.

Critical Care Management The most common septal defect, the perimembranous defect, is often repaired through the tricuspid valve (TV) from a right atriotomy. However, lesions in the inferior apical muscular septum or those high in the ventricular outflow tract may require a left or right ventriculotomy. If so, postoperative ventricular function may be impaired. Concomitant RV muscle bundle resection can further impair ventricular function. Before repair, measures that decrease PVR may appreciably increase left-to-right shunting in patients with a nonrestrictive defect and may increase the degree of CHF. Postoperative RV or LV failure may be a manifestation of the preoperative status of the myocardium, a result of the ventriculotomy and CPB, or both.

397

Small infants who fail to thrive, who are malnourished, and who have significant CHF preoperatively may have excessive lung water and may require prolonged mechanical ventilation postoperatively.158 Such infants may have limited intraoperative tolerance for anesthetics that depress the myocardium or for maneuvers that increase pulmonary blood flow. Persistent CHF and an audible murmur postoperatively, evidence of low cardiac output, or the need for extensive inotropic support intraoperatively suggest that a residual or previously unrecognized additional VSD is continuing to place a volume and pressure load on the ventricles. When PVR is increased preoperatively, the increase in RV afterload caused by closure of the VSD may be poorly tolerated, leading to the need for inotropic support of the heart and measures to decrease PVR. Rarely, ventricular outflow tract obstruction is caused by placement of the septal patch. Transesophageal echocardiography performed in the operating room is an important tool in diagnosing this problem so that it can be addressed prior to complete separation from CPB. Aortic regurgitation caused by prolapse of one of the aortic valve cusps can develop in subaortic or subpulmonic VSDs. In addition, heart block may occur after patch closure of a VSD. Temporary pacing may be needed to maintain an adequate heart rate and cardiac output. Generally a permanent pacemaker is indicated when there is evidence of pacemaker dependence beyond 7 to 10 days.159

Critical Care Management for Late Postoperative Care In the absence of residual VSDs, outflow obstruction, or heart block, most of these patients regain relatively normal myocardial function, especially if the VSD is repaired early.160 However, a small percentage of patients, especially those in whom a large defect was repaired late in childhood, continue to have some degree of ventricular dysfunction and some pulmonary hypertension.161

Atrioventricular Canal Defects Pathophysiology The endocardial cushion defect, or complete common AV canal, consists of defects in the atrial, ventricular, and atrioventricular septa and the AV valvular tissue. All four chambers communicate and share a single common AV valve. The atrial and ventricular shunts communicate volume and systemic pressures to the right ventricle and pulmonary artery. The ventricular shunt orifice usually is nonrestrictive (simple shunt); therefore, PVR governs the degree of excess pulmonary blood flow. Left AV valve regurgitation and direct left-ventricular-to-right-atrial shunting may further contribute to atrial hypertension and total left-to-right shunting. Critical Care Management Surgical repair of this lesion consists of division of the common AV valve and closure of the ASD and VSD with either a singlepatch or two-patch repair technique. In addition, the left AV valve (and sometimes the right AV valve) requires suture approximation and resuspension of the separated portions. Prior to surgical repair, these patients have large left-to-right shunts. As a result of their high pulmonary blood flows, they have CHF and increased pulmonary vascular reactivity. Myocardial depressants and therapies that decrease PVR and thereby increase shunt flow may be poorly tolerated before repair. The occasional patients, especially older children and those with trisomy 21, may have developed true pulmonary vascular disease. All of the potential complications of ASD and VSD closures are seen in these

398

SECTION IV



Pediatric Critical Care: Cardiovascular

patients. In addition, the left AV valve may be severely regurgitant.162 Inotropic support for the failing heart, afterload reduction for mitral regurgitation, and measures to decrease PVR may be required perioperatively. Patients with trisomy 21 frequently have an associated complete AV canal. Measures to decrease PVR and the use of prolonged ventilatory support are often necessary because of their tendency toward upper airway obstruction, sleep-disordered breathing, and abnormal pulmonary vascular reactivity. The large tongue, hypotonia, upper airway obstruction, and difficult vascular access of these patients pose additional problems. The most frequent postoperative problems in patients with trisomy 21 are residual VSDs, left AV valve insufficiency, and pulmonary hypertension.163

Truncus Arteriosus Communis Pathophysiology With truncus arteriosus communis, the embryonic truncus fails to septate normally into the two great arteries. A single great artery leaves the heart and gives rise to the coronary, pulmonary, and systemic arteries. The truncus straddles a large VSD and receives blood from both ventricles. Complete mixing of systemic and pulmonary venous blood in the single great artery causes mild hypoxemia. Both pulmonary arteries usually originate from the ascending truncus, but occasionally only a single PA originates from the common trunk; the pulmonary artery orifice is seldom restrictive. The resulting shunt (simple) produces excessive pulmonary blood flow early in life as the PVR decreases. This pulmonary steal may decrease the systemic blood flow; however, the presence of two functional ventricles often prevents as significant a clinical manifestation as in patients with true single ventricle. Patients with truncus arteriosus may have anatomically anomalous coronary origins; the addition of a coronary runoff lesion may make them prone to early coronary ischemia and subsequent ventricular compromise. Children with truncus arteriosus are at risk for developing early pulmonary vascular obstructive disease, especially if delayed in their repair.164 Regurgitation of blood through the truncal valve may place an additional volume load on the ventricles. Critical Care Management Complete repair of this lesion should be performed early in the neonate, before the pulmonary vascular resistance drops further, resulting in clinical compromise.165 The VSD is closed with a synthetic patch, and the pulmonary arteries are detached from the truncus. Continuity is established between the RV and the pulmonary arteries with a valved conduit.166 The truncal valve may require valvuloplasty if a significant amount of blood regurgitates through it. The presence of a dysplastic and moderately regurgitant truncal valve poses additional challenges; most data suggest that these patients are best served long term by cardiac transplantation. Pulmonary arterial banding or valve replacement may be considered as an interim bridging strategy to this destination. Critical care management centers on control of pulmonary blood flow and ventricular support. Pulmonary blood flow may increase further with anesthetic agents, hyperventilation, alkalosis, and oxygen administration, resulting in hypotension and acute ventricular failure. If measures for increasing PVR do not decrease pulmonary flow, temporary occlusion of one branch of the pulmonary artery with a tourniquet limits pulmonary flow and restores systemic perfusion pressure until CPB can be

instituted. Because these patients are often in high-output CHF, myocardial depressants should be used with caution. Immediately after repair, the combination of persistent pulmonary arterial hypertension and RV failure can be fatal. Hence, aggressive measures should be taken to support myocardial function and lower PVR adequately. A residual VSD adds volume and pressure load on the ventricles and may have a devastating impact on the patient’s hemodynamics and oxygenation. This should be suspected in patients who are not doing well postoperatively, and any residual VSD should be repaired, if feasible. Truncal valve regurgitation or stenosis may induce LV failure early during the postoperative period.

Critical Care Management for Late Postoperative Care Obstruction of the pulmonary conduit and the accompanying RV hypertension may occur early or late during the postoperative course. Usually, the conduit is unable to support flow in the growing child after several postoperative years. Late development of truncal (aortic) valve regurgitation is possible. For patients who underwent initial repair beyond late infancy, residual persistent pulmonary hypertension will most likely be a problem.

Total Anomalous Pulmonary Venous Connection Pathophysiology Patients with TAPVC are cyanotic because their pulmonary veins connect to a systemic vein (complete mixing), and they have varying degrees of pulmonary venous obstruction. The venous connection may be supracardiac (e.g., to the SVC, innominate, or azygos vein), cardiac (e.g., to the coronary sinus), or infracardiac (e.g., to the hepatic veins, portal vein, or ductus venosus). Patients with this anomaly must have a patent foramen ovale or an ASD that allows blood flow to the left side of the heart. This anatomic arrangement provides complete mixing of all systemic and pulmonary venous blood in the right atrium. Unless there is significant stenosis of the pulmonary venous connection, most of this RA blood passes through the RV into the pulmonary artery, which increases pulmonary blood flow. If pulmonary venous return is significantly diminished due to obstruction, there is increased pulmonary venous congestion and decreased pulmonary blood flow. Critical Care Management Patients with significant obstruction, typically those with PV that drain into subdiaphragmatic vessels, may be very ill with hypoxemia, severe pulmonary edema, and pulmonary artery hypertension. Resuscitation, including mechanical ventilation, PEEP, and inotropic support of the myocardium, is followed by early surgical intervention to relieve the pulmonary venous obstruction. Although patients are hypoxemic, their primary pathology is caused by obstructed venous return from the lungs. Therapy that increases pulmonary blood flow (e.g., PGE1 or iNO) must be avoided. Surgical repair of TAPVC requires attachment or redirection of the pulmonary venous confluence to the left atrium. Intraoperative and postoperative problems often are related to residual or recurrent stenosis of the pulmonary veins. In patients who have severe and prolonged (often fetal) preoperative pulmonary venous obstruction, the pulmonary vascular bed is poorly reactive, reflected by highly pulmonary vascular resistance indices (PVRi). This elevation in PVRi results in high pulmonary artery pressures and poor RV function after bypass and during the early postoperative period. Critical care management of these patients after completion of the repair should emphasize inotropic support of the RV,



CHAPTER 36 Critical Care After Surgery for Congenital Cardiac Disease

Critical Care Management for Late Postoperative Care Other than the potential for late development of recurrent pulmonary venous obstruction, these patients generally do well and have good cardiovascular reserve once recovery from the surgery is complete.167 The size of the pulmonary veins at birth may be a predictor of late complications with recurrent pulmonary vein stenosis.168

Transposition of the Great Arteries Pathophysiology With transposition of the great arteries (d-TGA), the right ventricle gives rise to the aorta. Almost 50% of patients with this anomaly have a VSD, and some have a variable degree of subpulmonic stenosis. Oxygenated pulmonary venous blood returns to the left atrium and is recirculated to the pulmonary artery without reaching the systemic circulation. Similarly, systemic venous blood returns to the RA and ventricle and is ejected into the aorta again. Obviously, this arrangement is compatible with life only for a few circulation cycles unless there is some mixing of pulmonary and systemic venous blood via a PDA or an opening in the atrial or ventricular septum at birth. The physiologic disturbance in these patients is one of inadequate mixing of pulmonary and systemic blood rather than one of inadequate pulmonary blood flow. Mixing of blood at the atrial level can be improved by balloon atrial septostomy. If dangerous levels of hypoxemia persist after the septostomy and metabolic acidosis ensues, an infusion of PGE1 can maintain the patency of ductus arteriosus, increase pulmonary blood flow (by increasing left-to-right shunting across the PDA), and thereby increase the volume of oxygenated blood entering the left atrium. The volume-overloaded LA is likely to shunt part of its contents into the RA and thereby improve the oxygen saturation of aortic blood. Unlike other lesions, increased left-to-right shunting of blood during anesthesia improves arterial oxygen saturation before correction of the transposition. Depending on the particular anatomy and the presence of a VSD or pulmonary stenosis, one of three corrective procedures is used. The intraoperative and postoperative problems encountered differ with each type of procedure. Atrial Switch Procedure (Mustard and Senning)

An atrial-level partition is created with baffling to redirect pulmonary venous blood across the TV to the RV and thus to the aorta.169 Systemic venous (SVC and IVC) return is directed across the atrial septum to the mitral valve, into the LV, and out the pulmonary artery. Although the pulmonary and systemic circuits are then connected serially instead of in parallel, this arrangement leaves the patient with a morphologic RV and TV in continuity with the aorta. Therefore, this ventricle must work against systemic arterial pressure and resistance. One problem with atrial baffles is that they can obstruct systemic and pulmonary venous return.170 When this occurs, the patient manifests signs and symptoms of systemic venous obstruction, as evidenced by signs of systemic venous hypertension. When the pulmonary venous pathway is obstructed, pulmonary venous hypertension may be manifested by respiratory failure, poor gas exchange, and pulmonary edema. Severe pulmonary

venous obstruction is manifested in the operating room by the presence of copious amounts of bloody fluid in the endotracheal tube, low cardiac output, and frequently poor oxygenation. Residual interatrial shunts also may cause intraoperative or postoperative hypoxemia. Long-term rhythm disturbances and the limitations of ventricular and AV valve function have made this operation nearly obsolete for standard transposition anatomy. Arterial Switch Operation (Jatene Procedure)

Because of the complications associated with atrial baffle procedures, Jatene and others explored whether anatomic correction of this lesion by dividing both great arteries and reattaching them to the opposite anatomically correct ventricle would improve survival.171,172 This procedure requires excision and reimplantation of the coronary arteries to the neoaorta (formerly the proximal main pulmonary artery). LV mass decreases progressively after birth; thus, the ASO is done in the early (day 2 to 10 of life) neonatal period when the PVR (LV afterload) and LV pressure are high. The success of the ASO depends on adequate conditioning of the LV and technical proficiency with the coronary transfer. If the LV’s ability to tolerate the work required is misjudged, the child may develop severe LV failure postoperatively, necessitating significant vasoactive or mechanical support to maintain cardiac output. In the instance of a late postnatal diagnosis of TGA with intact ventricular septum, the LV may require “reconditioning” by banding the pulmonary artery to encourage LV hypertrophy and hyperplasia, as well as a modified Blalock-Taussig shunt (BT shunt) to augment pulmonary blood flow.173 Although the ASO can often be accomplished 1 week later, during this interval, these patients are often cyanotic and require considerable pharmacologic support.174 In contrast, if the neonate has a nonrestrictive VSD, the LV is accustomed to high systemic resistances and will tolerate the increased workload at any age. Myocardial ischemia or infarction may occur after mobilization and reimplantation of the coronary arteries, especially if they are stretched or twisted. Inotropic support, maintenance of coronary perfusion pressures, control of heart rate, and treatment with vasodilators may be particularly useful, as in adult patients with myocardial ischemia. Postoperative bleeding and tamponade occur more commonly with this operation because of the presence of multiple arterial anastomoses. At experienced centers, mortality after neonatal repair of transposition of the great arteries now is less than 3% and may be less than 2% for most anatomic arrangements of coronary arteries if the aortic arch is normal.175 Midterm and longer-term outcomes for the ASO are excellent, demonstrating a 25-year survival and freedom from reoperation rates of 96.7% and 75%, respectively.176 Alternative operations are reserved almost exclusively for patients with particularly difficult coronary anatomy177,178 or pulmonic (neoaortic) stenosis. ​

avoidance of myocardial depressant drugs, and minimization of PVR. Prolonged mechanical ventilation with gentle hyperventilation and other postoperative therapy to decrease PVR are required. Inhaled NO has been particularly useful in this population, provided that there is no residual pulmonary vein obstruction.49

399

Ventricular Switch (Rastelli Procedure)

This procedure can be used in TGA with VSD or double-outlet RV when there is an unrestrictive outlet VSD and when coexisting pulmonary valve stenosis precludes a standard arterial switch operation. The pulmonary valve is oversewn and the RV is connected to the pulmonary artery with a conduit.179 Complications of the Rastelli procedure include obstruction of LV outflow as a result of the narrowing of the subaortic region by the VSD patch. The conduit also may obstruct during or after the immediate postoperative period. A small but significant incidence of heart block in these patients can be a difficult postoperative problem.

400

SECTION IV



Pediatric Critical Care: Cardiovascular

Critical Care Management Management of patients following an atrial switch procedure rests on optimizing systemic oxygen delivery, monitoring for signs of baffle obstruction, and control of atrial arrhythmias. Most patients post-ASO without a VSD have an unremarkable postoperative course. Persistent ventricular dysfunction heralded by LA hypertension, hemodynamic instability, ventricular arrhythmia, or evidence of ischemic changes should prompt an intensive evaluation of the adequacy of the coronary anastomosis. Any of these perioperative issues or the unanticipated need for extracorporeal support necessitates immediate coronary evaluation and revision. Special attention should be paid to postoperative bleeding, given the extensive arterial suture lines. Patients with delayed intervention (either due to late presentation or comorbidities) usually will need to have a careful assessment for possible LV insufficiency. For these, an aggressive strategy of systemic afterload reduction, deep sedation, and muscle relaxation while expecting a more protracted ICU course is often the norm. Occasionally, these patients will benefit from the use of ECLS or temporary LVAD support for retraining of the LV in the postoperative course. Late Complications Following atrial baffle, patients can be regarded as having a physiologic or functional two-ventricle repair (i.e., the morphologic LV is the pulmonary ventricle and the morphologic RV remains the systemic ventricle). Actuarial survival rates at 15 years have been quoted to be up to 85%; however, significant long-term functional deterioration is likely with increasing risk for right heart failure, sudden death, and dysrhythmias.180,181 This situation is evidenced by systemic (right) ventricular dysfunction and TV regurgitation long after the repair.182 These patients also are prone to develop significant atrial dysrhythmias, including supraventricular tachyarrhythmias and sick sinus syndrome later in life.183 Virtually all coronary artery patterns are amenable to ASO. No particular pattern has been associated with late death. A report of coronary artery angiography in 366 patients following ASO (median age at follow-up, 7.9 years) revealed coronary artery stenosis or occlusion in 3% of patients.184 Despite the angiographic findings, evaluation with serial ECG, exercise testing, and wallmotion abnormalities on echocardiography rarely demonstrate evidence of ischemia.185 After repair, the native pulmonary valve becomes the neoaortic valve. A 30% incidence of trivial to mild aortic regurgitation has been reported on intermediate-term follow-up, without significant hemodynamic changes.186 Severe regurgitation is unusual. There appears to be a very low incidence of significant rhythm disturbances after ASO.187 Supravalvar pulmonary AS was an early complication but now is less common with surgical techniques that extensively mobilize, augment, and reconstruct the pulmonary arteries. Supravalvar AS may develop but is rare. Assessment of myocardial performance using echocardiography, cardiac catheterization, and exercise testing following ASO has demonstrated function identical to that in age-matched controls. Based on the currently available clinical, functional, and hemodynamic data, a patient who has undergone ASO with no evidence of subsequent problems should be treated like any patient with a structurally normal heart when presenting for noncardiac surgery. Late complications of the Rastelli procedure include progressive conduit obstruction and RV hypertension, residual VSDs,

and occasional subaortic obstruction from diversion of LV outflow across the VSD to the aorta.

Tetralogy of Fallot Pathophysiology The four anatomic features of TOF are VSD, RV outflow tract obstruction, overriding of the aorta, and RV hypertrophy. There may be additional muscular VSDs, and obstruction of the pulmonary valve and main and branch pulmonary arteries. Resistance to RV outflow forces systemic venous return from right to left across the VSD and into the aorta, producing arterial desaturation. The amount of blood that shunts right to left through the VSD varies with the magnitude of the RV outflow tract obstruction and with SVR. Distal PVR is low and has minimal influence on shunting. Systemic vasodilation, in conjunction with increasing dynamic infundibular stenosis, intensifies rightto-left shunting and can lead to hypercyanotic spells. Such spells can occur at any time before surgical correction of the anomalies and can be life-threatening. Because the morbidity associated with recurrent hypercyanotic spells is significant, many physicians consider recurrent episodes of hypercyanosis an indication for corrective surgery at any age. Critical care management of TOF patients with hypercyanotic episodes should focus on minimizing oxygen consumption, acidosis, tachycardia, and acute elevations in PVR while augmenting preload and SVR. Hypercyanotic spells in nonanesthetized children should initially be managed with 100% oxygen by facemask, a knee-chest position or squat position (to increase SVR), and sedation. Classically, intravenous morphine is used. However, intranasal medications, such as fentanyl, have been used effectively when IV access has not yet been established. Dexmedetomidine has been reported in the management of hypercyanotic spells, as it provides not only sedation but also the additional benefit of lowering the heart rate.188 Regardless of the specific drug used, this regimen can usually stabilize the dynamic infundibular stenosis while keeping SVR elevated. Deeply cyanotic and lethargic patients are given rapid IV crystalloid infusions to augment circulating blood volume. Continued severe hypoxemia should be treated with a vasopressor bolus (e.g., phenylephrine 1–2 mg/kg, titrated up to 5 mg/kg) to further augment SVR, and judicious use of IV propranolol or esmolol to slow the heart rate may be considered as necessary. The latter allows more filling time and relaxes the infundibulum. If a hypercyanotic spell persists despite treatment, either stabilization on VA ECMO or immediate surgical intervention (palliative aortopulmonary shunt or complete repair) is indicated. Induction of anesthesia must proceed cautiously after minimal fasting times and administration of a premedication, if possible. The child can be anesthetized with IV narcotics and/or inhalational agents, but care must be taken to minimize reduction in SVR. The pattern of mechanical ventilation is critical, as excessive intrathoracic pressure can further reduce antegrade flow across the RV outflow. Critical Care Management for the Early Postoperative Course The surgical approach to the patient with TOF who presents with recurrent early hypercyanotic spells is variable. Traditionally, complete repair is completed at 3 to 6 months of age. Delayed repair also is often necessary when a coronary artery crosses the RV outflow tract, precluding transannular patch repair. There are currently two palliative interventions to facilitate a delayed repair



CHAPTER 36 Critical Care After Surgery for Congenital Cardiac Disease

strategy. The first is the use of a systemic-to-pulmonary artery shunt. Excellent outcomes have been achieved with this approach, and the need for a transpulmonary valve annulus outflow patch at the time of definitive surgery is reduced.189 However, the risks of cyanosis and complications related to a systemic-to-pulmonary artery shunt argue for early complete repair of TOF. The alternative approach, developed more recently, is stenting the RV outflow tract in the cardiac catheterization laboratory.190 This procedure has the additional benefit of improving pulmonary artery growth prior to the definitive repair.191 Another approach for patients with TOF is to proceed with an early complete repair. For that method, a ventriculotomy is performed in the RV outflow tract and frequently is extended distally through the pulmonary valve annulus and beyond any associated pulmonary artery stenosis. The outflow tract is enlarged with pericardium or synthetic material, and obstructing muscle bundles are resected to relieve the outflow tract obstruction.192 Because they are smaller and younger, these patients may be at increased risk for complications associated with CPB. Pulmonary regurgitation results after a transannular incision that may compromise ventricular function in the postoperative period. In approximately 8% of patients, abnormalities in the origin and distribution of the coronary arteries preclude placement of the RV outflow patch, making it necessary to bypass the stenosis by placing an external conduit from the body of the right ventricle to the pulmonary artery.193,194 An analysis of 3059 TOF repairs between 2002 and 2007 demonstrated that 83% (2534) had a complete repair as their initial index procedure (with 6%, 19%, 38%, and 24% undergoing operation at the ages of 0 to 1, 1 to 3, 3 to 6, and 6 to 12 months, respectively).195 There were 217 (7%) patients who underwent complete repair following initial palliation. Rates of ventriculotomy, transannular patch, and RV-PA conduit use were significantly higher in those requiring initial palliation. Discharge mortality was higher in palliative patients versus initial complete repair (7.5% vs. 1.3%). There was less disparity in discharge mortality between the two approaches among neonates (6.2% vs. 7.8%).195 When weaning patients from CPB following TOF repair, the aim of therapy is to support RV function and minimize afterload on the right ventricle. This is particularly important following repair in neonates or small infants. Although systolic dysfunction of the RV may occur following neonatal ventriculotomy, the clinical picture is more commonly one of a restrictive physiology reflecting reduced RV compliance or diastolic dysfunction.36,37 Factors contributing to diastolic dysfunction include ventriculotomy, lung and myocardial edema following CPB, inadequate myocardial protection of the hypertrophied ventricle during aortic cross-clamp, coronary artery injury, residual outflow tract obstruction, volume load on the ventricle from a residual VSD or pulmonary regurgitation and arrhythmias. Patients usually separate from CPB with satisfactory blood pressure and atrial filling pressures less than 10 mm Hg on modest inotropic support. However, in neonates during the first 6 to 12 hours after surgery, a low–cardiac output state with increased right-sided filling pressures from diastolic dysfunction is common following a right ventriculotomy. Continued sedation and paralysis usually are necessary for the first 24 to 48 hours to minimize the stress response and associated myocardial work. Preload must be maintained despite elevation of RA pressure. In addition to high right-sided filling pressures, pleural effusions or ascites may develop. Inotropic support is often required and, if the blood pressure can tolerate it, introduction of a

401

phosphodiesterase inhibitor, such as milrinone, is beneficial because of its lusitropic properties. Because of the restrictive physiology, even a relatively small volume load from a residual VSD or pulmonary regurgitation is often poorly tolerated in the early postoperative period; 2 to 3 days may be required before RV compliance improves and cardiac output increases. Although the patent foramen ovale or any ASD is usually closed in older patients at the time of surgery, it is beneficial to leave a small atrial communication following neonatal repair. In the face of diastolic dysfunction and increased RV end-diastolic pressure, a right-toleft atrial-level shunt maintains preload to the LV and, therefore, cardiac output. Patients may be desaturated initially following surgery because of this shunting. As RV compliance and function improve, the amount of shunt decreases and both antegrade pulmonary blood flow and systemic arterial oxygen saturation increase. Arrhythmias following repair include heart block, ventricular ectopy, and junctional ectopic tachycardia (JET). Maintaining sinus rhythm is important to optimize end-diastolic filling and minimize end-diastolic pressure. AV pacing may be necessary for heart block. Complete right bundle branch block is typical on the postoperative ECG. Although JET is typically transient, it can result in significant deleterious effects on the child’s hemodynamics. Treatment of JET may include sedation, cooling, paralysis, antiarrhythmic medications, temporary pacing, and, in rare circumstances, mechanical support to maintain hemodynamics. Most patients recover systolic ventricular function postoperatively. However, in a small group of patients, especially those repaired at older ages, significant ventricular dysfunction remains.196,197 These patients can have left ventricular subendocardial ischemia that impairs LV myocardial mechanics.198 Pulmonary valve insufficiency may contribute to residual ventricular systolic dysfunction.199 The most common cause of systolic dysfunction immediately after repair of TOF is a residual or previously unrecognized VSD, which causes a volume load on the LV and pressure load on an already stressed RV, leading to RV failure and poor cardiac output.25 A residual VSD combined with residual RV outflow obstruction is particularly deleterious. In some patients, the distal pulmonary arteries may be so hypoplastic and stenotic that they cannot be satisfactorily corrected. Suprasystemic pressure develops in the RV, which can be ameliorated in some cases by partially opening the VSD to allow an intracardiac right-to-left ventricular shunt. This shunt unloads the compromised RV at the expense of decreased arterial oxygen saturation.

Critical Care Management for Late Postoperative Care Reconstruction of the RV outflow tract may lead to significant problems that affect RV function and risk for arrhythmias over time. Although most of the long-term outcome data pertain to patients following TOF repair, similar complications and risks are likely for those who have undergone an extensive RV outflow reconstruction, such as placement of a conduit from the RV to the pulmonary artery for correction of pulmonary atresia, truncus arteriosus, and the Rastelli procedure for transposition of the great arteries with pulmonary stenosis. Complete surgical repair of TOF has been successfully performed for more than 40 years, with studies reporting a 30- to 35-year actuarial survival of approximately 85%.200 Many patients report leading relatively normal lives, but RV dysfunction may progress after repair and may be evident only on exercise stress testing or echocardiography. A spectrum of problems may develop, ranging from a dilated RV with systolic dysfunction to

402

SECTION IV



Pediatric Critical Care: Cardiovascular

diastolic dysfunction from a poorly compliant RV. Continued evaluation is necessary because of the increased risk for ventricular arrhythmias and late sudden death. Factors that may adversely affect long-term survival include older age at initial repair, initial palliative procedures, and residual chronic pressure or volume load as occurs from pulmonary insufficiency or stenosis. Systolic dysfunction secondary to a residual volume load from pulmonary regurgitation after tetralogy repair is a predictor of late morbidity. It is reflected as cardiomegaly on chest radiograph, an increase in RV end-diastolic volume and regurgitant volume by echocardiography and cardiac MRI,12 and a reduction in anaerobic threshold, maximal exercise performance, and endurance on exercise testing.201 Patients who have significant pulmonary regurgitation, RV dilation, and reduced RV function are at potential risk for a fall in cardiac output during anesthesia, particularly as positive-pressure ventilation may increase the amount of regurgitation. These patients currently benefit from early surgical pulmonary valve replacement to reduce these symptoms and risks. An important group to distinguish consists of those who have continued restrictive physiology or diastolic dysfunction secondary to reduced ventricular compliance. They usually do not have cardiomegaly, they demonstrate better exercise tolerance, and the risk for ventricular dysrhythmias is possibly decreased. Although the RV is hypertrophied, function is generally well preserved on echocardiography, with minimal pulmonary regurgitation. The incidence of significant RV outflow obstruction developing over time is low. Residual obstruction contributes to early mortality within the first year after surgery but is well tolerated in the long term. A wide variation in the incidence of ventricular ectopy has been reported in numerous follow-up studies, including up to 15% of patients on routine ECG and up to 75% of patients on Holter monitor. Multiple risk factors—including older age at repair, the extent of ventriculotomy, residual hemodynamic abnormalities, and duration of follow-up—have all been considered important. In common with these factors are probable myocardial injury and fibrosis from chronic pressure and volume overload, combined with cyanosis. Although ventricular ectopy is common in asymptomatic patients during ambulatory ECG, Holter monitoring, and exercise stress testing, it often is low grade and does not identify those patients at risk for sudden death. Electrophysiologic induction of sustained ventricular tachycardia (VT), especially when monomorphic, is suggestive of the presence of a reentrant arrhythmic pathway. Although dependent on the stimulation protocol used to induce VT, the presence of monomorphic VT in a symptomatic patient with syncope and palpitations is significant and indicates treatment with radiofrequency ablation, surgical cryoablation, antiarrhythmic drugs, or placement of an implantable cardioverter-defibrillator.202 The risk for ventricular dysrhythmias during anesthesia and ICU care for subsequent hospitalizations is unknown. Although preoperative prophylaxis with antiarrhythmic drugs is not recommended, a means for external defibrillation and pacing must be readily available.

Pulmonary Atresia Pathophysiology Atresia of the pulmonary valve or main pulmonary artery forms a spectrum of cardiac defects. Management depends on the extent of atresia, size of the RV and TV, presence of a VSD and collateral vessels, surface area of the pulmonary vascular bed, and coronary artery anatomy. The timing of developmental abnormality defines

the associated lesions. Pulmonary atresia with intact ventricular septum (PAIVS) is primarily an abnormality of TV development that subsequently affects the pulmonary valve through its effects on fetal RV growth. Because the impact on the pulmonary valve is late, the fetal truncus arteriosus has already divided and the mesenchymal distal pulmonary vasculature can connect to a main pulmonary artery pressure head appropriately. As a result, this lesion predictably has well-developed central pulmonary arteries and pulmonary artery arborization. At one end of the spectrum of PAIVS, platelike pulmonary atresia overlaps with critical pulmonary stenosis where there is a mild or negligible degree of hypoplasia of the RV and TV. In these lesions, a fixed obligatory right-to-left atrial-level shunt of all systemic venous return exists. Some blood may flow into the RV, but because there is no outlet, blood regurgitates back across the TV and eventually reaches the LA and LV. Pulmonary blood flow is derived exclusively or predominantly from a PDA. As a rule, these patients do not have extensive aortopulmonary collateral blood flow; consequently, they often become cyanotic when the PDA closes after birth. Critical pulmonary valve stenosis can be effectively treated by balloon valvuloplasty in the catheterization laboratory. Antegrade flow across the RV outflow may not improve immediately but may gradually increase over days as RV compliance improves. In platelike pulmonary atresia, radiofrequency perforation precedes balloon valvuloplasty. Most patients with PAIVS have an underdeveloped TV and RV. Depending on the degree of RV hypoplasia (which is directly related to the TV annulus z-score), the patient may be unsuitable for a biventricular repair in the long term. In this situation, initial palliation with an aortopulmonary shunt is necessary. However, if the RV is deemed to be of suitable size, then reconstruction of RV outflow with a pericardial patch or interventional catheter techniques may be considered (similar to the approach described earlier for platelike PA with mild RV hypoplasia). A large conal branch or aberrant left coronary artery across the RV outflow tract may restrict the size of a ventriculotomy and placement of a patch or conduit. At the other end of the spectrum, severe pulmonary atresia may be associated with an extremely hypoplastic RV that is not suitable for biventricular repair. A palliative procedure with a modified BT or central shunt usually is necessary at first to improve pulmonary blood flow, followed by staged single-ventricle repair (see the Fontan Procedure section). Patients with PAIVS and severe RV hypoplasia may have numerous fistulous connections (sinusoids) between the small hypertensive RV cavity and coronary circulation.203 This is distinct from RV-dependent coronary circulation (RVDCC), in which these sinusoidal connections are accompanied by proximal stenoses in the true coronary arteries. If RV decompression or even staged palliation is being considered in such a patient, then a coronary angiogram is highly desirable to exclude this phenomenon. In PAIVS with documented RVDCC, cardiac transplantation is often the treatment of choice. In contrast to PAIVS, PA/VSD and TOF with PA represent an early failure of proper conotruncal development. As a result, the mesenchymal segmental pulmonary arteries do not see the main pulmonary artery pressure head and instead form connections with the nearest alternative (aorta). This is the nature of the development of the aortopulmonary collaterals (APCs) associated with the lesion. If the collaterals are substantial, defining a large pulmonary arterial segment, they are referred to as major aortopulmonary collateral arteries (MAPCAs). As a rule, these patients



CHAPTER 36 Critical Care After Surgery for Congenital Cardiac Disease

403

have two good-sized ventricles (unless accompanied by other lesions, such as an unbalanced AV septal defect). This disease can exist in a spectrum with confluent central pulmonary arteries and a short-segment main pulmonary artery atresia to discontinuous pulmonary arteries without a true central pulmonary artery. In the former situation, the pulmonary artery arborization is often close to normal, with few MAPCAs. When antegrade flow is established from the RV into the main pulmonary artery by a reparative procedure, the left-to-right shunt via collateral flow will impose a diastolic load on the LV. Preoperative occlusion of these collateral vessels can be accomplished by interventional techniques in the cardiac catheterization laboratory but may leave the child precariously cyanotic in the hours before operation. The alternative is to complete the surgical procedure in a hybrid suite. This spectrum of the lesion is completely repaired in the neonatal period. As mentioned, MAPCAs may be present to varying degrees, supplying some or all segments of the lung. They can be associated with a large left-to-right shunt, contributing to volume overload and pulmonary hypertension. Larger collateral vessels supplying significant portions of the lung can be anastomosed or “unifocalized” to the native pulmonary arteries, with the ultimate aim being to establish full antegrade pulmonary blood flow. Smaller vessels to some segments of the lung can be coiled in the cardiac catheterization laboratory provided that there is antegrade flow from the native pulmonary arteries to those lung segments (dual supply). When the pulmonary arteries are diminutive, it is important to establish early antegrade flow from the RV to the pulmonary artery in an effort to promote growth and establish a pathway to the pulmonary arteries for subsequent balloon dilation. A modified BT or Mee shunt may initially be necessary to provide sufficient pulmonary blood flow if the pulmonary arteries are exceedingly small. Initially, the VSD can be left open; postoperative management of cyanosis or CHF will be determined by the size of and resistance offered by the pulmonary circulation. The course in these patients can be dynamic and demanding for even the most experienced practitioners. When collaterals are unifocalized in the operating room and RV to diminutive pulmonary artery continuity is established, cyanosis may ensue, and therapy is aimed at lowering PVR or (re)establishing adequate pulmonary blood flow. On the other hand, if the child is fully saturated in the aorta with elevated pulmonary artery oxygen saturation and LA pressure, then a left-to-right shunt through the VSD may be developing, which will produce a volume load on the LV and an unstable postoperative course, dictating VSD closure. When the patient is not fully saturated in the aorta but is suffering from a volume-loaded LV with low cardiac output and high LA pressure postoperatively, excessive systemic-to-pulmonary collaterals may be the culprits, requiring catheterization laboratory investigation and occlusion or immediate reoperation.

noncompliant postoperatively. This may manifest as requirements for high filling pressures and consequent right-to-left shunting through a purposefully patent foramen ovale. With growth and improved compliance of the RV, the right-to-left shunting diminishes and the infant’s oxygenation improves substantially. If hypoxemia persists, a PGE1 infusion can be reinitiated to temporize either until accommodation of the restrictive RV physiology or for a BT shunt. In patients with long-segment pulmonary atresia, the need for a conduit to bridge the gap between the RV and pulmonary artery complicates the repair. Again, RV failure may occur postoperatively, especially when there is a residual VSD or RV outflow obstruction. The conduit may obstruct acutely during chest closure, further elevating pressure in the RV. The relationship of the conduit to either normal or variant coronary anatomy is vital, as impingement of these structures can present as sudden inexplicable deterioration at the time of sternal closure. After the VSD is closed and blood flow from the RV to the pulmonary arteries is enhanced, there may be excessive pulmonary blood flow (Qp/Qs .1) as a result of the combined flow into the pulmonary arteries from the RV and from aortopulmonary collaterals. If this occurs, the patient develops CHF and requires intraoperative inotropic support and an extended period of postoperative mechanical ventilation. With large collateral flow, the pulse pressure is wide and diastolic pressure low. The patient may require surgery to ligate the collateral vessels or may require embolization.

Critical Care Management Critical care management of patients with pulmonary atresia is similar to that for TOF. Maintaining the patency of the ductus for the perioperative treatment of neonates with pulmonary atresia and critical pulmonary stenosis is essential prior to additional interventions to establish more permanent pulmonary blood flow. Following pulmonary valvotomy (interventional or surgical), the goal of therapy is to improve oxygenation and decrease RV afterload. Because the LV may be more pressure and volume loaded in this lesion than in the normal fetus, the RV may be relatively

Pathophysiology In this condition, an imperforate TV and hypoplasia of the RV are present, often accompanied by a VSD of variable size and by pulmonic stenosis. The most common form of tricuspid atresia has normally related great arteries; when associated with transposition of the great vessels, the clinical presentation is similar to that of hypoplastic left heart syndrome. In the usual type of tricuspid atresia, an obligatory atrial-level shunt exists from the RA through the patent foramen ovale or ASD into the LA, where complete mixing takes place. The degree

Critical Care Management for Late Postoperative Care Patients with TOF and pulmonary atresia are subject to the same late problems and complications as patients with TOF alone. In addition, they will require conduit revisions over their lifetime. Some patients have accelerated conduit obstruction after surgery, which is often related to the presence of a porcine valve.204 Consequently, many experts now prefer bovine-valved conduits or homografts except in certain situations, such as when the distal pulmonary vascular impedance is high, resulting in central pulmonary artery hypertension. In these situations, a valveless conduit that permits pulmonary insufficiency (and thus relieves central pulmonary artery hypertension) is preferred. In cases when pulmonary regurgitant volume load to the RV is undesirable, some favor the placement of a heterologous bovine jugular vein bioprosthesis with a trileaflet venous valve.205 This has been the valved conduit of choice in patients younger than 18 years whenever pulmonary regurgitant volume load to the RV is undesirable.205 Small-caliber bovine jugular vein conduits may result in significantly improved freedom from dysfunction at 5 and 10 years’ follow-up compared with pulmonary homografts in patients who received the operation during the first 2 years of life.206

Tricuspid Atresia

404

SECTION IV



Pediatric Critical Care: Cardiovascular

of hypoxemia depends on the amount of pulmonary blood flow, which is regulated by the severity of the pulmonic stenosis. This, in turn, is regulated by the size of the VSD; patients with an extremely small/restrictive VSD will have pulmonary atresia in addition to tricuspid atresia. The common presentation is characterized by significant hypoxemia caused by decreased pulmonary blood flow.

Critical Care Management The ultimate palliative surgical course for tricuspid atresia is a modified Fontan procedure. Neonatal hypoxemia prior to surgery can be effectively stabilized with a PGE1 infusion. An initial palliative procedure may be required to improve pulmonary blood flow (modified BT shunt) if the child becomes severely hypoxemic. In contrast, if there is excessive pulmonary blood flow, a pulmonary artery band may be needed. The critical care management and complications are those discussed in the sections on shunts, banding, and modified Fontan procedures. Complications of chronic hypoxemia and cyanosis are also present.

Left-Sided Obstructive Lesions Pathophysiology This category includes valvar, subvalvar, and supravalvar mitral and aortic stenosis, aortic coarctation, IAA, and HLHS. Although these lesions can occur as isolated defects, they often coexist (Shone syndrome) or are accompanied by other congenital cardiac defects. Identification of additional structural defects is necessary for optimal preoperative, surgical, and postoperative treatment. Patients with LV outflow tract obstruction tend to present either as neonates or as young infants with significant LV dysfunction and CHF or later in childhood with subclinical LV hypertrophy. The dramatic presentation of a neonate with circulatory collapse typically occurs with lesions that obstruct systemic blood flow so severely that right-to-left shunting at the ductus arteriosus is required to perfuse the body. As the ductus significantly narrows or closes, the LV proves inadequate to support the systemic circulation or fails, leading to pulmonary edema and respiratory distress. When systemic perfusion becomes inadequate, the patient may quickly develop cardiogenic shock. Classic examples of this physiology include severe (or “critical”) valvar AS, coarctation of the aorta, and HLHS (see the earlier single-ventricle discussion). If the obstruction is less severe, the child can make the transition through ductal closure without notable LV dysfunction and maintain an adequate cardiac output. Over time, however, the pressure overload on the LV stimulates generalized hypertrophy. If untreated and significant, long-term pressure overload can cause LV diastolic dysfunction (compliance falls and end-diastolic pressure rises, causing pulmonary edema), LV systolic dysfunction, and episodic myocardial ischemia. Clinical manifestations of these changes can include reduced exercise tolerance, exertional chest pain, ventricular dysrhythmias, syncope, and sudden death. In this situation, significant LV dilation or clinical signs of CHF are ominous findings associated with a poor prognosis and an increased surgical mortality rate. Aortic Stenosis Of the three anatomic subtypes of AS, valvar AS occurs more frequently than subvalvar or supravalvar AS. The newborn with critical valvar AS who develops hypotension and acidosis as the ductus arteriosus closes requires resuscitation with PGE1 to

restore aortic flow plus mechanical ventilation and inotropic support to achieve stabilization before an intervention is performed. Currently, balloon dilation of the stenotic aortic valve during cardiac catheterization is the preferred intervention at most centers. A surgical valvotomy under direct vision using CPB is the surgical alternative. Despite successful relief of obstruction, significant LV dysfunction and low cardiac output often persist for days after the procedure and require continued treatment with mechanical ventilation and vasoactive drugs. Until LV function recovers and can support the entire cardiac output, a PGE1 infusion may be necessary to maintain a PDA. Patients should be carefully evaluated after balloon aortic valvuloplasty for residual AS and aortic insufficiency, the chief potential complication of valve dilation, especially if cardiac output does not improve over several days. Older infants, children, and adolescents with moderate (pressure gradient 50–70 mm Hg at catheterization) or severe (pressure gradient .70 mm Hg at catheterization) valvar AS also are generally good candidates for balloon aortic valvuloplasty. Usually, if more than mild aortic regurgitation coexists with AS, surgical intervention is preferred to balloon valvuloplasty. The pathophysiology produced by all types of aortic outflow obstruction is similar—that is, the pressure-overloaded LV becomes progressively hypertrophied and develops reduced compliance and an abnormally elevated end-diastolic pressure. Initial assessment of obstruction relief can occur when the patient is still in the catheterization laboratory or operating room by either direct pressure measurements or echocardiography. Nevertheless, reevaluation for residual obstruction by physical examination or echocardiography in the ICU as patients recover from anesthesia and baseline physiology returns is important because outflow gradients can change. A significant residual obstruction should be suspected in any patient with persistent low cardiac output following the intervention. Poor recovery of LV function after surgery can occur secondary to inadequate myocardial protection with cardioplegia in hearts with significant ventricular hypertrophy. Patients with marked hypertrophy are also at greater risk for developing VT and ventricular fibrillation early after surgery. In patients with preserved LV systolic function who undergo an uncomplicated procedure, such as aortic valvuloplasty or subvalvar membrane resection, myocardial recovery after CPB is typically rapid, and inotropic support is usually not required. Systemic hypertension is more common following relief of LV outflow obstruction, especially during emergence from anesthesia and sedation. Antihypertensive therapy in the initial 24 to 48 hours may be necessary to prevent an aortic suture line and reconstructed valve leaflet disruption from excessive stress and to allow adequate hemostasis. Initially, both titratable b-blockers (e.g., labetalol and esmolol) and vasodilators (e.g., nitroprusside), alone or in combination, are effective in lowering the blood pressure for these patients. However, when using vasodilators, caution must be taken to maintain adequate coronary perfusion for the hypertrophied ventricle. In addition to assessing aortic valve and LV function, an evaluation for complications specific to each procedure is required. For example, if a myectomy is performed as part of the resection of fibromuscular subvalvar AS, the possibility of a new VSD, mitral valve injury, and left bundle branch block should all be assessed. Following the Ross procedure, it is important to assess patients for RV outflow tract and LV outflow tract obstruction, because the



CHAPTER 36 Critical Care After Surgery for Congenital Cardiac Disease

RV outflow tract is also reconstructed with a valved conduit. This procedure involves reimplantation of the native coronary arteries into the new pulmonary autograft placed in the aortic position; signs of coronary ischemia, unexplained hemodynamic collapse, or ventricular arrhythmia should prompt aggressive reevaluation of coronary flow adequacy.

Coarctation of the Aorta Coarctation of the aorta is a narrowing in the descending aorta located at the level of insertion of the ductus arteriosus (juxtaductal or contraductal coarctation). Narrowing of the aortic lumen is asymmetric, with the majority of the obstruction occurring because of posterior tissue infolding, leading to the common description of a posterior aortic shelf. Depending on the severity and rapidity of development of the narrowing, patients can present as neonates with severe obstruction (a critical coarctation of the aorta) upon ductal closure, as infants with CHF, or as children/ adolescents with asymptomatic upper extremity hypertension (especially with exercise). Neonates presenting with critical coarctation of the aorta can often be distinguished clinically from patients with critical AS by their clearly discrepant upper versus lower body pulses, perfusion, and blood pressures. Other features at presentation are similar, including evidence of CHF and inadequate blood flow to tissues. Because ductal narrowing or closure is common after hospital discharge, these patients often become critically ill and suffer endorgan damage before the ductus arteriosus can be reopened and resuscitation complete. Intestinal and renal ischemia leading to necrotizing enterocolitis and renal failure, respectively, are wellknown complications of critical coarctation of the aorta. Echocardiography often reveals additional left-sided defects, such as bicuspid aortic valve, valvar AS, aortic arch hypoplasia, and VSD. Preoperative stabilization and management in this clinical scenario include initiation of PGE1, mechanical ventilation, and inotropic agents as needed. In addition, these patients require adequate time for end-organ recovery before performing an intervention. In rare situations, when the PGE1 infusion is unable to open the ductal and periductal arch tissue to diminish the afterload on the ventricle and facilitate cardiac and end-organ recovery, the patient should be taken to the operating room urgently for coarctation repair. Coarctation of the aorta also occurs in association with complex defects, such as D-transposition of the great arteries, single ventricle, and complete AV canal defect. If the ductus arteriosus is patent during echocardiographic evaluation of a neonate with suspected CHD, it often is not possible to predict the severity of coarctation of the aorta with confidence. A patient can have an abnormally narrowed aorta just proximal to the site of ductal insertion (i.e., the aortic isthmus) and a posterior shelf but still not develop a severe coarctation of the aorta following ductal closure. Therefore, evaluation of the potential severity of coarctation of the aorta in the ICU often involves discontinuing the PGE1 infusion to allow for the ductus to close, followed by close clinical and echocardiographic reassessment for clinical manifestations of coarctation. An intervention to reduce aortic arch obstruction is indicated in any neonate with clinical or echocardiographic evidence of reduced ventricular function or impaired cardiac output related to coarctation development. These indications are more important than the systolic blood pressure difference between upper and lower body per se, although differences greater than 30 mm Hg often are accompanied by diminished ventricular function. The postoperative management of patients following surgical repair of coarctation of the aorta can vary depending on the age

405

at intervention. However, the key issues for assessment in all patients are adequate relief of obstruction and preservation of spinal cord function. Upper and lower body blood pressures and pulses should be compared serially and the lower extremities monitored closely for the return of sensation and voluntary movement in the early postoperative period. Equal pulses and a reproducible systolic blood pressure difference less than 10 to 12 mm Hg between upper and lower extremities indicate an excellent repair. Neonates and young infants typically require 1 to 2 days of mechanical ventilation postoperatively. Older children and adolescents often can be extubated in the operating room and rarely require inotropic support. In fact, these patients repaired at an older age are increasingly likely to have significant hypertension,207 which should be treated aggressively early after surgery to reduce the risk of aortic suture disruption and bleeding. b-Blockers and vasodilators, along with adequate analgesia and sedation, are effective. Patients with longstanding coarctation of the aorta frequently have persistent systemic hypertension despite adequate surgical repair; continued treatment with angiotensin-converting enzyme inhibitors is advocated to achieve normal blood pressures. Due to the challenges in managing this cohort of older patients after coarctation repair and the advent of interventional catheterization approaches, including the use of covered stents, a surgical repair strategy is increasingly falling out of favor. Four uncommon complications are associated with surgical repair of coarctation of the aorta. Postcoarctectomy syndrome manifests as abdominal pain or distension in older patients and is presumably caused by mesenteric ischemia from reflex vasoconstriction after restoration of pulsatile aortic flow. Recurrent laryngeal nerve and phrenic nerve trauma can cause vocal cord paralysis and hemidiaphragm paresis or paralysis, respectively, with neonates and infants at highest risk. Disruption of lymphatic vessels or thoracic duct trauma can produce a chylous effusion that may require treatment by drainage, dietary modification, or surgical ligation of the ductus. Catheter-directed balloon angioplasty is used to treat both native and residual coarctation of the aorta.208 The results of native coarctation of the aorta dilation after early follow-up appear similar to published surgical results, but aortic aneurysm formation has been reported. Balloon angioplasty of recurrent coarctation of the aorta after surgery is effective and is generally preferred to reoperation.

Interrupted Aortic Arch Patients with IAA typically present as neonates with either a loud systolic murmur or circulatory compromise as the ductus arteriosus closes. Therefore, patient presentation can be similar to other critical left-sided obstructive lesions. Unlike either critical AS or coarctation of the aorta, however, severe pressure overload on the LV does not occur in the presence of an unrestrictive VSD, which functions as a “pop-off” for LV outflow. The approach to resuscitation is similar to that described for the other ductal-dependent left-sided obstructive lesions, with attention to the possibility of pulmonary overcirculation as for HLHS. Postoperative management issues specific to patients with IAA include assessment of possible residual left-sided obstruction both in the aortic arch and in the subaortic region, shunting across a residual VSD, hypocalcemia (related to the 22q11 syndrome), dysrhythmias, and LV dysfunction with low cardiac output secondary to global effects of CPB and deep hypothermic circulatory arrest. Left-lung hyperinflation on postoperative chest radiographs suggests compression of the left main stem bronchus. This

406

SECTION IV



Pediatric Critical Care: Cardiovascular

complication tends to occur after difficult arch reconstructions when tension on the aorta causes it to press on the anterior surface of the bronchus, thus producing distal air trapping.

Hypoplastic Left Heart Syndrome Pathophysiology

Among the congenital heart lesions, perhaps the most controversial has been HLHS. Left untreated, HLHS is a uniformly fatal disease; debate continues regarding the optimal management strategy (i.e., staged palliation, neonatal transplantation, or comfort care). Currently, the 14-month transplant-free survival ranges from 65% to 75%, making the transplantation waitlist mortality unacceptable for these patients. The results of surgical management vary among institutions and are clearly dependent on expertise and experience,209 the clinical condition of the neonate at presentation,210 degree of prematurity, associated congenital anomalies, presence of an intact atrial septum, age at presentation,211,212 and degree of hypoplasia of left heart structures.212,213 This common example of single-ventricle physiology also represents the most severe form of obstructive left heart lesion. An anatomic spectrum of disease is implied for the lesion; however, in its most severe and common presentation, there is atresia or marked hypoplasia of the aortic and mitral valves with critical underdevelopment of the left atrium, left ventricle, and ascending aorta. A 1or 2-mm diameter ascending aorta gives rise to the coronary circulation and to the head vessels before convergence with the ductus arteriosus, where the aorta becomes larger and supplies the circulation to the lower body. Pulmonary venous blood returns to the diminutive left atrium and cannot cross the atretic mitral valve. It is instead directed to the right atrium and RV, where it mixes with systemic venous return and is ejected into the pulmonary artery. Systemic blood flows from the pulmonary artery, across the PDA, to the aorta. As the PDA constricts in the neonatal period, systemic blood flow decreases and all ventricular output is directed to the lungs. The Qp/Qs ratio approaches infinity as Qs nears zero. The patient consequently presents with a high Pao2 (70–200 mm Hg), shock, and profound metabolic acidosis. When the ductus arteriosus is reopened with PGE1, systemic perfusion is reestablished, the acidosis resolves, and the Pao2 returns to the range of 40 to 60 mm Hg, representative of a Qp/Qs ratio between 1 and 2. Critical Care Management

Preoperative resuscitation with PGE1, correction of metabolic acidosis, and recovery from end-organ dysfunction are crucial to the stabilization and management of patients with this lesion. Resuscitation is enhanced by the judicious use of inotropic agents, which optimize cardiac output and end-organ perfusion. However, excessive delay in the timing of surgical intervention may result in a gradual decline in PVR, excessive pulmonary blood flow, and inadequate systemic perfusion. The surgical reconstructive approach to this lesion now commonly entails three operations intended to provide the child with a reconstructed aortic arch and Fontan-type single-ventricle physiology by 2 to 5 years of age.214,215 In the first stage of the reconstruction (Norwood operation),215 the pulmonary artery is transected at the bifurcation and an anastomosis is performed connecting the pulmonary trunk to the ascending aorta (above the coronary arteries) to form a neoaorta, connecting RV, diminutive LV, coronary arteries, and systemic circulations in an unobstructed fashion. Pulmonary blood flow is established via a modified BT shunt (usually 3.5 mm in diameter) or a valveless RV-PA (Sano) conduit. The atrial septum is excised to ensure free flow of pulmonary venous return

to the TV. The Norwood operation may also be performed to repair other complex single-ventricle defects that include systemic outflow obstruction or hypoplasia.216 The critical care considerations are the same as those outlined for patients with single-ventricle physiology. Perioperative management requires optimization of combined ventricular output, minimization of systemic oxygen consumption, and careful manipulation of Qp/Qs. Postoperative Management Evolution of Treatment Strategies. Common teaching has  

held that postoperative mortality and hemodynamic instability result from myocardial dysfunction and the physiologic burden imposed by shunt-dependent pulmonary blood flow parallel to the systemic circulation. Treatment strategies have emphasized factors that affect the balance between pulmonary and systemic blood flow. Immediately following a Norwood operation, PVR may be transiently elevated but soon declines. Once PVR falls, treatment is aimed at raising resistance to blood flow through the lungs and redirecting cardiac output to the systemic circulation. High inspired concentration of oxygen, hyperventilation, alkalosis, systemic vasoconstriction, and anemia will cause a further increase in pulmonary blood flow and should be avoided. Therapies designed to raise PVR and thereby direct aortic blood flow to the systemic circulation have focused on lowering the Fio2 or on allowing the Paco2 to rise and the pH to fall toward 7.3. Ventilation with hypoxic gas mixtures or added carbon dioxide has been advocated by some centers and has been intermittently embraced and abandoned by others. Validation of the effectiveness of these techniques to balance the pulmonary and systemic circulations has been difficult.217,218 The clinical focus of care after Norwood palliation for HLHS has evolved since the 1990s. The early emphasis was on manipulating the PVR by optimizing mechanical ventilation (i.e., mean airway pressure, tidal volume, rate and inspiratory time, Fio2, and Paco2). The objective was to raise PVR and lower pulmonary blood flow while increasing systemic blood flow. Like any therapeutic strategy, manipulating gas exchange and inspired gases had its own set of adverse effects. Delivering low Fio2 (below room air concentrations) leads to alveolar hypoxia and can be life-threatening. Furthermore, if treatment to elevate PVR is prolonged in preparation for reconstructive or transplantation surgery, PVR may be persistently elevated during and after weaning from CPB. In centers where neonates are allowed to awaken and breathe spontaneously in the immediate postoperative period, pulmonary blood flow may become excessive. Adding carbon dioxide to the inspired gas may reverse the tendency toward respiratory alkalosis and stabilize the relative balance of the pulmonary and systemic circulations if the forces that drive minute ventilation are suppressed by sedation or analgesia.217,218 However, the metabolic cost of carbon dioxide breathing in an awakening child may discourage the widespread application of this technique until the physiologic advantage over conventional means of controlling alveolar ventilation and Paco2 has been demonstrated. This is especially true in unsedated preoperative patients for whom factors controlling respiration during carbon dioxide breathing may limit the change of Paco2 yet substantially increase the respiratory rate and work of breathing. By the late 1980s it was apparent that (Norwood) palliated circulation was needed for only 10 to 12 weeks until the infant reached adequate size for a second stage of palliation

407



CHAPTER 36 Critical Care After Surgery for Congenital Cardiac Disease



to blunt systemic vascular reactivity and dilate the peripheral circulation,219 but its potent, long-lasting hypotensive effects and its limited reversibility by alpha-agonists has proven challenging. The phosphodiesterase inhibitors then came to enjoy widespread use in pediatric critical care. These agents lower SVR, increase cardiac output, and reduce filling pressures. The new strategy was to monitor (arteriovenous) Do2, support cardiac output, and reduce SVR. Later in the decade, it was observed that some patients had limited coronary reserve, low mixed venous oxygen saturation, rising lactate, and collapsed in the first 48 hours after operation. This emphasized the fundamental limitation of myocardial function and cardiac output of these infants in the early postoperative period. The morphologic RV and TV seem ill-suited to support both adequate systemic and optimal pulmonary blood flow after the Norwood procedure. Several centers then embraced temporary mechanical support of the circulation for the failing Norwood patient in the early postoperative period. Specific Considerations for the Norwood Operation. Management of patients following a Norwood-type operation is complex. Intensive monitoring is essential because the patient’s clinical status can change abruptly with rapid deterioration. Persistent or progressive metabolic acidosis is an ominous prognostic sign that must be aggressively managed. Considerations in the assessment of the circulation following the Norwood operation are given in Table 36.5. ​

(bidirectional Glenn). The introduction of a 3.5-mm systemicto-pulmonary shunt (rather than a 4-mm shunt) and the appreciation of the surgical complexity of an appropriately placed shunt did as much to reduce excessive pulmonary blood flow in infants as did manipulation of the ventilator. Although the smaller shunts were associated with a rare but real incidence of shunt thrombosis, those involved in postoperative care found patient management with a small shunt substantially easier than struggling with the tendency for pulmonary overcirculation with a 4-mm shunt. While a relative increase in cyanosis (from a smaller shunt) was believed to be justified by the greater early postoperative stability, mortality rates did not plummet with recognition of the benefits of smaller shunts and PVR manipulation. In the early 1990s it was appreciated that arterial oxygen saturation was only one variable in the assessment of Qp/Qs. A perfectly acceptable arterial oxygen saturation of 80% in this disease may represent severe pulmonary overcirculation if the mixed venous oxygen saturation is very low (e.g., 20%). Hence, interest emerged in measuring and monitoring both arterial and mixed venous oxygen saturations. Thus, in patients with excessive pulmonary blood flow despite a small (3.5-mm) fixed-diameter shunt from the subclavian artery, there was less emphasis on micromanagement of PVR and enhanced interest in pharmacologic support of cardiac output by reducing systemic afterload and diminishing the driving pressure across the shunt. Some have advocated the use of the alpha-blocker phenoxybenzamine

TABLE Management Considerations for Patients Following a Norwood Procedure 36.5

Etiology

Management

Sao2 ,80% Svo2 ,60% Normotensive

Balanced flow (Qp 5 Qs)

No intervention

Sao2 .90% Hypotension

Overcirculated (Qp . Qs) Low PVR Large BT shunt Residual arch obstruction

Raise PVR • Controlled hypoventilation • Low Fio2 (0.17–0.19) • Add CO2 (3%–5%) • Increase systemic perfusion • Afterload reduction, vasodilation • Inotropic support • Surgical shunt revision

Sao2 ,75% Hypertension

Undercirculated (Qp , Qs) High PVR Small, kinked, thrombosed BT shunt

Lower PVR • Controlled hyperventilation • Alkalosis • Sedation/paralysis Increase cardiac output • Inotropic support Hematocrit .40% Surgical intervention

Sao2 ,75% Hypotension Low Svo2

Low cardiac output Ventricular failure Myocardial ischemia Residual arch obstruction Atrioventricular valve regurgitation

Minimize stress response Inotropic support Surgical revision Consider mechanical support Consider transplantation























Scenario

BT, Blalock-Taussig; Fio2, inspired oxygen concentration; PVR, pulmonary vascular resistance; Qp, pulmonary blood flow; Qs, systemic blood flow; Sao2, arterial oxygen saturation; Svo2, mixed venous oxygen saturation.

408

SECTION IV



Pediatric Critical Care: Cardiovascular

After a Norwood operation, a pH of 7.4, Paco2 of 40 mm Hg, and Pao2 of 40 mm Hg in room air, with a mixed venous oxygen saturation of 60%, connote a well-balanced circulation. Higher saturations can be achieved if the systemic circulation is well dilated without compromising perfusion pressure. Frequent adjustments in mechanical ventilation settings and Fio2 may be necessary in the first few hours after surgery. However, manipulations of Fio2 in the face of a restrictive 3.5-mm shunt may have less impact on pulmonary blood flow than systemic vasodilation.214 Leaving the sternum open after a Norwood operation may facilitate lower filling pressures, balanced circulation, and a stable ventilation pattern. Deep sedation or even muscle paralysis and anesthesia often are continued after surgery to minimize the stress response until the patient has stable circulation and gas exchange. Inotropic support with dopamine or epinephrine usually is required, titrated to systemic pressure and perfusion. Afterload reduction with milrinone as a second-line agent is beneficial to reduce myocardial work and improve systemic perfusion. Monitoring SVC oxygen saturations as a measure of Svo2 and cardiac output is useful in this assessment. Volume replacement to maintain preload is essential, aiming for a common atrial pressure approximating 8 to 10 mm Hg. The type, diameter, length, and position of the shunt affect the balance of pulmonary and systemic flow. Generally, a 3.5-mm modified BT shunt from the distal innominate artery provides adequate pulmonary blood flow without excessive steal from the systemic circulation for most full-term neonates. Nevertheless, a shunt resulting in a low diastolic pressure (,30 mm Hg) in turn affects perfusion to other vascular beds, particularly the coronary, cerebral, renal, and splanchnic perfusion. This may contribute to a prolonged and difficult postoperative course. Overcirculation in the immediate postoperative period with an Sao2 greater than 90% may reflect a low PVR or increased flow across the shunt if the shunt size is too large or the perfusion pressure is increased from residual aortic arch obstruction distal to the shunt insertion site. The increased volume load on the systemic ventricle results in CHF and progressive systemic hypoperfusion with cool extremities, oliguria, and metabolic acidosis. Although manipulation of mechanical ventilation and inspired oxygen concentration may help limit pulmonary blood flow, surgical revision to reduce the shunt size may be necessary. If there is significant diastolic runoff through a large shunt, coronary perfusion will be reduced and lead to ischemia, low output, arrhythmias, and cardiac arrest. Rhythm disturbances are uncommon in the immediate postoperative period following a Norwood operation, and a sudden loss of sinus rhythm, particularly heart block or ventricular fibrillation, should increase the suspicion of myocardial ischemia and impending circulatory collapse. In the immediate postoperative period, mild hypoxemia with an Sao2 of 70% to 75% and Pao2 of 30 to 35 mm Hg is preferable to an overcirculated state with high systemic oxygen saturations and falling mixed venous oxygen saturation. Pulmonary blood flow often increases on the first postoperative day as ventricular function improves and PVR falls during recovery from CPB. Pulmonary venous desaturation from parenchymal lung diseases—such as atelectasis, pleural effusions, and pneumothorax—requires aggressive management. Persistent desaturation and hypotension reflect low cardiac output from poor ventricular function, thereby decreasing the perfusion pressure across the shunt. Svo2 is low (often ,40%),

and treatment is directed first at augmenting contractility with inotropic agents and subsequently reducing afterload with a vasodilator. This is a serious clinical problem with high mortality after a Norwood operation. The related myocardial ischemia and acidosis further impair myocardial function and systemic perfusion, leading to circulatory collapse. Atrioventricular valve regurgitation and residual aortic arch obstruction are important causes of persistent low cardiac output and inability to wean from mechanical ventilation. Echocardiography is useful for assessing valve and ventricular function but is less accurate for assessing the degree of residual arch obstruction. Cardiac catheterization is sometimes necessary and will enable fine-tuning of hemodynamic support or balloon dilation of a hypoplastic segment of narrowed aorta. Occasionally, surgical revision of the aortic arch or atrioventricular valve is necessary, although this is seen more commonly in the interval before the bidirectional cavopulmonary shunt. The advocated Sano modification of the Norwood procedure involves placement of a conduit from the RV to the PA confluence (RV-PA shunt).220,221 The primary advantage of this procedure in the immediate postoperative period is improved diastolic perfusion without runoff across an aortopulmonary shunt. Ventricular function is less likely to be compromised after surgery because the volume load to the ventricle is reduced from a lower Qp/Qs, along with a reduced risk for myocardial ischemia because of improved coronary perfusion. Perfusion to cerebral, renal, and splanchnic circulations also is likely to be improved with the lack of diastolic runoff to the pulmonary circulation, which may enhance postoperative recovery. Because pulmonary blood flow occurs only during ventricular systole across the RV to the pulmonary artery conduit, there may be a critical reduction in pulmonary blood flow and excessive hypoxemia, especially during periods of low cardiac output or if there is dynamic obstruction to flow at the ventricular insertion site. Efforts to overcome this limitation by creating a larger RV incision run the longerterm risk of ventricular dysfunction, arrhythmias, or aneurysm formation. A multicenter, randomized, controlled trial involving 549 neonates with HLHS demonstrated a significantly improved transplantation-free survival for patients randomized to receive an RV-PA shunt compared to those palliated with a modified BT shunt (74% vs. 64%, P 5 .01).222 The major benefit of the RV-PA shunt seems to be an increased survival rate in the early period after the procedure. The short-term survival advantage of the Sano modification of the Norwood operation for centers where the mortality rate after the Norwood operation was already below 15% will be hard to demonstrate. More recent long-term followup, however, demonstrates an elimination of this survival advantage at 3 years of age with the RV-PA shunt group.223 In addition, this group had worse RV ejection fraction, validating concerns regarding the impact of an incision in the systemic ventricle.223 Orthotopic heart transplantation has gained acceptance as an alternative treatment for HLHS.224 Neonatal transplants appear to be well-tolerated, and some centers have avoided maintenance steroid therapy while achieving excellent midterm results using transplantation as the sole therapeutic option for this disease.225,226 Others have successfully advocated a combined approach using either transplantation or staged reconstruction, depending on the pathophysiologic state of the child and the availability of a donor heart. However, the critical shortage of donor organs places a marked limitation on the correction of this common congenital heart lesion.



CHAPTER 36 Critical Care After Surgery for Congenital Cardiac Disease





Many children have derived benefit from a completed, staged reconstruction or heart transplantation for this previously fatal illness. They are often able to lead active, productive lives, and many do develop normally.227,228 Both survival and developmental outcomes for this disease are improving worldwide. However, the long-term prognosis for this evolving therapy will not be known for several years. Hybrid Approach. An alternative to the stage I Norwood palliation that eliminates the insult associated with CPB in the fragile neonate with HLHS has been developed. This approach combines interventional cardiac catheterization techniques with less-involved surgery and is referred to as the hybrid procedure.229,230 The goal is to replicate the physiologic state of the Norwood procedure by the combination of three interventions: (1) placement of bilateral pulmonary artery bands to limit flow to the lungs, (2) placement of an endovascular stent to maintain the long-term patency of the ductus arteriosus, and (3) balloon atrial septostomy with or without stenting of the atrial communication to ensure adequate mixing and unrestricted left-to-right atrial flow.229–232 This procedure is accomplished through a standard median sternotomy and does not require CPB. The procedure requires superb coordination between the surgical and cardiac catheterization teams. It is generally performed in a hybrid suite consisting of a large modern cardiac catheterization laboratory with enough room to accommodate the surgeons and supporting operating room staff in addition to the interventional cardiology team and anesthesiologists. Outcomes have been encouraging, with early mortality rates comparable to those of standard protocols (about 15%–20%).233,234 Direct comparison of the two methods is complicated because, in most centers, the hybrid procedure generally has been reserved for patients regarded as high risk for bypass surgery (e.g., low birthweight, unstable hemodynamics, and poor ventricular function). Interstage mortality can be significant (15%–20%),233 and case-matched studies have shown no benefit over conventional surgery.232 One barrier to the widespread implementation of the hybrid procedure is that results of conventional surgery in the low-risk groups are now so good that many centers of excellence have been reluctant to undertake a new procedure with its attendant learning curve. The hybrid procedure poses some unique technical challenges: the ductal stent position is crucial and, if patients have a diminutive ascending and transverse aorta, then the procedure does not address this impediment to coronary flow. If the transverse aorta is small, the stent itself might distort or interfere with retrograde flow into the arch. For this reason, most centers do not recommend the hybrid approach in the setting of a small transverse arch. Nevertheless, the hybrid approach undoubtedly has a role in the management of patients with HLHS. With continued encouraging results235 beyond the centers of excellence that developed and advanced this technique, it has found a niche among high-risk patients or as a bridge to transplantation. Up to 50% of patients who survive the stage I hybrid procedure require catheter reintervention due to stent migration or restrictive flow across the atrial communication.233 The stage II procedure becomes much more extensive than the conventional stage II because the aortic arch needs to be reconstructed (excising the ductal stent), the bands removed, and the pulmonary arteries repaired with a patch in addition to creating the cavopulmonary connection. Consequently, stage II after the hybrid procedure carries higher operative mortality (10% to 15%). This needs to be taken into account when comparing it with conventional techniques.233,235 No absolute consensus exists on the future of the

409

hybrid procedure. This innovative approach offers potential benefits that should be scrutinized through careful study and longterm follow-up.

Summary The cardiac ICU has become the epicenter of activity in large cardiovascular programs. Nowhere are collaborative practices and multidisciplinary skills more valued or necessary. A curriculum in cardiac intensive care is now formally incorporated into cardiology training. Pediatric intensive care training programs have a mandate to include curricula and experience in the management of postoperative cardiac patients. Additional cardiac intensive care training is offered in selected centers to pediatric intensive care specialists wishing to pursue a career in the cardiac ICU. Specialists in this field must have in-depth training in pediatric intensive care and cardiology, as the scope of practice goes well beyond the cardiovascular system and requires expertise in complex respiratory physiology, diagnosis and management of multiorgan system dysfunction, and the various supportive techniques vital to the discipline of intensive care, to name a few. Increased complexity of disease, advances in technology and applied research, shortened LOSs, and improved survival in contemporary series all describe the fast-paced specialized environment that has accompanied the development of this new specialty of pediatric cardiac intensive care. The dramatic reduction in cardiac intensive care mortality has been gratifying. It is attributable to many factors. Achieving 100% survival with minimal morbidity remains our elusive goal. It will challenge the next generation of practitioners.

Acknowledgment We gratefully acknowledge the contributions of Peter C. Laussen and David L. Wessel, who authored this chapter in previous editions.

Key References Agus MS, Steil GM, Wypij D, et al. Tight glycemic control versus standard care after pediatric cardiac surgery. N Engl J Med. 2012;367:1208-1219. Almond CS, Singh TP, Gauvreau K, et al. Extracorporeal membrane oxygenation for bridge to heart transplantation among children in the United States: analysis of data from the Organ Procurement and Transplant Network and Extracorporeal Life Support Organization Registry. Circulation. 2011;123:2975-2984. Atz AM, Wessel DL. Sildenafil ameliorates effects of inhaled nitric oxide withdrawal. Anesthesiology. 1999;91:307-310. Booth KL, Roth SJ, Thiagarajan RR, Almodovar MC, del Nido PJ, Laussen PC. Extracorporeal membrane oxygenation support of the Fontan and bidirectional Glenn circulations. Ann Thorac Surg. 2004;77:1341-1348. Chang AC, Hanley FL, Wernovsky G, et al. Early bidirectional cavopulmonary shunt in young infants. Postoperative course and early results. Circulation. 1993;88:II149- II58. d’Udekem Y, Iyengar AJ, Galati JC, et al. Redefining expectations of long-term survival after the Fontan procedure: twenty-five years of follow-up from the entire population of Australia and New Zealand. Circulation. 2014;130:S32-S38. Galantowicz M, Cheatham JP. Lessons learned from the development of a new hybrid strategy for the management of hypoplastic left heart syndrome. Pediatr Cardiol. 2005;26:190-199. Hoffman GM, Tweddell JS, Ghanayem NS, et al. Alteration of the critical arteriovenous oxygen saturation relationship by sustained

410

SECTION IV



Pediatric Critical Care: Cardiovascular

afterload reduction after the Norwood procedure. J Thorac Cardiovasc Surg. 2004;127:738-745. Jatene AD, Fontes VF, Souza LC, Paulista PP, Neto CA, Sousa JE. Anatomic correction of transposition of the great arteries. J Thorac Cardiovasc Surg. 1982;83:20-26. Lang P, Chipman CW, Siden H, Williams RG, Norwood WI, Castaneda AR. Early assessment of hemodynamic status after repair of tetralogy of Fallot: a comparison of 24 hour (intensive care unit) and 1 year postoperative data in 98 patients. Am J Cardiol. 1982;50:795-799.

Norwood WI, Lang P, Hansen DD. Physiologic repair of aortic atresiahypoplastic left heart syndrome. N Engl J Med. 1983;308:23-26. Ohye RG, Sleeper LA, Mahony L, et al. Comparison of shunt types in the Norwood procedure for single-ventricle lesions. N Engl J Med. 2010;362:1980-1992. Wernovsky G, Mayer Jr JE, Jonas RA, et al. Factors influencing early and late outcome of the arterial switch operation for transposition of the great arteries. J Thorac Cardiovasc Surg. 1995;109:289-301.

The full reference list for this chapter is available at ExpertConsult.com.

e1























































































1. Murphy SL, Mathews TJ, Martin JA, Minkovitz CS, Strobino DM. Annual Summary of Vital Statistics: 2013-2014. Pediatrics. 2017; 139:e20163239. 2. Gilboa SM, Devine OJ, Kucik JE, et al. Congenital heart defects in the United States: estimating the magnitude of the affected population in 2010. Circulation. 2016;134:101-109. 3. Friedman WF. The intrinsic physiologic properties of the developing heart. Prog Cardiovasc Dis. 1972;15:87-111. 4. Romero TE, Friedman WF. Limited left ventricular response to volume overload in the neonatal period: a comparative study with the adult animal. Pediatr Res. 1979;13:910-915. 5. Mills AN, Haworth SG. Greater permeability of the neonatal lung. Postnatal changes in surface charge and biochemistry of porcine pulmonary capillary endothelium. J Thorac Cardiovasc Surg. 1991;101:909-916. 6. Hanley FL, Heinemann MK, Jonas RA, et al. Repair of truncus arteriosus in the neonate. J Thorac Cardiovasc Surg. 1993;105:1047-1056. 7. McCrindle BW, Tchervenkov CI, Konstantinov IE, et al. Risk factors associated with mortality and interventions in 472 neonates with interrupted aortic arch: a Congenital Heart Surgeons Society study. J Thorac Cardiovasc Surg. 2005;129:343-350. 8. Planche C, Serraf A, Comas JV, Lacour-Gayet F, Bruniaux J, Touchot A. Anatomic repair of transposition of great arteries with ventricular septal defect and aortic arch obstruction. One-stage versus two-stage procedure. J Thorac Cardiovasc Surg. 1993;105:925-933. 9. Gladman G, McCrindle BW, Williams WG, Freedom RM, Benson LN. The modified Blalock-Taussig shunt: clinical impact and morbidity in Fallot’s tetralogy in the current era. J Thorac Cardiovasc Surg. 1997;114:25-30. 10. Fisher DJ, Heymann MA, Rudolph AM. Fetal myocardial oxygen and carbohydrate consumption during acutely induced hypoxemia. Am J Physiol. 1982;242:H657-H661. 11. Skoff RP, Bessert D, Barks JD, Silverstein FS. Plasticity of neurons and glia following neonatal hypoxic-ischemic brain injury in rats. Neurochem Res. 2007;32:331-342. 12. Geva T. Introduction: magnetic resonance imaging. Pediatr Cardiol. 2000;21:3-4. 13. Chung T. Assessment of cardiovascular anatomy in patients with congenital heart disease by magnetic resonance imaging. Pediatr Cardiol. 2000;21:18-26. 14. Freed MD, Heymann MA, Lewis AB, Roehl SL, Kensey RC. Prostaglandin E1 infants with ductus arteriosus-dependent congenital heart disease. Circulation. 1981;64:899-905. 15. Yokota M, Muraoka R, Aoshima M, et al. Modified Blalock-Taussig shunt following long-term administration of prostaglandin E1 for ductus-dependent neonates with cyanotic congenital heart disease. J Thorac Cardiovasc Surg. 1985;90:399-403. 16. Lewis AB, Freed MD, Heymann MA, Roehl SL, Kensey RC. Side effects of therapy with prostaglandin E1 in infants with critical congenital heart disease. Circulation. 1981;64:893-898. 17. Lim DS, Kulik TJ, Kim DW, Charpie JR, Crowley DC, Maher KO. Aminophylline for the prevention of apnea during prostaglandin E1 infusion. Pediatrics. 2003;112:e27-e29. 18. Donahoo JS, Roland JM, Kan J, Gardner TJ, Kidd BS. Prostaglandin E1 as an adjunct to emergency cardiac operation in neonates. J Thorac Cardiovasc Surg. 1981;81:227-231. 19. Moler FW, Khan AS, Meliones JN, Custer JR, Palmisano J, Shope TC. Respiratory syncytial virus morbidity and mortality estimates in congenital heart disease patients: a recent experience. Crit Care Med. 1992;20:1406-1413. 20. Jonas RA, Lang P, Mayer JE, Castaneda AR. The importance of prostaglandin E1 in resuscitation of the neonate with critical aortic stenosis. J Thorac Cardiovasc Surg. 1985;89:314-315. 21. Jonas RA, Hansen DD, Cook N, Wessel D. Anatomic subtype and survival after reconstructive operation for hypoplastic left heart syndrome. J Thorac Cardiovasc Surg. 1994;107:1121-1127; discussion 1127-1128.

22. Hirsch JC, Charpie JR, Ohye RG, Gurney JG. Near-infrared spectroscopy: what we know and what we need to know—a systematic review of the congenital heart disease literature. J Thorac Cardiovasc Surg. 2009;137:154-159, 159e1-12. 23. Hoffman GM, Brosig CL, Mussatto KA, Tweddell JS, Ghanayem NS. Perioperative cerebral oxygen saturation in neonates with hypoplastic left heart syndrome and childhood neurodevelopmental outcome. J Thorac Cardiovasc Surg. 2013;146:1153-1164. 24. Dodge-Khatami J, Gottschalk U, Eulenburg C, et al. Prognostic value of perioperative near-infrared spectroscopy during neonatal and infant congenital heart surgery for adverse in-hospital clinical events. World J Pediatr Congenit Heart Surg. 2012;3:221-228. 25. Lang P, Chipman CW, Siden H, Williams RG, Norwood WI, Castaneda AR. Early assessment of hemodynamic status after repair of tetralogy of Fallot: a comparison of 24 hour (intensive care unit) and 1 year postoperative data in 98 patients. Am J Cardiol. 1982;50:795799. 26. Wernovsky G, Wypij D, Jonas RA, et al. Postoperative course and hemodynamic profile after the arterial switch operation in neonates and infants. A comparison of low-flow cardiopulmonary bypass and circulatory arrest. Circulation. 1995;92:2226-2235. 27. Friedberg MK, Dubin AM, Van Hare GF, McDaniel G, Niksch A, Rosenthal DN. Acute effects of single-site pacing from the left and right ventricle on ventricular function and ventricular-ventricular interactions in children with normal hearts. Congenit Heart Dis. 2009;4:356-361. 28. Petrucci O, O’Brien SM, Jacobs ML, Jacobs JP, Manning PB, Eghtesady P. Risk factors for mortality and morbidity after the neonatal Blalock-Taussig shunt procedure. Ann Thorac Surg. 2011;92:642-651; discussion 651-652. 29. Ross HJ, Law Y, Book WM, et al. Transplantation and mechanical circulatory support in congenital heart disease: a scientific statement from the American Heart Association. Circulation. 2016;133:802-820. 30. Wolf MJ, Kanter KR, Kirshbom PM, Kogon BE, Wagoner SF. Extracorporeal cardiopulmonary resuscitation for pediatric cardiac patients. Ann Thorac Surg. 2012;94:874-879; discussion 879-880. 31. Delmo Walter EM, Alexi-Meskishvili V, Huebler M, et al. Extracorporeal membrane oxygenation for intraoperative cardiac support in children with congenital heart disease. Interact Cardiovasc Thorac Surg. 2010;10:753-758. 32. Almond CS, Singh TP, Gauvreau K, et al. Extracorporeal membrane oxygenation for bridge to heart transplantation among children in the United States: analysis of data from the Organ Procurement and Transplant Network and Extracorporeal Life Support Organization Registry. Circulation. 2011;123:2975-2984. 33. Mascio CE, Austin EH III, Jacobs JP, et al. Perioperative mechanical circulatory support in children: an analysis of the Society of Thoracic Surgeons Congenital Heart Surgery Database. J Thorac Cardiovasc Surg. 2014;147:658-664: discussion 664-665. 34. Brunetti MA, Gaynor JW, Retzloff LB, et al. Characteristics, risk factors, and outcomes of extracorporeal membrane oxygenation use in pediatric cardiac ICUs: a report from the Pediatric Cardiac Critical Care Consortium Registry. Pediatr Crit Care Med. 2018;19: 544-552. 35. Peberdy MA, Gluck JA, Ornato JP, et al. Cardiopulmonary resuscitation in adults and children with mechanical circulatory support: a scientific statement from the American Heart Association. Circulation. 2017;135:e1115-e1134. 36. Cullen S, Shore D, Redington A. Characterization of right ventricular diastolic performance after complete repair of tetralogy of Fallot. Restrictive physiology predicts slow postoperative recovery. Circulation. 1995;91:1782-1789. 37. Redington AN, Penny D, Rigby ML, Hayes A. Antegrade diastolic pulmonary arterial flow as a marker of right ventricular restriction after complete repair of pulmonary atresia with intact septum and critical pulmonary valvar stenosis. Cardiol Young. 1992;2:382-386. 38. Zuppa AF, Nicolson SC, Adamson PC, et al. Population pharmacokinetics of milrinone in neonates with hypoplastic left heart

References



59.

















































60. Lopez-Gutierrez JC, Tovar JA. Chylothorax and chylous ascites: management and pitfalls. Semin Pediatr Surg. 2014;23:298-302. 61. Kreutzer J, Kreutzer C. Lymphodynamics in congenital heart disease: the forgotten circulation. J Am Coll Cardiol. 2017;69:2423-2427. 62. Starling EH. On the absorption of fluids from the connective tissue spaces. J Physiol. 1896;19:312-326. 63. Yeh J, Brown ER, Kellogg KA, et al. Utility of a clinical practice guideline in treatment of chylothorax in the postoperative congenital heart patient. Ann Thorac Surg. 2013;96:930-936. 64. Panthongviriyakul C, Bines JE. Post-operative chylothorax in children: an evidence-based management algorithm. J Paediatr Child Health. 2008;44:716-721. 65. Maldonado F, Hawkins FJ, Daniels CE, Doerr CH, Decker PA, Ryu JH. Pleural fluid characteristics of chylothorax. Mayo Clin Proc. 2009;84:129-133. 66. Allen EM, van Heeckeren DW, Spector ML, Blumer JL. Management of nutritional and infectious complications of postoperative chylothorax in children. J Pediatr Surg. 1991;26:1169-1174. 67. Buttiker V, Fanconi S, Burger R. Chylothorax in children: guidelines for diagnosis and management. Chest. 1999;116:682-687. 68. Roehr CC, Jung A, Proquitte H, et al. Somatostatin or octreotide as treatment options for chylothorax in young children: a systematic review. Intensive Care Med. 2006;32:650-657. 69. Caverly L, Rausch CM, da Cruz E, Kaufman J. Octreotide treatment of chylothorax in pediatric patients following cardiothoracic surgery. Congenit Heart Dis. 2010;5:573-578. 70. Nath DS, Savla J, Khemani RG, Nussbaum DP, Greene CL, Wells WJ. Thoracic duct ligation for persistent chylothorax after pediatric cardiothoracic surgery. Ann Thorac Surg. 2009;88:246-251; discussion 251-252. 71. Engum SA, Rescorla FJ, West KW, Scherer LR III, Grosfeld JL. The use of pleuroperitoneal shunts in the management of persistent chylothorax in infants. J Pediatr Surg. 1999;34:286-290. 72. Dori Y, Keller MS, Rome JJ, et al. Percutaneous lymphatic embolization of abnormal pulmonary lymphatic flow as treatment of plastic bronchitis in patients with congenital heart disease. Circulation. 2016;133:1160-1170. 73. Majdalany BS, Saad WA, Chick JFB, Khaja MS, Cooper KJ, Srinivasa RN. Pediatric lymphangiography, thoracic duct embolization and thoracic duct disruption: a single-institution experience in 11 children with chylothorax. Pediatr Radiol. 2018;48:235-240. 74. Itkin M. Interventional Treatment of Pulmonary Lymphatic Anomalies. Tech Vasc Interv Radiol. 2016;19:299-304. 75. Choi I, Lee S, Hong YK. The new era of the lymphatic system: no longer secondary to the blood vascular system. Cold Spring Harb Perspect Med. 2012;2:a006445. 76. Dori Y, Keller MS, Fogel MA, et al. MRI of lymphatic abnormalities after functional single-ventricle palliation surgery. AJR Am J Roentgenol. 2014;203:426-431. 77. Mittnacht AJ, Hollinger I. Fast-tracking in pediatric cardiac surgery—the current standing. Ann Card Anaesth. 2010;13:92-101. 78. Vricella LA, Dearani JA, Gundry SR, Razzouk AJ, Brauer SD, Bailey LL. Ultra fast track in elective congenital cardiac surgery. Ann Thorac Surg. 2000;69:865-871. 79. Limperopoulos C, Majnemer A, Shevell MI, Rosenblatt B, Rohlicek C, Tchervenkov C. Neurodevelopmental status of newborns and infants with congenital heart defects before and after open heart surgery. J Pediatr. 2000;137:638-645. 80. Marino BS, Lipkin PH, Newburger JW, et al. Neurodevelopmental outcomes in children with congenital heart disease: evaluation and management: a scientific statement from the American Heart Association. Circulation. 2012;126:1143-1172. 81. Donofrio MT, Bremer YA, Schieken RM, et al. Autoregulation of cerebral blood flow in fetuses with congenital heart disease: the brain sparing effect. Pediatr Cardiol. 2003;24:436-443. 82. Licht DJ, Shera DM, Clancy RR, et al. Brain maturation is delayed in infants with complex congenital heart defects. J Thorac Cardiovasc Surg. 2009;137:529-536; discussion 536-537.





58.





57.





56.





55.





54.





53.





52.





51.





50.



49.



48.



47.



46.



45.



44.



43.



42.





41.

syndrome undergoing stage I reconstruction. Anesth Analg. 2006; 102:1062-1069. Hoffman TM, Wernovsky G, Atz AM, et al. Efficacy and safety of milrinone in preventing low cardiac output syndrome in infants and children after corrective surgery for congenital heart disease. Circulation. 2003;107:996-1002. Heath D, Edwards JE. The pathology of hypertensive pulmonary vascular disease; a description of six grades of structural changes in the pulmonary arteries with special reference to congenital cardiac septal defects. Circulation. 1958;18:533-547. Wessel DL. Current and future strategies in the treatment of childhood pulmonary hypertension. Prog Pediatr Cardiol. 2001;12:289-318. Ignarro LJ, Buga GM, Wood KS, Byrns RE, Chaudhuri G. Endotheliumderived relaxing factor produced and released from artery and vein is nitric oxide. Proc Natl Acad Sci U S A. 1987;84:9265-9269. Frostell C, Fratacci MD, Wain JC, Jones R, Zapol WM. Inhaled nitric oxide. A selective pulmonary vasodilator reversing hypoxic pulmonary vasoconstriction. Circulation. 1991;83:2038-2047. Atz AM, Wessel DL. Inhaled nitric oxide in the neonate with cardiac disease. Semin Perinatol. 1997;21:441-455. Journois D, Pouard P, Mauriat P, Malhere T, Vouhe P, Safran D. Inhaled nitric oxide as a therapy for pulmonary hypertension after operations for congenital heart defects. J Thorac Cardiovasc Surg. 1994;107:1129-1135. Miller OI, Tang SF, Keech A, Pigott NB, Beller E, Celermajer DS. Inhaled nitric oxide and prevention of pulmonary hypertension after congenital heart surgery: a randomised double-blind study. Lancet. 2000;356:1464-1469. Goldman AP, Delius RE, Deanfield JE, et al. Pharmacological control of pulmonary blood flow with inhaled nitric oxide after the fenestrated Fontan operation. Circulation. 1996;94:II44-II48. Adatia I, Atz AM, Wessel DL. Inhaled nitric oxide does not improve systemic oxygenation after bidirectional superior cavopulmonary anastomosis. J Thorac Cardiovasc Surg. 2005;129:217-219. Adatia I, Atz AM, Jonas RA, Wessel DL. Diagnostic use of inhaled nitric oxide after neonatal cardiac operations. J Thorac Cardiovasc Surg. 1996;112:1403-1405. Atz AM, Munoz RA, Adatia I, Wessel DL. Diagnostic and therapeutic uses of inhaled nitric oxide in neonatal Ebstein’s anomaly. Am J Cardiol. 2003;91:906-908. Atz AM, Wessel DL. Sildenafil ameliorates effects of inhaled nitric oxide withdrawal. Anesthesiology. 1999;91:307-310. Drummond WH, Gregory GA, Heymann MA, Phibbs RA. The independent effects of hyperventilation, tolazoline, and dopamine on infants with persistent pulmonary hypertension. J Pediatr. 1981;98:603-611. Keung CY, Smith KR, Savoia HF, Davidson AJ. An audit of transfusion of red blood cell units in pediatric anesthesia. Paediatr Anaesth. 2009;19:320-328. Guzzetta NA, Allen NN, Wilson EC, Foster GS, Ehrlich AC, Miller BE. Excessive postoperative bleeding and outcomes in neonates undergoing cardiopulmonary bypass. Anesth Analg. 2015;120:405-410. Iyengar A, Scipione CN, Sheth P, et al. Association of complications with blood transfusions in pediatric cardiac surgery patients. Ann Thorac Surg. 2013;96:910-916. Cholette JM, Swartz MF, Rubenstein J, et al. Outcomes using a conservative versus liberal red blood cell transfusion strategy in infants requiring cardiac operation. Ann Thorac Surg. 2017;103: 206-214. Floh AA, Zafurallah I, MacDonald C, Honjo O, Fan CS, Laussen PC. The advantage of early plication in children diagnosed with diaphragm paresis. J Thorac Cardiovasc Surg. 2017;154:1715-1721.e4. Mery CM, Moffett BS, Khan MS, et al. Incidence and treatment of chylothorax after cardiac surgery in children: analysis of a large multi-institution database. J Thorac Cardiovasc Surg. 2014;147: 678-686.e1; discussion 685-686. Zuluaga MT. Chylothorax after surgery for congenital heart disease. Curr Opin Pediatr. 2012;24:291-294.





40.



39.



e2

e3













































105. Stevens PE, Levin A, Kidney Disease: Improving Global Outcomes Chronic Kidney Disease Guideline Development Work Group Members. Evaluation and management of chronic kidney disease: synopsis of the kidney disease: improving global outcomes 2012 clinical practice guideline. Ann Intern Med. 2013;158:825-830. 106. Sutherland SM, Byrnes JJ, Kothari M, et al. AKI in hospitalized children: comparing the pRIFLE, AKIN, and KDIGO definitions. Clin J Am Soc Nephrol. 2015;10:554-561. 107. Devarajan P. Biomarkers for the early detection of acute kidney injury. Curr Opin Pediatr. 2011;23:194-200. 108. Mishra J, Dent C, Tarabishi R, et al. Neutrophil gelatinase-associated lipocalin (NGAL) as a biomarker for acute renal injury after cardiac surgery. Lancet. 2005;365:1231-1238. 109. Zappitelli M, Washburn KK, Arikan AA, et al. Urine neutrophil gelatinase-associated lipocalin is an early marker of acute kidney injury in critically ill children: a prospective cohort study. Crit Care. 2007;11:R84. 110. Dent CL, Ma Q, Dastrala S, et al. Plasma neutrophil gelatinaseassociated lipocalin predicts acute kidney injury, morbidity and mortality after pediatric cardiac surgery: a prospective uncontrolled cohort study. Crit Care. 2007;11:R127. 111. Elliott M. Modified ultrafiltration and open heart surgery in children. Paediatr Anaesth. 1999;9:1-5. 112. Fivez T, Kerklaan D, Mesotten D, et al. Early versus late parenteral nutrition in critically ill children. N Engl J Med. 2016;374:1111-1122. 113. Berenholtz SM, Pronovost PJ, Lipsett PA, et al. Eliminating catheter-related bloodstream infections in the intensive care unit. Crit Care Med. 2004;32:2014-2020. 114. Nelson-McMillan K, Hornik CP, He X, et al. Delayed Sternal closure in infant heart surgery—the importance of where and when: an analysis of the STS Congenital Heart Surgery Database. Ann Thorac Surg. 2016;102:1565-1572. 115. Falcao G, Ulate K, Kouzekanani K, Bielefeld MR, Morales JM, Rotta AT. Impact of postoperative hyperglycemia following surgical repair of congenital cardiac defects. Pediatr Cardiol. 2008;29: 628-636. 116. Yates AR, Dyke PC II, Taeed R, et al. Hyperglycemia is a marker for poor outcome in the postoperative pediatric cardiac patient. Pediatr Crit Care Med. 2006;7:351-355. 117. van den Berghe G, Wouters P, Weekers F, et al. Intensive insulin therapy in critically ill patients. N Engl J Med. 2001;345:1359-1367. 118. Vlasselaers D, Milants I, Desmet L, et al. Intensive insulin therapy for patients in paediatric intensive care: a prospective, randomised controlled study. Lancet. 2009;373:547-556. 119. Macrae D, Grieve R, Allen E, et al. A randomized trial of hyperglycemic control in pediatric intensive care. N Engl J Med. 2014;370: 107-118. 120. Agus MS, Steil GM, Wypij D, et al. Tight glycemic control versus standard care after pediatric cardiac surgery. N Engl J Med. 2012; 367:1208-1219. 121. Faustino EVS, Hirshberg EL, Asaro LA, et al. Short-term adverse outcomes associated with hypoglycemia in critically ill children. Crit Care Med. 2019;47:706-714. 122. Sadhwani A, Asaro LA, Goldberg C, et al. Impact of tight glycemic control on neurodevelopmental outcomes at 1 year of age for children with congenital heart disease: a randomized controlled trial. J Pediatr. 2016;174:193-198.e2. 123. Atz AM, Feinstein JA, Jonas RA, Perry SB, Wessel DL. Preoperative management of pulmonary venous hypertension in hypoplastic left heart syndrome with restrictive atrial septal defect. Am J Cardiol. 1999;83:1224-1228. 124. Vlahos AP, Lock JE, McElhinney DB, van der Velde ME. Hypoplastic left heart syndrome with intact or highly restrictive atrial septum: outcome after neonatal transcatheter atrial septostomy. Circulation. 2004;109:2326-2330. 125. Glatz JA, Tabbutt S, Gaynor JW, et al. Hypoplastic left heart syndrome with atrial level restriction in the era of prenatal diagnosis. Ann Thorac Surg. 2007;84:1633-1638.



















































83. Back SA, Han BH, Luo NL, et al. Selective vulnerability of late oligodendrocyte progenitors to hypoxia-ischemia. J Neurosci. 2002; 22:455-463. 84. Mahle WT, Tavani F, Zimmerman RA, et al. An MRI study of neurological injury before and after congenital heart surgery. Circulation. 2002;106:I109-I114. 85. Galli KK, Zimmerman RA, Jarvik GP, et al. Periventricular leukomalacia is common after neonatal cardiac surgery. J Thorac Cardiovasc Surg. 2004;127:692-704. 86. Petit CJ, Rome JJ, Wernovsky G, et al. Preoperative brain injury in transposition of the great arteries is associated with oxygenation and time to surgery, not balloon atrial septostomy. Circulation. 2009;119: 709-716. 87. Newburger JW, Jonas RA, Wernovsky G, et al. A comparison of the perioperative neurologic effects of hypothermic circulatory arrest versus low-flow cardiopulmonary bypass in infant heart surgery. N Engl J Med. 1993;329:1057-1064. 88. Limperopoulos C, Majnemer A, Shevell MI, et al. Predictors of developmental disabilities after open heart surgery in young children with congenital heart defects. J Pediatr. 2002;141:51-58. 89. Anand KJ, Hickey PR. Halothane-morphine compared with highdose sufentanil for anesthesia and postoperative analgesia in neonatal cardiac surgery. N Engl J Med. 1992;326:1-9. 90. van Dijk M, de Boer JB, Koot HM, Tibboel D, Passchier J, Duivenvoorden HJ. The reliability and validity of the COMFORT scale as a postoperative pain instrument in 0 to 3-year-old infants. Pain. 2000;84:367-377. 91. Johansson M, Kokinsky E. The COMFORT behavioural scale and the modified FLACC scale in paediatric intensive care. Nurs Crit Care. 2009;14:122-130. 92. Kerson AG, DeMaria R, Mauer E, et al. Validity of the Richmond Agitation-Sedation Scale (RASS) in critically ill children. J Intensive Care. 2016;4:65. 93. Garcia Guerra G, Robertson CM, Alton GY, et al. Neurotoxicity of sedative and analgesia drugs in young infants with congenital heart disease: 4-year follow-up. Paediatr Anaesth. 2014;24:257-265. 94. Traube C, Silver G, Gerber LM, et al. Delirium and mortality in critically ill children: epidemiology and outcomes of pediatric delirium. Crit Care Med. 2017;45:891-898. 95. Herrup EA, Wieczorek B, Kudchadkar SR. Characteristics of postintensive care syndrome in survivors of pediatric critical illness: a systematic review. World J Crit Care Med. 2017;6:124-134. 96. Girard TD, Jackson JC, Pandharipande PP, et al. Delirium as a predictor of long-term cognitive impairment in survivors of critical illness. Crit Care Med. 2010;38:1513-1520. 97. Sockalingam S, Parekh N, Bogoch, II, et al. Delirium in the postoperative cardiac patient: a review. J Card Surg. 2005;20:560-567. 98. Traube C, Silver G, Reeder RW, et al. Delirium in critically ill Children: An International Point Prevalence Study. Crit Care Med. 2017;45:584-590. 99. Patel AK, Biagas KV, Clarke EC, et al. Delirium in children after cardiac bypass surgery. Pediatr Crit Care Med. 2017;18:165-171. 100. Lucas SS, Nasr VG, Ng AJ, Joe C, Bond M, DiNardo JA. Pediatric cardiac intensive care society. 2014 consensus statement: pharmacotherapies in cardiac critical care: sedation, analgesia and muscle relaxant. Pediatr Crit Care Med. 2016;17:S3-S15. 101. Traube C, Silver G, Kearney J, et al. Cornell Assessment of Pediatric Delirium: a valid, rapid, observational tool for screening delirium in the PICU*. Crit Care Med. 2014;42:656-663. 102. Carollo DS, Nossaman BD, Ramadhyani U. Dexmedetomidine: a review of clinical applications. Curr Opin Anaesthesiol. 2008; 21:457-461. 103. Rodriguez-Gonzalez R, Sobrino T, Veiga S, et al. Neuroprotective effects of dexmedetomidine conditioning strategies: Evidences from an in vitro model of cerebral ischemia. Life Sci. 2016;144:162-169. 104. Akcan-Arikan A, Zappitelli M, Loftis LL, Washburn KK, Jefferson LS, Goldstein SL. Modified RIFLE criteria in critically ill children with acute kidney injury. Kidney Int. 2007;71:1028-1035.

e4







148.





149.





150.





151.



152.











147.





153.





154.















155.







156.





158.





159.





160.





161.



162.







157.











163.







164.





165.





166.





167.



168.









































126. Kalish BT, Tworetzky W, Benson CB, et al. Technical challenges of atrial septal stent placement in fetuses with hypoplastic left heart syndrome and intact atrial septum. Catheter Cardiovasc Interv. 2014; 84:77-85. 127. Marshall AC, van der Velde ME, Tworetzky W, et al. Creation of an atrial septal defect in utero for fetuses with hypoplastic left heart syndrome and intact or highly restrictive atrial septum. Circulation. 2004;110:253-258. 128. Vida VL, Bacha EA, Larrazabal A, et al. Hypoplastic left heart syndrome with intact or highly restrictive atrial septum: surgical experience from a single center. Ann Thorac Surg. 2007;84:581585; discussion 586. 129. Chang AC, Hanley FL, Wernovsky G, et al. Early bidirectional cavopulmonary shunt in young infants. Postoperative course and early results. Circulation. 1993;88:II149-II158. 130. Fontan F, Kirklin JW, Fernandez G, et al. Outcome after a “perfect” Fontan operation. Circulation. 1990;81:1520-1536. 131. Castaneda AR. From Glenn to Fontan. A continuing evolution. Circulation. 1992;86:II80-II84. 132. Gentles TL, Mayer Jr JE, Gauvreau K, et al. Fontan operation in five hundred consecutive patients: factors influencing early and late outcome. J Thorac Cardiovasc Surg. 1997;114:376-391. 133. d’Udekem Y, Iyengar AJ, Galati JC, et al. Redefining expectations of long-term survival after the Fontan procedure: twenty-five years of follow-up from the entire population of Australia and New Zealand. Circulation. 2014;130:S32-S38. 134. Redington AN, Penny D, Shinebourne EA. Pulmonary blood flow after total cavopulmonary shunt. Br Heart J. 1991;65:213-217. 135. Penny DJ, Redington AN. Doppler echocardiographic evaluation of pulmonary blood flow after the Fontan operation: the role of the lungs. Br Heart J. 1991;66:372-374. 136. Walsh MA, Merat M, La Rotta G, et al. Airway pressure release ventilation improves pulmonary blood flow in infants after cardiac surgery. Crit Care Med. 2011;39:2599-2604. 137. Shekerdemian LS, Bush A, Shore DF, Lincoln C, Redington AN. Cardiopulmonary interactions after Fontan operations: augmentation of cardiac output using negative pressure ventilation. Circulation. 1997;96:3934-3942. 138. Fishberger SB, Wernovsky G, Gentles TL, et al. Factors that influence the development of atrial flutter after the Fontan operation. J Thorac Cardiovasc Surg. 1997;113:80-86. 139. Stewart RD, Pasquali SK, Jacobs JP, et al. Contemporary Fontan operation: association between early outcome and type of cavopulmonary connection. Ann Thorac Surg. 2012;93:1254-1260; discussion 1261. 140. Booth KL, Roth SJ, Thiagarajan RR, Almodovar MC, del Nido PJ, Laussen PC. Extracorporeal membrane oxygenation support of the Fontan and bidirectional Glenn circulations. Ann Thorac Surg. 2004;77:1341-1348. 141. Sivarajan VB, Almodovar MC, Rodefeld MD, Laussen PC. Pediatric extracorporeal life support in specialized situations. Pediatr Crit Care Med. 2013;14:S51-S61. 142. Spach MS, Serwer GA, Anderson PA, Canent Jr RV, Levin AR. Pulsatile aortopulmonary pressure-flow dynamics of patent ductus arteriosus in patients with various hemodynamic states. Circulation. 1980;61:110-122. 143. Bancalari E, Jesse MJ, Gelband H, Garcia O. Lung mechanics in congenital heart disease with increased and decreased pulmonary blood flow. J Pediatr. 1977;90:192-195. 144. Gerhardt T, Bancalari E. Lung compliance in newborns with patent ductus arteriosus before and after surgical ligation. Biol Neonate. 1980;38:96-105. 145. Rashkind WJ, Mullins CE, Hellenbrand WE, Tait MA. Nonsurgical closure of patent ductus arteriosus: clinical application of the Rashkind PDA Occluder System. Circulation. 1987;75:583-592. 146. Laborde F, Noirhomme P, Karam J, Batisse A, Bourel P, Saint Maurice O. A new video-assisted thoracoscopic surgical technique for

interruption of patient ductus arteriosus in infants and children. J Thorac Cardiovasc Surg. 1993;105:278-280. Burke RP, Wernovsky G, van der Velde M, Hansen D, Castaneda AR. Video-assisted thoracoscopic surgery for congenital heart disease. J Thorac Cardiovasc Surg. 1995;109:499-507; discussion 8. Robinson S, Gregory GA. Fentanyl-air-oxygen anesthesia for ligation of patent ductus arteriosus in preterm infants. Anesth Analg. 1981; 60:331-334. Fan LL, Campbell DN, Clarke DR, Washington RL, Fix EJ, White CW. Paralyzed left vocal cord associated with ligation of patent ductus arteriosus. J Thorac Cardiovasc Surg. 1989;98:611-613. Zbar RI, Chen AH, Behrendt DM, Bell EF, Smith RJ. Incidence of vocal fold paralysis in infants undergoing ligation of patent ductus arteriosus. Ann Thorac Surg. 1996;61:814-816. Odegard KC, Kirse DJ, del Nido PJ, et al. Intraoperative recurrent laryngeal nerve monitoring during video-assisted thoracoscopic surgery for patent ductus arteriosus. J Cardiothorac Vasc Anesth. 2000;14:562-564. Baylen B, Meyer RA, Korfhagen J, Benzing G III, Bubb ME, Kaplan S. Left ventricular performance in the critically ill premature infant with patent ductus arteriosus and pulmonary disease. Circulation. 1977;55:182-188. Haworth SG. Pulmonary vascular disease in secundum atrial septal defect in childhood. Am J Cardiol. 1983;51:265-272. Collins G, Calder L, Rose V, Kidd L, Keith J. Ventricular septal defect: clinical and hemodynamic changes in the first five years of life. Am Heart J. 1972;84:695-705. Rabinovitch M, Haworth SG, Castaneda AR, Nadas AS, Reid LM. Lung biopsy in congenital heart disease: a morphometric approach to pulmonary vascular disease. Circulation. 1978;58:1107-1122. Friedman WF, Heiferman MF. Clinical problems of postoperative pulmonary vascular disease. Am J Cardiol. 1982;50:631-636. Haworth SG, Sauer U, Buhlmeyer K, Reid L. Development of the pulmonary circulation in ventricular septal defect: a quantitative structural study. Am J Cardiol. 1977;40:781-788. Vincent RN, Lang P, Elixson EM, et al. Measurement of extravascular lung water in infants and children after cardiac surgery. Am J Cardiol. 1984;54:161-165. Tucker EM, Pyles LA, Bass JL, Moller JH. Permanent pacemaker for atrioventricular conduction block after operative repair of perimembranous ventricular septal defect. J Am Coll Cardiol. 2007; 50:1196-1200. Cordell D, Graham Jr TP, Atwood GF, Boerth RC, Boucek RJ, Bender HW. Left heart volume characteristics following ventricular septal defect closure in infancy. Circulation. 1976;54:294-298. Jablonsky G, Hilton JD, Liu PP, et al. Rest and exercise ventricular function in adults with congenital ventricular septal defects. Am J Cardiol. 1983;51:293-298. Studer M, Blackstone EH, Kirklin JW, et al. Determinants of early and late results of repair of atrioventricular septal (canal) defects. J Thorac Cardiovasc Surg. 1982;84:523-542. Rizzoli G, Mazzucco A, Maizza F, et al. Does Down syndrome affect prognosis of surgically managed atrioventricular canal defects? J Thorac Cardiovasc Surg. 1992;104:945-953. Marcelletti C, McGoon DC, Mair DD. The natural history of truncus arteriosus. Circulation. 1976;54:108-111. Bove EL, Lupinetti FM, Pridjian AK, et al. Results of a policy of primary repair of truncus arteriosus in the neonate. J Thorac Cardiovasc Surg. 1993;105:1057-1065; discussion 1065-1066. Ebert PA, Robinson SJ, Stanger P, Engle MA. Pulmonary artery conduits in infants younger than six months of age. J Thorac Cardiovasc Surg. 1976;72:351-356. Raisher BD, Grant JW, Martin TC, Strauss AW, Spray TL. Complete repair of total anomalous pulmonary venous connection in infancy. J Thorac Cardiovasc Surg. 1992;104:443-448. Jenkins KJ, Sanders SP, Orav EJ, Coleman EA, Mayer Jr JE, Colan SD. Individual pulmonary vein size and survival in infants with

e5





183.





184.





185.





186.





188.





187.











182.







181.







180.







179.







178.







177.







176.







175.







174.







173.







172.







171.







170.

189. Karl TR, Sano S, Pornviliwan S, Mee RB. Tetralogy of Fallot: favorable outcome of nonneonatal transatrial, transpulmonary repair. Ann Thorac Surg. 1992;54:903-907. 190. Dryzek P, Mazurek-Kula A, Moszura T, Sysa A. Right ventricle outflow tract stenting as a method of palliative treatment of severe tetralogy of Fallot. Cardiol J. 2008;15:376-379. 191. Dohlen G, Chaturvedi RR, Benson LN, et al. Stenting of the right ventricular outflow tract in the symptomatic infant with tetralogy of Fallot. Heart. 2009;95:142-147. 192. Pigula FA, Khalil PN, Mayer JE, del Nido PJ, Jonas RA. Repair of tetralogy of Fallot in neonates and young infants. Circulation. 1999;100:II157-II161. 193. Castaneda AR, Freed MD, Williams RG, Norwood WI. Repair of tetralogy of Fallot in infancy. Early and late results. J Thorac Cardiovasc Surg. 1977;74:372-381. 194. Dabizzi RP, Caprioli G, Aiazzi L, et al. Distribution and anomalies of coronary arteries in tetralogy of Fallot. Circulation. 1980;61:95-102. 195. Al Habib HF, Jacobs JP, Mavroudis C, et al. Contemporary patterns of management of tetralogy of Fallot: data from the Society of Thoracic Surgeons Database. Ann Thorac Surg. 2010;90:813-819; discussion 819-820. 196. Borow KM, Green LH, Castaneda AR, Keane JF. Left ventricular function after repair of tetralogy of Fallot and its relationship to age at surgery. Circulation. 1980;61:1150-1158. 197. Nudel DB, Berman MA, Talner NS. Effects of acutely increasing systemic vascular resistance on oxygen tension in tetralogy of Fallot. Pediatrics. 1976;58:248-251. 198. Reduto LA, Berger HJ, Johnstone DE, et al. Radionuclide assessment of right and left ventricular exercise reserve after total correction of tetralogy of Fallot. Am J Cardiol. 1980;45:1013-1018. 199. Bove EL, Byrum CJ, Thomas FD, et al. The influence of pulmonary insufficiency on ventricular function following repair of tetralogy of Fallot. Evaluation using radionuclide ventriculography. J Thorac Cardiovasc Surg. 1983;85:691-696. 200. Nollert G, Fischlein T, Bouterwek S, Bohmer C, Klinner W, Reichart B. Long-term survival in patients with repair of tetralogy of Fallot: 36-year follow-up of 490 survivors of the first year after surgical repair. J Am Coll Cardiol. 1997;30:1374-1383. 201. Carvalho JS, Shinebourne EA, Busst C, Rigby ML, Redington AN. Exercise capacity after complete repair of tetralogy of Fallot: deleterious effects of residual pulmonary regurgitation. Br Heart J. 1992;67:470-473. 202. Gatzoulis MA, Balaji S, Webber SA, et al. Risk factors for arrhythmia and sudden cardiac death late after repair of tetralogy of Fallot: a multicentre study. Lancet. 2000;356:975-981. 203. Hanley FL, Sade RM, Blackstone EH, Kirklin JW, Freedom RM, Nanda NC. Outcomes in neonatal pulmonary atresia with intact ventricular septum. A multiinstitutional study. J Thorac Cardiovasc Surg. 1993;105:406-423, 424-427; discussion 423-424. 204. Heck Jr HA, Schieken RM, Lauer RM, Doty DB. Conduit repair for complex congenital heart disease. Late follow-up. J Thorac Cardiovasc Surg. 1978;75:806-814. 205. Brown JW, Ruzmetov M, Rodefeld MD, Vijay P, Darragh RK. Valved bovine jugular vein conduits for right ventricular outflow tract reconstruction in children: an attractive alternative to pulmonary homograft. Ann Thorac Surg. 2006;82:909-916. 206. Fiore AC, Ruzmetov M, Huynh D, et al. Comparison of bovine jugular vein with pulmonary homograft conduits in children less than 2 years of age. Eur J Cardiothorac Surg. 2010;38:318-325. 207. Anyanwu E, Klemm C, Achatzy R, et al. Surgery of coarctation of the aorta: a nine-year review of 253 patients. Thorac Cardiovasc Surg. 1984;32:350-357. 208. McCrindle BW, Jones TK, Morrow WR, et al. Acute results of balloon angioplasty of native coarctation versus recurrent aortic obstruction are equivalent. Valvuloplasty and Angioplasty of Congenital Anomalies (VACA) Registry Investigators. J Am Coll Cardiol. 1996;28:1810-1817.





169.

totally anomalous pulmonary venous connection. J Am Coll Cardiol. 1993;22:201-206. Senning A. Surgical correction of transposition of the great vessels. Surgery. 1966;59:334-336. Hagler DJ, Ritter DG, Mair DD, Davis GD, McGoon DC. Clinical, angiographic, and hemodynamic assessment of late results after Mustard operation. Circulation. 1978;57:1214-1220. Castaneda AR, Norwood WI, Jonas RA, Colon SD, Sanders SP, Lang P. Transposition of the great arteries and intact ventricular septum: anatomical repair in the neonate. Ann Thorac Surg. 1984; 38:438-443. Jatene AD, Fontes VF, Souza LC, Paulista PP, Neto CA, Sousa JE. Anatomic correction of transposition of the great arteries. J Thorac Cardiovasc Surg. 1982;83:20-26. Di Donato RM, Fujii AM, Jonas RA, Castaneda AR. Age-dependent ventricular response to pressure overload. Considerations for the arterial switch operation. J Thorac Cardiovasc Surg. 1992;104:713-722. Wernovsky G, Giglia TM, Jonas RA, Mone SM, Colan SD, Wessel DL. Course in the intensive care unit after ‘preparatory’ pulmonary artery banding and aortopulmonary shunt placement for transposition of the great arteries with low left ventricular pressure. Circulation. 1992;86:II133-II139. Wernovsky G, Mayer Jr JE, Jonas RA, et al. Factors influencing early and late outcome of the arterial switch operation for transposition of the great arteries. J Thorac Cardiovasc Surg. 1995;109: 289-301. Khairy P, Clair M, Fernandes SM, et al. Cardiovascular outcomes after the arterial switch operation for D-transposition of the great arteries. Circulation. 2013;127:331-339. Day RW, Laks H, Drinkwater DC. The influence of coronary anatomy on the arterial switch operation in neonates. J Thorac Cardiovasc Surg. 1992;104:706-712. Wernovsky G, Sanders SP. Coronary artery anatomy and transposition of the great arteries. Coron Artery Dis. 1993;4:148-157. Rastelli GC, McGoon DC, Wallace RB. Anatomic correction of transposition of the great arteries with ventricular septal defect and subpulmonary stenosis. J Thorac Cardiovasc Surg. 1969;58:545-552. Helbing WA, Hansen B, Ottenkamp J, et al. Long-term results of atrial correction for transposition of the great arteries. Comparison of Mustard and Senning operations. J Thorac Cardiovasc Surg. 1994;108:363-372. Merlo M, de Tommasi SM, Brunelli F, et al. Long-term results after atrial correction of complete transposition of the great arteries. Ann Thorac Surg. 1991;51:227-231. Benson LN, Bonet J, McLaughlin P, et al. Assessment of right ventricular function during supine bicycle exercise after Mustard’s operation. Circulation. 1982;65:1052-1059. Gelatt M, Hamilton RM, McCrindle BW, et al. Arrhythmia and mortality after the Mustard procedure: a 30-year single-center experience. J Am Coll Cardiol. 1997;29:194-201. Tanel RE, Wernovsky G, Landzberg MJ, Perry SB, Burke RP. Coronary artery abnormalities detected at cardiac catheterization following the arterial switch operation for transposition of the great arteries. Am J Cardiol. 1995;76:153-157. Massin M, Hovels-Gurich H, Dabritz S, Messmer B, von Bernuth G. Results of the Bruce treadmill test in children after arterial switch operation for simple transposition of the great arteries. Am J Cardiol. 1998;81:56-60. Jenkins KJ, Hanley FL, Colan SD, Mayer Jr JE, Castaneda AR, Wernovsky G. Function of the anatomic pulmonary valve in the systemic circulation. Circulation. 1991;84:III173-III179. Rhodes LA, Wernovsky G, Keane JF, et al. Arrhythmias and intracardiac conduction after the arterial switch operation. J Thorac Cardiovasc Surg. 1995;109:303-310. Senzaki H, Ishido H, Iwamoto Y, et al. Sedation of hypercyanotic spells in a neonate with tetralogy of Fallot using dexmedetomidine. J Pediatr (Rio J). 2008;84:377-380.

e6

































222. Ohye RG, Sleeper LA, Mahony L, et al. Comparison of shunt types in the Norwood procedure for single-ventricle lesions. N Engl J Med. 2010;362:1980-1992. 223. Newburger JW, Sleeper LA, Frommelt PC, et al. Transplantationfree survival and interventions at 3 years in the single ventricle reconstruction trial. Circulation. 2014;129:2013-2020. 224. Bailey LL, Gundry SR, Razzouk AJ, Wang N, Sciolaro CM, Chiavarelli M. Bless the babies: one hundred fifteen late survivors of heart transplantation during the first year of life. The Loma Linda University Pediatric Heart Transplant Group. J Thorac Cardiovasc Surg. 1993;105:805-814; discussion 814-815. 225. Armitage JM, Fricker FJ, del Nido P, Starzl TE, Hardesty RL, Griffith BP. A decade (1982 to 1992) of pediatric cardiac transplantation and the impact of FK 506 immunosuppression. J Thorac Cardiovasc Surg. 1993;105:464-472; discussion 472-473. 226. Starnes VA, Griffin ML, Pitlick PT, et al. Current approach to hypoplastic left heart syndrome. Palliation, transplantation, or both? J Thorac Cardiovasc Surg. 1992;104:189-194; discussion 194-195. 227. Freedom RM. Neurodevelopmental outcome after the Fontan procedure in children with the hypoplastic left heart syndrome and other forms of single ventricle pathology: challenges unresolved. J Pediatr. 2000;137:602-604. 228. Wernovsky G, Stiles KM, Gauvreau K, et al. Cognitive development after the Fontan operation. Circulation. 2000;102:883-889. 229. Barron DJ, Kilby MD, Davies B, Wright JG, Jones TJ, Brawn WJ. Hypoplastic left heart syndrome. Lancet. 2009;374:551-564. 230. Galantowicz M, Cheatham JP. Lessons learned from the development of a new hybrid strategy for the management of hypoplastic left heart syndrome. Pediatr Cardiol. 2005;26:190-199. 231. Akinturk H, Michel-Behnke I, Valeske K, et al. Hybrid transcatheter-surgical palliation: basis for univentricular or biventricular repair: the Giessen experience. Pediatr Cardiol. 2007;28:79-87. 232. Pizarro C, Derby CD, Baffa JM, Murdison KA, Radtke WA. Improving the outcome of high-risk neonates with hypoplastic left heart syndrome: hybrid procedure or conventional surgical palliation? Eur J Cardiothorac Surg. 2008;33:613-618. 233. Galantowicz M, Cheatham JP, Phillips A, et al. Hybrid approach for hypoplastic left heart syndrome: intermediate results after the learning curve. Ann Thorac Surg. 2008;85:2063-2070; discussion 2070-2071. 234. Pizarro C, Murdison KA, Derby CD, Radtke W. Stage II reconstruction after hybrid palliation for high-risk patients with a single ventricle. Ann Thorac Surg. 2008;85:1382-1388. 235. Bacha EA, Daves S, Hardin J, et al. Single-ventricle palliation for high-risk neonates: the emergence of an alternative hybrid stage I strategy. J Thorac Cardiovasc Surg. 2006;131:163-171.e2.

































209. Ashburn DA, McCrindle BW, Tchervenkov CI, et al. Outcomes after the Norwood operation in neonates with critical aortic stenosis or aortic valve atresia. J Thorac Cardiovasc Surg. 2003;125: 1070-1082. 210. Kumar RK, Newburger JW, Gauvreau K, Kamenir SA, Hornberger LK. Comparison of outcome when hypoplastic left heart syndrome and transposition of the great arteries are diagnosed prenatally versus when diagnosis of these two conditions is made only postnatally. Am J Cardiol. 1999;83:1649-1653. 211. Iannettoni MD, Bove EL, Mosca RS, et al. Improving results with first-stage palliation for hypoplastic left heart syndrome. J Thorac Cardiovasc Surg. 1994;107:934-940. 212. Donner RM. Hypoplastic left heart syndrome. Curr Treat Options Cardiovasc Med. 2000;2:469-480. 213. Mahle WT, Spray TL, Wernovsky G, Gaynor JW, Clark BJ III. Survival after reconstructive surgery for hypoplastic left heart syndrome: A 15-year experience from a single institution. Circulation. 2000;102:III136-III141. 214. Mosca RS, Bove EL, Crowley DC, Sandhu SK, Schork MA, Kulik TJ. Hemodynamic characteristics of neonates following first stage palliation for hypoplastic left heart syndrome. Circulation. 1995;92:II267-II271. 215. Norwood WI, Lang P, Hansen DD. Physiologic repair of aortic atresia-hypoplastic left heart syndrome. N Engl J Med. 1983;308: 23-26. 216. Daebritz SH, Nollert GD, Zurakowski D, et al. Results of Norwood stage I operation: comparison of hypoplastic left heart syndrome with other malformations. J Thorac Cardiovasc Surg. 2000;119:358-367. 217. Tabbutt S, Ramamoorthy C, Montenegro LM, et al. Impact of inspired gas mixtures on preoperative infants with hypoplastic left heart syndrome during controlled ventilation. Circulation. 2001;104:I159-I164. 218. Bradley SM, Simsic JM, Atz AM. Hemodynamic effects of inspired carbon dioxide after the Norwood procedure. Ann Thorac Surg. 2001;72:2088-2093; discussion 2093-2094. 219. Hoffman GM, Tweddell JS, Ghanayem NS, et al. Alteration of the critical arteriovenous oxygen saturation relationship by sustained afterload reduction after the Norwood procedure. J Thorac Cardiovasc Surg. 2004;127:738-745. 220. Sano S, Ishino K, Kawada M, et al. Right ventricle-pulmonary artery shunt in first-stage palliation of hypoplastic left heart syndrome. J Thorac Cardiovasc Surg. 2003;126:504-509; discussion 509-510. 221. Sano S, Ishino K, Kawada M, Honjo O. Right ventricle-pulmonary artery shunt in first-stage palliation of hypoplastic left heart syndrome. Semin Thorac Cardiovasc Surg Pediatr Card Surg Annu. 2004;7:22-31.

e7

Abstract: The neonatal myocardium is less compliant than that of the older child, less tolerant of increases in afterload, and less responsive to increases in preload. A predictable decrease in cardiac index typically occurs 6 to 12 hours after separation from cardiopulmonary bypass, but milrinone administration during the early postoperative period may attenuate this phenomenon. Patients with postoperative low cardiac output require careful evaluation for unanticipated residual lesions. Patients with restrictive physiology from hypertrophy and diastolic dysfunction of the right ventricle may require high right-sided filling pressures to achieve adequate cardiac output, making them prone to hepatic congestion, anasarca, pleural effusions, and ascites. Inhaled nitric oxide plays an important role in the management of postoperative pulmonary hypertension in the cardiac intensive care unit. Hypoxemia after bidirectional cavopulmonary anastomosis generally is a sign of decreased pulmonary blood flow related to reduced cardiac output. Liberation from positive-pressure mechanical ventilation should be accomplished as soon as feasible, particularly in patients

after a cavopulmonary anastomosis (bidirectional Glenn) or Fontan operation because spontaneous breathing improves pulmonary blood flow, arterial oxygen saturation, and ventricular preload. Ventricular ectopy and elevated atrial pressures after the arterial switch operation should raise suspicion of myocardial ischemia from insufficient coronary blood flow. Postoperative care of the patient with hypoplastic left heart syndrome after stage I palliation (Norwood procedure) may require delicate balancing of the pulmonary and systemic blood flows. A high arterial oxygen saturation denotes excessive pulmonary blood flow and in patients with impaired ventricular output is generally accompanied by inadequate systemic blood flow, acidosis, and end-organ dysfunction. Key words: cavopulmonary shunt, hybrid procedure, Fontan procedure, Norwood procedure, interrupted aortic arch, coarctation of the aorta, hypoplastic left heart syndrome, transposition of the great arteries, single ventricle

10 37 ChapterTransplantation Cardiac Title CHAPTER SO NYA KIRMANI AUTHOR AND MICHAEL CARBONI

PEARLS • •

















Most pediatric transplantations are performedoffor cardiomyopTo gain basic knowledge of the development the eye. athy or congenital heart disease not how amenable to correction To develop essential understanding abnormalities at or palliation. various stages of development can arrest or hamper normal Individuals a good quality ofvisual life following heart formation ofmay thehave ocular structures and pathways. transplantation but are likely destined for a repeat transplant. ABO-incompatible heart transplantation is possible in young children and infants with outcomes comparable to ABO-compatible transplantation. Diastolic dysfunction may persist for several months following transplantation.

Background It has now been nearly 4 decades since the first successful pediatric heart transplantation was performed at Stanford University.1 Approximately 700 pediatric heart transplantations are now performed annually worldwide, with over 400 of those performed in the United States. A quarter of these transplantations are performed in children younger than 1 year. The remainder are split fairly evenly between children 1 to 10 years of age and adolescents younger than 18 years.2 The majority of infant transplantations are performed owing to congenital heart disease not amenable to correction or palliation, while the majority of adolescent transplantations are for cardiomyopathy. With improvements in immunosuppression and medical care, the overall transplant survival at 1 year has improved to over 90%, and we now expect 5-year survival to exceed 80%.2 From the newest survival data (International Society for Heart and Lung Transplantation 2018 Report2), the time at which 50% of recipients remain alive is 13.3 years for teenagers and 22.3 years for infants. Late death after transplantation is the result of posttransplant coronary vasculopathy, malignancy, or rejection due to nonadherence to immunosuppression regimens.2,3 Rehospitalization after the first year is rare, and quality of life has been excellent.4 Advances in our understanding of the immune system and improvements in critical care management of both donor and recipient patients have resulted in an increased survival benefit. Although overall perioperative mortality has greatly improved over the past 25 years, increased risk of mortality has been seen in patients with the diagnosis of congenital heart disease—especially infants—and those undergoing retransplant.2 One cause of early

• • • •









Right heartadequate support isinformation important after postcardiac transplantation. To acquire about normal anatomy of Immunosuppression should start following transthe eye and related structures andimmediately develop a strong foundation plantation. for the understanding of common ocular problems and their The high-risk period for acute allograft rejection is in the first consequences. month after transplantation. Complications of immunosuppression are infection, acute renal failure, malignancy, and hyperglycemia.

mortality is failure of the allograft to perform adequately, also known as primary graft dysfunction. Reasons for primary graft dysfunction include myocardial injury from inadequate organ preservation or long ischemic times, and elevated pulmonary vascular resistance (PVR) with pulmonary hypertension causing right ventricular failure. Acute allograft rejection is an exceedingly rare event immediately after transplantation. The physiology of the transplanted heart as well as preoperative and perioperative critical care play important roles in the successful transplantation of critically ill children with limited other options. This chapter reviews critical care management of the pediatric patient undergoing heart transplantation.

Indications for Transplant In 1995, the International Society for Heart and Lung Transplantation identified the following reasons as indications for pediatric heart transplantation5: • Need for ongoing intravenous (IV) inotropic or mechanical circulatory support • Complex congenital heart disease not amenable to conventional surgical repair or palliation, or for which the surgical procedure carried a higher risk of mortality than transplantation • Progressive deterioration of ventricular function or functional status despite optimal medical care with digitalis, diuretics, angiotensin-converting enzyme inhibitors, b-blockers, and/or other oral heart failure therapies • Malignant arrhythmia or survival of cardiac arrest unresponsive to medical treatment, catheter ablation, or an automatic implantable defibrillator







411

412

SECTION IV



Pediatric Critical Care: Cardiovascular

• Progressive pulmonary hypertension that could preclude cardiac transplantation at a later date • Growth failure secondary to severe congestive heart failure unresponsive to conventional medical treatment • Unacceptable poor quality of life secondary to heart failure





Transplant Evaluation Once a child is seriously considered for a heart transplant, an extensive evaluation is required to determine suitability for transplantation. This evaluation includes laboratory and imaging studies to elucidate the degree of heart failure and confirm that no further medical or surgical options are available for treatment as well as to investigate any potential contraindications to transplantation.6 Major contraindications to pediatric heart transplantation include: • Presence of any noncardiac condition that would significantly shorten life expectancy beyond what would otherwise be expected with cardiac transplantation. Examples include severe neuromuscular disease, active neoplasm, genetic disorders with poor prognosis, and severe immunodeficiency. • Presence of any noncardiac condition that may affect the transplanted heart and therefore also shorten life expectancy. Examples include significantly elevated and nonreactive PVR, mitochondrial disease, active infection, and morbid obesity (body mass index . 35). • Social factors—including unstable/unreliable caregiver system, history of recurrent medical noncompliance, and history of or current recreational drug use. In addition to these factors, relative contraindications include elevated but reactive PVR, history of malignancy, stroke, elevated panel reactive antibody (PRA), significant hepatic dysfunction, and/or significant renal insufficiency. In these cases, multiorgan transplantation might be considered. A typical transplant evaluation consists of laboratory and imaging studies, pulmonary function testing and exercise testing, cardiac catheterization, and various consultations. These consultations include social work, financial, nutrition, psychology, and indicated subspecialty consults. Box 37.1 summarizes this evaluation. The goal of laboratory evaluation, in addition to blood typing and PRA profile, is to identify infectious risk factors and overall end-organ and metabolic health. Imaging studies—including magnetic resonance imaging (MRI)/magnetic resonance angiography and/or computed tomography (CT) angiography—may assist surgical teams in planning the transplant procedure as well as assessing suitability of potential donors, particularly in children with complex congenital heart disease. A cardiac catheterization is generally indicated to determine cardiac indices as well as PVR and reactivity.



• BOX 37.1 Heart Transplant Evaluation Laboratory Testing • Chemistry • Complete metabolic profile with electrolytes, hepatic panel, uric acid, lactate dehydrogenase, and blood urea nitrogen and creatinine • Fasting lipid profile • Thyroid function testing, other metabolic evaluation as indicated • Urinalysis • Hematology • Complete blood count with differential • Coagulation panel • Blood typing and blood group antibody screening • Immunology • Panel of reactive antibody • Infectious disease • Viral testing for cytomegalovirus, Epstein-Barr virus, herpes, human immunodeficiency virus, human T- cell leukemia virus, varicella, toxoplasmosis, and other studies as indicated • Hepatitis panel Cardiac catheterization and other imaging as needed (magnetic resonance imaging, computed tomography) Exercise testing Psychosocial evaluation Financial consultation Other subspecialty consultations as required





























Panel Reactive Antibody The PRA is an immunologic test routinely performed to determine a recipient’s antibody response to human leukocyte antigens, a protein that coats human cells and is unique to each individual. Exposure to nonself human proteins results in production of antihuman leukocyte antigen (anti-HLA) antibodies, a phenomenon known as sensitization. Sensitization to HLA can occur with administration of blood products, implantation of nonself human tissue into the body, use of a ventricular assist device, or pregnancy. The former two often occur in children undergoing palliative or corrective surgery for congenital heart disease. Anti-HLA antibodies may also develop spontaneously.

HLAs are divided into two classes, class I and class II, and are specified by a letter (A, B, C, DR, DQ) and number code (e.g., A2, B8, C6) based on the molecular structure. The PRA test is performed by exposing a potential recipient’s serum to the lymphocytes of a panel of about 100 blood donors. Results are reported in percentage of positive reactions within the panel from 0% to 100%. Thus, if a recipient reacts to 50 of 100 samples, the PRA is 50%. This test is reported for class I and class II antigens separately. Current testing is predominately performed with Luminex beads, which allows for HLA specificities to be reported. In other words, a list of antigen codes producing a positive antibody reaction can be generated. These would be the undesirable antigens in a donor that could stimulate production of undesirable donor-specific antibodies (DSAs) and give a positive crossmatch. Knowing these antigens allows a virtual crossmatch to be performed comparing the donor HLAs with the list of anti-HLA antibodies of the recipient. A positive virtual crossmatch is likely to give a positive retrospective crossmatch if that heart were to be implanted. The presence of DSAs is associated with antibody-mediated rejection and can reduce the longevity of the cardiac allograft. A PRA of more than 10% has historically been associated with decreased posttransplant 1-year graft survival.2 Strategies exist to reduce antiHLA antibodies pretransplant, although variable in success, as well as strategies posttransplant to reduce the impact of DSAs, predominately with plasmapheresis. PRA in patients with repaired or palliated congenital heart disease can often be over 80%.

Transplant Listing Once all the components of the evaluation are complete, candidates are presented at the respective center’s listing conference. Key members of the listing conference include the transplant medical director, medical transplant team and transplant coordinators, transplant surgeons, social workers, and the center’s financial counselors. Each candidate is presented and all medical data



CHAPTER 37

• BOX 37.2 United Network for Organ Sharing

Allocation Status Criteria Status 1A













Candidate must meet one of the following: • Significant congenital heart disease diagnosis requiring infusion of multiple intravenous inotropes or a high dose of a single intravenous inotrope and admitted to the hospital that registered the candidate • Ductal-dependent pulmonary or systemic circulation with ductal dependency maintained by prostaglandin E infusion or stent and admitted to the transplant hospital that registered the candidate • Continuous mechanical ventilation and admitted to the hospital that registered the candidate • Requires assistance with a mechanical circulatory support device (e.g., extracorporeal membrane oxygenation, left ventricular assist device). Does not require hospitalization. • Requires assistance with an intraaortic balloon pump and admitted to the hospital that registered the candidate • Patient does not meet any of these specified criteria but has a suspected life expectancy of ,14 days without heart transplantation (e.g., refractory arrhythmia)

Status 1B





Candidate must meet one of the following: • Continuous infusion of one or more inotropes and does not qualify for pediatric status 1A. Does not require hospitalization. • Less than 1 year of age at the time of initial listing and has restrictive or hypertrophic cardiomyopathy

Status 2

• Individuals not meeting pediatric status 1A or 1B criteria

Status 7

• Listed but inactive. May be too well or too ill for transplant, or may have other patient/center-specific issues precluding transplant

are reviewed, along with recommendations from all consultants. At that time, a determination is made as to whether or not it is appropriate to list a candidate for transplant; if a candidate is found to be unsuitable, the team should determine what steps, if any, could be taken to make the candidate more suitable. If a candidate is suitable for listing, as of 2020, there are currently three active listing statuses in which to place candidates and one inactive status. Box 37.2 summarizes the United Network for Organ Sharing (UNOS) status allocation for pediatric heart transplantation. Wait list times vary from region to region for each listing status and vary greatly by recipient age. When an organ offer is received, the medical and surgical members of the transplant team review the offer for suitability for the intended recipient. Several factors require careful consideration, including size match, antibody profile, infectious profile, and quality of the donor heart, among others.

Management of the Potential Heart Transplant Recipient The pretransplant management of the critically ill patient with end-stage myocardial dysfunction can determine the outcome of that patient after thoracic transplant. In general, the more comorbidities a patient has pretransplant, the longer and more complicated their posttransplant clinical course may be. Many pediatric heart transplant candidates can be managed in the outpatient setting on an oral heart failure regimen with close follow-up. These

Cardiac Transplantation

413

patients may be on a combination of angiotensin-converting enzyme inhibitors, b-blockers, potassium-sparing agents (i.e., spironolactone), and diuretics. However, once a patient becomes symptomatic at rest, inpatient heart failure treatment is indicated. The principles of inotropic support, preservation of end-organ function, and attention to issues of nutrition and infection are the same as those for all other critically ill patients in the pediatric intensive care unit (ICU). In addition, careful attention must be paid to the presence and management of dysrhythmias. Critically ill children with myopathic ventricular dysfunction severe enough for them to be in the ICU generally require inotropic support. These agents increase contractility through a common pathway of increasing intracellular levels of cyclic adenylate monophosphate (cAMP). Increased cytoplasmic levels of cAMP cause increased release of calcium from the sarcoplasmic reticulum and increase contractile force generation. Increases in cAMP occur either by b-adrenergic–mediated stimulation (increased production) or phosphodiesterase 3 (PDE3) inhibition (decreased degradation). Milrinone has been well studied in the pediatric population and proven to be well tolerated. IV administration of milrinone increases cardiac output and reduces cardiac filling pressures and PVR and systemic vascular resistance (SVR) with minimal effect on heart rate.7,8 The increase in cardiac output is primarily due to its effect on PVR and SVR rather than a direct inotropic effect on the myocardium. Milrinone is generally initiated at doses of 0.25 mg/kg per minute and may be increased to 1 mg/kg per minute without significant adverse effects. This drug is primarily excreted in the urine; thus, serum concentrations can increase in the presence of renal insufficiency and dose adjustments should be made. Although atrial and ventricular ectopy are less common with milrinone than with other inotropes, ventricular arrhythmias can occur with the initiation of milrinone therapy, especially in the presence of renal dysfunction. Milrinone has a long half-life and should be used cautiously in patients with hypotension. The addition of low-dose epinephrine (0.01–0.05 mg/kg per minute), dopamine (3–5 mg/kg per minute), or dobutamine (2.5–5 mg/kg per minute) can help stabilize the critically ill child who is not responding adequately to milrinone therapy alone or who needs blood pressure augmentation prior to initiating milrinone therapy. Tachyphylaxis is unusual with milrinone. Milrinone effects may persist for several hours or days after discontinuation of a prolonged infusion. It is important to note that wait list times may be long even for the sickest children at the highest status awaiting a transplant, sometimes several months. In fact, pediatric wait list mortality overall is about 15% and can be significantly higher in select populations, such as those with congenital heart disease.9 Most patients waiting for heart transplantation who are on inotropic support do not remain hemodynamically stable indefinitely. Progressive end-organ dysfunction eventually ensues, requiring escalation of support that includes multiple inotropic agents in addition to respiratory and circulatory support. Mechanical ventilatory support may be needed to manage respiratory failure, control the effects of pulmonary edema, and reduce metabolic demand. Use of mechanical ventilation is a risk factor for wait list mortality and reduced survival following pediatric heart transplantation. It is currently a criterion for 1A status. Mechanical circulatory support has become an important addition to the treatment armamentarium for the infant or child with decompensated heart failure and low cardiac output unresponsive to pharmacologic maneuvers. Options include extracorporeal membrane oxygenation (ECMO), intraaortic balloon, and left

414

SECTION IV



Pediatric Critical Care: Cardiovascular

ventricular and right ventricular assist devices. Experience with ECMO as a bridge to heart transplantation has been reported by several pediatric transplant centers.10,11 ECMO support generally can be used for 2 to 3 weeks without major complications from bleeding or infection, extending the window for donor organ availability. However, ECMO use increases the risk of wait list mortality and is associated with reduced posttransplant survival.2,9 Isolated ventricular support devices, such as the Thoratec, Berlin Heart, HeartWare, and Heartmate pumps, are also available for use in some children.12–15 However, the Berlin Heart EXCOR is the only device approved by the US Food and Drug Administration for use in children. Placement of a device should be considered for signs and symptoms of persistent heart failure refractory to inotropic support and/or with concern for multiorgan dysfunction. Placement of a device in a patient with multisystem organ failure usually results in a poor outcome. We propose that these devices be placed early, before end-organ dysfunction. Doing so enables rehabilitation of the patient, who then becomes a more optimal candidate for organ transplantation. The use of a ventricular assist device has been associated with improved wait list mortality and posttransplant outcomes.2,9,16

Anticoagulation All patients with severe myocardial dysfunction are at risk for complications of systemic and pulmonary emboli. While there are no established guidelines for thrombosis prophylaxis in pediatric patients, most centers consider antiplatelet therapy with aspirin and/or anticoagulation with heparin or warfarin. Low-molecularweight heparin is generally not used, as it cannot be easily reversed in a patient who must go to the operating room urgently once a donor heart has been identified.

ABO-Incompatible Listing and Transplantation At present, it is possible to perform an ABO-incompatible (ABOi) heart transplantation without risk of hyperacute rejection if performed prior to maturation of the immune system and development of isohemagglutinins, generally under 12 months of age and possibly under 24 months.17 ABOi posttransplant outcomes have been shown not to be statistically different from ABO-compatible transplantation outcomes, and there has also been some improvement in wait list outcomes.18–20 The UNOS allows listing of patients 1 to 2 years of age as eligible for ABOi transplantation with an A or B isohemagglutinin titer less than 1:16 and no limit on titer for those younger than 1 year. Most centers now have protocols regarding pre- and posttransplant care for individuals listed as eligible for and undergoing an ABOi transplantation. Predominately, the management centers on avoiding sensitization events related to administration of blood products and strict adherence to blood-typing protocol when blood products are needed.

Critical Care Management of the Orthotopic Heart Transplant Recipient Intraoperative Considerations Posttransplant cardiac function is affected by multiple factors, such as recipient pretransplant characteristics, donor characteristics and management, preservation techniques, and total ischemic

time of the donor heart. Total ischemic time is defined as the total time starting from placement of the cross-clamp on the donor aorta to release of the recipient cross-clamp and is a major factor in early posttransplant cardiac function and transplant outcomes. Total ischemic times of less than 4 hours typically are associated with better graft function and posttransplant outcomes.2 Because of this, a great deal of planning goes into keeping ischemic time as low as possible. This includes planning for repeat sternotomies or performing any reconstruction that needs to be done prior to donor heart implantation, considering the logistics and travel times to donor site, and using implant techniques that reduce ischemic time. Techniques of implantation have not changed significantly since the original description.1,21 Although modifications may be necessary in transplantation for complex congenital heart disease patients, heart transplantation can be accomplished in virtually any congenital heart anomaly because the anatomic position of the aorta, pulmonary arteries, and left atrium are in a relatively constant position near the midline. Regardless of malposition or positional relationships of the great arteries, the aorta of the recipient can be mobilized to make anastomosis with the donor aorta possible. The left atrium is a midline structure; even when anomalies of pulmonary venous return exist, pulmonary veins usually approach the midline and can be incorporated into the repair.22 However, aortic root size mismatch can cause technical difficulty. Implantation of recipients with complex congenital heart disease can usually be accomplished by harvesting additional donor pulmonary artery, aorta, and caval tissue to replace deficient recipient tissue or to correct malposition of the vena cava or great arteries.22,23 There are generally two standard implantation techniques, classified by use of either a biatrial or bicaval anastomosis. In both techniques, left atrial tissue surrounding the recipient pulmonary veins is anastomosed to the donor left atrium. This is done to reduce implantation time and to avoid individual pulmonary vein anastomoses and subsequent stenosis. The traditional biatrial technique uses a relatively large cuff of recipient right and left atrial tissue anastomosed to a portion of both donor atria. This results in the presence of large atrial suture lines that can be a source for arrhythmia; possible injury to the sinus node, causing bradycardia; and atrial dilation resulting in alterations of atrial contractility, tricuspid valve function, and flow dynamics. This technique is primarily now used for infants and small children to avoid end-to-end anastomoses with small venae cavae or for patients with venous anomalies in which using the recipient atria is beneficial to the transplant. The newer bicaval technique removes the recipient right atrium and a large portion of the left atrium with preservation of the superior and inferior venae cavae and a cuff of tissue around the pulmonary veins.24,25 End-to-end anastomoses of recipient and donor venae cavae are then used. This technique has the advantage of preserving sinus node function and atrial geometry. Besides the issues discussed earlier, other intraoperative considerations include the administration of immunosuppressants during or immediately after implantation, delivery of inhaled nitric oxide (iNO) to support the donor right ventricle following implantation, and intraoperative plasmapheresis for any HLA antibody concerns. In the event of acute graft dysfunction and inability to separate from bypass, or for any other concerns about graft function, the use of temporary mechanical circulatory support should be considered.



CHAPTER 37

Early Perioperative Management The early perioperative management of the recipient is not significantly different from the management of any postcardiac surgical patient. The physiologic responses of the newly transplanted heart are altered because of denervation. The major changes related to autonomic denervation include diastolic dysfunction and exaggerated response to exogenously administered catecholamines. The transplanted heart must also adapt to a new environment related to recipient lung function and elevated PVR. Autonomic system denervation results in a relatively fixed heart rate without respiratory variation. Heart rates are between 90 and 110 beats per minute but can be faster because of exogenous catecholamine administration. Heart rates can also be slower if the recipient has been exposed to amiodarone before transplantation or if there was injury to the donor sinus node due to cold preservation or to disruption of blood supply. Temporary pacing or isoproterenol infusion may be needed to support heart rate and cardiac output during the postoperative period. Permanent pacing is rarely necessary. Early blood pressure instability is common because of loss of baroreceptor regulation and dependence of the transplanted heart on endogenous or exogenous catecholamines. Hypotension can have multiple causes, most related to the usual postcardiac bypass surgery issues. In addition, hypotension can be seen with primary graft dysfunction (discussed more later) and with vasoplegia in patients with a history of prolonged milrinone or amiodarone use. Hypertension in the perioperative period has multiple causes. An increased stroke volume from the healthier transplanted heart into a systemic vascular bed that is abnormal because of a longstanding increase in SVR secondary to chronic heart failure is one such cause. Donor hearts are also often oversized, resulting in a larger stroke volume. This larger stroke volume can contribute to hypertension and to hyperperfusion syndrome, a condition in which an acute increase in cardiac output increases cerebral blood flow, resulting in cerebral vasoconstriction, symptomatic seizures, headache, or changes in mental status in a patient with a previously low cardiac output.26 The oversized donor heart eventually undergoes remodeling with regression of hypertrophy. The use of steroids for immunosuppression is an additional cause of posttransplant hypertension. Any symptomatic hypertension should be treated aggressively early in the perioperative period, but hypotension should be avoided in order to reduce the risk of renal dysfunction. Systolic and/or diastolic dysfunction of the transplanted heart are often seen in the early postoperative period. The right ventricle is especially at risk owing to its susceptibility to preservation injury with thinner myocardium and increased myocardial stress from elevated PVR. Infusions of b-adrenergic agents for several days are often necessary to maintain optimal heart allograft function, as early myocardial function of the transplanted heart is dependent on catecholamine support.27 Milrinone, as well as other agents that improve contractility or decrease afterload, may be advantageous to support right ventricular function. The hemodynamics of the transplanted heart reflect a significant shift to the left of the pressure/volume curve such that small increases in preload can result in significant elevations in ventricular end-diastolic and atrial pressures. Diastolic dysfunction is a significant impediment to early allograft function, limiting cardiac output. Some degree of diastolic dysfunction can persist well into the recovery phase and even several months after transplantation, with biopsy specimens often displaying evidence of persistent preservation injury.

Cardiac Transplantation

415

Management of Early Heart Allograft Dysfunction Early allograft dysfunction may be related to unsuspected injury prior to organ procurement or because of preservation injury. The other major cause of primary allograft failure is elevated PVR in the recipient. Allograft failure rarely is caused by acute antibody-mediated injury. Donor risk factors for allograft dysfunction include “down time of the donor” (length of initial resuscitation), evidence of myocardial injury with elevation of troponin or ventricular dysfunction on echocardiogram, and use of high-dose inotropic support (dopamine/dobutamine .20 mg/kg per minute or epinephrine/norepinephrine .0.1 mg/kg per minute) in the donor. Abnormal heart function seen in the midst of the catecholamine storm of brain death can recover. Repeating the echocardiogram remote from the initial resuscitation period or after the reduction or discontinuation of inotropic support can increase confidence in accepting the donor heart. The other major reason for primary donor heart dysfunction is right heart failure from high PVR. We have known since the early days of heart transplantation that the donor right ventricle will not function well when exposed to an abnormal pulmonary circulation. High PVR in the recipient increases perioperative morbidity and mortality and can affect late survival. Potential heart recipients generally undergo cardiac catheterization before heart transplantation to document the anatomy of systemic and pulmonary venous connections, determine pulmonary artery size and distribution, and calculate PVR. The upper limit of PVR associated with successful orthotopic heart transplantation is not known. Criteria developed from the adult heart transplant experience indicate that a PVR greater than 6 Wood units or a transpulmonary gradient (pulmonary artery mean pressure minus left atrial mean pressure) greater than 15 mm Hg is associated with increased perioperative mortality. The transpulmonary gradient is the most useful surrogate for PVR because estimation of cardiac output in the catheterization laboratory can be flawed if using the Fick equation. In children, the PVR index (PVRI), determined by dividing the transpulmonary gradient by the cardiac index, is more useful, because children come in all sizes. A PVRI less than 6 index units is associated with low perioperative mortality. Orthotopic heart transplants have been successful with a PVRI greater than 6 and as high as 10 index units but with increased morbidity and mortality rates. The diagnosis of high PVR is evident as the recipient is weaned from cardiopulmonary bypass. Intraoperative transesophageal echocardiography demonstrates dilation of the right ventricle and a small, underfilled left heart. Acute management of high PVR and right heart dysfunction includes ventilation with a high fraction of inspired oxygen and administration of iNO at 20 to 40 ppm. The need for continuous pulmonary vasodilator medications in the immediate perioperative period is unusual, but prostacyclin and sildenafil have both proved effective in this situation.28

Immunosuppression and Heart Allograft Rejection It is imperative that immunosuppression be initiated early after heart transplantation. Solid-organ transplants transfer antigenpresenting cells (APCs) that are recognized as foreign by the recipient’s HLA immune system, which sets up a cascade of lymphocyte stimulation and proliferation. These lymphocytes then migrate to the heart allograft, where they can adhere to myocytes and endothelial receptors and cause tissue destruction. T-cell activation is the prime promoter of allograft rejection.

416

SECTION IV



Pediatric Critical Care: Cardiovascular

The initial signal is T-cell receptor binding of antigen on the surface of an APC. The APC is derived from the donor in the form of a monocyte, or a tissue macrophage. Interaction of the APC and T-cell receptor causes release of interleukin (IL)-1 from the APC, which activates the T cell. Activated T cells secrete IL-2 and other lymphokines that induce proliferation of activated T cells, which migrate to the allograft and cause tissue damage.

Immunosuppression protocols are similar from center to center with generally only small variations. Initial immunosuppression protocols typically include high-dose corticosteroids, induction with IL-2 receptor blockade (e.g., basiliximab) or antithymocyte globulin (ATG), followed within approximately 48 hours by introduction of a calcineurin inhibitor (CNI), such as cyclosporine or tacrolimus (Table 37.1).

TABLE Immunosuppression in the Intensive Care Unit 37.1

Agent

Mechanism of Action

Dose

Monitoring

Major Side Effect(s)

Induction Immunosuppression Corticosteroids

Redistribution of peripheral lymphocytes, inhibition of lymphokine IL-2 production, impairment of macrophage response to lymphocyte signals

Center specific: Generally high-dose steroids followed by taper

Glucose

Infection, cushingoid appearance, hypertension, hyperlipidemia, glucose intolerance

Basiliximab

Monoclonal antibody binds to IL-2 receptor

20 mg .35 kg 10 mg ,35 kg, administer days 1 and 4

CBC

Anaphylaxis

Antithymocyte globulin Nonspecific T-cell lysis

1.5 mg/kg/day for 3–7 days (1/2 dose for platelet count 50,000–75,000, WBC 2000– 3000; hold for platelets ,50,000, WBC ,2,000)

T-lymphocyte subsets, platelets

Thrombocytopenia, anaphylaxis, infection, PTLD, localized pain with RATG administration, serum sickness

Cyclosporine

Calcineurin inhibitor, inhibition of T-cell receptor lymphokine production and T-cell proliferation

IV: 2.5-5 mg/kg/day Oral: 5–10 mg/kg/day divided q12h (may need q8h in infants)

Monoclonal whole blood assay 100–400 ng/mL, depending on time since transplantation

Nephrotoxicity, central nervous system seizures, decreased magnesium, hypertension, hirsutism, gingival hyperplasia

Tacrolimus

Calcineurin inhibitor, inhibition of T-cell receptor lymphokine production and T-cell proliferation

Oral: 0.5–1.0 mg (or 0.1–0.3 mg/ kg) q12h and increase the dose to therapeutic level

10–15 ng/mL, whole blood

Nephrotoxicity, anemia/ neutropenia, headache, tremors, insomnia, glucose intolerance

Maintenance Immunosuppression Cyclosporine

IV: 2.5-5 mg/kg/day Oral: 5–10 mg/kg/day divided q12h (may need q8h in infants)

Tacrolimus

Oral: 0.5–1.0 mg (or 0.1–0.3 mg/ kg) q12h and increase the dose to therapeutic level

Sirolimus

Inhibition of T-cell activation and proliferation by preventing translation of mRNA

1 mg/m2 daily

Triglycerides, platelets, level 5–10 mg/mL

Nephrotoxicity, hyperlipidemia, thrombocytopenia, leukopenia, gastrointestinal intolerance, mouth sores

Azathioprine

Antimetabolite inhibits purine and DNA synthesis

1–2 mg/kg/day

WBC ,4000 ANC .1500

Bone marrow suppression

Mycophenolate mofetil Inhibits proliferation of T and B cells

Mycophenolate mofetil or mycophenolic acid 30–60 mg/kg/ day or 300–600 mg/m2/day

WBC ,4000 ANC .1500

Bone marrow suppression, gastrointestinal intolerance

Prednisone

Variable

Acute Cellular Rejection Methylprednisolone Antithymocyte globulin

1.5 mg/kg/day for 5–7 days

Peripheral lymphocyte count, platelet count

ANC, Absolute neutrophil count; CBC, complete blood cell count; IL-2, interleukin-2; IV, intravenous; mRNA, messenger ribonucleic acid; PTLD, posttransplant lymphoproliferative disease; RATG, rabbit antithymocyte globulin; WBC, whole blood cell count.



CHAPTER 37

Induction protocols using IL-2 receptor antagonists or ATG are effective in delaying the time to first allograft rejection episode and the time needed to initiate CNI medications, which is especially useful when there is significant renal dysfunction. Induction therapy also reduces the risk of death due to rejection, although it does not appear to have a long-term survival benefit, except possibly in those with a PRA greater than 50% or diagnosis of congenital heart disease.2,29–31 Induction therapy may also be useful in steroid avoidance protocols.32 Some centers do not use induction therapy in particular circumstances owing to concerns for risk of infection or viral reactivation, although that has not been borne out in recent studies.33 Corticosteroids have been part of standard protocols since the early days of solid-organ transplantation. High-dose methylprednisolone (5–10 mg/kg) is administered at the time of aortic crossclamp removal and continued in tapering doses over the first several days after surgery. Corticosteroids have immunosuppressive properties and benefit the allograft because of membranestabilizing and antioxidant effects on the graft. Steroid-sparing/ steroid-avoidance protocols exist for other solid-organ transplants and are in development for heart transplantation.32 More controversial is the timing of the introduction of the CNIs cyclosporine and tacrolimus. A major complication in the early perioperative course after heart transplantation is renal dysfunction; in the past, CNIs were major contributors. The APC and lymphocyte receptor interaction occurs within hours of the transplant; therefore, early introduction of CNIs is important. Because oral bioavailability of these drugs is so variable, continuous IV administration of these drugs is sometimes more desirable. However, IV administration of these drugs can be associated with acute renal failure in the posttransplant setting. Because of these factors, there is variation in CNI protocols among centers. Some, for instance, use IV cyclosporine until oral tacrolimus can be delivered reliably. Others wait until an oral CNI can be started. Most protocols have some delay in starting a CNI until renal function is stable, depending on whether induction therapy was used. Most protocols are based on oral/nasogastric administration of a standard dose of tacrolimus within 48 hours of transplantation. Target levels of this drug are reached 3 to 5 days after transplantation if subsequent doses are based on the trough level obtained each morning (see Table 37.1). The high-risk period for acute cellular rejection (ACR) is the first month after transplantation. ACR is a phenomenon that rarely occurs in the first week after transplantation. Hyperacute rejection is uncommon but can occur when a heart transplant recipient has preformed HLA antibody that reacts with a donor who has those specific HLA antigens. A positive crossmatch will be reported at the time of transplantation, which means that the recipient’s serum causes lysis of donor T cells obtained from lymph nodes from the donor at the time of organ procurement (in the case of cytotoxic testing) or that there is binding of recipient immunoglobulin G after exposure to donor lymphocytes (in the case of flow cytometry crossmatch). Heart transplant recipients at risk for hyperacute rejection are identified by measuring the presence of HLA antibody in their serum pretransplant. Specificity and quantification of these HLA antibodies can be measured by Luminex beads, which enable a virtual crossmatch (comparison of recipient HLA antibodies to donor HLA antigens) to be done between potential donors and recipients at the time of an offer. Children with palliated congenital heart disease are at particular risk for HLA sensitization because of exposure to blood products at the time of their previous surgical procedures. Institution of plasmapheresis immediately after implantation and

Cardiac Transplantation

417

continuing through the first several days after the operation is the preferred way to clear the offending antibody causing heart allograft dysfunction. The diagnosis of ACR is made by endomyocardial biopsy. Histopathology of cardiac tissue obtained by endomyocardial biopsy remains the gold standard for diagnosis of acute cardiac allograft rejection. The amount of infiltrating lymphocytes and the presence of myocyte injury are used to grade rejection (International Society for Heart and Lung Transplantation Grade 0R-3R) and to guide allograft rejection therapy. Surveillance endomyocardial biopsies are generally performed within the first 2 weeks after transplantation and then at strategic times depending on age and size of the child, available access, and technical difficulty of obtaining tissue. Follow-up protocols vary by center. Clinical recognition of acute allograft rejection can be subtle but is obviously important because tissue diagnosis is not always possible and surveillance techniques using peripheral blood, electrocardiography, and echocardiography have limitations. ACR can be present in the allograft without any symptoms or clinical findings. When ACR has progressed to hemodynamically significant allograft dysfunction, then symptoms of abdominal pain and vomiting are prevalent, and findings of systemic venous congestion, liver enlargement, and low cardiac output predominate. Symptoms of pulmonary venous congestion/pulmonary edema are rare findings. Graft dysfunction can be severe enough that cardiogenic shock is present, requiring inotropes or mechanical circulatory support. When ACR is suspected, histologic confirmation is always desirable if it can be safely performed. The principles of management are to acutely augment immunosuppression with methylprednisolone or a lympholytic agent depending on the histologic and clinical severity of the heart allograft dysfunction. Following acute treatment, increases in maintenance of immunosuppression agents are prescribed and a follow-up endomyocardial biopsy is scheduled. In addition to ACR, humoral or antibody-mediated rejection (AMR) may also present in similar fashion. Acute AMR is treated as in ACR, with the addition of plasmapheresis to reduce the presence of any DSA. In severe cases of graft dysfunction, therapy may be initiated prior to biopsy or results of antibody testing.

Complications of Immunosuppression in Heart Transplant Recipients That Occur in the Pediatric Intensive Care Unit Infection Infections are a major cause of mortality and morbidity in the early period after heart transplantation.2,34 Factors that predispose to infection can be divided into preexisting factors related to the donor and recipient and factors secondary to events in the intraoperative and postoperative periods. For example, the site of the organ transplanted provides a clue to the site of infection. Renal transplant recipients acquire urinary tract infections, whereas heart transplant recipients are exposed to chest cavity infections. The type and severity of the underlying illness leading to organ failure can increase the risk for rejection. Children with cardiomyopathy can be severely malnourished, require prolonged mechanical respiratory or circulatory support, and have chronic indwelling venous catheters, all of which predispose to infection. Pretransplant ventricular assist devices or ECMO can also increase risk of infection at cannulation and driveline sites. The

418

SECTION IV



Pediatric Critical Care: Cardiovascular

presence of a pretransplant pulmonary infarction is associated with lung abscess in the posttransplant recovery period.35 Neonates may experience severe sepsis from coagulase-positive staphylococci more often than older children. The herpes virus family plays a significant role in infections occurring after transplantation. The clinical expression of cytomegalovirus (CMV) and Epstein-Barr virus infection in the young patient is more severe because it is often a primary exposure.36 Clinical infections related to these viruses rarely present before 1 month following organ transplantation and are most common in the first 6 months after heart transplantation. However, the presence of a CMV mismatch (donor and recipient CMV status differ) can place the recipient at risk of significant CMV illness and requires prophylactic therapy with ganciclovir/ valganciclovir. Herpes simplex virus (HSV) can also be an issue posttransplant in the presence of immunosuppression. Currently, the donor’s HSV status is usually not known and recommendation is for universal prophylaxis with acyclovir/valaciclovir or ganciclovir/valganciclovir. Antibiotic management of the heart transplant recipient in the ICU can be guided by clinical suspicion, timing of infection after transplantation, and predisposing factors. Immunosuppression is selective and targets T cells. Neutrophil function is normal except for the effect of high-dose corticosteroids. Neutropenia can occasionally be a problem because of bone marrow suppression caused by antimetabolites and tacrolimus. Prophylactic antibiotics, in the form of third-generation cephalosporins, are used for patients after sternotomy and generally are continued until chest tubes and central lines are removed. The strategy to prevent infection also includes initial isolation, routine surveillance cultures, and regular replacement of indwelling catheters. In the early setting after transplantation, temperature elevation should indicate active infection and serious complication until proven otherwise. If an infection is suspected, early and aggressive investigation is necessary and broad-spectrum antibiotics/antifungal agents should be initiated until the source of the fever is identified.

Renal Function Acute kidney injury is a major complication following orthotopic heart transplantation. It is often multifactorial in etiology, as the premorbid risk factors of heart transplant recipients cannot necessarily be controlled. One must monitor and control use of CNI immunosuppression agents, especially when renal dysfunction is present or expected. Therapeutic strategies include delaying the initiation of cyclosporine and tacrolimus by using ATG or IL-2 receptor blockade for induction of immunosuppression. The other option is to use a modified oral/nasogastric protocol for tacrolimus administration.37 This protocol targets tacrolimus levels to below 6 ng/mL in the first 3 days after transplantation, followed by rapid increases in dosing and target level over the next 4 days. It is important to avoid early IV administration of these agents because they invariably lead to renal afferent arteriolar vasoconstriction and oliguria. If renal dysfunction is complicating the posttransplant course, it is still difficult to withdraw CNIs completely, but lowering the target level to ,6 ng/L and substituting higher doses of mycophenolate mofetil and adding sirolimus are reasonable options.38 Other medications, such as diuretics and CMV prophylaxis, can contribute to posttransplant renal dysfunction. Often, all of these medications are started at approximately the same time, thus increasing the risk for renal dysfunction.

Diabetes Mellitus Hyperglycemia is common after heart transplantation with highdose steroid and tacrolimus-based immunosuppression. The combination of decreased insulin production from islet cells caused by tacrolimus and decreased peripheral utilization related to highdose corticosteroids results in nonketotic hyperglycemia. Insulin therapy is commonly used early posttransplant until the point at which steroid dosing can be reduced and tacrolimus adjusted.

Future Management Strategies for Critical Care of Infants and Children With Cardiopulmonary Failure Heart transplantation in children has gained wide acceptance as an important adjunct to treatment of children with end-stage cardiomyopathic function from cardiomyopathy and palliated congenital heart disease. Successful transplantation has produced greater longevity and better quality of life for many infants and children. Ten-year survival free of malignancy and coronary vasculopathy is now the expected outcome.2 Although the future appears to be moving toward fewer transplant procedures in children owing to better surgical options for patients with congenital heart disease, the total number of transplants yearly continues to grow. There are many possible reasons for this, including better wait list survival to transplant, more retransplants, and increased numbers of children unable to complete intended staged palliations. Also, children who in the past were failing palliation and were not transplant candidates owing to their high risk are now undergoing transplants with reasonable results. The use of ventricular assist devices has also allowed patients to get to transplant that may not have in the past. The natural history of cardiomyopathy is changing because of our understanding of the cellular mechanisms of myocardial function. New treatment strategies using angiotensin receptor blockers, neprilysin inhibitors, and b-adrenergic blockade therapy are delaying or replacing the need for heart transplantation. Circulatory support is being miniaturized with the development of the extracorporeal Berlin Heart and with ongoing trials of implantable pediatric ventricular assist devices. Current adult devices continue to be used in ever smaller pediatric patients. These devices can cause reversed ventricular modeling, allowing discontinuation of support without heart replacement therapy (bridge to recovery). The other major benefit of circulatory support is that, if initiated early, it can rehabilitate the child, recover end-organ function, and reduce the risks of heart transplantation surgery. Cardiac failure and ventricular assist devices are covered in greater detail in Chapter 28. Other strategies are under investigation that would expand the donor pool and potentially reduce wait times and wait list mortality. These include novel methods of organ transport to allow continued perfusion of the beating heart. This allows procurement of hearts from greater distances and allows for observation of a heart that may be borderline for use in transplantation. In addition, heart procurement in donation after circulatory death is an approach that may allow for an expanded donor pool. As technology continues to advance, so do the possibilities for in vitro growth of a complete organ or partial replacement of a diseased heart with healthy tissue.



CHAPTER 37

Key References Almond CS, Morales DL, Blackstone EH, et al. Berlin Heart EXCOR pediatric ventricular assist device for bridge to heart transplantation in US children. Circulation. 2013;127:1702-1711. Butts R, Davis M, Savage A, et al. Effect of induction therapy on graft survival in primary pediatric heart transplantation: a propensity score analysis of the UNOS database. Transplantation. 2017;101(6): 1228-1233. Del Nido PJ, Bailey L, Kirklin JK. Surgical techniques in pediatric heart transplantation. In: Canter CE, Kirklin JK, eds. ISHLT Monograph Series: Pediatric Heart Transplantation. Vol 2. Philadelphia: Elsevier; 2007:83-102. Green M, Michaels MG. Infections in pediatric solid organ transplant recipients. J Pediatric Infect Dis Soc. 2012;1:144-151. Hoffman TM, Wernovsky G, Atz AM, et al. Prophylactic intravenous use of milrinone after cardiac operations in pediatrics (PRIMACORP) study. Am Heart J. 2002;143:15. Kirshborn PM, Bridges ND, Myung RJ, et al. Use of extracorporeal membrane oxygenation in pediatric thoracic organ transplantation. J Thorac Cardiovasc Surg. 2002;1223:130.

Cardiac Transplantation

419

Rossano JW, Cherikh WS, Chambers DC, et al. The International Thoracic Organ Transplant Registry of the International Society for Heart and Lung Transplantation: Twenty-first pediatric heart transplantation report; Focus them: multiorgan transplantation. J Heart Lung Transplant. 2018;37:1184-1195. Rossano JW, Lorts A, VanderPluym CJ, et al. Outcomes of pediatric patients supported with continuous-flow ventricular assist devices: a report from the Pediatric Interagency Registry for Mechanical Circulatory Support (PediMACS). J Heart Lung Transplant. 2016;35(5): 585-590. Wessel DL. Managing low cardiac output syndrome after congenital heart surgery. Crit Care Med. 2001;29:S220. West LJ, Pollock-Barziv SM, Dipchand, AI, et al. ABO-incompatible (ABOi) heart transplantation in infants. N Engl J Med. 2001;344: 793-800.

The full reference list for this chapter is available at ExpertConsult.com.

e1

























































































1. Baum D, Stinson EB, Shumway NE. The place for heart transplantation in children. In: Godman MJ, ed. Pediatric Cardiology. Vol 4. London: Churchill Livingstone; 1981. 2. Rossano JW, Cherikh WS, Chambers DC, et al. The International Thoracic Organ Transplant Registry of the International Society for Heart and Lung Transplantation: Twenty-first pediatric heart transplantation report; Focus them: multiorgan transplantation. J Heart Lung Transplant. 2018;37:1184-1195. 3. Ringewald JM, Gidding SS, Crawford SE, et al. Non-adherence is associated with late rejection in pediatric heart transplant recipients. J Pediatr. 2001;139:75. 4. Lawrence KS, Fricker FJ. Pediatric heart transplantation: quality of life. J Heart Lung Transplant. 1987;6:329. 5. Canter CE, Shaddy RE, Berstein D. Indications for heart transplantation. Circulation. 2007;115:658. 6. Gajarksi RJ, Bennett Pearce F. Recipient evaluation: medical and psychological morbidities. In: Canter CE, Kirklin JK, eds. ISHLT Monograph Series: Pediatric Heart Transplantation. Philadelphia: Elsevier; 2007:19-32. 7. Hoffman TM, Wernovsky G, Atz AM, et al. Prophylactic intravenous use of milrinone after cardiac operations in pediatrics (PRIMACORP) study. Am Heart J. 2002;143:15. 8. Wessel DL. Managing low cardiac output syndrome after congenital heart surgery. Crit Care Med. 2001;29:S220. 9. Almond CS, Thiagarajan RR, Piercey GE, et al. Waiting list mortality among children listed for heart transplantation in the United States. Circulation. 2009;119(5):717-727. 10. Kirshborn PM, Bridges ND, Myung RJ, et al. Use of extracorporeal membrane oxygenation in pediatric thoracic organ transplantation. J Thorac Cardiovasc Surg. 2002;1223:130-136. 11. Mehta U, Laks H, Sadeghi A, et al. Extracorporeal membrane oxygenation for cardiac support in pediatric patients. Am Surg. 2000;66:879-886. 12. Almond CS, Morales DL, Blackstone EH, et al. Berlin Heart EXCOR pediatric ventricular assist device for bridge to heart transplantation in US children. Circulation. 2013;127:1702-1711. 13. Rossano JW, Lorts A, VanderPluym CJ, et al. Outcomes of pediatric patients supported with continuous-flow ventricular assist devices: a report from the Pediatric Interagency Registry for Mechanical Circulatory Support (PediMACS). J Heart Lung Transplant. 2016;35(5): 585-590. 14. Reinhartz O, Keith FM, EL-Banayosy A, et al. Multicenter experience with the Thoratec ventricular assist device in children and adolescents. J Heart Lung Transplant. 2001;20:439. 15. Conway J, Miera O, Adachi I, et al. Worldwide experience of a durable centrifugal flow pump in pediatric patients. Semin Thorac Cardiovasc Surg. 2018;30:327-335. 16. Blume ED, Naftel DC, Bastardi HJ, et al. Pediatric Heart Transplant Study Investigators outcomes of children bridged to heart transplantation with ventricular assist devices: a multi-institutional study. Circulation. 2006;113(19):2313-2319. 17. West LJ, Pollock-Barziv SM, Dipchand, AI, et al. ABO-incompatible (ABOi) heart transplantation in infants. N Engl J Med. 2001;344: 793-800. 18. West LJ, Karamlou T, Dipchand AI, et al. Impact on outcomes after listing and transplantation, of a strategy to accept ABO blood groupincompatible donor hearts for neonates and infants. J Thorac Cardiovasc Surg. 2006;131:455-461.

19. Everitt MD, Donaldson AE, Casper TC, et al. Effect of ABO-incompatible listing on infant heart transplant waitlist outcomes: analysis of the United Network for Organ Sharing (UNOS) database. J Heart Lung Transplant. 2009;28:1254-1260. 20. Almond CSD, Gauvreau K, Thiagarajan RR, et al. Impact of ABOincompatible listing on wait-list outcomes among infants listed for heart transplantation in the United States: a propensity analysis. Circulation. 2010;121:1926-1933. 21. Shumway NE. Heart transplantation 1958-1995. J Ir Coll Physicians Surg. 1995;24:7-8. 22. Mayer JE, Perry S, O’Brien P. Orthotopic heart transplantation for complex congenital heart disease. J Thorac Cardiovasc Surg. 1990;99:484. 23. Del Nido PJ, Bailey L, Kirklin JK. Surgical techniques in pediatric heart transplantation. In: Canter CE, Kirklin JK, eds. ISHLT Monograph Series: Pediatric Heart Transplantation. Vol 2. Philadelphia: Elsevier; 2007:83-102. 24. Tsilimingas NB. Modification of bicaval anastomosis: an alternative technique for orthotopic cardiac transplantation. Ann Thorac Surg. 2003;75:1333. 25. Kirklin JK, McGriffin DC, Pinderski LJ, et al. Selection of patients and techniques of heart transplantation. Surg Clin North Am. 2004;84:257. 26. Razzouk AJ, Johnston JK, Larsen RL, et al. Effect of over-sizing cardiac allografts on survival in pediatric patients with congenital heart disease. J Heart Lung Transplant. 2005;24:195. 27. Stover EP, Siegel LC. Physiology of the transplanted heart. Int Anesthesiol Clin. 1995;33:11. 28. Kulkarni A, Singh TP, Sarniak A, et al. Sildenaphil for pulmonary hypertension after heart transplantation. J Heart Lung Transplant. 2004;23:1441. 29. Di Filippo S, Boissonnat P, Sassolas F, et al. Rabbit antithymocyte globulin as induction immunotherapy in pediatric heart transplantation. Transplantation. 2003;75(3):354-358. 30. Grundy N, Simmonds J, Dawkins H, et al. Pre-implantation basiliximab reduces incidence of early acute rejection in pediatric heart transplantation. J Heart Lung Transplant. 2009;28:1279. 31. Butts R, Davis M, Savage A, et al. Effect of induction therapy on graft survival in primary pediatric heart transplantation: a propensity score analysis of the UNOS database. Transplantation. 2017;101(6): 1228-1233. 32. Singh TP, Faber C, Blume ED, et al. Safety and early outcomes using a corticosteroid-avoidance immunosuppression protocol in pediatric heart transplant recipients. J Heart Lung Transplant. 2010;29(5):517-522. 33. Gajarski RJ, Blume ED, Urschel S, et al. Infection and malignancy after pediatric heart transplantation: the role of induction therapy. J Heart Lung Transplant. 2011;30(3):299-308. 34. Rostad CA, Wehrheim K, Kirklin JK, et al. Bacterial infections after pediatric heart transplantation: epidemiology, risk factors and outcomes. J Heart Lung Transplant. 2017;36:996-1003. 35. Hsu DT, Addonizio LJ, Hordof AJ, et al. Acute pulmonary embolism in pediatric patients awaiting heart transplantation. J Am Coll Cardiol. 1991;17:1621. 36. Green M, Michaels MG. Infections in pediatric solid organ transplant recipients. J Pediatric Infect Dis Soc. 2012;1:144-151. 37. Baran DA, Galin I, Sandle D, et al. Tacrolimus in cardiac transplantation: efficacy and safety of a novel dosing protocol. Transplantation. 2002;74:1136. 38. Lobach NE, Pollock-Barziv SM, West LJ, Dipchand AI. Sirolimus immunosuppression in pediatric heart transplant recipients: a singlecenter experience. J Heart Lung Transplant. 2005;24:184-189.

References

e2

Abstract: The physiology of the transplanted heart as well as preoperative and perioperative critical care all play important roles in the successful transplantation of critically ill children with limited other options. The overall transplant survival at 1 year has improved to more than 90%, with an expected 5-year survival exceeding 80%. The time at which 50% of recipients remain alive is 13.3 years for teenagers and 22.3 years for infants. Individuals may have a good quality of life following heart transplantation

but are likely destined for a repeat transplant. This chapter reviews critical care management of the pediatric patient undergoing heart transplantation. Key words: pediatric, heart transplant, critical care, immunosuppression, congenital, perioperative, posttransplant, mechanical support, milrinone, graft dysfunction

38 Physiologic Foundations of Cardiopulmonary Resuscitation ADNAN M. BAKAR, KENNETH E. REMY, SAREEN SHAH, AND CHARLES L. SCHLEIEN

With the development of basic cardiopulmonary resuscitation (CPR) in the early 1960s, skilled resuscitation teams both in and out of the hospital were formed. The development of CPR saved lives; previously, every victim of cardiac arrest had died. Soon thereafter, successful resuscitation of patients by basic life support measures, defibrillation, and medications became common even as long as 5 hours after commencement of CPR. Over the past decade, both survival and neurologic outcomes after in-hospital cardiac arrest have improved in adults and children.1–3 Data show that the success of CPR depends on many factors. Rapid institution of basic life support measures (i.e., bystander CPR for sudden out-of-hospital cardiac arrest and immediate electrical countershock for ventricular fibrillation [VF]) improve the chances of survival for patients experiencing sudden out-of-hospital cardiac arrest.4 These measures led to the growing deployment of automatic external defibrillators (AEDs) in public places. Immediate defibrillation is currently the standard of care in witnessed VF arrests. However, evidence indicates that basic life support and other measures directed at restoring energy substrates to the myocardium before countershock in patients with unwitnessed, outof-hospital arrest may further improve outcome.5–8 Other preexisting factors that play a role in successful resuscitation include the patient’s age, prior medical condition, presenting cardiac rhythm, and the etiology of cardiac arrest. In 2008, a multi-institutional prospective study was published that examined these preexisting factors and further described in two additional studies the clinical characteristics, hospital course, and outcomes of a cohort of children after in-hospital or out-of-hospital arrest. In 420









Both the cardiac and thoracic pump mechanisms play a role in infants and children during cardiopulmonary resuscitation. Thus, attention to excellent chest compression technique— with an emphasis on “push hard, push fast”—is critical to attaining sufficient cardiac output to maintain coronary and cerebral blood flow. Use of any vasoconstrictor should be sufficient to raise aortic diastolic pressure during cardiopulmonary resuscitation above the critical level for resuscitation success (.15–20 mm Hg).













PEARLS Amiodarone or lidocaine may be the most effective pharmacologic treatments for shock-resistant ventricular tachycardia or fibrillation. Use of the biphasic defibrillator is an important advance in the treatment of tachyarrhythmias and has advantages in its safety profile compared with monophasic defibrillators. Resuscitative care following cardiac arrest is critical to survival and includes appropriate uses of inodilators and neuroprotective strategies, including avoidance of hyperthermia.

addition to demonstrating differences in clinical characteristics, these studies offered future considerations for the care of children who had experienced cardiac arrest and postresuscitative care, including hypothermia.9–12 The low resuscitation rate in children, even when the patient does not have preexisting disease, probably results from the high incidence of asystole as the presenting rhythm. Asystole is the most common presenting rhythm in both in-hospital and out-of-hospital arrests, noted in 25% to 70% of victims.13–17 Bradycardia and pulseless electrical activity (PEA) are other common rhythms. The high incidence of asystole in children who experience cardiac arrest can be explained by systemic disturbances—such as hypoxia, acidosis, sepsis, and hypovolemia— that commonly precede the arrest. Although ventricular arrhythmias usually are reported to be infrequent (range, 1.3%–3.8%), out-of-hospital series report VF in 10% to 19% of victims younger than 20 years.18,19 These series, along with the observation that the frequency of witnessed arrest is much lower than in adults, suggests that ventricular rhythms may be more common than usually estimated and that delay in resuscitation results in progression of nonperfusing rhythms to asystole. Increasing availability of AEDs may be contributing to the increased recognition of ventricular arrhythmias in out-of-hospital pediatric cardiac arrest.20 In specialized cardiac intensive care units (ICUs), ventricular arrhythmias account for as many as 30% of the arrests.3,21,22 In their original work on CPR, Kouwenhoven et al.23 proposed that blood flow during closed-chest compressions resulted from squeezing of the heart between the sternum and vertebral column, now termed the cardiac blood flow mechanism. In fact, the precise



CHAPTER 38

mechanism by which forward circulatory flow is generated during closed-chest cardiac massage has major implications for current approaches to CPR. Other methods—such as vest CPR, active compression-decompression CPR (ACD-CPR) both without and with an impedance threshold valve (ITV), and interposed abdominal compressions with CPR (IAC-CPR)—take into account advances in our understanding of the mechanism of blood flow during resuscitation. The pharmacology of resuscitation remains controversial; these controversies have led to major changes in the guidelines for CPR. Use of sodium bicarbonate, calcium chloride, and glucose remains unresolved at this time. The role of high-dose epinephrine has been minimized because of concerns over postresuscitation deleterious effects on myocardial performance and poor outcomes. Evidence for a role for vasopressin, with a relatively pure vasoconstrictor effect, is accumulating. Data have been accumulating for the use of amiodarone or lidocaine as the antiarrhythmics of choice for ventricular ectopy in persons in cardiac arrest.3 Research is ongoing into alternative vasoconstrictors and the use of pharmacologic cocktails that may include b-blockers, antiarrhythmic agents, antioxidants, nitroglycerin, and a vasoconstrictor in attempts to improve the resuscitation outcome and postresuscitation cardiac function.24–26 Developments in the use of direct current countershock have occurred. Biphasic defibrillators are now widely in use and appear to improve the success of defibrillation at lower delivered energies. It is hoped that they decrease myocardial injury. As noted, the role of “shock first” is being reassessed because the success of electrical countershock in restoring spontaneous circulation declines rapidly after 3 to 4 minutes have elapsed. Although postresuscitation cerebral preservation has become an important area of focus, therapeutic hypothermia has not been found to improve neurologic outcome after pediatric cardiac arrest.11,12 This chapter discusses the physiologic foundations of CPR. In the first section, the possible mechanisms of blood flow by the thoracic and cardiac pump mechanisms are discussed, including how the specific chest geometry of children and infants helps decide which of these mechanisms applies. Newer CPR techniques, which consider the physiologic mechanisms discussed in the first section, are then discussed. Controversies and advances in pharmacologic management during CPR and current guidelines for use of drugs for resuscitation are addressed. New developments in the use of countershock—including the timing of shocks, energy used, and type of current delivery system used (biphasic or monophasic)—are discussed. Finally, the role of therapeutic hypothermia is reviewed.

Physiologic Foundations of Cardiopulmonary Resuscitation

421

Numerous clinical observations have conflicted with the cardiac pump hypothesis of blood flow. In 1964, Mackenzie et al.27 found that closed-chest CPR produced similar elevations in arterial and venous intravascular pressures, the result of a generalized increase in intrathoracic pressure. In 1976, Criley et al.28 made the dramatic observation that several patients in whom VF developed during cardiac catheterization produced enough blood flow to maintain consciousness by repetitive coughing. The production of blood flow by increasing thoracic pressure without direct cardiac compression describes the thoracic pump mechanism of blood flow during CPR. During normal cardiac function, the lowest pressure in the vascular circuit occurs on the atrial side of the atrioventricular valves. This low-pressure compartment is the downstream pressure for the systemic circulation, which allows venous return to the heart. Angiographic studies show that blood passes from the vena cava through the right heart into the pulmonary artery and from the pulmonary veins through the left heart into the aorta during a single chest compression. Echocardiographic studies show that, unlike normal cardiac activity or during open-chest CPR, during closed-chest CPR in both dogs and humans, the atrioventricular valves are open during blood ejection and aortic diameter decreases rather than increases during blood ejection.29,30 These findings during closedchest CPR support the thoracic pump theory and argue that the heart is a passive conduit for blood flow (Fig. 38.1).31 Initial measurements of hemodynamic data during chest compression for CPR found the generation of almost equal pressures in the left ventricle, aorta, right atrium, pulmonary artery, and esophagus.32 The finding that all intrathoracic vascular pressures are equal implies that suprathoracic arterial pressures must be higher than suprathoracic venous pressures. The unequal transmission of intrathoracic pressure to the suprathoracic vasculature establishes the gradient necessary for blood flow. The transmission of intrathoracic pressure to the suprathoracic veins may be modulated by venous valves. The presence of these jugular venous valves has been demonstrated in animals and humans undergoing Direct Cardiac Compression

Sternum

Mitral valve closed

Thoracic Pump Mitral valve opened

Vertebra

Mechanisms of Blood Flow

• Fig. 38.1

​Possible mechanisms for blood flow during cardiopulmonary resuscitation include direct cardiac compression (left) and the thoracic pump (right). With direct cardiac compression, an increase in chest compression rate causes an increase in blood flow by squeezing the heart between the vertebral column and sternum. With the thoracic pump mechanism, factors that increase pleural pressure cause an increase in pressure within the heart chambers and, ultimately, an increase in blood flow. (Modified from Schleien CL et al. Controversial issues in cardiopulmonary resuscitation, Anesthesiology. 1989;71:135.)  

The cardiac pump hypothesis holds that blood flow is generated during closed-chest compressions when the heart is squeezed between the sternum and vertebral column. This mechanism of flow implies that ventricular compression causes closure of the atrioventricular valves and that ejection of blood reduces ventricular volume. During chest relaxation, ventricular pressure falls below atrial pressure, allowing the atrioventricular valves to open and the ventricles to fill. This sequence of events resembles the normal cardiac cycle and occurs during cardiac compression when openchest CPR is used.

Chest compression force ↑ Rate of chest compression and and duty cycle cause ↑ Force of chest compression cause ↑ Pleural cavity pressure ↑ Blood flow from heart ↑ Pressure of heart chambers



Cardiac Versus Thoracic Pump Mechanism

422

SECTION IV



Pediatric Critical Care: Cardiovascular

CPR.33–35 An ultrasonography study of healthy children confirmed the presence of these valves in 84% of 239 jugular veins studied. The valves were bilateral in 74% of children.36 Transmission of intrathoracic pressure to the intracranial vault during CPR indicates that any such valve function is partial. Pathologic studies have also identified valves in the subclavian vein in the large majority of cadavers studied (87%). The absence of these valves in some patients is postulated to lead to failure of closed-chest CPR.37 Subsequent hemodynamic and echocardiographic studies found different results. Deshmukh et al. demonstrated in a porcine model that mitral valve function persisted throughout resuscitation in 17 of 22 animals and that in successfully resuscitated animals, maximal aortic pressure exceeded that in the right atrium throughout the resuscitation.33 In another porcine model of resuscitation, Hackl et al. manipulated the compressive force and depth of resuscitation by using a mechanical resuscitator.38 The frequency of mitral valve closure during compressive systole was directly proportional to the force and depth of chest compression. When the depth of compression reached 25% of the anteroposterior diameter, valve closure occurred in 95% of cycles. They concluded that the mechanism of blood flow was dependent on the force and depth of compression. In a study of CPR using transesophageal Doppler echocardiography in adults, Porter et al. demonstrated mitral valve closure in compressive systole in the majority of patients (12 of 17) but not all patients.39 Peak mitral flow occurred in diastole and was significantly higher in the group with mitral valve closure. Peak mitral flow occurred during compressive systole in those without valve closure. Left ventricular (LV) fractional shortening correlated with change in anteroposterior chest wall diameter and not mitral valve flow. These authors concluded that nonuniform increased intrathoracic pressure plays a role in determining whether valve closure occurs during chest compressions. As noted, a decrease in aortic dimension during CPR has been demo nstrated by echocardiography and taken as evidence for the thoracic pump mechanism of blood flow. Hwang et al. readdressed this issue using transesophageal echocardiography.40 They studied the aortic dimension of the proximal and distal thoracic aorta and noted a decrease in the aortic dimension in the distal aorta directly inferior to the zone of direct compression and an increase in the dimension of the proximal aorta. They also noted mitral valve closure in all subjects and a decrease in LV volume of almost 50% at end compression. These findings were believed to be most consistent with the cardiac pump mechanism of blood flow.41 Kim et al. also used transesophageal echocardiography to explore the role of the left ventricle during nontraumatic arrests.42 They noted that during the compression phase of CPR, there was anterograde flow from the ventricle to the aorta and retrograde flow toward the mitral valve. The mitral valve remained closed during compression and open during relaxation, while the aortic valve remained open during compression and closed during relaxation, which they concluded to be consistent with the cardiac pump mechanism. The cardiac pump mechanism appears to predominate during closed-chest CPR in specific clinical situations. As noted, increasing the applied force during chest compressions increases the likelihood of direct cardiac compression. A smaller chest size may allow for more direct cardiac compression.38,42 Adult dogs with small chests have better hemodynamics during closed-chest CPR than do dogs with large chests. Because the infant chest is smaller and more compliant than the adult chest, direct compression of the heart during CPR is more likely to occur. Blood flow during closed-chest CPR in a piglet model of cardiac arrest is higher than

that achieved in adult models.43 A study of 20 randomized swine to either a patient-centric blood pressure targeted approach with titration of compression depth to a systolic blood pressure of 100 mm Hg and vasopressors to a coronary perfusion pressure greater than 20 mm Hg or current usual practice, the blood pressure targeted group demonstrated improved 24-hour survival (8 of 10 vs 0 of 10 survival; P 5 .001).44 This study suggests that physiologic targets rather than absolute depths by age may in fact confer better outcomes.

Rate and Duty Cycle In 2015, the American Heart Association (AHA) recommended a rate of chest compressions of at least 100 per minute.45–49 At faster rates, blood flow is enhanced whether the thoracic pump mechanism or cardiac pump mechanism is invoked. Duty cycle is defined as the ratio of the duration of the compression phase to the entire compression-relaxation cycle, expressed as a percent. For example, at a rate of 30 compressions/min, a 1.2-second compression time produces a 60% duty cycle. If blood flow is generated by direct cardiac compression, then the stroke volume is determined primarily by the force of compression. Prolonging the compression (increasing the duty cycle) beyond the time necessary for full ventricular ejection should have no additional effect on stroke volume. Increasing the rate of compressions should increase cardiac output because a fixed, relatively small volume of blood is ejected with each cardiac compression. In contrast, if blood flow is produced by the thoracic pump mechanism, the volume of blood to be ejected comes from a large reservoir of blood contained within the capacitance vessels in the chest. With the thoracic pump mechanism, flow is enhanced by increasing either the force of compression or the duty cycle but is not affected by changes in compression rate over a wide range of rates.34 Additionally, the “push hard, push fast” recommendation is based on the maintenance of a higher compression rate with a higher force of compression. Allowing total recoil of the chest allows for full blood return during the relaxation phase of the cycle.50 It appears from experimental animal data that both the thoracic pump and cardiac pump mechanisms can effectively generate blood flow during closed-chest CPR. Differences between various studies may be attributed to differences in animal models or compression techniques. Important differences in animal models include chest wall geometry, compliance and elastic recoil, compliance of the diaphragm, and intraabdominal pressure. Differences in technique include the magnitude of sternal displacement; compression force; and momentum of chest compression, compression rate, and duty cycle. Experimental and clinical data support both mechanisms of blood flow during CPR in human infants. Results of several studies in dogs demonstrated a benefit of a compression rate of 120 per minute compared with slower rates during conventional CPR.51,52 In studies of piglets, puppies, and humans, no differences were found comparing different rates of compression during conventional CPR.34,53–55 In a study of piglet CPR, duty cycle was the major determinant of cerebral perfusion pressure (CPP). The duty cycle at which venous return became limited varied with age. A longer duty cycle was more effective in younger piglets.55 In a more recent study of 22 pigs randomized to head-up tilt versus supine CPR using an automated CPR device plus an impedance threshold device after VF arrest, CPP and ICP improved substantially with head-up tilt position.56 This suggests that gravity has an effect on the venous circulation with

high-quality CPR and head tilted up positioning on cerebral perfusion. The discrepant importance of rate and duty cycle in various models (by different investigators) is confusing; however, increasing the rate of compressions during conventional CPR to 100 per minute satisfies both those who prefer the faster rates and those who support a longer duty cycle. Appropriate chest compression rate, depth, and fraction are still being investigated. Niles et al. characterized these in-hospital CPR metrics and compliance to the 2015 AHA guidelines by evaluating the pediRES-Q database.57 They found that meeting all three of these AHA-derived CPR targets was extremely difficult to accomplish. Their study was not powered to address short- or long-term survival but was an important step in evaluating the optimal cutoffs for survival.

Chest Geometry Chest geometry plays an important role in the ability of extrathoracic compressions to generate intrathoracic pressure. Shape, compliance, and deformability, which change greatly with age, are the chest characteristics that have the greatest impact during CPR. The change in cross-sectional area of the chest during anterior to posterior delivered compressions is related to its shape (Fig. 38.2).58 The ratio of the chest anteroposterior diameter to the lateral diameter is referred to as the thoracic index. A keel-shaped chest, as seen in an adult dog, has a greater anteroposterior diameter and, thus, a thoracic index greater than 1. A flat chest, as in a thin human, has a greater lateral diameter and, thus, a thoracic index less than 1. A circular chest has a thoracic index equal to 1. A circle has a larger cross-sectional area than either of these elliptical chests. As an anteroposterior compression flattens a circle, the cross-sectional area decreases and compresses its contents. In contrast, as an anteroposterior compression is applied to the keelshaped chest, the cross-sectional area increases as a circular shape is approached. The cross-sectional area of the keel-shaped chest

does not decrease until the chest compression continues past the circular shape to flatten the chest. This implies a threshold past which the compression must proceed before intrathoracic contents are decreased and squeezed.58 Thus, the rounder, flatter chests of small dogs and pigs may require less chest displacement than the keel-shaped chests of adult dogs to generate thoracic ejection of blood. This dynamic has been demonstrated in small dogs having round chests compared with adult dogs having keelshaped chests.59 As humans age, the cartilage of the rib cage calcifies and chest wall compliance decreases. Older patients may require greater compression force to generate the same sternal displacement. A 3-month-old piglet requires a much greater compression force for anteroposterior displacement than its 1-month-old counterpart.58 Direct cardiac compression is more likely to occur in the more compliant chest of younger animals. Cerebral and myocardial blood flow during closed-chest CPR was much higher in infant piglets than in adults (Figs. 38.3 and 38.4).60 This finding supports the cardiac pump mechanism of blood flow in infants because the level of organ blood flow achieved during closed-chest CPR in piglets approaches the level achieved during open-chest cardiac massage in adults. Marked deformation of the chest can occur during prolonged CPR and may alter the effectiveness of CPR (Fig. 38.5).60 Over time, the chest assumes a flatter shape, producing a larger percent decrease in cross-sectional area at the same absolute chest displacement. Progressive deformation may be beneficial if it leads to more direct cardiac compression. Unfortunately, too much deformation may decrease the recoil of the chest wall during the 50

rlat

Area = 13%

A rap

*

20



10

1.0

rap

rlat

*

30

20% rap



0.8 0.6

Area = 13%

B • Fig. 38.2



​ hanges in area of ellipses with constant circumference. Each C ellipse is labeled with the anteroposterior (ap) and lateral (lat) radii, and a 20% anteroposterior compression is applied. Indicated change in area equals relaxed area – compressed area. (A) Initial anteroposterior/lateral ratio 5 0.7, and compression leads to positive ejection because relaxed area – compressed area is negative. (B) Initial anteroposterior/lateral ratio 5 1.4, and compression toward a circular shape results in an increase in area. (Modified from Dean JM, Koehler RC, Schleien CL, et al. Age-related changes in chest geometry during cardiopulmonary resuscitation. J Appl Physiol. 1987;62[6]:2212–2219.) 











*







‡ Without epinephrine With epinephrine P < .05 from prearrest P < .05 between groups



*

5 Cerebral O2 uptake

rlat





0.4 rlat

*

0

Cerebral fractional O2 extraction

rap



40

RELAXATION PHASE COMPRESSION PHASE 20%

423

Physiologic Foundations of Cardiopulmonary Resuscitation

Cerebral blood flow



CHAPTER 38

4

*

3 2



1

* ‡

0 Pre- 0 arrest

• Fig. 38.3

10

20

30

*

‡ ‡ 40

50

Time (min)



​Total cerebral blood flow, cerebral fractional oxygen (O2) extraction, and cerebral O2 uptake before cardiac arrest and during 50 minutes of cardiopulmonary resuscitation in the groups with and without epinephrine.

424

SECTION IV



Pediatric Critical Care: Cardiovascular

80 Without epinephrine With epinephrine

Epinephrine Norepinephrine

100

Entire heart

Blood flow (mL/min/100 g)

40

% of prearrest anteroposterior diameter

60

* *

20

Chest relaxation position

80

Displacement

60

Chest compression position 40 20 Displacement

80

60

RV LV Septum

40

RV

20

LV Septum

Prearrest 5

20

35

50

Duration of CPR (min) LSD

• Fig.

38.5 ​Piston position during chest compression and relaxation phases of the cycle, and net piston displacement expressed as a percent of prearrest anteroposterior chest diameter (12.0 6 0.3 cm) in piglets. Note that displacement was essentially unchanged over the 50-minute duration, but marked deformation occurred during the relaxation phase by 5 minutes and continued to further deform over the 50-minute period in the groups with or without epinephrine. CPR, Cardiopulmonary resuscitation. (Modified from Schleien CL, Dean JM, Koehler RC, et al. Effect of epinephrine on cerebral and myocardial perfusion in an infant animal preparation of cardiopulmonary resuscitation. Circulation. 1986;73[4]:809–817.)

*





0

10

20

30

40

50

Duration of CPR (min)

• Fig. 38.4

​Top, Total myocardial blood flow during cardiopulmonary resuscitation (CPR) in piglets with and without epinephrine. Asterisk indicates significant difference between groups at 5 and 20 minutes. Bottom, Blood flow to right ventricular free wall (RV, circle), left ventricular free wall (LV, squares), and interventricular septum (triangles) in the groups with and without epinephrine. Standard error bars are omitted for clarity, but the least significant difference bar (LSD, derived from Duncan multiple-range test) is shown for comparisons among heart regions within an animal group. (Means must differ by height of bar for P , .05.) LSD for comparing means between groups is twice that shown for within-group LSD. Asterisk indicates that RV blood flow was greater than LV and septal blood flows at 5 minutes in the group without epinephrine. Flows in all three regions in the group with epinephrine were greater than those in the respective regions in the group without epinephrine at 5 and 20 minutes. (Modified from Schleien CL, Dean JM, Koehler RC, et al. Effect of epinephrine on cerebral and myocardial perfusion in an infant animal preparation of cardiopulmonary resuscitation. Circulation. 1986;73[4]:809–817.)  



relaxation phase, leading to decreased cardiac filling. A progressive decrease in the effectiveness of chest compressions to produce blood flow is seen in piglets receiving conventional CPR.60 Permanent deformation of the chest in this model approaches 30% of the original anteroposterior diameter. Using a thoracic vest to limit deformation when performing CPR greatly decreased permanent chest deformation (3% vs. 30%) but did not attenuate the deterioration of vital organ blood flow with time.61 The characteristics of chest geometry of animals may relate to that in humans. Body weight, surface area, chest circumference, and diameter did not correlate with the magnitude of aortic pressure produced during CPR in a study of nine adults already declared dead.61 A direct comparison of adult and pediatric human CPR has not been performed. The higher intravascular pressures and organ blood flow during CPR in infants compared with

adults may result from more effective transmission of the force of chest compression because of the higher compliance and greater deformability of the infant chest.

Effects of Cardiopulmonary Resuscitation on Intracranial Pressure When chest compressions are applied, the increase in intrathoracic pressure is transmitted through the venous system of the head and neck to the intracranial vault, resulting in an increased intracranial pressure (ICP). Pressure is transmitted via the paravertebral veins and the cerebrospinal fluid during CPR in dogs.62 Large swings in ICP corresponding to chest compressions occur in children undergoing CPR.63 This transmission of intrathoracic pressure to the intracranial contents accounts for the low CPP by increasing the downstream pressure and cerebral blood flow during closed-chest CPR. However, in a porcine model of CPR with an impedance threshold device, it was found that CPR done in a reverse Trendelenburg position (head up at 30 degrees) reduced ICP and improved cerebral perfusion, likely because gravity improved venous drainage and, thus, reduced impedance to forward flow.56 The relationship of ICP to intrathoracic pressure during CPR is linear. In dogs receiving conventional CPR, ICP increased by one-third of the rise of intrathoracic pressure in a range from 10 to 90 mm Hg.62 However, some modes of CPR change the intrathoracic to ICP relationship. In dogs, abdominal binding increases the transmission of pressure to the intracranial space to one-half of the rise of intrathoracic pressure.64 Open-chest CPR decreases the transmission of pressure and improves CPP compared with conventional CPR. Thus, increasing intrathoracic pressure may decrease cerebral blood flow because of the increase in downstream pressure, the ICP. In this regard, ACD-CPR and ACD-ITV-CPR may have an advantage over conventional CPR. These techniques are designed



CHAPTER 38

to reduce intrathoracic pressure. Lindner et al. showed in a porcine model that cerebral perfusion is increased with ACD-ITVCPR compared with standard CPR.65 Using an adult porcine model of hypothermic VF arrest, the same group demonstrated by micro-dialysis techniques improved lactate/pyruvate ratios and reduced glucose accumulation in the ACD-ITV group compared with standard CPR.66 In a pediatric porcine model of resuscitation, Voelckel et al. found that ACD-CPR with ITV provided superior cerebral blood flow compared with standard CPR.67

Newer Cardiopulmonary Resuscitation Techniques A recent Cochrane systematic review of 11 trials including 12,944 adult patients with use of mechanical chest compression devices for CPR has not suggested that these mechanical devices are superior to conventional therapy.68 Similar recent reviews have had the same findings.69 However, pediatric trials were not included in this review and the authors did conclude that mechanical devices are a reasonable alternative where consistent, high-quality manual chest compressions are not possible or dangerous for the provider. Thus, simultaneous compression ventilation CPR, vest approaches, interposed abdominal compression CPR, and openchest modalities are discussed. More recent approaches, including tourniquet-assisted CPR to augment myocardial perfusion, have been presented in limited animal case series and will be reserved for future editions.70 Simultaneous compression ventilation cardiopulmonary resuscitation (SCV-CPR) is a technique designed to increase blood flow during conventional CPR by increasing the thoracic pump mechanism contribution to blood flow. Delivering a breath simultaneously with every compression, instead of after every fifth compression, increases intrathoracic pressure and augments blood flow produced by closed-chest CPR. Survival has been shown to be equivalent or significantly worse in both animals and humans who received SCV-CPR compared with conventional CPR.55,59,60,71–74 No study has shown an increased survival rate with this CPR technique. Interposed abdominal compression cardiopulmonary resuscitation (IAC-CPR) is the delivery of an abdominal compression during the relaxation phase of chest compression. An extensive review by Babbs has been published.75 IAC-CPR may augment conventional CPR in several ways. First, IAC-CPR may return venous blood to the chest during chest relaxation.76,77 Second, IAC-CPR increases intrathoracic pressure and augments the duty cycle of chest compression.76,78 Third, IAC-CPR may compress the aorta and return blood retrograde to the carotid or coronary arteries.77 IAC-CPR is an attractive alternative to some of the newer techniques of CPR because it requires no additional equipment for implementation. However, it does require training and manpower. Four randomized controlled trials have compared IAC-CPR with standard CPR. The first trial, reported in 1985 by Mateer et al., was the largest and included 291 patients.79 IAC-CPR was applied in the field by paramedics until ambulance transport. No differences in mortality were found. The later trials involved a total of 279 hospitalized patients.80–82 The results from these trials are more positive; a meta-analysis of these studies found an increased likelihood of return of spontaneous circulation (ROSC) and intact survival to discharge with IAC-CPR versus standard CPR.75,83 Although no intraabdominal trauma was detected in

Physiologic Foundations of Cardiopulmonary Resuscitation

425

any of the 426 patients in these trials, one pediatric case report demonstrated direct pancreatic injury. More recent studies evaluating IAC-CPR to standard CPR on end-tidal carbon dioxide (ETCO2) and ROSC demonstrated improvement in ETCO2 without a difference in ROSC.84 This suggests that IAC-CPR may indicate improved cardiac output via this method without clinical improvement. Application of IAC-CPR is limited by the need for training and additional manpower. Although it has not been studied in a pediatric group, with skilled personnel available, IAC-CPR could be considered for use with inpatient arrests. Active compression-decompression cardiopulmonary resuscitation (ACD-CPR) involves a negative-pressure “pull” on the thorax during the release phase of chest compression using a hand-held suction device (Fig. 38.6).85 This technique improves vascular pressures and minute ventilation during CPR in animals and humans.65,85–87 The mechanism of benefit of this technique is attributed to enhancement of venous return by the negative intrathoracic pressure generated during the decompression phase. In addition, it reverses the chest wall deformation that accompanies standard CPR.88 Preliminary results in adults were promising, and a large multi-institutional study of ACD-CPR completed in Europe found that ACD-CPR was superior to standard CPR.87,89,90 In this study, a total of 750 patients were randomly assigned to receive standard CPR or ACD-CPR. In the experimental group, 5% survived to 1 year (12 patients with intact neurologic status) versus 2% (three patients with intact neurologic status) in the standard group.91 However, a number of other trials have not shown a difference between standard CPR and ACD-CPR, and a Cochrane Database Systematic Review concluded that there was no consistent benefit from use of this technique.92 The effectiveness of ACD-CPR appears to be relatively site specific. Explanations for this variability have focused on the effectiveness of training for providers and intersite variation of on-scene advanced life support techniques.88 Use of ACD-CPR requires significantly more physical effort than conventional CPR; this requirement may have influenced outcome.93 No device is cleared for clinical use in the United States at this time. Use of an ITV has been evaluated in attempts to improve the outcome with ACD-CPR.91,94 This technique involves the use of a valve placed between the ventilating bag and airway, which is designed to close when the tracheal pressure falls below atmospheric pressure, enhancing the development of negative intrathoracic pressure during ACD-CPR (Figs. 38.7 and 38.8). Animal studies, including a young porcine model, showed improved organ perfusion, and brain micro-dialysis studies demonstrated decreased lactate accumulation and improved glucose utilization.66,67,95,96 In a small series of patients, diastolic pressure was raised along with CPP and ETCO2 release.97 These studies led to an inclusion of the technique as an acceptable alternative to standard CPR in the 2000 AHA guidelines and subsequent revised guidelines.47,49 Plaisance et al. reported on a series of 400 patients randomly assigned to ACD-CPR with ITV or sham ITV. Survival at 24 hours was significantly improved.98 There was a nonsignificant trend toward improved neurologic survival, with 6 of 10 discharged patients having intact survival compared with 1 of 8 discharged survivors in the sham ITV group. In a randomized controlled study by Wolcke et al.99 of 610 adults in cardiac arrest in the out-of-hospital setting, use of ACD-CPR plus the ITV was associated with improved ROSC and 24-hour survival rates when compared with CPR alone. The addition of the ITV was associated with improved hemodynamics during standard CPR in one

426

SECTION IV



Pediatric Critical Care: Cardiovascular

A

B • Fig. 38.6

​Device for performing active compression-decompression cardiopulmonary resuscitation. The upper part is a handle and the lower part is a suction cup. (Courtesy AMBU Corporation.)  

No ventilation or CPR

Active manual ventilation

Chest compression or exhalation

Chest Spontaneous decompression inhalation No airflow

Ventilation port Silicone diaphragm Airflow

Safety check valve Patient port

• Fig. 38.7

​Schematic diagram of impedance threshold valve. During positive-pressure ventilation, the valve is open and gas flows. During chest compression or exhalation, air moves freely through the valve. During chest decompression, airflow is impeded by the valve, decreasing intrathoracic pressure. During spontaneous ventilation, the check valve opens, allowing gas flow. CPR, Cardiopulmonary resuscitation. (Modified from Lurie KG, Barnes TA, Zielinski TM, et al. Evaluation of a prototypic inspiratory impedance valve designed to enhance the efficiency of cardiopulmonary resuscitation. Resp Care. 2003;48[1]:52–57.)  



clinical study.100 The ultimate role of this technique, which requires specialized equipment and significant resuscitator training, remains to be determined.91,101 Vest CPR uses an inflatable bladder resembling a blood pressure cuff that is wrapped circumferentially around the chest with phased inflation to increase intrathoracic pressure. Because chest dimensions are changed minimally, direct cardiac compression is unlikely. In addition, the even distribution of the force of compression over the entire chest wall decreases the likelihood of trauma to the skeletal chest wall and its thoracic contents. In a human study, vest CPR increased aortic systolic pressure but had little effect on aortic diastolic pressure compared with conventional CPR.102 Despite its late application, vest CPR improved the hemodynamics and rate of ROSC in adult patients in

another study.103 Evidence from a case control study of 162 adults documented improvement in survival to the emergency department when vest CPR was administered by adequately trained personnel to patients in cardiac arrest in the out-of-hospital setting.104 The lack of metallic parts has allowed vest CPR to be used experimentally during nuclear magnetic resonance spectroscopy to study brain intracellular pH.105 Clinically, the use of vest CPR depends on sophisticated equipment and remains experimental at this time.

Abdominal Binding Abdominal binders and military antishock trousers have been used to augment closed-chest CPR. Both methods apply continuous



CHAPTER 38

Physiologic Foundations of Cardiopulmonary Resuscitation

427

ACD CPR with Active Impedance Threshold Device +10 mm Hg 0 mm Hg –10 mm HgP1 Positive pressure ventilation

Compression Decompression

ACD CPR with Sham Impedance Threshold Device +10 mm Hg 0 mm Hg –10 mm Hg

P1

• Fig. 38.8



​Example of intratracheal pressures, a surrogate for intrathoracic pressures, in a patient undergoing cardiopulmonary resuscitation (CPR) with an automated compression device (ACD) with and without an impedance threshold device attached to a face mask. CPR was delivered at 100 compression/ decompression cycles/min with a synchronized compression/ventilation ratio of 15:2. Note the absence of significant decreases in intratracheal pressures with a sham device. With the active impedance threshold device, wide fluctuations in intratracheal pressure are seen with each compression and decompression. (Courtesy M. Lurie, MD, and Advanced Circulatory Systems, Inc.)

compression circumferentially below the diaphragm. Three mechanisms have been proposed for augmentation of CPR by these binders. First, binding the abdomen decreases the compliance of the diaphragm and raises intrathoracic pressure. Second, blood may be moved out of the intrathoracic structures to increase circulating blood volume. Third, applying pressure to the subdiaphragmatic vasculature and increasing its resistance may increase suprathoracic blood flow. These effects increase aortic pressure and carotid blood flow in both animals and humans.32,106 Unfortunately, as aortic pressure increases, the downstream component of CPP, namely, right atrial pressure, increases to an even greater extent, resulting in decreased CPP and myocardial blood flow.29 These techniques also lower the CPP by enhanced transmission of intrathoracic pressure to the intracranial vault, which raises ICP (the downstream component of CPP). Clinical studies have failed to show an increased survival when an abdominal binder or military antishock trouser suit was used to augment CPR.

Open-Chest Cardiopulmonary Resuscitation Use of open-chest cardiac massage has generally been replaced by closed-chest CPR. Compared with closed-chest CPR, open-chest CPR generates higher cardiac output and vital organ blood flow.107 During open-chest CPR, there is less elevation of intrathoracic, right atrial, and intracranial pressures, resulting in higher coronary and cerebral perfusion pressures and higher myocardial and cerebral blood flows.108 Open-chest CPR is not a technique that can be applied by most health care personnel. It can be used in the operating room, ICU, or emergency department equipped with the necessary surgical and technical equipment and personnel. It is easily used in the operating room or ICU after cardiac surgery when the open chest can be easily accessed. Open-chest CPR is indicated for cardiac arrest resulting from cardiac tamponade, hypothermia, critical aortic stenosis, and ruptured aortic aneurysm. Other indications include cardiac arrest resulting from penetrating or

crushed chest wall abnormalities that make closed-chest CPR impossible or ineffective.109 Open-chest CPR is indicated for select patients when closed-chest CPR has failed, although exactly which patients should receive this method of resuscitation under this condition is controversial. When initiated early after failure of closed-chest CPR, open-chest CPR may improve outcome.110 When performed after 15 minutes of closed-chest CPR, openchest CPR significantly improves CPP and the rate of successful resuscitation.111

Cardiopulmonary Bypass and Extracorporeal Cardiopulmonary Resuscitation Because of the low rate of survival after prolonged CPR, more aggressive methods have been suggested to improve its success: cardiopulmonary bypass (CPB) and extracorporeal membrane oxygenation CPR (E-CPR).112 CPB is one of the most effective ways to restore circulation after cardiac arrest. Animal studies show that CPB increases survival at 72 hours, increases recovery of consciousness, and preserves the myocardium better than does conventional CPR.113 In dogs, CPB resulted in better neurologic outcome than conventional CPR after a 4-minute ischemic period. However, neurologic outcome was dismal in both groups when the ischemic period lasted 12 minutes.113 Some 90% of dogs survived 24 hours after 15 to 20 minutes of cardiac arrest, but only 10% survived when the arrest time was prolonged to 30 minutes when CPB was used for stabilization during defibrillation.114 CPB decreased myocardial infarct size in a model involving coronary artery occlusion compared with conventional CPR.115 In all animal models, CPB improves the success of resuscitation compared with conventional CPR. Human experience with CPB for cardiac arrest outside the operating room is growing. In the first major series of pediatric patients undergoing E-CPR, reported by Morris et al., 64 children underwent 66 extracorporeal membrane oxygenation (ECMO)

428

SECTION IV



Pediatric Critical Care: Cardiovascular

runs initiated during active resuscitation with chest compressions or internal cardiac massage.116 Of these patients, 33 (50%) were decannulated and survived for more than 24 hours, 21 (33%) survived to hospital discharge, and 16 (26%) reportedly had no major changes in neurologic outcome. The average duration of CPR before cannulation in the survivors was 50 minutes. Of the 6 surviving children who required more than 60 minutes of CPR before ECMO, 3 had no apparent change in neurologic status. During the same period, 73 children underwent standard CPR; 10 received CPR for more than 30 minutes, with no survivors. Duncan et al. reported a series of 18 pediatric cardiac surgical patients at the Boston Children’s Hospital who received ECMO during active chest compressions.117 Of the first 7 patients, only 29% survived. This led to the development of a rapid ECMO deployment strategy in which an ECMO pump is kept saline-primed in the ICU at all times, allowing initiation of extracorporeal support within 15 minutes. Precannulation support times dropped from an average of 90 minutes but still remained high at an average of 50 minutes. Of the remaining 11 patients, 10 were decannulated successfully, with 6 long-term survivors, 5 of whom were in New York Heart Association class I. This rapid deployment strategy likely will become more commonplace in large pediatric centers. Many subsequent pediatric and adult studies have shown both the feasibility and varied success of E-CPR. Clear indications for its use include witnessed arrest in a biventricular circulation, while contraindications include inability to provide effective CPR. There is uncertainty for the role of E-CPR in prolonged conventional resuscitation, as there are no guidelines on when to initiate or which patient population it would be best suited for. This lack of criteria may explain the wide range of success of E-CPR, with survival ranging between 33% and 100%.118 Subgroup analysis from the Therapeutic Hypothermia after Cardiac Arrest In-Hospital trial demonstrated, in a univariate analysis of 56 children receiving open-chest CPR, that approximately one-half survived with good neurobehavioral outcomes at 1 year from index hospitalization.119 On multivariate analysis, the use of ECMO or other extracorporeal therapies demonstrated worse survival at 12 months and worsened neurobehavioral outcomes. However, this trial was designed to evaluate hypothermia as an intervention and many potential confounders may have biased these findings. Conversely, Lasa et al. compared 591 pediatric patients who received E-CPR versus 3165 that received CPR alone and showed that survival to hospital discharge and survival with a favorable neurologic outcome was more favorable with ECPR compared with CPR alone.120 Bembea et al. linked data from the Extracorporeal Life Support Organization and AHA Get with the Guidelines–Resuscitation registries to determine risk factors related to unfavorable outcomes with E-CPR among 593 children.121 In this study, they found that odds of death were increased with a noncardiac diagnosis and preexisting renal insufficiency and that for each additional 5 minutes of CPR prior to ECMO initiation, the odds risk of death increased by 1.04. Under the 2015 AHA guidelines, centers should consider E-CPR for in-hospital cardiac arrest refractory to standard resuscitation attempts if the condition leading to cardiac arrest is reversible or amenable to heart transplantation, if excellent conventional CPR has been performed after no more than several minutes of no-flow cardiac arrest, and if the institution is able to rapidly perform ECMO. Long-term survival has been reported even after more than 150 minutes of CPR in select patients.49 Data are emerging involving the role of ECMO in persons with refractory VF.122 These data have solely been in the form of

case reports but may represent a future direction for the care of patients with VF. In 2006, Samson et al. reported successful treatment of in-hospital VF in children with ECMO after cardiac arrest who had an initial rhythm of VF and immediate initiation of CPR.123 CPB and ECMO require a great deal of technical support and sophistication. In units with preprimed circuits on standby, CPB can be implemented quickly and with moderate success in a population of children who would otherwise almost certainly die. The success with some patients undergoing very long CPR times followed by ECMO use is encouraging and suggests the possibility of reversible myocardial injury as a cause of resuscitation failure in a subset of patients.

Transcutaneous Cardiac Pacing TCP is used as a method for noninvasive pacing of the ventricles for a relatively short period. Emergency cardiac pacing is successful in resuscitation only if it is initiated soon after the onset of arrest. In the absence of in situ pacing wires or an indwelling transvenous or esophageal pacing catheter, TCP is the preferred method for temporary electrical cardiac pacing. Since 1992, the AHA Advanced Cardiac Life Support (ACLS) guidelines have recommended the early use of an external pacemaker in patients with symptomatic bradycardia or asystole.124 Since Zoll established TCP in 1952 as a clinically useful method of pacing adult patients during ventricular standstill (Stokes-Adams attacks) and bradycardia-associated hypotension, numerous anecdotal reports have supported its use for bradycardic or asystolic arrests.125 Zoll et al. reported successful inhospital resuscitation of 12 of 16 patients with hypotensive bradycardia or asystole if TCP was initiated within 5 minutes of the arrest.126 In contrast, if TCP was started between 5 and 30 minutes after the arrest, only 8 of 44 patients with either of these rhythms could be resuscitated.126 In two controlled clinical trials of prehospital TCP, no differences in the survival rate or success of resuscitation were observed in paced and nonpaced patients who had asystole or PEA.127,128 In patients with symptomatic bradycardia, TCP improved resuscitation and the survival rate.129 To date, the efficacy of TCP in resuscitation of children has not been studied. Beland et al. showed that effective TCP could be achieved in hemodynamically stable children during induction of anesthesia for heart surgery.130 They were successful in 53 of 56 pacing trials, and the patients experienced no complications.130 TCP is indicated for patients whose primary problem is impulse formation or conduction and who have preserved myocardial function. TCP is most effective in patients with sinus bradycardia or high-grade atrioventricular block with slow ventricular response who also have a stroke volume sufficient to generate a pulse. TCP is not indicated for patients in prolonged arrest because, in this situation, TCP usually results in electrical but not mechanical cardiac capture and its use may delay or interfere with other resuscitative efforts. To set up pacing, one electrode is placed anteriorly at the left sternal border and the other posteriorly just below the left scapula. Smaller electrodes are available for infants and children; adultsized electrodes can be used in children weighing more than 15 kg.130 Electrocardiographic leads should be connected to the pacemaker, the demand or asynchronous mode selected, and an ageappropriate heart rate used. The stimulus output should be set at zero when the pacemaker is turned on and then increased gradually until electrical capture is seen on the monitor. The output



CHAPTER 38

required for a hemodynamically unstable rhythm is higher than that for a stable rhythm in children in whom the mean stimulus required for capture was between 52 and 65 mA. After electrical capture is achieved, one must ascertain whether an effective arterial pulse is generated. If pulses are not adequate, other resuscitative efforts should be used. The most serious complication of TCP is induction of a ventricular arrhythmia.131 Fortunately, this complication is rare and may be prevented by pacing only in the demand mode. Mild transient erythema beneath the electrodes is common. Skeletal muscle contraction can be minimized by using large electrodes, a 40-ms pulse duration, and the smallest stimulus required for capture. Sedatives or analgesics may be necessary in the patient who is awake. If defibrillation or cardioversion is necessary, one must allow a distance of 2 to 3 cm between the electrode and paddles to prevent arcing of the current.

Pharmacology Adrenergic Agonists In 1963, only 3 years after the original description of closed-chest CPR, Pearson and Redding described the use of adrenergic agonists for resuscitation.132 They subsequently showed that early administration of epinephrine in a canine model of cardiac arrest improved the success rate of CPR. They also demonstrated that the increase in aortic diastolic pressure by administration of a-adrenergic agonists was responsible for the improved success of resuscitation. They theorized that vasopressors such as epinephrine were of value because the drug increased peripheral vascular tone, not because of a direct effect on the heart.133 Yakaitis et al. investigated the relative importance of a- and b-adrenergic agonist actions during resuscitation.134 Only 27% of dogs that received a pure b-adrenergic receptor agonist along with an a-adrenergic antagonist were resuscitated successfully compared with all of the dogs that received a pure a-adrenergic agonist and a b-adrenergic antagonist.134 Later studies reconfirmed this finding.135 Michael et al. demonstrated that the a-adrenergic effects of epinephrine result in intense vasoconstriction of the resistance vessels of all organs of the body, except those supplying the heart and brain.43 Because of the widespread vasoconstriction in nonvital organs, adequate perfusion pressure—and, thus, blood flow to the heart and brain—can be achieved despite the fact that cardiac output is very low during CPR.60 The increase in aortic diastolic pressure associated with epinephrine administration during CPR is critical for maintaining coronary blood flow and enhancing the success of resuscitation. Even though the contractile state of the myocardium is increased by use of b-adrenergic agonists in the spontaneously beating heart, b-adrenergic agonists actually may decrease myocardial blood flow during CPR by increasing intramyocardial wall pressure and vascular resistance. This decrease in myocardial blood flow could redistribute intramyocardial blood flow away from the subendocardium, increasing the likelihood of ischemic injury to this region.136 Moreover, evidence indicates that left ventricular end-diastolic pressure (LVEDP) rises with epinephrine use, reducing the overall impact of the vasoconstrictor effects of epinephrine on CPP. Tang et al. showed elevated LVEDP and decreased measures of diastolic performance in epinephrine-resuscitated rats after induced VF compared with phenylephrine-resuscitated animals or epinephrine-resuscitated animals who also received a b-blocker.137 Similar data were found by McNamara, who used a rat pup model

Physiologic Foundations of Cardiopulmonary Resuscitation

429

• BOX 38.1 a-Adrenergic vs b-Adrenergic

Agonist Effects a-Adrenergic Effects • • • •

Vasoconstrict peripheral vessels Maintain aortic diastolic pressure Improve coronary blood flow No metabolic stimulatory effect

b-Adrenergic Effects • • • • • •

Vasodilate peripheral vessels Decrease aortic diastolic pressure Increase cellular metabolic rate Positive inotrope Increase intensity of ventricular fibrillation Increase heart rate and/or dysrhythmias following resuscitation

of asphyxial arrest.138 LVEDP was increased and diastolic function indices were decreased with epinephrine compared with either saline solution alone or epinephrine combined with verapamil. These data imply that excessive b-adrenergic effects prevent the intracellular calcium reuptake during diastole that is required for myocardial relaxation. By its inotropic and chronotropic effects, b-adrenergic stimulation increases myocardial oxygen demand, which increases the risk of ischemic injury when superimposed on low coronary blood flow. This combination of increased oxygen demand by b-adrenergic agonists and decreased oxygen supply may damage an already ischemic heart, raising the question of whether a pure a-adrenergic agonist would be better than epinephrine, with its significant b-adrenergic effects (Box 38.1).139 The effects on energy utilization and oxygen supply not only have implications for the success of the initial resuscitation but also for the postresuscitation function of the myocardium. In an effort to address the balance of the risks and benefits of epinephrine, a large randomized controlled trial was conducted among five National Health Service ambulance companies in the United Kingdom.140 The PARAMEDIC2 trial enrolled 8014 adult patients with out-of-hospital cardiac arrest and randomized them to either standard-dose epinephrine or saline placebo. Overall survival was improved in the epinephrine group (3.2% vs. 2.4%), but there was no difference in rates of survival with favorable neurologic outcomes. This finding is likely due to the much higher rates of severe neurologic impairment in the epinephrine survivors and is similar to results found in observational studies.141 A meta-analysis that included this study showed improved survival to admission with epinephrine but no improvement with survival to discharge or good neurologic outcome.142 Timing of epinephrine administration may have an impact. Recent analyses of large resuscitation databases have shown that survival is greatest with early administration of epinephrine in both adult and pediatric patients.143–145 Another interesting finding from analyzing these large databases is that longer time periods between epinephrine doses than what is currently recommended may be associated with better survival to hospital discharge. However, as this was a retrospective analysis, it warrants further investigation.146 A number of studies have attempted to compare a-adrenergic agonists to epinephrine during CPR. Phenylephrine and methoxamine are two pure a-adrenergic agonists that have been used in animal models of CPR with success equal to that of epinephrine.60,134,136 More recently, vasopressin (discussed in depth later) has been studied as a noncatecholamine vasoconstrictor in the

SECTION IV



Pediatric Critical Care: Cardiovascular

management of patients who experience cardiac arrest.18 These agents cause peripheral vasoconstriction and increase aortic diastolic pressure, resulting in improved myocardial and cerebral blood flows. This effect results in a higher oxygen supply/demand ratio in the ischemic heart and a theoretical advantage over the combined a- and b-adrenergic agonist effects of epinephrine. These agonists, as well as vasopressors such as vasopressin, have been used successfully for resuscitation.18,133,147 These drugs maintain blood flow to the heart during CPR with similar performance to epinephrine. In an animal model of VF cardiac arrest, a resuscitation rate of 75% was reported for both epinephrine- and phenylephrine-treated groups. In this study, the ratio of endocardial to epicardial blood flow was lower in the group treated with epinephrine, suggesting the presence of subendocardial ischemia.60 However, studies of this kind are difficult to interpret because of the inability to measure the degree of a-receptor activation by the different vasopressors. The higher subendocardial blood flow in the phenylephrine group may have been the result of less a-receptor activation.148–150 Moreover, some investigators have questioned the merits of using a pure a-adrenergic agonist during CPR. Although the inotropic and chronotropic effects of b-adrenergic agonists may have deleterious hemodynamic effects during CPR administered for VF, increases in both heart rate and contractility increase cardiac output when spontaneous coordinated ventricular contractions are achieved. Cerebral blood flow during CPR, like coronary blood flow, depends on peripheral vasoconstriction and is enhanced by use of a-adrenergic agonists. This action produces selective vasoconstriction of noncerebral peripheral vessels to areas of the head and scalp without causing cerebral vasoconstriction.60 As with myocardial blood flow, pure a-agonist agents are as effective as epinephrine in generating and sustaining cerebral blood flow during CPR in adult animal models and in infant models.60,147 No difference in neurologic deficits was found at 24 hours after cardiac arrest between animals receiving either epinephrine or phenylephrine during CPR.151 Animal studies have shown transient improvements in cerebral oxygenation,152 and similarly in a prospective study of 36 adult patients, a bolus of epinephrine showed a small increase in cerebral oxygenation within the first 5 minutes after administration.153 Analogous to the heart, b-adrenergic agonists could increase cerebral oxygen uptake if a sufficient amount of drug crosses the blood-brain barrier during or after resuscitation. In addition, adrenergic agonists may vasoconstrict or dilate cerebral vessels depending on the balance between a- and b-adrenergic receptors. Epinephrine and phenylephrine had similar effects on cerebral blood flow and metabolism, maintaining normal cerebral oxygen uptake for 20 minutes of CPR in dogs. This finding implies that cerebral blood flow was high enough to maintain adequate cerebral metabolism and that b-receptor stimulation did not increase cerebral oxygen uptake, despite the fact that the combined effects of brain ischemia and CPR can increase the permeability of the blood-brain barrier to drugs used during CPR or when enzymatic barriers to vasopressors (e.g., by monoamine oxidase) are overwhelmed during tissue hypoxia. Mechanical disruption of the barrier could occur during chest compressions by large fluctuations in cerebral venous and arterial pressures or as a result of hyperemia, the large increase in cerebral blood flow that occurs during the early reperfusion period when the cerebral vascular bed is maximally dilated following resuscitation, particularly if systemic hypertension occurs.154 No blood-brain barrier permeability changes during CPR immediately after resuscitation or 4 hours



&RQWURO &35 /RQJ&35 PLQSRVW KSRVW

.L P/JPLQ

430









• Fig. 38.9

3RQV

'LH

0&$

​Transfer coefficient (Ki) of a-aminoisobutyric acid for pons, diencephalon (DIE), and middle cerebral (MCA) artery regions. Control group: 8 minutes ischemia and 10 minutes cardiopulmonary resuscitation (CPR); 8 minutes ischemia and 40 minutes cardiopulmonary resuscitation; 3 minutes after resuscitation; 4 hours after resuscitation. Group 5 in each region, *P , .05, different from group 1 by one-way analysis of variance and Dunnett test for all three regions. (Modified from Schleien CL, Koehler RC, Schaffner DH, et al. Blood-brain barrier disruption after cardiopulmonary resuscitation in immature swine. Stroke. 1991;22:477.)  



after resuscitation were found in adult dogs.154 However, after 8 minutes of cardiac arrest and 6 minutes of CPR in piglets, the blood-brain barrier was permeable to the small neutral amino acid a-aminoisobutyric acid 4 hours after cardiac arrest (Fig. 38.9).155,156 The increase in permeability could be prevented by prearrest administration of conjugated superoxide dismutase and catalase, indicating a role of oxygen free radicals in the pathogenesis of this injury to the blood-brain barrier (Fig. 38.10).157 These endothelial membrane changes frequently were associated with the presence of intravascular polymorphonuclear and monocytic leukocytes.158 Whether leukocytes disrupt the blood-brain barrier by release of toxic substances, such as oxygen free radicals or proteases, or appear in the postischemic microvessels as an epiphenomenon of a more important derangement is unknown (Fig. 38.11).

Vasopressin The role of vasopressin as a noncatecholamine vasoconstrictor in the management of patients who experience cardiac arrest has received a great deal of interest. Work by Lindner in Europe and Landry in the United States had established sufficient evidence of efficacy for its use to be included in the 2010 AHA Guidelines for Cardiopulmonary Resuscitation and Emergency Cardiovascular Care.112,159 Subsequent adult studies comparing standard-dose epinephrine with vasopressin alone or in combination with standard-dose epinephrine have showed that vasopressin offered no advantage in ROSC or survival to discharge.160 It has subsequently been removed from the 2015 AHA guidelines. However, increasing evidence indicates that vasopressin is a useful agent in the management of shock of multiple etiologies. Therefore, it may have a role in postresuscitation management.

4 AIB transfer coefficien (µL/g per min)

1

Midbrain

Spinal cord

B

Pons

0

Posteriormiddle border

Anteriormiddle border

Posterior cerebral artery

2

Thalamus

A

Middle cerebral artery

Hippocampus

0

Anterior cerebral artery

1

3

Medulla

2

431

Group 1 Group 2 Group 3

Cerebellum

Group 1 Group 2 Group 3

Caudate nucleus

AIB transfer coefficien (µL/g per min)

3

Physiologic Foundations of Cardiopulmonary Resuscitation

Superior colliculus



CHAPTER 38

• Fig. 38.10





​(A) Transfer coefficient of a-aminoisobutyric acid (AIB) from plasma to brain in hippocampus, caudate nucleus, and primary supply and border regions of cerebral arteries in nonischemic time controls (group 1, n 5 5), ischemia group treated with polyethylene glycol (PEG; group 2, n 5 8), and ischemia group treated with PEG-superoxide dismutase and PEG-catalase (group 3, n 5 8). Error bars represent standard error of the mean (SEM). *P , .05 between groups 2 and 3 by Mann-Whitney U test. (B) Transfer coefficient of AIB from plasma to brain in caudal brain regions in nonischemic time controls (group 1, n 5 5), ischemia group treated with PEG (group 2, n 5 8), and ischemia group treated with PEG-superoxide dismutase and PEG-catalase (group 3, i5 8). Error bars represent SEM. *P , .05 between groups 2 and 3 by Mann-Whitney U test. (Modified from Schleien CL, Eberle B, Schaffner DH, et al. Reduced blood-brain barrier permeability after cardiac arrest by conjugated superoxide dismutase and catalase in piglets. Stroke. 1994;25[9]:1830–1834.)

• Fig. 38.11



​Transmission electron micrograph of an infant piglet brain 4 hours after 8 minutes of cardiac arrest and 6 minutes of cardiopulmonary resuscitation (magnification 35000). An intravascular leukocyte, which has the morphologic features of a monocyte, is adherent to the endothelial surface of a venule and appears to be occluding the lumen. The luminal surface of the endothelial cell contains membrane blebs and discontinuities. (From Caceres MJ, Schleien CL, Kuluz JW, et al. Early endothelial damage and leukocyte accumulation in piglet brains following cardiac​ arrest. Acta Neuropathol. 1995;90[6]:582.)

Arginine vasopressin is a short peptide hormone secreted by the posterior pituitary gland in response to changes in tonicity and in effective intravascular volume, signaled primarily via baroreceptor unloading in the aorta. Severe shock is the most potent stimulus to vasopressin secretion. Serum levels 20- to 200-fold

higher than normal may be found immediately after cardiac arrest and in other severe shock states. Despite these observations, lower than expected vasopressin levels have been found in some patients with profound shock, and patients dying of cardiac arrest have been found to have significantly lower vasopressin levels than do survivors.161–165 The cause of lower than expected vasopressin levels in some patients is unclear. Depletion of vasopressin stores may be a potential mechanism. Dogs subjected to profound hemorrhagic shock have an early massive elevation of vasopressin levels immediately after the event, followed by a depression below expected levels within 1 hour of the insult. Severe depletion of vasopressin stores from the posterior hypophysis was noted.166 These animals developed a catecholamine-refractory vasodilatory shock that responded dramatically to low doses of vasopressin.167 These observations have led to an exploration of the use of vasopressin in both cardiac arrest and shock states. Vasopressin is an extremely potent vasoconstrictor. Its effects on vascular tone are primarily mediated through interaction with a specific G protein–coupled receptor referred to as the V1a-receptor, which is distributed widely throughout vascular beds.165 Of note, the V1a-receptor is linked to the same second messenger system as the a-adrenergic receptor that mediates vasoconstriction through alteration of intracellular calcium levels. However, in the pulmonary circulation, vasopressin activation of V1-receptors mediates the release of nitric oxide and causes pulmonary vasodilatation. Vasopressin also interacts with its V2-receptor, which regulates aquaporin expression on the renal collecting duct epithelium. Stimulation of the V2-receptor occurs at substantially lower levels than those required to activate the V1a-receptor. Vasopressin use during resuscitation has been studied in animals and humans. In an adult porcine model of VF, vasopressin

432

SECTION IV



Pediatric Critical Care: Cardiovascular

at a dose of 0.8 mg/kg was found to be superior to the maximally effective dose of epinephrine 200 mg/kg in restoring LV myocardial blood flow, increasing diastolic CPP and total cerebral blood flow as well as rates of ROSC.138 Moreover, the duration of the effect was sustained for 4 minutes, compared with 1.5 minutes for epinephrine.18 Adverse effects noted in the postresuscitation phase included decreased renal and adrenal blood flows and reduced cardiac output.168,169 In a pediatric porcine model of cardiac arrest, vasopressin at a dose of 0.8 mg/kg was not as effective as epinephrine at 200 mg/kg in restoring LV myocardial blood flow or achieving ROSC.170 Only 1 of 6 animals achieved ROSC compared with 6 of 6 in the epinephrine group. A combination group that received both epinephrine at 45 mg/kg and vasopressin at 0.8 mg/kg fared better with ROSC in 4 of 6 animals. Possible explanations for the difference between adult and juvenile animals include different doseresponse curves for the two drugs, failure of maturation of vasopressin receptors, a different distribution of vasopressin receptors, or the different experimental model. In an initial small, randomized clinical trial of vasopressin compared with epinephrine for refractory VF, the rate of achieving ROSC was higher in the vasopressin group.171 A large multicenter randomized trial of vasopressin for cardiac arrest in adults has been reported.172 More than 1200 patients were randomly assigned in the field to receive 2 doses of either 40 international units (IU) of vasopressin or 1 mg of epinephrine followed by additional treatment with epinephrine, if necessary. In patients in whom ROSC was not achieved after two doses of medication, a third dose of medication such as epinephrine could be added at the resuscitating physician’s discretion. Initial dose vasopressin was equivalent to epinephrine in achieving survival to both hospital admission and discharge in patients with either PEA or VF. In patients with asystole, vasopressin was superior to epinephrine in achieving both survival to admission and discharge, although intact neurologic outcome was not improved. In the group receiving a third dose of medication such as epinephrine, survival was greater in the vasopressin group. In a study of 200 patients with in-hospital cardiac arrest, patients were randomly assigned to receive either 1 mg of epinephrine or 40 IU of vasopressin. Again, no statistical difference in survival to 1 hour or to hospital discharge was found between groups or subgroups.173 The results of these studies led to the classification of evidence for vasopressin for use in adults as indeterminate in the 2010 AHA guidelines.112 Subsequently, a meta-analysis of 10 randomized trials, including a total of 6120 adult patients, showed that the use of vasopressin was neither beneficial nor harmful in an unselected patient population in terms of ROSC, survival to hospital admission or discharge, or favorable neurologic outcome. However, chance of ROSC was significantly higher in inhospital cardiac arrest patients when vasopressin was used.174 The published experience with vasopressin in children who experience cardiac arrest is limited. The first case series of vasopressin use in CPR reported the outcome of four children with six prolonged refractory cardiac arrests that were unresponsive to standard resuscitation efforts.175 Each child received one or more bolus doses of vasopressin (0.4 U/kg) as rescue therapy. In all children, the initial rhythm was a form of PEA that deteriorated to asystole in four of six events. Three children had ROSC for more than 60 minutes, including one child with asystole. Two children survived for more than 24 hours and one survived to hospital discharge. A review of a national registry of in-hospital CPR showed that patients who received vasopressin had a lower incidence of ROSC

greater than 20 minutes (22 [34%] of 64) than patients who did not receive the medication (675 [55%] of 1229).176 The association of poor outcome with vasopressin persisted even with multivariate analysis with logistic regression to attempt to control for other factors that might affect ROSC. The effect of vasopressin in pediatric arrest that was refractory to an initial epinephrine dose was evaluated in a pilot study. Patients were given vasopressin after an initial dose of epinephrine and were compared with a retrospective matched cohort of patients who experienced cardiopulmonary arrest that required greater than two doses of a vasopressor, not including vasopressin. Ten patients were enrolled; while there was an increased 24-hour survival, there was no difference in ROSC, survival to hospital discharge, or favorable neurologic status at discharge.177 The current evidence examines the use of vasopressin only as a potential alternative when standard therapies, such as epinephrine, fail to cause ROSC. Unfortunately, variables such as dosing, timing of vasopressin infusion, or pediatric risk of mortality scores have not been controlled for in these studies. No double-blinded, randomized controlled studies have been performed; thus, no firm recommendations are available concerning the use of vasopressin for CPR in infants and children. The current recommended dose of vasopressin for adults in cardiac arrest is 40 IU. No data comparing this dose to other doses are available, and concern exists regarding postresuscitation complications related to this dose. We have selected 0.5 IU/kg as the standard for cardiac arrest. Further data are needed before more definitive dosing recommendations can be made. Use of vasopressin in postresuscitation management may be considered. A relative vasopressin deficiency has been noted in a number of shock states, including hemorrhage, sepsis, and postCPB, as well as in patients who have unsuccessful resuscitations. In these settings, shock may be refractory to catecholamines. These patients may respond to a vasopressin infusion, allowing the weaning of high-dose catecholamines. Although a role in the postresuscitation setting has not been demonstrated based on the data related to refractory shock, consideration of the use of vasopressin for refractory hypotension may be appropriate. Additionally, the additive effects of combination drug therapy for adults with cardiac arrest may be most beneficial. In a recent randomized, double-blind placebo control study investigating vasopressin plus epinephrine or saline placebo plus epinephrine with or without methylprednisolone, the group receiving combination therapy with vasopressin-epinephrine-methylprednisolone with CPR resulted in improved survival to hospital discharge with improved neurologic status.178,179

High-Dose Epinephrine The physiologic responses of animals and humans to higher doses of epinephrine include higher cerebral blood flow, increased myocardial and submyocardial blood flow, improved oxygen delivery relative to oxygen consumption, and less depletion of myocardial adenosine triphosphate (ATP) stores with more rapid repletion of phosphocreatine.149,180–184 Contrary results, with increased myocardial oxygen consumption and decreased myocardial blood flow, have been demonstrated during CPR following VF cardiac arrest.41,185 In a piglet model, high-dose epinephrine (HDE) produced lower myocardial blood flow than standard-dose epinephrine (SDE).180 In neonatal lambs following asphyxia-induced bradycardia, HDE resulted in a higher heart rate but lower stroke volume and cardiac output.186 Additionally, prolonged peripheral



CHAPTER 38

vasoconstriction and excessive doses of epinephrine may delay or impair reperfusion of systemic organs, particularly the kidneys and gastrointestinal tract. Studies regarding survival of patients who were given HDE have been contradictory. In out-of-hospital patients who experienced cardiac arrest, HDE produced higher aortic diastolic pressure during CPR and increased the rate of ROSC compared with standard doses of epinephrine. Gonzalez et al. demonstrated a dose-dependent increase in aortic blood pressure by epinephrine in patients who failed to respond to prolonged resuscitative efforts.187 Paradis et al.187–189 showed that HDE increased aortic diastolic pressure and improved the rate of successful resuscitation in patients in whom ACLS protocols had failed. This group also reported on a series of 20 children treated with HDE and compared them with 20 historic control subjects consisting of children with cardiac arrest treated with SDE.190 They reported that 14 of the children in the HDE group had ROSC, 8 survived to hospital discharge, and 3 were neurologically intact. There were no survivors in the SDE comparison group. Other centers have claimed that higher-than-standard doses of epinephrine during CPR in children improve the hemodynamics and increase the success of CPR. However, no one has provided any valid data suggesting that HDE improves survival beyond the immediate postresuscitation period.91,189,191,192 Based on these studies, the 1992 AHA guidelines for pediatric advance life support recommended HDE if an initial SDE failed to resuscitate the child. Three large multicenter studies were subsequently published that dampened enthusiasm for the use of HDE. Stiell et al. studied 650 cardiac arrest adult patients who were randomly assigned to receive either an SDE or HDE (7 mg) protocol.193 No differences were observed between the groups with regard to 1-hour survival (23% vs. 18%), rate of hospital discharge (5% vs. 3%), or neurologic outcome. Brown et al. reported on 1280 cardiac arrest adult patients who received either SDE (0.02 mg/kg) or HDE (0.2 mg/kg).24 Again, no differences in ROSC, short-term survival, survival to hospital discharge, or neurologic outcome were observed between the two groups of patients. In a study of 816 adults, Callaham et al. reported a higher ROSC in the HDE group.194 However, there were no differences in the rate of hospital discharge or ultimate survival of these patients. In addition to these studies, a specific pediatric animal study was published that failed to demonstrate a clear survival benefit for HDE, although the occurrence of ROSC appeared to be greater.185 The 2000 AHA guidelines changed the recommendation for HDE to an option for second and subsequent doses of epinephrine. A prospective, randomized, double-blind clinical trial of HDE in 68 pediatric inpatients was reported by Perondi et al.15 ROSC for more than 20 minutes was achieved in 15 of 34 patients who received HDE but in only 8 of 34 patients who received SDE (P 5 .07). However, survival to 24 hours occurred in only two of the HDE group versus seven of the SDE group (P 5 .05). In the group that experienced an asphyxial arrest, none of 12 treated with HDE were alive at 24 hours, whereas 7 of 18 patients in the SDE group survived. Four survived to hospital discharge, and two patients were neurologically normal.15 A meta-analysis of epinephrine use in adult cardiac arrest showed no difference in survival to discharge or neurologic outcome when using high dose over standard dose.195 These studies reinforced concerns that HDE may account for some of the adverse effects that occur after resuscitation and is the basis of the 2015 AHA guidelines’ recommendation against the use of HDE during CPR.156,196,197

Physiologic Foundations of Cardiopulmonary Resuscitation

433

Atropine Atropine is a parasympatholytic agent that acts by blocking cholinergic stimulation of the muscarinic receptors of the heart, which usually results in an increase in the sinus rate and shortening of the atrioventricular node conduction time. Atropine may also activate latent ectopic pacemakers. It has little effect on systemic vascular resistance, myocardial perfusion pressure, or contractility.198 Atropine is indicated for treatment of asystole, PEA, bradycardia associated with hypotension, second- and third-degree heart block, and slow idioventricular rhythms. In children who present in cardiac arrest, sinus bradycardia and asystole are the most common initial rhythms, which make atropine useful as a first-line drug. Atropine is particularly effective in clinical conditions associated with excessive parasympathetic tone. The recommended dose of atropine is 0.02 mg/kg, with a minimum dose of 0.1 mg and a maximum dose of 0.5 mg/dose. Smaller doses than 0.1 mg, even in small infants, may result paradoxically in bradycardia because of a central stimulatory effect on the medullary vagal nuclei by a dose that is too low to provide anticholinergic effects on the heart, although this phenomenon has come under debate.198a,198b Atropine may be given by any route, including intravenous, endotracheal, interosseous, intramuscular, and subcutaneous. Its onset of action occurs within 30 seconds, and its peak effect occurs between 1 and 2 minutes after an intravenous dose. The recommended adult dose is 0.5 mg every 5 minutes until the desired heart rate is obtained up to a maximum of 3 mg. For asystole, 1 mg is given intravenously and repeated every 5 minutes if asystole persists. Full vagal blockade usually is obtained with a dose of 2 mg in adults. Because of its parasympatholytic effects, atropine should not be used in patients in whom tachycardia is undesirable. In patients after myocardial infarction or ischemia with persistent bradycardia, atropine should be used in the lowest dose possible to increase heart rate. Using the lowest possible dose will limit tachycardia, a potent contributor to increased myocardial oxygen consumption, which could lead to VF. In addition, atropine should not be used in patients with pulmonary or systemic outflow tract obstruction or idiopathic hypertrophic subaortic stenosis because tachycardia decreases ventricular filling and lowers cardiac output in this setting.

Sodium Bicarbonate The administration of sodium bicarbonate results in an acid-base reaction in which bicarbonate combines with hydrogen to form carbonic acid, which dissociates into water and carbon dioxide. Because of the generation of carbon dioxide, adequate alveolar ventilation must be present to achieve the normal buffering action of bicarbonate. Use of sodium bicarbonate during CPR remains controversial because of its potential adverse effects and the lack of evidence showing any benefit from its use during CPR.199,200 Sodium bicarbonate is indicated for correction of significant metabolic acidosis, especially when signs of cardiovascular compromise are present. Acidosis itself may have a number of negative effects on the circulation, including depression of myocardial function by prolonging diastolic depolarization, depressing spontaneous cardiac activity, decreasing the electrical threshold for VF, decreasing the inotropic state of the myocardium, and reducing the cardiac response to catecholamines. Acidosis also decreases systemic vascular resistance and attenuates the vasoconstrictive

434

SECTION IV



Pediatric Critical Care: Cardiovascular

response of peripheral vessels to catecholamines. This effect is contrary to the desired effect during CPR. In addition, particularly in patients with a reactive pulmonary vascular bed, pulmonary vascular resistance is inversely related to pH. Rudolph and Yuan observed a twofold increase in pulmonary vascular resistance in calves when pH was lowered from 7.4 to 7.2 under normoxic conditions.201 Therefore, correction of even mild acidosis may be helpful in resuscitating patients who have the potential for increased right-to-left shunting through a cardiac septal defect, patent ductus arteriosus, or aortic-to-pulmonary shunt during periods of elevated pulmonary vascular resistance. Multiple adverse effects of bicarbonate administration include metabolic alkalosis, hypercapnia, hypernatremia, and hyperosmolality. All of these adverse effects are associated with a high mortality rate. Alkalosis causes a leftward shift of the oxyhemoglobin dissociation curve, thus, impairing release of oxygen from hemoglobin to tissues at a time when oxygen delivery already may be low. Alkalosis can result in hypokalemia, by enhancing potassium influx into cells, and ionic hypocalcemia, by increasing protein binding of ionized calcium. Hypernatremia and hyperosmolality may decrease tissue perfusion by increasing interstitial edema in microvascular beds. The marked hypercapnic acidosis that occurs during CPR on the venous side of the circulation, including the coronary sinus, may be worsened by administration of bicarbonate.202,203 Myocardial acidosis during cardiac arrest is associated with decreased myocardial contractility. The mean venoarterial partial pressure of carbon dioxide difference was 24 6 15 mm Hg in 5 patients during CPR and actually increased from 16 to 69 mm Hg in 1 patient after administration of bicarbonate.204 Another group showed a mean difference of 42 mm Hg between partial pressure of carbon dioxide in mixed venous blood and partial pressure of arterial carbon dioxide (Paco2) during CPR. Paradoxical intracellular acidosis after bicarbonate administration is possible because of rapid entry of carbon dioxide into cells with a slow egress of hydrogen ion out of cells. Paradoxical intracellular acidosis in the central nervous system after bicarbonate administration has been proposed but not definitively shown. In neonatal rabbits recovering from hypoxic acidosis, bicarbonate administration increased both arterial pH and intracellular brain pH as measured by nuclear magnetic resonance spectroscopy.205 In another study, intracellular brain ATP concentration in rats did not change during severe intracellular acidosis in the brain produced by extreme hypercapnia.206 The rats who maintained ATP concentration even in the face of severe brain acidosis had no functional or histologic differences from normal control subjects. Using nuclear magnetic resonance spectroscopy of the brain in dogs during cardiac arrest and CPR, intracellular brain pH decreased to 6.29, with total depletion of brain ATP after 6 minutes of cardiac arrest. Following effective CPR, ATP levels rose to 86% of prearrest levels and to normal by 35 minutes of CPR despite ongoing peripheral arterial acidosis (Fig. 38.12).105 However, cerebral pH decreased in parallel with blood pH when CPR was started immediately after arrest. Bicarbonate administration ameliorated and did not worsen the cerebral acidosis, indicating that the blood-brain pH gradient is maintained during CPR.207 A Cochrane study looking at the use of empirical sodium bicarbonate administration versus placebo in out-of-hospital cardiac arrests in 874 adults found no difference in survival to the hospital.208 Levy reviewed more than 30 animal studies evaluating the efficacy of sodium bicarbonate administration during CPR.209 Among studies with survival as the primary outcome, four showed

Minute 35 of CPR

D

Minute 6 of CPR

C

Minute 6 of V-Fib

B

PCr Pi PME PDE Control

A 10

• Fig. 38.12

0

–10

–20 PPM

31 ​ P magnetic resonance spectroscopy spectra from in situ dog brain during vest cardiopulmonary resuscitation (CPR) after a 6-minute delay in the onset of CPR from time of arrest. Each spectrum was acquired in 1 minute. The frequency of the inorganic phosphate (Pi) peak is pH dependent. Note complete absence of adenosine triphosphate (ATP) and phosphocreatine (PCr), and pHi 5 6.28 in trace B after 6 minutes of ventricular fibrillation (v-fib) without CPR. After 6 minutes of CPR (trace C), ATP is more than 85% recovered, but pH is only 6.61. After 35 minutes of CPR (trace D), pHi has returned to 7. PDE, phosphodiesters; PME, phosphomonoesters; PPM, parts per million. (Modified from Eleff SM, Schleien CL, Koehler RC et al: Brain bioenergetics during cardiopulmonary resuscitation in dogs, Anesthesiology 1992;76:77.)  

benefit and seven did not. When assessing myocardial performance, 12 studies concluded that sodium bicarbonate worsened performance, 2 studies showed no difference, and no study showed benefit. When reviewing 19 retrospective human adult studies examining mortality rates, 8 suggested a deleterious effect of sodium bicarbonate, 11 showed no difference in outcomes, and none showed benefit.209 A large prospectively conducted observational study of adult out-of-hospital cardiac arrest patients demonstrated outcomes for those who received sodium bicarbonate during resuscitation.210 Out of 13,865 patients, 5165 (37%) received sodium bicarbonate. Using 1:1 propensity matching to account for confounding variables, sodium bicarbonate administration was associated with worse survival to discharge and with a lower likelihood of achieving a favorable neurologic outcome. In a large retrospective study of pediatric patients with inhospital cardiac arrest, it was found that survivors were less likely to receive sodium bicarbonate than nonsurvivors. However, nonsurvivors were also observed to have a longer CPR duration as well as more doses of calcium, vasopressin, and epinephrine.9 A subsequent retrospective study by Raymond et al. examined sodium bicarbonate use after the institution of the 2010 AHA guidelines. While they found that sodium bicarbonate use for inhospital cardiac arrest had been decreasing, its use was still associated with decreased survival at 24 hours and hospital discharge when given outside current PALS recommendations.211 Due to potential adverse effects of bicarbonate, the indications for its use are limited to cardiac arrest associated with hyperkalemia, patients with preexisting metabolic acidosis, and after approximately 10 minutes of CPR. When Paco2 and pH are known,



CHAPTER 38

the dose of bicarbonate to correct the pH to 7.4 is calculated using the following equation: Sodium bicarbonate (mEq) 5 0.3 3 weight (kg) 3 Base deficit Because of its possible adverse effects and the large venous to arterial carbon dioxide gradient that develops during CPR, we recommend giving half the dose that would be given based on a volume of distribution of 0.6. If blood gases are not available, the initial dose is 1 mEq/kg, followed by 0.5 mEq/kg every 10 minutes of ongoing arrest. Alveolar ventilation must be maintained because of the generation of carbon dioxide and can be assessed only by serial measurements of arterial blood gases and pH. ETCO2 monitoring is useful during CPR because it provides important information regarding both pulmonary and cardiac function. ETCO2 is measured instantaneously in the exhaled gas of every breath. In the absence of lung disease, ETCO2 correlates closely with Paco2 provided that pulmonary blood flow is at least 20% to 25% of normal. As a respiratory monitor, ETCO2 analyzers accurately distinguish a tracheal (ETCO2 .10) from an esophageal (ETCO2 ,5) intubation in infants and children.212–217 Because measurements are made with every breath, dislodgment of the endotracheal tube from the trachea can be identified immediately. When cardiac output is extremely low, as occurs during ineffective CPR, delivery of carbon dioxide to the lungs is so limited that the total amount exchanged across the alveolar-capillary membrane is markedly reduced. In this situation, the measured ETCO2 is very low even when Paco2 is elevated. As cardiac output increases, ETCO2 increases and the difference between end-tidal and arterial CO2 becomes smaller.218 ETCO2 has been correlated with CPP, the critical parameter for resuscitation of the heart.218 However, a low ETCO2 may occur in the presence of adequate cardiac output during CPR after the administration of epinephrine because of its ability to increase intrapulmonary shunting.191,219,220 In this case, a low ETCO2 underestimates cardiac output. Other causes of low ETCO2 include airway obstruction, tension pneumothorax, pericardial tamponade, pulmonary embolism, hypothermia, severe hypocapnia (which occurs commonly with overaggressive hand ventilation), and esophageal intubation. In a large prospective observational study of adult in-hospital arrests, Sutton et al. showed that an ETCO2 greater than 10 mm Hg was associated with improved ROSC and survival.221 Levine et al. monitored ETCO2 in 150 adults with an out-of-hospital cardiac arrest who had electrical activity but no pulse.222 They found that after 20 minutes of ACLS, an ETCO2 level of 10 mm Hg successfully predicted survival to hospital admission with a sensitivity, specificity, and positive- and negative-predictive value of 100%. Grmec and Klemen prospectively studied ETCO2 as a prognostic indicator for outcomes in adult resuscitation and found that using an initial, average, and final ETCO2 level of 10 mm Hg identified 100% of patients who were successfully resuscitated, with specificities of 74%, 90%, and 81%, respectively.223 The 2015 AHA guidelines state that the use of ETCO2 as an indicator of cardiac output may be useful in adults (evidence class IIb).112 Pediatric data are limited at this time. One small prospective study of pediatric arrests evaluated whether an ETCO2 greater than 20 mm Hg during resuscitation was associated with survival but found no difference.224 There have been no human trials studying whether titrating resuscitation efforts to a specific number can affect clinical outcomes. In a piglet model of CPR, it was found that CPR guided by a target ETCO2 was as effective as CPR guided by depth monitor, video monitor, and verbal feedback. While no specific

Physiologic Foundations of Cardiopulmonary Resuscitation

435

value has yet to be established, given the noninvasive nature of ETCO2 monitoring and extrapolating from adult data,225 maintaining an ETCO2 of greater than 10 to 15 mm Hg through the use of ETCO2 monitoring is recommended during pediatric arrests.159

Other Alkalinizing Agents A number of other alkalinizing agents have been used experimentally in animals and humans. However, none have demonstrated any real advantages over sodium bicarbonate. Carbicarb, a solution of equimolar amounts of sodium bicarbonate and sodium carbonate, corrects metabolic acidosis without many of the adverse effects of sodium bicarbonate.76 The buffering action of sodium carbonate occurs by consumption of carbon dioxide with generation of bicarbonate ion, as illustrated in the following equation: Na2CO3 1 CO2 1 H2O 5 2HCO32 1 2Na1 During CPR, Carbicarb administration resulted in a greater increase in arterial pH and smaller increases in Paco2, lactate, and serum osmolality in animals.76,226,227 However, Carbicarb was not superior to sodium bicarbonate when used for hypovolemic shock in rats.228 Dichloroacetate (DCA) increases the activity of pyruvate dehydrogenase, which facilitates the conversion of lactate to pyruvate.229 When administered to patients with lactic acidosis, DCA decreased lactate concentration by half and increased bicarbonate concentration and pH.230 DCA improved cardiac output in other studies, possibly by increasing myocardial metabolism of lactate and carbohydrate.231,232 DCA did not improve outcome when compared with sodium bicarbonate in a multicenter trial of patients with lactic acidosis.233 Tromethamine (THAM; tris-hydroxymethyl-aminomethane) is an organic amine that combines with hydrogen ion, causing CO2 and H2O to combine to form bicarbonate and hydrogen ion. A dose of 3 mL/kg should raise the bicarbonate concentration by 3 mEq/L. Adverse effects of THAM include hyperkalemia, hypoglycemia, and acute hypocarbia, resulting in apnea. In addition, peripheral vasodilatation may occur after administration of THAM during CPR, which is an undesirable effect. THAM is contraindicated in patients with renal failure.

Calcium Recommendations for the use of calcium in CPR are restricted to a few specific situations: hypocalcemia, hyperkalemia, hypermagnesemia, and calcium channel blocker overdose. These restrictions are based on the possibility that exogenously administered calcium may worsen ischemia/reperfusion injury. Intracellular calcium overload occurs during cerebral ischemia by the influx of calcium through voltage- and agonist-dependent (e.g., N-methyl-d-aspartate) calcium channels. Calcium plays an important role in the process of cell death in many organs, possibly by activating intracellular enzymes such as nitric oxide synthase, phospholipases A and C, and others.234,235 Calcium channel blockers improve blood flow and function after ischemia to the heart, kidney, and brain.236–238 Calcium channel blockers also raise the threshold of the ischemic heart to VF.239 For these reasons, it appears that the recommended restrictions for use of calcium during CPR are well founded. On the other hand, no studies have shown that elevation of plasma calcium concentration, which occurs after calcium administration,

436

SECTION IV



Pediatric Critical Care: Cardiovascular

worsens outcome of cardiac arrest. Because the normal ratio of extracellular to intracellular calcium is on the order of 1000:1 to 10,000:1, it seems unlikely that the rate of influx of calcium into cells would be influenced by a relatively small increase in its extracellular concentration. The calcium ion is essential in myocardial excitation-contraction coupling, in increasing ventricular contractility, and in enhancing ventricular automaticity during asystole. Ionized hypocalcemia is associated with decreased ventricular performance and peripheral blunting of the hemodynamic response to catecholamines.240,241 In addition, severe ionized hypocalcemia has been documented in adults experiencing out-of-hospital cardiac arrest (mean Ca, 0.67 mmol/L), during sepsis, and in animals during prolonged CPR.241–243 Thus, patients at risk for ionized hypocalcemia should be identified and treated expeditiously. Both total and ionized hypocalcemia may occur in patients with chronic or acute disease. Total body calcium depletion leading to total serum hypocalcemia occurs in patients with hypoparathyroidism, DiGeorge syndrome, renal failure, pancreatitis, and long-term use of loop diuretics. Ionized hypocalcemia occurs after massive or rapid transfusion of blood products, a result of citrate and other preservatives in stored blood products binding calcium. The magnitude of hypocalcemia in this setting depends on the rate of blood administration, total dose, and hepatic and renal function of the patient. Administration of 2 mL/kg per minute of citrated whole blood causes a significant decrease in ionized calcium concentration in anesthetized patients. The pediatric dose of calcium chloride for resuscitation is 20 mg/kg. The adult dose is 500 to 1000 mg. Calcium gluconate is as effective as calcium chloride in raising ionized calcium concentration during CPR. Calcium gluconate is given at a dose of 60 mg/kg, with a maximum dose of 3 g in pediatric patients. Calcium should be given slowly through a large-bore, free-flowing intravenous line, preferably a central venous line. Severe tissue necrosis occurs when calcium infiltrates into subcutaneous tissue. When administered too rapidly, calcium may cause bradycardia, heart block, or ventricular standstill. Srinivasan et al. reviewed 1477 consecutive pediatric cardiopulmonary events submitted to the National Registry of Cardiopulmonary Resuscitation and reported on the prevalence of calcium administration.244 Calcium was given in 659 of these events. Calcium was more likely to be used in pediatric facilities, ICUs, in the setting of recent cardiac surgery, CPR performed for more than 15 minutes, asystole, and concurrently with other advanced life support medications. After controlling for confounding factors, calcium administration during CPR was independently associated with poor survival to discharge and unfavorable neurologic outcomes. They found that 21% of patients survived to hospital discharge when calcium was used, compared with 44% who survived when calcium was not used. Only 15% of patients had a favorable neurologic outcome when calcium was used, compared with 35% with a favorable outcome when calcium was not administered.244

Glucose Administration of glucose during CPR should be restricted to patients with documented hypoglycemia because of the possible detrimental effects of hyperglycemia on the brain during or following ischemia. Myers found that infant monkeys receiving glucose before cardiac arrest were more likely to develop seizures, prolonged coma, and brain death with cerebral necrosis than were

those that received saline solution.245 Siemkowicz and Hansen confirmed this finding when they demonstrated that after 10 minutes of global brain ischemia, the neurologic recovery of hyperglycemic rats was worse than that of normoglycemic control subjects.246 The mechanism by which hyperglycemia exacerbates ischemic neurologic injury may be increased production of lactic acid in the brain by anaerobic metabolism. During ischemia under normoglycemic conditions, brain lactate concentration reaches a plateau. In a hyperglycemic milieu, however, brain lactate concentration continues to rise for the duration of the ischemic period. The severity of intracellular acidosis during ischemia is directly proportional to the preischemic glucose concentration.247 The negative effect of hyperglycemia during brain ischemia is predicated on the presence of at least a small amount of blood flow to brain tissue. In one study, collaterally perfused but not end-arterial brain tissue had greater neuronal damage during hyperglycemic focal ischemia (Fig. 38.13).248 Clinical studies show a direct correlation between the initial post–cardiac arrest serum glucose concentration and poor neurologic outcome.237,249 However, a higher glucose concentration may simply reflect an endogenous response to severe stress and not the proximate cause of more severe brain injury.77 In piglets, postischemic administration of glucose did not worsen neurologic outcome after global hypoxia-ischemia.250 However, given the likelihood of additional ischemic and hypoxic events in the postresuscitation period, it seems prudent to maintain serum glucose in the normal range. Administration of insulin Glucose 2 NADH +2Pi

2 NAD+

2ATP

2 NADH

2 NADH

2 NAD+

2 Pyruvate− Ischemia Hypoxia



2 Lactate−+2H+

PDH

+/− 6 O2 Ca2+ 36 ATP

6 CO2

• Fig.

38.13 ​Schematic diagram illustrating the aerobic/anaerobic metabolism of glucose. Oxidation of pyruvate to CO2 (and H2O) by pyruvate dehydrogenase and citric acid cycle enzymes is retarded to blocked by oxygen deficiency, causing a reduction of pyruvate to lactate. If the adenosine triphosphate (ATP) formed during glycolysis is hydrolyzed (i.e., if the ATP concentration stays constant), one molecule of H1 is released for each molecule of lactate formed. If the mitochondria retain a membrane potential, they will sequester excess calcium entering the cell. However,​ if they deenergized (with collapse of their membrane potential), they will release their calcium content. ADP, Adenosine diphosphate; NAD, nicotinamide adenine dinucleotide; NADH, reduced nicotinamide adenine dinucleotide; Pi, intracellular phosphorus; PDH, pyruvate dehydrogenase.  



CHAPTER 38

to hyperglycemic rats after global brain ischemia improved neurologic outcome.251 The effect of insulin may be independent of its ability to lower blood glucose, because these investigators later showed that normoglycemic insulin-treated rats had a better outcome than normoglycemic placebo-treated control subjects.252 Using intensive insulin therapy, Van den Berghe et al. strictly controlled the blood glucose levels of adults in a surgical ICU, maintaining levels between 80 and 110 mg/dL.253,254 This control of blood glucose levels appeared to reduce mortality and protect the central and peripheral nervous systems. However, a subsequent study in a medical ICU showed no difference in mortality between the intensive insulin therapy and control groups.255 Another study showed that the use of intensive insulin therapy to maintain normoglycemia was associated with increased episodes of hypoglycemia.256 The HALF-PINT study was a large multicenter randomized controlled trial that assigned critically ill children to one of two different tight glycemic control targets: 80 to 110 mg/dL or 150 to 180 mg/dL.257 The trial was stopped early owing to low likelihood of benefit and possibility of induced harm. There was no difference in ICU-free days, mortality, organ dysfunction, or ventilator-free days between groups. The lowertarget group had a higher rate of health care–associated infections and more cases of severe hypoglycemia. Some groups of patients, including premature infants and debilitated patients with low endogenous glycogen stores, are more prone to the development of hypoglycemia during and after a physiologic stress (e.g., surgery).258 Hypoglycemia poses a higher risk in the immature pediatric brain compared with the adult brain. Bedside monitoring of serum glucose is critical during and after a cardiac arrest and allows for intervention before the critical point of low substrate delivery is reached. The dose of glucose needed to correct hypoglycemia is 0.5 to 1.0 g/kg given as 10% dextrose in infants. The osmolarity of 50% dextrose is approximately 2700 Osm/L and is associated with intraventricular hemorrhage in neonates and infants.

Physiologic Foundations of Cardiopulmonary Resuscitation

cardiac arrest.18,259 The incidence increases with age. Approximately 25% of children experiencing in-hospital cardiac arrest have ventricular tachycardia or fibrillation.123 Moreover, the growing and aging population of children palliated for complex congenital heart disease in which the occurrence of ventricular arrhythmias may be much higher than in the general pediatric population requires greater attention to ventricular arrhythmias than in the past. Other potential causes of ventricular arrhythmias include familial and acquired prolonged QT syndrome, other arrhythmogenic ventricular conditions, cardiomyopathies, myocarditis, drug intoxications (such as illicit and accidental ingestion and therapeutic misadventures), electrolyte derangements (e.g., magnesium, calcium, potassium, or glucose), and hypothermia.260,261 Advances have been made in the management of ventricular arrhythmias. Rapid access to defibrillation has been shown to reduce mortality in adults; the development of public access defibrillation and AEDs has flowed from this knowledge. Initially, AED devices had little utility for children, but the development of current-reducing electrodes and specific pediatric algorithms has made public access defibrillation a reality for children. Consequently, AEDs have been deployed in the many environments in which children would be the primary beneficiaries (e.g., schools and public swimming pools). The wearable cardioverterdefibrillator has been shown to be safe and effective in the pediatric population.262 When automated external defibrillators are used within 3 minutes of adult-witnessed VF in children, longterm survival can occur in more than 70% of cases.263–265 The technique of current delivery has undergone change with the development and deployment of biphasic defibrillators, which may offer increased efficacy with reduced risk of myocardial injury (Fig. 38.14). Finally, amiodarone is joined by lidocaine as the drugs of choice for refractory ventricular arrhythmias. The role of each of these factors in the resuscitation of pediatric arrest victims is discussed in the following section.

Defibrillation

Management of Ventricular Fibrillation The management of lethal ventricular arrhythmias traditionally has not played a major role in resuscitation teaching or management for children because of the low incidence of these arrhythmias. Newer evidence gathered in the environment of rapid access defibrillation suggests that as many as 19% of the presenting rhythms in pediatric arrests are ventricular in origin, represent as many as 5% to 15% of all pediatric victims of out-of-hospital

VF is the chaotic electrical excitation of the ventricle. The definitive treatment in accordance with the 2015 AHA guidelines is defibrillation.47,196,266 The electrical mechanism is usually explained as a reentrant depolarization of the myocardium, initially in waves, that then take more circuitous routes and degenerate into smaller reentry circuits resulting in loss of the rhythmic contractile function of the ventricles.267 This changing pattern of reentry circuits corresponds with the change from coarse to fine VF

'(),%5,//$7,21:$9()2506

MRXOHVDWRKPV 





7LPH PV  

• Fig. 38.14 waveforms.



7\SLFDOELSKDVLF %LSKDVLFWUXQFDWHGH[SRQHQWLDO &XUUHQW DPSV

&XUUHQW DPSV

&RQYHQWLRQDOPRQRSKDVLF 'DPSHGVLQHZDYH       − −

437

      − −

MRXOHVDWRKPV 







7LPH PV

​Schematic patterns of current flow for conventional monophasic and typical biphasic defibrillator

438

SECTION IV



Pediatric Critical Care: Cardiovascular

as the duration of fibrillation persists and may correlate with deterioration in energy stores associated with persistence of fibrillation.5,268,269 Similarly, most cases of ventricular tachycardia (VT) are attributable to reentrant mechanisms, although increased automaticity is the likely mechanism in persons with drug-induced torsades de pointes and electrolyte disturbances, such as hypokalemia and hypomagnesemia.270 Nonpulsatile VT with loss of effective contractile function of the heart rapidly deteriorates into VF. Loss of effective ventricular function with these arrhythmias requires emergent management. The standard for management of VF and pulseless VT is immediate defibrillation and high-quality CPR. Although the lowest energy dose for effective defibrillation and the upper limit for safe defibrillation in infants and children are not known, energy doses greater than 4 J/kg (up to 9 J/kg) have effectively defibrillated children.159 The standard voltage dose for pediatric defibrillation is 2 J/kg. If unsuccessful, successive doses of defibrillation are repeated at 4 J/kg or more, not to exceed 10 J/kg or the maximum adult dose.109 This dosage is based on data reported by Gutgesell et al. in 1976.271 They reported 71 defibrillation attempts in 27 children. Efficacy was 91% with 2 J/kg and 100% with 4 J/kg. After initial defibrillation, CPR is performed for 2 minutes, followed by a rhythm check and then repeat shock if required. This sequence may then be repeated, with consideration given to initiating vasopressor therapy. Rhythms that fail to respond to three rounds are defined as “shock resistant.” In this setting, the standard as defined in the AHA guidelines is amiodarone or lidocaine (or magnesium for torsades de pointes), followed by 2 minutes of CPR and continuation of the rhythm check-shock-CPR/vasopressor cycle (Fig. 38.15). Reversible causes of VT/VF should also be investigated. It is important to continue to deliver appropriate CPR while gathering defibrillation equipment.272,273 Additional important considerations when delivering shocks include paddle size, position, contact pressure, and use of electrode paste. Large paddles reduce thoracic impedance, and infants older than 1 year or weighing more than 10 kg should be treated with adult paddles.274 Adhesive patch electrodes are an acceptable alternative to paddles and can be used when available if their use does not cause a delay in therapy.275–277 Paddles should be positioned to achieve current flow through the heart; an anterior-apex, or anterior-posterior placement is selected. Contact pressure has been demonstrated to reduce impedance. Firm pressure, which commonly is not properly applied, is required.278 Proper electrode paste or gel is needed. Care is required to avoid smearing paste across the chest wall. Doing so can lead to arcing of the circuit and a resultant short circuit. Bare paddles, ultrasound gel, pads soaked in saline solution, and alcohol pads are not acceptable alternatives to electrode cream or paste.196 The role of immediate defibrillation in pediatric patients has come under question. The efficacy of defibrillation declines rapidly as fibrillation persists. When an arrest is witnessed and a defibrillator is immediately available, defibrillation likely will be successful. With any delay in resuscitation, the success of initial defibrillation declines at a rate estimated at between 7% and 10% per minute of continued fibrillation.160 A number of studies have demonstrated in both animals and adults that if more than 3 to 5 minutes of fibrillation have occurred before institution of defibrillation, use of CPR for 90 to 180 seconds to restore myocardial energy stores will improve the likelihood of conversion to a perfusing rhythm with defibrillation.6,8,279,280 Mitani et al. showed that children with out-of-hospital cardiac arrest due to a shockable rhythm had a higher rate of 1-month

survival and better neurologic outcome after bystander-initiated AED usage compared with those who received defibrillation by emergency medical services.281 Hunt et al. analyzed data obtained from the Get with the Guidelines–Resuscitation national registry to evaluate time to defibrillation and survival in pediatric in-hospital arrest.282 They found that time to first shock, whether less than or greater than 2 minutes, did not impact survival, ROSC, or favorable neurologic outcome. Thus, immediate recognition and initiation of CPR in a highly monitored pediatric setting may attenuate the effects of time to the first defibrillation attempt. Use of biphasic defibrillators is another important advance in the management of tachyarrhythmias. Studies suggest that defibrillation with a biphasic waveform can be achieved with lower energy and less myocardial injury than with a standard monophasic defibrillator current.283,284 The first commercially available devices were approved by the US Food and Drug Administration in 1996. An evidence-based review was undertaken by the AHA and published in 1998.45 The reviewers concluded that “low-energy, non-progressive biphasic waveform defibrillators may be used for both out-of-hospital and in-hospital VF arrest, including persistent or recurrent VF that does not respond to the initial lowenergy shock.” These conclusions were based on observational studies and case reports. Subsequently, Schneider et al. reported a randomized controlled trial of biphasic versus monophasic defibrillation for out-of-hospital cardiac arrest.285 Of 338 arrests, 115 patients had VF and were shocked with an AED. Defibrillation in the initial shock series was successful in 98% of patients receiving biphasic shocks but in only 69% of those receiving monophasic shocks (P , .0001), providing further evidence that biphasic waveforms are more efficacious than monophasic waveforms. Biphasic shocks appear to be at least as effective as monophasic shocks and less harmful. Published data on children are limited to case reports.286 Animal data are supportive of the use of biphasic defibrillators in infants and children. Clark et al. demonstrated in a piglet model that low-energy biphasic shocks were superior to monophasic shocks for converting catheter-induced VF.287 Both Tang et al. and Berg et al. studied the use of AEDs equipped with energy-reducing electrodes and found increased efficacy compared with monophasic waveforms.5,288 Berg et al. also demonstrated improved LV function 5 hours after resuscitation. According to the 2015 AHA recommendations, with a manual defibrillator, dosage recommendations for children remain 2 J/kg for the first attempt followed by 4 J/kg for subsequent attempts.5,49,289 This dose was increased from the 2005 guidelines, which cited studies that demonstrated the ineffectiveness of 2 J/ kg at achieving ROSC. Subsequent to the publication of the 2010 AHA recommendations, a review of the National Registry of Cardiopulmonary Resuscitation data compared the effect of 2 J/ kg with historical controls and a 4 J/kg initial shock dose in VF and pulseless VT. The authors found that termination of the arrhythmia with an initial shock dose of 2 J/kg was significantly less effective than historical controls (56% vs. 91%). A higher initial dose of 4 J/kg was associated with less successful ROSC than the 2 J/kg dose. Based on this conflicting data, they concluded that the optimal initial shock dose has yet to be determined. In the early 1990s, as part of an AHA campaign to improve the abysmal rates of resuscitation from out-of-hospital cardiac arrest in adults, the development and deployment of AEDs was initiated. Both fixed and escalating dose devices were developed; however, the initial dose usually was at least 150 J in adults. Because of the high fixed-energy doses, the devices were not recommended for use in children younger than 9 years. Moreover, their



CHAPTER 38

Physiologic Foundations of Cardiopulmonary Resuscitation

439

PEDIATRIC TACHYCARDIA With a pulse and poor perfusion

1

Identify and treat underlying cause • Maintain patent airway; assist breathing as needed • Oxygen • Cardiac monitor to identify rhythm; monitor blood pressure and oximetry • IO/IV access • 12-lead ECG if available; don’t delay therapy 2

Narrow (≤0.09 sec)

Evaluate QRS duration

Wide (>0.09 sec)

3 Evaluate rhythm with 12-lead ECG or monitor 4

5 Probable sinus tachycardia • Compatible history consistent with known cause • P waves present/normal • Variable R-R; constant PR • Infants: rate usually 5 years old, with no developmental delay) Assess for delirium • Preschool Confusion Assessment Method for the ICU (psCAM-ICU) each shift: (in child 6 months-5 years old)

Delirium present if:

• CAPD score of 9 or higher* • pCAM-ICU positive • psCAM-ICU positive

• Fig. 134.3

​Detection of delirium. *In a child with developmental delay, must also establish alteration from baseline. PICU, Pediatric intensive care unit.  





Address underlying disease:

Minimize iatrogenic factors:

• Assess for infection • Optimize pain control • Assess for hypoxemia • Assess for withdrawal

• Minimize sedatives • Avoid benzodiazepines • Avoid anticholinergics • Avoid restraints

Optimize environment • Frequently reorient the child • Communicate clearly and concisely (as age appropriate) • Create a quiet, well-lit space with familiar objects from the child’s home • Encourage cognitive stimulation and mobilization during the day • Cluster care to allow for uninterrupted sleep

Pharmacologic management of delirium may be indicated and can be implemented at the treating physician’s discretion. • Fig. 134.4  



​Delirium treatment algorithm.

minimized, as clinically appropriate. A patient on invasive mechanical ventilation, whose sedative regimen has been steadily escalated over the course of hours to days, may be delirious as a direct result of the sedatives and will often improve with removal of the offending agents.64 As described previously, numerous studies have indicated that benzodiazepines are particularly problematic and that escalating dosages may result in increased risk for delirium and excess ventilator days. In years past, clinicians hoped to decrease the traumatic effect of hospitalization by sedating the child; recent data indicate that this is counterproductive, interferes with the clinician’s ability to adequately recognize and treat pain, and may increase the risk of delusional memories and posttraumatic stress disorder.65,66 Many pediatric intensivists have adopted the philosophy that it is better for a critically ill patient to be awake and occasionally upset—with reassurance provided by the parents and the clinical care team—than to “sleep” through the ICU stay.67 Opiate and benzodiazepine withdrawal can precipitate delirium, but it is important to differentiate the physiologic signs of abstinence from the behavioral symptoms of hyperactive delirium. In fact, a child with hyperactive delirium who has never been on an opiate will score positive on a number of withdrawal assessment tools.68,69 If delirium is triggered by opiate withdrawal, simply replacing the missing opiate is often insufficient to reverse the symptoms. Judicious replacement of the opiate is necessary,

but it is important to then approach the delirium as a primary problem rather than continue to escalate the opiate dosage, as this will simply prolong the delirium exposure.70 Reviewing the patient’s medication list is essential. Anticholinergic agents and medications with anticholinergic effects may be routinely and often unnecessarily used in the PICU and can frequently be discontinued or replaced.71 If a child with hyperactive delirium is on corticosteroids, a taper should be considered. Attention should be paid to minimizing the use of benzodiazepines and replacing these agents with dexmedetomidine when sedation is necessary.

Environment Optimizing the environment is an achievable goal.72,73 It is important to ensure that children who require corrective lenses have their glasses on when awake. Creating a well-lit and uncluttered space during the day is feasible and effective. Educating parents and staff to limit guests and avoid commotion at the bedside is essential. Familiar items from home, such as a favorite blanket or stuffed toy, are more effective than new gifts. Helium balloons can be disconcerting to a lightly sedated child as they move spontaneously and can contribute to hallucinations. In adults, early mobilization has been shown to decrease delirium rates and shorten delirium once it occurs.74,75 Implementation



CHAPTER 134 Pediatric Delirium

1623

of an early mobilization protocol in the PICU does not have to be as dramatic as walking an intubated child; it can start as a simple change in approach and mindset. A child on invasive mechanical ventilation should be permitted to have periods of wakefulness as well as sleep; deep sedation should be avoided. When awake, passive range-of-motion activities in bed can be performed by the nurse or an appropriately supervised family member. Even young children can be safely maintained with a light level of sedation, awake on the ventilator, and interacting with caregivers, while older children and adolescents can often cooperate with a more aggressive mobilization strategy. In these patients, getting out of bed to a chair and/or walking may be attainable goals.76 A difficult challenge is establishing a sleep-friendly environment within the PICU.77 Asking the parents to describe the child’s preferred sleep position can be helpful—a toddler who always sleeps in the prone position at home will have enormous difficulty remaining asleep on the back while critically ill; thus, repositioning may help (especially when mechanically ventilated). Attempts to ensure a quiet, dark environment at night and encouraging parents to continue their usual bedtime routines (such as a storybook or back rub) can help facilitate sleep, even in the ICU.78

Prevention of delirium remains the ultimate goal. Data in adults suggest that early mobilization of critically ill patients will reduce the incidence of delirium and shorten its duration.74,75 In hospitalized adults who were not critically ill, delirium rates were decreased using a multicomponent intervention, with protocols for cognition, sleep, mobility, vision, hearing, and hydration.73 There is no evidence that prophylactic administration of antipsychotics decreases delirium rates in the general ICU population. There are several prospective studies describing effective preventive strategies for the development of delirium in critically ill children. A single-center PICU study decreased delirium rates by 19% with implementation of universal delirium screening and minimization of benzodiazepine-based sedation.27,42 A large quality improvement initiative in another PICU showed a stepwise decrease in delirium rates with implementation of universal delirium screening followed by protocol-based sedation and initiation of early mobilization.43 A systematic approach to family engagement, designed to operationalize the parents’ role in delirium prevention, has shown promising results in pilot studies.67 Further research on prevention of pediatric delirium is necessary and underway.

Pharmacotherapy

Conclusion

In 5% to 10% of cases, attempts to address the underlying illness, minimize therapeutic triggers, and optimize the environment are not enough, and persistent delirium symptoms interfere with necessary medical interventions. In such instances, pharmacologic therapy for delirium may be considered.72 It is important to stress that this is an off-label use; currently, no drug has been approved by the US Food and Drug Administration to treat delirium in either adults or children. Consensus approach is to use antipsychotic agents (either haloperidol or the atypical antipsychotics due to their more favorable side effect profiles) only when absolutely necessary.4 These procognitive drugs regulate neurotransmitters and can help organize thoughts and calm the delirious patient. This may facilitate the patient’s cooperation with ongoing medical care and allow the opportunity for the patient to wean from other medications that promote delirium.79 Evidence for pharmacotherapy is scarce. There are no published data showing that treatment with haloperidol reduces the duration of delirium in adults or children. One randomized, placebo-controlled study showed a reduction in the duration of delirium with quetiapine (an atypical antipsychotic medication) in critically ill adults.80 There have been several case series describing the successful use of atypical antipsychotic agents (including olanzapine, quetiapine, and risperidone) to treat delirium in young children, but no prospective efficacy studies have been conducted to date.81–85 A retrospective study designed to evaluate the safety of quetiapine in treating delirium in critically ill children found no serious adverse events with greater than 2400 doses administered.86 Further research is needed to systematically assess the safety and efficacy of antipsychotics in the treatment of pediatric delirium.

Delirium is prevalent in critical illness and is associated with significant short- and longer-term morbidity. Children are at significant risk of developing delirium while in the PICU. With increased awareness of this common problem, pediatric intensivists can detect delirium sooner and implement strategies to treat and prevent this serious complication of PICU care.

Prevention

Key References Mody K, Kaur S, Mauer EA, et al. Benzodiazepines and development of delirium in critically ill children: estimating the causal effect. Crit Care Med. 2018;46(9):1486-1491. Patel AK, Biagas KV, Clarke EC, et al. Delirium in children after cardiac bypass surgery. Pediatr Crit Care Med. 2017;18(2):165-171. Simone S, Edwards S, Lardieri A, et al. Implementation of an ICU bundle: an interprofessional quality improvement project to enhance delirium management and monitor delirium prevalence in a single PICU. Pediatr Crit Care Med. 2017;18(6):531-540. Traube C, Silver G, Gerber LM, et al. Delirium and mortality in critically ill children: epidemiology and outcomes of pediatric delirium. Crit Care Med. 2017;45(5):891-898. Traube C, Silver G, Reeder RW, et al. Delirium in critically ill children: an international point prevalence study. Crit Care Med. 2017;45(4): 584-590. Traube C, Silver G, Kearney J, et al. Cornell assessment of pediatric delirium: a valid, rapid, observational tool for screening delirium in the PICU. Crit Care Med. 2014;42(3):656-663. Meyburg J, Dill ML, Traube C, Silver G, von Haken R. Patterns of postoperative delirium in children. Pediatr Crit Care Med. 2017;18(2):128-133.

The full reference list for this chapter is available at ExpertConsult.com.

e1







































































































1. American Psychiatric Association. Diagnostic and Statistical Manual of Mental Disorders, ed 5. Arlington, VA: American Psychiatric Association; 2013. 2. Peterson JF, Pun BT, Dittus RS, et al. Delirium and its motoric subtypes: a study of 614 critically ill patients: delirium subtypes in the critically ill. J Am Geriatr Soc. 2006;54(3):479-484. 3. Ely EW, Siegel MD, Inouye M D SK. Delirium in the Intensive Care Unit: an under-recognized syndrome of organ dysfunction. Semin Respir Crit Care Med. 2002;22(02):115-126. 4. Barr J, Fraser GL, Puntillo K, et al. Clinical practice guidelines for the management of pain, agitation, and delirium in adult patients in the Intensive Care Unit. Crit Care Med. 2013;41(1):278-280. 5. Ouimet S, Kavanagh BP, Gottfried SB, Skrobik Y. Incidence, risk factors and consequences of ICU delirium. Intensive Care Med. 2006; 33(1):66-73. 6. Ely EW, Shintani A, Truman B, et al. Delirium as a predictor of mortality in mechanically ventilated patients in the intensive care unit. JAMA. 2004;291(14):1753-1762. 7. Ely EW, Gautam S, Margolin R, et al. The impact of delirium in the intensive care unit on hospital length of stay. Intensive Care Med. 2001;27(12):1892-1900. 8. Girard TD, Jackson JC, Pandharipande PP, et al. Delirium as a predictor of long-term cognitive impairment in survivors of critical illness. Crit Care Med. 2010;38(7):1513-1520. 9. Shehabi Y, Riker RR, Bokesch PM, Wisemandle W, Shintani A, Ely EW. Delirium duration and mortality in lightly sedated, mechanically ventilated intensive care patients. Crit Care Med. 2010; 38(12):2311-2318. 10. Pisani MA, Kong SYJ, Kasl SV, Murphy TE, Araujo KLB, Van Ness PH. Days of delirium are associated with 1-year mortality in an older intensive care unit population. Am J Respir Crit Care Med. 2009; 180(11):1092-1097. 11. Milbrandt EB, Deppen S, Harrison PL, et al. Costs associated with delirium in mechanically ventilated patients. Crit Care Med. 2004; 32(4):955-962. 12. Traube C, Greenwald BM. “The times they are a-changin”: universal delirium screening in pediatric critical care. Pediatr Crit Care Med. 2017;18(6):594-595. 13. Maldonado JR. Pathoetiological model of delirium: a comprehensive understanding of the neurobiology of delirium and an evidencebased approach to prevention and treatment. Crit Care Clin. 2008; 24(4):789-856. 14. Maldonado JR. Neuropathogenesis of delirium: review of current etiologic theories and common pathways. Am J Geriatr Psychiatry. 2013; 21(12):1190-1222. 15. Trzepacz PT. Update on the neuropathogenesis of delirium. Dement Geriatr Cogn Disord. 1999;10(5):330-334. 16. Trzepacz PT. Is there a final common neural pathway in delirium? Focus on acetylcholine and dopamine. Semin Clin Neuropsychiatry. 2000;5(2):132-148. 17. Sarter M, Bruno JP. Cortical cholinergic inputs mediating arousal, attentional processing and dreaming: differential afferent regulation of the basal forebrain by telencephalic and brainstem afferents. Neuroscience. 1999;95(4):933-952. 18. Flacker JM, Cummings V, Mach JR, Bettin K, Kiely DK, Wei J. The association of serum anticholinergic activity with delirium in elderly medical patients. Am J Geriatr Psychiatry. 1999;6(1):31-41. 19. Tune LE. Serum anticholinergic activity levels and delirium in the elderly. Semin Clin Neuropsychiatry. 2000;5(2):149-153. 20. Cerejeira J, Firmino H, Vaz-Serra A, Mukaetova-Ladinska EB. The neuroinflammatory hypothesis of delirium. Acta Neuropathol. 2010;119(6):737-754. 21. van Munster BC, Korevaar JC, Korse CM, Bonfrer JM, Zwinderman AH, de Rooij SE. Serum S100B in elderly patients with and without delirium. Int J Geriatr Psychiatry. 2010;25(3):234-239. 22. de Rooij SE, van Munster BC, Korevaar JC, Levi M. Cytokines and acute phase response in delirium. J Psychosom Res. 2007;62(5):521-525.

23. McGrane S, Girard TD, Thompson JL, et al. Procalcitonin and Creactive protein levels at admission as predictors of duration of acute brain dysfunction in critically ill patients. Crit Care. 2011;15(2):R78. 24. Elie M, Cole MG, Primeau FJ, Bellavance F. Delirium risk factors in elderly hospitalized patients. J Gen Intern Med. 1998;13(3):204-212. 25. Franco J, Valencia C, Bernal C, et al. Relationship between cognitive status at admission and incident delirium in older medical inpatients. J Neuropsychiatry Clin Neurosci. 2010;22(3):329-337. 26. Murray C, Sanderson DJ, Barkus C, et al. Systemic inflammation induces acute working memory deficits in the primed brain: relevance for delirium. Neurobiol Aging. 2012;33(3):603-616.e3. 27. Silver G, Traube C, Gerber LM, et al. Pediatric delirium and associated risk factors: a single-center prospective observational study. Pediatr Crit Care Med. 2015;16(4):303-309. 28. Engel GL, Romano J. Delirium, A Syndrome of Cerebral Insufficiency. 2014 [cited 2015 May 18]. Available at: http://neuro.psychiatryonline.org/doi/10.1176/jnp.16.4.526. 29. Seaman JS, Schillerstrom J, Carroll D, Brown TM. Impaired oxidative metabolism precipitates delirium: a study of 101 ICU patients. Psychosomatics. 2006;47(1):56-61. 30. Schoen J, Meyerrose J, Paarmann H, Heringlake M, Hueppe M, Berger KU. Preoperative regional cerebral oxygen saturation is a predictor of postoperative delirium in on-pump cardiac surgery patients: a prospective observational trial. Crit Care. 2011;15(5):R218. 31. Dinges D. The state of sleep deprivation: From functional biology to functional consequences. Sleep Med Rev. 2006;10(5):303-305. 32. Mistraletti G, Carloni E, Cigada M, et al. Sleep and delirium in the intensive care unit. Minerva Anestesiol. 2008;74(6):329-333. 33. Smith HAB, Brink E, Fuchs DC, Ely EW, Pandharipande PP. Pediatric delirium: monitoring and management in the Pediatric Intensive Care Unit. Pediatr Clin North Am. 2013;60(3):741-760. 34. Traube C, Silver G, Reeder RW, et al. Delirium in critically ill children: an international point prevalence study. Crit Care Med. 2017;45(4):584-590. 35. Smith HAB, Boyd J, Fuchs DC, et al. Diagnosing delirium in critically ill children: Validity and reliability of the Pediatric Confusion Assessment Method for the Intensive Care Unit. Crit Care Med. 2011;39(1):150-157. 36. Traube C, Ariagno S, Thau F, et al. Delirium in hospitalized children with cancer: incidence and associated risk factors. J Pediatr. 2017; 191:212-217. 37. Patel AK, Biagas KV, Clarke EC, et al. Delirium in children after cardiac bypass surgery. Pediatr Crit Care Med. 2017;18(2):165-171. 38. Alvarez RV, Palmer C, Czaja AS, et al. Delirium is a common and early finding in patients in the Pediatric Cardiac Intensive Care Unit. J Pediatr. 2018;195:206-212. 39. Meyburg J, Dill ML, Traube C, Silver G, von Haken R. Patterns of postoperative delirium in children. Pediatr Crit Care Med. 2017;18(2):128-133. 40. Smith HAB, Gangopadhyay M, Goben CM, et al. The preschool confusion assessment method for the ICU: valid and reliable delirium monitoring for critically ill infants and children. Crit Care Med. 2016;44(3):592-600. 41. Traube C, Silver G, Kearney J, et al. Cornell assessment of pediatric delirium: a valid, rapid, observational tool for screening delirium in the PICU. Crit Care Med. 2014;42(3):656-663. 42. Traube C, Silver G, Gerber LM, et al. Delirium and mortality in critically ill children: epidemiology and outcomes of pediatric delirium. Crit Care Med. 2017;45(5):891-898. 43. Simone S, Edwards S, Lardieri A, et al. Implementation of an ICU bundle: an interprofessional quality improvement project to enhance delirium management and monitor delirium prevalence in a single PICU. Pediatr Crit Care Med. 2017;18(6):531-540. 44. Inouye SK, Westendorp RG, Saczynski JS. Delirium in elderly people. Lancet. 2014;383(9920):911-922. 45. Meyburg J, Dill ML, von Haken R, et al. Risk factors for the development of postoperative delirium in pediatric intensive care patients. Pediatr Crit Care Med. 2018;19(10):e514-e521.

References

e2

46. Deeter KH, King MA, Ridling D, Irby GL, Lynn AM, Zimmerman JJ. Successful implementation of a pediatric sedation protocol for mechanically ventilated patients. Crit Care Med. 2011;39(4):683-688. 47. Penk JS, Lefaiver CA, Brady CM, Steffensen CM, Wittmayer K. Intermittent versus continuous and intermittent medications for pain and sedation after pediatric cardiothoracic surgery; a randomized controlled trial. Crit Care Med. 2018;46(1):123-129. 48. Smith HAB, Gangopadhyay M, Goben CM, et al. Delirium and benzodiazepines associated with prolonged ICU stay in critically ill infants and young children. Crit Care Med. 2017;45(9):1427-1435. 49. Mody K, Kaur S, Mauer EA, et al. Benzodiazepines and development of delirium in critically ill children: estimating the causal effect. Crit Care Med. 2018;46(9):1486-1491. 50. Robins JM, Hernan MA, Brumback B. Marginal structural models and causal inference in epidemiology. Epidemiology. 2000;11(5): 550-560. 51. Aydogan MS, Korkmaz MF, Ozgül U, et al. Pain, fentanyl consumption, and delirium in adolescents after scoliosis surgery: dexmedetomidine vs midazolam. Paediatr Anaesth. 2013;23(5):446-452. 52. Nellis ME, Goel R, Feinstein S, Shahbaz S, Kaur S, Traube C. Association between transfusion of RBCs and subsequent development of delirium in critically ill children. Pediatr Crit Care Med. 2018;19(10):925-929. 53. Traube C, Mauer EA, Gerber LM, et al. Cost associated with pediatric delirium in the ICU. Crit Care Med. 2016;44(12):e1175-e1179. 54. Shann F, Pearson G, Slater A, Wilkinson K. Paediatric index of mortality (PIM): a mortality prediction model for children in intensive care. Intensive Care Med. 1997;23(2):201-207. 55. Silver G, Kearney J, Traube C, Atkinson TM, Wyka KE, Walkup J. Pediatric delirium: Evaluating the gold standard. Palliat Support Care. 2015;13(3):513-516. 56. Silver G, Kearney J, Traube C, Hertzig M. Delirium screening anchored in child development: The Cornell Assessment for Pediatric Delirium. Palliat Support Care. 2015;13(4):1005-1011. 57. Schieveld JNM, Leroy PLJM, Os J, Nicolai J, Vos GD, Leentjens AFG. Pediatric delirium in critical illness: phenomenology, clinical correlates and treatment response in 40 cases in the pediatric intensive care unit. Intensive Care Med. 2007;33(6):1033-1040. 58. Silver G, Traube C, Kearney J, et al. Detecting pediatric delirium: development of a rapid observational assessment tool. Intensive Care Med. 2012;38(6):1025-1031. 59. Inouye SK, Schlesinger MJ, Lydon TJ. Delirium: a symptom of how hospital care is failing older persons and a window to improve quality of hospital care. Am J Med. 1999;106(5):565-573. 60. Schieveld JN, Janssen NJ. Delirium in the pediatric patient: on the growing awareness of its clinical interdisciplinary importance. JAMA Pediatr. 2014;168(7):595-596. 61. Harris J, Ramelet AS, van Dijk M, et al. Clinical recommendations for pain, sedation, withdrawal and delirium assessment in critically ill infants and children: an ESPNIC position statement for healthcare professionals. Intensive Care Med. 2016;42(6):972-986. 62. Patel AK, Bell MJ, Traube C. Delirium in pediatric critical care. Pediatr Clin North Am. 2017;64(5):1117-1132. 63. Tobias JD. Acute pain management in infants and children—Part 2: intravenous opioids, intravenous nonsteroidal anti-inflammatory drugs, and managing adverse effects. Pediatr Ann. 2014;43(7): e169-e175. 64. Patel SB, Poston JT, Pohlman A, Hall JB, Kress JP. Rapidly reversible, sedation-related delirium versus persistent delirium in the Intensive Care Unit. Am J Respir Crit Care Med. 2014;189(6):658-665. 65. Colville G, Kerry S, Pierce C. Children’s factual and delusional memories of intensive care. Am J Respir Crit Care Med. 2008; 177(9):976-982. 66. Clukey L, Weyant RA, Roberts M, Henderson A. Discovery of unexpected pain in intubated and sedated patients. Am J Crit Care. 2014;23(3):216-220.

67. Silver G, Traube C. A systematic approach to family engagement: feasibility pilot of a pediatric delirium management and prevention toolkit. Palliat Support Care. 2019;17(1):42-45. 68. Franck LS, Harris SK, Soetenga DJ, Amling JK, Curley MA. The Withdrawal Assessment Tool-1 (WAT-1): an assessment instrument for monitoring opioid and benzodiazepine withdrawal symptoms in pediatric patients. Pediatr Crit Care Med. 2008;9(6):573-580. 69. Ista E, de Hoog M, Tibboel D, Duivenvoorden HJ, van Dijk M. Psychometric evaluation of the sophia observation withdrawal symptoms scale in critically ill children. Pediatr Crit Care Med. 2013;14(8):761-769. 70. Traube C, Silver G. Iatrogenic withdrawal syndrome or undiagnosed delirium? Crit Care Med. 2017;45(6):e622-e623. 71. Madden K, Hussain K, Tasker RC. Anticholinergic medication burden in pediatric prolonged critical illness: a potentially modifiable risk factor for delirium. Pediatr Crit Care Med. 2018;19(10):917-924. 72. Hipp DM, Ely EW. Pharmacological and nonpharmacological management of delirium in critically ill patients. Neurotherapeutics. 2012;9(1):158-175. 73. Inouye SK, Bogardus Jr ST, Charpentier PA, et al. A multicomponent intervention to prevent delirium in hospitalized older patients. N Engl J Med. 1999;340(9):669-676. 74. Schweickert WM, Pohlman MC, Nigos C, et al. Early physical and occupational therapy in mechanically ventilated, critically ill patients: a randomised controlled trial. Lancet. 2009;373(9678): 1874-1882. 75. Needham DR, Zanni J, Pradhan P, et al. Early physical medicine and rehabilitation for patients with acute respiratory failure: a quality improvement project. Arch Phys Med Rehabil. 2010;91(4):536-542. 76. Wieczorek B, Ascenzi J, Kim Y, et al. PICU Up!: impact of a quality improvement intervention to promote early mobilization in critically ill children. Pediatr Crit Care Med. 2016;17(12):e559-e566. 77. Kudchadkar SR, Yaster M, Punjabi NM. Sedation, sleep promotion, and delirium screening practices in the care of mechanically ventilated children: a wake-up call for the pediatric critical care community. Crit Care Med. 2014;42(7):1592-1600. 78. Kawai Y, Weatherhead JR, Traube C, et al. Quality improvement initiative to reduce pediatric intensive care unit noise pollution with the use of a pediatric delirium bundle. J Intensive Care Med. 2019; 34(5):383-390. 79. Lonergan E, Britton AM, Luxenberg J. Antipsychotics for delirium. In: The Cochrane Collaboration, Lonergan E, eds. Cochrane Database of Systematic Reviews [Internet]. Chichester, UK: John Wiley & Sons, Ltd; 2007. 80. Devlin JW, Roberts RJ, Fong JJ, et al. Efficacy and safety of quetiapine in critically ill patients with delirium: a prospective, multicenter, randomized, double-blind, placebo-controlled pilot study. Crit Care Med. 2010;38(2):419-427. 81. Traube C, Witcher R, Mendez-Rico E, Silver G. Quetiapine as treatment for delirium in critically ill children: a case series. J Pediatr Intensive Care. 2013;2(3):121-126. 82. Karnik NS, Joshi SV, Paterno C, Shaw R. Subtypes of pediatric delirium: a treatment algorithm. Psychosomatics. 2007;48(3):253-257. 83. Turkel SB, Hanft A. The pharmacologic management of delirium in children and adolescents. Paediatr Drugs. 2014;16(4):267-274. 84. Silver GH, Kearney JA, Kutko MC, Bartell AS. Infant delirium in pediatric critical care settings. Am J Psychiatry. 2010;167(10):1172-1177. 85. Traube C, Augenstein J, Greenwald B, LaQuaglia M, Silver G. Neuroblastoma and pediatric delirium: a case series: Neuroblastoma and Delirium. Pediatr Blood Cancer. 2014;61(6):1121-1123. 86. Joyce C, Witcher R, Herrup E, et al. Evaluation of the safety of quetiapine in treating delirium in critically ill children: a retrospective review. J Child Adolesc Psychopharmacol. 2015;25(9):666-670.





































































































































































e3

Abstract: Delirium is a frequent and serious complication of pediatric critical illness. It is independently associated with delayed time to extubation, increased hospital length of stay, and higher medical costs. After controlling for severity of illness, children with delirium have been shown to have excess mortality rates. Modifiable risk factors for delirium have been identified, including benzodiazepine-based sedation. With implementation of routine screening for all children in the pediatric intensive care unit

(PICU), delirium can be detected early, when it is most amenable to treatment. A change in PICU culture can decrease the burden of delirium in critically ill children. Key words: Delirium, pediatric, critical care, epidemiology, risk factor, outcome, prevention, treatment, screening, diagnosis, CAPD

135 Procedural Sedation for the Pediatric Intensivist NIR ATLAS, RAHUL C. DAMANIA, AND PRADIP P. KAMAT

In the last 2 decades, there has been a robust demand for outpatient pediatric procedural sedation, now provided by myriad pediatric subspecialists, including pediatric intensivists.1,2 The intensivist who is trained in the early recognition and management of airway and cardiopulmonary issues is a perfect fit to provide procedural sedation outside the pediatric intensive care unit (PICU).3 A 2015 survey by Kamat et al. reported that intensivists staffed 78% of all sedation programs within the Society for Pediatric Sedation (SPS).4 Pediatric intensivists no longer solely sedate within the PICU; they also provide procedural sedation in sedation suites, radiology suites, oncology clinics, and endoscopy suites.1,5 Conventional procedures for which the pediatric intensivist provides procedural sedation are shown in Box 135.1.

Differences Between Outpatient and Inpatient Sedation Within the PICU, intensivists have the luxury of assistance from multiple team members, including nurses, respiratory therapists, an intravenous catheter placement team, and back-up emergency services (such as rapid response or the resource nurse). In addition, the PICU is equipped with advanced airway equipment, capnography, and extensive hemodynamic monitoring capabilities. In contrast, in the outpatient setting, the sedationist is usually partnered with a single nurse who is exclusively dedicated to the sedation.6 An additional nurse may be assisting the proceduralist and not directly involved with the process of sedation. A goal of outpatient sedation is maintenance of the natural airway during sedation. Thus, the need for intubation may be deemed as a failure. Monitoring of exhaled end-tidal carbon dioxide with capnography may not be readily available, though highly recommended.7 Physical access to the patient may be particularly challenging in certain locations, such as magnetic resonance imaging (MRI) suites. Given the distinctive challenges that arise from outpatient procedural sedation, the intensivist must be trained to perform sedation safely in a variety of clinical settings.

Outpatient Procedural Sedation Training During Pediatric Critical Care Fellowship As the demand for procedural sedation outside the PICU increases, so does the need for the intensivist to demonstrate 1624

proficiency in outpatient procedural sedation management and monitoring. A recent article describing trends in outpatient procedural sedation reports a consistent intensivist presence in the provision of procedural sedation over the last 10 years.2 Despite the important role of the intensivist in procedural sedation, a recent survey of pediatric critical care medicine (PCCM) fellowship directors reported that only one-third of PCCM fellowship trainees received formal procedural sedation training during their fellowship.8 Additionally, only 61% of fellows felt adequately prepared to provide procedural sedation on their own after finishing fellowship.8 Current training for procedural sedation in most PCCM fellowship programs appears to be inconsistent and optional, highlighting a training gap that must be addressed.9 Given that most PCCM fellows likely will be required to perform procedural sedation outside the PICU, it is imperative that fellowship programs incorporate outpatient procedural sedation training in their academic and training curricula. The SPS (www.pedsedation.org) provides simulationbased training in procedural sedation during its annual conference. Simulation has been shown to be effective in teaching sedation competencies and enhancing team dynamics; it should be routinely employed to train providers of procedural sedation.10

Sedation Team Structure Considering the increasing role of the intensivist in procedural sedation outside the PICU, the newly graduated PCCM fellow will likely be required to incorporate this practice into clinical service. Some institutions allow intensivists to “moonlight” in the sedation service for compensation in addition to their PICU responsibilities. Transition to being a sedationist (part time or even full time) may also appeal to senior intensivists trying to decrease PICU clinical service or on-call duties before retirement or those experiencing burnout or moral distress while working in the PICU. Some programs use a hybrid of full-time sedationists (usually senior physicians from PCCM or Pediatric Emergency Medicine) and other intensivist and emergency medicine specialists who cover sedation shifts when not working in their primary clinical sites.4 In addition to the intensivist, most sedation programs allow a dedicated trained sedation nurse to assist mostly with sedation monitoring. An observer (usually another nurse) may be involved in monitoring but may also help with interruptible tasks. All personnel providing procedural sedation must have training in pediatric advanced life support.6

• BOX 135.1 Examples of Common Procedures

Requiring Pediatric Procedural Sedation Radiology Imaging Magnetic resonance imaging Computed tomography scan Positron emission tomography scan Nuclear medicine scans

Hematology-Oncology Bone marrow biopsy/aspiration Lumbar puncture 6 intrathecal administration of medications

Gastroenterology Colonoscopy Upper endoscopy Percutaneous endoscopic gastrostomy/gastrostomy tube placement/change

Surgical Abscess drainage Biopsies (renal, liver, thyroid) Fracture reduction and cast placement Wound dressing/vacuum-assisted closure Central venous line or peripherally inserted central catheter placement Chest tube placement Suture removal Laceration repair

Neurology Brainstem auditory response test Electroencephalography Electromyography Epidural blood patching Lumbar puncture (diagnostic) Magnetoencephalography Somatosensory evoked potentials

Other Eye examination Sexual assault examination Painful procedures not otherwise defined

Classification of Sedation The American Society of Anesthesiology (ASA) classifies sedation as mild, moderate, deep, and general anesthesia.11 The classification is based on responsiveness to voice, touch, painful stimulus, and the ability to maintain airway reflexes and cardiovascular function. The type of medication used does not define the level of sedation. Mild sedation is a drug-induced state in which the patient responds to verbal commands. For example, mild sedation may be employed in a child to reduce anxiolysis before a nonpainful imaging study. Moderate sedation is a state of depressed consciousness in which the patient may respond to verbal commands or tactile stimulation. No airway interventions are required, and cardiovascular function is maintained. For example, moderate sedation may be appropriate in a cooperative adolescent during suture repair of a laceration. Deep sedation is defined as a depression of consciousness in which patients cannot be easily aroused but respond purposefully following repeated or painful stimulation. The patient may require assistance in maintaining a patent airway, but the cardiovascular system is usually maintained. In outpatient procedural sedation, the intensivist commonly induces



CHAPTER 135

Procedural Sedation for the Pediatric Intensivist 1625

deep sedation, typically with propofol. The sedation specialist must have in-depth knowledge of monitoring as well as rescue of pediatric patients undergoing deep sedation. Last, general anesthesia is a drug-induced loss of consciousness during which patients are not arousable, even by painful stimulation. The ability to maintain ventilatory function is commonly impaired and patients often require positive-pressure ventilation. Neuromuscular and cardiovascular functions may also be impaired. For these reasons, general anesthesia is reserved for operating rooms and not performed in the outpatient procedural sedation setting. It is important to recognize that sedation is a continuum and that any child can slip from moderate sedation to deep sedation, making airway rescue skills paramount.6

Equipment, Monitoring, and Rescue Drugs Essential components that must be available in any outpatient sedation location include an emergency cart, hemodynamic monitoring devices, and rescue drugs. Equipment in emergency carts must include bag-valve-mask devices, oral and nasopharyngeal airways, laryngeal mask airways (LMAs), endotracheal tubes, laryngoscopy blades, and intravenous lines. In addition to these lifesaving devices, sedation specialists should have access to portable telemetry, pulse oximetry, capnography, blood pressure monitoring, and defibrillators.12 Last, common rescue medications—such as albuterol, atropine, diphenhydramine, dextrose, epinephrine, flumazenil, lidocaine, lorazepam, methylprednisolone, naloxone, oxygen, racemic epinephrine, and sodium bicarbonate—should be readily available.6 Pediatric intensivists are trained to rescue a child using these tools, making them uniquely suited to provide outpatient sedation for diagnostic and therapeutic procedures.

Sedation Prescreening Not all patients are candidates for procedural sedation. Pediatric intensivists should be aware that certain conditions necessitate the services of a pediatric anesthesiologist. Children with difficult airway (determined by history or physical examination), microcephaly, micrognathia, retrognathia, mandibular/midface hypoplasia, or genetic syndromes with known complex airway anatomy are best referred to the anesthesiologist. In addition, patients with ASA physical status classification of IV or higher (Table 135.1), severe obstructive sleep apnea (apnea-hypopnea index .10), morbid obesity (body mass index .95th percentile), and complex

TABLE American Society of Anesthesiology 135.1 Classification

Class

Description

I

Normal healthy patient

II

Patient with mild systemic disease

III

Patient with severe systemic disease

IV

Patient with severe systemic disease that is a constant threat to life

V

Moribund patient who is not expected to survive without an operation

VI

Declared brain-dead patient whose organs are being removed for donor purposes

1626 S E C T I O N X I V



Pediatric Critical Care: Anesthesia Principles in the Pediatric Intensive Care Unit

TABLE Modified Fasting Periods for Pediatric 135.2 Procedural Sedationa

Food/Beverage

Fasting Period (hours)

Clear liquids (water, clear beverages, nonpulp fruit juice, tea/coffee without milk)

2

Breast milk

4

Infant formula/nonhuman milk

6

Meal (light meal or a meal with fried or fatty foods)

6–8

a Recent studies suggest minimal to low risk of aspiration with shorter duration of fasting. Fasting periods may be reduced under appropriate circumstances or per institutional policies.

cardiopulmonary disease (such as unrepaired congenital heart disease) may not be suitable candidates for outpatient procedural sedation and are best discussed with the anesthesiologist.13 Once a patient is deemed to be a candidate for natural airway sedation, instructions for nil per os (NPO; nothing by mouth) status should be given. Most institutions follow the American Academy of Pediatrics (AAP) fasting guidelines for procedural sedation.6 Table 135.2 shows the modified NPO guidelines commonly used in this setting. Recent evidence reporting minimal to no aspiration risk with a reduced fasting duration prior to procedural sedation will likely result in future changes in NPO guidelines.14–16 On the day of sedation, the pediatric intensivist must perform a focused medical history that, at a minimum, includes history of current illness, past medical history, current medications, allergies, adherence to fasting guidelines, and a family history of adverse reactions to anesthesia.17 An informed consent should be obtained from the parents or the legal guardian prior to procedural sedation. The physical examination on the day of sedation must include vital signs and a focused evaluation of the airway anatomy, including Mallampati scoring (see Fig. 127.5), cervical range of motion, and detailed cardiopulmonary assessment. Before sedation is initiated, intensivists should have a systematic approach so as not to overlook vital equipment, monitors, and rescue drugs. SOAP-ME is a commonly used acronym for presedation planning. It stands for suction (including age-appropriate catheters and a suction apparatus), oxygen (ensuring a functioning delivery device), airway (including bag-valve-mask, oral and nasopharyngeal airways, LMA, endotracheal tubes, and laryngoscope blades), pharmacy (sedation drugs, emergency lifesupport drugs, and reversal agents), monitors (functional pulse oximetry, end-tidal monitoring, noninvasive blood pressure, telemetry, and a stethoscope), equipment (including a nearby defibrillator).6 The sedationist should also have a backup emergency plan in the event of a life-threatening complication. Most sedation programs use a sedation rescue kit composed of flumazenil, naloxone, succinylcholine (for laryngospasm), and an anticholinergic agent (atropine or glycopyrrolate).

Considerations in Choosing Commonly Used Medications Before making medication choices, one must clearly identify the specific goals of a particular procedural sedation. The objectives of

procedural sedation are to (1) maintain patient safety, (2) minimize pain (i.e., provide analgesia), (3) decrease anxiety and psychological trauma (i.e., provide anxiolysis/amnesia), (4) provide immobility if required (i.e., adjust sedation depth), and (5) return child to a baseline state for safe discharge (follow AAP or institutional guidelines for discharge).6 Key properties of sedative, analgesic, and hypnotic agents used in outpatient procedural sedation are rapid onset, short duration of action, and preservation of airway/respiratory reflexes at appropriate weight-based doses. For this reason, most sedationists have embraced propofol, fentanyl, ketamine, and dexmedetomidine as pharmacologic agents of choice (Table 135.3). Recent studies from the Pediatric Sedation Research Consortium have demonstrated decreasing trends in the use of chloral hydrate and pentobarbital over the past 10 years, as agents with faster onset and recovery time are now available for use outside the operating room for nonanesthesiology providers. Propofol is particularly valuable in children because of its predictable pharmacokinetics and favorable adverse effect profile. Propofol provides amnesia, deep sedation, and immobility but no analgesia. Fentanyl is an excellent analgesic but can cause apnea and, when rapidly administered, a rigid chest. Naloxone is a fentanyl reversal agent that should be included in the emergency kit. Ketamine has the advantage of maintaining an excellent hemodynamic profile while providing potent analgesia and amnesia.12 Dexmedetomidine induces an arousable sleep state. By itself, it is not a good sedative for long or painful procedures but may be useful for short radiologic imaging.18–20 Recent studies have demonstrated the potentially advantageous role of dexmedetomidine as an adjuvant in procedural sedation. The use of a dexmedetomidine infusion followed by a propofol infusion (without propofol bolus) resulted in fewer adverse airway events and interventions when compared with propofol alone during procedural sedation for MRI.21 Much consideration should be given to the choice of sedation agent. For example, painless procedures, such as radiology imaging, may require propofol or dexmedetomidine. Painful procedures, such as bone marrow aspiration/biopsy or renal biopsy (both of which require immobility), may require a combination of propofol and fentanyl or ketamine.22,23 Last, providers should be facile in the use of inhaled nitrous oxide or an intranasal sedative (midazolam or dexmedetomidine) for anxiolysis prior to intravenous catheter placement, Foley catheter insertion, and other distressing procedures.24 Topical anesthetics—such as lidocaine-prilocaine cream, lidocaine jelly, and anesthetic skin refrigerant—are useful adjuncts. Midazolam may be a suitable alternative when nitrous oxide is contraindicated or not available. Intranasal dexmedetomidine or midazolam have been used successfully for short imaging procedures without injection of contrast when a peripheral intravenous catheter is not necessary.25 Useful information on intranasal medications is available at www.intranasal.net.

Nonpharmacologic Approach to Outpatient Sedation The involvement of a child life specialist during procedural sedation is invaluable. The specialist can help prepare the child for sedation and decrease anxiety through distraction, among other techniques. Certain vulnerable pediatric populations—including children with special needs, autism, and developmental delay— greatly benefit from coping plans put in place by child life specialists



CHAPTER 135

Procedural Sedation for the Pediatric Intensivist

1627

TABLE 135.3 Summary of Medications, Adjuncts, and Reversal Agents Commonly Used in Procedural Sedation

Agent

Route

Dose

Onset (min)

Duration (min)

Applications/Commentsa

Propofol

IV

1–2 mg/kg every 2–3 min Infusion 125–300 mg/kg/min

2–3

10–15

Procedures requiring immobility Add analgesic for painful procedure

Ketamine

IV

1 mg/kg initial 0.5 mg/kg subsequent

1–2

10–15

IM

4–6 mg/kg initial 2–4 mg/kg subsequent

10–15

30–40

Generally increases blood pressure and heart rate May increase secretions Provides short to medium duration sedation and analgesia for painful procedures

Fentanyl

IV

0.5–1 mg/kg initial 0.5 mg/kg subsequent

2–3

30–45

Analgesia with good hemodynamic profile Rapid intravenous administration can result in rigid chest

Dexmedetomidine

IV

Induction 1–3 mg/kg Infusion 1–2 mg/ kg/h

0–15

30–45

Noninvasive procedures Does not blunt respiratory drive

IN

2–4 mg/kg

30–45

45–60

IV

0.05–0.1 mg/kg (maximum 4 mg/dose)

2–5

30–45

PO

0.5–0.7 mg/kg

15–20

Up to 60

IN

0.2–0.4 mg/kg

5–10

30–45

Inhaled

50%–70%

3–5

5–10

Short noninvasive or minimally invasive

Naloxone

IV

0.01–0.02 mg/kg

1–2

30–45

Opioid overdose Reverses sedation and respiratory depression

Flumazenil

IV

0.01–0.02 mg/kg

1–2

30–45

Benzodiazepine overdose Reverses sedation and paradoxical reaction to benzodiazepines

IV

1–2 mg/kg

Rapid

4–6

Risk of malignant hyperthermia

IM

3–4 mg/kg

2–3

10–30

IV

0.02 mg/kg (minimum 0.1 mg/dose)

Rapid

1–4

Sedatives

Midazolam

Nitrous oxide

Anxiolysis prior to procedure Adjunct/premedication for deeper regimens

Reversal

Laryngospasm Succinylcholine Atropine

Antimuscarinic Increases heart rate and decreases secretions

a Applies to all routes unless otherwise specified. IM, Intramuscular; IN, intranasal; IV, intravenous; PO, oral.

created in collaboration with families.26 Distraction by means of virtual reality goggles, tablet computers, and video games have been used across the entire age spectrum.27 Neonates can have short, nonpainful procedures performed using feeding and bundling, swaddling, or pneumatic papoose.28

Sedation Adverse Events Sedation adverse events can be classified as minor without intervention, minor with intervention, and major/serious. Minor events that do not require intervention may include crying during propofol administration or oxygen desaturation below baseline but greater than 90% on pulse oximetry. Minor adverse events with intervention commonly include agitation, brief apnea, desaturation (oxygen saturation ,90% for .30 seconds), coughing, hypotension, and stridor. Serious adverse events include airway obstruction (lack of air movement despite respiratory effort), laryngospasm (lack of air movement with respiratory effort not relieved by repositioning or airway adjuncts), need for emergency anesthesia consultation, unplanned hospital admission, cardiac

arrest, or death. Serious adverse events can cause irreversible neurologic injury and are best anticipated and acted on quickly.1,22 A large series of approximately 91,000 sedation encounters from the Pediatric Sedation Research Consortium reported a low overall serious adverse event rate of 2.5% when propofol was used by pediatric intensivists for procedural sedation.1 More recently, using trends data from 432,842 encounters, overall, the serious adverse events by all providers (including pediatric intensivists) was reported to be even lower, at 1.78%. Recent studies have also highlighted the potential for neurotoxicity that sedation and anesthesia may pose in the developing brain of infants and children under 3 years of age.29,30 Therefore, every effort should be made to minimize duration of sedation and exposure to multiple agents.

Key References American Society of Anesthesiologists Task Force on Sedation, Analgesia by Non-Anesthesiologists. Practice guidelines for sedation and analgesia by non-anesthesiologists. Anesthesiology. 2002;96(4):1004-1017.

1628 S E C T I O N X I V



Pediatric Critical Care: Anesthesia Principles in the Pediatric Intensive Care Unit

Green SM, Leroy PL, Roback MG, et al. An international multidisciplinary consensus statement on fasting before procedural sedation in adults and children. Anaesthesia. 2020;75(3):374-385. Kamat PP, Ayestaran FW, Gillespie SE, et al. Deep procedural sedation by a sedationist team for outpatient pediatric renal biopsies. Pediatr Transplant. 2016;20(3):372-377. Kamat PP, McCracken CE, Simon HK, et al. Trends in outpatient procedural sedation: 2007-2018. Pediatrics. 2020;145(5):e20193559. Kudchadkar SR. Pediatric intensivists and elective procedural sedation: paradox or perfect pair? Pediatr Crit Care Med. 2015;16(1):77-78.

Mason KP, Zurakowski D, Zgleszewski SE, et al. High dose dexmedetomidine as the sole sedative for pediatric MRI. Paediatr Anaesth. 2008;18(5):403-411. Sulton C, Kamat P, Mallory M, Reynolds J. The use of intranasal dexmedetomidine and midazolam for sedated magnetic resonance imaging in children: a report from the Pediatric Sedation Research Consortium. Pediatr Emerg Care. 2020;36(3):138-142.

The full reference list for this chapter is available at ExpertConsult.com.

e1







































































1. Kamat PP, McCracken CE, Gillespie SE, et al. Pediatric critical care physician-administered procedural sedation using propofol: a report from the Pediatric Sedation Research Consortium Database. Pediatr Crit Care Med. 2015;16(1):11-20. 2. Kamat PP, McCracken CE, Simon HK, et al. Trends in outpatient procedural sedation: 2007-2018. Pediatrics. 2020;145(5):e20193559. 3. Kudchadkar SR. Pediatric intensivists and elective procedural sedation: paradox or perfect pair? Pediatr Crit Care Med. 2015;16(1):77-78. 4. Kamat PP, Hollman GA, Simon HK, Fortenberry JD, McCracken CE, Stockwell JA. Current state of institutional privileging profiles for pediatric procedural sedation providers. Hosp Pediatr. 2015;5(9):487-494. 5. Cravero JP. Pediatric sedation with propofol-continuing evolution of procedural sedation practice. J Pediatr. 2012;160(5):714-716. 6. Cote CJ, Wilson S, American Academy of Pediatrics, American Academy of Pediatric Dentistry. Guidelines for monitoring and management of pediatric patients before, during, and after sedation for diagnostic and therapeutic procedures. Pediatrics. 2019;143(6): e20191000. 7. Langhan ML, Mallory M, Hertzog J, Lowrie L, Cravero J, Pediatric Sedation Research Consortium. Physiologic monitoring practices during pediatric procedural sedation: a report from the Pediatric Sedation Research Consortium. Arch Pediatr Adolesc Med. 2012; 166(11):990-998. 8. Hooper MC, Kamat PP, Couloures KG. Evaluating the need for pediatric procedural sedation training in pediatric critical care medicine fellowship. Pediatr Crit Care Med. 2019;20(3):259-261. 9. Nadkarni V. Our survey says...pediatric procedural sedation training should not be a postscript! Pediatr Crit Care Med. 2019;20(3):296-297. 10. Hollman GA, Banks DM, Berkenbosch JW, et al. Development, implementation, and initial participant feedback of a pediatric sedation provider course. Teach Learn Med. 2013;25(3):249-257. 11. American Society of Anesthesiologists Task Force on Sedation, Analgesia by Non-Anesthesiologists. Practice guidelines for sedation and analgesia by non-anesthesiologists. Anesthesiology. 2002;96(4):1004-1017. 12. Berkenbosch JW. Options and considerations for procedural sedation in pediatric imaging. Paediatr Drugs. 2015;17(5):385-399. 13. Grunwell JR, Marupudi NK, Gupta RV, et al. Outcomes following implementation of a pediatric procedural sedation guide for referral to general anesthesia for magnetic resonance imaging studies. Paediatr Anaesth. 2016;26(6):628-636. 14. Beach ML, Cohen DM, Gallagher SM, Cravero JP. Major adverse events and relationship to nil per os status in pediatric sedation/anesthesia outside the operating room: a report of the Pediatric Sedation Research Consortium. Anesthesiology. 2016;124(1):80-88. 15. Bhatt M, Johnson DW, Taljaard M, et al. Association of preprocedural fasting with outcomes of emergency department sedation in children. JAMA Pediatr. 2018;172(7):678-685.

16. Green SM, Leroy PL, Roback MG, et al. An international multidisciplinary consensus statement on fasting before procedural sedation in adults and children. Anaesthesia. 2020;75(3):374-385. 17. Daud YN, Carlson DW. Pediatric sedation. Pediatr Clin North Am. 2014;61(4):703-717. 18. Mason KP, Robinson F, Fontaine P, Prescilla R. Dexmedetomidine offers an option for safe and effective sedation for nuclear medicine imaging in children. Radiology. 2013;267(3):911-917. 19. Mason KP, Zgleszewski SE, Dearden JL, et al. Dexmedetomidine for pediatric sedation for computed tomography imaging studies. Anesth Analg. 2006;103(1):57-62, table of contents. 20. Mason KP, Zurakowski D, Zgleszewski SE, et al. High dose dexmedetomidine as the sole sedative for pediatric MRI. Paediatr Anaesth. 2008;18(5):403-411. 21. Boriosi JP, Eickhoff JC, Klein KB, Hollman GA. A retrospective comparison of propofol alone to propofol in combination with dexmedetomidine for pediatric 3T MRI sedation. Paediatr Anaesth. 2017;27(1):52-59. 22. Kamat PP, Ayestaran FW, Gillespie SE, et al. Deep procedural sedation by a sedationist team for outpatient pediatric renal biopsies. Pediatr Transplant. 2016;20(3):372-377. 23. Patel KN, Simon HK, Stockwell CA, et al. Pediatric procedural sedation by a dedicated nonanesthesiology pediatric sedation service using propofol. Pediatr Emerg Care. 2009;25(3):133-138. 24. Tsze DS, Mallory MD, Cravero JP. Practice patterns and adverse events of nitrous oxide sedation and analgesia: a report from the Pediatric Sedation Research Consortium. J Pediatr. 2016;169:260265.e262. 25. Sulton C, Kamat P, Mallory M, Reynolds J. The use of intranasal dexmedetomidine and midazolam for sedated magnetic resonance imaging in children: a report from the Pediatric Sedation Research Consortium. Pediatr Emerg Care. 2020;36(3):138-142. 26. Kamat PP, Bryan LN, McCracken CE, Simon HK, Berkenbosch JW, Grunwell JR. Procedural sedation in children with autism spectrum disorders: a survey of current practice patterns of the society for pediatric sedation members. Paediatr Anaesth. 2018;28(6):552-557. 27. Salas M. Using Technology for distraction during imaging in a pediatric population. Clin J Oncol Nurs. 2015;19(4):409-410. 28. Parad RB. Non-sedation of the neonate for radiologic procedures. Pediatr Radiol. 2018;48(4):524-530. 29. Kamat PP, Sulton C, Kudchadkar SR, et al. Procedural sedation outside the operating room and potential neurotoxicity: analysis of an at-risk pediatric population. Acad Pediatr. 2019;19(8):978-984. 30. Kamat PP, Kudchadkar SR, Simon HK. Sedative and anesthetic neurotoxicity in infants and young children: not just an operating room concern. J Pediatr. 2019;204:285-290.

References

e2

Abstract: Pediatric intensivists are comprehensively trained in airway management techniques, experts in pediatric cardiorespiratory physiology, and independently responsible for routine airway management in the pediatric intensive care unit. Therefore, it follows that the pediatric intensivist may also have a role in outpatient procedural sedation. This chapter outlines the intensivist’s role in outpatient procedural sedation, the sedation team structure, types of procedures for which sedation is provided,

medications, and adverse events encountered in procedural sedation. It also discusses nuances of sedation prescreening since not every patient is a candidate for routine procedural sedation and some require referral to the anesthesiologist. Key words: Procedural sedation, natural airway sedation, deep sedation, outpatient, sedation prescreening

SECTION

XV

Pediatric Critical Care: Board Review Questions

e1

136 Board Review Questions Chapter 1: History of Pediatric Critical Care Medicine 1. Treatment for which of the following disease entities was not an important trigger in the early development of distinct, full-time, multidisciplinary pediatric intensive care units: A. Measles B. Poliomyelitis C. Reye syndrome D. Tetanus Preferred response: A

Rationale In Europe, pediatric intensive care followed shortly after the poliomyelitis epidemic in Denmark in 1952. In 1955, Dr. Goran Haglund, a pediatric anesthesiologist, established the first medical-surgical pediatric intensive care unit (PICU) for infants and children at the Children’s Hospital in Göteborg in Sweden. In France, in 1963, a newborn presented with tetanus and was admitted to l’Hôpital des Enfants Malades of Paris. Shortly afterward, Dr. Gilbert Huault and J.B. Joly, both neonatologists, opened the first multidisciplinary PICU in France at Saint Vincent de Paul Children’s Hospital. This unit was the first pediatrician-directed PICU in Europe; it soon became a major influence on the development of PICUs. In the mid-to-late 1970s, as pediatric cardiovascular surgery for more complex lesions in infants was developing, nurses provided postoperative care in designated units. Children with Reye syndrome suddenly appeared, requiring complex multisystem care. In addition, in the 1980s, emergency medical services (EMS) systems began transporting severely injured children to hospitals, where they required rapid assessment and intervention by nurses and physicians and initiation of cardiorespiratory and neurologic support. 2. Of the following, which most profoundly influenced the development of early distinct, geographically separate, multidisciplinary pediatric intensive care units? A. Advanced forms of mechanical ventilation B. Federal government finance programs C. New therapeutic interventions for oncology patients D. Nursing Preferred response: D

e2

Rationale Pediatric critical care medicine (PCCM) developed initially through the efforts of pediatric anesthesiologists, as well as pediatric general and cardiac surgeons, and neonatologists. In fact, most of the original PICUs were founded by pediatric anesthesiologists. Before discrete, geographically separate, intensive care units evolved, critically ill children often received close monitoring, intensive nursing care, and pulmonary support in the postanesthetic recovery room.

Chapter 2: High-Reliability Pediatric Intensive Care Unit: Role of Intensivist and Team in Obtaining Optimal Outcomes 1. The three dimensions of healthcare quality stated by Donabedian are: A. Reliability, operations, resilience B. Safety, efficiency, outcome C. Structure, process, outcome D. Structure, system, efficiency Preferred response: C

Rationale Avedis Donabedian, an early “systems thinker” in healthcare, stated that healthcare quality should be based upon three dimensions: structure-process-outcomes. Structure is the setting in which care is delivered. Process refers specifically to how care is provided, including incorporation of high reliability principles into daily activities. Outcomes refer to endpoints of care, including commonly used quality and safety measures, and other key outcomes such as length of stay, patient/family experience, and cost/value. 2. A care bundle is defined as: A. A flow-chart to guide provider decision making for a disease process B. A list of standardized, best practice interventions for a patient population or disease C. A way of designing an intensive care unit to cohort patients according to their disease process D. Collaboration between multiple hospitals to provide the best possible patient care Preferred response: B

CHAPTER 136  Board Review Questions

Rationale A care bundle is a relatively short list of standardized, generally evidence-based or best practice interventions for a patient population or disease that, when implemented consistently, leads to improved outcomes. It is the combination of elements performed consistently and in aggregate that drive improvement. A clinical pathway is a flow chart to guide provider decision making. 3. The PICU leadership team at your hospital is discussing the results of a recent analysis of a quality and safety benchmark report showing a drop in performance measures over the previous three quarters. The PICU nursing director states that several PICUs have improved quality by learning from so-called high-reliability organizations. Which of the following industries are commonly cited as high-reliability organizations? A. Coal mining B. Commercial aviation C. Healthcare D. High-speed passenger railroad Preferred response: B

Rationale The most commonly cited examples of high-reliability organizations include U.S. Navy aircraft carrier flight deck operations, commercial aviation, nuclear power, and wilderness/forest firefighting. Whereas many healthcare organizations certainly aspire to become high-reliability organizations, there are few examples (if any) of high-reliability organizations in healthcare today. 4. High-reliability organizations are characterized by five core principles. Which of the following is a core principle of all high-reliability organizations? A. Commitment to resilience B. Deference to hierarchy (“command and control”) C. Lack of transparency D. Preoccupation with success Preferred response: A

Rationale The five core principles of high-reliability organizations include deference to expertise, reluctance to simplify, sensitivity to operations (which often means greater transparency), preoccupation with failure (and learning from failures), and commitment to resilience.

Chapter 3: Critical Communication in the Pediatric Intensive Care Unit 1. Which is an important factor in the successful implementation of huddles? A. Designating a leader B. Encouraging the elective participation of team members C. Holding the huddle in a remote location D. Performing on an ad hoc basis Preferred response: A

e3

Rationale All huddles should be led by a designated leader to keep the discussions brief and focused. Mandatory participation of all team members is required for a successful huddle. Huddles should be held in a central location to encourage all team members to attend. Huddles should be incorporated in an institution’s standard work. The duration of huddles should be 10 minutes or less, whenever possible. 2. What is the focus of team training programs? A. Avoiding errors during training to prevent confusion B. Developing communication strategies that promote hierarchy among team members C. Emphasizing individual tasks, duties, and responsibilities D. Facilitated debriefings Preferred response: D

Rationale Team training programs focus on facilitated debriefings to transform experiences into retained knowledge. Members are encouraged to learn all team tasks, duties, and responsibilities. Incorporating errors during training allows for the creation of contingency plans. The focus of communication strategies is to flatten hierarchy and encourage assertiveness. Closed-loop communication maintains a shared mental model among all team members.

Chapter 4: Professionalism in Pediatric Critical Care 1. Which of the following best describes the Charter on Medical Professionalism? A. It depicts the good and virtuous doctor. B. It focuses on putting the patient first. C. It portrays professionalism as a social contract. D. It recognizes the patient’s right to access all medical resources. Preferred response: C

Rationale The charter sets forth a social contract by which professionals justify the privileges of their profession by performing as professionals. It lays out principles that underlie 10 commitments of physicians to patients and society. It is not a description of the good and virtuous doctor. All recognize that doctor by his or her behaviors toward patients and others. It views the social contract in the context of the “medical marketplace” and social and financial pressure. Although it emphasizes altruism and putting the patient first, it goes well beyond that in defining broad commitments to society. The charter recognizes the patient’s right to make informed choices among the available options, but it also lays out commitments under which the physician distributes finite resources equitably and informs patients honestly and as completely as possible so that they can choose wisely.

e4

S E C T I O N XV   Pediatric Critical Care: Board Review Questions

2. Which is true of the Physician Charter? A. It describes professional responsibilities. B. It expresses ethical values. C. It mandates professional behaviors. D. It sets guidelines for regulatory agencies. Preferred response: A

Rationale The charter describes the responsibilities that fulfill the physician’s professional contract with society. It does not have the force of law and has no power to mandate professional behavior. It does not reduce physician responsibilities to guidelines for actions. It is not so much a statement of ethical values as it is a commitment to 10 practical physician responsibilities.

Chapter 5: Leading and Managing Change in the Pediatric Intensive Care Unit The following questions are based on this vignette: You are the medical director of a “closed” PICU in which surgical patients are co-managed by the surgical and critical care services with all other medical admissions admitted to the critical care service, and consultant services do not enter patient orders without express discussion with the critical care team. One common exception to this delineation of medical team hierarchy involves the anesthesia pain service which has maintained direct management of all patient controlled anesthesia (PCA) infusions even in the PICU. In the past 3 months, (1) two new orthopedic surgeons joined the practice and have contributed to a doubling of the usual admission volume for postoperative spinal fusion admissions, (2) a new delivery system was deployed for all PCA pumps that requires a code to be entered to reprogram the PCA settings, (3) a new hospital policy now restricts prescription of additional opioid medications to only pain service physicians and nurse practitioners for all patients who have PCA orders, and (4) the pain service has lost personnel going from 2 physicians and 4 nurse practitioners providing 24/7 coverage to 1 physician and 1 nurse practitioner that provide daytime in-person coverage with nights and weekend coverage provided by the on-call anesthesia team. At the same time, patient satisfaction scores have declined for these patients, anonymous family surveys report feeling like their child’s pain was ignored in the postoperative period, and multiple adverse event reports have been submitted detailing challenging conversations and delays in patient care. Specifically, families have complained, and the bedside nursing team has reported that acute pain episodes not resolved with PCA parameters are left unaddressed for too long while the critical care team calls

the pain service to adjust analgesia and add adjunctive therapies in response to breakthrough pain. Also, the PICU bedside team reports frustration that the pain service does not take into account nursing pain assessments when making decisions. 1. You have been tasked with implementing a new care guideline that addresses the perception that there is a delay in responding immediately to patient reported pain symptoms. As you consider how to address this patient care issue, the initial approach MOST LIKELY to achieve rapid and sustained improvement in management of postoperative pain in this patient population with the LEAST amount of disruption of established practice and work flow among involved parties is: A. Meet with the medical director of anesthesia pain service and explain that for all other consulting services on PICU patients, the PICU has direct management of all patient care. Therefore, the PICU will now assume primary responsibility of all pain management including PCA prescriptions. The pain service would be consulted only if the patient’s pain is not adequately controlled by the PICU team’s approach and at time of transfer from the PICU, at which time the pain service will assume primary responsibility for pain management as currently occurs in all other surgical inpatient admissions. B. Request a comprehensive report from the hospital Process Improvement Committee of the past year’s posterior spinal fusion post-op PICU admissions that details patient data from the first 24 hours of PICU admission related to pain scores, total patient opioid exposure, frequency of prn doses of rescue medications for increased patient pain, and pain service documentation. C. Meet with the PICU RN leading Process Improvement initiatives in the PICU to discuss the situation and identify multi-disciplinary and multi-professional representatives to investigate the scope of the concerns, to understand the current work flow and impacted team members, to list key drivers impacting this aspect of care, and to identify current “best practice.” D. Develop a comprehensive re-education process targeting new PICU RNs on the physiologic and psychologic aspects of pediatric pain and nonpharmacologic interventions that have been proven to reduce or eliminate need for pharmacologic intervention. E. Empower bedside PICU RNs to advocate for their patient and re-emphasize the role of the PICU charge RN to support and guide individual RNs through difficult situations. Preferred response: C

CHAPTER 136  Board Review Questions

Rationale While all five answers are reasonable components of developing a new patient care guideline, only answer C describes an initial planning process that maps out a plan for gaining understanding of the scope of the challenge and for identifying needed participants in the development and implementation of the new guideline. Depending on this preliminary planning work, any of the other four approaches could be needed components of the overall strategy. However, beginning with any of the other four answers could create significant resistance to change (answer A), create significant time-intensive data collection that does not answer targeted questions (answer B), repeat training that does not address the necessary gaps in knowledge (answer D), or fail to recognize the role of other PICU team members in addressing the situation (answer E). 2. You and your team have developed a new patient care guideline for post-op pain management for these patients. The MOST EFFECTIVE and MOST RESPONSIVE means of evaluating the initial impact of these new guidelines is to: A. Develop data collection tools that populate quarterly reports for review. B. Track monthly patient satisfaction scores and surveys and review the results with PICU staff. C. Perform rapid, real-time data collection for each patient impacted by the new analgesia and solicit verbal feedback on whether or not the patient’s pain has been adequately managed. D. Perform rapid, real-time data collection of pre-determined objective and subjective data elements and solicit input from bedside PICU team members for ways to improve the new guideline that can be immediately incorporated and evaluated after ad hoc review by a core team of identified project leaders. E. Hold monthly meetings during which the current bedside team caring for any patients undergoing posterior spinal fusions can provide their input on how the guidelines are working. Preferred response: D

Rationale Rapid Plan-Do-Study-Act (PDSA) cycles allow immediate assessment of the impact of change as well as timely and responsive adjustments to new initiatives that address unanticipated consequences of the new guidelines. PDSA cycles evaluate predetermined outcomes and promote transparency about the effort for all involved PICU team members. In order to avoid premature changes based on single instances, a core leadership team should evaluate proposed changes and adjust planned performance metrics accordingly.

e5

3. Since deployment 6 months ago, these postoperative analgesia management guidelines have been very successful and are being used as a model for interprofessional pain management and team communication among other surgical specialty patients. A senior anesthesiologist who started the pain service and was away on extended sabbatical over the past year returns to clinical service. When first informed, he was skeptical about the guidelines, citing 30 years of personal experience in postoperative pain management as being superior to the approach used in the guidelines. He has cancelled two previously informally scheduled meetings with you to review the new guidelines and discuss his specific concerns. In the past 2 weeks, he has become more vocal in his objections and has voiced his disagreement in front of families in the middle of morning rounds, leading to multiple complaints by PICU team member and family members. You have firsthand experience working with this physician on other successfully implemented patient care initiatives and have always found his brusque comments and opinions to be insightful and well-meaning. You believe that you have a productive, honest, and friendly relationship with him based on your prior shared successes. The BEST initial method for addressing this physician’s disruptive behavior is: A. Meet with the hospital Chief Medical Officer and the department chair of anesthesiology to discuss this individual’s behavior and request that he be removed from participation in clinical care for these patients until he has been counseled and completed education and training on these guidelines. B. Meet him in his office with two cups of coffee to begin an impromptu conversation to better understand the basis of his disagreements with the guidelines. C. Hold an intervention meeting with the anesthesiologist, yourself, and one or two members of your core guideline development leadership team to acknowledge that the anesthesiologists recent behavior has been disruptive and counterproductive to your shared goals for delivering optimal patient care, to describe the ways in which the guidelines have improved patient care, acknowledge the instances in which the guidelines could still be optimized, and to ask for specific thoughts on how the guidelines can be modified to address his specific concerns and misgivings. D. Collect statements from witnesses of his disruptive behavior to share with him and his department chair as examples of the negative impact of his actions, demand that he apologize to the PICU team, and set the expectation that he comply with the analgesia guidelines. E. Refer the situation to the hospital professional standards committee. Preferred response: C

e6

S E C T I O N XV   Pediatric Critical Care: Board Review Questions

Rationale Addressing disruptive behavior directly can be uncomfortable and challenging but is essential in maintaining morale and momentum when leading and implementing change initiatives. Depending on the situation, the specific approach can be preventative and informal (answer B), accusatory (answer D), or punitive (answers A and E). However, in a situation in which the disruptive individual has cancelled previously scheduled meetings, has not commented on the strengths and weaknesses of the guideline, and is interfering with constructive interprofessional team communication and collaboration, a direct confrontation that emphasizes a shared common goal (improving patient care) and outlines unacceptable behavior that still leaves room for education (review of the guideline development process and observed outcomes) and dialogue (specific solicitation of guideline critique) can preserve relationships and avenues for partnership on future projects.

Chapter 7: Fostering a Learning Healthcare Environment in the Pediatric Intensive Care Unit 1. Which of the following elements provides a foundation for a learning healthcare environment? A. Best practice B. Clinical research C. Professionalism D. Shared educational model Preferred response: C

Rationale The elements of professionalism, namely accountability, respect, and teamwork, need to be consistently present in order for best practice, clinical research, and shared education to occur. 2. Which of the following accurately characterizes gender disparity in the medical workspace? A. About 10% of female physicians have experienced sexual harassment. B. A significant gender pay gap persists in medicine. C. Male and female physicians are both likely to be introduced as “doctor.” D. Women receive similar research start-up funding when compared to men. Preferred response: B

Rationale 30% of female physicians have experienced sexual harassment. Women receive significantly less research start-up funding when compared to men. Male physicians are more likely to be introduced as “doctor,” compared to female physicians. In addition, women are less likely to be first authors in top tier journals; women are less likely to be included on expert guideline consensus panels; there are fewer women in leadership positions, even in pediatrics; and there are fewer women full professors as compared to men.

3. Which of the following characteristics of clinical standard work is correct? A. Amplifies occurrence of nuisance variables B. Complicates communication among providers C. Establishes a baseline for continuous improvement D. Increases waste transiently with implementation Preferred response: C

Rationale Standardization utilizing clinical standard work facilitates identifying and eliminating waste, communicating between providers, establishing a baseline for continuous improvement, and minimizing noise/controlling for nuisance variables. Standardization represents the foundation for iterative improvement, and without standardization, measurements of improvement are not possible. 4. Which of the following outcomes has been reported in a doseresponse fashion as a function of proportional compliance with the ICU Liberation ABCDEF bundle elements? A. Decreased incidence of anemia requiring transfusion B. Decreased ventilator-associated lung injury C. Improved ICU and hospital survival D. Increased proportion of rapid eye-movement sleep Preferred response: C

Rationale Two independent cohort analyses demonstrated that proportional compliance with ABCDEF bundle elements resulted in significant and dose-related improvements in patient-centered, clinically meaningful outcomes such as survival, duration of mechanical ventilation, neurological organ dysfunction (i.e., delirium and coma), use of physical restraints, ICU readmission rates, and discharge disposition of ICU survivors. Anemia, ventilator-associated lung injury, and sleep quality data have not been published. 5. Which aspect of simulation in an interdisciplinary teaching model is particularly effective? A. Real time debriefing B. Role playing C. Systems thinking D. Teamwork practicing Preferred response: A

Rationale All activities related to simulation-based education are important, but real-time team debriefing around critical events (doing in context) represents a particularly effective interdisciplinary (simulation) teaching modality. 6. In addition to identification of best available evidence to support best practice, what is the other benefit of a learning healthcare environment? A. Encouraging empiric treatment modalities B. Facilitating craftsman care attitude C. Promoting wellness and resiliency D. Training basic researchers Preferred response: C

CHAPTER 136  Board Review Questions

Rationale The second, sometimes less obvious benefit of a learning healthcare environment is promoting wellness and resiliency among critical care providers. Participation of the interdisciplinary team in shared education and research/quality improvement activities provides opportunities for critical care providers to debrief and reflect, to provide mutual support, and to reinvigorate a sense of purpose.

Chapter 8: Challenges of Pediatric Critical Care in Resource-Poor Settings 1. Which of the following is the leading cause of childhood deaths under the age of 5 years in low-middle income countries? A. Diarrheal illnesses B. Lower respiratory illnesses C. Malaria D. Neonatal problems E. Road traffic injuries Preferred response: D

Rationale The majority of childhood deaths under the age of 5 years in lowand middle income countries are related to neonatal problems (34%), followed by lower respiratory (16%) and diarrheal illnesses (10%), as well as malaria (7%). Road traffic injuries play an important role in children 5 to 14 years old. 2. You are the attending consultant in a tertiary care center in a low-middle income country. The unit is equipped with facilities for mechanical ventilation (1 available ventilator), intermittent arterial blood gases, vasopressors, intermittent blood products. You are on call, and there are two patient calls: The first call is from the casualty senior resident for an 11-yearold child with chronic liver disease with pneumonia of 5 days duration with pediatric acute respiratory distress syndrome (PARDS) requiring mechanical ventilation and monitoring. The second call is for a child admitted in the pediatric ward. A colleague of yours is requesting shifting this 10-yearold child with a rare genetic disorder, who he followed since birth to the ICU with septic shock, PARDS, and multiorgan dysfunction. The child is also being referred from the ministry of health. Which of the following policies regarding admission to PICU are to be considered while deciding admissions in units in low resource settings? A. Chronological order in which PICU requests are made do not matter. B. Optimization of overall benefit that could result from use of PICU resources C. Preference is to be given for requests made by the central and state health authorities or hospital administration. D. The trainee can decide “who to admit” or “who not to admit’” without discussion with the attending physician. Preferred response: B

Rationale In resource limited countries, many children do not have access to ICUs. With a growing demand for intensive care and availability of limited resources, having policies for ICU admission therefore is of significant importance in such settings. Guidelines available for admission and discharge policies from high income countries

e7

may not be applicable to low- and middle-income countries with high burden of mortality from acute illnesses. However, the broad principles that could be adopted based on the patient population and resource availability have been discussed by Argent et al. At Red Cross War Memorial Children’s in South Africa, for example, general principles for PICU admission are the following: • Children will be considered for admission in terms of the chronological order in which that request is made • Preference will not be given to children from the Red Cross War Memorial Children’s Hospital. In fact, where possible, preference will be given to children who are referred from other areas (where they have very limited access to alternative intensive care) • No child will be refused admission to the PICU without a decision by the ICU attending on call at the time • No trainee will be asked to make a final decision about the appropriateness of PICU admission. This MUST be a consultant decision. 3. Which of the following statements is false regarding performance of PCCM research in low resource settings (LRS)? A. Investigators in LRS would benefit from participation in networks that include other investigators within LRS and collaborations with investigators outside of LRS. B. Research programs based on high-income countries should lead efforts to solve problems in LRS as they have better resources. C. Research led by investigators in LRS have informed important changes in clinical guidelines for fluid resuscitation for shock and for cost-effective delivery of noninvasive respiratory support. D. The research agenda for LRS should be informed by research priorities identified in LRS. Preferred response: B

Rationale Research priorities are ideally based on indicators such as the leading causes of child morbidity and mortality and thus vary by region. Clinicians and researchers in low-resource regions should lead prioritization of the research agenda, even though there are less research resources available including training opportunities and funding. Collaboration, research network, and training opportunities in concert with investigators and programs in highresourced regions may serve to benefit overall strengthening of research infrastructure in low-resourced regions.

Chapter 9: Public Health Emergencies and Emergency Mass Critical Care 1. Your hospital receives news from the World Health Organization and the Centers for Disease Control that there is an emerging infectious disease rapidly spreading internationally. Estimates indicate your area will start seeing cases within the week. What actions should the ICU team take to prepare? A. Activate crisis standards of care B. Review the incident command system with ICU leaders C. Teach staff to provide manual ventilation D. Utilize personal protective equipment for all patients Preferred response: B

Rationale Preparing for a potential pandemic should prompt each unit to review their own internal emergency plans and ensure all key staff

e8

S E C T I O N XV   Pediatric Critical Care: Board Review Questions

are aware of how decisions will be made and communicated during the public health emergency (PHE). The primary way decisions are centralized and made with the best available information is the Incident Command System (ICS). ICU leaders will need to know what information they are expected to provide the incident command center and what decisions the incident commander will make versus what the local leaders will make. As the current resources are adequate to provide for the patients there is not a need to utilize crisis standards of care yet; hopefully conservation and contingency strategies will help avoid the need to utilize crisis standards of care. Personal protective equipment (PPE) should not be used for all patients as this will exhaust supplies of critical equipment. Manual ventilation should only be used as part of emergency mass critical care (EMCC) and when mechanical ventilators are not available; as such, this technique should not be a high priority for education and instead educational efforts should focus on reviewing the hospital and unit disasters plan for infectious outbreaks. 2. The emergency department calls to alert the ICU that a local shopping center building has collapsed on a Saturday with many people inside, and they are expecting many casualties to arrive within minutes. What is the correct order of steps to take? A. Communicate with the incident commander, send a triage team to the ED, rapid transfer/discharge of any non-ICU level patients B. Rapid assessment of surge capacity, communicate with the incident commander, send a triage team to the ED C. Rapid transfer/discharge any non-ICU level patients, communicate with the ICS, assemble a triage team D. Send a triage team to the ED, communicate with the incident commander, rapid assessment of surge capacity Preferred response: B

Rationale In any potential surge knowing the available hospital surge capacity is a critical first step. Upon notice of an incident each unit performs a rapid assessment of which patients can be safely transferred or discharged and communicates this information succinctly to the incident commander. Units should not proactively transfer or discharge patients without word from the ICS as this could result in uncoordinated, unneeded, or inappropriate allocation of resources. After alerting the incident commander, members of the ICU designated for the triage team should proceed to the ED to aid in helping with identifying patients appropriate for rapid transfer to the ICU. 3. A prolonged surge event due to severe influenza has caused the hospital to activate their incident command center (ICS) and utilize strategies to meet surge needs. The incident commander sends word that the ICU should begin converting OR spaces to ICU spaces, that personal protective equipment goggles should be cleaned and reused whenever possible, and that ICU staff are being asked to work overtime. What level of surge response is this? A. Conventional B. Contingency C. Crisis D. Emergency mass critical care Preferred response: B

Rationale Converting reasonably equipped hospital spaces to ICU space, selective reuse of supplies, and extension of existing ICU staffing pool are all contingency strategies. Conventional strategies lie in conserving resources, utilizing available staff without significantly extending staffing, substituting only where equivalent, and making use of supply caches. Crisis strategies include converting poorly equipped or non-hospital areas to ICU spaces, using nonICU staff for ICU care, and reallocating (rationing) scarce resources. EMC refers to the global category of needing to provide ICU care to a much larger than usual number of patients. 4. Another hospital in the area calls to request that some of their ICU level patients be moved to your hospital’s ICU. They were the hospital closest to a school bus crash and received 15 critically ill pediatric victims from the crash. Their usual ICU capacity is 16 patients, and they were at capacity before the event. Your ICU’s usual capacity is 80 patients and it is usually .90% occupied. What size of surge did the 15 victims create at the outside hospital and what size surge would your ICU have if you accepted all 15? A. Major surge at outside hospital, minor surge for your ICU B. Minor surge for outside hospital, usual volume for your ICU C. Moderate surge for both D. Moderate surge for outside hospital, minor surge for your ICU Preferred response: D

Rationale For a 16 bed ICU an additional 15 patients is a 94% increase, which meets the range of a moderate surge (20–100% increase from usual capacity). For an 80 bed ICU, an additional 15 patients is an 18% increase which is a minor surge (up to 20% above usual capacity). This scenario would not constitute a major surge (.100%) for either unit.

Chapter 10: Lifelong Learning in Pediatric Critical Care 1. You are preparing to teach a group of intensive care fellows about how to conduct an effective family meeting. Interpersonal and communication skills are an example of which of the following: A. Competency domain B. Easily testable skill C. Entrustable professional activity D. Milestone E. Stage of skill acquisition Preferred response: A

Rationale The six core competency domains established by the ACGME are: • Practice-Based Learning and Improvement • Patient Care and Procedural Skills • Systems-Based Practice • Medical Knowledge • Interpersonal and Communication Skills • Professionalism These six domains should be used to guide and coordinate evaluation of all residents or fellows in their development.

CHAPTER 136  Board Review Questions

2. A critical care fellow is preparing a simulation scenario to practice communication skills during cardiopulmonary resuscitation with the bedside team. This type of simulation is an example of which of the following? A. Crew Resource Management B. Procedural Training C. Resuscitation Manikin Teaching D. Standardized Role Play E. Task Training Preferred response: A

Rationale Crew resource management was initiated in the aviation industry to improve safety in critical environments by focusing on interpersonal communication, leadership, and decision making. 3. You are preparing a curriculum to teach residents about pediatric acute respiratory distress syndrome. Which of the following is an example of a method you might use that embraces adult learning principles? A. Behavior management strategy B. Didactic lecture using PowerPoint C. See one, do one, teach one D. Small group learning E. Videotaped lecture with question and answer period Preferred response: D

Rationale Adults are self-directed and autonomous. They learn best when they are active participants in the learning process and are allowed to practice newly acquired skills and concepts. As a consequence, education is typically most effective when programs facilitate selflearning with specific goals of acquiring practical information. Efforts to be inclusive of curricular methods that support adult learning principles are occurring in undergraduate, graduate, and continuing medical education. Problem-based and small group learning, flipped classrooms, and simulation exercises allow many venues for reaching learners in different ways. 4. You are preparing to take the American Board of Pediatrics Critical Care Subspecialty examination. Which of the following is not required to take the examination? A. A job as a pediatric intensivist B. Evidence of meaningful scholarly project during pediatric critical care fellowship C. Initial certification in General Pediatrics D. Successful completion of a Critical Care Medicine fellowship program E. Valid unrestricted license to practice Preferred response: A

Rationale To qualify for the ABP subspecialty examination, applicants are required to have a valid unrestricted allopathic and/or osteopathic license to practice, initial certification in general pediatrics, and

e9

successful completion of critical care medicine training in a program accredited by the ACGME. The applicant must also provide evidence of meaningful scholarship during training which can include research, an educational project, or a quality project. 5. You are planning to teach a group of critical care fellows how to manage a patient with acute respiratory distress syndrome. Which of the following instructional techniques takes best advantage of the qualities of the adult learner? A. Attending a webinar about ventilation strategies followed by a short quiz B. Engaging in a detailed review of real patient cases followed by discussion of ventilation strategies C. Attending a lecture with a PowerPoint presentation followed by a quiz D. Reading a research article on ARDS and then discussing the merits of the study Preferred response: B

Rationale Adult learners thrive with active, engaged learning environments. Although all options have the potential for active, engaged learning, response B, review of real patient cases followed by discussion of care strategy, is an example of problem-based learning and requires active engagement by the learners. It is also clearly related to their experience. 6. Which of the following is an example of an entrustable professional activity (EPA)? A. Identifies subtle or unusual physical exam findings B. Recognizes a patient requiring urgent or emergent care and initiates evaluation and management C. Routinely provides vital signs and other laboratory data on rounds D. Shows self-awareness of one’s own knowledge, skill, and emotional limitations that leads to appropriate help- seeking behaviors Preferred response: B

Rationale Domains of competence refer to the six groups of physician subcompetencies defined by ACGME, including medical knowledge, patient care, practice-based learning and improvement, interpersonal and communication skills, professionalism, and systemsbased practice. Milestones are the developmental levels of knowledge, skills, and attitudes for each subcompetency and describe the progression of a physician’s ability. Entrustable professional activities (EPAs) are tasks or responsibilities that trainees are entrusted to perform unsupervised once they have attained sufficient competency. EPAs have been developed with patient safety in mind, but they also provide a framework for curricular development and lifelong progression of learning. Response B, “recognizes a patient requiring urgent or emergent care and initiates evaluation and management,” is a discrete, concrete skill that can be observed and entrusted.

e10

S E C T I O N XV   Pediatric Critical Care: Board Review Questions

Chapter 11: Essential Concepts in Clinical Trial Design and Statistical Analysis 1. In a randomized controlled trial to evaluate the effects of prone positioning compared to supine positioning in pediatric acute respiratory distress syndrome (PARDS), bedside nurses and treating physicians are asked to complete daily questionnaires classifying whether the patient has had clinical improvement, stability, or deterioration. This study is at risk of what kind of bias? A. Ascertainment bias B. Bias due to loss of data C. Lead time bias D. Selection bias Preferred response: A

Rationale Ascertainment bias exists when those responsible for determining outcomes are aware of treatment group assignment. In this study, it would not be possible to blind bedside nurses and treating physicians to study group assignment, and knowledge of the treatment group assignment might influence their responses regarding illness trajectory. Bias due to loss of data occurs when substantial patient outcome data is missing, or when data is missing related to treatment arm assignment. Lead time bias is a risk in screening intervention studies, where time to an outcome may increase only because an illness was identified earlier than it would have naturally presented, but the screening intervention did not actually impact the occurrence of the outcome. Finally, selection bias occurs when certain patients are preferentially enrolled in the treatment or control arm of a study, creating imbalance between the study arms. 2. A randomized controlled trial of a biologic immunomodulating drug in patients with severe sepsis decreased mortality to 7% among patients receiving treatment, compared to 21% among patients in the control arm. Among patients with similar baseline risk of mortality, what is the number needed to treat (NNT) with this medication to save one life? A. 3 B. 3.5 C. 7 D. 14 E. Not enough information to answer Preferred response: C

Rationale NNT is the inverse of the absolute risk reduction (ARR) 3 100, or 1/(riskuntreated – risktreated) 3 100. In this case, ARR 5 21% 2 7% 5 14%, and 100/14 ,7. The NNT can be adjusted for baseline risk; among septic patients with twice the baseline risk of mortality (,42%), the NNT would be halved, at 3.5. 3. An interventional study for patients with traumatic brain injury randomizes half to receive an early physical therapy and mobilization bundle, compared to standard of care. What is the most appropriate statistical test to compare the hospital length of stay between the two groups? A. ANOVA B. Chi square test C. T-test D. Wilcoxon rank-sum test Preferred response: D

Rationale Hospital length of stay is a good example of a continuous variable with a skewed distribution, as most patients have short lengths of stay, but a handful of patients have very long lengths of stay. A nonparametric test, like the Wilcoxon rank-sum test, is most appropriate for comparing the median of this kind of outcome measure between two groups. A t-test is appropriate for comparing the mean of a normally distributed outcome measure between two groups (e.g., low-density lipoprotein cholesterol.) ANOVA is used to compare means of a normally distributed outcome measure between multiple groups. Finally, a chi square test is used to compare frequencies in a 2 3 2 table, typically used for binary outcomes. 4. An investigator wishes to conduct a randomized trial examining continuous renal replacement therapy (CRRT) for fluid overload in patients with severe acute respiratory distress syndrome (ARDS). Which of the following is true for this study? A. If the study results demonstrate a relative risk of mortality of 0.8 in the treatment group, with a 95% CI of 0.55-1.05 and a p value of 0.12, then we can conclude that CRRT is not effective in improving outcomes for patients with severe ARDS. B. Patients who are randomized to CRRT but do not receive it because their hemodialysis catheter malfunctioned should be analyzed along with patients who are in the control group. C. The outcomes should be compared based on original treatment group assignment, regardless of what therapy the patient actually received. D. The outcomes should be compared based on what therapy the patient actually received, regardless of original treatment group. E. This study cannot be conducted because it is impossible to blind patients and providers to the treatment group assignment. Preferred response: C

Rationale Intention to treat analysis preserves the benefits of randomization, and patients who are randomized to a treatment group should be analyzed along with that group. It is legitimate to conduct studies in which blinding is not possible; however, study staff who ascertain subjective outcomes should be blinded to treatment group assignment. These study results suggest that there may be a treatment effect—the majority of the 95% confidence interval lies below 1.0—but, there is a reasonable probability (12%) that this estimate of effect was found by chance. It may be that there is no effect, but it may also be that this study was underpowered to identify this 20% decrease in mortality.

CHAPTER 136  Board Review Questions

Chapter 12: Prediction Tools for Short-Term Outcomes Following Critical Illness in Children 1. A 4-year-old previously healthy boy is admitted to the intensive care unit with septic shock. He is intubated on arrival to the unit, started on broad spectrum antibiotics, and fluid resuscitated. He has ongoing hypotension and multiple vasoactive infusions are initiated. His laboratory studies demonstrate evidence of disseminated intravascular coagulation (DIC) and his most recent arterial blood gas analysis demonstrates a severe metabolic acidosis with a lactate of 8. The resident on call tells you that his Pediatric Risk of Mortality (PRISM) score two hours after admission is 19. The most correct interpretation of the PRISM score in this patient is: A. His risk of mortality during this admission is 19%. B. His risk of mortality in the next two hours is 19%. C. It cannot be interpreted because he has not yet been fully resuscitated. D. It cannot be interpreted because he has not yet been admitted long enough. E. It cannot be interpreted because PRISM does not apply to individual patients. Preferred response: E

Rationale PRISM scores are not designed to be interpreted for an individual patient, but instead provide a mechanism by which case-mix can be measured over a large group of patients. PRISM scores are calculated during the first 4 hours of a patient’s admission to the ICU and include physiologic and laboratory data for 2 hours prior to and 4 hours after admission. This time limitation is intended to more accurately represent a patient’s presenting physiologic status, as opposed to the provision of intensive care. 2. As part of a quality improvement initiative, a colleague has created a model to examine patients at risk of developing pressure injuries while in the intensive care unit. She reports in a divisional research meeting that the discrimination of the model is 0.92. Which of the following is the most correct interpretation of this value? A. The model’s overall sensitivity is 92%. B. The model’s overall specificity is 92%. C. The positive predictive value of the test is 92%. D. 92% of variation in a patient’s likelihood of developing a pressure injury is explained by the model. E. 92% of patients with pressure injuries will meet all the components of the model. Preferred response: B

Rationale Discrimination is a model’s ability to correctly differentiate patients with a specified outcome from those without the outcome. This is in contrast to calibration, which is the model’s ability to predict overall event rates. The model’s discrimination is often reported as the C statistic, which represents the average sensitivity of the model over the range of possible specificity values.

e11

3. A fellow is planning a research project to study patient risk factors for mortality in patients with traumatic brain injury. She is hoping to include patients from multiple centers and would like your advice about how to control for variation in risk between hospitals. Which of the following is most useful for the research team to assess? A. The sample size from each hospital. B. The standardized mortality ratio for each hospital. C. The hospital length of stay for each included patient. D. The yearly number of traumatic brain injury patients admitted to each hospital. E. The presence or absence of neurosurgical services at each hospital. Preferred response: B

Rationale Standardized mortality ratios represent the ratio of observed to expected mortalities and provide a mechanism by which researchers can control for case mix variation between institutions. This can help to isolate patient characteristics, which are of interest to the fellow, from systems-level characteristics. While the other choices represent important considerations, the standardized mortality ratio is most important to allow for control. 4. A 4-month-old former 35-week gestational age girl is admitted to the intensive care unit for hypoxic respiratory failure requiring mechanical ventilation. She was intubated at an outside hospital and was started on broad spectrum antibiotics for concern for pneumonia with an elevated white blood cell count (WBC) and infiltrate on chest radiograph. As part of her admission process, the bedside nurse completes a worksheet with information for the Pediatric Risk of Mortality (PRISM) score as well as the Paediatric Index of Mortality-3 (PIM3) score. Which of the following pieces of information will be included in the PIM3 but not PRISM? A. The pH from an arterial blood gas obtained one hour after admission. B. The measured systolic blood pressure at admission. C. The results of pupillary reflex testing. D. The fact that the child is undergoing mechanical ventilation. E. The measured WBC obtained at the outside hospital one hour prior to admission. Preferred response: D

Rationale Both PIM3 and PRISM include evaluation of physiologic and clinical data for a patient but the methods differ slightly in the amount of included data, the observation period, and inclusion of nonphysiologic data. Specifically, PIM3 includes mechanical ventilation within an hour of admission, while PRISM includes more laboratory values. Both scoring systems include evaluation of systolic blood pressure and pupillary reflexes.

e12

S E C T I O N XV   Pediatric Critical Care: Board Review Questions

5. A researcher develops a model to predict the risk of patients admitted to the PICU of developing a deep venous thrombosis based on a number of demographic and clinical factors. The model is developed on 1000 patients with an area under the receiver operating characteristic curve (AUC) of 0.8. The model is subsequently validated on a separate dataset of 1000 patients with an AUC 5 0.7. What term best describes the model’s performance as quantified by the AUC? A. Calibration B. External validity C. Discrimination D. Internal validity Preferred response: C

Rationale Discrimination is the accuracy of a model in differentiating outcomes groups and is most often assessed by the AUC. External validity is the extent to which the results of a study can be generalized to other situations and other people. External validity is typically assessed through the process of study replication. Calibration refers to the ability of a model to assign the correct probability of outcome to patients over the entire range of risk prediction. The most accepted method for measuring calibration is the Hosmer-Lemeshow goodness-of-fit test. Internal validity relates to the extent to which a causal conclusion based on study findings is warranted and is assessed by the degree to which the study design minimized systematic error or bias. The precision of a measurement system is the degree to which repeated measurements under unchanged conditions show the same results. 6. A previously healthy 7-year-old male is admitted to the PICU of a tertiary care children’s hospital in septic shock secondary to lobar pneumonia. He is tachycardic (heart rate, 130 beats per minute) and tachypneic (respiratory rate, 55 breaths per minute) and undergoes intubation of the trachea within 30 minutes of PICU admission for respiratory failure. Arterial blood gas reveals pH 5 7.2, Paco2 5 40 mm Hg, Pao2 5 80 mm Hg, and HCO2 3 5 12 mEq/L on Fio2 of 0.6. Coagulation studies reveal an elevated INR 5 1.8. Which scoring system is most appropriate to assess this patient’s physiologic status on the initial day of PICU admission? A. PELOD B. PRISM III C. SNAPPE-II D. STS-EACTS score Preferred response: B

Rationale CRIB II and SNAPPE-II are scoring systems for neonates. STS-EACTS score was developed for patients with congenital heart disease. PELOD is a scoring system designed to quantify the degree of organ dysfunction in PICU patients and correlates well with mortality. PELOD, however, is calculated based on the greatest degree of physiologic dysfunction to occur at any point during the entire PICU admission, not just the initial day of admission. PRISM III is a physiology-based acuity score that is calculated in the interval from the 2 hours preceding PICU admission to 4 hours following PICU admission.

7. You are asked to accept the transfer of a 2-year old child with respiratory failure secondary to viral pneumonitis from a rural emergency department (ED). The child was seen and discharged from that ED 2 days ago. This morning, her parents awoke to her loud breathing and called emergency medical services. The emergency medical technicians assessed the child, determined that she had respiratory failure, stabilized her airway, provided bag-valve-mask ventilation with oxygen, and provided respiratory treatments during transport to the local ED. They notified the ED of the child’s condition. The ED is performing additional steps for stabilization, along with some diagnostic tests, including a chest radiograph and basic laboratory studies. The ED has explained to the family that their child will need to be transported to the local children’s hospital, and someone from the ED is calling you for a helicopter transport. This scenario demonstrates appropriate system design for the critically ill child around which of the following issues? A. Access B. Cost C. Outcome D. Process Preferred response: A

Rationale Systems are designed to facilitate easy access to services for those who need them. Access (response A) describes organized and integrated care for critically ill children and their families who are in need of tertiary pediatric services. Cost (response B) is an important element of healthcare delivery that focuses on the monetary resources devoted to patient care. However, economic terms are not discussed in this scenario. Research is an important activity in generating new knowledge, but this scenario is concerned with the delivery of care to an individual patient. Although the child ultimately may be enrolled in a research project or database evaluating outcomes, research is not the primary focus of her care. Process measures (response D) are those that occur between patients and providers or between various providers. Although a number of clinical processes have occurred in the scenario, the scenario also includes a number of interactions between other structural components, such as institutions and emergency services, which establish an organized hierarchy of interactions for the delivery of care. The final outcomes of care from the ICU perspective (response C) are not discussed in the scenario. Although the child was effectively rescued and delivered to the rural ED and a number of intermediate outcomes may be described, the final outcomes in terms of vitality, economics, or morbidity are yet to be determined. Therefore, the most appropriate answer is A. 8. After the first ICU day, the child’s PRISM III score is 8. What should you do? A. Ignore the score for today. B. Remove the child from mechanical ventilation and provide comfort care. C. Rescore the patient using the Pediatric Index of Mortality (PIM). D. Recognize that this score is correlated with a population risk of mortality. Preferred response: D



CHAPTER 136 Board Review Questions

e13

Rationale

Rationale

The PRISM score is a physiologically based severity of illness measure calculated from variables determined within the first 24 hours of care. It is helpful for understanding the risks for mortality and length of stay for populations of patients with a given severity of illness. However, the score cannot be applied for prognostication at the level of individual patients; hence it should not be used to guide clinical decisions as suggested in response B. Repetitive scoring using PRISM during hospitalization (response A) has not been validated. Using the PIM (response C), which captures care at the point of admission rather than at 24 hours, provides a different measure of severity that also can be compared across populations. Importantly, a bias exists in relation to the inclusion of mechanical ventilation as a predictor variable in PIM. Therefore the most appropriate answer is D.

Boyle’s law (P1V1 5 P2V2) describes the relationship between pressure and volume of a gas in an enclosed space at a constant temperature. Given that this patient will be rising in altitude, atmospheric pressure is expected to decrease with increasing height. Any gases trapped in enclosed spaces are expected to expand proportionately to any decreases in atmospheric pressure. Substituting the known pressures and volumes into the equation allows for calculation of the unknown cuff volume at the highest altitude:

Chapter 13: Pediatric Critical Care Transport 1. What is the most common mode of inter-facility transport for pediatric patients? A. Ferry B. Fixed-wing C. Ground D. Rotor-wing Preferred response: C

(741  2  630  V2 ) V2  2.4 mL 3. According to the Emergency Medical Treatment and Labor Act (EMTALA) established by the COBRA legislation, an appropriate transfer criterion includes which of the following? A. The referring hospital must provide care and stabilization within its ability. B. The referring physician certifies that the medical benefits expected from the transfer outweigh the risks. C. The referring physician consents to transfer after being informed of the risks of transfer. D. The receiving facility must have available space and qualified personnel and agree to accept the transfer. Preferred response: B

Rationale

Rationale

The advantages of ground transport include easier and direct access to referring and receiving locations, lower cost, and ability to respond in most weather conditions. Circumventing traffic rules via use of lights and sirens does not improve the outcomes of pediatric transports and significantly increase the risk of an accident. Rotor-wing (helicopter) transport may be faster than ground transport for patients located 45–60 miles away from admitting institution, particularly in geographically challenging areas such as waterways, and can help to minimize out-of-hospital time. Fixed-wing transports are typically reserved for long-distance flights and have the ability for cabin pressurization. Disadvantages of both rotor-wing and fixed-wing transports are limited space and high cost.

EMTALA delineates rules for inter-facility transfer. Appropriate transfers must meet the following criteria: (1) the transferring hospital must provide care and stabilization within its ability; (2) the referring physician certifies that the medical benefits expected from the transfer outweigh the risks; (3) the patient consents to transfer after being informed of the risks of transfer; (4) the receiving facility must have available space and qualified personnel and agree to accept the transfer; (5) copies of medical records and imaging studies should accompany the patient; and (6) the inter-facility transport must be made by qualified personnel with the necessary equipment.

2. A pediatric patient is intubated for acute respiratory failure secondary to pneumonia at a critical access hospital and now requires rotor-wing transport to the nearest pediatric intensive care unit for ongoing care. The referring facility is located at an altitude of 718 ft above sea level where the atmospheric pressure is 741 mm Hg. The flight path will take the patient over a mountain pass with an unpressurized cabin altitude equivalent to 5100 ft above sea level and an atmospheric pressure of 630 mm Hg. If the endotracheal tube cuff is inflated with 2 mL of air prior to departure from the referral facility, what is the expected volume of air in the cuff when at the highest altitude? A. 1.5 mL B. 2.4 mL C. 3.0 mL D. 3.2 mL Preferred response: B

4. Which of the following characterizes a typical skilled inter- facility pediatric transport team in comparison to care providers at a referring hospital? A. Administration of drugs not typically available at nontertiary centers B. Aggressive, early institution of simple therapies C. Availability of superior airway and breathing support modalities D. Superior diagnostic capabilities Preferred response: B

Rationale For most pediatric critical illnesses, definitive care, ideally beginning with a skilled inter-facility transport team, does not involve miracle drugs or technologies but rather the early, aggressive administration of simple therapies. Therapy that includes timely initiation of resuscitation fluids, early administration of inotropes (frequently via peripheral intravenous or intraosseous catheters), and early antibiotic therapy can improve outcomes.

e14

S E C T I O N XV   Pediatric Critical Care: Board Review Questions

Chapter 14: Pediatric Vascular Access and Centeses 1. Which of the following is a recommended practice to reduce catheter-related blood stream infections (CRBSI) from central venous catheters (CVC)? A. Full barrier precautions at the time of insertion B. Giving all intravenous antibiotics through the catheter C. Povidone-iodine ointment applied to insertion site daily D. Vancomycin containing heparin flushes Preferred response: A

Rationale CRBSI is the most common complication related to CVCs. In children, the location of the insertion site is not related to infection risk. The risk of infection is decreased by the use of a bundle of practices during insertion and ongoing maintenance of the CVC. The insertion bundle includes strict maximal sterile barrier precautions and aseptic technique. Dressing changes with chlorhexidine skin prep, minimizing catheter access, and daily assessment of the need of the catheter are all recommended as a part of CVC maintenance. Antimicrobial-impregnated catheters may decrease the risk of catheter-related infection, but more pediatric studies are needed. 2. Which of the following is the most common complication of intraosseous (IO) infusion? A. Fat embolus requiring mechanical ventilation B. Fracture requiring internal fixation C. Hypercalcemia from bone demineralization D. Infiltration of fluids into the surrounding tissues Preferred response: D

Rationale Significant complications of IO insertion and infusion are rare. The most common complication is extravasation of fluid. The causes of extravasation include incomplete penetration of the bony cortex, movement of the needle such that the hole is larger than the needle, dislodgment of the needle, penetration of the posterior cortex, and leakage of fluid through another hole in the bone, such as a previous IO site or fracture. Extravasation of a small amount of fluid is usually not problematic, but with larger volumes, compartment syndrome can develop and may require fasciotomy and even amputation. Use of the IO line for prolonged periods or with pressure bags increases the risk for this complication. If extravasation occurs, the needle should be removed, and the extremity diligently observed for signs of compartment syndrome. Experience to date suggests that the complications of the new mechanical insertion devices are similar to manual IO needle use. Other rare complications include infection and bone fracture. Osteomyelitis, cellulitis, and sepsis have been reported in conjunction with IO infusion. Risk for infection is increased when IO access is prolonged, and these devices are used in patients with bacteremia.

3. Ultrasound guidance for needle pericardiocentesis should be standard practice for which of the following indications? A. Cardiac tamponade with ongoing cardiopulmonary resuscitation B. Elective pericardiocentesis with idiopathic pericarditis C. Penetrating trauma of the right ventricle D. Second procedure after a failed blind, landmark-based attempt Preferred response: B

Rationale Except in life-threatening tamponade, ultrasound imaging should be used to improve success and reduce complications. Drainage of a pericardial effusion due to any cause is absolutely indicated when cardiac tamponade is present. Often drainage is recommended if the effusion is large, even in the absence of tamponade, for diagnosis and fluid removal. For small effusions, pericardiocentesis may be indicated for diagnosis alone. With purulent pericarditis, open surgical drainage may be more effective because of the difficulty in draining purulent exudate. Traumatic pericardial effusions secondary to penetrating trauma often require surgical drainage of the blood, because tamponade is common. Pneumopericardium secondary to pulmonary air leaks in mechanically ventilated patients is usually well tolerated hemodynamically but may require drainage, especially in small infants, because of the development of tamponade. There is no absolute contraindication to pericardiocentesis in an emergency situation. The presence of aortic dissection or myocardial rupture is considered a major contraindication. The presence of a bleeding diathesis or coagulopathy is another contraindication. Open drainage is preferred over closed drainage when the patient has traumatic tamponade and is in cardiac arrest. When the effusion is loculated in a location not easily reached via the subxiphoid approach, needle pericardiocentesis is contraindicated because the risk of complications increases, while the possibility of successful drainage is low. 4. Which of the following is most consistent with an exudative pleural effusion? A. Pleural to serum lactate dehydrogenase (LDH) ratio , 0.4 B. Pleural fluid LDH , the upper limit of normal serum LDH value C. Pleural to serum protein ratio . 0.5 D. Pleural fluid WBC count ,5% of peripheral WBC count Preferred response: C

Rationale Analysis of pleural fluid is separated into two basic diagnostic categories: exudates and transudates. Transudates arise from imbalances of hydrostatic or oncotic pressures, such as seen in congestive heart failure or nephrotic syndrome. Exudates can be caused by a variety of mechanisms, most commonly from pleural and lung inflammation or impaired lymphatic drainage. The criteria used to distinguish between the two have evolved, but are rooted in Light’s Criteria Rule (see below). The updated combination of two or more of these criteria increases the diagnostic sensitivity for the rule. More recent diagnostic rules, including the “two-test rule” and “three-test rule,” include pleural fluid cholesterol level greater than 45 mg/dL and do not require concomitant serum levels to be obtained.

CHAPTER 136  Board Review Questions

Additional pleural fluid studies should be sent to aid in diagnosis, especially fluid for culture, cell count with differential, and cytology. Low pleural glucose (,60 mg/dL) and pleural pH (,7.3) with very elevated nucleated cell counts (.50,000/mL) are highly suggestive of empyema. Elevated pleural triglyceride levels (.110 mg/dL) and lymphocyte predominance suggest chylothorax, while triglycerides ,50 mg/dL effectively rule it out. Elevated amylase suggests pancreatitis or esophageal rupture. Advances in polymerase chain reaction (PCR) technology allow for rapid and accurate diagnosis of viruses and bacteria in pleural fluid. Light’s criteria rule for diagnosing exudative effusions are as follows: If at least one of the following present, effusion defined as exudate: • Pleural to serum protein ratio .0.5 • Pleural to serum lactate dehydrogenase (LDH) ratio .0.6 • Pleural fluid LDH more than twice the upper limit of normal serum LDH value 5. Which of the following is suggestive of spontaneous bacterial peritonitis when evaluating fluid obtained from a paracentesis? A. Ascitic fluid glucose 22 mg/dL with serum glucose 110 mg/dL B. Multiple organisms in the culture C. Neutrophils 250/mm3 D. Total protein 2.2 g/dL Preferred response: C

Rationale Condition

Clinical Characteristics

Laboratory Findings

Spontaneous bacterial peritonitis

Cloudy or turbid Gram stain positive ,10%, cultures may be negative, single organism

Neutrophils .250/mm3 Total protein ,1 g/dL LDH and glucose similar to serum

6. Initial management of limb ischemia after arterial catheterization should include which of the following? A. Aspirin and local application of nitroglycerin paste B. Catheter removal and systemic heparin anticoagulation C. Local infusion of alteplase D. Thrombectomy within 4 hours Preferred response: B

Rationale Limb ischemia that develops after arterial catheterization first requires immediate catheter removal. If ischemia does not rapidly resolve and no contraindications to anticoagulation exist, heparinization should be considered. In larger vessels, thrombectomy or local infusion of thrombolytic agents such as alteplase may be considered in consultation with vascular surgeons and interventional radiologists.

e15

7. During an ultrasound-guided pericardiocentesis, saline microbubble contrast (saline solution in a syringe that has been agitated) injected through the introducer needle demonstrates the appearance of contrast in the left ventricle. The most appropriate next step is: A. Alerting a cardiovascular operative team for emergency open pericardiotomy B. Catheter placement, followed by high-resolution computed tomography coronary arteriography C. Insertion of the guidewire D. Removal of the needle Preferred response: D

Rationale Several techniques are helpful to determine if blood obtained during pericardiocentesis is of cardiac or pericardial origin. One technique involves injection of small amounts of saline microbubble contrast (saline solution in a syringe that has been agitated) through the introducer needle while imaging with echocardiography. If contrast bubbles are seen in the heart, then the tip of the needle is intracardiac and should be removed. If bubbles appear in the pericardial sac, then the needle is appropriately placed in the pericardium.

Chapter 15: Ultrasonography in the Pediatric Intensive Care Unit 1. Which of the following statements is true regarding ultrasound guidance by intensivists for vascular access in children? A. It is not useful for the placement of umbilical arterial or venous catheters. B. It should be performed using active (dynamic) ultrasound guidance, which is superior to physically marking the vessel location preprocedure (static guidance). C. It should be performed with a transverse visualization of the vessel, as this is demonstrably superior to needle insertion in the longitudinal plane. D. It sufficiently increases safety in multiple studies of femoral and subclavian central venous catheter insertion to be considered the standard of care. Preferred response: B

Rationale Placement of central venous catheters using active ultrasound guidance has been demonstrated to be superior to marking position prior to the procedure. Generalizable studies that demonstrate a significant safety benefit for subclavian vein central venous cannulation under ultrasound in children remain lacking. Advantages of ultrasound use for arterial catheter and umbilical vein catheter placement have been described. There have not been sufficient studies to determine whether transverse or longitudinal guidance approaches for vascular access are advantageous.

e16

S E C T I O N XV   Pediatric Critical Care: Board Review Questions

2. Which of the following statements is true regarding cardiac ultrasonography by intensivists in children? A. It demonstrates good concordance with studies performed and interpreted by expert echocardiographers. B. It is useful in cardiac arrest to determine the likelihood of failed return of spontaneous circulation (ROSC) if the heart is not moving. C. It is not affected by high intraabdominal pressure. D. When used for determining intravascular volume status, identical criteria for IVC measurement should be used for mechanically ventilated and spontaneously breathing patients. Preferred response: A

Rationale Cardiac ultrasonography by pediatric intensivists and emergency medicine physicians has demonstrated good concordance in terms of both interpretation and image acquisition with ICU ultrasound equipment. Though adult cardiac arrest studies have demonstrated cardiac standstill may predict failure to achieve ROSC, recovery of cardiac function in children on ECMO initiated during CPR has also been described. Increased intrathoracic pressure with mechanical ventilation affects the measurement of the inferior vena cava (IVC) for volume status assessment as well. Accordingly, heart position is also affected by elevated intrathoracic or intraabdominal pressures due to position of the diaphragm and lungs in these clinical scenarios. 3. What is true regarding pulmonary ultrasound in critically ill children? A. It demonstrates reduced visibility of lung parenchyma and pulmonary vascular structures as consolidation progresses. B. It is affected by changes in body composition, development, and size throughout the growth of a child. C. It is prone to artifactual findings, which should be ignored during interpretation of the image. D. It should be performed with a phased array transducer. Preferred response: B

Rationale The appearance of the lung changes throughout pediatric development as infant body water decreases and bones ossify. Early in life, intrathoracic structures may be more visible because of less impedance from lung and bony structures. Through late childhood into adolescence, ongoing growth affects available windows and the depth of visualization in patients. A phased array transducer can visualize areas within the thorax and the pleural line; however, this is not optimal for visualizing pleural sliding because the field of view with this type of sector transducer is narrow near the skin. Artifactual pulmonary ultrasound findings, such as B- and Z-lines, have been described as useful for assessing pediatric pulmonary pathology and should be incorporated into the assessment. Consolidation of lung parenchyma increases visualization of structures due to less interference from air as it is excluded from the tissue. B-lines are not apparent in pneumothorax due to separation of the visceral pleura from the chest wall.

Chapter 16: Patient- and Family-Centered Care in the Pediatric Intensive Care Unit 1. High quality and collaborative communication includes: A. Avoidance of conflict B. Decreased used of empathetic statements C. Explicit support of family decision making D. Guarantee of positive outcome E. Higher proportion of physician speech Preferred response: C

Rationale Explicit support of family decision-making, higher proportion of family speech, increased use of empathetic statements and expressing nonabandonment are all elements of high quality communication that is associated with increased patient survival, family and patient satisfaction, and is protective against physician and nursing burnout. Conflict is abundant in clinical practice. It may be unavoidable. Rather than side-stepping conflict, collaborative communication helps physicians identify conflict as an opportunity to develop a more complete understanding of the family’s differing perspectives. 2. Parental presence during invasive procedures and cardiopulmonary resuscitation has been shown to: A. Complicate the ability of clinicians to teach trainees procedural skills B. Increase the likelihood of adverse events C. Increase parental emotional lability D. Negatively impact physicians’ technical performance E. Reduce parental anxiety surrounding their child’s care Preferred response: E

Rationale Parental presence during invasive procedures and cardiopulmonary resuscitation is associated with improved satisfaction, better understanding, reduced anxiety, better coping, more emotional stability, and improved adjustment to a child’s death. Parental presence during procedures or resuscitation has not been associated with negative impacts on technical performance, ability to teach, adverse events, or clinical decision making. 3. In general, what is the primary cause of stress in parents related to the admission of their child to an ICU? A. They are concerned about the cost of care. B. They are afraid their child will die. C. They are stressed about being separated from their child. D. They are concerned their child will not be the same after leaving the hospital. Preferred response: C

Rationale The admission process can be frightening for the parents and child, especially when the admission is emergent or unplanned. Every effort should be extended to help the parents acclimate to the new environment; they should be treated with compassion and courtesy, and time should be taken to meet their needs. Parents report a loss of control, which can be unbearable when they are separated from their ill child. To support the child and the parents, caregivers should invite the parents to be part of the admission process and enable them to remain with their child to the extent possible.



CHAPTER 136 Board Review Questions

Chapter 17: Pediatric Critical Care Ethics 1. The use of substituted judgment is most appropriate for decision-making on behalf of which of the following patients? A. 18-month-old male with severe ARDS on ECLS B. 7-year-old female with an accidental ingestion C. 13-year-old male with 60% total body surface area burns who has been sedated and intubated since his accident D. 16-year-old female with cystic fibrosis end-stage lung disease who is ventilated and sedated Preferred response: D

Rationale The substituted judgment standard applies to situations in which the surrogate decision makers know what the patient would have wanted. Very young children are unable to communicate what they want. Even older children often have not have developed the ability to consider and verbalize their preferences about complex medical decisions prior to becoming incapacitated. Therefore, utilization of the best interest standard (as opposed to substituted judgement) as the basis of surrogate decision-making is most common in pediatrics. In situations where an older adolescent (commonly living with chronic medical conditions) has considered and conveyed their wishes to adults prior to the losing the ability to participate in decision-making, substituted judgment is appropriate to utilize in making surrogate medical decisions. In all situations where the wishes of the patient are unknown, the best-interest standard is the most appropriate decision-making tool. 2. Which of the following statements regarding adolescent decisionmaking is true? A. A 15-year-old patient can seek medical care for reproductive and mental health needs without parental consent. B. Adolescents are typically not developmentally ready to participate in discussions about their medical care. C. Adolescents are legally required to provide assent as part of the informed consent process. D. Pediatric patients designated as mature minors are still required to have adult caregiver consent for significant medical decisions. Preferred response: A

Rationale In pediatrics, parents or legal guardians must provide informed consent for non-adult children. If children are 7 years of age or older, their informed assent is desired but not legally required. However, there are exceptions. Adolescents who qualify as mature minors can provide consent for themselves. Similarly, all states allow adolescents to provide consent for specific healthcare needs like reproductive health and mental health services after a certain age (usually 14 or 15). Regardless of legal decision-making authority, clinicians should strongly consider including adolescents in medical decisions and should work with adolescents and their families to achieve agreement whenever possible. 3. During an influenza outbreak, the PICU is running out of ventilators. The MOST appropriate allocation strategy is: A. Adhere to your institution’s predetermined standardized triage criteria B. Perform a lottery C. Prioritize neonates over older children since they have a longer life expectancy D. Withdraw the ventilators from patients least likely to survive Preferred response: A

e17

Rationale Allocation decisions for scarce resources are best made in advance, before a period of surge. This prevents triage decisions from being made at individual bedsides and eliminates potential injustices. Every institution should have a disaster plan in place to deal with surges—natural or manmade. 4. A 3-year-old girl suffered a cardiac arrest following cardiac surgery. She has had minimal neurologic recovery. She has been in the CICU for 5 months with chronic critical illness. Her parents adamantly demand that everything medically possible be done for her despite ongoing communication with the medical team. Many members of the medical team are distressed at caring for this patient and advocate for withdrawal of life support. The most appropriate response from the medical team is: A. Allow distressed staff to refuse to care for the child B. Continue to tell the family daily that their child is suffering and that life support should be withdrawn C. Continue to support the family and the medical team, and engage the ethics consult service to apply a fair processbased approach to evaluate if continuing medical therapies is potentially inappropriate D. The medical team should tell the family that life support will be withdrawn against their wishes Preferred response: C

Rationale Clinicians cannot simply refuse to care for a child based on a claim of moral distress, or unilaterally withdraw life-support against a family’s wishes without a fair process. The medical team should continue to support the family and staff through these challenging times, working diligently to maintain open lines of communication and trust with the family. 5. Which of the following is a true statement regarding the doctrine of informed consent? A. Decisions must be voluntary and not subject to coercion, manipulation, or undue influence. B. It does not apply to decisions involving research. C. The patient needs to understand all the minutia of the treatment being discussed. D. The decision making should be shared with a competent medical provider. Preferred response: A

Rationale The doctrine of informed consent applies to both medical decisions and research. Informed consent must satisfy four requirements that apply when surrogates provide permission as well as when consent is obtained directly from patients. With disclosure, the clinician should supply the patient with sufficient information that a “reasonable person” would desire to be able to make an informed medical decision. With understanding, the clinician should assess the patient’s understanding of the proposed course of action, the risks and benefits of that course of action, and any available alternatives along with the risks and benefits associated with those actions. Understanding may be particularly impaired in the critical care setting where the high stakes and time pressures can impact the ability to achieve optimal understanding. With capacity, the patient must meet legal requirements for competency, be able to understand the medical decision, form a reasonable judgment based on the consequences of the decision, and be

e18

S E C T I O N XV   Pediatric Critical Care: Board Review Questions

able to communicate that decision to others. Legally, children under the age of 18 are not considered competent for medical decision making with the exception of emancipated and mature minors. Emancipated minors are considered competent based on characteristics that are defined by state law, but may include pregnancy, parenthood, or establishing financial independence. Mature minors represent another category that is defined by state law whereby a minor, usually above a certain age, can be judged competent to make certain medical decisions. Most states require a judge to make these determinations, and the judge may restrict the determination to the medical decisions at hand. With voluntariness, decisions must be voluntary and not subject to coercion, manipulation, or undue influence. Importantly, physicians should not withhold or deemphasize information in an effort to manipulate patients. 6. Which of the following is an important consideration that may justify seeking legal intervention to override a parent’s decision to refuse a medical therapy? A. Mediation and negotiation efforts are initiated and conducted by the state courts. B. Parental authority is absolute and cannot be challenged legally by medical professionals. C. The intervention refused by the parent is one that is not commonly performed. D. The parent’s decision places the child at a significant risk of serious harm. Preferred response: D

Rationale In most situations, parents are granted wide latitude in the decisions they make on behalf of their children, and the law has respected those decisions except when they place the child’s health, well-being, or life in jeopardy. Parental authority is not absolute, however, and when a parent or guardian fails to adequately guard the interests of a child, the decision may be challenged, and the state may intervene. A clinician’s authority to interfere with parental decision making is limited. Except in emergency situations where a child’s life is threatened imminently or a delay would result in significant suffering or risk to the child, the physician cannot do anything to a child without the permission of the child’s parent or guardian. Touching (physical examination, diagnostic testing, or administering a medication) without consent is generally considered battery under the law. The clinician’s options include either tolerating the parents’ decision (while continuing to try to convince them to act otherwise) or involving a state agency to assume medical decision-making authority on behalf of the child. Only the state can order a parent to comply with medical recommendations. This can take different forms, but most frequently either includes involvement of child protective services (under a claim of medical neglect) or a court order. Both of these options represent a serious challenge to parental authority, and parents will generally perceive them as disrespectful and adversarial. Such action interferes with family autonomy, can adversely affect the family’s future interactions with medical professionals, and can negatively impact the emotional well-being of the child. Neither should be undertaken without serious consideration. Before initiating the involvement of state agencies to limit parental authority and override parental refusal, the clinician must establish that (1) the recommended course of action is likely to benefit the child in an important way, (2) the treatment is of proved efficacy with

a reasonable likelihood of success, (3) the parent or surrogate’s decision to refuse intervention places the child at significant risk of serious harm in comparison to the recommendations of the healthcare team (applying the harm principle), and (4) all attempts at mediation and negotiation to find a mutually acceptable solution have been exhausted.

Chapter 18: Ethical Issues Around Death and Dying 1. A 10-year old patient with recurrent acute myelogenous leukemia (AML) is now 10 days post-stem cell transplant and is admitted to the Pediatric Intensive Care Unit in critical condition from septic shock. Despite maximal support, the patient continues to deteriorate from multiple organ failure, including acute respiratory distress syndrome (ARDS), ventricular dysfunction, hepatic failure and diffuse coagulopathy with bleeding from every orifice. The patient’s parents now demand a trial of extracorporeal membrane oxygenation (ECMO). The most appropriate response to this request is: A. Conduct a family meeting to discuss prognosis B. Continue with current therapy C. Consult hospital legal office D. Provide a trial of ECMO E. Write a unilateral DNR order Preferred response: A

Rationale The parents have a long established legal and ethical right to make decisions for their child, but they may not demand therapy that the physician deems inappropriate. However, before escalating the situation further, it is most appropriate to invest all necessary time and energy into listening to their concerns and fostering respectful communication by holding a family meeting as the next best step. 2. The parents and the attending physician have made the decision to withdraw life-sustaining treatment from the 10-yearold patient with recurrent AML, now one month post-stem cell transplant with progressive multiple organ failure. Which of the following options reflects current thinking in medical ethics: A. Neither withholding life-sustaining treatments nor withdrawing life-sustaining treatments is allowed without the assent of the 10-year old child in addition to the permission of the parents. B. There is no ethical distinction between withholding lifesustaining treatments and withdrawing life-sustaining treatments C. Withdrawing life-sustaining treatments is more ethical than withholding life-sustaining treatments D. Withholding life-sustaining treatments is more ethical than withdrawing life-sustaining treatments. Preferred response: B

Rationale There is no morally relevant or logically valid distinction between withholding and withdrawing life-sustaining treatments in mainstream bioethics or in the law in the United States. They are considered to be the same.

CHAPTER 136  Board Review Questions

e19

Chapter 19: Palliative Care in the Pediatric Intensive Care Unit

legal repercussions should not prevent physicians from treating patients appropriately. It is also generally accepted, and supported by a series of court cases, that any medical technology can be discontinued if it is no longer providing a benefit to the patient, no longer desired by the competent adult patient, or merely prolonging imminent death. For children, parents are assumed to be the legal decision-makers except in unusual circumstances. Typically, there is little controversy when parents and the medical team agree on goals of care, the withdrawal of technology that is no longer helpful, and the management of suffering. The Doctrine of Double Effect is often used as an ethical justification for providing medications to treat pain and suffering at the end of life, since there is also a possibility that they will hasten death. The components that make the unintended consequence justifiable include: 1. The act itself (treating suffering) must be inherently good 2. The agent intends the good effect (treating suffering) rather than the bad effect (hastening death) 3. The good effect must outweigh the bad effect (e.g., hastening death by many years for mild pain would be unacceptable) Many add that the bad effect must not be a means to the good effect—meaning death is not used as the means to end suffering. Practically, these arguments can shed light on what medications may or may not be acceptable to use at the end of life. Medications that treat pain or other symptoms should be used in reasonable doses based on the patient’s prior exposure and expected tolerance and may be escalated rapidly as needed for untreated symptoms. Medications that are unacceptable are those that would merely hasten the dying process without treating suffering (e.g., neuromuscular blockade, potassium chloride). In the vignette, it would be appropriate to give any of the sedative or opioid agents listed with an intent to treat pain or dyspnea, even if the respiratory drive might be suppressed or the intracranial pressure affected. Vecuronium, however, would be inappropriate as the ventilator is being withdrawn since neuromuscular blockade would hasten death without treating suffering (and in fact would potentially make it difficult to detect distress). Agents that are considered to provide deep sedation (e.g., ketamine or propofol) are typically considered only when other agents have proven ineffective or when the patient is known to be very tolerant to less sedating agents, but their use can still be justified.

1. 12-year-old boy with a history of glioblastoma is in the ICU with respiratory failure and mental status changes. An MRI of his brain is obtained and identifies extensive leptomeningeal spread of his tumor. The oncology team has met with his parents to let them know there are no further cancer-directed therapies to offer. The family and medical team decide that it is time to discontinue invasive interventions and focus on comfort, and begin to plan for a compassionate extubation. Which of the following medications would be inappropriate to give as the ventilator is discontinued? A. Hydromorphone B. Ketamine C. Lorazepam D. Propofol E. Vecuronium Preferred response: E

2. You are caring for an 8-year-old boy with end-stage metastatic osteosarcoma, with extensive bony and lung metastases. He was admitted to the PICU from home due to a pain crisis that could not be adequately controlled by his hospice team. He is currently on a hydromorphone PCA (patient-controlled anesthesia), with both a basal rate and a demand dose. You are escalating the dose aggressively, and he continues to complain of refractory pain, particularly in his right humerus and right scapula. Which of the following adjunctive pain treatments would be LEAST appropriate in this setting? A. Bisphosphonates B. Epidural analgesia C. Ketamine D. Lidocaine patch E. Steroids Preferred response: B

Rationale There is long-standing legal and moral precedent that pain and suffering at the end of life can and should be treated, and fears of

Rationale Steroids and bisphosphonates are important adjuvant therapies for bone pain that is refractory to opiates. Ketamine would be a

3. The decision is made to withdraw mechanical ventilation on a 10-year-old patient with recurrent AML, now one month post-stem cell transplant with progressive multiple organ failure and allow her to die. Which of the following options reflects current thinking in medical ethics on the goal of administering sedatives and analgesics in this context: A. To hasten the patient’s death B. To relieve the parent’s anxiety C. To relieve the team’s anxiety D. To treat patient discomfort Preferred response: D

Rationale The goal of administering these therapies is only to treat patient discomfort and not to hasten the dying process or treat concerns of family members or the team with medication to the patient. 4. The decision is made to withdraw mechanical ventilation on a 10-year-old patient with recurrent AML, now one month post-stem cell transplant with progressive multiple organ failure and allow her to die. Which of the following might be indicated in this context? A. Antisialagogue B. Caffeine C. Neuromuscular blocking agent D. Potassium Preferred response: A

Rationale Medications that have no comfort relieving properties, but will hasten the death of the patient, such as neuromuscular blocking agents, potassium, and calcium are contraindicated in this context. Similarly, caffeine would work at odds with sedatives and analgesics in a patient in this context. An antisialagogue might be indicated in this context to reduce the secretion burden in this patient being extubated from mechanical ventilation with the goal of allowing her to die.

e20

S E C T I O N XV   Pediatric Critical Care: Board Review Questions

reasonable addition to the hydromorphone PCA, given its distinct mechanism of action (NMDA-antagonist) and its potential to help alleviate any component of neuropathic pain. Lidocaine patches can be useful adjuncts, particularly when the pain is highly localized to one or two areas. Epidural analgesia can be an invaluable adjunctive therapy for refractory pain in end-stage cancer, as long as the site of pain is at the chest or below. This patient’s site of pain (shoulder and scapula) would be inappropriately high to place an epidural catheter. 3. You are caring for a 2-year-old boy, formerly a 26-week premature infant, with chronic respiratory failure due to bronchopulmonary dysplasia, dependent on mechanical ventilation via a tracheostomy tube. He also has global developmental delay and a seizure disorder. He is admitted to the PICU with severe pneumonia, sepsis, and multiorgan dysfunction syndrome. What is the best strategy to begin a discussion of limitation of interventions with the family? A. Ask the family to share their understanding of their son’s current medical condition, as well as their values, hopes, and goals with regard to their son and his care. B. Impress upon the family that the severity of the child’s illness and poor overall prognosis do not justify further aggressive life support given the child’s underlying developmental delay. C. Inform the family that the patient is not going to live through the night. D. Present a list of interventions and ask whether or not the family would like them performed. Preferred response: A

Rationale Discussions of this nature should always begin by allowing the family to articulate their understanding of the child’s current condition, as well as their hopes, values, and goals for their child overall. After the clinician has heard this information, recommendations for or against various interventions can be made as they pertain to the family’s stated goals. Although it is important to ensure that the family understands the severity of illness and poor prognosis, it is not appropriate to make a recommendation against continuing life support based only on the presence of developmental delay. Physicians are poor at predicting the exact time of death for patients, so statements of certainty around death should be avoided. Physicians should avoid presenting families with a “menu” of options for care; rather they should elicit the family’s goals and make recommendations for pursuing or limiting certain interventions based on their stated goals. Physicians must avoid phrases such as “do everything,” as they are nonspecific and imply that “doing” is always the best course of action. 4. Which of the following is true of methadone use for symptom management? A. It has a toxic metabolite that can accumulate and lead to seizures. B. It has a long and variable half-life, which can lead to accumulation over time. C. It is renally excreted and therefore should be avoided in patients with renal failure. D. It shortens the QT interval. Preferred response: B

Rationale The half-life of methadone varies from less than 10 hours to greater than 75 hours, depending on a variety of host factors. It can easily accumulate as it is reaching steady state and lead to oversedation and even obtundation. Therefore dosing and titration must be performed carefully. Morphine is renally excreted and should be avoided in renal failure. Meperidine has a toxic metabolite that can accumulate and lead to seizures. Fentanyl is the only opiate available as a transdermal patch. Methadone prolongs the QT interval, and a screening electrocardiogram should be considered prior to its initiation.

Chapter 20: Organ Donation Process and Management of the Organ Donor 1. You are caring for a 4-week-old infant who had a prolonged cardiac arrest after being placed face down in the crib. The child has sustained significant anoxic brain injury. On the third hospital day, the infant has a Glasgow Coma Score of 3 with fixed and dilated pupils. The infant is mechanically ventilated and receiving a dopamine infusion of 4 mg/kg/min. Which statement is most correct about ongoing medical care for this infant? A. Authorization for donation after circulatory death (DCD) should be obtained from the family during discussions and decisions about end-of-life care. B. Early referral for organ donation to the organ procurement organization (OPO) enhances recovery of organs for transplantation. C. Organs cannot be recovered from neonatal donors. D. The role of the pediatric intensivist ends after providing support for the family during end-of-life care and declaring death because of potential conflict of interest with organ donation. Preferred response: B

Rationale Early referral for organ donation to the organ procurement organization (OPO) is considered a best practice and enhances recovery of organs for transplantation. Donor management should be viewed as a continuum of care provided by the pediatric intensivist and the critical care team from the time of admission to recovery of organs for transplantation. Involvement of intensivists results in better donor management and recovery of more organs with better graft function following transplantation. The discussions and decision to authorize organ donation after cardiac death (DCD) should only occur after an independent decision to withdraw medical support has been decided upon by the parents or legal guardian. This firewall avoids the perceived ethical conflict that the patient is being allowed to die to recover organs. Authorization must occur from parents or legal guardians.

CHAPTER 136  Board Review Questions

2. A 10-month-old unrestrained child who is a victim of a motor vehicle crash is admitted to your PICU. The child has sustained significant intracranial injury with subdural blood noted on the initial computed axial tomography scan of the head. The neurosurgical team has indicated that no surgical intervention is required at this time. This child has a Glasgow Coma Score of 3. No sedation or analgesia has been administered for the past 8 hours. The child is endotracheally intubated and mechanically ventilated with fraction of inspired oxygen (Fio2), 0.8; peak inspiratory pressure, 30 cm H2O; positive end expiratory pressure, 10 cm H2O; ventilator rate, 22 breaths per minute. The chest radiographs show bilateral pulmonary contusions. Agonal respirations noted 24 hours ago have ceased. Blood pressure is being supported with an epinephrine infusion of 0.7 mg/kg/min. Urine output has remained acceptable since admission 2 days ago. The bedside nurse notifies you that the child has fixed and dilated pupils. Which of the following statements is most correct? A. Ancillary studies are not required to make the diagnosis of neurologic death in infants younger than one year of age. B. An observation period of 24 hours between neurologic examinations is recommended to determine neurologic death in this patient. C. The partial pressure of carbon dioxide (Paco2) of 59 mm Hg during the apnea exam is sufficient to confirm brain death. D. Two separate examinations and a single apnea test are required to determine neurologic death in infants and children. Preferred response: A

Rationale Determination of neurologic death is a clinical diagnosis that relies on the neurologic examination and an apnea test. Neurologic death can be determined in term infant 37 weeks estimated gestational age (EGA) to 30 days of age to 18 years of age. It can be more difficult to make a determination of neurologic death in the neonate and younger child; therefore, serial examinations are essential to ensure the clinical examination remains consistent throughout the observation and testing period. The recommended observation period for infants 37 weeks EGA to 30 days of age is 24 hours. A 12-hour observation period is recommended for children older than 30 days of age. Apnea testing must be performed with each examination, and the Paco2 must rise to .60 mm Hg and 20 mm Hg above the baseline Paco2 for the apnea test to be valid by current national guidelines. Ancillary studies are not required to make a determination of neurologic death in any patient of any age unless the clinical examination and apnea test cannot be completed, making choice A the correct response.

e21

3. You have been asked by the organ procurement organization to assist with management of a 4-year-old child. This patient has been declared dead by neurologic criteria, and the parents have authorized donation. This patient has required extensive fluid resuscitation for blood pressure support. The child continues to receive vasopressor support with an epinephrine infusion of 0.6 mg/kg/min and dopamine of 12 mg/kg/min. Mechanical ventilation is instituted as follows: fraction of inspired oxygen (Fio2), 0.65; peak inspiratory pressure, 28 cm H2O; positive end expiratory pressure, 12 cm H2O; ventilator rate, 22 breaths per minute. Laboratory evaluation reveals: hemoglobin, 7.6 g/dL; hematocrit, 22.3%; serum sodium, 162 mEq/L; potassium, 3.1 mEq/L; chloride, 119 mEq/L; blood urea nitrogen (BUN), 19 mg/dL; creatinine, 1.2 mg/dL. Which of the following statements is correct? A. Corticosteroids can stabilize lung function and reduce free water accumulation in a donor B. Desmopressin for treatment of diabetes insipidus can improve blood pressure and potentially reduce vasopressor requirements in a donor C. Intranasal vasopressin is the preferred treatment for diabetes insipidus in a donor D. Thyroid hormone prevents the anaerobic to aerobic cellular metabolic shift in a donor Preferred response: A

Rationale Hormonal replacement therapy (HRT) restores aerobic metabolism, replaces hormone derived from the hypothalamus and pituitary, augments blood volume, and minimizes the use of inotropic support while optimizing cardiac output. Common agents used for HRT include thyroid hormone, corticosteroids, and vasopressin or desmopressin for treatment of diabetes insipidus. Corticosteroids such as hydrocortisone are another pharmacologic agent routinely used by many centers for HRT to assist with hemodynamic support. Treatment of the donor with high doses of corticosteroids reduces inflammation associated with neurologic death and modulates immune function. The potential benefit of hydrocortisone and other steroids may lie in their ability to alter adrenergic receptors and regulate vascular tone by increasing sensitivity to catecholamines. Steroids have also been shown to stabilize pulmonary function, reduce lung water accumulation, and increase lung recovery from donors, making A the correct response. The effects of thyroid hormone on myocardial contractility can be immediate or delayed. Thyroid hormone is commonly used in the hemodynamically unstable donors. The acute inotropic properties of thyroid hormone may occur as a result of beta-adrenoreceptor sensitization. Additionally, thyroid hormone administration may play an important role in maintaining aerobic metabolism at the tissue level after neurologic death has occurred. Levothyroxine (Synthroid) and triiodothyronine (T3) are the two intravenous thyroid agents available for administration. Vasopressin is commonly used to treat diabetes insipidus (DI) in the donor. This agent is not as potent as desmopressin on a weight per weight basis. Use of vasopressin for treatment of DI can reduce the need for vasopressor support. Vasopressin and desmopressin should be administered by the intravenous route. Intranasal administration is not recommended because of erratic or no absorption in the brain-dead donor.

e22

S E C T I O N XV   Pediatric Critical Care: Board Review Questions

4. You are caring for a critically ill 15-month-old patient who was a victim of abusive head trauma admitted to your PICU 5 days ago. This child is comatose with a Glasgow Coma Score of 3, fixed and dilated pupils, and no response to noxious stimulation. The child is mechanically ventilated with the following settings: fraction of inspired oxygen (Fio2), 1.0; peak inspiratory pressure, 32 cm H2O; positive end expiratory pressure, 8 cm H2O. Blood pressure is being supported with an epinephrine infusion of 0.9 mg/kg/min, and 15 mg/kg/min of dopamine. The child has a markedly distended abdomen and has minimal urine output. A chest radiograph revealed posterior and lateral rib fractures and bilateral pleural effusions with a right upper lobe consolidation. Computed tomography scan of the head revealed bilateral subdural and subarachnoid hemorrhages, diffuse cerebral edema, and effacement of the lateral ventricles. Serum laboratory studies revealed white blood cell count, 10.6 3 103/µL; hemoglobin 7.8 g/dL; hematocrit, 24.2%; sodium, 157 mEq/L; potassium, 4.1 mEq/L; chloride, 116 mEq/L; bicarbonate, 16 mEq/L, blood urea nitrogen, 24 mg/dL; creatinine, 1.92 mg/dL; glucose, 136 mmol/L; lactate, 4.3 mg/dL; alanine aminotransferase, 93 U/L; aspartate aminotransferase,126 U/L; albumin, 2.6 g/dL; protein, 4.1 g/dL; prothrombin time (PT), 15 seconds; international normalized ration (INR), 1.6; activated partial thromboplastin time (aPTT), 62 seconds. Arterial blood gas revealed pH, 7.392; partial pressure of carbon dioxide (Paco2), 30 mm Hg; partial pressure of oxygen (Pao2), 83 mm Hg; bicarbonate, 18 mEq/L; base excess, 25 mmol/L. Which of the following statements is correct? A. Early referral of the organ procurement organization after neurologic death has been declared can enhance chances for organ recovery. B. Maintenance of euvolemia and correction of metabolic derangements will have little effect on graft function and viability of organs for transplantation. C. Notifying the medical examiner or coroner’s office prior to death may expedite and facilitate organ donation. D. Organ donation is not possible because of multisystem organ dysfunction. Preferred response: C

Rationale Involvement of the pediatric intensivist and critical care team in the management of critically ill and injured children, especially in pediatric donation, where there is a limited and decreasing number of donors improves the quality and number of organs recovered. Preconceptions about eligibility for donation by the critical care team may not be current or accurate. Donor organ suitability for transplantation is best assessed by the organ procurement organization (OPO). Thresholds for acceptable organ dysfunction can vary according to time of evaluation, transplant program comfort levels, and recipient urgency. In many instances management of the potential organ donor can improve organ function and increase chances of successful organ recovery. Serial echocardiograms may demonstrate donor response to effective medical therapy enabling cardiac recovery for transplantation; a positive blood culture or bacterial meningitis may not preclude organ donation if antibiotic therapy has been administered. Organs

from HIV positive donors can be transplanted into HIV-positive recipients, and organs from hepatitis positive donors are now being transplanted with good success. Successful recovery of organs and the prosecution of the perpetrator may still occur in most cases of child homicide with close cooperation between forensic investigators, treating physicians, the transplant team, and OPO. Early involvement of the medical examiner or coroner prior to determination of death and protocols to facilitate organ recovery in homicide cases may reduce denials for organ donation, making answer C the correct choice. Efforts to reduce the number of medical examiner denials for donation are supported in the position statement by the National Association of Medical Examiners. Early involvement and timely notification of the OPO prior to determination of death allows a greater amount of time for collaboration to coordinate the donation process. Notification of the OPO after determination of death is considered a late referral and may not allow adequate time for the OPO and critical care team to fully discuss donation opportunities. The OPO can also work closely with the medical examiner or coroner to facilitate donation in cases of homicide. Ensuring OPO engagement early in the course of caring for a critically ill patient allows the intensive care team to better understand the entire donation process and eliminate confusion that may disrupt end-of-life care and the process of donation. Early referral improves authorization rates and can assist families with understanding and coping with end-of-life care issues. 5. Which of the following substances is markedly elevated immediately following brain death: A. Antidiuretic hormone B. Catecholamines C. Cortisol D. Insulin E. Thyroid hormone Preferred response: B

Rationale Neurologic death resulting from cerebral ischemia increases circulating cytokines, reduces cortisol production, and precipitates massive catecholamine release. Hemodynamic deterioration associated with neurologic death is initiated by a massive release of catecholamines, commonly referred to as sympathetic, catecholamine, or autonomic storm. The sympathetic storm results in intense but transient hypertension. The tremendous physiologic derangements associated with neuroendocrine dysfunction require specific interventions to restore normal physiology. Agents such as thyroid hormone, corticosteroids, vasopressin, and insulin are commonly employed during donor management. Hormonal replacement therapy can reduce circulatory instability associated with thyroid and cortisol depletion, especially in situations where significant inotropic support is required. 6. Which of the following metabolic derangements is common following neurologic death? A. Hypercalcemia B. Hyperglycemia C. Hypermagnesemia D. Hypochloremia E. Hyponatremia Preferred response: B

CHAPTER 136  Board Review Questions

Rationale The critical care team should actively manage the potential donor and correct existing physiologic and metabolic derangements that follow neurologic death to preserve the option of organ donation for the family. Metabolic derangements such as iatrogenic hypernatremia from hyperosmolar therapy and hyperglycemia associated with catecholamine release and reduced cerebral metabolism should be corrected. Volume loss from osmotic diuresis associated with hyperglycemia and diabetes insipidus (DI) following neurologic death must be anticipated and addressed to prevent cardiovascular collapse. Without substrate consumption by the brain, glucose needs are reduced, and the patient is prone to hyperglycemia. As neurologic death occurs, cerebral metabolism is further decreased and co2 production falls resulting in a reduction in Paco2. Hypothermia should be anticipated as a result of hypothalamic failure and loss of thermoregulation. Additionally, impaired adrenergic stimulation results in loss of vascular tone with systemic vasodilation and amplified heat losses. Hypocalcemia occurs commonly secondary to large volume replacement with colloids such as albumin, massive blood transfusions that result in large amounts of citrate reducing free calcium concentrations, and sepsis. Calcium is necessary for myocardial contraction and hypocalcemia can depress cardiac output, affect SVR, and organ perfusion. The use of calcium supplementation should be guided by ionized calcium levels. 7. Evaluation of a 3-year-old child for brain death is being considered. To determine brain death in this child, which of the following is most accurate? A. A longer period of observation in this young child is required because of legal issues. B. An ancillary study is required to make the diagnosis of brain death in children. C. Ancillary studies such as electroencephalogram, radionuclide cerebral blood flow study, and computed tomography angiography are used in children. D. A thorough neurologic examination and apnea test are required to make the diagnosis of brain death. Preferred response: D

Rationale In a child older than 1 year, brain death is a clinical diagnosis and does not require an ancillary study unless the examination and apnea test cannot be completed. A thorough neurologic examination and apnea test that is repeated after a specified observation period is required to determine brain death based on clinical criteria. A longer observation period is currently recommended in infants younger than 30 days and for children younger than 1 year. For children older than 1 year, an observation period of 12 hours is recommended. Ancillary studies such as an electroencephalogram and cerebral blood flow study can be used to assist with the diagnosis of brain death. Computed tomography angiography has not been validated in children and cannot be relied on as a diagnostic ancillary test to determine brain death in children. The apnea test must achieve a Pco2 of 60 mm Hg to be consistent with brain death.

e23

8. Which statement about neurologic death in infants and children is true? A. Ancillary studies are required to make the diagnosis of neurologic death in infants younger than 1 year of age. B. A Paco2 of 59 mm Hg in a child with acute lung injury who desaturates 3 minutes into the apnea exam is consistent with brain death. C. Neurologic death can be determined in term newborns at 37 weeks’ estimated gestational age. D. The revised pediatric brain death guidelines require two separate examinations and a single apnea test to determine neurologic death in infants and children. Preferred response: C

Rationale Determination of neurologic death can occur in a term infant at 37 weeks’ estimated gestational age (EGA) to 30 days of age. It can be more difficult to make a determination of neurologic death in the neonate; therefore serial examinations are essential to ensure that the clinical examination remains consistent throughout the observation and testing period. The recommended observation period for infants at 37 weeks’ EGA to 30 days of age is 24 hours. Apnea testing must be performed with each examination, and the Paco2 must rise to .60 mm Hg and 20 mm Hg above the baseline Paco2 to document apnea. Two neurologic examinations and apnea tests separated by an observation period are required to establish the diagnosis of neurologic death in the United States. The duration of observation between examinations is based on age. Ancillary studies are not mandatory to make a determination of neurologic death. The physician caring for the child will determine the need for ancillary studies based on history and the ability to complete the clinical examination and apnea testing. If clinical examination and apnea testing cannot be safely completed, an ancillary study should be used to assist with determination of death.

Chapter 21: Long-Term Outcomes following Critical Illness in Children 1. How does health-related quality of life (HRQL) differ from quality of life? A. Consideration of entirety of past medical history for HRQL B. Dimension of personal judgment over one’s health and disease C. Inclusion of functional status during assessment of HRQL D. Utilization of prospective as opposed to retrospective evaluation Preferred response: B

Rationale Quality of life is defined as an individual’s perception of his or her position in life in relation to the individual’s goals, expectations, standards, and concerns. HRQL is defined as quality of life in which a dimension of personal judgment over one’s health and disease is added, and encompasses the impact of health status on physical, mental, emotional, and social functioning.

e24

S E C T I O N XV



Pediatric Critical Care: Board Review Questions

2. A group of investigators is designing an interventional trial and planning to utilize health-related quality of life (HRQL) as the primary endpoint. Which of the following protocol elements would optimize assessment for this outcome measure? A. Ascertain baseline medical complexity for chronic comorbid conditions B. Conduct paired baseline and follow-up HRQL surveys C. Control for duration of intensive care unit stay D. Undertake illness severity measures (e.g., PRISM, PIM) of all patient participants Preferred response: B

Rationale Conducting paired HRQL assessments controls for each subject’s baseline status and permits change from baseline (paired) analyses. Semi-quantification of baseline chronic comorbid conditions represents an alternative but less specific approach. 3. Which of the following represents a risk factor for prolonged deterioration of functional status from baseline following pediatric critical illness? A. Duration of stay .28 days B. Elective PICU admission C. Nononcologic diagnoses D. Younger age at PICU admission Preferred response: A

Rationale Longer duration of PICU stay, oncologic diagnoses, older age, and emergent PICU admission are all risk factors for prolonged deterioration of functional status assessed by Pediatric Overall Performance Category. 4. Which of the following adverse events was most commonly documented among a cohort of children surviving critical illness at a median follow-up time of 5 months after PICU discharge? A. Fatigue disorder B. Post-traumatic stress disorder C. Psychiatric disorders D. Sleep disorder Preferred response: D

Rationale One study found that at a median of 5 months after discharge, 20% of PICU survivors were at risk for psychiatric disorders, 34% were at risk for PTSD, 38% were at risk for fatigue disorder, and 80% were at risk for a sleep disorder.

Chapter 22: Burnout and Resiliency 1. Which of the following is a common characteristic of compassion fatigue? A. Alcohol use B. Low energy in the workplace C. Reduced capacity for empathy D. Unrelieved stress and tension Preferred response: C

Rationale Compassion fatigue is broadly defined as reduced capacity and interest in being empathetic for those who are suffering. Although often used interchangeably, burnout consists of three dimensions including depersonalization, emotional exhaustion, and diminished feelings of personal accomplishment. While compassion fatigue may result in burnout, they are not synonymous. Low energy in the workplace, unrelieved stress and tension, and alcohol use may all contribute to or be signs or symptoms of burnout; none is specific to define compassion fatigue. 2. Which of the following factors has been found to be significant in the development of burnout among pediatric critical care physicians? A. Female gender B. Older age of a practitioner C. Presence of a palliative care consult team D. Years in pediatric critical care practice Preferred response: A

Rationale A recent national cross-sectional online survey exploring burnout and psychological distress among over 250 pediatric critical care physicians in the United States found that the risk of any burnout was about two times more in women physicians (odds ratio, 1.97; 95% CI, 1.2–3.4). Association between other personal or practice characteristics and burnout was not evident in the study, while regular physical exercise appeared to be protective. 3. Which of the following measures is considered a successful organizational strategy for mitigating burnout in critical care providers? A. Administering annual staff satisfaction surveys B. Increasing helpful pop-up alerts within the EMR C. Limiting family visitation hours D. Using ethics and palliative care consultations in the ICU Preferred response: D

Rationale The Critical Care Societies Collaborative’s Call to Action to address burnout identified that ethics and palliative care consultations is a successful strategy to mitigate burnout by divesting some of the burden of end-of-life discussions to clinical experts. Additionally, providing healthy food options for clinicians, providing an on-site gymnasium, and offering stress-reduction courses are all considered to be useful for mitigating burnout in critical care providers. 4. What factor most specifically impacts the development of burnout in pediatric critical care providers? A. Dealing with dying children B. ICU shift length C. Open visitation in the pediatric ICU D. Pediatric resident and fellow rotation schedules Preferred response: A

Rationale Dealing with the death of a critically ill pediatric patient has been demonstrated to be a specific factor impacting the development of burnout in pediatric critical care providers compared to general ICU related factors.



CHAPTER 136 Board Review Questions

Chapter 23: Structure and Function of the Heart 1. When examining a cardiac pathology specimen, atrial anatomy is defined by: A. The venous return to the heart (typically the inferior vena cava or IVC, superior vena cava or SVC, and coronary sinus to the right atrium and pulmonary veins to the left atrium), the sinus node as a right atrial structure, and the atrial relationship to the atrioventricular (tricuspid or mitral) valves. B. The constant defining features of a right versus left atrium are the atrial appendage and its extent of pectinate muscles and the venous return to the heart (IVC, SVC, and coronary sinus to the right atrium and pulmonary veins to the left atrium). C. The constant defining features of a right versus left atrium are the atrial appendage and its extent of pectinate muscles and typically the venous return to the heart (IVC, SVC, and coronary sinus to the right atrium and pulmonary veins to the left atrium). D. Pulmonary venous return, a broad based atrial appendage, and sinus node as constant features of a right atrium. Preferred response: C

Rationale Ventricular morphology is defined by the atrioventricular (AV) valve it contains; however, the atria are not always related to the same sided AV valve (i.e., AV discordance), making answer A incorrect. Systemic venous return is typically to the right atrium and pulmonary venous return to the left atrium, but not always, making answer B incorrect. Answer D is also incorrect, since pulmonary venous return is typically a feature of the left atrium. 2. Which statement is true regarding the transition from fetal to postnatal circulation? A. Compared to an infant at 6 weeks of age, the infant at birth relies mostly on stroke volume in order to have adequate cardiac output. B. The ductus arteriosus is kept open in fetal circulation by a balance between prostaglandin E2 (PGE2) and endothelin-1 (ET-1). After birth, the vasoconstrictive effects of PGE2 become more dominant and the ductus constricts, leading to closure of the ductus within 24 hours to 3 weeks C. There is a significant decrease in total body oxygen consumption and cardiac output after birth. Over time, with increased distensibility of the ventricular myocardium, the infant relies less on heart rate to augment cardiac output. D. With cord clamping, the baby experiences a rise in systemic vascular resistance (SVR) and a reduction in pulmonary vascular resistance (PVR). Preferred response: D

Rationale During transition to postnatal circulation, there is a rise in SVR and a reduction in PVR, and there is a significant increase in total body oxygen consumption and cardiac output, which initially is heavily reliant on heart rate to increase output until ventricular distensibility improves. Cardiac output in the fetus is determined mainly by heart rate because of a limited capacity to increase stroke volume that results mainly from decreased diastolic distensibility. Consequently, fetal bradycardia is detrimental to blood flow and

e25

oxygen delivery. In addition, because at birth approximately 80% of the infant’s hemoglobin is in the form of fetal hemoglobin, the reduced ability of this hemoglobin to unload oxygen at the tissue level compels the infant to have a higher cardiac output than the infant will have 4 to 6 weeks later. Therefore the neonate has limited cardiac output reserve, and the heart has near-maximal contractility. These features make the neonate unusually susceptible to diseases that impair cardiac function. 3. When assessing a child with cardiomyopathy who has left ventricular dilation and moderate mitral regurgitation, the preferred method to evaluate and monitor systolic function via echocardiography is: A. Change in ventricular pressure during isovolemic contraction before the aortic valve opens (dP/dT), taken from a continuous wave Doppler of the mitral regurgitation jet. B. M-mode derived TAPSE (tricuspid annular plane systolic excursion). C. M-mode generated ejection fraction. D. No reliable echocardiographic method is available, and this patient should undergo cardiac MRI for functional assessments. Preferred response: A

Rationale TAPSE (tricuspid annular plane systolic excursion) is used to assess right ventricular systolic function (answer B is incorrect). Although an M-mode generated ejection fraction is a popular and common method used to assess systolic function in children, it is less accurate in the context of mitral regurgitation and left ventricular (LV) dilation (answer C is incorrect). Although cardiac MRI is useful for obtaining accurate chamber volumes and measures of systolic function, echocardiography is more practical and remains an accurate method for noninvasive regular surveillance of ventricular systolic function (answer D is incorrect). In the context of LV dilation and mitral valve regurgitation (MR), dP/dT (rate of pressure change over time during isovolumic contraction) is an accurate method of monitoring ongoing changes in contractility and is relatively unaffected by preload changes. However, the clinician must keep in mind that dP/dT is affected by changes in afterload. Therefore, in the context of heart failure therapy with afterload reducing agents, changes in dP/dT measurements of systolic function will need to be considered in this clinical setting. 4. Which statement is true in the context of reduced left ventricular contractility? A. Due to the steep slope of the ventricular elastance line, small changes in afterload result in large increases in stroke volume and decreases in end diastolic pressure and volume. B. In order to eject a normal stroke volume, even in the face of a normal afterload, the ventricle compensates by increasing end diastolic volume. C. In order to maintain stroke volume and cardiac output, afterload is increased. D. In order to keep ejecting the same stroke volume, preload reserve is maintained during periods of increased afterload. Preferred response: B

Rationale See Figure 136.1. With decreased LV contractility, the ventricle compensates by increasing end diastolic volume (EDV) in order to maintain a normal stroke volume (answer B is correct). A higher afterload causes even further increases in EDV and EDP,

e26

S E C T I O N XV   Pediatric Critical Care: Board Review Questions

which eventually leads to pulmonary edema (answer C is incorrect). In this setting, the normal preload reserve has already been used to maintain stroke volume (answer D is incorrect). Afterload reduction in this context is useful because a small decrease in afterload results in a large increase in stroke volume and large decrease in end-diastolic volume (EDV) and end-diastolic pressure (EDP). There is a relatively flat slope of the maximal ventricular elastance line, NOT a steep slope. Answer A is incorrect.

Pressure

I 3

2 1

V0

A

D

1

2

3

Volume

7. The true statement concerning endothelins is: A. Bosentan is an endothelin agonist. B. Endothelins act on ET-A receptor and cause vasodilation. C. Endothelins act on ET-B receptor and cause vasoconstriction. D. Endothelins cause either vasoconstriction or vasodilation. Preferred response: D

Pressure

I 3

2 1

D EDV

V0

B

1

2

3

4

Pressure

Volume

2

3

1

EDV EDV

V0

C

1

2

3 I

II

Rationale In all except young infants, the preferential source of energy for myocardial function comes from the b-oxidation of long-chain fatty acids. After fatty acids enter the cell, they are activated to fatty acid (or acyl) coenzyme A (CoA) compounds by palmitoyl-CoA synthetase, then linked by carnitine palmitoyl transferase I to carnitine to form acylcarnitines, thus releasing CoA. The acylcarnitines cross the mitochondrial membrane, and at the inner surface of the membrane another enzyme, carnitine palmitoyl transferase II, transfers the fatty acids back to CoA. The fatty acids can now undergo b-oxidation with the production of ATP. These enzymes also help transport acylcarnitine esters of CoA out of the mitochondria. These esters are toxic in high concentrations. Fetuses and neonates have decreased activity of carnitine palmitoyl transferase and palmitoyl-CoA synthetase, so glucose, lactate, and short-chain fatty acids are the preferred myocardial energy substrates at this age.

III

Volume

• Fig. 136.1  ​ 5. The main difference in the fetal/neonatal myocardium compared with mature adult myocardium is: A. Complete coupling of b adrenoreceptors to G-proteins B. More b adrenoreceptors C. More contractile components D. Sparse, disorganized, and immature T-tubule and sarcoplasmic reticular system Preferred response: D

Rationale The fetal/neonatal myocardium contains less b adrenoreceptors and less contractile components when compared to the adult myocardium. There is incomplete uncoupling of b adrenoreceptors to G-proteins in the fetal/neonatal myocardium as well. The T-tubule and sarcoplasmic reticular system of the fetal/neonatal myocardium are sparse and disorganized. 6. Except in young infants, the preferential source of energy for myocardial function comes from: A. Glucose B. Glycogen C. Ketones D. Long-chain fatty acids Preferred response: C

Rationale The vascular endothelium elaborates the endothelins (ET-1, ET-2, ET-3), a family of compounds that are vasoactive, structurally related peptides. ET-1 is the most potent vasoconstrictor known. It also promotes mitogenesis and stimulates the renin-angiotensin-aldosterone system and the release of vasopressin and atrial natriuretic peptide. These peptides act on one of two receptor subtypes: ET(A) and ET(B). ET(A) is located mainly on vascular smooth muscle cells and is responsible for mediating vasoconstriction and cell proliferation. ET(B) is present predominantly on endothelial cells and mediates vasorelaxation, as well as ET-1 clearance. Endothelins cause local vasoconstriction or vasodilation, depending on dose and location in the circulation. Individual endothelins occur in low levels in the plasma, generally below their vasoactive thresholds. This finding suggests they are primarily effective at the local site of release. Even at these levels, however, they may potentiate the effects of other vasoconstrictors such as norepinephrine and serotonin. Endothelin antagonists, such as bosentan, now are being used specifically in the setting of pulmonary arterial hypertension. 8. Regarding the regulation of vasomotor tone: A. Adenosine causes vasoconstriction. B. Hypokalemia causes vasodilation. C. Hyperkalemia causes vasodilation. D. Neuropeptide Y causes vasodilation. Preferred response: C

Rationale Local metabolic regulation of vasomotor tone provides an ideal homeostatic mechanism whereby metabolic demand can directly influence perfusion. Adenosine, which accumulates locally when tissue metabolism is high and tissue oxygenation is marginal, causes pronounced vasodilation in the coronary, striated muscle, splanchnic, and cerebral circulations. Potassium is released from muscle in response to increased work, ischemia, and hypoxia. Hypokalemia causes vasoconstriction. Hyperkalemia, within the physiological range, causes vasodilation by stimulating KIr channels.



CHAPTER 136 Board Review Questions

A

• Fig. 136.2



Volume ​

Rationale The figure depicts the pressure-volume relationship for a single cardiac cycle. During diastolic filling, volume increases and diastolic pressure rises slightly because of the increase in passive tension. At the end of diastole, isovolumic systole occurs, and ventricular pressure rises with no change in volume. When ventricular pressure exceeds aortic pressure, the aortic valve opens, blood is ejected, and ventricular volume decreases. Ejection ends, and pressure falls to diastolic levels as isovolumic relaxation occurs. The pressure and volume reached at the end of systole are those that would have been attained by the isolated ventricle at that same end-systolic volume. In other words, at a given volume, no higher pressure can be generated. The decrease in volume during ejection is the stroke volume which, divided by the end-diastolic volume, gives the ejection fraction; normally, ejection fraction is greater than 65%. 10. In what situation would the left ventricular (LV) pressurevolume relationship in a previously healthy 12-year-old be expected to change from loop abcd to loop abcd in Figure 136.3, below? A. Blood transfusion B. Hemorrhage C. Myocardial contusion D. Use of low-dose epinephrine Preferred response: C

cʹ c

100 b





a 50

100 LV volume (mL)

• Fig. 136.3



LV pressure (mm Hg)

dʹ d



Rationale Loop abcd represents the pressure-volume relationship in an intact heart. Contraction begins at the end-diastolic pressure and volume of point a. Line ab represents isovolumetric contraction. At point b, the aortic valve opens, since left ventricular (LV) pressure exceeds that in the aorta. Ejection begins at point b and ends at point c. At this point, the aortic pressure is equal to the maximal force produced by the ventricular wall for that specific endsystolic fiber length. Isovolumetric relaxation starts at point c and the aortic valve closes. The ventricular pressure drops (line cd). The mitral valve will open when LV pressure falls below left atrial pressure and blood will flow into the LV. The difference between lines ab and cd is the stroke volume. Point a represents preload and point b represents afterload. Loop abcd reflects a decrease in contractility, which could be the result of myocardial contusion. There is a decrease in stroke volume despite a larger end-diastolic volume at a similar level of LV pressure. Afterload reduction and increase in inotropy (low-dose epinephrine or milrinone) would shift the PV curve to the left and upward. A blood transfusion increases preload, resulting in an increase in stroke volume (see Figure 136.4 below, loop 2). The contractility has not changed (both end-systolic points are on the same line). 200

2 1 100

50

100 LV volume (mL)

• Fig. 136.4



Pressure

9. What does the segment “A” denote in the Pressure-Volume loop depicted in Figure 136.2 below? A. Ejection fraction B. End diastolic pressure C. Ejection time D. Stroke volume Preferred response: D

200

LV pressure (mm Hg)

In all organs, sensory and efferent nerve endings contain nonadrenergic, noncholinergic (NANC) peptides, for example, neuropeptide Y, VIP, calcitonin gene-related peptide (CGRP), and substance P. Neuropeptide Y is colocalized and released with norepinephrine, and VIP is colocalized with acetylcholine and released upon stimulation of vagal nerve endings. Most of these peptides except neuropeptide Y are vasodilatory, and they help modulate blood pressure and regional flows.

e27



11. At this same level of contractility, what strategies would restore the stroke volume for the patient with myocardial contusion and decreased contractility to his baseline? A. Increase in preload and inotropic support B. Increase in preload and vasoconstrictor therapy C. Increase in preload and vasodilator therapy D. Use of diuretics and inotropic support Preferred response: C

e28

S E C T I O N XV



Pediatric Critical Care: Board Review Questions

Rationale In patients with decreased myocardial contractility, stroke volume can be restored with either an increase in preload (loop 4) or a decrease in afterload (loop 3) (see Figure 136.5, below).

LV pressure (mm Hg)

200

2

1

4 3

100

50

100 LV volume (mL)

• Fig. 136.5





12. Loop 1 in Figure 136.6, below, represents the pressure-volume loop in an intact heart. The correct statement regarding the other loops (2 and 3) is: A. Loop 2 is reflective of a decrease in preload, as would occur with diuretics. B. Loop 3 is reflective of a decrease in preload, as would occur with diuretics. C. Loop 2 demonstrates the effect of increasing afterload. D. Loop 2 demonstrates the effect of decreasing afterload. Preferred response: C

LV pressure (mm Hg)

2

3 1

LV volume (mL)

• Fig. 136.6





Rationale The figure above represents the response of a normal heart to an increase in afterload. Loop 1 represents the normal physiologic pressure-volume loop. Stroke volume is diminished in loop 2 due to an increase in afterload. Loop 3 represents a compensatory response to the increase in afterload. Contraction begins at a higher end-diastolic volume. The heart ejects the same stroke volume at a higher afterload. Even though the stroke volumes for loops 1 and 3 are the same, the ejection fraction is lower for loop 3, since the end-diastolic volume is increased.

Chapter 24: Regional and Peripheral Circulation 1. The difference between autoregulated and maximal coronary flow is termed coronary flow reserve. What does the coronary flow reserve indicate? A. Maximal coronary artery vasoconstriction to meet increased demands for oxygen B. Maximal cardiac oxygen consumption C. Maximal ventricular contraction at any given coronary flow D. The amount of myocardial blood flow that can increase at any given pressure in order to meet increased oxygen demands Preferred response: D

Rationale At any given pressure, the difference between autoregulated and maximal flows is termed coronary flow reserve. Coronary flow reserve indicates how much extra flow the myocardium can get at a given pressure to meet increased demands for oxygen; if reserve is much reduced, then flow cannot increase sufficiently to meet demands and myocardial ischemia will occur. Coronary flow reserve is normally lower in the subendocardium than in the subepicardium, and decreases in coronary flow reserve are always more profound in the subendocardium than in the subepicardium. If autoregulated flow is normal but maximal flow is decreased, then coronary flow reserve will be reduced. Coronary flow reserve also can be reduced if maximal flows are normal but autoregulated flows increase. 2. Nitric oxide (NO) is a labile humoral factor produced by nitric oxide synthase from l-arginine in the vascular endothelial cell. Which of the following is true of nitric oxide? A. Decreases significantly in the pulmonary vasculature immediately after birth B. Is important in basal vascular tone, but less so for dynamic changes in vascular tone C. Is increased in response to increases in shear/flow across endothelial cells D. Produces vascular smooth muscle cell contraction by increasing the concentration of cGMP via the activation of soluble guanylate cyclase Preferred response: C

Rationale NO is a labile humoral factor produced by NO synthase from L-arginine in the vascular endothelial cell. NO diffuses into the smooth muscle cell and produces vascular relaxation by increasing concentrations of cGMP via the activation of soluble guanylate cyclase. NO is released in response to a variety of factors, including shear stress (flow) and the binding of certain endothelium-dependent vasodilators (such as acetylcholine, adenosine triphosphate [ATP], and bradykinin) to receptors on the endothelial cell. Basal NO release is an important mediator of both resting pulmonary and systemic vascular tone in the fetus, newborn, and adult, as well as a mediator of the fall in pulmonary vascular resistance normally occurring at the time of birth. Dynamic changes in NO release are fundamental to the regulation of all vascular beds.

CHAPTER 136  Board Review Questions

e29

3. Of the following agents, which has the most potent vasoconstricting properties? A. Angiotensin II B. Aldosterone C. Endothelin-1 D. Thromboxane A2 Preferred response: C

pulmonary artery pressure. These units are perfused in proportion to the driving pressure across them, which is approximately pulmonary artery pressure less vertical height (or critical closing pressure, whichever is higher). Zone III vessels lie at a vertical height less than outflow pressure. Driving pressure across these units is independent of height because inflow and outflow pressures are comparably influenced by gravity.

Rationale Endothelin-1 (ET-1) is produced by vascular endothelial cells. It has complex vasoactive properties, the most striking of which is its sustained hypertensive action. ET-1 is the most potent vasoconstricting agent discovered, with a potency 10 times that of angiotensin II. Humoral regulators of vascular tone include angiotensin, arginine vasopressin, bradykinin, histamine, and serotonin. Of less importance are thyroxine, aldosterone, and antinatriuretic peptide. Angiotensin plays a special role in the homeostasis of blood pressure and is produced in persons with hemorrhagic or hypovolemic shock. It causes generalized vasoconstriction in both systemic and pulmonary circulations, but locally it stimulates the release of vasodilating prostaglandins in lung and kidney. Bradykinin is a potent pulmonary and systemic vasodilator released locally by the action of the proteolytic enzymes on kallikrein after tissue injury. Breakdown of phospholipids within vascular endothelial cells results in production of important byproducts of arachidonic acid, including prostacyclin (PGI2) and thromboxane (TXA2). PGI2 activates adenylate cyclase, resulting in increased cyclic adenosine monophosphate (cAMP) production and subsequent vasodilation, whereas TXA2 results in vasoconstriction via phospholipase C signaling.

5. In which of the following vascular beds do neural stimuli exert little effect on basal blood flow under physiologic conditions? A. Cerebral B. Coronary C. Muscular D. Pulmonary Preferred response: A

4. Which of the West zones of the lung are effectively perfused in proportion to their height above the left atrium? A. Zones I and II B. Zone II only C. Zones I and III D. Zone III only Preferred response: B

Rationale The inflow pressure of the pulmonary circulation is low, thus creating a vertical gradation to the distribution of blood flow in the lung. Hydrostatic pressure must be adjusted for vertical height above the left atrium, both at the inflow and at the outflow of every alveolar capillary unit. For example, given a pulmonary artery mean pressure of 20 cm H2O (zeroed at the level of the left atrium), an alveolar capillary unit 12 cm above the left atrium faces an inflow pressure of only 8 cm H2O. A left atrial pressure of 5 cm H2O generates no opposing outflow pressure to an alveolar capillary unit more than 5 cm above the left atrium. Critical closing pressure of postcapillary vessels therefore sets outflow pressure for a unit 10 cm above the left atrium. If intrinsic vascular resistance were identical throughout the lung, flow at any vertical height would be determined by hydrostatic driving pressure (inflow-outflow) and would be greatest at the base and least at the apex of the lung. This phenomenon partitions the lung into three vertical regions, named West zones. Zone I vessels are higher above the left atrium than pulmonary artery pressure and are not perfused by the pulmonary artery. Zone II vessels lie above the height defined by the hydrostatic left atrial pressure but below the height of the

Rationale In marked contrast to other vascular beds, neural stimuli have relatively little effect on basal cerebral blood flow (CBF). Cerebral vessels display extensive perivascular innervations, especially the sympathetic nerves arising from the superior cervical sympathetic ganglia, but the brain is well protected from circulating catecholamines by the blood-brain barrier. Thus many of the vasoactive agents used in the critical care setting (a- and b-adrenergic agonists) have minimal effects on resting cerebral vascular tone. Mild to moderate electrical stimulation, as well as surgical resection of both the sympathetic and parasympathetic nervous system, does not alter cerebral vascular tone under resting conditions. However, vigorous sympathetic stimulation, as occurs with strenuous exercise or hypertension, does result in vasoconstriction of largeand medium-size cerebral vessels. Thus a neurogenic mechanism may not mediate cerebral vascular resistance under normal conditions, but it does provide protection at times of stress. 6. Which of the following substrates can the brain use during periods of starvation? A. Arginine B. Glutamine C. Glycogen D. Ketones Preferred response: D

Rationale At rest, cerebral blood flow is approximately 50 mL/100 g tissue/ min. Cerebral oxygen consumption is surprisingly high, averaging 3.2 mL/100 g tissue/min. Glucose is the primary energy substrate, although ketones can be utilized during periods of starvation. The brain has no functional capacity to store energy and thus is completely dependent on a steady supply of O2 because up to 92% of its adenosine triphosphate (ATP) production results from the oxidative metabolism of glucose. 7. Which of the following drugs should be used with caution in patients with increased intracranial pressure due to a risk of precipitating herniation? A. Esmolol B. Lorazepam C. Nitroprusside D. Pentobarbital Preferred response: C

e30

S E C T I O N XV



Pediatric Critical Care: Board Review Questions

Rationale Nitroprusside and other nitric oxide donor compounds can dilate cerebral vessels. This process greatly complicates the management of hypertension in patients with increased intracranial pressure. In such patients, nitroprusside may reduce arterial pressure but raise cerebral blood flow and blood volume, thereby causing herniation. 8. Which of the following mediators has been implicated in the vasospasm following subarachnoid hemorrhage? A. Adenosine B. Carbon dioxide C. Cyclic AMP (cAMP) D. Endothelin-1 (ET-1) Preferred response: D

Rationale Substance P, acetylcholine, oxytocin, ET-1, adenosine diphosphate, ATP, and prostaglandin cause nitric oxide (NO)-dependent cerebral vasodilation. Impaired NO signaling is important in the pathophysiology of subarachnoid hemorrhage in which endothelial dysfunction has been well documented, leading to the important clinical problem of vasospasm. ET-1 is an important mediator of cerebrovascular tone. Both ETA and ETB receptors have been identified in the cerebral vasculature. ET-1 given in high concentrations constricts cerebral vessels, probably via ETA receptor activation. However, ET-1 given in low concentrations relaxes cerebral vessels via endothelial cell ETB receptor activation, a response that is NO dependent. ET-1 has been identified as an important mediator of vasospasm following subarachnoid hemorrhage. ET-1 levels are increased following subarachnoid hemorrhage. In association with the increase in ET-1 levels are increases in ETA receptor levels, smooth muscle cell ETB receptor levels (which mediate vasoconstriction), and endothelin-converting enzyme activity. Adenosine leads to cerebral vasodilation through an increase in cAMP and increases more than fivefold with hypoxia. Adenosine is critical in CBF autoregulation. Carbon dioxide plays a critical role in regulation of CBF. A linear increase in CBF is seen with increasing Paco2, making CO2 one of the most potent known cerebral vasodilators. Carbon dioxide exerts its effect via reduction in perivascular pH. Arterial H1 cannot cross the blood-brain barrier, but CO2 can easily diffuse into the brain. Perivascular acidosis dilates the cerebral vasculature, whereas alkalosis leads to vasoconstriction. 9. In the normal heart, which of the following regions is more prone to ischemia because of low perfusion pressure? A. Right ventricle subepicardial regions B. Right ventricle subendocardial regions C. Left ventricle subepicardial regions D. Left ventricle subendocardial regions Preferred response: D

Rationale Myocardial perfusion over a cardiac cycle is normally approximately the same per gram of tissue in the outer (subepicardial), mid, and inner (subendocardial) layers of the left ventricle, but the dynamics during the cardiac cycle are complicated. At the end of diastole, when the ventricle is relaxed and tissue pressures

are probably less than 10 mm Hg in any layer of the left ventricle, pressures in the intramural arteries are probably similar to each other and to aortic pressure. At the beginning of systole, tissue pressure rises to equal intracavitary pressure in the subendocardium but then falls off linearly across the wall to about 10 mm Hg in the subepicardium. These pressures are for an instant added to those inside the vessels because the vessels’ walls are not rigid, and as a result, intravascular pressures in subendocardial arteries exceed aortic pressures, but aortic pressures are higher than are pressures in subepicardial arteries. These pressure gradients and the greater shortening of subendocardial versus subepicardial muscle fibers during systole compress the subendocardial vessels and squeeze blood out of them both forward into the coronary sinus and backward toward the epicardium. In fact, narrowing of the subendocardial vessels facilitates thickening and shortening of the myocytes. This backflow enters the subepicardial arteries to supply their systolic flow. In systole, some forward flow into the orifices of the coronary arteries does indeed occur, but this forward flow does not perfuse the myocardium; it merely fills the extramyocardial arteries. In fact, there is often reverse flow in the epicardial coronary arteries. In early diastole, blood flows first into the subepicardial vessels that have not been compressed, but it takes longer to refill the narrowed subendocardial vessels. Given enough time and perfusing pressure, all the myocardium will be perfused, but if diastole is too short or perfusion pressure is too low, subendocardial ischemia occurs. Right ventricular myocardium, on the other hand, normally is perfused both in systole and in diastole, because of lower tissue pressures. 10. Based on the graph for pressure-flow relations in the left coronary artery during normal flow and maximal vasodilation, which of the following statements is correct? A. Coronary vascular reserve is independent of perfusion pressure. B. For normal maximal flow, flow is uncoupled from metabolism. C. For normal autoregulated flow, flow is uncoupled from metabolism. D. For normal autoregulated flow, an increase in heart rate will increase maximal flow at any perfusion pressure. Preferred response: C

Rationale Coronary vascular resistance has three components: a basal low resistance in the arrested heart with maximally dilated vessels, an added resistance when vessels have tone, and a phasic resistance added whenever the ventricle contracts. In the beating heart with vessels maximally dilated by a pharmacologic dilator, the second of these resistances is absent. Perfusion of the left ventricular myocardium then produces a steep pressure-flow relation that is linear at higher flows but usually curvilinear at low pressures and flows (Figure 136.7). Because the vessels are maximally dilated, flow is uncoupled from metabolism and depends only on driving pressure and resistance. If heart rate is increased, maximal flow at any perfusion pressure decreases because the heart is in a relaxed state for a smaller proportion of each minute. If tone is allowed to return to the coronary vessels, then the pressure-flow relationship can be assessed at different perfusion pressures after cannulating the left coronary artery. It is necessary to do this because when cardiac metabolism and blood



CHAPTER 136 Board Review Questions

e31



flow are coupled, increasing aortic blood pressure will increase coronary flow not only by increasing perfusion pressure but also by increasing myocardial oxygen demand. Under normal conditions coronary blood flow is autoregulated, such that if perfusion pressure is raised or lowered from its normal value, there is a range over which almost no change in flow occurs; a rise in pressure has caused vasoconstriction, and a fall in pressure has caused vasodilation. At perfusion pressures above some upper limit, flow increases, probably because the pressure overcomes the constriction. More important, at pressures below about 40 mm Hg (but varying, as discussed later), flow decreases predominantly in the deep subendocardial muscle (Figure 136.8), indicating that some vessels have reached maximal vasodilation and can no longer decrease resistance to compensate for the decreased perfusion pressure. In these vessels, flow and pressure are directly related. If this pressure dependency occurs, then a further decrease in perfusion pressure decreases local blood flow below the required amount, or if myocardial oxygen demands increase

Rationale Coronary flow reserve also can be reduced if maximal flows are normal but autoregulated flows increase (Figure 136.9). Increased myocardial flows above normal values can occur with exercise, tachycardia, anemia, carbon monoxide poisoning, leftward shift of the hemoglobin oxygen dissociation curve (as in infants with a high proportion of fetal hemoglobin), hypoxemia, thyrotoxicosis, acute ventricular dilation (because of increased wall stress), inotropic stimulation by catecholamines, and acquired ventricular hypertrophy. If autoregulated flow is normal but maximal flow is decreased, as indicated by the decreased slope of the pressure-flow relation during maximal dilation (Figure 136.10), then coronary flow reserve will be reduced. Such a change can occur with marked tachycardia; a decrease in the number of coronary vessels due to

Normal maximal flow

600

400

400

Flow (mL/min)

500

300 R1 200

R2 Normal autoregulated flow

100 0 0

40

80

120

160

Perfusion pressure (mm Hg)

• Fig. 136.8



Normal maximal flow

600

500



Flow (mL/min)

11. Regarding coronary artery perfusion, a shift from normal autoregulated flow to increased autoregulated flow may be seen in which of the following conditions? A. Myocardial ischemia B. Coronary spasm C. Increased blood viscosity D. Carbon monoxide poisoning Preferred response: D

300

Increased autoregulated flow

R2

200

R1 Normal autoregulated flow

100 0 0

40

80

120

160

Perfusion pressure (mm Hg)

• Fig. 136.9



• Fig. 136.7



at the same low perfusion pressure (as will occur if the ventricle becomes dilated), the requisite increase in flow will not occur. These two conditions cause subendocardial ischemia. At any given pressure, the difference between autoregulated and maximal flows is termed coronary flow reserve. (Coronary flow reserve can be measured in units of mL/min but also can be assessed by a dimensionless flow reserve ratio derived by dividing maximal flow by resting flow.) Flow reserve depends on perfusion pressure because of the steepness of the pressure-flow relation in maximally dilated vessels. Coronary flow reserve indicates how much extra flow the myocardium can get at a given pressure to meet increased demands for oxygen; if reserve is much reduced, then flow cannot increase sufficiently to meet demands and myocardial ischemia will occur. What the figure does not show is that coronary flow reserve is normally lower in the subendocardium than in the subepicardium and that decreases in coronary flow reserve are always more profound in the subendocardium than in the subepicardium.



e32

S E C T I O N XV



Pediatric Critical Care: Board Review Questions

small vessel disease, as in some collagen vascular diseases, especially systemic lupus erythematosus; increased resistance to flow in one or more large coronary vessels because of embolism, thrombosis, atheroma, or spasm; impaired myocardial relaxation due to ischemia; myocardial edema; a marked increase in left ventricular diastolic pressure; marked increase in left ventricular systolic pressure if coronary perfusion pressure is not also increased, as in aortic stenosis or incompetence; and an increase in blood viscosity, most commonly seen with hematocrit levels higher than 65%. Normal maximal flow

600

Flow (mL/min)

500 400 300

Reduced maximal flow

R1

200 R2

100

Normal autoregulated flow

0 0

40

80

120

160

Perfusion pressure (mm Hg)

• Fig. 136.10





12. Pulmonary embolism and subsequent right ventricular failure develops in a previously normal 17-year-old patient. Which of the following drugs would help restore right ventricular function to normal? A. Dobutamine B. Esmolol C. Milrinone D. Norepinephrine Preferred response: D

Rationale Right ventricular myocardial blood flow is affected by the low right ventricular systolic pressure and the fact that changes in aortic pressure alter coronary perfusion pressure without altering right ventricular pressure work. If the normal right ventricle is acutely distended—for example, by pulmonary embolism— eventually right ventricular failure occurs. The increased wall stress increases oxygen consumption but the raised systolic pressure reduces coronary flow, so when supply cannot match demand, right ventricular myocardial ischemia occurs. Raising aortic perfusing pressure mechanically or with a-adrenergic agonists increases right ventricular myocardial blood flow, relieves ischemia, and restores right ventricular function to normal. Improved coronary flow is not the only mechanism of this improvement; increased left ventricular afterload moves the ventricular septum toward the right ventricle and improves left ventricular performance.

Chapter 25: Endothelium and Endotheliopathy 1. The endothelium plays an important role in controlling the vascular tone. Which of the following endothelium-derived factors is least likely to cause vasodilation? A. Acetylcholine B. Endothelin C. Endothelium-derived hyperpolarizing factor D. Nitric oxide (NO) Preferred response: B

Rationale Endothelin is an endothelium-derived vasoconstrictor. All other choices are endothelium vasodilators. The vasodilatory action of acetylcholine is derived through NO. Both NO and prostacyclin can be used clinically. 2. Binding of thrombin to cell surface thrombomodulin facilitates activation of which of the following? A. Antithrombin III B. Platelets C. Protein C D. Protein S Preferred response: C

Rationale Thrombin activity is also modulated by endothelial cell synthesis of thrombomodulin. The binding of thrombin to thrombomodulin facilitates the enzyme’s activation of the anticoagulant protein C. Activated protein C (APC) activity is enhanced by cofactor C, also called protein S, which is synthesized by endothelial cells as well as other cells (Figure 136.11). APC inhibits factor Va and factor VIIIa.

Tissue factor

Tissue factor inhibitor

Factor VIIa ; TF

Factor VII

Factor VIIIa

Factor X

Factor Xa

Prothrombin

Thrombin

Fibrinogen Antithrombin

Endothelium Factor VIII

Factor Va

Activated protein C

Fibrin

Protein C

Thrombomodulin

• Fig. 136.11





APC has a variety of antiinflammatory activities. It suppresses inflammatory cytokine during sepsis, inhibits leukocyte adhesion, decreases leukocyte chemotaxis, reduces endothelial cell apoptosis, helps maintain endothelial cell barrier function through activation of the sphingosine-1 phosphate receptor, and minimizes the decrease in blood pressure associated with severe sepsis. Another interesting effect of APC is its plasminogen activator inhibitor 1 (PAI-1) neutralizing effect. PAI-1 is a glycoprotein that acts as an acute-phase protein during acute inflammation. Its primary role in vivo is the inhibition of both tissue- and urokinase-type plasminogen activators. PAI-1 is the most efficient inhibitor of APC and thrombin in the absence of heparin. PAI-1



CHAPTER 136 Board Review Questions

3. Protein C acts by inhibiting the activities of which of the following? A. Factors II and V B. Factors II and Xa C. Factors Va and VIIIa D. Factors VIIa and Xa Preferred response: C

Rationale See Fig. 136.11. 4. Exposure and binding of endothelial tissue factor to which of the following factors initiates the coagulation cascade, resulting in the formation of fibrin? A. Factor II B. Factor Va C. Factor VIIa D. Factor VIII Preferred response: C

Rationale The expression and release of tissue factor is the pivotal step in transforming the endothelium from an anticoagulant to a procoagulant surface. Tissue factor accelerates factor VIIa–dependent activation of factors X and IX. The synthesis of tissue factor is induced by a number of agonists, including thrombin, endotoxin, several cytokines, shear stress, hypoxia, oxidized lipoproteins, and other endothelial insults. Once endothelial cells expressing tissue factor are exposed to plasma, prothrombinase activity is generated and fibrin is formed on the surface of the cells. Tissue factor also can be found in plasma as a soluble protein. Its role there is not well understood, but it probably plays a role in the initiation of coagulation. 5. Which of the following causes the production of inducible NO synthase (iNOS)? A. Bradykinin B. Histamine C. Insulin D. Tumor necrosis factor-a Preferred response: D

Rationale NO is generated from the conversion of L-arginine to NO and L-citrulline by the enzyme nitric oxide synthase (NOS). Two general forms of NOS exist: constitutive and inducible. In the unstimulated state, NO is continuously produced by constitutive NOS (cNOS). The activity of cNOS is modulated by calcium that is released from endoplasmic stores in response to the activation of certain receptors. Substances such as acetylcholine, bradykinin, histamine, insulin, and substance P stimulate NO production through this mechanism. Similarly, shearing forces acting on the endothelium are another important mechanism regulating the release of NO. The inducible form of NOS is not calcium dependent but instead is stimulated by the actions of cytokines (e.g.,

tumor necrosis factor-a and interleukins) or bacterial endotoxins (e.g., lipopolysaccharide). Induction of iNOS occurs over several hours and results in NO production that may be more than 1000fold greater than that produced by cNOS. This mechanism is important in the pathogenesis of inflammation (see Figure 136.12). Endotoxins Cytokines

Shearing forces

L-arg

+

Ca2+

+

iNOS cNOS NO + Cit

+ Ca2+

CaM

Vasoactive agents +

R

Endothelial cell NO + GTP

GC

cGMP-PK cGMP

cGMP-P

Smooth muscle cell

• Fig. 136.12 6. Where is NO produced? A. Endothelial cell B. Neutrophils C. Platelets D. Red blood cell



competes with thrombomodulin for binding with thrombin, which, in combination with its inhibition of APC, makes it a strong local procoagulant by a combined action of displacing and inactivating anticoagulant thrombin from thrombomodulin. In this way PAI-1 has important pathophysiologic effects in acute and chronic diseases.

e33



Preferred response: A

Rationale Furchgott and Zawadzki first postulated the existence of an endothelial relaxing factor in 1980, when they noticed that the presence of endothelium was essential for rabbit aortic rings to relax in response to acetylcholine. Later, it was determined that the biological effects of endothelial relaxing factor are mediated by NO. NO is generated in the endothelial cell from the conversion of L-arginine to NO and L-citrulline by the enzyme NOS. 7. NO acts by which of the following mechanisms? A. Binding to cyclic adenosine monophosphate (cAMP) B. Binding to cyclic guanosine monophosphate (cGMP) C. Binding to adenyl cyclase D. Binding to guanylyl cyclase Preferred response: D

Rationale Once NO is formed by an endothelial cell, it readily diffuses out of the cell and into adjacent smooth muscle cells, where it binds to and activates the soluble form of guanylyl cyclase, resulting in the production of cGMP from guanosine triphosphate. 8. Prostacyclin acts by which of the following mechanisms? A. Binding to cAMP B. Binding to cGMP C. Binding to guanylyl cyclase D. Binding to prostacyclin receptors Preferred response: D

Rationale Prostacyclin is a potent vasodilator that is active in both the pulmonary and systemic circulations. In addition to its vasodilatory effects, prostacyclin also has antithrombotic and antiplatelet activity. Its release may be stimulated by bradykinin and adenine nucleotides. Like NO, it is chemically unstable with a short half-life. However, unlike NO, prostacyclin activity in arterial beds de-



Pediatric Critical Care: Board Review Questions

pends on its ability to bind to specific receptors in vascular smooth muscle. Its vasodilator activity is therefore determined by the expression of such receptors. Prostacyclin receptors are coupled to adenylate cyclase to elevate cAMP levels in vascular smooth muscle. The increase in cAMP results in (1) stimulation of adenosine triphosphate–sensitive K1 channels, resulting in hyperpolarization of the cell membrane and inhibition of the development of contraction, and (2) increased efflux of Ca21 from the smooth muscle cell and inhibition of the contractile machinery. 9. Which of the following statements regarding the interaction between nitric oxide (NO) and prostacyclin is correct? A. NO inhibits the action of prostacyclin. B. NO and prostacyclin act sequentially to cause vasodilation. C. Prostacyclin inhibits the action of NO. D. Prostacyclin facilitates the cellular release of NO. Preferred response: D

Rationale Prostacyclin facilitates the release of NO by endothelial cells, and the action of prostacyclin in vascular smooth muscle is potentiated by NO. Interestingly, NO also may potentiate the effects of prostacyclin. The NO-mediated increase in cGMP in smooth muscle cells inhibits a phosphodiesterase that breaks down cAMP, therefore indirectly prolonging the half-life of the second messenger of prostacyclin.

Chapter 26: Principles of Invasive Cardiovascular Monitoring 1. A critically ill 4-year-old child has been admitted to the pediatric ICU after endotracheal intubation, central venous catheter placement in the right internal jugular vein, and arterial catheter placement in the left radial artery. Which set of findings below most reliably predict that a fluid bolus will increase cardiac output? A. A central venous pressure of 7 mm Hg while in normal sinus rhythm B. A pulse pressure variation of 20% during deep sedation and paralysis C. A pulmonary artery occlusion pressure of 12 mm Hg during deep sedation and paralysis D. An increase of his mean arterial pressure by 10 when his liver is manually compressed Preferred response: B

Rationale Due to cardiopulmonary interactions, when a patient receives a positive pressure breath their right ventricular preload will decrease, an effect accentuated by hypovolemia. The result of this temporary decrease in right ventricular preload is a decrease in left ventricular preload which follows a few cardiac cycles later and manifests itself as a decrease in pulse pressure during insufflation of the lungs due to the decreased blood volume ejected. A is incorrect as only central venous pressures ,5 mm Hg have been reliably shown to predict an increase in cardiac output following a fluid bolus. C is incorrect because a pulmonary artery occlusion pressure of 12 mm Hg during deep sedation and paralysis is higher than normal suggesting that the left atrium or ventricle is stressed, a condition likely to be worsened by increasing its volume load.

D is incorrect as there is insufficient evidence for manual liver palpation as a surrogate or replacement for passive leg raising, particularly without the explicit inclusion of sedation and paralysis. Liver compression may raise a child’s blood pressure due to pain, discomfort, anxiety, or increased alertness. 2. In a well calibrated, well-functioning arterial line of appropriate size, cannulation of which site will report the largest pulse pressure? A. Axillary artery B. Brachial artery C. Common femoral artery D. Radial artery Preferred response: D

Rationale Since impedance increases in the circulatory system the further a measurement is taken from the heart, the blood pressure increases with distance from the heart while the flow velocity decreases. The effect of this will be to raise the systolic peak while accelerating the diastolic downslope of the pressure reading. Mathematically the area under this curve will remain the same, though the pulse pressure will vary depending on where it is measured, with the highest measurements being those taken most distally to the heart (Figure 136.13). Pressure (cm H2O)

S E C T I O N XV

Velocity (cm/sec)

e34

260 220 180 140 90 50

Ascending Thoracic Abdominal Abdominal aorta aorta aorta aorta middle distal Flow velocity

Femoral

0 30

• Fig. 136.13





3. Which patient is most likely to display pulsus alternans on their arterial line tracing? A. An infant newly diagnosed with anomalous left coronary artery from the pulmonary artery (ALCAPA) B. A child on noninvasive positive pressure ventilation with community acquired pneumonia C. A teenager with primary pulmonary arterial hypertension newly diagnosed with corona virus infection D. A child who is postoperative day one from deceased donor renal transplant Preferred response: A

Rationale Pulsus alternans is an arterial line tracing which alternates between weak and strong beats and is associated with left ventricular systolic impairment. The systolic impairment results in an increased end-diastolic volume which leads to an increased stroke volume on the next cardiac cycle. The most likely patient to have left ventricular systolic impairment is the child with ALCAPA due to the deoxygenated blood from the pulmonary artery supplying their left coronary artery.



CHAPTER 136 Board Review Questions

4. Central venous pressure is most frequently used as a measure of which of the following? A. Left ventricular preload B. Left ventricular afterload C. Right ventricular preload D. Right ventricular afterload Preferred response: C

Rationale The central venous pressure is measured in the right atrium, super vena cava, or inferior vena cava and is the pressure at the end of diastole. Thus it represents the force, or load, on the right ventricle during diastole which is also called preload. Left ventricular preload cannot be measured from the central venous system, nor can the afterload of either ventricle. 5. A 7-day-old male with d-transposition of the great arteries presents 8 hours after an arterial switch operation with a heart rate of 180 beats per minute and hypotension (mean arterial pressure of 35 mm Hg). He appears to have A-V dissociation on telemetry. What would be the most likely pattern seen on the central venous pressure (CVP) monitor? A. Broad, flat a waves B. Enlarged a waves C. Fused c and v waves D. Steep abrupt x and y descent Preferred response: B

Rationale Cannon a waves are enlarged a waves seen when the right atrium is ejecting against a closed tricuspid valve, and they may be seen when atrioventricular discordance occurs (in this case, during an episode of postoperative junctional ectopic tachycardia). They will resolve once the underlying illness is treated. Response C occurs in tricuspid regurgitation, when the backflow of blood out of the right ventricle obliterates the x descent and the c wave becomes accentuated. Response D occurs during pericardial constriction or tamponade physiology. Although the patient is tachycardic and hypotensive, there are no other signs, from the stem, that the patient has this physiology. Response A, or loss of a wave, occurs during atrial fibrillation because loss of atrial contraction results in missing the a wave (Figure 136.14).

e35

6. An otherwise healthy infant has a peripheral arterial line placed in his dorsalis pedis for monitoring during an elective surgery. What discrepancy is likely to be found between a cuff blood pressure on the arm and the readings assessed from the arterial line? A. The mean arterial pressure (MAP) measurements will be widely different. B. The systolic blood pressure (SBP) measurement will be lower on the intraarterial measurement. C. The diastolic blood pressure (DBP) measurement will be lower on the intraarterial measurement. D. The MAPs will be the same, but the SBP on the cuff pressure will be higher. Preferred response: C

Rationale This is referred to as distal pulse amplification and occurs due to the nature of the vascular tree; the systolic blood pressure increases, and diastolic blood pressure decreases as you move peripherally, with a more exaggerated pulse pressure. The MAPs remain the same between the peripheral and central sites. Pressure waveform measurements from different sites of the arterial tree have varying morphologies depending on the properties of the vascular bed. The changes that occur as you move peripherally have to do with alterations in impedance and harmonic resonance. 7. A 10-year-old child has been admitted to the intensive care unit (ICU) in shock. The child is endotracheally intubated, mechanically ventilated, and has a pulmonary artery catheter in place. The following information is available: • Heart rate: 140 beats/min • Blood pressure: 90/60 mm Hg • Central venous pressure (CVP): 10 mm Hg • Mean arterial pressure (MAP): 70 mm Hg • Pulmonary arterial pressure: 45/25 mm Hg • Mean pulmonary arterial pressure: 32 mm Hg • Venous oxygen saturation: 65% • Pulmonary artery occlusion pressure: 15 mm Hg • Carbon monoxide (CO): 3 L/min What is the calculated systemic vascular resistance (SVR)? A. 1600 dynes sec/cm5 B. 30 Woods units C. 1200 dynes sec/cm5 D. 1200 mm Hg/m2 Preferred response: A

Rationale SVR is calculated by (MAP 2 CVP)/CO 5 (70 2 10)/3 5 60/ 3 5 20 in Woods units. Woods units are converted to dynes sec/cm5 by a multiplicative factor of 80. Thus 20 3 80 5 1600 dynes sec/cm5. V

C X

Y

A C

V X

• Fig. 136.14

Y  

A



e36

S E C T I O N XV   Pediatric Critical Care: Board Review Questions

8. A 3-year-old child is brought to the emergency department with fever and increasing lethargy during the past 24 hours. On initial evaluation the child is minimally responsive and has poor distal perfusion with cool hands and feet. Blood pressure is 70/30 mm Hg, and oxygen saturation is 85% in room air. The child is intubated, central venous access is obtained, and intravenous fluids are administered. An established goal of continued resuscitation for this child is which of the following? A. Achieve a pulmonary artery occlusion pressure of greater than 15 mm Hg B. Achieve a central venous pressure (CVP) of 15 mm Hg or higher C. Achieve a mixed venous saturation of 70% or higher D. Achieve a mean arterial pressure (MAP) of 80 mm Hg or higher Preferred response: C

Rationale Achieving a mixed venous saturation of 70% or higher is one of the goals of early goal-directed therapy for persons with septic shock. Early goal-directed therapy has been shown to decrease mortality in persons with septic shock.

Chapter 27: Assessment of Cardiovascular Function 1. Two infants with dextro-transposition of the great arteries have had arterial switch operations earlier in the day. Approximately 6 hours postoperatively, infant A is on a milrinone infusion of 0.3 mg/kg/min with a mean arterial blood pressure of 45 mm Hg and a central venous pressure of 8 cm H2O. The last lactate level was 3.2 mmol/L (previously 4.5 mmol/L) and a recent mixed venous O2 saturation is 75%. Infant B is on a milrinone infusion of 0.3 mg/kg/min with a mean arterial blood pressure of 45 mm Hg and a central venous pressure of 8 cm H2O. The last lactate is 3.2 mmol/L (previously 2.1 mmol/L) and a recent mixed venous saturation is 60%. Infant B has received a total of 20 mL/kg of crystalloid in the last 4 hours. Both infants have 100% arterial oxygen saturations and identical mechanical ventilation parameters. Based on the information above, which infant is at higher risk of clinical decompensation in the next 6 hours: A. Infant A B. Infant B C. Both are at equal risk based on the information provided Preferred response: B

Rationale While both infants have identical vital signs and are on the same vasoactive infusions, infant B has received a higher “quantity of therapy” (QOT) due to the volume infusions in the last 4 hours to maintain the hemodynamics listed. In addition, infant B has a rising lactate and a lower mixed venous saturation (higher arteriovenous oxygen difference) which may signify inadequate oxygen delivery. Infant B should be carefully monitored, and additional investigations into the cause for the higher QOT should be considered.

2. A 6-month-old infant with tetralogy of Fallot is recovering in the intensive care unit after a complete surgical repair. Approximately 6 hours after the operation is completed, the central venous pressure (CVP) is 15 mm Hg. The patient is hypotensive and receives 10 mL/kg of crystalloid. Which of the following is the least likely to be observed in the physiology described? A. Fluid accumulation in the thoracic and abdominal cavities B. Increased chest wall stiffness C. Increased intrathoracic pressures D. Increased peripheral and pulmonary edema E. Increased venous return Preferred response: E

Rationale The patient described in this question with tetralogy of Fallot is likely to have significant right ventricular diastolic dysfunction resulting in high filling pressures (high CVP). Leaky vasculature as a result of inflammation due to cardiopulmonary bypass can result in significant third-spacing of fluid. A fluid bolus for hypotension causes increased tissue edema and fluid accumulation in the thoracic and abdominal cavities which results in increased chest wall stiffness and increased intrathoracic pressure. As a result, there is decreased venous return and therefore decreased preload to the right ventricle which results in further hypotension and diminished cardiac output. 3. An infant with hypoplastic left heart syndrome is recovering after a Norwood Stage I procedure with a modified BlalockTaussig shunt. The patient’s arterial oxygen saturation and mean arterial blood pressure are within normal limits. The infant has a lactate level that has been rising over the last 4 hours and a wide arteriovenous O2 (AVO2) saturation difference. Which of the following will not increase systemic oxygen delivery in this patient? A. Addition of inhaled nitric oxide to improve pulmonary blood flow B. Afterload reduction C. Blood transfusion D. Increased inotropic support with dobutamine E. Sedation and paralysis Preferred response: A

Rationale This patient with single ventricle physiology has complete mixing of systemic and pulmonary venous blood. The rising lactate, minimal urine output, and widened AVO2 difference suggested compromised oxygen delivery despite a normal oxygen saturation and mean arterial blood pressure. The addition of a pulmonary vasodilator to improve pulmonary blood flow would likely exacerbate the problem further by increasing pulmonary blood flow at the expense of systemic blood flow. In other words, the Qp:Qs ratio would increase. All of the other options would increase systemic oxygen delivery by improving Qs (options B and D), decreasing systemic oxygen demand (option E), or increasing oxygen carrying capacity and likely mixed venous saturation (option C).

CHAPTER 136  Board Review Questions

4. A 12-year-old girl is diagnosed with viral myocarditis and has an echocardiogram showing severely depressed biventricular systolic function. Her blood pressure as measured by invasive monitoring is normal for age. Which of the following factors is compatible with a relatively low quantity of therapy? A. A central venous pressure of 6 mm Hg B. A lidocaine infusion to suppress ventricular tachycardia C. An epinephrine infusion D. Need for mechanical ventilation E. Two sodium bicarbonate boluses due to metabolic acidosis Preferred response: A

Rationale The patient is presenting with decreased systolic ventricular function in the setting of viral myocarditis. The cardiac output is low; however, the patient’s blood pressure is normal due to the compensatory rise in systemic vascular resistance (SVR). As a result of these factors, one would expect that the patient might require increased filling pressures, a need of inotropic support, and/or mechanical ventilation to provide positive pressure ventilation to decrease oxygen demands and decrease left ventricular afterload. In addition, a patient with myocarditis is at risk of ventricular arrhythmias due to the irritable myocardium. The presence of a metabolic acidosis can also signify poor systemic oxygen delivery. In this particular case, the presence of a low central venous pressure would be reassuring that the patient has not required fluid resuscitation and therefore a significant quantity of therapy. All of the other options would signify a patient that is more critically ill. In this scenario, it would be important to consider advanced therapies such as mechanical circulatory support if there are markers of poor oxygen delivery despite the interventions listed above. 5. What is a primary determinant of systemic oxygen delivery (Do2)? A. Hemoglobin concentration B. Intrathoracic pressure C. Oxygen consumption D. Pulmonary venous oxygen content Preferred response: A

Rationale Do2 5 Cardiac output (CO) 3 arterial content of oxygen (Cao2), where Cao2 5 (Sao2 3 Hgb 3 1.36) 1 (Pao2 3 0.0031). Therefore because hemoglobin concentration is a key variable that determines systemic arterial O2 content, it also is a primary determinant of Do2. 6. A 3-year-old child with hypoplastic left heart syndrome returns from the operating room following a lateral tunnel fenestrated Fontan procedure. The initial postoperative arterial blood gas shows an oxygen saturation of 80% on a Fio2 1.0 via the mechanical ventilator. Postoperative chest x-ray is unremarkable, and the echocardiogram shows good right ventricular systolic function, a patent 3-mm fenestration with right-to-left shunting, trace tricuspid regurgitation, no systemic outflow tract obstruction, and an unobstructed Fontan pathway with good flow to both pulmonary arteries. The patient is mildly hypotensive in sinus rhythm with a hemoglobin level of 15 gm/dL and no postoperative bleeding. What is the most appropriate initial management to improve blood pressure and oxygenation? A. Cool the patient to decrease oxygen consumption. B. Remove the patient from positive pressure mechanical ventilation. C. Start an inotropic agent to improve myocardial contractility. D. Transfuse with red blood cells to increase hemoglobin. Preferred response: B

e37

Rationale Positive pressure mechanical ventilation decreases systemic venous return and increases pulmonary vascular resistance, both of which are detrimental to Fontan physiology. Spontaneous respiration with negative intrathoracic pressure will provide the optimal conditions for systemic venous return and decreased pulmonary vascular resistance resulting in improved pulmonary blood flow (oxygenation), preload, and cardiac output. 7. A 3-year-old girl with history of a large ventriculoseptal defect is admitted to your pediatric intensive care unit after ventriculoseptal defect closure. A chest radiograph shows that the cardiothoracic ratio is 0.4. The cardiac apex is displaced upward. What is the most likely reason for these radiographic changes? A. Large pericardial effusion B. Left ventricular (LV) enlargement C. Patent foramen ovale D. Right ventricular (RV) enlargement Preferred response: D

Rationale The cardiothoracic ratio gives a quantitative estimate of cardiac size, which is obtained by dividing the transverse measurement of the cardiac shadow in the posteroanterior view by the width of the thoracic cavity. Cardiomegaly is present if this value is greater than 0.5 in adults and 0.6 in infants. Although useful for assessing LV enlargement, the cardiothoracic ratio is not as sensitive to RV enlargement. RV enlargement results in lateral and upward displacement of the cardiac apex on the posteroanterior view and filling of the retrosternal space on the lateral view. It is perhaps more important to know what the chest x-ray may not reveal: significant cardiac problems, such as constrictive pericarditis, acute fulminant myocarditis, and even acute pericardial tamponade are often associated with a normal-sized heart on a chest radiograph. 8. A 1-year-old patient has a hemoglobin of 9 mg/dL and arterial oxygen saturation of 86%. The child’s arterial blood gas results reveal the following values: pH, 7.38; pco2, 53 mm Hg; and Pao2, 56 mm Hg. Cardiac output is estimated to be 3.2 L/minute. The child’s weight is 10 kg. What is the approximate quantity of oxygen delivered to the tissues in 1 minute (Do2)? A. 10 mL O2/kg/min B. 20 mL O2/kg/min C. 30 mL O2/kg/min D. 40 mL O2/kg/min Preferred response: C

Rationale Tissue oxygenation is directly related to both Do2 and systemic arterial blood pressure. Do2, the quantity of O2 delivered to the tissues per minute, is the product of systemic blood flow (SBF), which equals cardiac output except in patients with certain cardiac malformations, and arterial O2 content:  L   mL   10  CO  D O2   minute   minute   mL   CaO2  mL blood   100 

e38

S E C T I O N XV   Pediatric Critical Care: Board Review Questions

where CO 5 cardiac output or SBF in L/min or L/min/m2 and CaO2 5 quantity of O2 bound to hemoglobin plus the quantity of O2 dissolved in the plasma in arterial blood. The O2 content of arterial blood (mL O2/dL blood) equals: CaO2  (SaO2  Hgb    1.36)    (PaO2     0.003) where Sao2 5 arterial O2 saturation, Hgb 5 hemoglobin concentration (g/dL), 1.36 (constant) 5 amount of O2 bound per gram of hemoglobin (mL) at 1 atmosphere of pressure, Pao2 5 arterial partial pressure of O2, and 0.003 (constant) 5 amount of O2 dissolved in plasma at 1 atmosphere. The quantity of dissolved O2 is generally considered to be negligible in the normal range of Pao2. In this case, the Cao2 is (9 3 1.36 3 0.86) 1 (0.0031 3 56) 5 10.53 1 0.17 5 10.7 mL O2/100 mL. The Do2 is therefore 10 3 3.2 3 10.7 5 342.4 mL O2/min or 34.24 mL O2/k.

Chapter 28: Cardiac Failure and Ventricular Assist Devices 1. A 5-year-old male has a 4-day history of persistent high fever (.38.5°C) and progressive lethargy. Mother reports that today he stopped eating and became tachypneic resulting in the child being transported to the emergency department. In triage, the child was noted to have the following vital signs: temperature, 38.8°C; heart rate, 140 beats per minute; respiration rate, 40 per minute; blood pressure, 75/60 mm Hg; Spo2, 85% in room air. The child appears to be in shock, with pallor, cyanosis, cold extremities with thready pulses, and he has an audible S3/S4 gallop rhythm with bilateral diffuse rales and hepatomegaly. The child was endotracheally intubated in the ED and ketamine and rocuronium were used during the procedure. The ED physician contacts you for additional recommendations and admission to the PICU. Your recommend the following NEXT IMPORTANT treatment for the child: A. Begin vasopressin infusion to increase the child’s blood pressure to .100 mm Hg systolic B. Begin epinephrine infusion to increase cardiac contractility, increase blood pressure to .85 mm Hg systolic, and improve central perfusion C. Give an intravenous bolus of 20 mL/kg of normal saline D. Start dexmedetomidine infusion to lower the child’s heart rate to ,100 beats/min Preferred response: B

Rationale The patient is presenting in cardiogenic shock requiring increase in cardiac output that can be obtained by epinephrine use due to its B1 effects on contractility which should lower the heart rate and increase blood pressure without increasing systemic vascular resistance. 2. The above child arrives in the PICU endotracheally intubated with two peripheral large bore IVs infusing the treatment you recommended above, and has the following vital signs: temperature, 38°C (after IV acetaminophen); heart rate, 120 beats per minute; respirations, 25 per minute (intubated on ventilator with Fio2 of 0.50); blood pressure, 95/50 mm Hg; Spo2, 98%. You immediately perform a point-of-care ultrasound (POCUS) of the heart from the apical four chamber view which reveals no tamponade but a left ventricular ejection fraction of 25% and normal proximal coronary arteries. An arterial blood gas is

obtained revealing: pH 7.20; pco2, 35 mm Hg; po2, 90 mm Hg; base excess, 29.0 mmol/L; lactate level, 8.2 mmol/L (normal: 1-2); B type natriuretic peptide level, 4675 pg/mL (normal: ,100 pg/mL). The complete blood count is normal. The child has a positive nasal rapid antigen test for SARS-CoV-2. Over the next 4 hours the child progresses downward with a worsening metabolic acidosis, increasing lactate level (12.4 mmol/L), lower mixed venous saturation level (,40%); anuria and decreased central and peripheral pulses. A discussion with your pediatric cardiac surgeon and interventional cardiologist leads to the need for mechanical circulatory support for this child. The best device choice for this child is: A. Percutaneously inserted Abiomed Impella 2.5 through the right femoral artery B. Thoratec Pediamag Right Ventricular Assist Device (RVAD) C. Veno-venous extracorporeal life support (VV-ECLS) with a double lumen catheter in the right internal jugular vein D. Veno-arterial extracorporeal life support (VA-ECLS) through the right internal jugular vein and carotid artery Preferred response: D

Rationale Only VA ECMO will provide increased systemic cardiac output (Qs) in this patient with cardiogenic shock. Theoretically, the Impella heart pump could provide LV assist in older adolescent but not in a 5-year-old. 3. The child stabilizes on the mechanical circulatory support you chose above but unfortunately after 10 days of aggressive therapy there is no recovery of cardiac function but all other organ function including neurologic status are intact. Your next BEST OPTION for this child’s long term survival is to: A. Continue on the current therapy for an additional 2 weeks hoping for cardiac recovery B. Continue on the current therapy but contact the heart transplant team to begin the process of listing the child status 1A C. Convert the child over to a paracorporeal Berlin Heart Excor Biventricular Assist Device using two 25-mL pumps as a bridge to cardiac transplant D. Speak to the family about the futility in continuing aggressive care and discontinue MCS Preferred response: C

Rationale VA ECMO has stabilized the child with preservation of the child’s other vital organs but this therapy is limited to short term use (despite a few outliers). For the child to be listed and receive a heart transplant, continued support would likely run months. Transitioning the child to a long term VAD is the best option for long term survival (to recovery or transplant). At this time ECLS/VAD are not futile or experimental therapies for this 5-year-old child. 4. A previously well 8-kg, 6-month-old girl is referred from an outside hospital with frequent episodes of new-onset polymorphic ventricular tachycardia. She has been cardioverted several times and commenced on a lidocaine infusion. The episodes are becoming more frequent and associated with significant hypotension. Echocardiogram reveals extensive left ventricular noncompaction with an ejection fraction of 31%. No intracardiac thrombus is reported. A venous saturation of 51% is obtained



CHAPTER 136 Board Review Questions

from an existing right internal jugular central venous catheter. What would be the appropriate subsequent management? A. Cannulate for venoarterial (VA) extracorporeal membrane oxygenation (ECMO) until arrhythmia control is achieved. B. Maximize antiarrhythmic medication and initiate anticoagulation and milrinone infusion. C. Place a left ventricular assist device (LVAD). D. Place on VA ECMO as a bridge to longer-term biventricular assist and heart transplantation. Preferred response: D

Rationale Refractory ventricular tachycardia with clinically significant hemodynamic compromise would necessitate biventricular support. The patient has impaired ventricular function with evidence of inadequate tissue oxygen delivery. The underlying disease process is not amenable to medical therapy alone, and the anticipated duration of mechanical support prior to orthotopic heart transplant would be in the order of weeks to months. 5. A 7-year-old, 22-kg boy is referred to the cardiology clinic with a 5-month history of increasing fatigue, exercise intolerance, anorexia, and dyspnea. His parents comment that his color is gray on minimal exertion. Echocardiography reveals a thin-walled, severely dilated left ventricle with an ejection fraction of 20%. There is moderate mitral regurgitation with an otherwise structurally normal heart and vasculature. A diagnosis of idiopathic dilated cardiomyopathy is made. He is admitted, a central line is placed, and he is started on a milrinone infusion and diuretics. He subsequently develops intermittent unifocal ventricular ectopy. After 24 hours, his parents comment on the improvement in his color, energy levels, and appetite. Listing for heart transplantation is considered. Venous oxygen saturation from his central line is initially 63%. Options for subsequent management include which of the following? A. Discharge to home on milrinone with regular visits with the cardiologist. B. Place on left ventricular assist device (LVAD) as a bridge to transplant. C. Remain in the PICU with current regimen until a donor heart is available. D. Wait for evidence of end-organ dysfunction or worsening symptoms of heart failure before intervening. Preferred response: A

Rationale This is a case of a different disease process in an older child where potential clinical improvement can occur in more than 50% of patients. Management will depend on response to initial therapy. Failure to improve clinically in association with evidence of inadequate tissue oxygen delivery will determine the timing of institution of mechanical support. The child’s size and expected duration of support precludes an adult VAD system or implantation of a short-term device. 6. You are caring for a newborn who underwent an arterial switch operation for d-transposition of the great vessels. Eight hours after surgery, his vital signs are as follows: temperature, 37.8°C; heart rate, 165 beats per minute (sinus rhythm); left atrial pressure, 12 cm H2O; central venous pressure, 15 cm H2O; and blood pressure, 54/31 mm Hg. On physical examination, he

e39

has slightly cold extremities and pulses are 11, breath sounds are clear bilaterally, the abdomen is soft, and the liver edge is palpated 3 cm below the right costal margin. Urine output has been 0.49 mL/kg/h for the past 4 hours. Arterial blood gas values show the following: pH, 7.23; pco2, 37 mm Hg; Pao2, 65 mm Hg; base deficit, 6; and lactic acid, 3.7 mmol/L. Which of the following interventions would potentially be detrimental to this patient? A. Dopamine, 10 µg/kg/min B. Epinephrine, 0.03 µg/kg/min C. Fentanyl, 3 µg/kg/h D. Milrinone, 0.5 µg/kg/min Preferred response: A

Rationale Low cardiac output state (LCOS) is commonly seen after cardiopulmonary bypass and is manifested by oliguria, followed by decreased perfusion, progressive acidosis, and hemodynamic compromise. The goal in management of LCOS is to provide adequate oxygen delivery (Do2) to the tissues, which can be achieved by decreasing oxygen consumption or increasing Do2. Maintaining normothermia or mild hypothermia, providing adequate analgesia and sedation, and sometimes neuromuscular blockade are common interventions that will help decreased oxygen consumption. Phosphodiesterase III inhibitors like milrinone are indicated in the postoperative management of LCOS. Milrinone favors calcium transport into the cell by increasing intracellular cyclic adenosine monophosphate. The increased intracellular calcium enhances the contractile state of the myocyte. Milrinone decreases systemic and pulmonary vascular resistance and improves diastolic relaxation by increasing the rate of calcium reuptake after systole. Although low-dose catecholamines are useful in improving contractility (via their b1-adrenergic effect), high-dose catecholamines (dopamine .5 µg/kg/min, epinephrine .0.05 µg/ kg/min) will increase heart rate and increase systemic vascular resistance, leading to increased myocardial oxygen consumption, decreased cardiac output, and decreased Do2. If optimized medical management is unsuccessful in reverting LCOS, mechanical support would be indicated.

Chapter 29: Echocardiographic Imaging 1. A patient undergoes surgical repair of tetralogy of Fallot with severe pulmonary stenosis. The postoperative transesophageal echocardiogram demonstrates no residual obstruction in the RV outflow track, with severe pulmonary insufficiency, and a hypertrophied right ventricle. On arrival to the ICU, the patient is noted to be cyanotic. Breath sounds are equal with appropriate readings on ventilator monitors. Which of the following would be an ideal early evaluation or intervention for this infant? A. Increase the Fio2 on the ventilator to 100% and hope the cyanosis resolves. B. Obtain an echocardiogram to evaluate for right-to-left shunting at the atrial level. C. Request that the surgeon take the patient back to the operating room immediately. D. Request that an interventionalist take the infant to the catheterization laboratory for device occlusion of a presumed right-to-left shunt. Preferred response: B

e40

S E C T I O N XV   Pediatric Critical Care: Board Review Questions

Rationale Echocardiography is useful (and the modality of choice) in the evaluation of the cyanotic patient, can often be quickly and easily obtained, and gives immediate answers on residual shunts in the postoperative patient. Prior to taking the infant back for revision of the operation, or to the catheterization laboratory for intervention, it is useful to understand the etiology of the cyanosis. An atrial level communication is at times left intentionally to allow a “pop-off” for the stiff right ventricle in the immediate postoperative period. A chest x-ray would be obtained prior to repositioning the ETT rather than assuming malposition in the setting of equal breath sounds. Increasing the Fio2 to 100% is not a longterm solution. 2. Which of the following statements is true regarding echocardiography? A. Echocardiography is limited to available acoustic windows. B. Echocardiography is uniquely a noninvasive modality. C. Echocardiography provides precise measurements of pulmonary arterial pressures. D. Echocardiography can provide histologic evaluation of cardiovascular tissues. Preferred response: A

Rationale Echocardiography is limited to the “windows” where the ultrasound beam can penetrate adequately to produce images. It does not penetrate well through air. This can be a major limitation to echocardiography. The remaining statements are not true. Transesophageal (TEE) and intracardiac echo (ICE) are examples of invasive echocardiography. Echocardiography provides estimates of pulmonary arterial pressures, which can be inaccurate for many different reasons. 3. What is the earliest gestational age that cardiac echocardiography can be used to assess fetal cardiac anatomy reliably? A. 8 weeks B. 14 weeks C. 20 weeks D. 28 weeks Preferred response: C

Rationale Complete anatomic and physiologic assessment can be obtained in the neonate and in the fetus at 20 weeks of gestation. 4. Which of the following is the mainstay of the anatomic diagnosis of congenital heart disease? A. Angiography B. Cardiac catheterization C. Computerized tomographic angiography D. Echocardiography Preferred response: D

Rationale The technique of echocardiography and the practice of echocardiology have changed the practice of pediatric cardiology by largely replacing the use of cardiac catheterization/angiography for the diagnosis of congenital malformations. Combined with the use of prostaglandin for maintaining the patency of the ductus arteriosus, echocardiography has dramatically reduced the need for emergency cardiac catheterization in neonates. Most patients with

congenital heart disease that is detected in the neonatal period can undergo palliative surgery without cardiac catheterization. Most definitive surgical repairs can be performed successfully without the risk of invasive studies. Pulsed, continuous-wave, color, and tissue Doppler have added important capabilities for anatomic and functional assessment. Intraoperative and postoperative management of congenital heart defects has been aided by the addition of transesophageal echocardiography. 5. Right ventricular pressure can be estimated with the use of echocardiography by the simplified Bernoulli equation, where v is peak velocity of which insufficiency jet? A. Pulmonary insufficiency jet 4v B. Pulmonary insufficiency jet 2v C. Tricuspid insufficiency jet 4v2 D. Tricuspid insufficiency jet 2v2 Preferred response: C

Rationale Estimation of right ventricular systolic pressure by Doppler echocardiography is done with use of the maximum velocity (v) of the regurgitant tricuspid jet. The systolic pressure gradient (delta P) between the right ventricle and the right atrium is calculated by using the simplified Bernoulli equation (delta P 5 4v2). 6. Segmental analysis of cardiac disease is classically based on which of the following? A. Great vessel arrangement, ventricular looping, situs B. Situs, great vessel arrangement, ventricular looping C. Situs, ventricular looping, great vessel arrangement D. Ventricular looping, great vessel arrangement, situs Preferred response: C

Rationale Comprehensive analysis of cardiovascular anatomy requires a step-by-step segmental approach. A complete step-by-step approach to cardiac diagnosis includes the diagnosis of atrial situs; identification of the chambers and their interconnections; and systematic assessment of valves, septa, coronaries, systemic and pulmonary veins, and aortic anatomy. Imaging of the thymus and diaphragm is part of the detailed echocardiographic examination. The segmental approach is based on the principle that all aspects of abnormal cardiovascular morphology can be broken down into discrete, mutually exclusive descriptors, allowing unambiguous delineation of any complex congenital malformation. The schema must include information on the presence, position, and connection of each cardiac segment. Classically, three segments have been recognized: atria, ventricles, and great arteries. By describing the anatomic segments and indicating the normality or abnormality of each, a complete description of the cardiac anatomy is possible. It now is possible to code cardiac anatomic abnormalities by segmental analysis. Determination of cardiac position and atrial-visceral situs is a standard portion of the echocardiographic assessment of congenital heart disease and is the foundation of the segmental approach. Atrial situs and atrial morphology are diagnosed together, and four possibilities exist: solitus (normal), inversus, and heterotaxy that may be right atrial isomerism or left atrial isomerism. Description of the connection of the atria and ventricles (i.e., AV connection) requires knowledge of both atrial and ventricular morphology. Four patterns of AV connection exist: concordant (i.e., normal); discordant; univentricular through a single inlet

CHAPTER 136  Board Review Questions

(i.e., tricuspid or mitral atresia), double inlet, or common inlet; and ambiguous (i.e., two ventricles with atrial isomerism). When the morphologic right atrium connects normally to the morphologic right ventricle and the left atrium connects to the left ventricle, AV concordance is present. When this connection is reversed and the morphologic right atrium connects to the morphologic left ventricle, AV connection is discordant and sometimes is referred to as ventricular inversion. Ventriculoarterial connection is the manner in which the great arteries and semilunar valves connect to the ventricular outflow tracts. Normally, the morphologic right ventricle connects to the pulmonary valve and the morphologic left ventricle connects to the aortic valve. Four possibilities exist: concordant (i.e., normal), discordant (i.e., right ventricle to the aorta and left ventricle to the pulmonary trunk), double outlet (usually the right ventricle), and single outlet (i.e., aortic or pulmonary atresia or truncus arteriosus). 7. Ventricular function can be assessed by the shortening fraction. Shortening fraction is calculated by which of the following equations? A. End-diastolic dimension minus end-systolic dimension divided by end-diastolic dimension B. End-diastolic dimension minus end-systolic dimension divided by end-systolic dimension C. End-systolic dimension minus end-diastolic dimension divided by end-systolic dimension D. End-systolic dimension minus end-diastolic dimension divided by end-diastolic dimension Preferred response: A

Rationale Echocardiography is a tomographic anatomic tool, but it also provides dynamic information about cardiac function and structure. Observations about the cardiac walls, including their movement, thickness, and degrees of shortening and thickening, can be extremely useful in determining segmental and global cardiac function. In general, the shortening fraction of the left ventricle should be at least 28% (end-diastolic minus end-systolic divided by end-diastolic dimension), and the walls of the left ventricle should move inward symmetrically. 8. In congenital heart disease, the major application of contrast echocardiography in the postoperative patient is which of the following? A. Assessment of ventricular function B. Assessment of coronary arterial flow C. Detection of residual shunt lesion D. Evaluation of mitral regurgitation Preferred response: C

Rationale An ultrasonic contrast agent is a substance that stabilizes microbubbles in solution, which are large enough to reflect ultrasound but small enough that they disappear rapidly and are physiologically safe. The agent may be as simple as an injection of saline solution into the circulation during two-dimensional echocardiographic imaging or as complex as precision-engineered microbubbles of polysaccharide that dissolve in the circulation after injection. Advances in bubble technology allow imaging of myocardial capillary perfusion. Contrast also can be useful in defining the identity of an imaged structure. For example, a structure under the aortic arch

e41

may be confusing but can be confirmed to be the innominate vein by echocardiographic contrast injection in a left arm vein. In persons with congenital heart disease, the major application of contrast echocardiography is in the postoperative patient with residual shunts or as a means to exclude congenital heart disease. Systemic venous injection of contrast fills the right side of the heart sequentially, and the site of residual right-to-left shunting can be defined. 9. The Ross procedure involves which of the following? A. Autologous autograft in the pulmonary position and mechanical prosthesis in the aortic position B. Autologous autograft in the aortic position and mechanical prosthesis in the pulmonary position C. Mechanical valve in the pulmonary position and autograft in the aortic position D. Placement of a pulmonary autograft in the aortic position Preferred response: D

Rationale The Ross procedure involves the substitution of the diseased aortic valve with the patient’s own pulmonary valve (autograft). A pulmonary homograft is then used to replace the patient’s pulmonary valve. The coronary arteries are translocated to the new aortic valve. Advantages of this procedure include the lack of need for anticoagulation because of the decreased incidence of thromboembolism, as well as growth of the valve as the child grows. One disadvantage is the treatment of one-valve disease with a two-valve replacement.

Chapter 30: Diagnostic and Therapeutic Cardiac Catheterization 1. A 2-week-old full term female with hypoplastic left heart syndrome has delayed convalescence following Norwood procedure with modified Blalock-Taussig shunt and is referred for cardiac catheterization. The patient has baseline hemoglobin of 14 mg/ dL and estimated oxygen consumption of 160 mL/minute/m2. The following saturations were obtained while intubated at a fraction of inspired oxygen of 21%: descending aorta, 88%; left upper pulmonary vein, 98%; mixed venous saturation obtained in the superior vena cava, 61%. What is the patient’s estimated ratio of pulmonary to systemic blood flow (Qp:Qs)? A. 0.4 B. 0.7 C. 1 D. 2.7 E. Cannot calculate with the given data Preferred response: D

Rationale The Fick method uses a patient’s oxygen consumption and oxygen carrying capacity across a vascular bed to calculate blood. In patients with single ventricle physiology, the systemic and pulmonary saturations are identical. For this reason, only three saturations are required for calculating Qp:Qs via the Fick method. As per equations in Table 30.1, the equation to calculate Qp:Qs can be simplified to include only saturations as many of the terms cancel out through division. For this question, Qp:Qs is calculated as: (systemic saturation – mixed venous saturation)  (pulmonary venous saturation – pulmonary artery saturation). Since the systemic and pulmonary arterial saturations are equivalent, (88 2 61)  (98 2 88) 5 2.7. Note that a saturation of 88% in a

e42

S E C T I O N XV   Pediatric Critical Care: Board Review Questions

patient with hypoplastic left heart syndrome after stage I palliation suggests significantly increased Qp:Qs as is demonstrated here. 2. A 12-year-old male with no prior history of cardiac disease is admitted to the pediatric intensive care unit with concerns for shock. He has a history of febrile illness with malaise several weeks prior. The patient is tachycardic and tachypneic with a pale appearance. The first Korotkoff sound is heard intermittently with respiration at a systolic blood pressure of 85 mm Hg and consistently at 72 mm Hg. On exam, the patient has weak pulses throughout, jugular venous distension, and distant heart sounds. The electrocardiogram demonstrates sinus tachycardia with beat-to-beat variation in the QRS amplitude. An echocardiogram shows a large pericardial effusion with respiratory variation in mitral inflow by pulse wave Doppler of .20%. Given concern for tamponade, the patient is prepared for a pericardiocentesis. For this patient, select the finding that is NOT representative of tamponade physiology: A. Hypotension B. Jugular venous distension C. Large pericardial effusion D. Pulsus paradoxus E. All of the above are required for tamponade physiology Preferred response: C

Rationale If the pericardial effusion accumulates slowly, patients can remain hemodynamically stable and potentially asymptomatic even with a large pericardial effusion. Tamponade is a clinical diagnosis that is characterized by Beck’s triad: distant heart sounds, hypotension, and jugular venous distension. Patients with tamponade also have tachycardia, tachypnea, narrow pulse pressures, and pulsus paradoxus. The ECG and echocardiogram changes may be suggestive of a diagnosis, but must be accompanied by the clinical signs and symptoms to meet a diagnosis. 3. A 2-year-old admitted with tachypnea is referred for catheterization after an echocardiogram demonstrated right heart enlargement and partial anomalous pulmonary venous return. The patient’s baseline hemoglobin is 12.5 mg/dL. The patient is sedated and intubated with fraction of inspired oxygen of 21%. Estimated oxygen consumption (Vo2) is 150 mL/min/ m2 and the following saturations are obtained: Innominate Vein

High Right Pulmonary Pulmonary Descending SVC Atrium Artery Vein Aorta

71%

70%

85%

86%

99%

99%

Calculate the pulmonary blood flow for this patient. A. 2.5 L/min/m2 B. 4.1 L/min/m2 C. 6.8 L/min/m2 D. 10 L/min/m2 E. Unable to calculate with the given information Preferred response: C

Rationale As per Table 30.1, pulmonary blood flow can be calculated as Vo2/10 3 [1.36 3 Hgb 3 (pulmonary vein saturation – pulmonary artery saturation)]. For the question, 150 / [10 3 1.36 3 12.5 3 (0.9920.86)] 5 150/(13.6 3 12.5 3 0.13) 5 6.8 L/min/m2.

4. A 10-month-old male is admitted for evaluation of persistent desaturations requiring nasal cannula support. He has a large ventricular septal defect. Despite poor outpatient follow-up, his growth has been normal. Infectious work up is unremarkable. He is referred for catheterization to more clearly delineate the source of desaturations and evaluate pulmonary vascular resistance prior to surgical referral. During the catheterization, the pulmonary venous saturation was 95% and pulmonary vascular resistance was 7 Wood units 3 m2 at fraction of inspired oxygen (Fio2) 0.21. At 1.0 Fio2 and 20 ppm inhaled nitric oxide, the pulmonary vascular resistance decreased to 4 Wood units 3 m2. Per AHA guidelines, what would be the most appropriate step in cardiac management? A. Initiate pulmonary vasodilator therapy with sildenafil and repeat catheterization prior to surgical repair. B. Initiate pulmonary vasodilator therapy for symptom management; however, he is not a surgical candidate due to pulmonary hypertension. C. Manage with diuretics as pulmonary vasodilator therapy is contraindicated. D. Refer for surgery, consider fenestrated patch repair and anticipate need for postoperative pulmonary hypertension therapy. Preferred response: D

Rationale Per recent AHA guidelines by Abman et al. entitled Pediatric Pulmonary Hypertension, patients younger than 1 year of age with simple shunting lesions and concern for increased pulmonary vascular resistance should undergo catheterization with acute vasodilator testing. For patients with indexed pulmonary vascular resistance greater than 6 Wood units 3 m2, vasodilator testing is recommended. If the pulmonary vascular resistance improves with testing, then it would be reasonable to refer the patient for surgical repair. 5. A 6-month-old infant is 4 days into the postoperative period after tetralogy of Fallot repair. There are persisting signs of right heart failure with hepatomegaly and ascites. An echocardiogram has inadequate windows to visualize the branch pulmonary arteries. What is the most appropriate management? A. Communicate to the cardiac surgeon and interventional cardiologist and advocate for a diagnostic catheterization and possible pulmonary artery intervention if significant residual stenotic lesions are diagnosed. B. Communicate to the cardiac surgeon and interventional cardiologist and advocate for a diagnostic catheterization and possible ASD closure. C. Continue to medically manage with the goal of spontaneous right heart recovery. D. Transfer the infant to the catheterization laboratory for pulmonary artery angiography. Preferred response: A

Rationale Communication is essential. The cardiac surgeon on a recent surgical patient will know the child’s anatomy intimately and also have an awareness of possible problems. The interventional cardiologist can advise on what is technically feasible. As the ICU attending, involving all providers is crucial for maximizing outcomes. Arranging a catheterization without consultation will result in a poorly planned and inefficient catheterization that may lead to unnecessary angiograms and worse outcomes. Right heart failure

CHAPTER 136  Board Review Questions

can spontaneously improve early after cardiac repair, but if recovery is prolonged 72 hours longer than anticipated, then strong consideration for residual cardiac defects is very important. There are now reports on safe early postoperative interventions, thus a catheterization for merely diagnostic purposes would be incorrect. Closing an ASD in the setting of right heart failure would worsen cardiac output and elevate central venous pressure. In fact if there is an intact atrial septum, one of the interventions to consider is creating an ASD. 6. When should cardiac surgical patients requiring venoarterial (VA) extracorporeal membrane oxygenation (ECMO) in the postoperative period be considered for cardiac catheterization? A. If there is left ventricle (LV) dysfunction, left atrial (LA) dilation, and pulmonary edema on chest radiograph B. Never; cardiac catheterization due to risks of the procedure and transportation is contraindicated when cannulated for VA ECMO C. Only as a last resort and should not be considered unless unable to be decannulated after 2 weeks D. Should not be considered unless unable to be decannulated after 1 week Preferred response: A

Rationale Left atrial decompression can be safely achieved in the catheterization laboratory and is important in the setting of LV dysfunction with distention. Decompression of the LA with interventional techniques (placing a cannula in the LA, creating an ASD) alleviates this problem. The other option is placement of a LA cannula by cardiac surgery, and this may be preferable in a small child with an open chest. Safe transport is challenging but should not be the deciding factor. Catheterization on VA ECMO requires a high degree of teamwork, particularly with transport to ensure that cannula position remains stable. Safe transportation is a reality in many centers. Procedural risks even with anticoagulation are minimal. It is important to have a high index of suspicion for residual lesions in postoperative cardiac surgery patients who cannot be weaned from ECMO. Unless there is clear evidence for myocardial recovery, consideration for catheterization by the ICU attending and direct consultation with cardiac surgery and interventional cardiology should occur early and within 72 hours of the start of the unexpected course. Interventions can be performed on VA ECMO and would be indicated if any residual lesions are diagnosed. 7. For an early postoperative patient who has persistent low cardiac output and is unable to be weaned from the ventilator, cardiac catheterization is characterized by which of the following? A. Carries a significantly increased risk of serious adverse events when compared with late postoperative catheterization B. May yield important physiologic and anatomic information that often leads to reintervention C. Should only be performed on a patient supported with extracorporeal membrane oxygenation (ECMO) D. Should be deferred until 6 weeks following surgery Preferred response: B

Rationale Cardiac catheterization should always be considered in the early postoperative period when a patient exhibits a persistent low output

e43

state or failure to wean from mechanical ventilation. Early postoperative catheterization often yields important physiologic and anatomic information and frequently leads to reintervention. Whereas elective catheter-based interventions are typically deferred until 6 weeks following surgery, the risk-benefit ratio favors early postoperative catheterization for the patient who does not follow a projected course. ECMO is used in postoperative patients who fail to separate from bypass or have progressive low output. These patients often require early postoperative catheterization to assess unfavorable physiology. In addition, ECMO support is often used to safely support the circulation in patients for whom a high-risk catheter intervention is planned. In the hands of an experienced team (i.e., interventional cardiologist, intensivist, anesthesia personnel, and nursing staff), serious adverse event rates are similar for early or late postoperative studies. 8. A balloon atrial septostomy in the preoperative patient with d-TGA is characterized by which of the following? A. Increases left atrial hypertension B. May increase the risk for pulmonary hypertension prior to cardiac surgery C. May be performed under fluoroscopic or echocardiographic guidance D. Worsens cyanosis by facilitating mixing at the atrial level Preferred response: C

Rationale Balloon atrial septostomy should be considered in any newborn with a confirmed diagnosis of d-TGA who demonstrates inadequate mixing at the atrial level. Signs of inadequate mixing include cyanosis, left atrial hypertension, and pulmonary hypertension. Balloon atrial septostomy improves cyanosis by facilitating mixing at the atrial level, reduces left atrial hypertension, and may reduce the risk for pulmonary hypertension prior to cardiac surgery. Balloon atrial septostomy may be performed in the catheterization laboratory (under fluoroscopic guidance) or in the intensive care unit (under echocardiographic guidance) at the discretion of the intensivist and cardiac interventionist.

Chapter 31: Pharmacology of the Cardiovascular System 1. A 4-month-old, 4 kg male with Trisomy 21, complete atrioventricular septal defect (AVSD), congestive heart failure, and failure to thrive is admitted to the pediatric intensive care unit (PICU) following surgical repair. The intraoperative course was complicated by a brief cardiopulmonary arrest, attributed to a pulmonary hypertensive crisis on separation from cardiopulmonary bypass. Spontaneous circulation was restored following 2 minutes of cardiopulmonary resuscitation. Milrinone was administered in 2 divided doses of 25 mg, each over 20 minutes, and infusions of epinephrine, 0.1 mg/kg/min, and milrinone, 0.5 mg/kg/min, were initiated. Nitric oxide at 10 ppm was added, and the patient was transferred to the PICU for further management. Vital signs on arrival are remarkable for a heart rate of 156 beats per minute and arterial blood pressure of 70/28 mm Hg. Oxygen saturation is 100% on Fio2 1.0 and full ventilator support. The central venous pressure is 4 mm Hg. On physical examination the infant is intubated, sedated, and medically paralyzed. His skin is warm and pink, with

e44

S E C T I O N XV   Pediatric Critical Care: Board Review Questions

pulses 11 to 21 in all 4 extremities and capillary refill time of 3 seconds. A soft diastolic rumble is present on cardiac auscultation. The lungs are clear. The liver edge is palpable 2 cm below the right costal margin. Pupils are 5 mm and sluggishly reactive to light bilaterally. The neurologic exam is otherwise obscured by neuromuscular blockade. The postoperative chest radiograph shows a well-positioned endotracheal tube and internal jugular vascular catheter, mild cardiomegaly and clear, well inflated lungs. Laboratory data reveal pH 7.38; Paco2, 36 mm Hg; Pao2, 223 mm Hg; HCO3, 22 mmol/L, base excess, 23 mmol/L; ionized calcium, 1.6 mmol/L (reference range 1.2–1.3 mmol/L); lactate, 1.9 mmol/L (reference range ,2 mmol/L); creatinine 1.6 mg/dL (reference range 0.2–0.6 mg/dL); hemoglobin 12 g/dL (reference range 9.0–13 g/dL) Ninety minutes later, the patient’s arterial blood pressure is 50/20 mm Hg, the heart rate is 182 per minute, and there has been only 1.5 mL of concentrated urine output from the urinary catheter since PICU admission. The bedside monitor shows sinus tachycardia and the physical exam is otherwise unchanged. After re-zeroing the arterial catheter system, the repeat blood pressure is the same. The most appropriate initial intervention is to: A. Confirm the blood pressure via sphygmomanometer B. Consult Cardiology for recommendations C. Increase the epinephrine infusion to 1 mg/kg/min D. Infuse 40 mL Ringer’s lactate over 5 minutes E. Start a vasopressin infusion 0.0008 U/kg/min Preferred response: D

Rationale An infusion of 10 mL/kg of isotonic fluid should be initiated and the response closely monitored. The observation that the patient’s perfusion appeared unchanged is due to exaggerated peripheral vasodilation that can occur with milrinone, along with a generalized sunburned appearance. This can be mistaken for adequate perfusion if consideration is not given to milrinone as the culprit. Although following blood pressure cuff readings may be helpful in certain scenarios, there is no reason to discount the invasive arterial blood pressure reading once the system has been interrogated and no problems have been identified. Increasing the epinephrine dose to 1 mg/kg/minute would be very poorly tolerated in this scenario. The patient is already tachycardic, and the risk of dysrhythmias is a known complication following repair of complete AVSD. In addition, the increased afterload to the left ventricle following closure of the ventricular septal defect and correction of mitral insufficiency would only be compounded by the increase in systemic vascular resistance (SVR) from this dose of epinephrine. For the same reason, adding a vasopressin infusion would not be the best choice. Although the oliguria in this scenario is likely due to a decrease in SVR, the resulting “relative” hypovolemia indicates that gentle fluid repletion should be initiated to optimize organ perfusion prior to increasing vascular tone with a vasoconstrictor. 2. While alerting the cardiologists to the change in the patient’s status is an important priority, waiting for their input will delay treatment of a problem that requires immediate attention. The effects of cardiopulmonary bypass and the associated inflammatory response, plus the impact of intraoperative events, combined with the major change in circulatory

physiology and potential for adverse medication effects must all be considered and quickly addressed in order to ensure optimal outcomes. The most likely reason for the hypotension is: A. Anaphylaxis from perioperative, prophylactic cefazolin B. Cumulative effect of the milrinone loading dose and infusion C. Equipment error D. Septic shock due to a urinary tract infection Preferred response: B

Rationale This patient’s presentation on admission to the PICU with widened pulse pressure with mild diastolic hypotension and prolonged capillary refill time despite the appearance of adequate perfusion is consistent with the peripheral vasodilation that can be seen in some patients on milrinone infusions, especially following a full loading dose. The response may be exaggerated in patients with mild acute kidney injury (such as this one) due to delayed drug clearance and those who are relatively volume depleted as was this patient. In the scenario presented here, renal hypoperfusion during cardiopulmonary bypass compounded by the brief cardiopulmonary arrest led to renal insufficiency. Approximately 85% of milrinone is cleared by the kidney. This property of the drug, along with its relatively long elimination half-life may result in accumulation of the medication, the consequences of which may persist for several hours. The need for dose adjustment should be evaluated for all medications whose effects may be intensified or prolonged by delayed renal clearance (e.g., certain sedatives and neuromuscular blocking agents). 3. Once the child’s hemodynamic status improves, which of the following is most likely to prevent recurrence of the hypotension? A. Decreasing the epinephrine to 0.05 mg/kg/min B. Decreasing the milrinone to 0.25 mg/kg/min C. Increasing the intravenous fluids to twice maintenance D. Increasing the nitric oxide to 20 ppm E. Increasing the dose of neuromuscular blocking agent Preferred response: B

Rationale Continuing the milrinone infusion is the best action, albeit at a lower rate, due to the patient’s history of congestive heart failure and pulmonary hypertension, along with the change in loading conditions following closure of the ventricular septum and recent cardiopulmonary arrest. In addition, there is evidence for improved outcomes in infants with postoperative cardiac dysfunction with administration of milrinone in the early postoperative period. Increasing the intravenous fluids to twice maintenance in this patient with kidney injury will likely do little to prevent the hypotension and instead worsen postoperative anasarca, while also increasing the workload on the heart. Likewise, increasing the nitric oxide, decreasing the epinephrine infusion rate, or increasing the neuromuscular blockade will not prevent recurrence of the hypotension. Following initial fluid administration and stabilization of hemodynamics, gentle up-titration of the epinephrine infusion may then be indicated to achieve optimal perfusion and to limit the number of fluid boluses.

CHAPTER 136  Board Review Questions

4. A 4-month-old male with Down syndrome is admitted to the PICU for postoperative care following patch repair of a large ventricular septal defect (VSD). He was managed preoperatively for congestive heart failure with furosemide and enalapril. His vital signs on admission to the PICU are significant for a temperature of 38.2°C; heart rate, 170 beats per minute; blood pressure, 86/38 mm Hg via a right radial arterial catheter (mean arterial pressure, 54 mm Hg); and oxygen saturation of 89% by pulse oximetry (with a probe on his great toe). On exam, his extremities are mottled and cool distally, with a capillary refill of 2.5 seconds. A bedside arterial blood gas suggests adequate oxygenation and ventilation. The arterial lactate is 6 mmol/L. You observe only a marginal change in his perfusion after three fluid boluses of 10 mL/kg. The central venous pressure is now 14 mm Hg. What is the most appropriate next step in his management? A. Administer an intravenous (IV) milrinone bolus of 50 µg/ kg over 15 minutes followed by a continuous infusion of milrinone at 0.5 µg/kg/minute. B. Continue to bolus with fluids, but change to albumin for more effective intravascular volume repletion. C. Start epinephrine at 0.5 µg/kg/minute, titrating to achieve a goal mean arterial pressure of 65 mm Hg. D. Start dopamine at 15 µg/kg/minute, and overdrive pace to ensure atrioventricular synchrony. Preferred response: A

Rationale This patient is exhibiting signs of postoperative, decompensated cardiogenic shock, with tachycardia, cool and mottled extremities, prolonged capillary refill, and an elevated lactate, all of which indicate inadequate cardiac output. The decreased saturation via pulse oximetry, while the arterial blood gas demonstrates adequate oxygenation, is a function of distal vasoconstriction, which impacts the ability of the pulse oximeter probe to provide an accurate reading. Fluid resuscitation was attempted to augment preload to optimize this patient’s ventricular filling (to take advantage of the Starling relationship), but it did not restore perfusion because there is significant myocardial dysfunction. Therefore the best intervention is to start milrinone, which provides both inotropy and afterload reduction, without significantly increasing myocardial oxygen demand. If a patient’s blood pressure and hemodynamics allow, a loading dose is given before initiating the infusion. A catecholamine such as epinephrine, as a sole agent and initiated at the dose provided in the scenario, would not be the best option for several reasons. First, the patient is already tachycardic, and epinephrine at 0.5 µg/kg/minute may increase the heart rate further and increase myocardial oxygen demand in a heart whose function is already impaired. Escalating this patient’s heart rate would also decrease the diastolic interval for ventricular filling, thus compromising cardiac output even further. In addition, patients who undergo repair of large ventricular septal defects are at increased risk for developing junctional ectopic tachycardia (JET), a tachyarrhythmia unique to the postoperative state in children, in which the loss of atrioventricular synchrony compromises cardiac output. Patients in whom this rhythm is poorly tolerated may require treatment with antiarrhythmics and active cooling if conservative measures such as avoiding fever and minimizing catecholamine-induced increases in adrenergic tone fail. Response B would not be the most appropriate next step because the central venous pressure is already elevated due to ventricular dysfunction, and rather than augmenting cardiac output,

e45

additional fluid boluses would likely produce pulmonary edema. High-dose dopamine would not be the best choice for similar reasons as those stated for epinephrine. In addition, at a dose of 15 µg/kg/minute, dopamine exhibits strong a-adrenergic (vasoconstrictive) properties, which would increase afterload and compromise distal perfusion even further. The consequences of increasing heart rate with overdrive pacing were detailed earlier. 5. Norepinephrine ___________ the heart rate. This action is mediated by ____________. Therefore ____________ would be an effective agent for reversing this response. A. Increases, the hypothalamus, isoproterenol B. Decreases, a reflex response, atropine C. Decreases, a vagal response, isoproterenol D. Has no effect on, the sinus node, ventricular pacing Preferred response: B

Rationale Norepinephrine exhibits strong a-adrenergic properties at doses used to reverse vasodilatory (hyperdynamic) shock. The resulting increase in vascular tone triggers a compensatory vagally mediated reflex slowing of the heart, which atropine will reverse via a muscarinic receptor blockade. Neither the hypothalamus nor the sinus node has been implicated. 6. You have decided to add an epinephrine infusion to the management of an 11-year-old girl with myocarditis. Your goal is to improve cardiac contractility via epinephrine’s effect on b receptors. A medical student on call with you asks why epinephrine is more selective for b receptors than norepinephrine. How do you respond to this question? A. Because of an increased number of hydroxyl groups on the phenylethylamine core B. Because of a decrease in size of the substituent on the amino group C. Because of an increase in size of the substituent on the amino group D. Because of a lack of hydroxyl group at position 4 on the phenylethylamine core Preferred response: C

Rationale Increasing the size of the substituent on the amino group enhances b selectivity, while decreasing the size favors a selectivity. Hydroxyl groups are the same in terms of number and location for all catecholamines. 7. Catecholamines and phosphodiesterase III (PDE III) inhibitors share a common mechanism in that both groups of agents do which of the following? A. Decrease intracellular calcium in the vascular smooth muscle B. Decrease the activity of protein kinase A C. Increase the activity of adenylate cyclase D. Increase the levels of cyclic adenosine monophosphate (cAMP) in the cardiac myocyte Preferred response: D

Rationale Catecholamines vary in terms of their effect on intracellular calcium in the vascular smooth muscle. PDE III inhibitors increase activity of adenylate cyclase. Both groups of drugs increase levels of the second messenger, cAMP.

e46

S E C T I O N XV   Pediatric Critical Care: Board Review Questions

8. You are taking care of a 6-month-old boy in the immediate postoperative period of a ventricular septal defect repair. The infant is normotensive but has decreased perfusion. You wish to add an inotropic agent. Which statement best describes an inotropic agent? A. Inotropic agents primarily increase blood pressure. B. Inotropic agents primarily increase systemic vascular resistance. C. Inotropic agents primarily increase stroke work. D. Inotropic agents primarily decrease myocardial oxygen consumption. Preferred response: C

Rationale Inotropic agents increase stroke work at a given preload and afterload; they do not necessarily increase blood pressure. Vasopressor agents increase blood pressure by increasing systemic vascular resistance. Inotropic agents may increase or decrease myocardial oxygen consumption, depending on the clinical scenario. 9. What is the result of an increase in cAMP in vascular smooth muscle? A. Inhibition of uptake of calcium by sarcoplasmic reticulum with resultant vasoconstriction B. Inhibition of uptake of calcium by sarcoplasmic reticulum with resultant vasodilation C. Uptake of calcium by sarcoplasmic reticulum with resultant vasoconstriction D. Uptake of calcium by sarcoplasmic reticulum with resultant vasodilation Preferred response: D

Rationale In vascular smooth muscle, both b1 and b2 receptors are present, although b2 receptors predominate. The b2 receptor is coupled to Gs; therefore activation of b2 receptors promotes formation of cAMP. The resulting activation of cAMP-dependent protein kinase in vascular smooth muscle, however, stimulates pumps that remove calcium from the cytosol and also promotes calcium uptake by the sarcoplasmic reticulum. As cytosolic calcium concentration decreases, smooth muscle relaxes and the blood vessel dilates. Adrenergic receptors also have been demonstrated on the endothelium and are capable of producing relaxation of the vessel. The exact mechanism involved, including the role of nitric oxide and the subtype of b receptors involved, remain under investigation. 1 0. What is the result of an increase in cAMP in myocyte? A. Closing of voltage-gated calcium channels with a decrease in inotropy B. Opening of voltage-gated calcium channels with an increase in inotropy C. Uptake of calcium by the sarcoplasmic reticulum with a resultant decrease in the force of the next contraction D. Uptake of calcium by the sarcoplasmic reticulum with a decrease in lusitropy Preferred response: B

Rationale Adrenergic receptors are typically coupled to one of three types of G proteins: Gs, Gi, or Gq. Gs proteins produce an increase in adenylate cyclase activity, whereas Gi proteins promote a decrease in adenylate cyclase activity. Gq protein receptors stimulate

phospholipase C to generate diacylglycerol and inositol 1,4,5- triphosphate (IP3). Myocardial b1-adrenergic receptors are associated with Gs. When an agonist agent engages this receptor type, the result is enhanced activity of adenylate cyclase and a rise in the concentration of cAMP, which activates protein kinase A (PKA). PKA in turn phosphorylates voltage-dependent calcium channels, increasing the fraction of channels that can be open and the probability that these channels are open, producing an increase in intracellular calcium concentration. Calcium then binds to troponin C, allowing for actin-myosin cross-bridge formation and sarcomere contraction. Also, PKA phosphorylates phospholamban, relieving the disinhibitory effect of the unphosphorylated form on calcium channels in the sarcoplasmic reticulum. The accumulation of calcium by the sarcoplasmic reticulum is thus enhanced, increasing the rate of sarcomere relaxation (lusitropy) and subsequently increasing the amount of calcium available for the next contraction. This leads to both enhanced contractility and active diastolic relaxation. 11. Activation of V1 receptors by arginine vasopressin (AVP) results in which of the following? A. Stimulation of Gq protein and activation of phospholipase C B. Stimulation of Gs protein and an increase in cAMP C. Stimulation of Gs protein and a decrease in cAMP D. Stimulation of Gi protein and an increase in cGMP Preferred response: A

Rationale Vasopressin receptors belong to the family of G protein–coupled receptors. V1 receptors are coupled to Gq, and V2 receptors are coupled to Gs. When vasopressin binds to the V1 receptor, phospholipase C (PLC) is activated with the eventual production of inositol 1,4,5-triphosphate (IP3) and 1,2 diacylglycerol (1,2 DG). These molecules increase the release of calcium from the endoplasmic reticulum and increase the entry of calcium through gated channels. The increase in intracellular calcium leads to an increase in the activity of myosin light chain kinase. This kinase acts on myosin to increase the number of actin-myosin cross-bridges, enhancing contraction of the myocyte. Of note, vasopressin has been shown to produce vasoconstriction in the skin, skeletal muscle, and fat while producing vasodilation in the renal, pulmonary, and cerebral vasculature. This effect may be mediated though nitric oxide or may be a function of the isoform of adenyl cyclase with which the receptor is coupled. AVP also has been shown to increase the pressor effects of catecholamines, although in two different vascular smooth muscle cell lines, AVP had opposing effects on isoproterenol-induced activation of adenyl cyclase. 12. Which of the following drugs has the least effect on increasing myocardial oxygen consumption? A. Dopamine B. Dobutamine C. Epinephrine D. Milrinone Preferred response: D

Rationale With the exception of the bipyridines (amrinone, milrinone), all inotropes increase myocardial oxygen consumption because they increase myocardial work.

CHAPTER 136  Board Review Questions

e47

13. Which of the following drugs worsens intrapulmonary shunt in acute respiratory distress syndrome? A. Atropine B. Digoxin C. Dopamine D. Milrinone Preferred response: C

16. An epinephrine infusion may result in which of the following adverse effects? A. Hypercalcemia B. Hyperkalemia C. Hypocalcemia D. Hypokalemia Preferred response: D

Rationale Dopamine depresses the ventilatory response to hypoxemia and hypercarbia by as much as 60%. Dopamine (and other b agonists) decreases Pao2 by interfering with hypoxic vasoconstriction. In one study, dopamine increased intrapulmonary shunting in patients with acute respiratory distress syndrome from 27% to 40%.

Rationale Hypokalemia is produced by an epinephrine infusion because of stimulation of b2-adrenergic receptors, which are linked to sodiumpotassium-adenosine triphosphatase (ATPase) located in skeletal muscle. Infusion of 0.1 µg/kg/min lowered serum potassium by 0.8 mEq/L. Hyperglycemia results from b-adrenergic–mediated suppression of insulin release.

14. Which of the following drugs may cause a decrease in heart rate? A. Dopamine B. Epinephrine C. Isoproterenol D. Norepinephrine Preferred response: D

17. Which of the following adverse effects is the most likely result of an isoproterenol infusion? A. Bradycardia B. Bronchoconstriction C. Decrease in myocardial oxygen consumption D. Decrease in serum theophylline concentrations Preferred response: D

Rationale Norepinephrine produces increases in systemic vascular resistance (SVR), arterial blood pressure, and urine flow. It is most valuable in the context of tachycardia because infusion of the drug does not produce significant elevation of heart rate and may even lower heart rate through reflex mechanisms. In a study of adults with abdominal sepsis, norepinephrine infusion was associated with increases in systemic blood pressure and SVR. Stroke volume increased as the heart rate declined. The cardiac index did not change, although creatinine clearance increased substantially. Norepinephrine has been shown to improve right ventricular performance in adults with hyperdynamic septic shock.

Rationale Isoproterenol enhances cardiac contractility and cardiac rate. Peripheral vasodilation produces a decrease in systemic vascular resistance (SVR), augmenting the direct chronotropic action of the drug. Significant tachycardia ensues. Systolic blood pressure increases, while mean and diastolic pressures decrease. If normal prior to infusion of isoproterenol, mesenteric and renal perfusions decrease; however, if the subject was in shock, then the increase in cardiac output associated with isoproterenol administration may result in an increase in blood flow to these tissues. Isoproterenol increases myocardial demand for oxygen and decreases supply by reducing diastolic coronary filling. If intravascular fluid of the patient is depleted, hypotension may complicate initiation of isoproterenol infusion. Pulmonary bronchial and vascular bed b2-adrenergic receptors produce bronchodilation and pulmonary vasodilation, respectively. Adverse effects associated with isoproterenol include fear, anxiety, restlessness, insomnia, and blurred vision. Other effects may include headache, dizziness, tinnitus, sweating, flushing, pallor, tremor, nausea, vomiting, and asthenia. Cardiovascular effects may include ventricular tachycardia and other ventricular dysrhythmias that may be life threatening. Isoproterenol may cause hypertension and also can cause severe hypotension. Isoproterenol decreases serum theophylline concentrations during concomitant therapy of status asthmaticus, and thus it may be necessary to increase theophylline dosage when isoproterenol therapy is initiated and reduce theophylline dosage when isoproterenol is discontinued.

15. Which of the following statements is true regarding the origin of these pharmacologic agents? A. Arginine vasopressin is synthesized in the posterior pituitary. B. Dobutamine is synthesized in the adrenal medulla from norepinephrine. C. Dopamine is the immediate precursor of norepinephrine in the adrenal medulla. D. Norepinephrine is synthesized in the adrenal medulla from epinephrine. Preferred response: C

Rationale Dopamine is a central neurotransmitter. It also is found in the sympathetic nerve terminals and in the adrenal medulla, where it is the immediate precursor of norepinephrine. Dopamine is hydroxylated at the b-carbon to produce norepinephrine, the principal neurotransmitter of the sympathetic nervous system. Arginine vasopressin is a nonapeptide hormone synthesized in the supraoptic and paraventricular nuclei of the hypothalamus. Dobutamine is a synthetic catecholamine. Isoproterenol is the synthetic N-isopropyl derivative of norepinephrine.

18. Which of the following is the major method of metabolism/ elimination of milrinone? A. Hepatic glucuronidation B. Metabolism by catechol O-methyltransferase (COMT) C. Metabolism by N-acetyltransferase D. Renal excretion Preferred response: D

e48

S E C T I O N XV



Pediatric Critical Care: Board Review Questions

Rationale Milrinone is approximately 70% bound to plasma proteins, with approximately 85% renal elimination. Hepatic glucuronidation accounts for a minor elimination pathway. Both renal dysfunction and congestive heart failure affect the elimination profile of milrinone, extending the elimination half-life to approximately 2 hours. Amrinone is metabolized by N-acetyltransferase. Epinephrine is metabolized by COMT to metanephrine in the liver and kidneys or deaminated via the action of MAO. Dopamine and dobutamine are metabolized by COMT and MAO. 1 9. What is the mechanism of action for digitalis? A. Closure of voltage-gated calcium channels B. Closure of K channels C. Inhibition of sodium/potassium (Na/K)-ATPase D. Opening of voltage-gated calcium channels Preferred response: C

Rationale Glycosides bind to and inhibit Na/K-ATPase. Binding of digoxin to ATPase is affected by serum potassium. Hyperkalemia depresses digoxin binding, whereas hypokalemia has the opposite effect, accounting in part for potentiation of digoxin-induced dysrhythmias during hypokalemia. Inhibition of ATPase produces an increase in intracellular calcium and enhances the inotropic state of the myocardium. 20. Which of the following abnormalities potentiates digitalis toxicity? A. Hypercalcemia B. Hyperkalemia C. Hypocalcemia D. Hypokalemia Preferred response: D

Rationale Digitalis toxicity is made more likely by factors that increase myocardial irritability, such as myocarditis, ischemia, hypoxemia, or catecholamine support. Hypokalemia and alkalosis also potentiate digoxin-induced dysrhythmias. Treatment of digoxin toxicity involves supportive treatment and correction of electrolyte disturbances. Specific pharmacologic support (via atropine, lidocaine, phenytoin, or magnesium sulfate) may be necessary (although frequently unsuccessful), and in life-threatening circumstances, treatment with digoxin-specific Fab antibody fragments is indicated.

Chapter 32: Cardiopulmonary Interaction 1. A child is receiving positive pressure ventilation for the acute respiratory distress syndrome. The child develops a large pleural effusion. For a given airway pressure, which of the following statements is true following the development of the effusion: A. End expiratory lung volume is increased B. Pleural pressure is increased C. The transpulmonary pressure is increased D. Tidal volume is unaffected Preferred response: B

Rationale For a given airway pressure, as chest wall elastance increases, pleural pressure rises and as a result the transpulmonary pressure

decreases throughout the respiratory cycle, decreasing tidal volume and end expiratory lung volume. 2. High airway pressure used for the management of hypoxic respiratory failure due to the acute respiratory distress syndrome may cause right ventricular stroke volume to decrease. Which of the following indicates the mechanism for decreased right heart output in this setting: A. Enlarged right ventricle with a flattened interventricular septum throughout the cardiac cycle B. Increasing arterial to end tidal CO2 gradient C. Lactic acidosis associated with hypotension D. Systolic pressure variation or pulse pressure variation Preferred response: A

Rationale High airway pressure may adversely affect right ventricular (RV) stroke volume and output due to a decrease in RV preload, afterload, or a combination of both. Lactic acidosis and hypotension are consistent with a shock state but do not indicate the mechanisms responsible. An increase in the arterial to end tidal CO2 gradient indicates an increase in wasted ventilation, which may be due to a decrease in pulmonary perfusion. However, if so, it does not indicate the mechanism. Systolic pressure variation is due to a decrease in RV stroke volume, which may result from altered RV loading conditions. An enlarged RV in association with elevated RV afterload (systolic flattening of the ventricular septum) would be consistent with positive pressure ventilation (PPV) increasing RV afterload and precipitating cor pulmonale. If the predominant effect of PPV was on pleural pressure and a decrease in systemic venous return, the RV would have decreased volumes throughout the cardiac cycle, and the ventricular septum would demonstrate normal position and orientation throughout the cardiac cycle. 3. Which of the following does not affect the mean systemic pressure (Pms): A. Venomotor tone B. Systemic arterial blood pressure C. Intravascular volume D. Neurohormonal activity Preferred response: B

Rationale The determinants of the Pms are intravascular volume and vascular capacitance (compliance); an alteration in intravascular volume or venomotor tone (affecting vascular capacitance) will alter Pms. Similarly, altered neurohormonal activity will indirectly impact these factors as well. Blood pressure does not impact Pms; cardiac output is responsible for filling the systemic venous reservoirs. Blood pressure is not responsible for driving blood back to the heart. 4. Why would positive pressure ventilation be expected to increase left ventricular stroke volume and cardiac output in patients with left ventricular systolic dysfunction? A. In expiration, positive pressure ventilation propels forward coronary blood flow. B. In patients with systolic ventricular dysfunction, the proportion of lung units with zone III conditions (pulmonary venous pressure . alveolar pressure) exceeds that of the normal circulation. C. Positive airway pressure reduces left ventricular afterload. D. Positive airway pressure raises transmural left atrial pressure. Preferred response: C



CHAPTER 136 Board Review Questions

Rationale Positive pressure ventilation raises juxtacardiac pressure. This reduces the left ventricular wall tension needed to generate systolic pressure within the left ventricle sufficient to open the aortic valve. In effect, positive juxtacardiac pressure reduces afterload to the left heart. Positive pressure ventilation also reduces demand for cardiac output, but this does not, itself, increase cardiac output. Positive airway pressure would not be expected to propel forward coronary blood flow. There would be an increase in zone III lung units when pulmonary venous pressure is elevated, and this might blunt the rise in pulmonary vascular resistance attributable to alveolar capillary compression. Overall, a rise in juxtacardiac pressure would be expected to reduce transmural atrial pressure. 5. Which of the following mechanisms mediates the positive pressure ventilation (PPV)-induced rise in right ventricle (RV) afterload? A. A compensatory increase in the mean systemic pressure (Pms) B. An increase in the proportion of lung units under zone III conditions (Pv . Ps) C. Compression of alveolar vessels D. Hypoxic pulmonary vasoconstriction Preferred response: C

Rationale Most of the alveolar surface is laced with a network of alveolar capillaries coursing within the alveolar septum. These sheets of epithelium and vessels lie between adjacent alveoli, so they are subject to the airway pressure that functionally surrounds them. Positive airway pressure compresses alveolar capillaries when alveolar pressure exceeds pulmonary capillary pressure, reducing the driving pressure that propels blood from pulmonary artery to pulmonary vein. There are also corner vessels that traverse the intersection of alveolar septae. These vessels are stretched and stented open at lung volumes above functional residual capacity, whether the lung is stretched by positive pressure or by spontaneous inspiration. This stenting of corner vessels reduces pulmonary vascular resistance. Positive airway pressure has little direct effect on mean systemic pressure but may, over a period of time, cause it to rise. A rise in Pms would increase venous return but would not increase right ventricular afterload. Hypoxic pulmonary vasoconstriction is a potent mechanism to alter the distribution of blood flow through the lungs. It does raise right ventricular afterload, but alveolar hypoxia should be reduced by positive pressure ventilation, not accentuated. As the lung is distended by positive airway pressure, the proportion of lung subject to zone III conditions is reduced, not increased. This reduction would raise right ventricular afterload. 6. Pulsus paradoxus is an exaggeration of the normal version of which of the following? A. Fall in systolic arterial pressure during spontaneous inspiration B. Fall in right ventricular filling during spontaneous inspiration C. Rise in aortic ejection velocity during spontaneous inspiration D. Rise in right ventricular stroke volume during spontaneous inspiration Preferred response: A

e49

Rationale Pulsus paradoxus is an exaggeration of the fall in systolic arterial pressure that normally occurs during spontaneous inspiration. It is a classical finding in pericardial tamponade but also occurs when pleural pressure drops precipitously during strained inspiration (e.g., in persons with croup, asthma, or pneumonia). In tamponade, end-diastolic cardiac volume is essentially fixed. Inspiration favors filling of the right side of the heart over filling of the left side of the heart, shifting the interventricular septum to the left and reducing left ventricular distention. This phenomenon decreases contractility by the Starling mechanism. Inspiration also increases left ventricular afterload by reducing juxtacardiac pressure. Restricting total cardiac volume accentuates these normal effects. Respiratory distress can accentuate these effects by the same mechanisms. Pulsus paradoxus during tamponade was defined before the advent of modern invasive monitoring. It was the appearance and disappearance of the Korotkoff sound over the respiratory cycle that defined the phenomenon. As the pressure in the cuff fell, the first appearance of the Korotkoff sound signaled the expiratory arterial pressure. As the mercury continued to fall, the sound went from intermittent to regular and more frequent when the inspiratory systolic pressure was reached, and it could be heard with every beat of the heart. Although there should be an associated rise in right ventricular stroke volume, that finding is not the defining event in tamponade.

Chapter 33: Disorders of Cardiac Rhythm 1. A 4-month-old child presents to the intensive care unit following operative repair of tetralogy of Fallot in complete AV block with an atrial rate of 120 bpm and a wide complex ventricular escape rate of 50 bpm. Temporary epicardial pacing wires were placed on the right atrium and right ventricle during surgery, with acceptable sensing and pacing characteristics. What would be the best immediate management step? A. Temporary AAI pacing B. Temporary VOO pacing C. Temporary DDD pacing D. Placement of a transvenous dual chamber permanent pacemaker Preferred response: C

Rationale With acceptable atrial and ventricular sensing and pacing, dual chamber pacing is the preferred pacing mode. AAI pacing would be inappropriate in the setting of AV block. Temporary VOO pacing is associated with an increased risk of inducing ventricular tachycardia or fibrillation. Placement of a permanent pacemaker system would be inappropriate on postoperative day 1, given that AV nodal conduction may recover. 2. What is the most common mechanism for supraventricular tachycardia (SVT) in infants and children? A. AV node reentry tachycardia B. Orthodromic reciprocating tachycardia (antegrade over accessory pathway, retrograde over atrioventricular [AV] node) C. Orthodromic reciprocating tachycardia (antegrade over AV node, retrograde over accessory pathway) D. Wolff-Parkinson-White syndrome Preferred response: C

e50

S E C T I O N XV



Pediatric Critical Care: Board Review Questions

Rationale Orthodromic reciprocating tachycardia utilizes an accessory pathway as the retrograde limb, but Wolff-Parkinson-White syndrome may or may not be present in sinus rhythm, making response D incorrect. 3. Which statement is most true regarding adenosine? A. Adenosine should never be administered in the presence of a wide QRS tachycardia. B. Adenosine terminates most tachycardias by blocking conduction in accessory pathways. C. Adenosine specifically blocks conduction in the AV node with little effect on blood pressure or sinus rate. D. Prompt access to defibrillation should be readily available when administering adenosine. Preferred response: D

Rationale Adenosine can cause profound hemodynamic compromise or even ventricular fibrillation, requiring access to immediate defibrillation, especially if it is being administered to patients with wide QRS tachycardia or known Wolff-Parkinson-White syndrome. 4. What is the most appropriate acute therapy for incessant supraventricular tachycardias (SVTs) such as persistent junctional reciprocating tachycardia in an infant? A. Intravenous verapamil B. Procainamide infusion C. Repeated doses of adenosine until sinus rhythm is sustained D. Vagal maneuvers such as ice to face Preferred response: B

Rationale The challenge in persons with incessant SVTs is the tendency for the SVTs to reinitiate. Measures directed at termination are of lesser utility. Repeated adenosine doses or repeated DC cardioversion may aggravate incessant behavior, and intravenous calcium channel blockers should not be administered to infants. 5. Regarding ventricular tachycardias (VTs), which of the following represents the most correct statement? A. Calcium channel blockers should never be administered for VT. B. Intravenously administered amiodarone is superior to intravenously administered lidocaine for treatment of sustained VT. C. The most appropriate acute therapy for polymorphic VT is intravenous magnesium. D. Unlike in adults, VTs in children are rarely life threatening. Preferred response: B

Rationale Certain idiopathic forms of VT are seen in healthy young patients; these forms of VT may respond to calcium channel blockers acutely and are amenable to catheter ablation. However, all ventricular tachycardias should be approached initially as potentially life threatening, and for polymorphic VT (including torsades de pointes) or pulseless monomorphic VT, immediate cardioversion should precede intravenous drug therapy.

Chapter 34: Shock States 1. You are called to the emergency department to see a 17-yearold male who presents as the driver in a high speed motor vehicle crash. He was restrained, vehicle intrusion was greater than 18 inches, and he was unresponsive at the scene. He was orotracheally intubated by EMS and presents with the following vital signs: heart rate, 165 beats/min; respiratory rate, 20/min; blood pressure, 75/35 mm Hg. He has sternal bruising, and his abdomen is distended and firm. In preparation for presentation to the operating room, your priorities are to: A. Initiate your institution’s massive transfusion policy with the goal of administering colloid fluids as soon as possible without delay in presentation to the operating room. B. Obtain an arterial blood gas and then adjust mechanical ventilator settings C. Obtain arterial and central venous access and then obtain a complete blood cell count, coagulation function tests and transaminases before clearing the patient to go to the operating room. D. Treat hypotension by administering 20 mL/kg NS intravenously Preferred response: A

Rationale This patient presents with a mechanism and clinical evidence suggestive of blunt thoracic and abdominal trauma. Initiating massive transfusion protocols for patients presenting in shock with suspected hemorrhage and reducing barriers to presentation to the operating room for damage control procedures are important. Room temperature normal saline can exacerbate the “triad of death” in trauma patients by worsening hypothermia, diluting coagulation, and contributing to hyperchloremic acidosis. 2. You are called to the emergency room to help evaluate two 3-year-old patients in separate rooms. The child in room A presents with a 3-day history of vomiting, diarrhea, and reduced eating and drinking. You find the child in room A to be awake, alert, but fussy; the child has dry mucous membranes; heart rate, 146 beats per minute; blood pressure, 73/52 mm Hg; warm extremities; and capillary refill of 3 seconds. The child in room B presents with a similar history of 3 days of vomiting, diarrhea, and reduced intake. The child in room B is sleepy but able to be aroused. She becomes agitated when aroused and has dry mucous membranes; heart rate, 144 beats per minute; blood pressure, 74/46 mm Hg; cool extremities, and capillary refill of 3 seconds. Your assessment is that one of these children is hypovolemic, and the other is in compensated hypovolemic shock. Therefore what is your recommendation? A. Both child A and B should be treated with oral rehydration over the next 24 to 48 hours per WHO guidelines. B. Both child A and B should be treated with 20 mL/kg of crystalloid delivered intravenously in #15 min. C. Child A should be treated with 20 mL/kg of crystalloid delivered intravenously in #15 min, and child B should be treated with oral rehydration over the next 24 to 48 hours per WHO guidelines. D. Child A should be treated with oral rehydration over the next 24 to 48 hours per WHO guidelines, and child B should be treated with 20 mL/kg of crystalloid delivered intravenously in #15 min. Preferred response: D

CHAPTER 136  Board Review Questions

Rationale Child A has hypovolemia due to gastroenteritis. Child A has normal compensatory responses without evidence of end-organ dysfunction. Child B has an abnormal neurologic exam and a widening pulse pressure. Child B is in compensated hypovolemic shock. Compensated shock should be initially treated with rapid volume administration, whereas patients with hypovolemia can be safely treated with oral rehydration per WHO recommendations. 3. A 5-year-old child with acute lymphoblastic leukemia and neutropenia presents to the pediatric intensive care unit with evidence of septic shock. Data suggest that children who receive ,40 mL/kg of intravenous fluids and are still in shock at the end of the first hour have worse outcomes than those who receive .40 mL/kg and are still in shock at the end of the first hour. Therefore what is your goal? A. Obtain a Scvo2 from the indwelling central catheter, place an arterial line and obtain a serum lactate, and then titrate fluid resuscitation until you see the Scvo2 and lactate improve to normal ranges. B. Treat with 20 mL/kg of crystalloid in #15 min, and then reevaluate for evidence of fluid response. C. Treat with 60 mL/kg of crystalloid fluid resuscitation within the first hour. D. Treat with 60 mL/kg of crystalloid and colloid as indicated within the first hour. Preferred response: B

Rationale Although data suggest that early, rapid fluid resuscitation improves mortality, there are accumulating data that suggest that fluid overload worsens mortality. Therefore the goal in fluid resuscitation is to give just what the patient requires. If the patient shows evidence of sustained response to fluid administration and resolution of shock, no further fluid boluses should be administered. If the patient shows refractory shock, fluid resuscitation should be continued for up to a total of 60 mL/kg in the first hour. After the first hour, if there is evidence of sustained shock, careful evaluation should be used to determine the indication for vasoactive/inotropic support with or without further fluid resuscitation. 4. You are working in the CICU and you receive a phone call from an outside provider in an emergency department caring for a 17-day-old newborn who weighs 3.2 kg and presented in shock. The patient is a full-term infant born to a G2P2 female who did not receive prenatal care. The patient was born at home and delivered by a midwife. Delivery was uncomplicated. The mother reports poor feeding for the last 24 hours, and this morning the newborn was found to be less responsive. In the ED, the patient presents with the following vital signs: temperature, 36.7°C; heart rate, 185 beats per minute; blood pressure, 52/28 mm Hg; respiratory rate, 74 per minute. The patient appears gray and is unresponsive. What do you recommend? A. Ceftriaxone, 20 mL/kg intravenous bolus of crystalloid, and transfer to the PICU B. Orotracheal intubation, ceftriaxone, 20 mL/kg intravenous bolus of crystalloid, and transfer to the PICU C. Orotracheal intubation, ampicillin and gentamicin, 10 mL/kg intravenous bolus of crystalloid, continuous infusion of prostaglandin E1, and transfer to the PICU D. Orotracheal intubation, ampicillin and gentamicin, 10 mL/kg intravenous bolus of crystalloid, echocardiogram, and transfer to the PICU Preferred response: C

e51

Rationale Newborns (,28 days) presenting in shock should be suspected for sepsis and a ductal-dependent congenital heart disease. Therefore fluid resuscitation should be provided, but it should start with a lower volume and then be reevaluated for responsiveness. Ceftriaxone may be okay for newborns, but risks in the newborn period make ampicillin and gentamicin a better empiric choice. Empirically starting a continuous infusion of prostaglandin E1 (PGE1) at 0.05 to 0.1 mg/kg/min may be lifesaving and should be started prior to obtaining the echocardiogram. PGE1 may cause apnea, but in an unresponsive infant presenting in shock, early intubation is indicated. 5. You are caring for a 5-year-old female with acute viral myocarditis. She is sedated, mechanically ventilated, and treated with a continuous infusion of epinephrine at 1 mg/kg/min. Her vital signs are as follows: temperature, 36.8°C; heart rate, 140 beats per min; blood pressure, 75/45 mm Hg; central venous pressure (CVP), 12 cm H2O. Her serum lactate level is rising. Echocardiogram shows that her right atrium is adequately filled, systolic ventricular function is depressed bilaterally with a left ventricular shortening fraction of 18% by M-mode. The septum is midline. The next best step in the management of this patient is to: A. Call your extracorporeal life support team because she is failing medical management of heart failure B. Deepen her sedation and start to cool her to reduce metabolic demand C. Treat with a pulmonary vasodilator because you are concerned that her CVP suggests right ventricular failure D. Treat with a vasodilator because you are concerned that her cardiac output is reduced to high systemic vascular resistance. Preferred response: A

Rationale Vasodilators may be indicated in a hypertensive patient with evidence of shock and low systolic function. However, this patient is hypotensive, suggesting that peripheral vasodilation may not be indicated. Although the patient’s CVP is greater than 10 cm H2O and the echo suggests low right ventricular systolic function, the septum remains midline, which suggests that reduction of pulmonary vascular resistance will not substantially increase cardiac output. Although reduction of metabolic demand is a reasonable next step, deepening sedation carries the risk of worsening myocardial function. Consideration of extracorporeal support is a reasonable next step in this patient with evidence of adequate right ventricular preload, low systolic function, and refractory shock despite high inotropic therapy.

Chapter 35: Pediatric Cardiopulmonary Bypass 1. When blood comes into contact with the cardiopulmonary bypass circuitry, these foreign surfaces will not do which of the following? A. Activate inflammatory response B. Cause hemolysis C. Disrupt hemostasis D. Shift the oxyhemoglobin curve leftward Preferred response: D

Rationale It is well documented that nonendothelial blood contact activates the systemic inflammatory response and causes hemolysis and

e52

S E C T I O N XV   Pediatric Critical Care: Board Review Questions

coagulopathy. Oxyhemoglobin shifts are caused by changes in blood pH, temperature, pco2, and 2,3-DPG. 2. Which of the following is an effective strategy to protect the immature myocardium during ischemic arrest? A. Increase the cardiac membrane resting potential. B. Keep the myocardium warm. C. Perfuse the coronaries with a hypotonic solution. D. Prevent intracellular calcium accumulation. Preferred response: D

Rationale Calcium shifting at the cellular membrane is an energy-dependent process, and minimizing intracellular calcium accumulation will help to prevent ATP depletion. Decreasing the cardiac membrane resting potential to achieve diastolic arrest is the goal of depolarizing cardioplegia. Delivering a cold cardioplegia solution will minimize metabolic demands and prolong the myocardial tolerance to ischemia. An isotonic or hypertonic cardioplegia solution is desired to reduce intracellular water accumulation and edema. 3. During the aortic cross clamp, which of the following perfusion techniques would be the most effective in reducing collateral blood flow to the heart and improving operative visibility? A. Administer more heparin and raise the activated coagulation time (ACT) B. Decrease arterial flow and patient temperature C. Increase the mean arterial pressure and cardiac index D. Transfuse an isotonic crystalloid solution and reduce viscosity Preferred response: B

Rationale Patients with pulmonary blood flow restrictions (e.g., tetralogy of Fallot, pulmonary atresia) can develop major aortopulmonary collateral arteries, and these collaterals can flood the heart during the aortic cross-clamp period if cardiopulmonary bypass flow is maintained. This excessive blood return not only obscures the surgical site but may also warm the cold arrested myocardium or wash out cardioplegia from the coronary arteries. Hypothermia and perfusion flow rate reduction can attenuate this collateral flow while maintaining adequate oxygenation delivery to the patient. 4. Which of the following factors does not increase the incidence of acute kidney injury (AKI) during cardiopulmonary bypass? A. Deep hypothermic circulatory arrest (DHCA) B. Hypotension C. Pulsatile perfusion flow D. Younger age Preferred response: C

Rationale The kidneys perceive nonpulsatile flow or a decrease in arterial flow as hypovolemia, and the resultant neurohormonal cascade is thought to trigger the AKI complex. Since most perioperative risk factors such as younger age and the incidence of higher surgical complexity are nonmodifiable, therapeutic strategies have focused on optimally managing perfusion flow rate, arterial pressure, and hematocrit.

Chapter 36: Critical Care After Surgery for Congenital Cardiac Disease 1. A 10-month-old girl (8 kg) is recovering from surgery to repair a ventricular septal defect. She has been taking her usual diet of breast milk and age-appropriate food since postoperative day 2. On postoperative day 3, while extubated and still requiring oxygen via nasal cannula, a chest radiograph showed a moderately sized right pleural effusion. A pigtail catheter was placed in the right pleural space and yielded 150 ml of white effluent containing 2000 WBCs (95% lymphocytes) and elevated triglycerides (410 mg/dL). The pleural catheter drains another 100 ml over the following day and follow up chest radiograph obtained on postoperative day 4 shows a small residual effusion. The MOST appropriate next step in the management of this patient is to: A. Begin a diet with medium-chain triglycerides (MCT) as the source of fat. B. Begin an intravenous infusion of octreotide. C. Continue current management and follow chest tube output. D. Discontinue enteral nutrition and begin total parenteral nutrition. E. Perform a lymphangiogram in preparation for lymphatic embolization or thoracic duct ligation. Preferred response: A

Rationale This patient developed chylothorax following cardiac surgery. With drainage of 12.5 mL/kg over 24 hours, this chylothorax would be classified in the “low volume” category. The initial management of low volume chylothoraces is centered on a diet low in long-chain triglycerides, where the source of fat is in the form of medium-chain triglycerides (MCT). This is often sufficient to resolve the chylothorax within 7 days. If persistent, additional treatments may be started sequentially; these include enteral fasting with total parenteral nutrition, octreotide, and thoracic duct embolization or surgical ligation. 2. A 5-day-old child born with d-transposition of the great arteries is recovering in the cardiac ICU following an arterial switch operation. Approximately twelve hours after successful separation from cardiopulmonary bypass, the patient is noted to have frequent premature ventricular contractions (PVCs). She is receiving mechanical ventilation and an infusion of epinephrine (0.05 mg/kg/min). The vital signs are heart rate, 170 beats per minute; respiratory rate, 28 per minute; blood pressure. 55/30 mm Hg (mean 41 mm Hg), central venous pressure, 14 mm Hg; left atrial pressure, 17 mm Hg. Electrolytes are normal, but an arterial blood gas shows a mild metabolic acidosis (7.34/38/123/-3/99%) and a lactate of 3.2 mmol/L. The MOST appropriate next step in the management of this patient is to: A. Administer a 10 mL/kg intravenous bolus of 5% albumin B. Obtain a STAT chest radiograph C. Obtain a STAT echocardiogram D. Start a furosemide infusion at 0.1 mg/kg/h E. Start a vasopressin infusion at 0.3 mU/kg/min Preferred response: C

CHAPTER 136  Board Review Questions

Rationale This patient is tachycardic, hypotensive, has a disproportionate elevation of the left atrial pressure, an elevated blood lactate, and new onset of frequent ventricular ectopy. Although it would be tempting to ascribe these findings to the vagaries of a post-cardiopulmonary bypass low cardiac output state, they are much more likely to be the manifestation of poor coronary blood flow. Therefore, it is imperative that a surgical issue, such as a coronary reimplantation kink or tamponade be ruled out. A STAT echocardiogram should be obtained to evaluate function, rule out tamponade, and ascertain adequate coronary blood flow. If the coronary buttons cannot be well visualized by echocardiogram, emergent cardiac catheterization should be performed. A fluid bolus would not be indicated in this patent with an already elevated left atrial pressure. Although a furosemide infusion is often initiated in the postoperative period, it would not address the primary concern of myocardial ischemia. Vasopressin would cause an increase in systemic vascular resistance, but it would not be a fruitful therapy if the patient indeed has a kink of the left coronary button. A chest radiograph would have been useful to rule out a new pulmonary process or pleural effusions, but would be low yield in this situation. 3. A 6-day-old child is returned to the cardiac intensive care unit after a stage I palliation (Norwood procedure) for hypoplastic left heart syndrome. He remains endotracheally intubated and has an open sternum. Heart rate is 180 beats/min and blood pressure is 55/28 mm Hg while on epinephrine (0.05 µg/kg/ min) and milrinone (0.5 µg/kg/min). He is being ventilated with a tidal volume of 8 mL/kg, PEEP of 5 cm H2O, rate of 26 breaths/min, and Fio2 of 0.4. An arterial blood gas analysis shows severe metabolic acidosis (pH 7.16, Paco2 40 mm Hg, Pao2 78 mm Hg, Sao2 94%, base deficit, 13). What is the next best step to manage this condition? A. Decrease the Fio2 to 0.21. B. Decrease the milrinone dose to 0.3 µg/kg/min. C. Increase the epinephrine dose to 0.08 µg/kg/min. D. Increase the respiratory rate to 30 breaths/min. Preferred response: A

Rationale The patient has classic signs of an unbalanced circulation following the Norwood procedure, with excess pulmonary blood flow and decreased systemic perfusion (high Qp:Qs). This is a serious emergency and requires prompt action. Decreasing the Fio2 to room air can be accomplished rapidly and will help balance the Qp:Qs. In some cases, carbon dioxide may need to be added to the inspired gas or subatmospheric Fio2 to help reduce excessive pulmonary blood flow. Decreasing the dose of milrinone would not be a reasonable strategy for this patient, who can in fact benefit from additional afterload reduction to increase systemic blood flow and help balance the Qp:Qs. For the same reason, increasing the dose of epinephrine could be detrimental as it will likely increase the systemic vascular resistance and increase pulmonary blood flow. Increasing the respiratory rate to 30 breaths/ min would lower the Paco2, and although this will help compensate the acidosis, it will also lower pulmonary vascular resistance and adversely contribute to the already increased pulmonary blood flow.

e53

4. A 6-month-old child with history of tricuspid atresia returns to the cardiac intensive care unit intubated following a bidirectional Glenn anastomosis. She is hemodynamically stable and in sinus rhythm but has significant hypoxemia (Sao2 of 65%) despite being offered a Fio2 of 1.0 through the ventilator. The hypoxemia persists despite adequate sedation and intravascular volume expansion with packed red blood cells to increase the hematocrit to 40%. Of the following, which intervention is least likely to meaningfully improve the arterial oxygen saturation in this patient? A. Controlled hypoventilation (permissive hypercapnia) B. Head elevation (30 degrees) C. Inhaled nitric oxide D. Intravenous milrinone and epinephrine Preferred response: C

Rationale Following the bidirectional Glenn anastomosis, arterial oxygen saturation should be in the 80% to 85% range; however, stabilization to this level can take a number of days. Persistent hypoxemia (Sao2 ,70%) can be secondary to a low cardiac output state (low Svo2), low pulmonary blood flow, or lung disease. Treatment is directed at improving contractility, increasing superior vena cava venous return, reducing afterload, and ensuring the patient has a normal rhythm and hematocrit. Increased pulmonary vascular resistance is an uncommon cause, and inhaled nitric oxide (NO) is not generally beneficial in these patients. This finding is not surprising because pulmonary artery (PA) pressure and resistance and vascular tone are not high enough following this surgery to see a demonstrable benefit from NO. Controlled hypoventilation targeting hypercapnia promotes increased cerebral blood flow and, consequently, increased blood flow through the superior vena cava (SVC)-PA anastomosis. Persistent profound hypoxemia should be investigated in the catheterization laboratory to evaluate hemodynamics, look for residual anatomic defects limiting pulmonary flow, such as SVC or PA stenosis or a restrictive atrial septal defect (ASD), and coil any significant venous decompressing collaterals (e.g., azygous vein), if present. 5. Following a Fontan operation for palliation of the hypoplastic left heart syndrome, which of the following postoperative strategies is commonly used? A. Early extubation whenever the patient is able to assume spontaneous breathing B. Forced diuresis to lower the central venous pressure C. High doses of dopamine and epinephrine to increase systemic vascular resistance and blood pressure D. Mechanical ventilation with high levels of positive end- expiratory pressure to optimize lung inflation and increase pulmonary blood flow Preferred response: A

Rationale Following a Fontan operation, liberation from positive pressure ventilation should be accomplished as soon as the patient is able to assume spontaneous breathing, provided there is no significant lung disease or atelectasis. For this reason, prolonged sedation and paralysis generally are not indicated. High doses of vasoactive drugs should be avoided because they increase the afterload to the systemic right ventricle. Lowering the central venous pressure is not indicated because pulmonary blood flow is largely dependent

e54

S E C T I O N XV



Pediatric Critical Care: Board Review Questions

on systemic venous return. A high level of positive end-expiratory pressure should be avoided in these patients because it increases intrathoracic pressure, reduces venous return, and consequently reduces pulmonary blood flow. 6. Which of the following is true for a patient with tetralogy of Fallot? A. A residual ventricular septal defect (VSD) promotes left-toright shunt and augments pulmonary blood flow. B. Epinephrine is the drug of choice in the postoperative period because it increases cardiac output and contractility of the poorly compliant right ventricle. C. Patients usually require high right atrial pressure postoperatively because of the hypertrophic and poorly compliant right ventricle. D. The presence of an atrial level communication is undesirable because it can lead to hypoxemia and decreased preload to the left ventricle. Preferred response: C

Rationale The poorly compliant right ventricle requires high right-sided filling pressures (right atrial pressure). Use of vasodilators should be avoided during a hypercyanotic spell, because lowering systemic vascular resistance will increase the right-to-left shunt and worsen hypoxemia and acidosis. An atrial level communication is highly desirable in patients with significant right ventricular diastolic dysfunction, because it allows for a right-to-left atrial level shunt that ensures adequate left ventricular preload (although it causes arterial oxygen desaturation). Milrinone is the drug of choice in the postoperative period because of its lusitropic effects. A residual ventricular septal defect (VSD) is deleterious, particularly in the setting of a persistent right ventricular outflow tract obstruction.

Chapter 37: Cardiac Transplantation 1. A 5-year-old patient with dilated cardiomyopathy presents to the cardiac ICU with severe decompensated congestive heart failure. Which one of the following therapeutic options would be the most successful bridge to transplant? A. Enalapril B. Extracorporeal membrane oxygenation (ECMO) C. Furosemide D. Mechanical ventilation E. Ventricular assist device Preferred response: E

Rationale While all options could be used to treat congestive heart failure, use of a ventricular assist device has been shown to improve waitlist mortality and improve posttransplant outcomes. Mechanical ventilator support and ECMO are risk factors for increased waitlist and posttransplant mortality. 2. Which of the following would be the strongest contraindication to isolated heart transplantation in a neonate? A. Common pulmonary vein atresia B. Dextrocardia C. Discontinuous pulmonary arteries D. Double aortic arch E. Total anomalous pulmonary venous return Preferred response: A

Rationale Isolated heart transplantation in the presence of pulmonary vein atresia would be technically difficult to perform, especially in a neonate. All other conditions presented could be overcome with good surgical planning. 3. In a 2-month-old infant with hypoplastic left heart syndrome who has undergone a Stage I reconstruction, the strongest indication for heart transplantation would be which of the following: A. Aortic arch obstruction with right ventricular dysfunction B. Marked cyanosis C. Protein losing enteropathy D. Restrictive atrial septal defect E. Severe tricuspid valve regurgitation Preferred response: E

Rationale Significant tricuspid valve regurgitation is a risk factor for failure of single ventricle palliation with poor outcomes. Aortic arch obstruction and restrictive atrial septal defect require intervention. Should that be unsuccessful, then transplantation could be considered. Protein losing enteropathy is generally not a complication of a stage I reconstruction, and cyanosis alone is not an indication for transplantation. 4. You admit a 12-year-old from the operating room following heart transplantation. The nurse is concerned about an episode of hypotension and asks which value on the monitor most likely indicates the presence of ventricular diastolic dysfunction. You answer that it is which of the following: A. Cardiac index 3.3 L/min/m2 B. CVP 22 mm Hg C. Heart rate 115 bpm D. NIRS 65% E. PA pressure 32/15 mm Hg Preferred response: B

Rationale Diastolic dysfunction of the right ventricle is common after heart transplantation. Central venous pressure (CVP) represents the end-diastolic pressure of the right ventricle in the setting of normal tricuspid valve function. Therefore, elevation of the CVP postcardiac surgery indicates the presence of ventricular diastolic dysfunction. The other parameters listed have values that are normal or near normal following heart transplantation. 5. The major cause of death in adolescents after heart transplant is which of the following? A. Dysrhythmia B. Noncompliance C. Renal failure D. Thromboembolism Preferred response: B

Rationale Currently the major cause of death in the adolescent heart transplant recipient is noncompliance with the medical regimen.

CHAPTER 136  Board Review Questions

e55

6. The preferred anticoagulant for patients in the intensive care unit who have myocardial dysfunction and are awaiting a heart transplant is which of the following? A. Aspirin B. Coumadin C. Heparin D. Lovenox Preferred response: C

the transplanted heart reflect a significant shift to the left of the pressure/volume curve. Diastolic dysfunction can be demonstrated from the early transplant period. Diastolic dysfunction is a significant impairment to early allograft function, limiting cardiac output. Diastolic dysfunction emphasizes the importance of heart rate and early sinus node function. The capability of temporary pacing in the early perioperative period is mandatory.

Rationale

9. Increased perioperative mortality for heart transplantation is associated with which of the following? A. Mitral valve insufficiency B. Tricuspid valve insufficiency C. Pulmonary arterial pressure ,15 mm Hg D. Pulmonary vascular resistance (PVR) .6 Wood units Preferred response: D

All patients waiting for heart transplantation should be managed with systemic anticoagulation. Heparin is preferred, but warfarin is acceptable in a stable patient who is receiving inotropic support. We add a word of caution regarding the use of low-molecularweight heparin for prophylaxis: Enoxaparin cannot be easily reversed in a patient who must go to the operating room emergently because a donor heart has been identified. Cardiovascular surgeons prefer using heparin for prophylactic anticoagulation. 7. First-line medication for management of hypotension in the potential heart donor is which of the following? A. Dobutamine B. Dopamine C. Epinephrine D. Vasopressin Preferred response: D

Rationale A catecholamine surge causing unnatural circulatory physiology that rapidly evolves is associated with brain death, making management of the donor difficult. This intense sympathomimetic outflow initially causes vasoconstriction resulting in tachycardia, hypertension, and increased myocardial oxygen demand. The result can be a direct injury to the myocardium in the potentially transplantable heart. Myocardial structural damage that includes myocytolysis, contraction band necrosis, subendocardial hemorrhage, edema formation, and interstitial mononuclear infiltration is seen. This initial sympathetic outflow is followed by a loss of sympathetic tone, resulting in marked vasodilation and hypotension. The hypotension and cardiovascular collapse are related to decreased systemic vascular resistance rather than primary myocardial dysfunction. Vasopressin is now the first-line blood pressure support medication because it treats diabetes insipidus in addition to supporting blood pressure. Vasopressin infusion of less than 2.5 units/hour usually is sufficient to increase mean arterial blood pressure and not cause end-organ injury. 8. Cardiac output in the heart immediately after transplantation is impaired primarily by which of the following? A. Arch obstruction B. Diastolic dysfunction C. Mitral valve insufficiency D. Tricuspid valve insufficiency Preferred response: B

Rationale The major changes in the physiology of the transplanted heart are related to autonomic denervation, including diastolic dysfunction and an exaggerated response to exogenously administered catecholamines. The transplanted heart also must adapt to a new environment related to the recipient’s lung function and elevated pulmonary vascular resistance (PVR). Hemodynamics of

Rationale High PVR in the recipient increases perioperative morbidity and mortality and can affect late survival. All potential heart recipients undergo cardiac catheterization prior to heart transplantation to document the anatomy of systemic and pulmonary venous connections, determine pulmonary artery size and distribution, and calculate PVR. The upper limit of PVR associated with successful orthotopic heart transplantation is not known. Criteria developed from the adult heart transplant experience indicate that a PVR greater than 6 Wood units or a transpulmonary gradient (pulmonary artery mean pressure – left atrial mean pressure) greater than 15 mm Hg is associated with increased perioperative mortality. The transpulmonary gradient is the most useful number for estimating PVR because measurement of cardiac output in the catheterization laboratory can be flawed. In children the PVR index, which is determined by dividing transpulmonary gradient by cardiac index, is more useful because children come in all sizes. A PVR index less than 6 index units is associated with low perioperative mortality. 10. Hyperglycemia following heart transplantation is most likely related to which of the following? A. Antithymocyte globulin B. Basiliximab C. Furosemide D. Tacrolimus Preferred response: D

Rationale Hyperglycemia is common after heart transplantation with tacrolimus-based immune suppression. The combination of decreased insulin production from islet cells caused by tacrolimus and decreased peripheral utilization related to high-dose corticosteroids results in nonketotic hyperglycemia. Insulin is initially mandatory in management but often can be discontinued if the tacrolimus dose is reduced and the corticosteroid portion of maintenance immune suppression is discontinued. 11. Diastolic dysfunction in a patient after heart transplantation most commonly will manifest as which of the following changes following a normal saline solution fluid challenge? A. An increase in right atrial pressure by 2 mm Hg B. An increase in left atrial pressure by 2 mm Hg C. No change in right atrial pressure D. No change in stroke volume Preferred response: B

e56

S E C T I O N XV   Pediatric Critical Care: Board Review Questions

Rationale Hypotension and cardiovascular collapse in patients after heart transplantation are related to decrease in systemic vascular resistance rather than primary myocardial dysfunction. Large fluid volumes and high-dose inotropic agents at a-adrenergic dosing range are administered, causing volume overload and vasoconstriction that can injure all donor organs. Hearts that are supported on high-dose inotropic agents likely will exhibit myocardial injury. A risk factor that predicts donor heart failure is a history of high-dose dopamine, dobutamine greater than 20 mg/ kg/min, and epinephrine greater than 0.1 mg/kg/min. 12. A 5-year old child with dilated cardiomyopathy in your pediatric intensive care unit is waiting for an orthotopic heart transplant. The child’s weight is 20 kg. She is receiving intermittent noninvasive continuous positive airway pressure. Her mixed venous oxygen saturation is 45%, and her left ventricular ejection fraction (LVEF) is less than 20%. Inotropic support includes a milrinone drip of 0.75 mg/kg/min and an epinephrine drip of 0.02 mg/kg/min. A donor organ becomes available within 1 hour of flying time from your medical center. The donor is a teenager who became brain dead after a motor vehicle accident 3 days ago. The donor weight is 40 kg. Resuscitation at the accident site was required with unknown down time. Vasopressin and epinephrine were initiated and given for the first 24 hours but subsequently weaned to low-dose dopamine. Current evaluation of the donor heart shows an LVEF of 60% with trivial mitral valve regurgitation. Would you recover this donor heart for your patient? A. No; the uncertain down time and high-dose inotropic support make the donor unacceptable. B. No; the donor is too large for my patient. C. No; the distance from the center makes it impossible for ischemic time to be less than 4 hours. D. Possibly; the donor is potentially acceptable. I would review all of the information from the donor site. Preferred response: D

Rationale Donor heart availability is limited, and the number of patients who die while waiting for a heart transplant is significant. Your recipient is critically ill, and the need for circulatory support is imminent (via extracorporeal membrane oxygenation or a ventricular support device), so all potential donor hearts should be critically evaluated. An estimate of donor “down time” is important but often inaccurate. Inotropic resuscitation of the donor is common. The use of high-dose inotropic support (i.e., epinephrine and norepinephrine) for a prolonged period of time (.24 hours) reflects significant donor heart ischemia, and in such cases the donor heart should not be used for transplantation. A donor/ recipient weight of greater than 2:1 is common. We use the donor/ recipient aortic root size rather than weight to estimate the size mismatch. We rarely use an undersized donor (i.e., weight less than 20% of the recipient) for transplantation. Flight time from the transplant center is relative. Ideally a total ischemic time of less than 4 hours is ideal (i.e., the time from aortic cross-clamp at the donor site to release of the aortic cross-clamp on the recipient). It is important to remember that a perfect heart to transplant is never available. We encourage direct verbal communication with the donor referral center.

13. A 15-year old patient with tricuspid atresia has had palliative surgery (atrial to pulmonary connection) (Fontan). A myopathic ventricular dysfunction and protein-losing enteropathy have developed. He has just had an orthotopic transplant. You are discussing potential immune suppression regimens. Pretransplant panel reactive antibody is 10% for class 1 human leukocyte antigens. Which of the following is of major concern for renal dysfunction in the early perioperative course after heart transplantation? A. Antithymocyte globulin (ATG) B. Basiliximab C. Methylprednisolone D. Mycophenolate E. Tacrolimus Preferred response: E

Rationale Immunosuppression protocols are similar from center to center with generally only small variations. Initial immune suppression protocols typically include high-dose corticosteroids, induction with IL-2 receptor blockade (e.g., basiliximab) or antithymocyte globulin (ATG), followed within approximately 48 hours by introduction of a calcineurin inhibitor (CNI) such as cyclosporine or tacrolimus. Induction protocols using IL-2 receptor antagonists or ATG are effective in delaying the time to first allograft rejection episode and the time needed to initiate CNI medications, which is especially useful when there is significant renal dysfunction. Induction therapy also reduces the risk of death due to rejection, although it does not appear to have a long-term survival benefit, except possibly in those with a PRA .50% or diagnosis of congenital heart disease. Induction therapy may also be useful in steroid avoidance protocols. Some centers do not use induction therapy under particular circumstances due to concerns for risk of infection or viral reactivation, although that has not been borne out in recent studies. Corticosteroids have been part of standard protocols since the early days of solid organ transplantation. High-dose methylprednisolone (5 to 10 mg/kg) is administered at the time of aortic cross-clamp removal and continued in tapering doses over the first several days after surgery. Corticosteroids have immunosuppressive properties and benefit the allograft because of membranestabilizing and antioxidant effects on the graft. Steroid-sparing/ steroid-avoidance protocols exist for other solid organ transplants and are in development for heart transplantation. More controversial is the timing of the introduction of the CNIs cyclosporine and tacrolimus. A major complication in the early perioperative course after heart transplantation is renal dysfunction; in the past, CNIs were major contributors. Acute kidney injury is a major complication following orthotopic heart transplantation. It is often multifactorial in etiology, as the premorbid risk factors of heart transplant recipients cannot necessarily be controlled. One must monitor and control use of calcineurin-inhibitor immune suppression agents, especially when renal dysfunction is present or expected. Therapeutic strategies include delaying the initiation of cyclosporine and tacrolimus by using antithymocyte globulin or IL-2 receptor blockade for induction of immune suppression. The other option is to use a modified oral/ nasogastric protocol for tacrolimus administration. This protocol targets tacrolimus levels to below 6 ng/mL in the first 3 days after transplantation, followed by a rapid increase in dosing and target

CHAPTER 136  Board Review Questions

level over the next 4 days. It is important to avoid early IV administration of these agents because they invariably lead to renal afferent arteriolar vasoconstriction and oliguria. If renal dysfunction is complicating the posttransplant course, it is still difficult to withdraw CNIs completely, but lowering the target level to less than 6 ng/L and substituting higher doses of mycophenolate mofetil and adding sirolimus are reasonable options. 14. An 8-year old girl with restrictive cardiomyopathy is being evaluated for transplant. A hemodynamic study is performed as part of the pretransplant evaluation. Hemodynamics are as follows: blood pressure, 110/80 mm Hg; pulmonary artery, 62/40 mm Hg, mean 50 mm Hg; pulmonary artery wedge, 35 mm Hg; left ventricular end-diastolic pressure, 35 mm Hg; and cardiac index, 2.5 L/min/m2. What is your decision regarding this patient’s suitability for orthotopic heart transplant? A. Before making a decision, hemodynamics should be repeated with a fraction of inspired oxygen of 1.0 and nitric oxide. B. Listing the patient for a heart transplant should be delayed until medical management of pulmonary hypertension can be initiated. C. The patient is a candidate for heart and lung transplant only. D. The patient is a candidate for orthotopic heart transplant with a recognized increased risk factor of elevated pulmonary vascular resistance (PVR). Preferred response: D

Rationale The evaluation of pulmonary vascular resistance (PVR) in the potential heart transplant recipient is a critical part of the evaluation. The right ventricle of the donor heart will acutely fail if it is exposed to excessive afterload from pulmonary vascular disease of the recipient. A pretransplant hemodynamic assessment with conditions favoring pulmonary vasodilation using oxygen, nitric oxide, and or nitroprusside is necessary to determine if the recipient is an orthotopic heart transplant candidate alone or will need to be referred for a heart and lung transplant. A transpulmonary gradient (pulmonary artery mean pressure–left atrial pressure) of less than 15 mm Hg or an estimated PVR index of less than 4 to 6 units/m2 is acceptable level of PVR for orthotopic heart transplantation.

Chapter 38: Physiologic Foundations of Cardiopulmonary Resuscitation 1. The odds of death are increased after extracorporeal cardiopulmonary resuscitation (E-CPR) with the following risk factors? A. Age B. Preexisting renal insufficiency C. Shorter CPR times prior to ECMO D. Venovenous (VV) ECMO Cannulation Preferred response: B

Rationale Data from the Extracorporeal Life Support Organization and AHA “Get with the Guidelines-Resuscitation” registries to determine risk factors related to unfavorable outcomes with E-CPR among 593 children. In this study they found that odds of death were increased with a noncardiac diagnosis and preexisting renal insufficiency, and that for each additional 5 minutes of CPR prior to ECMO initiation, the odds risk of death increased by 1.04.

e57

Age was not found to increase the odds of death. Longer CPR times (rather than shorter times) were found to have increased risk of death. E-CPR refers to veno-arterial rather than venovenous cannulation. 2. Cerebral blood flow during CPR is increased with the following medication? A. Dexmedetomidine B. Metoprolol C. Milrinone D. Phenylephrine Preferred response: D

Rationale Cerebral blood flow during CPR depends on peripheral vasoconstriction that can be enhanced using an a-adrenergic agonist or arginine vasopressin receptor agonist (V1 receptor). This action produces selective vasoconstriction of noncerebral peripheral vessels to areas of the head and scalp without causing cerebral vasoconstriction. As with myocardial blood flow, pure a-agonist agents are as effective as epinephrine in generating and sustaining cerebral blood flow during CPR in adult animal models and in infant models. A b-blocking agent such as metoprolol (b1 adrenergic receptor) would have a negative effect on cardiac output and no direct effect on systemic vascular resistance. A selective a2 adrenoreceptor agonist such as dexmedetomidine would activate G-proteins via a2a adrenoreceptors in the brainstem and inhibit norepinephrine release, causing no effect on the peripheral vasculature. Finally, use of a selective phosphodiesterase inhibitor such as milrinone would induce inhibition of cardiac and vascular tissue, resulting in vasodilation. 3. What is the most common presenting rhythm in a pediatric patient in cardiac arrest? A. Asystole B. Bradycardia C. Torsades de pointes D. Ventricular fibrillation Preferred response: A

Rationale Asystole is the most common presenting rhythm in a pediatric patient who presents in cardiac arrest, noted in 25–70% of victims. Bradycardia and pulseless electrical activity (PEA) are other common rhythms, while ventricular rhythms are infrequent. Systemic disturbances, such as hypoxia, acidosis, sepsis, and hypovolemia, often precede the arrest and lead to the asystole rhythm. 4. A 16-year-old male with no previous medical history had a witnessed collapse on the basketball court, and the automated external defibrillator (AED) recommended defibrillation, which was given. The patient arrives to your emergency room in cardiac arrest, having received several rounds of CPR with defibrillation attempts. Which antiarrhythmic would NOT be appropriate? A. Amiodarone B. Lidocaine C. Procainamide D. Sotalol Preferred response: D

e58

S E C T I O N XV



Pediatric Critical Care: Board Review Questions

Rationale Sotalol is not an appropriate antiarrhythmic choice for a ventricular arrhythmia in an acute cardiac arrest. According to the 2015 AHA guidelines, amiodarone and lidocaine are appropriate first-line agents, though neither has been shown to have a survival benefit over the other. Amiodarone given intravenously may depress myocardial function and lead to hemodynamic collapse, which may not be a concern in a patient undergoing active cardiopulmonary resuscitation. The halflife of amiodarone is on the magnitude of days, so it will take several doses to appropriately load a patient. It is contraindicated in patients with torsades de pointes as it increases the QTc and would exacerbate the arrhythmia. Lidocaine has a shorter half-life (5–10 minutes) though it may depress myocardial function at high plasma concentrations. Procainamide may also be used to treat ventricular arrhythmias, though like amiodarone it also prolongs the QTc. 5. What is the major pharmacologic effect of epinephrine during cardiopulmonary resuscitation? A. It raises aortic systolic pressure. B. It raises aortic diastolic pressure. C. It raises the heart rate via its b-agonist actions. D. It improves cardiac compliance by relaxing myocardium during diastole. Preferred response: B

Rationale At resuscitative doses of epinephrine, the a-adrenergic receptor effect predominates, increasing systemic vascular resistance in addition to increasing cardiac output. The a-adrenergic–mediated vasoconstriction of epinephrine increases aortic diastolic pressure and thus coronary perfusion pressure, a critical determinant of successful resuscitation. At nonresuscitative doses, epinephrine has potent b1-adrenergic receptor activity and moderate b2- and a1-adrenergic receptor ef-

Bed

5

23 JUN 98 11:52

***VENT TACHY HR 136

II

VPB 11

fects. Clinically, low doses of epinephrine increase cardiac output because of the b1-adrenergic receptor inotropic and chronotropic effects, whereas the a-adrenergic receptor–induced vasoconstriction is often offset by the b2-adrenergic receptor vasodilation. The result is an increased cardiac output, with decreased systemic vascular resistance and variable effects on the mean arterial pressure. 6. Compared with monophasic defibrillators, which of the following is true about biphasic defibrillators? A. They are too expensive and have an unacceptable fault rate. B. They have been shown to burn too much myocardium in smaller children. C. They require less energy output to be effective. D. They should be used at one-fourth the starting dose in children. Preferred response: C

Rationale Defibrillators can deliver energy in a variety of waveforms that are broadly characterized as monophasic or biphasic. Biphasic waveforms defibrillate more effectively and at lower energies than do monophasic waveforms. The value of higher energy biphasic shocks was demonstrated in the BIPHASIC trial. 7. A 16-year-old female with a history of chronic medications, including metoclopramide, ondansetron, and erythromycin, presents conscious with the tracing shown in Figure 136.15, below. Which of the following antiarrhythmic agents would be most appropriate in the acute setting? A. Adenosine B. Amiodarone C. Lidocaine D. Procainamide Preferred response: C

Sp02 98

ST1 0.0

ST2 –0.8

ST3 0.4

PULSE 68 25 mm/sec

(0BMIA10) [AI002]

aVL

• Fig. 136.15

Rationale Due to her medication profile, this patient is at risk for acquired long QT syndrome (LQTS); this tracing represents torsades de pointes (TdP). TdP is a form of polymorphic ventricular tachycardia that occurs in the setting of either acquired or congenital LQTS, where it appears as though the QRS is twisting around an isoelectric axis. Delay in treatment can cause the rhythm to degenerate into ventricular fibrillation. The first-line treatment for TdP is IV magnesium, which is effective in both treatment and prevention of TdP. It is effective even in the





setting of a normal serum magnesium level. Of the antiarrhythmics presented, lidocaine would be the most appropriate choice in the acute management of TdP, as it shortens the duration of the action potential. Both amiodarone and procainamide prolong the QT interval, which worsens the pathophysiology of this arrhythmia. Adenosine is used primarily to diagnose and treat supraventricular tachycardias. It would not treat TdP. In the setting of an unstable patient, ventricular defibrillation would be the first-line therapy.

CHAPTER 136  Board Review Questions

Chapter 39: Performance of Cardiopulmonary Resuscitation in Infants and Children 1. A 6-year-old child with a history of asthma is brought to the emergency department by paramedics with cardiopulmonary resuscitation (CPR) in process. She developed severe respiratory distress and was intubated during the ambulance ride to the hospital. Appropriate position of the endotracheal tube was documented, including exhaled end-tidal carbon dioxide (ETCO2). About 2 minutes prior to her arrival, she became progressively hypoxemic and bradycardic and cardiopulmonary resuscitation (CPR) was started. Upon arrival to the resuscitation bay, she is transferred to the stretcher with ongoing CPR, ETCO2 monitoring is confirmed, and a CPR quality recording defibrillator is placed. Which of the following is the correct compression-to-ventilation ratio and frequency of chest compressions in this situation? A. 15 compressions: 2 ventilations, at least 100 compressions/ minute B. 30 compressions: 2 ventilations, no more than 120 compressions/ minute C. Continuous chest compressions at a rate of 100 to 120 compressions/minute with 10 breaths per minute (asynchronous ventilation) D. 100–120 compressions/minute without ventilations (“handsonly” CPR) Preferred response: C

Rationale When a child has an invasive airway in place during CPR, guidelines recommend the provision of continuous chest compressions at a rate of 100 to 120 per minute with asynchronous ventilations. To avoid excessive ventilation, ventilations should be provided at a rate of 1 breath every 6 seconds (10 breaths per minute). 2. During ongoing resuscitation of the child above, an arterial line is placed in the femoral artery. Transduction during active CPR demonstrates a blood pressure of 75/22 mm Hg. Among these markers of CPR quality, which one has been associated with improved survival to hospital discharge with favorable neurological outcome after pediatric cardiac arrest? A. Chest compression rate 100–120 per minute B. Depth of compression .50 mm C. Diastolic blood pressure 25 mm Hg in infants and 30 mm Hg in older children D. ETCO2 during CPR 20 mm Hg Preferred response: C

Rationale In a recent large registry study of the Collaborative Pediatric Critical Care Research Network (CPCCRN), among children with an arterial line in place at the time of the arrest, a diastolic blood pressure 25 mm Hg in infants and 30 mm Hg in older children was associated with improved survival to discharge and survival to discharge with favorable neurological outcome compared to lower blood pressures.

e59

3. Following successful resuscitation from cardiac arrest, which of the following represent an optimal set of goals during the postarrest period? A. Blood pressure .5th percentile for age; Paco2 35–50 mm Hg; normal Pao2; avoidance of fever B. Core body temperature 32–34°C; Paco2 ,30 mm Hg; blood pressure .5th percentile for age C. Core body temperature 35–37°C; Paco2 ,30 mm Hg; blood pressure .5th percentile for age D. Serum glucose 60–100 mg/dL; blood pressure .5th percentile for age; normal Paco2 Preferred response: A

Rationale Goals for the provision of post cardiac arrest care include: • Avoidance of hypotension (provision of isotonic fluids, vasopressors, and inotropic agents to maintain blood pressure .5th percentile for age). • Avoidance of fever with continuous core body temperature monitoring and targeted temperature management to maintain normothermia. Therapeutic hypothermia can be considered following out-of-hospital cardiac arrest (OHCA). • Avoidance of significant hypocapnia/hypercapnia or hypoxemia/ hyperoxia. • Monitoring for seizures. 4. A 14-year-old girl was found unresponsive and CPR is being provided for pulseless electrical activity cardiac arrest. During a pulse and rhythm check, there is no pulse palpable and the monitor and defibrillator display a disorganized rhythm that appears to be ventricular fibrillation (VF). You direct the team to resume chest compressions. What is your next step in management? A. Amiodarone 5 mg/kg intravenously B. Continued CPR according to the PEA algorithm, as this was the original rhythm C. Defibrillation for VF of 2 J/kg D. Synchronized cardioversion for VF: 0.5–1 J/kg Preferred response: C

Rationale Though they encompass a smaller proportion of arrest rhythms relative to adults, ventricular fibrillation (VF) and pulseless ventricular tachycardia (pVT) do occur in children and require vigilance by clinicians to identify and appropriately treat. Moreover, a substantial number of children have VF or pVT as a subsequent cardiac arrest rhythm—that is, they initially have a nonshockable rhythm for which CPR is initiated and then develop VF/pVT. Therefore constant reassessment during resuscitation is paramount. In addition to ongoing high-quality CPR, the first-line treatment of VF or pVT is defibrillation, following which CPR should be immediately resumed. If two defibrillation attempts are unsuccessful, amiodarone or lidocaine should be administered, in addition to ongoing defibrillation attempts and CPR. Minimization of peri-defibrillation interruptions in chest compressions has been associated with improved outcomes.

e60

S E C T I O N XV   Pediatric Critical Care: Board Review Questions

5. A healthy 12-year-old is playing baseball and is hit in the center of the chest by a pitch. He suddenly collapses and loses consciousness, unresponsive to stimulation, with an occasional gasp (agonal) breath. When paramedics arrive, the first ECG rhythm they are likely to see, is: A. Asystole B. Pulseless electrical activity C. Sinus bradycardia D. Ventricular fibrillation Preferred response: D

Rationale Following a sharp blow to the chest with sudden collapse, called “commotion cordis,” the most common cause of arrest is an R on T phenomena leading to ventricular fibrillation. If an AED (automated external defibrillator) had been applied prior to EMS arrival, a shock would have been advised. 6. A 12-year-old with severe asthma has respiratory distress unresponsive to albuterol aerosols and oxygen. The child becomes confused, then unconscious and unresponsive. The Spo2 was 90%, then 70% and now is not picking up. There is no pulse palpable and the arterial catheter tracing is flat (nonpulsatile) on the monitor. The child has an occasional gasp (agonal) breath. The ECG rhythm on the monitor is most likely: A. Pulseless electrical activity B. Sinus bradycardia C. Ventricular fibrillation D. Ventricular tachycardia Preferred response: A

Rationale This child has progressive respiratory failure due to lower airway obstruction (asthma) with severe arterial desaturation, and has progressed to a pulseless rhythm, with evidence of no perfusion and no pulse (by palpation or arterial line). This is most likely pulseless electrical activity. One should review the H’s and T’s for potential causes of PEA, with high suspicion for pneumothorax in a severe asthmatic. 7. During cardiopulmonary resuscitation (CPR), which of the following bedside monitoring tools is most predictive of return of spontaneous circulation (ROSC)? A. Exhaled CO2 capnograph of .15 Torr B. Femoral pulse palpable with chest compressions C. Pacer spikes visible on electrocardiogram when transcutaneous pacing is initiated D. Pulse oximeter waveform detected with chest compressions Preferred response: A

Rationale During the low-flow phase of CPR, achieving optimal cardiac output/coronary perfusion pressure is consistently associated with an improved chance of return of spontaneous circulation. Bedside capnography can be useful as a rough estimate of pulmonary blood flow. Achieving an optimal exhaled carbon dioxide concentration .15 Torr has been associated with improved short-term outcome in both animal and human studies.

8. A previously healthy 7-year-old boy under evaluation for syncope suddenly collapses. His cardiac arrest rhythm looks like ventricular fibrillation (VF) on the monitor. If high-quality standard CPR was provided (with chest compressions and 100% fraction of inspired oxygen rescue breathing), an arterial blood gas (ABG) and venous blood gas (VBG) drawn just prior to defibrillation would most likely show: A. ABG: pH 7.10, Pco2 65, Po2 300 VBG: pH 7.35, Pco2 65, Po2 40 B. ABG: pH 7.10, Pco2 55, Po2 50 VBG: pH 7.10, Pco2 55, Po2 50 C. ABG: pH 7.05, Pco2 25, Po2 20 VBG: pH 7.25, Pco2 55, Po2 35 D. ABG: pH 7.30, Pco2 30, Po2 300 VBG: pH 7.10, Pco2 75, Po2 30 Preferred response: D

Rationale Provision of high-quality CPR (i.e., push hard, push fast but not too fast, allow full chest recoil, minimize interruptions, and don’t overventilate) can result in cardiac output that approaches 50% of normal. The other choices have either inappropriately low Po2 levels given this scenario or a venous pH that is higher than the one recorded from the corresponding ABG.

Chapter 40: Structure and Development of the Upper Respiratory System 1. A child with 22q11 deletion syndrome and congenital heart disease presents with a difficult intubation due to a laryngeal web. Which abnormality in the normal larynx embryogenesis caused this to occur? A. Atresia as the most common result of failure to recanalize B. Cartilages and muscles derived from third and fourth branchial arches C. Derivation of epithelium from mesoderm of the laryngotracheal tube D. Formation of epiglottis by mesenchyme proliferation of the fourth and sixth branchial arches E. Temporary occlusion of larynx until the 10th week when recanalization occurs Preferred response: E

Rationale As epithelium proliferates rapidly, the temporary occlusion of the larynx ends by the 10th week when recanalization occurs. Atresia is the least common result of failure to recanalize. Laryngeal webs and/ or stenosis may be common results of failure to recanalize. Branchial arches involved in embryogenesis of the larynx are as follows: cartilages and muscles are derived from the fourth and sixth branchial arches, whereas the third and fourth branchial arches form the epiglottis by mesenchyme proliferation. Derivation of epithelium is from endoderm of the laryngotracheal tube, not mesoderm.

CHAPTER 136  Board Review Questions

2. A newborn intubation maybe challenging due to the anterior superior location of the larynx compared to a toddler. What oropharyngeal changes with growth and development account for this? A. Growth in the oropharynx is primarily in the anteroposterior direction. B. Lateral walls of the pharynx consist of a pair of constrictor muscles innervated by cranial nerves V, IX, and X. C. Oropharynx is the crossroads for the soft palate above and the hypopharynx below. D. The oropharynx is prominent in young infants. E. The oropharynx is first evident in children between ages 2 and 3 years. Preferred response: E

Rationale In young infants there is no defined oropharynx. The nasopharynx and hypopharynx are contiguous. Over the first 2–3 years, growth is primarily in the vertical direction such that a distinct oropharynx becomes evident, usually between 2 and 3 years of age. Three pairs of constrictor muscles are seen in the lateral walls of the pharynx, not just one pair. The innervation is via cranial nerves, V, IX, and X. The oropharynx forms the crossroads for the nasopharynx above and the hypopharynx below. The soft palate is the muscular extension of the bony hard palate and a critical structure for occlusion of the nasal cavity while eating and drinking, as well as retraction is important for speech production. 3. What is narrowest part of the upper airway in infants? A. Larynx B. Nasal valve C. Nasopharynx D. Subglottis Preferred response: B

Rationale The structure of the upper airway differs in the infant, young child, and young adult. Preferential nasal breathing is present in neonates and persists until 6 months of age because of the highriding larynx in the neck with the soft palate and vallecula in close anatomic approximation. The nasal tip—in particular, the nasal valve area—is the area of highest resistance in the upper airway of the infant. 4. A pediatric patient requires intubation. Laryngoscopy shows an easy grade 1 view; however, the age-appropriate endotracheal tube is difficult to pass. Which is not a possible reason for this to happen? A. Laryngeal web B. Subglottic stenosis C. Tracheal stenosis D. Tracheomalacia Preferred response: D

Rationale Tracheomalacia would not obstruct passing of the endotracheal tube because it is weakness or external compression of the trachea. It may hinder ventilation but the endotracheal tube should be able to pass easily.

e61

5. What method can be used to reach a definitive diagnosis of a laryngeal cleft? A. Fiberoptic endoscopic examination of swallowing (FEES) B. Flexible nasopharyngoscopy C. Operative endoscopy with palpation of the larynx D. Videofluoroscopic swallow study (VFSS) Preferred response: C

Rationale To assess for a laryngeal cleft, palpation of the interarytenoid area is the gold standard.

Chapter 41: Structure and Development of the Lower Respiratory System 1. A baby is born at 24 weeks’ gestation and has a difficult respiratory course in the NICU. She eventually gets a tracheostomy for chronic ventilation and is transferred to the PICU at 6 months of age. Her oxygen requirement has been decreasing, and she is discharged home on a ventilator. Which of the following is a correct statement regarding the development of this child’s lower respiratory system? A. Alveolarization is complete in this child. B. The acinus is the gas exchange area of the lung. C. The pulmonary veins course with the airways and the bronchioles throughout the lung. D. Type I alveolar epithelial cells produce and secrete surfactant. Preferred response: B

Rationale Surfactant is produced by type II cells, not type I cells. Gas exchange occurs in the acinus. Alveolarization appears to be nearly complete at about age 2 years and so should continue to occur in this child. The pulmonary veins do not normally course with the airways and bronchioles. When they do, this suggests alveolarcapillary dysplasia. 2. The primary role of the lung is gas exchange. What structural feature of the lung supports this function? A. Alveolar macrophages are scarce in the normal lung. B. The connective tissue space, or interstitium of the lung at the alveolar level, has abundant lymphatics. C. The internal surface area of the adult lung is 70 to 80 m2, of which 90% covers the pulmonary capillaries. D. When fully matured the pulmonary artery and its thickness is approximately 90% that of the aorta. Preferred response: C

Rationale The internal surface area of the adult lung is 70 to 80 m2, of which 90% covers the pulmonary capillaries; thus the air-blood surface available for gas exchange is 60 to 70 m2. When fully matured, the pulmonary artery and its thickness is only about 60% that of the aorta. Alveolar macrophages are abundant and form an important arm of the defense mechanism of the lung. The connective tissue space, or interstitium of the lung at the alveolar level, does not have lymphatics, but it can accumulate fluid that can be absorbed into the lymphatic system, which ends usually at the respiratory bronchiolar level.

e62

S E C T I O N XV   Pediatric Critical Care: Board Review Questions

3. Which of the following is the most predominant bronchial mucosal cell type? A. Basal cells B. Brush cells C. Ciliated cells D. Neuroendocrine cells Preferred response: C

Rationale The bronchial mucosa contains several epithelial cell types: ciliated, mucus producing (goblet cells), basal, brush, and neuroendocrine. Ciliated cells constitute more than 90% of the epithelial cell population in the conducting airways, but the proportion and number of cilia per cell decrease from the proximal to the distal airways. In addition to its ciliary beating movement, the ciliated columnar cells regulate the depth of the composition of the periciliary fluid and transport ions across the epithelium. The basal cell has a progenitor cell role and also functions to maintain adherence of columnar cells to the basement membrane. The brush cell, thought to have a role in fluid absorption or chemoreceptor function, is found rarely in the tracheobronchial and alveolar epithelia. Although mucous goblet cells secrete mucin, it is the submucosal glands that produce more than 90% of the mucus needed for mucociliary function. Neuroendocrine cells can be solitary near the basal lamina between columnar cells or in collections called neuroepithelial bodies that occur near branch points of bronchi. A number of neural markers are expressed (e.g., 5-hydroxytryptamine, chromogranin A, neuron-specific enolase, and synaptophysin), and a number of hormones are produced (e.g., endothelin, calcitonin, and bombesin [gastrin-releasing peptide]). They are more abundant in the fetus and likely have a role in lung growth or maturation. 4. It is not known with certainty when alveolar development is completed. However, based on our current knowledge, we believe which of the following statements? A. All alveoli are present at birth. B. All alveoli are present at 16 weeks’ gestation. C. Alveoli continue to develop until 2 to 8 years of age. D. Alveoli continue to develop into adulthood. Preferred response: C

Rationale At birth, primitive alveoli called saccules are evident, but approximately 50 million alveoli are already formed. The number of alveoli in a normal adult can vary from 300 to 500 million, and they have a diameter of 150 to 200 µm. The early work by Dunnell suggesting that new alveolar formation ceased at about age 8 years has been challenged by Thurlbeck, who has shown that alveolarization appears to be nearly complete at about age 2 years. Lung volume correlates with body size, but alveolar surface area correlates with metabolic activity; thus alveoli become more complex in shape during maturation and as increasing O2 is required.

Chapter 42: Physiology of the Respiratory System 1. Regarding respiratory physiology, which one of the following statements is least accurate? A. Peripheral airway resistance in children ,5 years is fourfold higher than in older children and adults. B. Specific compliance is the same for adults and children, but specific conductance is higher in children. C. With laminar flow, resistance to flow is proportional to viscosity. D. With turbulent flow, resistance to flow is proportional to density. Preferred response: B

Rationale Compliance is a measure of the elastic nature of the chest wall and lungs. In children, due to decreased development of the structure of the lungs and developing calcification of the ribs, the compliance of both the lungs and chest wall is decreased. 2. Which one of the following clinical conditions is not expected to be associated with sudden decline in end-tidal CO2 as it relates to the alveolar gas equation? A. Air embolism B. Cardiac standstill C. Hypoventilation D. Obstruction of the endotracheal tube Preferred response: C

Rationale The volume of air entering the lungs each minute that actually participates in gas exchange is called the alveolar ventilation (A). It is therefore the difference between the total volume of air entering the lungs each minute (minute ventilation, E) and the volume of air entering the lungs that does not participate in gas exchange (dead space: D): A 5 E 2 D. Anything decreasing the amount of CO2 seen by the end-tidal sensor will decrease the level displayed, but a sudden decrease represents a sudden change, whereas hypoventilation would demonstrate a gradual increase. 3. Lack of oxygen equilibration due to diffusion limitation (alveolarcapillary block) can be evaluated by measuring which of the following? A. Diffusing capacity of CO (carbon monoxide) B. Diffusing capacity of CO2 (carbon dioxide) C. Distribution of an inhaled gas mixture containing a radioactive marker D. FEV1/FVC when inhaling pure oxygen Preferred response: A

Rationale DCO is a good index of the diffusion capacity of oxygen (Do2). 4. Reduction of the pulmonary diffusing (D) capacity to onefourth of its normal value would be expected to have what effect on systemic arterial oxygen and carbon dioxide partial pressures (compared to normal)? A. Decrease Pao2 and decrease Paco2 B. Decrease Pao2 but no change in Paco2 C. Increase Pao2 and decrease Paco2 D. Increase Pao2 and increase Paco2 Preferred Response: B

CHAPTER 136  Board Review Questions

e63

Rationale Pao2 decreases when the diffusion capacity of oxygen (Do2) decreases to less than one-third its normal value. But Do2 is so high normally that even a decrease to one-fourth will still permit carbon dioxide to equilibrate in the time that blood passes through pulmonary capillaries.

3. A standard pulse oximeter is able to detect: A. Carboxyhemoglobin B. Methemoglobin C. Oxygenated hemoglobin only D. Oxygenated and deoxygenated hemoglobin Preferred response: D

Chapter 43: Noninvasive Respiratory Monitoring and Assessment of Gas Exchange

Pulse oximetry is based on the observation that the attenuation of light passing through blood-perfused tissue changes with pulsation of blood and that the alternating component of the light attenuation results from the composition of arterial blood. Hemoglobin has characteristic light-absorbing properties that change with oxygen binding. The deoxy form of hemoglobin (deoxyHb) has a single peak in the visible and near-infrared region. Oxyhemoglobin (oxyHb) has two peaks in the visible region but no significant peak in the near-infrared region. At any given wavelength, there is a difference in absorption between oxyHb and deoxyHb except where the spectra cross, at wavelengths called isosbestic wavelengths, where the absorption is the same for each state. At nonisosbestic wavelengths, the difference in absorption can be used to determine the fraction of oxyhemoglobin. Saturation of hemoglobin is defined as follows:

1. Which of the following is least likely associated with a source of error when measuring oxygen saturation via pulse oximetry (Spo2)? A. Increased proportion of the oxidized form of hemoglobin B. Presence of fetal hemoglobin (HgbF) C. Probe placement on a digit of a patient immediately status post–cold water submersion D. Probe placement on the right second digit of a patient experiencing bilateral upper extremity tonic-clonic seizure activity Preferred response: B

Rationale Fetal hemoglobin and adult hemoglobin have nearly identical absorption. Studies have demonstrated that Spo2 measurements by pulse oximetry are unaffected by the presence of HgbF in neonates. All other choices are associated with known sources of error for pulse oximetry readings. Specifically, patient movement (such as seizure activity in answer D) can be associated with poor signal and inaccurate pulse oximetry readings. Additionally, local perfusion and patient temperature may affect the probe’s ability to measure Spo2 due to alterations in the local pulsatile blood flow. In patients with cold extremities and significant peripheral vasoconstriction (expected in a patient status post–cold water submersion in answer C), perfusion to the digits is likely significantly impaired and will affect the accuracy of pulse oximetry readings. 2. Which of the following is true regarding near-infrared spectroscopy (NIRS)? A. NIRS measures only light absorbed by hemoglobin. B. NIRS measurement is not affected by skin temperature. C. Values from NIRS may be helpful for trending a patient’s oxygenation and hemodynamic status. D. When placed on the forehead, NIRS provides a measurement that is equivalent to the mixed venous saturation. Preferred response: C

Rationale Near-infrared spectroscopy (NIRS) is a noninvasive method utilized to measure the Hgb-oxygen saturation of a local region of interest. This is achieved through the use of multiple (2–4) wavelengths of near-infrared light directed via a cutaneous probe into the underlying tissue, which are absorbed by pigments such as myoglobin, hemoglobin, and cytochrome. Studies have reported varying degrees of accuracy for cerebral NIRS oximeters and have also found large variation in reading errors. Extracranial tissue changes due to superficial vasoconstriction secondary to vasoactive medication (e.g., norepinephrine, phenylephrine) or sympathetic response to pain or hypothermia can be additional sources of error with cerebral NIRS oximetry. Based on these findings, many authors recommend the use of cerebral oximetry as trend monitors and not absolute tissue oxygenation measures or injury threshold determinants.

Rationale

Hbsat  [OxyHb]/([OxyHb]  [DeoxyHb]) where Hbsat 5 fractional saturation of hemoglobin, [oxyHb] 5 concentration of oxyhemoglobin, and [deoxyHb] 5 concentration of deoxyhemoglobin. Hemoglobin percent saturation, as commonly reported, is determined by multiplying Hbsat by 100. 4. Which of the following are the most reliable sites for monitoring rapidly changing core temperature? A. Distal esophagus, tympanic membrane, pulmonary artery, and nasopharynx B. Distal esophagus, pulmonary artery, bladder, and rectum C. Skin, bladder, and rectum D. Skin, bladder, and nasopharynx Preferred response: A

Rationale Commonly used core temperature monitoring sites include the distal esophagus, tympanic membrane, pulmonary artery, and nasopharynx. These sites detect core temperature changes rapidly, in contrast to urinary bladder or rectal measurements, which are good reflections of core temperature during steady-state conditions. Cutaneous temperature monitoring is the least reliable indicator of rapid core temperature changes. However, monitoring peripheral temperatures can be useful in defining core peripheral gradients in temperature and assist in tracking vasoconstriction and vasodilation. Oral probes are used as thermometers, and some have been attached to pacifiers. A thermometer that scans the temporal artery is also available. The ideal spot for continuous core temperature monitoring is a pulmonary artery catheter, but because of the invasive nature of this monitor, it would never be placed for temperature monitoring alone. An esophageal temperature probe positioned in the lower third of the esophagus is a good alternative. In this position, the temperature sensor is immediately behind the left atrium and accurately tracks core temperature without significant time lag in the majority of situations. If a gastric tube with applied suction is present next to the temperature probe, it must be on the low intermittent setting or the temperature readings will be falsely

S E C T I O N XV



Pediatric Critical Care: Board Review Questions

lowered. Nasopharyngeal and tympanic membrane temperatures are good indicators of cerebral temperature but can be inaccurate as a result of sensor positioning. Axillary and peripheral skin probably are the most convenient sites for monitoring temperatures, but they also are the most inaccurate because of skin perfusion. 5. You are leading resuscitation for a 2-year-old boy who suffers a bradycardic cardiac arrest requiring cardiopulmonary resuscitation (CPR). He receives approximately 2 minutes of highquality CPR and then a code dose of epinephrine. After approximately 30 seconds you noticed a change in his end-tidal CO2 waveform (Figure 136.16, below). What does this change in capnography most likely indicate? A. Dislodgement of the endotracheal tube B. Inadequate depth of chest compressions C. Presence of lower airway obstruction D. Restoration of pulmonary blood flow through return of spontaneous circulation (ROSC) Preferred response: D

ETCO2

40

15

Time

• Fig. 136.16





CO2 (mm Hg)

e64

50 37

Real-time

Trend

0

• Fig. 136.17





Rationale Incorrect placement of the endotracheal tube in the esophagus results in initial detection of trace PETCO2, which is not sustained over time, and an uncharacteristic waveform that lacks a defined respiratory upstroke, plateau, or inspiratory downstroke. Hypoventilation may be indicated by a gradual rise of PETCO2, while airway obstruction is demonstrated by a change in the shape of the capnography waveform (absence of expiratory or alveolar plateau). Sudden loss of PETCO2 with a waveform that transitions from normal to flat line is an indication of laryngospasm or apnea.

Chapter 44: Overview of Breathing Failure 1. Compared with adults, neonates are more predisposed to respiratory failure because: A. Airway resistance is lower in neonates. B. Neonates have a greater proportion of type 1 fibers in the diaphragm. C. The neonatal diaphragm has a greater angle from the vertical than the adult. D. The neonatal chest wall is less compliant than the adult. E. The ribs are more vertical than the adult. Preferred response: C

Rationale Capnography has been shown to be a useful tool in many clinical situations. Recent studies have demonstrated that use of end-tidal CO2 monitoring during CPR can help improve the quality of CPR and provide insight into the patient’s physiology. Recent Advanced Cardiac Life Support (ACLS) and PALS guidelines recommend using capnography to monitor the effectiveness of chest compressions during CPR. A sudden decrease in PETCO2 is seen with loss of pulmonary blood flow in cardiac arrest. Increasing PETCO2 values generated during CPR are associated with chest compression depth and ventilation rate, and an acute rise in PETCO2 exceeding 10 mm Hg is seen with return of spontaneous circulation. Patients with ROSC after CPR have statistically higher levels of PETCO2, suggesting better lung perfusion and cardiac output. Current guidelines suggest achieving a threshold of 10 to 15 mm Hg to ensure adequate delivery of chest compressions, although an average PETCO2 level of 25 mm Hg was found in patients with ROSC in a recent systematic review and meta-analysis. In the above waveform, we see initial PETCO2 values of approximately 15–20 mm Hg, suggesting adequate chest compression depth. We then see an acute rise in PETCO2 up to levels of 40 mm Hg and above. This acute rise is characteristic of increased pulmonary blood flow and is suggestive of achieving ROSC.

Rationale Neonates are at a number of mechanical disadvantages for breathing when compared with older children and adults. Because the neonatal chest wall is more compliant than the adult, the ribs recoil more and are more horizontal. This in turn promotes the diaphragm to be less apposed to the chest wall and a greater angle of the diaphragm to the chest well. Both of these make the diaphragm less efficient. These effects all act together to promote restrictive lung disease in the normal neonate. Therefore, they are more tachypneic at baseline and susceptible to any further restrictive process. In addition, neonates have smaller airways, which greatly increases resistance to airflow and predisposes to more severe obstructive airway disease (both inspiratory and expiratory). Airway resistance is inversely proportional to the fourth power of the radius of the airway. (R5 where h is gas viscosity, L is airway length, and r is radius of the airways). Thus, small changes in the airway caliber due to edema or mucus can cause severe increase in the respiratory work a neonate must perform. Because the neonatal diaphragm is composed of a greater proportion of type 2 fibers, it tolerates prolonged loads less well and becomes fatigued more easily. Type 1 fibers, which are more prevalent in the adult and well-trained diaphragm, resist fatigue better than type 2.

6. Which of the following clinical scenarios best explains this capnography waveform (Figure 136.17)? A. Airway obstruction B. Apnea C. Esophageal intubation D. Hypoventilation Preferred response: C

2. Of the following, which is a cause of central hypoventilation? A. Botulism B. Cerebral palsy C. Metabolic acidosis D. Phox2B mutation E. Tetrodotoxin Preferred response: D

CHAPTER 136  Board Review Questions

Rationale Central hypoventilation is caused by a decreased stimulus, or altered threshold, for respiratory rhythm generators in the central nervous system. Patients with the congenital central hypoventilation syndrome lack normal automatic control of breathing due to mutations in Phox2B, which is expressed in the retrotrapezoid nucleus of the medulla, one of the major chemosensitive regions controlling ventilation. Because chemosensitive regions in both the periphery and the brain sense pco2 through local changes in pH, metabolic acidosis increases the drive to breathe, as opposed to decreases it. Patients with central hypoventilation lack effort despite increased CO2 tension. Disorders of the peripheral nervous system and muscles may also have inadequate respiratory effort. Tetrodotoxin, via inhibition of nerve impulses, suppresses breathing, though the central drive is intact. Similarly, botulism inhibits neurotransmission at the motor endplate and prevents muscle response to central stimuli. Patients with cerebral palsy have a number of factors that predispose to respiratory failure. These include increased (and inappropriate) airway tone as well as scoliosis and other chest wall deformities that occur over time. However, these patients nearly always have adequate respiratory drive at baseline. A patient with cerebral palsy who lacks adequate respiratory drive should be evaluated for other causes (i.e., seizures, sepsis, drug overdose). 3. What is the primary mechanism by which shock causes respiratory failure? A. Cortical brain ischemia. B. Inadequate muscle blood flow. C. Metabolic acidosis. D. Overstimulation of medullary respiratory centers. Preferred response: B

Rationale In shock, blood flow to respiratory muscles becomes inadequate for aerobic muscle work. Respiratory muscles first develop an oxygen debt and then lose their ability to contract. Metabolic acidosis stimulates chemoreceptors and brainstem respiratory sites, causing tachypnea and dyspnea, but not preventing muscle work. The high respiratory muscle workload of respiratory failure does increase muscle blood flow to the extent supportable by the circulation and is a strain on the heart, but it is not the cause of respiratory failure in shock. Cortical brain ischemia might impair volitional responses to respiratory distress, but it is the brainstem that governs automatic responses to respiratory stimuli. Overstimulation of medullary respiratory centers may drive tachypnea but is not the cause of respiratory muscle failure from shock. 4. A child fell while rock climbing and suffered injury to his thoracic spinal cord at the level of T6. He has lower extremity flaccidity and a sensory level at the lower sternum and at the level of T6 posteriorly. What would be the greatest impact of this injury on breathing? A. Dependence on accessory muscles of respiration B. Diminished strength of cough C. Loss of the cough reflex D. Reduced inspiratory capacity Preferred response: B

e65

Rationale The muscles of the abdomen are used for active expiration and are innervated by the lower thoracic spinal cord. They are important for coughing, sneezing, and for forced expiration, but play little role in passive expiration. The cough reflex involves abdominal muscles as well as deep intercostals, which are innervated by both upper and lower thoracic motor neurons. The reflex would be intact despite a T6 injury, but the cough itself would be weak. Inspiration is powered by the superficial and parasternal intercostal muscles and by the accessory muscles of respiration, so inspiratory capacity should be preserved. Dependence on accessory muscles for normal inspiratory power should not be necessary. Retractions reflect inspiratory effort, which should not be directly altered by this injury. 5. Which of the following is a true statement about control of breathing? A. Aortic and carotid chemoreceptors directly modulate respiratory drive. B. Brainstem respiratory control is located in a single medullary nucleus. C. Volitional and automatic control of breathing may be independently impaired. D. Volitional control of breathing requires medullary modulation. Preferred response: C

Rationale Volitional control of respiration normally may be asserted in the awake state, but only up to a point. One cannot consciously hold the breath beyond a certain level of medullary stimulation. Supratentorial injury may impair volitional control of breathing but spare automatic control. On the other hand, medullary stroke may impair automatic control of breathing but leave volitional control intact. Brainstem respiratory control is widely distributed in the pons and medulla and involves numerous other brainstem nuclei. All mechanoreceptor and chemoreceptor input that modulates respiratory drive must be processed in the brain before effector responses can occur. Aortic and carotid chemoreceptors modulate breathing only after integration in the medulla. Patients with loss of all cortical behavior may exhibit respiratory distress and use accessory muscles of breathing. 6. Which of the following is a true statement about respiratory muscle exhaustion? A. Myoglobin provides adequate oxygen reserves to protect against respiratory muscle exhaustion from hypoxemia. B. Muscle training can prevent fatigue of respiratory muscles. C. Rapid shallow breathing is a sign of respiratory muscle exhaustion. D. Shock may precipitate respiratory muscle exhaustion and respiratory arrest. Preferred response: D

Rationale Shock can diminish oxygen supply to the respiratory muscles and precipitate muscle exhaustion. Respiratory arrest is a final common pathway to death in all forms of untreated shock. Hypoxemia also can cause respiratory muscle exhaustion. Although myoglobin may provide a short-term buffer to transient oxygen deficits, it cannot prevent persistent hypoxia or blood flow limitation from progressing to respiratory muscle exhaustion.

e66

S E C T I O N XV



Pediatric Critical Care: Board Review Questions

Krebs cycle production of adenosine triphosphate is very inefficient. Most Krebs cycle adenosine triphosphate is generated by oxidation of Krebs cycle reducing fragments. When there is insufficient oxygen to feed oxidative phosphorylation, reducing fragments build up and inhibit Krebs cycle enzymes. Anaerobic metabolism cannot prevent respiratory muscle exhaustion. Muscle training improves the ability of respiratory muscles to meet excessive demand, although only to a point. Rapid shallow breathing is a compensatory mechanism to deal with elevated work of breathing and occurs before respiratory muscles become exhausted. 7. Which of the following values is the least likely to stimulate the peripheral or central chemoreceptors, resulting in an increase in respiratory rate? A. Low hemoglobin B. Paco2 C. Pao2 D. pH Preferred response: A

Rationale Hypoxemia is a powerful stimulus to ventilation mediated by sensory input originating in the carotid body chemoreceptor. Peripheral chemoreceptor activity (reflected in minute ventilation) increases slightly with decrements in Pao2 below 500 mm Hg and rises steeply as Pao2 falls below 50 mm Hg. Low oxygen tension, rather than low oxygen content, is the important ventilator stimulus. Little carotid body response results from profound anemia. Carotid chemoreceptor response also contributes to arousal from sleep during episodes of hypoxia. Hydrogen ion concentration and carbon dioxide tension independently activate chemoreceptors in the carotid body and in the brainstem. The simultaneous presence of hypoxia augments the hypercapnic ventilatory response.

Chapter 45: Ventilation/Perfusion Inequality 1. The primary abnormality of gas exchange in all patients with respiratory distress syndrome (ARDS) include A. Decreased alveolar-end capillary diffusion capacity B. Decreased ventilation/perfusion (V/Q) matching C. Increased intrapulmonary shunt D. Hypoventilation Preferred response: C

Rationale The primary gas exchange abnormality of ARDS is intrapulmonary shunt. Some, but not all, patients will have areas of low V/Q in addition to shunt. Consequently, increases in the fraction of inspired oxygen (Fio2) usually have little influence on arterial oxygenation. Patients with ARDS do not suffer from abnormalities in alveolar-end capillary diffusion capacity or hypoventilation unless there are other conditions complicating the ARDS. Patients with ARDS do experience elevated alveolar Pco2 and hypercapnia, which may be made worse if excessive levels of positive-end expiratory pressure (PEEP) are employed. According to the alveolar gas equation, elevations in the alveolar partial pressure of carbon dioxide (Paco2) will result in a fall in alveolar and arterial oxygenation. These decrements are small, responsive to increases in Fio2, and not the primary cause of hypoxemia in patients with ARDS.

2. The abnormalities of gas exchange and the site of airflow obstruction in patients with asthma are, respectively: A. Atelectasis and distal airways B. Decreased alveolar-end capillary diffusion capacity and distal airways C. Decreased ventilation/perfusion (V/Q) matching and proximal airways D. Hypoventilation and distal airways E. Intrapulmonary shunt and proximal airways Preferred response: C

Rationale The primary gas exchange abnormality of asthma is V/Q mismatch which occurs in the distal airways due to edema and/or mucus production while airflow obstruction occurs due to bronchoconstriction in larger more proximal airways. Although the same pathology produces the two pathophysiologic mechanisms, no correlation exists between measurements of airway obstruction and gas exchange. Bronchodilators may augment flow to areas of low V/Q match, temporarily worsening hypoxemia. 3. Which of the following is most accurate regarding West zones of the lung A. In zone 2, perfusion is determined by the difference between alveolar pressures and pulmonary venous pressures. B. In zone 3, the greatest pressure is the alveolar pressure. C. Zone 1 approximates dead space areas of the lung. D. Zone 1 conditions are commonly seen with increased pulmonary blood flow. Preferred response: C

Rationale The West zones describe the forces affecting pulmonary blood flow (PBF), which are determined by the gravitational influences on the lung. In zone 1, the alveolar pressure (PA) exceeds both the pulmonary arterial (PPA) and venous pressures (PV). In this zone there is little to no blood flow so it approximates dead space, but such conditions are rare except in cases of diminished PBF. In zone 2, PPA exceeds PA, which is greater than PV, and perfusion in this zone is determined by the pressure difference between alveolar and pulmonary venous pressures. In zone 3, the pressures descend from PPA to PV and finally PA. 4. Shunt fraction can be calculated. Which of the following data would be required? A. Arterial oxygen content, alveolar-arterial oxygen difference, cardiac output B. Arterial oxygen content, venous oxygen content, assumption of oxygen content of blood undergoing gas exchange C. Cardiac output, arterial oxygen saturation, and oxygen consumption D. Dead space fraction, minute ventilation, and cardiac output Preferred response: B

Rationale The equation for calculating the shunt fraction is based on the Fick equation Q s / Q t   

C c O2   C a O2 C c O2  C V O2

CHAPTER 136  Board Review Questions

where Qs and Qt represent shunt and total pulmonary blood flow, respectively, and CaO2, CvO2, and CcO2 are the arterial, venous, and pulmonary capillary oxygen contents, respectively. The dead space fraction can be derived from measurement of mixed endtidal CO2 and arterial Pco2 but is not needed for calculation of shunt fraction. Minute ventilation also is not needed for the calculation of the shunt fraction, but total cardiac output is part of the equation as seen above. A similar equation employing carbon dioxide contents in place of oxygen contents could be constructed. Although cardiac output is derived in the calculation of shunt fraction, response C is incomplete as the equation also requires contents. Alveolar ventilation and barometric pressure are not required for the calculation. Response A has only one of the necessary variables for the equation. Establishment of a significant shunt fraction will inform the clinician that there will be decreased alveolar-arterial oxygen difference locally. 5. Regarding the effect of intrapleural pressures and alveolar size, which of the following statements is accurate? A. The intrapleural pressure at the apex is more negative than at the base, so the alveoli are larger at the apex. B. The intrapleural pressure at the apex is more positive than at the base, so alveoli are smaller at the apex. C. The intrapleural pressures do not vary between the apex of the lung and the base and therefore do not affect alveolar size. D. The intrapleural pressure at the base is more positive than at the apex, so alveoli are larger at the base. Preferred response: A

Rationale The lung is a viscoelastic structure encased in the supporting chest wall with gravity imposing a globular shape on the lung. Pleural pressure is more negative at the apex of the lung compared with the base, increasing approximately 0.25 cm H2O per centimeter of vertical distance toward the lung base. Thus transpulmonary pressure is more marked at the apex so apical alveoli are large and at the upper end of the normal pressure-volume curve. They distend less for a given pressure change, that is, they are less compliant. In the spontaneously breathing upright human, maximal gas distribution occurs at the base and progressively diminishes toward the lung apex. This gradient also exists when inhalation occurs in the supine or lateral decubitus position, although to a lesser degree. 6. A 7-year-old child with status asthmaticus is undergoing treatment in your pediatric intensive care unit with systemic corticosteroids, b2-agonists, ipratropium, and 0.60 fraction of inspired oxygen. She has moderate air entry, bilateral wheezes, no nasal flaring, and mild intercostal retractions. Her respiratory rate is 22 per minute. Her pulse oximetry saturations prior to and after initiation of therapy were 91% and 86%, respectively. Which of the following is the most likely explanation for this observed change in oxygen saturation? A. Excessive fatigue with hypoventilation and resultant hypoxemia B. Increase in airway secretion due to the institution of ipratropium C. Increase in ventilation/perfusion mismatch due to b2agonist D. Mucus plugging of the airways due to institution of ipratropium Preferred response: C

e67

Rationale In persons with asthma, high inspired oxygen concentrations may prevent hypoxic pulmonary vasoconstriction and place low alveolar ventilation (Va)/perfusion (Q) regions at risk for absorption atelectasis, and high doses of bronchodilators may enhance the perfusion of low Va/Q areas, exacerbating Va/Q mismatch. However, the beneficial effects of bronchodilators on airway resistance generally outweigh the worsening in Va/Q mismatch. This child is not showing signs of excessive fatigue. There is no nasal flaring, and retractions are only mild. The air entry is moderate and wheezes are present, therefore response A is not correct. Ipratropium is an anticholinergic that causes a decrease in airway secretion (thus response B is incorrect), and there are no clinical signs that this child has a mucus plug in her airway (thus response D is incorrect). 7. One of the proposed mechanisms for improvement in oxygenation in patients with acute respiratory distress syndrome (ARDS) placed in the prone position is: A. Perfusion is greater in the nondependent areas in the prone position. B. Perfusion is greater in the dependent areas in the prone position. C. Ventilation is greater in the dependent areas in the prone position. D. Ventilation is lower in the nondependent areas in the prone position. Preferred response: A

Rationale In ARDS and other lung injury models, nonaerated or poorly aerated portions of the lung are found mainly in the dependent areas. Perfusion is largely gravity-independent, especially in West zone 3 conditions. The majority of perfusion goes through dorsal lung regions, whether in the prone or supine position. Consequently, perfusion is greatest to the dependent lung in the supine position and to the nondependent lung in the prone position. Positive pressure, especially positive end-expiratory pressure, redistributes perfusion toward the dependent portion of the lungs by creating the condition of West zones 2 and 1. This redistribution may increase the vertical perfusion gradient in the supine position but may reduce it in the prone position. These various physiologic factors contribute to the increase in the uniformity of perfusion in the prone position. 8. You are caring for a 4-month-old child with bronchiolitis who has developed respiratory failure. You instruct the medical student rotating on your service that bronchiolitis has features of both restrictive and obstructive lung diseases. Common pulmonary abnormalities that occur in both restrictive as well as obstructive lung diseases are: A. Altered closing capacity especially with respect to functional residual capacity (FRC) B. Increased intrapulmonary shunt fraction C. Increased time constants D. Increased zone 3 pulmonary blood flow conditions Preferred response: A

Rationale In almost all lung diseases alterations in closing capacity and functions residual capacity (FRC) occur resulting in altered gas exchange. Obstructive lung diseases (e.g., asthma) are characterized by increases in FRC and TLC, whereas in restrictive lung diseases

e68

S E C T I O N XV   Pediatric Critical Care: Board Review Questions

(e.g., ARDS) both capacities are decreased. However, closing capacity changes as well and so in asthma there is increased Va/Q mismatch, whereas in ARDS there is increased intrapulmonary shunt. Many other lung diseases result in Va/Q mismatch or increased intrapulmonary shunt, and some of these diseases are discussed further in the chapter. Increased alveolar-capillary diffusion time is seen in diseases that result in abnormalities of the alveolar-capillary interface, such as pulmonary fibrosis, and is not a defining feature of either obstructive or restrictive lung diseases. The time constant of lung segments is defined by the equation

   R  C where the time constant (t) is the product of resistance (R) and compliance (C) or R divided by elastance (E), the inverse of compliance. In general the time constant is increased in obstructive lung diseases and decreased in restrictive lung disease. However, lung diseases tend to be inhomogeneous and so some local areas may have time constants that differ markedly from the lung as a whole. This is especially true in bronchiolitis. Finally, zone 3 in the West model of pulmonary blood flow occurs when pulmonary arterial and venous pressures exceed pulmonary alveolar pressures. In obstructive lung disease, airways become hyperinflated, increasing alveolar pressure, and this will likely diminish the amount of zone 3 areas. Similarly, in restrictive diseases, there is loss of lung volume or pulmonary edema that increases the amount of zone 4 regions. 9. A 7-month-old girl with severe pneumococcal pneumonia develops respiratory failure requiring intubation of her trachea and institution of mechanical ventilation. Shortly after intubation an arterial blood gas shows the following results: pH 7.32, paco2 36 mm Hg, pao2 57 mm Hg on and Fio2 of 0.6. Her pulse oximeter reads 88%. Explanation for differences in the abnormalities seen in the levels of carbon dioxide and oxygen include: A. Carbon dioxide has greater diffusion in blood than oxygen does. B. The difference is entirely explained by the alveolar gas equation. C. The greater density of CO2 results in better gas exchange in gravitationally dependent areas of the lung. D. The hemoglobin-oxygen binding curve has limited influence on blood CO2 levels. Preferred response: D

Rationale Carbon dioxide does have greater density than oxygen and does diffuse more readily than oxygen. However, neither of these features of carbon dioxide would explain the differences in levels of oxygen and carbon dioxide in this patient with pneumonia. In patients with diffusion abnormalities (e.g., pulmonary fibrosis), especially in states of increased cardiac output, pulmonary capillary blood will equilibrate with the CO2 in alveolar gas, but equilibrium may not be reached with the oxygen in alveolar gas. Still, pneumonia is not distinguished by decreased diffusion capacity. In patients with gas intrapulmonary shunt or Va/Q mismatch abnormalities the carbon dioxide levels in affected pulmonary capillary units will be elevated, but often this can be corrected by increases in alveolar minute ventilation. In more normal areas the CO2 in the pulmonary capillary blood is below normal and corrects the high CO2 of these areas when the blood mixes in the large pulmonary veins, and this probably is the case for this infant. In contrast, since the majority of

oxygen in the blood is carried by hemoglobin, areas of abnormal gas exchange are characterized by pulmonary capillary blood that is not fully saturated. This cannot be corrected by less diseased areas of the lung because the small amount of dissolved oxygen in areas that are more normal will not improve the Po2 enough to bring the saturations of arterial blood to normal levels. 10. The correct statement regarding alveolar size and compliance in different lung regions in a healthy child in the upright position is: A. Apical alveoli are larger and therefore more compliant than alveoli at the base of the lungs. B. Alveolar size is the same at the apex and the base of the lungs, so compliance is equal. C. Alveoli at the base of the lungs are larger and therefore more compliant than apical alveoli. D. Alveoli at the base of the lungs are smaller and therefore more compliant than apical alveoli. Preferred response: D

Rationale Apical alveoli are large and at the upper end of the normal pressure-volume curve. They distend less for a given pressure change, that is, they are less compliant. Alveoli at the base are smaller and more compliant than apical alveoli. 1 1. What is the definition of time constant? A. The time required to inflate 63% of final lung volume and is equal to the product of resistance and compliance B. The time required to inflate 95% of final lung volume and is equal to the product of resistance and compliance C. The time required to inflate 63% of final lung volume and is equal to resistance/compliance D. The time required to deflate 95% of final lung volume and is equal to resistance/compliance Preferred response: A

Rationale The time constant (the product of resistance and compliance) is defined as the time required for inflation to 63% of final lung volume, inflation being indefinitely prolonged. Therefore a given lung unit with a slow time constant will fill more slowly than one with a fast time constant and also will empty more slowly. Should the time constants of different lung units vary, as frequently happens in pulmonary illness, gas distribution will be determined in part by the rate, duration, and frequency of inhalation. 12. In an upright individual with normal lung perfusion, which West zone represents a region where PA . PPV . PPA, where PA is alveolar pressure, PPA is pulmonary artery pressure, and PPV is pulmonary venous pressure? A. Zone I B. Zone II C. Zone III D. This representation does not exist Preferred response: D

Rationale The three-zone model of pulmonary blood flow has been widely used to explain the heterogeneity of perfusion within the lung.



CHAPTER 136 Board Review Questions

Three variables comprise the components of this model: pulmonary arterial (Ppa), alveolar (Pa), and pulmonary venous (Pv) pressures. The degree of blood flow within the lung depends on the relative magnitudes of these pressures within that zone. Zone 1 (Pa . Ppa . Pv) has negligible blood flow, because the higher alveolar pressure is believed to compress collapsible capillaries. This region is one of minimal gas exchange and “wasted” ventilation. Zone 1 conditions are rare except in cases of diminished pulmonary blood flow (e.g., hypotension and cardiac failure) or increased Pa encountered during positive pressure ventilation. Zone 2 consists of the mid portions of the lungs in which Ppa . Pa . Pv, where flow rate is determined by the difference between pulmonary arterial and alveolar pressure. Venous pressure does not influence the flow rate. Blood flow progressively increases with descent through this zone, because Ppa increases whereas Pa remains relatively constant. In the lowest zone of the lung described by West, zone 3, Ppa . Pv . Pa, therefore the arteriovenous pressure gradient (Ppa-Pv) determines flow rate. This gradient remains relatively constant descending through this zone, although because pleural pressures increase less, blood flow is greater in more dependent areas of zone 3. A zone 4 region in the most dependent areas of lung also has been described. In this region, transudated pulmonary interstitial fluid increases interstitial pressures, thereby reducing blood flow; this effect is exaggerated as lung volume diminishes from total lung capacity to residual volume. 13. Which West zone represents a region where Ppa . Pa . Pv, where Pa is alveolar pressure, Ppa is pulmonary artery pressure, and Ppv is pulmonary venous pressure? A. Zone I B. Zone II C. Zone III D. This representation does not exist Preferred response: B

Rationale See the rationale for Question 12.

Chapter 46: Mechanical Dysfunction of the Respiratory System 1. A child with advanced cirrhosis is admitted to the ICU for management of acute gastrointestinal bleeding. The child has severe ascites and has become progressively more obtunded and hypoxemic. Which of the following mechanisms is most likely to result in a negative cardiovascular response to the institution of positive pressure ventilation in this patient? A. A disproportionate increase in pleural pressure as lung volume increases B. An increased need for sedation after endotracheal intubation C. A negative effect of PEEP on left ventricular contractility D. A reduction in lung compliance after the addition of PEEP Preferred response: A

Rationale Abdominal distention decreases chest wall compliance and, as a result, pleural pressure increases markedly as the lungs expand, decreasing venous return.

e69

2. Which of the following statements describes most accurately the relationship between the pressure displayed by the ventilator (measured at the endotracheal tube connector) and alveolar pressure during volume-controlled (constant flow) and pressure-controlled (decelerating inspiratory flow) mechanical ventilation? A. The difference between ventilator pressure and alveolar pressure is determined by lung compliance in both modes. B. The difference between ventilator pressure and alveolar pressure is not affected by air leaks around the endotracheal tube during pressure-controlled ventilation. C. Ventilator pressure is higher than alveolar pressure at endinspiration during volume-controlled ventilation. D. Ventilator pressure is higher than alveolar pressure at endinspiration during pressure-controlled ventilation. Preferred response: D

Rationale During volume-controlled ventilation, inspiratory flow continues throughout inspiration. Consequently, at the end of inspiration, there is a gradient of pressure between the endotracheal tube connector and the alveoli. During pressure-controlled ventilation, flow decreases and ultimately ceases during inspiration. Thus the two pressures tend to equilibrate at the end of inspiration. 3. Which of the following conditions is most likely to increase the volume displacement of the diaphragm in a spontaneously breathing infant? A. Abdominal distention B. Croup C. Pulmonary edema D. Spinal cord section at the level of C8 Preferred response: D

Rationale Intercostal muscle paralysis causes severe rib cage distortion during inspiration, adding to the volume that the diaphragm has to displace during a breath. All the other conditions tend to decrease tidal volume. 4. Regarding transmural pressures of thorax (PTH), lung (PL), and chest wall (PW), which of the following is correct (PA 5 alveolar pressure, Pb 5 atmospheric pressure, and Ppl 5 pressure at pleural surface)? A. PL 5 PA 2 Pb B. PL 5 PA 2 Ppl C. PTH 5 Ppl 2 Pb D. PW 5 PA 2 Ppl Preferred response: B

Rationale Different pressures are needed to inflate the thorax, the lungs, and the chest wall. When the respiratory muscles are completely relaxed, the thorax, the lungs, and the chest wall are all held at their respective volumes by outward-acting pressure gradients across their walls. These pressure gradients or transmural pressures are defined by the following equations (see Fig. 136.18): (1) PTH 5 PA 2 Pb, (2) PL 5 PA 2 Ppl, and (3) PW 5 Ppl 2 Pb.

e70

S E C T I O N XV



Pediatric Critical Care: Board Review Questions

5. Which term is used to describe the property responsible for the development of pressure-volume loops of the respiratory system during breathing? A. Compliance B. Elastance C. Hysteresis D. Resistance Preferred response: C

Lung volume

Rationale Analysis of the volume-pressure relationships of the thorax and its components becomes more complicated when pressure changes generated by gas flow and by the movement of the lung and chest wall tissue as the lungs inflate and deflate are considered. These pressure changes result from molecular interactions between the gas and the airway walls, within the gas stream itself, and among the components of the gas-liquid interface and the tissue. These molecular interactions always result in a net loss or dissipation of energy from the respiratory system. The lost energy can no longer be used to perform work, and consequently dissipative pressure losses cause the volume-pressure relationships of the respiratory system to follow a different trajectory depending on the direction of the volume change. This property, known as hysteresis, is responsible for the development of loops when the volume-pressure relationships are plotted continuously during a breath. In this graphic representation, the dissipative pressures can be easily identified as the horizontal distance between the volume-pressure tracing and the corresponding point on the elastic volume-pressure relationship. The work done against these pressures can be quantified as the area enclosed by the loop (see Figure 136.18, below).

Pres Pressure

• Fig. 136.18





6. Regarding elastic properties of the lung and chest wall, which of the following statements is correct? A. Elasticity is a dissipative property of the lungs and chest wall. B. Outward elastic recoil of the lungs is counterbalanced by the inward elastic recoil of the chest wall. C. The relaxation volume of the lungs in an adult is the residual volume. D. The negative pressure generated between the lung tissue and chest wall contributes to venous return at normal lung volumes. Preferred response: D

Rationale Elasticity is typically a nondissipative process because the energy needed to produce elastic deformation during inspiration is accumulated in the tissues and then used to empty the lungs during expiration. In contrast, all resistive processes are dissipative: The energy liberated by the friction of the gas against the airway walls or by the molecular interactions within the tissue is transformed into heat and transported outside of the system by the blood or the expired gas. Elastic pressures result from the tendency of the components of the lungs and chest wall to recover their original shape after undergoing deformation. By definition, elastic recoil drives the thorax and its individual components to adopt a volume, known as the relaxation volume, at which recoil itself is extinguished. The relaxation volumes of the lungs and the chest wall are the volumes that each of these components would adopt if all the mechanical constraints imposed by their mutual attachments and interactions were removed. The relaxation volume of the thorax, in contrast, is defined by the mechanical interaction of the lungs and the chest wall. It coincides with the point at which the opposing elastic recoils of these two components neutralize each other. In the adult, the relaxation volume of the lungs is lower than the residual volume (the volume of gas contained in the lungs at the end of a forced expiration). The relaxation volume of the chest wall, by contrast, exceeds 50% of the vital capacity (the maximal volume of gas that can be inhaled from residual volume). This discrepancy in the relaxation volumes of the lungs and chest wall has three important consequences. First, it forces the relaxation volume of the thorax as a whole to occupy a position intermediately between the relaxation volumes of the lungs and the chest wall (at approximately 35% of vital capacity). Under most circumstances, this volume coincides with the functional residual capacity (FRC), which is the volume contained in the lungs at the end of a tidal expiration. Second, as the thorax starts to rise above its relaxation volume during inspiration, the outward recoil of the chest wall contributes to the expansion of the lungs, thereby reducing the work that the respiratory muscles need to perform during normal circumstances. Finally, at normal breathing volumes, the opposing actions of the lungs and chest wall recoils create a negative pressure at the boundaries of the lung tissue with the chest wall and the other intrathoracic structures. This negative pressure is an important contributor to the return of venous blood to the chest. In an infant, the chest wall generates remarkably little outward recoil within the normal range of breathing volumes. Because the inward recoil of the lungs varies little with respect to lung size and age during development, the relaxation volume of the infant’s thorax is proportionally smaller than that of the adult. If, as occurs in the adult, the FRC coincided with this relaxation volume (15% of vital capacity compared with 35% in the adult), then the infant would be at a definite disadvantage in terms of alveolar stability and oxygenation. Newborns of most mammalian species have developed physiologic strategies to maintain their FRC above the relaxation volume of the thorax. 7. Maintenance of alveolar stability and oxygenation in neonates depend on physiologic strategies to do which of the following? A. Increase closing capacity B. Increase residual volume C. Increase vital capacity D. Maintain the FRC above the relaxation volume of the thorax Preferred response: D

CHAPTER 136  Board Review Questions

Rationale Newborns of most mammalian species have developed physiologic strategies to maintain their FRC above the relaxation volume of the thorax. These strategies are generally directed at interrupting expiratory flow before expiration is complete and include shortening of the expiratory time, contraction of adductor muscles of the glottis to retard exhalation, and persistence of the tonic activity of the inspiratory muscles during expiration. 8. Compared with adults and older children, premature infants and neonates are able to tolerate high lung volumes during mechanical ventilation without cardiovascular compromise, possibly because of which of the following conditions? A. Their lower airway resistance B. Their higher cardiac contractility C. Their higher cardiac output D. Their higher chest wall compliance Preferred response: D

Rationale The pressure inside the pleural space is really determined by the elastic recoil of the chest wall and the volume of the thoracic contents. As long as lung volume is not forced above its normal range and chest wall compliance is unaltered by disease, pleural pressure (and thus the pressure around the major vessels and the heart) remains low, regardless of the airway pressures. Conversely, excessive lung distention (e.g., in asthma) is always associated with a high pleural pressure and, for that reason, is less well tolerated from a cardiovascular point of view. The dependence of pleural pressure on chest wall compliance explains why premature infants and newborns, who have very large chest wall compliance, have limited changes in this pressure during positive pressure ventilation, even if physiologic lung volumes are exceeded. Disease-induced reductions in chest wall compliance, on the other hand, always increase pleural pressure and reduce venous return to the heart. This is one reason why patients with abdominal distention typically have low cardiac output and why relief of the distention (e.g., by paracentesis in patients with ascites) reduces pleural pressure and increases cardiac output. 9. Which one of the following areas is considered the zero reference during inspiration? A. The alveoli B. The extrathoracic airway C. The intrapleural space D. The mouth Preferred response: D

Rationale Airway transmural pressure varies during breathing. Its variations result from the fact that inspiration and expiration have very different effects on the pressures inside and outside the airways. The pressure inside all airways undergoes qualitatively similar changes during each phase of the breathing cycle. During inspiration, for instance, there is a gradient of increasingly negative pressures from the mouth, where pressure is atmospheric (or the zero reference), to the alveolar spaces, where the pressure must be negative (or subatmospheric) for gas to flow in. This negative pressure is of course driven by the actions of the respiratory muscles and transmitted to the lungs via the link between the chest wall and lungs at the pleural space. During expiration, alveolar pressure becomes positive and the gradient is inverted, with the pressures inside the airways being always positive but diminishing toward the mouth.

e71

10. What happens to the extrathoracic and intrathoracic portion of the airways during a normal respiratory cycle of inspiration and expiration with airflow? A. During inspiration, the pressure surrounding the extrathoracic airway is more positive than the lumen, resulting in narrowing of the airway. B. During inspiration, the pressure surrounding the extrathoracic airway is more negative than the lumen, resulting in narrowing of the airway. C. During expiration, the pressure surrounding the intrathoracic airway is more negative than the lumen, resulting in narrowing of the airway. D. During expiration, the pressure surrounding the intrathoracic airway is more negative than the lumen, resulting in dilation of the airway. Preferred response: A

Rationale During inspiration, there is a gradient of increasingly negative pressures from the mouth (zero reference), where the pressure is atmospheric, distally to the airway. The pressure at the alveolar spaces must be negative (or subatmospheric) for air to flow. The pressure inside the airways is always negative, regardless of intraor extrathoracic location. During expiration, for air to move from the alveolar spaces to the mouth, the pressures must be more positive distally and decrease toward the mouth. Normal breathing changes in airway caliber depend on the location of the airway (intrathoracic versus extrathoracic) and the phase of the respiratory cycle (inspiration versus expiration). Extrathoracic airways include the pharynx, larynx, and extrathoracic portion of the trachea and are surrounded by neck tissue, maintaining constant atmospheric pressure around these areas. During inspiration, the pressures distal to the extrathoracic airway become more negative, causing the extrathoracic portions of the airway to narrow. The opposite holds true for the intrathoracic airway. During inspiration, the pressure surrounding their walls (intrapleural pressure) becomes more negative compared with the pressure inside the lumen, which causes these portions of the airway to dilate. To be correct, responses B through D should read as follows: B. During inspiration, the pressure surrounding the extrathoracic airway is more positive than the lumen resulting in narrowing of the airway. C. During expiration, the pressure surrounding the extrathoracic airway is less positive than the lumen, resulting in dilation of the airway. D. During expiration, the pressure surrounding the intrathoracic airway is more positive than the lumen, resulting in narrowing of the airway. 1 1. What is the equal pressure point? A. During inspiration, the point at which transmural pressure is positive B. During inspiration, the point at which transmural pressure is negative C. During expiration, the point at which the pleural pressure equals alveolar pressure D. The pressure midway in the airway, where the pressure distal to the mouth equals the pressure proximal to the alveoli Preferred response: C

e72

S E C T I O N XV



Pediatric Critical Care: Board Review Questions

Rationale The narrowing of the intrathoracic airways during expiration is contingent on the existence of a pressure gradient from the alveoli to the mouth. Alveolar pressure (Pa) must always exceed pleural pressure (Ppl) by a magnitude equivalent to the elastic recoil of the lungs. As the gas progresses downstream during expiration, frictional pressure losses lower the pressure inside the airways. Eventually the cumulative pressure losses can be as large as the pulmonary elastic recoil, and the pressure inside the airways becomes equal to Ppl. Beyond this equal pressure point, airway transmural pressure becomes negative (i.e., the pressure outside exceeds the pressure inside the airway) and acts to collapse the airway. 1 2. Stridor in a child with croup occurs during which process? A. Expiration due to a more positive pressure downstream from the obstruction B. Expiration due to a more negative pressure downstream from the obstruction C. Inspiration due to a more negative pressure downstream from the obstruction D. Inspiration due to a more positive pressure downstream from the obstruction Preferred response: C

Rationale When airway obstruction is extrathoracic (e.g., with croup, glossoptosis, or tonsil or adenoid hypertrophy), the person must create a more negative pressure inside the airway segment downstream from the obstruction to overcome the increased resistance during inspiration. Therefore this segment of the airway tends to collapse, worsening the obstruction and producing a characteristic turbulent noise (inspiratory stridor) as gas accelerates through the narrowest point and induces vibrations in the airway mucosa, creating in the process a decrease in inside pressure that approximates the walls of the airway even further. The obstruction is relieved during expiration because the pressure inside the airway segment, now upstream from the obstruction, must become more positive with respect to atmospheric pressure to force gas flow through the obstruction (see Figure 136.19, below).

• Fig. 136.19





13. When does wheezing in a child with tracheobronchial compression occur? A. During inspiration and expiration due to a more positive pressure downstream from the obstruction B. During expiration due to a less positive pressure downstream from the obstruction C. During expiration due to a more positive pressure downstream from the obstruction D. During inspiration due to a more positive pressure downstream from the obstruction Preferred response: C

Rationale When airway obstruction is intrathoracic (e.g., with extrinsic compression of the trachea and bronchi, tracheobronchomalacia, and asthma), during inspiration, the pressure inside the airways downstream from the obstruction has to become more negative than that inside the airways upstream. However, no matter how negative, the pressure inside the airways still must be less negative than the pleural pressure because otherwise the lung recoil (Pa – Ppl) would be negative, which is unimaginable. Thus during inspiration the transmural pressure of intrathoracic airways remains positive. In contrast, during expiration the pressure inside the airway segment located between the obstruction and the thoracic outlet may become lower than the pleural pressure at some point. This situation, coupled with the convective acceleration of flow at the obstructed segments, causes these airways to collapse and produce high-pitched vibrations (wheezing), expiratory delay, and dynamic hyperinflation.

Chapter 47: Diseases of the Upper Respiratory Tract 1. An 18-month-old boy arrives to the emergency department with sudden onset high fever, toxic appearance, stridor, and drooling. His oxygen saturation on blow-by oxygen supplementation is 96%. Lateral neck radiograph shows a “thumb sign.” What is the most appropriate next step in the treatment of his condition? A. Bedside nasal fiberoptic examination of the pharynx to confirm the diagnosis B. Conservative management with noninvasive ventilation and antibiotics C. Endotracheal intubation in the emergency room D. Transport to the operating room for intubation Preferred response: D

Rationale This patient shows the classic signs and symptoms of epiglottitis. It is now a rare disease among vaccinated young children, but the sudden onset of severe airway obstruction requires prompt recognition and treatment. At the same time, care should be taken not to disturb the patient, as agitation can result in complete airway obstruction. For this reason, fiberoptic examination of the epiglottis in the patient who is awake is usually not advisable. Similarly, attempts to examine the oropharynx directly or to start an intravenous line should be discouraged. If the patient will tolerate it, humidified oxygen should be administered, preferably through a plastic hose held by the parent. If the suspicion for epiglottitis is high, the child should go to the operating room as quickly as possible. In the operating room, the patient is anesthetized with an inhaled anesthetic (sevoflurane) and oxygen while the patient is spontaneously breathing.

CHAPTER 136  Board Review Questions

Once the patient has been anesthetized, an intravenous catheter is inserted. Laryngoscopy is then performed. It may be exceedingly difficult to obtain a direct view of the glottis and trachea because of the large swollen epiglottis. Nevertheless, it is almost always possible to pass an endotracheal tube through the edematous tissues and into the trachea. Nasotracheal intubation is preferred to orotracheal intubation because the tube is more readily secured to the face, the patient cannot bite the tube, and salivation is decreased. An otolaryngologist should be in the operating room and ready to do an emergency tracheostomy if an airway cannot be secured by endotracheal intubation, although this is rarely necessary. Endotracheal intubation is preferred to tracheostomy because it has been shown that complications are more common when a tracheostomy has been routinely used to treat epiglottitis. After the airway is secured, blood cultures and cultures of the epiglottis are obtained, and antibiotic therapy is initiated with a penicillinase-resistant antibiotic because of the high incidence of H. influenzae resistance to ampicillin. Patients usually require endotracheal intubation for 24 to 72 hours while the swollen epiglottis returns to normal size. The patient may be allowed to breathe spontaneously through the endotracheal tube or may undergo mechanical ventilation. 2. A 5-year-old child with newly diagnosed leukemia is admitted to the pediatric intensive care unit in severe respiratory distress. Chest radiograph shows a widened mediastinum. Which one of the following measures is to be avoided during airway management in this child? A. Administration of muscle relaxants B. Endotracheal intubation C. Heliox administration D. Left lateral decubitus position Preferred response: A

Rationale This child likely has an anterior mediastinal mass compressing the intrathoracic trachea. Because the symptoms produced by a malignant mass impinging on the trachea can worsen dramatically over several days, the child with respiratory compromise resulting from a mediastinal mass deserves rapid evaluation and aggressive medical therapy. The child’s refusal of certain body positions (mostly the supine position) to avoid dyspnea often precedes other signs and symptoms (cough, tachypnea, respiratory distress) of an anterior mediastinal mass. Forcing the child to lie down may result in airway obstruction or even cardiac arrest. Conversely, airway obstruction by the mediastinal mass in the supine position is sometimes relieved by changing one’s body position (lateral decubitus, prone, sitting). Heliox (a mixture of 70% helium and 30% oxygen) administration may be beneficial in case there is severe narrowing of the trachea, because the characteristics of this mixture permit greater gas flow past areas of airway narrowing. Endotracheal intubation is indicated only if respiratory function becomes severely compromised. This measure, of course, is only of benefit if the tip of the endotracheal tube can be advanced distal to the site of tracheal compression, which often means that main stem intubation is necessary to bypass the lesion if it is at the level of the distal trachea or carina. If intubation is necessary, sedation/anesthesia before laryngoscopy should be carried out while maintaining spontaneous ventilation, as positive pressure ventilation might be impossible. Thus muscle relaxants in this situation should be avoided.

e73

Mechanical support with ECMO has been used to support patients with large mediastinal masses; however, the mass may distort the great vessels and pose unusual challenges for the ECMO team. Unfortunately, if the airway is lost or if the degree of cardiac compression is grave, ECMO might be the only option left to provide tissue oxygenation until the compression is relieved.

Chapter 48: Pediatric Acute Respiratory Distress Syndrome and Ventilator-Associated Lung Injury 1. You are called to the emergency department to assess a previously healthy 3-year-old male who presented with a 2-day history of fever and cough. He had been started on high flow nasal cannula and was escalated to full face bilevel positive airway pressure (BiPAP) with the following settings: inspiratory pressure, 16 cm H2O; expiratory pressure 5 8 cm H2O; fraction of inspired oxygen (Fio2), 0.8; oxygen saturation as measured by pulse oximetry (Spo2), 100%. His chest radiograph (CXR) revealed a right-sided opacity. You recall that mortality patients with pediatric acute respiratory distress syndrome (PARDS) treated with noninvasive ventilation is 15%. Which of the following statements regarding diagnosing PARDS in this child is correct? A. He has PARDS based on his timing, trigger, CXR, and Spo2/Fio2 of 125. B. He is admitted to the inpatient unit since he cannot have PARDS with a unilateral opacity on chest imaging. C. In order to diagnose PARDS, Pao2/Fio2 should be used so an arterial blood gas should be performed. D. The Fio2 should be titrated down until his Spo2 is #97% so that you can calculate the Spo2/Fio2. Preferred response: D

Rationale There is a strong linear association between Spo2 and Pao2 as long as the Spo2 is #97%. The Pediatric Acute Lung Injury Consensus Conference (PALICC) definition of PARDS recommends use of the oxygenation index or oxygenation saturation index when a Pao2 is not available. Although the PALICC definition of PARDS does not require new bilateral infiltrates, it recommends future studies investigate whether bilateral versus unilateral infiltrates improves discrimination of risk stratification. 2. The patient in question 1 is admitted to the PICU since he required endotracheal intubation due to increased work of breathing and persistent hypoxemia. He is currently medically paralyzed after intubation, and he is placed on the following ventilator settings: volume control mode with tidal volume 6 mL/kg; inspiratory plateau pressure, 28 cm H2O; positive-end expiratory pressure (PEEP), 10 cm H2O; fraction of inspired oxygen (Fio2), 0.9; Spo2, 85%. The respiratory therapist recommends increasing the PEEP and you: A. Agree and recommend increasing PEEP to 14 cm H2O B. Disagree because you are worried about raising his plateau pressure and causing barotrauma C. Disagree because he is treated with low tidal volume, and moderate hypoxemia is probably okay in a previously healthy 3 year old D. Suggest that the extracorporeal life support team be consulted because he is failing maximal support Preferred response: A

e74

S E C T I O N XV



Pediatric Critical Care: Board Review Questions

Rationale The optimal method to set PEEP for an individual patient remains a research question, and existing data do not support a single PEEP level (high or low) that improves outcome for adults or children with ARDS. However, multiple studies suggest high variability in use of PEEP by PICU providers, and a retrospective study suggest that mortality in children was lower when providers were adherent with the ARDSNet lower PEEP/higher Fio2 table. Lower PEEP/Higher Fio2 Fio2

0.3

0.4

0.4

0.5

0.5

0.6

0.7

0.7

PEEP

5

5

8

8

10

10

10

12

Fio2

0.7

0.8

0.9

0.9

0.9

1.0

PEEP

14

14

14

16

18

18-24

These data suggest that this PEEP/Fio2 table is a reasonable starting point, but adherence requires attention to the effect on plateau pressure, driving pressure, markers of oxygenation, and hemodynamics. 3. The patient in question 1 is now breathing spontaneously and is treated with pressure control synchronized intermittent mandatory ventilation on the following settings: pressure control, 28 cm H2O; positive-end expiratory pressure (PEEP), 12 cm H2O; pressure support, 20 cm H2O, ventilator rate, 20 per minute; fraction of inspired oxygen (Fio2), 0.7. His tidal volumes range from 5–6 mL/kg, and his oxygen saturation as measured by pulse oximetry (Spo2) is 94%. The patient is breathing above the ventilator at a rate of 36 breaths/minute, he has high inspiratory work with retractions, his end-tidal CO2 is 45 mm Hg and his Paco2 is 55 mm Hg. The nurse is concerned that the patient is uncomfortable and is asking if the patient can be sedated more deeply. You: A. Agree because most children who are intubated and mechanically ventilated need to be deeply sedated B. Agree and you consider whether neuromuscular blockade is indicated because you are worried that the patient’s respiratory effort is contributing to lung injury C. Disagree and raise the tidal volume because the patient’s effort and Paco2 suggest that he cannot be supported with low tidal volume ventilation D. Disagree because you are worried about causing delirium with more sedation and you think he is adequately supported. Preferred response: C

Rationale Although a randomized controlled trial of protocolized sedation did not reduce the number of days of mechanical ventilation, there are substantial data suggesting multiple harmful effects of sustained deep sedation of critically ill children and adults, including delirium and prolonged duration of hospitalization. The amount of patient effort required to identify patient self-inflicted lung injury (P-SILI) is difficult to determine without esophageal manometry. Although prevention of P-SILI is likely one mechanism of benefit of early neuromuscular blockade, a recent randomized controlled trial did not show benefit of neuromuscular blockade in adults with ARDS.

4. On the third day of the hospitalization, the patient in question 1 develops progressively worse hypoxemia. His oxygen saturation index (OSI) is now 22. Data suggest that pulmonary- specific ancillary therapies that are indicated in children include which of the following? A. Exogenous surfactant B. High-frequency oscillatory ventilation (HFOV) C. Inhaled nitric oxide D. There are no data in children supporting specific pulmonary ancillary therapies. Preferred response: D

Rationale There are no conclusive data supporting the specific pulmonary or pulmonary ancillary therapies in children. The most conclusive data supporting pulmonary therapy address limiting tidal volume in adults. Limiting the driving pressure and prone positioning seem to be beneficial in adults with ARDS, but there are no pediatric data. There are no studies supporting the use of exogenous surfactant, HFOV, or inhaled nitric oxide to improve mortality in children or adults.

Chapter 49: Acute Viral Bronchiolitis 1. A 9-month-old previously healthy infant is admitted to the pediatric intensive care unit with respiratory distress including nasal flaring, grunting, subcostal retractions, and bilateral scattered wheezing. Current symptoms were preceded by 3 days of rhinorrhea, cough, and fever. Which of the following treatment options is most associated with improved outcomes in bronchiolitis and is suggested to have utility in some inpatients by both the American Academy of Pediatrics (AAP) and the Canadian Pediatric Society (CPS)? A. Albuterol B. Heliox C. Inhaled epinephrine D. Inhaled hypertonic saline Preferred response: D

Rationale Hypertonic saline (HTS) may improve respiratory mechanics by increasing mucociliary clearance and reducing airway edema. HTS was prescribed to 13% of critical bronchiolitis subjects in one recent multicenter report and one-third of surveyed intensivists report prescribing HTS. In general, treatment guidelines do not encourage its routine use in children hospitalized with bronchiolitis, though the AAP and Canadian Pediatric Society (CPS) suggest it may have some utility in inpatients. Some data suggest that heliox may improve respiratory distress, but not necessarily clinical outcomes. Recent studies show that albuterol improves respiratory resistance by .20% in ,40% of subjects, though clinicians’ ability to identify “responders” based on subjective clinical exams was generally no better than a coin flip. Meta-analyzed data support that albuterol does not shorten hospital length of stay. The AAP, the Australasian Guidelines, and the United Kingdom’s National Institute for Health and Care Excellence (NICE) Guidelines state that neither epinephrine nor albuterol should be used in inpatients, though the AAP points out that critically ill children are generally excluded from the trials supporting that recommendation. Both the AAP and CPS suggest considering epinephrine specifically in select circumstances (e.g., “as a rescue agent in severe disease”).

CHAPTER 136  Board Review Questions

2. Which of the following is true about the spread of respiratory syncytial virus (RSV)? A. Hand washing with soap and water is the only acceptable method of hand hygiene after examining a patient with RSV. B. Hospitalized children with RSV infection do not require precautions to minimize spread of disease. C. Palivizumab prevents all cases of bronchiolitis, so hospitalized children receiving palivizumab prophylaxis may share a room with an RSV-infected child. D. RSV can remain infectious on counter tops for over 6 hours Preferred response: D

Rationale Viruses that cause bronchiolitis, like RSV and rhinovirus, are spread via multiple mechanisms including aerosols, direct contact with virus-containing secretions, and indirect contact (e.g., fomites). RSV can survive on surfaces for several hours. Many patients in the PICU without bronchiolitis have risk factors for severe disease, so preventing spread to these susceptible children may help to improve their outcomes. Hand disinfection, with either alcohol-based rubs or soap and water (if hands are visibly soiled), is recommended by the AAP before and after direct contact with patients, after contact with nearby inanimate objects, and after removing gloves. Use of gowns, gloves, and face protection also reduces transmission. Children at high-risk for severe and lifethreatening disease from RSV may be candidates for prophylactic passive immunization. Palivizumab, a monoclonal antibody against the RSV F glycoprotein, reduces the rates of hospitalization and ICU admission by 50% in high-risk children. Local guidelines vary by region, but prophylaxis may be warranted in children born extremely premature and those with chronic lung disease, hemodynamically significant congenital heart disease, immunodeficiency, and other comorbidities. Palivizumab does not improve clinical outcomes when given during acute illness to lower risk children. Vaccines against RSV are under development. 3. A 3-month-old infant is admitted to the pediatric intensive care unit with acute respiratory failure secondary to bronchiolitis, and high-flow nasal cannula (HFNC) at a flow rate of 2 L/kg/minute is initiated. What is the primary mechanism by which HFNC improves work of breathing? A. Application of positive-end expiratory pressure at 5–10 cmH2O B. Provision of oxygen to the anatomic dead space C. Increased laminar flow in the bronchioles D. Activation of alpha and beta adrenergic receptors Preferred response: B

Rationale HFNC improves a patient’s respiratory status via several mechanisms, including reduced metabolic work of the nasopharyngeal tissues, improved mucociliary function, and reduced inspiratory resistance. Substantial effects of HFNC are due to wash out of the nasopharyngeal anatomical dead space, replacing the CO2-rich and O2-poor air that remains in the nasopharynx at the end of exhalation with CO2-free and O2-rich gas, thereby improving CO2 removal and oxygenation. This may be particularly beneficial in young children—such as those with bronchiolitis—given the

e75

higher ratio of anatomic dead space to tidal volume in infants. HFNC is intended to be an open system, with the nares .50% unobstructed by the cannula to enable wash out, thereby limiting the amount of positive airway pressure generated. Nasopharyngeal pressures may reach 4–8 cm-H2O with flows up to ,2.5 L/ kg/min, but depend heavily on flow rate and whether the child’s mouth is open or closed, and vary widely between patients even if those factors are equivalent. How much of this pressure is transmitted to the alveoli is unclear. Regardless of the mechanisms, HFNC reduces work of breathing in children with bronchiolitis, with maximal effects seen at 1.5–2.0 L/kg/min, and its introduction into clinical practice has been associated with reduced rates of intubation for bronchiolitis. 4. In RSV bronchiolitis, what is the primary mechanism of respiratory pathophysiology? A. Endothelial damage of small blood vessels with associated focal hemorrhagic necrosis, and mononuclear infiltration of alveolar walls and fibrinous exudates with macrophages in the alveoli B. Viral particles replicating inside the type II pneumocytes initiate cellular apoptosis C. Viral replication initiates a cascade of T and B cell infiltration into the peribronchiolar tissue, leading to edema, mucous secretion and cellular sloughing D. Viral surface glycoproteins F and G inactivate surfactant leading to increased alveolar permeability, airway edema, and impaired gas exchange Preferred response: C

Rationale In an autopsy study, children who died from RSV infection were found to have extensive RSV antigen in lung epithelium, sloughed epithelial cells blocking the small airways, significant apoptosis, low quantities of lymphocyte cytokines, and a near absence of CD81 lymphocytes and NK cells. These findings suggest that the pathogenesis of fatal RSV infection may be secondary to the failure of the child to develop an appropriate adaptive cytotoxic T cell response to infection. A larger autopsy study included 250 children who died from a variety of acute respiratory infections including RSV, adenovirus, influenza, and parainfluenza. This study described RSV as causing the most profound damage and inflammation to the bronchiolar epithelial cells. More recent studies have described a pattern of necrotic RSV infected epithelial cells contributing to small airway inflammation. Furthermore, RSV likely destroys ciliated cells, contributing to impaired mechanical clearance of the distal airways. Young children and infants are disproportionately burdened by viral lower respiratory tract disease. In addition to functionally immature immune systems, infant respiratory anatomy and mechanics predispose to severe disease. The viral induced inflammatory response occurring in proportionally smaller bronchioles leads to alveolar obstruction and collapse with edema, mucus, and cellular debris. The increased resistance affects both inspiration and expiration in the small airways, ultimately leading to a “ballvalve” mechanism of air-trapping, hyperinflation, and resorption atelectasis. The subsequent pulmonary ventilation and perfusion mismatch may lead to hypoxemia.

e76

S E C T I O N XV



Pediatric Critical Care: Board Review Questions

Chapter 50: Asthma 1. An 8-year-old with a past medical history of moderate persistent asthma is brought to the emergency department due to progressive respiratory distress and wheezing after running out of his albuterol rescue inhaler. The patient is in moderate respiratory distress with diffuse inspiratory and expiratory wheezing and moderate subcostal retractions. Vital signs are pulse, 148 beats per minute; respiratory rate, 34 breaths per minute; blood pressure, 115/78 mm Hg, oxygen saturation as measured by pulse oximetry (Spo2), 96%. The patient is started on the standard asthma acute care protocol. After 30 minutes, he is less distressed, with scattered expiratory wheeze and improved work of breathing. Subsequent vital signs are heart rate, 164 beats per minute; respiratory rate, 22 breaths per minute; blood pressure, 122/82 mm Hg; Spo2, 88%. Which of the following interventions best explains the decrease in Spo2? A. Intramuscular ceftriaxone B. Intravenous fluid bolus with 0.9% saline C. Intravenous magnesium sulfate D. Intravenous methylprednisolone E. Nebulized albuterol Preferred response: E

Rationale A significant adverse effect of b-agonist agents is hypoxia. This is related to drug mediated pulmonary vasodilation overcoming local hypoxic vasoconstriction and increasing perfusion to poorly ventilated lung units creating intrapulmonary shunt. Intravenous fluids, corticosteroids, magnesium sulfate, and ceftriaxone are all common therapeutic interventions used in the emergency department setting to treat acute severe asthma. However none of these interventions would explain hypoxia in this particular patient. 2. Which of the following statements regarding the assessment of a child with critical asthma is most accurate? A. Lactic acidosis in a child with critical asthma is typically due to tissue hypoxia (type A) and should be treated with fluid administration and vasoactive medications B. Less severe pulsus paradoxus (a smaller decrease in blood pressure with inspiration) is typically a sign of a less severe asthma exacerbation C. Serial measurements of blood chemistry is indicated in critical asthma to assess for beta-agonist induced hyperkalemia D. The degree of wheezing heard upon auscultation correlates well with disease severity, with the sickest patients having the loudest wheezing Preferred response: B

Rationale Children with more severe asthma exacerbations have more prominent pulsus paradoxus via multiple mechanisms, including that the increased effort of breathing leads to a more negative intrapleural pressure and therefore a larger increase in left heart afterload. Some children with very severe asthma exacerbations have reduced air entry and quieter breath sounds, leading to reduced or even inaudible wheezing. Lactic acidosis in children with critical asthma is typically due to excess sympathetic stimulation (type B) from adrenergic medications (e.g., albuterol, terbutaline).

Beta-agonist medications can cause hypokalemia, not hyperkalemia. 3. Which of the following is the predominant mechanism by which terbutaline causes bronchodilation? A. Activating the b-2 receptor, which increases cyclic adenosine monophosphate (cAMP) levels by augmenting its synthesis by adenylate cyclase B. Blocking the acetylcholine receptor, reducing the level of cyclic guanosine monophosphate (cGMP) C. Blocking the N-methyl-D-aspartate receptors in airway smooth muscle D. Inhibiting phosphodiesterase, which increases cAMP levels by preventing its degradation Preferred response: A

Rationale An increase in cyclic adenosine monophosphate (cAMP) levels through adenylate cyclase–mediated synthesis is the predominant mechanism by which the b-agonists cause bronchodilation. Response B is the mechanism by which anticholinergic medications (e.g., ipratropium bromide) work. Response C is one mechanism by which ketamine causes bronchodilation. The methylxanthines (e.g., aminophylline) also cause bronchodilation by increasing cAMP. However, they work by inhibiting phosphodiesterase 4 and preventing the breakdown of cAMP (response D). 4. Which of the following statements regarding near-fatal asthma is true? A. A ventilation strategy of low respiratory rates (,12 breaths/min), moderate-to-high tidal volumes (8–12 mL/kg), and permissive hypercapnia has been proved to be associated with increased mortality and a rate of pneumothorax approaching 100%. B. Increasing positive end-expiratory pressure (PEEP) in mechanically ventilated asthma patients receiving neuromuscular blockade has been shown to have unfavorable effects on lung volumes, airway pressure, and hemodynamics. C. Ketamine is contraindicated during intubation due to its slow onset of action and tendency to cause bronchoconstriction. D. Nearly all subjects have a history of severe persistent asthma with frequent ICU admissions in the 1 year preceding the episode of near-fatal asthma. Preferred response: B

Rationale Tuxen established in 1989 that increasing PEEP in patients with airway obstruction who are receiving neuromuscular blockade is associated with unfavorable increases in hyperinflation and intrathoracic pressures and frequent decreases in systemic blood pressure and venous oxygen saturation. In our practice, we set PEEP to zero in mechanically ventilated asthmatic patients during neuromuscular blockade (barring extenuating circumstances) and use minimal PEEP (less than auto-PEEP and not more than 8 cm H2O) during spontaneous breathing. Mortality from near-fatal asthma is approximately 4% in the United States. In a study by the Collaborative Pediatric Critical Care Research Network, 11 fatalities were observed out of 261 children with near-fatal asthma (4.2%). Of these 11 children, 10 had suffered a cardiac arrest prior to PICU admission. In that same publication the authors reported that 13% of subjects had no prior history of



CHAPTER 136 Board Review Questions

asthma (response D) and that only 29% of patients had an admission for asthma in the preceding year. Similarly, among 51 subjects who died from asthma in Australia, 32% of subjects had never been admitted to the hospital because of asthma prior to their death. Ketamine is commonly employed when intubating a child with near-fatal asthma (response C) due to its rapid onset, bronchodilatory effects, and beneficial hemodynamic outcomes. As described in the text, we suggest a ventilatory strategy that employs relatively low respiratory rates (6–12 breaths/min), tidal volumes of 8 to 12 mL/kg, and permissive hypercapnia (response A). When a similar strategy was used in 26 subjects, all patients survived, and pneumothorax was uncommon. 5. A 10-year-old boy is admitted to the pediatric intensive care unit with status asthmaticus refractory to initial treatment in the emergency department. The patient is agitated and exhibits severe intercostal and subcostal retractions with barely audible breath sounds. Mechanical ventilation is initiated because of hypoxemia and impending respiratory failure. Based on current best standards of care, which of the following ventilation strategies should be used after intubation? A. Permissive hypercapnia with the use of low respiratory rates and long expiratory times to avoid dynamic hyperinflation B. Pressure-controlled ventilation with a long inspiratory time and a short expiratory time C. The application of high levels of positive end-expiratory pressure (10 to 15 cm H2O) to prevent derecruitment and atelectasis in the patient who has undergone neuromuscular blockade D. The use of high tidal volumes and rapid respiratory rates to deliver supraphysiologic minute ventilation and correct the respiratory acidosis Preferred response: A

Rationale Mechanical ventilation of a child with asthma should follow a strategy based on long expiratory times, slow respiratory rates, and permissive hypercapnia to avoid dynamic hyperinflation. Attempts at normalizing arterial blood gases by delivering supraphysiologic minute ventilation increase the risk of air leak, hemodynamic instability, and death.

Chapter 51: Neonatal Pulmonary Disease 1. The rapid physiologic response to surfactant therapy is: A. Decreased lung compliance B. Increased functional residual capacity (FRC) C. Increased lung resistance D. Increased surface tension Preferred response: B

Rationale Exogenous surfactant delivery of natural or synthetic surfactant proteins and phospholipids acts to rapidly decrease alveolar surface tension (Laplace’s Law: distending pressure 5 2 3 surface tension/alveolar radius) and increase lung compliance (volume/pressure), effectively increasing FRC. A direct effect of surfactant on lung resistance is unlikely, which can be seen by Ohm’s Law (resistance 5 pressure/flow), but with the resultant airway opening would act to either decrease or maintain lung resistance.

e77

2. What is the stage of lung maturation in a fetus at 28 weeks of gestation? A. Alveolar B. Embryonic C. Pseudoglandular D. Saccular Preferred response: D

Rationale The progression of human lung development begins with the embryonic stage (,0–5 weeks’ gestation), progressing to the pseudoglandular stage (,5–16 weeks), to canalicular stage (,16–25 weeks), to the terminal sac or saccular stage (,25–36 weeks), and finally entering the early alveolar stage at near term (.36 weeks). Not unexpectedly, human newborns in the late canalicular or saccular stage of lung development may require supplemental oxygen and respiratory support for respiratory insufficiency. Premature birth halts the normal intrauterine progression of lung development, believed to contribute to the development of “new” bronchopulmonary dysplasia (BPD) predominantly in infants ,28 weeks’ gestation. 3. The consequences of exogenous surfactant therapy for respiratory distress syndrome (RDS) include: A. Decreased risk of pulmonary hemorrhage B. Enhanced ventilation/perfusion C. Increased risk of air leaks D. Inhibited endogenous surfactant production Preferred response: B

Rationale Exogenous surfactant rapidly improves lung compliance and functional residual capacity (FRC) allowing for enhanced ventilation/perfusion, resulting in improved oxygenation and resolution of hypercarbia. Surfactant is also recycled and either catabolized or reutilized and does not decrease endogenous production by the type II alveolar cells. In randomized controlled trials (RCTs), surfactant reduced the incidence of air leak syndromes when compared to mechanical ventilation alone for RDS. In meta-analyses of RCTs, surfactant did not impact pulmonary hemorrhage and, in fact, a listed complication of exogenous surfactant labelling is pulmonary hemorrhage. Rather than a direct effect of surfactant, it is hypothesized that a rapid drop in pulmonary vascular resistance with improved oxygenation results in increased shunting of systemic blood to the lungs across a patent ductus arteriosus, leading to over-circulation and capillary leakage or pulmonary hemorrhage. 4. A 36-week-old male infant born by spontaneous vaginal delivery is found to have biphasic stridor when he is agitated. His oxygen saturations are 100%, but he shows increased work of breathing when he becomes upset. He had mild hypotonia, and brain computed tomography demonstrated a mild subarachnoid hemorrhage. Based on these findings, what is the most likely cause of the infant’s stridor? A. Laryngomalacia B. Tracheomalacia C. Unilateral vocal cord paralysis D. Vascular ring Preferred response: A

e78

S E C T I O N XV   Pediatric Critical Care: Board Review Questions

Rationale Biphasic stridor is usually the result of laryngeal obstruction. It tends to worsen with agitation, and it is the most common type of stridor in the neonatal period. Of the various causes of laryngeal obstruction, laryngomalacia is the most common laryngeal anomaly. 5. A 3-month-old term baby presents with a history of chronic wheezing that is unresponsive to bronchodilators and difficulty feeding. A chest x-ray reveals an abnormally shaped mediastinum. Which of the following is most likely? A. Gastroesophageal reflux B. Laryngomalacia C. Tracheoesophageal fistula D. Vascular ring Preferred response: D

Rationale Vascular compression of the trachea or main stem bronchus can result from improper regression of the embryonic branchial arch arteries during fetal development. The most common anomaly is a vascular “ring” consisting of a double aortic arch, in which the vessel completely encircles the trachea and esophagus. Other variants include an ectopic aortic arch (passing behind the esophagus) or an aberrant origin of the right brachiocephalic artery. Vascular rings cause inspiratory stridor and expiratory wheezing, neither of which change appreciably with the infant’s position. Intermittent worsening of symptoms is sometimes seen when the infant is feeding, as boluses passing down the esophagus further compress the trachea. Feeding difficulty also may be present because of esophageal compression. When a previously undiagnosed child first presents with these symptoms, the child may be misdiagnosed as having tracheomalacia, bronchomalacia, or tracheobronchomalacia until further imaging confirms the cardiovascular abnormality. This condition also can be detected antenatally. An abnormally shaped mediastinum on a chest film often provides a clue to this diagnosis. Endoscopy may identify tracheal or esophageal compression. An echocardiogram may define the nature of the vascular anomaly, but cardiac catheterization is sometimes necessary. Decisions regarding surgical correction of this defect depend on the relative compromise of the trachea and esophagus; however, the degree of compression may actually worsen as the infant grows. 6. A 37-week gestational age infant is born to a diabetic mother. The infant is vigorous at birth with a strong cry. His extremities are pink, and perfusion is excellent. At 4 hours of life, tachypnea and irritability develop, progressing to apnea. Oxygen saturation is 93% in room air. His lungs are clear on auscultation, and no murmur is present. What is the most probable cause for this child’s symptoms? A. Apnea of prematurity B. Congenital heart disease C. Hyperviscosity syndrome D. Laryngomalacia Preferred response: C

Rationale This child most likely has hyperviscosity syndrome. Hyperviscosity syndrome is a result of polycythemia and can occur in several situations, including twin-twin transfusion, maternal-fetal

transfusion, delayed cord clamping, home delivery, maternal diabetes, small-for-gestational-age infants, postmature infants, and infants with Down syndrome or Beckwith-Wiedemann syndrome. Symptoms generally relate to the degree of hyperviscosity and can range from tachypnea to apnea, listlessness to irritability, and jitteriness to seizures. Hyperviscosity syndrome presents in the first few hours of an infant’s life, and the presentation can mimic that in infants with congenital pneumonia, meconium aspiration, persistent pulmonary hypertension, or congenital heart disease. This child is a full-term infant, thus discarding the possibility of apnea of prematurity. Because this child was born with a strong cry, it is unlikely that he has vocal cord paralysis. Unilateral vocal cord paralysis usually presents with a weak or sometimes hoarse cry. Bilateral vocal cord paralysis manifests with moderate to severe stridor, and some infants may have near total airway obstruction. Infants with tracheomalacia may be mildly or severely affected, depending on the extent of involvement and the ability of surrounding supporting tissues to maintain airway patency. Affected infants generally have symptoms in the newborn period, but presentation may be delayed for many days or weeks if the defect is mild. In these milder cases, infants may remain symptom free until an intercurrent infection leads to increased airway secretions and increased work of breathing. Symptoms include expiratory wheezing and respiratory distress including tachypnea and retractions, and the infant may receive a mistaken diagnosis of reactive airway disease; however, the use of bronchodilators may actually worsen the condition. In the neonate, significant congenital heart disease typically presents in one of two ways: (1) cyanosis with minimal or no respiratory distress or (2) cardiorespiratory failure. Cyanotic lesions, such as transposition of the great vessels or tetralogy of Fallot, usually present in an infant who is blue but comfortable. Characteristically, congenital cyanotic heart disease is suspected in such an infant, even if a murmur is absent, particularly if the infant remains desaturated or hypoxemic in 100% oxygen. Less common in the neonate are the conditions that lead to early heart failure, such as an atrioventricular canal defect or a large ventricular septal defect. These infants generally are not cyanotic but are pale with marked respiratory distress and often a loud murmur; chest radiographs may show the classic signs of cardiomegaly and pulmonary vascular congestion.

Chapter 52: Pneumonitis and Interstitial Disease 1. Pulmonary hemorrhage, proteinuria and/or hematuria as well as anti–basement membrane antibody confirms the diagnosis of? A. Goodpasture syndrome B. Heiner syndrome C. Systemic lupus erythematosus D. Wegener granulomatosis Preferred response: A

Rationale Goodpasture syndrome is associated with auto-antibodies to the alveolar and glomerular basement membrane causing the symptoms of hemoptysis, proteinuria, or hematuria initially. Wegener granulomatosis is a primary vasculitis while lupus is related to immune complex deposition. Heiner syndrome is a food hypersensitivity pulmonary disease.

CHAPTER 136  Board Review Questions

2. In which of the following disease processes is direct injury to the mucosal surface the most common mode of pulmonary injury, causing epithelial cells of air passages to become necrotic and desquamate, resulting in marked airway obstruction and bronchospasm? A. Fungal infection B. Inhalational injury C. Interstitial lung disease D. Viral pneumonitis Preferred response: B

Rationale Injury of the airways in inhalational injury may be manifested as upper airway obstruction resulting in laryngotracheitis, bronchitis, and upper airway edema. More peripheral airway obstruction may present with classic findings of asthma and airway edema with hypersecretion. In cases of massive exposure, the presenting symptoms may be those associated with acute respiratory distress syndrome, manifested by profound ventilation/perfusion (V/Q) mismatch, cyanosis, dyspnea, and respiratory failure. In inhalational injury, the injury is directly to the tissues within the airway; in infectious or interstitial lung diseases, it is infiltration by other cell types that results in significant injury. 3. Which of the following is the leading cause of lower respiratory tract infection in infants and children? A. Fungus B. Staphylococcus aureus C. Streptococcus pneumoniae D. Viral agents Preferred response: D

Rationale Viral agents are the leading cause of lower respiratory tract infection in infants and children. Respiratory syncytial virus (RSV) followed by parainfluenza are the most commonly isolated viral agents. Streptococcus followed by Staphylococcus aureus are the leading causes of bacterial pneumonia but are far less common than viruses. Fungus and atypical pathogens are less common causes of lower respiratory infection in infants and children. 4. What is the appropriate order of therapeutic objectives in the management of massive pulmonary hemorrhage? A. Bronchoscopy (therapeutic or diagnostic), endotracheal intubation, fluid resuscitation B. Endotracheal intubation, increase positive end-expiratory pressure (PEEP), fluid resuscitation, bronchoscopy (therapeutic or diagnostic) C. Embolization of bleeding vessel, fluid resuscitation, endotracheal intubation D. Fluid resuscitation, endotracheal intubation, embolization of bleeding vessel Preferred response: B

Rationale Securing a safe and stable airway is always the primary goal of any resuscitation. Increasing the end-expiratory pressure may provide a tamponade to the site of hemorrhage in an effort to prevent further profuse bleeding. After establishing an airway, the patient needs to be resuscitated to allow for increased stability for future short-term control of the bleeding. These measures could include bronchoscopy or embolization of the bleeding vessel.

e79

5. Which of the following is present in the early stage of interstitial lung disease? A. Decreased expiratory flow rates B. Decreased resting arterial oxygen tension C. Diminished carbon monoxide diffusing capacity D. Increased carbon dioxide tension Preferred response: C

Rationale Changes in lung volumes in pulmonary parenchymal disease depend primarily on the intensity of the alveolitis and the stage of the disease process. Acute severe pneumonitis with an intense alveolitis is characterized by moderate to severe reduction in both vital capacity and total lung capacity. It also is associated with a reduction in pulmonary compliance. In the early stages, patients with chronic interstitial diseases involving the lung parenchyma often have normal vital capacity and total lung capacity. There is subsequent reduction in lung volumes and pulmonary compliance as the disease progresses and pulmonary fibrosis ensues. Expiratory flow rates usually are preserved in pneumonitis involving the lung parenchyma, and major obstructive defects, although reported, are rare. The carbon monoxide diffusing capacity, one of the earliest and most sensitive tests of parenchymal inflammation, is diminished in persons with interstitial lung disease. A reduction in carbon monoxide diffusing capacity is not specific and may be found with other parenchymal disorders. In early parenchymal disease, resting arterial oxygen tension may be normal, but there is often mild alveolar hyperventilation with a reduction in alveolar carbon dioxide tension and widening of the alveolar-arterial oxygen gradients (Pao2 – Pao2). With exercise, hypoxemia and an increased Pao2 – Pao2 become exaggerated because of ventilation/ perfusion imbalance. Ventilation/perfusion mismatch is attributed to regional alterations of flow, altered parenchymal compliance, and increased obstruction to pulmonary airflow. Progressive alveolitis and subsequent derangement of gas exchange lead to deterioration of ventilatory efficiency and markedly increased work of breathing. Adequate oxygenation may become impossible even with the use of high-flow supplemental oxygen. Resting hypercapnia, pulmonary hypertension, and eventual right ventricular dysfunction with heart failure are common sequelae. 6. A 15-month-old child is admitted to the pediatric intensive care unit with severe anemia (hemoglobin of 5 g/dL) due to pulmonary hemorrhage. The child’s previous medical history is significant for six episodes of otitis media in the previous 12 months. The child was born full term with a birth weight of 3.2 kg. Currently the child weighs 6 kg. Besides pallor, tachycardia, and tachypnea, no other abnormalities are noticed on physical examination. Of the following, which is the most likely diagnosis for this child? A. Behçet syndrome B. Goodpasture syndrome C. Heiner syndrome D. Wegener granulomatosis Preferred response: C

Rationale Pulmonary hemorrhage (PH) is defined as extravasation of blood into airways or lung parenchyma. Massive PH in adults is defined as blood loss of 600 mL or more in 24 hours. Loss of 10% of a patient’s circulating blood volume into the lungs regardless of age causes a significant alteration in cardiorespiratory function and should be considered massive. The diagnosis of PH following an

e80

S E C T I O N XV   Pediatric Critical Care: Board Review Questions

episode of silent bleeding is established by pulmonary hemosiderosis, which is the abnormal accumulation of iron within lung parenchyma and alveolar macrophages. The classic clinical triad of hemoptysis, microcytic hypochromic anemia, and diffuse alveolar-filling opacities on chest radiograph is found in most episodes of diffuse/immune PH. Although the lung may be the only organ affected, more frequently multiple organs are involved. In patients with PH, establishing which extrapulmonary organs are involved by the disease helps to narrow the differential diagnosis of which of the immune-mediated disorders is most likely present. Diffuse parenchymal bleeding without evidence of extrapulmonary involvement occurs in idiopathic pulmonary hemosiderosis, Heiner syndrome, and drug-induced PH. Idiopathic pulmonary hemosiderosis, a disease of childhood, is a diagnosis of exclusion. Clinically, episodes of PH recur, with 30% to 50% of patients eventually dying of exsanguination or respiratory failure. Microscopic examination of the lungs is compatible with nonspecific injury rather than a specific cause such as vasculitis or immune deposits. Heiner syndrome, which affects children between the ages of 6 months and 2 years, usually manifests other symptoms, such as chronic rhinitis, recurrent otitis media, and growth retardation. Tests for precipitating antibodies to milk proteins are positive. Symptoms resolve when milk and milk products are eliminated from the diet. In children with PH and either proteinuria, hematuria, or red cell casts, Goodpasture syndrome is the most likely etiology. The presence of a linear immunofluorescent staining of immunoglobulin and C3 along glomerular capillary walls and antibasement membrane antibody in the serum confirms the diagnosis of Goodpasture syndrome. Fifty percent of patients with Goodpasture syndrome die of asphyxia as a result of massive PH. The presence of sinusitis or bilateral, multiple cavitary pulmonary nodules and evidence of glomerulonephritis in patients with PH help distinguish Wegener granulomatosis from the other vasculitides. The immune-mediated causes of PH with multisystem organ involvement often have characteristic physical findings to suggest the diagnosis. Serositis, arthritis, facial erythema, fever, and glomerulonephritis are present prior to the development of PH in patients with systemic lupus erythematosus (SLE). Ten percent of all cases of immune-mediated PH are associated with SLE. The onset of PH in persons with SLE is abrupt. Recurrent uveitis, mucocutaneous ulcerations, and genital ulcerations in a patient with PH suggests Behçet syndrome as the etiology. Other clinical features seen with Behçet syndrome include arthritis, gastrointestinal disease, cardiovascular involvement, and central nervous system disease. A necrotizing vasculitis of small- to medium-sized arteries and veins and thromboses of the terminal vascular beds or vena cava confirm the diagnosis.

Chapter 53: Diseases of the Pulmonary Circulation 1. Pulmonary vasodilators that do not cause systemic vasodilation and systemic hypotension are referred to as selective pulmonary vasodilators. Which of the following agents is a selective pulmonary vasodilator? A. Inhaled nitric oxide B. Intravenous milrinone C. Intravenous sildenafil D. Oral sildenafil Preferred response: A

Rationale Inhaled nitric oxide (iNO) is a specific pulmonary vasodilator and does not cause systemic hypotension. Although it has some effects on systemic circulation, it typically does not cause systemic hypotension in patients with pulmonary hypertension. It is inactivated by combining with hemoglobin to form methemoglobin with minimal effects on systemic vasculature. 2. In the management of pulmonary arterial hypertension in children, which statement is most appropriate regarding the use of phosphodiesterase 5 inhibitors? A. All patients in the START-1 trial had exercise tolerance performed by bicycle ergometry as the primary end point at 16 weeks. B. The FDA recommends the use of low-dose sildenafil in the chronic management of PH in children, as it improves exercise ability. C. In the START-1 trial, the Kaplan-Meier estimates of 3-year survival rates from the start of sildenafil were 94%, 93%, and 88%, respectively, for patients randomized to low-, medium-, and high-dose sildenafil. D. Tadalafil has a lower affinity for PDE-5 compared with the other PDE inhibitors. Preferred response: C

Rationale The START-1 (sildenafil in treatment-naïve children aged 1 to 17 years with pulmonary arterial hypertension) pediatric trial was a landmark study, with the primary end point of exercise tolerance using bicycle ergometry testing only in those subjects able to perform (about half of study patients). The controversy arose when the mortality was assessed on the yearly basis in the low-dose (10 mg), medium-dose (10–40 mg), and high-dose (20–80 mg) groups. By 3 years, the hazard ratios for mortality were 3.95 (CI: 1.46–10.65) for high- versus low-dose and 1.92 (CI: 0.65–5.65) for mediumversus low-dose sildenafil. Kaplan-Meier estimates of 3-year survival rates from the start of sildenafil were 94%, 93%, and 88%, respectively, for patients randomized to low-, medium-, and high-dose sildenafil in the START-1 trial. In August 2012, the FDA issued a strong warning against the chronic use of sildenafil for pediatric patients (1–17 years of age). The warning states that (1) children taking a high dose of Revatio had a higher risk of death than children taking a low dose and (2) low doses of Revatio are not effective in improving exercise ability. However, future statements clarified that the earlier FDA statement was not intended to suggest that Revatio should never be used in children and can be used in acute critical care settings and in infants with bronchopulmonary dysplasia and pulmonary hypertension. One of the advantages of tadalafil is its strong affinity for PDE-5 compared to other PDE inhibitors and hence its longer duration of action.

CHAPTER 136  Board Review Questions

3. What is the least accurate statement regarding the management of pulmonary arterial hypertension (PAH) in children? A. Coadministration of sildenafil with ketoconazole or rifampin should be avoided. B. Due to the risk of hepatic toxicity, the FDA requires that liver function tests be performed at least once in 3 months in patients on endothelial receptor antagonists such as bosentan. C. Nitric oxide is currently the first-line drug in the acute management of PAH or in cases of postoperative PAH arising from congenital heart disease (CHD) repair. D. There is a mutual pharmacokinetic interaction between bosentan and sildenafil that may influence the dosage of each drug in a combination treatment. Preferred response: B

Rationale As sildenafil is metabolized by hepatic CYP450, coadministration of sildenafil with CYP3A inducers or inhibitors such as ketoconazole or rifampin should be avoided. There is a mutual pharmacokinetic interaction between bosentan and sildenafil that may influence the dosage of each drug in a combination treatment. Bosentan decreases the maximum plasma concentration of sildenafil (Cmax) by 55.4% on day 16, whereas sildenafil increased bosentan Cmax by 42%, hence close monitoring is advisable with coadministration. Nitric oxide (NO) is currently the first-line drug in the acute management of PAH or in cases of postoperative PAH arising from CHD repair or other causes. Due to the risk of hepatic toxicity, the FDA requires that liver function tests be performed at least monthly and hematocrit every 3 months on patients on endothelial receptor antagonists such as bosentan. There is concern that the endothelin antagonists as a class may be capable of causing testicular atrophy and male infertility. 4. Which of the following statements most accurately reflects acute vasoreactivity testing in persons with pulmonary arterial hypertension (PAH)? A. Acute response to vasodilator testing is defined as a decrease in mean pulmonary arterial pressure by at least 10 mm Hg to an absolute level of less than 40 mm Hg with a decrease in cardiac output. B. Acute vasodilator testing is indicated in patients with significantly elevated left ventricular filling pressures. C. Inhaled nitric oxide (iNO) is the most commonly used drug for acute vasoreactivity. D. Responders to acute vasodilator testing (AVT) are more likely to respond to any therapy specific to PAH than are nonresponders. Preferred response: C

Rationale Vasoreactivity testing is an essential component in the diagnostic evaluation of PAH because it has therapeutic implications. Responders are more likely to respond to oral CCBs than are nonresponders and seem to have a better prognosis. AVT is usually performed during the same procedure as the diagnostic right heart catheterization. AVT is performed using iNO, intravenous PGI2, or intravenous adenosine. The most commonly used drug for AVT is iNO at doses of 20 to 40 ppm for 5 minutes. Hemodynamic measurements are recorded prior to and after administration of iNO, and the drug is discontinued completely. Based largely on data from Sitbon and colleagues, the European Society

e81

of Cardiology and American College of Chest Physicians guidelines propose that the acute response to vasodilator testing be defined as a decrease in mean pulmonary arterial pressure by at least 10 mm Hg to an absolute level of less than 40 mm Hg without a decrease in cardiac output. Patients with PAH due to conditions other than idiopathic PAH, such as BMPR2 genotype or anorexigen-induced PAH, have a very low rate of long-term responsiveness to oral CCB therapy. Accordingly, the decision to proceed with AVT in such patients should be individualized. AVT testing is not indicated, and may be harmful, in patients with significantly elevated left heart filling pressures. 5. Which of the following statements is true regarding vasoactive mediators and their effect on vascular smooth muscle cells? A. Nitric oxide stimulates the formation of cyclic adenosine monophosphate (cAMP). B. Phosphodiesterase 3 (PDE-3) inhibitors such as milrinone increase cAMP levels. C. Phosphodiesterase 5 (PDE-5) inhibitors such as sildenafil increase cAMP levels. D. Prostacyclin increases the formation of cyclic guanosine monophosphate (cGMP). Preferred response: B

Rationale Vasodilators such as NO and prostacyclin relax vascular smooth muscle by increasing intracellular concentrations of the second messengers cGMP and cAMP, respectively. Inhibition of the cGMP degrading phosphodiesterase (PDE-5) by sildenafil and inhibition of the cAMP degrading phosphodiesterase (PDE-3) by milrinone offer additional tools to achieve pulmonary vasodilation in patients with persistent pulmonary hypertension. Type 5 phosphodiesterase (PDE-5) is primarily responsible for degradation of cGMP to the inactive metabolite GMP. One way of augmenting the concentration of cGMP in pulmonary vessels is by inhibiting the activity of PDE-5. PDE-5 inhibitors, such as sildenafil and tadalafil, might therefore be expected to prolong the vasodilating effects of cGMP. Sildenafil is a potent and selective inhibitor of cGMP-specific PDE-5. 6. Which of the following statements is true regarding fetal circulation and transition at birth? A. All the blood draining into the right atrium from the inferior vena cava (IVC) enters the left atrium. B. Pulmonary vascular resistance falls and systemic vascular resistance increases soon after birth. C. The ductus arteriosus shunts blood from right to left (pulmonary artery to aorta) soon after birth. D. The highest Pao2 levels in the fetus are in the umbilical arteries. Preferred response: B

Rationale Oxygenated blood (Pao2 approximately 30–40 mm Hg) returning from the placenta in the umbilical vein splits in the liver, with slightly more than half passing through the ductus venosus to the IVC. The oxygenated blood streams along the medial aspect of the IVC as it enters the right atrium. Approximately two-thirds of the IVC flow is directed toward the foramen ovale by the eustachian valve and the septum primum and enters the left atrium. The remaining third of IVC flow mixes with the blood from the superior vena cava and enters the right ventricle. The majority of

e82

S E C T I O N XV   Pediatric Critical Care: Board Review Questions

the right ventricular output enters the ductus arteriosus and the descending aorta. A small portion enters the lungs via the pulmonary arteries. The ratio of blood flow to the pulmonary arteries to the flow that traverses the ductus arteriosus is determined by the fetal pulmonary vascular resistance (PVR). The first stage of transitional circulation is essentially a fetal pulmonary circulation that is characterized by high pressure and low flow because of both passive and active elevation of PVR. Thus PVR exceeds systemic vascular resistance, resulting in right atrial and ventricular pressures exceeding left atrial and ventricular pressures. High PVR results in right-to-left shunting of blood across the foramen ovale, and most of the blood ejected by the right ventricle flows across the ductus arteriosus into the descending aorta. The second stage of normal transition is accomplished when the fluid-filled fetal lungs are distended with air during the first breath. A rapid decrease in PVR occurs with mechanical distention of the pulmonary vascular bed. The entry of air into the alveoli improves oxygenation of the pulmonary vascular bed, further decreasing PVR. At birth, PVR decreases dramatically. As PVR becomes less than systemic, flow across the ductus reverses. Within the first 5 minutes after birth, oxygen-induced vasodilation and lung expansion decrease PVR to approximately half of systemic resistance. During the first few hours after birth, the ductus arteriosus closes, largely in response to the increase in oxygen tension. At this point the normal postnatal circulatory pattern is established.

Chapter 54: Mechanical Ventilation and Respiratory Care 1. Which of the following is a described advantage of using a decelerating flow pattern for mechanically ventilated patients? A. Decreased length of ventilation in children with asthma B. Higher mean airway pressures C. Higher peak inspiratory pressures D. Improved triggering Preferred response: B

Rationale Given that the majority of flow occurs early in a breath delivered with decelerating flow pattern, pressure rises faster, functionally raising mean airway pressure, along with a lower peak inspiratory pressure for the same delivered tidal volume. While decelerating flow is a more natural breath and may improve synchrony, the flow pattern has no effect on triggering, nor has it been associated with improved outcomes. 2. Which of the following is most consistent with a lung protective ventilation strategy in a child with new bilateral infiltrates, oxygenation index of 23, Spo2 88%, and hemodynamic instability? A. Paco2 36 mm Hg B. PEEP 14 cm H2O C. Pplat 34 cm H2O D. Tidal volume (Vt) of 10 mL/kg ideal body weight (IBW) Preferred response: B

Rationale Lung protective strategies avoid 4 types of lung injury: barotrauma, volutrauma, atelectotrauma, and biotrauma. The former two types are avoided by limiting Vt to ,8 mL/kg, keeping Pplat ,28 cm H2O, and allowing permissive hypercapnia. Elevated PEEP helps avoid atelectotrauma.

3. Restrictive lung disease may be associated with which of the following: A. Decreased compliance B. Elevated functional residual capacity (FRC) C. Increased airways resistance D. Prolonged time constant Preferred response: A

Rationale Restrictive lung diseases are a state of reduced compliance, either due to parenchymal disease or reduced chest wall compliance. Generally, frictional resistance may be increased, but airways resistance is not significantly affected. Because of the marked reduction in compliance, time constant is shortened, and FRC is reduced. 4. A 6-month old child with is endotracheally intubated and begun on mechanical ventilation for bronchiolitis. Ventilator settings are pressure control mode with positive end-expiratory pressure (PEEP) 6 cm H2O; peak inspiratory pressure (PIP), 14 cm H2O; measured tidal volume (Vt) of 7 mL/kg of ideal body weight (IBW); pressure support (PS) 12 cm H2O; fraction of inspired oxygen (Fio2), 0.6; ventilator rate, 28 per minute. Inspiratory time (Ti) is 1.2 sec. As neuromuscular blockade wears off, the patient becomes agitated and asynchronous with the ventilator. Evaluation of the patient reveals that the patient is awake, agitated, and not breathing over the ventilator. Based on this information, what would be a reasonable next step? A. Decrease Ti B. Increase trigger sensitivity C. Increase inspiratory flow demand D. Re-dose neuromuscular blocking agent Preferred response: A

Rationale The patient likely has asynchrony due to delayed cycling, and wants to start exhalation before the ventilator will allow it. The inspiratory time is set at more than half of the entire cycle time, and every breath in a controlled breath with a set inspiratory time. The patient would also likely benefit from a decreased respiratory rate to allow longer expiratory times and more supported breaths, where he is allowed to set his own inspiratory time. 5. Which of the following is correct in describing pressure-regulated volume control mode of ventilation? A. Patient-triggered, flow-controlled, flow-cycled ventilation B. Patient-triggered, flow-controlled, time-cycled ventilation C. Patient/machine-triggered, pressure-controlled, time-cycled ventilation D. Patient/machine-triggered, pressure-controlled, flow-cycled ventilation Preferred response: C

Rationale PRVC or pressure-regulated volume control mode of ventilation is a dual-control mandatory mode. Each breath is either patienttriggered (SIMV or Assist-control) or machine-triggered. The primary control is pressure. Between breaths, volume is controlled primarily by adjusting the pressure control level. Cycling is by time. All the other modes are common misconceptions of this mode.

CHAPTER 136  Board Review Questions

6. Which of the following is associated with a high risk (.25%) of extubation failure? A. Dynamic compliance of 0.9 mL/kg/cm H2O B. Fraction of inspired oxygen (Fio2) of 0.45 to maintain Spo2 of 95% C. Fraction of minute ventilation provided by the ventilator of 20% D. Spontaneous tidal volume without pressure-support of 6 mL/kg Preferred response: B

Rationale Answers A, C, and D are associated with ,10% risk of failure. Only Answer B is associated with at least a 25% of failure. Respiratory Parameter

Low Risk (10%)

High Risk (25%)

Spontaneous tidal volume (mL/kg)

.6.5

,3.5

Fio2

,0.3

.0.4

Mean airway pressure (cm H2O)

,5

.8.5

Peak inspiratory pressure (cm H2O)

,25

.30

Dynamic compliance (mL/kg/cm H2O)

.0.9